Всі тести з Крок 1 - Медицина
1 / 6307
Хворий на гіпертонічну хворобу, який лікувався гипотиазидом, скаржиться на загальну слабкість, втрату апетиту, серцебиття. Спостерігається гіпотонія м'язів, мляві паралічі, ослаблення перистальтики кишечника. Що може бути причиною такого стану?

Гіперурикемія

Гипонатриемия

Гіперкаліємія

Гипокалиемия

Гіперкальціємія

2 / 6307
При диспансерному обстеженні хлопчику 7 років встановлено діагноз - дальтонізм. Батьки здорові, кольорове зір нормальний. Але у дідуся по материнській лінії така ж аномалія. Який тип успадкування цієї аномалії?

Домінантний, зчеплений з підлогою

Аутосомно - рецесивний

Рецесивна, зчеплений з підлогою

Неповне домінування

Аутосомно - домінантний

3 / 6307
Методом непрямої калориметрії встановлено, що основний обмін досліджуваного на 40% нижче належного. Порушення діяльності якої ендокринної залози є причиною?

Підшлункова залоза

Тимус

Епіфіз

Надпочечники

Щитовидна залоза

4 / 6307
Проводячи хірургічне втручання на жовчному міхурі лікарю необхідно визначити загальний міхурцевий проток. В якому анатомічному утворенні очеревини він знаходиться?

Lig. Hepatorenale

Lig. Hepatoduodenal

Lig. Gastrocolicum

Lig. Duodenorenale

Lig. Hepatogastricum

5 / 6307
Під час розтину померлого в комі молодого людини виявлені великий тромбоемболічний інфаркт лівого півкулі мозку, велика септична селезінка, імунокомплексний гломерулонефрит, виразки на стулках аортального клапана, покриті поліпоподобнимі тромбами з колоніями стафілококів. Яке забол Еван викликало церебральну тромбоемболію?

Септикопіємія

Ревматичний тромбендокардіт

Септичний бактеріальний ендокардит

Гострий ревматичний вальвулит

Септицемія

6 / 6307
Провідником наукової експедиції по Індії був місцевий житель, який ніколи НЕ розлучався зі своєю собакою. Якими інвазійними захворюваннями можуть бути заражені члени експедиції при контакті з цією собакою, якщо вона є джерелом інвазії?

ехінококоз

фасциолезом

теніозом

парагонімоз

дикроцеліоз

7 / 6307
У хворого камінь загального жовчного протока припинив надходження жовчі в кишечник. Порушення якого процесу травлення при цьому спостерігається?

Переваривание білків

Переваривание вуглеводів

Всмоктування вуглеводів

Переваривание жирів

Всмоктування білків

8 / 6307
Больног про з явищами енцефалопатії госпіталізували в неврологічний стаціонар і виявили кореляцію між наростанням енцефалопатії і речовинами, які надходять з кишечника в загальний кровотік. Які сполуки, що утворюються в кишечнику, можуть викликати ендотоксемії?

Біотин

Ацетоацетат

Орнитин

Індол

Бутират

9 / 6307
При реєстрації ЕКГ хворого з гіперфункцією щитовидної залози зареєстровано збільшення частоти серцевих скорочень. Скорочення якого елемента ЕКГ про це свідчить?

Інтервалу PQ

Інтервалу RR

Комплексу QRS

Інтервалу Р-Т

Сегмента PQ

10 / 6307
Хворий 62 років поступив в неврологічне відділення в зв'язку з мозковим крововиливом. Стан важкий. Спостерігається наростання глибини і частоти дихання, а потім його зменшення до апное, сел ле чого цикл дихальних рухів відновлюється. Який тип дихання виник у хворого?

Чейна - Стокса

Апнейстичне

Кусмауля

Біота

Гаспінг - дихання

11 / 6307
Внаслідок блокади іонних каналів мембрани клітини її потенціал спокою зменшився з -90 мВ до -70 мВ. Які канали заблоковані?

Магнієві

кальцієві

Калієві

Натрієві

Хлорні

12 / 6307
У чоловіка 45 років після значної психоемоційної навантаження несподівано з'явилася стискає біль в області серця з іррадіацією в ліву руку, шию, під ліву лопатку. Особа стало блідим, покрилося холодним потім. Нітрогліцерин зняв напад болю. Який процес розвинувся у хворого?

Стенокардія

Психогенний шок

Інсульт

Інфаркт міокарда

Перфорація виразки шлунка

13 / 6307
На час профогляду у людини, НЕ має скарг на стан здоров'я, виявили лейкоцитоз. Причиною цього може бути те, що кров для аналізу здана після:

Розумової роботи

Значного споживання води

Фізичної навантаження

Відпочинку на курорті

Вживання алкоголю

14 / 6307
У препараті черв ого кісткового мозку людини визначаються скупчення гігантських клітин, що знаходяться в тісному контакті з синусоїдними капілярами. Які формені елементи крові утворюються з цих клітин?

Лімфоцити

Еритроцити

Кров'яні пластинки

Моноцити

Лейкоцити

15 / 6307
У крові пацієнта вміст глюкози натще було 5,65 ммоль / л, через 1 годину після цукрового навантаження становило 8,55 ммоль / л, а через 2 години - 4,95 ммоль / л. Такі показники характерні для:

Хворого з інсуліннезалежний цукровий діабет

Хворого з прихованим цукровим діабетом

Здорової людини

Хворого з тиреотоксикозом

Хворого з інсулінозалежним цукровим діабетом

16 / 6307
Жінка 30 років хворіє близько року, коли вперше з'явилися болі в області суглобів, їх припухлість, почервоніння шкіри над ними. Попередній діагноз - ревматоїдний артрит. Однією з ймовірних причин цього захворювання є зміна в структурі білка сполучної тканини:

овальбумін

муцин

Коллагена

міозин

тропонин

17 / 6307
У результаті безконтрольного прийому вітамінного препарату у дитини з'явилися анорексія, нудота, блювота, пронос, гіпертермія, з'явилися крововиливи на шкірі та слизових, а також явища менингизма. Який препарат приймав дитина?

Соматотропин

Ретинолу ацетат

Нікотинамід

Токоферолу ацетат

Рутин

18 / 6307
У породіллі через 24 години після пологів діагностовано емболія легеневої артерії. З'явилася задишка з частим глибоким диханням. АТ - 90/40 мм рт. ст, частота серцевих скорочень - 99 / хв. Газовий аналіз: рН = 7,48; pC Про 2 = 32 мм рт. ст.; [ НСО 3 ] = 20 ммоль / л ; [BE] = +1 ммоль / л. Яке з перерахованих нижче розладів кислотно - лужного стану має місце в цьому випадку?

Компенсований респіраторний алкалоз

Де компенсований метаболічний ацидоз

Респіраторний алкалоз + метаболічний ацидоз

Декомпенсований респіраторний алкалоз

Компенсований метаболічний ацидоз

19 / 6307
Хворому при подагрі лікар призначив алопуринол. Яке фармакологічне властивість алопуринолу забезпечує терапевтичний ефект в даному випадку?

Прискорення катаболізму піримідинових нуклеотидів

Уповільнення реутилізацію піримідинових нуклеотидів

Конкурентне інгібування ксантиноксидази

Збільшення швидкості виведення азотовмісних речовин

Прискорення синтезу нуклеїнових кислот

20 / 6307
Є одноклітинний організм, що характеризується набором хромосом 2 п = 8, що розмножується безстатевим шляхом. Генетичне різноманітність особин в популяції складе (без урахування мутацій):

256 типів

128 типів

32 типу

8 типів

1 тип

21 / 6307
Під час розтину померлого від поширеного перит ВНІТ в дистальних відділах тонкої кишки виявлено численні виразки овальної форми, розташовані уздовж кишки. Дно виразок чисте, гладке, утворене м'язовою або серозною оболонкою, краї виразок рівні, закруглені. У двох виразках є перфоративні отвори діаметром до 0,5 см. Яке захворювання можна припустити?

Висипний тиф

Туберкульоз

Дизентерія

Холера

Черевний тиф

22 / 6307
З метою серологічної діагностики коклюшу поставлена розгорнута реакція з кашлюковим і паракоклюшная діагностикумами. На дні пробірок, в які було внесено діагностикум з Bordetella parapertussis, утворився зернистий осад. Які антитіла виявила ця реакція?

Аглютиніни

Антитоксини

бактеріолізини

Опсоніни

преціпітіни

23 / 6307
У дитини після перенесеної ангіни різко збільшилися лімфатичні вузли: паратрахеальние, біфуркаційні, шийні. При мікроскопії шийного лімфатичного вузла виявлені вогнища некрозу, обмежені лімфоцитами, епітеліоїдними клітинами і клітинами Пирогова - Лангханса. Яка найбільш ймовірна патологія?

Саркоидоз

Сап

Сифіліс

Туберкульоз

риносклероми

24 / 6307
У працівника хімчистки виявлена жирова дистрофія печінки. Порушення синтезу якого речовини в печінці може привести до даної патології?

фосфатидними кислоти

Сечовини

фосфатидилхолін

холевой кислоти

тристеарин

25 / 6307
У інфекційне відділення поступив дитина 9 років з скаргами на біль в животі, пронос, втрата апетиту. У випорожненнях - еритроцити. Аналіз показав наявність дизентерійної флори. Який препарат краще призначити біль ному?

Норсульфазол

Бісептол

Етазол

Фталазол

фенілсаліцілат

26 / 6307
На розтині жінки 33 років виявлено потовщення стінки шлунка в пілоричному відділі (на розрізі шари стінки помітні) з розростанням щільної білуватої тканини в підслизовому шарі і дрібними тяжами її в м'язовому шарі. Рельєф слизової оболонки збережений, складки ригідні, нерухомі. Яка макроскопічна форма пухлини в даному випадку?

Інфільтративно-виразкова форма

Вузол

Виразка

Інфільтрат

Кіста

27 / 6307
У людини зменшений діурез, гіпернатріємія, гіпокаліємія. Гиперсекреция якого гормону може бути причиною таких змін?

Передсердний натрійуретичний фактор

Вазопрессин

Паратгормон

Адреналін

Альдостерон

28 / 6307
У хворого виявлено порушення прохідності дихальних шляхів на рівні дрібних і середніх бронхів. Які порушення кислотно - лужного рівноваги можна визначити в крові в даному випадку?

Респіраторний алкалоз

Метаболічний ацидоз

Метаболічний алкалоз

Респіраторний ацидоз

29 / 6307
При розтині жінки 40 років, стра дає ревматоїдний артрит, виявлено збільшену щільну селезінку. На розрізі її тканина коричнево - червоного кольору з збільшеними фолікулами, які мають вигляд напівпрозорих сірувато - білуватих зерен. Який з перерахованих патологічних процесів найбільш вірогідний?

Гіаліноз селезінки

Сальна селезінка

Сагова селезінка

глазурного селезінка

порфірний селезінка

30 / 6307
Жінка 58 років. Стан важкий, помутніння свідомості, шкіра суха, очі запалі, ціаноз, запах гнилих яблук з рота. Об'єктивно: глюкоза крові 15,1 ммоль / л, глюкоза сечі 3,5%. Яка найбільш ймовірна причина цього стану?

Гипогликемическая кома

Гіповалеміческая кома

Гипергликемическая кома

Уремічна кома

Анафілактичний шок

31 / 6307
У травматологічний пункт доставлений хворий з пошкодженням м'язів нижніх кінцівок. За рахунок яких клітин можлива репаративна регенерація м'язових волокон і відновлення функції м'язів?

Клітин - сателітів

Фібробластів

міоепітеліальние клітин

міофібробласти

міобласти

32 / 6307
До лікаря - інфекціоніста на прийом прийшов хворий з скаргами на лихоманку, що триває три дні, загальну слабкість, безсоння, погіршення апетиту. Лікар запідозрив черевної тиф. Який метод лабораторної діагностики найбільш доцільно призначити для підтвердження діагнозу?

Виділення білікультури

Виділення копрокультури

Виділення міелокультури

Виділення урінокультури

Виділення гемокультури

33 / 6307
У хворого з хронічно протікає запальним процесом шкіри і підшкірної клітковини виявлено переважання процесів проліф ераціі. Брак якого гормону може призвести до цього?

СТГ

Альдостерон

Тироксин

Кортизон

Інсулін

34 / 6307
Хворий на цукровий діабет вранці натщесерце отримав призначену дозу інсуліну пролонгованої дії. Пропустив черговий прийом їжі і незабаром відчув ознаки гіпоглікемії. Застосування глюкози стан НЕ полегшило. Який препарат необхідно ввести для купірування даного стану?

Преднізолон

Норадреналин

Адреналін

Триамцинолон

Гидрокортизон

35 / 6307
При мікроскопічному дослідженні прямої кишки виявлені великі осередки некрозу слизової оболонки, некротичні маси просякнуті фібрином, утворюючи плівку. Слизова і підслизова оболонка по периферії ділянок некрозу повнокровні, набряклі з крововиливами і лейкоцитарної інфільтрацією. Яке захворювання можна припустити?

Дизентерія

Черевний тиф

Холера

Амебіаз

Сальмонельоз

36 / 6307
Пацієнт звернувся в клініку з скаргами на загальну слабкість, ниючі болі в животі, поганий апетит, з підозрою на жовтяницю. У сироватці крові виявлено 77,3 мкмоль / л загального білірубіну і 70,76 мкмоль / л кон'югованого білірубіну. Який найбільш ймовірний вид жовтяниці?

Цироз печінки

Гемолітична жовтяниця

Обтурационная жовтяниця

Гострий гепатит

Механічна жовтяниця

37 / 6307
При гепатиті, інфаркті міокарда в плазмі крові хворих різко зростає активність аланін - і аспартатамінотрансферази. Які причини зростання активності цих ферментів в крові?

Збільшення швидкості розпаду амінокислот в тканинах

Пошкодження мембран клітин і вихід ферментів у кров

Зростання швидкості синтезу амінокислот у тканинах

Збільшення активності ферментів гормонами

Нестача піридоксину

38 / 6307
Хворому 50 років з метою лікування черевного тифу призначений левоміцетин, але на наступний день стан хворого погіршився, температура піднялася до 39,6 ° С. Чим пояснити погіршення стану хворого?

реінфекцій

Алергічної реакцією

Дією ендотоксинів збудника

Приєднання вторинної інфекції

нечутливість збудника до левоміцетину

39 / 6307
При обстеженні хворого виявлено наступні клінічні прояви: шкірні покриви рожеві, теплі на дотик, сухі, ЧСС - 92 / хв., ЧД - 22 / хв., Температура тіла - 39,2 ° С. Яке співвідношення процесів утворення і віддачі тепла в описаному періоді лихоманки?

Посилення теплопродукції без зміни тепловіддачі

Теплопродукция нижче тепловіддачі

Теплопродукция равн а тепловіддачі

Теплопродукция перевищує тепловіддачу

Зниження тепловіддачі на фоні незміненої теплопродукції

40 / 6307
У спортсмена на старті перед змаганнями відзначається підвищення артеріального тиску і частоти серцевих скорочень. Впливом яких відділів ЦНС можливо пояснити зазначені зміни?

Середнього мозку

Кори великих півкуль

Проміжного мозку

довгаста мозку

Гіпоталамуса

41 / 6307
У експерименті на кролі через 2 тижні після звуження ниркової артерії виявлено збільшення кількості еритроцитів і гемоглобіну в крові внаслідок стимуляції еритропоезу еритропоетинами. Що підсилює утворення еритропоетинів?

Гиперосмия

гіпоосмія

Гіперкапнія

Гиповолемия

Гипоксемия

42 / 6307
Хворий, який три місяці тому отримав травму голови, чує мова, розуміє її, але НЕ може правильно назвати предмет. В який частці кори великих півкуль є ушкодження?

Верхня лобова

Середня лобова

Верхня скронева

Середня скронева

Нижня лобова

43 / 6307
При розподілі клітини досліднику вдалося спостерігати фазу, при якій були відсутні мембрани ядра і ядерце, а центриоли перебували на полюсах клітини. Хромосоми мали вигляд клубка ниток, вільно розташованих в цитоплазмі. Для якої фази це характерно?

Інтерфаз

телофаза

метафазі

анафазу

профазі

44 / 6307
Під час патронажу лікар виявив у дитини симетричну шорсткість щік, діарею, порушення нервової діяльності. Недолік будь харчових факторів є причиною такого стану?

Треонін, пантотенова кислота

Лізин, аскорбінова кислота

Фенілаланін, пангамовая кислота

Нікотинова кислота, триптофан

Метіонін, ліпоєва кислота

45 / 6307
Хворий гіпотонією, відчувши погіршення загального стану прийняв без контролю лікаря кілька таблеток ефедрину через короткі проміжки часу. Але деякий поліпшення спостерігав тільки після прийому першої таблетки. Яким явищем обумовлено таке дію?

Кумуляция

Сенсибілізація

Звикання

Ідіосинкразія

Тахіфілаксія

46 / 6307
У чоловіка 60 років діагностовано інсульт в області латеральних ядер гіпоталамуса. Які зміни поведінки слід очікувати при цьому?

Ненаситність

Агресивність

Відмова від їжі

Жага

Депресія

47 / 6307
У хворого на ЕКГ виявлено збільшення тривалості інтервалу QT. Це може бути наслідком зменшення в шлуночках швидкості:

деполяризація

Скорочення

Деполяризації і реполяризації

Розслаблення

реполяризації

48 / 6307
Хворий скаржиться на болі в шлунку, печію. При огляді виявлено про підвищення кислотності шлункового соку. Що необхідно призначити хворому для нейтралізації кислотності шлункового соку?

Алмагель

Бензогексоний

Папаверина гідрохлорид

Атропіну сульфат

Прозерін

49 / 6307
У хворих на алкоголізм часто спостерігається гіповітаміноз Вх, який є наслідком порушення харчування. Симптомами гіповітамінозу В 1 є розлади нервової системи, психози, втрата пам'яті. Чому до дефіциту вітаміну Вх особливо чутливі клітини нервової тканини?

Посилюється ліполіз жирової тканини

Підвищується інтенсивність гліколізу

Знижується інтенсивність гліколізу

Порушується окислення жирних кислот

Порушується аеробний розпад глюкози

50 / 6307
Чоловік 37 років доставлений в хірургічне відділення з явищами гострого панкреатиту: блювота, пронос, брадикардія, гіпотензія, слабкість, явища зневоднення організму. Який препарат найбільш доцільно використовувати в першу чергу?

Ефедрин

Контрикал

Але - шпу

Платифиллин

Етаперазін

51 / 6307
У тимусі призупинено освіту Т - лімфоцитів - хелперів. Які процеси імуногенезу, що відбуваються в сполучної тканини, будуть порушуватися в першу чергу?

опсонізації

Перетворення В-лімфоцитів в плазматичні клітини

фагоцітірованія антигенів макрофагами

Освіти попередників Т-лімфоцитів

фагоцітірованія чужорідних тіл

52 / 6307
Жінка 52 років, хворіє на цукровий діабет, скаржиться на головний біль, стомлюваність, безсоння. Під час огляду встановлено високий рівень артеріального тиску - 200/100 мм рт. ст. Який препарат найбільш доцільно використовувати д ля швидкого зниження артеріального тиску?

Каптоприл

Анаприлин

Папаверин

Празозин

Резерпін

53 / 6307
Хворому з закритим переломом плечової кістки накладена гіпсова пов'язка. На наступний день з'явилася припухлість, синюшність і похолодання кисті травмованої руки. Про якому розладі периферичного кровопостачання свідчать ці ознаки?

Венозна гіперемія

Тромбоз

Артеріальна гіперемія

Ішемія

Емболія

54 / 6307
До отоларинголога звернувся хворий, у якого при огляді гортані виявлено неповне змикання голосових зв'язок при фонації. Голосова щілина при цьому приймає форму овалу. Функція який м'язи гортані порушена у хворого?

М. cricoaryttenoideus lateralis

M. vocalis

М. aryttenoideus transversus

M. cricoar yttenoideus posterior

М. thyroaryttenoideus

55 / 6307
Лікування дитини, хворої на рахіт, з допомогою вітаміну D 3 НЕ дало позитивного результату. Яка найбільш ймовірна причина неефективності лікування?

Порушення включення вітаміну D3 в фермент

Підвищений використання вітаміну D3 мікрофлорою кишечника

Недостатність ліпідів в їжі

Порушення гідроксилювання вітаміну D3

Порушення транспорту вітаміну D3 білками крові

56 / 6307
У жінки виявлено пухлину яєчника. Показана операція. Яку зв'язку повинен перерізати хірург, щоб відділити яєчник від матки?

Широку зв'язку матки

Власну зв'язку яєчника

Круглу зв'язку матки

Бічну пупкову зв'язку

Зв'язку, підвішуючу яєчник

57 / 6307
У хворого за добу виділяється води з організму менше, ніж надходить. Яке захворювання може привести до такого стану?

Цистит

Панкреатит

Інфекційні хвороби

Серцева недостатність

Гепатит

58 / 6307
У хворого в 2 рази збільшена щитовидна залоза. При пальпації заліза щільна, поверхня нерівномірна, горбиста. При гістологічному дослідженні - дифузна інфільтрація тканин залози лімфоцитами, плазматичними клітинами з утворенням фолікул і посилене розростання сполучної тканини. Який найбільш ймовірний діагноз?

Ендемічний зоб

Дифузний токсичний зоб

Зоб Хасімото

Спорадичний зоб

Зоб Ріделя

59 / 6307
У пацієнтки з постійною гіпоглікемією аналіз крові після введення адреналіну суттєво НЕ змінився. Лікар припустив порушення в печінці. Про зміну який функції печінки може йти мова?

Екскреторної

Глікогендепонірующей

Холестерінообразующей

Кетогенной

гликолитическому

60 / 6307
У результаті черепно - мозкової травми у хворого були виявлені наступні симптоми: інтенційний тремор, дисметрія, адиадохокинез, дизартрія. Яка структура голів ного мозку пошкоджена?

Стріатум

Мозочок

Рухова кора

Блідий куля

Чорне речовина

61 / 6307
У хворого М., 45 років, при аналізі ЕКГ виявлено: ритм синусовий, число передсердних комплексів більше числа шлуночкових комплексів ; прогресуюче подовження інтервалу Р -Q від комплексу до комплексу ; випадання окремих шлуночкових комплексів ; зубці Р і комплекси QRST без змін. Назвіть тип порушень серцевого ритму?

Повна атріовентрикулярна блокада

Атріовентрикулярна блокада I ступеня

Атриовентрикулярная блокада II ступеня

Сіноаурікуляр ная блокада

внутрішньопередсердну блокада

62 / 6307
У лікарню доставлений дворічний дитина з уповільненим розумовим і фізичним розвитком, що страждає частими рвотами після прийому їжі. У сечі визначена фенилпировиноградная кислота. Наслідком порушення якого обміну є дана патологія?

Ліпідного обміну

Обміну амінокислот

Водно - сольового обміну

Вуглеводного обміну

Фосфорно - кальцієвого обміну

63 / 6307
У жінки 42 років, яка перенесла операцію на нирці, після наркозу розвинулися явища рекурарізаціі і припинилося дихання. В якості миорелаксанта був використаний дитилін. Яке засіб найбільш доцільно використовувати для відновлення тонусу м'язів? - Стрихніну нітрат

Галантоміна гідробромід

Прозерін

Плазму крові

кофеі н

64 / 6307
У клініку доставлений дитина 8 років з різаною раною підошви правої ноги. При хірургічної обробці виявлена глибока рана з розтином сухожилля м'язи на підошовної поверхні, ближче до латерального краю стопи. У хворого обмежене підняття латерального краю стопи. Функція який м'язи швидше за все порушена?

М. extensor digitorum longus

M. tibialis anterior

М. peroneus longus

М. quadriceps femoris

M. triceps surae

65 / 6307
Хворий 40 років госпіталізований з скаргами на загальну слабкість, судоми верхніх і нижніх кінцівок, АТ - 160/100 мм рт. ст. Результати дослідження: глюкоза крові - 6,5 ммоль / л, холестерин - 6 ммоль / л, кальцій - 2 ммоль / л, фосфор - 1 ммоль / л, натрій -160 ммоль / л. Сечовиділення - 700 мл в добу. Як перша патологія найбільш імовірно буде причиною такого стану?

гіперпаратиреоїдизмом

Рахіт

Тиреотоксикоз

Гіперальдостеронізм

Гіпоальдостеронізм

66 / 6307
Хворому на туберкульоз, в анамнезі якого була відкрита легенева форма захворювання, проведено мікроскопічне дослідження мокротиння з метою визначення збудника. Який метод фарбування доцільно при цьому використовувати?

Метод Романовського - Гімза

Метод Ціля - Нільсона

Метод Грама

Метод Бурри - Гінса

Метод Нейссера

67 / 6307
При томографії у хворого в середостінні виявлені збільшені лімфатичні вузли. При гістологічному дослідженні в лімфовузлі виявлені циркулярні розростання сполучної тканини, навколишнього гранулемоподобная освіти з лімфоцитів, плазмоцитів і великих двоядерних их клітин Березовського - Штернберга. Який найбільш ймовірний діагноз?

Лімфогранулематоз

Лімфолейкоз

Саркоидоз

Лімфосаркома

Туберкульоз

68 / 6307
У експерименті на постсинаптическую мембрану нейрона подіяли речовиною, яке викликало її гіперполяризацію. Проникність для яких іонів на постсинаптичні мембрані збільшилася в даній ситуації?

Калію

Кальцію

Натрію

Магнію

Марганця

69 / 6307
Вивчення відбитків виступів епідермісу пальців рук (т. Н. Дактилоскопія) використовується в криміналістиці для ідентифікації особи, а також для діагностики генетичних аномалій, в зокрема хвороби Дауна. Який шар шкіри визначає індивідуальність відбитків?

Роговий

Блискучий

Сітчастий

Сосочковий

Базальний

70 / 6307
У інфекційне відділений ие госпіталізовано хворого з скаргами на багаторазовий пронос і блювоту, біль в м'язах ніг, слабкість, запаморочення, ірач поставив попередній діагноз - 'холера'. Як необхідно досліджувати матеріал від хворого для експрес - діагнозу?

Пряма і непряма РІФ

Бактеріологічним методом

РА

Біологічним методом

71 / 6307
Після впливу мутагена в метафазної платівці людини виявлено на три хромосоми менше норми. Зазначена мутація відноситься до:

Транслокация

Анеуплоїдія

Політенія

Інверсія

Полиплоидия

72 / 6307
У лабораторії при мікроскопії мокротиння хворого на пневмонію випадково виявлені личинки. При аналізі крові виявлено еозинофілія. Який гельмінтоз можна припустити?

Описторхоз

Трихоцефальоз

Парагонімоз

Аскаридоз

Ентеробіоз

73 / 6307
Клітку лабораторного тваринного піддали надлишкового рентгенівському опроміненню. В результаті утворилися білкові фрагменти в цитоплазмі. Який органоид клітини прийме участь в їх утилізації?

Клітинний центр

Рибосоми

Комплекс Гольджі

Лізосоми

Ендоплазматичний Ретик лум

74 / 6307
У дівчинки 12 років при розтині виявлені множинні крововиливи в шкірі (переважно сідниць, нижніх кінцівок), серозних і слизових оболонках, в головному мозку. У надниркових залозах - вогнищевий некроз і масивні крововиливи, в нирках - некротичний нефроз, гнійний артрит, іридоцикліт, васкуліт. Який найбільш ймовірний діагноз?

Системна червона вовчак

Вузликовий періартеріїт

Променева хвороба

менінгококцемія

Висипний тиф

75 / 6307
На базарі громадянин А. продавав ковбасу під назвою ' свиняча домашня '. У Держсанінспекцією виникло підозра фальсифікації ковбаси. З допомогою якої серологічної реакції імунітету можна ідентифікувати харчовий продукт?

преципітацією

РСК

РНГА

імунофлуоресценція

Аглютинації

76 / 6307
У породіллі 35 років визначається больовий синдром, пов'язаний з затримкою першого періоду пологів. Який препарат краще за все застосувати для зменшення болю?

Кодеїн

Парацетамол

Анальгін

Морфін

Промедол

77 / 6307
У добовому раціоні дорослої здорової людини повинні бути жири, білки, вуглеводи, вітаміни, мінеральні солі і вода. Вкажіть кількість білка, що забезпечує нормальну життєдіяльність організму.

100-120

70-80

10-20

50-60

40-50

78 / 6307
У хворого виник спазм гладкої мускулатури бронхів. Використання будь активаторів для зняття нападу буде фізіологічно обгрунтованим?

М - холінорецепторів

α - і β - адреноблокатори

Β - адреноблокатори

α - адренорецепторів

Н - холінорецепторів

79 / 6307
Хворому з значними опік ами зробили пересадку донорської шкіри, але на 8- перші добу трансплантат набряк, змінився його колір і на 11 добу він почав відторгатися. Які клітини беруть в цьому участь?

Еозинофіли

Еритроцити

В - лімфоцити

Базофіли

Т - лімфоцити

80 / 6307
У клініку доставлений пацієнт з ознаками гострого алкогольного отруєння. Які зміни вуглеводного обміну характерні для цього стану?

У м'язах посилюється аеробний розпад глюкози

У м'язах переважає анаеробний розпад глюкози

У печінці посилюється розпад глікогену

В печінці знижується швидкість глюконеогенезу

У печінці посилюється глюконеогенез

81 / 6307
При дослідженні гнійних виділень з шийки матки бактериоскопически виявлено наявність грамнегативних бобовидних диплококков, що знаходяться як в середині, так і поза лейкоцитів. Назвіть збудника гнійного запалення шийки матки. -Calymmatobacterium granulomatis

Neisseria gonorrhoeae

Chlamydia trachomatis

Haemophilus vaginalis

Trichomonas vaginalis

82 / 6307
У дитини 7 років на шкірі розгинальних поверхонь Локтєв их і колінних суглобів з'явилися щільні, безболісні вузлики розміром 1-2 мм. У біоптаті вузликів - великий осередок фибриноидного некрозу сполучної тканини з лімфоцитами і макрофагами по периферії. При якому захворюванні спостерігаються такі вузлики?

Вузликовий періартеріїт

Ревматоїдний артрит

Склеродермия

Системна червона вовчак

Ревматизм

83 / 6307
У тварин в експерименті реєструють електричну активність нейронів спірального вузла, що дозволяє аналізувати афферентную імпульсацію від рецепторів:

Кортиєва органу

Вестибулярних

напівкружними каналів

Вестибулярних і Кортиєва органу

пристеночно

84 / 6307
Хвора 25 років звернулася з скаргами на погіршення зору. При огляді виявлено порушення акомодації, зіниця розширена, НЕ реагує на світло. Функція яких м'язів порушена?

М'яз, що розширює зіницю, ресничная

Верхня коса, ресничная

Латеральна пряма м'яз, що звужує зіницю

звужує і розширює зіницю м'язи

М'яз, що звужує зіницю, ресничная

85 / 6307
У клініку доставили пацієнта 32 років з масивною крововтратою внаслідок автодорожньої травми. Пульс 110 уд / хв., Частота дихання - 22 за 2 хв., АТ - 100/60 мм рт. ст. Яке зміна крові з перерахованих буде найбільш характерним через 1 годину після крововтрати?

Еритропенія

Гипопротеинемия

Гиповолемия

Гіпохромія еритроцитів

Лейкопенія

86 / 6307
Під час хірургічної операції пацієнту проведено переливання крові. На антигени якого збудника необхідно перевірити цю кров?

Вірусу гепатиту В

Вірусу гепатиту А

ентеровірус

Вірусу гепатиту Е

аденовіруси

87 / 6307
Пацієнт скаржиться на задишку після фізичної навантаження. Об'єктивно: анемія, наявність парапротеина в зоні гамма - глобулінів. Який показник в сечі необхідно визначити для підтвердження діагнозу мієломи?

Білірубін

Гемоглобін

Церулоплазмін

Білок Бене - Джонса

антитрипсину

88 / 6307
У хворого 45 років на правій нозі спостерігається блідість шкіри гомілки і стопи, визначаться відсутність пульсації тильної артерії стопи та задньої великогомілкової артерії. Пульсація стегнової артерії збережена. Поразка який артерії сталося?

Зовнішньої клубової

Низхідній колінної

Підколінної

Глибокої артерії стегна

Малогомілкової

89 / 6307
Хворий тривалий час приймав глюкокортикоїди. Після різкої відміни препарату скаржиться на міалгію ію, підвищену стомлюваність, емоційну нестабільність, головний біль, безсоння, втрату апетиту, нудоту. Розвинувся синдром відміни глюкокортикоїдів. Призначення яких препаратів показано для корекції даного стану?

Адреналін

Глюкокортикоїди

Мінералокортикоїди

АКТГ

Кортикостероїди

90 / 6307
При обстеженні міміки хворого виявлено, що він НЕ може скласти губи трубочкою, НЕ може свистіти, під час сміху куточки рота НЕ піднімаються вгору, ротова щілина розтягується в боки (попе чная посмішка). Пацієнт страждає на міопатію - спадкове захворювання з дистрофічних пошкодженням м'язів. На атрофію який м'язи вказують дані симптоми?

Круговий м'язи рота

Великий виличної м'язи

М'язи сміху

Жувальною м'язи

Шийної м'язи

91 / 6307
У експерименті на тварині видалення ділянки кори півкуль мозку усунуло раніше вироблені умовні рефлекси на світлове роздратування. Який ділянку кори був видалений?

прецентральная звивина

Скронева частка

Постцентральна звивина

Лімбічна кора

Потилична кора

92 / 6307
У чоловіка 36 років з черепно - мозкової травмою дихання слабке, пульс ніткообразний, рефлекси відсутні. Який шлях введення пірацетаму найбільш доцільний в даному випадку?

Пероральний

Ректальний

Інгаляційний

Підшкірний

Внутрішньовенний

93 / 6307
У хворого інфаркт передньої стінки лівого шлуночка. У басейні якого судини виникло порушення кровообігу?

Лівою крайової гілки лівої вінцевої артерії

Передньої міжшлуночкової гілки лівої вінцевої артерії

Задній міжшлуночкової гілки правої вінцевої артерії

огинає гілок і лівої вінцевої артерії

Предсердно - шлуночкових гілок правої вінцевої артерії

94 / 6307
Юнак 16 років, що страждає на цукровий діабет з 10 років, почув про можливості замінити ін'єкції інсуліну таблетками глібенкламіду. Однак лікар, до якого він звернувся, категорично йому відмовив. Чому глібенкламід не можна призначити в даному випадку?

Викликає гіперпродукцію гідрокортизону

Стимулює альфа - клітини

Чи не стимулює бета - клітини

Підсилює виділення адреналіну

Швидко деградує в печінці

95 / 6307
У хворого на туберкульоз після тривалого лікування з'явилися шум і дзвін у вухах, зниження слуху, висипання на шкірі, набряк слизових оболонок і порушилася координація рухів. Після скасування препарату стан хворого значно покращився. Який препарат приймав хворий?

Етамбутол

Рифампіцин

Стрептомицина сульфат

Бепаск

Ізоніазид

96 / 6307
При обстеженні у хворого виявлено підвищений вміст ліпопротеїнів низької щільності в сироватці крові. Яке захворювання можна очікувати у цього хворого?

Запалення легенів

Пошкодження нирок

Гострий панкреатит

Гастрит

Атеросклероз

97 / 6307
При черговому нападі бронхіальної астми хворий з допомогою інгалятора вдихнув препарат, який в вигляді інгаляцій використовується як бронхолитик, а в ін'єкціях - в акушерській практиці для попередження викиднів. Який препарат був використаний хворим?

Адреналін

Фенотерол

Еуфілін

Ефедрин

Сальбутамол

98 / 6307
На електронній мікрофотографії біопсійного матеріалу представлено легке недоношеної дитини. Виявлено спадання стінки альвеол з - за відсутності сурфактанту. Порушення функції яких клітин стінки альвеоли обумовлюють дану картину?

Альвеолоцитами II типу

Секр еторних клітин

Фібробластів

альвеолоцитами I типу

Альвеолярних макрофагів

99 / 6307
У здорового дорослого людини проводять зондування порожнин серця і великих судин. Де знаходиться зонд, якщо на протязі серцевого циклу зареєстровані зміни тиску від 0 до 120 мм рт. ст?

Легенева артерія

Правий шлуночок

Аорта

Передсердя

Лівий шлуночок

100 / 6307
Пацієнту, який проживає на специфічній геохимической території, поставлений діагноз - ендемічний зоб. Який вид посттрансляционной модифікації тиреоглобуліну порушений в організмі хворого?

Фосфорилування

Йодування

Ацетилювання

Метилирование

Глікозилювання

101 / 6307
У хворого діагностовано гострий інфаркт міокарда, що супроводжується стійкими болями за грудиною. Неефективність раніше введених препаратів дала підставу вр Ачу провести нейролептанальгезию. Який нейролептик треба використовувати?

Аміназин

Дроперидол

Галоперидол

Резерпін

метеразін

102 / 6307
Дитина 5 років поступив в ЛОР - відділення з діагнозом гнійне запалення середнього вуха. Захворювання почалося з запалення носоглотки. Через якийсь канал скроневої кістки інфекція потрапила в барабанну порожнину?

канадець барабанної струни

Барабанний каналець

Сонний канал

Сонно - барабанні канальці

Мишечнотрубний канал

103 / 6307
У дівчинки 5 років спостерігається висока температура і біль в горлі. Об'єктивно: набряк м'якого піднебіння, на мигдалинах сірі плівки, які важко відділяються, залишаючи глибокі кровоточать дефекти тканини. Яке з нижчеперелічених захворювань найбільш ймовірно?

Ангіна Симонівського - Венсана

Інфекційний мононуклеоз

Дифтерія зіва

Некротична ангіна

Лакунарна ангіна

104 / 6307
При дисбактеріозах, що супроводжуються процесом гниття (протея, псевдомонад) і підвищенням рН фекалій, необхідно призначати біологічні препарати, Подкисляющие середу і проявляють антагоністичну дію. Які мікроорганізми для цього підходять?

азотобактер

серрацій

Клебсієли

ентеробактерій

Біфідумбактеріі

105 / 6307
Хворий скаржиться на загальну слабкість, головний біль, нудоту, блювоту, рідкі випорожнення з домішкою слизу і крові. При мікроскопії дуоденального вмісту і при дослідженні свіжих фекалій виявлено рухомі личинки. Поставте діагноз.

Стронгілоїдоз

Трихоцефальоз

Ентеробіоз

дракункульоз

Анкілостомоз

106 / 6307
В клініку потрапив чоловік 54 років з скаргами на болі в правій підреберній області, блювоту з кров'ю. Об'єктивно: збільшення розмірів печінки, варикозне розширення вен стравоходу і шлунка, кровотечі з них. Порушення функції якого судини найімовірніше за все мало місце?

Aorta abdominalis

Vena porta,

Vena hepatica

Vena cava inferior

Vena cava superior

107 / 6307
Хворий помер від інфаркту міокарда. При патогистологическом дослідженні міокарда виявлені значні контрактурной зміни в кардіоміоцитах. Це обумовлено нагромадженням в кардіоміоцитах іонів:

Водню

Кальцію

Натрію

Магнію

Хлора

108 / 6307
Однорічний дитина відстає в розумовому розвитку від своїх однолітків. За вранці: блювота, судоми, втрата свідомості. У крові - гіпоглікемія натще. З дефектом якого ферменту це пов'язано?

глікогенсінтази

аргінази

фосфорилазу

Сахарози

лактози

109 / 6307
З метою аналгезії можуть бути використані речовини, що імітують ефекти морфіну, але продукуються в ЦНС. Вкажіть їх.

Β - ендорфін

Кальцитонин

Вазопрессин

Окситоцин

Соматоліберин

110 / 6307
Хворий звернувся з скаргами на порушення зору, що супроводжується опущеними століття, неможливістю повернути око вгору і до середини. Об'єктивно: очей відведений назовні, зіниця розширена і НЕ реагує на світло, хворий НЕ бачить близько. Який нерв пошкоджений?

Окоруховий

блоковий

Трійчастого

Відвідний

Зоровий

111 / 6307
Через кілька хвилин після введення препарату хворому на правець з'явилася задишка, частий пульс, зниження артеріального тиску. Який препарат міг бути найбільш імовірною причиною виниклого ускладнення?

Антитоксична сироватка

Донорський гамма-глобулін

Анатоксин

Антибіотик

Сульфаніламід

112 / 6307
При розтині померлого, що страждав від атеросклерозу, в головному мозку виявлено тромбоз гілки внутрішньої сонної артерії і сірого кольору вогнище вологого розм'якшення тканин. Який патологічний процес виявлений в головному мозку?

Геморагічна інфільтрація

Ішемічний інфаркт

Енцефаліт

Пухлина мозку

Гематома

113 / 6307
При моделюванні запалення нижньої кінцівки у тварини підвищилася температура тіла, збільшилася зміст антитіл та лейкоцитів у крові. Які речовини зумовили розвиток цих загальних реакцій організму при запаленні?

Мінералокортикоїди

Лейкотрієни

соматомедину

Глюкокортикоїди

Інтерлейкіни

114 / 6307
Потерпілий доставлений в клініку з відкритим переломом гілки нижньої щелепи, з великим кровотечею в області перелому. Пошкодження який артерії найімовірніше за все мало місце?

Осередковою нижньої

Висхідною піднебінної

Мовне

Середній скроневій

Особовий

115 / 6307
У потерпілого - різана рана шиї, розташована уздовж заднього краю m. sternocleidomastoideus, ослаблення чутливості шкіри задньої поверхні вушної раковини. Який нерв може бути пошкодженим?

n. auriculotemporalis

N. auricularis magnus

nn. auriculares anteriores

п. auricularis posterior

г. auricularis nervi vagi

116 / 6307
У хворого з гострим ринітом виявлена гіперемія і підвищене утворення слизу в носовій порожнині. Активність яких клітин епітелію слизової оболонки підвищена?

Мікроворсінчатих

Бокаловидних

Ендокринних

ресничном

Базальних

117 / 6307
Хвора звернулася в травмпункт по приводу нагноєння різаної рани. Лікар для очищення рани від гнійних виділень промив її 3% розчином перекису водню. При цьому піна НЕ утворилася. З чим пов'язано відсутність дії препарату?

Спадкова недостатність фосфатдегідрогенази еритроцитів

Неглибока рана

Низька концентрація Н 202

Спадкова недостатність каталази

Наявність в рані гнійного вмісту

118 / 6307
В результаті виснажливої м'язової роботи у робочого значно зменшилася буферна ємність крові. Надходженням якого кислого речовини в кров можна пояснити це явище?

3- фосфогліцерата

α - кетоглутарата

1,3- бісфосфогліцерата

Лактату

піруват

119 / 6307
Хворий безконтрольно приймав дігоксин, що викликало порушення діяльності серця: зниження скоротливості, брадикардію, шлуночкову екстрасистолію. Яке з перерахованих речовин є препаратом вибору для купірування виникли порушень?

етмозін

Верапаміл

Калію хлорид

Новокаинамид

Анаприлин

120 / 6307
У дитини, яка народилася в пізньому шлюбі, малий зріст, відставання в розумовому розвитку, товстий ' географічний ' язик, вузькі очні щілини, плоске обличчя з широкими вилицями. Яка найбільш ймовірна причина розвитку описаного синдрому?

Внутрішньоутробна інфекція

Родова травма

Спадкова хромосомна патологія

Внутрішньоутробний імунний конфлікт

Внутрішньоутробна інтоксикація

121 / 6307
В приймальне відділення доставлений хворий з скаргами на сухість у роті, світлобоязнь та порушення зо ня. Шкіра гіперемована, суха, зіниці розширені, тахікардія. При подальшому обстеженні був встановлений діагноз: отруєння алкалоїдами беладони. Яке з лікарських засобів доцільно застосувати?

Армін

Діазепам

дипироксим

Прозерін

Пілокарпін

122 / 6307
Хворий 50 років скаржиться на спрагу, п'є багато води, виражена поліурія. Глюкоза крові 4,8 ммоль / л, в сечі глюкози і ацетонових тіл немає, сеча безбарвна, питома вага 1,002-1,004. Яка причина полиурии?

Недолік АДГ

Інсулінова недостатність

Тиреотоксикоз

Гіпотиреоз

Альдостеронізм

123 / 6307
У хворого визначена хворобливість по ходу великих нервових стовбурів і збільшене зміст пірувату в крові. Недолік якого вітаміну може викликати такі зміни?

РР

В2

Пантотенова кислота

Біотин

B1

124 / 6307
Юнак 16 років скаржиться на свербіж між пальцями рук і на животі, який посилюється вночі. При огляді на шкірі були виявлені тоненькі смуги сірого кольору і дрібна висипка. Який найбільш ймовірний збудник цієї хвороби?

Sarcoptes scabiei

Ixodes ricinus

Ixodes persulcatus

Ornithodorus papillipes

Dermacentor pictus

125 / 6307
Проводять дослідження на ізольованій порушуємо клітці. Встановлено, що поріг сили подразнення клітини суттєво зменшився. Що з зазначеного може бути причиною цього?

Активація натрієвих каналів мембрани

Блокада енергоутворення в клітині

Активація калієвих каналів мембрани

Інактивація натрієвих каналів мембрани

Інактивація кальцієвих каналів мембрани

126 / 6307
Пацієнту з гострим інфарктом міокарда внутрішньовенно крапельно введено 1500 мл різних розчинів в протягом 8 годин, кисень інтраназально. Смерть наступила від набряку легенів. Що стало причиною набряку легенів?

Зменшення онкотичного тиску за рахунок гемодилюції

Нейрогенная реакція

Перевантаження лівого шлуночка об'ємом

Інгаляція кисню

Алергічна реакція

127 / 6307
У хворого через 12 годин після гострого нападу загрудинної болю визначено різке підвищення активності АсАТ в сироватці крові. Яка з перерахованих патологій буде найбільш імовірною?

Нецукровий діабет

Вірусний гепатит

Інфаркт міокарда

Цукровий діабет

колагенози

128 / 6307
Хворому з прогресуючою м'язовою дистрофією було проведено біохімічне дослідження сечі. Поява якого речовини в великому кількості в сечі може підтвердити захворювання м'язів у даного хворого?

Гіппурова кислота

Порфірини

Креатин

Сечовина

Креатинін

129 / 6307
У жінки з первинним гіперпаратиреоїдизмом періодично повторюються напади ниркової коліки. Ультразвукове обстеження показало наявність дрібних каменів в нирках. Яка найбільш ймовірна причина освіти цих каменів?

Гиперкальциемия

Гіперхолестеринемія

Гіперкаліємія

Гіперурикемія

Гіперфосфатемія

130 / 6307
У дитини з розумовою відсталістю виявлено зелена забарвлення сечі після додавання 5% розчину FeCl 3. Про порушення обміну якої амінокислоти свідчить позитивний результат цієї діагностичної проби?

Триптофану

Глутаміну

Аргініну

Тирозину

фенілаланін

131 / 6307
При бактеріологічному дослідженні випорожнень чотиримісячної дитини з симптомами гострої кишкової інфекції в середовищі Ендо виросли в великому кількості червоні колонії. Які це можуть бути мікроорганізми?

Ешерихії

Стафілококи

Стрептококи

Сальмонели

Шигели

132 / 6307
Велика частина навчаючи стніков експедиції Магеллана в Америку загинула від захворювання авітамінозом, яке виявлялося загальної слабкістю, підшкірними крововиливами, випаданнями зубів, кровотечами з ясен. Вкажіть назву цього авітамінозу.

Анемія Бирмера

Поліневрит (бері - бері)

Пелагра

Рахіт

Скорбут (цинга).

133 / 6307
У 40- річного чоловіка з стенозирующим (без метастазів) раком стравоходу виявляються наступні зміни: атрофія скелетних м'язів, жирової клітковини. Шкіра землисто - коричневої забарвлення, епідерміс витончений, серце зменшено по розмірам. Міокард і печінку бурого кольору. Яка найбільш ймовірна причина такого стану?

Хвороба Аддісона

Ракова кахексія

Алиментарная кахексія

Міастенія

Бура атрофія

134 / 6307
При дослідженні каріотипу у пацієнта були виявлені два типи клітин в однаковому кількості з хромосомними наборами 46XY і 47XXY. Який найбільш ймовірний діагноз?

Синдром Патау

Синдром Клайнфельтера

Синдром Дауна

Нормальний каріотип

Моносомія - Х

135 / 6307
У чоловіка 44 років, мисливця на ондатр, підвищилася температура тіла до 38 ° С, з'явилися головний біль, набряк повік, гіперемія кон'юнктив, на шкірі в області шиї утворилася неглибока виразка. Вкажіть ймовірного збудника захворювання.

Brucella suis

Yersinia pseudotuberculosis

Francisella tularensis

Leptospira interrogans

Bacillus anthracis

136 / 6307
Чоловікові 35 років з виразковою хворобою зроблено резекцію антрального відділу шлунка. Секреція якого гастроінтестинального гормону внаслідок операції буде порушена найбільш за все?

Сек ретин

Гістамін

Нейротензин

Холецистокінін

Гастрін

137 / 6307
Під час розтину в верхній частці правого легкого виявлений великий клиноподібний осередок темно - червоною щільною тканини. Під час гістологічного дослідження в ній виявлений некроз стінок альвеол, просвіт альвеол щільно заповнений еритроцитами. Який процес розвинувся в легенях?

Крововилив в легені

карніфікація легких

Гангрена легенів

Геморагічний інфаркт легень

Ателектаз легенів

138 / 6307
Хворий 2 роки тому переніс операцію резекції пілоричного відділу шлунка. Спостерігається слабкість, періодичне поява темних кіл під очима, задишка. В аналізі крові: НЬ - 70 г / л, ер. - 3,0 * 1012 / л, ц. п. - 0,7. Які зміни еритроцитів в мазках крові найбільш характерні для даного стану?

Овалоціти

Мак РОЦИТ

мегалоціти

шізоціти

мікроцітов

139 / 6307
При обстеженні чоловіка 45 років, який перебував тривалий час на рослинній дієті, виявлено негативний азотистий баланс. Яка особливість раціону стала причиною цього явища? - Недостатнє кількість жирів і білків

Надмірне кількість води

Недостатнє кількість жирів

Надмірне кількість вуглеводів

Недостатнє кількість білків

140 / 6307
Куди потрібно провести катетер для забору лімфи з грудного лімфатичного протока?

В лівий венозний кут

У правий венозний кут

У ліву пахову вену

У верхню порожнисту вену

В нижню порожнисту вену

141 / 6307
У хворого 27 років виявлені патологічні зміни печінки і головного мозку. У плазмі крові виявлено різке зниження, а в сечі збільшення вмісту міді. Поставлено діагноз - хвороба Вільсона. Активність якого ферменту в сироватці крові необхідно дослідити для підтвердження діагнозу?

Карбоангідрази

ксантиноксидазу

Церулоплазміну

лейцінамінопептідази

Алкогольдегідрогенази

142 / 6307
У собаки в експериментальних НТЕ дратували на шиї периферичний відрізок блукаючого нерва, при цьому спостерігали такі зміни серцевої діяльності:

Збільшення атріовентрикулярного проведення

Збільшення сили скорочень

Збільшення збудливості міокарда

Зменшення частоти скорочень

Збільшення частоти і сили скорочень

143 / 6307
Хворому виконана трансплантація рогівки. Які особливості будови рогівки дозволяють сподіватися на її приживлення, а НЕ відторгнення?

Надмірна іннервація

Відсутність кровоносних і типових лімфатичних судин

Наявність сполучної тканини

Наявність одношарового плоского епітелію

Наявність багатошарового переднього епітелію

144 / 6307
У хворої на бронхіальну астму вірусне інфікування спровокувало астматичний статус зі смертельним результатом. Під час г істологіческого дослідження легких виявлено спазм і набряк бронхіол, в їх стінках виражена інфільтрація лімфоцитами, еозинофілами і іншими лейкоцитами, а також дегрануляция лаброцитов. Який механізм гіперчутливості лежить в основі описаних змін?

Реагінового реакція

Запальний

Іммуннообусловленний клітинний цитоліз

Аутоімунний

Імунокомплексний

145 / 6307
У жінки напередодні пологів ШОЕ 40 мм / год. Така величина ШОЕ обумовлена тим, що в крові підвищений вміст:

Еритроцитів

Білків

ліпопротеїнів

Фібриногену

альбумін

146 / 6307
У хворого 42 років при дослідженні периферичної крові виявлено: гемоглобін 80 г / л, еритроцитів 3,2 * 1012 / л, лейкоцитів 25 * 109 / л ; лейкоцитарна формула: базофілів - 5%, еозинофілів - 9%, мієлобластів - 3%, промиелоцитов - 8%; нейтрофілів: мієлоцитів -11%, метамиелоцитов - 22%, паличкоядерних - 17%, сегментоядерних - 19%, лімфоцитів - 3%, моноцитів - 3%. Яка патологія крові найбільш імовірна у хворого?

панміелофтіз

Хронічний мієлолейкоз

Мієлобластний лейкоз

промієлоцитарному лейкоз

Еритромієлоз

147 / 6307
У хворого 17 років інтраопераційно на нижній поверхні печінки виявлена пухлина з розмірами 4,5x5,0x3,5 см з субсерозной локалізацією, темно - червоного кольору, на розрізі представлена поло ня зі значним вмістом крові. Який попередній діагноз?

Лимфангиома

Гемангіоперицитома

Кавернозная гемангіома

Гемангіоендотеліома

Капілярна гемангіома

148 / 6307
Після травми передньої поверхні верхньої третини передпліччя у хворого спостерігається утруднення пронации, ослаблення ладонного згинання кисті і порушення чутливості шкіри 1-3 пальців на долоні. Який нерв пошкоджений?

П. medianus

п. radialis

п. musculocutaneus

п. cutaneus antebrachii medialis

п. ulnaris

149 / 6307
У жінки 63 років є ознаки ревматоїдного артриту. Підвищення рівня якого з нижчеперелічених показників крові буде найбільш значущим для підтвердження діагнозу?

Сумарних гликозаминогликанов

ліпопротеїди

Спільного холестерину

R-глікозідази

Кислого фосфатази

150 / 6307
На аутопсії померлої, яка тривалий час хворіла на цистит і дискінезією сечоводів, виявлені морфологічні ознаки уремії. Нирка була нерівномірно рубцево - зморщена. В просвіті мисок - дрібні уратні камені і пісок. Гістологічно виявлено ' щитовидну нирку ', осередки інтерстіціонного запалення. Який з нижчеперелічених діагнозів є найбільш імовірним?

Первинно зморщена нирка

атеросклеротичні зморщена нирка

амілоїдні зморщена нирка

Гострий пієлонефрит

Хронічний пієлонефрит

151 / 6307
У хворого на глаукому спостерігається підвищення внутрішньоочного тиску при нормальній секреції водянистої вологи циліарним тілом. З пошкодженням яких структур стінки очного яблука пов'язане порушення відтоку рідини з передньої камери?

Заднього епітелію рогівки

Венозного синуса

цилиарном тіла

ресничном м'язи

Судинної оболонки

152 / 6307
Зменшення кровопостачання органів обумовлює розвиток гіпоксії, яка активізує функцію фібробластів. Обсяг яких елементів збільшується в цій ситуації?

Паренхіматозних елементів органу

Лімфатичних судин

Міжклітинних речовини

Судин мікроциркуляторного русла

Нервових елементів

153 / 6307
У хворого на цукровий діабет змінилося значення рН крові і стало рівним 7,3. Визначення компонентів який буферної системи використовується для діагностики розладів кислотно - лужного рівноваги?

фосфатні

Білкової

бікарбонатному

гемоглобінового

Оксигемоглобіновой

154 / 6307
Згідно моделі подвійної спіралі ДНК, запропонованої Уотсоном і Криком, було встановлено, що одна з ланцюгів зберігається при реплікації, а друга синтезується комплементарно першої. Як називається цей спосіб реплікації?

Дисперсний

Консервативний

Аналогічний

напівконсервативним

Ідентичний

155 / 6307
Підліток 15 років скаржиться на брак повітря, загальну слабкість, серцебиття. ЧСС -130 уд./ хв., АТ -100/60 мм рт. ст, на ЕКГ комплекс QRS нормальної форми і тривалості. Число зубців Р і шлуночкових комплексів однакове, зубець Т слі т з зубцем Р. Яка аритмія серця спостерігається у підлітка?

Синусова тахікардія

Синусова екстрасистолія

Мерехтіння передсердь

Тремтіння передсердь

Передсердна пароксизмальна тахікардія

156 / 6307
При мікроскопічному дослідженні пухлини верхньої губи виявлено, що вона побудована з численних щілиноподібних порожнин. Стінки порожнин встелені плоским ендотелієм. Порожнини заповнені рідкою кров'ю і згустками. Поставте діагноз.

Венозна гемангіома

гломуса - ангіома

Гемангіоперицитома

Капілярна гемангіома

Кавернозная гемангіома

157 / 6307
Пацієнт скаржиться на запаморочення, відчуття спраги, ускладнене ковтання, погане бачення близьких предметів. Об'єктивно: часте дихання, розширені зіниці, загальне збудження, балакучість, але мова малозрозумілий. АТ - 110 /70 мм рт. ст., пульс 110 хв. Про передозуванні якого препарату можуть свідчити зазначені симптоми?

Атропіну

Аміназину

Кофеїн

Морфін

Ефедрину

158 / 6307
У дитини, який одужує після кору, розвинулася пневмонія, викликана умовно - патогенними бактеріями. Яка найбільш ймовірна форма цієї інфекції?

Реінфекція

Вторинна інфекція '

Госпітальна інфекція

Суперінфекція

Персистирующая інфекція

159 / 6307
У чоловіка, при житті страждав бронхоектатичної хворобою, пневмосклерозом з вираженими явищами кахексії, на розтині серце зменшене в розмірах, стінки стоншені, в'ялої консистенції, на розрізі тканина бурого кольору. Відкладення якого пігменту спостерігається в міокарді?

Ліпофусцин

Цероїд

Гематоїдин

Ліпохроми

Гемосидерин

160 / 6307
В гастроентерологічне відділення потрапив хворий 57 років з підозрою на синдром Золлінгера - Еллісона, про що свідчило різке збільшення рівня гастрину в сироватці крові. Яке порушення секреторної функції шлунка найбільш імовірно?

Гіпосекреція гіперацидних

Гіпосекреція гіпоацидного

Гиперсекреция гіпоацидного

Гиперсекреция гіперацидних

Ахілія

161 / 6307
В інфекційне відділення лікарні госпіталізовано хворого з діагнозом бактеріальної дизентерії. Лабораторними дослідженнями встановлено, що збудник чутливий до багатьох протимікробних засобів, але у хворого виявлені явища анемії. Який препарат протипоказаний хворому?

Ампіцилін

Ентеросептол

Фталазол

Левоміцетин

Фуразолідон

162 / 6307
Мати виявила у 5 річної дочки на періанальних складках білих ' черв'ячків ',' які викликали у дівчинки свербіж і занепокоєння, і доставила їх в лабораторію. Виявлено білі гельмінти 0,5-1 см завдовжки, ниткоподібної форми з загостреними кінцями, у деяких вони закручені. Який діагноз можна поставити?

Опісторхоз

Дифиллоботриоз

Ентеробіоз

Теніоз

Аскаридоз

163 / 6307
Який механізм тепловіддачі найбільш ефективно спрацьовує при перебуванні людини в умовах 80% вологості повітря і температурі навколишнього середовища +35°С?

Радіація

Теплопровідність

Потовиділення

Конвекція

164 / 6307
У жінки 68 років після інсульту відсутні рухи в верхній і нижній правій кінцівках. Тонус м'язів цих кінцівок і рефлексів у них збільшені. Є патологічні рефлекси. Яка це форма паралічу?

параплена Егія

Геміплегія

моноплегии

Тетраплегія

Дисоціація

165 / 6307
Для гістологічного дослідження доставлена віддалена на операції матка. Під слизової оболонкою визначені численні округлої форми вузли, чітко відокремлені від навколишнього тканини. Мікроскопічно пухлина побудована з пучків гладкої мускулатури, з явищами тканинного атипизма. Який найбільш ймовірний діагноз?

Хоріонепітеліома

Лейоміосаркома

Фіброміоми

Лейоміома

Рак матки

166 / 6307
В інфекційну лікарню поступив хворий з періодично повторюється гарячкою. У препараті крові (товста крапля), пофарбованому по методу Романовського - Гімза, виявлені спіралеподібні мікроорганізми з гострими кінцями синьо - фіолетового кольору. Який збудник виявлено?

висипного тифу

Черевного тифу

Малярії

Поворотного тифу

лептоспіроз

167 / 6307
В травмпункт звернувся чоловік 45 років після побутової травми плеча. Об'єктивно: відсутні функції розгинання, привиди і пронації плеча. Пошкодження який м'язи викликало таке стан?

підлопаткова м'язи

Надосной м'язи

Великий круглої м'язи

Малої круглої м'язи

підоснов м'язи

168 / 6307
Людина отримав електротравму. При цьому струм пройшов через серцевий м'яз. Які небезпечні порушення в роботі серця, що вимагають невідкладного втручання, можуть виникнути в цій ситуації?

Фибрилляция шлуночків

Фібриляція передсердь

Брадикардія

Екстрасистолія

Тахікардія

169 / 6307
В сім'ї росте донька 14 років, у якої спостерігаються деякі відхилення від норми: зріст її нижче, ніж у одноліток, відсутні ознаки статевого дозрівання, шия дуже коротка, плечі широкі, інтелект в нормі. Яке захворювання можна припустити?

Синдром Шерешевського - Тернера

Синдром Едвардса

Синдром Дауна

Синдром Патау

Синдром Клайнфельтера

170 / 6307
В приймальне відділення доставлений чоловік у непритомному зі стоянні. Об'єктивно: на зовнішні подразники хворий НЕ реагує, дихання періодичне по типу Чейн - Стокса, зіниці звужені, зіничний рефлекс відсутній. Було встановлено, що дані симптоми обумовлені використанням морфіну. Призначити антидотную терапію.

апоморфін гідрохлорид

Унітіол

Протаміну сульфат

Налоксон

Кальцію хлорид

171 / 6307
У хворої 43- х років на тлі септичного шоку відзначається тромбоцитопенія, зменшення фібриногену, поява в крові продуктів деградації фібрину, петехіальні крововиливи. Яка найбільш ймовірна причина даних змін?

Екзогенна інтоксикація

Аутоіммунна тромбоцитопенія

Порушення вироблення тромбоцитів

ДВС - синдром

Геморагічний діатез

172 / 6307
У хворого похилого віку спостерігали збільшення і потовщені ення пальців, кистей, стоп, носа і нижньої щелепи. З збільшенням виділення якого гормону пов'язані зазначені порушення?

Соматотропіну

Інсуліну

Паратгормону

адренокортикотропіну

Тиреотропіну

173 / 6307
При огляді 3 річного дитини виявлено порушення звапніння кісток, деформацію хребта і грудної клітини, викривлення нижніх кінцівок. Який препарат є найбільш ефективним в даній ситуації?

Глюкоза

Тіаміну бромід

Адіурекрин

Токоферол

Ергокальциферол

174 / 6307
В гістологічному препараті стінки серця між ендокардит і міокардом визначаються великі клітини зі світлою цитоплазмою і ексцентрично розміщеним ядром. Які це клітини?

Ендокринні клітини

липоцитов

Клітини Пуркіньє

Пейсмейкерови клітини

Скоротливі кардіоміоцити

175 / 6307
Хворий на ішемічну хворобу серця НЕ повідомив лікаря, що у нього бувають напади бронхоспазмов. Лікар призначив препарат, після прийому якого напади стенокардії стали рідше, але почастішали напади бронхоспазму. Вкажіть, який препарат був призначений.

Верапаміл

Дилтиазем

Анаприлин

Нитросорбид

Атенолол

176 / 6307
Хворому проведена субтотальна субфасціальна резекція щитовидної залози. В післяопераційному періоді тривалий час зберігається охриплість голосу. Який нерв пошкоджений в ході операції?

Верхній гортанний нерв

Під'язиковий нерв

Нижньощелепний нерв

Язичний нерв

Поворотний гортанний нерв

177 / 6307
Хворий 45 років госпіталізований в хірургічне відділення з скаргами на раптову гостру біль в надчеревній області. Після обстеження поставлений діагноз: перфоративная (проривна) виразка задньої стінки шлунка. Куди вилилося вміст шлунка в момент перфорації?

У передшлункову сумку

В лівий брижових синус

В печінкову сумку

В сальниковую сумку

У правий брижових синус

178 / 6307
У чоловіка 60 років після крововиливу в головний мозок настав тривалий сон. Пошкодження яких структур найбільш ймовірно призвело до цього стану?

Чорної субстанції

Кори великих півкуль

ядер черепних нервів

гіпокамп

Ретикулярної формації

179 / 6307
В культурі тканин ядерним випромінюванням пошкоджені ядерця ядер. Оновлення яких органел в цитоплазмі клітин стає проблематичним?

ендоплазматичний сітки

Комплексу Гольджі

Рибосом

Мікротрубочок

Лізосом

180 / 6307
При відтворенні артеріальної гіпертензії у собаки через 1 місяць товщина стінки лівого шлуночка збільшилася в 1,7 рази, а обсяг циркулюючої крові НЕ змінився порівняно з вихідними даними. Яка стадія гіпертрофії міокарда спостерігається у тварини?

Декомпенсації

Початкова

Прогресуючого кардіосклерозу

Закінченої гіпертензія рофіі

Аварійна

181 / 6307
В біоптаті бронха хворого, зловживає курінням, в потовщеною слизової оболонки виявлено хронічне запалення і трансформацію одношарового війчастого епітелію в багатошаровий плоский епітелій. Який з процесів найбільш вірогідний?

Плоскоклітинний рак

Лейкоплакия

Гіперплазія епітелію

Метаплазія

Гіпертрофія епітелію

182 / 6307
В препаратах представлені зрізи органів кровотворення і імуногенезу людини, для яких характерне наявність лімфоїдної тканини, яка формує різні структури (лімфатичні вузлики, часточки, тяжі). Визначте, в якому з органів відбувається антігеннезавісімая проліферація і диференціювання лімфоцитів.

гемолімфатичних вузли

Тимус

Селезінка

Лімфатичні вузли

Мигдалик

183 / 6307
При огляді зіву у хворого на ангіну визначається гіперемія слизової оболонки неба, мигдалини збільшені в розмірах, червоні, на їх поверхні помітні дрібні біло - жовті осередки. Який клініко - морфологічний варіант ангіни найбільш імовірний в даному випадку?

Катаральна

Фібринозна

Некротична

Гнійна

Лакунарна

184 / 6307
Хлопчик 13 років скаржиться на загальну слабкість, запаморочення, стомлюваність. Спостерігається відставання в розумовому розвитку. У крові і сечі висока концентрація валина, ізолейцину, лейцину. Сеча специфічного запаху. Який найбільш ймовірний діагноз?

Хвороба Аддісона

Базедова хвороба

Тирозиноз

Хвороба кленового сиропу

Гістидинемія

185 / 6307
В лікарню потрапив хворий з Східної Сибіру з жал Обами на біль в печінці. У фекаліях знайдені яйця до 30 мкм, що нагадують по формі насіння огірків. Який попередній діагноз можна поставити хворому?

Описторхоз

Гіменолепідоз

Дікроцеліоз

Парагонімоз

Тениаринхоз

186 / 6307
Після введення жабі стрихніну вона на мінімальне роздратування відповідає генералізованими судомами. Блокада який структури ЦНС є причиною цього?

Гальмівних синапсів

Клітин Реншоу

адренорецептори

холінорецептори

збуджує синапсів

187 / 6307
піддослідних собаці через зонд в порожнину шлунка ввели 150 мл м'ясного бульйону. Зміст якого з наведених речовин швидко збільшиться в крові тваринного?

Інсулін

Нейротензин

Гастрін

вазоінтестінальний поліпептид

Соматостатін

188 / 6307
Хворий 65 років, який страждав на атеросклероз, госпо алізірован в хірургічне відділення з приводу розлитого гнійного перитоніту. Під час операції діагностовано тромбоз брижових артерій. Яка найбільш ймовірна причина перитоніту?

Геморагічний інфаркт

Ішемічний інфаркт

Стазі

Ішемія компресійна

Ішемія ангіоспастична

189 / 6307
З випорожнень хворого дитини 6 місячного віку, який знаходився на штучному вигодовуванні виділена культура кишкової палички з антигенною структурою 0-111. Який діагноз можна поставити?

холероподібний захворювання

Харчове отруєння

Гастроентерит

коліентеріт

дізентеріеподобние захворювання

190 / 6307
У хворого діагностовано тиреотоксикоз. У крові знайдено антитиреоїдні тіла. Який тип алергічної реакції по Кумбсу і Джелу спостерігається п ри розвитку цього захворювання?

Гіперчутливість сповільненого типу

Цитотоксичний

Стимулюючий

Анафілактичний

Імунокомплексний

191 / 6307
У чоловіка 52 років через 3 роки після операції видалення шлунка зміст еритроцитів у крові становить 2,0 × 10 12 / л, Hb - 85 г / л, до. п. - 1,27. Порушення засвоєння якого вітаміну викликало такі зміни?

A

С

В12

Р

В6

192 / 6307
При пункційної біопсії в трансплантованою нирці виявлена дифузна інфільтрація строми лімфоцитами, плазмоцитами, лімфобластами, плазмобласти, а також некротичний артеріїт. Який патологічний процес розвинувся в трансплантаті?

Ішемічне пошкодження нирки

Тубулонекроз

Пієлонефрит

Гломерулонефрит

Імунне відторгнення

193 / 6307
При визначенні групи крові по системі АВ 0 аггл ютінацію еритроцитів досліджуваної крові викликали стандартні сироватки I і II груп та НЕ викликала III групи. Які агглютіногени містяться в цих еритроцитах?

С

А і В

B

D і C

А

194 / 6307
У людини 70 років швидкість поширення пульсової хвилі виявилася суттєво вище, ніж у 25- річного. Причиною цього є зниження:

Швидкості кровотоку

Частоти серцевих скорочень

Серцевого викиду

Еластичності судинної стінки

Артеріального тиску

195 / 6307
В групі дітей, які їли солодкий соковитий кавун, у двох з'явилися ознаки отруєння: різка слабкість, запаморочення, головний біль, блювота, задишка, тахікардія, синюшність губ, вух, кінчиків пальців. Лабораторний аналіз кавуна показав високу зміст нітратів. Який провідний механізм в патоген Езе отруєння тільки у двох дітей?

Недостатність каталази

Недостатність глутатіон - піроксідази

Недостатність супероксиддисмутази

Недостатність мет -Hb- редуктази

Блокада цитохромоксидази

196 / 6307
У хворого через 9 діб після введення лікувальної сироватки з'явилася кропив'янка, свербіж шкіри, набряк її та слизових оболонок, припухання лімфатичних вузлів. Яке захворювання розвинулося?

Сироваткова хвороба

Феномен Швартцмана

Поліноз

Феномен Овері

Набряк Квінке

197 / 6307
Хворий з інфекційним мононуклеозом на протязі 2 х тижнів приймав глюкокортикостероїдні препарати. Настала ремісія, але у нього виникло загострення хронічного тонзиліту. Результатом якого дії глюкокортикостероїдів є дане ускладнення?

імунодепресивними

протишокова

антиалергічні

антитоксически

Протизапальної

198 / 6307
При травмі у пацієнта 44 років виник розрив сухожиль м'язів лівої долоні, поверхневих кровоносних судин. Після оперативного втручання і видалення більшої частини некротичних зміненої м'язової тканини кровотік був відновлений. За рахунок яких судин?

Aa. metacarpeae palmares

Arcus palmaris profundus

Arcus palmaris superficialis

Аа. digitales palmares communes

Аа. perforantes

199 / 6307
При дослідженні каріотипу 5 річної дівчинки виявлено 46 хромосом. Одна з хромосом 15 й пари довше звичайної, т. до. до неї приєдналася хромосома з 21- ї пари. Який вид мутації має місце у цій дівчинки?

Інверсія

Делеция

Брак

Транслокация

Дуплікація

200 / 6307
У хворого з значними периферійними набряками почергове застосування діхлотіазіда, етакриновою кислоти і фуросеміду НЕ сприяло значному діуретичній ефекту. У крові значне підвищення кількості альдостерону. Вкажіть препарат вибору.

Амілорид

Клопамід

Сечовина

Спіронолактон

Вабить

201 / 6307
Через деякий час після інтенсивної фізичної тренування у спортсмена активується глюконеогенез, основним субстратом якого є:

Глутаминовая кислота

Лактат

альфа - кетоглутарат

Аспарагінова кислота

Серин

202 / 6307
На місці автокатастрофи пораненому водію наклали кровоспинний джгут на верхню третину стегна. У хірургічне відділення хворий був доставлений в задовільному стані тільки через 3 години. Після зняття джгута з'явився різкий набряк тканини стегна, частий пульс, холодний піт, різка гіпотонія . Яке патологічне явище розвинулося у потерпілого?

Анафілактичний шок

Кардіогенний шок

Токсеміческій шок

Колапс

Геморагічний шок

203 / 6307
Хворому з значними опіками зробили пересадку донорської шкіри, але на 8-му добу трансплантат набряк, змінився його колір і на 11-ту добу він почав відторгатися. Які клітини беруть в цьому участь?

В - лімфоцити

Еозинофіли

Базофіли

Еритроцити

Т - лімфоцити

204 / 6307
Хворому для купірування кишкової коліки призначений атропіну сульфат. Яке з названих захворювань може служити протипоказанням в цьому випадку?

Синусова брадикардія

Бронхіальна астма

Глаукома

Запаморочення

Гіпотонія

205 / 6307
Біля інфікованої рани збільшилися регіонарні лімфовузли. При гістологічному дослідженні в них виявлено збільшення кількості макрофагів, лімфоцитів і лімфатичних фолікулів в кірковому шарі, а також велику кількість плазматичних клітин. Який процес в лімфатичних вузлах відображають виявлені гістологічні зміни?

Реакцію гіперчутливості

Пухлинну трансформацію

Антигенну стимуляцію

Уроджену недостатність лімфоїдної тканини

Придбану недостатність лімфоїдної тканини

206 / 6307
У дитини підвищилася температура тіла, з'явилися болі при ковтанні. При огляді: піднебінні мигдалини збільшені, темно - червоні, покриті сіро - жовтими плівками, щільно спаяні з поверхнею мигдаликів. Яке запалення розвинулося в мигдалинах?

Крупозне

Геморагічне

Гнійне

Катаральне

Дифтеритичне

207 / 6307
У пацієнта, які тривалий час хворіє на туберкульоз, виявлено внутрішньоклітинне розташування мікобактерій. Який з вказаних препаратів обов'язково повинен бути включений в комплексну терапію туберкульозу?

Етіонамід

Натрію парааміносаліцілат

Етамбутол

Рифампіцин

Ізоніазид

208 / 6307
Після прийому жирної їжі у хворого з'являються нудота і печія, має місце стеаторея. Причиною такого стану може з'явитися:

Брак амілази

Порушення синтезу трипсину

Брак жовчних кислот

Підвищений виділення ліпази

Порушення синтезу фосфоліпази

209 / 6307
Під час здачі іспиту у студентів ' пересихає в роті '. Посилена реалізація яких рефлексів є механізмом, що обумовлює розвиток цього стану?

Незаперечних парасимпатических

Незаперечних симпатичних

умовних парасимпатических

Умовних симпатичних

Незаперечних периферичних

210 / 6307
Хворому з цирозом печінки разом з лікарськими препаратами внутрішньовенно ввели 500,0 мл 5% розчину глюкози. Яке порушення водного балансу найбільш ймовірно може виникнути у хворого?

Гиперосмолярная гипергидратация

Ізоосмолярна гипергидратация

Гіпоосмолярна гипергидратация

Гіпоосмолярна дегідратація

Порушень водного балансу НЕ відбудеться

211 / 6307
У гістологічному препараті визначається ться рецепторная зона сенсоепітеліального органу почуттів. Клітини даної зони лежать на базальної мембрані і включають такі види: зовнішні і внутрішні сенсорні, зовнішні і внутрішні фалангові, стовпові, зовнішні прикордонні і зовнішні підтримують. Якому органу почуттів належить дана рецепторная зона?

Органу смаку

Органу нюху

Органу зору

Органу рівноваги

Органу слуху

212 / 6307
У чоловіка при ураженні одного з відділів ЦНС спостерігається астенія, м'язова дистонія, порушення рівноваги. Який з відділів ЦНС вражений?

Вестибулярні ядра

Чорна субстанція

Червоні ядра

Ретикулярная формація

Мозочок

213 / 6307
Після введення пирога у хворого підвищилася температура, шкірні покриви стали блідими, холодними на дотик, з'явився озноб, зрілось споживання кисню. Як змінюються процеси терморегуляції в описаному періоді лихоманки?

Знижується тепловіддача

Збільшується теплопродукція

Тепловіддача дорівнює теплопродукції

Знижується теплопродукція

Збільшується тепловіддача

214 / 6307
У травмпункт звернувся чоловік 45 років після побутової травми плеча. Об'єктивно: відсутні функції розгинання, приведення і пронації плеча. Пошкодження який м'язи викликало таке стан?

підлопатковий м'яз

надостьовий м'яз

підостьовий м'яз

малий круглий м'яз

великий круглий м'яз

215 / 6307
У препараті червоного кісткового мозку людини визначаються скупчення гігантських клітин, що знаходяться в тісному контакті з синусоїдними капілярами. Які формені елементи крові утворюються з цих клітин?

Кров'яні пластинки

Моноцити

Лейкоцити

Лімфоцити

Еритроцити

216 / 6307
При гістологічному дослідженні стулки мітрального клапана серця виявлено: мукоїдному набряк, пошкодження ендотелію, утворення по замикаючому краю фібринових тромбів. Яка форма ревматичного ендокардиту спостерігається?

Зворотно - бородавчастий ендокардит

Поліпозно - виразковий ендокардит

Дифузний ендокардит

Фібропластичний ендокардит

Гострий бородавчастий ендокардит

217 / 6307
У дівчинки 5 років спостерігається висока температура і біль в горлі. Об'єктивно: набряк м'якого піднебіння, на мигдалинах сірі плівки, які відділяються з працею, залишаючи глибокі кровоточать, дефекти тканини. Яке з нижчеперелічених захворювань найбільш ймовірно?

Некротична ангіна

Інфекційний мононуклеоз

Дифтерія зіва

Лакунарна ангіна

Ангіна Симон івського - Венсана

218 / 6307
У хворого 70 років діагностовано крововилив у стовбур мозку. Обстеження виявило підвищення тонусу м'язів - згиначів на тлі зниження тонусу мишцразгібателей. Роздратуванням яких структур мозку можна пояснити зміни в тонусі м'язів?

Чорної субстанції

ядер Голя

ядер Бурдаха

Червоних ядер

ядер Дейтерса

219 / 6307
У хворого спостерігається атонія м'язів. Назвіть фермент м'язової тканини, активність якого може бути знижена при такому стані.

Креатинфосфокиназа

транскетолази

Каталаза

Амілаза

7- глутамінтрансферази

220 / 6307
У хворого виявлено різке зниження активності сурфактанту легень. Які зміни слід очікувати у цього хворого?

Розростання сполучної тканини легенів

Зміна еластичних властивостей легенів

Схильність альвеол до спадання і неможливість їх швидкого розправлення

Зменшення трахеобронхіального секрету

Порушення кровообігу в легенях

221 / 6307
52- річну пацієнтку на протязі останніх декількох днів турбують напади болю в правому підребер'ї після прийому жирної їжі. Візуально визначається пожовтіння склер і шкіри, ахолічний кал, сеча ' кольору пива? Присутність якого речовини в сечі пацієнтки зумовило темний колір сечі при обтураційній жовтяниці?

білірубінглюкуроніди

уробилин

стеркобіліна

Кетонових тел

Глюкози

222 / 6307
На обмеженому ділянці епідермісу внаслідок травми відсутні шари аж до росткового. Які клітини послужать джерелом його регенерації?

Шари крилатих і зернистих клітин незруйнованим ділянки

Шари базальних клітин

Шари крилатих і базальних клітин

Шар базальних клітин незруйнованим ділянки

Шари крилатих клітин

223 / 6307
У хворого спостерігається сльозотеча, підвищене слиновиділення. Це стан, спільно з іншими симптомами, расц енівается як роздратування волокон одного з черепних нервів. Якого саме нерва і яких його волокон? - Соматичні рухові волокна лицьового нерва

Парасимпатические волокна лицьового нерва

Парасимпатические волокна блукаючого нерва

Соматичні рухові волокна окорухового нерва

Парасимпатические волокна окорухового нерва

224 / 6307
У хворого з Тімом (пухлина вилочкової залози) спостерігається ціаноз, розширення підшкірної венозної сітки і набряк м'яких тканин обличчя, шиї, верхньої половини тулуба і верхніх кінцівок. Який венозний стовбур перетиснено пухлиною?

Передня яремна вена

Зовнішня яремна вена

Верхня порожниста вена

Підключичної вена

Внутрішня яремна вена

225 / 6307
Під час розтину в верхній частці правого легкого про назовні великий клиновидний вогнище темно - червоною щільною тканини. Під час гістологічного дослідження в ній виявлений некроз стінок альвеол ; просвіт альвеол щільно заповнений еритроцитами. Який процес розвинувся в легенях?

Геморагічний інфаркт легень

Гангрена легенів

карніфікація легких

Ателектаз легенів

Крововилив в легені

226 / 6307
При яких групах крові батьків по системі резус можлива резус - конфліктна ситуація під час вагітності?

Дружина Rh +, чоловік Rh + (гомозигота)

Дружина Rh-, чоловік Rh-

Дружина Rh + (гетерозигота), чоловік Rh + (гомозигота)

Дружина Rh-, чоловік Rh + (гомозигота)

Дружина Rh +, чоловік Rh + (гетерозигота)

227 / 6307
Мукополісахарідоз відноситься до хвороб накопичення. З - за відсутності ферментів порушується розщеплення полісахаридів. У хворих спостерігається вище ня виділення їх з сечею і накопичення в одному з органоїдів клітин. В яких органелах накопичуються мукополісахариди?

В ендоплазматичному ретикулуме

У клітинному центрі

У мітохондріях

В лізосомах

У комплексі Гольджі

228 / 6307
З метою аналгезії можуть бути використані речовини, що імітують ефекти морфіну, але виробляються в ЦНС. Вкажіть таке речовина.

Кальцитонин

Окситоцин

бетаендорфін

Соматоліберин

Вазопрессин

229 / 6307
У хворого, які тривалий час кращого тютюн, розвинувся рак легені. Які з перерахованих канцерогенних речовин містяться в тютюновому димі і відносяться до ПАУ (поліненасичених ароматичним вуглеводів)?

диметиламіноазобензолу

Діетілнітрозамін

бета - нафтиламин

Ортоаміноазотолуол

Бензпирен

230 / 6307
У померлого 58 років на розтині мітральний клапан деформований, потовщений, змикається НЕ до кінця. Мікроскопічно: вогнища колагенових волоконец еозинофільно, дають плюсову реакцію на фібрин. Найбільш ймовірно це:

Мукоїдне набухання

Амилоидоз

Гіаліноз

Фібринозне запалений ие

Фібриноїдне набухання

231 / 6307
Хворий 45 років госпіталізований в хірургічне відділення з скаргами на раптову гостру біль в надчеревній області. Після обстеження поставлений діагноз: перфоративная (проривна) виразка задньої стінки шлунка. Куди вилилося вміст шлунка в момент перфорації?

В лівий брижових синус

В сальниковую сумку

У передшлункову сумку

В печінкову сумку

У правий брижових синус

232 / 6307
При попаданні чужорідного тіла в дихальні шляхи на який бронх має бути направлено увагу лікаря в першу чергу для пошуку стороннього тіла і його видалення?

Ліві часткові бронхи

Лівий головний бронх

Ліві сегментарні бронхи

Праві часткові бронхи

Правий головний бронх

233 / 6307
Смерть хворого 16 років настала від розлитого (поширеного) фібрінозногнойного перитоніту. На розтині в нижньому ділянці тонкої кишки виявлена виразка, що повторює форму пеєрових бляшки, з перфорацією стінки кишки. Мікроскопічна дослідження виявило невизначеність малюнка лімфоїдної тканини, вите снення її проліферуючими моноцитами, які формують гранульоми. Ускладнення якого захворювання стало причиною смерті?

дизентерія

неспецифічні виразкового коліту

черевного тифу

бруцельоз

холери

234 / 6307
Після перенесеного запального захворювання у хворого виникло неповне відведення очного яблука в латеральну сторону. Який нерв пошкоджений у хворого?

Окоруховий

Особовий

Зоровий

Бічний

Відвідний

235 / 6307
У чоловіка 52 років діагностовано системний амебіаз з ураженням кишечника, печінки, легенів. Який препарат слід призначити?

Метронідазол

Ентеросептол

Хиниофон

Хінгамін

Тетрациклін

236 / 6307
У крові хворого виявлено низький рівень альбумінів і фібриногену. Зниження активності яких органел гепатоцитів печінки наиб олее ймовірно обумовлює це явище?

Гранулярності ендоплазматичної мережі

Лізосом

гранулярной ендоплазматичної мережі

Комплексу Гольджі

Мітохондрій

237 / 6307
У хворого виявлено множинні синці на тілі, тривалість кровотечі по Дуке 25 хвилин, число тромбоцитів крові 25 * 109 / л. Для якого захворювання характерні такі ознаки?

Гемофілія В

Гемофілія А

Авітаміноз С

Спадковий дефект освіти тромбоцитів

Хвороба Віллебранда

238 / 6307
У хворого жовтяницею отримані такі дані лабораторного обстеження: HbsAg-, HBeAs-, анти HBsG +, анти HBsM-, HCAg +. Який діагноз підтверджується на основі цих даних?

Гепатит С

Гепатит С, в анамнезі - гепатит

Рецидив гепатиту В

Хронічний гепатит В з низькою реплікативної активністю

П овторное зараження гепатитом В

239 / 6307
Хворому з бактеріальною інфекцією лікар призначив сульфадимезин в таблетках, порадивши запивати його 1,5-2 л лужної мінеральної води щодоби. Чим обумовлена необхідність даної рекомендації?

Для нейтралізації кислоти шлункового соку

Для зниження дратівної впливу на шлунок

Для зсуву рН крові в лужну сторону

Для пролонгування дії

Для профілактики кристалізації ацетильних похідних препарату в ниркових канальцях

240 / 6307
У хворого 30 років з дизентерією, підтвердженої бактеріологічно, з'явилися ознаки парапроктиту. Про який стадії місцевих змін найбільш ймовірно йде мова у даного хворого?

Стадія загоєння виразок

Фолікулярний коліт

Катаральний коліт

Стадія освіти виразок

Фіб рінозний коліт

241 / 6307
У клінічно здорових людей в умовах високогір'я виявляються ознаки анемії. При аналізі крові виявляють серповидні клітини. Визначте генотип цих людей:

Aa

ХсХс

аа

АА

ХСХс

242 / 6307
При визначенні групи крові по системі АВ 0 агглютинацию еритроцитів досліджуваної крові викликали стандартні сироватки Г і II груп і НЕ викликала сироватка III групи. Які агглютіногени містяться в цих еритроцитах?

С

В

А

А і В

D і С

243 / 6307
З випорожнень хворого дитини 6 місячного віку, який знаходився на штучному вигодовуванні, виділена культура кишкової палички з антигенною структурою 0-111. Який діагноз можна поставити?

Коли - ентерит

холероподібний захворювання

Харчове отруєння

Гастроентерит

дізентеріеподобние захворювання

244 / 6307
Малюк попросив Вас надути повітряний кульку як можна сильніше за один видих. Яким обсягом повітря Ви скористаєтеся?

Ємністю вдиху

Життєвої ємністю легких

Резервним об'ємом вдиху

Загальною ємністю легких

Функціональної залишкової ємністю

245 / 6307
Під час хірургічної операції пацієнту проведено переливання крові. На антигени якого збудника необхідно перевірити цю кров?

Вірусу гепатиту В

Вірусу гепатиту Е

Вірусу гепатиту А

аденовіруси

ентеровірус

246 / 6307
У 6 місячного дитини спостерігалися часті і сильні підшкірні кровотечі. Призначення синтетичного аналога вітаміну К (вікасолу) дало позитивний ефект. У гамма - карбоксилювання глутамінової кислоти якого білка системи згортання крові приймає участь цей вітамін?

Фібриногену

Фактора Розенталя

протромбіну

антигемофільний глобуліну А

Фактора Хагемана

247 / 6307
Хворому з гіперсекрецією шлункового соку лікар рекомендував виключити з дієти насичені бульйони і овочеві відвари, тому що вони стимулюють шлункову секрецію. Який переважний механізм стимуляції шлункової секреції в даному випадку?

Роздратування смакових рецепторів

Роздратування механорецепторів шлунка

Стимуляція вироблення гастрит ина G-клітинами

Стимуляція вироблення секретину в 12-палої кишці

Роздратування механорецепторів порожнини рота

248 / 6307
Яка частина товстої кишки найбільш ймовірно буде пошкоджена при пораненні в праву половину живота?

Сигмовидная ободова кишка

Пряма кишка

Поперечна ободова кишка

Низхідна ободова кишка

Висхідна ободова кишка

249 / 6307
Дитина 7 років НЕ може відвести плече і підняти його до горизонтального рівня, до особі руку приводить лише тильною стороною при деякій абдукції плеча (за рахунок надостной м'язи) - рука ' сурмача '. Активна функція якої м'язи відсутній?

Великий круглої

Великий грудної

Малої круглої

подостной

дельтоподібними

250 / 6307
З нітратів, нітритів і нітрозамінів в організмі утворюється азотистая кислота, яка обумовлює окисне дезамінування азотистих про снованіі нуклеотидів. Це може привести до точкової мутації - заміни цитозину на:

Тимин

Урацил

Инозин

Аденін

гуанін

251 / 6307
Мати виявила у 5 річної дочки на періанальних складках білих ' черв'ячків ',' які викликали у неї свербіння і занепокоєння, і доставила їх в лабораторію. При огляді лікар побачив білих гельмінтів 0,5-1 см завдовжки, ниткоподібної форми з загостреними кінцями, у деяких вони закручені. Який діагноз можна поставити?

Аскаридоз

Описторхоз

Теніоз

Ентеробіоз

Дифиллоботриоз

252 / 6307
У людини діагностовано галактоземія - хвороба накопичення. Внаслідок порушення якої клітинної структури виникла ця хвороба?

Мітохондрій

Клітинного центру

Комплексу Гольджі

Центросоми

Лізосом

253 / 6307
При дисбактеріозах, що супроводжуються процесом гниття (протей, псевдомонади) і підвищенням рН фекалій, необхідно призначати біологічні препарати, Подкисляющие середу і проявляють антагоністичну дію. Які мікроорганізми для цього підходять?

Клебсієли

Біфідумбактеріі

ентеробактерій

азотобактер

серрацій

254 / 6307
Після введення жабі стрихніну вона на мінімальне роздратування відповідає генералізованими судомами. Блокада який структури ЦНС є причиною цього?

Клітин Реншоу

холінорецептори

Гальмівних синапсів

збуджує синапсів

адренорецептори

255 / 6307
Хворому з підозрою на черевний тиф лікар - інфекціоніст призначив бактеріологічне дослідження крові. Доцільність цього призначення пояснюється тим, що в першу тиждень захворювання тифо - паратифами спостерігається:

Вірусемія

Септицемія

Септикопіємія

токсінеміі

Бактериемия

256 / 6307
У жінки 32 років з безсимптомним перебігом хвороби у другій раз народився мертвий дитина з вираженою мікроцефалією. Про якому захворюванні в першу чергу слід подумати лікаря?

Сифіліс

Токсоплазмоз

Гістоплазмоз

Бруцельоз

Лістеріоз

257 / 6307
Для попередження нападу бронхіальної астми лікар призначив хворому кромолин натрію. Який з наведених механізмів характерний для цього кошти?

Зниження концентрації імуноглобулінів

Зв'язування вільного гістаміну

Інактивація гістаміну

Блокада гістамінових рецепторів

Стабілізація мембран огрядних кліток

258 / 6307
Дівчинка 10 років часто хворіє на гострі респіраторні інфекції, після яких спостерігаються множинні точкові крововиливи я в місцях тертя одягу. Гіповітаміноз якого вітаміну має місце у дівчинки?

B1

B2

В 6

A

C

259 / 6307
Людина отримав електротравму. При цьому струм пройшов через серцевий м'яз. Які небезпечні порушення в роботі серця, що вимагають невідкладного втручання, можуть виникнути в цій ситуації?

Екстрасистолія

Брадикардія

Фібриляція передсердь

Тахікардія

Фибрилляция шлуночків

260 / 6307
З допомогою шпателя зроблений зішкріб слизової рота людини. У незруйнованих епітеліальних клітинах пофарбованого мазка добре видно овальні ядра, неоднакові по розмірах. Яким шляхом відбувалося поділ цих клітин?

Митоз

Шизогонія

Бінарне розподіл

Мейоз

Амитоз

261 / 6307
Під час ЕКГ - дослідження хворий виявлено періодичне поява шлуночкової екстрасистоли. При цьому встановлено, що перед екстрасистолою відсутня зубець Е. Яка найбільш ймовірна причина його зникнення? - Виникнення рефрактерного періоду в шлуночках

Блокада проведення імпульсу по передсердях

Неможливість ретроградного проведення через AV-вузол

Блокада імпульсу в синусовому у злі

Виникнення рефрактерного періоду в передсерді

262 / 6307
54- річний чоловік скаржиться на відсутність чутливості шкіри нижнього століття, зовнішньої латеральної поверхні носа, верхньої губи. Лікар при огляді встановлює запалення другої гілки трійчастого нерва. Через якийсь отвір виходить з черепа ця гілка?

рвані отвір

Овальний отвір

Кругле отвір

Верхня глазничная щілину

Остистий отвір

263 / 6307
Хворий з діагнозом вогнищевого туберкульозу верхньої частки правого легкого отримує ізоніазид в складі комбінованої терапії. Через деякий час пацієнт почав скаржитися на м'язову слабкість, зниження чутливості шкіри, порушення зору, координації рухів. Який вітамінний пре парат доцільно використовувати для усунення цих явищ?

Вітамін D

Вітамін В12

Вітамін В6

Вітамін А

Вітамін С

264 / 6307
Хворий скаржиться на диспептичні розлади, мелену, гемороїдальні кровотечі. При дослідженні виявлено розширення сітки венозних судин на передній стінці живота в комплексі з збільшенням його розмірів. Яка патологія шлунково-кишкового тракту може проявитися такими симптомами?

Ентерит

Коліт

Портальная гіпертензія

Кишкова аутоинтоксикация

Виразкова хвороба

265 / 6307
У хворого з гіпертонічною хворобою при систематичному лікуванні антигіпертензивним засобом з'явився кашель. Яке з названих засобів може бути причиною даного побічного ефекту?

Клофелін

Еналаприл

Діхлотіазід

Празозин

Верапаміл

266 / 6307
У експерименті досліджували поріг сили подразнення клітин різних тканин. В яких з наступних клітин він виявиться найнижчими?

В мотонейронах спинного мозку

У залізистих клітинах

В кардіоміоцитах

У міоцитах гладкких м'язів

У міоцитах скелетних м'язів

267 / 6307
У хворого, які тривалий час брав глюкокортикоїди, в результаті відміни препарату виникло загострення наявного захворювання, зниження артеріального тиску, слабкість. Чим можна пояснити ці явища?

гіперпродукція АКТГ

Виникненням недостатності надниркових залоз

сенсибілізація

звикання до препарату

кумуляції

268 / 6307
У психіатричну клініку доставлений хворий 40 років в стані збудження, агресії, марення. Який препарат слід ввести хворому?

Настоянку валеріани

Аміназин

Резерпін

Седуксен (діазепам)

Натрію бромід

269 / 6307
Хворий госпіталізований з скаргами на блювоту, запаморочення, двоїння в очах, утруднене ковтання. Лікар запідозрив ботулізм. Які методи діагностики доцільно використовувати для підтвердження діагнозу?

Біологічну пробу, бактеріологічний

Алергійну пробу, серологічний

Протозоологіческій, мікроскопічний

Бактеріологічний, мікологічний

270 / 6307
У хворого нефритом виявлена глюкозурія і аминоацидурия. Порушення якого механізму реабсорбції глюкози і амінокислот є причиною цього?

фагоцитозу

Первинного активного транспорту

Простий дифузії

піноцитоз

Вторинного Na+ - залежного транспорту

271 / 6307
У хворого 27 років виявлені патологічні зміни печінки і головного мозку. У плазмі крові виявлено різке зниження, а в сечі збільшення вмісту міді. Поставлено діагноз - хвороба Вільсона. Активність якого ферменту в сироватці крові необхідно дослідити для підтвердження діагнозу?

Церулоплазміну

Алкогольдегідрогенази

ксантиноксидазу

Карбоангідрази

лейцінамінопептідази

272 / 6307
У юнака під час фізичної навантаження хвилинне споживання кисню і хвилинне виділення вуглекислого газу дорівнюють 1000 мл. Які субстрати окислюються в клітинах його організму?

Білки

Жири

Вуглеводи

Вуглеводи і білки

Вуглеводи і жири

273 / 6307
У гастроентерологічне відділення потрапив хворий 57 років з підозрою на синдром Золінгера - Еллісона, про що свідчило різке збільшення рівня гастрину в сироватці крові. Яке порушення секреторної функції шлунка найбільш імовірно?

Гіпосекреція гіпоацидного

Гіпосекреція гіперацидних

Ахілія

Гиперсекреция гіперацидних

Гиперсекреция гіпоацидного

274 / 6307
Дитина 5 років поступив в ЛОР - відділення з діагнозом 'гнійне запалення середнього вуха.' Захворювання почалося з запалення носоглотки. Через якийсь канал скроневої кістки інфекція потрапила в барабанну порожнину?

Мишечнотрубний канал

канадець барабанної струни

Сонно - барабанні канальці

Сонний канал

Барабанний каналець

275 / 6307
При видаленні зуба для знеболювання використовують розчин новокаїну. З якої метою його вводять НЕ в ясна біля зуба, а в область проходження чутливого нерва?

Блокується проведення больових імпульсів

Порушується формування потенціалу дії больових рецепторів

Підвищується збудливість больових рецепторів

Пригноблюється аксональний транспорт

Змінюється рН тканин в області анестезії

276 / 6307
У хворого сухим плевритом вислуховується шум тертя плеври. При ураженні якого виду епітелію відзначається цей симптом?

Перехідний епітелій

Одношаровий плоский епітелій

Одношаровий призматичний епітелій

Одношаровий кубічний епітелій

багатошарове ий епітелій

277 / 6307
метильних групи (-СН3) використовуються в організмі для синтезу таких важливих сполук як креатин, холін, адреналін і ін. Яка з незамінних амінокислот, є джерелом цих груп є одна?

Триптофан

Метіонін

Ізолейцин

Лейцин

Валін

278 / 6307
При розтині померлого дитини, який страждав діареєю, виявлено ексікоз і розлитої фібринозний коліт. В мазку - відбитку слизової оболонки виявлені грамнегативні палички. Який найбільш ймовірний діагноз?

Стафилококковая кишкова інфекція

Холера

Сальмонельоз

Черевний тиф

Дизентерія

279 / 6307
Поступив з їжею глікоген Гідролізований в шлунково - кишковому тракті. Який кінцевий продукт утворився в результаті цього процесу?

Фруктоза

Лактат

Лактоза

Галактоза

Глюкоза

280 / 6307
При обстеженні хворого виявлено новоутворення в білому речовині півкуль великого мозку з локалізацією в коліні і передньому відділі задньої ніжки внутрішньої капсули. Волокна якого провідного шляху мозку будуть зруйновані?

Tr. thalamocorticalis

Tr. pyramidalis

Tr. frontothalamicus

Tr. frontopontinus

Tr. parietooccipitopontinus

281 / 6307
У людини внаслідок довільної затримки дихання на 40 с зросли частота серцевих скорочень і системне артеріальний тиск. Реалізація будь механізмів регуляції зумовила зміна показників?

Умовні парасимпатичні рефлекси -

Безумовні парасимпатичні рефлекси

Умовні симпатичні рефлекси

Безумовні симпатичні рефлекси

282 / 6307
На протязі двох тижнів хвора приймала мікстуру, прописану невропатологом по приводу неврастенії. Самопочуття хворої декілька покращилось, однак незабаром з'явилися скарги на нежить, кон'юнктивіт, шкірну висип, млявість і ослаблення пам'яті. Був встановлений діагноз ' бромізм '. Що доцільно призначити для послаблення симптомів?

Полиглюкин

Розчин глюкози 5%

Аспаркам

Натрію хлорид

283 / 6307
Хворий НЕ може підняти опущену вниз нижню щелепу. Які м'язи НЕ можуть виконати свої функції?

Кругова м'яз рота

Жувальні м'язи

Мімічні м'язи

надчерепной м'яз

М'яз, що піднімає кут рота

284 / 6307
До лікаря звернувся хворий з пошкодженої стопою, забинтованою брудною марлевою пов'язкою, просоченої гнійними виділеннями. Спроба зняти пов'язку для огляду і обробки рани викликає гостру біль, так як вона прилипла до рани поверхні. Який антисептик полегшить видалення пов'язки і очистить рану від бруду і гною?

Етакрідін

Перекис водню

Етоній

Калію перманганат

Фурацилин

285 / 6307
Хворому з непереносимістю антибіотиків для лікування пневмонії призначений сульфален. Через кілька днів у хворого розвинувся гемоліз еритроцитів. Недостатність якого ферменту в організмі хворого сприяла розвитку цього побічного ефекту?

Глюкозо-6-фосфатдегідрогенази

ацетальдегіддегідрогенази

холінестерази

Урідіндіфосфатглюкуроновой трансферази

N-ацетилтрансферази

286 / 6307
До лікаря звернувся хворий з скаргами на постійну спрагу. Виявлено гіперглікемія, поліурія та підвищений змісту 17- кетостероїдів в сечі. Яке захворювання найбільш ймовірно?

Глікогеноз I типу

Інсулінозалежний діабет

Стероїдний діабет

Аддисонова хвороба

Мікседема

287 / 6307
При рентгенологічному дослідженні кісток основи черепа виявлено збільшення порожнини турецького сідла, витончення передніх похилих відростків, руйнування різних ділянок турецького сідла. Пухлина який ендокринної залози може викликати таке руйнування кісток?

Епіфіза

щитовидної залози

вилочкова залози

Гіпофіза

Наднирників

288 / 6307
Людина стоїть в кімнаті в легкій одязі, температура повітря +14°С. Вікна та двері закриті. Яким шляхом він віддає більше всього тепла?

Конвекція

перспірація

Теплорадіація

Теплопроведение

Випаровування

289 / 6307
Клінічні дослідження крові рекомендується проводити натще і вранці. Які зміни компонентів крові можливі, якщо провести забір крові після прийому їжі?

Зниження числа еритроцитів

Зниження числа тромбоцитів

Збільшення числа лейкоцитів

Збільшення білків плазми

Збільшення числа еритроцитів

290 / 6307
Після фізичної навантаження підвищився артеріальний тиск крові. Чому?

Збільшилася кількість гемоглобіну

Збільшилася зміст води в плазмі крові

Збільшився хвилин ний обсяг кровообігу

Збільшилася кількість функціонуючих капілярів

Збільшилася зміст іонів калію в плазмі крові

291 / 6307
У хворого з запаленням легенів непереносимість антибіотиків. Який з комбінованих сульфаніламідних препаратів слід призначити хворому?

Сульфацил натрію

Етазол

Сульфадиметоксин

Бісептол

Стрептоцид

292 / 6307
Тривале перебування в умовах спеки викликало у людини спрагу. Сигналізація від яких рецепторів, в першу чергу, зумовила її розвиток?

барорецепторами дуги аорти

натрієві рецепторів гіпоталамуса

осморецептори печінки

Осморецептори гіпоталамуса

глюкорецептори гіпоталамуса

293 / 6307
У юнака 20 років травмовано праве яєчко. Яку небезпеку це становить для лівого (здорового) яєчка?

Розвиток гіпертрофії

Чи не загрожує нічим

Демаскування антигену і виникнення пошкодження антитілами

Розвиток инфекц іонного процесу

Розвиток атрофії

294 / 6307
При обстеженні хворого визначається наявність гіперглікемії, кетонурія, поліурії, гиперстенурия і глюкозурії. Яка форма порушення кислотно - лужного балансу має місце в даній ситуації?

Газовий алкалоз

Метаболічний алкалоз

Метаболічний ацидоз

Газовий ацидоз

Негазовий алкалоз

295 / 6307
У хворого 45 років на правій нозі спостерігається блідість шкіри гомілки і стопи, визначається відсутність пульсації тильної артерії стопи та задньої великогомілкової артерії. Пульсація стегнової артерії збережена. Поранення який артерії сталося?

Зовнішньої клубової

Підколінної

Низхідній колінної - Малогомілкової

Глибокої артерії стегна

296 / 6307
Чоловікові 70 років, яка страждає хронічним бронхітом, призначений противокашлевой препарат - кодеїн. Який механізм забезпечує противокашлевой ефект?

Центральний

Рефлекторний

Периферичний дію

Місцеве дію

Конкурентний

297 / 6307
Людина сидів у кріслі з закритими очима, коли задзвонив телефон. Як зміниться у нього ЕЕГ?

Збільшиться тета - ритм

Збільшиться альфа - ритм

Тета - ритм зміниться на альфа - ритм

Альфа - ритм зміниться на бета - ритм

Бета - ритм зміниться на тета - ритм

298 / 6307
У приймальне відділення доставлений хворий у непритомному стані. Шкіра холодна, зіниці звужені, дихання з утрудненням, відзначається періодичність по типу Чейна - Стокса, артеріальний тиск знижений, сечовий міхур переповнений. Отруєння яким речовиною най більш ймовірно?

транквілізатори

Наркотичними анальгетиками

ненаркотичних анальгетиками

М-холиноблокаторами

299 / 6307
У хворого гострим мієлобластний лейкоз виявлено: збільшення печінки, селезінки, анемія, мієлобласти в периферичної крові. Який принциповий ознака дозволяє відрізнити гострий мієлобластний лейкоз від хронічного?

Владні клітини в периферичної крові

Анемія

Панцитопенія

Лейкемічний провал

Тромбоцитопенія

300 / 6307
При дослідженні крові хворого виявлено значне збільшення активності МВ - форм КФК (креатинфосфокінази) та ЛДГ-1. Яка найбільш ймовірна патологія?

Холецистит

Ревматизм

Гепатит

Панкреатит

Інфаркт міокарда

301 / 6307
Сульфаніламідні препарати нагадують по структурі парааминобензойную кислоту. В ніж з остоит молекулярна основа їх фармакологічного ефекту?

В порушення синтезу вітаміну

В руйнуванні клітинної мембрани

В активації ліполізу

У ингибировании гліколізу

У зв'язуванні з ДНК

302 / 6307
На прийом до лікаря прийшов пацієнт дуже високого зростання, з довгими товстими пальцями рук, великою нижньою щелепою і відвислою нижньою губою. Підвищену секрецію якого гормону який залози можна підозрювати?

антидіуретичного гормону задньої частки гіпофіза

Гормонів надниркової залози з групи глюкокортикоїдів

гонадотропні гормону передньої частки гіпофіза

Соматотропного гормону передньої частки гіпофіза

Гормонів щитовидної залози

303 / 6307
Хворий 37 років, що страждає на облітеруючий ендартеріїтом судин нижніх кінцівок, отримує фенилин в добовій дозі 60 мг / кг. У зв'язку з проявами судомного синдрому (в анамнезі ЧМТ) призначений фенобарбітал, після відміни якого у хворого виникло носове кровотеча. Дане ускладнення пов'язане з:

аліфатичні гідроксилюванням фенобарбіталу

Гальмуванням фенобарбіталом мікросомального окислення в печінці

окислювальний дезаминированием фенилина

Індукцією феноба рбіталом ферментів мікросомального окислення в печінці

кон'югації фенилина з глюкуроновою кислотою

304 / 6307
У хворого 30 років діагностовано ішемічна хвороба серця. Напади стенокардії турбують вже 3 роки. Виявлено спадковий характер захворювання. Який вид гиперлипопротеидемии найбільш ймовірно буде виявлено у хворого?

III (дісбеталіпопротеідемія).

I (гіперхіломікронемія).

II (гіпербеталіпопротеідемія).

V (гіперпребеталіпопротеідемія і гіперхіломікронемія).

IV (гіперпребеталіпопротеідемія).

305 / 6307
Жінка 53 років, зріст 163 см, вага 92 кг, рівномірне відкладення жиру, особа одутле, малорухлива, апатична. При натисканні шкіри ноги залишається ямка. Порушенням функції якої залози обумовлено стан хворої?

Полових

надниркові

Щитоподібної

Паращитовидних

Гіпофіза

306 / 6307
В приймальне відділення доставлений дитина 1,5 років з ознаками отруєння нітратами: стійкий ціаноз, задишка, судоми. Який пат огенетіческій механізм лежить в основі цих симптомів?

Освіта карбогемоглобін

Освіта метгемоглобіну

Освіта карбоксигемоглобина

Освіта скороченої гемоглобіну

Освіта оксигемоглобина

307 / 6307
Під час обіду дитина поперхнувся і аспірованої їжу. Почався сильний кашель, шкіра і слизові ціанотичні, пульс прискорений, дихання рідкісне, видих подовжений. Яке порушення зовнішнього дихання розвинулося у дитини?

Дихання Біота

Стенотичне дихання

альтернирующая дихання

Стадія инспираторной задишки при асфіксії

Стадія експіраторной задишки при асфіксії

308 / 6307
У померлого від малярії виражена жовтушність шкіри, склер та слизових оболонок. При розтині: селезінка збільшена в розмірі, аспидно - сірого кольору. Аспидно - сіра забарвлення селезінки обуслов лена наявністю:

гемомеланін

Гемопорфіріна

гемосидерину

Меланіну

ліпофусцин

309 / 6307
Хворому з маніакально - депресивним синдромом в стадії депресії, скаржилися на відчуття тривоги, страху, був призначений антидепресант з супутнім псіхоседатівним ефектом. Який це був препарат?

Ніаламід

інказан

Имизин (іміпрамін)

Амитриптилин

Сіднофен

310 / 6307
Який препарат слід призначити хворому, у якого з - за висипу алергічного характеру з почервонінням, набряком, сильним свербінням з'явилася безсоння?

Хлоралгідрат

Фенобарбітал

Натрію оксибутират

Нітразепам

Димедрол

311 / 6307
З метою перевірки крові донорів на наявність антигенів гепатиту B необхідно застосовувати високочутливі методи. Яку з названих реакцій слід застосує ь з вказаною метою?

Іммуноелектрофорез

Твердофазний імуноферментний аналіз

Реакцію непрямої гемаглютинації

Реакцію зв'язування комплементу

Реакцію непрямої імунофлуоресценції

312 / 6307
У пацієнта після травми виникли паралічі, розлади больової чутливості справа ; зліва паралічі відсутні, але порушена больова і температурна чутливість. Яка причина такого явища?

Пошкодження мозочка

Пошкодження рухової зони кори головного мозку

Пошкодження стовбура мозку

Одностороннє ураження спинного мозку з правої сторони

Пошкодження середнього мозку

313 / 6307
При обстеженні хворого встановлено діагноз - кліщовий поворотний тиф. Яким шляхом міг заразитися хворий?

Через укус селищного кліща

Через укус собачого кліща

Через укус малярійного комара

Через укус москіта

Через укус тайгового кліща

314 / 6307
В шкірі виявлена щільна, рухома, чітко відмежована від оточуючих тканин пухлина. У розрізі вона білого кольору, представлена волокнистою тканиною. Мікроскопічно - хаотично переплетені колагенові волокна, клітин мало. Яка це пухлина?

Дерматофіброма

Тверда фіброма

М'яка фіброма

гістіоцитома

Десмоїд

315 / 6307
В печінці хворого порушена детоксикація природних метаболітів і ксенобіотиків. Активність якого цитохрому може бути знижена?

цитохроми

Цитохроми Р-450

Гемоглобіну

цитохромоксидази

цитохроми З-1

316 / 6307
У хворого тромбоз коронарних судин. Необхідно введення речовини, що відновлює їх прохідність. Яке з пропонованих речовин про ладает необхідними властивостями?

Стрептокиназа

Фенилин

Гепарин

Ацетилсаліцилова кислота

Пентоксифілін

317 / 6307
При обстеженні хворого з гемофілією виявлено зміна деяких показників крові. Який з перерахованих ознак відповідає цьому захворюванню?

Тромбоцитопенія

афібриногенемією

Еритроцитоз

Час згортання крові загальмовано

Еозинофілія

318 / 6307
У хворого діагностовано гострий інфаркт міокарда, що супроводжується стійкими болями за грудиною. Неефективність раніше введених препаратів дала підставу лікарю провести нейролептанальгезию. Який нейролептик треба використовувати?

Галоперидол

Резерпін

Аміназин

Дроперидол

Метаперазін

319 / 6307
Відомо, що вірус імунодефіциту людини належить до сімейства ретровірусів. Вкажіть осно вной ознака, що характеризує дане сімейство.

Наявність мінус - РНК

Наявність ферменту зворотної транскриптази

Виявлення антигенів реакцією імуноферментного аналізу

Відсутність інтеграції нуклеїнової кислоти в геном господаря

320 / 6307
При мікроскопічному дослідженні нефробіоптата виявлено наявність півмісяців більш ніж в 50% клубочків, капілярні петлі некротизованих, в просвіті їх виявлено фібринові тромби, виражений тубуло - інтерстиціальний компонент. Про якому захворюванні нирок слід думати?

Амилоидоз

Швидкопрогресуючий гломерулонефрит

Хронічний гломерулонефрит

Ліпоїдний нефроз

Некротический нефроз

321 / 6307
Морфологічні дослідження селезінки виявили активізацію імунних реакцій в організмі. В яких структурах даного органу починається антигензависимая проліферація Т - лімфоцитів?

Червона пульпа

Центральна зона білої пульпи

Періартеріального зона білої пульпи

Маргінальна зона білої пульпи

Мантійна зона білої пульпи

322 / 6307
При обстеженні хворого з травматичним пошкодженням головного мозку виявлено, що він перестав розрізняти переміщення предмета по шкірі. Який відділ кори мозку пошкоджений?

Тім'яна частка кори

Задня центральна звивина

Лобова частка кори

Передня центральна звивина

Потилична частка кори

323 / 6307
Після внебольничного аборту у жінки прогресував гнійний ендоміометрит зі смертельним результатом. При розтині померлої виявлені численні абсцеси легких, субкапсулярні гнійники в нирках, гіперплазія селезінки. Яка форма сепсису виникла у хворої?

уросепсисі

Легеневий сепсис

З ептікопіемія

Септицемія

хроніосепсису

324 / 6307
Хвора 45 років скаржиться на задишку при невеликій фізичної навантаженні, набряки на балках, в анамнезі часті ангіни, хворіє на протязі двох років. Діагностовано міокардит, комбінований мітральний порок серця, недостатність кровообігу. Який гемодинамічний механізм декомпенсації серця у хворої?

Підвищення артеріального тиску

Зменшення обсягу циркулюючої крові

Зниження венозного тиску

Тахікардія

Зменшення хвилинного обсягу серця

325 / 6307
У дитини 7 років на шкірі розгинальних поверхонь ліктьових і колінних суглобів з'явилися щільні, безболісні вузлики розміром 1-2 мм. У біоптаті вузликів - великий осередок фибриноидного некрозу сполучної тканини з лімфоцитами і макрофагами по периферії. При якому захворюванні спостерігаються такі вузлики?

Системна червона вовчак

Вузликовий періартеріїт

Ревматизм

Ревматоїдний артрит

Склеродермия

326 / 6307
Для лікування урогенітальних інфекцій використовують хінолони - інгібітори ферменту ДНК - гірази. Який процес порушується під впливом хінолонів в першу чергу?

Ампліфікація генів

Репарація

Реплікація

Рекомбінація генів

Зворотній транскрипція

327 / 6307
При мікроскопічному дослідженні тканини печінки було виявлено, що деякі клітини розпалися на невеликі фрагменти з окремими органелами і залишками ядра, оточені мембраною. Запальна реакція відсутня. Для якого патологічного процесу характерні ці зміни?

Апоптоз

Некроз

Пла зморексіс

Плазмоліз

каріорексисом

328 / 6307
У хворого має місце позаклітинний набряк тканин (збільшені розміри м'яких тканин кінцівок, печінки і т. д.). Зменшення якого параметра гомеостазу є найбільш вірогідною причиною розвитку набряку?

Осмотичного тиску плазми крові

Онкотичного тиску плазми крові

в'язкості

гематокриту

329 / 6307
При обстеженні юнака з розумовою відсталістю виявлено евнухоідний будова тіла, недорозвиненість статевих органів. У клітинах порожнини рота - статевий хроматин. Який метод генетичного дослідження слід використовувати для уточнення діагнозу?

дерматогліфіки

Популяційно-статистичний

Клініко-генеалогічний

Цитологічний

Біохімічний

330 / 6307
У людини зареєстрована електрокардіограма зі зниженою амплітудою зубця R. Що означає цей зубець на ЕКГ?

Поширення збудження по шлуночках

Поширення збудження від передсердь до шлуночків

Електричну систолу серця

Поширення збудження по передсердям

Електричну діастолу серця

331 / 6307
У жінки 68 років після інсульту відсутні рухи в верхній і нижній правих кінцівках. Тонус м'язів цих кінцівок і рефлекси в них підвищені. Є патологічні рефлекси. Яка це форма паралічу?

Геміплегія

Тетраплегія

моноплегии

Дисоціація

Параплегія

332 / 6307
У хворого 17 років інтраопераційно на нижній поверхні печінки виявлена пухлина розмірами 4,5x5,0x3,5 см з субсерозной локалізацією, темно - червоного кольору, на розрізі представлена порожнинами зі значним вмістом крові. До акой найбільш ймовірний попередній діагноз?

Капілярна гемангіома

Гемангіоендотеліома

Лимфангиома

Кавернозная гемангіома

Гемангіоперицитома

333 / 6307
У хворого при гастроскопії виявлено недостатнє кількість слизу, що покриває слизову оболонку. З порушенням функції яких клітин стінки шлунка це пов'язано?

B- клітин призматичного залозистого епітелію

ендокріноціти

шеечную клітин

Парієтальних клітин залоз шлунка

Головних екзокриноцитів

334 / 6307
Особам, які бажають схуднути, рекомендують включати в харчовий раціон побільше нежирної яловичини. Якими властивостями білків це пояснюється?

Низькою калорійністю

Наявністю найбільшого специфічно-динамічного дії

Швидким насиченням

Поганим всмоктуванням

Тривалою затримкою в шлунку

335 / 6307
Чоловік чину 65 років, що страждає на подагру, скаржиться на болі в області нирок. При ультразвуковому дослідженні встановлено наявність ниркових каменів. В результаті якого процесу утворюються ниркові камені?

Розпаду пуринових нуклеотидів

орнітінового циклу

Відновлення цистеїну

катаболізм білків

Розпаду гема

336 / 6307
У хворого з діагнозом ' злоякісний карціноід ' різко підвищено вміст серотоніну в крові. З якої амінокислоти може утворитися даний біогенний амін?

Метіоніну

треоніну

Триптофану

аланін

лейцин

337 / 6307
При електронномікроскопіческом дослідженні гіалінового хряща виявляються клітини з добре розвиненою гранулярной ендоплазматичної мережею, комплексом Гольджі. Яку функцію виконують ці клітини?

Руйнування межкл еточного речовини хряща

Освіта міжклітинної речовини

Депонування глікогену

Депонування жиру

Трофіка хрящової тканини

338 / 6307
У хворого на шкірі обличчя поступово розвинулася бляшка з некрозом і виразкою в центрі. При патогистологическом дослідженні біоптату виявлено розростання атипових епітеліальних клітин з великою кількістю патологічних мітозів. Який найбільш ймовірний діагноз?

Саркома

Трофическая виразка

Фиброма

Рак шкіри

Папілома

339 / 6307
У хворого міастенією після призначення прозерину з'явилася нудота, діарея, посмикування м'язів мови і скелетних м'язів. Чим можна купірувати інтоксикацію?

ізадрину

фізостигмін

метацин

мезатон

Пиридостигмина бромидом

340 / 6307
У хворого внутрішньосуглобове перелом шийки стегнової кістки. Спостерігається асиметрія головки. До акая структура буде найбільш ймовірно пошкоджена?

Стегновий нерв

Запірательний нерв

Зв'язка головки стегнової кістки

запирательной артерія

Круговий пояс

341 / 6307
У хворого інфаркт міокарда в області передньої стінки лівого шлуночка. У басейні якої артерії виникло порушення кровообігу?

Предсердно - шлуночкових гілок лівої вінцевої артерії

Огинає гілки лівої вінцевої артерії

Передніх шлуночкових гілок правої вінцевої артерії

Передньої міжшлуночкової гілки лівої вінцевої артерії

Лівою крайової гілки лівої вінцевої артерії

342 / 6307
Хворий, що страждав хронічним гнійним остеомієлітом, помер від хронічної ниркової недостатності. При розтині виявлено великі щільні нирки беложелтого кольору з сальним блиском на зрізі. Який найбільш веро ятний діагноз?

Хронічний гломерулонефрит

Амилоидоз нирок

Гострий некротичний нефроз

Підгострий гломерулонефрит

Септичний нефрит

343 / 6307
До лікаря звернулися пацієнти з подібними скаргами: слабість, болі в кишечнику, розлад шлунково-кишкового тракту. Після дослідження фекалій з'ясувалося, що термінової госпіталізації підлягає один з пацієнтів, у якого виявлено цисти з чотирма ядрами. Для якого найпростішого характерні такі цисти?

Кишкова амеба

Тріхомонада

Лямблії

Дизентерийная амеба

балантидії

344 / 6307
У хворого після важкої травми розвинувся шок і з'явилися ознаки гострої ниркової недостатності (ОПН). Що є провідним механізмом розвитку гострої ниркової недостатності в даному випадку?

Підвищення тиску в капсулі клубочка

Підвищення тиску в ниркових артеріях

Зниження онкотичного тиску крові

Порушення відтоку сечі

Падіння артеріального тиску

345 / 6307
У немовляти відзначається блювота і пронос, загальна дистрофія, гепато- і спленомегалія. При припинення вигодовування молоком симптоми зменшуються. Який основний наследст венний дефект буде відзначатися в патогенезі?

Гиперсекреция залоз зовнішньої секреції

Порушення обміну тирозину

Недостатність глюкозо-6-фосфатдегідрогенази

Порушення обміну галактози

Порушення обміну фенілаланіну

346 / 6307
З хімічного виробництва в токсикологічне відділення доставлений хворий з отруєнням ртуттю. Який препарат слід використати в даній ситуації?

Налоксон

ізонітрозину

Унітіол

Активоване вугілля

Ентеросорбент СКН

347 / 6307
У хворого в 2 рази збільшена щитовидна залоза. При пальпації заліза щільна, поверхня нерівномірна, горбиста. При гістологічному дослідженні - дифузна інфільтрація тканин залози лімфоцитами, плазматичними клітинами з утворенням фолікулів і посилене розростання сполучної тканини. Який най більш ймовірний діагноз?

Ендемічний зоб

Зоб Ріделя

Зоб Хасімото

Спорадичний зоб

Дифузний токсичний зоб

348 / 6307
Дитина 10 років переніс кілька атак ревматизму. При його клінічному обстеженні було встановлено, що мали місце запальні явища в суглобах, і виявилися ознаки недостатності мітрального клапана. Яке з патологічних явищ у даного хворого можна віднести до поняття 'хвороба'?

Порок мітрального клапана

Недостатність мітрального клапана

Артрит

Запалення суглобів

Ревматизм

349 / 6307
У хворого діагностовано мегалобластная анемія. Недостатнє кількість якого речовини може призводити до розвитку цієї хвороби?

холекальциферол

ціанокобаламін

Меди

Гліцину

Магнію

350 / 6307
При дослідженні крові у хворого в иявлена виражена гіпоглюкоземія натщесерце. При дослідженні біоптату печінки виявилося, що в клітинах печінки НЕ відбувається синтез глікогену. Недостатність якого ферменту є причиною захворювання?

Фруктозодіфосфатази

глікогенсинтетазу

альдолаза

фосфорилазу

піруваткарбоксілази

351 / 6307
При розтині жінки 40 років, яка страждала ревматоїдним артритом, виявлено збільшену щільну селезінку. На розрізі її тканина коричнево - червоного кольору з збільшеними фолікулами, які мають вигляд напівпрозорих сірувато - білуватих зерен. Який патологічний процес найбільш вірогідний?

Гіаліноз селезінки

глазурного селезінка

Сальна селезінка

Сагова селезінка

порфірний селезінка

352 / 6307
В лабораторії досліджують мокроту хворого на туберкульоз. Який метод пофарбовані ания слід використовувати для виявлення збудників туберкульозу?

Гімза - Романовського

Циля - Нільсена

Бурри - Гінса

Нейссера

Грама - Синьова

353 / 6307
У новонародженого діагностовано порушення розвитку міокарда шлуночка. З порушенням розвитку якого ембріонального джерела пов'язана ця патологія?

парієтальної спланхноплеври

ентодерми

ектодерма

мезенхіми

Вісцеральної спланхноплеври

354 / 6307
У хворого запальний гнійний процес шкіри першого межпальцевого проміжку ноги. Які лімфатичні вузли є регіональними для зазначеного ділянки і відреагують болем і припухлістю?

Передні великогомілкової

Поверхневі пахові

Зовнішні клубові

Задні великогомілкової і підколінні

Поверхневі і глибокі пахові

355 / 6307
Аспірин надає проти вовоспалітельное дію, так як пригнічує активність циклооксигенази. Рівень будь біологічно активних речовин буде при цьому знижуватися?

йодтироніни

Катехоламінів

простагландинів

лейкотрієнів

біогенні амінів

356 / 6307
З гнійної рани хворого виділений патогенний стафілокок і визначена чутливість його до антибіотиків: пеніцилін - зона затримки росту 8 мм ; оксацилін - 9 мм ; ампіцилін - 10 мм ; гентаміцин - 22 мм ; линкомицин - 11 мм. Який антибіотик необхідно вибрати для лікування хворого?

Ампіцилін

Пеніцилін

Линкомицин

Оксациллин

Гентамицин

357 / 6307
В медико - генетичну консультацію звернулася жінка з приводу ризику захворювання на гемофілію у свого сина. Її чоловік страждає цим захворюванням з народження. Жінка і її батьки здорові в відношенні гемофілії. Яка ймовірність появи хвороби у хлопчика в даній родині?

Все хлопчики будуть хворими

75% хлопчиків будуть хворими

25% хлопчиків будуть хворими

Усі хлопчики будуть здоровими

50% хлопчиків будуть хворими

358 / 6307
Хворий 55 років спостерігається у ендокринолога з приводу порушення ендокринної функції підшлункової залози, що проявляється зменшенням кількості гормону глюкагону в крові. Функція яких клітин цієї залози порушена в цьому випадку?

А-клітини острівців Лангерганса

S-клітини острівців Лангерганса

B2-клітини острівців Лангерганса

бета-клітини острівців Лангерганса

РР-клітини острівців Лангерганса

359 / 6307
У чоловіка 42 років, який страждає на подагру, в крові підвищена концентрація сечової кислоти. Для зниження рівня сечової кислот и йому призначений аллопуринол. Конкурентною інгібітором якого ферменту є алопуринол?

Гуаніндезамінази

аденозіндезамінази

Аденінфосфорібозілтрансферази

Гіпоксантінфосфорібозілтрансферази

ксантиноксидазу

360 / 6307
У молодого людини в м'яких тканинах лівого стегна з'явилось безболісне новоутворення без чітких меж. У біоптаті тканини новоутворення нагадують риб'яче м'ясо, складаються з незрілих фібробластоподібних клітин з численними митозами, проростають в м'язи. Який найбільш ймовірний діагноз?

Міома

Фібросаркома

Рак

Фиброма

міосаркома

361 / 6307
У хворого 24 років через півтори тижні після важкої стрептококової ангіни проявилася набряклість обличчя, підвищився артеріальний тиск. Гематурія і протеїнурія 1,2 г / л. У крові виявлено антистрептококових анти тіла і зниження компонентів комплементу. У микрососудах яких структур найбільш імовірна локалізація скупчень імунних комплексів, що зумовили розвиток нефропатії?

Петля Генле

Клубочки

Проксимальний відділ канальців

Піраміди

Спадний відділ канальців

362 / 6307
У чоловіка 36 років з черепно - мозкової травмою дихання слабке, пульс ниткоподібний, рефлекси відсутні. Який шлях введення пірацетаму найбільш доцільний в даному випадку?

Інгаляційний

Пероральний

Підшкірний

Внутрішньовенний

Ректальний

363 / 6307
При мікроскопічному дослідженні біоптату шийки матки у хворої на хронічний ендоцервікоз виявлена клітинна і ядерна атипія багатошарового плоского епітелію, патологічні мітози, а також рогові перлини в глибині епітеліальних шарів. Який наиболе е ймовірний діагноз?

Плоскоклітинний рак без зроговіння

Перехідно рак

Плоскоклітинний рак з зроговінням

Анапластичний рак

Залозистий рак

364 / 6307
У хворого виник спазм гладенької мускулатури бронхів. Фізіологічно обгрунтованим буде dикористання для зняття нападу активаторів:

α- і β-адренорецепторів

Н-холінорецепторів

М-холінорецепторів

Β-адренорецепторів

α-адренорецепторів

365 / 6307
При дослідженні каріотипу у пацієнта були виявлені два типи клітин в однаковому кількості з хромосомними наборами 46XY і 47XXY. Який найбільш ймовірний діагноз?

Синдром Дауна

Нормальний каріотип

Синдром Патау

Моносомія - Х

Синдром Клайнфельтера

366 / 6307
При всіх формах розмноження (статеве і нестатеве розмноження) елементарними дискретними єдиний іцамі спадковості є:

Одна пара нуклеотидів

Дві ланцюжка молекули ДНК

Один ген

Одна ланцюжок молекули ДНК

Один нуклеотид

367 / 6307
В хірургічний кабінет звернувся чоловік, якого покусала невідома собака. Широкі рвані рани локалізовані на обличчі. Яку лікувально - профілактичну допомогу потрібно надати для профілактики сказу?

Терміново ввести нормальний гамма - глобулін

Почати імунізацію антирабічною вакциною

Госпіталізувати хворого і містити під наглядом лікаря

Призначити комбіновану антибіотикотерапію

Терміново ввести вакцину АКДС

368 / 6307
При аналізі крові у хворого залишковий азот склав 48 ммоль / л, сечовина -15,3 ммоль / л. Про захворюванні якого органу свідчать результати цього аналізу?

Кишечника

Нирок

селезінки

Шлунка

Печінки

369 / 6307
У жінки 62 років розвинулася катаракта (помутніння кришталика) на тлі цукрового діабету. Посилення якого процесу при діабеті є причиною помутніння кришталика?

Глікозіт ілірованія білків

кетогенез

протеолізу білків

ліполіз

глюконеогенез

370 / 6307
Терапія анаприліном позитивно вплинула на динаміку хвороби у жінки 44 років, яка страждає стенокардією. Який головний механізм дії цього препарату?

Зменшення окислювального обміну в міокарді внаслідок блокади ферментів циклу Кребса

Зниження потреби і збільшення надходження кисню в міокард

Блокада β - адреноблокатори і зниження потреби міокарда в кисні

Зменшення енерговитрат міокарда внаслідок зниження навантаження

Збільшення надходження кисню в міокард

371 / 6307
У людини внаслідок тривалого голодування швидкість клубочкової фільтрації зросла на 20%. Що є найбільш вірогідною причиною зміни фільтрації в зазначених умовах?

Зниження онкотичного тиску плазми крові

Підвищення коефіцієнта фільтрації

Підвищення ниркового плазмотока

Вище ня проникності ниркового фільтру

Підвищення системного артеріального тиску

372 / 6307
В яких клітинах на протязі життя НЕ спостерігається мітоз, і кількісне зміст ДНК залишається постійним?

В м'язових (гладких)

В кровотворних

У епідермісі

В м'язових (поперечно - смугастих)

В нейронах

373 / 6307
У хворої 55 років виявлено збільшення розмірів гіпофіза, гіперплазія кори надниркових залоз. АТ - 190/90 мм рт. ст.; зміст глюкози в крові - 20 ммоль / л, має місце глюкозурія, ожиріння, гірсутизм. Для якої патології характерні виявлені зміни?

Адіпозогенітальная дистрофія

Хвороба Аддісона

Хвороба Барракера - Симмондса

Хвороба Іценко - Кушинга

Синдром Іценко - Кушинга

374 / 6307
При аутопсії померлого на шкірі дистальних відділів нижніх кінцівок виявлено багряно - червоні плями, бляшки і вузли (саркома Капоши). Виявлено також гостра пневмонія, викликана пневмоцистами. Для якого захворювання характерні дані зміни?

Сибірська виразка

СНІД

Грип

Дифтерія

Кір

375 / 6307
У жінки 45 років в період цвітіння трав з'явилося гостре запальне захворювання верхніх дихальних шляхів і очей: гіперемія, набряк, слизові виділення. Який вид лейкоцитозу буде найбільш характерним при цьому?

Моноцитоз

Базофілія

Еозинофілія

Нейтрофілія

Лимфоцитоз

376 / 6307
У жінки виявлено пухлину яєчника. Показана операція. Яку зв'язку повинен перерізати хірург, щоб відділити яєчник від матки?

Круглу зв'язку матки

Бічну пупкову зв'язку

Власну зв'язку яєчника

Широку зв'язку матки

Зв'язку, підвішуючу яєчник

377 / 6307
У хворого в анамнезі відзначений тривалий зростання кінцівок, подовжені ' павукові ' пальці, дефекти кришталика ока, аномалії серцево - судинної системи. Інтелект в нормі. Які ф енотіпіческіе ознаки ще можуть бути у цього хворого:

Недорозвинення нижньої щелепи

Плоске обличчя і широке перенісся

Порушення розвитку сполучної тканини

Недорозвинення гонад

Розщеплення м'якого та твердого піднебіння

378 / 6307
У хворого з головним болем, високою температурою, ознобом, кашлем з мокротиння виділили палички овоидной форми з біполярної забарвленням, грамнегативні. В мазку з бульонной культури розташовуються ланцюжками, на агарі утворюють колонії R- форми. Для якого захворювання це характерно?

Туберкульоз

Дифтерія

Стрептококова ангіна

Менінгококовий назофарингіт

Чума

379 / 6307
При гістологічному дослідженні стінки аорти виявлено велику кількість ксантомних клітин, розташованих переважно в інтимі. При якому захворюванні можлива така картина?

нодозной периартериит

Атеросклероз

Сифілітичний мезаортит

Гіпертонічна хвороба

Неспецифічний аортоартеріїт

380 / 6307
В приймально - діагностичне відділення доставлена жінка 38 років з маточним кровотечею. Які найбільш ймовірні змінений ія з боку крові відбудуться?

Лейкопенія

Лейкоцитоз

Зменшення гематокритного числа

Збільшення гематокритного числа

Еритроцитоз

381 / 6307
У хворого камінь загального жовчного протока припинив надходження жовчі в кишечник. Порушення якого процесу травлення при цьому спостерігається?

Переваривание білків

Всмоктування білків

Переваривание жирів

Переваривание вуглеводів

Всмоктування вуглеводів

382 / 6307
В лікарню доставили хворого на цукровий діабет в несвідомому стані. Дихання типу Кусмауля, артерії ально тиск 80/50 мм рт. ст., запах ацетону з рота. Накопиченням в організмі яких речовин можна пояснити виникнення даних розладів?

Модифікованих ліпопротеїдів

Вугільної кислоти

Кетонових тел

Молочної кислоти

Складних вуглеводів

383 / 6307
В відділення травматології доставлений хворий з розчавлених м'язової тканини. Який біохімічний показник сечі при цьому буде збільшений?

Глюкоза

Загальні ліпіди

Мінеральні солі

Сечова кислота

Креатинін

384 / 6307
У хворого на туберкульоз після тривалого лікування з'явилися шум і дзвін у вухах, зниження слуху, висип на шкірі, набряк слизових оболонок і порушилася координація рухів. Після скасування препарату стан хворого значно покращився. Який препарат приймав хворий?

Рифампіцин

Бепаск

Ізоніазид

Стрептомицина сульфат

Етамбутол

385 / 6307
В лікарню швидкої допомоги доставили дитину 7 років в стані алергічного шоку, що розвинувся після укусу оси. У крові підвищена концентрація гістаміну. В результаті якої реакції утворюється цей амін?

декарбоксилюванню

Відновлення

Гідроокісленія

дегидрированием

Дезамінування

386 / 6307
У дитини, хворої на дифтерію, через 10 днів після введення антитоксичної протидифтерійної сироватки з'явилися висипання на шкірі, які супроводжувалися сильним свербінням, підвищилася температура тіла до 38 ° С, з'явилися болі в суглобах. Яка причина цих явищ?

Сироваткова хвороба

Гіперчутливість сповільненого типу

Атопія

Анафілактична реакція

Контактна алергія

387 / 6307
У пацієнта, який півтора місяці тому назад переніс інфаркт міокарда, діагностований синдром Дресслера з характерною тріадою: перикардит, плеврит, пневмонія. Причиною його розвитку вважається:

Активація сапрофітної мікрофлори

Зниження резистентності до інфекційних агентів

Інтоксикація організму продуктами некрозу

Викид в кров міокардіальних ферментів

Сенсибілізація організму антигенами міокарда

388 / 6307
Під час емоційного збудження частота серцевих скорочень у людини 30 років досягла 112 в хв. Який відділ проводить системи серця відповідає за це зміна?

Синоатріальна вузол

Пучок Гіса

Волокна Пуркіньє

Ніжки пучка Гіса

Атріовентрикулярний вузол

389 / 6307
Хворий госпіталізований з гострою серцево - судинної недостатністю. Який препарат необхідно використовувати в даному випадку?

Дигитоксин

Адреналін

Строфантин

Дигоксин в таблетках

Аміодарон

390 / 6307
Гризуни є резервуаром збудників багатьох хвороб. З чим це пов'язано в першу чергу?

Біологічні особливості гризунів, що сприяють обміну паразитами і збудниками з людиною

Властивість гризунів швидко розмножуватися

Приналежність гризунів до найбільш численному ряду класу ссавців

Приналежність гризунів до важливим компонентам наземних біоцен озов

Проживання в умовах, де ектопаразити використовують гризунів як джерело живлення

391 / 6307
У 4 місячного дитини яскраво виражені прояви рахіту. Розладів травлення НЕ відзначається. Дитина багато знаходиться на сонці. В протягом 2 х місяців дитина отримував вітамін D 3, однак прояви рахіту НЕ зменшилися. Порушенням синтезу якої речовини можна пояснити розвиток рахіту у цього дитини?

Інсуліну

Паратгормону

кальцитріолом

Тироксину

кальцитоніну

392 / 6307
Тварині, сенсибілізованій туберкуліном, внутрибрюшинно введений туберкулін. Через 24 години при лапаратомии виявлено венозну гіперемію і набряк очеревини. В мазках - відбитках з очеревини велика кількість лімфоцитів і моноцитів. Який патологічний процес у тварини?

Гнійне запалення

Асептичне запалення

Алергічне запалення

Фібринозне запалення

Серозное запалення

393 / 6307
Альбіноси погано переносять сонячне загар, у них з'являються опіки. Порушення метаболізму який кислоти лежить в основі цього явища?

глутамінова кислоти

Триптофану

фенілаланін

Метіоніну

Гістидину

394 / 6307
При повному (з водою) аліментарному голодуванні розвинулись генералізовані набряки. Який з патогенетичних факторів у цьому випадку є провідним?

Зниження гідростатичного тиску міжклітинної рідини

Зниження онкотичного тиску плазми крові

Підвищення осмотичного тиску міжклітинної рідини

Підвищення онкотичного тиску тканинної рідини

Зниження осмотичного тиску плазми крові

395 / 6307
При штовханні штанги спортсмен закидає голову назад для максимального підвищення тонус а м'язів - розгиначів верхніх кінцівок. Де розташовані центри рефлексів, що виникають при цьому?

У спинному мозку

У рухової корі

В базальних гангліях

В ядрах Дейтерса

В червоних ядрах

396 / 6307
Хворому проводять правобічну пульмонектомію по приводу раку легкого. Який порядок розміщення анатомічних утворень кореня правого легкого (в напрямку зверху вниз)?

Артерія, вени, бронх

Артерія, бронх, вени

Бронх, артерія, вени

Відня, артерія, бронх

397 / 6307
При реєстрації ЕКГ хворого з гіперфункцією щитовидної залози зареєстровано збільшення частоти серцевих скорочень. Скорочення якого елемента ЕКГ про це свідчить?

Сегмента PQ

Інтервалу PQ

Інтервалу RR

Інтервалу Р-Т

Комплексу QRS

398 / 6307
При розтині труп а хворого, який помер від легеневої недостатності, виявлено збільшене легке з вогнищами темно - червоного, білого, рожево - жовтого кольору, некротичний трахеобронхіт. Яким захворюванням найбільш ймовірно страждав хворий?

Фиброзирующий альвеоліт

Грипозна пневмонія

Корова пневмонія

Токсична форма грипу

Крупозная пневмонія

399 / 6307
При обстеженні у хворого виявлено підвищений вміст ліпопротеїнів низької щільності в сироватці крові. Яке захворювання можна очікувати у цього хворого?

Пошкодження нирок

Гастрит

Атеросклероз

Запалення легенів

Гострий панкреатит

400 / 6307
Внаслідок блокади іонних каналів мембрани клітини її потенціал спокою зменшився з -90 мВ до -70 мВ. Які канали заблоковані?

Калієві

Хлорні

Кальцієві

Натрієві

Магнієві

401 / 6307
Підвищена ламкість судин, руйнування емалі і дентину у хворих на цингу здебільшого обумовлені порушенням дозрівання колагену. Який етап модифікації проколагену порушений при цьому авітамінозі?

Гідроксилювання проліну

Утворення поліпептидних ланцюгів

Видалення з проколагену С-кінцевого пептиду

Глікозилювання гидроксилізинових залишків

Відщеплення N-кінцевого пептиду

402 / 6307
Дитина 10 років скаржиться на слабкість, нудоту, дратівливість. На білизні знайдені гельмінти білого кольору завдовжки 5-10 мм. Під час мікроскопії зіскрібка з періанальних складок виявлені безбарвні яйця несиметричної форми. Укажіть, який гельмінт паразитує у дитини?

Трихінела

Аскарида людська

Гострик

Кривоголовка

Волосоголовець

403 / 6307
Під час розтину мертвонародженої дитини виявлено аномалію розвитку серця: шлуночки не розмежовані, з правої частини виходить суцільний артеріальний стовбур. Для якого класу хребетних тварин характерна подібна будова серця?

Ссавці

Риби

Птахи

Рептилії

Амфібії

404 / 6307
Чоловік 37 років надійшов до хірургічного відділення з явищами гострого панкреатиту: блювання, пронос, брадикардія, гіпотензія, слабість, явища зневоднювання організму. Який препарат найбільш доцільно використати в першу чергу?

Ефедрин

Но-шпу

Контрикал

Етаперазин

Платифілін

405 / 6307
У 12-річного хлопця часто виникають вірусні та бактеріальні інфекції, екзематозні ураження шкіри. Під час обстеження виявлено зменшення Тлімфоцитів та IgМ при нормальному вмісті IgA і IgG. Якій вид патології імунної системи спостерігається у хворого?

Гіпоплазія тимуса

Спадковий дефіцит системи комплементу

Синдром Шерешевського-Тернера

Комбінований імунодефіцит

Гіпогаммаглобулінемія Брутона

406 / 6307
Під час обстеження людини встановлено, що хвилинний об'єм серця дорівнює 3500 мл, систолічний об'єм – 50 мл. Якою є частота серцевих скорочень у цієї людини?

70 скорочень за хвилину

60 скорочень за хвилину

50 скорочень за хвилину

80 скорочень за хвилину

90 скорочень за хвилину

407 / 6307
У жінки виявлено пухлину яєчника. Показана операція. Яку зв`язку повинен перерізати хірург, щоб відділити яєчник від матки?

Зв’язку, що підвішує яєчник

Круглу зв’язку матки

Власну зв’язку яєчника

Широку зв’язку матки

Бічну пупкову зв’язку

408 / 6307
На рентгенограмі таза у новонародженого визначаються три самостійні кістки, які з’єднані хрящем у ділянці кульшової западини. Які це кістки?

Клубова, лобкова, сіднична

Лобкова, сіднична, стегнова

Крижова, лобкова, куприкова

Клубова, крижова, куприкова

Сіднична, стегнова, крижова

409 / 6307
У крові чоловіка 26 років виявлено 18% еритроцитів сферичної, сплощеної, кулеподібної та остистої форми. Інші еритроцити були у формі двоввігнутих дисків. Як називається таке явище?

Патологічний пойкілоцитоз

Патологічний анізоцитоз

Еритроцитоз

Фізіологічний анізоцитоз

Фізіологічний пойкілоцитоз

410 / 6307
Під час розтину тіла чоловіка, який тривалий час зловживав алкоголем, виявлено: печінка малих розмірів, щільна, дрібногорбиста. Мікроскопічно: псевдочасточки дрібні, розподілені вузькими прошарками сполучної тканини з лімфомакрофагальними інфільтратами; гепатоцити у стані великокрапельної жирової дистрофії. Який найбільш імовірний діагноз?

Алкогольний цироз

Хронічний активний алкогольний гепатит

Хронічний персистуючий алкогольний гепатит

Токсична дистрофія печінки

Жировий гепатоз

411 / 6307
У крові пацієнта вміст глюкози натщесерце був 5,65 ммоль/л, через 1 годину після цукрового навантаження становив 8,55 ммоль/л, а через 2 години – 4,95 ммоль/л. Такі показники характерні для:

Здорової людини

Хворого з інсулінозалежним цукровим діабетом

Хворого з прихованим цукровим діабетом

Хворого з інсуліннезалежним цукровим діабетом

Хворого з тиреотоксикозом

412 / 6307
У хворого, що тривалий час приймав глюкокортикоїди, в результаті відміни препарату виникло загострення наявного захворювання, зниження артеріального тиску, слабкість. Чим можна пояснити ці явища?

Виникненням недостатності наднирників

Сенсибілізацією

Кумуляцією

Звиканням до препарату

Гіперпродукцією АКТГ

413 / 6307
Хворий похилого віку скаржиться на головний біль, запаморочення, швидку втомлюваність, погіршення пам’яті. В анамнезі черепно-мозкова травма. Препарат якої групи необхідно призначити хворому?

Транквілізатори

Ноотропні

Снодійні

Нейролептики

Аналгетики

414 / 6307
З хімічного виробництва в токсикологічне відділення доставлено хворого з отруєнням ртуттю. Який препарат слід використати в даній ситуації?

Унітіол

Ізонітрозин

Активоване вугілля

Ентеросорбент СКН

Налоксон

415 / 6307
При обстеженні хворого виявлене новоутворення в білій речовині півкуль великого мозку з локалізацією у коліні та передньому відділі задньої ніжки внутрішньої капсули. Волокна якого провідного шляху мозку будуть зруйновані?

Tr. frontothalamicus

Tr. parietooccipitopontinus

Тr. thalamocorticalis

Tr. frontopontinus

Tr. pyramidalis

416 / 6307
При бактеріологічному дослідженні випорожнень чотиримісячної дитини з симптомами гострої кишкової інфекції на середовищі Ендо виросли у великій кількості червоні колонії. Які це можуть бути мікроорганізми?

Шигели

Сальмонели

Стрептококи

Ешерихії

Стафілококи

417 / 6307
Хворий протягом двох тижнів отримував медикаментозну терапію з приводу психозу. Стан хворого поліпшився, однак невдовзі з’явились ригідність, тремор, гіпокінезія. Який з наведених препаратів викликає такі ускладення?

Аміназин

Імізин

Сиднокарб

Хлордіазепоксид

Дифенін

418 / 6307
На розтині тіла померлого виявлено: м’яка мозкова оболонка верхніх відділів півкуль головного мозку різко повнокровна, жовто-зеленого кольору, просочена гнійним та фібринозним ексудатом, що нагадує чіпець. Для якого захворювання характерна така картина?

Менінгококовий менінгіт

Грипозний менінгіт

Менінгіт при сибірці

Менінгіт при висипному тифі

Туберкульозний менінгіт

419 / 6307
У молодих здорових батьків народилася дівчинка, білява, з блакитними очима. У перші ж місяці життя у дитини розвинулися дратівливість, неспокій , порушення сну і харчування, а обстеження невропатолога показало відставання у розвитку дитини. Який метод генетичного обстеження дитини слід застосувати для точного встановлення діагнозу?

Біохімічний

Цитологічний

Популяційно-статистичний

Близнюковий

Генеалогічний

420 / 6307
Дистрофічні зміни серцевого м’яза супроводжуються розширенням порожнин серця, зниженням сили серцевих скорочень, збільшенням об’єму крові , що залишається під час систоли в порожнині серця, переповненням вен. Для якого стану серця це характерно?

Кардіосклероз

Аварійна стадія гіперфункції та гіпертрофії

Міогенна дилатація

Тампонада серця

Тоногенна дилатація

421 / 6307
При визначенні групи крові за системою АВ0 аглютинацію еритроцитів досліджуваної крові викликали стандартні сироватки I та II груп і не викликала III групи. Які аглютиногени містяться в цих еритроцитах?

А та В

В

D та C

А

С

422 / 6307
У жінки з первинним гіперпаратиреоїдизмом періодично повторюються напади ниркової коліки. Ультразвукове обстеження показало наявність дрібних каменів у нирках. Яка найбільш імовірна причина утворення цих каменів?

Гіперкаліємія

Гіперкальціємія

Гіперхолестеринемія

Гіперфосфатемія

Гіперурикемія

423 / 6307
У хворого на глаукому спостерігається підвищення внутрішньоочного тиску при нормальній секреції водянистої вологи циліарним тілом. З ушкодженням яких структур стінки очного яблука пов’язане порушення відтоку рідини з передньої камери?

Венозного синуса

Заднього епітелію рогівки

Циліарного тіла

Війкового м’яза

Судинної оболонки

424 / 6307
Який механізм тепловіддачі найбільш ефективно спрацьовує при перебуванні людини в умовах 80% вологості повітря та температурі навколишнього середовища +35°С?

Конвекція

Радіація

--

Випаровування

Теплопроведення

425 / 6307
Провідником наукової експедиції по Індії був місцевий житель, який ніколи не розлучався зі своїм улюбленим собакою. Якими інвазійними захворюваннями можуть бути заражені члени експедиції при контакті з цим собакою, якщо він є джерелом інвазії?

Дикроцеліозом

Ехінококозом

Фасціольозом

Теніозом

Парагонімозом

426 / 6307
Хворій 34 років 3 роки тому було встановлено діагноз гломерулонефриту. За останні 6 місяців з'явилися набряки. Що лежить в основі їх розвитку?

Порушення білковоутворюючої функції печінки

Протеїнурія

Гіперосмолярність плазми

Гіперпродукція вазопресину

Гіперальдостеронізм

427 / 6307
При жировій інфільтрації печінки порушується синтез фосфоліпідів. Вкажіть, яка з наведених речовин може посилювати процеси метилювання в процесі синтезу фосфоліпідів?

Глюкоза

Гліцерин

Цитрат

Аскорбінова кислота

Метіонін

428 / 6307
Хворому 50 років з метою лікування черевного тифу призначений левоміцетин, але на наступний день стан хворого погіршився, температура підвищилася до 39,6°С. Чим пояснити погіршення стану хворого?

Дією ендотоксинів збудника

Нечутливістю збудника до левоміцетину

Приєднанням вторинної інфекції

Реінфекцією

Алергічною реакцією

429 / 6307
У хворого 30 років з дизентерією, підтвердженою бактеріологічно, з'явились ознаки парапроктиту. Про яку стадію місцевих змін найбільш імовірно йде мова у даного хворого?

Фібринозний коліт

Фолікулярний коліт

Стадія утворення виразок

Катаральний коліт

Стадія загоєння виразок

430 / 6307
Тривале перебування в умовах спеки викликало у людини спрагу. Сигналізація від яких рецепторів, перш за все, зумовила її розвиток?

Барорецепторів дуги аорти

Осморецепторів печінки

Глюкорецепторів гіпоталамусу

Натрієвих рецепторів гіпоталамусу

Осморецепторів гіпоталамусу

431 / 6307
Протягом двох тижнів хвора приймала мікстуру, призначену невропатологом з приводу неврастенії. Самопочуття хворої дещо поліпшилося, однак незабаром з’явились скарги на нежить, кон’юнктивіт, шкірні висипи, млявість та послаблення пам’яті. Був встановлений діагноз “бромізм”. Що доцільно призначити для послаблення симптомів?

Розчин глюкози 5%

Поліглюкін

Аспаркам

Натрію хлорид

432 / 6307
Хворого з явищами енцефалопатії госпіталізували до неврологічного стаціонару і виявили кореляцію між наростанням енцефалопатії і речовинами, що надходять із кишечнику до загального кровотоку. Які з'єднання, що утворюються в кишечнику, можуть викликати ендотоксемію?

Індол

Біотин

Ацетоацетат

Бутират

Орнітин

433 / 6307
В експерименті у кролика було видалено верхній шийний вузол симпатичного стовбура. На боці видалення спостерігаються почервоніння і підвищення температури шкіри голови. Яка форма порушень периферичного кровообігу розвинулася у

Метаболічна артеріальна гіперемія

Нейротонічна артеріальна гіперемія

Венозна гіперемія

Нейропаралітична артеріальна гіперемія

Стаз

434 / 6307
До травматологічного відділення доставлено чоловіка з закритою травмою живота праворуч та підозрою на розрив печінки. В якому з приведених утворень очеревини слід чекати накопичення крові?

Fossa ischio-analis

Recessus duodenalis inferior

Excavatio rectovesicalis

Recessus intersigmoideus

Bursa omentalis

435 / 6307
У пацієнта, який чітко виконував рекомендації по дотриманню певної дієти протягом 10 днів, було проведене дослідження величини дихального коефіцієнту. Встановлено, що він дорівнює 1. Якої дієти дотримувався пацієнт?

З переважним вмістом вуглеводів

Змішаної

З переважним вмістом жирів і вуглеводів

З переважним вмістом білків і жирів

З переважним вмістом білків і вуглеводів

436 / 6307
У препаратах подані зрізи органів кровотворення та імуногенезу людини, для яких характерна наявність лімфоїдної тканини, що формує різноманітні структури (лімфатичні вузлики, дольки, тяжі). Визначте, в якому з органів відбувається антигеннезалежна проліферація та диференціювання лімфоцитів?

Лімфатичні вузли

Тимус

Селезінка

Гемолімфатичні вузли

Мигдалик

437 / 6307
За даними бактеріоскопії мазку з уретри у хворого виявлено гонорею. Враховуючи, що препаратами вибору для лікування гонореї є фторхінолони, хворому необхідно призначити:

Уросульфан

Фуразолідон

Цефазолін

Ципрофлоксацин

Фторурацил

438 / 6307
Хворому з метою відновлення дихання при отруєнні чадним газом було введено аналептичний засіб рефлекторного типу дії з групи Н-холіноміметиків. Який засіб було призначено хворому?

Адреналіну гідрохлорид

Лобеліну гідрохлорид

Атропіну сульфат

Пентамін

Мезатон

439 / 6307
Молода людина під час активного підтягування на перекладині відчула різкий біль у спині. Об'єктивно: біль при спробах руху верхньою кінцівкою, обмеження функцій приведення та пронації. Розтягнення якого м’яза, найбільш імовірно, відбулося?

М. levator scapulae

М. subscapularis

М. latissimus dorsi

М. romboideus major

М. trapezius

440 / 6307
З метою серологічної діагностики коклюшу поставлена розгорнута реакція з коклюшним та паракоклюшним діагностикумами. На дні пробірок, до яких було внесено діагностикум з Bordetella parapertussis, утворився зернистий осад. Які антитіла виявила ця реакція?

Антитоксини

Опсоніни

Преципітини

Бактеріолізини

Аглютиніни

441 / 6307
При тиреотоксикозі підвищується продукція тиреоїдних гормонів Т3 та Т4, розвивається схуднення, тахікардія, психічне збудження та інше. Як саме впливають тиреоїдні гормони на енергетичний обмін в мітохондріях клітин?

Активують окисне фосфорилювання.

Роз`єднують окислення та окисне фосфорилювання

Блокують дихальний ланцюг

Активують субстратне фосфорилювання

Блокують субстратне фосфорилювання

442 / 6307
У комплексному лікуванні гіпертонічної хвороби хворому був призначений сечогінний препарат. Через кілька днів АТ знизився, але виникли ознаки гіпокаліємії. Який препарат міг викликати таке ускладнення?

Спіронолактон

Еналаприл

Фуросемід

Клофелін

Тріамтерен

443 / 6307
Хворий 45 років госпіталізований до хірургічного відділення зі скаргами на раптовий гострий біль в надчеревній ділянці. Після обстеження встановлено діагноз: перфоративна (проривна) виразка задньої стінки шлунка. Куди вилився вміст шлунка в момент перфорації?

В правий брижовий синус

В лівий брижовий синус

В сальникову сумку

В передшлункову сумку

В печінкову сумку

444 / 6307
В медико-генетичну консультацію звернулась жінка з приводу ризику захворювання на гемофілію у свого сина. Її чоловік страждає на дане захворювання з народження. Жінка та її батьки здорові стосовно гемофілії. Яка вірогідність появи хвороби у хлопчика в даній сім'ї?

50% хлопчиків будуть хворими

75% хлопчиків будуть хворими

Всі хлопчики будуть хворі

Усі хлопчики будуть здорові

25% хлопчиків будуть хворими

445 / 6307
При розтині тіла жінки 40 років, яка страждала ревматоїдним артритом, знайдено збільшену щільну селезінку. На розрізі її тканина коричневочервоного кольору зі збільшеними фолікулами, які мають вигляд напівпрозорих сірувато-білуватих зерен. Який патологічний процес найбільш вірогідний?

Сагова селезінка

Гіаліноз селезінки

Глазурна селезінка

Масна селезінка

Порфірна селезінка

446 / 6307
У хворого відзначається підвищена чутливість шкіри до сонячного світла. Його сеча при тривалому стоянні набуває темно-червоного кольору. Яка найбільш імовірна причина такого стану?

Пелагра

Альбінізм

Гемолітична жовтяниця

Порфірія

Алкаптонурія

447 / 6307
Терапія анаприліном позитивно вплинула на динаміку хвороби у жінки 44 років, яка страждає стенокардією. Який головний механізм дії цього препарату?

Зменшення окислювального обміну в міокарді внаслідок блокади ферментів циклу Кребса

Зменшення енергозатрат міокарда внаслідок зниження навантаження

Збільшення надходження кисню в міокард

Зниження потреби і збільшення надходження кисню в міокард

Блокада β-адренорецепторів і зниження потреби міокарда в кисні

448 / 6307
В хірургічний кабінет звернулась людина, яку покусав невідомий собака. Широкі рвані рани локалізовані на обличчі. Яку лікувально-профілактичну допомогу потрібно надати для профілактики сказу?

Призначити комбіновану антибіотикотерапію

Госпіталізувати хворого і утримувати під наглядом лікаря

Терміново ввести вакцину АКДП

Розпочати імунізацію антирабічною вакциною

Терміново ввести нормальний гама-глобулін

449 / 6307
До приймального відділення доставлений хворий зі скаргами на сухість у роті, світлобоязнь та порушення зору. Шкіра гіперемійована, суха, зіниці розширені, тахікардія. При подальшому обстеженні був встановлений діагноз: отруєння алкалоїдами красавки. Який із лікарських засобів доцільно застосувати?

Діазепам

Прозерин

Пілокарпін

Дипіроксим

Армін

450 / 6307
При моделюванні запалення нижньої кінцівки у тварини підвищилася температура тіла, збільшився вміст антитіл та лейкоцитів у крові. Які речовини зумовили розвиток цих загальних реакцій організму при запаленні?

Глюкокортикоїди

Соматомедини

Інтерлейкіни

Лейкотриєни

Мінералокортикоїди

451 / 6307
В крові хворого виявлено низький рівень альбумінів і фібриногену. Зниження активності яких органел гепатоцитів печінки найбільш вірогідно обумовлює це явище?

Мітохондрій

Гранулярної ендоплазматичної сітки

Агранулярної ендоплазматичної сітки

Комплексу Гольджі

Лізосом

452 / 6307
Під час патронажу лікар виявив у дитини симетричну шорсткість щік, діарею, порушення нервової діяльності. Нестача яких харчових факторів є причиною такого стану?

Нікотинова кислота, триптофан

Лізин, аскорбінова кислота

Метіонін, ліпоєва кислота

Фенілаланін, пангамова кислота

Треонін, пантотенова кислота

453 / 6307
Дівчинка 10 років часто хворіє на гострі респіраторні інфекції, після яких спостерігаються множинні точкові крововиливи в місцях тертя одягу. Гіповітаміноз якого вітаміну має місце в дівчинки?

А

В2

В1

В6

С

454 / 6307
У хворого з гіпохромною анемією січеться і випадає волосся, відзначається підвищена ламкість нігтів, порушений смак. Який механізм розвитку зазначених симптомів?

Зниження продукції тиреоїдних гормонів

Дефіцит залізовмісних ферментів

Зниження продукції паратирину

Дефіцит вітаміну В12

Дефіцит вітаміну А

455 / 6307
Аспірин має протизапальну дію, оскільки пригнічує активність циклооксигенази. Рівень яких біологічно активних речовин буде при цьому знижуватися?

Йодтиронінів

Лейкотриєнів

Біогенних амінів

Катехоламінів

Простагландинів

456 / 6307
У чоловіка 60 років після крововиливу в головний мозок настав тривалий сон. Пошкодження яких структур найімовірніше призвело до цього стану?

Ретикулярної формації

Кори великих півкуль

Чорної субстанції

Гіпокампа

Ядер черепних нервів

457 / 6307
Клітину лабораторної тварини піддали надмірному рентгенівському опроміненню. У результаті утворились білкові фрагменти в цитоплазмі. Який органоїд клітини візьме участь у їх утилізації?

Лізосоми

Комплекс Гольджі

Клітинний центр

Рибосоми

Ендоплазматичний ретикулум

458 / 6307
У чоловіка 52 років діагностовано системний амебіаз з ураженням кишечнику, печінки, легень. Який препарат слід призначити?

Метронідазол

Хінгамін

Тетрациклін

Ентеросептол

Хініофон

459 / 6307
До лікаря звернувся хворий зі скаргами на постійну спрагу. Виявлена гіперглікемія, поліурія та підвищений вміст 17-кетостероїдів у сечі. Яке захворювання найбільш імовірне?

Стероїдний діабет

Глікогеноз I типу

Інсулінозалежний діабет

Мікседема

Аддісонова хвороба

460 / 6307
У біоптаті бронха хворого, який зловживає палінням, у потовщеній слизовій оболонці виявлено хронічне запалення і трансформацію одношарового війчастого епітелію в багатошаровий плоский епітелій. Який із процесів найбільш імовірний?

Гіперплазія епітелію

Гіпертрофія епітелію

Плоскоклітинний рак

Лейкоплакія

Метаплазія

461 / 6307
Пацієнту з гострим інфарктом міокарда внутрішньовенно крапельно введено 1500 мл різних розчинів протягом 8 годин, кисень інтраназально. Смерть настала від набряку легень. Що спричинило набряк легень?

Перевантаження лівого шлуночка об'ємом

Алергічна реакція

Інгаляція кисню

Нейрогенна реакція

Зменшення онкотичного тиску за рахунок гемодилюції

462 / 6307
У хворої 43 років на фоні септичного шоку визначається тромбоцитопенія, зменшення фібриногену, поява в крові продуктів деградації фібрину, петехіальні крововиливи. Яка найбільш вірогідна причина цих змін?

Порушення виробляння тромбоцитів

Аутоімунна тромбоцитопенія

Екзогенна інтоксикація

Геморагічний діатез

ДВС-синдром

463 / 6307
До лікаря звернулися пацієнти з подібними скаргами: слабкість, болі в кишечнику, розлад ШКТ. Після дослідження фекалій з'ясувалось, що терміновій госпіталізації підлягає один з пацієнтів, у якого були виявлені цисти з чотирма ядрами. Для якого найпростішого характерні такі цисти?

Кишкова амеба

Дизентерійна амеба

Трихомонада

Балантидій

Лямблія

464 / 6307
До клініки доставили пацієнта 32 років з масивною крововтратою внаслідок автодорожної травми. Пульс 110 уд/хв., частота дихання - 22 за хв., АТ - 100/60 мм рт.ст. Яка зміна крові із перелічених буде найбільш характерною через 1 годину після крововтрати?

Лейкопенія

Гіповолемія

Гіпохромія еритроцитів

Еритропенія

Гіпопротеїнемія

465 / 6307
При диспансерному обстеженні хлопчику 7 років встановлено діагноз - дальтонізм. Батьки здорові, кольоровий зір нормальний. Але у дідуся по материнській лінії така ж аномалія. Який тип успадкування цієї аномалії?

Домінантний, зчеплений зі статтю

Аутосомно-домінантний

Неповне домінування

Аутосомно-рецесивний

Рецесивний, зчеплений зі статтю

466 / 6307
Під час мікроскопічного дослідження збільшеного шийного лімфатичного вузла дівчинки 14 років було знайдено, що тканинна будова вузла порушена, лімфоїдні фолікули відсутні, є ділянки склерозу та вогнища некрозу, клітинний склад вузла поліморфний, присутні лімфоцити, еозинофіли, атипові клітини великих розмірів з багаточасточковими ядрами (клітини БерезовськогоШтернберга) та одноядерні клітини також великих розмірів. Який найбільш імовірний діагноз?

Хронічний лімфолейкоз

Гострий лімфолейкоз

Лімфома Беркітта

Грибоподібний мікоз

Лімфогранулематоз

467 / 6307
Хворий на цукровий діабет після ін'єкції інсуліну знепритомнів, почалися судоми. Який результат може дати біохімічний аналіз крові на вміст цукру?

5,5 ммоль/л

10,0 ммоль/л

8,0 ммоль/л

3,3 ммоль/л

1,5 ммоль/л

468 / 6307
Жінка 45 років страждає на сезонний алергічний риніт, пов’язаний з цвітінням амброзії. Який лікарський засіб з групи стабілізаторів тучних клітин можна застосувати для профілактики даного захворювання?

Кетотифен

Діазолін

Фенкарол

Димедрол

Тавегіл

469 / 6307
Тварині в експерименті перерізали передні корінці п’яти сегментів спинного мозку. Які зміни відбудуться в зоні

Втрата рухів

Втрата дотикової чутливості

Гіперчутливість

Втрата пропріоцептивної чутливості

Втрата температурної чутливості

470 / 6307
У хворого із значними периферійними набряками почергове застосування дихлотіазиду, етакринової кислоти і фуросеміду не сприяло значному діуретичному ефекту. У крові - значне підвищення кількості альдостерону. Вкажіть препарат вибору.

Сечовина

Маніт

Клопамід

Спіронолактон

Амілорид

471 / 6307
У досліді на тварини зруйновано середню частину завитка внутрішнього вуха. Це призведе до порушення сприйняття звуків такої частоти:

Порушень не буде

Середня

Висока та низька

Низька

Висока

472 / 6307
Після накладання джгута у досліджуваного виявили точкові крововиливи. З порушенням функції яких клітин крові це пов’язано?

Нейтрофіли

Еозинофіли

Лімфоцити

Моноцити

Тромбоцити

473 / 6307
Чоловік 65 років, який страждає на подагру, скаржиться на болі в області нирок. При ультразвуковому обстеженні встановлена наявність ниркових каменів. У результаті якого процесу утворюються ниркові камені?

Катаболізму білків

Розпаду гему

Відновлення цистеїну

Орнітинового циклу

Розпаду пуринових нуклеотидів

474 / 6307
Артеріальна гіпертензія у хворого обумовлена стенозом ниркових артерій. Активація якої системи є головною ланкою в патогенезі цієї форми гіпертензії?

Парасимпатична

Симпатоадреналова

Гіпоталамо-гіпофізарна

Калікреїн-кінінова

Ренін-ангіотензинова

475 / 6307
З носоглотки дитини 5 років виділено мікроорганізм, який за морфологічними та біохімічними ознаками ідентичний Corynebacterium diphtheriae, але не утворює екзотоксин. У результаті якого процесу цей мікроорганізм може стати токсигенним?

Хромосомна мутація

Вирощування у присутності антитоксичної сироватки

Пасаж через організм чутливих тварин

Культивування на телуритовому середовищі

Фагова конверсія

476 / 6307
В експерименті ізольований м'яз жаби ритмічно подразнюють електричними імпульсами. Кожний наступний імпульс припадає на період розслаблення попереднього скорочення. Яке скорочення м'язу виникає?

Суцільний тетанус

Одиночне

Зубчастий тетанус

Тонічне

Асинхронне

477 / 6307
Для підвищення результатів спортсмену рекомендовано застосовувати препарат, що містить карнітин. Який процес у найбільшому ступені активується карнітином?

Синтез ліпідів

Тканинне дихання

Синтез стероїдних гормонів

Транспорт жирних кислот у мітохондрії

Синтез кетонових тіл

478 / 6307
Хлопчик 13 років скаржиться на загальну слабість, запаморочення, втомлюваність. Спостерігається відставання у розумовому розвитку. У крові та сечі висока концентрація валіну, ізолейцину, лейцину. Сеча специфічного запаху. Який найбільш імовірний діагноз?

Хвороба Аддісона

Базедова хвороба

Тирозиноз

Хвороба кленового сиропу

Гістидинемія

479 / 6307
У собаки в досліді подразнювали на шиї периферичний відрізок блукаючого нерва, при цьому спостерігали такі зміни серцевої діяльності:

Збільшення частоти та сили скорочень

Збільшення атріовентрикулярного проведення

Зменшення частоти скорочень

Збільшення збудливості міокарда

Збільшення сили скорочень

480 / 6307
Після позалікарняного аборту у жінки прогресував гнійний ендоміометрит зі смертельним наслідком. При розтині померлої виявлені чисельні абсцеси легень, субкапсулярні гнійнички в нирках, гіперплазія селезінки. Яка форма сепсису виникла у хворої?

Уросепсис

Легеневий сепсис

Хроніосепсис E. Септикопіємія

Септицемія

481 / 6307
До лікаря-інфекціоніста з хворою дитиною звернулися батьки, які тривалий час працювали в одній азіатській країні. У дитини шкіра землистого кольору, втрата апетиту, в’ялість, збільшені печінка, селезінка, периферичні лімфатичні вузли. Яке протозойне захворювання можна припустити у дитини?

Вісцеральний лейшманіоз

Токсоплазмоз

Амебіаз

Балантидіаз

Лямбліоз

482 / 6307
До травматологічного пункту доставлено хворого з пошкодженням м'язів нижніх кінцівок. За рахунок яких клітин можлива репаративна регенерація м'язових волокон і відновлення функції м'язів?

Міофібробластів

Клітин-сателітів

Міобластів

Фібробластів

Міоепітеліальних клітин

483 / 6307
Хворого госпіталізовано зі скаргами на блювання, запаморочення, двоїння в очах, утруднене ковтання. Лікар запідозрив ботулізм. Які методи діагностики доцільно використати для підтвердження діагнозу?

Біологічну пробу, бактеріологічний

Протозоологічний, мікроскопічний

Бактеріологічний, мікологічний

Алергічну пробу, серологічний

484 / 6307
У хворого на ревматоїдний артрит, який лікувався індометацином, виникли ознаки гастропатії. З якою дією препарату можна пов’язати виникнення цього ускладнення?

Антикінінова

Місцевоподразнююча

Антигістамінна

Антисеротонінова

Антициклооксигеназна

485 / 6307
На розтині виявлено: на зовнішній поверхні аортального клапана великі (1-2 см) буровато-червоні, крихкі нашарування, які прикривають виразкові дефекти. Який найбільш імовірний діагноз?

Зворотній бородавчастий ендокардит

Гострий бородавчастий ендокардит

Фібропластичний ендокардит

Дифузний ендокардит

Поліпозно-виразковий ендокардит

486 / 6307
Під час інтраопераційної біопсії молочної залози виявлено ознаки тканинного атипізму, що виражається у порушенні співвідношення паренхіми і строми з переважанням останньої, різних розмірів і форми залозисті структури, вистелені одношаровим проліферуючим епітелієм. Який найбільш імовірний діагноз?

Фіброаденома

Неінфільтруючий рак

Папілома

Інфільтруючий рак

Мастит

487 / 6307
Хворий скаржиться на слабкість, задишку, набряки нижніх кінцівок. Діагноз – хронічна серцева недостатність. Який засіб необхідно призначити хворому в першу чергу?

Анаприлін

Дигітоксин

Раунатин

Папаверин

Кофеїн

488 / 6307
При нанесенні стоматологом пероксиду водню на слизову оболонку порожнини рота з'явилася інтенсивна піна. Який фермент спричиняє такий ефект?

Метгемоглобінредуктаза

Ацетилтрансфераза

Глюкозо-6-фосфатдегідрогеназа

Холінестераза

Каталаза

489 / 6307
Через декілька годин після опіку в ділянці гіперемії та набряку шкіри у хворого з’явилось вогнище некрозу. Який головний механізм забезпечує посилення руйнівних явищ в осередку запалення?

Еміграція лімфоцитів

Первинна альтерація

Вторинна альтерація

Проліферація фібробластів

Діапедез еритроцитів

490 / 6307
У потерпілого травма ліктьового суглоба з відриванням медіального надвиростка плечової кістки. Який нерв може бути пошкоджено при цій травмі?

Ліктьовий

М'язово-шкірний

Серединний

Медіальний шкірний нерв передпліччя

Променевий

491 / 6307
У хворого на цукровий діабет змінилось значення рH та стало дорівнювати 7,3. Визначення компонентів якої буферної системи використовується для діагностики розладів кислотно-лужної рівноваги?

Білкової

Гемоглобінової

Оксигемоглобінової

Фосфатної

Бікарбонатної

492 / 6307
У хворого М., 45 років, при аналізі ЕКГ встановлено: ритм синусовий, число передсердних комплексів більше числа шлуночкових комплексів; прогресуюче подовження інтервалу P-Q від комплексу до комплексу; випадання окремих шлуночкових комплексів; зубці Р та комплекси QRST без змін. Назвіть тип порушення серцевого ритму.

Синоаурикулярна блокада

Атріовентрикулярна блокада II ступеня

Внутрішньопередсердна блокада

Повна атріовентрикулярна блокада

Атріовентрикулярна блокада I ступеня

493 / 6307
Хвора 27 років закрапала в очі краплі, до складу яких входить пеніцилін. Через декілька хвилин з’явилися свербіння та печіння шкіри, набряк губ та повік, кашель зі свистом, став знижуватися артеріальний тиск. Які імуноглобуліни беруть участь у розвитку цієї алергічної реакції?

IgE і IgG

IgA і IgM

IgM і IgG

IgG і IgD

IgM і IgD

494 / 6307
У клітині, яка мітотично ділиться, спостерігається розходження дочірніх хроматид до полюсів клітини. На якій стадії мітотичного циклу знаходиться клітина?

Метафаза

Інтерфаза

Профаза

Телофаза

Анафаза

495 / 6307
В приймальне відділення доставлено хворого у непритомному стані. Шкіра холодна, зіниці звужені, дихання з утрудненням, відзначається періодичність по типу Чейна-Стокса, артеріальний тиск знижений, сечовий міхур переповнений. Отруєння якою речовиною найбільш вірогідне?

М-холіноблокаторами

Наркотичними аналгетиками

Транквілізаторами

Ненаркотичними аналгетиками

496 / 6307
При дослідженні гнійних виділень з шийки матки бактеріоскопічно виявлено присутність грамнегативних бобоподібних диплококів, які знаходилися як всередині, так і поза лейкоцитами. Назвіть чинника гнійного запалення шийки матки.

Calymmatobacterium granulomatis

Trichomonas vaginalis

Neisseria gonorroeae

Chlamidia trachomatis

Haemophilus vaginalis

497 / 6307
У хворого 60 років унаслідок злоякісної пухлини великого сосочка дванадцятипалої кишки виникла обтураційна жовтяниця. Просвіт якої анатомічної структури стискується пухлиною?

Міхурова протока

Права печінкова протока

Загальна печінкова протока

Ліва печінкова протока

Печінково-підшлункова ампула

498 / 6307
Хворий 60 років госпіталізований до хірургічного відділення в зв’язку з інфекцією, викликаною синьогнійною паличкою, чутливою до антибіотика пеніцилінового ряду. Вкажіть, який з наведених пеніцилінів має виражену активність по відношенню до Pseudomonas aeruginosa?

Карбеніцилін

Метицилін

Оксацилін

Феноксиметилпеніцилін

Бензилпеніцилін

499 / 6307
На електронній мікрофотографії біопсійного матеріалу подано легеню недоношеної дитини. Виявлено злипання стінки альвеол через відсутність сурфактанту. Порушення функції яких клітин стінки альвеоли зумовлює дану картину?

Секреторних клітин

Альвеолоцитів II типу

Альвеолярних макрофагів

Альвеолоцитів I типу

Фібробластів

500 / 6307
В приймально-діагностичне відділення доставлено жінку 38 років з матковою кровотечею. Які найбільш вірогідні зміни зі сторони крові відбудуться?

Лейкоцитоз

Еритроцитоз

Лейкопенія

Збільшення гематокритного числа

Зменшення гематокритного числа

501 / 6307
На базарі громадянин А. продавав ковбасу під назвою “свиняча домашня”. У держсанінспекції виникла підозра фальсифікації ковбаси. За допомогою якої серологічної реакції імунітету можна ідентифікувати харчовий продукт?

РЗК

Імунофлуоресценції

Аглютинації

Преципітації

РНГА

502 / 6307
У хворого виник спазм гладенької мускулатури бронхів. Використання активаторів яких мембранних циторецепторів фізіологічно обгрунтовано для зняття нападу?

Н-холінорецепторів

М-холінорецепторів

β-адренорецепторів

α-адренорецепторів

α- та β-адренорецепторів

503 / 6307
У хворої з клінічними ознаками імунодефіциту кількість та функціональна активність Т- і В-лімфоцитів не змінені. Під час обстеження на молекулярному рівні виявлено дефект, при якому порушена функція антигенпрезентації імунокомпетентним клітинам. Дефект яких клітин є найбільш імовірним?

Т-лімфоцити, В-лімфоцити

0-лімфоцити

Макрофаги, моноцити

Фібробласти, Т-лімфоцити, В-лімфоцити

NK-клітини

504 / 6307
Хвора 25 років звернулася зі скаргами на погіршення зору. При огляді виявлено порушення акомодації, зіниця розширена, не реагує на світло. Функція яких м'язів порушена?

М'яз, що звужує і м'яз, що розширює зіницю

Верхній навскісний, війковий

Латеральний прямий, м'яз, що звужує зіницю

М'яз, що звужує зіницю, війковий

М'яз, що розширює зіницю, війковий

505 / 6307
Внаслідок дії електричного струму на збудливу клітину виникла деполяризація її мембрани. Рух яких іонів через мембрану є причиною деполяризації?

Cl–

К+

НСО3–

Са2+

Na+

506 / 6307
У міокарді шлуночків досліджуваної людини порушені процеси реполяризації. Це призведе до порушення амплітуди, конфігурації, тривалості зубця:

P

Т

Q

S

R

507 / 6307
Хворий 50 років скаржиться на спрагу, п'є багато води, виражена поліурія. Глюкоза крові 4,8 ммоль/л, в сечі глюкози і ацетону немає, сеча безбарвна, питома вага 1,002-1,004. Яка причина поліурії?

Тиреотоксикоз

Альдостеронізм

Нестача вазопресину

Інсулінова недостатність

Гіпотиреоз

508 / 6307
Хвора на бронхіальну астму приймала таблетки, які викликали безсоння, головний біль і підвищення артеріального тиску. Який препарат міг стати причиною таких ускладнень?

Ефедрин

Адреналін

Ізадрин

Кромолін натрію

Еуфілін

509 / 6307
До гастроентерологічного відділення потрапив хворий 57 років з підозрою на синдром Золінгера-Еллісона, про що свідчило різке збільшення рівня гастрину у сироватці крові. Яке порушення секреторної функції шлунка найбільш імовірне?

Гіперсекреція гіперацидна

Ахілія

Гіперсекреція гіпоацидна

Гіпосекреція гіперацидна

Гіпосекреція гіпоацидна

510 / 6307
У хворого 17 років інтраопераційно на нижній поверхні печінки виявлена пухлина розмірами 4,5х5,0х3,5 см із субсерозною локалізацією, темночервоного кольору, на розрізі представлена порожнинами зі значним вмістом крові. Поставте попередній діагноз.

Гемангіоендотеліома

Лімфангіома

Кавернозна гемангіома

Гемангіоперицитома

Капілярна гемангіома

511 / 6307
У дитини виявлено галактоземію. Концентрація глюкози в крові суттєво не змінена. Дефіцитом якого ферменту зумовлене це захворювання?

Галактокіназа

Аміло-1,6-глюкозидаза

Фосфоглюкомутаза

Галактозо-1-фосфат-уридилтрансфераза

Гексокіназа

512 / 6307
Під час гістологічного дослідження шлунка виявлено, що у залозах міститься дуже мало парієтальних клітин або вони повністю відсутні. Слизову оболонку якої ділянки шлунка вивчали?

Тіло шлунка

Пілоричний відділ

Дно шлунка

Кардіальний відділ

513 / 6307
У результаті черепно-мозкової травми у хворого були виявлені такі симптоми: інтенційний тремор, дисметрія, адіадохокінез, дизартрія. Яка структура головного мозку ушкоджена?

Стріатум

Чорна речовина

Рухова кора

Блідий шар

Мозочок

514 / 6307
До травмпункту звернувся чоловік 45 років після побутової травми плеча. Об'єктивно: відсутні функції розгинання, приведення та пронації плеча. Пошкодження якого м’яза викликало такий стан?

Надосний м’яз

Підосний м’яз

Великий круглий м’яз

Підлопатковий м’яз

Малий круглий м’яз

515 / 6307
У крові хворого збільшена концентрація пірувату, значна кількість його екскретується з сечею. Який авітаміноз спостерігається у хворого?

Авітаміноз В2

Авітаміноз В3

Авітаміноз Е

Авітаміноз В6

Авітаміноз В1

516 / 6307
Внаслідок впливу гама-випромінювання ділянка ланцюга ДНК повернулась на 180 градусів. Який з наведених видів мутацій відбувся в ланцюгу ДНК?

Реплікація

Дуплікація

Транслокація

Делеція

Інверсія

517 / 6307
Куди треба провести катетер для забору лімфи з грудної лімфатичної протоки?

У лівий венозний кут

У правий венозний кут

У ліву пахову вену

У верхню порожнисту вену

У нижню порожнисту вену

518 / 6307
У хворого із запаленням легень непереносимість антибіотиків. Який з комбінованих сульфаніламідних препаратів слід призначити хворому?

Бісептол

Сульфацил натрію

Стрептоцид

Сульфадиметоксин

Етазол

519 / 6307
У хворого видалено дванадцятипалу кишку. Це призвело до зменшення секреції, перш за все, таких гастроінтестинальних гормонів:

Гастрин та гістамін

Гістамін

Холецистокінін та секретин

Нейротензин

Гастрин

520 / 6307
У померлого 58 років на розтині мітральний клапан деформований, потовщений, змикається не до кінця. Мікроскопічно: вогнища колагенових волокнинок еозинофільні, дають плюсову реакцію на фібрин. Найвірогідніше це:

Амілоїдоз

Мукоїдне набухання

Фібринозне запалення

Фібриноїдне набухання

Гіаліноз

521 / 6307
В організм людини введено живу вакцину. На підвищення активності яких клітин сполучної тканини можна очікувати?

Адипоцити і адвентиційні клітини

Плазмоцити і лімфоцити

Фібробласти і лаброцити

Пігментоцити і перицити

Макрофаги і фібробласти

522 / 6307
Тварині, сенсибілізованій туберкуліном, внутрішньоочеревинно введений туберкулін. Через 24 години при лапаротомії виявлено венозну гіперемію та набряк очеревини. У мазках-відбитках з очеревини велика кількість лімфоцитів та моноцитів. Який патологічний процес у тварини?

Алергійне запалення

Гнійне запалення

Серозне запалення

Асептичне запалення

Фібринозне запалення

523 / 6307
У чоловіка 42 років, який страждає на подагру, в крові підвищена концентрація сечової кислоти. Для зниження рівня сечової кислоти йому призначено алопуринол. Конкурентним інгібітором якого ферменту є алопуринол?

Аденозиндезамінази

Гіпоксантинфосфорибозилтрансферази

Гуаніндезамінази

Ксантиноксидази

Аденінфосфорибозилтрансферази

524 / 6307
Під час мікроскопії волосини хворого, взятої з уражених ділянок, виявлені обривки міцелію гриба, спори, пухирці повітря і крапельки жиру. Для якого грибкового захворювання характерна така мікроскопічна картина волосини?

Епідермофітія

Фавус

Мікроспорія

Споротрихоз

Трихофітія

525 / 6307
У людини зменшений діурез, гіпернатріємія, гіпокаліємія. Гіперсекреція якого гормону може бути причиною таких змін?

Адреналін

Альдостерон

Вазопресин

Паратгормон

Передсердний натрійуретичний фактор

526 / 6307
Послаблення кровопостачання органа зумовлює розвиток гіпоксії, яка активізує функцію фібробластів. Об’єм яких елементів нарощується в цій ситуації?

Міжклітинної речовини

Судин мікроциркуляторного русла

Паренхіматозних елементів органа

Нервових елементів

Лімфатичних судин

527 / 6307
Хворий скаржиться на часте та утруднене сечовиділення. Порушення структури якого з наведених утворень є причиною цього?

Сім’яні міхурці

Простата

Придатки яєчка

Яєчка

Бульбоуретральні залози

528 / 6307
У результаті виснажливої м'язової роботи у робіткика значно зменшилась буферна ємність крові. Надходженням якої кислої речовини до крові можна пояснити це явище?

Лактату

α-кетоглутарату

3-фосфогліцерату

1,3-бісфосфогліцерату

Пірувату

529 / 6307
У групі дітей, які їли солодкий соковитий кавун, у двох з'явились ознаки отруєння: різка слабість, запаморочення, головний біль, блювання, задишка, тахікардія, синюшність губ, вух, кінчиків пальців. Лабораторний аналіз кавуна показав високий вміст нітратів. Який провідний механізм у патогенезі отруєння тільки у двох дітей?

Недостатність каталази

Блокада цитохромоксидази

Недостатність мет-Hb-редуктази

Недостатність глутатіон-пероксидази

Недостатність супероксиддисмутази

530 / 6307
У спортсмена на старті перед змаганнями відзначається підвищення артеріального тиску та частоти серцевих скорочень. Впливом яких відділів ЦНС можна пояснити вказані зміни?

Довгастого мозку

Гіпоталамуса

Кори великих півкуль

Проміжного мозку

Середнього мозку

531 / 6307
Під час запалення відзначається підвищення проникливості судин мікроциркуляторного русла, збільшення в них гідродинамічного тиску крові. У міжклітинній рідині має місце підвищення осмотичної концентрації і дисперсності білкових структур. Який вид набряку буде спостерігатися в цьому

Змішаний

Мембраногенний

Колоїдно-осмотичний

Гідродинамічний

Лімфогенний

532 / 6307
На розтині виявлено, що нирки збільшені в розмірах, поверхня крупногорбиста за рахунок наявності численних порожнин з гладенькою стінкою, заповнених прозорою рідиною. Яке захворювання нирок мало місце у хворого?

Інфаркт

Некротичний нефроз

Полікістоз

Пієлонефрит

Гломерулонефрит

533 / 6307
Для лікування злоякісних пухлин призначають метотрексат - структурний аналог фолієвої кислоти, який є конкурентним інгібітором дигідрофолатредуктази. На якому рівні метотрексат пригнічує синтез нуклеїнових кислот?

Репарація

Реплікація

Процесинг

Транскрипція

Синтез мононуклеотидів

534 / 6307
У результаті пошкодження одного з реакторів АЕС відбулося витікання радіоактивних продуктів. Люди, які знаходилися в зоні підвищеної радіації, орієнтовно отримали по 250-300 Р. Їх негайно госпіталізовано. Які зміни складу крові будуть характерними для потерпілих?

Лімфопенія

Анемія

Нейтропенія

Тромбоцитопенія

Лейкопенія

535 / 6307
Тестостерон і його аналоги збільшують масу скелетний м'язів, що дозволяє використовувати їх для лікування дистрофій. Взаємодією гормону з яким клітинним субстратом зумовлена ця дія?

Мембранні рецептори

Ядерні рецептори

Хроматин

Рибосоми

Білки-активатори транскрипції

536 / 6307
До інфекційної лікарні надійшов пацієнт з діареєю. Під час бактеріоскопічного дослідження фекальних мас виявили грамнегативні зігнуті палички. Яке захворювання можна припустити у хворого?

Сальмонельозний гастроентерит

Дифтерія

Кишкова форма чуми

Черевний тиф

Холера

537 / 6307
При дослідженні каріотипу 5-річної дівчинки виявлено 46 хромосом. Одна з хромосом 15-ї пари довша від звичайної, тому що до неї приєдналась хромосома з 21-ї пари. Який вид мутації має місце у цієї дівчинки?

Делеція

Транслокація

Дуплікація

Інверсія

Нестача

538 / 6307
Дитина 7 років не може відвести плече і підняти його до горизонтального рівня, до обличчя руку приводить лише тильною стороною при деякій абдукції плеча (за рахунок надостного м'яза) - рука “сурмача”. Активна функція якого м'яза

Малого круглого

Великого круглого

Дельтоподібного

Підостного

Великого грудного

539 / 6307
Трансмуральний інфаркт міокарда у хворого ускладнився розвитком гострої лівошлуночкової недостатності. Що є найбільш типовим для цього стану?

Набряк кінцівок

Ціаноз

Артеріальна гіпертензія

Набряк легенів

540 / 6307
У крові хворих на цукровий діабет спостерігається підвищення вмісту вільних жирних кислот. Причиною цього може бути:

Зниження активності фосфатидилхолін-холестерин-ацилтрансферази плазми E. Підвищення активності тригліцеридліпази адипоцитів

Накопичення в цитозолі пальмітоїл-КоА

Активація синтезу аполіпопротеїнів А-1, А-2, А-4

Активація утилізації кетонових тіл

541 / 6307
Для постановки туберкулінової проби дитині внутрішньошкірно введено туберкулін. Через 24 години в місці введення відзначена виражена гіперемія, ущільнення тканин. Який механізм лежить в основі розвитку даних змін?

Утворення гранульом

Імунокомплексна цитотоксичність

Цитотоксичність реагінового типу

Антитільна цитотоксичність

Клітинна цитотоксичність

542 / 6307
У жінки 63 років є ознаки ревматоїдного артриту. Підвищення рівня якого з перерахованих нижче показників крові буде найбільш значущим для підтвердження діагнозу?

Кислої фосфатази

R-глікозидази

Сумарних глікозаміногліканів

Ліпопротеїдів

Загального холестерину

543 / 6307
Чоловік 59 років має ознаки паренхіматозної жовтяниці та портальної гіпертензії. Під час гістологічного дослідження пункційного біоптату печінки знайдено: балково-часточкова будова порушена, частина гепатоцитів має ознаки жирової дистрофії, утворюються порто-портальні сполучнотканинні септи з формуванням псевдочасточок, з наявністю перипортальних лімфомакрофагальних інфільтратів. Який найбільш імовірний діагноз?

Вірусний гепатит

Цироз печінки

Токсична дистрофія

Хронічний гепатоз

Алкогольний гепатит

544 / 6307
Жінка 49 років тривалий час хворіла на хронічний гломерулонефрит, який призвів до смерті. На розтині встановлено, що нирки мають розміри 7х3х2.5 см, масу 65,0 г, щільні, дрібнозернисті. Мікроскопічно: фібринозне запалення серозних і слизових оболонок, дистрофічні зміни паренхіматозних органів, набряк головного мозку. Яке ускладнення призвело до вказаних змін серозних оболонок і

Уремія

Анемія

ДВЗ-снндром

Сепсис

Тромбоцитопенія

545 / 6307
У внутрішньоутробному періоді розвитку в судинній системі плода функціонує крупна артеріальна (боталова) протока, яка після народження перетворюється в lig. arteriosum. Які анатомічні утворення з’єднує між собою ця протока?

Легеневий стовбур та аорта

Легеневий стовбур та верхня порожниста вена

Аорта та верхня порожниста вена

Аорта та нижня порожниста вена

Праве та ліве передсердя

546 / 6307
Внаслідок активації іонних каналів зовнішньої мембрани збудливої клітини значно збільшився її потенціал спокою. Які канали були активовані?

Повільні кальцієві

Натрієві

Натрієві та кальцієві

Швидкі кальцієві

Калієві

547 / 6307
Біля інфікованої рани збільшилися регіонарні лімфовузли. При гістологічному дослідженні в них виявлено збільшення кількості макрофагів, лімфоцитів і лімфатичних фолікулів в кірковому шарі, а також велику кількість плазматичних клітин. Який процес в лімфатичних вузлах відображають виявлені гістологічні зміни?

Набуту недостатність лімфоїдної тканини

Природжену недостатність лімфоїдної тканини

Пухлинну трансформацію

Антигенну стимуляцію

Реакцію гіперчутливості

548 / 6307
Хворому 56 років із скаргами на спрагу та часте сечовиділення ендокринологом було встановлено діагноз цукрового діабету та призначено бутамід. Яким є механізм дії цього препарату?

Полегшує транспорт глюкози через клітинні мембрани

Сприяє засвоєнню глюкози клітинами тканин організму

Пригнічує альфа-клітини острівців Лангерганса

Стимулює бета-клітини острівців Лангерганса

Пригнічує всмоктування глюкози в кишечнику

549 / 6307
Хвора звернулася до травмпункту з приводу нагноєння різаної рани. Лікар для очищення рани від гнійних виділень промив її 3% розчином перекису водню. При цьому піна не утворилася. З чим пов'язана відсутність дії препарату?

Спадкова недостатність каталази

Неглибока рана

Низька концентрація H2O2

Спадкова недостатність фосфатдегідрогенази еритроцитів

Наявність у рані гнійного вмісту

550 / 6307
Хворий 62 років надійшов до неврологічного відділення з приводу мозкового крововиливу. Стан тяжкий. Спостерігається наростання глибини і частоти дихання, а потім його зменшення до апное, після чого цикл дихальних рухів відновлюється. Який тип дихання виник у хворого?

Апнеїстичне

Гаспінг–дихання

Кусмауля

Біота

Чейна–Стокса

551 / 6307
Хворому з гіперсекрецією шлункового соку лікар рекомендував виключити з дієти насичені бульйони і овочеві відвари, тому що вони стимулюють шлункову секрецію. Який переважний механізм стимуляції шлункової секреції у цьому випадку?

Подразнення механорецепторів ротової порожнини

Стимуляція вироблення секретину в 12-палій кишці

Подразнення смакових рецепторів

Подразнення механорецепторів шлунка

Стимуляція вироблення гастрину G-клітинами

552 / 6307
При дослідженні крові хворого виявлено значне збільшення активності МВ-форм КФК (креатинфосфокінази) та ЛДГ-1. Яка найбільш імовірна патологія?

Холецистит

Панкреатит

Ревматизм

Інфаркт міокарда

Гепатит

553 / 6307
До інфекційного відділення лікарні госпіталізовано хворого з діагнозом бактеріальної дизентерії. Лабораторними дослідженнями встановлено, що збудник чутливий до багатьох протимікробних засобів, однак у хворого виявлені явища анемії. Який препарат протипоказаний хворому?

Левоміцетин

Фталазол

Ампіцилін

Ентеросептол

Фуразолідон

554 / 6307
На мікропрепараті червоного кісткового мозку виявляються численні капіляри, через стінку яких у кровоносне русло виходять зрілі формені елементи крові. До якого типу належать ці капіляри ?

Соматичні

Фенестровані

Вісцеральні

Синусоїдні

Лімфатичні

555 / 6307
В експерименті на тварині видалення ділянки кори півкуль мозку усунуло раніше вироблені умовні рефлекси на світлове подразнення. Яку ділянку кори було видалено?

Постцентральна звивина

Прецентральна звивина

Лімбічна кора

Потилична кора

Скронева доля

556 / 6307
В експерименті на спинному мозку при збудженні альфа-мотонейронів м'язів-згиначів відзначено гальмування альфа-мотонейронів м'язів-розгиначів. Який вид гальмування лежить в основі цього явища?

Латеральне

Зворотне

Деполяризаційне

Пресинаптичне

Реципрокне

557 / 6307
Під час емоційного збудження частота серцевих скорочень у людини 30 років досягла 112 на хв. Який відділ провідної системи серця є відповідальним за цю зміну?

Синоатріальний вузол

Пучок Гіса

Волокна Пуркіньє

Ніжки пучка Гіса

Атріовентрикулярний вузол

558 / 6307
При розтині померлого від поширеного перитоніту в дистальних відділах тонкої кишки виявлено численні виразки овальної форми, які розташовані вздовж кишки. Дно виразок чисте, гладеньке, утворене м'язовою або серозною оболонкою, краї виразок рівні, закруглені. У двох виразках є перфоративні отвори діаметром до 0,5 см. Яке захворювання треба

Холера

Черевний тиф

Туберкульоз

Висипний тиф

Дизентерія

559 / 6307
Малюк попросив Вас надути повітряну кульку якомога дужче за один видих. Яким об'ємом повітря Ви скористаєтесь?

Життєвою ємністю легень

Функціональною залишковою ємністю

Ємністю вдиху

Резервним об'ємом вдиху

Загальною ємністю легень

560 / 6307
Під час вивчення родоводу сім'ї, в якій спостерігається гіпертрихоз (надмірне оволосіння вушних раковин), виявлено, що ця ознака зустрічається в усіх поколіннях тільки у чоловіків і успадковується від батька до сина. Визначте тип успадкування гіпертрихозу:

Зчеплений з Х-хромосомою рецесивний

Аутосомно-рецесивний

Аутосомно-домінантний

Зчеплений з Х-хромосомою домінантний

Зчеплений з Y-хромосомою

561 / 6307
Приймання оральних контрацептивів, які містять статеві гормони, пригнічує секрецію гормонів гіпофіза. Секреція якого з наведених гормонів пригнічується при прийманні оральних контрацептивів, які містять статеві гормони?

Окситоцин

Вазопресин

Фолікулостимулюючий

Тиреотропний

Соматотропний

562 / 6307
Внаслідок росту пухлини у порожнину III шлуночка головного мозку у пацієнта розвиваються вегетативні розлади у вигляді порушення сну, терморегуляції, усіх видів обміну, нецукрового діабету. Подразнення ядер якої ділянки головного мозку викликає ці симптоми?

Гіпоталамус

Довгастий мозок

Міст

Покришка середнього мозку

Ніжки мозку

563 / 6307
У хворого виявлена болючість по ходу великих нервових стволів та підвищений вміст пірувату в крові. Нестача якого вітаміну може викликати такі зміни?

РР

Біотин

Пантотенова кислота

В2

В1

564 / 6307
Територію старого худобомогильника, який не використовувався більше 50 років, планується відвести під житлове будівництво. Однак дослідження грунту виявило наявність життєздатних спор збудника особливо небезпечного захворювання. Який із вказаних мікроорганізмів найбільш імовірно міг зберігатися у грунті протягом такого тривалого

Yersinia pestis

Brucella abortus

Bacillus anthracis

Francisella tularensis

Mycobacterium bovis

565 / 6307
Дитина 5 років надійшла до ЛОР-відділення з діагнозом гнійне запалення середнього вуха. Захворювання розпочалося із запалення носоглотки. Через який канал скроневої кістки інфекція потрапила в барабанну порожнину?

Сонний канал

М’язовотрубний канал

Барабанний каналець

Сонно-барабанні канальці

Каналець барабанної струни

566 / 6307
Хворий на цукровий діабет вчасно не отримав ін’єкцію інсуліну, що призвело до розвитку гіперглікемічної коми (вміст глюкози в крові – 50 ммоль/л). Який механізм є головним у розвитку цієї коми?

Гіпокаліємія

Ацидоз

Гіперосмія

Гіпоксія

Гіпонатріємія

567 / 6307
У тяжко травмованої людини поступово настала біологічна смерть. Свідченням цього є:

Відсутність рухливості

У клітинах відбувається автоліз і розкладання

Відсутність серцебиття і дихання

Невпорядкованість хімічних процесів

Непритомність

568 / 6307
У жінки 68 років після інсульту відсутні рухи в верхній та нижній правій кінцівках. Тонус м’язів цих кінцівок і рефлекси в них підвищені. Є патологічні рефлекси. Яка це форма паралічу?

Тетраплегія

Геміплегія

Моноплегія

Дисоціація

Параплегія

569 / 6307
У хворого з верхнім типом ожиріння клінічно тривало відзначалися артеріальна гіпертонія, гіперглікемія, глюкозурія. Смерть настала від крововиливу в головний мозок. Під час патоморфологічного дослідження виявлені базофільна аденома гіпофіза, гіперплазія кори наднирників. Який найбільш імовірний діагноз?

Акромегалія

Гіпофізарний нанізм

Хвороба Іценко-Кушінга

Адипозогенітальна дистрофія

Цукровий діабет

570 / 6307
У хворого після важкої травми розвинувся шок та з’явилися ознаки гострої ниркової недостатності [ГНН]. Що є провідним механізмом розвитку ГНН в даному випадку?

Порушення відтоку сечі

Підвищення тиску в капсулі нефрону

Падіння артеріального тиску

Підвищення тиску в ниркових артеріях

Зниження онкотичного тиску крові

571 / 6307
У людини внаслідок тривалого голодування швидкість клубочкової фільтрації зросла на 20%. Найбільш імовірною причиною змін фільтрації в зазначених умовах є:

Зменшення онкотичного тиску плазми крові

Збільшення коефіцієнту фільтрації

Збільшення проникності ниркового фільтру

Збільшення ниркового плазмотоку

Збільшення системного артеріального тиску

572 / 6307
При розтині померлої в комі молодої людини виявлено поширений тромбоемболічний інфаркт лівої півкулі мозку, велика септична селезінка, імунокомплексний гломерулонефрит, виразки в стулках аортального клапана, прикриті поліпоподібними тромбами з колоніями стафілококів. Яке захворювання викликало церебральну тромбоемболію?

Ревматичний тромбоендокардит

Септицемія

Гострий ревматичний вальвуліт

Септичний бактеріальний ендокардит

Септикопіємія

573 / 6307
Жінці з дисфункціональною матковою кровотечею зробили діагностичне вишкрябання. Гістологічно у зіскрібку виявлено велику кількість звивистих залоз, просвіти окремих залоз кістозно розширені. Назвіть різновид загальнопатологічного процесу в ендометрії:

Метаплазія

Залозисто-кістозна гіперплазія

Гіпертрофічні розростання

Дисплазія

Атрофія

574 / 6307
У людини частота серцевих скорочень утримується на рівні, що не перевищує 40 разів за хвилину. Що є водієм ритму серця у цієї людини?

Ніжки пучка Гіса

Пучок Гіса

Синоатріальний вузол

Атріовентрикулярний вузол

Волокна Пуркіньє

575 / 6307
У хворого виявлено порушення прохідності дихальних шляхів на рівні дрібних і середніх бронхів. Які порушення кислотно-лужної рівноваги можна виявити у крові в даному

Респіраторний алкалоз

Респіраторний ацидоз

Метаболічний ацидоз

Метаболічний алкалоз

576 / 6307
У хворих з непрохідністю жовчовивідних шляхів пригнічується зсідання крові, виникають кровотечі, що є наслідком недостатнього засвоєння вітаміну:

Каротину

Е

К

А

D

577 / 6307
На розтині тіла померлого виявлено, що вся права легеня збільшена у розмірі, щільна, на плеврі нашарування фібрину, на розрізі тканина легені сірого кольору, з неї стікає каламутна рідина. Для якого захворювання легень характерна така картина?

Крупозна пневмонія

Гангрена легені

Вогнищева пневмонія

Фіброзуючий альвеоліт

Інтерстиціальна пневмонія

578 / 6307
У людини вимірюють внутрішньоплевральний тиск. У якій фазі людина затримала дихання, якщо величина тиску дорівнює – 25 см вод.ст.?

Форсований вдих

Спокійний видих

Спокійний вдих

Форсований видих

579 / 6307
У хворого 27 років виявлено патологічні зміни печінки і головного мозку. У плазмі крові виявлено різке зниження, а в сечі підвищення вмісту міді. Поставлено діагноз - хвороба Вільсона. Активність якого ферменту в сироватці крові необхідно дослідити для підтвердження діагнозу?

Лейцинамінопептидази

Карбоангідрази

Алкогольдегідрогенази

Ксантиноксидази

Церулоплазміну

580 / 6307
До лікаря звернувся хворий зі скаргами на постійну спрагу. Виявлена гіперглікемія, поліурія та підвищений вміст 17-кетостероїдів у сечі. Яке захворювання найбільш імовірне?

Аддісонова хвороба

Інсулінозалежний діабет

Мікседема

Стероїдний діабет

Глікогеноз I типу

581 / 6307
Під час обіду дитина похлинулася і аспірувала їжу. Почався сильний кашель, шкіра і слизові ціанотичні, пульс прискорений, дихання рідке, видих подовжений. Яке порушення зовнішнього дихання розвинулось у дитини?

Альтернуюче дихання

Дихання Біота

Стадія експіраторної задишки при асфіксії

Стенотичне дихання

Стадія інспіраторної задишки при асфіксії

582 / 6307
У лабораторії при мікроскопії харкотиння хворого на пневмонію випадково виявлені личинки. При аналізі крові виявлена еозинофілія. Який гельмінтоз можна передбачити?

Аскаридоз

Трихоцефальоз

Парагонімоз

Ентеробіоз

Опісторхоз

583 / 6307
У хворого з тимомою (пухлиною вилочкової залози) спостерігається ціаноз, розширення підшкірної венозної сітки і набряк м’яких тканин обличчя, шиї, верхньої половини тулуба і верхніх кінцівок. Який венозний стовбур перетиснено

Передня яремна вена

Зовнішня яремна вена

Верхня порожниста вена

Внутрішня яремна вена

Підключична вена

584 / 6307
Під час дослідження первинної структури молекули глобіну виявлено заміну глутамінової кислоти на валін. Для якої спадкової патології це характерно?

Гемоглобіноз

Серпоподібноклітинна анемія

Хвороба Мінковського-Шоффара

Фавізм

Таласемія

585 / 6307
При травмі у пацієнта 44 років виник розрив сухожилків м’язів лівої долоні, поверхневих кровоносних судин. Після оперативного втручання і видалення більшої частини некротично зміненої м’язової тканини кровотік був відновлений. За рахунок яких судин?

Arcus palmaris profundus

Arcus palmaris superficialis

A. digitales palmares communes

A. metacarpeae palmares

A. perforantes

586 / 6307
У хворої, яка страждає на тромбофлебіт глибоких вен гомілки, раптово настала смерть. На розтині трупа у загальному стовбурі і біфуркації легеневої артерії знайдено червоні пухкі маси з тьмяною гофрованою поверхнею, що лежать вільно. Який патологічний процес виявив патологоанатом?

Тканинна емболія

Тромбоемболія

Жирова емболія

Емболія стороннім тілом

Тромбоз

587 / 6307
При оформленні дитини до школи для вирішення питання про необхідність ревакцинації поставлено пробу Манту, яка виявилася негативною. Про що свідчить такий результат

Про відсутність антитоксичного імунітету до туберкульозу

Про наявність антитіл до туберкульозних бактерій

Про відсутність антитіл до туберкульозних бактерій

Про наявність клітинного імунітету до туберкульозу

Про відсутність клітинного імунітету до туберкульозу

588 / 6307
Під час бактеріологічного дослідження промивних вод хворого на харчове отруєння висіяли чисту культуру бактерій з такими властивостями: грамнегативна рухлива паличка, на середовищі Ендо росте у вигляді безбарвних колоній. Представником якого роду було зумовлене захворювання?

Citrobacter

Escherichia

Salmonella

Shigella

Yersinia

589 / 6307
До клініки потрапив чоловік 54 років зі скаргами на болі в правій підреберній ділянці, блювоту з кров'ю. Об'єктивно: збільшення розмірів печінки, варикозне розширення вен стравоходу і шлунка, кровотеча з них. Порушення функції якої судини, ймовірніше за все, мало місце?

Aorta abdominalis

Vena cava inferior

Vena hepatica

Vena porta

Vena cava superior

590 / 6307
Під час обстеження донора, який тривалий час не здавав кров, за допомогою методу ІФА виявлені анти-НBs антитіла. Про що свідчить у цьому випадку позитивний результат ІФА?

Про хронічний гепатит В

Про гострий гепатит В

Про хронічний гепатит С

Про гострий гепатит С

Про перенесений гепатит В

591 / 6307
Дитина вдихнула ґудзик. Куди найбільш імовірно він потрапить?

У стравохід

У трахею

У гортань

У лівий головний бронх

У правий головний бронх

592 / 6307
У молодої людини в м'яких тканинах лівого стегна з'явилось безболісне новоутворення без чітких меж. В біоптаті тканини новоутворення нагадують риб'яче м'ясо, складаються з незрілих фібробластоподібних клітин з численними мітозами, які проростають в м'язи. Який найбільш вірогідний діагноз?

Рак

Фібросаркома

Міома

Фіброма

Міосаркома

593 / 6307
У дитини 2 років виникли судоми внаслідок зниження концентрації іонів кальцію в плазмі крові. Це зумовлено зниженням функції:

Тимуса

Шишкоподібної залози

Кори наднирників

Прищитоподібних залоз

Гіпофіза

594 / 6307
У процесі метаболізму в організмі людини утворюються активні форми кисню, у тому числі супероксидний аніон-радикал. За допомогою якого ферменту інактивується цей аніон?

Каталаза

Пероксидаза

Глутатіонпероксидаза

Глутатіонредуктаза

Супероксиддисмутаза

595 / 6307
У загальному вигляді генетичний апарат еукаріот є таким: екзон – інтрон – екзон. Така структурно-функціональна організація гена зумовлює особливості транскрипції. Якою буде про-іРНК відповідно до згаданої схеми?

Екзон-інтрон-екзон

Екзон-екзон-інтрон

Екзон-інтрон

Інтрон-екзон

Екзон-екзон

596 / 6307
При отруєнні аманітином (отрутою блідої поганки) блокується РНКполімераза В(ІІ). При цьому припиняється:

Зворотна транскрипція

Дозрівання мРНК

Синтез мРНК

Синтез праймерів

Синтез тРНК

597 / 6307
Чоловіку 70 років, який страждає на хронічний бронхіт, призначений протикашльовий препарат - кодеїн. Який механізм забезпечує протикашльовий ефект?

Периферична дія

Рефлекторний

Конкурентний

Центральний

Місцева дія

598 / 6307
Хворий 46 років скаржиться на утруднення носового дихання. У біоптаті потовщеної слизової носа знайдені клітини Мікуліча, скупчення епітеліоїдних клітин, плазмоцити, лімфоцити, гіалінові кулі. Який найбільш імовірний діагноз?

Аденовірусний риніт

Склерома

Алергічний риніт

Риновірусна інфекція

Менінгококовий назофарингіт

599 / 6307
З випорожнень хворої дитини 6-місячного віку, яка знаходилась на штучному вигодовуванні, виділена культура кишкової палички з антигенною структурою 0-111. Який діагноз можна апідозрити?

Колі-ентерит

Харчове отруєння

Дизентерієподібне захворювання

Холероподібне захворювання

Гастроентерит

600 / 6307
На препараті яєчника, забарвленому гематоксиліном-еозіном, визначається фолікул, в якому клітини фолікулярного епітелію розміщені в 1-2 шари і мають кубічну форму, навколо овоцита видно оболонку яскраво-червоного кольору. Назвіть цей фолікул:

Вторинний

Первинний

Атретичний

Примордіальний

Зрілий

601 / 6307
Пацієнт під час роботи швидко стомлюється. У положенні стоячи з заплющеними очима він похитується, втрачає рівновагу. Тонус скелетних м'язів знижений. Яка з наведених структур мозку уражена у цієї людини?

Мозочок

Базальні ганглії

Гіпоталамус

Прецентральна звивина кори великих півкуль

Таламус

602 / 6307
На розтині тіла чоловіка 35 років у другому сегменті правої легені виявлено вогнище ущільнення діаметром 5 см, оточене тонкою капсулою. Вогнище представлене сухою крихкою тканиною з тьмяною поверхнею. Для якого захворювання характерні такі морфологічні зміни?

Хондрома

Туберкульома

Післязапальний пневмосклероз

Рак легені

Туморозна форма силікозу

603 / 6307
Під час розтину тіла померлого 58 років знайдено, що мітральний клапан деформований, потовщений, змикається не до кінця. Мікроскопічно: вогнища колагенових волокнинок еозинофільні, дають позитивну реакцію на фібрин. Найвірогідніше це:

Амілоїдоз

Гіаліноз

Фібринозне запалення

Мукоїдне набухання

Фібриноїдне набухання

604 / 6307
У чоловіка 42 років, який страждає на подагру, в крові підвищена концентрація сечової кислоти. Для зниження рівня сечової кислоти йому призначено алопуринол. Конкурентним інгібітором якого ферменту є алопуринол?

Гуаніндезаміназа

Гіпоксантинфосфорибозилтрансфераза

Аденінфосфорибозилтрансфераза

Аденозиндезаміназа

Ксантиноксидаза

605 / 6307
У юнака енерговитрати збільшились з 500 до 2000 кДж за годину. Що з наведеного може бути причиною цього?

Розумова праця

Прийом їжі

Підвищення зовнішньої температури

Перехід від сну до бадьорості

Фізичне навантаження

606 / 6307
Під час розтину тіла померлої в комі молодої людини виявлено поширений тромбоемболічний інфаркт лівої півкулі мозку, велика септична селезінка, імунокомплексний гломерулонефрит, виразки в стулках аортального клапана, прикриті поліпоподібними тромбами з колоніями стафілококів. Яке захворювання викликало церебральну тромбоемболію?

Септикопіємія

Септичний бактеріальний ендокардит

Ревматичний тромбоендокардит

Септицемія

Гострий ревматичний вальвуліт

607 / 6307
У чоловіка 60 років після крововиливу в головний мозок настав тривалий сон. Пошкодження якої структури найімовірніше призвело до цього стану?

Ядра черепних нервів

Ретикулярна формація

Гіпокамп

Чорна субстанція

Кора великих півкуль

608 / 6307
Тестостерон і його аналоги збільшують масу скелетних м'язів, що дозволяє використовувати їх для лікування дистрофій. Взаємодією з яким клітинним субстратом зумовлена ця дія?

Мембранні рецептори

Хроматин

Ядерні рецептори

Білки-активатори транскрипції

Рибосоми

609 / 6307
При жировій інфільтрації печінки порушується синтез фосфоліпідів. Яка з перелічених речовин може посилювати процеси метилювання в синтезі фосфоліпідів?

Аскорбінова кислота

Гліцерин

Метіонін

Глюкоза

Цитрат

610 / 6307
Під час обстеження людини встановлено, що хвилинний об'єм крові дорівнює 3500 мл, систолічний об'єм - 50 мл. Якою є частота серцевих скорочень у цієї людини?

70 скорочень за хвилину

50 скорочень за хвилину

60 скорочень за хвилину

90 скорочень за хвилину

80 скорочень за хвилину

611 / 6307
Чоловік 40 років пробіг 10 км за 60 хвилин. Які зміни енергетичного обміну відбудуться у його м'язах?

Посилиться протеоліз

Посилиться гліколіз

Посилиться глюконеогенез

Посилиться глікогеноліз

Збільшиться швидкість окислення жирних кислот

612 / 6307
У результаті виснажливої м'язової роботи у робітника значно зменшилась буферна ємність крові. Надходженням якої кислої речовини до крові можна пояснити це явище?

Лактат

α-кетоглутарат

Піруват

3-фосфогліцерат

1,3-бісфосфогліцерат

613 / 6307
До фібрилярних елементів сполучної тканини належать колаген, еластин та ретикулін. Вкажіть амінокислоту, яка входить тільки до складу колагену, і визначення якої в біологічних рідинах використовується для діагностики захворювань сполучної тканини?

Фенілаланін

Гідроксипролін

Пролін

Лізин

Гліцин

614 / 6307
У тяжко травмованої людини поступово настала біологічна смерть. Свідченням цього є:

Невпорядкованість хімічних процесів

Відсутність серцебиття і дихання

У клітинах відбувається автоліз і розкладання

Непритомність

Відсутність рухливості

615 / 6307
Хворому на тромбофлебіт призначено комплексну терапію, яка діє на різні етапи тромбоутворення. Яке з перелічених засобів сприяє відновленню прохідності судини?

Неодикумарин

Гепарин

Дипіридамол

Фібринолізин

Ацетилсаліцилова кислота

616 / 6307
Жінка 42 років із невралгією трійчастого нерва скаржиться на періодичне почервоніння правої половини обличчя та шиї, відчуття припливу тепла та підвищення шкірної чутливості. Якою у цьому випадку є артеріальна гіперемія за патофізіологічним механізмом?

Метаболічна

Реактивна

Нейропаралітична

Робоча

Нейротонічна

617 / 6307
У мікропрепараті, виготовленому з пунктату регіонарного лімфовузла хворого та зафарбованому за Романовським-Гімзою, лікар виявив тонкі мікроорганізми з 12-14 рівномірними завитками, з гострими кінцями, довжиною 10-13 мкм блідо-рожевого кольору. Про збудника якої інфекційної хвороби може йти мова у даному випад- ку?

Сифіліс

Поворотний тиф

Трипаносомоз

Лейшманіоз

Лептоспіроз

618 / 6307
Група чоловіків звернулася до лікаря зі скаргами на підвищення температури, головні болі, набряки повік та обличчя, болі в м'язах. З анамнезу стало відомо, що всі вони мисливці і часто вживають в їжу м'ясо диких тварин. Який найбільш імовірний діагноз?

Філяріатоз

Цистицеркоз

Трихінельоз

Теніоз

Теніаринхоз

619 / 6307
У загальному вигляді генетичний апарат еукаріот є таким: екзон-інтрон-екзон. Така структурно-функціональна організація гена зумовлює особливості транскрипції. Якою буде про-іРНК відповідно до згаданої схеми?

Екзон-інтрон-екзон

Екзон-інтрон

Екзон-екзон

Екзон-екзон-інтрон

Інтрон-екзон

620 / 6307
Хворий 43 років чотири місяця тому переніс травматичну ампутацію лівої нижньої кінцівки. На момент огляду скаржиться на відчуття наявності ампутованої кінцівки і постійний сильний, іноді нестерпний біль у ній. Який вид болю у хворого?

Фантомний

Невралгія

Каузалгія

Таламічний

Рефлекторний

621 / 6307
Під час футбольного матчу гравець отримав травму колінного суглоба. На рентгенівському знімку виявлено перелом кістки, що лежить у товщі сухожилка чотириголового м'яза стегна. До якої з перелічених груп кісток належить ця кістка?

Трубчасті

Сесамоподібні

Повітроносні

Змішані

Плоскі

622 / 6307
У пацієнта після травми виникли паралічі, розлади больової чутливості справа; зліва - паралічі відсутні, але порушена больова і температурна чутливість. Яка причина такого явища?

Пошкодження стовбура мозку

Однобічне пошкодження спинного мозку з правого боку

Пошкодження рухової зони кори головного мозку

Пошкодження середнього мозку

Пошкодження мозочка

623 / 6307
Під час дослідження сироватки крові у пацієнта з ознаками імунодефіциту виявлено антитіла до білків gP120 і gP41. Наявність якої інфекції у хворого підтверджує цей результат?

ЕСНО-інфекція

ВІЛ-інфекція

HBV-інфекція

TORCH-інфекція

HLTV-1-інфекція

624 / 6307
У дитини було діагностовано перелом плечової кістки. Зламана рука почала відставати в рості. Яка частина кістки постраждала?

Діафіз

Метафіз

Кістковомозковий канал

Апофіз

Епіфіз

625 / 6307
Через деякий час після інтенсивного фізичного тренування у спортсмена активується глюконеогенез. Що є його основним субстратом?

Аспарагінова кислота

Глутамінова кислота

Серин

α-кетоглутарат

Лактат

626 / 6307
До інфекційного відділення лікарні госпіталізовано хворого з діагнозом бактеріальної дизентерії. Лабораторними дослідженнями встановлено, що збудник чутливий до багатьох протимікробних засобів, однак у хворого виявлені явища анемії. Який препарат протипоказаний хворому?

Фуразолідон

Ампіцилін

Ентеросептол

Левоміцетин

Фталазол

627 / 6307
У людини частота серцевих скорочень утримується на рівні, що не перевищує 40 разів за хвилину. Що є водієм ритму серця у цієї людини?

Пучок Гіса

Волокна Пуркіньє

Атріовентрикулярний вузол

Ніжки пучка Гіса

Синоатріальний вузол

628 / 6307
У пацієнта під час відвідування стоматолога виникла різка гіпотензія. Який із пре- паратів, що стимулює адренергічні структури, слід використати для нормалізації артеріального тиску?

Мезатон

Ксилометазолін

Доксазозин

Санорин

Ерготамін

629 / 6307
Прозерин при систематичному введенні щуру підвищує тонус скелетних м'язів. Фторотан викликає релаксацію скелетних м'язів і послаблює ефекти прозерину. Визначте характер взаємодії прозерину і фторотану.

Незалежний антагонізм

Прямий функціональний антагонізм

Непрямий функціональний антагонізм

Конкурентний антагонізм

Неконкурентний антагонізм

630 / 6307
У зв'язку з підозрою на внутрішньолікарняну інфекцію у відділенні новонароджених пологового будинку проведено обстеження. У кількох дітей, а також на деяких предметах догляду виявлено золотистий стафілокок. Які властивості виділених культур дають можливість встановити їхнє походження з одного джерела?

Антигенна структура

Пігментоутворення

Антибіотикограма

Біохімічна активність

Фаготип

631 / 6307
На базарі громадянин продавав ковбасу під назвою 'свиняча домашня'. У держсанінспекції виникла підозра фальсифікації ковбаси . За допомогою якої серологічної реакції імунітету можна ідентифікувати харчовий продукт?

Імунофлуоресценції

Аглютинації

Преципітації

PЗK Е. РНГА

632 / 6307
У хворого після важкої травми розвинувся шок та з'явилися ознаки гострої ниркової недостатності [ГНН]. Що є провідним механізмом розвитку ГНН у даному випадку?

Порушення відтоку сечі

Підвищення тиску в ниркових артеріях

Зниження онкотичного тиску крові

Падіння артеріального тиску

Підвищення тиску в капсулі нефрону

633 / 6307
Студент старанно конспектує лекцію. Якість конспектування значно погіршилася, коли сусіди стали розмовляти. Який вид гальмування в корі головного мозку є причиною цього?

Зовнішнє

Позамежне

Згасаюче

Диференціювальне

Запізніле

634 / 6307
У хворого 39 років після променевої терапії з приводу пухлини печінки утворилася виразка тонкої кишки внаслідок пригнічення мітотичної активності клітин, за рахунок яких відбувається поновлення покривного епітелію тонкої кишки. Мітотичну активність яких клітин пригнічено у даного хворого?

Екзокриноцитів з ацидофільною зернистістю

Ендокринних клітин

Стовпчастих клітин крипт без облямівки

Келихоподібних екзокриноцитів

Стовпчастих епітеліоцитів

635 / 6307
Хворій 34 років 3 роки тому було встановлено діагноз гломерулонефриту. За останні 6 місяців з'явилися набряки. Що лежить в основі їх розвитку?

Протеїнурія

Порушення білковоутворюючої функції печінки

Гіперальдостеронізм

Гіперосмолярність плазми

Гіперпродукція вазопресину

636 / 6307
Під час бактеріологічного дослідження гнійних виділень з уретри було встановлено наявність бактерій, які за Грамом фарбувалися негативно, нагадували кавові зернини, розщеплювали глюкозу і мальтозу до кислоти, знаходилися всередині лейкоцитів. Збудниками якої хвороби є ці мікроорганізми?

Венеричний лімфогранулематоз

Меліоїдоз

М'який шанкр

Сифіліс

Гонорея

637 / 6307
Проводиться каріотипування клітин здорової людини. У каріотипі знайдено дрібну акроцентричну непарну хромосому. Якою хромосомою вона може бути?

Хромосома групи С

Y-хромосома

Хромосома групи В

Х-хромосома

Хромосома групи А

638 / 6307
При травмі у пацієнта 44 років виник розрив сухожилків м'язів лівої долоні, поверхневих кровоносних судин. Після оперативного втручання і видалення більшої частини некротично зміненої м'язової тканини кровотік був відновлений. За рахунок яких судин?

Аа. metacarpeae palmares

Аа. perforantes

Arcus palmaris profundus

Аа. digitales palmares communes

Arcus palmaris superficialis

639 / 6307
У хворого 30 років з різаною раною передпліччя виникло порушення розгинання пальців кисті. Про пошкодження якого нерва це свідчить?

Ліктьовий

Медіальний шкірний нерв передпліччя

Серединний

Променевий

М'язовошкірний

640 / 6307
До приймального відділення доставлено хворого у непритомному стані. Шкіра холодна, зіниці звужені, дихання з утрудненням, відзначається періодичність по типу Чейна-Стокса, артеріальний тиск знижений, сечовий міхур переповнений. Отруєння якою речовиною найбільш вірогідне?

М-холіноблокаторами

Ненаркотичними анальгетиками

Транквілізаторами

Наркотичними анальгетиками

641 / 6307
Малюк попросив Вас надути повітряну кульку якомога дужче за один видих. Яким об'ємом повітря Ви скористаєтесь?

Резервний об'єм вдиху

Функціональна залишкова ємність

Ємність вдиху

Загальна ємність легень

Життєва ємність легень

642 / 6307
Жінка 33 років, яка тривалий час лікується з приводу хронічного поліартриту, скаржиться на підвищення артеріального тиску, зміни розподілу жирової тканини, порушення менструального циклу. З прийманням якого препарату пов'язані ці скарги?

Індометацин

Преднізолон

Синафлан

Беклометазон

Бутадіон

643 / 6307
Хворому на сечокам'яну хворобу після обстеження призначили алопуринол - конкурентний інгібітор ксантиноксидази. Підставою для цього був хімічний аналіз ниркових каменів, які складалися переважно з:

Дигідрату оксалату кальцію

Урату натрію

Моногідрату оксалату кальцію

Сульфату кальцію

Фосфату кальцію

644 / 6307
У людини хірургічно видалили ушкоджену патологічним процесом дистальну чверть тонкої кишки. Як це позначиться на всмоктуванні поживних речовин при звичайному харчовому раціоні?

Зменшиться всмоктування вуглеводів

Всмоктування суттєво не зміниться

Зменшиться всмоктування жирів

Зменшиться всмоктування води

Зменшиться всмоктування білків

645 / 6307
Під час дослідження біоптату шкіри хворого на алергічний васкуліт знайдено: стінка судин потовщена, гомогенна, пікрофуксином забарвлюється у жовтий колір, ШІК-позитивна. Який патологічний процес розвинувся в стінках судин?

Гіаліноз

Мукоїдне набухання

Фібриноїдне набухання

Амілоїдоз

Ліпідоз

646 / 6307
У людини внаслідок патологічного процесу збільшена товщина альвеолокапілярної мембрани. Безпосереднім наслідком цього буде зменшення:

Хвилинного об'єму дихання

Альвеолярної вентиляції легень

Дифузійної здатності легень

Кисневої ємкості крові

Резервного об'єму видиху

647 / 6307
З метою серологічної діагностики коклюшу поставлена розгорнута реакція з коклюшним та паракоклюшним діагностикумами. На дні пробірок, до яких було внесено діагностикум з Bordetella parapertussis, утворився зернистий осад. Які антитіла виявила ця реакція?

Преципітини

Антитоксини

Бактеріолізини

Опсоніни

Аглютиніни

648 / 6307
У хворих на цукровий діабет і під час голодування в крові збільшується вміст ацетонових тіл, що використовуються в якості енергетичного матеріалу. З якої речовини вони синтезуються?

Кетоглутарат

Ацетил-КоА

Малат

Цитрат

Сукциніл-КоА

649 / 6307
На мікропрепараті червоного кісткового мозку виявляються численні капіляри, через стінку яких у кровоносне русло виходять зрілі формені елементи крові. До якого типу належать ці капіляри?

Фенестровані

Соматичні

Лімфатичні

Вісцеральні

Синусоїдні

650 / 6307
Хворий помер при явищах уремії. На розтині нирки збільшені, в'ялої консистенції, кірковий шар широкий, набряклий, з червоним крапом; мозкова речовина темночервона. Мікроскопічно у порожнині капсули клубочків виявлені 'півмісяці', які стискають капіляри, дистрофія нефроцитів, набряк та інфільтрація строми. Яке захворювання було причиною смерті цього чоловіка?

Пієлонефрит

Нефротичний синдром

Нефролітіаз

Амілоїдоз нирок

Гломерулонефрит

651 / 6307
Під час обстеження хворого виявлене новоутворення у білій речовині півкуль великого мозку з локалізацією у коліні та передньому відділі задньої ніжки внутрішньої капсули. Волокна якого провідного шляху будуть зруйновані?

Tr. parietooccipitopontinus

Tr. frontopontinus

Tr. frontothalamicus

Tr. thalamocorticalis

Tr. pyramidalis

652 / 6307
Під час обстеження юнака з розумовою відсталістю виявлено євнухоїдну будову тіла, недорозвиненість статевих органів. У клітинах порожнини рота - статевий хроматин. Який метод генетичного дослідження слід застосувати для уточнення діагнозу?

Біохімічний

Цитологічний

Популяційно-статистичний

Дерматогліфіку

Клініко-генеалогічний

653 / 6307
У хворого лікар діагностував гостру гонорею. З анамнезу стало відомо, що раніше він переніс гонорею і вилікування було повним. До якої категорії інфекцій можна віднести це нове захворювання?

Суперінфекція

Автоінфекція

Вторинна інфекція

Рецидив

Реінфекція

654 / 6307
Недбалий студент раптово зустрівся з деканом. Концентрація якого гормону найшвидше збільшиться в крові студента?

Соматотропін

Кортикотропін

Кортизол

Адреналін

Тироліберин

655 / 6307
Хворий 50 років скаржиться на спрагу, п'є багато води, виражена поліурія. Глюкоза крові 4,8 ммоль/л, в сечі глюкози і ацетону немає, сеча безбарвна, питома вага 1,002-1,004. Яка причина поліурії?

Нестача вазопресину

Інсулінова недостатність

Альдостеронізм

Гіпотиреоз

Тиреотоксикоз

656 / 6307
Після аналізу родоводу, лікар – генетик встановив: ознака проявляється у кожному поколінні, жінки та чоловіки успадковують ознаку з однаковою частотою, батьки в однаковій мірі передають ознаку своїм дітям. Який тип успадкування має досліджувана ознака?

Х-зчеплене рецесивне успадкування

Y-зчеплене успадкування

Х-зчеплене домінантне успадкування

Аутосомно- домінантний

Аутосомно- рецесивний

657 / 6307
Чоловік 59 років має ознаки паренхіматозної жовтяниці та портальної гіпертензії. Під час гістологічного дослідження пункційного біоптату печінки знайдено: балково-часточкова будова порушена, частина гепатоцитів має ознаки жирової дистрофії, утворюються порто-портальні сполучнотканинні септи з формуванням псевдочасточок, з наявністю перипортальних лімфо-макрофагальних інфільтратів. Який найбільш імовірний діагноз?

Токсична дистрофія

Цироз печінки

Хронічний гепатоз

Вірусний гепатит

Алкогольний гепатит

658 / 6307
В експерименті у кроля було видалено верхній шийний вузол симпатичного стовбура. На боці видалення спостерігається почервоніння і підвищення температури шкіри голови. Яка форма порушень периферичного кровообігу розвинулася в цьому випадку?

Венозна гіперемія

Метаболічна артеріальна гіперемія

Нейропаралітична артеріальна гіперемія

Стаз

Нейротонічна артеріальна гіперемія

659 / 6307
Забір крові для загального аналізу рекомендують проводити натщесерце і зранку. Які зміни складу крові можливі, якщо провести забір крові після прийому їжі?

Збільшення кількості лейкоцитів

Збільшення кількості еритроцитів

Зниження кількості еритроцитів

Збільшення білків плазми

Зниження кількості тромбоцитів

660 / 6307
У хворого із значними периферійними набряками почергове застосування дихлотіазиду, етакринової кислоти і фуросеміду не сприяло значному діуретичному ефекту. У крові - значне підвищення кількості альдостерону. Вкажіть препарат вибору.

Сечовина

Амілорид

Спіронолактон

Клопамід

Маніт

661 / 6307
Під час мікроскопії біоптату печінки виявлені гранульоми, які складаються з плазматичних, лімфоїдних, гігантських багатоядерних клітин типу Пирогова- Ланхганса, дрібних судин з ознаками ендо- і периваскуліту, зустрічаються осередки казеозного некрозу. Для якого захворювання характерно утворення таких гранульом?

Лепра

Риносклерома

Сифіліс

Сап

Туберкульоз

662 / 6307
Хвора 25 років звернулася зі скаргами на погіршення зору. При огляді виявлено порушення акомодації, зіниця розширена, не реагує на світло. Функція яких м'язів порушена?

М'яз, що розширює зіницю, війковий

Верхній косий, війковий

М'яз, що звужує і м'яз, що розширює зіницю

М'яз, що звужує зіницю, війковий

Латеральний прямий, м'яз, що звужує зіницю

663 / 6307
У 6-місячної дитини спостерігалися часті та інтенсивні підшкірні крововиливи. Призначення синтетичного аналога вітаміну К (вікасолу) дало позитивний ефект. В гама-карбоксилюванні глутамінової кислоти якого білку згортаючої системи крові бере участь цей вітамін?

Фібриноген

Фактор Розенталя

Фактор Хагемана

Протромбін

Антигемофільний глобулін А

664 / 6307
У результаті землетрусу чоловік 50 років два дні перебував під завалом. Після звільнення з-під завалу у нього було встановлено діагноз синдрому тривалого розчавлення. Виникнення якого ускладнення в подальшому найбільш імовірне у цьому випадку?

Гостра дихальна недостатність

Гостра печінкова недостатність

Гостра серцева недостатність

Гостра ниркова недостатність

Гостра судинна недостатність

665 / 6307
У хворого 27 років діагностовано гнійне запалення жовчного міхура. До якого відділу очеревинної порожнини потрапить гній під час розриву жовчного міхура при його типовому положенні.

До передшлункової сумки

До печінкової сумки

До лівого бічного каналу

До верхнього дванадцятипалого закутка

До чепцевої сумки

666 / 6307
До хірурга звернулася хвора з приводу наявності ущільнення у молочній залозі. У якому напрямку має здійснити розріз хірург під час операції для зменшення травмування часточки?

Поперечно

Вертикально

Дугоподібно

Радіально

667 / 6307
Хворий помер на 3-й день після операції з приводу перфорації стінки товстої кишки з явищами розлитого гнійного перитоніту. На розтині: слизова оболонка стінки товстої кишки потовщена, вкрита фібриновою плівкою , поодинокі виразки проникають на різну глибину. Гістологічно: некроз слизової, інфільтрація лейкоцитами з фокусами геморагій. Ускладнення якого захворювання стало причиною смерті хворого?

Дизентерія

Неспецифічний виразковий коліт

Черевний тиф

Хвороба Крона

Амебіаз

668 / 6307
Під час обертання на каруселі у жінки 25 років з'явилися нудота, блювання, посилення потовиділення. Активація яких рецепторів зумовила рефлекторний розвиток цих симптомів?

Вестибулярних рецепторів півколових каналів

Зорових рецепторів

Отолітових вестибулярних рецепторів

Рецепторів кортієвого органу

Пропріорецепторів скелетних м'язів

669 / 6307
До хірургічного відділення надійшов чоловік 35 років з гнійною раною шиї попереду трахеї (в ділянці передвісцерального простору). Куди може поширитися інфекція, якщо терміново не буде проведене оперативне втручання?

У ретровісцеральний простір

У середнє середостіння

У переднє середостіння

У заднє середостіння

У надгруднинний міжапоневротичний простір

670 / 6307
Через два тижні після переливання крові у реципієнта виникла пропасниця. Про яке протозойне захворювання можна думати?

Амебіаз

Лейшманіоз

Трипаносомоз

Малярію

Токсоплазмоз

671 / 6307
До лікаря звернулися пацієнти з подібними скаргами: слабкість, болі у животі, рідкі випорожнення. Після дослідження фекалій з'ясувалося, що терміновій госпіталізації підлягає один з пацієнтів, у якого були виявлені цисти з чотирма ядрами. Для якого найпростішого характерні такі цисти?

Кишкова амеба

Дизентерійна амеба

Трихомонада

Балантидій

Лямблія

672 / 6307
У людини внаслідок втрати 1,5 л крові різко зменшився діурез. Посилена секреція якого гормону спричинила зміни діурезу?

Вазопресин

Натрійуретичний

Паратгормон

Кортикотропіну

Кортизол

673 / 6307
Хворому 56 років із скаргами на спрагу та часте сечовиділення було встановлено діагноз цукрового діабету та призначено бутамід. Яким є механізм дії цього препарату?

Стимулює бета-клітини острівців Лангерганса

Сприяє засвоєнню глюкози клітинами тканин організму

Пригнічує альфа-клітини острівців Лан герганса

Полегшує транспорт глюкози через клітинні мембрани

Пригнічує всмоктування глюкози в кишечнику

674 / 6307
Для знеболювання маніпуляції, пов'язаної із обробкою опеченої поверхні, ввели внутрішньовенно препарат для короткочасного наркозу. Наркоз настав через хвилину, під час нього спостерігалися підвищення AT, тахікардія, підвищення тонусу скелетних м'язів; рефлекси збережені. Після виходу із наркозу у хворого відзначалися дезорієнтація, зорові галюцинації. Який препарат ввели хворому?

Тіопентал натрію

Сомбревін

Ефір для наркозу

Кетамін

Закис азоту

675 / 6307
Під час бактеріологічного дослідження випорожнень чотиримісячної дитини з симптомами гострої кишкової інфекції на середовищі Ендо виросли у великій кількості червоні колонії. Які це можуть бути мікроорганізми?

Стафілококи

Сальмонели

Шигели

Стрептококи

Ешерихії

676 / 6307
У здорової дорослої людини швидкість проведення збудження через атріовентрикулярний вузол дорівнює 0,02-0,05 м/с. Що забезпечує атріовентрикулярна затримка?

Послідовність скорочення передсердів та шлуночків

Достатню силу скорочення шлуночків

Достатню силу скорочення передсердь

Одночасність скорочення обох шлуночків

Одночасність скорочення обох перед- сердів

677 / 6307
У дитини, яка страждає на пілоростеноз, що супроводжується частими блюваннями, розвинулися ознаки зневоднення організму. Яка форма порушення кислотнолужного стану може розвинутися у даному випадку?

Газовий ацидоз

Метаболічний ацидоз

Негазовий ацидоз

Газовий алкалоз

Негазовий алкалоз

678 / 6307
При ентеробіозі призначають акрихін - структурний аналог вітаміну В2. Порушення синтезу яких ферментів у мікроорганізмів викликає цей препарат?

Пептидази

Цитохромоксидази

Амінотрансферази

ФАД-залежні дегідрогенази

НАД-залежні дегідрогенази

679 / 6307
Хворому на бронхіальну астму призначено інгаляції 0,5% розчину ізадрину. Бронхоспазм був усунутий, але пацієнт почав скаржитися на болі в серці та серцебиття. Це пов'язано зі стимуляцією:

β2-адренорецепторів

α1-адренорецепторів

Синтезу ацетилхоліну

М-холінорецепторів

β1-адренорецепторів

680 / 6307
Хвора 13 років знаходиться на стаціонарному лікуванні в гематологічному відділенні обласної дитячої лікарні з діагнозом залізодефіцитна анемія. Який тип гіпоксії має місце у цієї хворої?

Змішана

Гемічна

Дихальна

Циркуляторна

Тканинна

681 / 6307
Для запобігання віддалених результатів чотириденної малярії пацієнту 42 років призначили примахін. Вже на 3 добу від початку лікування терапевтичними дозами препарату у пацієнта з'явилися болі в животі та в ділянці серця, диспептичні розлади, загальний ціаноз, гемоглобінурія. Що є причиною розвитку побічної дії препарату?

Генетична недостатність глюкозо-6-фосфат-дегідрогенази

Зниження активності мікросомальних ферментів печінки

Сповільнення екскреції препарату з сечею

Кумуляція лікарського засобу

Потенціювання дії іншими препаратами

682 / 6307
У хворого, який проходить курс лікувального голодування, нормальний рівень глюкози у крові підтримується головним чином за рахунок глюконеогенезу. З якої амінокислоти у печінці людини найбільш активно синтезується глюкоза?

Лізин

Аланін

Валін

Глутамінова кислота

Лейцин

683 / 6307
Хворому з метою відновлення дихання при отруєнні чадним газом було введено аналептичний засіб рефлекторного типу дії з групи Н-холіноміметиків. Який засіб було призначено хворому?

Мезатон

Лобеліну гідрохлорид

Атропіну сульфат

Пентамін

Адреналіну гідрохлорид

684 / 6307
У людини внаслідок довільної затримки дихання на 40 с зросли системний артеріальний тиск та частота серцевих скорочень. Реалізація яких механізмів регуляції зумовлює зміни показників?

Безумовні симпатичні рефлекси

Умовні симпатичні рефлекси

Умовні парасимпатичні рефлекси

Безумовні парасимпатичні рефлекси

685 / 6307
У хлопчика 2 років спостерігається збільшення у розмірах печінки та селезінки, катаракта. У крові підвищена концентрація цукру, однак тест толерантності до глюкози в нормі. Спадкове порушення обміну якої речовини є причиною цього стану?

Мальтоза

Фруктоза

Сахароза

Галактоза

Глюкоза

686 / 6307
Протягом двох тижнів хвора приймала мікстуру, призначену невропатологом з приводу неврастенії. Самопочуття хворої дещо покращилося, однак незабаром з'явилися скарги на нежить, кон'юнктивіт, шкірні висипи, млявість та послаблення пам'яті. Був встановлений діагноз 'бромізм'. Що доцільно призначити для послаблення симптомів?

Поліглюкін

Аспаркам

Натрію хлорид

Розчин глюкози 5%

687 / 6307
Під час вивчення фаз мітотичного циклу корінця цибулі знайдено клітину, в якій хромосоми лежать в екваторіальній площині, створюючи зірку. На якій стадії мітозу перебуває клітина?

Метафаза

Телофаза

Інтерфаза

Профаза

Анафаза

688 / 6307
У хворих на колагеноз має місце процес деструкції сполучної тканин. Це підтверджується збільшенням у крові:

Активності трансаміназ

Вмісту уратів

Активності ізоферментів ЛДГ

Вмісту креатину та креатиніну

Вмісту оксипроліну та оксилізину

689 / 6307
У хворого з опіковою хворобою наступила смерть внаслідок прогресування сепсису. Під час розтину тіла в ділянці передньої черевної стінки за допомогою мікроскопічного дослідження виявлено дифузну інфільтрацію міжм'язевих проміжків сегментоядерними лейкоцитами, набряк тканин, лізис м'язових волокон. Визначте характер патологічного процесу.

Дифтеритичне запалення

Катаральне запалення

Некроз

Абсцес

Флегмона

690 / 6307
У хворого з періодичними нападами ядухи, які виникають при вдиханні різних ароматичних речовин, діагностована атопічна бронхіальна астма. Визначено збільшення Ig Е. Для якого типу реакцій це характерно?

Анафілактичні

Імунокомплексні

Автоімунні

Цитотоксичні

Гіперчутливості сповільненого типу

691 / 6307
В експерименті збільшили проникність мембрани збудливої клітини для іонів калію. Які зміни електричного стану мембрани виникатимуть при цьому?

Змін не буде

Потенціал дії

Гіперполяризація

Локальна відповідь

Деполяризація

692 / 6307
Жінка 26 років скаржиться на сухість у роті, спрагу. Під час обстеження: глюкоза крові - 6,5 ммоль/л, глюкозурія. Для якого з зазначених нижче станів найбільш характерні ці симптоми?

Нецукровий діабет

Цукровий діабет

Аліментарна глюкозурія

Стероїдний діабет

Нирковий діабет

693 / 6307
У хворого 30 років з дизентерією, підтвердженою бактеріологічно, з'явились ознаки парапроктиту. Про яку стадію місцевих змін найбільш імовірно йде мова у даного хворого?

Фолікулярний коліт

Катаральний коліт

Утворення виразок

Фібринозний коліт

Загоєння виразок

694 / 6307
На прийом до лікаря звернувся хворий з симетричним дерматитом відкритих ділянок шкіри. З бесіди з пацієнтом встановлено, що він харчується здебільшого крупами і їсть мало м'яса, молока і яєць. Дефіцит якого з перерахованих вітамінів домінує у цього пацієнта?

Біотин

Токоферол

Нікотинамід

Кальциферол

Фолієва кислотна

695 / 6307
Хворий надійшов до клініки з пораненням у ділянці шиї. Під час обстеження виявлено пошкоджений нерв, розташований попереду переднього драбинчастого м'яза. Який нерв ушкоджено?

Блукаючий

Шийний відділ симпатичного стовбуру

Діафрагмальний

Під'язиковий

Язикоглотковий

696 / 6307
У харкотинні хворого з підозрою на пневмонію виявлено грампозитивні диплококи, подовжені, з дещо загостреними протилежними кінцями. Які мікроорганізми виявлені у харкотинні?

Neisseria meningitidis

Neisseria gonorrhoeae

Staphylococcus aureus

Streptococcus pneumoniae

Klebsiella pneumoniae

697 / 6307
Під час інтраопераційної біопсії щитоподібної залози гістологічно серед фолікулів, заповнених колоїдом, виявлені лімфоїдні структури із центрами росту. Яке захворювання у хворого?

Базедів зоб

Зоб Ріделя

Ендемічний зоб

Спорадичний зоб

Зоб Хашимото

698 / 6307
Скарги та об'єктивні дані дозволяють припустити наявність у хворого запального процесу в жовчному міхурі, порушення колоїдних властивостей жовчі, імовірність утворення жовчних каменів. Що головним чином може спричинити утворення каменів?

Хлориди

Урати

Фосфати

Оксалати

Холестерин

699 / 6307
Людина після травми головного мозку втратила зір. Пошкодження якої зони кори головного мозку може бути причиною цього?

Тім'яна

Потилична

Скронева

Скронева та тім'яна

Лобна

700 / 6307
У хворого спостерігаються часті кровотечі з внутрішніх органів, слизових оболонок. Аналіз виявив недостатність гідроксипроліну та гідроксилізину у складі колагенових волокон. Через нестачу якого вітаміну порушено в організмі пацієнта процеси гідроксилювання названих амінокислот?

Вітамін А

Вітамін Н

Вітамін С

Вітамін К

Вітамін РР

701 / 6307
У хворого на сухий плеврит вислуховується шум тертя плеври. При ураженні якого виду епітелію відмічається цей симптом?

Перехідний епітелій

Одношаровий кубічний епітелій

Одношаровий плоский епітелій

Багатошаровий епітелій

Одношаровий призматичний епітелій

702 / 6307
До лікаря звернулася хвора зі скаргами на розлади травлення, розлитий біль у животі. Під час обстеження лікар виявив різко виражене зниження гемоглобіну в крові. З опитування виявилося, що, проживаючи на Далекому Сході, хвора часто вживала у їжу малосольну рибну ікру. Аналогічний стан відзначений у деяких родичів, що проживають з нею. Яке захворювання діагностував лікар у цієї хворої?

Аскаридоз

Трихінельоз

Теніоз

Дифілоботріоз

Ехінококоз

703 / 6307
Хворому на миготливу аритмію, у якого в анамнезі бронхіальна астма, треба призначити протиаритмічний засіб. Який препарат з цієї групи протипоказаний хворому?

Анаприлін

Новокаїнамід

Аймалін

Верапаміл

Ніфедипін

704 / 6307
У чоловіка 52 років діагностовано системний амебіаз з ураженням кишечнику, печінки, легень. Який препарат слід призначити?

Ентеросептол

Метронідазол

Тетрациклін

Хініофон

Хінгамін

705 / 6307
У хворого з розладом мозкового кровотоку порушено акт ковтання, він може поперхнутися під час прийому рідкої їжі. Який відділ мозку уражено?

Шийний відділ спинного мозку

Мозочок

Проміжний мозок

Середній мозок

Довгастий мозок

706 / 6307
Під час постсинтетичного періоду мітотичного циклу було порушено синтез білків - тубулінів, які беруть участь у побудові веретена поділу Це може призвести до порушення:

Деспіралізації хромосом

Цитокінезу

Розходження хромосом

Тривалості мітозу

Спіралізації хромосом

707 / 6307
У 2-річної дитини кишковий дисбактеріоз, на фоні якого виник геморагічний синдром. Що є найбільш імовірною причиною геморагій у цієї дитини?

Нестача вітаміну К

Гіповітаміноз РР

Дефіцит фібриногену

Гіпокальціємія

Активація тромбопластину тканин

708 / 6307
До лікаря-генетика звернувся юнак 18 років астенічної будови тіла: вузькі плечі, широкий таз, високий зріст; незначне оволосіння обличчя. Виражена розумова відсталість. Було встановлено попередній діагноз: синдром Клайнфельтера. Який метод медичної генетики дозволяє підтвердити цей діагноз?

Популяційно-статистичний

Близнюковий

Дерматогліфіка

Цитогенетичний

Генеалогічний

709 / 6307
У хворого на глаукому спостерігається підвищення внутрішньоочного тиску при нормальній секреції водянистої вологи циліарним тілом. З ушкодженням яких структур стінки очного яблука пов'язане порушення відтоку рідини з передньої камери?

Війковий м'яз

Судинна оболонка

Задній епітелій рогівки

Венозний синус

Циліарне тіло

710 / 6307
Під час вивчення родоводу сім'ї, в якій спостерігається гіпертрихоз (надмірне оволосіння вушних раковин) встановлено, що ця ознака трапляється в усіх поколіннях тільки у чоловіків і успадковується від батька до сина. Який тип успадкування гіпертрихозу?

Зчеплений з Y-хромосомою

Аутосомно-рецесивний

Аутосомно-домінантний

Зчеплений з Х-хромосомою домінантний

Зчеплений з Х-хромосомою рецесивний

711 / 6307
У положенні на спині дитина 6 місяців задихається. Пальпаторно на передній стінці трахеї до яремної вирізки визначається пухлиноподібне утворення, яке про- довжується в переднє середостіння. Що з наведеного може стискати трахею?

Навколотрахеальні лімфатичні вузли

Навколощитоподібні лімфатичні вузли

Паращитоподібні залози

Загруднинна залоза

Щитоподібна залоза

712 / 6307
При визначенні групи крові за системою АВ0 аглютинацію еритроцитів досліджуваної крові викликали стандартні сироватки І та II груп і не викликала III групи. Які аглютиногени містяться в цих еритроцитах?

А та В

А

D та C

C

В

713 / 6307
У померлого від ядухи чоловіка, який багато років страждав на бронхіальну астму, при гістологічному дослідженні легень виявлено: у просвіті бронхіол та дрібних бронхів багато слизу з вмістом еозинофілів, склероз міжальвеолярних перетинок, розширення просвіту альвеол. Який з механізмів розвитку реакції гіперчутливості має місце?

Гранулематоз

Імунокомплексний

Цитоліз, обумовлений лімфоцитами

Цитотоксичний

Реагіновий

714 / 6307
У хворого спостерігаються напади артеріальної гіпертензії, які супроводжуються тахікардією, різким болем у надчеревній ділянці. Для якого з перелічених захворювань найбільш характерні ці симптоми?

Пухлина яєчників

Аденома клубочкової зони наднирників

Аденома щитовидної залози

Базофільна аденома гіпофіза

Феохромоцитома

715 / 6307
У жінки 35 років із хронічним захворюванням нирок розвинувся остеопороз. Дефіцит якого з нижчеперерахованих речовин є основною причиною цього ускладнення?

D3

D2

25OHD3

1.25(ОH)2D3

Холестерин

716 / 6307
У хворого спостерігається ішемія тканин нижче колінного суглоба, що супроводжується «переміжною кульгавістю». Про оклюзію якої артерії слід думати?

Малогомілкова

Задня великогомілкова

Підколінна

Проксимальна частина стегнової

Передня великогомілкова

717 / 6307
У хворого з захворюванням печінки виявлено зниження вмісту протромбіну в крові. Це призведе, перш за все, до порушення:

Судинно-тромбоцитарного гемостазу

Антикоагулянтних властивостей крові

Фібринолізу

Першої фази коагуляційного гемостазу

Другої фази коагуляційного гемостазу

718 / 6307
Під час фарбування мазка з харкотиння хворого з підозрою на крупозну пневмонію були використані наступні барвники і реактиви: розчин генціанвіолету, розчин Люголя, 96° спирт, водний фуксин. Який спосіб фарбування застосований у даному випадку?

За Лефлером

За Романовським

За Грамом

За Цилем-Нільсеном

За Нейсером

719 / 6307
У чоловіка 50 років раптово виникли сильне серцебиття, біль у серці, різка слабкість, підвищення AT; пульс аритмічний, з дефіцитом. На ЕКГ виявлено відсутність зубців Р і різна тривалість інтервалів R-R. Яке порушення серцевого ритму у хворого?

Синусова екстрасистолія

Поперечна блокада серця

Пароксизмальна тахікардія

Миготлива аритмія

Дихальна аритмія

720 / 6307
У збудливій клітині заблокували іонні канали. Це суттєво не змінило рівень потенціалу спокою, але клітина втратила здатність до генерації ПД. Які канали заблоковано?

Калієві

Натрієві та калієві

Натрієві

Кальцієві

Хлорні

721 / 6307
У печінці хворого порушена детоксикація природних метаболітів та ксенобіотиків. Знижена активність якого хромопротеїду може бути причиною цього?

Цитохром С-1

Гемоглобін

Цитохром В

Цитохром Р-450

Цитохромоксидаза

722 / 6307
У хворого із запаленням легень непе- реносимість антибіотиків. Який з комбінованих сульфаніламідних препаратів слід призначити хворому?

Бісептол

Сульфацил натрію

Стрептоцид

Сульфадиметоксин

Етазол

723 / 6307
Хворому 65 років з ознаками загального ожиріння, небезпекою жирової дистрофії печінки рекомендована дієта, збагачена ліпотропними речовинами. Вміст якої речовини у раціоні має найбільше значення у даному випадку?

Вітамін С

Метіонін

Глюкоза

Гліцин

Холестерин

724 / 6307
У хворого на гострий мієлобластний лейкоз виявлено: збільшення печінки, селезінки, анемія, мієлобласти в периферичній крові. Яка принципова ознака дозволяє відрізнити гострий мієлобластний лейкоз від хронічного?

Панцитопенія

Анемія

Бластні клітини в периферичній крові

Лейкемічний провал

Тромбоцитопенія

725 / 6307
У молодої людини в м'яких тканинах лівого стегна з'явилося безболісне новоутворення без чітких меж. У біоптаті тканини новоутворення нагадує риб'яче м'ясо, складаються з незрілих фібробластоподібних клітин з численними мітозами, які проростають в м'язи. Який найбільш імовірний діагноз?

Міома

Рaк

Фібросаркома

Міосаркома

Фіброма

726 / 6307
Літня людина перенесла інфаркт правої півкулі головного мозку. Через рік, враховуючи відсутність рухів лівих кінцівок, проведено комп'ютерну томографію мозку, під час якої в правій півкулі знайдено порожнину з гладенькими стінками, заповнену ліквором. Який патологічний процес виявлено у головному мозку?

Сіре розм'якшення мозку

Гідроцефалія

Гематома

Інфаркт мозку

Постінфарктна кіста

727 / 6307
У хворого, який страждає на вроджену еритропоетичну порфірію, відзначена світлочутливість шкіри. Накопиченням якої сполуки в клітинах шкіри це обумовлено?

Копропорфіриноген III

Уропорфіриноген II

Уропорфіриноген І

Протопорфірин

Гем

728 / 6307
На 8 день після введення протиправцевої сироватки з приводу брудної рани стопи у пацієнта температура підвищилася до 38°C, з'явилися болі в суглобах, висипка, свербіж. З боку крові спостерігається лейкопенія і тромбоцитопенія. Алергічна реакція якого типу розвинулася в цьому випадку?

Стимулюючого

Гіперчутливості уповільненого типу

Цитотоксичного

Анафілактичного

Імунокомплексного

729 / 6307
Хвора 44 років надійшла до терапевтичного відділення з діагнозом правобічний плеврит. Обстеження підтвердило наявність рідини в плевральній порожнині. У якому закутку плеври буде найбільше накопичення серозної рідини?

Лівому середостінно-діафрагмальному

Правому реберно-діафрагмальному

Правому реберно-середостінному

Лівому реберно-середостінному

Правому середостінно-діафрагмальному

730 / 6307
В експерименті ізольований м'яз жаби ритмічно подразнюють електричними імпульсами. Кожний наступний імпульс припадає на період розслаблення попереднього скорочення. Яке скорочення м'язу виникає?

Асинхронне

Одиночне

Зубчастий тетанус

Тонічне

Суцільний тетанус

731 / 6307
Після радіаційного опромінення у хворого зруйновано стовбурові клітини. Відновлення яких клітин пухкої волокнистої сполучної тканини буде порушено?

Макрофаги

Перицити

Адипоцити

Пігментні клітини

Фібробласти

732 / 6307
У малярійного плазмодія – збудника триденної малярії - розрізняють два штами: південний та північний. Вони відрізняються тривалістю інкубаційного періоду: у південного він короткий, а у північного - довгий. Дія якого відбору проявляється у даному випадку?

Штучний

Розсікаючий

Стабілізуючий

Статевий

Руховий

733 / 6307
У хворого в крові підвищений вміст сечової кислоти, що клінічно проявляється больовим синдромом внаслідок відкладення уратів у суглобах. У результаті якого процесу утворюється ця кислота?

Катаболізм гему

Розщеплення білків

Розпад піримідинових нуклеотидів

Реутилізація пуринових основ

Розпад пуринових нуклеотидів

734 / 6307
У хворого після видалення зуба з'явився стійкий біль за грудиною. Після вживання під язик антиангінального засобу біль за грудиною зник, але хворий поскаржився на головний біль і запаморочення. Який препарат використав хворий?

Нітрогліцерин

Верапаміл

Метопролол

Анаприлін

Валідол

735 / 6307
До терапевтичного відділення надійшов хворий з тривалою бронхопневмонією. Антибіотикотерапія не дала належного ефекту. Який препарат для підвищення імунного статусу слід додати до комплексної терапії цього хворого?

Парацетамол

Сульфокамфокаїн

Димедрол

Аналгін

Тималін

736 / 6307
Дистрофічні зміни серцевого м'яза супроводжуються розширенням порожнин серця, зниженням сили серцевих скорочень, збільшенням об'єму крові, що залишається під час систоли в порожнині серця, переповненням вен. Для якого стану серця це характерно?

Міогенна дилатація

Аварійна стадія гіперфункції та гіпертрофії

Кардіосклероз

Тампонада серця

Тоногенна дилатація

737 / 6307
У гістопрепараті представлено паренхіматозний орган, що має кіркову і мозкову речовину. Кіркова речовина утворена тяжами епітеліоцитів, між якими проходять кровоносні капіляри; тяжі формують три зони. Мозкова речовина складається з хромафіноцитів і венозних синусоїдів. Який орган має дані морфологічні ознаки?

Наднирник

Лімфатичний вузол

Тимус

Нирка

Щитоподібна залоза

738 / 6307
Під час огляду хворого з ранами, що кровоточать, лікар виявив пошкодження тканин личинками, а також локальні місця нагноєння, та встановив діагноз: облігатний міаз. Личинки яких комах є збудниками цього захворювання?

Муха хатня

Муха вольфартова

Жигалка осіння

Муха цеце

Тріатомовий клоп

739 / 6307
У мазку крові, забарвленому за Романовським-Гімзою, виявляється 20% великих (діаметром 20 мкм), округлих клітин з блідобазофільною цитоплазмою і бобоподібним ядром. Як клінічно характеризується це явище?

Нейтрофільоз

Лейкопенія

Лімфоцитоз

Моноцитоз

Ретикулоцитоз

740 / 6307
Під час тяжких пологів відбувся розрив лобкового симфізу. Який орган найбільш імовірно буде травмований?

Сечовий міхур

Яєчники

Матка

Пряма кишка

Маткові труби

741 / 6307
Під час гістологічного дослідження серця померлого від гострої серцевої недостатності у міокарді лівого шлуночка виявлена ділянка омертвіння, відмежована від неушкодженої тканини зоною повнокровних судин, дрібних крововиливів та лейкоцитарною інфільтрацією. Який діагноз найбільш імовірний?

Дифузний ексудативний міокардит

Інфаркт міокарда

Продуктивний міокардит

Осередковий ексудативний міокардит

Ішемічна дистрофія міокарда

742 / 6307
У міокарді шлуночків досліджуваної людини порушені процеси реполяризації. Це призведе до порушення амплітуди, конфігурації та тривалості зубця:

R

Q

P

S

Т

743 / 6307
У чоловіка 46 років на шкірі голо- ви визначалася пігментна пляма темного кольору, яка дещо вибухала та не спричиняла турбот. Останнім часом пляма почала збільшуватися, з'явився біль, колір став чорно-коричневим, у її основі пальпується вузлик . Під час гістологічного дослідження видаленої тканини визначалися веретеноподібні і поліморфні клітини, цитоплазма яких вміщувала пігмент бурого кольору. Про яку пухлину іде мова?

Базаліома

Меланома

Карциноїд

Гемангіома

Гематома

744 / 6307
У процесі метаболізму в організмі людини утворюються активні форми кисню, у тому числі супероксидний аніон-радикал. За допомогою якого ферменту інактивується цей аніон?

Супероксиддисмутаза

Глутатіонредуктаза

Пероксидаза

Каталаза

Глутатіонпероксидаза

745 / 6307
До медико-генетичної консультації звернулася жінка з приводу ризику захворювання на гемофілію у свого сина. Її чоловік страждає на дане захворювання з народження. Жінка та її батьки здорові стосовно гемофілії. Яка імовірність народження хворого хлопчика в цій сім'ї?

Усі хлопчики будуть хворі

Всі хлопчики будуть здорові

50% хлопчиків будуть хворими

25 % хлопчиків будуть хворими

75 % хлопчиків будуть хворими

746 / 6307
Під час огляду хворого, який знаходився на гірських пасовиськах і надійшов у тяжкому стані з високою температурою, лікар відзначив, що пахвинні лімфатичні вузли збільшені до 8 см, зпаяні з оточуючими тканинами, нерухливі, шкіра над ними червона, різко болюча. Під час мікроскопічного дослідження в лімфатичному вузлі найгостріше серозно-геморагічне запалення. Для якого захворювання це характерно?

Туляремія

Чума

Сифіліс

Сибірка

Бруцельоз

747 / 6307
У хворого на цукровий діабет з'явився різкий біль у правій стопі. Під час огляду великий палець стопи чорного кольору, тканини стопи набряклі, осередки відшарування епідермісу, виділення з неприємним запахом. Яка клініко-морфологічна форма некрозу розвинулася у хворого?

Секвестр

Гангрена суха

Пролежень

Інфаркт

Гангрена волога

748 / 6307
У хворої 38 років після прийому аспірину і сульфаніламідів спостерігається посилений гемоліз еритроцитів, викликаний недостатністю глюкозо-6фосфатдегідрогенази. Порушенням утворення якого коферменту зумовлена ця патологія?

НАДФН

Піридоксальфосфат

ФАДН2

ФМНН2

Убіхінон

749 / 6307
Під час ультразвукового обстеження серця лікар спостерігає за стулками мітрального клапана. Що відбувається з ними під час систоли передсердь?

Стуляються, закриваючи просвіт отвору

Притискаються до стінки судини

Вивертаються в порожнину передсердя

Вивертаються в порожнину шлуночка

Притискаються до стінок передсердя

750 / 6307
У мікропрепараті спинного мозку необхідно проаналізувати стан ядра, нейрони якого утворюють моторні закінчення у скелетній мускулатурі. Про яке ядро спинного мозку йдеться?

Проміжне латеральне ядро

Власне ядро переднього рога

Власне ядро заднього рога

Грудне ядро

Власне ядро сірої речовини

751 / 6307
Пацієнтка 58 років скаржиться на підвищену втомлюваність, зниження працездатності, сонливість, задишку під час швидкої ходи. У крові: еритроцити - 4,6 * 1012 /л, гемоглобін - 92 г/л, кольоровий показник - 0,6. У мазку крові - велика кількість анулоцитів і мікроцитів. Для якої анемії це характерно?

Постгеморагічна

Гемолітична

Залізодефіцитна

Перніціозна

Серпоподібноклітинна

752 / 6307
Під час дослідження первинної структури молекули глобіну виявлено заміну глутамінової кислоти на валін. Для якої спадкової патології це характерно?

Гемоглобіноз

Хвороба Мінковського-Шоффара

Фавізм

Таласемія

Серпоподібноклітинна анемія

753 / 6307
У хворої на дифтерію дитини через 10 днів після введення антитоксичної протидифтерійної сироватки з'явилася висипка на шкірі, яка супроводжувалася сильним свербінням, підвищилася температура тіла до 38°С, з'явилися болі в суглобах. Яка причина цих явищ?

Анафілактична реакція

Контактна алергія

Атопія

Гіперчутливість сповільненого типу

Сироваткова хвороба

754 / 6307
У хворого внаслідок хронічного захворювання органів дихання на фоні задишки, тахікардії і ціанозу під час дослідження газового складу крові виявлено розвиток гіпоксемії і гіперкапнії. Яке порушення зовнішнього дихання спостерігається у хворого?

Гіповентиляція

Гіпердифузія

Гіпоперфузія

Гіперперфузія

Гіпервентиляція

755 / 6307
У людини в стані спокою збільшена робота м'язів, що забезпечують вдих. Що з наведеного може бути причиною цього?

Негативний внутрішньоплевральний тиск

Звуження дихальних шляхів

Зменшення хвилинного об'єму дихання

Поверхневе дихання

Рідке дихання

756 / 6307
Біля інфікованої рани збільшилися регіонарні лімфовузли. Під час гістологічного дослідження в них виявлено збільшення кількості макрофагів, лімфоцитів і лімфатичних фолікулів у кірковому шарі, а також велику кількість плазматичних клітин. Який процес у лімфатичних вузлах відображують виявлені гістологічні зміни?

Набуту недостатність лімфоїдної тканини

Реакцію гіперчутливості

Антигенну стимуляцію

Природжену недостатність лімфоїдної тканини

Пухлинну трансформацію

757 / 6307
Послаблення кровопостачання органа зумовлює розвиток гіпоксії, яка активізує функцію фібробластів. Об'єм яких елементів збільшується в цій ситуації?

Судини мікроциркуляторного русла

Міжклітинна речовина

Нервові елементи

Паренхіматозні елементи органа

Лімфатичні судини

758 / 6307
Хворому, який звернувся до лікаря з приводу безпліддя, було призначено обстеження на токсоплазмоз і хронічну гонорею. Яку реакцію слід поставити для виявлення прихованого токсоплазмозу і хронічної гонореї у цього хворого?

РЗК

РЗПГА

РЗНГА

Імуноблотингу

РІФ

759 / 6307
Хвора на бронхіальну астму приймала протягом 2-х місяців преднізолон у таблетках. Внаслідок значного покращання стану раптово припинила його прийом. Розвиток якого ускладнення високоімовірний у цьому випадку?

Шлункова кровотеча

Синдром відміни

Синдром Іценко-Кушінга

Гіпотонія

Ожиріння верхньої половини тулуба

760 / 6307
До ендокринолога звернувся хворий зі скаргами на схуднення на 10 кг за 2 місяці, серцебиття, витрішкуватість. Для гіперфункції якої ендокринної залози (яких залоз) ці скарги найбільш характерні?

Підшлункова

Надниркові залози

Яєчники

Щитоподібна

Паращитоподібні залози

761 / 6307
Чоловік 63 років страждає на рак стравоходу, є метастази в лімфатичні вузли середостіння, ракова кахексія. Яка патогенетична стадія пухлинного процесу у цьому випадку?

Ініціації

Прогресії

Трансформації

Промоції

762 / 6307
У хворого через наявність каменя у загальній жовчній протоці припинилося надходження жовчі у кишечник. Порушення якого з процесів спостерігається при цьому?

Перетравлення білків

Перетравлення вуглеводів

Перетравлення жирів

Всмоктування білків

Всмоктування вуглеводів

763 / 6307
У пацієнта 48 років після сильного пси- хоемоційного навантаження раптово з'явився гострий біль у ділянці серця з іррадіацією у ліву руку. Нітрогліцерин зняв напад болю через 10 хвилин. Який патогенетичний механізм є провідним у розвитку болю у даному випадку?

Закупорка коронарних судин

Підвищення потреби міокарда у кисні

Спазм коронарних судин

Стиснення коронарних судин

Розширення периферичних судин

764 / 6307
Під час розтину тіла померлого виявлено, що печінка збільшена у розмірах, щільна, краї заокруглені, на розрізі тканина жовтувато-коричневого кольору з темно-червоними крапками та смужками, що нагадує малюнок мускатного горіху. Який патологічний процес лежить в основі таких змін печінки?

Гостре венозне повнокров'я

Артеріальне недокрів'я

Хронічне венозне повнокров'я

Артеріальне повнокров'я

Хронічна кровотеча

765 / 6307
У здорової людини визначили величину енерговитрат. У якому стані знаходилася людина, якщо її енерговитрати виявилися меншими за основний обмін?

Нервове напруження Е. Сон

Відпочинок

Легка робота

Спокій

766 / 6307
У чоловіка 58 років клінічна картина гострого панкреатиту. Підвищення в сечі якої з нижчеперерахованих речовин буде підтвердженням діагнозу?

Амілаза

Залишковий азот

Сечовина

Сечова кислота

Альбумін

767 / 6307
Під час інтраопераційної біопсії молочної залози виявлено ознаки тканинного атипізму, що виражається у порушенні співвідношення паренхіми і строми з переважанням останньої, різних розмірів і форми залозисті структури, вистелені одношаровим проліферуючим епітелієм. Який найбільш імовірний діагноз?

Інфільтруючий рак

Мастит

Неінфільтруючий рак

Папілома

Фіброаденома

768 / 6307
До інфекційної лікарні надійшов пацієнт з клінічними ознаками енцефаліту. В анамнезі - укус кліща. Під час постановки реакції затримки гемаглютинації вияв- лено антитіла проти збудника кліщового енцефаліту в розведенні 1:20, що не є діагностичним. Які наступні дії лікаря після одержання вказаного результату?

Дослідити цю ж сироватку повторно

Повторити дослідження з іншим діагностикумом

Повторити дослідження із сироваткою, взятою через 10 днів

Відхилити діагноз кліщового енцефаліту

Використати чутливішу реакцію

769 / 6307
У 40-річної вагітної проведено амніоцентез. Під час дослідження каріотипу плоду одержано результат: 47XY+21. Яку патологію плода виявлено?

Хвороба Шерешевського-Тернера

Хвороба Патау

Фенілкетонурія

Синдром Клайнфельтера

Синдром Дауна

770 / 6307
До травмпункту звернувся чоловік 45 років після побутової травми плеча. Об'єктивно: відсутні функції розгинання, приведення та пронації плеча. Пошкодження якого м'яза викликало такий стан?

Великого круглого

Підосного

Надостного

Підлопаткового

Малого круглого

771 / 6307
У хворого 43 років спостерігається хронічний атрофічний гастрит, мегалобластна гіперхромна анемія. Підвищено виділення метилмалонової кислоти з сечею. Недостатністю якого вітаміну обумовлене виникнення зазначеного симптомокомплексу?

Вітамін В3

Вітамін В1

Вітамін В5

Вітамін В12

Вітамін В2

772 / 6307
Під час обстеження у іноземного громадянина виявили кишковий шистосомоз. Яким шляхом він міг заразитися?

При вживання в їжу риби

При вживанні в їжу м'яса

Через укуси комарів

Під час купання в річці

Через брудні руки

773 / 6307
Після вживання ацетилсаліцилової кислоти у пацієнта з'явився біль у ділянці шлунка як наслідок загострення виразкової хвороби. Що лежить в основі ульцерогенності вказаного препарату?

Імунодепресивний ефект

Антипростагландинова дія

Жовчогінна дія

Спазм судин

Стимуляція виділення пепсину

774 / 6307
До лікаря звернулася жінка зі скаргами на те, що у весняний період у неї з'являється нежить, сиплість голосу, почервоніння повік зі сльозотечею. Який тип алергічної реакції за Джелом і Кумбсом розвивається у цьому випадку?

Імунокомплексний

Гіперчутливості сповільненого типу

Стимулюючий

Цитотоксичний

Анафілактичний

775 / 6307
У чоловіка 39 років спостерігається підвищений ризик розвитку інфекційних процесів, гіперкератоз, порушення присмеркового зору. Який вітамінний препарат необхідно призначити?

Ергокальциферол

Ретинолу ацетат

Піридоксину гідрохлорид

Рибофлавін

Токоферолу ацетат

776 / 6307
У жінки, яка тривалий час знаходилася на дієті з використанням очищеного рису, виявлено поліневрит (хвороба бері-бері). Відсутність якого вітаміну в раціоні призводить до розвитку цього захворювання?

Фолієва кислота

Піридоксаль

Рибофлавін

Тіамін

Аскорбінова кислота

777 / 6307
При повному (з водою) аліментарному голодуванні розвинулись генералізовані набряки. Який з патогенетичних факторів у цьому випадку є провідним?

Зниження осмотичного тиску плазми крові

Підвищення онкотичного тиску міжклітинної рідини

Зниження гідростатичного тиску міжклітинної рідини

Зниження онкотичного тиску плазми крові

Підвищення осмотичного тиску міжклітинної рідини

778 / 6307
Куди треба провести катетер для забору лімфи з грудної лімфатичної протоки?

У лівий венозний кут

У верхню порожнисту вену

У правий венозний кут

У ліву пахову вену

У нижню порожнисту вену

779 / 6307
До лікаря звернувся хворий із скаргами на постійну спрагу. Виявлена гіперглікемія, поліурія та підвищений вміст 17кетостероїдів у сечі. Яке захворювання найбільш імовірне?

Інсулінозалежний діабет

Глікогеноз І типу

Мікседема

Стероїдний діабет

Аддісонова хвороба

780 / 6307
Смерть хворого 16 років настала від розлитого (поширеного) фібринозногнійного перитоніту. Під час розтину в нижній ділянці тонкої кишки виявлена виразка, яка повторювала форму пейєрової бляшки, з перфорацією стінки кишки. Мікроскопічне дослідження виявило невизначеність малюнку лімфоїдної тканини, витіснення її проліферуючими моноцитами, які формують гранульоми. Ускладнення якого захворювання є причиною смерті?

Холера

Черевний тиф

Неспецифічний виразковий коліт

Дизентерія

Бруцельоз

781 / 6307
Чоловік 42 років надійшов до кардіологічного відділення з діагнозом стенокардія. До комплексу препаратів, призначених хворому, включений інгібітор ферменту фосфодіестерази. Концентрація якої речовини в серцевому м'язі буде збільшуватись?

АМФ

АДФ

АТФ

ГМФ

Цикло-АМФ

782 / 6307
У хворого спостерігається гемералопія (куряча сліпота). Яка з перелічених речовин матиме лікувальну дію?

Каротин

Креатин

Карнітин

Кератин

Карнозин

783 / 6307
Хворому на аскаридоз призначили лікарський засіб, який має також вплив на імунну систему і використовується як імуномодулятор. Який це препарат?

Пірантел В. Фенасал

Піперазину адипінат

Нафтамон

Левамізол

784 / 6307
У хворого інфаркт міокарда в ділянці передньої стінки лівого шлуночка. В басейні якої артерії виникло порушення кровообігу?

Передня міжшлуночкова гілка лівої вінцевої артерії

Огинаюча гілка лівої вінцевої артерії

Передсердно-шлуночкова гілка лівої вінцевої артерії

Ліва крайова гілки лівої вінцевої артерії

Передні шлуночкові гілки правої вінце- вої артерії

785 / 6307
У хворого напади бронхіальної астми виникають звичайно вночі, супроводжуються брадикардією, спастичними болями у кишечнику, проносом. Препарати якої групи можуть усунути ці симптоми?

Салуретики

Н-холіноблокатори

М-холіноблокатори

β-адреноблокатори

Симпатолітики

786 / 6307
Під час аналізу електрокардіограми встановлено, що тривалість серцевого циклу у людини дорівнює 1 сек. Якою є частота серцевих скорочень за хвилину?

60

50

70

100

80

787 / 6307
У хворого на хронічну серцеву недостатність у процесі лікування препаратами наперстянки виникли симптоми, що свідчать про початок токсичної дії серцевих глікозидів. Який препарат необхідно призначити для зменшення негативної дії серцевих глікозидів?

Калію хлорид

Атропіну сульфат

Дипіроксим

Етимізол

Кофеїн-натрія бензоат

788 / 6307
Хворого госпіталізовано зі скаргами на блювання, запаморочення, двоїння в очах, утруднене ковтання. Лікар запідозрив ботулізм. Які методи діагностики доцільно використати для підтвердження діагнозу?

Бактеріологічний, мікологічний

Алергічну пробу, серологічний

Біологічну пробу, бактеріологічний

Протозоологічний, мікроскопічний

789 / 6307
Під час розтину тіла чоловіка, який багато років працював на шахті і помер від хронічної легенево-серцевої недостатності, виявлено, що легені малоповітряні, значно ущільнені, склерозовані, верхівки емфізематозно змінені, поверхня сірочорного кольору, на розрізі тканина легенів аспідно-чорного кольору. Яке захворювання стало причиною смерті цього чоловіка?

Силікоз

Талькоз

Алюміноз

Антракоз

Азбестоз

790 / 6307
У чоловіка після гіпертонічного кризу відзначається відсутність довільних рухів у правих руці і нозі, тонус м'язів у цих кінцівках підвищений. Який вид порушення функції нервової системи спостерігається у даному випадку?

Рефлекторний парез

Периферичний параліч

Периферичний парез

Центральний парез

Центральний параліч

791 / 6307
Під час патронажу лікар виявив у дитини симетричну шорсткість щік, діарею, порушення нервової діяльності. Нестача яких харчових факторів є причиною такого стану?

Фенілаланін, пангамова кислота

Метіонін, ліпоєва кислота

Лізин, аскорбінова кислота

Нікотинова кислота, триптофан

Треонін, пантотенова кислота

792 / 6307
Під час мікроскопічного дослідження змиву з рани хворого з гострим рановим процесом гомілки виявлено велику кількість клітин неправильної витягнутої форми, з щільним ядром, у базофільній цитоплазмі яких міститься багато лізосом, фагосом, піноцитозних пухирців. Які клітини виявлено в рані?

Фіброцити

Фібробласти

Тканинні базофіли

Плазмоциди

Макрофаги сполучної тканини

793 / 6307
Чоловік 40 років госпіталізований до хірургічного відділення з діагнозом розрив селезінки. У якому анатомічному утворі буде накопичуватися кров?

У правому бічному каналі

У печінковій сумці

У прямокишково-міхуровій заглибині

У передшлунковій сумці

У чепцевій сумці

794 / 6307
На електронній мікрофотографії фрагменту нирки представлена приносна артеріола, у якій під ендотелієм видно великі клітини, які містять секреторні гранули. Назвіть цей вид клітин:

Мезангіальні

Гладеньком'язові

Інтерстиціальні

Юкставаскулярні

Юкстагломерулярні

795 / 6307
Хворий взимку впав в ополонку, замерз на вітрі, захворів. Температура тіла підвищилася до 39,7°С і коливалася від 39,0°С до 39,8°С. Назвіть тип температурної кривої у хворого.

Febris remittens

Febris recurrens

Febris continua

Febris hectica

Febris intermittens

796 / 6307
У жінки, що хворіє на міастенію, виникли розлади дихання, що вимагало застосування штучної вентиляції легень. Який вид дихальної недостатності розвинувся у даної хворої?

Обструктивний

Рестриктивний

Центрогенний

Нервово-м'язовий

Торакодіафрагмальний

797 / 6307
Піддослідному собаці через зонд у порожнину шлунка ввели 150 мл м'ясного бульйону. Вміст якої з приведених речовин швидко збільшиться в крові тварини?

Нейротензин

Вазоінтестінальний поліпептид

Гастрин

Інсулін

Соматостатин

798 / 6307
У хворого, що тривалий час палить тютюн, розвинувся рак легені. Які з перерахованих канцерогенних речовин містяться в тютюновому диму і відносяться до ПАВ (поліненасичених ароматичних вуглеводів)?

β-нафтиламін

Діетилнітрозамін

Бензпірен

Диметиламіноазобензол

Ортоаміноазотолуол

799 / 6307
У хворого 56 років з серцевою недостатністю спостерігається набряк стоп та гомілок, шкіра в місці набряку бліда і холодна. Яка провідна ланка патогенезу набряку у цього хворого?

Підвищення гідростатичного тиску в венулах

Зменшення онкотичного тиску в капілярах

Позитивний водний баланс

Підвищення проникливості капілярів

Порушення лімфовідтоку

800 / 6307
При спалаху гострої респіраторної інфекції з метою встановлення діагнозу грип проводиться експрес-діагностика, яка грунтується на виявленні специфічного вірусного антигену в досліджуваному матеріалі (змив носоглотки). Яку серологічну реакцію використовують для цього?

Реакція аглютинації

Реакція імунофлюоресценції

Реакція опсонізації

Реакція зв’язування комплементу

Реакція преципітації

801 / 6307
До хірургічного відділення надійшов чоловік 35 років з гнійною раною шиї попереду трахеї (у ділянці передвісцерального простору). Куди може поширитися інфекція, якщо терміново не буде проведене оперативне втручання?

У заднє середостіння

У надгруднинний міжапоневротичний простір

У середнє середостіння

У переднє середостіння

У ретровісцеральний простір

802 / 6307
У хворого з гіпохромною анемією січеться та випадає волосся, відзначається підвищена ламкість нігтів, порушений смак. Якою є причина розвитку зазначених симптомів?

Дефіцит вітаміну В12

Зниження продукції паратирину

Зниження продукції тиреоїдних гормонів

Дефіцит залізовмісних ферментів

Дефіцит вітаміну А

803 / 6307
До медико-генетичної консультації звернулася жінка з приводу ризику захворювання на гемофілію у свого сина. Її чоловік страждає на це захворювання від народження. Жінка та її родичі не страждали на це захворювання. Визначте вірогідність народження хлопчика з гемофілією у цій сім’ї:

50% хлопчиків будуть хворими

0%

100%

25% хлопчиків будуть хворими

75% хлопчиків будуть хворими

804 / 6307
Послаблення кровопостачання органа зумовлює розвиток гіпоксії, яка активізує функцію фібробластів. Об’єм яких елементів збільшується в цій ситуації?

Судини мікроциркуляторного русла

Паренхіматозні елементи органа

Лімфатичні судини

Нервові елементи

Міжклітинна речовина

805 / 6307
При мікроскопії зіскобу з періанальних складок виявлені безбарвні яйця, що мають форму несиметричних овалів, розміром 50x23 мкм. Про який вид гельмінту йде мова?

Кривоголовка

Карликовий ціп’як

Гострик

Аскарида

Волосоголовець

806 / 6307
У здорового обстежуваного в стані спокою кількість еритроцитів становить 5,65*1012/л. Причиною цього може бути те, що обстежуваний:

Шахтар

Відповідальний працівник міністерства

Вагітна жінка

Студент

Мешканець високогір’я

807 / 6307
До лікаря звернулася мати, син якої за літо виріс на 18 см. При обстеженні хлопця 12 років: зріст - 180 см, вага 68 кг. З гіперфункцією якої ендокринної залози це може бути пов’язано?

Підшлункова

Епіфіз

Щитоподібна

Гіпофіз

Наднирник

808 / 6307
Хворий не розуміє змісту слів, а також не розуміє власної мови (словесна глухота). Яка із структур кори великих півкуль головного мозку уражена?

Нижня тім’яна часточка

Верхня тім’яна часточка

Верхня скронева звивина

Нижня лобова звивина

Задня центральна звивина

809 / 6307
Територію старого худобомогильника, який не використовувався більше 50 років, планується відвести під житлове будівництво. Однак дослідження грунту виявило наявність життєздатних спор збудника особливо небезпечного захворювання. Який із вказаних мікроорганізмів найбільш вірогідно міг зберігатися у грунті протягом такого тривалого часу?

Batillus anthratis

Frantisella tularensis

Brucella abortus

Yerarna pestis

Mycobacterium bovis

810 / 6307
Молода людина під час активного підтягування на перекладині відчула різкий біль у спині. Об’єктивно: біль при спробах руху верхньою кінцівкою, обмеження функцій приведення та пронації. Розтягнення якого м’яза, найбільш вірогідно, відбулося?

М.latissimus dorsi

М.levator scapulae

М.subscapularis

М.romboideus major

M.trapezrns

811 / 6307
Людина хворіє на цукровий дiабет, що супроводжується гіперглікемією натще понад 7,2 ммоль/л. Рівень якого білка плазми крові дозволяє ретроспективно (за попередні 4-8 тижні до обстеження) оцінити рівень глікемії?

Фібриноген

С-реактивний білок

Альбумін

Церулоплазмін

Ілікозильований гемоглобін

812 / 6307
У хворого із значними периферічними набряками почергове застосування дихлотіазиду, етакринової кислоти та фуросеміду не викликало значного діуретичного ефекту. У крові значне підвищення кількості альдостерону. Вкажіть препарат вибору:

Клопамід

Маніт

Спіронолактон

Сечовина

Амілорид

813 / 6307
Під час лапаротомії хірург виявив гангренозне ураження низхідної ободової кишки. Тромбоз якої артерії зумовив цей стан?

Права ободова

Верхня брижова

Ліва ободова

Серединна ободова

Клубово-ободова

814 / 6307
Під час розтину тіла померлої в комі молодої людини, виявлено: поширений тромбоемболічний інфаркт лівої півкулі мозку, велика септична селезінка, імунокомплексний гломерулонефрит, виразки на стулках аортального клапану, що прикриті поліпоподібними тромбами з колоніями стафілококів. Яке захворювання викликало церебральну тромбоемболію?

Септичний бактеріальний ендокардит

Гострий ревматичний вальвуліт

Септицемія

Септикопіємія

Ревматичний тромбендокардит

815 / 6307
Внаслідок дії електричного струму на збудливу клітину виникла деполяризація її мембрани. Рух яких іонів через мембрану є причиною деполяризації?

НСО

Cl-

Са2+

Na+

K+

816 / 6307
Постраждалому з колотою раною передньої стінки шлунка надається хірургічна допомога. До якого утворення порожнини очеревини потрапив вміст шлунка?

Лівий мезентеріальний синус

Печінкова сумка

Передшлункова сумка

Правий мезентеріальний синус

Сальникова сумка

817 / 6307
Вагітній жінці, що мала в анамнезі декілька викиднів, призначена терапія, яка містить вітамінні препарати. Укажіть вітамін, який сприяє виношуванню вагітності:

Рутін

Піридоксальфосфат

Цианкобаламін

α-токоферол

Фолієва кислота

818 / 6307
У хворого з аневризмою правої підключичної артерії спостерігається осиплість голосу. З подразненням якого нерву це може бути пов’язане?

N.laringeus inferior smster

N.laringeus superior smster

N.laringeus reccurens dexter

N.laringeus reccurens srnlster

N.laringeus superior dexter

819 / 6307
До приймального відділення доставлений хворий зі скаргами на сухість у роті, світлобоязнь та порушення зору. Шкіра гіперемована, суха, зіниці розширені, тахікардія. При подальшому обстеженні був встановлений діагноз: отруєння алкалоїдами красавки. Який із лікарських засобів доцільно застосувати?

Дипіроксим

Пілокарпін

Армін

Діазепам

Прозерин

820 / 6307
У пацієнта після травми виникли паралічі, розлади дотикової чутливості з правого боку; зліва паралічі відсутні, але порушена больова та температурна чутливість. Яка причина цього явища?

Пошкодження рухової зони кори головного мозку

Пошкодження стовбура мозку

Пошкодження мозочка

Пошкодження середнього мозку

Однобічне пошкодження спинного мозку з правого боку

821 / 6307
Оглядаючи черевну порожнину потерпілого з проникним пораненням передньої черевної стінки, хірург помітив пошкодження малого сальника, з якого витікала яскраво-червона (артеріальна) кров. Яка артерія виявилася пошкодженою?

A.gastroepiploica sinistra

A.hepatica propria

A.gastroepiploica dextra

A.lienalis

A.hepatica communis

822 / 6307
У пологовому будинку народилася дитина з численними порушеннями як зовнішніх так і внутрішніх органів - серця, нирок, травної системи. Був встановлений попередній діагноз - синдром Дауна. За допомогою якого методу можна підтвердити цей діагноз?

Близнюковий

Цитогенетичний

Біохімічний

Популяційно-статистичний

Генеалогічний

823 / 6307
У процесі метаболізму в організмі людини утворюються активні форми кисню, у тому числі супероксидний аніонрадикал. За допомогою якого ферменту інактивується цей аніон?

Глутатіонредуктаза

Пероксидаза

Глутатіонпероксидаза

Супероксиддисмутаза

Каталаза

824 / 6307
Експериментатору необхідно якнайшвидше виробити умовний рефлекс у собаки. На базі якого безумовного рефлексу доцільно виробляти умовний рефлекс у цьому випадку?

Орієнтувальний

Захисний

Статевий

Травний

Міостатичний

825 / 6307
Студент старанно конспектує лекцію. Якість конспектування значно погіршилася, коли сусіди стали розмовляти. Який вид гальмування в корі головного мозку є причиною цього?

Згасаюче

Позамежове

Диференціювальне

Запізніле

Зовнішнє

826 / 6307
Під час патронажу лікар виявив у дитини симетричну шорсткість щік, діарею, порушення нервової діяльності. Нестача яких харчових факторів є причиною такого стану?

Нікотинова кислота, триптофан

Лізин, аскорбінова кислота

Метіонін, ліпоєва кислота

Фенілаланін, пангамова кислота

Треонін, пантотенова кислота

827 / 6307
Для вивчення локалізації біосинтезу білка в клітинах, миші ввели мічені амінокислоти аланін та триптофан. Біля яких органел буде спостерігатися накопичення мічених амінокислот?

Лізосоми

Гладенька ЕПС

Клітинний центр

Рибосоми

Апарат Гольджі

828 / 6307
У хворих на колагеноз має місце процес деструкції сполучної тканини. Це підтверджується збільшенням у крові:

Вмісту уратів

Вмісту оксипроліну та оксилізину

Активності трансаміназ

Активності ізоферментів ЛДГ

Вмісту креатину та креатиніну

829 / 6307
У 6-місячної дитини спостерігалися часті та інтенсивні підшкірні крововиливи. Призначення синтетичного аналога вітаміну K (вікасолу) дало позитивний ефект. У укарбоксилюванні глутамінової кислоти якого білка зсідальної системи крові бере участь цей вітамін?

Антигемофільний глобулін A

Фактор Розенталя

Фібриноген

Протромбін

Фактор Хагемана

830 / 6307
У хворого 60 років виявлено розширення вен стравоходу, прямої кишки та підшкірних вен передньої черевної стінки. Система якої вени ушкоджена?

Ворітна

Верхня брижова

Непарна

Верхня порожниста

Нижня порожниста

831 / 6307
У хворого лікар діагностував гостру гонорею. З анамнезу відомо, що раніше він переніс гонорею та вилікування було повним. До якої категорії інфекцій можна віднести це нове захворювання?

Рецидив

Аутоінфекція

Реінфекція

Вторинна інфекція

Суперінфекція

832 / 6307
У людини зменшений діурез, гіпернатріємія, гіпокаліємія. Гіперсекреція якого гормону може бути причиною таких змін?

Передсердний натрійуретичний фактор

Вазопресин

Адреналін

Альдостерон

Паратгормон

833 / 6307
Під час вивчення фаз мітотичного циклу корінця цибулі знайдено клітину, в якій хромосоми лежать в екваторіальній площині, утворюючи зірку. На якій стадії мітозу перебуває клітина?

Телофаза

Інтерфаза

Анафаза

Метафаза

Профаза

834 / 6307
У хворого напад тахікардії. Які мембранні циторецептори кардіоміоцитів доцільно заблокувати, щоб припинити напад?

М- та Н-холінорецептори

М-холінорецептори

в-адренорецептори

а-адренорецептори

Н-холінорецептори

835 / 6307
У крові хворого концентрація альбумінів складає 2,8 г/л, підвищена концентрація лактатдегідрогенази 5 (ЛДГ5). Про захворювання якого органа це свідчить?

Серце

Нирка

Легеня

Печінка

Селезінка

836 / 6307
Під час розтину тіла мертвонародженої дитини виявлено аномалію розвитку серця: шлуночки не розмежовані, з правої частини виходить суцільний артеріальний стовбур. Для якого класу хребетних тварин характерна подібна будова серця?

Рептилії

Риби

Птахи

Ссавці

Амфібії

837 / 6307
У чоловіка 59 років спострерігаються ознаки паренхіматозної жовтяниці та портальної гіпертензії. Під час гістологічного дослідження пункційного біоптату печінки знайдено: балково-часточкова будова порушена, частина гепатоцитів має ознаки жирової дистрофії, утворюються порто-портальні сполучнотканинні септи з формуванням псевдочасточок, з наявністю пері-портальних лімфомакрофагальних інфільтратів. Який найбільш вірогідний діагноз?

Токсична дистрофія

Хронічний гепатоз

Алкогольний гепатит

Цироз печінки

Вірусний гепатит

838 / 6307
До гастроентерологічного відділення надійшов хворий із запаленням жовчних шляхів. У порціях жовчі виявлено рухомі найпростіші грушоподібної форми, двоядерні, з опорним стрижнем - аксостилем. Яке протозойне захворювання діагностується у хворого?

Балантидіаз кишковий

Амебна дизєнтєрія

Трихомоноз

Амебіаз кишковий

Лямбліоз

839 / 6307
У хворого 30 років з рiзаною раною передпліччя виникло порушення розгинання пальців кисті. Про пошкодження якого нерва це свідчить?

М’язовошкірний

Присередній шкірний нерв передпліччя

Серединний

Ліктьовий

Променевий

840 / 6307
Під час обробки атипових кардіо-міоцитів синоатріального вузла біологічно активною речовиною, зареєстровано збільшення їх мембранного потенціалу через збільшену проникність для іонів калію. Яка біологічно активна речовина впливала на кардіоміоцити?

Тироксин

Атріопептид

Ацетилхолін

Адреналін

Норадреналін

841 / 6307
Під час експерименту подразнюють скелетний м’яз серією електричних імпульсів. Який вид м’язового скорочення буде виникати, якщо кожний наступний імпульс надходить у періоді вкорочення попереднього поодинокого м’язового скорочення?

Контрактура м’яза

Суцільний тетанус

Серія поодиноких скорочень

Асинхронний тетанус

Зубчастий тетанус

842 / 6307
У хворого через декілька годин після опіку в ділянці гіперемії та набряку шкіри з’явилося вогнище некрозу. Який головний механізм забезпечує посилення руйнівних явищ в осередку запалення?

Проліферація фібробластів

Еміграція лімфоцитів

Первинна альтерація

Вторинна альтерація

Діапедез еритроцитів

843 / 6307
У чоловіка 42 років, який страждає на подагру, в крові підвищена концентрація сечової кислоти. Для зниження рівня сечової кислоти йому призначено алопуринол. Конкурентним інгібітором якого ферменту є алопуринол?

Гіпоксантинфосфорибозилтрансфераза

Ксантиноксидаза

Аденозиндезаміназа

Аденінфосфорибозилтрансфераза

Гуаніндезаміназа

844 / 6307
Хлопчик 13 років скаржиться на загальну слабкість, запаморочення, втомлюваність. Спостерігається відставання у розумовому розвитку. При обстеженні виявлено високу концентрацію валіну, ізолейцину, лейцину в крові та сечі. Сеча специфічного запаху. Який найбільш вірогідний діагноз?

Хвороба 'кленового сиропу'

Тирозиноз

Базедова хвороба

Хвороба Аддісона

Гістидинемія

845 / 6307
Хворий звернувся із скаргами на загальну слабкість, головний біль, нудоту, блювання, рідкі випорожнення з домішками слизу та крові. При мікроскопії дуоденального вмісту та при дослідженні свіжих фекалій виявлено рухомі личинки. Який найбільш вірогідний діагноз?

Анкілостомідоз

Трихоцефальоз

Стронгілоїдоз

Ентеробіоз

Дракункульоз

846 / 6307
Біля інфікованої рани збільшилися регіонарні лімфовузли. Під час гістологічного дослідження в них виявлено збільшення кількості макрофагів, лімфоцитів та лімфатичних фолікулів у кірковому шарі, а також велику кількість плазматичних клітин. Який процес відображують виявлені у лімфатичних вузлах гістологічні зміни?

Набута недостатність лімфоїдної тканини

Антигенна стимуляція

Природжена недостатність лімфоїдної тканини

Пухлинна трансформація

Реакція гіперчутливості

847 / 6307
Жінка 62 років скаржиться на частий біль у ділянці грудної клітки та хребта, переломи ребер. Лікар припустив мієломну хворобу (плазмоцитому). Який з перерахованих нижче лабораторних показників буде мати найбільше діагностичне значення?

Парапротеїнемiя

Протєїнурія

Гіпопротеїнемія

Гiперальбумiнемiя

Гіпогло6улінємія

848 / 6307
У гістопрепараті представлено паренхіматозний орган, що має кіркову та мозкову речовину. Кіркова речовина утворена тяжами епітеліоцитів, між якими проходять кровоносні капіляри; тяжі формують три зони. Мозкова речовина складається з хромафіноцитів та венозних синусоїдів. Який орган має зазначені морфологічні ознаки?

Щитовидна залоза

Наднирник

Лімфатичний вузол

Нирка

Тимус

849 / 6307
Хворий надійшов зі скаргами на диспептичні розлади, мелену, гемороїдальні кровотечі. Під час дослідження виявлено розширення сітки венозних судин на передній стінці живота, збільшення його розмірів. Яка патологія може мати такі симптоми?

Кишкова аутоінтоксикація

Ентерит

Виразкова хвороба

Портальна гіпертензія

Коліт

850 / 6307
Хворому, що страждає на склеродермію (колагенове захворювання), було призначено імунодепресант, який відноситься до групи протималярійних засобів, похідних хіноліну. Хворий приймав препарат тривало. Внаслідок цього у нього знизилася вага, посивіло волосся, зменшилася гострота зору, з’явилося миготіння перед очима. Було призначено обстеження у окуліста, який відмінив препарат. Визначте, яким препаратом лікували хворого:

Кризанол

Хінгамін

Хіноцид

Метотрексат

Циклофосфан

851 / 6307
У загальному вигляді генетичний апарат еукаріот є таким: ек-зон-інтрон-екзон. Така структурно-функціональна організація гена зумовлює особливості транскрипції. Якою буде про-і-РНК відповідно до згаданої схеми?

Інтрон-екзон

Екзон-інтрон-екзон

Екзон-екзон

Екзон-інтрон

Екзон-екзон-інтрон

852 / 6307
хворого виявлено підвищення активності ЛДГ1 ЛДГ2 52. , АсАТ, креатинкінази. В якому органі хворого найбільш вірогідний розвиток патологічного процесу?

Нирки

Підшлункова залоза

Скелетні м’язи

Серце

Печінка

853 / 6307
Споживання пацієнтом протягом тривалого часу забруднених овочів та фруктів призвело до отруєння нітратами. Яке похідне гемоглобіну утворилося у крові даного хворого?

Hb O2

Hb NHCOOH

Hb CN

Hb-OH

Hb СО

854 / 6307
3 роки тому хворій 34 років було встановлено діагноз хронічного гломерулонефриту. За останні 6 місяців з’явилися набряки. Що лежить в основі їх розвитку?

Гіперальдостеронізм

Гіперпродукція вазопресину

Протеїнурія

Гіперосмолярність плазми

Порушення білковоутворюючої функції печінки

855 / 6307
Під час роботи щодо ліквідації наслідків аварії на АЕС, робітник одержав дозу опромінення 500 рентген. Скаржиться на головний біль, нудоту, запаморочення. Які зміни кількості лейкоцитів можна очікувати у хворого через 10 годин після опромінення?

Лейкемія

Агранулоцитоз

Лейкопенія

Нейтрофільний лейкоцитоз

Лімфоцитоз

856 / 6307
З хімічного виробництва до токсикологічного відділення доставлений хворий з отруєнням парами ртуті. Який препарат слід використати в даній ситуації?

Ентеросорбент СКН

Активоване вугілля

Унітіол

Налоксон

Ізонітрозин

857 / 6307
У раціоні людини велика кількість вуглеводів. Кількість яких структур збільшиться у цитоплазмі гепатоцитів?

Вільні рибосоми

Краплини жиру

Лізосоми

Гранули глікогену

Включення ліпофусцину

858 / 6307
Жінка 63 років має ознаки ревматоїдного артриту. Підвищення рівня якого з перерахованих нижче показників крові буде найбільш значущим для підтвердження діагнозу?

R-глікозидаза

Кисла фосфатаза

Сумарні глікозаміноглікани

Ліпопротеїди

Загальний холестерин

859 / 6307
Під час розтину на зовнішній поверхні аортального клапана, виявлені великі (1-2 см) буровато-червоні, крихкі нашарування, які прикривають виразкові дефекти. Який найбільш вірогідний діагноз?

Дифузний ендокардит

Поворотній бородавчастий ендокардит

Гострий бородавчастий ендокардит

Фібропластичний ендокардит

Поліпозно-виразковий ендокардит

860 / 6307
Після крововиливу в мозок у хворого стали неможливими активні рухи лівих руки та ноги. Тонус м’язів цих кінцівок підвищено, їх спінальні рефлекси різко підсилені, розширено зони рефлексів. Рефлекс Бабінського зліва. Який вид розладу рухів має місце у хворого?

В’ялий параліч

Центральний параліч

Периферичний параліч

Спінальний шок

Рефлекторний параліч

861 / 6307
Під час рентгенологічного дослідження кісток основи черепа виявлено збільшення турецького сідла та деструкція окремих його ділянок. Пухлинне розростання якого анатомічного утворення може спричинити таке руйнування кістки?

Гіпофіз

Чотиригорбове тіло

Епіфіз

Зоровий горб

Колінчасті тіла

862 / 6307
Хворий відзначає часті проноси, особливо після вживанняжирної їжі, схуднення. Лабораторні дослідження показали наявність стеатореї; кал гіпохолічний. Що може бути причиною такого стану?

Недостатність панкреатичної фосфо-ліпази

Обтурація жовчних шляхів

Незбалансована дієта

Запалення слизової оболонки тонкої кишки

Недостатність панкреатичної ліпази

863 / 6307
Під час розтину тіла чоловіка 56 років, який страждав на фіброзно-кавернозний туберкульоз легень, знайдено збільшену у розмірах щільну селезінку. На розрізі тканина її коричнево-рожевого кольору, гладенька, з віскоподібною поверхнею. Який з перелічених патологічних процесів у селезінці найбільш вірогідний?

Сагова селезінка

Цианотична індурація

Сальна селезінка

Порфірна селезінка

Глазурна селезінка

864 / 6307
У хворого з гнійничковими ураженнями шкіри виділений збудник, який на кров’яному агарі утворює жовті колонії округлої форми, середніх розмірів, оточені зоною гемолізу. У мазках з колоній - коки, розташовані скупченнями неправильної форми, грампозитивні. Виділена культура оксидазо- і каталазопозитивна, ферментує маніт, синтезує плазмокоагулазу. Який вид збудника виділений?

Staphylococcus epidermidis

Staphylococcus aureus

Streptococcus agalactiae

Staphylococcus saprophyticus

Streptococcus pyogenes

865 / 6307
До стоматолога звернулася мати зі скаргами на руйнування зубів у дитини 2-х років. Під час огляду молочні зуби деформовані, уражені карієсом, у шийки коричнева облямівка. З анамнезу встановлено, що мати під час вагітності приймала антибіотики без контролю лікаря. Вкажіть, антибютики якої групи, що мають найбільш виражену тератогенну дію, могла приймати мати?

Макроліди

Тетрацикліни

Пєніциліни

Цефалоспорини

Аміноглікозиди

866 / 6307
У хворого на хронічну серцеву недостатність у процесі лікування препаратами наперстянки виникли симптоми, що свідчать про початок токсичної дії серцевих глікозидів. Який препарат необхідно призначити для зменшення негативної дії серцевих глікозидів?

Калію хлорид

Етімізол

Кофеїн-бензоат натрію

Діпіроксим

Атропіну сульфат

867 / 6307
До лікаря звернувся хворий зі скаргами на непереносимість сонячної радіації. Мають місце опіки шкіри та порушення зору. Попередній діагноз: альбінізм. Порушення обміну якої амінокислоти відзначається у цього пацієнта?

Триптофан

Лізин

Пролін

Аланін

Тирозин

868 / 6307
До лікарні швидкої допомоги доставили дитину 7 років у стані алергічного шоку, який розвинувся після того, як її вжалила оса. У крові підвищена концентрація гістаміну. В результаті якої реакції утворився цей амін?

Дегідрування

Декарбоксилювання

Дезамінування

Гідрооксилювання

Відновлення

869 / 6307
Під час огляду хворого лікар запідозрив синдром Іценка-Кушинга. Визначення якої речовини в крові хворого підтвердить припущення лікаря?

Адреналін

Холестерин

Ретинол

Кортизол

Токоферол

870 / 6307
У дитини 6 місяців спостерігається різке відставання в психомоторному розвитку, напади судом, бліда шкіра з екзематозними змінами, біляве волосся, блакитні очі. У цієї дитини найбільш вірогідно дозволить встановити діагноз визначення концентрації у крові та сечі:

Лейцину

Триптофану

Валінду

Гістидину

Фенілпірувату

871 / 6307
Тромбоз коронарної артерії спричинив розвиток інфаркту міокарда. Які механізми ушкодження кардіоміоцитів є домінуючими при цьому захворюванні?

Ацидотичні

Ліпідні

Протеїнові

Електролітно-осмотичні

Кальцієві

872 / 6307
У крові хворого виявлено низький рівень альбумінів та фібриногену. Зниження активності яких органел гепатоцитів найбільш вірогідно обумовлює це явище?

Гранулярна ендоплазматична сітка

Мітохондрії

Лізосоми

Агранулярна ендоплазматична сітка

Комплекс Гольджі

873 / 6307
Під час обстеження хворого була виявлена недостатня кількість імуноглобулінів. Які з клітин імунної системи їх продукують?

Т-супресори

Плазматичні

Т-хелпери

Т-кілери

Плазмобласти

874 / 6307
Жінка 33 років страждає на гепатоцеребральну дистрофію (хвороба Вільсона). У крові - знижений вміст церулоплазміну. У сечі - різко підвищений вміст амінокислот. Посиленням якого процесу зумовлені ці зміни?

Комплексоутворення амінокислот з міддю

Розпад тканинних 6ілків

Переамінування амінокислот

Ілюконеогенез

Синтез сечовини

875 / 6307
В одному з гірських селищ спостерігалася масова загибель гризунів, що супроводжувалася захворюванням мешканців селища. Для цієї хвороби було притаманне швидке підвищення t0 до 400 С, виражена інтоксикація, збільшення пахвинних лімфовузлів. У препаратах-мазках з трупного матеріалу виявлені грамнегативні палички овоїдної форми з біполярним забарвленням. Які мікроорганізми є збудниками цього інфекційного захворювання?

Збудник сибірки

Збудник туляремії

Стафілокок

Збудник чуми

Клостридії

876 / 6307
До терапевтичного відділення надійшов хворий з тривалою бронхопневмонією. Антибіотикотерапія не дала належного ефекту. Який препарат для підвищення імунного статусу доцільно додати до комплексної терапії цього хворого?

Парацетамол

Сульфокамфокаїн

Тималін

Аналгін

Димедрол

877 / 6307
У хворого на хронічний цистіт у біоптаті слизової оболонки сечового міхура разом з перехідним епітелієм виявлені вогнища багатошарового плоского незроговілого епітелію. Який процес лежить в основі зазначених змін в епітелії?

Дисплазія

Метаплазія

Гіперкератоз

Дистрофія

Гіперплазія

878 / 6307
Який механізм тепловіддачі найбільш ефективно реалізується під час перебування людини при температурі навколишнього середовища +350 С та 80% вологості повітря?

Радіація

Конвекція

Випаровування

Теплопроведення

879 / 6307
У хворого спостерігається підвищення опору вигнанню крові з лівого шлуночка. При якому із перерахованих патологічних процесів може виникнути така ситуація?

Артеріальна гіпотензія

Аортальний стеноз

Недостатність аортального клапана

Мітральний стеноз

Емболія легеневої артерії

880 / 6307
У представників однієї з людських популяцій тіло подовжене, широка варіабельність зросту, знижений об’єм м’язової маси, подовжені кінцівки, зменшена у розмірах і об’ємі грудна клітка, підвищене потовиділення, знижені показники основного обміну та синтезу жирів. До якого адаптивного типу людей відноситься дана популяція?

Гірський

Тропічний

Проміжний

Тип зони помірного клімату

Арктичний

881 / 6307
Під час гістологічного дослідження біопсійного матеріалу шлунка виявили малу кількість чи повну відсутність парієтальних клітин у залозах. Слизову оболонку якої ділянки шлунку вивчали?

Кардіальний відділ

Пілоричний відділ

Дно

Тіло

882 / 6307
До травматологічного пункту доставлено хворого з ушкодженням нижньої кінцівки внаслідок прямого удару по внутрішній поверхні середньої третини гомілки. Перелом якого анатомічного утворення найбільш вірогідний?

Діафіз великогомілкової кістки

Проксимальний епіфіз малогомілкової кістки

Дистальний епіфіз малогомілкової кістки

Проксимальний епіфіз великогомілкової кістки

Дистальний епіфіз великогомілкової кістки

883 / 6307
Тестостерон та його аналоги збільшують масу скелетних м’язів, що дозволяє використовувати їх для лікування дистрофій. Взаємодією з яким клітинним субстратом зумовлена ця дія?

Ядерні рецептори

Мембранні рецептори

Рибосоми

Хроматин

Білки-активатори транскрипції

884 / 6307
У вогнищі запалення підвищується проникність судин мікроциркуляторного русла, у них збільшується гідродинамічний опір. У міжклітинній рідині підвищується осмотична концентрація та дисперсність білкових структур. Який вид набряку буде спостерігатися у даному випадку?

Колоїдно-осмотичний

Мембраногенний

Гідродинамічний

Змішаний

Лімфогенний

885 / 6307
Під час дослідження коронарних артерій виявлені атеросклеротичні бляшки з кальцинозом, що закривають просвіт судин на 1/3. У м’язі дрібні множинні білуваті прошарки сполучної тканини. Як називається процес, виявлений у міокарді?

Міокардит

Тигрове серце

Післяінфарктний кардіосклероз

Інфаркт міокарда

Дифузний кардіосклероз

886 / 6307
У процесі онтогенезу у людини на організменному рівні проявилися наступні зміни: зменшилася життєва ємність легень, збільшився артеріальний тиск, розвинувся атеросклероз. Який період онтогенезу найбільш вірогідний у цьому випадку?

Похилий вік

Юнацький

Підлітковий

Молодий вік

Початок зрілого віку

887 / 6307
Хворий 50 років з метою лікування черевного тифу почав приймати левоміцетин, але на наступний день його стан погіршився, температура підвищилася до 39,60 С. Чим пояснити погіршення стану хворого?

Дією ендотоксинів збудника

Нечутливістю збудника до левоміцетину

Реінфекцією

Алергічною реакцією

Приєднанням вторинної інфекції

888 / 6307
Як зміниться фармакологічна активність препарату з високою спорідненістю до білків плазми крові при виникненні гіпоальбумінемії?

Дещо зменшиться

Не зміниться

Зникне

Суттєво зменшиться

Підвищиться

889 / 6307
У людини в стані спокою значно збільшена робота м’язів видиху. Що з наведеного може бути причиною цього?

Звуження дихальних шляхів

Рідке дихання

Поверхневе дихання

Негативний внутрішньоплевральний тиск

Зменшення хвилинного об’єму дихання

890 / 6307
У місті епідемія грипу. Який препарат доцільно використати для неспецифічної профілактики захворювання?

Пеніцилін

Протигрипозний імуноглобулін

Протигрипозна сироватка

Лейкоцитарний інтерферон

Протигрипозна вакцина

891 / 6307
Під час розтину трупа чоловіка 56 років у термінальному відділі тонкої кишки виявлено декілька виразок діаметром 4-5 см. Краї виразок піднімаються над поверхнею слизової оболонки; стінки вкриті сірувато-жовтуватими масами, які кришаться. Реакція Відаля позитивна. Який найбільш вірогідний діагноз?

Паратиф

Поворотний тиф

Хвороба Крона

Черевний тиф

Дизентерія

892 / 6307
Під час огляду дитини, яка перехворіла на кір, у м’яких тканинах щік та промежини виявлено нечітко відмежовані, набряклі, червоно-чорного кольору ділянки, у яких виявляється помірна флюктуація. Яке ускладнення розвинулося у дитини?

Пролежень

Волога гангрена (нома)

Трофічна виразка

Газова гангрена

Суха гангрена

893 / 6307
У людини вимірють внутрішньо-плевральний тиск. У якій фазі людина затримала дихання, якщо величина тиску дорівнює - 7,5 см вод.ст.?

Форсований вдих

Спокійний видих

Спокійний вдих

Форсований видих

894 / 6307
В умовах експерименту у кролика перев’язали ниркову артерію, що через 2 тижні призвело до суттєвого збільшення артеріального тиску. У результаті збільшення секреції якої біологічно активної речовини це відбулося?

Ренін

Адреналін

Норадреналін

Натрійуретичний гормон

Вазопресин

895 / 6307
Куди треба провести катетер для забору лімфи з грудної лімфатичної протоки?

У лівий венозний кут

У верхню порожнисту вену

У нижню порожнисту вену

У ліву пахвинну вену

У правий венозний кут

896 / 6307
У дитячому садку планується проведення вакцинації проти коклюшу. Який препарат необхідно використати з цією метою?

Вакцина БЦЖ

АДП анатоксин

Нормальний у-глобулін

Вакцина АКДП

Типоспецифічна сироватка

897 / 6307
У студента перед іспитом виникла тахікардія. Які зміни на ЕКГ свідчитимуть про її наявність?

Подовження інтервалу R-R

Вкорочення інтервалу P-Q

Подовження сегменту Q-T

Вкорочення інтервалу R-R

Подовження комплексу QRS

898 / 6307
Перед відрядженням за кордон лікарю, з метою профілактики малярії, призначено засіб з гістомизонтоцидною дією. Який препарат він одержував?

Хлоридин

Примахін

Хінін

Доксициклін

Бісептол

899 / 6307
У хлопчика 5 років, якого непокоїть анальний свербіж, виявлені черв’яки класу нематод (гострики). Оберіть лікарський засіб для дегельмінтації дитини:

Празіквантель

Фенасал

Сім’я гарбуза

Аміноакрихін

Мебендазол

900 / 6307
Після ремонту автомобілю в гаражному приміщенні водій потрапив до лікарні з симптомами отруєння вихлопними газами. Вміст якої речовини у крові буде підвищено?

Глікозильований гемоглобін

Карбгемоглобін

Оксигемоглобін

Карбоксигемоглобін

Метгемоглобін

901 / 6307
У жінки через 6 місяців після пологів розвинулася маткова кровотеча. Під час гінекологічного обстеження у порожнині матки виявлена тканина темно-червоного кольору з множинними порожнинами, що нагадує 'губку'. Під час мікроскопічного дослідження пухлини у лакунах крові виявлені атипові світлі епітеліальні клітини Лангханса та гігантські клітини синцитіотрофобласта. Яка це пухлина?

Міхуровий занос

Плоскоклітинний незроговілий рак

Аденокарцинома

Фіброміома

Хоріонепітеліома

902 / 6307
Хвора 13 років знаходиться на стаціонарному лікуванні в гематологічному відділенні обласної дитячої лікарні з діагнозом залізодефіцитна анемія. Який тип гіпоксії має місце у цієї хворої?

Тканинна

Циркуляторна

Дихальна

Змішана

Гемічна

903 / 6307
Хворому, що страждає на тромбоемболічну хворобу, призначений штучний антикоагулянт пелентан. Антагоністом якого вітаміну є цей препарат?

Вітамін С

Вітамін Е

Вітамін А

Вітамін D

Вітамін К

904 / 6307
У хворої гінекологічного відділення виникли симптоми внутрішньої кровотечі. Який засіб слід призначити з метою пригнічення фібринолізу та зупинки кровотечі?

Дицинон

Контрикал

Фібриноген

Хлористий кальцій

Вікасол

905 / 6307
У хворого з клінічними ознаками первинного імунодефіциту виявлено порушення функції антигенпрезентації імунокомпетентним клітинам. Порушення функціонування яких клітін може бути причиною цього?

В-лімфоцити

Т-лімфоцити

Макрофаги, моноцити

0-лімфоцити

Фібробласти

906 / 6307
У хворого 37 років на фоні тривалого застосування антибіотиків спостерігається підвищена кровоточивість при невеликих пошкодженнях. У крові - зниження активності II, VII, X факторів згортання крові; подовження часу згортання крові. Нестачею якого вітаміну обумовлені ці зміни?

Вітамін С

Вітамін Е

Вітамін К

Вітамін А

Вітамін D

907 / 6307
У хворого на хронічний гломерулонефрит швидкість клубочкової фільтрації (ШКФ) знижена до 20% від нормальної. Що є основною причиною зниження ШКФ у даному випадку?

Зменшення кількості функціонуючих нефронів

Тромбоз ниркових артерій

Ішемія нирок

Тубулопатія

Обтурація сечовивідних шляхів

908 / 6307
5 років температура підвищилася до 40о 108. С, з’явився різкий головний біль, блювання, неспокій, озноб. Через 4 дні з’явилася геморагічна висипка на шкірі, олігоурія та надниркова недостаність, що і стало причиною смерті. При бактеріологічному дослідженні мазків з глотки виявлено менінгокок. Яка форма хвороби виявлена?

Менінгоенцефаліт

Менінгококцемія

Менінгококовий менінгіт

Гідроцефалія

Менінгококовий назофарингіт

909 / 6307
У жінки 37 років під час оперативного втручання на органах малого тазу виникла необхідність перев’язати маткову артерію. Яке з утворень може бути випадково перев’язаним разом з нею?

Сечовід

Внутрішня клубова вена

Маткова труба

Кругла зв’язка матки

Сечівник

910 / 6307
Під час гістологічного дослідження тимуса чоловіка 40 років, визначено зменшення частки паренхіматозних елементів залози, збільшення частки жирової та пухкої сполучної тканини, збагачення її тимусними тільцями при незмінній загальній масі органу. Як зветься таке явище?

Гіпотрофія

Вікова інволюція

Акцідентальна інволюція

Атрофія

Дистрофія

911 / 6307
Підліток 12 років протягом 3 місяців втратив 7 кг маси тіла. Вміст глюкози у крові становить 20 ммоль/л. Несподівано розвинулася кома. Який вид цукрового діабету найбільш вірогідний у хлопчика?

Гіпертіреоїдний

Гіпофізарний

Інсулінозалежний (I тип)

Стероїдний

Інсулінонезалежний (II тип)

912 / 6307
Внаслідок тривалого голодування в організмі людини швидко зникають резерви вуглеводів. Який з процесів метаболізму підтримує при цьому вміст глюкози в крові?

Глікогеноліз

Анаеробний гліколіз

Пентозофосфатний цикл

Глюконеогенез

Аеробний гліколіз

913 / 6307
У чоловіка 55 років, який протягом багатьох років страждає на недостатність мітрального клапана, виникла гостра серцева недостатність. Який патофізіологічний варіант недостатності серця спостерігається у цьому випадку?

Гіпоксичне ушкодження серця

Гостра тампонада серця

Перевантаження серця об’ємом

Коронарогенне ушкодження серця

Нейрогенне ушкодження серця

914 / 6307
У бактеріологічній лабораторії проводиться дослідження м‘ясних консервів на вміст ботулінічного токсину. Для цього дослідній групі мишей ввели екстракт із досліджуваного матеріалу та антитоксичну протиботулінічну сироватку типів А, В, Е; контрольній групі мишей ввели екстракт без протиботулінічної сироватки. Яку серологічну реакцію було використано?

Преципітації

Опсоно-фагоцитарна

Зв‘язування комплементу

Подвійної імунної дифузії

Нейтралізації

915 / 6307
Хворому, що страждає на хронічну серцеву недостатність, лікар порадив провести профілактичний курс лікування кардіотонічним препаратом з групи серцевих глікозидів, який приймають ентерально. Який препарат було рекомендовано хворому?

Кордіамін

Дигоксин

Строфантин

Кордарон

Корглікон

916 / 6307
Офтальмолог з діагностичною метою (розширення зіниць для огляду очного дна) використав 1 % розчин мезатону. Чим обумовлений мідріаз, викликаний препаратом?

Активація α2 адренорецепторів

Активація М-холінорецепторів

Активація α1 адренорецепторів

Блокада α1 адренорецепторів

Активація β1 адренорецепторів

917 / 6307
Чоловіка 45 років протягом останніх 3 років непокоїв сухий кашель; наростала задишка, легенева недостатність, швидко втрачалася вага. На розтині: легеневе серце; у легенях різко виражений фіброз з наявністю порожнин, що створюють картину 'медових сот'. Гістологічно: інтерстиційний фіброз з вираженою інфільтрацією строми лімфогістіоцитами з домішками нейтрофілів. Який найбільш вірогідний діагноз?

Бронхоектатична хвороба

Післязапальний пневмосклероз

Фіброзуючий альвеоліт

Хронічна бульозна емфізема

Пиловий пневмосклероз

918 / 6307
Під час огляду дитини педіатр відзначив відставання у фізичному та розумовому розвитку. У сечі різко підвищений вміст кетокислоти, що дає якісну кольорову реакцію з хлорним залізом. Яке порушення обміну речовин було виявлено?

Алкаптонурія

Фенілкетонурія

Цистинурія

Альбінізм

Тирозинемія

919 / 6307
Хвора 40 років надійшла до інфекційного відділення лікарні з високою температурою тіла. Об’єктивно: виражені менінгеальні симптоми. Проведено спиномозкову пункцію. Яке анатомічне утворення було пропунктовано?

Cisterna cerebellomedullaris posterior

Spatium subarachnoideum

Spatium subdurale

Spatium epidurale

Cavum trigeminale

920 / 6307
Тварині внутрішньовенно ввели концентрований розчин хлориду натрію, що зумовило зниження реабсорбції іонів натрію у канальцях нирок. Внаслідок яких змін секреції гормонів це відбувається?

Збільшення вазопресину

Збільшення альдостерону

Зменшення вазопресину

Зменшення альдостерону

Зменшення натрійуретичного фактора

921 / 6307
У холодну погоду з вітром люди замерзають швидше, ніж за відсутності вітру. Причиною цього є те, що вітер, вперш за все, збільшує віддачу тепла шляхом:

Конвекції

Випаровування

Радіації

Теплопроведення

922 / 6307
Під час бігу на довгі дистанції скелетна мускулатура тренованої людини використовує глюкозу з метою отримання енергії АТФ для м’язового скорочення. Вкажіть основний процес утилізації глюкози в цих умовах:

Ілікогеноліз

Аеробний гліколіз

Ілюконеогенез

Анаеробний гліколіз

Ілікогенез

923 / 6307
Дитина 10 років під час гри порізала ногу відламком скла та була направлена до поліклініки для введення протиправцевої сироватки. З метою попередження розвитку анафілактичного шоку лікувальну сироватку вводили за Безредкою. Який механізм лежить в основі подібного способу гіпосенсибілізації організму?

Зв’язування рецепторів до IgE на тучних клітинах

Блокування синтезу медіаторів тучних клітин

Стимуляція синтезу антиген-специфічних IgG2

Зв’язування фіксованих на тучних клітинах IgE

Стимуляція імунологічної толерантності до антигену

924 / 6307
У хворого через добу після апендектомії у крові визначається нейтрофільний лейкоцитоз із регенеративним зсувом. Який найбільш вірогідний механізм розвитку лейкоцитозу у даному випадку?

Уповільнення еміграції лейкоцитів у тканини

Уповільнення руйнування лейкоцитів

Посилення лейкопоезу та уповільнення еміграції лейкоцитів у тканини

Посилення лейкопоезу

Перерозподіл лейкоцитів у організмі

925 / 6307
У хворого з широким інфарктом міокарда розвинулася серцева недостатність. Який патогенетичний механізм її розвитку?

Гостра тампонада серця

Перевантаження серця об’ємом

Реперфузійне ураження міокарда

Перевантаження серця тиском

Зменшення маси функціонуючих кардіоміоцитів

926 / 6307
Під час визначення групової належності крові за системою АВ0, аглютинацію еритроцитів досліджуваної крові викликали стандартні сироватки I та II груп, та не викликала сироватка III групи. Кров якої групи досліджується?

АВ(IV)

Неможливо визначити

А(II)β

В(III)α

O(I)α та β

927 / 6307
Хворий 50 років скаржиться на спрагу, п’є багато води; виражена поліурія. Глюкоза крові - 4,8 ммоль/л. У сечі глюкози та ацетонових тіл немає, сеча безбарвна, питома вага- 1,002 - 1,004. Яка причина поліурії?

Нестача вазопресину

Гіпотиреоз

Інсулінова недостатність

Альдостеронізм

Тиреотоксикоз

928 / 6307
До ендокринолога звернулася хвора 45 років із скаргами на підвищенний апетит, сухість слизових оболонок ротової порожнини, зростання діурезу. При обстеженні вперше виявлено інсуліннезалежний діабет. Який з названих препаратів доцільно призначити хворій?

Адіурекрин

Глібенкламід

Вазопресин

Окситоцин

Інсулін

929 / 6307
До реанімаційного відділення надійшов хворий із симптомами гострого отруєння морфіном - непритомність, гіпотермія, дихання Чейн-Стокса, гіпотензія, брадикардія, міоз. Який з перерахованих препаратів буде найефективнішим у даній ситуації?

Камфора

Кордіамін

Етимізол

Кофеїн

Налоксон

930 / 6307
До лікаря звернувся хворий із скаргами на біль у лівій половині шиї, який посилюється при рухах голови. Положення при якому біль не турбує - це нахил голови ліворуч з поверненим обличчям праворуч. Ураження якого м’яза спричиняє біль у даному випадку?

M.longus со11і

M.stemocleidomastoideus dexter

М.stemocleidomastoideu sinister

M.platisma sister

M.platisma dexter

931 / 6307
Хвора у віці 69 років страждає хронічними запорами, в основі яких лежить гіпотонія товстої кишки. Який з перерахованих препаратів доцільно призначити?

Бісакоділ

Натрію сульфат

Касторова олія

Прозерин

Магнію сульфат

932 / 6307
Хворий 45 років госпіталізований до лікарні зі скаргами на високу температуру, біль під час дихання, задишку та кашель. Після обстеження, лабораторної та рентгенодіагностики було встановлено діагноз - плеврит. Для евакуації ексудату була призначена плевральна пункція. В якому місці плевральної порожнини буде знаходитися найбільша кількість ексудату?

Реберно-медіастинальний синус

Під куполом плеври

Під коренем легенів

Діафрагмально-медіастинальний синус

Реберно-діафрагмальний синус

933 / 6307
Під час розтину трупа жінки 69 років підвищеного живлення, яка померла від гострого інфаркту міокарда, в інтимі коронарних артерій знайдені численні білуваті, щільні, вибухаючі формування, що різко звужують просвіт судин. Для якої стадії атеросклерозу властиві такі зміни?

Стадія утворення атероматозної виразки

Ліпосклероз

Атерокальциноз

Атероматоз

Ліпоїдоз

934 / 6307
Під час обстеження чоловіка 45 років, який перебуває довгий час на вегетеріанській рослинній дієті, виявлено негативний азотистий баланс. Яка особливість раціону стала причиною цього?

Недостатня кількість жирів

Надлишкова кількість вуглеводів

Надлишкова кількість води

Недостатня кількість вітамінів

Недостатня кількість білків

935 / 6307
Хворій на ревматоїдний поліартрит призначили нестероїдний протизапальний засіб - диклофенак натрію. Через деякий час у хворої виникло загострення супутнього захворювання, що змусило відмінити препарат. Яке супутнє захворювання могло призвести до відміни препарату?

Виразкова хвороба шлунка

Цукровий діабет

Бронхіальна астма

Гіпертонічна хвороба

Ішемічна хвороба серця

936 / 6307
Хворий, що лікувався з приводу неврозу сибазоном, скаржиться на зубний біль. Лікар призначив йому знеболювальний засіб у дозі, яка меньша за середню терапевтичну. Яке явище взяв до уваги лікар, зменшуючи дозу препарату?

Толерантність

Потенціювання

Кумуляція

Лікарська залежiсть

Сумація

937 / 6307
Під час дослідження сироватки крові пацієнта з ознаками імунодефіциту виявлено антитіла до білків gP120 та gP41. Наявність якої інфекції у хворого підтверджує цей результат?

ВІЛ-інфекція

HLTV-1-інфекція

TORCH-інфекція

HBV-інфекція

ЕСНО-інфекція

938 / 6307
На розтині тіла чоловіка 46 років на слизовій оболонці прямої та сигмовидної кишок виявилено множинні коричнево-зелені нашарування, крововиливи; у просвіті кишки слиз, невелика кількість крові; гістологічно - фібринозний коліт. При бактеріологічному дослідженні вмісту кишки висіяно S.Sonne. Який найбільш вірогідний діагноз?

Хвороба Крона

Дизентерія

Холера

Сальмонельоз

Ієрсініоз

939 / 6307
Під час розтину тіла чоловіка, який служив на підводному атомному човні, виявили наступне: спустошення в кістковому мозку (панмієлофтиз), анемія, лейкопенія, тромбоцитопенія, розпад лімфоцитів у лімфатичних вузлах, селезінці, лімфатичному апараті шлунково-кишкового тракту, крововиливи в на-днирники. Який найбільш вірогідний діагноз?

Кесонна хвороба

Гостра анемія

Вібраційна хвороба

Гостра променева хвороба

Гострий лейкоз

940 / 6307
Пацієнтка 58 років скаржиться на підвищену втомлюваність, зниження працездатності, сонливість, задишку під час швидкої ходи. У крові: ер.-4,61012/л, Hb- 92 г/л, кол.показн.- 0,6. У мазку

крові велика кількість анулоцитів та мікроцитів. Для якої анемії це притаманно?

Залізодефіцитна

Постгеморагічна

Перніціозна

Серповидноклітинна

Гемолітична

941 / 6307
Хворий багато років страждав на бронхіальну астму та помер від нападу ядухи. Під час гістологічного дослідження легень виявлено: в просвіті бронхіол та дрібних бронхів багато слизу з домішкою еозинофілів, склероз міжальвеолярних перетинок, розширення просвіту альвеол. Який механізм розвитку реакції гіперчутливості мав місце?

Реагінова реакція

Цитоліз, обумовлений лімфоцитами

Гранулематоз

Імунокомплексна реакція

Цитотоксична реакція

942 / 6307
У хворого 53 років після поранення в ділянку промежини відмічається довільне сечовиділення. Який з м’язів найбільш вірогідно ушкоджений?

M.transversus реrineі superficialis

M.bulbospongiosus

M.sphyncter uretrae

M.ischiocavemosus

M.transversus реrіnеі profundus

943 / 6307
При обстеженні на бактеріоносійство черевного тифу у сироватці крові кухарки шкільної їдальні виявлені Vі-антитіла. Яка з перелічених реакцій була використана у данному випадку?

Реакція Відаля

ІФА

РЗК

РНГА

РІФ

944 / 6307
Хвора надійшла до клініки зі скаргами на загальну слабкість, запаморочення, задишку. Незадовго до звернення у клініку вона приймала левоміцетин для профілактики кишкових інфекцій. У крові:

ер.- 1,91012/л, Hb- 58 г/л, колірний показник- 0,9; лейк.- 2,2109/л. Про яку анемію це свідчить?

Гіпопластична

Апластична

Метапластична

Гемолітична

Залізодефіцитна

945 / 6307
До клініки потрапила дитина 1 року з ознаками ураження м’язів. Після обстеження виявлений дефіцит карнитину у м’язах. Порушення якого процесу є біохімічною основою цієї патології?

Синтез актину та міозину

Транспорт жирних кислот у мітохондрії Регуляція рівня Ca2+ B. в мітохондріях

Субстратне фосфорилювання

Утилізація молочної кислоти

946 / 6307
Жінка 38 років скаржиться на підвищену пітливість, серцебиття, підвищення температури тіла у вечірні години. Основний обмін збільшений на 60%. Лікар встановив діагноз тиреотоксикоз. Які властивості тироксину приводять до підсилення теплопродукції?

Підвищує спряження окиснення та фосфорилування

Зменшує дезамінування амінокислот

Сприяє накопиченню ацетил-КоА

Зменшує в-окиснення жирних кислот

Розщеплює окисне фосфорилування

947 / 6307
У людини збережена смакова, але втрачена загальна чутливість структур ротової порожнини. Про ураження якого нерва це свідчить?

N.glossopharyngeus та n.vagus

N.trigemmus

N.glossopharyngeus

N.hypoglossus

N.vagus

948 / 6307
До приймального відділення лікарні надійшов непритомний юнак з ознаками отруєння морфіном. Відзначається поверхневе та рідке дихання, яке обумовлене пригніченням дихального центру. Який тип недостатності дихання виник при цьому?

Вентиляційна рестриктивна

Перфузійна

Дифузійна

Вентиляційна дисрегуляторна

Вентиляційна обструктивна

949 / 6307
У хворого з серцевою недостатністю виникла аритмія, під час якої частота скорочень передсердь була 70, а шлуночків - 35/хв. Порушення якої функції провідної системи серця спостерігалося у хворого?

Автоматизм

Збудливість та провідність

Провідність

Скоротливість

Збудливість

950 / 6307
До лікаря звернувся пацієнт з приводу пожовтіння склер та шкіри. Під час обстеження не виявлено ознак енцефалопатії, холемічного та ахолічного синдромів. Яка жовтяниця розвинулася у цього пацієнта?

Гемолітична

Ядерна

Механічна

Ензимопатична

Паренхіматозна

951 / 6307
Жінка 63 років приймала пірацетам для відновлення функції ЦНС після ішемічного інсульту. Стан хворої значно покращився. Який механізм дії даного препарату?

Блокада холінацетази

Блокада дофамін-гідроксилази

Блокада катехол-о-метилтрансферази

Блокада моноамінооксидази

Покращення метаболізму в ЦНС

952 / 6307
У хворого на рентгенограмі легень виявлено затемнення. Під час діагностичної експресбіопсії лімфатичного вузла бронха виявлено: сирний некроз, навколо якого розташовані епітеліоїдні та лімфоїдні пласти з домішками багатоядерних гігантських клітин. Вкажіть причину лімфаденіту:

Метастази раку

Аденовірусна інфекція

Туберкульоз

Сифіліс

Пневмонія

953 / 6307
Хворий помер з явищами уремії. На розтині нирки збільшені, в’ялої консистенції, кіркова речовина широка, набрякла, з червоним крапом; мозкова речовина темно-червона. Мікроскопічно у порожнині капсули ниркового тільця виявлені “півмісяці' які стискають капіляри, дистрофія нефроцитів, набряк та інфільтрація строми. Яке захворювання стало причиною смерті цього чоловіка?

Амілоїдоз нирок

Нефротичний синдром

Нефролітіаз

Гломерулонефрит

Пієлонефрит

954 / 6307
У хворого струс головного мозку, що супроводжується повторним блюванням та задишкою. В артерiальнiй крові: рН- 7,62; PC02 - 40 мм рт.ст. Яке порушення кислотноосновного стану у хворого?

Газовий ацидоз

Газовий алкалоз

Негазовий ацидоз

Негазовий алкалоз

955 / 6307
У хворого на жовчокам’яну хворобу розвинулася механічна жовтяниця. Під час обстеження встановлено, що камінь знаходиться у загальній жовчній протоці. Які жовчовивідні протоки утворюють обтуровану протоку?

Ductus hepaticus commurns et ductus cysticus

Ductus hepaticus dexter et ductus cysticus

Ductus hepaticus dexter et smster

Ductus hepaticus commurns et ductus choledochus

Ductus hepaticus smster et ductus cysticus

956 / 6307
У дитини першого року життя під час профілактичного огляду виявлено порушення мінералізації кісток. Нестача якого вітаміну може бути причиною цього?

Фолієва кислота

Токоферол

Рибофлавін

Кальциферол

Кобаламін

957 / 6307
У чоловіка 30 років методом непрямої калориметрії встановлено, що його основний обмін на 30% менше від належного. Знижена секреція гормонів якої залози (яких залоз) є причиною цього?

Наднирники

Підшлункова

Прищитоподібні

Епіфіз

Щитовидна

958 / 6307
Новонароджена дитина має недорозвинений тимус. Який вид гемопоезу буде порушений?

Моноцитопоез

Лімфопоез

Мегакаріоцитопоез

Гранулоцитопоез

Еритропоез

959 / 6307
На мікропрепараті серця розрізняються клітини прямокутної форми, розмірами від 50 до 120 мкм, з центрально розташованим ядром, розвиненими міофібрилами, зв’язані між собою вставними дисками. Яку функцію виконують ці клітини?

Скорочення серця

Ендокринна

Захисна

Регенераторна

Проведення імпульсів

960 / 6307
У збудливій клітині заблокували іонні канали. Це суттєво не змінило рівень потенціалу спокою, але клітина втратила здатність до генерації ПД. Які канали заблоковано?

Хлорні

Натрієві

Калієві

Натрієві та калієві

Кальцієві

961 / 6307
У людини крововилив у задню центральну звивину призвів до порушення чутливості з протилежного боку. Який вид чутливості порушений?

Нюхова та смакова

Шкірна та пропріоцептивна

Зорова

Слухова

Слухова та зорова

962 / 6307
У пацієнта розвинулися порушення рухової активності: тремор, атаксія та асинергія рухів, дизартрія. Яка структура найбільш вірогідно уражена?

Лімбічна система

Базальні ганглії

Мозочок

Стовбур мозку

Довгастий мозок

963 / 6307
Хворому на миготливу аритмію, в анамнезі якого бронхіальна астма, треба призначити протиаритмічний засіб. Який препарат з цієї групи ПРОТИПОКАЗАНИЙ хворому?

Верапаміл

Анаприлін

Ніфедипін

Новокаїнамід

Аймалін

964 / 6307
На електронній мікрофотографії представлена клітина нейрального походження, що знаходиться у складі епітелію слизової оболонки. Дистальна частина периферійного відростку клітини має булавоподібне потовщення, від якого відходять 10-12 війок. Що це за клітина?

Нюхова

Палочкова зорова клітина

Біполярний нейрон спинномозкового вузла

Колбочкова зорова клітина

Сенсорна клітина смакової цибулини

965 / 6307
Ізольована клітина серця людини автоматично генерує імпульси збудження з частотою 60 разів за хвилину. З якої структури серця отримано цю клітину?

Пучок Гіса

Синоатриальний вузол

Атріовентрикулярний вузол

Передсердя

Шлуночок

966 / 6307
У людини через 10 хвилин після початку інтенсивної фізичної роботи кількість еритроцитів у крові збільшилася з 4,01012/л до 4,51012/л. Що є основною причиною цього?

Активація еритропоезу

Пригнічення руйнування еритроцитів

Втрата води організмом

Вихід еритроцитів з депо

Збільшення хвилинного об’єму крові

967 / 6307
У людини в артеріальній крові напруга кисню збільшена до 104 мм рт.ст., а вуглекислого газу зменшена до 36 мм рт.ст. Що може бути причиною цього?

Довільна гіпервентиляція

Перебування у горах

Помірне фізичне навантаження

Затримка дихання

Інтенсивне фізичне навантаження

968 / 6307
У чоловіка 33 років діагностовано прободіння шлунка та запалення очеревини, що призвело до напруження м’язів передньої черевної стінки (“доскоподібний живіт”). Який рефлекс забезпечує цей симптом?

Кутанно-вісцеральний

Вісцеро-вісцеральний

Вісцеро-соматичний

Сомато-вісцеральний

Вісцеро-кутанний

969 / 6307
У хворого глибока рвана рана із нерівними краями, вкрита гноєм. У крайових відділах сочна грануляційна тканина, яка не здіймається над рівнем рани. Назвіть вид загоєння рани:

Первинним натягом

Вторинним натягом

Під струпом

Безпосереднє закриття дефекту епітеліальної тканини

970 / 6307
Під час гістологічного дослідження лімфовузла хворого 18 років з ділянки заднього трикутника шиї морфолог виявив скопичення клітин, серед яких поодинокі багатоядерні клітини Березовського-Штернберга, великі клітини Ходжкіна, малі клітини Ходжкіна та багато лімфоцитів, поодинокі плазматичні клітини, еозинофіли. Яке захворювання у хворого?

Лімфоцитарна лімфома

Лімфогранулематоз

Пухлина Беркіта

Хронічний лімфоїдний лейкоз

Нодулярна лімфома

971 / 6307
Дівчинка 6 років захворіла на дифтерію та померла від асфіксії на третю добу. На аутопсії слизова оболонка трахеї та бронхів стовщена, набрякла, тьмяна, вкрита сіруватими плівками, які легко відокремлюються. Про який вид запалення свідчать морфологічні зміни?

Дифтеритичне

Геморагічне

Серозне

Катаральне

Крупозне

972 / 6307
Хворому з травмою передпліччя під час репозиції кісток для міорелаксації введено дитилін. Повне відновлення тонусу та функції м’язів відбулося більш, ніж через годину. Чим можна пояснити значне подовження курареподібної дії препарату?

Генетичним дефіцитом бутирилхолінестерази

Утворенням активного метаболіту

Пригнічєнням мікросомного окиснення

Генетичним дефіцитом гідроксилази

Генетичним дєфіцитом моноамінооксидази

973 / 6307
У людини, що виконувала важку фізичну роботу в умовах підвищеної температури навколишнього середовища, змінилася кількість білків плазми крові. Що саме має місце у даному випадку?

Диспротеїнемія

Абсолютна гіпепротеїнемія

Парапротеїнемія

Абсолютна гіпопротеїнемія

Відносна гіперпротеїнемія

974 / 6307
Хворому, що страждає на стенокардію та приймає ізосорбіда мононітрат, було додатково призначено лікарський засіб з дезагрегантним ефектом. Визначте цей препарат:

Ніфедипін

Валідол

Анаприлін

Ацетилсаліцилова кислота

Нітрогліцерин

975 / 6307
Хворому в післяопераційному періоді для стимуляції перистальтики кишечника та тонусу сечового міхура було призначено препарат з групи антихолінестеразних засобів. Визначте його серед нижченаведених препаратів:

Дихлотіазид

Прозерин

Резерпін

Анаприлін

Маніт

976 / 6307
У хворого діагностовано ураження стулок правого передсердно-шлуночкового клапана. Внаслідок запального процесу якої анатомічної структури серця відбулися зміни у стулках?

Міокард

Фіброзний перикард

Епікард

Ендокард

Серозний перикард

977 / 6307
У хворого на обличчі вугрі. Під час мікроскопії зскрібків із уражених ділянок виявлені живі членистоногі розміром 0,2-0,5 мм., які мають витягнуту червоподібну форму, чотири пари коротких кінцівок, що розташовані у середній частині тіла. Який лабораторний діагноз?

Міаз

Короста

Фтиріоз

Педикульоз

Демодекоз

978 / 6307
У хворого, який тривалий час страждає на хронічний ентероколіт, після вживання молока виникли метеоризм, діарея, коліки. З нестачею якого ферменту в кишечнику це пов’язано?

Амілаза

Мальтаза

Сахараза

Глікогенситнетаза

Лактаза

979 / 6307
У новонародженної дитини виявлено гнійні виділення з кон’юктиви ока. Під час мікроскопії мазка з кон’юктиви знайдено велику кількість лейкоцитів та грамнегативні бобоподібні диплококи, що знаходяться всередині лейкоцитів. Який збудник є причиною цього захворювання?

Streptococcus pyogenes

Staphylococcus aureus

Neisseria catarrhalis

Neisseria gonorrhoeae

Staphylococcus epidermidis

980 / 6307
Жінці 30 років на 32 тижні вагітності лікар, в складі комплексної терапії виразкової хвороби шлунка, призначив антибіотик. Вкажіть, який препарат можна застосувати у даному випадку?

Тетрациклін

Гентаміцин

Левоміцетин

Азитроміцин

Бензилпеніцилін

981 / 6307
Під час ультразвукового обстеження вагітної жінки було діагностовано багатоводдя. З порушенням діяльності яких позазародкових органів можна пов’язати даний патологічний стан?

Алантоїс

Плацента

Жовтковий мішок

Хоріон

Амніотична оболонка

982 / 6307
У хворого з гострим циститом під час дослідження сечі виявили лейкоцити та багато грамнегативних паличок. При посіві виросли колонії слизового характеру, які утворювали зелений розчинний пігмент. Який мікроорганізм, найбільш вірогідно, є причиною захворювання?

Pseudomonas aeruginosa

Salmonella enteritidis

Escherihia coli

Klebsiella pneumoniae

Proteus mirabilis

983 / 6307
Чоловіка 49 років доставлено з місця автомобільної аварії до лікарні у непритомному стані. Шкірні покриви бліді, пульс частий та поверхневий. Переломів кісток та пошкодження головного мозку не виявлено. Під час пункції черевної порожнини отримано значну кількість крові. Що є первинною причиною важкого стану потерпілого?

Гіповолемія

Гіпопротеїнемія

Гіпонатріємія

Еритропенія

Гіпоінсулінемія

984 / 6307
У жінки 45 років народився хлопчик з розщепленням верхньої щелепи (“заяча губа” та “вовча паща”). Під час додаткового обстеження виявлені значні порушення з боку нервової, серцево-судинної систем та зору. При дослідженні каріотипу діагностована трісомія по 13 хромосомі. Який синдром має місце у хлопчика?

Клайнфельтера

Шерешевського-Тернера

Дауна

Едвардса

Патау

985 / 6307
Під час медико-генетичного консультування родини зі спадковою патологією виявлено, що аномалія проявляється через покоління у чоловіків. Який тип успадковування притаманний для цієї спадкової аномалії?

Х-зчеплене домінантне

Y-зчеплене

Аутосомне рецесивне

Аутосомне домінантне

Х-зчеплене рецесивне

986 / 6307
Людина 28 років споживає надмірну кількість вуглеводів (600 г на добу), що перевищує її енергетичні потреби. Який процес буде активуватися у даному випадку?

Ліполіз

Окислення жирних кислот

Глюконеогенез

Гліколіз

Ліпогенез

987 / 6307
У дитини 5 років діагностовано хворобу Брутона, яка проявляється у важкому перебігу бактеріальних інфекцій, відсутності В-лімфоцитів та плазматичних клітин. Які зміни вмісту імуноглобулінів будуть спостерігатися у сироватці крові цієї дитини?

Збільшення IgD,IgE

Зменшення IgD,IgE

Зменшення IgA, IgM

Збільшення IgA, IgM

Змін не буде

988 / 6307
У жінки 30 років виявлено недостатність зовнішньосекреторної функції підшлункової залози. Гідроліз яких поживних речовин буде порушений?

Жири, вуглеводи

Білки, жири, вуглеводи

Білки

Білки, жири

Білки, вуглеводи

989 / 6307
На практичному занятті з мікробіології студентам запропоновано пофарбувати готові зафіксовані мазки із мокротиння хворого на туберкульоз. Який метод фарбування треба використати у данному випадку?

Циля-Нільсена

Грама

Романовського-Гімза

Буррі

Гінса

990 / 6307
Під час обстеження на бактеріоносійство працівників дитячих закладів у виховательки виділена С.diphtheriae. Було проведено дослідження на токсигенність збудника, яке показало, що цей штам С.diphtheriae не продукує екзотоксин. Яку реакцію провели при дослідженні на токсигенність дифтерійних бактерій?

Реакція імунофлуоресценсії

Реакція аглютинації

Реакція кільцепреципітації

Реакція зв’язування комплементу

Реакція преципітації в агаровому гелі

991 / 6307
У хворої параліч мімічної мускулатури, розлад сприйняття смаку в передніх 2/3 язика, зменшення слиновиділення. Який з черепних нервів уражений?

N.hypoglossus

N.glossphapyngeus

N.trigeminus

N.facialis

N.vagus

992 / 6307
Лікар-дослідник у складі альпіністської експедиції піднявся у базовий табір, розташований на висоті 5000 м. На 3-й день перебування у нього з’явилися ознаки гірської хвороби: задишка, головний біль, втрата апетиту, загальна слабкість, ціаноз. Який тип гіпоксії має місце в цьому випадку?

Тканинна

Гіпоксична

Гемічна

Змішана

Циркуляторна

993 / 6307
У хворого на колагеноз після тривалого прийому преднізолону з’явився спастичний біль скелетних м’язів внаслідок розвитку гипокаліємії. Який препарат треба використати для корекції обміну калію?

Діазепам

Тирокальцитонін

Дитилін

Панангін

Но-шпа

994 / 6307
У робітника 37 років, який працював у кесоні, після підйому на поверхню раптово з’явилися ознаки гострого порушення мозкового кровообігу, непритомність. Через декілька днів він помер. На розтині в лівій півкулі головного мозку виявлено вогнище сірого кольору м’якої консистенції, неправильної форми, розмірами 5х6х3,5 см. Який процес мав місце у головному мозку?

Ішемічний інсульт

Геморагічний інсульт

Кіста

Абсцес

Пухлина

995 / 6307
Зі слизових оболонок та з харкотиння хворого, який тривалий час приймав імунодепресанти, були виділені грампозитивні великі овальні клітини з брунькуванням, що розташовані хаотично, та подовжені клітини у вигляді ланцюжків. Який збудник виділений?

Стрептококи

Кандиди

Стрептобактерії

Ієрсінії

Актиноміцети

996 / 6307
У здорової дорослої людини проводять зондування порожнин серця. Зонд знаходится у лівому шлуночку. Під час якої фази (періоду) серцевого циклу буде зареєстровано збільшення тиску від 8 до 70 мм рт.ст.?

Період вигнання

Фаза швидкого вигнання

Фаза ізометричного скорочення

Фаза повільного вигнання

Фаза асинхронного скорочення

997 / 6307
Людина за призначенням лікаря тривалий час приймала препарат з групи глюкокортикоїдних гормонів. Секреція якого (яких) з наведених гормонів буде пригнічена внаслідок цього?

Статеві

Соматотропний

Мінералокортикоїди

Кортикотропний

Тиротропний

998 / 6307
У студента через 2 години після іспиту в аналізі крові виявлено лейкоцитоз без істотних змін у лейкоцитарній формулі. Який найбільш вірогідний механізм розвитку лейкоцитозу?

Уповільнення міграції лейкоцитів у тканини

Уповільнення руйнування лейкоцитів

Посилення лейкопоезу та зменшення руйнування лейкоцитів

Посилення лейкопоезу

Перерозподіл лейкоцитів в організмі

999 / 6307
У молодої жінки видалили пухлину дистального кінця стегнової кістки, яка швидко зростала. Макроскопічно: пухлина строкатого вигляду - від біло-сірого до коричневочервоного кольору, пухкої консистенції. Мікроскопічно: основний тканинний компонент пухлини представлений кістковими та остеоїдними структурами, вистеленими атиповими остеобластами з патологічними мітозами; безліч тонкостінних судин. Який найбільш вірогідний діагноз?

Саркома Юїнга

Остеома

Ангіосаркома

Остеосаркома

Хондрома

1000 / 6307
У хворого підвищений вміст глюкози в крові; є цукор в сeчі. Під час пункційної біопсії нирки виявлено: розширення мезангія з осередковим накопиченням мембраноподібної речовини з перигломерулярним склерозом деяких клубочків, гіаліноз та плазматичне просякання артеріол, лімфогістіоцитарна інфільтрація строми з наявністю поліморфноядерних лейкоцитів; глікогенна інфільтрація нефроцитів вузького сегмента. Який найбільш вірогідний діагноз?

Хронічний гломерулонефрит

Підгострий гломерулонефрит

Гострий гломерулонефрит

Пієлонефрит

Діабетичний гломерулосклероз

1001 / 6307
До хірургічного відділення надійшов чоловік 35-ти років з гнійною раною шиї попереду трахеї (в ділянці передвісцерального простору). Куди може поширитися інфекція, якщо терміново не буде проведене оперативне втручання?

У заднє середостіння

У переднє середостіння

У ретровісцеральний простір

У середнє середостіння

У надгруднинний міжапоневротичний простір

1002 / 6307
Тривале перебування в умовах спеки викликало в людини спрагу. Сигналізація від яких рецепторів, перш за все, зумовила її розвиток?

Натрієві рецептори гіпоталамуса

Глюкорецептори гіпоталамуса

Осморецептори печінки

Осморецептори гіпоталамуса

Барорецептори дуги аорти

1003 / 6307
Людина, яка проживала в ендемічному вогнищі, перехворіла триденною малярією. Через півтора року після переїзду в іншу місцевість захворіла малярією знову. Яка найбільш вірогідна форма цього захворювання?

Вторинна інфекція

Суперінфекція

Реінфекція

Персистуюча інфекція

Рецидив

1004 / 6307
До приймального відділення в важкому стані надійшов чоловік 38-ми років, який отруївся сулемою. Який антидот треба негайно ввести хворому?

Ізонітрозин

Унітіол

Дипіроксим

Атропін

Налорфін

1005 / 6307
Людина хворіє на цукровий діабет, що супроводжується гіперглікемією натще понад 7,2 ммоль/л. Рівень якого білка плазми крові дозволяє ретроспективно (за попередні 4-8 тижні до обстеження) оцінити рівень глікемії?

С-реактивний білок

Глікозильований гемоглобін

Церулоплазмін

Фібриноген

Альбумін

1006 / 6307
У дітей часто можна спостерігати утруднене носове дихання, яке пов’язане з надмірним розвитком лімфоїдної тканини слизової оболонки глотки. Розростання яких мигдаликів може спричинити це явище?

Tonsilla lingualis

Tonsilla palatina

Tonsilla pharyngea

Усі названі мигдалики

Tonsilla tubaria

1007 / 6307
У чоловіка 50-ти років раптово виникли сильне серцебиття, біль у серці, різка слабкість, підвищення артеріального тиску; пульс аритмічний, з дефіцитом. На ЕКГ виявлено відсутність зубців Р та різна тривалість інтервалів R-R. Яке порушення серцевого ритму в хворого?

Синусова екстрасистолія

Пароксизмальна тахікардія

Миготлива аритмія

Дихальна аритмія

Поперечна блокада серця

1008 / 6307
У пацієнта 60-ти років виявлено погіршення сприйняття звуків високої частоти. Порушення стану яких структур слухового аналізатора зумовило ці зміни?

М’язи середнього вуха

Барабанна перетинка

Основна мембрана завитки біля овального віконця

Основна мембрана завитки біля гелікотреми

Євстахієва труба

1009 / 6307
У хворого спостерігається ішемія тканин нижче колінного суглоба, що супроводжується 'переміжною кульгавістю'. Про оклюзію якої артерії слід думати?

Проксимальна частина стегнової

Передня великогомілкова

Підколінна

Задня великогомілкова

Малогомілкова

1010 / 6307
В експерименті було показано, що опромінені ультрафіолетом клітини шкіри хворих на пігментну ксеродерму, через дефект ферменту репарації, повільніше відновлюють нативну структуру ДНК, ніж клітини здорових людей. За допомогою якого ферменту відбувається цей процес?

РнК-лігаза

Ендонуклеаза

ДНК-полімераза III

Праймаза

ДНК-гіраза

1011 / 6307
Ветеринарний фельдшер, що працював на тваринницькій ферму звернувся до лікаря зі скаргами на 6іль у суглобах, лихоманку, нездужання, пітливість по ночам. Хворіє близько місяця. Враховуючи скарги та професійний анамнез, лікар запідозрив у нього бруцельоз. Який матеріал, взятий у цього хворого, підлягає дослідженню в звичайній мікробіологічній лабораторії?

Блювотні маси

Випорожнення

Спинномозкова рідина

Сироватка крові

Сеча

1012 / 6307
У хворого виявлено зниження вмісту іонів магнію, які потрібні для прикріплення рибосом до гранулярної ендоплазматичної сітки. Відомо, що це призводить до порушення біосинтезу білка. Який саме етап біосинтезу білка буде порушено?

Трансляція

Транскрипція

Реплікація

Активація амінокислот

Термінація

1013 / 6307
У пацієнта після травми виникли паралічі, розлади больової чутливості праворуч; з лівого боку - паралічі відсутні, але порушена больова та температурна чутливість. Яка причина такого явища?

Пошкодження середнього мозку

Пошкодження рухової зони кори головного мозку

Пошкодження стовбура мозку

Однобічне пошкодження спинного мозку з правого боку

Пошкодження мозочка

1014 / 6307
Студент старанно конспектує лекцію. Якість конспектування значно погіршилася, коли сусіди стали розмовляти. Який вид гальмування в корі головного мозку є причиною цього?

Запізніле

Зовнішнє

Позамежове

Диференціювальне

Згасаюче

1015 / 6307
При рентгенологічному дослідженні кісток основи черепа виявлено збільшення порожнини турецького сідла, витончення передніх нахилених відростків, руйнування різних ділянок турецького сідла. Пухлина якої ендокринної залози може спричинити таке руйнування кісток?

Епіфіз

Вилочкова залоза

Наднирники

Гіпофіз

Щитоподібна залоза

1016 / 6307
До лікаря-невропатолога звернулася хвора 52-х років із скаргами на втрату чутливості шкіри правої половини обличчя в ділянці нижньої повіки, спинки носа та верхньої губи. Вкажіть, яка гілка якого нерва при цьому ушкоджена:

Нижньощелепна гілка трійчастого нерва

Барабанна струна лицевого нерва

Великий кам’янистий нерв лицевого нерва

Верхньощелепна гілка трійчастого нерва

Очна гілка трійчастого нерва

1017 / 6307
Хворий з нейродермітом протягом тривалого часу вживав преднізолон. При обстеженні в нього виявили підвищення рівня цукру в крові. Вплив препарату на яку ланку вуглеводного обміну призводить до виникнення цього ускладнення?

Активація розщеплення інсуліну

Пригнічення синтезу глікогену

Активація глюконеогенезу

Активація глікогеногенезу

Посилення всмоктування глюкози в кишечнику

1018 / 6307
Надмірне оволосіння вушних раковин (гіпертрихоз) визначається геном, локалізованим у Y-хромосомі. Цю ознаку має батько. Яка вірогідність народження хлопчика з такою аномалією?

25%

75%

100%

35%

0%

1019 / 6307
У дівчини виявлена диспропорція тіла, крилоподібні складки на шиї. При цитогенетичному дослідженні в ядрах лейкоцитів не виявлені 'барабанні палички', а в ядрах букального епітелію відсутні тільця Барра. Який найбільш вірогідний діагноз?

Синдром Едвардса

Синдром Клайнфельтера

Синдром Патау

Синдром Дауна

Синдром Шерешевського-Тернера

1020 / 6307
У хворих на колагеноз має місце процес деструкції сполучної тканини. Це підтверджується збільшенням у крові:

Вмісту креатину та креатиніну

Вмісту уратів

Активності трансаміназ

Вмісту оксипроліну та оксилізину

Активності ізоферментів ЛДГ

1021 / 6307
Хворому з частими нападами стенокардії був призначений сустак-форте по 1 табл. 2 рази на день. Спочатку відмічався позитивний ефект, однак, на другу добу, напади стенокардії відновилися. Чим можна пояснити неефективність призначеного препарату?

Сенсибілізація

Кумуляція

Ідіосинкразія

Тахіфілаксія

Залежність

1022 / 6307
На розтині тіла чоловіка 35-ти років у другому сегменті правої легені виявлено вогнище ущільнення діаметром 5 см, оточене тонкою капсулою. Вогнище представлене сухою крихкою тканиною з тьмяною поверхнею. Для якого захворювання характерні такі морфологічні зміни?

Туберкулома

Рак легені

Хондрома

Післязапальний пневмосклероз

Туморозна форма силікозу

1023 / 6307
У хворого лікар діагностував гостру гонорею. З анамнезу стало відомо, що раніше він переніс гонорею і вилікування було повним. До якої категорії інфекцій можна віднести це нове захворювання?

Рецидив

Вторинна інфекція

Суперінфекція

Автоінфекція

Реінфекція

1024 / 6307
Хворій проводиться операція на щитоподібній залозі. Гілки яких артерій має перев’язати хірург під час операції?

Верхня щитоподібна та щитошийний стовбур

Верхня та нижня щитоподібні

Верхня щитоподібна та висхідна артерія шиї

Висхідна та глибока артерії шиї

Верхня та нижня гортанні

1025 / 6307
Під час вивчення фаз мітотичного циклу корінця цибулі знайдено клітину, в якій хромосоми лежать в екваторіальній площині, утворюючи зірку. На якій стадії мітозу перебуває клітина?

Анафаза

Інтерфаза

Телофаза

Метафаза

Профаза

1026 / 6307
Запалення барабанної порожнини (гнійний середній отит) у хворого ускладнилося запаленням комірок соскоподібного відростка. Через яку стінку барабанної порожнини проникнув гній до комірок?

Передня

Верхня

Латеральна

Задня

Медіальна

1027 / 6307
На спеціальному живильному середовищі після посіву виділення гною з уретри, виросли ніжні голубуваті колонії. При мікроскопії препаратів з них виявлені грамнегативні бобовинні диплококи. Збудником якої хвороби вони є?

Сифіліс

Хламідіоз

Туляремія

Гонорея

Меліоїдоз

1028 / 6307
В бактеріологічній лабораторії досліджували в’ялену рибу домашнього виготовлення, яка стала причиною важкого харчового отруєння. При мікроскопії виділеної на середовищі Кіта-Тароцці культури виявлені мікроорганізми, схожі на тенісну ракетку. Який найбільш вірогідний діагноз?

Дизентерія

Черевний тиф

Ботулізм

Холера

Сальмонельоз

1029 / 6307
До лікаря звернулася хвора зі скаргами на розлади травлення, розлитий біль у животі. При обстеженні лікар виявив різко виражене зниження гемоглобіну в крові. З опитування виявилося, що, під час проживання на Далекому Сході, хвора часто вживала в їжу малосольну рибну ікру. Аналогічний стан відзначений у деяких родичів, що проживають з нею. Яке захворювання діагностував лікар у цієї хворої?

Дифілоботріоз

Трихінельоз

Аскаридоз

Теніоз

Ехінококоз

1030 / 6307
У клітині повністю заблокований синтез АТФ. Як зміниться величина мембранного потенціалу спокою цієї клітини?

Істотно збільшиться

Зникне

Спочатку збільшиться, потім зменшиться

Незначно збільшиться

Спочатку зменшиться, потім збільшиться

1031 / 6307
Хворій, що перенесла мастектомію в зв’язку з раком молочної залози, був призначений курс променевої терапії. Який з перерахованих вітамінних препаратів має виражену радіопротекторну дію, зумовлену антиоксидантною активністю?

Тіаміну хлорид

Ергокальциферол

Токоферолу ацетат

Рибофлавін

Фолієва кислота

1032 / 6307
У померлого від хронічної серцево-судинної недостатності на розтині виявлене 'тигрове серце'. З боку ендокарду помітна жовтувато-біла посмугованість; міокард тьмяний, глинисто-жовтий. Який процес зумовив дану патологію?

Гіаліново-краплинна дистрофія

Жирова судинно-стромальна дистрофія

Амілоїдоз

Жирова паренхіматозна дистрофія

Вуглеводна дистрофія

1033 / 6307
У хворої дитини періодично з’являються рідкі випорожнення, іноді біль у ділянці живота, нудота, блювання. Зі слів матері, одного разу в дитини з блювотними масами виділився гельмінт веретеноподібної форми, розміром 20 см. Причиною такого стану може бути:

Дракункульоз

Трихоцефальоз

Трихінельоз

Анкілостомоз

Аскаридоз

1034 / 6307
Дистрофічні зміни серцевого м’яза супроводжуються розширенням порожнин серця, зниженням сили серцевих скорочень, збільшенням об’єму крові, що залишається під час систоли в порожнині серця, переповненням вен. Для якого стану серця це характерно?

Кардіосклероз

Аварійна стадія гіперфункції та гіпертрофії

Тоногенна дилатація

Міогенна дилатація

Тампонада серця

1035 / 6307
У хворого 27-ми років діагностовано гнійне запалення жовчного міхура. До якого відділу очеревинної порожнини потрапить гній під час розриву жовчного міхура при його типовому положенні?

До верхнього дванадцятипалого закутка

До передшлункової сумки

До печінкової сумки

До чепцевої сумки

До лівого бічного каналу

1036 / 6307
У хворого, який тривалий час страждає на переміжну кульгавість, тканини пальців стопи сухі, чорного кольору, нагадують мумію. На невеликій відстані від почорнілої ділянки розташована двоколірна лінія (червона лінія прилягає до практично незмінених тканин, а біло-жовта - до змінених тканин). Який вид некрозу в даного хворого?

Секвестр

Інфаркт

Гангрена

Пролежень

Мацерація

1037 / 6307
Збудник гепатиту D (дельта-агент) є дефектним вірусом, який може реплікуватись лише в клітинах, що вже інфіковані одним з вірусів. Який саме вірус необхідний для розмноження дельта-вірусу?

Вірус гепатиту В

Вірус імунодефіциту людини

Вірус гепатиту Е

Вірус Епстайна-Барр

Вірус гепатиту А

1038 / 6307
Хворому призначено дигоксин. Через декілька днів виявлено ознаки передозування препарату, його вміст у крові значно перевищує верхню межу терапевтичної концентрації. Як називається такий варіант дії лікарських речовин?

Тахіфілаксія

Антагонізм

Потенціювання

Звикання

Кумуляція

1039 / 6307
Хворий чоловік госпіталізований на 5-й день хвороби з проявами жовтяниці, болем у м’язах, ознобом, носовими кровотечами. Під час проведення лабораторної діагностики бактеріолог виконав темнопольну мікроскопію краплини крові хворого. Назвіть збудника хвороби:

Calymmatobacterium granulomatis

Rickettsia mooseri

Borrelia dutlonii

Leptospira interrogans

Bartonella bacilloformis

1040 / 6307
При переведенні погляду на близько розташований об’єкт, заломна сила оптичних середовищ ока збільшилася на 10 діоптрій. Це є результатом зміни стану такої структури ока:

Волога передньої камери ока

Скловидне тіло

Кришталик

М’яз, що розширює зіницю

Рогівка

1041 / 6307
Хворому з метою відновлення дихання, при отруєнні чадним газом, було введено аналептичний засіб рефлекторного типу дії з групи Н-холіноміметиків. Який засіб було призначено хворому?

Лобеліну гідрохлорид

Пентамін

Атропіну сульфат

Адреналіну гідрохлорид

Мезатон

1042 / 6307
У хворого, прооперованого з приводу ускладненого апендициту, в крові відзначаються наступні зміни: ер.-4,0•1012/л, Нb-120 г/л, КП- 0,9, лейк.-18•109 /л, б - 0, е - 0, мієлоц.- 0, ю - 0, п - 20, с - 53, л - 21, м - 5. Як називається такий ядерний зсув лейкоцитарної формули?

Регенеративний зсув вліво

Дегенеративний зсув вліво

Зсув вправо

Регенеративно-дегенеративний

Гіперрегенеративний

1043 / 6307
Забір крові для загального аналізу рекомендують проводити натщесерце та зранку. Які зміни складу крові можливі, якщо провести забір крові після приймання їжі?

Зниження кількості еритроцитів

Зниження кількості тромбоцитів

Збільшення кількості лейкоцитів

Збільшення кількості еритроцитів

Збільшення білків плазми

1044 / 6307
Для серологічної діагностики сифілісу в реакції Вассермана лікар-лаборант підготував такі реактиви: кардіоліпіновий антиген, спиртовий екстракт ліпоїдів із серцевого м’яза бика з холестерином, антиген з трепонем, зруйнованих ультразвуком, гемолітична система, фізіологічний розчин, досліджувані сироватки. Який ще компонент необхідний для постановки реакції?

Діагностична преципітуюча сироватка

Комплемент

Антиглобулінова сироватка

Живі трепонеми

Еритроцити барана

1045 / 6307
До відділення реанімації надійшов чоловік 47-ми років з діагнозом інфаркт міокарду. Яка з фракцій лактатдегідрогенази (ЛДГ) буде переважати в сироватці крові протягом перших двох діб?

ЛДГ1

ЛДГ3

ЛДГ4

ЛДГ5

ЛДГ2

1046 / 6307
Під час операції на головному мозку відмічено, що подразнення певних зон кори великих півкуль викликало в хворого i тактильні і температурні відчуття. На яку зону кори діяли подразники?

Прецентральна звивина

Верхня латеральна звивина

Поясна звивина

Постцентральна звивина

Парагіпокампова звивина

1047 / 6307
До відділення травматології надійшов хворий із значним пошкодженням м’язової тканини. Який біохімічний показник сечі буде збільшений при цьому?

Креатинін

Мінеральні солі

Сечова кислота

Глюкоза

Загальні ліпіди

1048 / 6307
У хворої 43-х років на фоні септичного шоку відзначається тромбоцитопенія, зниження фібриногену, поява в крові продуктів дегенерації фібрину, поява петехіальних крововиливів. Вкажіть причину виникнення даних змін:

Автоімунна тромбоцитопенія

Геморагічний діатез

Екзогенна інтоксикація

ДВЗ-синдром

Порушення утворення тромбоцитів

1049 / 6307
Хворому з великими опіками зробили пересадку донорської шкіри. На 8-му добу трансплантат набряк, змінився його колір; на 11 добу почав відторгатися. Які клітини беруть у цьому участь?

Т-лімфоцити

Базофіли

Еозинофіли

В-лімфоцити

Еритроцити

1050 / 6307
Хворий з інфекційним мононуклеозом протягом двох тижнів приймав глюкокортикостероїдні препарати. Наступила ремісія, проте в нього виникло загострення хронічного тонзиліту. Результатом якої дії глюкокортикостероїдів є дане ускладнення?

Протизапальна

Імунодепресивна

Протишокова

Антитоксична

Антиалергічна

1051 / 6307
Під час розтину трупа дівчинки 12-ти років виявлено: множинні крововиливи у шкірі (переважно сідниць, нижніх кінцівок), серозних та слизових оболонок, у головному мозку. У наднирниках вогнищевий некроз та масивні крововиливи, у нирках - некротичний нефроз, гнійний артрит, іридоцикліт, васкуліт. Який найбільш вірогідний діагноз?

Висипний тиф

Менінгококцемія

Системний червоний вівчак

Вузликовий періартеріїт

Променева хвороба

1052 / 6307
Більша частина учасників експедиції Магелана до Америки загинула від авітамінозу. Це захворювання проявлялося загальною слабкістю, підшкірними крововиливами, випадінням зубів, кровотечею з ясен. Як називається цей авітаміноз?

Пелагра

Анемія Бірмера

Скорбут (цинга)

Рахіт

Поліневрит (бері-бері)

1053 / 6307
При розтині тіла жінки 40-а років, яка страждала на ревматоїдний артрит, знайдена збільшена щільна селезінка. На розрізі її тканина коричнево-червоного кольору зі збільшеними фолікулами, які мають вигляд напівпрозорих сірувато-білуватих зерен. Вкажіть, який з перелічених патологічних процесів найбільш вірогідний?

Сальна селезінка

Порфірна селезінка

Гіаліноз селезінки

Сагова селезінка

Глазурна селезінка

1054 / 6307
Чоловік 28-ми років надійшов зі скаргами на нудоту, блювання, біль у правому підребер’ї. Об’єктивно: жовтяничність шкіри, склер; температура тіла підвищена, печінка збільшена, сеча темна, кал гіпохолічний. Гіпербілірубінемія (білірубін прямий та непрямий), білірубінурія, уробілінурія, гіпопротеїнемія, зниження зсідання крові. Для якого з перелічених нижче станів найбільш характерні ці зміни?

Підпечінкова жовтяниця

Гострий панкреатит

Клітинно-паренхіматозна жовтяниця

Гострий холецистит

Надпечінкова гемолітична жовтяниця

1055 / 6307
Хворому на сечокам’яну хворобу після обстеження призначили алопуринол - конкурентний інгібітор ксантиноксидази. Підставою для цього був хімічний аналіз ниркових каменів, які складалися переважно з:

Сульфату кальцію

Моногідрата оксалату кальцію

Урату натрію

Дигідрата оксалату кальцію

Фосфату кальцію

1056 / 6307
Під час обстеження підлітка, що страждає на ксантоматоз, виявлена сімейна гіперхолестеринемія. Концентрація яких ліпопротеїнів значно підвищена в крові при цій патології?

Хіломікрони

ЛПВГ

ЛПДНГ

НЕЖК

ЛПНГ

1057 / 6307
У жінки, що хворіє на міастенію, виникли розлади дихання, що вимагало застосування штучної вентиляції легень. Який вид дихальної недостатності розвинувся в даної хворої?

Центрогенний

Торакодіафрагмальний

Нервово-м’язовий

Рестриктивний

Обструктивний

1058 / 6307
У вагітної жінки розвинувся токсикоз з важкими повторними блюваннями протягом доби. До кінця доби почали з’являтися тетанічні судоми та зневоднення організму. Який зсув кислотно-лужного стану викликав вказані зміни?

Видільний ацидоз

Видільний алкалоз

Метаболічний ацидоз

Газовий алкалоз

Газовий ацидоз

1059 / 6307
У хворої 18-ти років пахові лімфатичні вузли збільшені в розмірах, не болючі, ущільнені при пальпації. У ділянці слизової оболонки геніталій невеликих розмірів виразка з ущільненими краями та 'лакованим'дном сіруватого кольору. Який найбільш вірогідний діагноз?

Гонорея

Лепра

Трофічна виразка

Туберкульоз

Сифіліс

1060 / 6307
У хворого з підозрою на дифтерію під час бактеріоскопічного дослідження мазка з зіву виявлені паличкоподібні бактерії з зернами волютину. Який етіотропний препарат є препаратом вибору в даному випадку?

Еубіотик

Дифтерійний анатоксин

Протидифтерійна антитоксична сироватка

Інтерферон

Бактеріофаг

1061 / 6307
Чоловік 26-ти років скаржиться на безплідність. Об’єктивно: зріст 186 см, довгі кінцівки, гінекомастія, гіпоплазія яєчок, у зіскрібку слизової оболонки щоки знайдені тільця Барра. Діагностований синдром Клайнфельтера. Який механізм хромосомної аномалії має місце при даному захворюванні?

Делеція хромосоми

Інверсія хромосоми

Нерозходження гетосом у мейозі

Нерозходження хроматид у мітозі

Транслокація

1062 / 6307
Чоловік 32-х років чотири роки страждає на хронічний гломерулонефрит з нефротичним синдромом. Відзначаються явні набряки на обличчі, в останній час з’явилися набряки на ногах та тулубі. Який із механізмів розвитку набряків найбільш вірогідний у цього хворого?

Утруднення лімфовідтоку

Підвищення проникливості капілярів

Зниження онкотичного тиску крові

Підвищення гідростатичного тиску крові в капілярах

Підвищення онкотичного тиску тканинної рідини

1063 / 6307
У жінки, яка тривалий час знаходилася на дієті з використанням очищеного рису, виявлено поліневрит (хвороба Бері-Бері). Відсутність якого вітаміну в раціоні призводить до розвитку цього захворювання?

Піридоксаль

Фолiєва кислота

Тіамін

Аскорбінова кислота

Рибофлавін

1064 / 6307
У хворого після видалення жовчного мiхура утруднені процеси всмоктування Ca через стінку кишечнику. Призначення якого вітаміну буде стимулювати цей процес?

РР

С

К

В12

D3

1065 / 6307
У хворого з гіпертонічною хворобою з’явилися головний біль, шум у вухах, блювання. Артеріальний тиск підвищився до 220/160 мм рт.ст. Під час обстеження виявлена асиметрія обличчя з правого боку, відсутність довільних рухів, підвищення сухожилкових рефлексів та тонусу м’язів правих руки і ноги. Яка форма розладів рухової функції має місце в цьому випадку?

Гіперкінез

Тетраплегія

Моноплегія

Параплегія

Геміплегія

1066 / 6307
У жінки, яка хворіє на гіпертонічну хворобу, розвинувся напад бронхіальної астми. Який засіб слід призначити для зняття нападу?

Еуфілін

Ізадрин

Ефедрин

Сальбутамол

Адреналін

1067 / 6307
Фенілкетонурія успадковується як аутосомно-рецесивна ознака. У здорових батьків народилася дитина, хвора на фенілкетонурію. Які генотипи батьків?

Аа х Аа

аа х аа

АА х АА

Аа х аа

АА х Аа

1068 / 6307
Хворий відзначає часті проноси, особливо після вживання жирної їжі, схуднення. Лабораторні дослідження показали наявність стеатореї; кал гіпохолічний. Що може бути причиною такого стану?

Недостатність панкреатичної ліпази

Незбалансована дієта

Обтурація жовчних шляхів

Недостатність панкреатичної фосфоліпази

Запалення слизової оболонки тонкої кишки

1069 / 6307
У хворої жінки після парентерального введення гормону відбулося підвищення артеріального тиску і також підвищилися рівні глюкози та ліпідів у крові. Який гормон було введено?

Інсулін

Фолікулін

Адреналін

Прогестерон

Глюкагон

1070 / 6307
У дівчинки 7-ми років ознаки анемії. Лабораторно встановлений дефіцит піруваткінази в еритроцитах. Порушення якого процесу грає головну роль в розвитку анемії у дівчинки?

Дезамінування амінокислот

Тканинне дихання

Окислювальне фосфорилювання

Анаеробний гліколіз

Розклад пероксидів

1071 / 6307
У дитини 5-ти років, що часто хворіє на респіраторні захворювання, відмічаються екзематозні явища після вживання деяких харчових продуктів, схильність до затяжного перебігу запальних процесів. Який вид діатезу можна припустити в даному випадку?

Ексудативно-катаральний

Лімфатико-гіпопластичний

Нервово-артритичний

Геморагічний

Астенічний

1072 / 6307
Хворий сонливий, свідомість потьмарена, реакції на сильні подразники загальмовані. Шкіра бліда, суха, виражені набряки. М’язові фібрилярні посмикування. Мідріаз. Дихання типу Чейна-Стокса з аміачним запахом. При аускультації серця - шум тертя перикарда. Який вид коми розвинувся в хворого?

Гіперосмолярна

Ниркова

Кетоацидотична

Апоплексична

Печінкова.

1073 / 6307
Дитина 10-ти місячного віку, батьки якої брюнети, має світлє волосся, дуже світлу шкіру та блакитні очі. Зовнішньо при народженні виглядала нормально, але протягом останніх 3 місяців спостерігалися порушення мозкового кровообігу, відставання в розумовому розвитку. Причиною такого стану може бути:

Глікогеноз

Гостра порфірія

Гістидинемія

Галактоземія

Фенілкетонурія

1074 / 6307
У грудної дитини спостерігається забарвлення склер, слизових оболонок. Виділяється сеча, яка темніє на повітрі. В крові та сечі виявлено гомогентизинову кислоту. Що може бути причиною даного стану?

Галактоземія

Альбінізм

Алкаптонурія

Цистинурія

Гістидинемія

1075 / 6307
У мазку дуоденального вмісту хворого з розладом травлення виявлено найпростіших розміром 10-18 мкм. Тіло грушоподібної форми, 4 пари джгутиків, у розширеній передній частині тіла два ядра, які розміщені симетрично. Який вид найпростіших найбільш вірогідний?

Дизентерійна амеба

Лямблія

Трихомонада

Балантидій

Кишкова амеба

1076 / 6307
У потерпілого травма м’яких тканин голови та тім’яних кісток в ділянці їх з’єднання між собою, яка супроводжується сильною кровотечею. Яке судинне утворення пошкоджене?

Sinus rectus

Sinus sagittalis inferior

Sinus sagittalis superior

Sinus petrosus superior

Sinus transversus

1077 / 6307
Жінка 44-х років скаржиться на загальну слабкість, біль у ділянці серця, значне збільшення маси тіла. Об’єктивно: обличчя місяцеподібне, гірсутизм, АТ- 165/100 мм рт.ст., зріст -164 см, вага - 103 кг; переважно накопичення жиру на шиї, верхньому плечовому поясі, животі. Що є основним патогенетичним механізмом ожиріння у жінки?

Зниження продукції тиреоїдних гормонів

Зниження продукції глюкагону

Підвищення продукції мінералокортикоїдів

Підвищення продукції глюкокортикоїдів

Підвищення продукції інсуліну

1078 / 6307
У дитини 3-х років після перенесеної важкої вірусної інфекції відзначаються повторне блювання, непритомність, судоми. При дослідженні виявлена гіперамоніємія. З чим може бути пов’язана зміна біохімічних показників крові у цієї дитини?

Порушення знешкодження біогенних амінів

Посилення гниття білків у кишечникУ

Активація процесів декарбоксилювання амінокислот

Порушення знешкодження аміаку в орнітиновому циклі

Пригнічення активності ферментів трансамінування

1079 / 6307
На гістологічному препараті в сполучній тканині знайдено великі клітини, заповнені базофільною метахроматичною зернистістю; гістохімічно встановлено, що гранули містять гепарин та гістамін. Які клітини найбільш вірогідно знайдено в препараті?

Фібробласти

Плазмоцити

Тучні клітини

Адипоцити

Макрофаги

1080 / 6307
Альбіноси погано переносять вплив сонця - засмага не розвивається, а з’являються опіки. Порушення метаболізму якої амінокислоти лежить в основі цього явища?

Триптофан

Метіонін

Глутамінова

Гістидин

Фенілаланін

1081 / 6307
У хворого 25-ти років з численних шкірних пустул висівається золотистий стафілокок в асоцiацiї з епідермальним стафілококом. В аналiзi харкотиння виявлена пневмоциста карінії, у випорожненнях - криптоспоридії, вульгарний протей та гриби роду кандіда. При якому захворюванні зустрічається таке множинне інфікування умовно-патогенними мікроорганізмами?

Дисбактерiоз

СНІД

Медикаментозний агранулоцитоз

Цукровий діабет

Сепсис

1082 / 6307
До терапевтичного відділєння надійшов хворий з тривалою бронхопневмонією. Антибіотикотерапія не дала належного ефекту. Який препарат для підвищення імунного статусу слід додати до комплексної терапії цього хворого?

Тималін

Сульфокамфокаїн

Анальгін

Димедрол

Парацетамол

1083 / 6307
У пацієнта цироз печінки. Дослідження якої з перелічених речовин, що екскретуються з сечею, може характеризувати стан антитоксичної функції печінки?

Амінокислоти

Амонійні солі

Креатинін

Сечова кислота

Гіпурова кислота

1084 / 6307
Під час розтину трупа чоловіка 60-ти років у міокарді передньої стінки лівого шлуночка виявлене сіре неправильної форми щільне вогнище 5х4 см з чіткими межами, волокнистої структури. Який діагноз найбільш вірогідний?

Дифузний дрібновогнищевий міокардіосклероз

Післяінфарктний міокардіосклероз

Міокардит

Кардіоміопатія

Інфаркт

1085 / 6307
При аналізі спірограми в обстежуваного встановлено зменшення частоти та глибини дихання. До зменшення якого показника це призведе?

Хвилинний об’єм дихання

Залишковий об’єм

Резервний об’єм вдиху

Життєва ємність легень

Резервний об’єм видиху

1086 / 6307
У внутрішньоутробному періоді розвитку в судинній системі плода функціонує крупна артеріальна (боталова) протока, яка після народження перетворюється в lig. arteriosum. Що з’єднує ця протока?

Праве та ліве передсердя

Легеневий стовбур та верхню порожнисту вену

Аорту та верхню порожнисту вену

Аорту та нижню порожнисту вену

Легеневий стовбур та аорту

1087 / 6307
Хворий під час курсу лікування метронідазолом вжив невелику кількість алкоголю, внаслідок чого розвинулось тяжке отруєння. Поясніть причину цього:

Накопичення ацетальдегіду

Серцево-судинна недостатність

Невралгічні розлади

Порушення функції нирок

Алергічна реакція

1088 / 6307
До пологового відділення надійшла вагітна жінка зі слабкою пологовою діяльністю. Призначте засіб гормональної природи для посилення пологової діяльності:

Метандростенолон

Гідрокортизон

Прогестерон

АКТГ

Окситоцин

1089 / 6307
Оперуючи молочну залозу, хірурги надають перевагу радіальним розрізам. З особливостями будови яких анатомічних структур пов’язана така техніка операції?

Основи часточок звернені до соска

Часточки залози розташовані поперечно

Часточки залози розташовані вертикально

Верхівки часточок сходяться до соска

1090 / 6307
У пацієнта, який суворо виконував рекомендації щодо дотримання певної дієти протягом 10 днів, було проведено дослідження величини дихального коефіцієнта. Результат: ДК=1,0. Якої дієти дотримувався пацієнт?

З переважним вмістом білків та вуглеводів

Змішана

З переважним вмістом вуглєводів

З переважним вмістом жирів та вуглеводів

З переважним вмістом білків та жирів

1091 / 6307
У мікропрепараті спинного мозку необхідно проаналізувати стан ядра, нейрони якого утворюють моторні закінчення в скелетній мускулатурі. Про яке ядро спинного мозку йде мова?

Проміжне латеральне ядро

Власне ядро заднього рога

Грудне ядро

Власне ядро сірої речовини

Власне ядро переднього рога

1092 / 6307
У процесі розвитку дитини хребет поступово набув два лордози та два кіфози. Це пояснюється розвитком здатності до:

Повзання

Лежання

Сидіння

Прямоходіння

Плавання

1093 / 6307
При лабораторному дослідженні сечі в нормі в ній не виявляються формені елементи крові. Яка структура нефрону найбільше перешкоджає їх надходженню до первинної сечі?

Епітелій зовнішнього листка капсули клубочка

Мезангіальні клітини

Базальна мембрана капілярів клубочка

Юкставаскулярні клітини

Епітелій петлі Іенле

1094 / 6307
На розтині тіла жінки 27-ми років, що померла від хронічної ниркової недостатності, виявлені множинні рубці та інфаркти в селезінці, нирках. Під час мікроскопічного дослідження виявлені зміни в артеріях середнього та дрібного калібру, які характеризуються склерозом стінок, помірною проліферацією ендотелію і вираженою лімфогістіоцитарною інфільтрацією навколо. На яке захворювання страждала померла?

Атеросклероз

Хвороба Марфана

Вісцеральний сифіліс

Гіпертонічна хвороба

Вузликовий періартеріїт

1095 / 6307
Хворому з облітеруючим ендартеріїтом проведена гангліонарна симпатектомія. Позитивний лікувальний ефект цієї операції пов’язаний з розвитком артеріальної гіперемії нижніх кінцівок. Як визначається ця гіперемія за механізмом розвитку?

Нейропаралітична

Метаболічна

Реактивна

Нейротонічна

Робоча

1096 / 6307
Під час аналізу електрокардіограми встановлено, що тривалість серцевого циклу в людини дорівнює 1 секунді. Яка частота серцевих скорочень за хвилину?

80

100

60

50

70

1097 / 6307
Нервово-м’язовий препарат жаби обробили отрутою. Після цього зберігається здатність м’яза до скорочення у відповідь на пряму стимуляцію, але втрачається у відповідь на стимуляцію нерва. Що блокує ця отрута?

Процеси енергоутворення

Нервово-м’язовий синапс

Калієві канали

Натрієві канали

Спряження збудження та скорочення в м’язі

1098 / 6307
У місті епідемія грипу. Який препарат доцільно використати для неспецифічної профілактики захворювання?

Протигрипозна вакцина

Лейкоцитарний інтерферон

Протигрипозний імуноглобулін

Протигрипозна сироватка

Пеніцилін

1099 / 6307
Швидкою допомогою до приймального відділення доставлений хворий з кривавим блюванням. В анамнезі - цироз печінки. Пошкодження яких вен найбільш вірогідне у даному випадку?

Печінкові

Нижня брижова

Верхня брижова

Стравохідні

Селезінкова

1100 / 6307
Хворий скаржиться на біль при рухах у лівій половині шиї. Найкраще положення, при якому хворий не відчуває болю - нахил шиї вліво з одночасним підняттям голови і поверненням обличчя в протилежний бік. Функція якого м’яза порушена?

M.sternohyoideus

M.sternocleidomastoideus зліва

M.trapezius зліва

M.sternocleidomastoideus з правого боку

M.trapezius з правого боку

1101 / 6307
У людини вимірюють внутрішньо-плевральний тиск. У якій фазі людина затримала дихання, якщо величина тиску дорівнює - 7,5 см вод.ст.?

Спокійний видих

Спокійний вдих

Форсований видих

Форсований вдих

1102 / 6307
Жінка 49 років звернулася до лікаря зі скаргами на підвищену втомлюваність та появу задишки під час фізичного навантаження. На ЕКГ: ЧСС-50/хв.; PQ- подовжений, QRS- не змінений, кількість зубців Р перевищує кількість комплексів QRS. Який вид аритмії у пацієнтки?

Синусова брадикардія

Миготлива аритмія

Синоатріальна блокада

Атріовентрикулярна блокада

Екстрасистолія

1103 / 6307
На препараті одного з відділів дихальної системи виявлений трубчастий орган, в якому визначається невисокий епітелій, добре розвинена м’язова оболонка, відсутні залози та хрящ. Назвіть цей орган:

Гортань

Великі бронхи

Середні бронхи

Трахея

Малі бронхи

1104 / 6307
На препараті яєчника, забарвленому гематоксиліном-еозіном, визначається фолікул, в якому клітини фолікулярного епітелію розміщені в 1-2 шари та мають кубічну форму, навколо овоциту видно оболонку яскраво-червоного кольору. Назвіть цей фолікул:

Атретичний

Зрілий

Первинний

Примордіальний

Вторинний

1105 / 6307
Для розвитку гарячкових станів характерним є зростання рівня білків 'гострої фази' церулоплазміну, фібриногену, С-реактивного протеїну. Вкажіть можливий механізм цього явища:

Стимулюючий вплив ІЛ-1 на гепатоцити

Дегрануляція тканинних базофілів

Проліферативна дія ІЛ-2 на Т-лімфоцити

Руйнівна дія підвищеної температури на клітини організму

1106 / 6307
Хворий протягом двох тижнів отримував медикаментозну терапію з приводу психозу. Стан хворого поліпшився, однак невдовзі з’явилися ригідність, тремор, гіпокінезія. Який з перелічених препаратів викликає вказані ускладнення?

Хлордіазепоксид

Імізін

Дифенін

Сиднокарб

Аміназин

1107 / 6307
При огляді лікарем у хворого виявлена стегнова кила. Через яке анатомічне утворення вона виходить під шкіру?

Anulus femoralis

Hiatus saphenus

Lacuna musculorum

Anulus inguinalis superficialis

Canalis adductorius

1108 / 6307
Внаслідок впливу у-випромінювання ділянка ланцюга ДНК повернулася на 180 градусів. Яка з перелічених видів мутацій відбулася в ланцюзі ДНК?

Делеція

Транслокація

Дуплікація

Реплікація

Інверсія

1109 / 6307
В експерименті збільшили проникність мембрани збудливої клітини для іонів калію. До яких змін мембранного потенціалу це призведе?

Потенціал дії

Локальна відповідь

Гіперполяризація

Змін не буде

Деполяризація

1110 / 6307
При розтині тіла чоловіка, який тривалий час зловживав алкоголем, печінка малих розмірів, щільна, дрібно-бугриста. Мікроскопічно: псевдочасточки дрібні, розподілені вузькими прошарками сполучної тканини з лімфомакрофагальними інфільтратами; гепатоцити в стані великокрапельної жирової дистрофії. Який з перелічених діагнозів найбільш вірогідний?

Хронічний персистуючий алкогольний гепатит

Токсична дистрофія печінки

Алкогольний цироз

Хронічний активний алкогольний гепатит

Жировий гепатоз

1111 / 6307
У чоловіка виявлено гонорею за даними бактеріоскопії мазка з уретри. Враховуючи, що препаратами вибору для лікування гонореї є фторхінолони, хворому необхідно призначити:

Фуразолідон

Фторурацил

Уросульфан

Цефазолін

Ципрофлоксацин

1112 / 6307
Хворий з ревматоїдним артритом приймає диклофенак-натрій. Після чергового аналізу крові лікар відмінив цей препарат. Яке ускладнення з боку крові викликав диклофенакнатрій?

Еозинофілія

Посилення гемокоагуляції

Лейкопенія

Лейкоцитоз

Зниження гемокоагуляції

1113 / 6307
У хворого, який тривалий час приймав тетрациклін, виник кандидоз слизових оболонок. Який лікарський препарат слід призначити для його лікування?

Фурадонін

Гризеофульвін

Амфотеріцин

Ітраконазол

Нітрофунгін

1114 / 6307
Хворий 40-а років висуває скарги на сильне серцебиття, пітливість, нудоту, порушення зору, тремор рук, підвищення артеріального тиску. З анамнезу: 2 роки тому було встановлено діагноз феохромоцитома. Гіперпродукція яких гормонів зумовлює цю патологію?

Тиреоїдні гормони

Глюкокортикоїди

АКТГ

Альдостерон

Катехоламіни

1115 / 6307
Хвора 56-ти років тривалий час хворіє на тиреотоксикоз. Який тип гіпоксії може розвинутися у цієї хворої?

Змішана

Дихальна

Тканинна

Циркуляторна

Гемічна

1116 / 6307
У хворої симптоми запального процесу сечостатевих шляхів. У мазку із слизової оболонки піхви виявлено великі одноклітинні організми грушоподібної форми з загостреним шипом на задньому кінці тіла, великим ядром та ундулюючою мембраною. Які найпростіші знайдені в мазку?

Trichomonas hominis

Lamblia intestinalis

Trichomonas buccalis

Trypanosoma gambiense

Trichomonas vaginalis

1117 / 6307
Ціаніди є надзвичайно потужними клітинними отрутами, які при надходженні до організму людини можуть спричинити смерть. Блокування якого ферменту тканинного дихання лежить в основі такої їх дії?

Гемоглобінредуктаза

Цитохромоксидаза

Ферохелатаза

Глюкозо-6-фосфатдегідрогеназа

Каталаза

1118 / 6307
Під час гістологічного дослідження тимуса чоловіка 40-а років, визначено зменшення частки паренхіматозних елементів залози, збільшення частки жирової та пухкої сполучної тканини, збагачення її тимусними тільцями при незмінній загальній масі органа. Як зветься таке явище?

Акцідентальна інволюція

Атрофія

Вікова інволюція

Дистрофія

Гіпотрофія

1119 / 6307
У хворого спостерігається збільшений тонус артеріол за нормальних показників насосної функції серця. Як це вплине на величину артеріального тиску?

Зросте переважно діастолічний

Зменшиться переважно діастолічний

Тиск не зміниться

Зменшиться переважно систолічний

Зросте переважно систолічний

1120 / 6307
Хворий 50-ти років страждає на гіпертонічну хворобу. Під час фізичного навантаження у нього з’явилися відчуття м’язової слабкості, нестачі повітря, синюшність губ, шкіри, обличчя; дихання супроводжувалося відчутними на відстані вологими хрипами. Який механізм лежить в основі виникнення такого синдрому?

Гостра лівошлуночкова недостатність

Колапс

Тампонада серця

Хронічна лівошлуночкова недостатність

Хронічна правошлуночкова недостатність

1121 / 6307
У хворого 58-ми років з гострою серцевою недостатністю спостерігається зменшення добової кількості сечі - олігурія. Який механізм цього явища?

Підвищення гідростатичного тиску крові в капілярах

Зниження клубочкової фільтрації

Зниження проникності ниркового фільтру

Зниження онкотичного тиску крові

Зниження кількості функціонуючих клубочків

1122 / 6307
Хворий 32-х років надійшов до стаціонару зі скаргами на загальне нездужання, блювання, біль праворуч внизу живота. Після огляду лікарем був поставлений діагноз - гострий апендицит. Який вид болю в хворого?

Соматичний глибокий

Соматичний поверхневий ранній

Соматичний поверхневий пізній

Фантомний

Вісцеральний

1123 / 6307
У новонародженого фізіологічна жовтяниця. Рівень вільного білірубіну в крові значно перевищує норму. Нестачею якого ферменту це обумовлено?

Ксантиноксидаза

Аденозиндезаміназа

Трансаміназа

УДФ-глюкуронілтрансфераза

Гем-оксигеназа

1124 / 6307
У бактеріологічній лабораторії проводиться дослідження м’ясних консервів на вміст ботулінічного токсину. Для цього дослідній групі мишей ввели екстракт із досліджуваного матеріалу та антитоксичну протиботулінічну сироватку типів А, В, Е; контрольній групі мишей ввели екстракт без протиботулінічної сироватки. Яку серологічну реакцію було використано?

Подвійної імунної дифузії

Опсонофагоцитарна

Преципітації

Нейтралізації

Зв’язування комплементу

1125 / 6307
У хворого на цукровий діабет після введення інсуліну настала непритомність, спостерігаються судоми. Який результат біохімічного аналізу крові на вміст цукру?

8 ммоль/л

10 ммоль/л

1,5 ммоль/л

5,5 ммоль/л

3,3 ммоль/л

1126 / 6307
При зовнішньому огляді новонародженого виявлені виражені зміни шкірних покривів усього тіла. Шкіра суха, тьмяна, з нерівною поверхнею та з наявністю сірих пластин, які відшаровуються. З яким видом дистрофії пов’язана ця патологія?

Гідропічна

Мукоїдне набухання

Рогова

Фібриноїдне набухання

Гіаліново-крапельна

1127 / 6307
Лікування туберкульозу здійснюється за допомогою комбінованої хіміотерапії, що включає речовини різного механізму дії. Яке з протитуберкульозних засобів пригнічує в мiкобактерiях транскрипцію РНК на ДНК?

ПАСК

Стрептоміцин

Етіонамід

Ізоніазид

Рiфампiцин

1128 / 6307
Чоловіка 45-ти років протягом останніх 3-х років непокоїв сухий кашель, наростала задишка, легенева недостатність, швидко втрачалася вага. На розтині: легеневе серце; у легенях різко виражений фіброз з наявністю порожнин, що створюють картину 'медових сот'. Гістологічно: інтерстиційний фіброз з вираженою інфільтрацією строми лімфогістіоцитами з домішками нейтрофілів. Який найбільш вірогідний діагноз?

Хронічна бульозна емфізема

Післязапальний пневмосклероз

Пиловий пневмосклероз

Фіброзуючий альвеоліт

Бронхоектатична хвороба

1129 / 6307
У хворого вдень піднялася температура тіла до 39oС і через 6 годин повернулася до норми. На другу добу напад повторився: в період пароксизму температура досягла 41oС, період апірексії настав через 8 годин. Як називається такий тип температурної кривої?

Зворотній

Гектичний

Постійний

Септичний

Інтермітуючий

1130 / 6307
Травма мозку викликала підвищене утворення аміаку. Яка амінокислота бере участь у видаленні аміаку з мозкової тканини?

Глутамінова

Триптофан

Тирозин

Валін

Лізин

1131 / 6307
Хвора 40-а років надійшла до інфекційного відділення лікарні з високою температурою тіла. Об’єктивно: виражені менінгеальні симптоми. Проведено спинномозкову пункцію. Яке анатомічне утворення було пропунктовано?

Spatium epidurale

Spatium subarachnoideum

Cisterna cerebellomedullaris posterior

Cavum trigeminale

Spatium subdurale

1132 / 6307
У вагітної жінки 29-ти років лікар визначив розміри таза. За допомогою циркуля була виміряна відстань між двома верхніми передніми клубовими остями. Який розмір великого таза був визначений?

Conjugata vera

Distantia cristarum

Distantia trochanterica

Distantia spinarum

Conjugata anatomica

1133 / 6307
Після проведення туберкулінової проби (проба Манту) у дитини через 48 годин на місці ведення туберкуліну утворилася папула до 10 мм у діаметрі. Який механізм гіперчутливості лежить в основі розвитку вказаних змін?

Анафілаксія

Гранулематоз

Антитілозалежна цитотоксичність

Імунокомплексна цитотоксичність

Клітинна цитотоксичність

1134 / 6307
У людини хірургічно видалили ушкоджену патологічним процесом дистальну чверть тонкої кишки. Як це позначиться на всмоктуванні поживних речовин при звичайному харчовому раціоні?

Зменшиться всмоктування вуглеводів

Зменшиться всмоктування води

Всмоктування не зміниться

Зменшиться всмоктування білків

Зменшиться всмоктування жирів

1135 / 6307
У новонародженого хлопчика під час огляду зовнішніх статевих органів виявлена розщілина сечівника, яка відкривається на нижній поверхні статевого члена. Про яку аномалію йдеться?

Монорхізм

Епіспадія

Гіпоспадія

Гермафродитизм

Крипторхізм

1136 / 6307
У ході катаболізму гістидину утворюється біогенний амін, що має потужну судинорозширюючу дію. Назвіть його:

Дофамін

Гістамін

ДОФА

Норадреналін

Серотонін

1137 / 6307
У чоловіка швидкість клубочкової фільтрації 80 мл/хв (норма - 125 ± 25 мл/хв). Збільшення якого показника може бути причиною цього?

Нирковий кровотік

Проникність ниркового фільтру

Ефективний фільтраційний тиск

Онкотичний тиск плазми крові

Гідростатичний тиск крові в капілярах клубочків

1138 / 6307
Хворому внутрішньовенно ввели гіпертонічний розчин глюкози. Це підсилить рух води:

З клітин до міжклітинної рідини

З міжклітинної рідини до клітин

З міжклітинної рідини до капілярів

З капілярів до міжклітинної рідини

Змін руху води не буде

1139 / 6307
Хвора 45-ти років звернулася до лікаря з скаргами на порушення сну, що проявлялося погіршенням засинання та пробудженнями серед ночі. Лікар призначив хворій снодійний засіб. Який вид фармакотерапії використав лікар?

Вибіркова

Патогенетична

Рефлекторна

Етіотропна

Симптоматична

1140 / 6307
Для запобігання віддалених результатів чотиридобової малярії пацієнту 42-х років призначили примахін. Вже на 3 добу від початку лікування терапевтичними дозами препарату в пацієнта з’явилися біль у животі та в ділянці серця, диспепсичні розлади, загальний ціаноз, гемоглобінурія. Що стало причиною розвитку побічної дії препарату?

Кумуляція лікарського засобу

Сповільнення екскреції препарату з сечею

Зниження активності мікросомаль-них ферментів печінки

Потенціювання дії іншими препаратами

Генетична недостатність глюкозо-6-фосфат-дегідрогенази

1141 / 6307
До ендокринолога звернулася хво- ра 45-ти років із скаргами на підвищений апетит, сухість слизових оболонок ротової порожнини, зростання діурезу. При обстеженні вперше виявлено інсулінонезалежний діабет. Який з названих препаратів доцільно призначити хворій?

Адіурекрин

Глібенкламід

Окситоцин

Вазопресин

Інсулін

1142 / 6307
У хворого спостерігається зниження тактильної та смакової чутливості слизової оболонки задньої третини спинки язика. З патологією якого нерва це пов’язано?

Лицевий

Язикоглотковий

II гілка трійчастого нерва

Під’язиковий

III гілка трійчастого нерва

1143 / 6307
На 8-й день після введення протиправцевої сироватки з приводу брудної рани стопи у пацієнта підвищилася температура тіла до 380 С, з’явилися біль у суглобах, висипка, свербіж. У крові - лейкопенія і тромбоцитопенія. Який тип алергічної реакції розвинувся?

Анафілактична

Гіперчутливість уповільненого типу

Цитотоксична

Імунокомплексна

Стимулююча

1144 / 6307
Юнак 17-ти років захворів гостро, температура тіла підвищилася до 38,5o С, з’явилися кашель, нежить, сльозотеча, виділення з носу. Яке запалення розвинулося у юнака?

Катаральне

Гнійне

Геморагічне

Фібринозне

Серозне

1145 / 6307
У добовій сечі хворого знайдені вилужені еритроцити. Для якої патології нирок найбільш притаманний виявлений симптом?

Пієлонефрит

Гостра ниркова недостатність

Нирковокам’яна хвороба

Нефротичний синдром

Дифузний гломерулонефрит

1146 / 6307
При аналізі родоводу лікар-генетик встановив, що хвороба зустрічається в осі6 чоловічої та жіночої статей, не в усіх поколіннях, і що у здорових батьків можуть народжуватися хворі діти. Який тип успадкування хвороби?

Аутосомно-рецесивний

Y-зчеплений

Х-зчеплений домінантний

Аутосомно-домінантний

Х -зчеплений рецесивний

1147 / 6307
Хворий на бронхіальну астму не повідомив лікаря, що в нього бувають напади стенокардії. Лікар призначив препарат, після прийому якого напади бронхіальної астми стали рідшими, однак, почастішали напади стенокардії. Який препарат був призначений?

Ізадрин

Еуфілін

Кромолін-натрій

Сальбутамол

Фенотерол

1148 / 6307
Хворий, що лікувався з приводу неврозу сибазоном, скаржиться на зубний біль. Лікар призначив йому знеболювальний засіб у дозі, яка менша за середню терапевтичну. Яке явище взяв до уваги лікар, зменшуючи дозу препарату?

Потенціювання

Лікарська залежність

Толерантність

Кумуляція

Сумація

1149 / 6307
Запальний процес видозміненої підсерозної основи навколо шийки матки спричинив інтенсивний больовий симптом у пацієнтки. Патологічний процес якої ділянки статевих органів встановив лікар?

Myometrium

Perimetrium

Endometrium

Parametrium

Mesometrium

1150 / 6307
При дослідженні амніотичної рідини, одержаної під час амніоцентезу (прокол амніотичної оболонки), виявлені клітини, ядра яких містять статевий хроматин (тільце Барра). Про що це може свідчити?

Трисомія

Розвиток плода чоловічої статі

Розвиток плода жіночої статі

Генетичні порушення розвитку плода

Поліплоїдія

1151 / 6307
На гістологічному препараті, забарвленому орсеїном, у середній оболонці судини виявлено від 40 до 60 вікончастих еластичних мембран. Назвіть цю судину:

Вена безм’язового типу

Вена м’язового типу

Артерія еластичного типу

Артерія м’язового типу

Артерія змішаного типу

1152 / 6307
У людей після тривалого фізичного навантаження виникає інтенсивний біль у м’язах. Які зміни в м’язах є найбільш вірогідною причиною цього?

Підвищена збудливість

Підвищення вмісту АДФ

Посилений розпад білків

Накопичення молочної кислоти

Накопичення креатиніну

1153 / 6307
Під час операції у пацієнта було видалено частину легені, яка вентилюється бронхом третього порядку, що супроводжується гілками легеневої артерії та інших судин. Яка частина легені була видалена?

Сегмент легені

Середня частка

Верхня частка

Нижня частка

Легенева часточка

1154 / 6307
При спадковій оротацидурії виділення оротової кислоти в багато разів перевищує норму. Синтез яких речовин буде порушений при цій патології?

Пуринові нуклеотиди

Піримідинові нуклеотиди

Сечовина

Біогенні аміни

Сечова кислота

1155 / 6307
На розтині тіла чоловіка 46-ти років на слизовій оболонці прямої та сигмоподібної кишок виявлено множинні коричнево-зелені нашарування, крововиливи; у просвіті кишки слиз, невелика кількість крові; гістологічно - фібринозний коліт. При бактеріологічному дослідженні вмісту кишки висіяна S.Sonne. Який найбільш вірогідний діагноз?

Холера

Ієрсиніоз

Сальмонельоз

Дизентерія

Хвороба Крона

1156 / 6307
Чоловік 40-а років протягом 10 років після перелому великогомілкової кістки страждав на хронічний остеомієліт. Три роки тому з’явився нефротичний синдром. Смерть настала від уремії. На секції: нирки щільні, білі, з рубцями в кірковому шарі на розрізі з сальним блиском. Яка патологія нирок розвинулася?

Хронічний пієлонефрит

Ідіопатичний амілоїдоз

Хронічний гломерулонефрит

Первинний амілоїдоз

Вторинний амілоїдоз

1157 / 6307
Пацієнтка 58-ми років скаржиться на підвищену втомлюваність, зниження працездатності, сонливість, задишку під час швидкої ходи. У крові: ер.-4,6•1012/л, Hb- 92 г/л, КП- 0,6. У мазку крові - велика кількість анулоцитів та мікроцитів. Для якої анемії це характерно?

Серповидноклітинна

Залізодефіцитна

Перніціозна

Гемолітична

Постгеморагічна

1158 / 6307
У жінки 30-ти років виникли набряки обличчя. При обстеженні виявлені протеїнурія (5,87 г/л), гіпопротеїнемія, диспротеїнемія, гіперліпідемія. Для якого стану характерно таке поєднання симптомів?

Хронічна ниркова недостатність

Нефротичний синдром

Хронічний пієлонефрит

Нефритичний синдром

Гостра ниркова недостатність

1159 / 6307
Хворий багато років страждав на бронхіальну астму та помер від нападу ядухи. Під час гістологічного дослідження легень виявлено: в просвіті бронхіол та дрібних бронхів багато слизу з домішкою еозинофілів, склероз міжальвеолярних перетинок, розширення просвіту альвеол. Який механізм розвитку реакції гіперчутливості мав місце?

Гранулематоз

Цитотоксична реакція

Цитоліз, обумовлений лімфоцитами

Імунокомплексна реакція

Реагінова реакція

1160 / 6307
У хворого знижений синтез вазопресину, що призводить до поліурії і, як наслідок, до вираженої дегідратації організму. У чому полягає механізм розвитку поліурії?

Збільшення швидкості клубочкової фільтрації

Зниження реабсорбції глюкози

Зниження канальцевої реабсорбції іонів Na

Зниження канальцевої реабсорбції білку

Зниження канальцевої реабсорбції води

1161 / 6307
У медичній практиці для профілактики алкоголізму широко використовують тетурам, який є інгібітором альдегіддегідрогенази. Підвищення в крові якого метаболіту викликає відразу до алкоголю?

Ацетальдегід

Етанол

Малоновий альдегід

Метанол

Пропіоновий альдегід

1162 / 6307
До клініки надійшла дитина 1 року з ознаками ураження м’язів. Після обстеження виявлений дефіцит карнітину в м’язах. Порушення якого процесу є біохімічною основою цієї патології?

Транспорт жирних кислот у мітохондрії

Синтез актину та міозину

Регуляція рівня Ca2+ в мітохондріях

Субстратне фосфорилювання

Утилізація молочної кислоти

1163 / 6307
В експерименті електричними імпульсами подразнюють нерв, що призводить до виділення привушною залозою великої кількості рідкої слини. Який нерв стимулюють?

N.glossopharyngeus

N.vagus

N. facialis

N.sympathicus

N.trigeminus

1164 / 6307
До щелепно-лицьового відділення надійшов хворий з переломом нижньої щелепи. Було вирішено з’єднання кісток провести хірургічним методом під наркозом. Після внутрішньовенного введення міорелаксанту спостерігались короткочасні фібрилярні скорочення м’язів обличчя хворого. Який міорелаксант було застосовано?

Тубокурарина хлорид

Пипекуроній бромід

Меліктин

Дитилін

Діазепам

1165 / 6307
У хворого відзначається атрофія альвеолярних відростків щелепи після видалення зубів. Це є прикладом:

структурного сліду адаптації

хвороби

патологічного стану

патологічної реакції

патологічного процесу

1166 / 6307
Під час гістологічного дослідження слизової оболонки матки знайдено звивисті залози, пилко- та штопороподібні, подовжені, розростання строми з гіперплазією її клітин. Який найбільш вірогідний діагноз?

Лейоміома

Пухирний занос

Плацентарний поліп

Залозиста гіперплазія ендометрія

Гострий ендометрит

1167 / 6307
При лабораторному дослідженні крові пацієнта 44-х років виявлено, що вміст білків в плазмі становить 40 г/л. Як це впливає на транскапілярний обмін води?

Зменшуються фільтрація та реабсорбція

Збільшуються фільтрація та реабсорбція

Обмін не змінюється

Збільшується фільтрація, зменшується реабсорбція

Зменшується фільтрація, збільшується реабсорбція

1168 / 6307
В гістологічному препараті органу ротової порожнини видно, що передня поверхня вистелена багатошаровим плоским незроговілим епітелієм, а задня поверхня - багаторядним війчастим епітелієм. Що це за орган?

Тверде піднебіння

Щока

М’яке піднебіння

Губа

Ясна

1169 / 6307
Хворому на гостру серцеву недостатність було введено серцевий глікозид швидкої дії. Який з перелічених засобів було введено?

Корглікон

Адонізид

Дигітоксин

Целанід

Мілринон

1170 / 6307
У нирках досліджуваного збільшена реабсорбція іонів кальцію і зменшена - фосфатних іонів. Впливом якого гормону це зумовлено?

Вазопресин

Паратгормон

Гормональна форма вітаміну D3

Альдостерон

Тирокальцитонін

1171 / 6307
У людини вміст глюкози в крові 15 ммоль/л (поріг реабсорбції - 10 ммоль/л). Наслідком цього буде:

Зменшення діурезу

Зменшення реабсорбції глюкози

Глюкозурія

Зменшення секреції вазопресину

Зменшення секреції альдостерону

1172 / 6307
Людина після травми головного мозку втратила зір. Пошкодження яких зон кори головного мозку може бути причиною цього?

Скронева та тім’яна

Скронева

Тім’яна

Потилична

Лобна

1173 / 6307
При аналізі ЕКГ людини з’ясовано, що у другому стандартному відведенні від кінцівок зубці Т позитивні, їх амплітуда та тривалість відповідає нормі. Вірним є висновок про те, що в шлуночках серця нормально відбувається процес:

Реполяризація

Збудження

Деполяризація

Скорочення

Розслаблення

1174 / 6307
Ізольована клітина серця людини автоматично генерує імпульси збудження з частотою 60 разiв за хвилину. З якої структури серця отримано цю клітину?

Шлуночок

Передсердя

Пучок Пса

Атріовентрикулярний вузол

Синоатрiальний вузол

1175 / 6307
Під час підготовки пацієнта до операції проведено вимірювання тиску в камерах серця. В одній з них тиск протягом серцевого циклу змінювався від 0 до 120 мм рт.ст. Назвіть цю камеру серця:

Лівий шлуночок

Правий шлуночок

Праве передсердя

Ліве передсердя

1176 / 6307
Під час помірного фізичного навантаження хвилинний об’єм крові в досліджуваного становить 10 л/хв. Який об’єм крові проходить у нього за хвилину через судини легень?

4 л/хв

6 л/хв

5 л/хв

7 л/хв

10 л/хв

1177 / 6307
У немовляти, народженої передчасно, частина альвеол не розправилася за рахунок підвищеної еластичної тяги легень. Яким чином можна зменшити цю силу?

Відсмоктування рідини з дихальних шляхів

Вдихання чистого кисню

Введення сурфактантів

Штучна вентиляція легень

Введення глюкози

1178 / 6307
Під час розтину тіла померлого в прямій та сигмоподібній кишках видно дефекти слизової оболонки неправильної форми з нерівними контурами, вони зливаються між собою, залишаючи невеликі островки збереженої слизової оболонки. Про який різновид коліту йдеться?

Катаральний

Фібринозний

Гнійний

Виразковий

Фолікулярний

1179 / 6307
У дитини з гострою кишковою інфекцією швидко розвинулись ознаки зневоднення, з’явилася кров у випорожненнях. Педіатром було запідозрено коліентерит. Яким методом необхідно скористатись для діагностики ентерального ешерихіозу?

Бактеріологічний

Алергічний

Біологічний

Серологічний

Мікроскопічний

1180 / 6307
У жінки 22-х років, при гістологічному дослідженні збільшених шийних лімфатичних вузлів, виявлені вузлики, що складаються переважно з плоских, дещо витягнутих, з блідо забарвленим ядром, гігантських клітин округло-овальної форми з блідо-рожевою цитоплазмою та з ядрами, розташованими на периферії (у вигляді частоколу), а також дрібних округлої форми клітин з вузьким обідком цитоплазми; у центрі деяких вузликів безструктурні маси, забарвлені в блідо-рожевий колір. Для якого захворювання характерні такі зміни?

Лімфогранулематоз

Неспецифічний гіперпластичний лімфаденіт

Бруцельоз

Туберкульоз

Саркоїдоз

1181 / 6307
У хворого під час об’єктивного огляду виявлено: тонка фігура, великий череп, сильно розвинена лобна частина обличчя, короткі кінцівки. Для якого конституціонального типу це характерно?

Мускульний

Церебральний

Респіраторний

Дигестивний

Змішаний

1182 / 6307
Хвора 38-ми років померла під час нападу бронхіальної астми, що не вдалося купірувати. Під час гістологічного дослідження в просвіті бронхів виявлені скупчення слизу, в стінці бронхів багато тучних клітин (лаброцитів), багато з них у стані дегрануляції, а також багато еозинофілів. Який патогенез цих змін у бронхах?

Гранулематоз

Імунокомплексний механізм

Клітинно обумовлений цитоліз

Цитотоксична, цитолітична дія антитіл

Атопія

1183 / 6307
У хворого на тромбофлебіт нижніх кінцівок з’явився біль у грудній клітці, кровохаркання, наростаюча дихальна недостатність, при явищах якої він помер. На розтині діагностовано множинні інфаркти легень. Яка найбільш вірогідна причина їх розвитку в цьому випадку?

Тромбоз бронхіальних артерій

Тромбоемболія бронхіальних артерій

Тромбоз гілок легеневої артерії

Тромбоемболія гілок легеневої артерії

Тромбоз легеневих вен

1184 / 6307
У клітинах мозку лисиці, яка була спіймана в межі міста, виявлені включення у вигляді тілець Бабеша-Негрі. Джерелом якого захворювання є ця тварина?

Кліщовий енцефаліт

Сказ

Вітряна віспа

Інфекційний мононуклеоз

Грип

1185 / 6307
Хворому, що страждає на стенокардію та приймає ізосорбіда мононітрат, було додатково призначено лікарський засіб з дезагрегантним ефектом. Визначте цей препарат:

Анаприлін

Ацетилсаліцилова кислота

Нітрогліцерин

Валідол

Ніфедипін

1186 / 6307
У хворого на обличчі вугрі. Під час мікроскопії зіскрібків із уражених ділянок виявлені живі членистоногі розміром 0,2-0,5 мм, які мають витягнуту червоподібну форму, чотири пари коротких кінцівок, що розташовані в середній частині тіла. Який лабораторний діагноз?

Фтиріоз

Міаз

Демодекоз

Короста

Педикульоз

1187 / 6307
У хворого гостра серцево-легенева недостатність, що супроводжується набряком легень. Який препарат з групи діуретиків необхідно призначити?

Спіронолактон

Тріамтерен

Фуросемід

Діакарб

Дихлотіазид

1188 / 6307
До лікаря звернулася хвора зі скаргами на нежить, який посилюється навесні в період цвітіння рослин. Було встановлено діагноз алергійного риніту. Які зміни лейкоцитарної формули можна очікувати в аналізі крові цієї хворої?

Лімфопенія

Зсув формули вліво

Еозинопенія

Лімфоцитоз

Еозинофілія

1189 / 6307
В крові пацієнта вміст глюкози натщесерце 5,6 ммоль/л, через 1 годину після цукрового навантаження - 13,8 ммоль/л, а через 3 години - 9,2 ммоль/л. Для якої патології характерні такі показники?

Хвороба Іценко-Кушінга

Прихована форма цукрового діабету

Здорова людина

Тиреотоксикоз

Акромегалія

1190 / 6307
До терапевтичного відділення надійшов хворий на виразкову хворобу шлунка з гіперацидним синдромом. Який препарат з перелічених груп лікарських засобів необхідно використати у комплексній терапії пацієнта?

Стероїдні протизапальні засоби

Блокатори Н1-гістамінових рецепторів

Блокатори кальцієвих каналів

Нестероїдні протизапальні засоби

Блокатори Н2-гістамінових рецепторів

1191 / 6307
Чоловік 45-ти років звернувся до лікаря з приводу бляшкоподібного утвору на шиї. В біоптаті шкіри гістологічно виявлено пухлинні клітини, розташовані гніздами, які мають круглу та овальну форму з вузьким ободком базофільної цитоплазми та схожі на клітини базального шару епідермісу. Як називається пухлина в пацієнта?

Базаліома

Сирінгоаденома

Гідраденома

Епідермальний рак

Трихоепітеліома

1192 / 6307
У хворого з діагнозом хвороба Іценка-Кушинга (гіперпродукція гормонів корою наднирників) в крові визначено підвищену концентрацію глюкози, кетонових тіл, натрію. Який біохімічний механізм є провідним у виникненні гіперглікемії?

Гліколіз

Аеробний гліколіз

Глікогеноліз

Глікогенез

Глюконеогенез

1193 / 6307
При цукровому діабеті внаслідок активації процесів окиснення жирних кислот виникає кетоз. До яких порушень кислотно-лужної рівноваги може призвести надмірне накопичення кетонових тіл у крові?

Метаболічний алкалоз

Змін не відбуватиметься

Дихальний алкалоз

Дихальний ацидоз

Метаболічний ацидоз

1194 / 6307
У жінки, що має 0 (I) групу крові, народилася дитина з групою крові AB. Чоловік цієї жінки має групу крові A. Які з наведених нижче видів взаємодії генів пояснюють це явище?

Неповне домінування

Кодомінування

Комплементарність

Полімерія

Епістаз рецесивний

1195 / 6307
У дитини 5-ти років діагностовано хворобу Брутона, яка проявляється у важкому перебігу бактеріальних інфекцій, відсутності В-лімфоцитів та плазматичних клітин. Які зміни вмісту імуноглобулінів будуть спостерігатися в сироватці крові цієї дитини?

Змін не буде

Збільшення IgA, IgM

Зменшення IgD, IgE

Зменшення IgA, IgM

Збільшення IgD, IgE

1196 / 6307
У жінки 30-ти років виявлено недостатність зовнішньосекреторної функції підшлункової залози. Гідроліз яких поживних речовин буде порушений?

Білки, вуглеводи

Білки

Жири, вуглеводи

Білки, жири, вуглеводи

Білки, жири

1197 / 6307
При розслідуванні спалаху внутрішньолікарняної інфекції виділені чисті культури золотистого стафілококу від хворих, медперсоналу та деяких об’єктів навколишнього середовища. Яке дослідження необхідно провести, щоб визначити ідентичність виділених стафілококів і встановити джерело госпітальної інфекції?

Фаготипування

Серотипування

Визначення біовару

Визначення патогенності

Зараження тварин

1198 / 6307
У студента через 2 години після іспиту в аналізі крові виявлено лейкоцитоз без істотних змін у лейкоцитарній формулі. Який найбільш вірогідний механізм розвитку лейкоцитозу?

Уповільнення міграції лейкоцитів у тканини

Уповільнення руйнування лейкоцитів

Посилення лейкопоезу та зменшення руйнування лейкоцитів

Перерозподіл лейкоцитів в організмі

Посилення лейкопоезу

1199 / 6307
У молодої жінки видалили пухлину дистального кінця стегнової кістки, яка швидко зростала. Макроскопічно: пухлина строкатого вигляду - від біло-сірого до коричневочервоного кольору, пухкої консистенції. Мікроскопічно: основний тканинний компонент пухлини представлений кістковими та остеоїдними структурами, вистеленими атиповими остеобластами з патологічними мітозами; безліч тонкостінних судин. Який найбільш вірогідний діагноз?

Ангіосаркома

Хондрома

Остеосаркома

Саркома Юїнга

Остеома

1200 / 6307
У жінки 39-ти років відмічається мокнуття в ділянці соска молочної залози, наявність неглибокої виразки з запальною гіперемією та набряком шкіри. При гістологічному дослідженні біоптату цієї ділянки в ростковому шарі потовщеного епідермісу виявлені атипові великі клітини зі світлою та оптично пустою цитоплазмою, з відсутністю міжклітинних містків. Такі клітини знайдені і в усті великих протоків залози. Який найбільш вірогідний діагноз?

Базально-клітинний рак

Внутрішньопротоковий рак

Меланома

Плоскоклітинний рак

Хвороба Педжета

1201 / 6307
Хворому перед операцією був ведений дитилін (лістенон) і проведена інтубація. Після закінчення операції та припинення наркозу самостійне дихання не відновилося. Дефіцит якого ферменту в організмі хворого подовжує дію м’язового релаксанту?

Карбангідраза

K — Na-АТФ-аза

N-ацетилтрансфераза

Псевдохолінестераза

Сукцинатдегідрогеназа

1202 / 6307
Людина хворіє на цукровий діабет, що супроводжується гіперглікемією натще понад 7,2 ммоль/л. Рівень якого білка плазми крові дозволяє ретроспективно (за попередні 4-8 тижні до обстеження) оцінити рівень глікемії?

Фібриноген

Церулоплазмін

Альбумін

Глікозильований гемоглобін

С-реактивний білок

1203 / 6307
У пацієнта 60-ти років виявлено погіршення сприйняття звуків високої частоти. Порушення стану яких структур слухового аналізатора зумовило ці зміни?

Євстахієва труба

Барабанна перетинка

М’язи середнього вуха

Основна мембрана завитки біля гелікотреми

Основна мембрана завитки біля овального віконця

1204 / 6307
У хворого на цукровий діабет з’явився різкий біль у правій стопі. Об’єктивно: великий палець стопи чорного кольору, тканини стопи набряклі, осередки відшарування епідермісу, виділення з неприємним запахом. Яка клініко-морфологічна форма некрозу розвинулася в хворого?

Гангрена волога

Секвестр

Пролежень

Іангрена суха

Інфаркт

1205 / 6307
Постраждалому з колотою раною передньої стінки шлунка наддається хірургічна допомога. До якого утворення порожнини очеревини потрапив вміст шлунка?

Передшлункова сумка

Правий мезентеріальний синус

Печінкова сумка

Лівий мезентеріальний синус

Сальникова сумка

1206 / 6307
До клініки надійшла дитина 4-х років з ознаками тривалого білкового голодування: затримка росту, анемія, набряки, розумова відсталість. Причиною розвитку набряків у цієї дитини є зниження синтезу:

Альбумінів

Глобулінів

Ілікопротеїнів

Ліпопротеїнів

Гемоглобіну

1207 / 6307
Під час розтину трупа чоловіка, що страждав на крупозну пневмонію і помер від пневмококового сепсису, у правій плевральній порожнині містилося 900 мл каламутної зеленувато-жовтого кольору рідини. Листки плеври тьмяні, повнокровні. Назвіть клінікоморфологічну форму запалення в плевральній порожнині:

Флегмона

Гострий абсцес

Хронічний абсцес

Фібринозне запалення

Емпієма

1208 / 6307
У хворого виявлено зниження вмісту іонів магнію, які потрібні для прикріплення рибосом до гранулярної ендоплазматичної сітки. Відомо, що це призводить до порушення біосинтезу білка. Який саме етап біосинтезу білка буде порушено?

Реплікація

Термінація

Трансляція

Активація амінокислот

Транскрипція

1209 / 6307
При дослідженні сироватки крові хворого виявлене підвищення рівня аланінамінотрансферази (АЛТ) та аспартатамінотрансферази (АСТ). Які зміни на клітинному рівні можуть призвести до подібної ситуації?

Руйнування клітин

Руйнування генетичного апарату клітин

Порушення міжклітинних взаємовідношень

Порушення ферментних систем клітин

Порушення функції енергозабезпечення клітин

1210 / 6307
У юнака енерговитрати збільшились з 500 до 2000 кДж за годину. Що з наведеного може бути причиною цього?

Перехід від сну до бадьорості

Підвищення зовнішньої температури

Прийом їжі

Розумова праця

Фізичне навантаження

1211 / 6307
У хворого виявлено підвищення активності ЛДГ1 ,2 , АсАТ, креатинфосфокінази. В якому органі (органах) найбільш вірогідний розвиток патологічного процесу?

Сполучна тканина

Печінка та нирки

Серцевий м’яз

Скелетні м’язи

Нирки та надниркові залози

1212 / 6307
За даними ВООЗ на малярію щорічно на Землі хворіють приблизно 250 млн. чоловік. Ця хвороба зустрічається переважно у тропічних і субтропічних областях. Межі її розповсюдження співпадають з ареалами комарів роду:

Анофелес

Кулізета

Аедес

Мансоніа

Кулекс

1213 / 6307
У померлого 58-ми років на розтині: мітральний клапан деформований, потовщений, змикається не до кінця. Мікроскопічно: вогнища колагенових волоконець еозінофільні, дають позитивну реакцію на фібрин. Найвірогідніше це:

Амілоїдоз

Мукоїдне набухання

Гіаліноз

Фібринозне запалення

Фібриноїдне набухання

1214 / 6307
Для вивчення локалізації біосинтезу білка в клітинах, миші ввели мічені амінокислоти аланін та триптофан. Біля яких органел буде спостерігатися накопичення мічених амінокислот?

Апарат Гольджі

Лізосоми

Рибосоми

Гладенька ЕПС

Клітинний центр

1215 / 6307
У районах Південної Африки у людей розповсюджена серпоподібно-клітинна анемія, при якій еритроцити мають форму серпа внаслідок заміни в молекулі гемоглобіну амінокислоти глутаміну на валін. Чим викликана ця хвороба?

Генна мутація

Порушення механізмів реалізації генетичної інформації

Геномні мутації

Кросинговер

Трансдукція

1216 / 6307
Хворий помер при явищах серцево-судинної недостатності. Результати розтину: післяінфарктний кардіосклероз, гіпертрофія міокарда і дилятація його порожнин, особливо правого шлуночка. Печінка збільшена, з гладенькою поверхнею, на розрізі повнокровна, з темно-червоними крапками на буруватому фоні тканини. Гістологічно: повнокров’я центральних відділів часточок; у периферичних відділах навколо портальних трактів - гепатоцити у стані жирової дистрофії. Як називаються описані вище зміни печінки?

Мускатна печінка

Несправжньомускатна печінка

Стеатоз печінки

Амілоїдоз

Цироз печінки

1217 / 6307
У померлого від хронічної серцево-судинної недостатності на розтині виявлене 'тигрове серце'. З боку ендокарду помітна жовтувато-біла посмугованість; міокард тьмяний, глинисто-жовтий. Який процес зумовив дану патологію?

Вуглеводна дистрофія

Жирова паренхіматозна дистрофія

Жирова судинно-стромальна дистрофія

Амілоїдоз

Гіаліново-краплинна дистрофія

1218 / 6307
Під час експерименту подразнюють скелетний м’яз серією електричних імпульсів. Який вид м’язового скорочення буде виникати, якщо кожний наступний імпульс надходить у періоді вкорочення попереднього поодинокого м’язового скорочення?

Суцільний тетанус

Контрактура м’яза

Асинхронний тетанус

Серія поодиноких скорочень

Зубчастий тетанус

1219 / 6307
Амоніак є дуже отруйною речовиною, особливо для нервової системи. Яка речовина бере особливо активну участь у знешкодженні амоніаку в тканинах мозку?

Гістидин

Глутамінова кислота

Лізин

Аланін

Пролін

1220 / 6307
Хворий чоловік госпіталізований на 5-й день хвороби з проявами жовтяниці, болем у м’язах, ознобом, носовими кровотечами. Під час проведення лабораторної діагностики бактеріолог виконав темнопольну мікроскопію краплини крові хворого. Назвіть збудника хвороби:

Borrelia dutlonii

Calymmatobacterium granulomatis

Bartonella bacilloformis

Leptospira interrogans

Rickettsia mooseri

1221 / 6307
У хворого поперечний розрив спинного мозку нижче VI грудного сегменту. Як внаслідок цього зміниться дихання?

Стане більш частим

Стане більш глибоким

Стане більш рідким

Зупиниться

Суттєво не зміниться

1222 / 6307
У хворого через 12 годин після гострого нападу загруднинного болю знайдено різке підвищення активності АсАТ у сироватці крові. Вкажіть патологію, для якої характерне це зміщення:

Колагеноз

Вірусний гепатит

Цукровий діабет

Інфаркт міокарда

Нецукровий діабет

1223 / 6307
Людина стоїть у кімнаті в легкому одязі, температура повітря +140 C, вікна і двері зачинені. Яким шляхом вона віддає найбільше тепла?

Теплопроведення

Випаровування

Конвекція

Перспірація

Теплорадіація

1224 / 6307
Жінка 30-ти років хворіє близько року, коли вперше з’явився біль у ділянці суглобів, їх припухлість, почервоніння шкіри над ними. Попередній діагноз - ревматоїдний артрит. Зміна якого компоненту в структурі білка сполучної тканини є однією з причин цього захворювання?

Муцин

Колаген

Овоальбумін

Тропонін

Міозин

1225 / 6307
Хворий з інфекційним мононуклеозом протягом двох тижнів приймав глюкокортикостероїдні препарати. Наступила ремісія, проте в нього виникло загострення хронічного тонзиліту. Результатом якої дії глюкокортикостероїдів є дане ускладнення?

Імунодепресивна

Протизапальна

Антитоксична

Протишокова

Антиалергічна

1226 / 6307
У собаки в досліді подразнювали на шиї периферичний відрізок блукаючого нерва. При цьому спостерігали такі зміни серцевої діяльності:

Збільшення збудливості міокарда

Збільшення швидкості атріовентрикулярного проведення

Зменшення частоти скорочень

Збільшення сили скорочень

Збільшення частоти та сили скорочень

1227 / 6307
У результаті виснажуючої м’язової праці у робочого значно зменшилася буферна ємність крові. Надходження якої речовини у кров може бути причиною цього явища?

1,3-бісфосфогліцерат

3-фосфогліцерат

Піруват

Лактат

1228 / 6307
Хвора доставлена бригадою швидкої допомоги. Об’єктивно: стан важкий, свідомість відсутня, адинамія. Шкірні покриви сухі, запалі очі, ціаноз обличчя, тахікардія, запах ацетону з рота. Результати аналізів: глюкоза крові -20,1 ммоль/л (у нормі - 3,3-5,5 ммоль/л), у сечі - 3,5% (у нормі - 0). Який найбільш вірогідний діагноз?

Гіперглікемічна кома

Гіпоглікемічна кома

Анафілактичний шок

Гостра серцева недостатність

Гостре алкогольне отруєння

1229 / 6307
Жінка 62-х років скаржиться на частий біль у ділянці грудної клітки та хребта, переломи ребер. Лікар припустив мієломну хворобу (плазмоцитому). Який з перерахованих нижче лабораторних показників буде мати найбільше діагностичне значення?

Гіперальбумінемія

Парапротеїнемія

Гіпопротеїнемія

Гіпоглобулінемія

Протеїнурія

1230 / 6307
У новонародженої дитини на пелюшках виявлені темні плями, що свідчать про утворення гомогентизинової кислоти. З порушенням обміну якої речовини це пов’язане?

Триптофан

Тирозин

Метіонін

Галактоза

Холестерин

1231 / 6307
Під час роботи щодо ліквідації наслідків аварії на АЕС, робітник одержав дозу опромінення 500 рентген. Скаржиться на головний біль, нудоту, запаморочення. Які зміни кількості лейкоцитів можна очікувати в хворого через 10 годин після опромінення?

Лейкопенія

Лімфоцитоз

Лейкемія

Нейтрофільний лейкоцитоз

Агранулоцитоз

1232 / 6307
У підлітка 12-ти років, який хворіє на бронхіальну астму, виник тяжкий напад астми: виражена експіраторна задишка, блідість шкірних покривів. Який вид порушення альвеолярної вентиляції має місце?

Центральний

Обструктивний

Торако-діафрагмальний

Рестриктивний

Нервово-м’язовий

1233 / 6307
У жінки, що тривалий час дотримувалася дієти з використанням очищеного рису, виявлений поліневрит (хвороба Бері-Бері). Відсутність якого вітаміну в їжі призводить до розвитку цього захворювання?

Рибофлавін

Піридоксин

Фолієва кислота

Тіамін

Аскорбінова кислота

1234 / 6307
У хворого після видалення жовчного міхура утруднені процеси всмоктування Ca через стінку кишечнику. Призначення якого вітаміну буде стимулювати цей процес?

D3

В12

C

K

PP

1235 / 6307
У населеному пункті зареєстрований спалах гепатиту, який зв’язують з водним фактором. Який вірус гепатиту міг викликати спалах захворювань у цьому населеному пункті?

C

G

B

D

E

1236 / 6307
У хворого 43-х років у шлунку погано перетравлюються білки. Аналіз шлункового соку виявив низьку кислотність. Функція яких клітин шлунка порушена в даному випадку?

Ендокринні клітини

Шиєчні мукоцити

Парієтальні екзокриноцити

Слизові клітини (мукоцити)

Головні екзокриноцити

1237 / 6307
До навчального закладу вступив юнак 16-ти років з сільської місцевості. При плановому проведенні реакції Манту виявилося, що у цього юнака вона негативна. Яка найбільш раціональна тактика лікаря?

Провести серодiагностику туберкульозу

Терміново ізолювати юнака з навчального колективу

Зробити щеплення БЦЖ

Повторити реакцію через 1 місяць

Провести прискорену дiагностику туберкульозу методом Прайса

1238 / 6307
Хворий відзначає частi проноси, особливо після вживання жирної їжі, схуднення. Лабораторні дослідження показали наявність стеатореї; кал гіпохолічний. Що може бути причиною такого стану?

Запалення слизової оболонки тонкої кишки

Обтурація жовчних шляхів

Недостатність панкреатичної фосфо-ліпази

Недостатність панкреатичної ліпази

Незбалансована дієта

1239 / 6307
Хворий звернувся до лікаря зі скаргами на дисфункцію кишечнику. Лікар констатував симптоми дуоденіту і ентериту. Під час лабораторного дослідження встановлено діагноз: лямбліоз. Застосування якого препарату показане?

Мономіцин

Тетрациклін

Хінгамін

Еритроміцин

Метронідазол

1240 / 6307
У групи альпіністів на висоті 3000 метрів було зроблено аналіз крові. Виявлене зниження HCO3 до 15 ммоль/л (норма 22-26 ммоль/л). Який механізм зниження HCO3 крові?

Посилення ацидогенезу

Зниження реабсорбції бікарбонатів у нирках

Гіповентиляція

Зниження амоніогенезу

Гіпервентиляція

1241 / 6307
Після введення лікарської речовини у піддослідної тварини зменшилося виділення слини, розширилися зіниці, а при наступному введенні у вену ацетилхоліну частота скорочень серця істотно не змінилася. Вкажіть назву цієї речовини:

Прозерин

Сальбутамол

Адреналін

Атропін

Анаприлін

1242 / 6307
При тривалому використанні препарату в хворого можуть мати місце остеопороз, ерозії слизової шлунка, гіпокаліємія, затримка натрію і води, зменшення вмісту кортикотропіну в крові. Укажіть цей препарат:

Резерпін

Індометацин

Дигоксин

Преднізолон

Гіпотіазид

1243 / 6307
Дитина квола, апатична. Печінка збільшена, при її біопсії виявлено значний надлишок глікогену. Концентрація глюкози в крові нижче норми. У чому причина зниженої концентрації глюкози у крові цієї хворої?

Підвищена активність глікогенсинтетази у печінці

Понижена (відсутня) активність гексокінази у печінці

Понижена (відсутня) активність глікоген-фосфорилази у печінці

Дефіцит гену, який відповідає за синтез глюкозо-1-фосфатуридинтрансферази

Понижена (відсутня) активність глюкозо-6-фосфатази у печінці

1244 / 6307
До лікарні швидкої допомоги госпіталізований чоловік 63-х років з явищами колапсу. Для боротьби з гіпотензією лікар вибрав норадреналін. Який механізм дії цього препарату?

Активація α-адренорецепторів

Активація β-адренорецепторів

Блокада M-холінорецепторів

Активація серотонінових рецепторів

Активація дофамінових рецепторів

1245 / 6307
У дитини з підозрою на дифтерію з зіву виділена чиста культура мікроорганізмів та вивчені їх морфологічні, тинкторіальні, культуральні та біохімічні властивості, які виявилися типовими для збудників дифтерії. Яке дослідження необхідно ще провести для видачі висновку про те, що виділена патогенна дифтерійна паличка?

Визначення цистиназної активносгі

Визначення властивосгі розщеплювати крохмаль

Визначення уреазної активності

Визначення протеолітичних властивостей

Визначення токсигенних властивостей

1246 / 6307
У дитини на слизовій оболонці щік та на язиці виявлені білуваті плями, які нагадують молоко, що скипілося. У виготовлених препаратах-мазках знайдені грампозитивні овальнi дріжджоподі-6ні клітини. Які це збудники?

Фузобактерії

Стафілококи

Актиноміцети

Дифтерійна паличка

Гриби роду Кандіда

1247 / 6307
У хворого 35-ти років, який часто вживає алкоголь, на фоні лікування сечогінними засобами, виникли сильна м’язова і серцева слабкість, блювання, діарея, АТ- 100/60 мм рт.ст., депресія. Причиною такого стану є посилене виділення з сечею:

Кальцію

Калію

Хлору

Натрію

Фосфатів

1248 / 6307
Хворий після вживання жирної їжі відчуває нудоту, млявість; з часом з’явилися ознаки стеатореї. У крові холестерин - 9,2 ммоль/л. Причиною такого стану є нестача у кишечнику:

Фосфоліпідів

Жовчних кислот

Хіломікронів

Тригліцеридів

Жирних кислот

1249 / 6307
У чоловіка, який тривалий час не вживав з їжею жирів, але отримував достатню кількість вуглеводів і білків, виявлено дерматит, погане загоювання ран, погіршення зору. Дефіцит яких компонентів є причиною порушення обміну речовин?

Олеїнова кислота

Вітаміни PP, H

Лінолева кислота, вітаміни A, D, E, K

Мінеральні солі

Пальмітинова кислота

1250 / 6307
Експериментальній тварині давали надлишкову кількість глюкози, міченої за вуглецем, протягом тижня. У якій сполуці можна виявити мітку?

Арахідонова кислота

Пальмітинова кислота

Метіонін

Холін

Вітамін A

1251 / 6307
РНК, що містить вірус імунодефіциту людини, проникла всередину лейкоцита і за допомогою ферменту ревертази змусила клітину синтезувати вірусну ДНК. В основі цього явища лежить:

Конваріантна реплікація

Зворотня транскрипція

Репресія оперона

Дерепресія оперона

Зворотня трансляція

1252 / 6307
У хворого з частими кровотечами з внутрішніх органів і слизових оболонок виявлені пролін і лізин у складі колагенових волокон. Через відсутність якого вітаміну порушено їх гідроксилювання?

Вітамін E

Вітамін K

Тіамін

Вітамін C

Вітамін A

1253 / 6307
У хворого із запаленням легень спостерігається підвищення температури тіла. Яка біологічно активна речовина відіграє провідну роль у виникненні цього прояву?

Брадикінін

Інтерлейкін-I

Гістамін

Серотонін

Лейкотрієни

1254 / 6307
Хвора 27-ми років закрапала в очі краплі, до складу яких входить пеніцилін. Через декілька хвилин з’явився свербіж та печіння тіла, набряк губ та повік, свистячий кашель; став падати артеріальний тиск. Які імуноглобуліни беруть участь в розвитку даної алергічної реакції?

IgG та IgD

IgA та IgM

IgM та IgD

IgE та IgG

IgM та IgG

1255 / 6307
У пацієнта цироз печінки. Дослідження якої з пєрєлічєних речовин, що екскретуються з сечею, може характеризувати стан антитоксичної функції печінки?

Амонійні солі

Гіпурова кислота

Сечова кислота

Креатинін

Амінокислоти

1256 / 6307
У дитини 2-х років виникли судоми внаслідок зниження концентрації іонів кальцію в плазмі крові. Функція якого ендокринного органу знижена?

Прищитоподібні залози

Тимус

Гіпофіз

Шишкоподібна залоза

Кора наднирників

1257 / 6307
При запаленні ока у хворого відмічалося накопичення мутної рідини з високим вмістом білку на дні передньої камери, яке отримало назву - гіпопіон. Який процес лежить в основі зазначених змін?

Вторинна альтерація

Первинна альтерація

Проліферація

Порушення мікроциркуляції

1258 / 6307
У людини частота серцевих скорочень постійно утримується на рівні 40 разів за хвилину. Що є водієм ритму серця у неї?

Волокна Пуркін’є

Атріовентрикулярний вузол

Пучок Гіса

Ніжки пучка Гіса

Синоатріальний вузол

1259 / 6307
Хвора 48-ми років надійшла до клініки із скаргами на слабкість, дратівливість, порушення сну. Об’єктивно: шкіра та склери жовтого кольору. У крові: підвищення рівня загального білірубіну з переважанням прямого. Кал - ахолічний. Сеча - темного кольору (жовчні пігменти). Яка жовтяниця має місце в хворої?

Гемолітична

Синдром Кріглера-Найяра

Синдром Жільбера

Паренхіматозна

Механічна

1260 / 6307
При бактеріологічному дослідженні промивних вод хворого на харчове отруєння висіяли чисту культуру бактерій з такими властивостями: грамнегативна рухлива паличка, на середовищі Ендо росте у вигляді безбарвних колоній. Представником якого роду було зумовлене захворювання?

Citrobacter

Shigella

Salmonella

Iersinia

Esherichia

1261 / 6307
Хворому встановлено діагноз - активний вогнищевий туберкульоз легень. Вкажіть, який із препаратів найбільш доцільно призначити в першу чергу?

Ізоніазид

Циклосерін

Етоксид

Сульфален

Етіонамід

1262 / 6307
При обстеженні молодого чоловіка у центрі по боротьбі зі СНІДом отримано позитивний результат ІФА з антигенами ВІЛ. Скарги на стан здоров’я відсутні. Про що може свідчити результат ІФА?

Про перенесене захворювання на СНІД

Про інфікування ВГВ

Про персистенцію ВГВ

Про захворювання на СНІД

Про інфікування ВІЛ

1263 / 6307
До лікаря звернулися батьки хлопчика 10-ти років, у якого відзначалося збільшення волосяного покриву на тілі, ріст бороди і вус, низький голос. Збільшення секреції якого гормону можна припустити?

Кортизол

Естроген

Тестостерон

Соматотропін

Прогестерон

1264 / 6307
У хворого, що страждає на важку форму порушення водно-сольового обміну, настала зупинка серця в діастолі. Який найбільш вірогідний механізм зупинки серця в діастолі?

Гіпернатріємія

Гіпокаліємія

Гіпонатріємія

Дегідратація організму

Гіперкаліємія

1265 / 6307
Під час дослідження коронарних артерій виявлені атеросклеротичні бляшки з кальцинозом, що закривають просвіт судин на 1/3. У м’язі дрібні множинні білуваті прошарки сполучної тканини. Як називається процес, виявлений у міокарді?

Післяінфарктний кардіосклероз

Інфаркт міокарда

Міокардит

Тигрове серце

Дифузний кардіосклероз

1266 / 6307
У реакції пасивної гемаглютинації, поставленої з еритроцитарним черевнотифозним Viдіагностикумом, виявлені антитіла у розведенні сироватки обстежуваного до 1:80, що вище діагностичного титру. Такий результат свідчить про наступне:

Реконвалесценції хворого на черевний тиф

Інкубаційний період черевного тифу

Рецидив черевного тифу

Гостре захворювання на черевний тиф

Можливе носійство паличок черевного тифу

1267 / 6307
У підлітка внаслідок радіоактивного опромінення значно постраждала лімфоїдна система, відбувся розпад великої кількості лімфоцитів. Відновлення нормальної формули крові можливо завдяки діяльності залози:

Підшлункова

Печінка

Щитоподібна

Тимус

Наднирники

1268 / 6307
До приймального відділення доставлено чоловіка з різаною раною підошви правої стопи. У постраждалого обмежене підняття латерального краю стопи. Під час обробки виявлено пошкодження сухожилка м’яза. Який м’яз ушкоджений?

Триголовий м’яз гомілки

Довгий розгинач пальців

Довгий малогомілковий

Короткий малогомілковий

Передній великогомілковий

1269 / 6307
У хворого, що страждає на серцеву недостатність, спостерігаються збільшення печінки, набряки нижніх кінцівок, асцит. Який механізм є провідним в утворенні даного набряку?

Лімфогенний

Мембраногенний

Колоїдно-осмотичний

Гідродинамічний

1270 / 6307
У мужчини 32-х років високий зріст, гінекомастія, жіночий тип оволосіння, високий голос, розумова відсталість, безпліддя. Попередній діагноз - синдром Клайнфельтера. Що необхідно дослідити для його уточнення?

Лейкоцитарна формула

Група крові

Сперматогенез

Родовід

Каріотип

1271 / 6307
У шахтаря виявлено фіброз легень, що супроводжувався порушенням альвеолярної вентиляції. Який механізм виникнення цього порушення є провідним?

Порушення нервової регуляції дихання

Спазм бронхів

Звуження верхніх дихальних шляхів

Обмеження рухомості грудної клітки

Обмеження дихальної поверхні легень

1272 / 6307
Людина зробила спокійних видих. Як називається об’єм повітря, який міститься у неї в легенях при цьому?

Резервний об’єм видиху

Життєва ємність легень

Дихальний об’єм

Залишковий об’єм

Функціональна залишкова ємність легень

1273 / 6307
При дослідженні ізольованого кардіоміоциту встановлено, що він не генерує імпульси збудження автоматично. З якої структури серця отримано кардіоміоцит?

Волокна Пуркін’є

Пучок Пса

Сино-атріальний вузол

Атріовентрикулярний вузол

Шлуночок

1274 / 6307
При обстеженні людини встановлено, що хвилинний об’єм серця дорівнює 3500 мл, систолічний об’єм - 50 мл. Якою є частота серцевих скорочень за хвилину у людини?

50

90

80

60

70

1275 / 6307
У людини, яка обертається на каруселі, збільшилися частота серцевих скорочень, потовиділення, з’явилася нудота. З подразненням яких рецепторів, перш за все, це пов’язано?

Зорові

Вестибулярні ампулярні

Пропріоцептори

Вестибулярні отолітові

Слухові

1276 / 6307
З метою встановлення токсигенно-ті виділених від пацієнтів збудників дифтерії, культури висіяли на чашку Петрі з поживним агаром по обидва боки від розташованої в центрі смужки фільтрувального паперу, змоченого протидифтерійною антитоксичною сироваткою. Після інкубації посівів в агарі між окремими культурами і смужкою фільтрувального паперу виявлено смужкоподібні ділянки помутніння середовища. Яку імунологічну реакцію було виконано?

Реакція преципітації в гелі

Реакція Кумбса

Реакція аглютинації

Реакція кільцепреципітації

Реакція опсонізації

1277 / 6307
У людини вимірюють внутрішньо-плевральний тиск. У якій фазі людина затримала дихання, якщо величина тиску дорівнює - 7,5 см вод.ст?

Спокійний вдих

Форсований вдих

Форсований видих

Спокійний видих

1278 / 6307
Жінка 49-ти років звернулася до лікаря зі скаргами на підвищену втомлюваність та появу задишки під час фізичного навантаження. На ЕКГ: ЧСС-50/хв, PQ- подовжений, QRS- не змінений, кількість зубців P перевищує кількість комплексів QRS. Який вид аритмії у пацієнтки?

Синусова брадикардія

Синоатріальна блокада

Миготлива аритмія

Екстрасистолія

Атріовентрикулярна блокада

1279 / 6307
У хворого з невритом стегнового нерва порушено згинання стегна та розгинання гомілки у колінному суглобі. Функція якого м’яза при цьому порушена?

Напівперетинчастий м’яз

Двоголовий м’яз стегна

Триголовий м’яз стегна

Чотирьохголовий м’яз стегна

Півсухожилковий м’яз

1280 / 6307
При розтині трупа новонародженого хлопчика виявлені полідактилія, мікроцефалія, незрощення верхньої губи та твердого піднебіння, а також гіпертрофія паренхіматозних органів. Вказані вади відповідають синдрому Патау. Яка найбільш вірогідна причина даної патології?

Нерозходження статевих хромосом

Трисомія 13-ої хромосоми

Часткова моносомія

Трисомія 21-ої хромосоми

Трисомія 18-ої хромосоми

1281 / 6307
На препараті яєчника, забарвленому гематоксиліном-еозіном, визначається фолікул, в якому клітини фолікулярного епітелію розміщені в 1-2 шари та мають кубічну форму, навколо овоциту видно оболонку яскраво-червоного кольору. Назвіть цей фолікул:

Первинний

Атретичний

Зрілий

Примордіальний

Вторинний

1282 / 6307
Після тижневого застосування нового косметичного засобу у жінки розвинулося запалення повік з гіперемією, інфільтрацією та болючістю. Алергічна реакція якого типу розвинулася у пацієнтки?

III

V

II

I

IV

1283 / 6307
Під час огляду хворого відзначаються різке звуження зіниць, сонливість, рідке дихання за типом Чейна-Стокса, затримка сечі, сповільнення серцевого ритму, підвищення спинномозкових рефлексів. Яка речовина викликала отруєння?

Барбітал

Кофеїн

Морфін

Атропін

Фосфакол

1284 / 6307
Хворий знаходиться на обліку в ендокринологічному диспансері з приводу гіпертиреозу. До схуднення, тахікардії, тремтіння пальців рук, приєдналися симптоми гіпоксії - головний біль, втомлюваність, мерехтіння 'мушок' перед очима. Який механізм дії тиреоїдних гормонів лежить в основі розвитку гіпоксії?

Роз’єднання окиснення та фосфорилювання

Конкурентне гальмування дихальних ферментів

Специфічне зв’язування активних центрів дихальних ферментів

Гальмування синтезу дихальних ферментів

Посилення синтезу дихальних ферментів

1285 / 6307
При алергічному дерматиті лікар призначив хворому в складі комплексної терапії Яігістаміноблокатор. Визначте цей препарат:

Преднізолон

Лоратадин

Адреналін

Кромолін-натрій

Гідрокортизон

1286 / 6307
У пораненого кровотеча із гілок сонної артерії. Для тимчасового припинення кровотечі сонну артерію треба притиснути до горбика поперечного відростка шийного хребця. До якого саме хребця притискається артерія у таких випадках?

VI

III

II

V

IV

1287 / 6307
Внаслідок аварії у постраждалого виникли сильний біль та набряк передньої поверхні гомілки; тильне згинання ступні утруднене. Функція якого з названих м’язів гомілки постраждала?

M.tibialis anterior

M.peroneus longus

M.flexor hallucis longus

M.flexor digitorum longus

M.peroneus brevis

1288 / 6307
В першому класі було проведене медичне обстеження учнів з метою відбору дітей для ревакцинації проти туберкульозу. Яку з наведених нижче проб при цьому використали?

Проба Бюрне

Проба Манту

Проба з антраксином

Нашкірна проба з тулярином

Проба Шика

1289 / 6307
В експерименті збільшили проникність мембрани збудливої клітини для іонів калію. До яких змін мембранного потенціалу це призведе?

Потенціал дії

Локальна відповідь

Змін не буде

Гіперполяризація

Деполяризація

1290 / 6307
Хворий на гіпертонічну хворобу II стадії з лікувальною метою приймав один з гіпотензивних препаратів. Через деякий час артеріальний тиск знизився, але хворий став скаржитися на в’ялість, сонливість, байдужість. Пізніше з’явився біль у шлунку. Було діагностовано виразкову хворобу. Який гіпотензивний препарат приймав хворий?

Резерпін

Каптоприл

Верапаміл

Фуросемід

Дибазол

1291 / 6307
У жінки через 6 місяців після пологів розвинулася маткова кровотеча. Під час гінекологічного обстеження у порожнині матки виявлена тканина темно-червоного кольору з множинними порожнинами, що нагадує 'губку'. Під час мікроскопічного дослідження пухлини, у лакунах крові виявлені атипові світлі епітеліальні клітини Лангханса та гігантські клітини синцитіотрофобласта. Яка це пухлина?

Фіброміома

Аденокарцинома

Міхуровий занос

Плоскоклітинний незроговілий рак

Хоріонепітеліома

1292 / 6307
Хворий 42-х років висуває скарги на сильне серцебиття, пітливість, нудоту, порушення зору, тремор рук, підвищення артеріального тиску. З анамнезу: 2 роки тому було встановлено діагноз феохромоцитома. Гіперпродукція яких гормонів зумовлює цю патологію?

Катехоламіни

АКТГ

Тиреоїдні гормони

Глюкокортикоїди

Альдостерон

1293 / 6307
Хвора 56-ти років тривалий час хворіє на тиреотоксикоз. Який тип гіпоксії може розвинутися у цієї хворої?

Циркуляторна

Змішана

Тканинна

Дихальна

Гемічна

1294 / 6307
Для підвищення спортивних результатів чоловіку рекомендували застосовувати препарат, що містить карнітин. Який процес у найбільшому ступені активізується карнітином?

Тканинне дихання

Синтез стероїдних гормонів

Синтез кетонових тіл

Синтез ліпідів

Транспорт жирних кислот до міто-хондрій

1295 / 6307
У хворої симптоми запального процесу сечостатевих шляхів. У мазку із слизової оболонки піхви виявлено великі одноклітинні організми грушоподібної форми з загостреним шипом на задньому кінці тіла, великим ядром та ундулюючою мембраною. Які найпростіші знайдені в мазку?

Trichomonas vaginalis

Lamblia intestinalis

Trypanosoma gambiense

Trichomonas hominis

Trichomonas buccalis

1296 / 6307
У хворого виявлено екстрасистолію. На ЕКГ при екстрасистолічному скороченні відсутній зубець P, комплекс QRS деформований, є повна компенсаторна пауза. Які це екстрасистоли?

Передсердно-шлуночкові

Шлуночкові

Синусні

Передсердні

1297 / 6307
У хворого на рак спинки язика виникла сильна кровотеча внаслідок ураження пухлиною дорзальної артерії язика. Яку судину повинен перев’язати лікар для зупинки кровотечі?

Дорзальна артерія язика

Висхідна артерія глотки

Лицева артерія

Глибока артерія язика

Язикова артерія

1298 / 6307
В експерименті певним чином зруйнована значна кількість стовбурових клітин червоного кісткового мозку. Оновлення яких популяцій клітин у складі пухкої сполучної тканини буде загальмовано?

Макрофаги

Перицити

Фібробласти

Ліпоцити

Пігментні клітини

1299 / 6307
Під час гістологічного дослідження тимуса чоловіка 40-ка років, визначено зменшення частки паренхіматозних елементів залози, збільшення частки жирової та пухкої сполучної тканини, збагачення її тимусними тільцями при незмінній загальній масі органу. Як зветься таке явище?

Дистрофія

Гіпотрофія

Вікова інволюція

Акцідентальна інволюція

Атрофія

1300 / 6307
На слизовій оболонці правого піднебінного мигдалика спостерігається безболісна виразка з гладеньким лакованим дном та рівними хрящоподібної консистенції краями. Мікроскопічно: запальний інфільтрат, що складається з лімфоцитів, плазмоцитів, невеликої кількості нейтрофілів та епітеліоїдних клітин, та наявність ендо- та периваскуліту. Про яке захворювання йдеться?

Виразково-некротична ангіна Венсана

Туберкульоз

Дифтерія зіву

Актиномікоз

Сифіліс

1301 / 6307
У бактеріологічній лабораторії проводиться дослідження м’ясних консервів на вміст ботулінічного токсину. Для цього дослідній групі мишей ввели екстракт із досліджуваного матеріалу та антитоксичну протиботулінічну сироватку типів А, В, Е; контрольній групі мишей ввели екстракт без протиботулінічної сироватки. Яку серологічну реакцію було використано?

Нейтралізації

Преципітації

Опсонофагоцитарна

Зв’язування комплементу

Подвійної імунної дифузії

1302 / 6307
Для запобігання післяопераційної кровотечі 6-ти річній дитині рекомендовано приймати вікасол, який є синтетичним аналогом вітаміну K. Вкажіть, які посттрансляційні зміни факторів згортання крові активуються під впливом вікасолу?

Карбоксилювання глутамінової кислоти

Полімеризація

Глікозилювання

Фосфорилювання радикалів серину

Частковий протеоліз

1303 / 6307
У чоловіка 33-х років внаслідок спинномозкової травми порушена больова та температурна чутливість, що обумовлено пошкодженням таких висхідних шляхів:

Задній спиномозочковий

Медіальний спинокортикальний

Спиноталамічні

Латеральний спинокортикальний

Передній спиномозочковий

1304 / 6307
У хворого для обробки опікової поверхні шкіри було використано препарат, антисептичні властивості якого забезпечуються вільним киснем, що відщеплюється у присутності органічних речовин. Який лікарський засіб був використаний?

Фурацилін

Натрію гідрокарбонат

Калію перманганат

Кислота борна

Хлоргексидин

1305 / 6307
У дитини 2-х років після грипу з’явилися скарги на біль у вусі. Лікар виявив зниження слуху та запалення середнього вуха. Яким шляхом інфекція потрапила до середнього вуха?

Через canalis caroticus

Через atrium mastoideum

Через слухову трубу

Через foramen jugularis

Через canalis nasolacrimalis

1306 / 6307
Хворому 50-ти років з хронічною серцевою недостатністю і тахіаритмією призначили кардіотонічний препарат. Який з препаратів призначили хворому?

Дофамін

Дигоксин

Аміодарон

Мілдронат

Добутамін

1307 / 6307
При мікроскопічному дослідженні оперативно видаленого апендикса відзначався набряк, дифузна нейтрофільна інфільтрація стінки з некрозом та наявністю дефекту слизової оболонки з ураженням її м’язової пластинки. Яка форма апендициту розвинулася в хворого?

Поверхнева

Флегмонозна

Апостематозна

Гангренозна

Флегмонозно-виразкова

1308 / 6307
У жінки 39-ти років під час операції були видалені збільшена у розмірах маткова труба та частина яєчника з великою кістою. При гістологічному дослідженні стінки труби виявлені децидуальні клітини, ворсини хоріону. Який найбільш вірогідний діагноз було встановлено при дослідженні маточної труби?

Плацентарний поліп

Літопедіон

Трубна вагітність

Хоріонкарцинома

'Паперовий' плід

1309 / 6307
У хлопчика 4-х років після перенесеного важкого вірусного гепатиту мають місце блювання, втрата свідомості, судоми. У крові - гіперамоніємія. Порушення якого біохімічного процесу викликало патологічний стан хворого?

Порушення знешкодження аміаку в печінці

Порушення знешкодження біогенних амінів

Пригнічення ферментів трансаміну-вання

Активація декарбоксилування амінокислот

Посилення гниття білків у кишечнику

1310 / 6307
У хворого виявлено гіперкаліємію та гіпонатріємію. Знижена секреція якого гормону може спричинити такі зміни?

Вазопресин

Альдостерон

Натрійуретичний

Паратгормон

Кортизол

1311 / 6307
При непрямому гістогенезі кісткової тканини трубчастих кісток між епіфізарним та діафізарним центрами окостеніння утворюється пластинка, що в подальшому забезпечує ріст кісток у довжину. Як називається ця структура?

Остеон

Кісткова пластинка

Кісткова манжетка

Шар внутрішніх генеральних пластинок

Метафізарна пластинка

1312 / 6307
Хвора 40-ка років надійшла до інфекційного відділення лікарні з високою температурою тіла. Об’єктивно: виражені менінгеальні симптоми. Проведено спинномозкову пункцію. Яке анатомічне утворення було пропунктовано?

Cavum trigeminale

Spatium subdurale

Cisterna cerebellomedullaris posterior

Spatium subarachnoideum

Spatium epidurale

1313 / 6307
Після резекції середньої третини облітерованої тромбом стегнової артерії нижня кінцівка кровопостачається за рахунок обхідних анастомозів. Назвіть артерію, яка має основне значення у відновленні кровотоку:

Глибока зовнішня соромітня артерія

Глибока стегнова артерія

Низхідна колінна артерія

Поверхнева огинальна артерія клубової кістки

Поверхнева надчеревна артерія

1314 / 6307
У хворого не розгинається колінний суглоб, відсутній колінний рефлекс, порушена чутливість шкіри на передній поверхні стегна. Який нерв уражений?

Верхній сідничний

Великий малогомілковий

Нижній сідничний

Затульний

Стегновий

1315 / 6307
На електронній мікрофотографії фрагменту нирки представлена приносна артеріола, у якій під ендотелієм видно великі клітини, що містять секреторні гранули. Назвіть даний вид клітин:

Мезангіальні

Гладеньком’язові

Юкстагломерулярні

Інтерстиційні

Юкставаскулярні

1316 / 6307
Для вирішення питання ретроспективної діагностики перенесеної бактеріальної дизентерії було призначено серологічне дослідження сироватки крові з метою встановлення титру антитіл до шигел. Яку з перелічених реакцій доцільно використати для цього?

Бактеріоліз

Преципітація

Гемоліз

Пасивна гемаглютинація

Зв’язування комплементу

1317 / 6307
У чоловіка 43-х років з видаленою ниркою були виявлені симптоми анемії. Що зумовило появу цих симптомів?

Нестача заліза

Підвищене руйнування еритроцитів

Нестача фолієвої кислоти

Нестача вітаміну B12

Зниження синтезу еритропоетинів

1318 / 6307
Хворий 50-ти років потрапив до лікарні зі скаргами на біль за грудниною, ядуху при фізичних навантаженнях. Після ангіографії виявлені патологічні зміни в задній міжшлуночковій гілці правої вінцевої артерії. Які ділянки серця уражені?

Правий передсердно-шлуночковий клапан

Ліве передсердя

Задня стінка правого і лівого шлуночків

Передня стінка правого і лівого шлуночків

Праве передсердя

1319 / 6307
Після проведення туберкулінової проби (проба Манту) у дитини через 48 годин на місці ведення туберкуліну утворилася папула до 10 мм у діаметрі. Який механізм гіперчутливості лежить в основі розвитку вказаних змін?

Імунокомплексна цитотоксичність

Клітинна цитотоксичність

Антитілозалежна цитотоксичність

Анафілаксія

Гранулематоз

1320 / 6307
У новонародженого хлопчика під час огляду зовнішніх статевих органів виявлена розщілина сечівника, яка відкривається на нижній поверхні статевого члена. Про яку аномалію йдеться?

Крипторхізм

Гермафродитизм

Гіпоспадія

Епіспадія

Монорхізм

1321 / 6307
У людини порушено всмоктування продуктів гідролізу жирів. Причиною цього може бути дефіцит у порожнині тонкої кишки наступних компонентів:

Жиророзчинні вітаміни

Жовчні кислоти

Жовчні пігменти

Іони натрію

Ліполітичні ферменти

1322 / 6307
У мешканців територій з холодним кліматом в крові збільшений вміст гормону, що має пристосувальне терморегуляторне значення. Про який гормон йдеться?

Соматотропін

Кортизол

Глюкагон

Тироксин

Інсулін

1323 / 6307
У результаті порушення техніки безпеки відбулося отруєння сулемою (хлористою ртуттю). Через 2 дні добовий діурез склав 620 мл. У хворого з’явилися головний біль, блювання, судоми, задишка, у легенях - вологі хрипи. Яка патологія має місце?

Пієлонефрит

Гломерулонефрит

Уремічна кома

Хронічна ниркова недостатність

Гостра ниркова недостатність

1324 / 6307
У новонародженої дитини з пілоростенозом часте блювання, що супроводжується апатією, слабкістю, підвищенням тонусу м’язів, інколи судомами. Яка форма порушення кислотно-основного стану розвинулася в хворого?

Видільний ацидоз

Газовий алкалоз

Негазовий алкалоз

Метаболічний ацидоз

Газовий ацидоз

1325 / 6307
При декарбоксилуванні глутамату в ЦНС утворюється медіатор гальмування. Назвіть його:

Глутатіон

Аспарагін

Серотонін

ГАМК

Гістамін

1326 / 6307
Під час катаболізму гістидину утворюється біогенний амін, що має потужну судинорозширюючу дію. Назвіть його:

Норадреналін

ДОФА

Серотонін

Дофамін

Гістамін

1327 / 6307
При утилізації арахідонової кислоти за циклооксигеназним шляхом утворюються біологічно активні речовини. Вкажіть їх:

Інсуліноподібні фактори росту

Біогенні аміни

Тироксин

Соматомедини

Простагландини

1328 / 6307
Внаслідок руйнування певних структур стовбуру мозку тварина втратила орієнтувальні рефлекси. Які структури було зруйновано?

Вестибулярні ядра

Чотиригорбкова структура

Червоні ядра

Чорна речовина

Медіальні ядра ретикулярної формації

1329 / 6307
У людини осмотичний тиск плазми крові 350 мосмоль/л (норма - 300 мосмоль/л). Це спричинить, перш за все, посилену секрецію такого гормону:

Альдостерон

Кортизол

Вазопресин

Адренокортикотропін

Натрійуретичний

1330 / 6307
Молодий чоловік звернувся до лікарні зі скаргами на порушення сечовипускання. Під час обстеження зовнішніх статевих органів виявлено, що сечівник розщеплений зверху і сеча витікає через цей отвір. Який вид аномалії розвитку зовнішніх статевих органів спостерігається у цьому випадку?

Епіспадія

Парафімоз

Гермафродитизм

Гіпоспадія

Фімоз

1331 / 6307
Хворий скаржиться на біль у ділянці печінки. При дослідженні жовчі, отриманої під час дуоденального зондування, виявлені жовтуваті яйця овальної форми, звужені до полюсів, на кінці одного полюсу знаходиться кришечка. Розміри цих яєць найменші серед яєць усіх гельмінтів. Який найбільш вірогідний діагноз?

Дифілоботріоз

Ехінококоз

Теніарінхоз

Теніоз

Опісторхоз

1332 / 6307
У хворого з дизентерією при колоноскопії виявлено, що слизова оболонка товстої кишки гіперемована, набрякла, її поверхня вкрита сіро-зеленими плівками. Назвіть морфологічну форму дизентерійного коліту:

Гнійний

Виразковий

Фібринозний

Катаральний

Некротичний

1333 / 6307
У хворого через добу після апендектомії у крові визначається нейтрофільний лейкоцитоз із регенеративним зсувом. Який найбільш вірогідний механізм розвитку лейкоцитозу в даному випадку?

Перерозподіл лейкоцитів у організмі

Уповільнення руйнування лейкоцитів

Посилення лейкопоезу

Посилення лейкопоезу та уповільнення міграції лейкоцитів у тканини

Уповільнення міграції лейкоцитів у тканини

1334 / 6307
У хірурга після проведення тривалої операції підвищився артеріальний тиск до 140/110 мм рт.ст. Які зміни гуморальної регуляції можуть бути причиною підвищення артеріального тиску в даному випадку?

Активація симпатоадреналової системи

Активація калікреїн-кінінової системи

Активація ренін-ангіотензинової системи

Активація утворення і виділення альдостерону

Гальмування симпатоадреналової системи

1335 / 6307
Хворому внутрішньовенно ввели гіпертонічний розчин глюкози. Це підсилить рух води:

З капілярів до міжклітинної рідини

Змін руху води не буде

З міжклітинної рідини до капілярів

З міжклітинної рідини до клітин

З клітин до міжклітинної рідини

1336 / 6307
У чоловіка 36-ти років черепно-мозкова травма. Об’єктивно: дихання слабке, пульс ниткоподібний, рефлекси відсутні. Який шлях введення пірацетама найбільш доцільний у даному випадку?

Внутрішньовенний

Підшкірний

Пероральний

Інгаляційний

Ректальний

1337 / 6307
Дитина скаржиться на загальну слабкість, відсутність апетиту, неспокійний сон, свербіж у періанальній ділянці. Встановлено діагноз: ентеробіоз. Для уточнення діагнозу слід провести:

Імунодіагностика

Аналіз дуоденального вмісту

Рентгеноскопічне дослідження

Зіскоб з періанальних складок

Біопсія м’язової тканини

1338 / 6307
У хворого опікова хвороба ускладнилася ДВЗ-синдромом. Яку стадію ДВЗ-синдрому можна запідозрити, якщо відомо, що кров хворого згортається менше ніж за 3 хвилини?

Гіпокоагуляції

Термінальна

Фібриноліз

Перехідна

Гіперкоагуляції

1339 / 6307
На практичному занятті студенти вивчали забарвлений мазок крові миші з бактеріями, фагоцитованими лейкоцитами. Яка органела клітини завершує перетравлення цих бактерій?

Лізосоми

Апарат Гольджі

Рибосоми

Гранулярна ендоплазматична сітка

Мітохондрії

1340 / 6307
Жінка 55-ти років звернулася зі скаргами на тривалі циклічні маткові кровотечі протягом року, слабкість, запаморочення. Об’єктивно: блідість шкіри. У крові: Hb- 70 г/л, ер.- 3,2•1012/л, КП- 0,6, лейк.- 6,0•109 /л, ретикулоцити -1%; гіпохромія еритроцитів. Яка анемія у хворої?

Гемолітична

Хронічна постгеморагічна

Залізодефіцитна

В12-фолієводефіцитна

Апластична

1341 / 6307
У пацієнта за даними аудіометрії виявлено порушення сприйняття звуків середньої частоти. Причиною цього може бути пошкодження:

Латеральних колінчастих тіл

Спірального ганглія

Чотиригорбикової структури

Кохлеарних ядер

Середньої частини завитки

1342 / 6307
Госпіталізовано хворого з діагнозом карциноїд кишечнику. Аналіз виявив підвищену продукцію серотоніну. Відомо, що ця речовина утворюється з амінокислоти триптофану. Який біохімічний механізм лежить в основі даного процесу?

Мікросомальне окиснення

Декарбоксилювання

Трансамінування

Дезамінування

Утворення парних сполук

1343 / 6307
-х років скаржиться на слабкість, підвищення температури до 38 — 400 143. C. Об’єктивно: печінка і селезінка збільшені. У крові: Hb- 100 г/л, ер.- 2,9•1012/л, лейк.- 4,4•109 /л, тромб.-48 • 109 /л, нейтрофіли сегментоядерні -17%, лімфоцити - 15%, бластні клітини - 68%. Всі цитохімічні реакції негативні. Дайте гематологічний висновок:

Гострий лімфобластний лейкоз

Гострий мієлобластний лейкоз

Недиференційований лейкоз

Гострий еритромієлоз

Хронічний мієлолейкоз

1344 / 6307
Хворий на бронхіальну астму не повідомив лікаря, що в нього бувають напади стенокардії. Лікар призначив препарат, після прийому якого напади бронхіальної астми стали рідшими, однак почастішали напади стенокардії. Який препарат був призначений?

Фенотерол

Кромолін-натрій

Сальбутамол

Еуфілін

Ізадрин

1345 / 6307
Хворий на хронічну серцеву недостатність протягом декількох місяців приймав в амбулаторних умовах дигоксин. На певному етапі лікування в нього виникли симптоми передозування препарату. Яке явище лежить в основі розвитку цього ускладнення?

Сенсибілізація

Матеріальна кумуляція

Тахіфілаксія

Звикання

Функціональна кумуляція

1346 / 6307
Хворий, що лікувався з приводу неврозу сибазоном, скаржиться на зубний біль. Лікар призначив йому знеболювальний засіб у дозі, яка менша за середню терапевтичну. Яке явище взяв до уваги лікар, зменшуючи дозу препарату?

Лікарська залежність

Толерантність

Сумація

Потенціювання

Кумуляція

1347 / 6307
У крові дитини виявлено високий вміст галактози, концентрація глюкози понижена. Спостерігаються катаракта, розумова відсталість, розвивається жирове переродження печінки. Яке захворювання має місце?

Лактоземія

Фруктоземія

Цукровий діабет

Стероїдний діабет

Галактоземія

1348 / 6307
З урахуванням клінічної картини хворому призначено піридоксальфосфат. Для корекції яких процесів рекомендований цей препарат?

Синтез білку

Трансамінування і декарбоксилювання амінокислот

Дезамінування пуринових нуклеотидів

Окисне декарбоксилювання кетокислот

Синтез пуринових та піримідинових основ

1349 / 6307
Під час операції у пацієнта було видалено частину легені, яка вентилюється бронхом третього порядку, що супроводжується гілками легеневої артерії та інших судин. Яка частина легені була видалена?

Нижня частка

Середня частка

Сегмент легені

Верхня частка

Легенева часточка

1350 / 6307
У хворого з клінічними ознаками імунодефіциту проведено імунологічні дослідження. Виявлено значне зниження кількості клітин, що утворюють розетки з еритроцитами барана. Який висновок слід зробити на основі даних аналізу?

Недостатність клітин-ефекторів гуморального імунітету

Зниження рівня системи комплементу

Зниження рівня Б-лімфоцитів

Зниження рівня натуральних кілерів (УХ-клітин)

Зниження рівня T-лімфоцитів

1351 / 6307
У жінки з III (Б), Rh- групою крові народилась дитина з II (A) групою крові. У дитини діагностовано гемолітичну хворобу новонародженого внаслідок резус-конфлікту. Яка група крові за системою АБо та резус-належність можливі у батька?

III (Б), Rh+

I (O), Rh+

II (A), Rh+

I (O), Rh

II (A), Rh

1352 / 6307
У жінки 45-ти років хвороба Іценко-Кушінга - стероїдний діабет. При біохімічному обстеженні: гіперглікемія, гіпохлоремія. Який з перерахованих нижче процесів активується у жінки в першу чергу?

Транспорт глюкози в клітину

Глікогеноліз

Глюконеогенез

Гліколіз

Реабсорбція глюкози

1353 / 6307
На гістологічному препараті видно судину, стінка якої складається з ендотелію, базальної мембрани та пухкої сполучної тканини. Назвіть тип судини:

Лімфокапіляр

Вена безм’язового типу

Артерія

Вена м’язового типу

Гемокапіляр

1354 / 6307
При розтині тіла померлого чоловіка 48-ми років, у ділянці 1-го сегменту правої легені виявлено округлий утвір діаметром 5 см з чіткими контурами, оточений тонким прошарком сполучної тканини, виповнений білими крихкими масами. Діагностуйте форму вторинного туберкульозу:

Гострий кавернозний туберкульоз

Туберкулома

Казеозна пневмонія

Фіброзно-кавернозний туберкульоз

Гострий вогнищевий туберкульоз

1355 / 6307
У хворого на гострий міокардит з’явилися клінічні ознаки кардіогенного шоку. Який із вказаних нижче патогенетичних механізмів є провідним в розвитку шоку?

Зниження діастолічного притоку до серця

Депонування крові в органах

Збільшення периферичного опору судин

Зниження насосної функції серця

Зниження судинного тонусу

1356 / 6307
У хворого з гострою нирковою недостатністю на 6-й день проведення терапевтичних заходів виникла поліурія. Чим зумовлене зростання діурезу на початку поліуричної стадії гострої ниркової недостатності?

Відновлення фільтрації в нефронах

Збільшенням об’єму циркулюючої крові

Збільшенням натрійуретичного фактора

Зменшенням альдостерону в плазмі

Зменшенням вазопресину в плазмі

1357 / 6307
При тривалому лікуванні голодуванням у пацієнта зменшилося співвідношення альбумінів і глобулінів у плазмі крові. Що з наведеного буде наслідком цих змін?

Зниження ШЗЕ

Зниження гематокритного показника

Збільшення ШЗЕ

Збільшення гематокритного показника

Гіперкоагуляція

1358 / 6307
Експериментальній тварині, після попередньої сенсибілізації, підшкірно введено дозу антигену. У місці ін’єкції розвинулось фібринозне запалення з альтерацією стінок судин, основної речовини та волокнистих структур сполучної тканини у вигляді мукоїдного та фібриноїдного набухання і некрозу. Яка імунологічна реакція має місце?

Нормергічна реакція

Реакція трансплантаційного імунітету

Гранульоматоз

Гіперчутливість негайного типу

Гіперчутливість сповільненого типу

1359 / 6307
У хворого спостерігається порушення зору - гемералопія ('куряча сліпота'). Який вітамінний препарат треба вживати хворому, щоб відновити зір?

Піридоксин

Токоферолу ацетат

Вікасол

Ретинолу ацетат

Тіаміну хлорид

1360 / 6307
До клініки госпіталізована дитина 1-го року з ознаками ураження м’язів. Після обстеження виявлений дефіцит карнітину в м’язах. Біохімічною основою цієї патології є порушення процесу:

Утилізації молочної кислоти

Транспорту жирних кислот до міто-хондрій

Регуляції рівня Ca2+ в мітохондріях

Субстратного фосфорилювання

Синтезу актину та міозину

1361 / 6307
У хворого важка післяопераційна псевдомонадна інфекція. Який з перелічених антибіотиків показаний хворому?

Доксициклін

Бензилпеніцилін

Цефазолін

Еритроміцин

Амікацину сульфат

1362 / 6307
У дитини 9-ти місяців спостерігається запізніле прорізування зубів, порушення порядку прорізування. Конфігурація верхньої щелепи - у горизонтальному напрямку ('високе піднебіння'); мікроскопічно - у зубах нерівномірна мінералізація емалі, зморщені емалеві призми, деякі з них вакуолізовані. Розширення зони предентину; зустрічаються поодинокі дентиклі. Яке захворювання у дитини?

Пізній рахіт

Ранній рахіт

Гіпервітаміноз D

Остеомаляція

Подагра

1363 / 6307
Під час обстеження у хворого виявлено абсцес крило-піднебінної ямки. Куди може розповсюдитись інфекція при несвоєчасному наданні медичної допомоги?

Орбіта

Лобова пазуха

Міжкрилоподібний простір

Барабанна порожнина

Підапоневротичний скроневий простір

1364 / 6307
При мікроскопічному дослідженні нирок померлої від ниркової недостатності жінки 36-ти років, в клубочках виявлено проліферацію нефротелію капсули, подоцитів та макрофагів з утворенням 'півмісяців', некроз капілярних петель, фібринові тромби в їх просвітах, а також склероз та гіаліноз клубочків, атрофію канальців та фіброз строми нирок. Який з перелічених діагнозів найбільш вірогідний?

Хронічний гломерулонефрит

Гострий гломерулонефрит

Фокальний сегментарний склероз

Мембранозна нефропатія

Підгострий гломерулонефрит

1365 / 6307
На судово-медичній експертизі знаходиться тіло чоловіка 58-ми років, який тривалий час зловживав алкоголем. Помер вдома. Макроскопічно: права легеня щільна і збільшена в розмірах, тканина на розрізі сіруватого кольору, однорідна, плевра вкрита сіруватими плівчастими нашаруваннями. Мікроскопічно - порожнини альвеол містять нитки фібрину, гемолізовані еритроцити. Діагностуйте захворювання:

Первинний туберкульоз легень

Казеозна пневмонія

Крупозна пневмонія

Інтерстиційна пневмонія

Вогнищева пневмонія

1366 / 6307
Через кілька днів після споживання копченої свинини у хворого з’явилися набряки обличчя та повік, шлунково-кишкові розлади, різке підвищення температури, м’язовий біль. В аналізі крові різко виражена еозинофілія. Яким гельмінтом могла заразитися людина через свинину?

Гострик

Анкілостома

Трихінела

Аскарида

Волосоголовець

1367 / 6307
У хворого після видалення зуба з’явився стійкий біль за грудниною. Після сублінгвального вживання антиангінального засобу біль за грудниною зник, але хворий поскаржився на головний біль та запаморочення. Для якого препарату притаманні ці властивості?

Метопролол

Валідол

Нітрогліцерин

Верапаміл

Анаприлін

1368 / 6307
До щелепно-лицьового відділення надійшов хворий з переломом нижньої щелепи. Було вирішено з’єднання кісток провести хірургічним методом під наркозом. Після внутрішньовенного введення міорелаксанту спостерігались короткочасні фібрилярні скорочення м’язів обличчя хворого. Який міорелаксант було застосовано?

Меліктин

Тубокурарина хлорид

Пипекуроній бромід

Діазепам

Дитилін

1369 / 6307
При розтині тіла померлого 56-ти років у правій скроневій частці головного мозку знайдено великий осередок розм’якшеної сірої речовини кашицеподібної консистенції, блідо-сірого кольору. В артеріях основи мозку чисельні білувато-жовті потовщення інтими, які різко звужують просвіт. Який найбільш вірогідний діагноз?

Крововилив

Набряк мозку

Ішемічний інсульт

Абсцес мозку

Геморагічний інсульт

1370 / 6307
При нестачі вітаміну A у людини відбувається порушення сутінкового зору. Вкажіть клітини, яким належить означена фоторецепторна функція:

Гангліонарні нервові клітини

Паличкові нейросенсорні клітини

Колбочкові нейросенсорні клітини

Біполярні нейрони

Горизонтальні нейроцити

1371 / 6307
У хворого 65-ти років під час неврологічного обстеження виявлено крововилив у межах верхньої скроневої звивини. У зоні кровопостачання якої артерії воно знаходиться?

Передня мозкова артерія

Задня мозкова артерія

Передня сполучна артерія

Основна артерія

Середня мозкова артерія

1372 / 6307
Чоловік 70-ти років під час гоління зрізав гнійник в ділянці соскоподібного відростка. Через 2 доби він був доставлений до лікарні з дiагнозом запалення оболонок головного мозку. Яким шляхом інфекція потрапила в порожнину черепа?

V.v.labyrinthi

V.emissariae mastoideae

V.v.tympanicae

V.v.auriculares

V.facialis

1373 / 6307
У жінки 22-х років виявлєні збіль-шєні лімфатичні вузли. Гістологічно: у лімфатичному вузлі наявні лімфоцити, гістіоцити, ретикулярні клітини, малі та великі клітини Ходжкіна, багатоядерні клітини Березовського-Штернберга, поодинокі осередки казе-озного некрозу. Для якого захворювання характерні такі зміни?

Метастаз рака легень

Лімфогранулематоз

Хронічний лейкоз

Лімфосаркома

Гострий лейкоз

1374 / 6307
У хворого з підозрою на одне з протозойних захворювань досліджено пунктат лімфатичного вузла. В препараті, забарвленому за Романовським-Гімзою, виявлені тільця півмісяцевої форми із загостреним кінцем, блакитною цитоплазмою, ядром червоного кольору. Яких найпростіших виявлено в мазках?

Малярійні плазмодії

Токсоплазми

Вісцеротропні лейшманії

Дерматотропні лейшманії

Трипаносоми

1375 / 6307
Під час статевого дозрівання клітини чоловічих статевих залоз починають продукувати чоловічий статевий гормон тестостерон, який обумовлює появу вторинних статевих ознак. Які клітини чоловічих статевих залоз продукують цей гормон?

Сустентоцити

Підтримуючі клітини

Клітини Лейдіга

Клітини Сертолі

Сперматозоїди

1376 / 6307
Ьольована клітина серця людини автоматично генерує імпульси збудження з частотою 60 разів за хвилину. З якої структури серця отримано цю клітину?

Атріовентрикулярний вузол

Шлуночок

Передсердя

Пучок Гіса

Синоатріальний вузол

1377 / 6307
При обстежені пацієнта встановили сильний, врівноважений, інертний тип вищої нервової діяльності за Павловим. Якому темпераменту за Гіппократом відповідає пацієнт?

Меланхолік

Флегматик

Сангвінік

Холерик

1378 / 6307
У хворого крововилив у задню центральну звивину. До порушення якого виду чутливості з протилежного боку це призведе?

Шкірна та пропріоцептивна

Зорова

Слухова

Слухова і зорова

Нюхова та смакова

1379 / 6307
У пункційному біоптаті печінки виявлена дистрофія гепатоцитів з некрозами, а також склероз з порушеннями балкової та часточкової будови, з утворенням несправжніх часточок і регенераторних вузлів. Оберіть найбільш вірогідний діагноз:

Прогресуючий масивний некроз печінки

Гострий гепатит

Хронічний гепатит

Хронічний гепатоз

Цироз печінки

1380 / 6307
У 60-ти річного пацієнта була виявлена гіперглікемія та глюкозурія. Для лікування цього хворого лікар призначив препарат для приймання всередину. Який це препарат?

Корглікон

Окситоцин

Глібенкламід

Фуросемід

Панкреатин

1381 / 6307
У людини, що виконувала важку фізичну роботу в умовах підвищеної температури навколишнього середовища, змінилася кількість білків плазми крові. Що саме має місце у даному випадку?

Диспротеїнемія

Відносна гіперпротеїнемія

Абсолютна гіпопротеїнемія

Парапротеїнемія

Абсолютна гіпепротеїнемія

1382 / 6307
У піддослідного щура з паралічем кінцівки спостерігається зникнення сухожилкових і шкірних рефлексів, зниження м’язового тонусу, при цьому зберігається здатність м’язів ураженої кінцівки відповідати збудженням на пряму дію постійного струму. Який тип паралічу відзначається у тварини?

В’ялий периферичний

Спастичний центральний

Екстрапірамідний

Спастичний периферичний

В’ялий центральний

1383 / 6307
Ліквідатора аварії на АЕС, який отримав опромінення, стало турбувати блювання, яке виникає несподівано. Який препарат слід призначити хворому?

Резерпін

Де-нол

Атропін

Метоклопрамід

Аерон

1384 / 6307
У хворого з гострим циститом під час дослідження сечі виявили лейкоцити та багато грамнегативних паличок. При посіві виросли колонії слизового характеру, які утворювали зелений розчинний пігмент. Який мікроорганізм, найбільш вірогідно, є причиною захворювання?

Proteus mirabilis

Salmonella enteritidis

Klebsiella pneumoniae

Pseudomonas aeruginosa

Escherihia coli

1385 / 6307
Чоловік 45-ти років звернувся до лікаря з приводу бляшкоподібного утвору на шиї. В біоптаті шкіри гістологічно виявлено пухлинні клітини, розташовані гніздами, які мають круглу та овальну форму з вузьким ободком базофільної цитоплазми та схожі на клітини базального шару епідермісу. Як називається пухлина в пацієнта?

Сирінгоаденома

Гідраденома

Епідермальний рак

Базаліома

Трихоепітеліома

1386 / 6307
У чоловіка з’явилися біль, набряк та почервоніння шкіри у передньо-верхній частині стегна та великого пальця стопи. Які лімфатичні вузли нижньої кінцівки відреагували на запальний процес?

Поверхневі повздовжні

Загальні повздовжні

Глибокі пахвинні

Внутрішні повздовжні

Поверхневі пахвинні

1387 / 6307
До лабораторії надійшов матеріал із рани хворого. Попередній діагноз - газова гангрена. Яким мікробіологічним методом можна встановити видову приналежність збудника?

РІА

Алергічний

Бактеріологічний

Серологічний

Бактеріоскопічний

1388 / 6307
У культурі клітин, отриманих від хворого з лізосомною патологією, виявлено накопичення значної кількості ліпідів у лізосомах. При якому з перелічених захворювань має місце це порушення?

Галактоземія

Хвороба Тея-Сакса

Подагра

Хвороба Вільсона-Коновалова

Фенілкетонурія

1389 / 6307
У чоловіка після черепно-мозкової травми виникла правобічна косоокість, що сходиться. Пошкодження якого черепно-мозкового нерва призвело до таких наслідків?

n.oculomotorius

n.facialis

n.trochlearis

n.abducens

n.trigeminus

1390 / 6307
При проникаючому пораненні передньої черевної стінки раневий канал пройшов над малою кривиною шлунка. Яке утворення очеревини найвірогідніше пошкоджене?

Ligamentum gastrocolicum

Ligamentum triangulare sinistrum

Ligamentum hepatorenale

Ligamentum hepatogastricum

Ligamentum hepatoduoduodenale

1391 / 6307
У кроля перерізали нерв, що іннервує праве вухо, і видалили правий верхній шийний симпатичний вузол. Одразу після операції провели вимірювання температури шкіри вух. Виявилося, що температура шкіри вуха кролика на боці денервації на 1,50 C вища, ніж на протилежному інтактному боці. Що з наведеного є найбільш вірогідною причиною вказаних явищ?

Реактивна артеріальна гіперемія

Артеріальна гіперемія нейропаралітичного типу

Артеріальна гіперемія нейротонічного типу

Фізіологічна артеріальна гіперемія

Артеріальна гіперемія, обумовлена метаболічними факторами

1392 / 6307
У чоловіка 63-х років захворювання почалось гостро з явищ гострого трахеїту і бронхіту, до яких приєдналася бронхопневмонія. На 10-ту добу хворий помер від легенево-серцевої недостатності. На розтині виявлений фібринозно-геморагічний ларинготра-хеобронхіт; легені збільшені в об’ємі, на розрізі мають 'пістрявий'вигляд за рахунок чергування ділянок бронхопневмонії, крововиливів у легеневу паренхіму, гострих абсцесів і ателектазів. У внутрішніх органах - дисциркуляторні та дистрофічні зміни. Який діагноз найбільш вірогідний?

Аденовірусна інфекція

Парагрип

Респіраторно-синцитіальна інфекція

Грип, важка форма

Грип середньої важкості

1393 / 6307
На аутопсії померлого від грипу чоловіка відзначено, що серце дещо збільшене у розмірах, пастозно, на розрізі міокард тьмяний, з крапом. Мікроскопічно: у міокарді на всьому протязі ознаки паренхіматозної жирової і гідропічної дистрофії, строма набрякла, з незначною макрофагально-лімфоцитарною інфільтрацією, судини повнокровні; периваскулярно - петехіальні крововиливи. Який вид міокардиту розвинувся в даному випадку?

Гнійний

Проміжний проліферативний

Серозний вогнищевий

Гранулематозний

Серозний дифузний

1394 / 6307
У хворого на хронічну серцеву недостатність, незважаючи на терапію кардіотонічними засобами і тіазидовим діуретиком, зберігаються набряки і виникла загроза асциту. Який препарат слід призначити для підсилення діуретичного ефекту застосованих ліків?

Клопамід

Спіронолактон

Фуросемід

Амілорид

Манітол

1395 / 6307
У хворого на колагеноз після тривалого прийому преднізолону з’явився спастичний біль скелетних м’язів внаслідок розвитку гипокаліємії. Який препарат треба використати для корекції обміну калію?

Діазепам

Тирокальцитонін

Панангін

Дитилін

Но-шпа

1396 / 6307
Хлопчик 7-ми років. Об’єктивно: на гіперемованому фоні шкіри дрібно-крапчастий яскраво-рожевий висип на лобі, шиї, внизу живота, підколінних ямках; носогубний трикутник блідий. В ротоглотці - відмежована яскраво-червона гіперемія; мигдалики набряклі, пухкі, в лакунах є гній, язик малиновий. Шийні лімфовузли збільшені, щільні, болючі. Який найбільш вірогідний діагноз?

Кашлюк

Скарлатина

Краснуха

Інфекційний мононуклеоз

Дифтерія

1397 / 6307
Пацієнт скаржиться на сухість шкіри голови, свербіж, ламкість і випадіння волосся. При обстеженні встановлений діагноз: себорея. З порушенням діяльності яких клітин це пов’язано?

Меланоцити

Адипоцити

Клітини сальних залоз

Епітеліоцити

Клітини потових залоз

1398 / 6307
В хірургічному відділенні лікарні виник спалах госпітальної інфекції, що проявлялася в частому нагноєнні післяопераційних ран. При бактеріологічному дослідженні гною був виділений золотистий стафілокок. Яке дослідження треба використати для виявлення джерела цього збудника серед персоналу відділення?

Серологічна ідентифікація

Фаготипування

Мікроскопія

Біохімічна ідентифікація

Визначення чутливості до антибіотиків

1399 / 6307
Внаслідок короткочасного фізичного навантаження у людини рефлекторно зросли частота серцевих скорочень та системний артеріальний тиск. Активація яких рецепторів найбільшою мірою зумовила реалізацію пресорного рефлексу в цій ситуації?

Пропріорецептори працюючих м’язів

Волюморецептори судин

Терморецептори гіпоталамуса

Хеморецептори судин

Барорецептори судин

1400 / 6307
В експерименті подразнюють скелетний м’яз серією електричних імпульсів. Який вид м’язового скорочення буде виникати, якщо кожний наступний імпульс припадає на період розслаблення поодинокого м’язового скорочення?

Асинхронний тетанус

Зубчастий тетанус

Контрактура м’яза

Суцільний тетанус

Серія поодиноких скорочень

1401 / 6307
Після вживання жирної їжі у хворого з’являються нудота та печія, має місце стеаторея. Причиною такого стану може бути:

Порушення синтезу трипсину

Підвищене виділення ліпази

Нестача амілази

Нестача жовчних кислот

Порушення синтезу фосфоліпази

1402 / 6307
У немовляти внаслідок неправильного годування виникла виражена діарея. Одним з основних наслiдкiв діареї є екскреція великої кількості бікарбонату натрію. Яка форма порушення кислотно-лужного балансу має місце в цьому випадку?

Респіраторний алкалоз

Респіраторний ацидоз

Не буде порушень кислотно-лужного балансу

Метаболічний ацидоз

Метаболічний алкалоз

1403 / 6307
У трирічної дитини з підвищеною температурою тіла після прийому аспірину спостерігається посилений гемоліз еритроцитів. Вроджена недостатність якого ферменту могла викликати у дитини гемолітичну анемію?

Глюкозо-6-фосфатдегідрогеназа

Глюкозо-6-фосфатаза

Глікогенфосфорилаза

Гліцеролфосфатдегідрогеназа

γ-глутамілтрансфераза

1404 / 6307
Людина хворіє на цукровий діабет, що супроводжується гіперглікемією натще понад 7,2 ммоль/л. Рівень якого білку плазми крові дозволяє ретроспективно (за попередні 4-8 тижні до обстеження) оцінити рівень глікемії?

С-реактивний білок

Альбумін

Церулоплазмін

Глікозильований гемоглобін

Фібриноген

1405 / 6307
Під час лапаротомії хірург виявив гангренозне ураження низхідної ободової кишки. Тромбоз якої артерії зумовив цей стан?

Верхня брижова

Серединна ободова

Ліва ободова

Права ободова

Клубово-ободова

1406 / 6307
У пацієнта 60-ти років виявлено погіршення сприйняття звуків високої частоти. Порушення стану яких структур слухового аналізатора зумовило ці зміни?

Євстахієва труба

Барабанна перетинка

М’язи середнього вуха

Основна мембрана завитки біля овального віконця

Основна мембрана завитки біля гелікотреми

1407 / 6307
З сироватки крові людини виділили п’ять ізоферментних форм лактатдегідрогенази і вивчили їх властивості. Яка властивість доводить, що виділені ізоферментні форми одного і того ж ферменту?

Однакова молекулярна маса

Тканинна локалізація

Однакова електрофоретична рухливість

Каталізують одну і ту ж реакцію

Однакові фізико-хімічні властивості

1408 / 6307
Було доведено, що молекула незрілої і-РНК (про-і-РНК) містить більше триплетів, чим знайдено амінокислот у синтезованому білку. Це пояснюється тим, що трансляції у нормі передує:

Репарація

Ініціація

Процесінг

Реплікація

Мутація

1409 / 6307
У хворого виявлено зниження вмісту іонів магнію, які потрібні для прикріплення рибосом до гранулярної ендоплазматичної сітки. Відомо, що це призводить до порушення біосинтезу білка. Який саме етап біосинтезу білка буде порушено?

Реплікація

Трансляція

Термінація

Транскрипція

Активація амінокислот

1410 / 6307
Студент старанно конспектує лекцію. Якість конспектування значно погіршилась, коли сусіди стали розмовляти. Який вид гальмування умовних рефлексів є причиною цього?

Згасаюче

Зовнішнє

Диференційоване

Запізніле

Позамежове

1411 / 6307
Хворий з нейродермітом протягом тривалого часу вживав преднізолон. При обстеженні в нього виявили підвищення рівня цукру в крові. Вплив препарату на яку ланку вуглеводного обміну призводить до виникнення цього ускладнення?

Активація розщеплення інсуліну

Пригнічення синтезу глікогену

Активація глікогеногенезу

Посилення всмоктування глюкози в кишечнику

Активація глюконеогенезу

1412 / 6307
Для вивчення локалiзацiї біосинтезу бiлка в клітинах, миші ввели мічені амінокислоти аланін та триптофан. Біля яких органел буде спостерігатися накопичення мічених амінокислот?

Лізосоми

Гладенька ЕПС

Клітинний центр

Рибосоми

Апарат Гольджі

1413 / 6307
У хворого виявили туберкульоз легень. Який антибіотик слід йому призначити поряд з іншими протитуберкульозними засобами?

Левоміцетин

Тетрациклін

Ріфампіцин

Азітроміцин

Кефзол

1414 / 6307
Запалення барабанної порожнини (гнійний середній отит) у хворого ускладнилося запаленням комірок соскоподібного відростка. Через яку стінку барабанної порожнини потрапив гній до комірок?

Передня

Верхня

Задня

Медіальна

Латеральна

1415 / 6307
Для прискорення загоєння рани слизової оболонки в ротовій порожнині хворому призначено препарат, який являє собою термостабільний білок, що міститься у людини в сльозах, слині, грудному молоці матері, а також його можна виявити в свіжознесеному курячому яйці. Відомо, що він являє собою фактор природної резистентності організму і має назву:

Іманін

Лізоцим

Інтерферон

Комплемент

Інтерлейкін

1416 / 6307
Внаслідок стресу у похилої людини підвищився артеріальний тиск. Причиною цього є активація:

Функції кори наднирників

Функції гіпофізу

Функції щитоподібної залози

Парасимпатичного ядра блукаючого нерва

Симпато-адреналової системи

1417 / 6307
Хворий 20-ти років скаржиться на загальну слабкість, запаморочення, швидку втомлюваність. У крові: Hb- 80 г/л. Мікроскопічно: еритроцити зміненої форми. Причиною цього стану може бути:

Паренхіматозна жовтяниця

Серпоподібноклітинна анемія

Гостра переміжна порфірія

Обтураційна жовтяниця

Хвороба Аддісона

1418 / 6307
У жінки 30-ти років хвилинний об’єм крові у стані спокою становить 5 л/хв. Який об’єм крові проходить у неї через судини легень за 1 хвилину?

2,0 л

2,5 л

1,5 л

5 л

3,75 л

1419 / 6307
У людини внаслідок тривалого голодування швидкість клубочкової фільтрації зросла на 20%. Найбільш вірогідною причиною змін фільтрації в зазначених умовах є:

Збільшення проникності ниркового фільтру

Збільшення ниркового плазмотоку

Збільшення коефіцієнта фільтрації

Збільшення системного артеріального тиску

Зменшення онкотичного тиску плазми крові

1420 / 6307
Під час експерименту подразнюють скелетний м’яз серією електричних імпульсів. Який вид м’язового скорочення буде виникати, якщо кожний наступний імпульс надходить у пєріоді вкорочення попереднього поодинокого м’язового скорочення?

Асинхронний тетанус

Суцільний тетанус

Контрактура м’яза

Серія поодиноких скорочень

Зубчастий тетанус

1421 / 6307
У пацієнта, що звернувся до лікаря, спостерігається жовте забарвлення шкіри, сеча темна, кал темно-жовтого кольору. Підвищення концентрації якої речовини буде спостерігатися в сироватці крові?

Вільний білірубін

Мезобілірубін

Вердоглобін

Білівердин

Кон’ югований білірубін

1422 / 6307
На прийом до лікаря звернувся хворий з симетричним дерматитом відкритих ділянок шкіри. З бесіди з пацієнтом встановлено, що він харчується, в основному, крупами і вживає мало м’яса, молока та яєць. Дефіцит якого вітаміну є провідним у цього пацієнта?

Біотин

Нікотинамід

Фолієва кислота

Токоферол

Кальциферол

1423 / 6307
Хворий 46-ти років звернувся до лікаря зі скаргою на біль в суглобах, який посилюється напередодні зміни погоди. У крові виявлено підвищення концентрації сечової кислоти. Посилений розпад якої речовини є найвірогіднішою причиною захворювання?

УМФ

УТФ

ТМФ

ЦМФ

АМФ

1424 / 6307
У хворої 38-ми років ревматизм в активній фазі. Визначення якого лабораторного показника сироватки крові має діагностичне значення при даній патології?

Трансферин

Сечова кислота

Сечовина

Креатинін

С-реактивний білок

1425 / 6307
Хворий з інфекційним мононуклеозом протягом двох тижнів приймав глюкокортикостероїдні препарати. Настала ремісія, проте в нього виникло загострення хронічного тонзиліту. Результатом якої дії глюкокортикостероїдів є дане ускладнення?

Протишокова

Імунодепресивна

Протизапальна

Антитоксична

Антиалергічна

1426 / 6307
Під час реєстрації ЕКГ хворого з гіперфункцією щитоподібної залози зареєстровано збільшення частоти серцевих скорочень. Вкорочення якого елементу ЕКГ про це свідчить?

Інтервал R — R

Комплекс QRS

Інтервал P — T

Сегмент P — Q

Інтервал P — Q

1427 / 6307
Чоловіку 70-ти років, що хворіє на хворобу Паркінсона, був призначений препарат леводопа. Через тиждень стан хворого значно покращився. Який механізм дії цього лікарського засобу?

Гальмування гістамінергічної системи

Активація дофамінової системи

Гальмування холінергічної системи

Гальмування серотонінергічної системи

Активація енкефалінової системи

1428 / 6307
Жінка 62-х років скаржиться на частий біль у ділянці грудної клітки та хребта, переломи ребер. Лікар припустив мієломну хворобу (плазмоцитому). Який з перерахованих нижче лабораторних показників буде мати найбільше діагностичне значення?

Парапротеїнемія

Протеїнурія

Гіпоглобулінемія

Гіперальбумінемія

Гіпопротеїнемія

1429 / 6307
Під час гри у волейбол спортсмен після стрибка приземлився на зовнішній край стопи. Виник гострий біль в гомілковостопному суглобі, активні рухи в ньому обмежені, пасивні - в повному обсязі, але болісні. Потім розвинулася припухлість у ділянці зовнішньої кісточки, шкіра почервоніла, стала теплішою на дотик. Який вид розладу периферичного кровообігу розвинувся в даному випадку?

Тромбоз

Емболія

Стаз

Венозна гіперемія

Артеріальна гіперемія

1430 / 6307
Хворий надійшов до стаціонару з гострою серцево-судинною недостатністю. Який препарат необхідно призначити хворому в даному випадку?

Аміодарон

Дигоксин в таблетках

Адреналіну гідрохлорид

Дигітоксин

Корглікон

1431 / 6307
У хворого з підозрою на дифтерію під час бактеріоскопічного дослідження мазка з зіву виявлені паличкоподібні бактерії з зернами волютину. Який етіотропний препарат є препаратом вибору в даному випадку?

Бактеріофаг

Інтерферон

Протидифтерійна антитоксична сироватка

Дифтерійний анатоксин

Еубіотик

1432 / 6307
У хворого після видалення жовчного міхура утруднені процеси всмоктування Ca через стінку кишечнику. Призначення якого вітаміну буде стимулювати цей процес?

В12

PP

K

C

D3

1433 / 6307
Жінці виконана операція з приводу позаматкової (трубної) вагітності. Гілки яких артерій має перев’язати хірург під час операції?

Маткова та верхня міхурна

Верхня міхурна та яєчникова

Нижня міхурна та яєчникова

Маткова та нижня міхурна

Маткова та яєчникова

1434 / 6307
Дитині 6-ти місяців з бронхітом зроблена рентгенограма грудної клітки. Крім змін пов’язаних з бронхами, на R-грамі визначається тінь вилочкової залози (тимуса). З чим можуть бути пов’язані ці зміни?

Є наслідком запалення вилочкової залози

Є наслідком бронхіту

Є наслідком пухлинного процесу

Для цього віку вказаний стан є варіантом норми

Є наслідком неправильного анатомічного розташування

1435 / 6307
Хворий відзначає часті проноси, особливо після вживання жирної їжі, схуднення. Лабораторні дослідження показали наявність стеатореї; кал гіпохолічний. Що може бути причиною такого стану?

Незбалансована дієта

Недостатність панкреатичної ліпази

Недостатність панкреатичної фосфоліпази

Обтурація жовчних шляхів

Запалення слизової оболонки тонкої кишки

1436 / 6307
У хворого з гнійничковими ураженнями шкіри виділений збудник, який на кров’яному агарі утворює жовті колонії округлої форми, середніх розмірів, оточені зоною гемолізу. У мазках з колоній - коки, розташовані скупченнями неправильної форми, грампозитивні. Виділена культура оксидазо- і каталазопозитивна, ферментує маніт, синтезує плазмокоагулазу. Який вид збудника виділений?

Staphylococcus aureus

Staphylococcus saprophyticus

Staphylococcus epidermidis

Streptococcus agalactiae

Streptococcus pyogenes

1437 / 6307
Чоловік 65-ти років, який страждає на подагру, скаржиться на біль в ділянці нирок. При ультразвуковому обстеженні встановлена наявність ниркових каменів. Підвищення концентрації якої речовини є найбільш вірогідною причиною утворення каменів у даному випадку?

Сечова кислота

Холестерин

Сечовина

Цистин

Білірубін

1438 / 6307
При мікроскопії зіскрібку з язика, забарвленого за Грамом, знайдені овальні, округлі, темно-фіолетового кольору, видовжені ланцюжки клітин, що брунькуються. Про збудника якого захворювання може йти мова?

Актиномікоз

Стафілококова інфекція

Кандидоз

Стрептококова інфекція

Дифтерія

1439 / 6307
Жінка 44-х років скаржиться на загальну слабкість, біль у ділянці серця, значне збільшення маси тіла. Об’єктивно: обличчя місяцеподібне, гірсутизм, АТ- 165/100 мм рт.ст., зріст -164 см, вага - 103 кг; переважно накопичення жиру на шиї, верхньому плечовому поясі, животі. Що є основним патогенетичним механізмом ожиріння у жінки?

Підвищення продукції глюкокорти-коїдів

Підвищення продукції мінералокор-тикоїдів

Зниження продукції глюкагону

Підвищення продукції інсуліну

Зниження продукції тиреоїдних гормонів

1440 / 6307
Кров, яку взяли у хворого з підозрою на сепсис, посіяли на цукровий бульйон. У цукровому бульйоні утворився придонний осад. При пересіві на кров’яний агар виросли дрібні, прозорі, кулясті колонії, оточені зоною гемолізу. У мазку, приготованому з осаду, визначалися грампозитивні коки, що розташовувалися у вигляді довгих ланцюжків. Які мікроорганізми присутні у крові цього хворого?

Сарцини

Стрептококи

Тетракоки

Мікрококи

Стафілококи

1441 / 6307
Хворий 2 роки тому переніс операцію резекції пілоричного відділу шлунка. Спостерігається слабкість, періодична поява темних кіл під очима, задишка. У крові: Hb70 г/л, ер.- 3,0 • 1012/л, КП- 0,7 Які зміни еритроцитів у мазках крові найбільш характерні для даного стану?

Овалоцити

Мегалоцити

Макроцити

Мікроцити

Шизоцити

1442 / 6307
У пацієнта цироз печінки. Дослідження якої з перелічених речовин, що екскретуються з сечею, може характеризувати стан антитоксичної функції печінки?

Гіпурова кислота

Креатинін

Амінокислоти

Амонійні солі

Сечова кислота

1443 / 6307
Територію старого худобомогильника, який не використовувався більше 50 років, планується відвести під житлове будівництво. Однак дослідження ґрунту показало наявність життєздатних спор збудника особливо небезпечного захворювання. Який із вказаних мікроорганізмів найбільш вірогідно міг зберегтися у ґрунті протягом такого тривалого часу?

Bacillus anthracis

Francisella tularensis

Yersinia pestis

Brucella abortus

Mycobacterium bovis

1444 / 6307
На розтині хворого, який багато років працював на шахті і помер від хронічної легеневосерцевої недостатності, виявлено, що легені малоповітряні, значно ущільнені, склерозовані, верхівки емфізематозно змінені, поверхня сіро-чорного кольору, на розрізі тканина легенів аспідно-чорного кольору. Від якої хвороби настала смерть?

Азбестоз

Антракоз

Алюміноз

Талькоз

Силікоз

1445 / 6307
У баклабораторії під час мікроскопії мазків з харкотиння хворого на хронічне легеневе захворювання, забарвлених за Цілем-Нільсеном, виявлені червоні палички. Яка властивість туберкульозної палички виявлена при цьому?

Спороутворення

Спиртостійкість

Кислотостійкість

Капсулоутворення

Лугостійкість

1446 / 6307
Під час дослідження коронарних артерій виявлені атеросклеротичні бляшки з кальцинозом, що закривають просвіт судин на 1/3. У м’язі дрібні множинні білуваті прошарки сполучної тканини. Як називається процес, виявлений у міокарді?

Післяінфарктний кардіосклероз

Міокардит

Тигрове серце

Інфаркт міокарда

Дифузний кардіосклероз

1447 / 6307
Встановлено ураження вірусом ВІЛ Т-лімфоцитів. При цьому фермент вірусу зворотня траскриптаза (РНК-залежна ДНК-полімераза) каталізує синтез:

і-РНК на матриці вірусного білка

Вірусна ДНК на матриці ДНК

Вірусна і-РНК на матриці ДНК

ДНК на матриці вірусної і-РНК

ДНК на вірусній р-РНК

1448 / 6307
До травматологічного відділення госпіталізовано чоловіка 35-ти років з травмою лівої кисті. Об’єктивно: різана рана долонної поверхні лівої кисті; середні фаланги II-V пальців не згинаються. Які м’язи пошкоджені?

Долонні міжкісткові м’язи

Глибокий м’яз-згинач пальців

Тильні міжкісткові м’язи

Поверхневий м’яз-згинач пальців

Червоподібні м’язи

1449 / 6307
При гістологічному дослідженні біоптату шкіри виявлені гранульоми, які складаються з макрофагальних вузликів з наявністю лімфоцитів та плазматичних клітин. Крім того, зустрічаються великі макрофаги з жировими вакуолями, які містять запакованих у вигляді куль збудників захворювання (клітини Вірхова). Грануляційна тканина добре васкуляризована. Для якого захворювання характерна описана гранульома?

Туберкульоз

Сифіліс

Лепра

Сап

Риносклерома

1450 / 6307
На препараті яєчника, забарвленому гематоксиліном-еозином, визначається фолікул, в якому клітини фолікулярного епітелію розміщені в 1-2 шари та мають кубічну форму, навколо овоциту наявна оболонка яскраво-червоного кольору. Назвіть цей фолікул:

Зрілий

Примордіальний

Вторинний

Первинний

Атретичний

1451 / 6307
До інфекційної лікарні надійшов пацієнт з клінічними ознаками енцефаліту. В анамнезі - укус кліща. В реакції гальмування гемаглютинації виявлено антитіла проти збудника кліщового енцефаліту в розведенні 1:20, що не є діагностичним. Вкажіть наступні дії лікаря після одержання зазначеного результату:

Дослідити цю ж сироватку повторно

Повторити дослідження із сироваткою, взятою через 10 днів

Повторити дослідження з іншим діагностикумом

Використати чутливішу реакцію

Відхилити діагноз кліщового енцефаліту

1452 / 6307
У чоловіка 40-ка років у ділянці шиї виникло почервоніння та набряк шкіри і з часом розвинувся невеликий гнійник. На розрізі осередок щільний, жовто-зеленого забарвлення. В гнійних масах видно білі крупинки. Гістологічно виявлено друзи грибка, плазматичні та ксантомні клітини, макрофаги. Який найбільш вірогідний вид мікозу?

Актиномікоз

Аспергільоз

Кокцидіомікоз

Споротрихоз

Кандидоз

1453 / 6307
Тривале вживання деяких лікарських засобів, що передують вагітності, збільшують ризик народження дитини з генетичними вадами. Як називається ця дія?

Фетотоксичний ефект

Ембріотоксичний ефект

Тератогенний ефект

Бластомогенний ефект

Мутагенний ефект

1454 / 6307
Під час розтину чоловіка, у якого після поранення кінцівки виникло тривале нагноєння рани, і який помер при явищах інтоксикації, знайдено загальне виснаження, зневоднення, бура атрофія печінки, міокарду, селезінки, поперечносмугастої мускулатури та амілоїдоз нирок. Який з перелічених діагнозів найбільш вірогідний?

Хроніосепсис

Септикопіємія

Хвороба Чорногубова

Бруцельоз

Септицемія

1455 / 6307
Жінка 19-ти років, хвора на первинний сифіліс, отримує комплексну терапію, до складу якої входить натрієва сіль бензилпеніциліну. Вкажіть механізм дії цього препарату:

Блокада синтезу білків цитоплазми

Блокада синтезу РНК

Блокада синтезу ДНК

Блокада синтезу петдидогліканів мікробної оболонки

Блокада тіолових груп ензимів

1456 / 6307
У жінки через 6 місяців після пологів розвинулася маткова кровотеча. Під час гінекологічного обстеження у порожнині матки виявлена тканина темно-червоного кольору з множинними порожнинами, що нагадує 'губку'. Під час мікроскопічного дослідження пухлини у лакунах крові виявлені атипові світлі епітеліальні клітини Лангханса та гігантські клітини синцитіотрофобласта. Яка це пухлина?

Аденокарцинома

Плоскоклітинний незроговілий рак

Фіброміома

Міхуровий занос

Хоріонепітеліома

1457 / 6307
У тварини з недостатністю аортальних клапанів розвинулась гіпертрофія лівого шлуночка серця. В окремих його ділянках визначаються локальні контрактури. Накопичення якої речовини в міокардіоцитах обумовило контрактури?

Вуглекислий газ

Калій

Натрій

Молочна кислота

Кальцій

1458 / 6307
У дівчинки діагностований адреногенітальний синдром (псевдогермафродитизм). Надмірна секреція якого гормону наднирників обумовила дану патологію?

Альдостерон

Кортизол

Адреналін

Естроген

Андроген

1459 / 6307
Під час гістологічного дослідження тимуса чоловіка 40-ка років, визначено зменшення частки паренхіматозних елементів залози, збільшення частки жирової та пухкої сполучної тканини, збагачення її тимусними тільцями при незмінній загальній масі органу. Як зветься таке явище?

Гіпотрофія

Дистрофія

Атрофія

Акцидентальна інволюція

Вікова інволюція

1460 / 6307
При обстеженні хворого 70-ти років виявлено інсулінонезалежний цукровий діабет. Який препарат доцільно призначити хворому?

Мерказоліл

Глібенкламід

Кортизон

Паратіреоїдин

Інсулін

1461 / 6307
Охолодження тіла людини у воді виникає значно швидше, ніж на повітрі. Який шлях тепловіддачі у воді значно ефективніший?

Випаровування поту

Конвекція

Тепловипромінювання

Теплопроведення

1462 / 6307
У бактеріологічній лабораторії проводиться дослідження м’ясних консервів на вміст ботулінічного токсину. Для цього дослідній групі мишей ввели екстракт із досліджуваного матеріалу та антитоксичну протиботулінічну сироватку типів А, В, Е; контрольній групі мишей ввели екстракт без протиботулінічної сироватки. Яку серологічну реакцію було використано?

Опсонофагоцитарна

Подвійної імунної дифузії

Зв’язування комплементу

Нейтралізації

Преципітації

1463 / 6307
Хворому, що страждає на хронічну серцеву недостатність, лікар порадив провести профілактичний курс лікування кардіотонічним препаратом з групи серцевих глікозидів, який приймають ентерально. Який препарат було рекомендовано хворому?

Кордіамін

Строфантин

Дигоксин

Кордарон

Корглікон

1464 / 6307
Під час диспансерного огляду у чоловіка 36-ти років, водія за професією, артеріальний тиск склав 150/90 мм рт.ст. Скарги на шум у вухах наприкінці робочого дня та загальне нездужання, які зникають після відпочинку. Діагностована гіпертонічна хвороба. Який провідний патогенетичний механізм у цьому випадку?

Нейрогенний

Гуморальний

Нирковий

Ендокринний

Рефлексогенний

1465 / 6307
Травма мозку викликала підвищене утворення аміаку. Яка амінокислота бере участь у видаленні аміаку з мозкової тканини?

Лізин

Глутамінова

Тирозин

Триптофан

Валін

1466 / 6307
Проводиться вивчення максимально спіралізованих хромосом каріотипу людини. При цьому процес поділу клітини припинили на стадії:

Профаза

Інтерфаза

Телофаза

Метафаза

Анафаза

1467 / 6307
Після проведення туберкулінової проби (проба Манту) у дитини через 48 годин на місці введення туберкуліну утворилася папула до 10 мм у діаметрі. Який механізм гіперчутливості лежить в основі розвитку вказаних змін?

Анафілаксія

Антитілозалежна цитотоксичність

Імунокомплексна цитотоксичність

Клітинна цитотоксичність

Гранулематоз

1468 / 6307
Хворий госпіталізований до хірургічного відділення з підозрою на запалення дивертикулу Меккеля. Обстеження якого відділу кишечнику необхідно провести для виявлення дивертикулу під час операції?

Ileum

Caecum

Duodenum

Colon ascendens

Jejunum

1469 / 6307
У людини визначили величину енерговитрат. У якому стані знаходилась людина, якщо її енерговитрати виявилися меншими за основний обмін?

Легка робота

Відпочинок

Сон

Нервове напруження

Спокій

1470 / 6307
У новонародженого хлопчика під час огляду зовнішніх статевих органів виявлена розщілина сечівника, яка відкривається на нижній поверхні статевого члена. Про яку аномалію йдеться?

Гермафродитизм

Крипторхізм

Монорхізм

Епіспадія

Гіпоспадія

1471 / 6307
Тварині внутрішньовенно ввели концентрований розчин хлориду натрію, що зумовило зниження реабсорбції іонів натрію у канальцях нирок. Внаслідок яких змін секреції гормонів це відбувається?

Збільшення вазопресину

Зменшення натрійуретичного фактора

Зменшення вазопресину

Зменшення альдостерону

Збільшення альдостерону

1472 / 6307
Внаслідок руйнування певних структур стовбуру мозку тварина втратила орієнтувальні рефлекси у відповідь на сильні світлові подразники. Які структури було зруйновано?

Червоні ядра

Передні горбки чотиригорбкового тіла

Чорна речовина

Задні горбки чотиригорбкового тіла

Вестибулярні ядра

1473 / 6307
У цитоплазмі міоцитів розчинена велика кількість метаболітів окиснення глюкози. Назвіть один з них, який безпосередньо перетворюється на лактат:

Піруват

Глюкозо-6-фосфат

Фруктозо-6-фосфат

Оксалоацетат

Гліцерофосфат

1474 / 6307
До стоматолога звернулася мати дитини 2-х років, яка під час вагітності, у зв’язку з інфекційним захворюванням, безсистемно приймала антибіотики. Під час огляду у дитини відзначається руйнування різців, емаль зубів жовтого кольору, у шийки зубів коричнева облямівка. Який з препаратів має виражену тератогенну дію?

Ксантинола нікотинат

Доксацикліну гідрохлорид

Фуросемід

Ампіокс

Октадин

1475 / 6307
У хворого 59-ти років, директора підприємства, після перевірки податкової інспекції з’явився інтенсивний пекучий біль, локалізований за грудниною, який іррадіює в ліву руку. Через 15 хвилин стан хворого нормалізувався. Який можливий механізм стенокардії є провідним у цього хворого?

Внутрішньосудинна агрегація формених елементів

Підвищення у крові рівня катехола-мінів

Тромбоз коронарних судин

Атеросклероз коронарних судин

Функціональне перевантаження серця

1476 / 6307
Пацієнт одужав після перенесеної дизентерії Зонне і повторно заразився цим же збудником. Як називається така форма інфекції?

Реінфекція

Рецидив

Персистуюча інфекція

Суперінфекція

Хронічна інфекція

1477 / 6307
Чоловік 56-ти років потрапив до клініки зі скаргами на загальну слабкість, біль і печіння в язиці, відчуття оніміння в кінцівках. У минулому переніс резекцію кардіального відділу шлунка. У крові: Hb- 80 г/л; ер.- 2,0 • 1012/л; КП-1,2, лейк.- 3,5 • 109 /л. Який вид анемії у цього хворого?

Гемолітична

Апластична

Постгеморагічна

В12 -фолієводефіцитна

Залізодефіцитна

1478 / 6307
Хворий 35-ти років звернувся до лікаря із скаргами на сильну нежить та втрату відчуття запахів протягом тижня. Об’єктивно: в носовій порожнині велика кількість слизу, що вкриває слизову оболонку та блокує рецептори нюху. Де в носовій порожнині розташовані ці рецептори?

Присінок носа

Нижня носова раковина

Загальний носовий хід

Середня носова раковина

Верхня носова раковина

1479 / 6307
Юнак 17-ти років захворів гостро, температура тіла підвищилася до 38,5o C, з’явилися кашель, нежить, сльозотеча, виділення з носу. Яке запалення розвинулося у юнака?

Катаральне

Геморагічне

Серозне

Фібринозне

Гнійне

1480 / 6307
У хворого з порушенням мозкового кровотоку порушений акт ковтання. Який відділ мозку постраждав?

Шийний відділ спинного мозку

Стовбур мозку

Проміжний мозок

Середній мозок

Передній мозок

1481 / 6307
Внаслідок дефіциту вітаміну B1 порушується окисне декарбоксилювання α-кетоглутарової кислоти. Синтез якого з наведених коферментів порушується при цьому?

Ліпоєва кислота

Коензим А

Тіамінпірофосфат

Нікотинамідаденіндинуклеотид

Флавінаденіндинуклеотид

1482 / 6307
Після хімічного опіку в хворого розвинувся стеноз стравоходу. Виникло різке схуднення від затрудненого прийому їжі. У крові: ер.- 3,0 • 1012/л, Hb-106 г/л, загальний білок - 57 г/л. Який вид голодування в хворого?

Водне

Повне

Абсолютне

Білкове

Неповне

1483 / 6307
У жінки 30-ти років виникли набряки обличчя. При обстеженні виявлені протеїнурія (5,87 г/л), гіпопротеїнемія, диспротеїнемія, гіперліпідемія. Для якого стану характерно таке поєднання симптомів?

Нефротичний синдром

Нефритичний синдром

Хронічна ниркова недостатність

Хронічний пієлонефрит

Гостра ниркова недостатність

1484 / 6307
У померлого внаслідок серцевої недостатності на шкірі відзначаються сліди висипу у вигляді плям і крапок. У ділянці крижі, остистих відростків хребців - пролежні. При мікроскопічному дослідженні ЦНС, шкіри, надниркових залоз, в судинах мікроциркуляторного русла і дрібних артеріях - деструктивно-проліферативний ендотромбоваскуліт з наявністю гранульом Попова; в серці - інтерстиційний міокардит. Який з перелічених діагнозів найбільш вірогідний?

Вузликовий періартеріїт

Висипний тиф

Черевний тиф

Ку -гарячка

ВІЛ-інфекція

1485 / 6307
На розтині померлого від сепсису, в стегновій кістці нижньої кінцівки виявлено флегмонозне запалення, що охоплює кістковий мозок, гаверсові канали та періост. Під періостом - множинні абсцеси, в навколишніх м’яких тканинах стегна - також флегмонозне запалення. Який патологічний процес має місце?

Остеопетроз

Гострий гематогенний остеомієліт

Остеопороз

Хронічний гематогенний остеомієліт

1486 / 6307
У хворого знижений синтез вазопресину, що призводить до поліурії і, як наслідок, до вираженої дегідратації організму. У чому полягає механізм розвитку поліурії?

Зниження канальцевої реабсорбції білку

Зниження канальцевої реабсорбції іонів Na

Зниження канальцевої реабсорбції води

Зниження реабсорбції глюкози

Збільшення швидкості клубочкової фільтрації

1487 / 6307
У хворого з нефротичним синдромом спостерігаються масивні набряки обличчя та кінцівок. Який патогенетичний механізм є провідним в розвитку набряків?

Підвищення судинної проникності

Лімфостаз

Підвищення гідродинамічного тиску крові

Зниження онкотичного тиску крові

Підвищення лімфовідтоку

1488 / 6307
Під шкірою піднижньощелепної ділянки у жінки 55-ти років виявлено рухоме утворення 1,0x0,7 см, з чіткими межами, тістуватої консистенції, з повільним ростом. При гістологічному дослідженні утворення виявлені ліпоцити, які формують часточки, різні за формою та розміром, розмежовані тонкими прошарками сполучної тканини із судинами. Який найбільш вірогідний діагноз?

Ліпосаркома

Ангіома

Фібросаркома

Ліпома

Фіброма

1489 / 6307
При щелепно-лицевій операції в стаціонарі, лікар-стоматолог для премедикації, з метою зменшення почуття страху та для потенціювання дії анестетиків, призначив хворому транквілізатор - похідне бензодіазепіну. Який з перерахованих засобів призначив лікар?

Дроперідол

Діазепам

Аміназин

Атропіну сульфат

Сульпірид

1490 / 6307
До приймального відділення лікарні надійшов непритомний юнак з ознаками отруєння морфіном. Відзначається поверхневе та рідке дихання, яке обумовлене пригніченням дихального центру. Який тип недостатності дихання виник при цьому?

Вентиляційна обструктивна

Вентиляційна дисрегуляторна

Перфузійна

Дифузійна

Вентиляційна рестриктивна

1491 / 6307
У хворого на жовчнокам’яну хворобу розвинулася механічна жовтяниця. Під час обстеження встановлено, що камінь знаходиться у загальній жовчній протоці. Які жовчовивідні протоки утворюють обтуровану протоку?

Ductus hepaticus communis et ductus choledochus

Ductus hepaticus dexter et ductus cysticus

Ductus hepaticus sinister et ductus cysticus

Ductus hepaticus communis et ductus cysticus

Ductus hepaticus dexter et sinister

1492 / 6307
При лабораторному дослідженні крові пацієнта 44-х років виявлено, що вміст білків у плазмі становить 40 г/л. Як це впливає на транскапілярний обмін води?

Зменшується фільтрація, збільшується реабсорбція

Зменшуються фільтрація та реабсорбція

Збільшується фільтрація, зменшується реабсорбція

Збільшуються фільтрація та реабсорбція

Обмін не змінюється

1493 / 6307
З метою диференційної діагностики менінгітів проводять дослідження спинномозкової рідини. В якому місці люмбальна пункція є безпечною?

LII -LIII

LI -LII

LV -SI

ThXII -LI

LIII -LIV

1494 / 6307
Ізольована клітина серця людини автоматично генерує імпульси збудження з частотою 60 разів за хвилину. З якої структури серця отримано цю клітину?

Пучок Гіса

Атріовентрикулярний вузол

Шлуночок

Передсердя

Синоатріальний вузол

1495 / 6307
Внаслідок посттрансляційних змін деяких білків, що приймають участь в зсіданні крові, зокрема протромбіну, вони набувають здатності зв’язувати кальцій. В цьому процесі бере участь вітамін:

B2

А

C

В1

K

1496 / 6307
У людини, що виконувала важку фізичну роботу в умовах підвищеної температури навколишнього середовища, змінилася кількість білків плазми крові. Що саме має місце у даному випадку?

Відносна гіперпротеїнемія

Абсолютна гіперпротеїнемія

Абсолютна гіпопротеїнемія

Парапротеїнемія

Диспротеїнемія

1497 / 6307
У хворого на тромбофлебіт нижніх кінцівок з’явився біль у грудній клітці, кровохаркання, наростаюча дихальна недостатність, при явищах якої він помер. На розтині діагностовано множинні інфаркти легень. Яка найбільш вірогідна причина їх розвитку в цьому випадку?

Тромбоз гілок легеневої артерії

Тромбоемболія бронхіальних артерій

Тромбоз бронхіальних артерій

Тромбоз легеневих вен

Тромбоемболія гілок легеневої артерії

1498 / 6307
У жінки, що хворіє на остеохондроз, з’явився різкий біль у плечовому суглобі, який посилювався при відведенні плеча. Ураження якого нерва може бути причиною цих симптомів?

Дорсальний нерв лопатки

Грудо-спинний нерв

Пахвовий нерв

Підключичний нерв

Підлопатковий нерв

1499 / 6307
У хворого глибока різана рана на задній поверхи плеча в середній його третині. Про ушкодження якого м’яза можна думати?

Дзьобоподібно-плечовий м’яз

Плечовий м’яз

Ліктьовий м’яз

Двоголовий м’яз плеча

Трьохголовий м’яз плеча

1500 / 6307
Хворому, що страждає на стенокардію та приймає ізосорбіду мононітрат, було додатково призначено лікарський засіб з дезагрегантним ефектом. Визначте цей препарат:

Ніфедипін

Ацетилсаліцилова кислота

Анаприлін

Нітрогліцерин

Валідол

1501 / 6307
Хворому в післяопераційному періоді для стимуляції перистальтики кишечника та тонусу сечового міхура було призначено препарат з групи антихолінестеразних засобів. Визначте його серед нижче наведених препаратів:

Маніт

Прозерин

Резерпін

Анаприлін

Дихлотіазид

1502 / 6307
У хворого на обличчі вугри. Під час мікроскопії зіскрібків із уражених ділянок виявлені живі членистоногі розміром 0,2-0,5 мм, які мають витягнуту червоподібну форму, чотири пари коротких кінцівок, що розташовані у середній частині тіла. Який лабораторний діагноз?

Фтиріоз

Міаз

Короста

Педикульоз

Демодекоз

1503 / 6307
У чоловіка 28-ми років після вогнепального поранення гомілки розвинулася виразка на боці пошкодження. Що є основним у патогенезі нейродистрофії в даному випадку?

Пошкодження тканини

Порушення мікроциркуляції

Інфекція

Психічний стрес

Травматизація периферичного нерва

1504 / 6307
Чоловік 45-ти років звернувся до лікаря з приводу бляшкоподібного утворення на шиї. В біоптаті шкіри гістологічно виявлено пухлинні клітини, розташовані гніздами, які мають круглу та овальну форму з вузьким ободком базофільної цитоплазми та схожі на клітини базального шару епідермісу. Як називається пухлина в пацієнта?

Епідермальний рак

Гідраденома

Базаліома

Сирінгоаденома

Трихоепітеліома

1505 / 6307
Хворому на гіпертонічну хворобу лікар запропонував засіб, який припиняє тромбоутворення, вводиться парентерально. Оберіть цей засіб:

Неодикумарин

Гепарин

Протаміну сульфат

Амбен

Синкумар

1506 / 6307
У людини до травми гематокритний показник 40%. Яким він буде через добу після втрати 750 мл крові?

50%

30%

45%

40%

55%

1507 / 6307
У студента через 2 години після іспиту в аналізі крові виявлено лейкоцитоз без істотних змін у лейкоцитарній формулі. Який найбільш вірогідний механізм розвитку лейкоцитозу?

Перерозподіл лейкоцитів в організмі

Посилення лейкопоезу та зменшення руйнування лейкоцитів

Уповільнення руйнування лейкоцитів

Посилення лейкопоезу

Уповільнення міграції лейкоцитів у тканини

1508 / 6307
Комплекс Гольджі виводить речовини із клітини завдяки злиттю мембранного мішечка з мембраною клітини. При цьому вміст мішечка виливається назовні. Який процес тут проявляється?

Екзоцитоз

Жодна відповідь невірна

Ендоцитоз

Полегшена дифузія

Активний транспорт

1509 / 6307
Відпочиваючи на дачі, хлопчик знайшов павука з наступними морфологічними особливостями: довжина - 2 см, кулясте черевце чорного кольору, на спинному боці якого видно червоні плямочки у два ряди, чотири пари членистих кінцівок вкриті дрібними чорними волосками. Визначте дане членистоноге:

Тарантул

Кліщ

Фаланги

Скорпіон

Каракурт

1510 / 6307
Дитина 3-х років із симптомами стоматиту, гінгівіту, дерматиту відкритих ділянок шкіри була госпіталізована. При обстеженні встановлено спадкове порушення транспорту нейтральних амінокислот у кишечнику. Нестача якого вітаміну зумовила дані симптоми?

Вітамін A

Біотин

Пантотенова кислота

Ніацин

Кобаламін

1511 / 6307
Хворому з підозрою на діагноз 'прогресуюча м’язова дистрофія' був зроблений аналіз сечі. Наявність якої сполуки в сечі підтверджує діагноз?

Кальмодулін

Креатин

Колаген

Порфірин

Міоглобін

1512 / 6307
У хворого, який знаходиться на лікуванні з приводу вірусного гепатиту В, з’явилися ознаки печінкової недостатності. Які зміни крові, що свідчать про порушення білкового обміну, найбільш вірогідно спостерігатимуться у цьому випадку?

Абсолютна гіпоальбумінемія

Абсолютна гіперальбумінемія

Абсолютна гіперглобулінемія

Абсолютна гіперфібриногенемія

Білковий склад крові не змінений

1513 / 6307
У вагітної жінки визначили групу крові. Реакція аглютинації еритроцитів відбулася зі стандартними сироватками груп 0ав (I), Bα (III) і не відбулася зі стандартною сироваткою групи Aβ (II). Досліджувана кров належить до групи:

Aβ (II)

Bα (III)

0αβ (I)

AB (IV)

1514 / 6307
Чутливий нервовий ганглій складається з нейроцитів кулястої форми з одним відростком, який на певній відстані від перикаріону поділяється на аксон і дендрит. Як називаються такі клітини?

мультиполярні

уніполярні

аполярні

біполярні

псевдоуніполярні

1515 / 6307
При обстеженні дитини лікар виявив ознаки рахіту. Нестача якої сполуки в організмі дитини сприяє розвитку цього захворювання?

1,25 [ОН]- дигідроксіхолекальциферол

Токоферол

Нафтохінон

Біотин

Ретинол

1516 / 6307
У зародка порушено процес сегментації дорзальної мезодерми та утворення сомітів. В якій частині шкіри можливі порушення розвитку?

Дерма

Епідерміс

Потові залози

Сальні залози

Волосся

1517 / 6307
Встановлено, що деякі сполуки, наприклад, токсини грибів та деякі антибіотики, можуть пригнічувати активність РНК-полімерази. Порушення якого процесу відбувається в клітині у випадку пригнічування даного ферменту?

Процесінг

Реплікація

Трансляція

Репарація

Транскрипція

1518 / 6307
Матеріалом від дитини з попереднім діагнозом 'ентеровірусна інфекція'заразили культуру клітин мавпи (Vero) і мишат-сисунків, в результаті не виявлено цитопатичного ефекту на культурі клітин, але зареєстрована загибель мишат-сисунків. Які ентеровіруси могли викликати захворювання у цієї дитини?

ECHO

Коксакі А

Некласифіковані ентеровіруси 68-71

Коксакі В

Поліовіруси

1519 / 6307
Хворий помер від наростаючої легенево-серцевої недостатності. При гістологічному дослідженні виявлено: дифузне ураження легенів з інтерстиціальним набряком, інфільтрацією інтерстиціальної тканини лімфоцитами, макрофагами, плазмоцидами; пневмофіброз, панацинарна емфізема. Який найбільш вірогідний діагноз?

Бронхопневмонія

Ателектаз легенів

Хронічний бронхіт

Фіброзуючий альвеоліт

Бронхіальна астма

1520 / 6307
В експерименті подразнюють гілочки блукаючого нерва, які іннервують серце. Це призвело до того, що припинилося проведення збудження від передсердь до шлуночків. Електрофізіологічні зміни в яких структурах серця є причиною цього?

Пучок Гіса

Шлуночки

Атріовентрикулярний вузол

Передсердя

Синоатріальний вузол

1521 / 6307
У людини з нападом бронхоспазму необхідно зменшити вплив блукаючого нерва на гладеньку мускулатуру бронхів. Які мембранні циторецептори доцільно заблокувати для цього?

H-холінорецептори

α- та β-адренорецептори

β-адренорецептори

M-холінорецептори

α-адренорецептори

1522 / 6307
У пацієнтки хірургічного відділення з’явилися скарги на біль у попереку та в низу живота, болісне і часте сечовипускання. Після бактеріологічного дослідження сечі виявлені грамнегативні оксидазапозитивні паличкоподібні бактерії, що утворюють мукоїдні колонії зеленуватого кольору зі специфічним запахом. Про який збудник можна думати?

E.coli

Proteus mirabilis

Str.pyogenes

Pseudomonas aeruginosa

Mycoplasma pneumonie

1523 / 6307
Виділяють декілька груп молекулярних механізмів, які мають важливе значення в патогенезі ушкодження клітин, що сприяє розвитку патології. Які процеси забезпечують протеїнові механізми ушкодження?

Перекисне окиснення ліпідів

Ацидоз

Пригнічення ферментів

Активація фосфоліпаз

Осмотичне розтягнення мембран

1524 / 6307
Дитина народилася з вовчою пащею. При обстеженні виявлено вади аорти, в крові - зменшення Т-лімфоцитів. Який імунодефіцитний синдром у новонародженого?

Луї-Бар

Ді Джорджі

Швейцарський тип

Чедіака-Хігасі

Віскотта-Олдріча

1525 / 6307
У юнака 16-ти років після перенесеного захворювання знижена функція синтезу білків у печінці внаслідок нестачі вітаміну K. Це може призвести до порушення:

Осмотичного тиску крові

Утворення еритропоетинів

Утворення антикоагулянтів

Зсідання крові

Швидкості зсідання еритроцитів

1526 / 6307
У життєвому циклі клітини відбувається процес самоподвоєння ДНК. В результаті цього однохроматидні хромосоми стають двохроматидними. У який період клітинного циклу спостерігається це явище?

M

S

G2

G0

G1

1527 / 6307
У водія, який потрапив у ДТП, отримав травму та знаходиться у стані шоку, спостерiгається зменшення добової кількості сєчі до 300 мл. Який основний патогенетичний фактор цієї зміни діурезу?

Падіння артеріального тиску

Підвищення проникності судин

Вторинний гіперальдостеронізм

Зниження онкотичного тиску крові

Зменшення кількості функціонуючих клубочків

1528 / 6307
У дорослої людини системний артеріальний тиск знизився з 120/70 до 90/50 мм рт.ст., що викликало рефлекторне звуження судин. У якому з зазначених органів звуження судин буде найбільшим?

Серце

Наднирники

Нирки

Кишечник

Головний мозок

1529 / 6307
Під час обстеження 12-ти річного хлопчика, який відстає у рості, виявлена ахондроплазія: непропорційна тілобудова з помітним вкороченням рук і ніг, внаслідок порушення росту епіфізарних хрящів довгих трубчастих кісток. Дане захворювання є:

Спадковим, зчепленим зі статтю

Набутим

Вродженим

Спадковим за рецесивним типом

Спадковим за домінантним типом

1530 / 6307
У хворого виявлена аутоімунна гемолітична анемія, що розвивається за цитотоксичним типом. Які речовини є антигенами при алергічних реакціях II типу?

Антибіотики

Сироваткові білки

Гормони

Модифіковані рецептори клітинних мембран

Модулятори запалення

1531 / 6307
При виконанні оперативного втручання в ділянці пахвової порожнини, хірургу необхідно виділити артеріальну судину, що оточена пучками плечового сплетіння. Яка це артерія?

A.profunda brachii

A.transversa colli

A.axillaris

A.vertebralis

A.subscapularis

1532 / 6307
Для лікування хронічної серцевої недостатності хворий приймає дигоксин. Який діуретичний засіб може збільшити токсичність дигоксину за рахунок посиленого виведення з організму іонів K+?

Панангін

Силібор

Лізиноприл

Спіронолактон

Гідрохлортіазид

1533 / 6307
Під час голодування м’язові білки розпадаються до вільних амінокислот. В який процес найімовірніше будуть втягуватися ці сполуки за таких умов?

Декарбоксилування

Синтез вищих жирних кислот

Глюконеогенез у м’язах

Глюконеогенез у печінці

Глікогеноліз

1534 / 6307
Хворій на виразкову хворобу шлунка в якості етіотропної терапії призначили препарат, що пригнічує ріст і розмноження H.pylori. Назвіть цей препарат:

Празозин

Glauvent

Метронідазол

Фуросемід

Корглікон

1535 / 6307
Після оперативного видалення частини шлунка у хворого порушилося всмоктування вітаміну B12, він виводиться з калом. Розвинулася анемія. Який фактор необхідний для всмоктування цього вітаміну?

Соляна кислота

Фолієва кислота

Гастрин

Пепсин

Гастромукопротеїн

1536 / 6307
Хворому на хронічну серцеву недостатність призначили дигоксин у середньотерапевтичній дозі. Через 2 тижні після початку прийому препарату у нього з’явились ознаки інтоксикації препаратом (брадикардія, екстрасистолія, нудота). Як називається явище, яке призвело до накопичення в організмі препарату?

Толерантність

Ідіосинкразія

Тахіфілаксія

Сенсибілізація

Кумуляція

1537 / 6307
При проходженні профілактичного огляду у шахтаря лікар встановив зміни функціонального стану серця, що свідчать про серцеву недостатність в стадії компенсації. Що з нижче перерахованого є головним підтвердженням компенсації діяльності серця?

Ціаноз

Гіпертрофія міокарда

Збільшення артеріального тиску

Задишка

Тахікардія

1538 / 6307
Потенціал спокою клітини дорівнює -80 мВ. Під час якої фази ПД величина мембранного потенціалу склала +30 мВ?

Деполяризації

Слідової гіперполяризації

Реверсполяризації

Слідової деполяризації

1539 / 6307
У результаті поранення у чоловіка 35-ти років настав повний розрив спинного мозку на рівні першого шийного сегменту. Як зміниться характер дихання?

Збережеться діафрагмальне, щезне грудне

Не зміниться

Зупиниться

Стане рідким та глибоким

Збережеться грудне, щезне діафрагмальне

1540 / 6307
У піддослідної тварини під час експерименту подразнюють периферичний відрізок блукаючого нерва. Які з наведених змін будуть спостерігатися при цьому?

Збільшення частоти серцевих скорочень

Збільшення частоти дихання

Розширення зіниць

Зменшення частоти серцевих скорочень

Розширення бронхів

1541 / 6307
У хворого 27-ми років з політравмою (закрита травма грудної клітини, закритий перелом правого стегна) через дві години після проведення скелетного витягнення різко погіршився стан і, на фоні гострої легенево-серцевої недостатності, настала смерть. При гістологічному дослідженні кровоносних судин легень та головного мозку померлого, при забарвленні суданом ІІІ, виявлені краплі помаранчевого кольору, які закупорюють просвіт судин. Яке ускладнення політравми розвинулося у хворого?

Мікробна емболія

Тромбоемболія

Повітряна емболія

Газова емболія

Жирова емболія

1542 / 6307
У хворого з важким перебігом респіраторної вірусної інфекції з’явилися клінічні ознаки прогресуючої серцевої недостатності, яка призвела до смерті хворого на 2-му тижні захворювання. На розтині серце зі значним розширенням порожнин, в’яле. Гістологічно в міокарді виявляється повнокров’я мікросудин і дифузна інфільтрація строми лімфоцитами та гістіоцитами. Вкажіть найбільш вірогідний діагноз:

Міокардит

Гостра коронарна недостатність

Кардіоміопатія

Інфаркт міокарда

Стенокардія

1543 / 6307
В препараті, зафарбованому за методом Ожешко, видно паличкоподібні мікроорганізми, зафарбовані в синій колір, в яких термінально розміщені компоненти округлої форми, зафарбовані в червоний колір. Як називаються ці компоненти?

Джгутики

Мезосоми

Спори

Війки

Капсули

1544 / 6307
Проведена пункційна біопсія печінки хворому 38-ми років з виразною жовтяницею, дрібними крововиливами у шкірі, загальною слабкістю, втратою апетиту. Патологічне дослідження виявило поширену дистрофію, некроз гепатоцитів, наявність тілець Каунсілмена. По периферії часточок значна інфільтрація лімфоцитами, зустрічаються окремі багатоядерні гепатоцити. Який найбільш вірогідний діагноз?

Хронічний гепатит

Вірусний гепатит гострий

Міліарний цироз печінки

Алкогольний гепатит гострий

Токсична дистрофія печінки

1545 / 6307
До реанімаційного відділення надійшов чоловік з пораненням задньої ділянки шиї (regio nuchae). Який з м’язів тіла займає цю ділянку?

M.latissimus dorsi

M.sternocleidomastoideus

M.trapezius

M.rhomboideus minor

M.scalenus anterior

1546 / 6307
В експерименті на жабі зруйнували лабіринт з правого боку. До зниження тонусу яких м’язів це призведе?

Екстензори праворуч та ліворуч

Флексори праворуч

Екстензори праворуч

Екстензори ліворуч

Флексори ліворуч

1547 / 6307
Хвора 20-ти років померла від інтоксикації через 8 днів після штучного позалікарняного аборту при терміні вагітності 14-15 тижнів. На розтині тіла померлої: жовтувате забарвлення склери очей, шкіри, гнійно-некротичний ендометрит, багаточисельні гнійники в легенях, гіперплазія селезінки з великою кількістю нейтрофілів в її синусах. Яке ускладнення після аборту розвинулося у хворої?

Септикопіємія

Геморагічний шок

Хроніосепсис

Вірусний гепатит А

Септицемія

1548 / 6307
Хворий скаржиться на запаморочення і втрату слуху. Який нерв ушкоджений?

Під’язиковий

Блоковий

Трійчастий

Присінково-завитковий

Блукаючий

1549 / 6307
До лікарні надійшла дитина 6-ти років. Під час обстеження було виявлено, що дитина не може фіксувати погляд, не слідкує за іграшками, на очному дні відзначається симптом 'вишневої кістки'. Лабораторні аналізи показали, що у мозку, печінці та селезінці - підвищений рівень гангліозиду глікометиду. Яке спадкове захворювання у дитини?

Хвороба Тея-Сакса

Хвороба Німана-Піка

Хвороба Мак-Аргдля

Синдром Шерешевського-Тернера

Хвороба Вільсона-Коновалова

1550 / 6307
У хворого на шиї виявлено пакет спаяних між собою лімфовузлів щільної консистенції. При гістологічному обстеженні видаленого лімфовузла відмічається проліферація ретикулярних клітин, наявність клітин Березовського-Штернберга. Про яке захворювання йдеться?

Лімфобластний лейкоз

Лімфогранулематоз

Мієлобластний лейкоз

Мієлоцитарний лейкоз

Лімфоцитарний лейкоз

1551 / 6307
Під час пальпації молочної залози у хворої виявлено ущільнення у вигляді вузла у нижньому медіальному квадранті. У які лімфатичні вузли можуть поширитися метастази при цьому?

Бронхолегеневі

Глибокі латеральні шийні

Задні середостінні

Верхні діафрагмальні

Пригруднинні

1552 / 6307
Вагітній жінці, під час встановлення на облік у жіночу консультацію, було проведено клінічне обстеження на ряд інфекцій. У сироватці крові були виявлені lgM до вірусу краснухи. Про що свідчить такий результат?

Загострення хронічного процесу

Повторне інфікування вірусом краснухи

Первинне зараження жінки

Хронічний процес

Жінка здорова

1553 / 6307
Батьки для профілактики кишкових інфекцій у дитини 3-х років тривало застосовували антибіотики. Через місяць стан дитини погіршився. У крові - виражена лейкопенія і гранулоцитопенія. Який найбільш вірогідний механізм виявлених змін у крові?

Гемолітичний

Мієлотоксичний

Аутоімунний

Перерозподільний

Віковий

1554 / 6307
У результаті патологічного процесу в бронхах відбувається десквамація епітелію. За рахунок яких клітин відбуватиметься регенерація бронхіального епітелію?

Ендокринні

Вставочні

Базальні

Війчасті

Келихоподібні

1555 / 6307
Хворий, що страждав на туберкульоз, помер від прогресуючої легенево-серцевої недостатності. На розтині в ділянці верхівки правої легені визначається порожнина діаметром 5 см, яка сполучається з просвітом сегментарного бронха. Стінки порожнини з середини покриті сирнистими масами, під якими знаходяться епітеліоїдні клітини і клітини Пирогова-Лангханса. Вкажіть морфологічну форму туберкульозу:

Гострий кавернозний туберкульоз

Туберкульома

Гострий осередковий туберкульоз

Казеозна пневмонія

Інфільтративний туберкульоз

1556 / 6307
Робітники парникового господарства працюють в умовах несприятливого мікроклімату: температура повітря +37oC, відносна вологість 90%, швидкість руху повітря 0,2 м/с. Яким шляхом здійснюється тепловіддача за цих умов?

Усі зазначені шляхи

Випаровування

Радіація

Конвекція

Теплопроведення

1557 / 6307
При огляді хворої лікар-гінеколог відмітив симптоми запалення статевих шляхів. У мазку, взятому із піхви виявлено овально-грушоподібні найпростіші з шипом, з передньої частини яких відходять джгутики; наявна ундулююча мембрана. Яке захворювання підозрює лікар у хворої?

Токсоплазмоз

Лямбліоз

Кишковий трихомоноз

Балантидіоз

Урогенітальний трихомоноз

1558 / 6307
У хлопчика I (IoIo) група крові, а в його сестри IV(IAIB). Які групи крові у батьків цих дітей?

II (ІАІА) та III (IBIo)

III (IBIo) та IV (IАIB)

II (IAIо) та III (IBIo)

I (IoIo) та IV (IAIB)

I (IoIo) та III (IBIo)

1559 / 6307
У хворого на ентерит, що супроводжувався значною діареєю, спостерігається зменшення кількості води в позаклітинному просторі, збільшення її всередині клітин та зниження осмолярності крові. Як називають таке порушення водно-електролітного обміну?

Гіпоосмолярна гіпогідратація

Гіперосмолярна гіпергідратація

Гіперосмолярна гіпогідратація

Осмолярна гіпогідратація

Гіпоосмолярна гіпергідратація

1560 / 6307
У клініці для лікування інфаркту міокарда пацієнту введено ембріональні стовбурові клітини, що одержано шляхом терапевтичного клонування в цього ж пацієнта. Як називається цей вид трансплантації?

Гетеротрансплантація

Ізотрансплантація

Алотрансплантація

Ксенотрансплантація

Аутотрансплантація

1561 / 6307
На гістологічному препараті селезінки виявлена судина, стінка якої складається з ендотелію та субендотеліального шару, середня оболонка відсутня, зовнішня оболонка зрощена зі сполучнотканинними прошарками селезінки. Що це за судина?

Артеріола

Вена безм’язового типу

Артерія м’язового типу

Капіляр

Вена м’язового типу

1562 / 6307
В умовному експерименті повністю інгібовано розвиток клітин мезенхіми. Порушення розвитку якої м’язової тканини при цьому буде спостерігатися?

Серцева м’язова тканина

М’язова тканина епідермального походження

Скелетна м’язова тканина

М ’язова тканина нейрального походження

Гладенька м’язова тканина

1563 / 6307
Хворому на хронічний гастрит зроблена внутрішньошлункова рН-метрія, за допомогою якої встановлено зниження кислотності шлункового соку. Функція яких клітин знижена?

Парієтальні екзокриноцити

Ендокриноцити

Шийкові клітини

Додаткові клітини

Головні екзокриноцити

1564 / 6307
В експерименті на тварині здійснили перерізку блукаючих нервів з обох боків. Як при цьому зміниться характер дихання?

Стане глибоким і частим

Дихання не зміниться

Стане поверхневим та частим

Стане поверхневим та рідким

Стане глибоким і рідким

1565 / 6307
Чоловік 70-ти років хворіє на атеросклероз судин нижніх кінцівок та ішемічну хворобу серця. Під час обстеження виявлено порушення ліпідного складу крові. Надлишок яких ліпопротеїнів є головною ланкою в патогенезі атеросклерозу?

Хіломікрони

Проміжної щільності

Високої щільності

Низької щільності

Холестерин

1566 / 6307
У хворого після операції резекції шлунка на 2-3-й день не відновилася перистальтика кишок. Що потрібно призначити хворому для стимуляції функції шлунково-кишкового тракту?

Норадреналіну гідротартат

Прозерин

Циклодол

Празозин

Атропіну сульфат

1567 / 6307
Хворому чоловіку 75-ти років, у якого частота серцевих скорочень була 40/хвилину, імплантували серцевий електростимулятор. Після цього частота серцевих скорочень зросла до 70/хв. Функцію якого відділу серця взяв на себе електростимулятор?

Волокна Пуркін’є

Атріовентрикулярний вузол

Волокна пучка Гіса

Ніжки Гіса

Синоатріальний вузол

1568 / 6307
Стоматолог з метою зменшення салівації під час пломбування зуба хворому призначив препарат. Що це за лікарський засіб?

Адреналіну гідрохлорид

Прозерин

Атропіну сульфат

Пілокарпіну гідрохлорид

Мезатон

1569 / 6307
У жінки 30-ти років після тривалого використання губної помади з флюоресцуючою речовиною на облямівці губ розвинулася обмежена еритема, незначне лущення, пізніше поперечні дрібні борозни та тріщини. Після спеціальних методик при мікроскопічному дослідженні цієї зони ураження: в сполучній тканині наявність сенсибілізованих лімфоцитів і макрофагів, явища цитолізу. Який тип імунологічної гіперчутливості розвинувся на губі?

Гранулематоз

IV тип (клітинна цитотоксичність)

II тип (антитільна цитотоксичність)

III тип (імунокомплексна цитотокси-чність)

I тип (реагінового типу)

1570 / 6307
У хворого на ессенціальну артеріальну гіпертензію розвинувся гіпертонічний криз, що призвело до нападу серцевої астми. Який механізм серцевої недостатності є провідним в даному випадку?

Пошкодження міокарда

Перевантаження серця підвищеним опором

Порушення надходження крові до серця

Абсолютна коронарна недостатність

Перевантаження серця збільшеним об’ємом крові

1571 / 6307
Людина, що тривалий час приймає ліки, не може різко припинити їх вживання, оскільки при цьому виникають порушення психічних та соматичних функцій. Як називається синдром різних порушень при відмові від прийому речовини?

Тахіфілаксія

Кумуляція

Сенсибілізація

Абстиненція

ідіосинкразія

1572 / 6307
Хворий звернувся до стоматолога зі скаргами на гнійне запалення ясен. Який препарат буде найбільш ефективним, якщо припускається анаеробна природа збудника?

Оксацилін-натрій

Нітроксолін

Гентаміцин

Ко-тримоксазол

Метронідазол

1573 / 6307
У щурів, що знаходяться у стані стресу, підвищені м’язовий тонус та артеріальний тиск, збільшений вміст глюкози у крові, посилена секреція кортикотропіну і кортикостероїдів. У якій фазі стресу знаходяться ці тварини?

Виснаження

Фаза шоку

Еректильна

Термінальна

Фаза протишоку

1574 / 6307
При ультрамікроскопічному дослідженні популяції 'темних' гепатоцитів у цитоплазмі клітин визначено розвинену гранулярну ендоплазматичну сітку. Яку функцію в даних клітинах виконує ця органела?

Депонування іонів кальцію

Синтез білків плазми крові

Синтез вуглеводнів

Дезінтоксикаційна

Продукція жовчі

1575 / 6307
В лікарню звернувся хворий зі скаргами на швидку стомлюваність і виражену м’язову слабкість. При обстеженні виявлено аутоімунне захворювання, внаслідок якого порушується функціональний стан рецепторів у нервово-м’язових синапсах. Дія якого медіатора буде заблокована?

Норадреналін

Гліцин

Ацетилхолін

Дофамін

Серотонін

1576 / 6307
У пацієнта 35-ти років виявили підвищену кислотність шлункового соку. Блокада яких рецепторів може спричинити її зниження?

α1 -адренорецептори

Гістамінові

α2 -адренорецептори

β2 -адренорецептори

β1 -адренорецептори

1577 / 6307
По ходу слухового нерва у молодої жінки виявлена пухлина у вигляді вузла до 3 см в діаметрі, м’яко-еластичної консистенції,

рожево-білого кольору, однорідна. Мікроскопічно пухлина містить пучки клітин з овальними ядрами. Клітинно-волокнисті пучки формують ритмічні структури, створені паралельними рядами, правильно орієнтованими клітинами, розташованими у вигляді частоколу, поміж яких знаходиться безклітинна гомогенна зона (тільця Верокаї). Що це за пухлина?

Гангліонейробластома

Злоякісна невринома

Невринома

Гангліоневрома

Нейробластома

1578 / 6307
Жінка 45-ти років хворіє на рак лівої молочної залози. На лівій руці є ознаки недостатності лімфатичної системи - набряк кінцівки, збільшення лімфовузлів. Яка форма недостатності лімфообміну спостерігається у хворої?

Динамічна недостатність

Змішана недостатність

Механічна недостатність

Резорбційна недостатність

1579 / 6307
Для морфологічного дослідження представлена ендокринна залоза, паренхіма якої складається з епітелію та нервової тканини. В епітеліальних трабекулах виявляється два типи клітин: хромофільні та хромофобні. Визначте даний орган:

Гіпоталамус

Надниркова залоза

Щитоподібна залоза

Гіпофіз

Прищитоподiбна залоза

1580 / 6307
Під час операції на пахвинному каналі з приводу грижі, хірург пошкодив його вміст. Що саме пошкодив хірург?

Funiculus spermaticus

Lig. teres uteri

Lig. inguinale

Urarchus

1581 / 6307
У дитини 12-ти років непереносимість ряду харчових продуктів. Їх вживання викликає алергічну реакцію у вигляді висипань на шкірі, що сверблять. Який протигістамінний засіб слід призначити, щоб не заважати шкільним заняттям дитини (не викликати сонливість)?

Лоратадин

Димедрол

Диклофенак-натрію

Мезатон

Еуфілін

1582 / 6307
Чоловік 26-ти років перебуває в торпідній стадії шоку внаслідок автомобільної аварії. В крові: лейк.- 3, 2 • 109 /л. Який головний механізм в розвитку лейкопенії?

Пригнічення лейкопоезу

Порушення виходу зрілих лейкоцитів з кісткового мозку в кров

Перерозподіл лейкоцитів у судинному руслі

Підвищення виділення лейкоцитів з організму

Руйнування лейкоцитів у кровотворних органах

1583 / 6307
У хворого геморагічний інсульт. Виявлено в крові підвищену концентрацію кінінів. Лікар призначив хворому контрікал. Для гальмування якої протеїнази було зроблено це призначення?

Калікреїн

Трипсин

Пепсин

Хімотрипсин

Колагеназа

1584 / 6307
Для лікування бактеріальної пневмонії було призначено бензилпеніциліну натрієву сіль. Який механізм антимікробної дії препарату?

Пригнічення SH-груп ферментів мікроорганізмів

Пригнічення внутрішньоклітинного синтезу білка

Пригнічення активності холінестерази

Пригнічення синтезу клітинної стінки мікроорганізмів

Антагонізм з параамінобензойною кислотою

1585 / 6307
Хвороба Хартнепа зумовлена точковою мутацією лише одного гена, наслідком чого є порушення всмоктування амінокислоти триптофану в кишечнику та реабсорбції її в ниркових канальцях. Це призводить до одночасних розладів у травній і сечовидільній системах. Яке генетичне явище спостерігається в цьому випадку?

Полімерія

Неповне домінування

Плейотропія

Кодомінування

Комплементарна взаємодія

1586 / 6307
З метою прискорення загоєння променевої виразки був призначений вітамінний препарат. Вкажіть його:

Метилурацил

Левамізол

Ретаболіл

Преднізолон

Ретинолу ацетат

1587 / 6307
При посіві матеріалу із зіву від хворого ангіною на кров’яно-телуритовий агар виросли колонії діаметром 4-5 мм, сірого кольору, радіально покреслені (у вигляді розеток). Під мікроскопом - грампозитивні палички із булавоподібними потовщеннями на кінцях, що розміщені у вигляді розтопирених пальців. Які це мікроорганізми?

Дифтероїди

Коринебактерії дифтерії

Стрептобацили

Стрептококи

Клостридії ботулізму

1588 / 6307
Плазмові фактори згортання крові зазнають посттрансляційної модифікації за участю вітаміну K. Як кофактор, він потрібен у ферментній системі у-карбоксилювання білкових факторів коагуляції крові, завдяки збільшенню спорідненості їх молекул з іонами кальцію. Яка амінокислота карбоксилюється в цих білках?

Аргінін

Валін

Глутамінова

Фенілаланін

Серин

1589 / 6307
Фармакологічні ефекти антидепресантів пов’язані з блокуванням (інгібуванням) ними ферменту, який каталізує розпад таких біогенних амінів, як норадреналін і серотонін в мітохондріях нейронів головного мозку. Який фермент бере участь у цьому процесі?

Моноамінооксидаза

Ліаза

Декарбоксилаза

Пептидаза

Трансаміназа

1590 / 6307
Онкологічному хворому призначили препарат метотрексат, до якого з часом клітинимішені пухлини втратили чутливість. Експресія гену якого ферменту при цьому змінюється?

Фолатоксидаза

Дегідрофолатредуктаза

Тиміназа

Фолатдекарбоксилаза

Дезаміназа

1591 / 6307
При диспансерному обстеженні хлопчику 7-ми років встановлено діагноз - синдром ЛешаНайхана (хворіють тільки хлопчики). Батьки здорові, але у дідуся за материнською лінією таке саме захворювання. Який тип успадкування захворювання?

Рецесивний, зчеплений із статтю

Домінантний, зчеплений із статтю

Автосомно-домінантний

Автосомно-рецесивний

Неповне домінування

1592 / 6307
Значну роль у профілактиці туберкульозу відіграє планова масова вакцинація проти туберкульозу новонароджених дітей віком 5-7 днів життя. При цьому застосовують вакцину:

БЦЖ

АДП

АКДП

АД

АКП

1593 / 6307
До кардіологічного відділення надійшов хворий з гіпертонічним кризом, йому внутрішньовенно ввели антигіпертензивний засіб - сіль лужноземельного металу. Який препарат ввели хворому?

Бензогексоній

Калію хлорид

Кальцію лактат

Магнію сульфат

Натрію гідрокарбонат

1594 / 6307
Під час оперативного втручання на малому тазі жінки виникла необхідність перев’язати яєчникову артерію. Яке з утворень може бути випадково перев’язаним разом з нею?

Маткова труба

Кругла зв’язка матки

Сечівник

Внутрішня клубова вена

Сечовід

1595 / 6307
До приймального відділення лікарні доставлено людину з гострим отруєнням морфіном. Який специфічний антагоніст наркотичних анальгетиків необхідно застосувати в цьому випадку?

Налоксон

Парацетамол

Дигоксин

Метацин

Унітіол

1596 / 6307
Верхня частка правої легені збільшена, сірого кольору, безповітряна, з поверхні розрізу стікає каламутна рідина, на плеврі багато фібринозних плівок; мікроскопічно в альвеолах виявляється ексудат з присутністю нейтрофілів, десквамованих альвеолоцитів і ниток фібрину. Стінка бронха інтактна. Який найбільш вірогідний діагноз?

Абсцес легень

Грипозна пневмонія

Крупозна пневмонія

Вогнищева пневмонія

Інтерстиціальна пневмонія

1597 / 6307
У хворого 28-ми років відзначалися підвищення артеріального тиску, гематурія та набряки на обличчі. Незважаючи на лікування, наростали явища ниркової недостатності. Через 6 місяців хворий помер від уремії. Мікроскопічно при дослідженні нирок у клубочках виявлена проліферація нефротелію капсули, подоцитів з утворенням 'півмісяців', склероз та гіаліноз клубочків. Який найбільш вірогідний діагноз?

Хронічний гломерулонефрит

Підгострий гломерулонефрит

Гострий пієлонефрит

Гострий гломерулонефрит

Нефротичний синдром

1598 / 6307
До клініки швидкої допомоги надійшов хворий з ножовим пораненням лівої поперекової ділянки. У ході операції хірург встановив, що внутрішні органи не пошкоджені, але лезо ножа пошкодило один із м’язів ниркового ложа. Назвіть цей м’яз:

Випрямляч хребта

Зовнішній косий м’яз живота

Великий поперековий м’яз

Клубовий м’яз

Внутрішній косий м’яз живота

1599 / 6307
Чоловік з колотою раною в ділянці чотирьохстороннього отвору звернувся до лікаря. При обстеженні виявлено, що потерпілий не може відвести руку від тулуба. Який нерв вірогідно ушкоджений?

N.subclavius

N.medianus

N.ulnaris

N.axillaris

N.radialis

1600 / 6307
Під час розтину тіла чоловіка, що за життя страждав на важку форму гіпотиреозу, виявлено: сполучна тканина, строма органів, жирова і хрящова тканини набряклі, напівпрозорі, слизоподібні. Мікроскопічно у тканинах виявляються зірчасті клітини з відростками, між якими знаходиться слиз. Назвіть вид дистрофії:

Стромально-судинна білкова

Стромально-судинна вуглеводна

Паренхіматозна білкова

Стромально-судинна жирова

Паренхіматозна жирова

1601 / 6307
Хворий 30-ти років звернувся до лікаря зі скаргами на пронос і біль у животі протягом 5-ти днів, підвищення температури тіла до 37,5oC з ознобами. Напередодні хворий був у лісі, де випив води з відкритого водоймища. Встановлено лабораторно підтверджений діагноз: амебна дизентерія. Вкажіть препарат вибору для лікування цього захворювання:

Левоміцетин

Фуразолідон

Фталазол

Еметина гідрохлорид

Метронідазол

1602 / 6307
Чоловік, що страждає на спадкову хворобу, одружився із здоровою жінкою. У них було 5 дітей, три дівчинки і два хлопчика. Усі дівчатка успадкували хворобу батька. Який тип спадкування цього захворювання?

Зчеплений з Y-хромосомою

Рецесивний, зчеплений з X -хромосомою

Домінантний, зчеплений з X -хромосомою

Аутосомно-рецесивний

Аутосомно-домінантний

1603 / 6307
На електронній мікрофотографії представлена клітина макрофагічної природи, вздовж відростків якої розташовуються еритроцити на різних стадіях диференціювання. Клітина якого органу представлена?

Мигдалик

Селезінка

Червоний кістковий мозок

Тимус

Лімфатичний вузол

1604 / 6307
Досить часто причиною набутих імунодефіцитів є інфекційне ураження організму, при якому збудники безпосередньо розмножуються в клітинах імунної системи і руйнують їх. Виберіть серед перерахованих ті захворювання, при яких має місце вищезгадане:

Ку-гарячка, висипний тиф

Інфекційний мононуклеоз, СНІД

Поліомієліт, гепатит А

Дизентерія, холера

Туберкульоз, мікобактеріоз

1605 / 6307
В шкірі виявлена щільна, рухома, чітко відмежована від оточуючих тканин пухлина. На розрізі вона білого кольору, представлена волокнистою тканиною. Мікроскопічно: хаотично переплетені колагенові волокна, клітин мало. Що це за пухлина?

Міома

Дерматофіброма

Фіброма

Десмоїд

Гістіоцитома

1606 / 6307
У хворого після оперативного видалення кісти підшлункової залози виник геморагічний синдром з вираженим порушенням зсідання крові. Розвиток цього ускладнення пояснюється:

Недостатнім утворенням фібрину

Активацією фактору Крисмаса

Активацією фібринолітичної системи

Активацією протизгортальної системи

Зменшенням кількості тромбоцитів

1607 / 6307
Мати звернулася до лікаря з приводу того, що у дитини 5-ти років під дією сонячних променів на шкірі з’являються еритеми, везикулярний висип, свербіж шкіри. Лабораторні дослідження виявили зменшення вмісту заліза у сироватці крові, збільшення виділення з сечею уропорфіриногену І. Найбільш вірогідною спадковою патологією у дитини є:

Еритропоетична порфірія

Метгемоглобінемія

Інтермітуюча порфірія

Печінкова порфірія

Копропорфірія

1608 / 6307
Немовля відмовляється від годування груддю, збудливе, дихання неритмічне, сеча має специфічний запах 'пивної закваски' або 'кленового сиропу'. Вроджений дефект якого ферменту викликав дану патологію?

Глюкозо-6-фосфатдегідрогеназа

Гліцеролкіназа

УДФ-глюкуронілтрансфераза

Дегідрогеназа розгалужених альфа-кетокислот

Аспартатамінотрансфераза

1609 / 6307
У мікропрепараті, виготовленому з пунктату регіонарного лімфовузла хворого, зафарбованому за Романовським-Гімзою, лікар виявив тонкі мікроорганізми з 12-14 рівномірними завитками з гострими кінцями, довжиною 10-13 мкм блідо-рожевого кольору. Про збудника якої інфекційної хвороби може йти мова в даному випадку?

Сифіліс

Поворотний тиф

Трипаносомоз

Лейшманіоз

Лептоспіроз

1610 / 6307
При санітарно-бактеріологічному дослідженні води методом мембранних фільтрів виявлено дві червоні колонії на мембранному фільтрі (середовище Ендо), через який пропустили 500 мл досліджуваної води. Розрахуйте колі-індекс та колі-титр досліджуваної води:

2 та 500

250 та 2

500 та 2

4 та 250

250 та 4

1611 / 6307
Лікар-отоларинголог при огляді хворого відмітив гіперемію, значний набряк мигдаликів з сірим нальотом на них. При мікроскопії нальоту було виявлено грам-позитивні палички, розташовані під кутом одна до одної. Про яке захворювання слід думати?

Епідемічний паротит

Ангіна

Менінгоназофарингіт

Скарлатина

Дифтерія

1612 / 6307
До приймального відділення доставлено жінку 38-ми років з матковою кровотечею, що триває другу добу. Що з наведеного буде виявлено при аналізі крові хворої?

Уповільнення ШОЕ

Зменшення гематокритного показника

Лейкоцитоз

Еозинофілія

Збільшення колірного показника

1613 / 6307
У чоловіка після гіпертонічної кризи відзначається відсутність довільних рухів в правих руці та нозі, тонус м’язів у цих кінцівках підвищений. Який вид розладу рухової функції спостерігається у даному випадку?

Периферичний парез

Центральний параліч

Рефлекторний парез

Периферичний параліч

Центральний парез

1614 / 6307
У хворого в організмі знижений вміст іонів магнію, які потрібні для прикріплення рибосом до гранулярної ендоплазматичної сітки. Відомо, що це призводить до порушення біосинтезу білка. Який саме етап біосинтезу білка буде порушено?

Активація амінокислот

Транскрипція

Трансляція

Реплікація

Термінація

1615 / 6307
Хворий з виразковою хворобою шлунка приймав антацидний препарат альмагель. Для лікування гострого бронхіту йому призначили антибіотик метициклін. Проте протягом 5- ти днів температура не знизилася, кашель і характер харкотиння не змінились. Лікар прийшов до висновку про несумісність ліків при їх взаємодії. Про який саме вид несумісності ліків йдеться?

Фармацевтична

Фармакодинамічна

Прямий антагонізм

Фармакокінетична на етапі біотрансформації

Фармакокінетична на етапі всмоктування

1616 / 6307
У хворого 70-ти років атеросклероз ускладнився тромбозом судин нижніх кінцівок, виникла гангрена пальців лівої стопи. Початок тромбоутворення, найбільш вірогідно, пов’язаний з:

Перетворенням фібриногену в фібрин

Перетворенням протромбіну в тромбін

Зниженням синтезу гепарину

Адгезією тромбоцитів

Активацією протромбінази

1617 / 6307
У хворого 44-х років на ЕКГ виявлені ознаки гіпертрофії обох шлуночків та правого передсердя. Діагностовано недостатність тристулкового клапана. Який патогенетичний варіант порушення функції серця має місце при цій недостатності?

Перевантаження серця об’ємом

Тампонада серця

Перевантаження серця опором

Коронарна недостатність

Первинна міокардіальна недостатність

1618 / 6307
Бактеріологічне дослідження гнійних виділень з уретри виявило наявність бактерій, які за Грамом фарбувалися негативно, нагадували кавові зернини, розкладали глюкозу і мальтозу до кислоти, розташовувалися в лейкоцитах. Збудники якої хвороби виявлені?

Гонорея

М ’який шанкр

Меліоїдоз

Сифілісу

Венеричний лімфогранулематоз

1619 / 6307
Прозерин при системному введенні підвищує тонус скелетних м’язів. Фторотан викликає релаксацію м’язів і послаблює ефекти прозерину. Який характер взаємодії прозерину та фторотану?

Прямий функціональний антагонізм

Незалежний антагонізм

Неконкурентний антагонізм

Конкурентний антагонізм

Непрямий функціональний антагонізм

1620 / 6307
У жінки 62-х років розвинулася катаракта (помутніння кришталика) на фоні цукрового діабету. Який тип модифікації білків має місце при діабетичній катаракті?

Метилювання

Обмежений протеоліз

Глікозилювання

АДФ-рибозилювання

Фосфорилювання

1621 / 6307
У дитини 2-х років виявлено відставання у фізичному розвитку, часті пневмонії. Встановлено діагноз - незарощення артеріальної протоки. Сполучення яких кровоносних судин викликало порушення гемодинаміки?

Верхня порожниста вена і легеневий стовбур

Аорта і легеневий стовбур

Верхня порожниста вена і аорта

Аорта і легеневі вени

Легеневий стовбур і легеневі вени

1622 / 6307
в середовищі з температурою 38o 22. C та відносною вологістю повітря 50%. Які шляхи тепловіддачі зумовлюють підтримку постійної температури ядра тіла за цих умов?

Конвекція і теплопроведення

Випаровування

Теплопроведення

Конвекція

Радіація

1623 / 6307
До медико-генетичної консультації звернулося подружжя з питанням про вірогідність народження у них дітей, хворих на гемофілію. Подружжя здорове, але батько дружини хворий на гемофілію. В цій родині на гемофілію можуть захворіти:

Половина дочок

Сини та дочки

Половина синів

Всі діти

Тільки дочки

1624 / 6307
У хворого з пересадженим серцем при фізичному навантаженні збільшився хвилинний об’єм крові. Який механізм регуляції забезпечує ці зміни?

Симпатичні безумовні рефлекси

Парасимпатичні безумовні рефлекси

Катехоламіни

Симпатичні умовні рефлекси

Парасимпатичні умовні рефлекси

1625 / 6307
На прийом до лікаря прийшла хвора зі скаргами на розлад травлення, розлитий біль у животі. При обстеженні виявлено різке зниження вмісту гемоглобіну в крові. З анамнезу відомо, що, під час перебування на Далекому Сході, вона часто вживала в їжу малосольну риб’ячу ікру. Аналогічний стан відзначений у деяких родичів, що проживають з нею. Яке захворювання найбільш вірогідне?

Трихінельоз

Теніоз

Ехінококоз

Дифілоботріоз

Аскаридоз

1626 / 6307
У людини нормальна чутливість шкіри пальця, але він не відчуває наявності на ньому обручки. Який процес, спричинений впливом обручки, є причиною цього?

Порушення структури епідермісу

Порушення кровообігу

Порушення структури рецепторів

Адаптація рецепторів

Розвиток фіброзної тканини

1627 / 6307
У хворого 45-ти років з підозрою на запалення оболонок мозку потрібно було отримати спинномозкову рідину Зроблено діагностичну пункцію між дугами поперекових хребців (L3 — S4).Через яку зв’язку пройшла голка під час пункції?

Міжпоперечна

Жовта

Передня поздовжня

Задня поздовжня

Клубово-поперекова

1628 / 6307
При аналізі ЕКГ людини з’ясовано, що у стандартних відведеннях від кінцівок зубці T позитивні, їх амплітуда та тривалість нормальні. Вірним є висновок, що у шлуночках серця нормально відбувається такий процес:

Скорочення

Розслаблення

Реполяризація

Збудження

Деполяризація

1629 / 6307
У хворого в крові збільшена концентрація пірувату. Значна його кількість екскретується з сечею. Дефіцит якого вітаміну має місце у хворого?

B1

E

В3

B6

В2

1630 / 6307
Людина захворіла на пелагру. При опитуванні стало відомо, що впродовж тривалого часу вона харчувалася переважно кукурудзою, мало вживала м’яса. Дефіцит якої речовини у кукурудзі спричинив розвиток хвороби?

Гістидин

Пролін

Тирозин

Триптофан

Аланін

1631 / 6307
При забарвленні бакпрепаратів, виготовлених з харкотиння, методом Ціля-Нільсена виявлено наявність яскраво-червоних паличок, які розміщувалися поодиноко або групами, не чутливі до дії кислот. На живильних середовищах перші ознаки росту з’являються на 10-і5-ту добу. До якої родини відносяться виявлені бактерії?

Klebsiella rhinoscleromatis

Micobacterium tuberculosis

Histoplasma dubrosii

Coxiella burnettii

Yersinia pseudotuberculosis

1632 / 6307
Пацієнтці з високим ступенем ожиріння у якості харчової добавки було рекомендовано карнітин для поліпшення 'спалювання'жиру. Яку безпосередню участь бере карнітин у процесі окиснення жирів?

Активація ВЖК

Бере участь в одній з реакцій бета-окиснення ВЖК

Активація внутрішньоклітинного ліполізу

Транспорт ВЖК з цитозоля до мітохондрій

Транспорт ВЖК з жирових депо до тканин

1633 / 6307
У дитини 2-х років з катаральними явищами та висипом на шкірі лікар запідозрив скарлатину. Внутрішньошкірно дитині ввели невелику кількість сироватки до еритрогенного токсину стрептокока, на місці ін’єкції висип зник. Що означають результати реакції?

У дитини підвищена чутливість до еритрогенного токсину

Захворювання викликав не гемолітичний стрептокок

Імунна система дитини дуже ослаблена

Всю дозу сироватки можна вводити внутрішньовенно

Клінічний діагноз підтвердився

1634 / 6307
У хворого поперечний розрив спинного мозку нижче VI грудного сегменту. Як внаслідок цього зміниться дихання?

Стане більш глибоким

Стане більш частим

Стане більш рідким

Суттєво не зміниться

Зупиниться

1635 / 6307
У хворої на бронхіальну астму вірусне інфікування спровокувало астматичний статус зі смертельним наслідком. При гістологічному дослідженні легень виявлено: спазм і набрякання бронхіол, в їх стінках виражена інфільтрація лімфоцитами, еозинофілами та іншими лейкоцитами, а також дегрануляція лаброцитів. Який механізм гіперчутливості лежить в основі зазначених змін?

Реагінова реакція

Аутоімунний

Імунокомплексний

Запальний

Імунозумовлений клітинний цитоліз

1636 / 6307
Мати виявила у 5-річної доньки на періанальних складках білих 'черв’ячків ', які викликали у дитини свербіж і неспокій, і доставила їх до лабораторії. Під час огляду лікар побачив білих гельмінтів 0,5-1 см довжиною, ниткоподібної форми з загостреними кінцями, у деяких кінці були закручені. Який найбільш вірогідний діагноз?

Опісторхоз

Ентеробіоз

Теніоз

Аскаридоз

Дифілоботріоз

1637 / 6307
У хворого 27-ми років виявлено патологічні зміни печінки і головного мозку. У плазмі крові виявлено різке зниження, а в сечі - підвищення вмісту міді. Встановлено діагноз - хвороба Вільсона. Активність якого ферменту в сироватці крові необхідно дослідити для підтвердження діагнозу?

Алкогольдегідрогеназа

Карбоангідраза

Ксантиноксидаза

Лейцинамінопептидаза

Церулоплазмін

1638 / 6307
Пацієнт звернувся до лікаря зі скаргами на задишку, що виникала після фізичного навантаження. Клінічне обстеження виявило анемію та наявність парапротеїну в зоні гамма-глобулінів. Який показник у сечі необхідно визначити для підтвердження діагнозу мієломи?

Білірубін

Гемоглобін

Церулоплазмін

Білок Бенс-Джонса

Антитрипсин

1639 / 6307
У чоловіка 45-ти років після зна- чного психоемоційного навантаження раптово з’явився стискаючий біль в ділянці серця з іррадіацією в ліву руку, шию, під ліву лопатку. Обличчя стало блідим, вкрилося холодним потом. Нітрогліцерин усунув напад болю. Який процес розвинувся у хворого?

Інфаркт міокарда

Психогенний шок

Стенокардія

Перфорація виразки шлунка

Інсульт

1640 / 6307
Жінка 62-х років скаржиться на частий біль у ділянці грудної клітки та хребта, переломи ребер. Лікар припустив мієломну хворобу (плазмоцитому). Який з перерахованих нижче лабораторних показників буде мати найбільше діагностичне значення?

Гіперальбумінемія

Гіпоглобулінемія

Парапротеїнемія

Протеїнурія

Гіпопротеїнемія

1641 / 6307
Дитині 8-ми років, що надійшла до інфекційного відділення з високою температурою (до 38oC), дрібноточковим яскраво-червоним висипом, було встановлено діагноз скарлатини. Об’єктивно: слизова оболонка зіву яскраво гіперемована, набрякла, мигдалики різко збільшені, з тьмяними вогнищами жовтувато-сірого кольору і ділянками чорного кольору. Яке запалення лежить в основі змін у зіві?

Гнійно-некротичне

Геморагічне

Серозне

Катаральне

Фібринозне

1642 / 6307
У новонародженої дитини на пелюшках виявлені темні плями, що свідчать про утворення гомогентизинової кислоти. З порушенням обміну якої речовини це пов’язане?

Тирозин

Метіонін

Холестерин

Галактоза

Триптофан

1643 / 6307
У гістологічному препараті визначаються рецепторна зона сенсоепітеліального органа чуттів. Клітини даної зони лежать на базальній мембрані і включають наступні види: зовнішні та внутрішні сенсорні, зовнішні та внутрішні фалангові, стовбурові, зовнішні межові і зовнішні підтримуючі. Вкажіть, якому органу чуттів належить зазначена рецепторна зона:

Слуху

Нюху

Рівноваги

Смаку

Зору

1644 / 6307
Хворий надійшов до відділення з нападом задухи, зумовленої спазмом гладенької мускулатури дихальних шляхів. Назвіть відділи повітроносних шляхів, зміна стану яких, в основному, спричинила даний напад:

Бронхи середнього калібру

Респіраторний відділ

Бронхи малого калібру

Кінцеві бронхіоли

Бронхи великого калібру

1645 / 6307
Під час обіду дитина поперхнулася і аспірувала їжу. Почався сильний кашель, шкіра і слизові ціанотичні, пульс частий, дихання рідке, видих подовжений. Яке порушення зовнішнього дихання розвинулося у дитини?

Експіраторна задишка при асфіксії

Альтернуюче дихання

Стенотичне дихання

Інспіраторна задишка при асфіксії

Дихання Біота

1646 / 6307
Хворий 50-ти років звернувся до клініки зі скаргами на загальну слабкість, втрату апетиту, аритмію серця. Спостерігається гіпотонія м’язів, мляві паралічі, послаблення перистальтики кишечнику. Причиною такого стану може бути:

Гіперкаліємія

Гіпопротеїнемія

Гіпофосфатемія

Гіпокаліємія

Гіпонатріємія

1647 / 6307
Під час гри у волейбол спортсмен після стрибка приземлився на зовнішній край стопи. Виник гострий біль у гомілковостопному суглобі, активні рухи в ньому обмежені, пасивні - в повному обсязі, але болісні. Потім розвинулася припухлість у ділянці зовнішньої щиколотки, шкіра почервоніла, стала теплішою на дотик. Який вид розладу периферичного кровообігу розвинувся в даному випадку?

Венозна гіперемія

Стаз

Тромбоз

Емболія

Артеріальна гіперемія

1648 / 6307
У хворого на міастенію після призначення прозерину з’явилися нудота, діарея, посмикування м’язів язика і скелетних м’язів. Чим можна усунути інтоксикацію?

Мезатон

Пірідостигміну бромід

Фізостигмін

Атропіну сульфат

Ізадрин

1649 / 6307
Чоловік помер від гострого інфекційного захворювання, яке супроводжувалось гарячкою, жовтяницею, геморагічною висипкою на шкірі та слизових оболонках, а також гострою нирковою недостатністю. При гістологічному дослідженні тканини нирки (забарвлення за Романовським-Гімзою) виявлені звивисті бактерії, які мають вигляд букв С та S. Які бактерії були виявлені?

Спіролли

Кампілобактерії

Лептоспіри

Боррелії

Трепонеми

1650 / 6307
У підлітка 12-ти років, який хворіє на бронхіальну астму, виник тяжкий напад астми: виражена експіраторна задишка, блідість шкірних покривів. Який вид порушення альвеолярної вентиляції має місце?

Нервово-м’язовий

Центральний

Обструктивний

Рестриктивний

Торако-діафрагмальний

1651 / 6307
Чоловіку 46-ти років, що хворіє на дифузний токсичний зоб, була проведена операція резекції щитоподібної залози. Після операції відмічаються відсутність апетиту, диспепсія, підвищена нервово-м’язова збудливість. Маса тіла не збільшилася. Температура тіла у нормі. Чим, із нижче переліченого, обумовлений стан хворого?

Підвищенням продукції кальцитоніну

Підвищенням продукції тиреоліберину

Підвищенням продукції тироксину

Зниженням продукції тироксину

Зниженням продукції паратгормону

1652 / 6307
Хворому, який переніс інфаркт міокарда, призначена ацетилсаліцилова кислота по 75 мг щоденно. З якою метою призначено препарат?

Зниження температури

Зменшення агрегації тромбоцитів

Зменшення болю

Розширення коронарних судин

Зменшення запалення

1653 / 6307
При медичному огляді у військкоматі був виявлений хлопчик 15-ти років, високого зросту, з євнухоїдними пропорціями тіла, гінекомастією, з ростом волосся на лобку за жіночим типом. Відмічається відкладання жиру на стегнах, відсутність росту волосся на обличчі, високий голос; коефіцієнт інтелекту знижений. Виберіть каріотип, що відповідає даному захворюванню:

46, XX

46, XY

47, XXX

45, XO

47, XXY

1654 / 6307
З метою попередження відторгнення трансплантата після пересадки органів обов’язковим є проведення курсу гормонотерапії з метою імуносупресії. Які гормони застосовують з цією метою?

Катехоламіни

Мінералокортикоїди

Глюкокортикоїди

Тиреоїдні

Статеві гормони

1655 / 6307
У хворої 53-х років внаслідок тупої травми живота діагностований розрив печінки. У якому анатомічному утворенні збереться кров, що вилилася?

Сальникова сумка

Лівий брижовий синус

Правий брижовий синус

Прямокишково-маткове заглиблення

Міхурово-маткове заглиблення

1656 / 6307
Хвора скаржиться на набряк ніг, посиніння шкіри, невеличкі виразки збоку латерального виростку. При обстеженні відмічено: припухлість, збільшення розмірів вен, утворення вузлів. З боку якої вени відмічається патологія?

V femoralis

V iliaca externa

V. saphena parva

V profunda femoris

V saphena magna

1657 / 6307
У дитини 1,5 років спостерігається відставання в розумовому і фізичному розвитку, посвітління шкіри і волосся, зниження вмісту в крові катехоламінів. При додаванні до свіжої сечі декількох крапель 5 % розчину трихлороцтового заліза з’являється оливковозелене забарвлення. Для якої патології обміну амінокислот характерні дані зміни?

Ксантонурія

Тирозиноз

Альбінізм

Фенілкетонурія

Алкаптонурія

1658 / 6307
При розтині тіла померлого чоловіка 73-х років, який довго хворів на ішемічну хворобу серця з серцевою недостатністю, знайдено: 'мускатна'печінка, бура індурація легень, ціанотична індурація нирок та селезінки. Вкажіть, який з видів порушення кровообігу в хворого призвів до таких наслідків?

Хронічне загальне венозне повнокров’я

Гостре загальне венозне повнокров’я

Гостре недокрів’я

Хронічне недокрів’я

Артеріальна гіперемія

1659 / 6307
Хворий з хронічним гіперацидним гастритом для усунення печії застосовує антацидний засіб, після прийому якого відзначає поліпшення, однак разом з тим виникає відчуття розпирання у шлунку. Який з наведених препаратів міг викликати вказаний побічний ефект?

Магнію окис

Магнію трисилікат

Натрію гідрокарбонат

Алюмінію гідроокис

Пепсин

1660 / 6307
При тривалому використанні препарату в хворого можуть мати місце остеопороз, ерозії слизової шлунка, гіпокаліємія, затримка натрію i води в організмі, зменшення вмісту кортикотропіну в крові. Укажіть цей препарат:

Дигоксин

Преднізолон

Резерпін

Гіпотіазид

Індометацин

1661 / 6307
Хворому тривалий час вводили високі дози гідрокортизону, внаслідок чого настала атрофія однієї з зон кори наднирників. Яка це зона?

Сітчаста

Клубочкова і сітчаста

Клубочкова

Пучкова

1662 / 6307
До лікарні швидкої допомоги госпіталізований чоловік 63-х років з явищами колапсу. Для боротьби з гіпотензією лікар вибрав норадреналін. Який механізм дії цього препарату?

Активація β-адренорецепторів

Активація α1 -адренорецепторів

Активація дофамінових рецепторів

Блокада M-холінорецепторів

Активація серотонінових рецепторів

1663 / 6307
Основна маса азоту з організму виводиться у вигляді сечовини. Зниження активності якого ферменту в печінці призводить до гальмування синтезу сечовини і нагромадження амоніаку в крові і тканинах?

Аспартатамінотрансфераза

Уреаза

Пепсин

Карбамоїлфосфатсинтаза

Амілаза

1664 / 6307
У хворого на сифіліс при лікуванні препаратами вісмуту з’явилися сірі плями на слизовій оболонці ротової порожнини та симптоми нефропатії. Який засіб доцільно використати у хворого для лікування отруєння препаратами вісмуту?

Унітіол

Бемегрид

Налорфін

Налоксон

Метиленовий синій

1665 / 6307
На ЕКГ пацієнта мають місце такі зміни: зубець P - нормальний, інтервал P — Q - вкорочений, шлуночковий комплекс QRST - розширений, зубець R -двогорбий або двофазний. Яка із форм аритмії має місце у даного пацієнта?

Миготлива аритмія

Атріовентрикулярна блокада

Синдром WPW (Вольфа-Паркінсона-Уайта)

Синдром Фредеріка (тріпотіння передсердь)

Миготіння шлуночків

1666 / 6307
У гістологічному препараті паренхіма органа представлена лімфоїдною тканиною, яка утворює лімфатичні вузлики; останні розташовуються дифузно і містять центральну артерію. Яке анатомічне утворення має таку морфологічну будову?

Червоний кістковий мозок

Селезінка

Тимус

Мигдалик

Лімфатичний вузол

1667 / 6307
У хворого з частими кровотечами з внутрішніх органів і слизових оболонок виявлені пролін і лізин у складі колагенових волокон. Через відсутність якого вітаміну порушено їх гідрокси-лювання?

Тіамін

Вітамін A

Вітамін K

Вітамін C

Вітамін E

1668 / 6307
При диспансерному обстеженні хлопчику 7-ми років встановлено діагноз - дальтонізм. Батьки здорові, кольоровий зір у них у нормі, але у дідуся по материнській лінії така ж аномалія. Який тип успадкування цієї аномалії?

Аутосомно-домінантний

Домінантний, зчеплений зі статтю

Неповне домінування

Рецесивний, зчеплений зі статтю

Аутосомно-рецесивний

1669 / 6307
У здорової дорослої людини швидкість проведення збудження через атріовентрикулярний вузол дорівнює 0,02-0,05 м/с. Атріовентрикулярна затримка забезпечує:

Одночасність скорочення обох шлуночків

Достатню силу скорочення передсердь

Одночасність скорочення обох передсердь

Достатню силу скорочення шлуночків

Послідовність скорочення передсердь та шлуночків

1670 / 6307
На гістологічному препарат нирки представлена ділянка дистального канальцю нефрону, що проходить між приносною та виносною артеріолами. В клітинах, що складають стінку ка-нальцю, наявні ущільнені ядра, відсутня базальна мембрана. Як зветься це структурне утворення?

Юкстагломерулярні клітини

Мезангіальні клітини

Клітини Гурмагтига

Щільна пляма

Юкставаскулярні клітини

1671 / 6307
Внаслідок дії електричного струму на збудливу клітину виникла деполяризація її мембрани. Вхід яких іонів через мембрану до клітини відіграв основну роль в розвитку деполяризації?

HCO3-

K+

Cl

Na+

Ca2+

1672 / 6307
Продуктами гідролізу та модифікації деяких білків є біологічно активні речовини - гормони. З якого із наведених білків у гіпофізі утворюються ліпотропін, кортикотропін, меланотропін та ендорфіни?

Проопіомеланокортин (ПОМК)

Нейроальбумін

Тиреоглобулін

Нейростромін

Нейроглобулін

1673 / 6307
У хворих з непрохідністю жовчовивідних шляхів пригнічується зсідання крові, виникають кровотечі, що є наслідком недостатнього засвоєння такого вітаміну:

A

E

K

D

C

1674 / 6307
Хвора 48-ми років надійшла до клініки із скаргами на слабкість, дратівливість, порушення сну. Об’єктивно: шкіра та склери жовтого кольору. У крові: підвищення рівня загального білірубіну з переважанням прямого. Кал - ахолічний. Сеча - темного кольору (жовчні пігменти). Яка жовтяниця має місце в хворої?

Гемолітична

Синдром Кріглера-Найяра

Механічна

Паренхіматозна

Синдром Жільбера

1675 / 6307
У баклабораторії під час мікроскопії мазків з харкотиння хворого на хронічне легеневе захворювання, забарвлених за Цілем-Нільсеном, виявлені червоні палички. Яка властивість туберкульозної палички виявлена при цьому?

Капсулоутворення

Спороутворення

Лугостійкість

Кислотостійкість

Спиртостійкість

1676 / 6307
Під час мікроскопічного дослідження збільшеного шийного лімфатичного вузла визначається стертя його структури, лімфоїдні фолікули відсутні, усі поля зору представлені клітинами з округлими ядрами і вузьким обідком базофільної цитоплазми. З клінічних даних відомо, що збільшені і інші групи лімфовузлів, а також селезінка та печінка. Про яке захворювання слід думати?

Мієлоїдний лейкоз

Мієломна хвороба

Лімфоїдний лейкоз

Лімфосаркома

Лімфогранулематоз

1677 / 6307
У юнака 20-ти років, через 2 тижні після перенесеної лакунарної ангіни, з’явилися скарги на загальну слабкість, набряки під очима. Після обстеження хворому встановлено діагноз: гострий гломерулонефрит. Які патологічні зміни у складі сечі найбільш вірогідні?

Циліндрурія

Натрійурія

Протеїнурія

Наявність свіжих еритроцитів

Піурія

1678 / 6307
До травматологічного пункту звернувся чоловік 38-ми років з травмою правої кисті. При огляді встановлено: різана рана в ділянці підвищення великого пальця правої кисті; дистальна фаланга I пальця не згинається. Який м’яз пошкоджено?

Короткий відвідний м’яз великого пальця

Протиставний м’яз великого пальця

Довгий м’яз-згинач великого пальця

Короткий м’яз-згинач великого пальця

Привідний м’яз великого пальця

1679 / 6307
У людини збільшена вентиляція легень внаслідок фізичного навантаження. Який з наведених показників зовнішнього дихання у неї значно більший, ніж у стані спокою?

Дихальний об’єм

Резервний об’єм видиху

Життєва ємність легень

Загальна ємність легень

Резервний об’єм вдиху

1680 / 6307
У людини внаслідок тривалого голодування швидкість клубочкової фільтрації зросла на 20%. Найбільш вірогідною причиною змін фільтрації в зазначених умовах є:

Збільшення ниркового кровотоку

Зменшення онкотичного тиску плазми крові

Збільшення системного артеріального тиску

Збільшення проникності ниркового фільтру

Збільшення коефіцієнта фільтрації

1681 / 6307
У пацієнта після переливання 200 мл крові підвищилася температура тіла до 37,9o C. Яка з наведених речовин найбільш вірогідно призвела до підвищення температури?

Фактор некрозу пухлин

Інтерлейкін-4

Інтерлейкін-1

Інтерлейкін-3

Інтерлейкін-2

1682 / 6307
У людини, яка обертається на каруселі, збільшилися частота серцевих скорочень, потовиділення, з’явилася нудота. З подразненням яких рецепторів, перш за все, це пов’язано?

Вестибулярні отолітові

Пропріоцептори

Слухові

Вестибулярні ампулярні

Зорові

1683 / 6307
Робітник тваринницької ферми гостро захворів і при наростаючих явищах інтоксикації помер. При розтині встановлено: селезінка збільшена, в’яла, на розрізі темно-вишневого кольору, зішкріб пульпи рясний. М’які мозкові оболонки на склепінні та основі мозку набряклі, просякнуті кров’ю, мають темно-червоний колір ('шапочка кардинала'). Мікроскопічно: серозно-геморагічне запалення оболонок і тканин головного мозку з руйнуванням стінок дрібних судин. Який найбільш вірогідний діагноз?

Сибірка

Холера

Туляремія

Чума

Бруцельоз

1684 / 6307
Жінка 49-ти років звернулася до лікаря зі скаргами на підвищену втомлюваність та появу задишки під час фізичного навантаження. На ЕКГ: ЧСС-50/хв, інтервал PQ- подовжений, комплекс QRS - не змінений, кількість зубців P перевищує кількість комплексів QRS. Який вид аритмії у пацієнтки?

Атріовентрикулярна блокада

Синусова брадикардія

Екстрасистолія

Синоатріальна блокада

Миготлива аритмія

1685 / 6307
Хворий на ішемічну хворобу серця, з метою усунення нападів стенокардії, впродовж дня багаторазово приймав препарат, який в подальшому з причини передозування привів до отруєння. Об’єктивно: ціаноз шкіри та слизових оболонок, різке зниження артеріального тиску, тахікардія, пригнічення дихання. У крові підвищений вміст метгемоглобіну. Препарат якої групи приймав хворий?

α-адреноблокатори

Міотропні спазмолітики

Органічні нітрати

Препарати аденозинового ряду

Блокатори кальцієвих каналів

1686 / 6307
У хворого з верхнім типом ожиріння тривало відзначалися артеріальна гіпертонія, гіперглікемія, глюкозурія. Смерть настала від крововиливу у головний мозок. Під час патоморфологічного дослідження виявлені базофільна аденома гіпофізу, гіперплазія кори наднирників. Який найбільш вірогідний діагноз?

Гіпофізарний нанізм

Акромегалія

Хвороба Іценка-Кушінга

Адипозогенітальна дистрофія

Цукровий діабет

1687 / 6307
Чоловік середнього віку виїхав до іншої країни на обіцяну йому роботу, але працевлаштуватися тривалий час йому не вдавалося. Які з ендокринних залоз були виснажені у цієї людини найбільше?

Наднирники

Сім’яники

Щитоподібна

Прищитоподібні

Підгрудинна

1688 / 6307
У дівчинки діагностований адреногенітальний синдром (псевдогермафродитизм). Надмірна секреція яких гормонів наднирників обумовила дану патологію?

Естрогени

Мінералокортикоїди

Катехоламіни

Глюкокортикоїди

Андрогени

1689 / 6307
У жінки народилась мертва дитина з багатьма вадами розвитку. Яке протозойне захворювання могло спричинити внутрішньоутробну загибель?

Лейшманіоз

Лямбліоз

Токсоплазмоз

Амебіаз

Малярія

1690 / 6307
У хворого на рак спинки язика виникла сильна кровотеча внаслідок ураження пухлиною дорзальної артерії язика. Яку судину повинен перев’язати лікар для зупинки кровотечі?

Дорзальна артерія язика

Висхідна артерія глотки

Язикова артерія

Лицева артерія

Глибока артерія язика

1691 / 6307
У підлітка 12-ти років, який впродовж 3-х місяців різко схуд, вміст глюкози у крові 50 ммоль/л. У нього розвинулася кома. Який головний механізм її розвитку?

Гіпоксичний

Кетонемічний

Лактацидемічний

Гіперосмолярний

Гіпоглікемічний

1692 / 6307
У хворого на мікросфероцитарну гемолітичну анемію (хворобу Мінковського-Шоффара), внаслідок підвищення проникливості мембрани еритроцитів, у клітину надходять іони натрію та вода. Еритроцити набувають форму сфероцитів і легко руйнуються. Який провідний механізм пошкодження еритроцитів має місце в даному випадку?

Електролітно-осмотичний

Ацидотичний

Протеїновий

Нуклеїновий

Кальцієвий

1693 / 6307
На гістологічному препараті представлено кровоносну судину. Внутрішня оболонка складається з ендотелію, підендотелію та внутрішньої еластичної мембрани. Середня оболонка збагачена гладенькими міоцитами. Вкажіть, для якої судини характерні дані морфологічні ознаки:

Вена безм’язового типу

Артерія еластичного типу

Вена м’язового типу

Капіляр

Артерія м’язового типу

1694 / 6307
Охолодження тіла людини у воді виникає значно швидше, ніж на повітрі. Який шлях тепловіддачі у воді значно ефективніший?

Тепловипромінювання

Конвекція

Випаровування поту

Теплопроведення

1695 / 6307
До лікарні після автокатастрофи надійшов юнак 18-ти років. У травматологічному відділенні виявлені численні травми м’яких тканин обличчя в ділянці медіального кута ока, які призвели до масивної кровотечі. Який артеріальний анастомоз міг бути пошкоджений у цьому регіоні?

a. carotis externa et a. carotis interna

a. carotis externa et a. subclavia

a. subclavia et a. ophthalmica

a. carotis interna et a. ophthalmica

a. carotis interna et a. subclavia

1696 / 6307
Жінці 36-ти років після хірургічного втручання внутрішньовенно ввели концентрований розчин альбуміну. Це спричинило посилений рух води у такому напрямку:

Змін руху води не відбуватиметься

Із клітин до міжклітинної рідини

З міжклітинної рідини до капілярів

З міжклітинної рідини до клітин

Із капілярів до міжклітинної рідини

1697 / 6307
Під час розтину трупа чоловіка зі злоякісною пухлиною шлунка, що помер від ракової інтоксикації, в задньонижніх відділах легень виявлені щільні сіро-червоного кольору неправильної форми осередки, які виступають над поверхнею розрізу. Мікроскопічно: у просвіті, стінках дрібних бронхів та альвеолах виявляється ексудат, в якому багато нейтрофілів. Про яке захворювання свідчать зміни у легенях померлого?

Гостра гнійна бронхопневмонія

Гостра серозна бронхопневмонія

Гострий бронхіт

Крупозна пневмонія

Проміжна пневмонія

1698 / 6307
Під час розтину тіла померлої дитини 1,5 років виявлені: геморагічний висип на шкірі, помірна гіперемія та набряк слизової оболонки носоглотки, дрібні крововиливи у слизових оболонках і внутрішніх органах, різкі дистрофічні зміни у печінці, міокарді, гострий некротичний нефроз, масивні крововиливи у наднирниках. Для якого захворювання найбільш характерні виявлені зміни?

Кір

Скарлатина

Висипний тиф

Дифтерія

Менінгококова інфекція

1699 / 6307
Під час огляду дитини 11-ти місяців педіатр виявив викривлення кісток нижніх кінцівок і затримку мінералізації кісток черепа. Нестача якого вітаміну призводить до даної патології?

Пантотенова кислота

Біофлавоноїди

Рибофлавін

Тіамін

Холекальциферол

1700 / 6307
У хворого на хронічний гепатит виявлено значне зниження синтезу і секреції жовчних кислот. Який процес у найбільшій мірі буде порушений у кишечнику цього хворого?

Травлення білків

Всмоктування гліцерину

Всмоктування амінокислот

Травлення вуглеводів

Емульгування жирів

1701 / 6307
Для вирішення питання ретроспективної діагностики перенесеної бактеріальної дизентерії було призначено серологічне дослідження сироватки крові з метою встановлення титру антитіл до шигел. Яку з перелічених реакцій доцільно використати для цього?

Гемоліз

Зв’язування комплементу

Бактеріоліз

Преципітація

Пасивна гемаглютинація

1702 / 6307
Вивчається робота оперону бактерії. Відбулося звільнення гена-оператора від білка репресора. Безпосередньо після цього в клітині почнеться:

Репресія

Реплікація

Трансляція

Транскрипція

Процесінг

1703 / 6307
При визначенні енерговитрат організму людини встановлено, що дихальний коефіцієнт дорівнює 1,0. Це означає, що у клітинах досліджуваного переважно окислюються:

Білки

Білки і вуглеводи

Жири

Вуглеводи та жири

Вуглеводи

1704 / 6307
У тварини в експерименті перерізали задні корінці спинного мозку. Які зміні відбуватимуться в зоні іннервації?

Втрата рухових функцій

Зниження тонусу м’язів

Втрата чутливості і рухових функцій

Підвищення тонусу м’язів

Втрата чутливості

1705 / 6307
Внаслідок руйнування певних структур стовбуру мозку тварина втратила орієнтувальні рефлекси. Які структури було зруйновано?

Чорна речовина

Вестибулярні ядра

Червоні ядра

Медіальні ядра ретикулярної формації

Чотиригорбкова структура

1706 / 6307
У людини осмотичний тиск плазми крові 350 мосмоль/л (норма - 300 мосмоль/л). Це спричинить, перш за все, посилену секрецію такого гормону:

Натрійуретичний

Адренокортикотропін

Альдостерон

Вазопресин

Кортизол

1707 / 6307
Під час бігу на короткі дистанції у нетренованої людини виникає м’язова гіпоксія. До накопичення якого метаболіту в м’язах це призводить?

Кетонові тіла

Оксалоацетат

Глюкозо-6-фосфат

Лактат

Ацетил-КоА

1708 / 6307
У цитоплазмі міоцитів розчинена велика кількість метаболітів окиснення глюкози. Назвіть один з них, який безпосередньо перетворюється на лактат:

Оксалоацетат

Гліцерофосфат

Фруктозо-6-фосфат

Піруват

Глюкозо-6-фосфат

1709 / 6307
Молодий чоловік звернувся до лікарні зі скаргами на порушення сечовипускання. Під час обстеження зовнішніх статевих органів виявлено, що сечівник розщеплений зверху і сеча витікає через цей отвір. Який вид аномалії розвитку зовнішніх статевих органів спостерігається у цьому випадку?

Парафімоз

Гермафродитизм

Епіспадія

Фімоз

Гіпоспадія

1710 / 6307
Хворий звернувся до лікаря-уролога зі скаргами на біль під час сечовипускання. У сечі, що отримана на аналіз у денний час, були виявлені яйця з характерним шипом. З анамнезу відомо, що хворий недавно повернувся з Австралії. Який найбільш вірогідний діагноз?

Дикроцеліоз

Опісторхоз

Шистосомоз урогенітальний

Шистосомоз кишковий

Шистосомоз японський

1711 / 6307
У хворого на дизентерію при колоноскопії виявлено, що слизова оболонка товстої кишки гіперемована, набрякла, її поверхня вкрита сіро-зеленими плівками. Назвіть морфологічну форму дизентерійного коліту:

Виразковий

Катаральний

Фібринозний

Гнійний

Некротичний

1712 / 6307
У хворого через добу після апендектомії у крові визначається нейтрофільний лейкоцитоз із регенеративним зсувом. Який найбільш вірогідний механізм розвитку лейкоцитозу в даному випадку?

Уповільнення міграції лейкоцитів у тканини

Перерозподіл лейкоцитів у організмі

Уповільнення руйнування лейкоцитів

Посилення лейкопоезу

Посилення лейкопоезу та уповільнення міграції лейкоцитів у тканини

1713 / 6307
Хворий 35-ти років звернувся до лікаря із скаргами на сильний нежить та втрату відчуття запахів протягом тижня. Об’єктивно: в носовій порожнині велика кількість слизу, що вкриває слизову оболонку та блокує рецептори нюху. Де в носовій порожнині розташовані ці рецептори?

Загальний носовий хід

Присінок носа

Нижня носова раковина

Верхня носова раковина

Середня носова раковина

1714 / 6307
У дитини 10-ти років поставлено пробу Манту (з туберкуліном). Через 48 годин на місці введення туберкуліну з’явилася папула розміром до 8 мм у діаметрі. Який тип реакції гіперчутливості розвинувся після введення туберкуліну?

Реакція гіперчутливості IV типу

Реакція гіперчутливості II типу

Реакція типу сироваткової хвороби

Атопічна реакція

Реакція типу феномен Артюса

1715 / 6307
вітаміну В1 115. порушується окисне декарбоксилювання α-кетоглутарової кислоти. Синтез якого з наведених коферментів порушується при цьому?

Ліпоєва кислота

Коензим А

Нікотинамідаденіндинуклеотид

Тіамінпірофосфат

Флавінаденіндинуклеотид

1716 / 6307
З урахуванням клінічної картини хворому призначено піридоксальфосфат. Для корекції яких процесів рекомендований цей препарат?

Синтез білку

Синтез пуринових та піримідинових основ

Трансамінування і декарбоксилювання амінокислот

Дезамінування пуринових нуклеотидів

Окисне декарбоксилювання кетокислот

1717 / 6307
У хворого з клінічними ознаками імунодефіциту проведено імунологічні дослідження. Виявлено значне зниження кількості клітин, що утворюють розетки з еритроцитами барана. Який висновок слід зробити аналізуючи дані дослідження?

Недостатність клітин-ефекторів гуморального імунітету

Зниження рівня натуральних кілерів (NK-клітин)

Зниження рівня В-лімфоцитів

Зниження рівня системи комплементу

Зниження рівня T-лімфоцитів

1718 / 6307
У померлого, що понад 20-ти років працював на вугільній шахті, при розтині тіла знайдені ущільнені легені сіро-чорного кольору зі значними ділянками новоутвореної сполучної тканини та наявністю великої кількості макрофагів з пігментом чорного кольору у цитоплазмі. Який з перелічених діагнозів найбільш вірогідний?

Сидероз

Антракосилікоз

Силікоантракоз

Талькоз

Антракоз

1719 / 6307
На розтині тіла померлого від сепсису, в стегновій кістці нижньої кінцівки виявлено флегмонозне запалення, що охоплює кістковий мозок, гаверсові канали та періост. Під періостом - множинні абсцеси, в навколишніх м’яких тканинах стегна - також флегмонозне запалення. Який патологічний процес має місце?

Хронічний гематогенний остеомієліт

Гострий гематогенний остеомієліт

Остеопетроз

Остеопороз

1720 / 6307
Після попередньої сенсибілізації експериментальній тварині підшкірно ввели дозу антигену. У місці ін’єкції розвинулось фібринозне запалення з альтерацією стінок судин, основної речовини та волокнистих структур сполучної тканини у вигляді мукоїдного та фібриноїдного набухання і некрозу. Яка імунологічна реакція має місцє?

Гранулематоз

Гіперчутливість негайного типу

Гіперчутливість сповільненого типу

Реакція трансплантаційного імунітету

Нормергічна реакція

1721 / 6307
Стоматолог під час огляду порожнини рота на межі середньої і задньої третини спинки язика виявив запалені сосочки. Які сосочки язика запалені?

Papillae fungiformes

Papillae filiformes

Papillae foliatae

Papillae conicae

Papillae vallatae

1722 / 6307
При розтині трупа чоловіка 50-ти років виявлено наступні зміни: права легеня у всіх відділах помірно щільна, на розрізі тканина безповітряна, дрібнозерниста, сухувата. Вісцеральна плевра з нашаруванням фібрину сіро-коричневого кольору. Який найбільш вірогідний діагноз?

Бронхопневмонія

Крупозна пневмонія

Інтерстиціальна пневмонія

Пневмофіброз

Туберкульоз

1723 / 6307
Під час статевого дозрівання клітини чоловічих статевих залоз починають продукувати чоловічий статевий гормон тестостерон, який обумовлює появу вторинних статевих ознак. Які клітини чоловічих статевих залоз продукують цей гормон?

Клітини Лейдіга

Сустентоцити

Клітини Сертолі

Сперматозоїди

Підтримуючі клітини

1724 / 6307
При огляді пацієнта виявлене надмірне розростання кісток і м’яких тканин обличчя, збільшені розміри язика, розширені міжзубні проміжки в збільшеній зубній дузі. Які зміни секреції гормонів у нього найбільш вірогідні?

Збільшена секреція соматотропного гормону

Зменшена секреція соматотропного гормону

Зменшена секреція інсуліну

Збільшена секреція інсуліну

Зменшена секреція тироксину

1725 / 6307
На розтині тіла померлого від ниркової недостатності, який протягом останніх 5-ти років хворів на бронхоектатичну хворобу, виявлені збільшені в розмірах нирки щільної консистенції з потовщеним кірковим шаром білого кольору та сальним блиском. Про яке захворювання нирок можна думати?

Вторинний амілоїдоз

Гломерулонефрит

Хронічний пієлонефрит

Некротичний нефроз

1726 / 6307
На розтині тіла жінки 49-ти років, що померла від хронічної ниркової недостатності, виявлено: нирки ущільнені, зменшені, строкаті, з ділянками крововиливів. Мікроскопічно: у ядрах епітелію канальців гематоксилінові тільця, потовщення базальних мембран капілярів клубочків, які мають вигляд дротяних петель, подекуди в капілярах - гіалінові тромби та вогнища фібриноїдного некрозу. Який найбільш вірогідний діагноз?

Артеріосклеротичний нефросклероз

Амілоїдоз

Ревматизм

Атеросклеротичний нефросклероз

Системний червоний вовчак

1727 / 6307
У хворого 37-ми років внаслідок тривалої антибіотикотерапії розвинувся дисбактеріоз кишечнику. Який вид препаратів необхідно використати для нормалізації кишкової мікрофлори?

Аутовакцини

Еубіотики

Сульфаніламіди

Вітаміни

Бактеріофаги

1728 / 6307
До генетичної консультації звернулася сімейна пара, в якій чоловік хворіє на інсулінозалежний цукровий діабет, а жінка здорова. Яка вірогідність появи інсулінозалежного діабету у дитини цього подружжя?

100%

Нижче, ніж в популяції

Більше, ніж в популяції

50%

Така сама, як в популяції

1729 / 6307
Хвора похилого віку хворіє на цукровий діабет 2-го типу, який супроводжується ожирінням, атеросклерозом, ішемічною хворобою серця. При цьому виявлена базальна гіперінсулінемія. Запропонуйте хворій адекватне лікування:

Ловастатин

Глібенкламід

Інсулін

Ретаболіл

Амлодипін

1730 / 6307
До лікаря звернулася жінка 32-х років зі скаргами на відсутність лактації після народження дитини. Дефіцитом якого гормону можна пояснити дане порушення?

Пролактин

Соматотропін

Вазопресин

Глюкагон

Тиреокальцитонін

1731 / 6307
У хворої під час профілактичного обстеження на медіальній стінці лівої пахвової западини виявлений збільшений лімфовузол метастатичного походження. Вкажіть найбільш вірогідну локалізацію первинної пухлини:

Легеня

Шлунок

Молочна залоза

Щитоподібна залоза

Піднижньощелепна слинна залоза

1732 / 6307
У чоловіка, що хворіє на остеохондроз, з’явився різкий біль у м’язах живота (бічних та передніх). При об’єктивному обстеженні лікар констатував підвищену больову чутливість шкіри підчеревної ділянки. Ураження якого нерва могло спричинити цій біль?

Клубово-підчеревний

Затульний

Статево-стегновий

Сідничний

Стегновий

1733 / 6307
На рентгенограмі нирок при пієлографії лікар виявив ниркову миску, в яку безпосередньо впадали малі чашечки (великі були відсутні). Яку форму сечовивідних шляхів нирки виявив лікар?

Ембріональна

Зріла

Деревоподібна

Фетальна

Ампулярна

1734 / 6307
На аутопсії померлого від грипу чоловіка відзначено, що серце дещо збільшене у розмірах, пастозне, на розрізі міокард тьмяний, з крапом. Мікроскопічно: у міокарді на всьому протязі ознаки паренхіматозної жирової і гідропічної дистрофії, строма набрякла, з незначною макрофагально-лімфоцитарною інфільтрацією, судини повнокровні; периваскулярно - петехіальні крововиливи. Який вид міокардиту розвинувся в даному випадку?

Гранулематозний

Гнійний

Проміжний проліферативний

Серозний вогнищевий

Серозний дифузний

1735 / 6307
У хворого на хронічну серцеву недостатність, незважаючи на терапію кардіотонічними засобами і тіазидовим діуретиком, зберігаються набряки і виникла загроза асциту. Який препарат слід призначити для підсилення діуретичного ефекту застосованих ліків?

Манітол

Клопамід

Спіронолактон

Фуросемід

Амілорид

1736 / 6307
При гінекологічному огляді жінки 30-ти років на шийці матки виявлені яскраво-червоні блискучі плями, які при дотику легко кровоточать. На біопсії: шматочок шийки матки вкритий циліндричним епітелієм із сосочковими виростами, в товщині тканини розростання залоз. Яка патологія шийки матки виявлена?

Справжня ерозія

Залозиста гіперплазія

Лейкоплакія

Псевдоерозія

Ендоцервіцит

1737 / 6307
У мертвонародженої дитини шкіра потовщена, нагадує панцир черепахи, вушні раковини недорозвинені. Гістологічно в шкірі: надмірне зроговіння, атрофiя зернистого шару епідермісу, відсутні запальні зміни. Яке захворювання найбільш вірогідно?

Ксеродермія

Іхтіоз

Дерматоміозит

Еритроплакія

Лейкоплакія

1738 / 6307
У хворого спостерігається погіршення сутінкового зору. Який з вітамінних препаратів слід призначити пацієнту?

Кислота нікотинова

Піридоксину гідрохлорид

Ціанокобаламін

Ретинолу ацетат

Кислота аскорбінова

1739 / 6307
Хвора звернулась до лікаря із скаргами на біль та обмеження рухів у колінних суглобах. Який з нестероїдних протизапальних засобів краще призначити, враховуючи наявність в анамнезі хронічного гастродуоденіту?

Промедол

Целекоксиб

Диклофенак-натрій

Бутадіон

Кислота ацетилсаліцилова

1740 / 6307
Депресії та емоційні розлади є наслідком нестачі у головному мозку норадреналіну, серотоніну та інших біогенних амінів. Збільшення їх вмісту у синапсах можна досягти за рахунок антидепресантів, які гальмують такий фермент:

Оксидаза L-амінокислот

Фенілаланін-4-монооксигеназа

Диамінооксидаза

Моноамінооксидаза

Оксидаза D-амінокислот

1741 / 6307
У хворого з’явилися жовтушність шкіри, склер та слизових оболонок. У плазмі крові підвищений рівень загального білірубіну, в калі - рівень стеркобіліну, в сечі - уробіліну. Який вид жовтяниці у хворого?

Хвороба Жільбера

Гемолітична

Холестатична

Обтураційна

Паренхіматозна

1742 / 6307
До патогістологічої лабораторії доставлено червоподібний відросток товщиною до 2,0 см. Серозна оболонка його тьмяна, потовщена, вкрита жовто-зеленими плівковими нашаруваннями. Стінка в’яла, сіро-червона. Просвіт відростка розширено, заповнено жовто-зеленими масами. При гістологічному дослідженні виявлено, що стінка інфільтрована нейтрофілами. Визначте захворювання апендикса:

Хронічний апендицит

Гострий гангренозний апендицит

Гострий флегмонозний апендицит

Гострий поверхневий апендицит

Гострий простий апендицит

1743 / 6307
Відпочиваючи на дачі, хлопчик знайшов павука з наступними морфологічними особливостями: довжина - 2 см, кулясте черевце чорного кольору, на спинному боці якого видно червоні плямочки у два ряди, чотири пари членистих кінцівок вкриті дрібними чорними волосками. Визначте дане членистоноге:

Кліщ

Скорпіон

Тарантул

Каракурт

Фаланги

1744 / 6307
У хворого внаслідок травми розвинувся травматичний шок, у перебігу якого мали місце наступні порушення: АТ- 140/90 мм рт.ст., Ps- 120/хв. Хворий метушливий, багатослівний, блідий. Якій стадії шоку відповідає цей стан?

Термінальна

Торпідна

Латентний період

Еректильна

Кінцева

1745 / 6307
У хворого, що надійшов до хірургічного відділення з ознаками гострого апендициту, виявлені наступні зміни білої крові: загальна кількість лейкоцитів - 16 • 109 /л. Лейкоцитарна формула: б.- 0, е.- 2%, ю.- 2%, п.- 8%, с.- 59%, л.- 25%, м.- 4%. Як класифікуються зазначені зміни?

Нейтрофілія з гіперрегенеративним зсувом вліво

Нейтрофілія з регенеративним зсувом вліво

Нейтрофілія з зсувом вправо

Лейкемоїдна реакція за нейтрофільним типом

Нейтрофілія з дегенеративним зсувом вліво

1746 / 6307
Після травми хворий не може розігнути руку в ліктьовому суглобі. Порушення функції якого з основних м’язів може це спричинити?

m. subscapularis

m. infraspinatus

m. triceps brachii

m. teres major

m. levator scapulae

1747 / 6307
До лікаря звернувся студент з проханням призначити препарат для лікування алергічного риніту, який виник у нього під час цвітіння липи. Який засіб можна застосувати?

Норадреналіну гідротартрат

Лозартан

Анаприлін

Амброксол

Лоратадин

1748 / 6307
У зародка порушено процес сегментації дорзальної мезодерми та утворення сомітів. В якій частині шкіри можливі порушення розвитку?

Потові залози

Дерма

Епідерміс

Волосся

Сальні залози

1749 / 6307
Дитина 9-ми місяців харчується штучними сумішами, які не збалансовані за вмістом вітаміну В6 . У дитини спостерігається пелагроподібний дерматит, судоми, анемія. Розвиток судом може бути пов’язаний з порушенням утворення:

ДОФА

Дофаміну

Гістаміну

ГАМК

Серотоніну

1750 / 6307
В експерименті на жабі вивчали міотатичний рефлекс. Однак при розтяганні скелетного м’яза він рефлекторно не скоротився. Порушення функції яких рецепторів може бути причиною цього?

Суглобові

Больові

М’язові веретена

Сухожильні рецептори Гольджі

Дотикові

1751 / 6307
В експерименті подразнюють гілочки блукаючого нерва, які іннервують серце. Це призвело до того, що припинилося проведення збудження від передсердь до шлуночків. Електрофізіологічні зміни в яких структурах серця є причиною цього?

Пучок Гіса

Шлуночки

Передсердя

Синоатріальний вузол

Атріовентрикулярний вузол

1752 / 6307
Чоловік протягом 3-х років працював в одній із африканських країн. Через місяць після переїзду до України звернувся до офтальмолога зі скаргами на біль в очах, набряки повік, сльозоточивість і тимчасове послаблення зору. Під кон’юнктивою ока були виявлені гельмінти розмірами 30-50 мм, які мали видовжене ниткоподібне тіло. Який найбільш вірогідний діагноз?

Аскаридоз

Дифілоботріоз

Трихоцефальоз

Ентеробіоз

Філяріоз

1753 / 6307
У юнака 16-ти років після перенесеного захворювання знижена функція синтезу білків у печінці внаслідок нестачі вітаміну K. Це може призвести до порушення:

Утворення еритропоетинів

Швидкості осідання еритроцитів

Утворення антикоагулянтів

Зсідання крові

Осмотичного тиску крові

1754 / 6307
На перехід із горизонтального положення у вертикальне система кровообігу відповідає розвитком рефлекторної пресорної реакції. Що з наведеного є її обов’язковим компонентом?

Зменшення об’єму циркулюючої крові

Зменшення насосної функції серця

Зменшення частоти серцевих скорочень

Системне звуження венозних судин ємності

Системне розширення артеріальних судин опору

1755 / 6307
У життєвому циклі клітини відбувається процес самоподвоєння ДНК. В результаті цього однохроматидні хромосоми стають двохроматидними. У який період клітинного циклу спостерігається це явище?

S

G0

M

G2

G1

1756 / 6307
У водія, який потрапив у ДТП, отримав травму та знаходиться у стані шоку, спостерігається зменшення добової кількості сечі до 300 мл. Який основний патогенетичний фактор цієї зміни діурезу?

Падіння артеріального тиску

Вторинний гіперальдостеронізм

Зниження онкотичного тиску крові

Зменшення кількості функціонуючих клубочків

Підвищення проникності судин

1757 / 6307
У хворого виявлена аутоімунна гемолітична анемія, що розвивається за цитотоксичним типом. Які речовини є антигенами при алергічних реакціях II типу?

Антибіотики

Гормони

Модулятори запалення

Модифіковані рецептори клітинних мембран

Сироваткові білки

1758 / 6307
Пацієнт страждає на геморагічний синдром, що проявляється частими носовими кровотечами, посттравматичними та спонтанними внутрішньо-шкірними та внутрішньосуглобовими крововиливами. Після лабораторного обстеження було діагностовано гемофілію В. Дефіцит якого фактора згортання крові обумовлює дане захворювання?

XI

V

VII

IX

VIII

1759 / 6307
Після черепно-мозкової травми у хворого спостерігається втрата можливості виконувати знайомі до травми складно-координовані рухи (апраксія). В якій ділянці кори великих півкуль найімовірніше локалізується ушкодження?

Gyrus supramarginalis

Gyrus angularis

Gyrus paracentralis

Gyrus lingualis

Gyrus parahippocampalis

1760 / 6307
Чоловік 58-ми років хворіє на атеросклероз судин головного мозку. При обстеженні виявлена гіперліпідемія. Вміст якого класу ліпопротеїдів у сироватці крові даного чоловіка найбільш вірогідно буде підвищений?

Ліпопротеїди низької щільності

Холестерин

Ліпопротеїди високої щільності

Хіломікрони

Комплекси жирних кислот з альбумінами

1761 / 6307
Хворий звернувся до лікаря зі скаргами на ригідність м’язів, скутість рухів, постійний тремор рук. Встановлено діагноз: хвороба Паркінсона. Який препарат найбільш раціонально призначити?

Етосуксимід

Дифенін

Фенобарбітал

Сибазон

Леводопа

1762 / 6307
До кардіологічного відділення надійшов хворий на ішемічну хворобу серця. Для профілактики нападів стенокардії призначено лікарський засіб з групи βадреноблокаторів. Назвіть цей препарат:

Морфіну гідрохлорид

Окситоцин

Метопролол

Фуросемід

Атропіну сульфат

1763 / 6307
До гінеколога звернулася жінка 28-ми років з приводу безпліддя. При обстеженні знайдено: недорозвинені яєчники та матка, нерегулярний менструальний цикл. При досліджєнні статевого хроматину в 6ільшості соматичних клітин виявлено 2 тільця Бара. Яка хромосомна хвороба найбільш вірогідна у жінки?

Синдром трипло-Х

Синдром Шерешевського-Тернера

Синдром Патау

Синдром Едвардса

Синдром Клайнфельтера

1764 / 6307
При отруєнні невідомим препаратом у пацієнта спостерігались сухість слизової оболонки рота та розширення зіниць. З яким впливом пов’язана дія цього препарату?

Стимуляція M-холінорецепторів

Блокада адренорецепторів

Стимуляція H-холінорецепторів

Стимуляція адренорецепторів

Блокада M-холінорецепторів

1765 / 6307
У новонародженого спостерігається диспепсія після годування молоком. При заміні молока розчином глюкози симптоми диспепсії зникають. Недостатня активність якого ферменту спостерігається у новонародженого?

Амілаза

Сахараза

Лактаза

Мальтаза

Ізомальтаза

1766 / 6307
У людини збільшена частота серцевих скорочень, розширені зіниці, сухість у роті. Наслідком активації в організмі якої системи регуляції функцій це викликано?

Парасимпатична

Метасимпатична

Симпатична

Ваго-інсулярна

Гипоталамо-гіпофізарно-наднирникова

1767 / 6307
Хворому з переломом кінцівки необхідно призначити препарат з групи міорелаксантів деполяризуючого типу дії для проведення нетривалого хірургічного втручання. Що це за засіб?

Пентамін

Атропіну сульфат

Тубокурарину хлорид

Дитилін

Цитітон

1768 / 6307
Пацієнт, що хворіє на хронічний бронхіт, приймає синтетичний муколітичний препарат, який сприяє розріджуванню харкотиння. Назвіть цей препарат:

Діазепам

Фуросемід

Гепарин

Еналаприл

Ацетилцистеїн

1769 / 6307
У хворого з масивними опіками розвинулась гостра недостатність нирок, що характеризується значним і швидким зменшенням швидкості клубочкової фільтрації. Який механізм її розвитку?

Зменшення ниркового кровотоку

Ушкодження клубочкового фільтра

Емболія ниркової артерії

Зменшення кількості функціонуючих нефронів

Збільшення тиску канальцевої рідини

1770 / 6307
У хворого з важким перебігом респіраторної вірусної інфекції з’явилися клінічні ознаки прогресуючої серцевої недостатності, яка призвела до смерті хворого на 2-му тижні захворювання. На аутопсії: серце зі значним розширенням порожнин, в’яле. Гістологічно в міокарді виявляється повнокров’я мікросудин і дифузна інфільтрація строми лімфоцитами та гістіоцитами. Який найбільш вірогідний діагноз?

Стенокардія

Інфаркт міокарда

Кардіоміопатія

Міокардит

Гостра коронарна недостатність

1771 / 6307
При поточному контролі санітарно-епідемічного стану аптеки проведено бактеріологічне дослідження повітря. Встановлено наявність у ньому бацил, дріжджеподібних грибів, гемолітичних стрептококів, мікрококів. Які з виявлених мікроорганізмів свідчать про пряму епідемічну небезпеку?

Мікрококи

Дріжджеподібні гриби

Бацили

Гемолітичні стрептококи

1772 / 6307
До реанімаційного відділення надійшов чоловік з пораненням задньої ділянки шиї (regio nuchae). Який з м’язів тіла займає цю ділянку?

m. sternocleidomastoideus

m. trapezius

m. latissimus dorsi

m. rhomboideus minor

m. scalenus anterior

1773 / 6307
На секції в лівій легені виявлено ділянку щільної тканини червоного кольору. Ділянка має форму конуса, чітко відмежована від здорової тканини, основою обернена до плеври. Тканина на розрізі зерниста, темно-червона. Який найбільш вірогідний діагноз?

Первинний туберкульозний афект

Крупозна пневмонія

Гангрена легені

Геморагічний інфаркт

Абсцес легені

1774 / 6307
У тварини збільшений тонус м’язів-розгиначів. Це є наслідком посиленої передачі інформації до мотонейронів спинного мозку такими низхідними шляхами:

Вестибулоспінальні

Ретикулоспінальні

Медіальні кортикоспінальні

Руброспінальні

Латеральні кортикоспінальні

1775 / 6307
Після опромінювання у людини з’явилася велика кількість мутантних клітин. Через деякий час більшість із них були розпізнані і знищені клітинами імунної системи, а саме:

Стовбуровими клітинами

В-лімфоцитами

T-лімфоцитами-супресорами

T-лімфоцитами-кілерами

Плазмобластами

1776 / 6307
У чоловіка 65-ти років розвинувся гнійний абсцес на шиї. Виділена культура грампозитивних коків, яка має плазмокоагулазну активність. Більш за все, це:

Streptococcus pyogenes

Staphylococcus epidermidis

Staphylococcus aureus

Staphylococcus saprophyticus

1777 / 6307
У клінічній практиці для лікування туберкульозу застосовують препарат ізоніазид - антивітамін, який здатний проникати у туберкульозну паличку. Туберкулостатичний ефект обумовлений порушенням процесів реплікації, окисно-відновних реакцій, завдяки утворенню несправжнього коферменту з:

ФМН

НАД

КоQ

ФАД

ТДФ

1778 / 6307
У новонародженої дитини спостерігаються зниження інтенсивності смоктання, часте блювання, гіпотонія. У сечі та крові значно підвищена концентрація цитруліну. Який метаболічний процес порушений?

Цикл Корі

Гліколіз

ЦТК

Орнітиновий цикл

Глюконеогенез

1779 / 6307
В експерименті на тварині здійснили перерізку блукаючих нервів з обох боків. Як при цьому зміниться характер дихання?

Дихання не зміниться

Стане глибоким і частим

Стане поверхневим та рідким

Стане поверхневим та частим

Стане глибоким і рідким

1780 / 6307
При підготовці до видалення зуба стоматолог порекомендував пацієнтові з метою профілактики кровотечі приймати препарат. Який засіб був рекомендований?

Магнію сульфат

Вікасол

Димедрол

Гепарин

Аспаркам

1781 / 6307
Людина, що тривалий час приймає ліки, не може різко припинити їх вживання, оскільки при цьому виникають порушення психічних та соматичних функцій. Як називається синдром різних порушень при відмові від прийому речовини?

Сенсибілізація

ідіосинкразія

Кумуляція

Тахіфілаксія

Абстиненція

1782 / 6307
У хворої 45-ти років невроз, що проявляється дратівливістю, безсонням, немотивованою тривогою. Який лікарський засіб усуне всі симптоми?

Діазепам

Кофеїн-бензоат натрію

Пірацетам

Екстракт валеріани

Леводопа

1783 / 6307
У хворого після вживання недоброякісної їжі розвинувся багаторазовий пронос. На наступний день у нього знизився артеріальний тиск, з’явились тахікардія, екстрасистолія. pH крові складає 7,18. Ці порушення є наслідком розвитку:

Негазового ацидозу

Газового ацидозу

Газового алкалозу

Метаболічного алкалозу

Негазового алкалозу

1784 / 6307
Яким буде скорочення м’язів верхньої кінцівки при утриманні (але не переміщенні) вантажу в певному положенні?

Ексцентричним

Концентричним

Ізотонічним

Ауксотонічним

Ізометричним

1785 / 6307
Хворий не відчуває дотику до шкіри в ділянці присередньої поверхні плеча. Порушення функції якого нерва спостерігається у хворого?

Шкірний присередній нерв плеча

Променевий нерв

Пахвовий нерв

Ліктьовий нерв

Шкірний присередній нерв передпліччя

1786 / 6307
По ходу слухового нерва у молодої жінки виявлена пухлина у вигляді вузла до 3 см в діаметрі, м’яко-еластичної консистенції, рожево-білого кольору, однорідна. Мікроскопічно пухлина містить пучки клітин з овальними ядрами. Клітинно-волокнисті пучки формують ритмічні структури, створені паралельними рядами, правильно орієнтованими клітинами, розташованими у вигляді частоколу, поміж яких знаходиться безклітинна гомогенна зона (тільця Верокаї). Що це за пухлина?

Гангліоневрома

Невринома

Злоякісна невринома

Гангліонейробластома

Нейробластома

1787 / 6307
До кардіологічного відділення надійшов хворий з інфарктом міокарда. Для усунення болю було вирішено потенціювати дію фентаніла нейролептиком. Який з перерахованих нейролептиків найбільш придатний для проведення нейролептаналгезії?

Дроперидол

Аміназин

Сульпірид

Галоперидол

Трифтазін

1788 / 6307
Для лікування бактеріальної пневмонії було призначено бензилпеніциліну натрієву сіль. Який механізм антимікробної дії препарату?

Пригнічення внутрішньоклітинного синтезу білка

Пригнічення активності холінестерази

Пригнічення SH-груп ферментів мікроорганізмів

Антагонізм з параамінобензойною кислотою

Пригнічення синтезу клітинної стінки мікроорганізмів

1789 / 6307
Хворий 49-ти років, водій за професією, скаржиться на нестерпний стискаючий біль за грудниною, що 'віддає' у ділянку шиї. Біль виник 2 години тому. Об’єктивно: стан важкий, блідість, тони серця послаблені. Лабораторне обстеження показало високу активність креатинкінази та ЛДГ1 . Для якого захворювання характерні такі симптоми?

Жовчнокам’яна хвороба

Стенокардія

Гострий інфаркт міокарда

Гострий панкреатит

Цукровий діабет

1790 / 6307
У чоловіка 60-ти років спостерігається послаблення перистальтики кишечнику. Який з наведених харчових продуктів буде стимулювати перистальтику найбільше?

Білий хліб

М’ясо

Чорний хліб

Сало

Чай

1791 / 6307
При посіві матеріалу із зіву від хворого ангіною на кров’яно-телуритовий агар виросли колонії діаметром 4-5 мм, сірого кольору, радіально посмуговані (у вигляді розеток). Під мікроскопом - грампозитивні палички із булавоподібними потовщеннями на кінцях, що розміщені у вигляді розчепірюваних пальців. Які це мікроорганізми?

Стрептококи

Коринебактерії дифтерії

Клостридії ботулізму

Стрептобацили

Дифтероїди

1792 / 6307
На мікропрепараті підщелепної слинної залози навколо кінцевих відділів і вивідних проток розрізняються кошикоподібні клітини, які охоплюють основи сероцитів і називаються міоепітеліоцити. До якої тканини належать ці клітини?

Сполучна зі спеціальними властивостями

Нервова

М’язова

Пухка волокниста сполучна

Епітеліальна

1793 / 6307
Встановлено, що аглютинація еритроцитів крові реципієнта викликали стандартні сироватки I та II груп і не викликали - сироватка III групи і антирезусна сироватка. Кров якої групи за системами AB0 і резус можна переливати реципієнту?

B,α (III)Rh-

AB(IV), Rh+

AB(IV), Rh-

0,α,β, (I)Rh+

1794 / 6307
Фармакологічні ефекти антидепресантів пов’язані з блокуванням (інгібуванням) ними ферменту, який каталізує розпад таких біогенних амінів, як норадреналін і серотонін в мітохондріях нейронів головного мозку. Який фермент бере участь у цьому процесі?

Ліаза

Моноамінооксидаза

Пептидаза

Декарбоксилаза

Трансаміназа

1795 / 6307
Онкологічному хворому призначили препарат метотрексат, до якого з часом клітинимішені пухлини втратили чутливість. Експресія гену якого ферменту при цьому змінюється?

Фолатоксидаза

Фолатдекарбоксилаза

Тиміназа

Дезаміназа

Дегідрофолатредуктаза

1796 / 6307
До кардіологічного відділення надійшов хворий з гіпертонічним кризом, йому внутрішньовенно ввели антигіпертензивний засіб - сіль лужноземельного металу. Який препарат ввели хворому?

Натрію гідрокарбонат

Кальцію лактат

Бензогексоній

Магнію сульфат

Калію хлорид

1797 / 6307
Постраждалий доставлений до хірургічного відділення із проникним пораненням у лівій бічній ділянці живота. Який відділ товстої кишки, найімовірніше, ушкоджений?

Colon ascendens

Colon transverses

Rectum

Colon descendens

Caecum

1798 / 6307
Постраждалого в аварії водія госпіталізовано до стаціонару з ушкодженням медіального надвиростка плечової кістки. Який нерв при цьому може бути ушкоджений?

n. axillaris

n. medianus

n. ulnaris

n. muscolocutaneus

n. radialis

1799 / 6307
Чоловік з колотою раною в ділянці чотирьохстороннього отвору звернувся до лікаря. При обстеженні виявлено, що потерпший не може відвести руку від тулуба. Який нерв вірогідно ушкоджений?

n. axillaris

n. subclavius

n. radialis

n. ulnaris

n. medianus

1800 / 6307
У вагітної жінки на передній черевній стінці виявлено пухлиноподібне утворення, яке виникло на місці видаленої два роки тому пухлини. Утворення має щільну консистенцію і розміри 2х1 см, з чіткими межами. При гістологічному дослідженні виявлено, що пухлина побудована з диференційованої сполучної тканини з переважанням колагенових волокон. Про яку пухлину слід думати?

Фібросаркома

Ліпома

Десмоїд

Гібернома

Лейоміома

1801 / 6307
Під час емоційного збудження частота серцевих скорочень (ЧСС) у людини 30-ти років досягла 112/хв. Зміна стану якої структури провідної системи серця є причиною збільшення ЧСС?

Ніжки пучка Пса

Синоатрiальний вузол

Атріовентрикулярний вузол

Волокна Пуркін’є

Пучок Гіса

1802 / 6307
При санітарно-бактеріологічному дослідженні води методом мембранних фільтрів виявлено дві червоні колонії на мембранному фільтрі (середовище Ендо), через який пропустили 500 мл досліджуваної води. Розрахуйте колі-індекс та колі-титр досліджуваної води:

2 та 500

250 та 4

250 та 2

500 та 2

4 та 250

1803 / 6307
Хвора 46-ти років довгий час страждає на прогресуючу м’язову дистрофію (Дюшена). Зміни рівня якого ферменту крові є діагностичним тестом в даному випадку?

Креатинфосфокіназа

Піруватдегідрогеназа

Лактатдегідрогеназа

Глутаматдегідрогеназа

Аденілаткіназа

1804 / 6307
Дівчинка 10-ти років часто хворіє на гострі респіраторні інфекції, після яких спостерігаються множинні точкові крововиливи в місцях тертя одягу. Який гіповітаміноз має місце в дівчинки:

B2

A

C

В6

B1

1805 / 6307
З сироватки крові людини виділили п’ять ізоферментних форм лактатдегідрогенази і вивчили їх властивості. Яка властивість доводить, що виділені ізоферментні форми одного і того ж ферменту?

Тканинна локалізація

Однакові фізико-хімічні властивості

Однакова електрофоретична рухливість

Каталізують одну і ту ж реакцію

Однакова молекулярна маса

1806 / 6307
Хворий з виразковою хворобою шлунка приймав антацидний препарат альмагель. Для лікування гострого бронхіту йому призначили антибіотик метициклін. Проте протягом 5- ти днів температура не знизилася, кашель і характер харкотиння не змінились. Лікар прийшов до висновку про несумісність ліків при їх взаємодії. Про який саме вид несумісності ліків йдеться?

Прямий антагонізм

Фармакокінетична на етапі всмоктування

Фармакодинамічна

Фармацевтична

Фармакокінетична на етапі біотрансформації

1807 / 6307
При гістологічному дослідженні біоптата перегородки носа хворого, який страждає на утруднене носове дихання, в слизовій оболонці знайдено гранулематозне запалення з наявністю в гранульомах клітин Микуліча і бактерій Волковича-Фріша. Який найбільш вірогідний діагноз?

Туберкульоз

Сап

Риносклерома

Сифіліс

Лепра

1808 / 6307
При аналізі ЕКГ необхідно визначити, що є водієм ритму серця. Зробити це можна на підставі вимірювання:

Напрямку зубців

Амплітуди зубців

Тривалості інтервалу R — R

Тривалості комплексу QRST

Тривалості зубців

1809 / 6307
У новонародженого хлопчика спостерігається деформація мозкового та лицьового черепа, мікрофтальмія, деформація вушної раковини, вовча паща, і т.ін. Каріотип дитини - 47, XY, 13+. Про яку хворобу йде мова?

Синдром Патау

Синдром Едвардса

Синдром Шерешевського-Тернера

Синдром Клайнфельтера

Синдром Дауна

1810 / 6307
Прозерин при системному введеннї підвищує тонус скелетних м’язів. Фторотан викликає релаксацію м’язів і послаблює ефекти прозерину. Який характер взаємодії прозерину та фторотану?

Неконкурентний антагонізм

Непрямий функціональний антагонізм

Незалежний антагонізм

Конкурентний антагонізм

Прямий функціональний антагонізм

1811 / 6307
Жінка в період вагітності тривалий час без контролю лікаря приймала антибіотик. Через деякий час у неї погіршився апетит, з’явились нудота, пронос, зміни слизових оболонок ротової порожнини та шлунково-кишкового тракту. З часом виникла жовтяниця. У новонародженого відмічено порушення росту кісток. Який препарат приймала жінка?

Еритроміцин

Бісептол

Доксациклін

Левоміцетин

Ампіцилін

1812 / 6307
До кардіологічного відділення доставлений хворий з діагнозом: гострий інфаркт міокарда. Для усунення больового синдрому (нейролептанальгезії) пацієнту необхідно ввести таку комбінацію лікарських препаратів:

Но-шпа + димедрол

Анальгін + піпольфен

Аміназин + діазепам

Морфін + атропін

Дроперідол + фентаніл

1813 / 6307
У хворого з пересадженим серцем при фізичному навантаженні збільшився хвилинний об’єм крові. Який механізм регуляції забезпечує ці зміни?

Катехоламіни

Симпатичні безумовні рефлекси

Парасимпатичні умовні рефлекси

Парасимпатичні безумовні рефлекси

Симпатичні умовні рефлекси

1814 / 6307
У хворого 53-х років, що тривало страждав на бронхоектатичну хворобу та кровохаркання, з’явилися набряки на обличчі і у ділянці попереку; у сечі - білок 33 мг/л. Смерть настала від легеневої кровотечі. Результати аутопсії: нирки збільшені в об’ємі, ущільнені, поверхня розрізу має сальний вигляд. Гістологічно відзначено відкладення у клубочках і по ходу канальців гомогенних еозинофільних мас, які вибірково забарвлюються конго-рот і дають метахромазію з метиловим фіолетовим. Який патологічний процес мав місце в нирках у даному випадку?

Амілоїдоз

Фібриноїдне набухання

Мукоїдне набухання

Жирова дистрофія

Гіаліноз

1815 / 6307
Людині внутрішньовенно ввели 0,5 л ізотонічного розчину лікарської речовини. Які з рецепторів насамперед прореагують на зміни водно-сольового балансу організму?

Осморецептори гіпоталамусу

Натрієві рецептори гіпоталамуса

Барорецептори дуги аорти

Осморецептори печінки

Волюморецептори порожнистих вен і передсердь

1816 / 6307
Отруєння ботулінічним токсином, який блокує вхід іонів кальцію до нервових закінчень аксонів мотонейронів, небезпечно для життя, бо загрожує:

Розладом тонусу судин

Зупинкою дихання

Розвитком блювання

Розвитком проносу

Зупинкою серця

1817 / 6307
Недбалий студент раптово зустрівся з деканом. Концентрація якого гормону найшвидше збільшиться в крові студента?

Кортикотропін

Тиреоліберин

Соматотропін

Кортизол

Адреналін

1818 / 6307
У студента після вживання м’яса, консервованого в домашніх умовах, з’явились диплопія, порушення мови та параліч дихання. Чим обумовлені такі симптоми ботулізму?

Активація аденілатциклази

Інвазія Cl. eotulinum в епітелій кишечнику

Дія нейротоксину

Секреція ентеротоксину

Ендотоксичний шок

1819 / 6307
У клінічно здорових батьків народилася дитина, хвора на фенілкетонурію (аутосомнорецесивне спадкове захворювання). Які генотипи батьків?

Аа х Аа

АА х АА

аа х аа

Аа х аа

АА х Аа

1820 / 6307
Хворого 55-ти років госпіталізовано до хірургічної клініки з підозрою на сепсис. Який матеріал для дослідження необхідно взяти від хворого і на яке середовище його слід засіяти?

Гній, жовточно-сольовий агар

Сеча, м’ясо-пептонний бульйон

Ліквор, сироватковий агар

Пунктат лімфовузла, цистеїновий агар

Кров, цукровий бульйон

1821 / 6307
У хворого діагностовано діабетичну кому. Концентрація цукру в крові становить 18,44 ммоль/л. Який з цукрознижуючих препаратів необхідно призначити даному хворому?

Інсулін середньої тривалості дії

Інсулін тривалої дії

Препарат із групи бігуанідів

Препарат із групи похідних сульфо-нілсечовини

Інсулін короткої дії

1822 / 6307
Людина захворіла на пелагру. При опитуванні стало відомо, що впродовж тривалого часу вона харчувалася переважно кукурудзою, мало вживала м’яса. Дефіцит якої речовини у кукурудзі спричинив розвиток хвороби?

Пролін

Аланін

Триптофан

Тирозин

Гістидин

1823 / 6307
До клініки надійшов хворий зі скаргами на біль у правій підреберній ділянці, блювання з кров’ю. При дослідженні було встановлено збільшення печінки, розширення підшкірних вен передньої стінки живота. В якій судині утруднений кровотік?

Черевна аорта

Нижня порожниста вена

Верхня порожниста вена

Печінкові вени

Ворітна вена

1824 / 6307
До лабораторії направлено матеріал білуватого нашарування із слизових оболонок ротової порожнини. Висів матеріалу зроблено на середовище Сабуро, відмічено ріст сметаноподібних колоній. Бактеріоскопія виявила короткі бруньковані нитки. До збудників якої інфекції відносять ізольовані мікроорганізми?

Мікоплазмоз

Спірохетоз

Мікоз

Рикетсіоз

Хламідіоз

1825 / 6307
У хворого, який проходить курс лікувального голодування, нормальний рівень глюкози у крові підтримується головним чином за рахунок глюконеогенезу. З якої амінокислоти при цьому у печінці людини найбільш активно синтезується глюкоза?

Глутамінова кислота

Лізин

Аланін

Лейцин

Валін

1826 / 6307
Оглядаючи дитину 6-ти років, лікар помітив на глоткових мигдаликах сірувату плівку, при спробі видалення якої виникла помірна кровотеча. Бактеріоскопія мазків з мигдаликів показала наявність грампозитивних бактерій булавоподібної форми. Які симптоми можуть виникнути у дитини у найближчі дні, якщо не буде проведене специфічне лікування?

Токсичні ураження серцевого м’яза, печінки, нирок

Набряк легенів

Папульозні висипи на шкірі

Дуже сильний нападоподібний кашель

Хвильоподібна лихоманка

1827 / 6307
Хворий 46-ти років звернувся до лікаря зі скаргою на біль в суглобах, який посилюється напередодні зміни погоди. У крові виявлено підвищення концентрації сечової кислоти. Посилений розпад якої речовини є найвірогіднішою причиною захворювання?

УТФ

УМФ

ЦМФ

ТМФ

АМФ

1828 / 6307
Чоловік 65-ти років надійшов до неврологічного відділєння з дiагнозом постінсультний синдром. Який препарат найбільш доцільно призначити хворому для прискорення одужання?

Галантаміну гідрохлорид

Ацеклідин

Іпратропіум бромід

Дипіроксим

Ізонітрозин

1829 / 6307
Через 2-3 години після парентерального введення препарату у пацієнта розвинувся коматозний стан, спостерігається дихання типу Чейн-Стокса, зіниці різко звужені, колінний рефлекс збережений. Який препарат міг спричинити отруєння?

Спирт етиловий

Аміназин

Морфін

Сибазон

Фенобарбітал

1830 / 6307
Дитина 3-х років надійшла до клініки з діагнозом отит. Є вірогідність розповсюдження гною із барабанної порожнини через задню стінку. Куди найвірогідніше може потрапити гній?

У внутрішнє вухо

В зовнішній слуховий прохід

В соскоподібну печеру

В задню черепну ямку

В слухову трубу

1831 / 6307
В медико-генетичній консультації при обстеженні хворого хлопчика в крові були виявлені нейтрофільні лейкоцити з однією 'барабанною паличкою'. Наявність якого синдрому можна запідозрити у хлопчика?

Синдром Шерешевського-Тернера

Синдром Дауна

Синдром трисомії-Х

Синдром Едвардса

Синдром Клайнфельтера

1832 / 6307
Після травми передньої поверхні верхньої третини передпліччя у хворого утруднення пронації, послаблення долонного згинання кисті та порушення чутливості шкіри 1-3 пальців на долоні. Який нерв ушкоджено?

n. ulnaris

n. medianus

n. radialis

n. musculocutaneus

n. cutaneus antebrachii medialis

1833 / 6307
До лікарні доставлено дитину 2-х років з уповільненим розумовим і фізичним розвитком, що страждає на часті блювання після прийому їжі. У сечі визначена фенілпіровиноградна кислота. Наслідком якого порушення є дана патологія?

Водно-сольовий обмін

Ліпідний обмін

Фосфорно-кальцієвий обмін

Обмін амінокислот

Вуглеводний обмін

1834 / 6307
Під час патронажу лікар виявив у дитини симетричну шорсткість щік, діарею, порушення нервової діяльності. Нестача яких харчових факторів є причиною такого стану?

Лізин, аскорбінова кислота

Фенілаланін, пангамова кислота

Метіонін, ліпоєва кислота

Нікотинова кислота, триптофан

Треонін, пантотенова кислота

1835 / 6307
Чоловік 38-ми років загинув при спробі підйому вантажу. Розвинувся колаптоїдний стан. На аутопсії виявлений розрив обширної аневризми грудного відділу аорти. За життя страждав на вісцеральний сифіліс. Який патологічний процес в даному випадку обумовив зменшення міцності стінки аорти, її розширення і розрив?

Атрофія м’язового шару

Зникнення еластичних волокон

Зміни інтими за типом 'шагреневої шкіри'

Зникнення колагенових волокон

Новоутворення судин

1836 / 6307
У хворого на міастенію після призначення прозерину з’явилися нудота, діарея, посмикування м’язів язика і скелетних м’язів. Чим можна усунути інтоксикацію?

Мезатон

Фізостигмін

Пірідостигміну бромід

Ізадрин

Атропіну сульфат

1837 / 6307
У хворого з типовою клінічною картиною дизентерії, внаслідок раннього застосування антибіотиків, під час бактеріологічного дослідження випорожнень шигели не виявлені. Титр антишигельозних антитіл в РПГА у парних сироватках у даного хворого підвищився в 4 рази. Про що це свідчить?

Неспецифічна реакція

Виключає діагноз дизентерії

Вакцинальна реакція

Підтверджує діагноз дизентерії

Переніс дизентерію раніше

1838 / 6307
Чоловік помер від гострого інфекційного захворювання, яке супроводжувалось гарячкою, жовтяницею, геморагічною висипкою на шкірі та слизових оболонках, а також гострою нирковою недостатністю. При гістологічному дослідженні тканини нирки (забарвлення за Романовським-Гімзою) виявлені звивисті бактерії, які мають вигляд букв С та S. Які бактерії були виявлені?

Борелії

Трепонеми

Спіроли

Кампілобактерії

Лептоспіри

1839 / 6307
Чоловіку 46-ти років, що хворіє на дифузний токсичний зоб, була проведена операція резекції щитоподібної залози. Після операції відмічаються відсутність апетиту, диспепсія, підвищена нервово-м’язова збудливість. Маса тіла не збільшилася. Температура тіла у нормі. Чим, із нижче переліченого, обумовлений стан хворого?

Зниженням продукції тироксину

Підвищенням продукції тироксину

Підвищенням продукції кальцитоніну

Зниженням продукції паратгормону

Підвищенням продукції тиреоліберину

1840 / 6307
з дихальною недостатністю рН крові 7,35. Визначення pC02 40. показало наявність гіперкапнії. При дослідженні рН сечі відзначається підвищення її кислотності. Яка форма порушення кислотно-основного стану в даному випадку?

Алкалоз газовий, декомпенсований

Ацидоз газовий, компенсований

Алкалоз газовий, компенсований

Ацидоз метаболічний, декомпенсований

Ацидоз метаболічний, компенсований

1841 / 6307
Чоловіку з виразковою хворобою дванадцятипалої кишки лікар після курсу терапії пропонує вживання соків із капусти та картоплі. Вміст яких речовин в цих овочах сприяє профілактиці та загоєнню виразок?

Пантотенова кислота

Вітамін K

Вітамін Бі

Вітамін U

Вітамін C

1842 / 6307
У 12-ти річного хлопчика в сечі виявлено високий вміст усіх амінокислот аліфатичного ряду. При цьому відмічена найбільш висока екскреція цистину та цистеїну. Крім того, УЗД нирок показало наявність каменів у них. Яка патологія найбільш вірогідна?

Хвороба Хартнупа

Фенілкетонурія

Алкаптонурія

Цистинурія

Цистит

1843 / 6307
У дитини 5-ти років гостра правобічна нижньодольова пневмонія. При посіві харкотиння виявлено, що збудник захворювання стійкий до пеніциліну, але чутливий до макролідів. Який препарат найбільш доцільно використати у даному випадку?

Азитроміцин

Ампіцилін

Гентаміцин

Тетрациклін

Стрептоміцин

1844 / 6307
У дитини 10-ти років хірург запідозрив запалення дивертикула Меккеля, що потребує оперативного втручання. Ревізія якої ділянки кишечника необхідна, щоб знайти дивертикул?

Низхідна ободова кишка

Висхідна ободова кишка

0,5 м порожньої кишки від зв’язки Трейца

1м клубової кишки від місця впадіння її в товсту кишку

20 см клубової кишки від клубово-сліпокишкового кута

1845 / 6307
Чоловік 44-х років з інфарктом міокарда, помер від лівошлуночкової недостатності. На аутопсії: набряк легень, дрібнокраплинні крововиливи у серозних та слизових оболонках. Мікроскопічно: дистрофічні та некробіотичні зміни епітелію проксимальних канальців нирок, у печінці - центролобулярні крововиливи та осередки некрозу. Який з видів порушення кровообігу найбільш вірогідний?

Хронічне загальне венозне повнокров’я

Хронічне недокрів’я

Гостре загальне венозне повнокров’я

Артеріальна гіперемія

Гостре недокрів’я

1846 / 6307
У хворого гнійне запалення клиноподібної пазухи. В яку частину носової порожнини витікає гній?

Meatus nasi communis

Meatus nasi medius

Meatus nasi inferior

Meatus nasi superior

1847 / 6307
До стаціонару надійшов хворий з діагнозом: виразкова хвороба 12-палої кишки у фазі загострення, виразка цибулини 12-палої кишки. Аналіз шлункового соку показав підвищення секреторної та кислотоутворюючої функцій шлунка. Оберіть препарат, що знижує секреторну функцію залоз шлунка за рахунок блокади Н2 -рецепторів:

Атропіну сульфат

Фамотидин

Метацин

Екстракт беладони сухий

Платифілін

1848 / 6307
У хворого 30-ти років із гострим запаленням підшлункової залози (панкреатитом) виявлено порушення порожнинного травлення білків. Це може бути пов’язано із недостатнім синтезом та виділенням залозою такого ферменту:

Пепсин

Ліпаза

Амілаза

Дипептидаза

Трипсин

1849 / 6307
Хворому тривалий час вводили високі дози гідрокортизону, внаслідок чого настала атрофія однієї з зон кори наднирників. Яка це зона?

Клубочкова і сітчаста

Клубочкова

Сітчаста

Пучкова

1850 / 6307
Жінка 25-ти років скаржиться на постійний біль у ділянці серця, задишку під час рухів, загальну слабкість. Об’єктивно: шкіра бліда та холодна, акроціаноз. Ps- 96/хв., АТ- 105/70 мм рт.ст. Межа серця зміщена на 2 см вліво. Перший тон над верхівкою серця послаблений, систолічний шум над верхівкою. Діагностовано недостатність мітрального клапана серця. Чим обумовлене порушення кровообігу?

Пошкодження міокарда

Перевантаження міокарда підвищеним опором відтоку крові

Збільшення об’єму судинного русла

Зниження об’єму циркулюючої крові

Перевантаження міокарда збільшеним об’ємом крові

1851 / 6307
У потерпілого в аварії спостерігається кровотеча з м’яких тканин попереду від кута нижньої щелепи. Яку судину необхідно перев’язати для зупинки кровотечі?

A.carotis interna

A.alveolaris inferior

A.facialis

A.temporalis superficialis

A.lingvalis

1852 / 6307
При мікроскопії зіскрібку з язика, забарвленого за Грамом, знайдені овальні, округлі, темно-фіолетового кольору, видовжені ланцюжки клітин, що брунькуються. Про збудника якого захворювання може йти мова?

Стафілококова інфекція

Кандидоз

Дифтерія

Актиномікоз

Стрептококова інфекція

1853 / 6307
В препараті представлений порожнистий орган. Слизова оболонка вкрита дворядним війковим епітелієм, що переходить в однорядний. М’язова пластинка слизової добре розвинена по відношенню до товщини всієї стінки. Хряща і залоз немає. Який орган представлений в препараті?

Сечовий міхур

Гортань

Дрібний бронх

Середній бронх

Трахея

1854 / 6307
Основна маса азоту з організму виводиться у вигляді сечовини. Зниження активності якого ферменту в печінці призводить до гальмування синтезу сечовини і нагромадження амоніаку в крові і тканинах?

Уреаза

Амілаза

Карбамоїлфосфатсинтаза

Пепсин

Аспартатамінотрансфераза

1855 / 6307
Хвора 36-ти років страждає на колагеноз. Збільшення вмісту якого метаболіту найбільш вірогідно буде встановлено у сечі?

Оксипролін

Індикан

Сечовина

Креатинін

Уробіліноген

1856 / 6307
При копрологічному дослідженні встановлено, що кал знебарвлений, у ньому знайдено краплі нейтрального жиру. Найбільш вірогідною причиною цього є порушення:

Кислотності шлункового соку

Секреції підшлункового соку

Секреції кишкового соку

Надходження жовчі до кишечнику

Процесів всмоктування в кишечнику

1857 / 6307
При обстеженні хворого виявлені наступні клінічні прояви: шкірні покриви рожеві, теплі на дотик, сухі, ЧСС-92/хв.,ЧДР- 22/хв., температура тіла -39,2oC. Яке співвідношення процесів утворення і віддачі тепла в описаному періоді лихоманки?

Зниження тепловіддачі на фоні незміненої теплопродукції

Теплопродукція дорівнює тепловіддачі

Посилення теплопродукції без зміни тепловіддачі

Теплопродукція нижче за тепловіддачу

Теплопродукція перевищує тепловіддачу

1858 / 6307
Хворий 21-го року надійшов до стаціонару з загостренням хронічного тонзиліту. Скаржиться на слабкість, задуху при помірному фізичному навантаженні. Температура 37,5oC. ЧСС- 110/хв. ЕКГ: ритм синусовий, інтервал PQ подовжений. Яка аритмія у хворого?

Внутрішньопередсердна блокада

Порушення внутрішньошлуночкової провідності

Передсердно-шлуночкова блокада ІІ ст.

Передсердно-шлуночкова екстрасистолія

Передсердно-шлуночкова блокада І ст.

1859 / 6307
Секреція яких гормонів гіпофізу гальмується після прийому оральних контрацептивів, які містять статеві гормони?

Тиреотропні

Соматотропний

Гонадотропні

Окситоцин

Вазопресин

1860 / 6307
У хворого на гіпертонічну хворобу виявлено високий рівень реніну в крові. Якому з гіпотензивних засобів слід віддати перевагу в цьому випадку?

Празозин

Лізиноприл

Ніфедіпін

Анаприлін

Дихлотіазид

1861 / 6307
При гістологічному дослідженні нирки у кірковій речовині визначається каналець, вистелений одношаровим кубічним облямівчастим епітелієм, цитоплазма якого забарвлена оксифільно. Який сегмент нефрону виявлений у препараті?

Проксимальний звивистий каналець

Дистальний прямий каналець

Збірна трубочка

Петля Генле

Дистальний звивистий каналець

1862 / 6307
Встановлено ураження вірусом ВШ Т-лімфоцитів. При цьому фермент вірусу зворотня траскриптаза (РНК-залежна ДНК-полімераза) каталізує синтез:

Вірусної і-РНК на матриці ДНК

ДНК на вірусній р-РНК

і-РНК на матриці вірусного білка

ДНК на матриці вірусної і-РНК

Вірусної ДНК на матриці ДНК

1863 / 6307
Людина зробила максимально глибокий видих. Як називається об’єм повітря, що знаходиться в її легенях після цього?

Функціональна залишкова ємність легень

Ємність вдиху

Альвеолярний об’єм

Резервний об’єм видиху

Залишковий об’єм

1864 / 6307
Внаслідок активації іонних каналів зовнішньої мембрани збудливої клітини значно збільшився її потенціал спокою. Які канали були активовані?

Повільні кальцієві

Натрієві та кальцієві

Калієві

Швидкі кальцієві

Натрієві

1865 / 6307
Хворому з кардіогенним шоком, гіпотензією, ядухою і набряками ввели неглікозидний кардіотонік. Який саме препарат був введений хворому?

Бемегрид

Добутамін

Кофеїн-бензоат натрію

Етимізол

Кордіамін

1866 / 6307
Після травми хворий не може розігнути руку в ліктьовому суглобі. Порушення функції якого м’яза може бути причиною цього?

Musculus subscapularis

Musculus brachialis

Musculus triceps brachii

Musculus biceps brachii

Musculus coraco-brachialis

1867 / 6307
Під час гістологічного дослідження стулок мітрального клапана серця жінки 30-ти років було встановлено, що ендотеліальні клітини вогнищево десквамовані, в цих ділянках на поверхні стулки розташовані дрібні тромботичні нашарування, сполучна тканина стулки з явищами мукоїдного набухання з ділянками склерозу та васкуляризації. Діагностуйте вид клапанного ендокардиту:

Дифузний

Гострий бородавчастий

Фібропластичний

Поліпозно-виразковий

Поворотньо-бородавчастий

1868 / 6307
У збільшеному шийному лімфатичному вузлі дівчинки 14-ти років мікроскопічно було встановлено, що тканинна будова вузла порушена, лімфоїдні фолікули відсутні, є ділянки склерозу та вогнища некрозу, клітинний склад вузла поліморфний, присутні лімфоцити, еозинофіли, атипові клітини великих розмірів з багаточасточковими ядрами (клітини Березовського-Штернберга) та також одноядерні клітини великих розмірів. Діагностуйте захворювання:

Хронічний лімфолейкоз

Гострий лімфолейкоз

Лімфома Беркіта

Лімфогранулематоз

Грибоподібний мікоз

1869 / 6307
Який з перерахованих сечогінних засобів НЕ БУДЕ проявляти діуретичного ефекту у пацієнта з хворобою Аддісона?

Гіпотіазид

Фуросемід

Спіронолактон

Етакринова кислота

Тріамтерен

1870 / 6307
Під час розтину чоловіка, у якого після поранення кінцівки виникло тривале нагноєння рани, і який помер при явищах інтоксикації, знайдено загальне виснаження, зневоднення, бура атрофія печінки, міокарду, селезінки, поперечносмугастої мускулатури та амілоїдоз нирок. Який з перелічених діагнозів найбільш вірогідний?

Септикопіємія

Хроніосепсис

Септицемія

Хвороба Чорногубова

Бруцельоз

1871 / 6307
Препарат чинить згубний вплив на еритроцитарні форми малярійних плазмодіїв, дизентерійну амебу. Застосовується для лікування та профілактики малярії, лікування амебіазу і колагенозів. Визначте цей препарат:

Еметину гідрохлорид

Хінгамін

Тетрациклін

Хінін

Еритроміцин

1872 / 6307
У хворого у крові збільшення концентрації пірувату, значна кількість його екскретується з сечею. Який авітаміноз спостерігається у хворого?

В6

В2

E

B3

B1

1873 / 6307
Дитина страждає на ідіосинкразію лікарської речовини. Чим зумовлена ця реакція?

Виснаження субстрату, з яким взаємодіє лікарська речовина

Спадкова ензимопатія

Інгібування мікросомальних ферментів печінки

Супутнє захворювання органу-мішені

Накопичення лікарської речовини в організмі

1874 / 6307
У одного з батьків запідозрили носійство рецесивного гена фенілкетонурії. Який ризик народження у цій сім’ї дитини, хворої на фенілкетонурію?

25%

100%

75%

50%

0%

1875 / 6307
Патологоанатом у біоптаті шкіри побачив гостре серозно-геморагічне запалення і ділянку некрозу. З анамнезу: захворювання почалося з появи невеликої червоної плями, в центрі якої утворився міхур із серозно-геморагічною рідиною. Згодом центральна частина стала чорною. Який діагноз найбільш вірогідний?

Карбункул стрептококовий

Сибірковий карбункул

Актиномікоз шкіри

Алергічний дерматит

Хімічний дерматит

1876 / 6307
В легенях пацієнта, який протягом 9-ти років працював шліфувальником каменю, виявлені дрібні округлої форми щільні вузлики, що складаються зі сполучної тканини. На периферії цих вузликів розташовані макрофаги. Проявом якого захворювання є зміни в легенях?

Бронхоектатична хвороба

Силікоз

Хронічний бронхіт

Гостра пневмонія

Бронхіальна астма

1877 / 6307
У пацієнта тривалість інтервалу P — Q на ЕКГ перевищує норму при нормальній тривалості зубця P. Причиною цього є зменшення швидкості проведення збудження:

Сино-атріальним вузлом

Волокнами Пуркін’є

Атріо-вентрикулярним вузлом

Ніжками пучка Гіса

Пучком Гіса

1878 / 6307
У підлітка 12-ти років, який впродовж 3-х місяців різко схуд, вміст глюкози у крові 50 ммоль/л. У нього розвинулася кома. Який головний механізм її розвитку?

Кетонемічний

Гіпоглікемічний

Лактацидемічний

Гіперосмолярний

Гіпоксичний

1879 / 6307
Який шлях віддачі тепла тілом робітників парникового господарства є найефективнішим при температурі повітря 36oC та відносній його вологості -70%?

Випаровування рідини

Конвекція

Теплорадіація

Теплопроведення

1880 / 6307
Внаслідок землетрусу чоловік 50-ти років дві доби перебував під завалом. Після звільнення з-під завалу рятівниками у нього був встановлений синдром тривалого розчавлення. Виникнення якого ускладнення в подальшому найбільш вірогідне?

Гостра ниркова недостатність

Гостра печінкова недостатшсть

Гостра серцева недостатшсть

Гостра дихальна недостатшсть

Гостра судинна недостатшсть

1881 / 6307
У ліквідатора наслідків аварії на АЕС під час перебігу гострої променевої хвороби виник геморагічний синдром. Що має найбільше значення в патогенезі цього синдрому?

Тромбоцитопенія

Порушення структури стінки судин

Підвищення активності факторів фібринолізу

Підвищення активності факторів систем протизсідання крові

Зменшення активності факторів зсідання крові

1882 / 6307
Для лікування жовтяниць показано призначення барбітуратів, які індукують синтез УДФглюкуронілтрансферази. Лікувальний ефект при цьому обумовлений утворенням:

Протопорфирину

Білівердину

Прямого (кон’югованого) білірубіну

Гему

Непрямого (некон’югованого) білірубіну

1883 / 6307
До лікарні потрапив чоловік 35-ти років, який втратив зір на одне око. З анамнезу відомо, що хворий часто вживав недостатньо просмажений шашлик. Після рентгенологічного обстеження та проведення імунологічних реакцій лікар поставив діагноз цистіцеркоз. Який гельмінт є збудником цього захворювання?

Trichocephalus trichiurus

Trichinella spiralis

Taeniarhynchus saginatus

Taenia solium

Diphyllobothrium latum

1884 / 6307
До лікарні після автокатастрофи надійшов юнак 18-ти років. У травматологічному відділенні виявлені численні травми м’яких тканин обличчя в ділянці медіального кута ока, які призвели до масивної кровотечі. Який артеріальний анастомоз міг бути пошкоджений у цьому регіоні?

a. subclavia et a. ophthalmica

a. carotis externa et a. carotis interna

a. carotis externa et a. subclavia

a. carotis interna et a. subclavia

a. carotis interna et a. ophthalmica

1885 / 6307
У чоловіка 75-ти років, який довгий час страждав на атеросклероз церебральних судин, на аутопсії у правій тім’яно-скроневої ділянці головного мозку виявлено вогнище неправильної форми, млявої консистенції, сірого кольору. Яка найбільш вірогідна причина розвитку цього процесу?

Тромбоз правої середньої мозкової артерії

Тромбоз судини м’якої мозкової оболонки

Тромбоз базилярної артерії

Тромбоз правої передньої мозкової артерії

Тромбоз правої задньої мозкової артерії

1886 / 6307
Під час розтину трупа чоловіка зі злоякісною пухлиною шлунка, що помер від ракової інтоксикації, в задньонижніх відділах легень виявлені щільні сіро-червоного кольору неправильної форми осередки, які виступають над поверхнею розрізу. Мікроскопічно: у просвіті, стінках дрібних бронхів та альвеолах виявляється ексудат, в якому багато нейтрофілів. Про яке захворювання свідчать зміни у легенях померлого?

Гостра серозна бронхопневмонія

Гостра гнійна бронхопневмонія

Гострий бронхіт

Проміжна пневмонія

Крупозна пневмонія

1887 / 6307
При оперативному втручанні з приводу каменів жовчних ходів хірург повинен знайти загальну печінкову протоку. Між листками якої зв’язки вона знаходиться?

Кругла зв’язка печінки

Печінково-шлункова

Печінково-дванадцятипала

Печінково-ниркова

Венозна зв’язка

1888 / 6307
Під час розтину тіла померлої дитини 1,5 років виявлені: геморагічний висип на шкірі, помірна гіперемія та набряк слизової оболонки носоглотки, дрібні крововиливи у слизових оболонках і внутрішніх органах, різкі дистрофічні зміни у печінці, міокарді, гострий некротичний нефроз, масивні крововиливи у наднирниках. Для якого захворювання найбільш характери виявлєні зміни?

Висипний тиф

Дифтєрія

Кір

Скарлатина

Менінгококова інфекція

1889 / 6307
Хвора 75-ти років доставлена до офтальмологічного відділення лікарні зі скаргами на погіршення зору. При об’єктивному дослідженні встановлена наявність пухлини мозку, розташованої в ділянці лівого зорового тракту. При цьому у хворої спостерігається випадіння поля зору в:

Правих і лівих половинах сітківки правого ока

Правих половинах сітківки обох очей

Правих і лівих половинах сітківок обох очей

Лівих половинах сітківки обох очей

Правих і лівих половинах сітківки лівого ока

1890 / 6307
У хворого на ішемічну хворобу серця відзначається гіпертрофія міокарда, тахікардія, зниження ХОК. Який з механізмів є провідним в ушкодженні кардіоміоцитів у даному випадку?

Втрата Ca2+ кардіоміоцитами

Пошкодження специфічних мембранних насосів

Збільшення числа а та β-адренорецепторів

Дегідратація кардіоміоцитів

Втрата Mg2+ кардіоміоцитами

1891 / 6307
У хворого на жовтяницю у крові підвищений вміст прямого білірубіну та жовчних кислот; у сечі відсутній стеркобіліноген. При якій жовтяниці можлива наявність цих ознак?

Механічна

Надпечінкова

Печінкова

Гемолітична

Паренхіматозна

1892 / 6307
Проводиться вивчення максимально спіралізованих хромосом каріотипу людини. При цьому процес поділу клітини припинили на стадії:

Інтерфаза

Телофаза

Метафаза

Анафаза

Профаза

1893 / 6307
У людини вимірюють енерговитрати натщесерце, лежачи, в умовах фізичного і психічного спокою, при температурі комфорту. В який час енерговитрати будуть найбільшими?

10-12 годин дня

14-16 годин дня

7-8 годин ранку

3-4 години ранку

17-18 годин вечора

1894 / 6307
При визначенні енерговитрат організму людини методом непрямої калориметрії встановлено, що за одну хвилину споживається 1000 мл кисню і виділяється 800 мл вуглекислого газу. Який дихальний коефіцієнт у досліджуваної людини?

0,9

1,25

1,0

0,8

0,84

1895 / 6307
Тварині внутрішньовенно ввели концентрований розчин хлориду натрію, що зумовило зниження реабсорбції іонів натрію у канальцях нирок. Внаслідок яких змін секреції гормонів це відбувається?

Зменшення натрійуретичного фактора

Зменшення вазопресину

Збільшення вазопресину

Збільшення альдостерону

Зменшення альдостерону

1896 / 6307
У хворого діагностовано алкаптонурію. Вкажіть фермент, дефект якого є причиною цієї патології:

Піруватдегідрогеназа

Глутаматдегідрогеназа

ДОФА-декарбоксилаза

Фенілаланінгідроксилаза

Оксидаза гомогентизинової кислоти

1897 / 6307
При хронічному передозуванні глюкокортикоїдів розвивається гіперглікемія. Назвіть процес вуглеводного обміну, за рахунок якого збільшується концентрація глюкози у плазмі крові:

Пентозофосфатний цикл

Глюконеогенез

Аеробний гліколіз

Шкогенез

Глікогеноліз

1898 / 6307
У хворої 49-ти років відзначається обмеження довільних рухів у лівих кінцівках. Тонус м’язів у лівих руці та нозі підвищений за спастичним типом, посилені місцеві сухожилкові рефлекси, виявляються патологічні рефлекси. Який найбільш вірогідний механізм призвів до розвитку м’язової гіпертонії та гіперрефлексії?

Зниження гальмівних низхідних впливів

Гальмування мотонейронів кори головного мозку

Активація синаптичної передачі імпульсів

Активація збуджуючих впливів з вогнища інсульту

Активація мотонейронів внаслідок інсульту

1899 / 6307
При виконуванні вправ на колоді гімнастка втратила рівновагу і впала. Із збудження, перш за все, яких рецепторів розпочнуться рефлекси, що забезпечать відновлення порушеної пози?

Вестибулорецептори

Рецептори завитки

Пропріорецептори

Отолітові вестибулорецептори

Ампулярні вестибулорецептори

1900 / 6307
Вірус імунодефіциту людини, маючи на своїй поверхні антигени gp — 41 та gp — 120, взаємодіє з клітинами-мішенями організму. Виберіть серед перерахованих антигени лімфоцитів людини, з якими комплементарно зв’язується gp — 120 вірусу:

CD 3

CD 8

CD 28

CD 19

CD 4

1901 / 6307
В анотації до препарату вказано, що він містить антигени збудника черевного тифу, адсорбовані на стабілізованих еритроцитах барана. З якою метою використовують цей препарат?

Для виявлення антитіл в реакції зв’язування комплементу

Для виявлення антитіл в реакції непрямої гемаглютинації

Для виявлення антитіл в реакції затримки гемаглютинації

Для серологічної ідентифікації збудника черевного тифу

Для виявлення антитіл в реакції Відаля

1902 / 6307
При обстеженні вагітної жінки з резус-негативною кров’ю знайдено високий рівень антиеритроцитарних антитіл, для зниження якого їй було підшито шкірний клапоть її резус-позитивного чоловіка. Через 2 тижні клапоть відторгнувся, мікроскопічно в ньому знайдено порушення кровообігу, набряк, клітинну інфільтрацію переважно лімфоцитами, нейтрофілами та макрофагами. Який з перелічених патологічних процесів найбільш вірогідний?

Інтерстиційне запалення

Реакція гіперчутливості сповільненого типу

Реакція гіперчутливості негайного типу

Гранулематозне запалення

Трансплантаційний імунітет

1903 / 6307
Дівчина 17-ти років з метою суїциду прийняла велику дозу фенобарбіталу. Після прибуття на місце події лікар швидкої допомоги швидко промив шлунок, ввів бемегрид і розчин натрію гідрокарбонату внутрішньовенно. З якою метою лікар ввів натрію гідрокарбонат?

Для інактивації фенобарбіталу

Для стимуляції дихання

Для нормалізації артеріального тиску

Для пробуджуючого ефекту

Для підвищення ниркової екскреції фенобарбіталу

1904 / 6307
Стоматолог під час огляду порожнини рота на межі середньої і задньої третини спинки язика виявив запалені сосочки. Які сосочки язика запалені?

Papillae vallatae

Papillae conicae

Papillae fungiformes

Papillae filiformes

Papillae foliatae

1905 / 6307
При лабораторному дослідженні крові пацієнта 44-х років виявлено, що вміст білків у плазмі становить 40 г/л. Як це впливає на транскапілярний обмін води?

Зменшуються фшьтращя та реабсорбція

Зменшується фшьтращя, збільшується реабсорбція

Обмін не змінюється

Збільшується фільтрація, зменшується реабсорбція

З6ільшуються фiльтрацiя та реабсорбція

1906 / 6307
У людини звужені зіниці. Чим це зумовлено?

Зростання тонусу симпатичних центрів

Дія норадреналіну

Збільшення активності симпато-адреналової системи

Зростання тонусу парасимпатичних центрів

Дія адреналіну

1907 / 6307
У чоловіка 46-ти років на шкірі визначалась темна пляма, яка вибухала та не спричиняла турбот. З часом пляма почала збільшуватись, з’явився біль, колір став чорно-коричневим і почав пальпуватися вузлик. При гістологічному дослідженні видаленої тканини визначалися веретеноподібні і поліморфні клітини з численними мітозами, цитоплазма яких вміщувала пігмент бурого кольору. Про яку пухлину йдеться?

Невус

Гемангіома

Базаліома

Меланома

1908 / 6307
У немовля с пілороспазмом внаслідок блювання, що часто повторювалося, з’явилися слабкість, гіподинамія, іноді судоми. Яка форма порушення кислотно-основного стану в нього спостерігається?

Видільний алкалоз

Видільний ацидоз

Газовий алкалоз

Екзогенний негазовий ацидоз

Метаболічний ацидоз

1909 / 6307
На електронній мікрофотографії клітини, яка має паличкоподібне ядро та веретеноподібну форму, у цитоплазмі спостерігається велика кількість проміжних мікрофіламентів, які містять десмін. З якої тканини зроблено зріз?

Сполучна

М’язова

Нервова

Епітеліальна

1910 / 6307
У важкоатлета при підйомі штанги відбувся розрив грудної лімфатичної протоки. Вкажіть найбільш вірогідне місце пошкодження:

У ділянці попереково-крижового зчленування

У ділянці шиї

У ділянці аортального отвору діафрагми

У задньому середостінні

У місці впадіння у венозний кут

1911 / 6307
У жінки, що хворіє на остеохондроз, з’явився різкий біль у плечовому суглобі, який посилювався при відведенні плеча. Ураження якого нерва може бути причиною цих симптомів?

Підключичний нерв

Грудо-спинний нерв

Підлопатковий нерв

Дорсальний нерв лопатки

Пахвовий нерв

1912 / 6307
На рентгенограмі нирок при пієлографії лікар виявив ниркову миску, в яку безпосередньо впадали малі чашечки (великі були відсутні). Яку форму сечовивідних шляхів нирки виявив лікар?

Ампулярна

Деревоподібна

Фетальна

Зріла

Ембріональна

1913 / 6307
У хворої внаслідок запалення порушена ендокринна функція фолікулярних клітин фолікулів яєчника. Синтез яких гормонів буде пригнічений?

Фолістатин

Фолікулостимулюючий гормон

Лютропін

Прогестерон

Естрогени

1914 / 6307
У хворої 19-ти років з дитинства відмічалося зниження гемоглобіну до 90-95 г/л. У крові під час госпіталізації: ер.- 3, 2 • 1012/л, Hb- 85 г/л, КП- 0,78; лейк.- 5, 6 • 109/л, тромб.- 210 • 109 /л. В мазку: анізоцитоз, пойкілоцитоз, мішенеподібні еритроцити. Ретикулоцити - 6%. Лікування препаратами заліза було неефективне. Яку патологію системи крові можна запідозрити в даному випадку?

Мембранопагія

Ферментопагія

Серпоподібноклітинна анемiя

Таласемія

Фавізм

1915 / 6307
Хворому на гострий інфаркт міокарда у комплексній терапії було призначено гепарин. Через деякий час після введення даного препарату з’явилась гематурія. Який антагоніст гепарину необхідно ввести хворому для усунення даного ускладнення?

Неодикумарин

Протаміну сульфат

Амінокапронова кислота

Вікасол

Фібриноген

1916 / 6307
Пацієнта турбують поліурія (7л на добу) і полідипсія. При обстеженні не виявлено ніяких розладів вуглеводного обміну. Дисфункція якої ендокринної залози може бути причиною даних порушень?

Нейрогіпофіз

Острівці підшлункової залози

Аденогіпофіз

Мозкова речовина наднирників

Кора наднирників

1917 / 6307
У чоловіка 30-ти років перед операцією визначили групову належність крові. Кров резуспозитивна. Реакцію аглютинації еритроцитів не викликали стандартні сироватки груп 0αβ (I), Аβ (II), Вα (III). Досліджувана кров належить до групи:

АВ (IV)

0αβ (I)

Аβ (II)

Вα (III)

1918 / 6307
У жінки 20-ти років під час медичного огляду при пальпації в молочній залозі виявлено щільний інкапсульований вузол діаметром 1,0 см. Результат післяопераційного біопсійного дослідження: розростання сполучної тканини навколо протоків молочної залози та залозисті елементи різного діаметру, що не утворюють часточок, без ознак клітинного атипізму. Яка відповідь патологоанатома?

Фіброзний рак

Аденома

Метастаз раку

Фіброаденома

Фіброма

1919 / 6307
У хворого 38-ми років на 3-му році захворювання на системний червоний вівчак виявлене дифузне ураження нирок, що супроводжується масивними набряками і вираженою протеїнурією. Що є найбільш вірогідною причиною розвитку протеїнурії у пацієнта?

Аутоімунне ушкодження нирок

Запальне ураження сечового міхура

Немічне ушкодження нирок

Запальне ураження сечовивідних шляхів

Асептичне ураження нирок

1920 / 6307
Студент на екзамені не зміг вірно відповісти на питання екзаменаційного білету, що супроводжувалося почервонінням шкіри обличчя, відчуттям жару і невпевненістю поведінки. Який вид артеріальної гіперемії розвинувся у даному випадку?

Нейротонічна

Постішемічна

Нейропаралітична

Патологічна

Метаболічна

1921 / 6307
У хворого з кровотечею розвинулась гостра ниркова недостатність, що спричинила смерть. На аутопсії макроскопічно: нирки збільшені з широким блідо-рожевим кірковим шаром, різко відмежованим від темно-червоних пірамід. Мікроскопічно: відсутність ядер епітелію звивистих канальців, тубулорексіс, венозний застій, ядра клітин судинних клубочків та прямих канальців збережені. Яка патологія нирок розвинулась у хворого?

Некронефроз

Пієлонефрит

Гломерулонефрит

Інфаркт

Нефроз

1922 / 6307
Хвора звернулась до лікаря із скаргами на біль та обмеження рухів у колінних суглобах. Який з нестероїдних протизапальних засобів краще призначити, враховуючи наявність в анамнезі хронічного гастродуоденіту?

Промедол

Бутадіон

Диклофенак-натрій

Целекоксиб

Кислота ацетилсаліцилова

1923 / 6307
У хворого з’явилися жовтушність шкіри, склер та слизових оболонок. У плазмі крові підвищений рівень загального білірубіну, в калі - рівень стеркобіліну, в сечі - уробіліну. Який вид жовтяниці у хворого?

Гемолітична

Холестатична

Паренхіматозна

Обтураційна

Хвороба Жільбера

1924 / 6307
У плазмі крові здорової людини знаходиться декілька десятків білків. При захворюванні організму з’являються нові білки, зокрема 'білок гострої фази'. Таким білком є:

С-реактивний білок

Імуноглобулін G

Протромбін

Фібриноген

Імуноглобулін А

1925 / 6307
Під час бійки у чоловіка виникла зупинка серця внаслідок сильного удару у верхню ділянку передньої черевної стінки. Який із зазначених механізмів спричинив зупинку серця?

Симпатичні умовні рефлекси

Парасимпатичні безумовні рефлекси

Периферичні рефлекси

Парасимпатичні умовні рефлекси

Симпатичні безумовні рефлекси

1926 / 6307
Для людини існує суворе обмеження в часі перебування на висоті понад 800 метрів над рівнем моря без кисневих балонів. Що є лімітуючим фактором для життя в даному випадку?

Сила земного тяжіння

Температура

Рівень вологості

Парціальний тиск кисню в повітрі

Рівень ультрафіолетового опромінення

1927 / 6307
У хворого із вираженим пневмосклерозом після перенесеного інфільтративного туберкульозу легень розвинулась дихальна недостатність. До якого патогенетичного типу вона відноситься?

Рестриктивний

Дисрегуляційний

Рефлекторний

Апнеїстичний

Обструктивний

1928 / 6307
Під час проведення хірургічних маніпуляцій було використано новокаїн з метою знеболення. Через 10 хвилин у хворого з’явилася блідість шкірних покривів, задишка, гіпотензія. Алергічну реакцію якого типу можна запідозрити?

Анафілактична

Стимулююча

Імунокомплексна

Клітинно-опосередкована

Цитотоксична

1929 / 6307
Чутливий нервовий ганглій складається з нейроцитів кулястої форми з одним відростком, який на певній відстані від перикаріону поділяється на аксон і дендрит. Як називаються такі клітини?

уніполярні

біполярні

псевдоуніполярні

аполярні

мультиполярні

1930 / 6307
У препараті трубчастого органу, зафарбованому орсеїном, виявлено близько 50 товстих мембран, які мають хвилясту форму і складають основу середньої оболонки органу. Який це орган?

Трахея

Артерія м’язового типу

Стравохід

Аорта

Стінка серця

1931 / 6307
Встановлено, що деякі сполуки, наприклад, токсини грибів та деякі антибіотики, можуть пригнічувати активність РНК-полімерази. Порушення якого процесу відбувається в клітині у випадку пригнічування даного ферменту?

Репарація

Процесінг

Транскрипція

Реплікація

Трансляція

1932 / 6307
У дорослої людини системний артеріальний тиск знизився з 120/70 до 90/50 мм рт.ст., що викликало рефлекторне звуження судин. У якому з зазначених органів звуження судин буде найменшим?

Серце

Скелетні м’язи

Кишечник

Шкіра

Печінка

1933 / 6307
На електронній мікрофотографії фрагменту власної залози шлунка представлена велика клітина неправильної кулястої форми, у цитоплазмі якої є велика кількість внутрішньоклітинних канальців та мітохондрій. Визначте дану клітину:

Головна

Парієтальна

Слизова

Недиференційована

Ендокринна

1934 / 6307
крові у спортсмена виявлено: ер.- 5,5 • 1012/л, Hb- 180 г/л, лейк.- 7-109 134. /л,н.- 64%, б.- 0,5%,е.- 0,5%, м.-8%,л.-27%. Такі показники свідчать про стимуляцію, перш за все:

Гранулоцитопоезу

Лімфопоезу

Лейкопоезу

Еритропоезу

Імуногенезу

1935 / 6307
Хворий помер від наростаючої легенево-серцевої недостатності. При гістологічному дослідженні виявлено: дифузне ураження легенів з інтерстиціальним набряком, інфільтрацією інтерстиціальної тканини лімфоцитами, макрофагами, плазмоцидами; пневмофіброз, панацинарна емфізема. Який найбільш вірогідний діагноз?

Бронхопневмонія

Бронхіальна астма

Ателектаз легенів

Фіброзуючий альвеоліт

Хронічний бронхіт

1936 / 6307
У людини з нападом бронхоспазму необхідно зменшити вплив блукаючого нерва на гладеньку мускулатуру бронхів. Які мембранні циторецептори доцільно заблокувати для цього?

α-адренорецептори

α- та β-адренорецептори

M-холінорецептори

H-холінорецептори

β-адренорецептори

1937 / 6307
У хворого виявлена виразка шлунка. При бактеріологічному дослідженні біоптату, отриманого із ушкодженої ділянки шлунка, на 5-й день на шоколадному агарі виросли дрібні колонії грамнегативних оксидоредуктазопозитивних звивистих бактерій. Який із перерахованих мікроорганізмів є найбільш вірогідним збудником?

Campilobacter jejuni

Chlamydia trachomatis

Campilobacter fetus

Helicobacter pylori

Mycoplasma hominis

1938 / 6307
Виділяють декілька груп молекулярних механізмів, які мають важливе значення в патогенезі ушкодження клітин, що сприяє розвитку патології. Які процеси забезпечують протеїнові механізми ушкодження?

Активація фосфоліпаз

Ацидоз

Осмотичне розтягнення мембран

Пригнічення ферментів

Перекисне окиснення ліпідів

1939 / 6307
У пацієнта діагностовано компресійний перелом поперекового хребця. При цьому різко збільшилась кривизна лордозу поперекового відділу хребта. Ушкодженням якої зв’язки може супроводжуватись така зміна кривизни хребта?

Задня поздовжня

Жовта

Клубово-поперекова

Передня поздовжня

Міжостиста

1940 / 6307
При розтині тіла померлого 49-ти років, який хворів на крупозну пневмонію і помер від пневмококового сепсису, в лівій плевральній порожнині містилось до 700 мл каламутної рідини зеленувато-жовтого кольору з неприємним запахом. Листки плеври тьмяні, повнокровні. Назвіть клініко-морфологічну форму запалення в плевральній порожнині:

Емпієма

Флегмона

Хронічний абсцес

Гострий абсцес

Фі6ринознє запалення

1941 / 6307
У чоловіка 35-ти років під час тривалого 6ігу виникла гостра серцева недостатність. Які зміни іонного складу спостерігаються у серцевому м’язі при цьому стані?

Збільшення в позаклітинному просторі іонів Na+ і Ca2+

Накопичення в клітинах міокарда іонів Na+ і Ca2+

Зменшення в позаклітинному просторі іонів K + і Mg2+

Зменшення в клітинах міокарда іонів Na+ і Ca2+

Накопичення в клітинах міокарда іонів K + і Mg2+

1942 / 6307
У дитини, що часто хворіє на ангіни і фарингіти, відзначається збільшення лімфовузлів та селезінки. Зовнішній вигляд характеризується пастозністю і блідістю, м’язова тканина розвинена слабко. У крові спостерігається лімфоцитоз. Як називається такий вид діатезу?

Астенічний

Лімфатико-гіпопластичний

Ексудативно-катаральний

Нервово-артритичний

Геморагічний

1943 / 6307
На перехід із горизонтального положення у вертикальне система кровообігу відповідає розвитком рефлекторної пресорної реакції. Що з наведеного є її обов’язковим компонентом?

Системне звуження венозних судин ємності

Зменшення частоти серцевих скорочень

Зменшення насосної функції серця

Системне розширення артеріальних судин опору

Зменшення об’єму циркулюючої крові

1944 / 6307
Дитині після аналізу імунограми встановили діагноз первинний імунодефіцит гуморальної ланки імунітету. Яка з причин може призвести до розвитку первинного імунодефіциту в організмі дитини?

Порушення обміну речовин в організмі матері

Порушення в процесі ембріонального розвитку

Спадкові порушення в імунній системі

Токсичне пошкодження В-лімфоцитів

Порушення реактивності та резистентності організму

1945 / 6307
У життєвому циклі клітини відбувається процес самоподвоєння ДНК. В результаті цього однохроматидні хромосоми стають двохроматидними. У який період клітинного циклу спостерігається це явище?

S

G2

G1

G0

M

1946 / 6307
Через дві доби після пологів у жінки розвинулася клініка шоку з ДВЗ-синдромом, внаслідок чого породілля померла. На аутопсії виявлений гнійний ендоміометрит, регіональний гнійний лімфангіт і лімфаденіт, гнійний тромбофлебіт. У паренхіматозних органах - дистрофічні зміни і проміжне запалення. Який діагноз найбільш вірогідний?

Деструюючий міхуровий занесок

Сепсис

Сифіліс

Туберкульоз статевих органів

Міхуровий занесок

1947 / 6307
Інозитолтрифосфати в тканинах організму утворюються в результаті гідролізу фосф атидилінозитолдифосфатів і відіграють роль вторинних посередників (месенджерів) в механізмі дії гормонів. Їхній вплив у клітині спрямований на:

Гальмування фосфодіестерази

Активацію аденілатциклази

Активацію протеїнкінази А

Вивільнення іонів кальцію з клітинних депо

Гальмування протеїнкінази С

1948 / 6307
Хворому на ішемічну хворобу серця з аритмією призначено препарат, який переважно блокує калієві канали, суттєво подовжує потенціал дії. Який препарат призначено?

Корглікон

Добутамін

Лізиноприл

Нітрогліцерин

Аміодарон

1949 / 6307
Для лікування хронічної серцевої недостатності хворий приймає дигоксин. Який діуретичний засіб може збільшити токсичність дигоксину за рахунок посиленого виведення з організму іонів K+?

Панангін

Силібор

Лізиноприл

Гідрохлортіазид

Спіронолактон

1950 / 6307
Під час голодування м’язові білки розпадаються до вільних амінокислот. В який процес найімовірніше будуть втягуватися ці сполуки за таких умов?

Глюконеогенез у печінці

Декарбоксилування

Синтез вищих жирних кислот

Глюконеогенез у м’язах

Глікогеноліз

1951 / 6307
Хвора звернулася до лікаря зі скаргами на неприємні відчуття в епігастрії, нудоту, зниження апетиту. При дослідженні дуоденального вмісту виявлено лямблії. Який препарат слід призначити?

Ізоніазид

Ацикловір

Хінгамін

Метронідазол

Рифампіцин

1952 / 6307
До гінеколога звернулася жінка 28-ми років з приводу безпліддя. При обстеженні знайдено: недорозвинені яєчники та матка, нерегулярний менструальний цикл. При дослідженні статевого хроматину в більшості соматичних клітин виявлено два тільця Бара. Яка хромосомна хвороба найбільш вірогідна у жінки?

Синдром трипло-Х

Синдром Клайнфельтера

Синдром Шерешевського-Тернера

Синдром Патау

Синдром Едвардса

1953 / 6307
До клініки потрапила дитина 1-го року з ознаками ураження м’язів кінцівок та тулуба. Після обстеження виявлений дефіцит карнітину в м’язах. Біохімічною основою цієї патології є порушення процесу:

Транспорту жирних кислот у мітохондрії

Субстратного фосфорилювання

Окисного фосфорилювання

Утилізації молочної кислоти

Регуляції рівня Ca2+ в мітохондріях

1954 / 6307
У хворого при проведенні наркозу виникли ознаки підвищення тонусу парасимпатичної нервової системи: гіперсалівація, ларингоспазм. Введенням якого препарату можна було попередити небажані ефекти?

Прозерин

Атропіну сульфат

Пірацетам

Анальгін

Адреналіну гідрохлорид

1955 / 6307
Після тривалого фізичного навантаження під час заняття з фізичної культури у студентів розвинулась м’язова крепатура. Причиною її виникнення стало накопичення у скелетних м’язах молочної кислоти. Вона утворилась після активації в організмі студентів:

Гліколізу

Ліполізу

Пентозофосфатного циклу

Глюконеогенезу

Глікогенезу

1956 / 6307
На прохання лікаря хворий після звичайного вдиху зробив максимально глибокий видих. Скорочення яких із наведених м’язів бере участь у такому видиху?

Діафрагма

Зовнішні міжреберні

Трапецієподібні

Живота

Грудні

1957 / 6307
На електронній мікрофотографії науковець виявив структуру, утворену вісьмома молекулами білків-гістонів і ділянкою молекули ДНК, що робить близько 1,75 обертів навколо них. Яку структуру виявив дослідник?

Хромосома

Елементарна фібрила

Хроматида

Нуклеосома

Напівхроматида

1958 / 6307
У хворого з масивними опіками розвинулась гостра недостатність нирок, що характеризується значним і швидким зменшенням швидкості клубочкової фільтрації. Який механізм її розвитку?

Збільшення тиску канальцевої рідини

Емболія ниркової артерії

Зменшення ниркового кровотоку

Ушкодження клубочкового фільтра

Зменшення кількості функціонуючих нефронів

1959 / 6307
У фекаліях трирічної дитини із сильновираженою діареєю, яка хворіє впродовж 3-х днів, за методом імунної електронної мікроскопії виявлено віруси із двошаровим капсидом і псевдоо-болонкою, які нагадують дрібні коліщатка із спицями. Які віруси найбільш вірогідно виявлені?

Коронавіруси

Ротавіруси

Реовіруси

ЕСНО-віруси

Коксакі-віруси

1960 / 6307
У хворого з лихоманкою та висипкою на шкірі після обстеження за допомогою серологічних реакцій поставлений діагноз фасціольоз. Було встановлено, що хворий заразився шляхом споживання сирої води з річки. Яка стадія життєвого циклу фасціоли інвазійна для людини?

Метацеркарій

Фіна

Мірацидій

Адолескарій

Яйце

1961 / 6307
У хворого з високим титром антинуклеарних антитіл смерть настала від наростаючої ниркової недостатності. При патологоанатомічному дослідженні виявлений мезангіопроліферативний гломерулонефрит, абактеріальний бородавчастий ендокардит; в селезінці - періартеріальний цибулинний склероз, в шкірі - продуктивнопроліферативний васкуліт. Який найбільш вірогідний діагноз?

Дерматоміозит

Нефротичний синдром

Вузликовий періартеріїт

Ревматизм

Системний червоний вівчак

1962 / 6307
При харчовому отруєнні виділена культура анаеробних грампозитивних спороутворюючих паличок. До якого виду, найбільш вірогідно, відноситься виділений збудник?

Vibrio parahemolyticus

Esherichia coli

Proteus vulgaris

P.mirabilis

C. perfringens

1963 / 6307
У тварини збільшений тонус м’язів-розгиначів. Це є наслідком посиленої передачі інформації до мотонейронів спинного мозку такими низхідними шляхами:

Медіальні кортикоспінальні

Ретикулоспінальні

Руброспінальні

Латеральні кортикоспінальні

Вестибулоспінальні

1964 / 6307
Хворий 70-ти років помер від гострої коронарної недостатності. За життя відзначалась припухлість, деформація і біль колінних суглобів. При патоморфологічному дослідженні деформованих суглобів і синовіальних оболонок виявлено: гіперемія оболонок з множинними периваскулярними запальними інфільтратами з лімфоцитів, плазмоцитів, макрофагів. Скупчення організованого фібрину, який вкриває ділянки синовіальної оболонки і визначається в суглобовій рідині у вигляді рисових зерен. Який найбільш вірогідний діагноз?

Туберкульозний артрит

Ревматоїдний артрит

Анкілозуючий спондилоартрит

Вузликовий періартеріїт

Деформуючий артроз

1965 / 6307
До відділення нейрохірургії було доставлено чоловіка з втратою слуху внаслідок травми голови. Порушення якої частки кори головного мозку може бути причиною цього?

скронева

тім’яна

лобова

потилична

постцентральна звивина

1966 / 6307
Вагітній жінці, під час встановлення на облік у жіночу консультацію, було проведено клінічне обстеження на ряд інфекцій. У сироватці крові були виявлені lgM до вірусу краснухи. Про що свідчить такий результат?

Жінка здорова

Загострення хронічного процесу

Первинне зараження жінки

Хронічний процес

Повторне інфікування вірусом краснухи

1967 / 6307
Хворий, що страждав на туберкульоз, помер від прогресуючої легенево-серцевої недостатності. На розтині в ділянці верхівки правої легені визначається порожнина діаметром 5 см, яка сполучається з просвітом сегментарного бронха. Стінки порожнини з середини покриті сирнистими масами, під якими знаходяться епітеліоїдні клітини і клітини Пирогова-Лангханса. Вкажіть морфологічну форму туберкульозу:

Гострий кавернозний туберкульоз

Гострий осередковий туберкульоз

Казеозна пневмонія

Інфільтративний туберкульоз

Туберкульома

1968 / 6307
До бактеріологічної лабораторії доставлені блювотні маси хворого з підозрою на холеру. З матеріалу приготований препарат 'висяча крапля'. Який метод мікроскопії буде використаний для виявлення збудника по його рухливості?

Імунна електронна

Фазово-контрастна

Шерстна

Електронна

Люмінесцентна

1969 / 6307
У хворого на ентерит, що супроводжувався значною діареєю, спостерігається зменшення кількості води в позаклітинному просторі, збільшення її всередині клітин та зниження осмолярності крові. Як називають таке порушення водно-електролітного обміну?

Осмолярна гіпогідратація

Гіперосмолярна гіпергідратація

Гіперосмолярна гіпогідратація

Гіпоосмолярна гіпергідратація

Гіпоосмолярна гіпогідратація

1970 / 6307
При огляді 6-місячної дитини лікар виявив незакрите заднє тім’ячко. В якому віці воно закривається при нормальному розвитку дитини?

До 3-х місяців

До кінця першого року життя

До 6-ти місяців

До народження

До кінця другого року життя

1971 / 6307
У новонародженої дитини спостерігаються зниження інтенсивності смоктання, часте блювання, гіпотонія. У сечі та крові значно підвищена концентрація цитруліну. Який метаболічний процес порушений?

Цикл Корі

Гліколіз

ЦТК

Глюконеогенез

Орнітиновий цикл

1972 / 6307
При обстеженні в клініці у чоловіка діагностували гостру променеву хворобу. Лабораторно встановлено різке зниження вмісту серотоніну в тромбоцитах. Порушення метаболізму якої речовини є можливою причиною зниження тромбоцитарного серотоніну?

Серин

Тирозин

5-окситріптофан

Фенілаланін

Гістидин

1973 / 6307
У хворої дитини виявлена затримка розумового розвитку, збільшення печінки, погіршення зору. Лікар пов’язує ці симптоми з дефіцитом в організмі галактозо-1- фосфатуридилтрансферази. Який патологічний процес має місце у дитини?

Гіпоглікемія

Гіперглікемія

Галактоземія

Гіперлактатацидемія

Фруктоземія

1974 / 6307
В умовному експерименті повністю інгібовано розвиток клітин мезенхіми. Порушення розвитку якої м’язової тканини при цьому буде спостерігатися?

М ’язова тканина нейрального походження

Гладенька м’язова тканина

М’язова тканина епідермального походження

Скелетна м’язова тканина

Серцева м’язова тканина

1975 / 6307
На гістологічному препараті паренхіма органу представлена лімфоїдною тканиною, яка утворює лімфатичні вузлики, останні розташовані дифузно і містять центральну артерію. Яке анатомічне утворення має дану морфологічну будову?

Селезінка

Червоний кістковий мозок

Лімфатичний вузол

Мигдалик

Тимус

1976 / 6307
У потерпілого з травмою голови у скроневій ділянці діагностовано епідуральну гематому. Яка з артерій найвірогідніше пошкоджена?

Середня мозкова

Задня вушна

Передня оболонкова

Поверхнева скронева

Середня оболонкова

1977 / 6307
У дитини спостерігається затримка фізичного та розумового розвитку, глибокі порушення з боку сполучної тканини внутрішніх органів, у сечі виявлено кератансульфати. Обмін яких речовин порушений?

Фібронектин

Колаген

Еластин

Глікозаміноглікани

Гіалуронова кислота

1978 / 6307
Видалення зуба у пацієнта з хронічним персистуючим гепатитом ускладнилось тривалою кровотечею. Яка причина геморагічного синдрому?

Зменшення утворення фібрину

Зменшення утворення тромбіну

Збільшення утворення тромбопла-стину

Збільшення синтезу фібриногену

Посилення фібринолізу

1979 / 6307
У жінки під час гаметогенезу (в мейозі) статеві хромосоми не розійшлися до протилежних полюсів клітини. Яйцеклітина була запліднена нормальним сперматозоїдом. Яке хромосомне захворювання може бути у дитини?

Синдром Патау

Синдром Едвардса

Синдром котячого крику

Синдром Дауна

Синдром Шерешевського-Тернера

1980 / 6307
Хворій для лікування алергічного хейліту призначили лоратадин. Який механізм дії даного лікарського засобу?

Підвищує активність моноамінокси-дази

Блокує Н1-гістамінові рецептори

Пригнічує активність Na,K — АТФ — ази

Блокує адренорецептори

Пригнічує активність холінестерази

1981 / 6307
Хворому після апендектомії з метою профілактики інфекції призначено антибіотик групи цефалоспоринів. Порушення якого процесу лежить в основі протимікробної активності антибіотиків цієї групи?

Рибосомальний синтез білку

Енергетичний обмін

Синтез нуклеїнових кислот

Утворення мікробної стінки

Блокада холінестерази

1982 / 6307
За медичним показанням пацієнту було проведено видалення частини однієї із структур ЦНС. В результаті видалення у пацієнта розвинулися атонія, астазія, інтенційний тремор, атаксія, адіадохокінез. Частина якої структури ЦНС була вилучена?

Лімбічна система

Мозочок

Гіпокамп

Базальні ганглії

Мигдалеподібний комплекс

1983 / 6307
У хворого діагностовано грип. Після прийому антипіретиків стан його різко погіршився: свідомість потьмарена, АТ- 80/50 мм рт.ст., Ps-140/хв., температура тіла різко знизилась до 35,8oC. Яке ускладнення виникло у даного хворого?

Алкалоз

Гіповолемія

Колапс

Ацидоз

Гіпертермія

1984 / 6307
Пацієнт звернувся до лікаря з приводу втрати здатності розрізняти смаки на корені язика. Лікар встановив, що це пов’язано з ураженням нерва. Якого?

Лицевий

Трійчастий

Верхньогортанний

Язикоглотковий

Блукаючий

1985 / 6307
Яким буде скорочення м’язів верхньої кінцівки при утриманні (але не переміщенні) вантажу в певному положенні?

Ексцентричним

Ауксотонічним

Ізометричним

Концентричним

Ізотонічним

1986 / 6307
Жінка. похилого віку госпіталізована зі скаргами на різкий 6іль, набряк в ділянці правого кульшового суглоба, що з’явилися після падіння. Об’єктивно: стегно приведене досередини, рухи в кульшовому суглобі порушені. Перелом якої кістки або її частини визначив лікар?

Лобкова кістка

Шийка стегнової кістки

Виростки стегнової кістки

Тіло стегнової кістки

Сіднична кістка

1987 / 6307
Жінка 45-ти років хворіє на рак лівої молочної залози. На лівій руці є ознаки недостатності лімфатичної системи - набряк кінцівки, збільшення лімфовузлів. Яка форма недостатності лімфообміну спостерігається у хворої?

Змішана недостатність

Динамічна недостатність

Резорбційна недостатність

Механічна недостатність

1988 / 6307
До лікарні доставлений хворий з отруєнням інсектицидом - ротеноном. Яка ділянка мітохондріального ланцюга переносу електронів блокується цією речовиною?

Сукцинат-коензим Q-редуктаза

Коензим Q-цитохром С-редуктаза

АТФ -синтетаза

Цитохром С-оксидаза

НАДН-коензим Q-редуктаза

1989 / 6307
Під час операції на пахвинному каналі з приводу грижі, хірург пошкодив його вміст. Що саме пошкодив хірург?

Lig. teres uteri

Urarchus

Lig. inguinale

Funiculus spermaticus

1990 / 6307
Чоловік 26-ти років перебуває в торпідній стадії шоку внаслідок автомобільної аварії. В крові: лейк.- 3, 2 • 109 /л. Який головний механізм в розвитку лейкопенії?

Руйнування лейкоцитів у кровотворних органах

Порушення виходу зрілих лейкоцитів з кісткового мозку в кров

Пригнічення лейкопоезу

Підвищення виділення лейкоцитів з організму

Перерозподіл лейкоцитів у судинному руслі

1991 / 6307
Хворий звернувся до лікаря зі скаргами на порушення відчуття рівноваги, що з’явилося після травми. Який нерв пошкоджено?

Проміжний

Лицевий

Трійчастий

Блукаючий

Присінково-завитковий

1992 / 6307
Хлопчик 5-ти місяців госпіталізований з приводу тонічних судом. Хворіє з народження. Об’єктивно: волосся жорстке, нігті витончені та ламкі, шкірні покриви бліді та сухі. В біохімічному аналізі крові: кальцій - 0,5 ммоль/л (норма - 0,75-2,5 ммоль/л), фосфор -1,9 ммоль/л (норма - 0,646-1,292 ммоль/л). З чим пов’язані ці зміни?

Гіпоальдостеронізм

Гіперальдостеронізм

Гіперпаратиреоз

Гіпотиреоз

Гіпопаратиреоз

1993 / 6307
У хворого на алкоголізм почався алкогольний психоз із вираженим психомоторним збудженням. Який препарат з групи нейролептиків слід призначити для швидкої допомоги?

Фторотан

Діазепам

Аміназин

Резерпін

Натрію бромід

1994 / 6307
При пункційній біопсії печінки хворого з клінікою печінково-клітинної недостатності виявлена вакуольна, балонна дистрофія гепатоцитів, некроз окремих клітин, тільця Каунсильмена, інфільтрація портальної та часточкової строми переважно лімфоцитами та макрофагами з незначною кількістю поліморфноядерних лейкоцитів. Який найбільш вірогідний діагноз?

Хронічний активний гепатит

Хронічний персистуючий гепатит

Алкогольний гепатит

Гострий вірусний гепатит

Аутоімунний гепатит

1995 / 6307
З метою прискорення загоєння променевої виразки був призначений вітамінний препарат. Вкажіть його:

Ретинолу ацетат

Метилурацил

Преднізолон

Левамiзол

Ретаболіл

1996 / 6307
Дівчина 15-ти років була доставлена до лікарні з запаленням червоподібного відростку. Аналіз крові показав ознаки анемії. У фекаліях було виявлено яйця гельмінта, які мають лимоноподібну форму (50х30 мкм), з 'пробочками'на полюсах. Який вид гельмінту паразитує у хворої?

Ехінокок

Волосоголовець

Карликовий ціп’як

Гострик

Анкілостома

1997 / 6307
У хворого, який страждає на вугри та запальні зміни шкіри обличчя, при мікроскопії матеріалу з осередків ураження виявлені живі членистоногі розміром 0,2-0,5 мм. Вони мають витягнуту червоподібну форму, чотири пари коротких кінцівок, розміщених у середній частині тіла. Виявлені членистоногі викликають:

Демодекоз

Коросту

Педикульоз

Шкірний міаз

Фтіріоз

1998 / 6307
При огляді дитини 4-х років зі скаргами на загальну слабкість, біль у горлі та утруднене ковтання лікар запідозрив дифтерію та направив матеріал до бактеріологічної лабораторії. На яке диференціально-діагностичне поживне середовище слід засіяти матеріал для виділення збудника дифтерії?

Середовище Левенштейна-Йєнсена

Кров’яно-телуритовий агар

Середовище Плоскірєва

Середовище Сабуро

Середовище Ендо

1999 / 6307
Серед антиатеросклеротичних препаратів, що застосовуються з метою профілактики та лікування атеросклерозу, є левостатин. Він діє шляхом:

Гальмування біосинтезу холестерину

Активації метаболізму холестерину

Пригнічення всмоктування холестерину в кишечнику

Стимулювання екскреції холестерину з організму

Усіма наведеними шляхами

2000 / 6307
У вагітної жінки на передній черевній стінці виявлено пухлиноподібне утворення, яке виникло на місці видаленої два роки тому пухлини. Утворення має щільну консистенцію і розміри 2х1 см, з чіткими межами. При гістологічному дослідженні виявлено, що пухлина побудована з диференційованої сполучної тканини з переважанням колагенових волокон. Про яку пухлину слід думати?

Фібросаркома

Десмоїд

Гібернома

Ліпома

Лейоміома

2001 / 6307
До хірургічного відділення доставлено чоловіка 35-ти років з гнійною раною на шиї попереду трахеї (в ділянці передвісцерального простору). Куди може розповсюдитись інфекція, якщо хворому терміново не зроблять операцію?

В ретровісцеральний простір

В грудну порожнину - заднє середостіння

В надгрудинний міжапоневротичний простір

В грудну порожнину - переднє середостіння

В грудну порожнину - середнє середостіння

2002 / 6307
Внаслідок інсульту (крововилив в головний мозок) у хворого відсутні вольові рухи м’язів голови і шиї. Обстеження головного мозку за допомогою ЯМР показало, що гематома знаходиться в коліні внутрішньої капсули. Який провідний шлях пошкоджено у хворого?

Tr.cortico-fronto-pontinus

Tr.thalamo-corticalis

Tr.cortico-nuclearis

Tr.cortico-thalamicus

Tr.cortico-spinalis

2003 / 6307
Хворому з гіперсекрецією шлункового соку лікар рекомендував виключити з дієти насичені бульйони і овочеві відвари, тому що вони стимулюють шлункову секрецію переважно через активацію:

Механорецепторів ротової порожнини

Механорецепторів шлунка

Смакових рецепторів

Вироблення гастрину

Вироблення секретину

2004 / 6307
У хворого після короткочасного оперативного втручання, проведеного із застосуванням дитиліну, понад 30 хвилин відмічалось пригнічення дихання, не відновився попередній тонус м’язів. Яку допомогу необхідно надати хворому?

Гемосорбція

Переливання крові

Форсований діурез

Перитонеальний діаліз

Гемодіаліз

2005 / 6307
Хворий з діагнозом 'вогнищевий туберкульоз верхньої долі правої легені' в складі комбінованої терапії одержує ізоніазид. Через деякий час пацієнт почав пред’являти скарги на м’язову слабкість, зниження шкірної чутливості, порушення зору, координації рухів. Який вітамінний препарат доцільно використати для усунення даних явищ?

Вітамін A

Вітамін D

Вітамін C

Вітамін В6

Вітамін В12

2006 / 6307
Хворому на гіпертонічну хворобу з супутнім обструктивним бронхітом призначили гіпотензивний засіб. Через деякий час у пацієнта почали з’являтися напади ядухи, розвинулась виражена брадикардія. На ЕКГ відмічались ознаки порушення атріовентрикулярного проведення. При призначенні якого препарату найбільш імовірний розвиток подібних симптомів?

Анаприлін

Верапаміл

Клофелін

Резерпін

Корданум

2007 / 6307
Хворий 60-ти років впродовж 9-ти років хворіє на цукровий діабет, отримує для корекції гіперглікемії інсулін-семіленте. 10 днів тому почав лікування гіпертонічної хвороби анаприліном. Через годину після прийому антигіпертензивного препарату розвинулась гіпоглікемічна кома. Який механізм виникнення гіпоглікемії за умови прийому анаприліну?

Зменшення періоду напіввиведення глюкагону

Пригнічення глікогенолізу

Збільшення періоду напіввиведення інсуліну-семіленте

Зменшення всмоктування глюкози

Збільшення біодоступності інсуліну-семіленте

2008 / 6307
Хворий надійшов до хірургічного відділення з діагнозом: гострий панкреатит. Розпочато консервативне лікування. Призначення якого препарату є патогенетично обґрунтованим?

Хімотрипсин

Трипсин

Панкреатин

Контрикал

Фібринолізин

2009 / 6307
До приймального відділення доставлений хворий зі скаргами на сухість в роті, світлобоязнь та порушення зору. Об’єктивно: шкіра гіперемована, суха, зіниці розширені, тахікардія. При подальшому обстеженні був встановлений діагноз: отруєння алкалоїдами беладонни. Який лікарський засіб доцільно застосувати?

Пілокарпін

Прозерин

Ацеклідин

Армін

Дипіроксим

2010 / 6307
При обстеженні 2-місячного хлопчика педіатр звернув увагу, що плач дитини схожий на нявкання кішки, відзначаються мікроцефалія і вада серця. За допомогою цитогенетичного методу був встановлений каріотип - 46 XY, 5р- . На якій стадії мітозу досліджували каріотип хворого?

Метафаза

Профаза

Анафаза

Прометафаза

Телофаза

2011 / 6307
Чоловік, що страждає на спадкову хворобу, одружився із здоровою жінкою. У них було 5 дітей, три дівчинки і два хлопчика. Усі дівчатка успадкували хворобу батька. Який тип спадкування цього захворювання?

Рецесивний, зчеплений з X -хромосомою

Аутосомно-рецесивний

Аутосомно-домінантний

Зчеплений з Y-хромосомою

Домінантний, зчеплений з X -хромосомою

2012 / 6307
Похідні птерину (аміноптерин і метотрексат) - є конкурентними інгібіторами дигідрофолатредуктази, внаслідок чого вони пригнічують регенерацію тетрагідрофолієвої кислоти з дигідрофолату. Ці лікарські засоби призводять до гальмування міжмолекулярного транспорту одновуглецевих груп. Біосинтез якого полімеру при цьому пригнічується?

Гомополісахариди

Глікозаміноглікани

Гангліозиди

ДНК

Білок

2013 / 6307
До лікарні надійшов хворий із скаргами на головний біль, біль у м’язах під час руху, слабкість, температуру, набряк повік і обличчя. Лікар пов’язує цей стан із вживанням свинини, купленої у приватних осіб. Який попередній діагноз може поставити лікар?

Опісторхоз

Трихінельоз

Фасціольоз

Теніарінхоз

Теніоз

2014 / 6307
В шкірі виявлена щільна, рухома, чітко відмежована від оточуючих тканин пухлина. На розрізі вона білого кольору, представлена волокнистою тканиною. Мікроскопічно: хаотично переплетені колагенові волокна, клітин мало. Що це за пухлина?

Дерматофіброма

Міома

Десмоїд

Фіброма

Гістіоцитома

2015 / 6307
На розтині тіла померлої виявлено морфологічні зміни: стеноз лівого атріовентрикулярного отвору, недостатність мітрального клапана. Гістологічно в міокарді - вогнищевий кардіосклероз, наявність квітучих гранульом Ашофф-Талалаєва. Який з перерахованих нижче діагнозів найбільш імовірний?

Вузликовий періартеріїт

Ревматизм

Склеродермія

Дерматоміозит

Системний червоний вовчак

2016 / 6307
У постраждалого виявлено рану верхньої частини передньої ділянки плеча. При обстеженні встановлена втрата активного згинання в ліктьовому суглобі і чутливості шкіри передньобічної поверхні передпліччя. Порушення функції якого нерва має місце?

Променевий

Пахвовий

Ліктьовий

М’язово-шкірний

Серединний

2017 / 6307
При мікроскопічному дослідженні шийного лімфатичного вузла виявлено скупчення епітеліоїдних клітин, лімфоцитів і гігантських клітин Пирогова-Лангханса. В центрі - казеозний некроз. Вкажіть найбільш імовірну патологію:

Туберкульоз

Сифіліс

Риносклерома

Сап

Саркоїдоз

2018 / 6307
При гістологічному дослідженні новоутворення шкіри виявлено: паренхіма сформована з покривного епітелію із збільшеним числом шарів. Строма разом з розростаннями епітелію формує сосочки. Вкажіть вид атипізму:

Клітинний

Метаболічний

Тканинний

Гістохімічний

Функціональний

2019 / 6307
Після перенесеного сепсису у хворої 27-ми років з’явився бронзовий колір шкіри, характерний для аддісонової хвороби. Механізм гіперпігментації полягає в підвищенні секреції такого гормону:

Меланоцитстимулюючий

Гонадотропний

Соматотропний

B-ліпотропний

Тиреотропний

2020 / 6307
У пацієнта, який півтора місяця тому переніс інфаркт міокарда, діагностовано синдром Дреслера з характерною тріадою: перикардит, плеврит, пневмонія. Який головний механізм цього ускладнення?

Активація сапрофітної мікрофлори

Зниження резистентності до інфекційних агентів

Викидання у кров міокардіальних ферментів

Сенсибілізація організму антигенами міокарда

Інтоксикація організму продуктами некрозу

2021 / 6307
Жінка після перенесеного нервового потрясіння погано спить. Якому снодійному засобу слід віддати перевагу для лікування безсоння?

Нітразепам

Барбаміл

Етамінал-натрій

Барбітал

Фенобарбітал

2022 / 6307
У хворої на дифтерію дитини через 10 днів після введення антитоксичної протидифтерійної сироватки з’явилася висипка на шкірі, яка супроводжувалася сильним свербежем, підвищенням температура тіла до 38oC, появою болю у суглобах. Яку причину цих явищ можна припустити?

Контактна алергія

Сироваткова хвороба

Атопія

Гіперчутливість уповільненого типу

Анафілактична реакція

2023 / 6307
У хворого з розладом мозкового кровообігу порушений акт ковтання, він може поперхнутися при прийомі рідкої їжі. Який відділ мозку уражений?

Мозочок

Середній мозок

Довгастий мозок

Шийний відділ спинного мозку

Проміжний мозок

2024 / 6307
Молодий чоловік 19-ти років страждав з раннього дитинства на бронхоектатичну хворобу, помер від ниркової недостатності. На розтині окрім множинних бронхоектатичних каверн, заповнених гнійним ексудатом, виявлено збільшені в розмірах нирки щільної консистенції, кірковий шар потовщений, білого кольору, щільний. Піраміди нирки анемічні, чіткі. Назвіть процес, який розвинувся в нирках:

Хронічний пієлонефрит

Вроджений кістоз нирок

Гломерулонефрит

Вторинний амілоїдоз

Вторинний нефросклероз

2025 / 6307
Дитина 3-х років з множинними порушеннями розвитку кісток лицевого відділу черепа померла. Причина смерті - сепсис, який розвинувся на фоні бронхопневмонії. В крові вміст імуноглобулінів в межах фізіологічної норми. На розтині встановлена відсутність тимусу. Назвіть головну причину хвороби дитини:

Вторинний імунодефіцитний синдром

Гострий лімфолейкоз

Синдром недостатності клітинного імунітету

Синдром хронічної інтоксикації

Синдром комбінованого імунодефіциту

2026 / 6307
До лікарні доставлено пораненого вогнепальною зброєю з сильною кровотечею. При огляді хірургом установлено, що кульовий канал пройшов через передню стінку живота, склепіння шлунка i вийшов на рівні ІХ ребра по лівій середній пахвовій лінії. Який орган постраждав разом з пораненням шлунка?

Ліва частка печінки

Селезінка

Поперечна ободова кишка

Ліва нирка

Підшлункова залоза

2027 / 6307
У хворого із захворюванням печінки виявлено зниження вмісту протромбіну в крові. Це призведе, перш за все, до порушення:

Другої фази коагуляційного гемостазу

Судинно-тромбоцитарного гемостазу

Першої фази коагуляційного гемостазу

Фібринолізу

Антикоагулянтних властивостей крові

2028 / 6307
в крові: ер.- 3,0 • 1012 28. /л; Hb- 90г/л; ретикул.- 0,5%. В мазку: пойкілоцити, гіпохромні еритроцити. Залізо сироватки крові - 80 мкмоль/л. Для якої патології це характерно?

Хвороба Мінковського-Шоффара

Серпоподібноклітинна анемія

Залізорефрактерна анемія

Залізодефіцитна анемія

Б12-дефіцитна анемія

2029 / 6307
У фізично здорових молодих курсантів після важкого фізичного навантаження при одноденному пішому переході на 50 км в сечі виявлено білок, рівень якого в середньому не перевищував 1 г/л. Який різновид протеїнурії мав місце?

Маршова

Дегідратаційна

Аліментарна

Органічна

Несправжня

2030 / 6307
Яким буде скорочення м’язів верхньої кінцівки при намаганні підняти непосильний вантаж?

Фазичне

Ауксотонічне

Ізометричне

Одиночне

Ізотонічне

2031 / 6307
У хворих на поворотний тиф виникає лихоманка, яка характеризується кількаденними періодами високої гарячки, що чергується з періодами нормальної температури. Така температурна крива називається:

Febris recurrens

Febris continua

Febris intermittens

Febris hectica

Febris atypica

2032 / 6307
У хворого з алкогольним цирозом печінки скарги на загальну слабкість, задишку. Встановлено зниження артеріального тиску, асцит, розширення поверхневих вен передньої стінки живота, спленомегалію. Яке порушення гемодинаміки спостерігається у хворого?

Недостатність лівого шлуночка серця

Синдром портальної гіпертензії

Тотальна серцева недостатність

Колапс

Недостатність правого шлуночка серця

2033 / 6307
Під час обертання на каруселі у жінки 25-ти років з’явилися нудота, блювання, посилення потовиділення. Активація яких рецепторів зумовила рефлекторний розвиток цих симптомів?

Отолітові вестибуляри

Пропріорецептори скелетних м’язів

Кортієвого органу

Вестибулярні півколові канали

Зорові

2034 / 6307
У студента 18-ти років під час фізичного навантаження реографічно зареєстровано перерозподіл кровотоку органів. У яких судинах кровотік підвищився найбільшою мірою?

Нирки

Печінка

Головний мозок

Скелетні м’язи

Шлунково-кишковий тракт

2035 / 6307
Під час ревізії порожнини очеревини з приводу перитоніту виявлений обмежений гнійник у кореня брижі сигмоподібної кишки. У якому утворенні очеревини знаходиться гнійник?

Правий бічний канал

Лівий брижовий синус

Лівий бічний канал

Правий брижовий синус

Міжсигмоподібна ямка

2036 / 6307
Під час розтину трупа шахтаря, що проробив у шахті більше 10-ти років, у легені виявлені тяжі білуватої волокнистої тканини і вузлики 0,2-0,3 см у діаметрі. При гістологічному дослідженні у вузликах невелика кількість коричневого пилу, концентричні розростання сполучної тканини, бідної на клітини, з вираженим гіалінозом. Про який пневмоконіоз слід думати у даному випадку?

Силікоз

Асбестоз

Сидероз

Бериліоз

Талькоз

2037 / 6307
При ендоскопічному дослідженні сечового міхура проведена біопсія пухлини, що складається з тонких, розгалужених сосочків, вкритих декількома рядами клітин перехідного епітелію. Назвіть пухлину:

Папілома

Перехідноклітинна карцинома

Плоскоклітинна карцинома

Базаліома

Фіброаденома

2038 / 6307
Хворий 23-х років надійшов до лікарні із черепно-мозковою травмою у важкому стані. Дихання характеризується судомним тривалим вдихом, який переривається коротким видихом. Для якого типу дихання це характерно?

Гаспінг-дихання

Апнейстичне

Чейн-Стокса

Біота

Куссмауля

2039 / 6307
Внаслідок дії на організм електричного струму міської електромережі впродовж 0,1 сек у напрямку 'права рука-голова'у постраждалого спостерігалась зупинка дихання. Вкажіть найбільш імовірний механізм цього ускладнення:

Параліч дихальних м’язів

Параліч центрів вдиху

Тотальний параліч дихального центру

Емоційний стрес

Рефлекторна зупинка дихання (больовий шок)

2040 / 6307
В експерименті досліджували поріг сили подразника клітин різних тканин. Де він виявився найменшим?

Міоцити скелетного м’яза

Типові кардіоміоцити

Міоцити гладенького м’яза

Залозисті клітини

Мотонейрони спинного мозку

2041 / 6307
На ізольованому серці вивчалась швидкість проведення збудження у різних його ділянках. Де була виявлена найменша швидкість?

Атріовентрикулярний вузол

Волокна Пуркін’є

Пучок Гіса

Міокард передсердь

Міокард шлуночків

2042 / 6307
Під час операції в печінці хворого виявлені дрібні міхурці малих розмірів з незначною кількістю рідини, які щільно прилягають один до одного. Який гельмінтоз виявився у хворого?

Дікроцеліоз

Опісторхоз

Альвеококоз

Клонорхоз

Фасціольоз

2043 / 6307
У хворого 70-ти років атеросклероз ускладнився тромбозом судин нижніх кінцівок, виникла гангрена пальців лівої стопи. Початок тромбоутворення, найбільш імовірно, пов’язаний з:

Перетворенням протромбіну в тром-бін

Перетворенням фібриногену в фібрин

Адгезією тромбоцитів

Зниженням синтезу гепарину

Активацією протромбінази

2044 / 6307
Жінку госпіталізовано в клініку з симптомами гострого живота. При обстеженні виникла підозра на позаматкову вагітність. Яке з анатомічних утворень таза необхідно пропунктувати для підтвердження діагнозу?

Processus vaginalis peritonei

Excavatio rectovesicalis

Excavatio vesicouterina

Excavatio rectouterina

Fossa ischiorectalis

2045 / 6307
Дитина 10-ти років страждає на стафілококовий дерматит. Лікування бензилпеніциліном не дало результатів. Призначення комбінованого препарату пеніциліну з клавулановою кислотою дало швидке одужання. Яка причина позитивної дії цього препарату?

Гальмування транспептидази

Активація фосфодіестерази

Інактивація бета-лактамази

Гальмування аденозиндезамінази

Блокада транслокази

2046 / 6307
У крові хворого виявлено підвищення активності ЛДГ 4,5, АлАТ, карбамоїлорнітинтрансферази. В якому органі можна передбачити розвиток патологічного процесу?

Нирки

Сполучна тканина

Печінка (можливий гепатит)

Скелетні м’язи

Серцевий м’яз (можливий інфаркт міокарда)

2047 / 6307
Апендикс, надісланий до патоморфологічного відділення після апендектомії, потовщений і збільшений у розмірах, серозна оболонка тьмяна, судини повнокровні, з просвіту відростка на розрізі виділяється рідина жовто-зеленого кольору. При якій формі апендициту виникають такі зміни?

Апостематозний

Гангренозний

Поверхневий катаральний

Флегмонозний

Простий катаральний

2048 / 6307
Внаслідок травми у чоловіка 40-ка років зруйновані задні корінці спинного мозку. Які розлади будуть спостерігатися в зоні іннервації цих корінців?

Втрата больової чутливості

Порушення функції посмугованих скелетних м’язів

Порушення функції гладеньких м’язів

Втрата всіх видів чутливості

Втрата температурної та вібраційної чутливості

2049 / 6307
До клініки госпіталізована пацієнтка із скаргами на кровохаркання, пітливість. Рентгенологічно виявлено вогнище туберкульозу у верхній частці правої легені. Показана операція. Яку кількість сегментів можна видалити в складі верхньої частки правої легені?

2

1

4

5

3

2050 / 6307
У товстій кишці під час колоноскопії виявлено дефект слизової оболонки діаметром 3,5 см з нерівним горбистим дном, нерівними краями, що підвищуються над дном на 1,7 см; межа цього підвищення нечітка. Тканина дна і країв дефекту щільна, білувата, шари стінки кишки у цій ділянці не розрізняються. Встановіть макроскопічну форму пухлини:

Інфільтрат

Вузол

Кіста

Виразка

Інфільтративно-виразкова форма

2051 / 6307
У препараті в одній з судин мікроциркуляторного русла середня оболонка утворена 1-2 шарами гладеньких міоцитів, які розташовані поодинці і мають спіралеподібний напрямок. Зовнішня оболонка представлена тонким шаром пухкої волокнистої сполучної тканини. Вкажіть вид судини:

Капіляр

Посткапіляр

Артеріоловенулярний анастомоз

Артеріола

Венула

2052 / 6307
У хворої людини посилений рух води з кровоносних капілярів до тканин, що викликало їх позаклітинний набряк (збільшені розміри м’яких тканин кінцівок, печінки тощо). Зменшення якого параметру гомеостазу є найбільш імовірною причиною розвитку набряку?

Гематокрит

Осмотичний тиск плазми крові

рН крові

Онкотичний тиск плазми крові

В’язкість крові

2053 / 6307
У постраждалого в дорожній пригоді лікар виявив перелом лівої ключиці та порушення кровообігу в кінцівці (немає пульсації променевої артерії). Стиснення якої судини є причиною порушення кровообігу в кінцівці?

Підключична артерія

Пахвова артерія

Пахвова вена

Хребтова артерія

Підключична вена

2054 / 6307
Хворого доставлено у медичний заклад в коматозному стані. Зі слів супроводжуючих вдалося з’ясувати, що він знепритомнів під час тренування на завершальному етапі марафонської дистанції. Який вид коми найімовірніше можна запідозрити у даного пацієнта?

Гіпотиреоїдна

Ацидотична

Печінкова

Гіпоглікемічна

Гіперглікемічна

2055 / 6307
У хворого коса пахвинна грижа. Яке анатомічне утворення стало слабким місцем передньої черевної стінки?

Надміхурова ямка

Стегнова ямка

Медіальна пахвинна ямка

Пахвинний трикутник

Латеральна пахвинна ямка

2056 / 6307
У хворих на тиреотоксикоз спостерігаються гіпертермія, булімія, зменшення маси тіла, що пов’язане з порушенням:

Циклу лимонної кислоти

Синтезу жирів

Розпаду АТФ

Спряження окислення і фосфорилю-вання

в-окиснення жирних кислот

2057 / 6307
Людині внутрішньовенно ввели 0,5 л ізотонічного розчину лікарської речовини. Які з рецепторів насамперед прореагують на зміни водно-сольового балансу організму?

Волюморецептори порожнистих вен і передсердь

Барорецептори дуги аорти

Осморецептори гіпоталамусу

Осморецептори печінки

Натрієві рецептори гіпоталамуса

2058 / 6307
Пацієнту 33 роки. Хворіє 10 років. Періодично звертається до лікаря зі скаргами на гострий біль у животі, судоми, порушення зору. У його родичів спостерігаються подібні симптоми. Сеча червоного кольору. Госпіталізований з діагнозом - гостра переміжна порфирія. Причиною захворювання може бути порушення біосинтезу такої речовини:

Жовчні кислоти

Простагландини

Гем

Колаген

Інсулін

2059 / 6307
У хворої 45-ти років при електрокардіографічному обстеженні виявлено такі зміни: інтервал P — Q подовжений, при цьому випадає кожен другий або третій комплекс QRST. Яке саме порушення провідності серця спостерігається?

Внутрішлуночкова блокада

Атріовентрикулярна блокада повна

Атріовентрикулярна блокада ІІІ ступеня

Атріовентрикулярна блокада І ступеня

Синоаурікулярна блокада

2060 / 6307
У вагітної жінки визначили групову належність крові. Реакцію аглютинації еритроцитів викликали стандартні сироватки I, III груп, та не викликала -сироватка II групи. Якою є група досліджуваної крові за системою АВ0?

B(III), α

АВ (IV)

A(II), β

O(I), α, β

2061 / 6307
При розтині тіла померлого чоловіка 65-ти років, який страждав на захворювання легень, патологічний процес переважно був локалізований у бронхах, де при гістологічному дослідженні були чітко видні залози, хрящові острівці та багаторядний циліндричний миготливий епітелій. В яких бронхах відбулися зміни?

Головні бронхи

Термінальні бронхіоли

Середні бронхи

Малі бронхи

Великі бронхи

2062 / 6307
У юнака 18-ти років діагностована м’язова дистрофія. Підвищення в сироватці крові вмісту якої речовини найбільш імовірне при цій патології?

Міоглобін

Міозин

Креатин

Лактат

Аланін

2063 / 6307
У хворого в крові збільшена концентрація пірувату. Значна його кількість екскретується з сечею. Дефіцит якого вітаміну має місце у хворого?

B3

B1

B6

B2

E

2064 / 6307
У 70-ті роки вчені встановили, що причиною важкої жовтяниці новонароджених є порушення зв’язування білірубіну в гепатоцитах. Яка речовина використовується для утворення кон’югату?

Глюкуронова кислота

Молочна кислота

Сечова кислота

Сірчана кислота

Піровиноградна кислота

2065 / 6307
Хворому, що страждає на хронічний гепатит, для оцінки знешкоджуючої функції печінки було проведене навантаження бензоатом натрію. За виділенням якої речовини з сечею судять про знешкоджуючу функцію печінки?

Валеріанова кислота

Щавелєва кислота

Гіпурова кислота

Фенілоцтова кислота

Лимонна кислота

2066 / 6307
У хворого, який проходить курс лікувального голодування, нормальний рівень глюкози у крові підтримується головним чином за рахунок глюконеогенезу. З якої амінокислоти при цьому у печінці людини найбільш активно синтезується глюкоза?

Лейцин

Глутамінова кислота

Аланін

Лізин

Валін

2067 / 6307
В сечі новонародженого визначається цитрулін та високий рівень аміаку. Вкажіть, утворення якої речовини, наймовірніше, порушене у цього малюка:

Креатин

Сечова кислота

Креатинін

Сечовина

Аміак

2068 / 6307
У здорової дорослої людини проводять зондування порожнин серця і великих судин. Де знаходиться зонд, якщо впродовж серцевого циклу зареєстровані зміни тиску від 0 до 120 мм рт.ст.?

Передсердя

Лівий шлуночок

Правий шлуночок

Легенева артерія

Аорта

2069 / 6307
При дослідженні гнійних виділень з шийки матки бактеріоскопічно виявлено присутність грамнегативних бобоподібних диплококів, які знаходилися як в середині, так і поза лейкоцитами. Назвіть збудника гнійного запалення шийки матки:

Trichomonas vaginalis

Calymmatobacterium granulomatis

Neisseria gonorroeae

Haemophilus vaginalis

Chlamidia trachomatis

2070 / 6307
Дитина 1-го року відстає в розумовому розвитку від своїх однолітків. Ранком відзначаються блювання, судоми, непритомність. У крові - гіпоглікемія натще. З дефектом якого ферменту це пов’язане?

Глікогенсинтетаза

Аргіназа

Фосфорилаза

Лактаза

Сахараза

2071 / 6307
Внаслідок черепно-мозкової травми у хворого розвинулись наступні симптоми: інтенційний тремор, дисметрія, адіадохокінез, дизартрія. Яка структура головного мозку ушкоджена?

Бліда куля

Мозочок

Чорна речовина

Стріатум

Рухова кора

2072 / 6307
У гістологічному препараті визначаються рецепторна зона сенсоепітеліального органа чуттів. Клітини даної зони лежать на базальній мембрані і включають наступні види: зовнішні та внутрішні сенсорні, зовнішні та внутрішні фалангові, стовбурові, зовнішні межові і зовнішні підтримуючі. Вкажіть, якому органу чуттів належить зазначена рецепторна зона:

Рівноваги

Слуху

Смаку

Зору

Нюху

2073 / 6307
Хвора скаржиться на набряк ніг, посиніння шкіри, невеличкі виразки збоку латерального виростку. При обстеженні відмічено: припухлість, збільшення розмірів вен, утворення вузлів. З боку якої вени відмічається патологія?

Vprofunda femoris

V saphena parva

V iliaca externa

V saphena magna

V femoralis

2074 / 6307
Хворому встановлений діагноз ураження голівки стегна ішемічного походження. Яка артерія ушкоджена?

Arteria femoralis

Arteria umbilicalis

Arteria illiaca externa

Arteria profunda femoris

Ramus acetabularum A.obturatoriae

2075 / 6307
Чоловік 44-х років з інфарктом міокарда, помер від лівошлуночкової недостатності. На аутопсії: набряк легень, дрібнокраплинні крововиливи у серозних та слизових оболонках. Мікроскопічно: дистрофічні та некробіотичні зміни епітелію проксимальних канальців нирок, у печінці - центролобулярні крововиливи та осередки некрозу. Який з видів порушення кровообігу найбільш імовірний?

Артеріальна гіперемія

Гостре недокрів’я

Гостре загальне венозне повнокров’я

Хронічне загальне венозне повнокров’я

Хронічне недокрів’я

2076 / 6307
У хворого гнійне запалення клиноподібної пазухи. В яку частину носової порожнини витікає гній?

Meatus nasi superior

Meatus nasi communis

Meatus nasi inferior

Meatus nasi medius

2077 / 6307
Для підвищення результатів спортсмену рекомендували застосовувати препарат, який містить у собі карнітин. Який процес в найбільшому ступені активується карнітином?

Синтез стероїдних гормонів

Синтез кетонових тіл

Тканинне дихання

Синтез ліпідів

Транспорт жирних кислот до мітохондрій

2078 / 6307
На ЕКГ пацієнта мають місце такі зміни: зубець P - нормальний, інтервал P — Q - вкорочений, шлуночковий комплекс QRST - розширений, зубець R -двогорбий або двофазний. Яка із форм аритмії має місце у даного пацієнта?

Миготіння шлуночків

Атріовентрикулярна блокада

Миготлива аритмія

Синдром WPW (Вольфа-Паркінсона-Уайта)

Синдром Фредеріка (тріпотіння передсердь)

2079 / 6307
У хворого з жовтяницею встановлено: підвищення у плазмі крові вмісту загального білірубіну за рахунок непрямого (вільного), в калі та сечі - високий вміст стеркобіліну, рівень прямого (зв’язаного) білірубіну в плазмі крові в межах норми. Про який вид жовтяниці можна думати?

Гемолітична

Жовтяниця немовлят

Хвороба Жильбера

Паренхіматозна (печінкова)

Механічна

2080 / 6307
Хвора 36-ти років страждає на колагеноз. Збільшення вмісту якого метаболіту найбільш імовірно буде встановлено у сечі?

Креатинін

Індикан

Сечовина

Оксипролін

Уробіліноген

2081 / 6307
У хворого на гіпертонічну хворобу виявлено високий рівень реніну в крові. Якому з гіпотензивних засобів слід віддати перевагу в цьому випадку?

Лізиноприл

Анаприлін

Дихлотіазид

Празозин

Ніфедипін

2082 / 6307
Причиною захворювання на пелагру може бути переважне харчування кукурудзою і зниження у раціоні продуктів тваринного походження. Відсутність у раціоні якої амінокислоти призводить до даної патології?

Гістидин

Фенілаланін

Метіонін

Ізолейцин

Триптофан

2083 / 6307
У людини збільшена вентиляція легень внаслідок фізичного навантаження. Який з наведених показників зовнішнього дихання у неї значно більший, ніж у стані спокою?

Резервний об’єм вдиху

Резервний об’єм видиху

Життєва ємність легень

Дихальний об’єм

Загальна ємність легень

2084 / 6307
Людина зробила максимально глибокий видих. Як називається об’єм повітря, що знаходиться в її легенях після цього?

Резервний об’єм видиху

Ємність вдиху

Функціональна залишкова ємність легень

Альвеолярний об’єм

Залишковий об’єм

2085 / 6307
Внаслідок активації іонних каналів зовнішньої мембрани збудливої клітини значно збільшився її потенціал спокою. Які канали були активовані?

Натрієві

Натрієві та кальцієві

Калієві

Повільні кальцієві

Швидкі кальцієві

2086 / 6307
Робітник тваринницької ферми гостро захворів і при наростаючих явищах інтоксикації помер. При розтині тіла встановлено: селезінка збільшена, в’яла, на розрізі темновишневого кольору, зішкріб пульпи рясний. М’які мозкові оболонки на склепінні та основі мозку набряклі, просякнуті кров’ю, мають темно-червоний колір ('шапочка кардинала'). Мікроскопічно: серозно-геморагічне запалення оболонок і тканин головного мозку з руйнуванням стінок дрібних судин. Який найбільш імовірний діагноз?

Холера

Туляремія

Бруцельоз

Чума

Сибірка

2087 / 6307
При медичному обстеженні у військкоматі у юнака 18-ти років виявлено опускання головки надп’ясткової кістки, що призвело до плоскостопості. Зі слабкістю якої зв’язки стопи це пов’язано?

Підошвова клино-кубоподібна

Підошвова п’ятково-човноподібна

Міжкісткова клино-кубоподібна

Роздвоєна

Надп’ятково-човноподібна

2088 / 6307
Після видалення у пацієнта 2/3 шлунка у крові зменшився вміст гемоглобіну, кількість еритроцитів, збільшилися розміри цих клітин крові. Дефіцит якого вітаміну призводить до таких змін у крові?

P

В12

C

В6

PP

2089 / 6307
У хворого 40-ка років при прогресуванні стафілококового гнійного періодонтиту виникло гнійне запалення кістково-мозкових просторів альвеолярного відростка, а потім тіла нижньої щелепи. Мікроскопічно кісткові балки витончені, вогнища некрозу, кісткові секвестри, оточені сполучнотканинною капсулою. Який найбільш імовірний діагноз?

Гострий остеомієліт

Хронічний фіброзний періостит

Гнійний періостит

Хронічний остеомієліт

Пародонтома

2090 / 6307
При розтині тіла дитини, померлої від серцевої недостатності, виявлено: розширення порожнин шлуночків серця. Мікроскопічно в стромі міокарда повнокров’я, набряк, розповсюджені інфільтрати з гістіоцитів, лімфоцитів, нейтрофілів, еозинофілів. Який найбільш імовірний діагноз?

Проміжний продуктивний мюкардит

Осередковий проміжний ексудативний міокардит

Вузликовий продуктивний міокардит

Дифузнопроміжний ексудативний міокардит

Альтернативний мюкардит

2091 / 6307
Хвора 75-ти років доставлена до офтальмологічного відділення лікарні зі скаргами на погіршення зору. При об’єктивному дослідженні встановлена наявність пухлини мозку, що розташована в ділянці лівого зорового тракту. При цьому у хворої спостерігається випадіння поля зору в:

Правих і лівих половинах сітківок обох очей

Лівих половинах сітківки обох очей

Правих і лівих половинах сітківки лівого ока

Правих половинах сітківки обох очей

Правих і лівих половинах сітківки правого ока

2092 / 6307
У хворого на ішемічну хворобу серця відзначається гіпертрофія міокарда, тахікардія, зниження ХОК. Який з механізмів є провідним в ушкодженні кардіоміоцитів у даному випадку?

Дегідратація кардіоміоцитів

Пошкодження специфічних мембранних насосів

Втрата Ca2+ кардіоміоцитами

Збільшення числа а та β-адренорецепторів

Втрата Mg2+ кардіоміоцитами

2093 / 6307
Тварині внутрішньовенно ввели концентрований розчин хлориду натрію, що зумовило зниження реабсорбції іонів натрію у канальцях нирок. Внаслідок яких змін секреції гормонів це відбувається?

Зменшення альдостерону

Зменшення натрійуретичного фактора

Зменшення вазопресину

Збільшення вазопресину

Збільшення альдостерону

2094 / 6307
У хворої 49-ти років відзначається обмеження довільних рухів у лівих кінцівках. Тонус м’язів у лівих руці та нозі підвищений за спастичним типом, посилені місцеві сухожилкові рефлекси, виявляються патологічні рефлекси. Який найбільш імовірний механізм призвів до розвитку м’язової гіпертонії та гіперрефлексії?

Активація синаптичної передачі імпульсів

Гальмування мотонейронів кори головного мозку

Активація збуджуючих впливів з вогнища інсульту

Зниження гальмівних низхідних впливів

Активація мотонейронів внаслідок інсульту

2095 / 6307
При виконуванні вправ на колоді гімнастка втратила рівновагу і впала. Із збудження, перш за все, яких рецепторів розпочнуться рефлекси, що забезпечать відновлення порушеної пози?

Рецептори завитки

Вестибулорецептори

Пропріорецептори

Ампулярні вестибулорецептори

Отолітові вестибулорецептори

2096 / 6307
В анотації до препарату вказано, що він містить антигени збудника черевного тифу, адсорбовані на стабілізованих еритроцитах барана. З якою метою використовують цей препарат?

Для виявлення антитіл в реакції Відаля

Для виявлення антитіл в реакції непрямої гемаглютинації

Для серологічної ідентифікації збудника черевного тифу

Для виявлення антитіл в реакції зв’язування комплементу

Для виявлення антитіл в реакції затримки гемаглютинації

2097 / 6307
На гістологічному препараті видно позазародковий орган, який являє собою міхурець, зв’язаний з кишковою трубкою. Стінка його зсередини вистелена епітелієм, зовні утворена сполучною тканиною. На ранніх етапах ембріогенезу він виконує функцію кровотворного органа. Назвіть цей орган:

Пуповина

Плацента

Амніон

Алантоїс

Жовтковий мішок

2098 / 6307
Хворий був доставлений до лікарні в коматозному стані. В анамнезі - цукровий діабет. Об’єктивно: дихання Кусмауля, зниження артеріального тиску, у видихуваному повітрі запах ацетону. Після проведеної невідкладної терапії стан покращився. Який препарат було введено хворому?

Інсулін

Ізадрин

Глібенкламід

Адреналін

Букаркам

2099 / 6307
У приймально-дiагностичне відділення доставили жінку 38-ми років з шлунковою кровотечею. Які зміни найбільш імовірні з боку крові через добу?

Еритроцитоз

Лейкопенія

Лейкоцитоз

Зменшення гематокритного числа

Збільшення гематокритного числа

2100 / 6307
При розтині трупа чоловіка 50-ти років виявлено наступні зміни: права легеня у всіх відділах помірно щільна, на розрізі тканина безповітряна, дрібнозерниста, сухувата. Вісцеральна плевра з нашаруванням фібрину сіро-коричневого кольору. Який найбільш імовірний діагноз?

Туберкульоз

Пневмофіброз

Інтерстиціальна пневмонія

Бронхопневмонія

Крупозна пневмонія

2101 / 6307
У людини звужені зіниці. Чим це зумовлено?

Дія адреналіну

Зростання тонусу парасимпатичних центрів

Дія норадреналіну

Збільшення активності симпато-адреналової системи

Зростання тонусу симпатичних центрів

2102 / 6307
При розтині тіла жінки 50-ти років, померлої від серцевої недостатності, знайдено дифузне збільшення щитоподібної залози. Мікроскопічно: перетворення призматичного епітелію фолікулів у циліндричний, проліферація епітелію з утворенням сосочків, розрідження колоїду та лімфоплазмоцитарна інфільтрація строми; в серці - гіпертрофія лівого шлуночка, серозний набряк та лімфоїдна інфільтрація строми, в печінці - серозний набряк. Який з перелічених діагнозів найбільш імовірний?

Дифузний токсичний зоб

Зоб Риделя

Зоб Хашімото

Гострий тиреоїдит

Ендемічний зоб

2103 / 6307
Пацієнт через 15 діб після повернення з багатомісячного плавання в районах Середземномор’я та Західної Африки відчув слабкість, головний біль, періодичні підвищення температури. Лікар запідозрив у хворого малярію. Який із перерахованих методів є найбільш адекватним в діагностиці даного захворювання?

Мікроскопічний

Мікробіологічний

Серологічний

Біологічний

Алергічний

2104 / 6307
У вагітної жінки взяли кров для підтвердження клінічного діагнозу 'токсоплазмоз'. Яка із перерахованих серологічних реакцій має діагностичне значення?

Реакція гальмування гемаглютинації

Реакція нейтралізації

Реакція аглютинації

Реакція зв’язування комплементу

Реакція гемадсорбції

2105 / 6307
У здорових батьків, спадковість яких не обтяжена, народилась дитина з чисельними вадами розвитку. Цитогенетичний аналіз виявив в соматичних клітинах дитини трисомію за 13-ю хромосомою (синдром Патау). З яким явищем пов’язане народження такої дитини?

Рецесивна мутація

Соматична мутація

Домінантна мутація

Порушення гаметогенезу

Хромосомна мутація

2106 / 6307
У хворого, який скаржиться на поліурію і полідипсію, знайдено цукор в сечі. Вміст цукру в плазмі крові у нормі. З чим пов’язаний механізм глюкозурії у хворого?

Iнсулiнорезистентнiсть рецепторів клітин

Недостатня продукція інсуліну підшлунковою залозою

Порушення реабсорбції глюкози в канальцях нефрону

Гіперпродукція глюкокортикоїдів наднирниками

Порушення фільтрації глюкози в клубочковому відділі нефрону

2107 / 6307
Під час об’єктивного обстеження хворого з діагнозом: атеросклеротичний міокардіосклероз, лікар встановив феномен дефіциту пульсу. При якій формі порушення серцевого ритму спостерігається такий феномен?

Миготлива аритмія

Ідіовентрикулярний ритм

Брадикардія

Передсердно-шлуночковий ритм

Синусова екстрасистолія

2108 / 6307
На препараті представлено орган, покритий сполучнотканинною капсулою, від якої відходять трабекули. В органі можна розрізнити кіркову речовину, де містяться лімфатичні вузлики та мозкову речовину, представлену тяжами лімфоїдних клітин. Який орган представлений на препараті?

Лімфатичний вузол

Мигдалики

Червоний кістковий мозок

Селезінка

Тимус

2109 / 6307
До лікаря звернулася жінка 25-ти років зі скаргами на дисменорею та безпліддя. При обстеженні виявлено: зріст жінки 145 см, недорозвинені вторинні статеві ознаки, на шиї крилоподібні складки. При цитологічному дослідженні в соматичних клітинах не виявлено тілець Барра. Який діагноз встановив лікар?

Синдром трисомії Х

Синдром Морріса

Синдром Шерешевського-Тернера

Синдром Клайнфельтера

2110 / 6307
У хворої внаслідок запалення порушена ендокринна функція фолікулярних клітин фолікулів яєчника. Синтез яких гормонів буде пригнічений?

Фолістатин

Лютропін

Фолікулостимулюючий гормон

Естрогени

Прогестерон

2111 / 6307
Хворому на гострий інфаркт міокарда у комплексній терапії було призначено гепарин. Через деякий час після введення даного препарату з’явилась гематурія. Який антагоніст гепарину необхідно ввести хворому для усунення даного ускладнення?

Фібриноген

Вікасол

Амінокапронова кислота

Неодикумарин

Протаміну сульфат

2112 / 6307
У хворого на хронічну серцеву недостатність, незважаючи на терапію кардіотонічними засобами і тіазидовим діуретиком, зберігаються набряки і виникла загроза асциту. Який препарат слід призначити для підсилення діуретичного ефекту застосованих ліків?

Клопамід

Манітол

Фуросемід

Спіронолактон

Амілорид

2113 / 6307
Пацієнта турбують поліурія (7 л на добу) і полідипсія. При обстеженні не виявлено ніяких розладів вуглеводного обміну. Дисфункція якої ендокринної залози може бути причиною даних порушень?

Аденогіпофіз

Острівці підшлункової залози

Мозкова речовина наднирників

Кора наднирників

Нейрогіпофіз

2114 / 6307
У жінки 20-ти років під час медичного огляду при пальпації в молочній залозі виявлено щільний інкапсульований вузол діаметром 1,0 см. Результат післяопераційного біопсійного дослідження: розростання сполучної тканини навколо протоків молочної залози та залозисті елементи різного діаметру, що не утворюють часточок, без ознак клітинного атипізму. Яка відповідь патологоанатома?

Фіброма

Аденома

Фіброаденома

Метастаз раку

Фіброзний рак

2115 / 6307
У хворого із швидко наростаючою внутрішньомозковою гіпертензією діагностована пухлина мозку. Під час операції видалена пухлина тім’яно-скроневої частки, м’якої консистенції, на розрізі строкатого вигляду. Гістологічно пухлина побудована з поліморфних гіперхромних клітин з утворенням псевдорозеток та великої кількості судин, ділянками некрозів і крововиливами. Який найбільш імовірний діагноз?

Гліобластома

Астроцитома

Арахноїдендотеліома

Олігодендрогліома

Менінгіома

2116 / 6307
У хворого з кровотечею розвинулась гостра ниркова недостатність, що спричинила смерть. На аутопсії макроскопічно: нирки збільшені з широким блідо-рожевим кірковим шаром, різко відмежованим від темно-червоних пірамід. Мікроскопічно: відсутність ядер епітелію звивистих канальців, тубулорексіс, венозний застій, ядра клітин судинних клубочків та прямих канальців збережені. Яка патологія нирок розвинулась у хворого?

Пієлонефрит

Інфаркт

Нефроз

Некронефроз

Гломерулонефрит

2117 / 6307
З метою масового обстеження студентів на носійство S.aureus перед виробничою практикою у дитячому відділенні клінічної лікарні було використано елективне середовище з метою отримання чистої культури цього збудника. Яке з перерахованих середовищ було використано?

Кров’яний телуритовий агар

Середовище Вільсона-Блера

Жовтково-сольовий агар

М’ясо-пептонний агар

Середовище Ендо

2118 / 6307
В крові хворого виявлено високий вміст галактози, концентрація глюкози знижена. Відмічена розумова відсталість, помутніння кришталика. Яке захворювання має місце?

Галактоземія

Лактоземія

Стероїдний діабет

Цукровий діабет

Фруктоземія

2119 / 6307
Під час бійки у чоловіка виникла зупинка серця внаслідок сильного удару у верхню ділянку передньої черевної стінки. Який із зазначених механізмів спричинив зупинку серця?

Парасимпатичні безумовні рефлекси

Симпатичні умовні рефлекси

Периферичні рефлекси

Симпатичні безумовні рефлекси

Парасимпатичні умовні рефлекси

2120 / 6307
У хворого, що надійшов до хірургічного відділення з ознаками гострого апендициту, виявлені наступні зміни білої крові: загальна кількість лейкоцитів - 16 • 109 /л. Лейкоцитарна формула: б.- 0, е.- 2%, ю.- 2%, п.- 8%, с.- 59%, л.-25%, м.- 4%. Як класифікуються зазначені зміни?

Нейтрофілія з регенеративним зсувом вліво

Нейтрофілія з гіперрегенеративним зсувом вліво

Нейтрофілія з дегенеративним зсувом вліво

Лейкемоїдна реакція за нейтрофільним типом

Нейтрофілія з зсувом вправо

2121 / 6307
У людини визначили частоту серцевих скорочень за пульсом. Вона дорівнює 120/хв. Якою при цьому є тривалість серцевого циклу?

0,9 с

0,5 с

1,0 с

0,7 с

0,8 с

2122 / 6307
У хворого із вираженим пневмосклерозом після перенесеного інфільтративного туберкульозу легень розвинулась дихальна недостатність. До якого патогенетичного типу вона відноситься?

Дисрегуляційний

Рефлекторний

Обструктивний

Апнеїстичний

Рестриктивний

2123 / 6307
Після перенесеної стрептококової інфекції у чоловіка діагностовано гострий гломерулонефрит. Найбільш імовірно, що ураження базальної мембрани ниркових тілець виникає внаслідок алергічної реакції такого типу:

Цитотоксична

Анафілактична

Стимулююча

Сповільнена

Імунокомплексна

2124 / 6307
До лікаря звернулась мати з приводу поганого самопочуття дитини - відсутність апетиту, поганий сон, дратівливість. При біохімічному дослідженні в крові виявлено відсутність ферменту глюкоцереброзидази. Для якої патології це характерно?

Хвороба Гірке

Хвороба Тея-Сакса

Хвороба Німана-Піка

Хвороба Гоше

Хвороба Помпе

2125 / 6307
Дитина 9-ми місяців харчується штучними сумішами, які не збалансовані за вмістом вітаміну B6. У дитини спостерігається пелагроподібний дерматит, судоми, анемія. Розвиток судом може бути пов’язаний з порушенням утворення:

ДОФА

Серотоніну

Дофаміну

Гістаміну

ГАМК

2126 / 6307
Внаслідок отруєння неякісною їжею хворому було промито шлунок розчином калію перманганату. Який механізм дії цього засобу?

Вивільнення йоду

Вивільнення атомарного кисню

Вивільнення хлору

Порушення мембран бактерій

Порушення синтезу ферментів дихального ланцюга

2127 / 6307
Чоловік протягом 3-х років працював в одній із африканських країн. Через місяць після переїзду до України звернувся до офтальмолога зі скаргами на біль в очах, набряки повік, сльозоточивість і тимчасове послаблення зору. Під кон’юнктивою ока були виявлені гельмінти розмірами 30-50 мм, які мали видовжене ниткоподібне тіло. Який найбільш імовірний діагноз?

Трихоцефальоз

Дифілоботріоз

Філяріоз

Ентеробіоз

Аскаридоз

2128 / 6307
У пацієнта з бронхіальною астмою за допомогою шкірних алергічних проб встановлено сенсибілізацію алергеном тополиного пуху. Який фактор імунної системи відіграє вирішальну роль в розвитку цього імунопатологічного стану?

IgG

IgM

Т-лімфоцити

IgD

IgE

2129 / 6307
У пацієнта діагностовано компресійний перелом поперекового хребця. При цьому різко збільшилась кривизна лордозу поперекового відділу хребта. Ушкодженням якої зв’язки може супроводжуватись така зміна кривизни хребта?

Задня поздовжня

Жовта

Клубово-поперекова

Передня поздовжня

Міжостиста

2130 / 6307
При дослідженні підозрілих м’ясних продуктів (сосиски), що мали характерних гнилісний запах, було виділено рухливі грамнегативні паличковидні мікроорганізми, що добре росли на МПА з ефектом 'роїння'. При посіві в конденсаційну воду мікроорганізми на поверхні середовища утворювали наліт димчасто-блакитного кольору. Який мікроорганізм міг спричинити гнилісний розпад даного продукту?

Шигели дизентерії

Протей

Кишкова паличка

Сальмонели

Холерний вібріон

2131 / 6307
До лабораторії доставлено кров хворого з підозрою на черевний тиф для проведення серологічного дослідження. Реакцію Відаля поставив недостатньо кваліфікований лаборант, який обмежився використанням тільки Ота Н-діагностикумів із сальмонел тифу. Які ще діагностикуми слід було використати для правильної постановки реакції Відаля?

Холери та дизентерії

Паратифів А та В

Еритроцитарні О- та Н-діагностикуми

Висипного та поворотного тифів

К та Vi діагностикуми сальмонел тифу

2132 / 6307
У дитини виявлена схильність до ожиріння, яка є результатом діатезу Назвіть вид діатезу, при якому найчастіше може розвинутись ожиріння:

Нервово-артритичний

Лімфатико-гіпопластичний

Ексудативно-катаральний

Астенічний

2133 / 6307
Інозитолтрифосфати в тканинах організму утворюються в результаті гідролізу фосфатидилінозитолдифосфатів і відіграють роль вторинних посередників (месенджерів) в механізмі дії гормонів. Їхній вплив у клітині спрямований на:

Вивільнення іонів кальцію з клітинних депо

Активацію аденілатциклази

Гальмування протеїнкінази С

Гальмування фосфодіестерази

Активацію протеїнкінази А

2134 / 6307
У хворого спостерігаються геморагії, в крові знижена концентрація протромбіну. Недостатність якого вітаміну призвела до порушення синтезу цього фактору згортання крові?

D

C

A

E

K

2135 / 6307
При отруєнні невідомим препаратом у пацієнта спостерігались сухість слизової оболонки рота та розширення зіниць. З яким впливом пов’язана дія цього препарату?

Стимуляція M-холінорецепторів

Блокада M-холінорецепторів

Стимуляція адренорецепторів

Блокада адренорецепторів

Стимуляція H-холінорецепторів

2136 / 6307
У відпочиваючого в санаторії у результаті сонячного опіку на шкірі спини утворилися міхурці, заповнені світлою рідиною, оточені зоною гіперемії, болісні. Який з перерахованих механізмів лежить в основі формування ексудації у вогнищі запалення?

Збільшення колоїдно-осмотичного тиску в тканині

Збільшення кількості лізосомальних ферментів

Зменшення виведення рідини з тканини

Зменшення рівня кейлонів у тканині

Еміграція лейкоцитів з судин

2137 / 6307
У лабораторії особливо небезпечних інфекцій проводиться мікроскопічне дослідження патологічного матеріалу від хворого з підозрою на чуму, забарвленого за Гінсом-Буррі. Яку властивість збудника дозволяє визначити даний метод?

Капсулоутворення

Лугостійкість

Наявність зерен волютину

Кислотостійкість

Спороутворення

2138 / 6307
Внаслідок вираженого зниження концентрації кальцію в плазмі крові у дитини 2-х років виникли тетанічні скорочення дихальних і глоткових м’язів. Зниження секреції якого гормону може бути причиною цього?

Кортизол

Паратгормон

Соматотропін

Тиреокальцитонін

Альдостерон

2139 / 6307
У хворого з лихоманкою та висипкою на шкірі після обстеження за допомогою серологічних реакцій поставлений діагноз фасціольоз. Було встановлено, що хворий заразився шляхом споживання сирої води з річки. Яка стадія життєвого циклу фасціоли інвазійна для людини?

Фіна

Яйце

Метацеркарій

Мірацидій

Адолескарій

2140 / 6307
До лікаря-педіатра звернулись батьки новонародженої дитини зі скаргами на виділення рідини (сечі) в ділянці пупка. Яка вроджена вада у дитини?

Незрощена сечова протока

Пупкова грижа

Розщеплення сечівника

Пахвинна грижа

Дивертикул Меккеля

2141 / 6307
При харчовому отруєнні виділена культура анаеробних грампозитивних спороутворюючих паличок. До якого виду, найбільш імовірно, відноситься виділений збудник?

Vibrio parahemolyticus

Proteus vulgaris

P.mirabilis

Esherichia coli

C. perfringens

2142 / 6307
До бактеріологічної лабораторії доставлені блювотні маси хворого з підозрою на холеру. З матеріалу приготований препарат 'висяча крапля'. Який метод мікроскопії буде використаний для виявлення збудника по його рухливості?

Шерстна

Імунна електронна

Фазово-контрастна

Електронна

Люмінесцентна

2143 / 6307
У результаті радіаційного випромінювання ушкоджені стовбурові гемопоетичні клітини. Утворення яких клітин сполучної тканини буде порушено?

Періцити

Адіпоцити

Фібробласти

Меланоцити

Макрофаги

2144 / 6307
У потерпілого з травмою голови у скроневій ділянці діагностовано епідуральну гематому. Яка з артерій найімовірніше пошкоджена?

Поверхнева скронева

Середня оболонкова

Передня оболонкова

Задня вушна

Середня мозкова

2145 / 6307
У дитини спостерігається затримка фізичного та розумового розвитку, глибокі порушення з боку сполучної тканини внутрішніх органів, у сечі виявлено кератансульфати. Обмін яких речовин порушений?

Фібронектин

Еластин

Гіалуронова кислота

Колаген

Глікозаміноглікани

2146 / 6307
У хворої 45-ти років невроз, що проявляється дратівливістю, безсонням, немотивованою тривогою. Який лікарський засіб усуне всі симптоми?

Леводопа

Діазепам

Пірацетам

Кофеїн-бензоат натрію

Екстракт валеріани

2147 / 6307
У хворого після вживання недоброякісної їжі розвинулася діарея. На наступний день у нього знизився артеріальний тиск, з’явились тахікардія, екстрасистолія. pH крові складає 7,18. Ці порушення є наслідком розвитку:

Газового алкалозу

Метаболічного алкалозу

Негазового алкалозу

Газового ацидозу

Негазового ацидозу

2148 / 6307
При ендоскопії шлунка виявлена атрофія слизової оболонки. Мікроскопічно в біоптаті виявлена пухлина, побудована з ланцюжків атипових епітеліальних клітин, розташованих серед тяжів грубоволокнистої сполучної тканини; строма значно переважає над паренхімою. Який найбільш імовірний діагноз?

Скирозний рак

Аденокарцинома

Дрібноклітинний рак

Медулярний рак

Солідний рак

2149 / 6307
Слизова оболонка трубчастого органу вкрита перехідним епітелієм, утворює поздовжні складки. М’язова оболонка складається з двох шарів у верхній половині і трьох у нижній. Який це орган?

Сечовий міхур

Сечовід

Маткова труба

Пряма кишка

Стравохід

2150 / 6307
До лікарні доставлений хворий з отруєнням інсектицидом - ротеноном. Яка ділянка мітохондріального ланцюга переносу електронів блокується цією речовиною?

Сукцинат-коензим Q-редуктаза

Коензим Q-цитохром С-редуктаза

НАДН-коензим Q-редуктаза

Цитохром С-оксидаза

АТФ -синтетаза

2151 / 6307
У хворого, який довготривало приймав преднізолон, в результаті відміни препарату виникло загострення захворювання, зниження артеріального тиску, слабкість. З чим можна зв’язати ці прояви?

Звикання до препарату

Гіперпродукція АКТГ

Сенсибілізація до препарату

Кумуляція препарату

Виникнення недостатності кори наднирників

2152 / 6307
У дитини 7-ми років, хворої на ангіну, був взятий матеріал (мазок з поверхні мигдаликів) і засіяний на кров’яний агар. Через добу виросли колонії стрептококів, навколо яких середовище стало прозорим. Наявність якого фактору патогенності у збудника виявило це дослідження?

Ендотоксин

Лейкоцидин

Нейрамінідаза

Гемолізин

Бета -лактамаза

2153 / 6307
У хворого на алкоголізм почався алкогольний психоз із вираженим психомоторним збудженням. Який препарат з групи нейролептиків слід призначити для швидкої допомоги?

Фторотан

Натрію бромід

Діазепам

Резерпін

Аміназин

2154 / 6307
При огляді дитини 4-х років зі скаргами на загальну слабкість, біль у горлі та утруднене ковтання лікар запідозрив дифтерію та направив матеріал до бактеріологічної лабораторії. На яке диференціально-діагностичне поживне середовище слід засіяти матеріал для виділення збудника дифтерії?

Середовище Сабуро

Середовище Плоскірєва

Середовище Левенштейна-Йєнсена

Середовище Ендо

Кров’яно-телуритовий агар

2155 / 6307
Серед антиатеросклеротичних препаратів, що застосовуються з метою профілактики та лікування атеросклерозу, є левостатин. Він діє шляхом:

Активації метаболізму холестерину

Пригнічення всмоктування холестерину в кишечнику

Гальмування біосинтезу холестерину

Усіма наведеними шляхами

Стимулювання екскреції холестерину з організму

2156 / 6307
У хворого під час ендоскопічного обстеження на слизовій оболонці шлунка виявлено кілька ерозій. Який із зазначених лікарських засобів міг спричинити таке ускладнення?

Діазепам

Диклофенак-натрій

Фамотидин

Глауцин

Дигоксин

2157 / 6307
При лабораторному дослідженні у хворого виявили стеаторею. Вкажіть фермент, недостатність дії якого призвела до виникнення цього симптому:

Пепсин

Хімотрипсин

Ліпаза

Лактаза

Амілаза

2158 / 6307
Людина впродовж тривалого часу вживала їжу, бідну на метіонін, внаслідок чого у неї спостерігалися розлади функції нервової та ендокринної систем. Це може бути наслідком порушення синтезу:

Жирних кислот

Адреналіну

Пірувату

Тироніну

Глюкагону

2159 / 6307
Жінка 45-ти років декілька років хворіє на системний червоний вовчак у легкій формі. При прогресуванні захворювання (з’явився міокардит) їй призначили преднізолон як імуносупресор. Через 2 місяці прийому у хворої виникла шлункова кровотеча. Яка найбільш імовірна її причина?

Збудження ЦНС

Ульцерогенна дія

Зменшення згортання крові

Подальше прогресування захворювання

Підвищення артеріального тиску

2160 / 6307
У новонародженої дитини виявлено вроджені вади розвитку травної системи, що пов’язано з дією тератогенних факторів на початку вагітності. На якій з зародкових листків подіяв тератоген?

Усі листки

Ендодерма

Ектодерма

Мезодерма

Ендодерма і мезодерма

2161 / 6307
Дитина під час гри порізала ногу осколком скла і була направлена у поліклініку для введення протиправцевої сироватки. З метою попередження розвитку анафілактичного шоку лікувальну сироватку вводили за Безредкою. Який механізм лежить в основі подібного способу гіпосенсибілізації організму?

Стимуляція імунологічної толерантності до антигену

Стимуляція синтезу антиген-специфічних IgG

Зв’язування фіксованих на тучних клітинах IgE

Зв’язування рецепторів до IgE на тучних клітинах

Блокування синтезу медіаторів у тучних клітинах

2162 / 6307
До токсикологічного відділення надійшов хворий з симптомами гострого отруєння сполуками ртуті. Який препарат необхідно призначити у якості антидоту?

Плантаглюцин

Нейромедин

Унітіол

Трифтазин

Баррол

2163 / 6307
Недостатність в організмі лінолевої та ліноленової кислот призводить до ушкоджень шкіри, випадіння волосся, сповільненого загоювання ран, тромбоцитопенії, зниження опірності до інфекційних захворювань. Порушення синтезу яких речовин найімовірніше зумовлює вказані симптоми?

Катехоламіни

Інтерлейкіни

Кортикостероїди

Інтерферони

Ейкозаноїди

2164 / 6307
У жінки, що тривало приймала антибіотики з приводу кишкової інфекції, розвинулось ускладнення з боку слизової порожнини рота у вигляді запального процесу і білого нальоту, у якому під час бактеріологічного дослідження були виявлені дріжджеподібні грибки Candida albicans. Який з перерахованих препаратів показаний для лікування цього ускладнення?

Флуконазол

Фуразолідон

Тетрациклін

Поліміксин

Бісептол

2165 / 6307
У хворого в лейкограмі: лейкоцити - 14 • 109 /л; мієлобласти - 71%, промієлоцити, мієлоцити, метамієлоцити - 0%, паличкоядерні нейтрофіли - 6%, сегментоядерні -13%; лімфоцити - 7%, моноцити - 3%. Яка патологія у хворого?

Мієлобластний лейкоз

Лімфобластний лейкоз

Нейтрофільний лейкоцитоз

Хронічний лімфолейкоз

Хронічний мієлолейкоз

2166 / 6307
Під час розтину тіла померлої жінки 54-х років з діагнозом 'вузликовий періартеріїт', мікроскопічно були виявлені такі змін в артеріолах: мукоїдне і фібриноїдне набухання, фібриноїдний некроз стінок, дифузна лімфогістіоцитарна інфільтрація, зміни ендотеліоцитів. Назвіть вид запалення в артеріолах:

Фібринозне запалення

Гостре не імунне запалення

Гостре імунне запалення

Дифузне ексудативне запалення

Дифтеритичне запалення

2167 / 6307
При дослідженні гостроти слуху в коваля виявили втрату слуху на 50% у діапазоні низьких частот і майже нормальну гостроту слуху в діапазоні високих частот. Порушення яких структур слухової системи призвело до такого стану?

М’язи середнього вуха

Барабанна перетинка

Середня частина кортієвого органу

Кортієв орган - ближче до овального віконця

Кортієв орган - ближче до гелікотреми

2168 / 6307
При відборі для ревакцинації вакциною БЦЖ у школяра поставлено пробу Манту, яка виявилася негативною. Результат проби свідчить про такі особливості імунітету до туберкульозу:

Наявність гуморального імунітету

Відсутність клітинного імунітету

Наявність клітинного імунітету

Відсутність гуморального імунітету

Відсутність антитоксичного імунітету

2169 / 6307
Людина вийшла з кондиціонованого приміщення назовні, де немає вітру, температура повітря +38oC, вологість 64%. За рахунок якого механізму буде здійснюватися віддача тепла організмом за цих умов?

Теплорадіація

Випаровування поту

Теплопроведення

Конвекція

Кондукція

2170 / 6307
Робітниця хімічного підприємства внаслідок порушення правил безпечної роботи зазнала токсичної дії азотистої кислоти та нітритів, які викликають дезамінування цитозину в молекулі ДНК. Який фермент ініціює ланцюг репараційних процесів?

ДНК-залежна-РНК-полімераза

Оротидилмонофосфат-декарбоксилаза

Уридин-ДНК-глікозидаза

Тимідилатсинтаза

Цитидинтрифосфатсинтетаза

2171 / 6307
У пацієнта з передозуванням наркотичної речовини відсутня свідомість, гіпотермія, гіпотензія, стійкий міоз, порушення дихання по типу Чейн-Стокса. Який функціональний антагоніст необхідно призначити для забезпечення виживання пацієнта?

Налоксон

Омепразол

Етимізол

Мезатон

Нітразепам

2172 / 6307
Хворий надійшов до клініки зі скаргами на загальну слабкість, порушення сну. Шкіра має жовтий колір. У крові: збільшена кількість прямого білірубіну, жовчних кислот. Кал ахолічний. Для якого стану характерні ці зміни?

Гемолітична жовтяниця

Хронічний холецистит

Надпечінкова жовтяниця

Механічна жовтяниця

Синдром Жільбера

2173 / 6307
У жінки 22-х років через 5 годин після вживання морепродуктів на шкірі тулуба та дистальних відділів кінцівок з’явились маленькі сверблячі папули, які частиною зливаються між собою. Через добу висипка самовільно зникла. Назвіть механізм гіперчутливості, що полягає в основі даних змін:

Системна анафілаксія

Імунокомплексна гіперчутливість

Атопія (місцева анафілаксія)

Антитілоопосередкований клітинний цитоліз

Клітинна цитотоксичність

2174 / 6307
Хворий на гіпертонічну хворобу разом з безсольовою дієтою та з антигіпертензивними засобами, довгий час приймав гідрохлортіазид, що зумовило порушення електролітного балансу. Яке порушення внутрішнього середовища виникло у хворого?

Гіпермагніємія

Збільшення об’єму циркулюючої крові

Метаболічний ацидоз

Гіпохлоремічний алкалоз

Гіперкаліємія

2175 / 6307
При авторадіографічному дослідженні епітелію тонкої кишки було виявлено, що його повне оновлення відбувається протягом 3-х діб за рахунок проліферації малодиференційованих клітин. Вкажіть їх локалізацію:

Основа ворсинок

Власна пластинка слизової оболонки

Верхівка ворсинок

Бічна поверхня ворсинок

Дно крипт

2176 / 6307
Пасажири автобуса у спекотну погоду попросили відкрити люки. Який шлях тепловіддачі при цьому зростає найбільше?

Теплопроведення

Випромінювання та теплопроведення

Конвекція

Випромінювання

Випаровування поту

2177 / 6307
У хворого через добу після апендектомії при аналiзi крові виявили нейтрофільний лейкоцитоз з регенеративним зсувом вліво. Який найбільш імовірний механізм розвитку абсолютного лейкоцитозу у периферичній крові хворого?

Зменшення руйнування лейкоцитів

Посилення лейкопоезу

Уповільнення еміграції лейкоцитів у тканині

Активація імунітету

Перерозподіл лейкоцитів в організмі

2178 / 6307
При загостренні ревматоїдного артриту хворому, в анамнезі якого супутній хронічний гастрит, призначений целекоксиб. Чим обумовлено зменшення побічної дії препарату на травний тракт?

Переважаюче пригнічення циклооксигенази-2

Пригнічення фосфоліпази А2

Переважаюча стимуляція аденіла-тциклази

Переважаюче пригнічення циклооксигенази-1

Пригнічення фосфодіестерази

2179 / 6307
В експерименті на кролі встановлено, що об’єм кисню, який споживається головним мозком за 1 хвилину, дорівнює об’єму CO2 , який виділяється клітинами мозку в кров. Це свідчить, що у клітинах головного мозку має місце:

Окислення вуглеводів

Окислення жирів

Гіпоксія

Окислення білків

Гіпокапнія

2180 / 6307
Хворому поставлено діагноз газова гангрена. Після ідентифікації збудника досліджуваний матеріал необхідно знищити. Який метод слід використати?

Тиндалізація

Кип’ятіння

Стерилізація парою під тиском

Пастеризація

Стерилізація текучою парою

2181 / 6307
Фенілкетонурія - це захворювання, яке зумовлено рецесивним геном, що локалізується в аутосомі. Батьки є гетерозиготами за цим геном. Вони вже мають двох хворих синів і одну здорову доньку. Яка імовірність, що четверта дитина, яку вони очікують, народиться теж хворою?

0%

50%

100%

25%

75%

2182 / 6307
До косметолога звернулася пацієнтка зі скаргами на появу чорних цяток на обличчі. Після обстеження було встановлено, що поява цяток пов’язана з порушенням виділення секрету сальних залоз. Який тип секреції характерний для цих залоз?

Мерокриновий та мікроапокриновий

Мерокриновий

Макроапокриновий

Мікроапокриновий

Голокриновий

2183 / 6307
У хворого 40-ка років ознаки гірської хвороби: запаморочення, задишка, тахікардія, рН крові - 7,50, pCO2 -30 мм рт.ст., зсув буферних основ +4 ммоль/л. Яке порушення кислотноосновного стану має місце?

Видільний ацидоз

Газовий алкалоз

Газовий ацидоз

Негазовий алкалоз

Негазовий ацидоз

2184 / 6307
Після ремонту автомобіля в закритому приміщенні при працюючому двигуні у чоловіка з’явилися задишка, запаморочення, акроціаноз, частота дихання 24-26/хв. Газовий склад крові: pO2 - 60 мм рт.ст., pCO2 - 30 мм рт.ст.; у крові наявний карбоксигемоглобін. Про який вид гіпоксії можна думати?

Гемічна

Циркуляторна

Респіраторна

Тканинна

Гіпоксична

2185 / 6307
У клітині в гранулярній ЕПС відбувається етап трансляції, при якому спостерігається просування і-РНК щодо рибосоми. Амінокислоти з’єднуються пептидними зв’язками в певній послідовності - відбувається біосинтез поліпептиду. Послідовність амінокислот у поліпептиді буде відповідати послідовності:

Антикодонів т-РНК

Нуклеотидів р-РНК

Нуклеотидів т-РНК

Антикодонів р-РНК

Кодонів і-РНК

2186 / 6307
При обстеженні у хворого виявлене порушення чутливості шкіри в ділянці передньої поверхні шиї. Який нерв уражений?

Поперечний нерв шиї

Малий потиличний

Великий вушний

Шийна петля

Надключичні

2187 / 6307
Хворий на хронічну серцеву недостатність тривалий час приймав препарат з групи серцевих глікозидів. У нього з’явились нудота, слабкість, екстрасистолія. Яке явище обумовило розвиток цих симптомів?

Звикання

Лікарська залежність

Ідіосинкразія

Матеріальна кумуляція

Функціональна кумуляція

2188 / 6307
При ультразвуковому обстеженні дитини був виявлений дивертикул Меккеля. Аномалію розвитку якої кишки діагностовано?

Сліпа

Ободова

Сигмоподібна

Порожня

Клубова

2189 / 6307
До реанімаційного відділення в тяжкому стані, без свідомості надійшов пацієнт. Діагностовано передозування барбітуратів, які спричинили феномен тканинної гіпоксії. На якому рівні відбулося блокування електронного транспорту?

Убіхінон

АТФ -синтаза

Цитохром b - цитохром c1

НАДН-коензимQ-редуктаза

Цитохромоксидаза

2190 / 6307
Встановлено, що в клітинах організмів відсутні мембранні органели та їх спадковий матеріал не має нуклеосомної організації. Що це за організми?

Еукаріоти

Віруси

Аскоміцети

Прокаріоти

Найпростіші

2191 / 6307
У пацієнта діагностований первинний туберкульоз легень. Призначення якого протитуберкульозного засобу з групи антибіотиків, що порушують синтез РНК мікобактерій, є бажаним?

Піразинамід

Етамбутол

Рифампіцин

ПАСК

Канаміцину сульфат

2192 / 6307
Хлопчик на другому році життя став часто хворіти на респіраторні захворювання, стоматити, гнійничкові ураження шкіри. Навіть невеликі пошкодження ясен і слизової ускладнюються запаленням, що протікає тривало. Встановлено, що у крові дитини практично відсутні імуноглобуліни усіх класів. Зниження функціональної активності якої клітинної популяції лежить в основі описаного синдрому?

В-лімфоцити

Т-лімфоцити

Макрофаги

NK-лімфоцити

Нейтрофіли

2193 / 6307
Секреція грудного молока у жінок обумовлена полімерними генами, причому кількість молока зростає із збільшенням числа домінантних алелів цих генів у генотипі жінки. Який генотип може мати породілля з відсутністю молока?

M1 M1m2m2

M1m1M2m2

m1m1m2m2

M1m1m2m2

m1m1M2m2

2194 / 6307
У хворого при томографічному обстеженні було виявлено пухлину відділу мозку з ушкодженням ядер XI та XII пар черепних нервів. Який це відділ мозку?

Mesencephalon

Diencephalon

Myelencephalon

Metencephalon

Telencephalon

2195 / 6307
У хворого 69-ти років на шкірі в ділянці нижньої повіки з’явилося невелике бляшкоподібне утворення з наступним виразкуванням, яке було оперативно видалене. При мікроскопічному дослідженні утворення: в дермі шкіри комплекси з атипових епітеліальних клітин; периферії клітини розташовані перпендикулярно до базальної мембрани. Клітини темні, призматичної полігональної форми з гіперхромними ядрами з частими мітозами. Іноді зустрічаються утворення, подібні до волосяного фолікула. Яка гістологічна форма рака у хворого?

Недиференційований рак

Плоскоклітинний рак з ороговінням

Базально-клітинний рак

Плоскоклітинний рак без ороговіння

Аденокарцинома

2196 / 6307
До пологового відділення госпіталізували жінку зі слабкістю пологової діяльності. Який засіб необхідно використати для стимуляції скорочень матки?

Кортиколіберин

Іонадоліберин

Соматостатин

Вазопресин

Окситоцин

2197 / 6307
У стоматолога на прийомі дуже неспокійний пацієнт, який ніяк не може зручно влаштуватися в кріслі, норовить схопити доктора за руку, заглядає на маніпуляційний стіл, цікавиться в медсестри, чи стерильні інструменти. Який темперамент у цього пацієнта?

Холерик

Меланхолік

Сангвінік

Флегматик

2198 / 6307
У хворого на слизовій оболонці ясен виразка овальної форми з припіднятими краями хрящоподібної щільності. Дно виразки м’ясисто-червоного забарвлення з нашаруваннями сірого кольору. При мікроскопічному дослідженні - проліферація ендотелію дрібних судин, периваскулярна лімфоплазмоцитарна інфільтрація. Про яке захворювання йдеться?

Сифіліс

Виразка-рак

Травматична виразка

Виразково-некротичний гінгівіт

Ерозивно-виразкова лейкоплакія

2199 / 6307
Визначте пульсовий і середньодинамічний артеріальний тиск (мм рт.ст.) у обстежуваного, якщо виміряний у нього артеріальний тиск становить 130/70 мм рт.ст.:

50,90

60, 90

50, 70

60, 100

60,80

2200 / 6307
У людини подразнення шкіри - свербіж, висипання, депігментація, збільшені лімфовузли. В оці знайдені філярії. Встановлений діагноз - онхоцеркоз. Які компоненти гнусу могли стати переносниками філярій p.Onchocerca?

Мошки

Мокрець

Комарі

Москіти

Ґедзі

2201 / 6307
Метильні групи (—CH3) використовуються в органiзмi для синтезу таких важливих сполук, як креатин, холін, адреналін, інші. Джерелом цих груп є одна з незамінних амінокислот, а саме:

Метіонін

Триптофан

Ізолейцин

Валін

Лейцин

2202 / 6307
Хвора 46-ти років скаржиться на сухість в роті, спрагу, почащений сечопуск, загальну слабкість. У крові: гіперглікемія, гіперкетонемія. У сечі: глюкоза, кетонові тіла. На ЕКГ: дифузні зміни в міокарді. Який найбільш імовірний діагноз?

Гострий панкреатит

Немічна хвороба серця

Цукровий діабет

Аліментарна гіперглікемія

Нецукровий діабет

2203 / 6307
В шкірі виявлена щільна, рухома, чітко відмежована від оточуючих тканин пухлина. На розрізі вона білого кольору, представлена волокнистою тканиною. Мікроскопічно: хаотично переплетені колагенові волокна, клітин мало. Що це за пухлина?

Дерматофіброма

Міома

Фіброма

Гістіоцитома

Десмоїд

2204 / 6307
У постраждалого виявлено рану верхньої частини передньої ділянки плеча. При обстеженні встановлена втрата активного згинання в ліктьовому суглобі і чутливості шкіри передньобічної поверхні передпліччя. Порушення функції якого нерва має місце?

Пахвовий

М’язово-шкірний

Серединний

Променевий

Ліктьовий

2205 / 6307
При підозрі на туберкульоз хворій дитині зробили пробу Манту. Через 24 години у місці введення алергену з’явились припухлість, гіперемія і болісність. Які основні компоненти визначають цю реакцію організму?

Гранулоцити, Т-лімфоцити і IgG

Мононуклеари, Т-лімфоцити і лімфокіни

Плазматичні клітини, Т-лімфоцити і лімфокіни

В-лімфоцити, IgМ

Макрофаги, В-лімфоцити і моноцити

2206 / 6307
На прийом до терапевта прийшов чоловік 37-ми років зі скаргами на періодичні інтенсивні больові напади у суглобах великого пальця стопи та їх припухлість. У сечі: різко кисла реакція і рожеве забарвлення. З наявністю яких речовин можуть бути пов’язані такі зміни?

Амонієві солі

Сульфат магнію

Хлориди

Солі сечової кислоти

Фосфат кальцію

2207 / 6307
При санітарно-бактеріологічному дослідженні водопровідної води отримані наступні результати: загальна кількість бактерій в 1,0 мл - 80, колі-індекс - 3. Як розцінити результат дослідження?

Вода є сумнівною

Вода є дуже забрудненою

Вода є забрудненою

Вода придатна для споживання

Вода є дуже сумнівною

2208 / 6307
в крові: ер.- 3,0 • 1012 8. /л; Hb-90г/л; ретикул.- 0,5%. В мазку: пойкілоцити, гіпохромні еритроцити. Залізо сироватки крові - 80 мкмоль/л. Для якої патології це характерно?

B12-дефіцитна анемія

Серпоподібноклітинна анемія

Залізодефіцитна анемія

Залізорефрактерна анемія

Хвороба Мінковського-Шоффара

2209 / 6307
Хворий 40-ка років впродовж тривалого часу страждає на бронхіальну астму і тахікардію. Вкажіть найдоцільніший в даній ситуації препарат для усунення бронхоспазму:

Ізадрин

Адреналіну гідрохлорид

Сальбутамол

Ефедрину гідрохлорид

Атропіну сульфат

2210 / 6307
Хворому на гіпертонічну хворобу був призначений препарат для зниження артеріального тиску з групи адренотропних засобів. Через деякий час у хворого тиск нормалізувався, але розвинулися брадикардія до 50/хв. та атріовентрикулярна блокада II ступеня. Який препарат було призначено?

Верапаміл

Празозин

Анаприлін

Мезатон

Клофелін

2211 / 6307
Хлопчик 12-ти років знаходиться у лікарні з підозрою на харчову токсикоінфекцію. При посіві фекалій хворого на середовище Ендо виросла велика кількість безбарвних колоній. Який мікроорганізм можна з найбільшою імовірністю ВИКЛЮЧИТИ з числа можливих збудників захворювання?

Salmonella enteritidis

Pseudomonas aeruginosa

Escherichia coli

Proteus vulgaris

Yersinia enterocolitica

2212 / 6307
Хворій 65-ти років, що страждає на інсулінонезалежний цукровий діабет, призначили всередину глібенкламід. Вкажіть механізм гіпоглікемічної дії цього препарату:

Пригнічує глюконеогенез у печінці

Пригнічує всмоктування глюкози у кишечнику

Пригнічує альфа-глюкозидазу і розпад полісахаридів

Посилює утилізацію глюкози периферичними тканинами

Стимулює виділення ендогенного інсуліну бета-клітинами

2213 / 6307
У тварини через 2 тижні після експериментального звуження ниркової артерії підвищився артеріальний тиск. Зі збільшенням дії на судини якого фактора гуморальної регуляції це пов’язано?

Ангіотензин II

Дофамін

Кортизол

Альдостерон

Вазопресин

2214 / 6307
У лабораторному експерименті на собаці вивчали будову центральних відділів слухової сенсорної системи. Була зруйнована одна з структур середнього мозку. Собака втратив орієнтувальний рефлекс на звукові сигнали. Яка структура була зруйнована?

Чорна речовина

Червоне ядро

Ядра ретикулярної формації

Верхні горбики чотиригорбикового тіла

Нижні горбики чотиригорбикового тіла

2215 / 6307
Хворий 23-х років надійшов до лікарні із черепно-мозковою травмою у важкому стані. Дихання характеризується судомним тривалим вдихом, який переривається коротким видихом. Для якого типу дихання це характерно?

Чейн-Стокса

Гаспінг-дихання

Біота

Апнейстичне

Куссмауля

2216 / 6307
При токсичному ушкодженні клітин печінки з порушенням її функцій у хворого з’явилися набряки. Які зміни складу плазми крові є провідною причиною розвитку набряків?

Зниження вмісту альбумінів

Збільшення вмісту глобулінів

Збільшення вмісту альбумінів

Зменшення вмісту фібриногену

Зменшення вмісту глобулінів

2217 / 6307
Під час операції в печінці хворого виявлені дрібні міхурці малих розмірів з незначною кількістю рідини, які щільно прилягають один до одного. Який гельмінтоз виявився у хворого?

Альвеококоз

Фасціольоз

Дікроцеліоз

Клонорхоз

Опісторхоз

2218 / 6307
Тварині, сенсибілізованій туберкуліном, внутрішньоочеревенно введений туберкулін. Через 24 години при лапаротомії виявлено венозну гіперемію та набряк очеревини. У мазках-відбитках з очеревини велика кількість лімфоцитів та моноцитів. Який патологічний процес у тварини?

Алергічне запалення

Серозне запалення

Гнійне запалення

Фібринозне запалення

Асептичне запалення

2219 / 6307
У хлопчика 3-х років з вираженим геморагічним синдромом відсутній антигемофільний глобулін А (фактор VIII) у плазмі крові. Яка фаза гемостазу первинно порушена у цього хворого?

Ретракція кров’яного згустку

Зовнішній механізм активації протромбінази

Перетворення протромбіну в тромбін

Перетворення фібриногену в фібрин

Внутрішній механізм активації протромбінази

2220 / 6307
У пацієнта з бронхіальною астмою за допомогою шкірних алергічних проб встановлено сенсибілізацію алергеном тополиного пуху. Який фактор імунної системи відіграє вирішальну роль в розвитку цього імунопатологічного стану?

IgE

IgG

IgD

IgM

Сенсибілізовані Т-лімфоцити

2221 / 6307
При недостатності кровообігу у період інтенсивної м’язової роботи у м’язі в результаті анаеробного гліколізу накопичується молочна кислота. Яка її подальша доля?

Використовується у тканинах для синтезу жирних кислот

Використовується у м’язі для синтезу амінокислот

Видаляється через нирки з сечею

Включається в глюконеогенез у печінці

Використовується тканинами для синтезу кетонових тіл

2222 / 6307
Дитина 10-ти років страждає на стафілококовий дерматит. Лікування бензилпеніциліном не дало результатів. Призначення комбінованого препарату пеніциліну з клавулановою кислотою дало швидке одужання. Яка причина позитивної дії цього препарату?

Гальмування аденозиндезамінази

Блокада транслокази

Інактивація бета-лактамази

Активація фосфодіестерази

Гальмування транспептидази

2223 / 6307
Хворий 18-ти років звернувся до лікаря зі скаргами на безсоння, що проявляється у важкому засинанні. В результаті цього він не висипається і на наступний день почуває втому, важко засвоює навчальний матеріал. Лікар встановив, що безсоння пов’язане з неврозоподібним станом. Зробіть раціональний вибір снодійного:

Фенобарбітал

Етамінал-натрій

Хлоралгідрат

Нітразепам

Бромізовал

2224 / 6307
У хворого, який страждає на вугрі та на запальні зміни шкіри обличчя, при мікроскопії матеріалу з осередків ураження виявлені живі членистоногі, довгастої форми, з 4 парами дуже редукованих кінцівок. Встановіть попередній діагноз:

Ураження шкіри коростяним свербуном

Алергія

Ураження шкіри блохами

Демодекоз

Педикульоз

2225 / 6307
Подразнення правого блукаючого нерва спричинило різке сповільнення атріовентрикулярного проведення. На ЕКГ при цьому буде подовжений:

Зубець T

Інтервал P — Q

Комплекс QRST

Інтервал R — R

Зубець P

2226 / 6307
У людини після укусу москітом виникли виразки шкіри. Аналіз вмісту виразки виявив всередині клітин людини безджгутикові одноклітинні організми. Який попередній діагноз?

Трипаносомоз

Лейшманіоз дерматотропний

Лейшманіоз вісцеральний

Токсоплазмоз

Балантидіоз

2227 / 6307
У чоловіка 63-х років рак стравоходу, метастази у лімфатичні вузли середостіння, ракова кахексія. Яка патогенетична стадія пухлинного процесу має місце?

Промоції

Ініціації

Трансформації

Прогресії

2228 / 6307
Жінку 44-х років вжалила оса, внаслідок чого розвинувся шок. В анамнезі - тяжка алергічна реакція на жалення оси. Об’єктивно: Ps- 179/хв, слабкий, АТ- 80/40 мм рт.ст., ЧД- 26/хв. Яка провідна ланка патогенезу анафілактичного шоку?

Зменшення ударного об’єму серця

Зниження периферійного опору судин

Тахікардія

Біль

Зменшення об’єму циркулюючої крові

2229 / 6307
Хворий надійшов до інфекційного відділення з підозрою на холеру. Який основний метод дослідження необхідно використати для підтвердження діагнозу?

Імунологічний

Алергічний

Серологічний

Бактеріологічний

Біологічний

2230 / 6307
При розтині тіла померлого виявлена гіперплазія кісткового мозку плоских і трубчастих кісток (піоїдний кістковий мозок), спленомегалія (6 кг), гепатомегалія (5 кг), збільшення всіх груп лімфатичних вузлів. Якому захворюванню відповідають виявлені зміни?

Лімфогрануломатоз

Мієломна хвороба

Справжня поліцитемія

Хронічний мієлолейкоз

Хронічний лімфолейкоз

2231 / 6307
Електрофоретичне дослідження сироватки крові хворого на пневмонію показало збільшення однієї з білкових фракцій. Вкажіть її:

Альбуміни

β-глобуліни

α1-глобуліни

α2-глобуліни

γ-глобуліни

2232 / 6307
У хворого коса пахвинна грижа. Яке анатомічне утворення стало слабким місцем передньої черевної стінки?

Пахвинний трикутник

Латеральна пахвинна ямка

Медіальна пахвинна ямка

Стегнова ямка

Надміхурова ямка

2233 / 6307
У хворого запалення легень ускладнилось ексудативним плевритом. В якому з перелічених анатомічних утворень переважно може накопичуватися рідина?

Sinus transversus pericardii

Sinus costodiaphragmaticus pleurae

Sinus phrenicomediastinalis pleurae

Sinus costomediastinalis pleurae

Sinus obliquus pericardii

2234 / 6307
Хворий не може відвести від тулуба верхню кінцівку. Який м’яз не виконує свою функцію?

Підосний м’яз

Малий круглий м’яз

Великий круглий м’яз

Дельтоподібний

Найширший м’яз спини

2235 / 6307
У хворої 45-ти років при електрокардіографічному обстеженні виявлено такі зміни: інтервал P — Q подовжений, при цьому випадає кожен другий або третій комплекс QRST. Яке саме порушення провідності серця спостерігається?

Синоаурікулярна блокада

Внутрішлуночкова блокада

Атріовентрикулярна блокада повна

Атріовентрикулярна блокада III ступеня

Атріовентрикулярна блокада I ступеня

2236 / 6307
У хворого порушена моторна функція язика. З патологією якого нерва це пов’язано?

Лицевий

Язикоглотковий

Під’язиковий

Додатковий

Блукаючий

2237 / 6307
При розтині тіла померлого чоловіка 65-ти років, який страждав на захворювання легень, патологічний процес переважно був локалізований у бронхах, де при гістологічному дослідженні були чітко видні залози, хрящові острівці та багаторядний циліндричний миготливий епітелій. В яких бронхах відбулися зміни?

Малі бронхи

Середні бронхи

Головні бронхи

Великі бронхи

Термінальні бронхіоли

2238 / 6307
До лікарні надійшов 9-річний хлопчик розумово і фізично відсталий. При біохімічному дослідженні крові: підвищена кількість фенілаланіну. Блокування якого ферменту може призве- сти до такого стану?

Глутаматдекарбоксилаза

Фенілаланін-4-монооксигеназа

Оксидаза гомогентизинової кислоти

Глутамінтрансаміназа

Аспартатамінотрансфераза

2239 / 6307
За клінічними показами хворому призначено піридоксальфосфат. Для корекції яких процесів рекомендований цей препарат?

Синтез білка

Синтез пуринових і піримідинових основ

Трансамінування і декарбоксилюван-ня амінокислот

Окисне декарбоксилювання кетокислот

Дезамінування амінокислот

2240 / 6307
До реанімаційного відділення надійшов хворий з ознаками гострого отруєння морфіном. Який засіб необхідно використати у даному випадку для промивання шлунку?

Розчин натрію хлориду

Фурацилін

Борна кислота

Натрію гідрокарбонат

Калію перманганат

2241 / 6307
У хворого відзначаються болі у ділянці кореня язика, зіву, піднебінних мигдаликів, у верхньому відділі глотки, вусі, втрачений смак у ділянці задньої третини язика. Ураженням якого нерва викликані ці порушення?

Язиковий

Блукаючий

Барабанна струна

Язикоглотковий

Великий кам’янистий

2242 / 6307
Для лікування деяких інфекційних захворювань, викликаних бактеріями, застосовуються сульфаніламідні препарати, що блокують синтез фактора росту бактерій. Назвіть механізм їх дії:

Є алостеричними інгібіторами ферментів

Є антивітамінами параамінобензойної кислоти

Є алостеричними ферментами

Беруть участь в окисно-відновних процесах

Інгібують всмоктування фолієвої кислоти

2243 / 6307
В сечі новонародженого визначається цитрулін та високий рівень амоніаку. Вкажіть, утворення якої речовини, найімовірніше, порушене у цього малюка:

Сечова кислота

Креатин

Креатинін

Амоніак

Сечовина

2244 / 6307
Жінка 45 -ти років, перукар, скаржиться на болі у ногах, що з’являються після роботи, ввечері та вночі. При огляді хворої виявлено варикозне розширені вени на присередній поверхні гомілки та стегна. Яка вена та її безпосередні протоки розширені?

Велика підшкірна

Передня великогомілкова вена

Глибока вена стегна

Стегнова

Мала підшкірна

2245 / 6307
У людини зменшений діурез, гіпернатріємія, гіпокаліємія. Гіперсекреція якого гормону може бути причиною таких змін?

Паратгормон

Вазопресин

Альдостерон

Передсердний натрійуретичний фактор

Адреналін

2246 / 6307
Після травми передньої поверхні верхньої третини передпліччя у хворого утруднення пронації, послаблення долонного згинання кисті та порушення чутливості шкіри 1-3 пальців на долоні. Який нерв ушкоджено?

n. ulnaris

n. musculocutaneus

n. cutaneus antebrachii medialis

n. radialis

n. medianus

2247 / 6307
У хворого 45-ти років при аналізі ЕКГ встановлено: ритм синусовий, число передсердних комплексів більше числа шлуночкових комплексів; прогресуюче подовження інтервалу P — Q від комплексу до комплексу; випадіння окремих шлуночкових комплексів; зубці P та комплекси QRST без змін. Назвіть тип порушення серцевого ритму:

Атріовентрикулярна блокада I ступеня

Внутрішньопередсердна блокада

Повна атріовентрикулярна блокада

Синоаурікулярна блокада

Атріовентрикулярна блокада II ступеня

2248 / 6307
В родині зростає дочка 14-ти років, у якої спостерігаються деякі відхилення від норми: зріст нижче, ніж у однолітків, відсутні ознаки статевого дозрівання, шия дуже коротка, плечі широкі. Інтелект в нормі. Яке захворювання можна припустити?

Синдром Патау

Синдром Дауна

Синдром Клайнфельтера

Синдром Шерешевського-Тернера

Синдром Едвардса

2249 / 6307
Після обстеження хворому на сечокам’яну хворобу призначили алопурінол - конкурентний інгібітор ксантиноксидази. Підставою для цього був хімічний аналіз ниркових каменів, переважною складовою яких є:

Сульфат кальцію

Моногідрат оксалату кальцію

Дигідрат оксалату кальцію

Урат натрію

Фосфат кальцію

2250 / 6307
У хворого 60-ти років, що багато років страждає на атеросклероз і переніс раніше інфаркт міокарда, розвинувся напад загрудинного болю. Хворий госпіталізований через 3 дні; на фоні прогресуючої серцево-судинної недостатності помер. Під час розтину тіла у ділянці задньої стінки лівого шлуночка і міжшлуночкової перегородки виявлена ділянка білого кольору близько 3 см у діаметрі, волокниста, западаюча, з чіткою межею. Прозектор трактував ці зміни як:

Ішемія міокарда

Міокардит

Дистрофія міокарда

Вогнищевий кардіосклероз

Інфаркт міокарда

2251 / 6307
Дослідник при мікроскопічному і електронно-мікроскопічному вивченні печінки звернув увагу, що деякі окремо розташовані клітини розпалися на дрібні фрагменти, оточені мембраною. У деяких з них наявні органели, інші включають фрагменти ядра, що розпалося. Запальна реакція навколо відсутня. Дослідник розцінив ці зміни, як:

Дистрофія

Апоптоз

Некроз

Гіпоплазія

Атрофія

2252 / 6307
У хворого на підгострий септичний ендокардит при огляді лікар відзначив загальну слабкість і іктеричність шкіри, склер і видимих слизових оболонок. У крові виявлена збільшена кількість непрямого білірубіну. Що зумовлює жовтяничність шкіри і слизових?

Підпечінкова жовтяниця

Гемосидероз

Жирова дистрофія

Надпечінкова жовтяниця

Печінкова жовтяниця

2253 / 6307
У хворого в анамнезі: з дитинства відмічався знижений рівень гемоглобіну. Лікування препаратами заліза не дає ефекту. У крові: ер.- 3,1 • 1012/л, ретик.-16%, Hb- 85 г/л, КП0,75; в мазку крові анізоцити, пойкілоцити, мішенеподібні еритроцити, еритроцити з базофільною зернистістю, рівень заліза у сироватці 30 мкмоль/л. Для якої патології системи крові характерні такі дані?

Фолієводефіцитна анемія

Залізодефіцитна анемія

B12-дефіцитна анемія

Гіпопластична анемія

Таласемія

2254 / 6307
До приймального відділення лікарні доставлено хворого з вираженими явищами гострої серцевої недостатності. Який препарат, із зазначених нижче, слід використати в першу чергу?

Фізостигмін

Корглікон

Етазол

Дигітоксин

Дитилін

2255 / 6307
Чоловік 30-ти років звернувся до стоматолога зі скаргою на розлади жування, у нього виникає біль при відтягуванні щелепи назад. Запалення якого з жувальних м’язів найімовірніше встановить лікар?

M. temporalis (задні волокна)

M. Мasseter

M. pterygoideus medialis

M. temporalis (передш волокна)

M. pterygoideus lateralis

2256 / 6307
Під час оперативного втручання на фоні використання гігронію різко знизився артеріальний тиск. Представники яких груп лікарських препаратів можуть нормалізувати артеріальний тиск?

α-адреноміметики

Н-холіноміметики

M-холіноміметики

Гангліоблокатори

α-адреноблокатори

2257 / 6307
з дихальною недостатністю рН крові 7,35. Визначення pCO2 57. показало наявність гіперкапнії. При дослідженні рН сечі відзначається підвищення її кислотності. Яка форма порушення кислотно-основного стану в даному випадку?

Алкалоз газовий, компенсований

Ацидоз метаболічний, компенсований

Алкалоз газовий, декомпенсований

Ацидоз газовий, компенсований

Ацидоз метаболічний, декомпенсований

2258 / 6307
У хворого, що страждає на порушення мозкового кровообігу, встановлено порушення функції лімбічної системи. Порушення кровопостачання у якій артерії мозку викликало ці симптоми?

Передня ворсинчаста

Задня мозкова

Середня мозкова

Хребтова

Передня мозкова

2259 / 6307
Хворому встановлений діагноз ураження голівки стегна ішемічного походження. Яка артерія ушкоджена?

Arteria profunda femoris

Arteria femoralis

Arteria illiaca externa

Arteria umbilicalis

Ramus acetabularum A.obturatoriae

2260 / 6307
В психіатричну клініку доставлений хворий 40-ка років у стані збудження, агресії, марення. Який препарат слід ввести хворому?

Натрію бромід

Седуксен

Настоянку валеріани

Резерпін

Аміназин

2261 / 6307
У нейрохірургічне відділення надійшов 54-річний чоловік із скаргами на відсутність чутливості шкіри нижньої повіки, латеральної поверхні носа, верхньої губи. Лікар при огляді встановив запалення другої гілки трійчастого нерва. Через який отвір виходить із черепа ця гілка?

Круглий отвір

Овальний отвір

Остистий отвір

Рваний отвір

Верхня очноямкова щілина

2262 / 6307
При бактеріологічному дослідженні проб сметани виділені ізольовані культури S.aureus. Як довести етіологічне значення ізольованої культури S.aureus як збудника харчового отруєння, яке виникло серед групи споживачів сметани?

Визначення цукролітичних властивостей

Виявлення ентеротоксину

Визначення плазмокоагулазної активності

Визначення лецитиназної активності

Визначення гемотоксинів

2263 / 6307
У дорослої людини за добу виділяється 20 л сечі з низькою відносною щільністю. Найбільш імовірною причиною цього є дефіцит в організмі:

Паратгормону

Реніну

Альдостерону

Вазопресину

Натрійуретичного фактора

2264 / 6307
У чоловіка 28-ми років при гістологічному дослідженні шийного лімфовузла виявлено: порушення малюнка внаслідок розростання епітеліоїдних, лімфоїдних клітин і макрофагів з ядрами у вигляді підкови, в центрі деяких скупчень клітин - безструктурні ділянки блідо-рожевого кольору з уламками ядер. Для якого захворювання характерні такі зміни?

Туберкульоз

Актиномікоз

Сифіліс

Метастаз пухлини

Лімфогрануломатоз

2265 / 6307
У хворого з жовтяницею встановлено: підвищення у плазмi крові вмісту загального білірубіну за рахунок непрямого (вільного), в калі та сечі - високий вміст стеркобіліну, рівень прямого (зв’язаного) білірубіну в плазмі крові в межах норми. Про який вид жовтяниці можна думати?

Механічна

Паренхіматозна (печінкова)

Жовтяниця немовлят

Хвороба Жильбера

Гемолітична

2266 / 6307
При штовханні штанги спортсмен закидає голову назад для максимального підвищення тонусу м’язів-розгиначів верхніх кінцівок. Де розташовані центри рефлексів, які при цьому виникають?

Рухова кора

Ядра Дейтерса

Спинний мозок

Базальні ганглії

Червоні ядра

2267 / 6307
Знешкодження ксенобіотиків (лікарських засобів, епоксидів, ареноксидів, альдегідів, нітропохідних тощо) та ендогенних метаболітів (естрадіолу, простагландинів, лейкотрієнів) відбувається в печінці шляхом їх кон’югації з:

Фосфоаденозином

Гліцином

S-Аденозилметіоніном

Аспарагіновою кислотою

Глутатіоном

2268 / 6307
При вивченні родоводу сім’ї, в якій спостерігається гіпертрихоз (надмірне оволосіння вушних раковин), виявлена ознака трапляється в усіх поколіннях тільки у чоловіків і успадковується від батька до сина. Визначте тип успадкування гіпертрихозу:

Зчеплений з Y-хромосомою

Аутосомно-домінантний

Зчеплений з Х-хромосомою домінантний

Аутосомно-рецесивний

Зчеплений з Х-хромосомою рецесивний

2269 / 6307
У спортсмена після інтенсивного тренування відзначається значне зниження тонусу судин у ділянці працюючих м’язів. Причиною розвитку такого ефекту є накопичення у працюючих тканинах:

Метаболітів

Ренін-ангіотензину

Натрійуретичного гормону

Серотоніну

Гістаміну

2270 / 6307
Внаслідок активації іонних каналів зовнішньої мембрани збудливої клітини значно збільшився її потенціал спокою. Які канали були активовані?

Натрієві

Натрієві та кальцієві

Швидкі кальцієві

Повільні кальцієві

Калієві

2271 / 6307
Чоловіку 40-ка років за вимогою діагностичних тестів зробили лімфографію органів грудної порожнини. Хірург встановив, що пухлина вразила орган, з лімфатичних судин якого лімфа безпосередньо переходить в грудну протоку. Який це орган?

Лівий головний бронх

Осердя

Стравохід

Трахея

Серце

2272 / 6307
У хірургічне відділення лікарні надійшла хвора з явищами гострого панкреатиту: блювання, пронос, сильний оперізуючий біль, слабкість, гіпотензія, зневодненням організму. Який препарат з антиферментною активністю показаний хворому?

Адреналін

Контрикал

Натрію гідрокарбонат

Анальгін

Атропіну сульфат

2273 / 6307
На препараті нирки розрізняються нефрони, які лежать на межі між кірковою та мозковою речовиною, мають однаковий діаметр приносних і виносних артеріол. Назвіть, яка функція буде порушена при їхньому пошкодженні?

Активність натрієвого рецептора

Синтез реніну

Шунтування крові при інтенсивному кровообігу

Синтез еритропоетину

Синтез простагландинів

2274 / 6307
Під час гістологічного дослідження стулок мітрального клапана серця жінки 30-ти років було встановлено, що ендотеліальні клітини вогнищево десквамовані, в цих ділянках на поверхні стулки розташовані дрібні тромботичні нашарування, сполучна тканина стулки з явищами мукоїдного набухання з ділянками склерозу та васкуляризації. Діагностуйте вид клапанного ендокардиту:

Поліпозно-виразковий

Фібропластичний

Гострий бородавчастий

Поворотньо-бородавчастий

Дифузний

2275 / 6307
Хворий переніс інсульт. Який з наведених препаратів слід включити до комплексної терапії з метою покращення кровообігу та метаболізму головного мозку?

Феназепам

Амітриптилін

Камфора

Пірацетам

Седуксен

2276 / 6307
Відзначте концентрацію етилового спирту, що має найбільш активну протимікробну дію за наявністю білку у середовищі:

70%

96%

40%

60%

15%

2277 / 6307
У хворого 40-ка років при прогресуванні стафілококового гнійного періодонтиту виникло гнійне запалення кістково-мозкових просторів альвеолярного відростка, а потім тіла нижньої щелепи. Мікроскопічно кісткові балки витончені, вогнища некрозу, кісткові секвестри, оточені сполучнотканинною капсулою. Який найбільш імовірний діагноз?

Хронічний остеомієліт

Хронічний фіброзний періостит

Гострий остеомієліт

Пародонтома

Гнійний періостит

2278 / 6307
Група чоловіків звернулася до лікаря зі скаргами на підвищення температури, головний біль, набряки повік та обличчя, біль у м’язах. З анамнезу: всі вони мисливці і часто вживають в їжу м’ясо диких тварин. Який найбільш імовірний діагноз?

Теніарінхоз

Філяріатоз

Трихінельоз

Теніоз

Цистицеркоз

2279 / 6307
Під час операції видалення матки з яєчниками і матковими трубами лікар перев’язує зв’язку, що підвішує яєчник. Які судини перев’язав лікар в цій зв’язці?

Внутрішня клубова артерія

Яєчникові артерія і вена

Маткові артерія і вена

Внутрішня клубова вена

Трубні артерія і вена

2280 / 6307
При проведенні операції на тонкій кишці лікар виявив ділянку слизової оболонки, де на фоні колових складок була присутня поздовжня складка. Який відділ тонкої кишки має таку будову?

Початковий eiddrn jejunum

Дистальний вiддiл ileum

Pars horizontalis duodeni

Pars ascendens duodeni

Pars descendens duodeni

2281 / 6307
Під час гістологічного дослідження легень хворого, що помер від серцевої недостатності, виявлені вогнища запалення з заповненням альвеол рідиною, забарвленою у блідорожевий колір, місцями з наявністю тонких рожевих ниток, що утворюють дрібнопетлисту сітку з невеликою кількістю лімфоцитів. Який характер ексудату у легенях?

Фібринозний

Серозно-фібринозний

Серозний

Гнійний

Геморагічний

2282 / 6307
У хворого нормально забарвлений кал, у складі якого з находиться велика кількість вільних жирних кислот. Причиною цього є порушення наступного процесу:

Жовчоутворення

Гідроліз жирів

Жовчовиділення

Всмоктування жирів

Секреція ліпаз

2283 / 6307
При термометрії встановлено, що температура відкритих ділянок шкіри на 1-1,5° нижче за температуру поруч розташованих ділянок, закритих одягом з натуральних тканин. Причиною цього є те, що одяг, перш за все, зменшує тепловіддачу таким шляхом:

Радіація

Конвекція

Проведення

Випаровування

2284 / 6307
У хворого діагностовано алкаптонурію. Вкажіть фермент, дефект якого є причиною цієї патології:

Фенілаланінгідроксилаза

Піруватдегідрогеназа

ДОФА-декарбоксилаза

Глутаматдегідрогеназа

Оксидаза гомогентизинової кислоти

2285 / 6307
На препараті м’якої мозкової оболонки виявляється судина, у стінці якої відсутня середня оболонка, зовнішня оболонка зрощена з оточуючою тканиною, внутрішня оболонка побудована із базальної мембрани та ендотелію. Що це за судина?

Вена волокнистого типу

Артеріола

Вена м’язового типу зі слабким розвитком м’язових елементів

Артерія змішаного типу

Артерія м’язового типу

2286 / 6307
На розтині тіла померлого від інтоксикації в тонкій кишці знайдено набряк групових лімфатичних фолікулів, виступаючих над поверхнею слизової оболонки у вигляді м’якоеластичних бляшок з нерівною поверхнею у вигляді борозен і звивин, що нагадують поверхню мозку. Який діагноз найбільш імовірний?

Холера

Черевний тиф

Дизентерія

Сальмонельоз

Гострий ентерит

2287 / 6307
При виконуванні вправ на колоді гімнастка втратила рівновагу і впала. Із збудження, перш за все, яких рецепторів розпочнуться рефлекси, що забезпечать відновлення порушеної пози?

Вестибулорецептори

Ампулярні вестибулорецептори

Пропріорецептори

Рецептори завитки

Отолітові вестибулорецептори

2288 / 6307
Під час обстеження хворої виникла підозра на наявність гнійного випоту в прямокишково-матковому заглибленні. Через яке анатомічне утворення найкраще пропунктувати дане заглиблення?

Ампула прямої кишки

Переднє склепіння піхви

Заднє склепіння піхви

Діафрагма тазу

Передня стінка піхви

2289 / 6307
Хворий 35-ти років звернувся зі скаргами на біль та набряк в ділянці дна ротової порожнини. Діагностовано запальний процес у ділянці вивідної протоки піднижньощелепної слинної залози. Куди відкривається ця протока?

Vestibulum oris

Recesus gingivalis

Caruncula sublingualis

Foramen caecum linguae

Plica fimbriata

2290 / 6307
До хірурга звернувся чоловік 60-ти років, що тривалий час хворіє на цукровий діабет. Об’єктивно: тканини правої стопи чорного кольору, щільні, з чіткими краями. Який діагноз поставив хірург?

Трофічна виразка

Суха гангрена

Газова гангрена

Пролежень

Волога гангрена

2291 / 6307
Запалення характеризується розширенням кровоносних судин на ділянці пошкодження, зменшенням кровообігу, підвищенням проникливості стінки судин. Яким з нижче наведених клітин належить головна роль в цьому?

Тканинні базофіли

Фібробласти

Плазмоцити

Еозинофіли

Макрофаги

2292 / 6307
Хворий після перенесеного епідемічного паротиту схуднув, постійно відчуває спрагу, п’є багато води, відмічає часте сечовиділення, підвищений апетит, шкірний свербіж, слабкість, фурункульоз. У крові: глюкоза - 16 ммоль/л, кетонових тіл - 100 мкмоль/л; глюкозурія. Яке захворювання розвинулось у пацієнта?

Стероїдний діабет

Інсулінонезалежний цукровий діабет

Цукровий діабет недостатнього харчування

Інсулінозалежний цукровий діабет

Нецукровий діабет

2293 / 6307
На гістологічне дослідження надіслано видалений червоподібний відросток. Розміри його збільшені, серозна оболонка тьмяна, повнокровна, вкрита плівками фібрину, стінки стовщені, на розрізі із просвіту виділяється гній. При мікроскопічному дослідженні спостерігається повнокров’я судин, набряк всіх шарів і дифузна інфільтрація їх лейкоцитами. Назвіть форму гострого апендициту:

Флегмонозний

Поверховий

Апостематозний

Простий

Гангренозний

2294 / 6307
У хворого 23-х років в результаті черепно-мозкової травми виник набряк мозку. Який механізм пошкодження клітин безпосередньо призвів до набряку мозку?

Електролітно-осмотичний

Ацидотичний

Ліпідний

Кальцієвий

Протеїновий

2295 / 6307
У хворого, який скаржився на біль у ділянці лівої лопатки, був діагностований інфаркт міокарду. Назвіть вид болю у хворого?

Фантомний

Іррадіюючий (відбитий)

Другий (епікритичний)

Перший (протопатичний)

Вісцеральний

2296 / 6307
Хвора 38-ми років надійшла до реанімаційного відділення в несвідомому стані. Рефлекси відсутні. Цукор крові -2,1 ммоль/л. В анамнезі - цукровий діабет з 18-ти років. Яка кома має місце у хворої?

Гіперосмолярна

Лактацидемічна

Гіперглікемічна

Гіпоглікемічна

Кетоацидотична

2297 / 6307
До фібрилярних елементів сполучної тканини належать колаген, еластин та ретикулін. Вкажіть амінокислоту, яка входить тільки до складу колагену і визначення якої в біологічних рідинах використовується для діагностики захворювань сполучної тканини:

Лізин

Пролін

Фенілаланін

Гліцин

Гідроксипролін

2298 / 6307
Пацієнт через 15 діб після повернення з багатомісячного плавання в районах Середземномор’я та Західної Африки відчув слабкість, головний біль, періодичні підвищення температури. Лікар запідозрив у хворого малярію. Який із перерахованих методів є найбільш адекватним в діагностиці даного захворювання?

Біологічний

Мікроскопічний

Серологічний

Мікробіологічний

Алергічний

2299 / 6307
У вагітної жінки взяли кров для підтвердження клінічного діагнозу 'токсоплазмоз'. Яка із перерахованих серологічних реакцій має діагностичне значення?

Реакція нейтралізації

Реакція гемадсорбції

Реакція гальмування гемаглютинації

Реакція аглютинації

Реакція зв’язування комплементу

2300 / 6307
У здорових батьків, спадковість яких не обтяжена, народилась дитина з чисельними вадами розвитку. Цитогенетичний аналіз виявив в соматичних клітинах дитини трисомію за 13-ю хромосомою (синдром Патау). З яким явищем пов’язане народження такої дитини?

Соматична мутація

Рецесивна мутація

Домінантна мутація

Порушення гаметогенезу

Хромосомна мутація

2301 / 6307
Для запобігання нападів гострого панкреатиту лікар призначив трасілол (контрікал, гордокс), який є інгібітором:

Трипсину

Еластази

Карбоксипептидази

Гастриксину

Хімотрипсину

2302 / 6307
Після травми на рентгенограмі постраждалого визначається перелом плеча у ділянці гребеня великого горбика. Функція якого м’яза буде у цьому випадку порушена?

Дельтоподібний

Великий грудний

Підключичний

Передній зубчастий

Малий грудний

2303 / 6307
Хворий помер від прогресуючої серцевої недостатності. На розтині серце розширене у поперечнику, мляве, м’яз на розрізі нерівномірного кровонаповнення, пістрявий, при гістологічному дослідженні у міокарді повнокров’я, у стромі лімфогістіоцитарні інфільтрати, що розсувають кардіоміоцити. Виявлені морфологічні зміни свідчать про:

Венозне повнокрів’я

Негнійний проміжний міокардит

Жирову дистрофії міокарда

Кардіосклероз

Інфаркт міокарда

2304 / 6307
В експерименті показано, що при саркомі Ієнсена споживання глюкози з привідної до пухлини артерії значно збільшується, має місце також приріст вмісту молочної кислоти у відвідній вені. Про що свідчить дане явище?

Посилення окисних процесів

Посилення анаеробного гліколізу

Зменшення окисних процесів

Посилення окиснення білків

Зменшення анаеробного гліколізу

2305 / 6307
У хворого 23-х років після перенесеної ангіни розвинувся сечовий синдром (гематурія, протеїнурія, лейкоцитурія). У пункційній біопсії нирок виявлена картина інтракапілярного проліферативного гломерулонефриту, а електронномікроскопічно виявлені великі субепітеліальні депозити. Який патогенез цього захворювання?

Імунокомплексний механізм

Клітинно обумовлений цитоліз

Грануломатоз

Атопія

Цитотоксична, цитолітична дія антитіл

2306 / 6307
У 29-річної породіллі на 3-й день після пологів виник дифузний набряк правої грудної залози, болючість при пальпації, гіперемія шкіри у вказаній ділянці, підвищення температури тіла до 38oC. При гістологічному дослідженні тканини залози виявлено: в стромі - дифузний клітинний інфільтрат, який складається з великої кількості нейтрофільних лейкоцитів, інтерстиційний набряк, гіперемія судин. Діагностуйте захворювання:

Гострий серозний мастит

Гострий флегмонозний мастит

Гострий апостематозний мастит

Хронічний гнійний мастит

Хронічний продуктивний мастит

2307 / 6307
У хворого на хронічну серцеву недостатність, незважаючи на терапію кардіотонічними засобами і тіазидовим діуретиком, зберігаються набряки і виникла загроза асциту. Який препарат слід призначити для підсилення діуретичного ефекту застосованих ліків?

Фуросемід

Манітол

Клопамід

Амілорид

Спіронолактон

2308 / 6307
До лікарні госпіталізовано хворого з підозрою на черевний тиф. Який матеріал необхідно взяти у нього з метою ранньої діагностики цього захворювання?

Кістковий мозок

Кров

Сеча

Жовч

Фекалії

2309 / 6307
У чоловіка 30-ти років перед операцією визначили групову належність крові. Кров резуспозитивна. Реакцію аглютинації еритроцитів не викликали стандартні сироватки груп 0αβ (I), Аβ (II), Вα (III). Досліджувана кров належить до групи:

АВ (IV)

Аβ (II)

Вα (III)

0αβ (I)

2310 / 6307
У людини збільшений вміст іонів кальцію в плазмі крові, зменшений - у кістках. Надмірна секреція якого гормону може спричинити такі зміни?

Трийодтиронін

Тиреокальцитонін

Паратгормон

Альдостерон

Тироксин

2311 / 6307
У студента, який складає іспит, вміст глюкози у плазмі крові складає 8 ммоль/л. Збільшена секреція якого з наведених гормонів сприяє розвитку гіперглікемії у студента?

Альдостерон

Глюкагон

Трийодтиронін

Тироксин

Інсулін

2312 / 6307
У хворого з неврологічними порушеннями діагностована пухлина головного мозку. Під час операції видалена пухлина, що має вид щільного вузла, пов’язаного з твердою мозковою оболонкою. Гістологічно пухлина побудована з ендотеліоподібних клітин, тісно прилеглих одна до одної. Який найбільш імовірний діагноз?

Менінгеальна саркома

Нейробластома

Менінгеома

Астроцитома

Гліобластома

2313 / 6307
На аутопсії жінки, що хворіла на хронічну дизентерію, при гістологічному дослідженні внутрішніх органів у стромі та паренхімі міокарда, нирок, у слизовій оболонці шлунка, у сполучній тканині легень виявлені аморфні відкладання фіолетового кольору, що дають позитивну реакцію за Коссом. Яке ускладнення розвинулось у хворої?

Метаболічне звапніння

Амілоїдоз

Метастатичне звапніння

Дистрофічне звапніння

Гіаліноз

2314 / 6307
Медсестра зі стажем роботи 10 років захворіла на контактний дерматит верхніх кінцівок. До якого типу імунної патології відноситься це захворювання?

Первинний імунодефіцит

Алергічна реакція сповільненого типу

Т-клітинний імунодефіцит

В-клітинний імунодефіцит

Алергічна реакція негайного типу

2315 / 6307
У хворого, прооперованого з приводу 'гострого живота', сеча коричневого кольору, кількість індикану в сечі вище 93 ммоль/добу. Про що це свідчить?

Збільшення інтенсивності гниття білків у кишечнику

Зниження інтенсивності знезараження амоніаку

Збільшення швидкості окисного дезамінування ароматичних амінокислот

Порушення фільтраційної здатності нирок

Зниження активності ферментів орнітинового циклу

2316 / 6307
У реанімаційному відділенні знаходиться хворий у коматозному стані. При дослідженні крові відзначено збільшення концентрації іонів K+ і зменшення - Ca++, ацидоз, збільшення рівнів сечовини, сечової кислоти. Який вид коми за етіологією найбільш імовірний?

Гіпоглікемічна

Печінкова

Ниркова

Діабетична

Нейрогенна

2317 / 6307
Жінка в період вагітності тривалий час безконтрольно приймала хіміотерапевтичний препарат. Через деякий час у неї погіршився апетит, з’явились нудота, пронос. З часом виникла жовтяниця. У новонародженого відмічено порушення росту кісток. Який препарат з групи тетрацикліну приймала жінка?

Азитроміцин

Бензилпеніциліну натрiєва сіль

Бісєптол

Доксицикліну гідрохлорид

Ципрофлоксацин

2318 / 6307
Під час розтину тіла дитини, яка померла при ознаках асфіксії, були виявлені в трахеї і головних бронхах сіруватого кольору плівки, які вільно лежали у просвіті дихальних шляхів, нагадуючи їх зліпки. Вкажіть вид запалення:

Дифтеритичне

Серозне

Крупозне

Гнійне

Катаральне

2319 / 6307
Чоловік 60-ти років скаржиться на біль у суглобах. У сироватці крові пацієнта виявлено підвищення концентрації С-реактивного білку та оксипроліну. Для якого захворювання характерні ці симптоми?

Гепатит

Ревматизм

Цукровий діабет

Жовтяниця

Подагра

2320 / 6307
До офтальмолога звернувся пацієнт зі скаргами на різі в очах. При обстеженні встановлена ерозія рогівки - відсутність поверхневого і шипуватого шарів епітелію. Які клітини будуть забезпечувати регенерацію ушкодженого епітелію?

Клітини рогового шару

Базальні

Клітини блискучого шару

Клітини зернистого шару

Клітини поверхневого шару

2321 / 6307
До інфекційного відділення госпіталізували хворого з ознаками загальної слабкості, сильними головними і м’язовими болями, високою температурою, гіперемією обличчя. Встановлено, що тиждень тому хворий відпочивав біля озера. Лікар запідозрив лептоспіроз. Яким чином лептоспіри могли потрапити до організму хворого?

З їжею

Через предмети вжитку

З ґрунтом

З повітрям

З водою

2322 / 6307
У дорослої людини системний артеріальний тиск знизився з 120/70 до 90/50 мм рт.ст., що викликало рефлекторне звуження судин. У якому з зазначених органів звуження судин буде найменшим?

Кишечник

Печінка

Шкіра

Скелетні м’язи

Серце

2323 / 6307
Під час хірургічного втручання на тонкій кишці у людини можлива рефлекторна зупинка серця. Які рецептори в міокарді необхідно заблокувати, щоб попередити зупинку?

α-адренорецептори

β-адренорецептори

H-холінорецептори

M-холінорецептори

Пуринові рецептори

2324 / 6307
У лікарню наприкінці робочого дня доставлений робітник 'гарячого' цеху, який скаржиться на головний біль, запаморочення, нудоту, загальну слабкість. Об’єктивно: свідомість збережена, шкірні покриви гіперемовані, сухі, гарячі на дотик. ЧСС- 130/хв. Дихання часте, поверхневе. Яке порушення процесів терморегуляції, найбільш імовірно, виникло у людини в даній ситуації?

Посилення тепловіддачі і зниження теплопродукції

Посилення тепловіддачі і теплопродукції

Зниження тепловіддачі

Посилення теплопродукції без змін тепловіддачі

Зниження теплопродукції без змін тепловіддачі

2325 / 6307
Експериментальне вивчення нового медичного препарату виявило блокуючий ефект на збирання білків-тубулінів, які є основою веретена поділу в клітинах, що діляться. Який етап клітинного циклу порушується цим препаратом?

Анафаза мітозу

Премітотичний період інтерфази

Телофаза мітозу

Синтетичний період

Постмітотичний період інтерфази

2326 / 6307
У чоловіка 35-ти років під час тривалого бігу виникла гостра серцева недостатність. Які зміни іонного складу спостерігаються у серцевому м’язі при цьому стані?

Накопичення в клітинах міокарда іонів Na+ і Ca2+

Зменшення в позаклітинному просторі іонів K+ і Mg2+

Зменшення в клітинах міокарда іонів Na+ і Ca2+

Накопичення в клітинах міокарда іонів K+ і Mg2+

Збільшення в позаклітинному просторі іонів Na+ і Ca2+

2327 / 6307
Під час аналізу ЕКГ людини з’ясовано, що у другому стандартному відведенні від кінцівок зубці P позитивні, їхня амплітуда 0,1 mV (норма - 0,05-0,25 mV), тривалість - 0,1 с (норма - 0,07-0,10 с). Вірним є висновок, що у передсердях нормально відбувається процес:

Збудження

Скорочення

Реполяризації

Розслаблення

Деполяризації

2328 / 6307
У юнака 18-ти років діагностовано хворобу Марфана. При дослідженні встановлено: порушення розвитку сполучної тканини, будови кришталика ока, аномалії серцевосудинної системи, арахнодактилія. Яке генетичне явище зумовило розвиток цієї хвороби?

Комплементарність

Кодомінування

Множинний алелізм

Неповне домінування

Плейотропія

2329 / 6307
Інозитолтрифосфати в тканинах організму утворюються в результаті гідролізу фосфатидилінозитолдифосфатів і відіграють роль вторинних посередників (месенджерів) в механізмі дії гормонів. Їхній вплив у клітині спрямований на:

Активацію аденілатциклази

Гальмування фосфодіестерази

Вивільнення іонів кальцію з клітинних депо

Активацію протеїнкінази А

Гальмування протеїнкінази С

2330 / 6307
Батьки дитини 3-х років звернули увагу на потемніння кольору його сечі при відстоюванні. Об’єктивно: температура у нормі, шкірні покриви чисті, рожеві, печінка не збільшена. Назвіть імовірну причину даного стану:

Гемоліз

Подагра

Фенілкетонурія

Алкаптонурія

Синдром Іценка-Кушінга

2331 / 6307
Хворий звернувся до лікаря зі скаргами на гнійничкові висипання на шкірі кінцівок. Який антисептик необхідно призначити хворому?

Преднізолон

Інсулін

Розчин йоду спиртовий

Гепарин

Сибазон

2332 / 6307
Після тривалого фізичного навантаження під час заняття з фізичної культури у студентів розвинулась м’язова крепатура. Причиною її виникнення стало накопичення у скелетних м’язах молочної кислоти. Вона утворилась після активації в організмі студентів:

Пентозофосфатного циклу

Ліполізу

Глюконеогенезу

Гліколізу

Глікогенезу

2333 / 6307
Під дією негативних чинників довкілля порушена функція міосателітоцитів. Зміну якої функції всього м’язового волокна слід очікувати в даному випадку?

Трофіка

Скорочення

Розслаблення

Скоротливий термогенез

Регенерація

2334 / 6307
У хворого сечокам’яна хвороба. При видаленні конкременту з правого сечоводу хірург розрізав стінку сечоводу. В яке анатомічне утворення потрапить сеча?

Заочеревинний простір

Правий брижовий синус

Правий бічний канал

Прямокишково-міхурове заглиблення

Лівий бічний канал

2335 / 6307
У дитячому колективі проведено планову вакцинацію проти кору. Яким методом можна перевірити ефективність проведеної вакцинації?

Вірусологічний

Серологічний

Вірусоскопічний

Біологічний

Алергопроба

2336 / 6307
У тварини збільшений тонус м’язів-розгиначів. Це є наслідком посиленої передачі інформації до мотонейронів спинного мозку такими низхідними шляхами:

Ретикулоспінальні

Вестибулоспінальні

Латеральні кортикоспінальні

Медіальні кортикоспінальні

Руброспінальні

2337 / 6307
Хвора 28-ми років потрапила до інфекційної лікарні з приводу пожовтіння шкіри, склер, слизових оболонок. Лабораторно встановлене підвищення рівня прямого білірубіну у крові. В сечі виявлений уробіліноген і білірубін. Для якого з перелічених захворювань характерні такі зміни?

Туберкульоз нирки

Механічна жовтяниця

Гемолітична жовтяниця

Паренхіматозна жовтяниця

Інфаркт нирки

2338 / 6307
У новонародженої дитини спостерігаються: судоми, блювання, жовтяниця, специфічний запах сечі. Лікар-генетик висловив підозру про спадкову хворобу обміну речовин. Який метод дослідження необхідно використати для постановки точного діагнозу?

Цитогенетичний

Біохімічний

Дерматогліфіка

Близнюковий

Популяційно-статистичний

2339 / 6307
У дитини спостерігається затримка фізичного та розумового розвитку, глибокі порушення з боку сполучної тканини внутрішніх органів; у сечі виявлено кератансульфати. Обмін яких речовин порушений?

Гіалуронова кислота

Фібронектин

Еластин

Колаген

Глікозаміноглікани

2340 / 6307
У деяких анаеробних бактерій піруват, що утворюється внаслідок гліколізу, перетворюється на етиловий спирт (спиртове бродіння). У чому біологічний сенс цього процесу?

Утворення АТФ

Утворення АДФ

Утворення лактату

Поповнення фонду НАД+

Забезпечення клітини НАДФН

2341 / 6307
У хворого, який довготривало приймав преднізолон, в результаті відміни препарату виникло загострення захворювання, зниження артеріального тиску, слабкість. З чим можна пов’язати ці прояви?

Звикання до препарату

Кумуляція препарату

Виникнення недостатності кори наднирників

Сенсибілізація до препарату

Гіперпродукція АКТГ

2342 / 6307
В експерименті необхідно оцінити рівень збудливості тканини. Для цього доцільно визначити:

Амплітуду ПД

Поріг деполяризації

Критичний рівень деполяризації

Потенціал спокою

Тривалість ПД

2343 / 6307
Працівник соціальної служби після тривалого спілкування з людиною без визначеного місця проживання захворів на туберкульоз. До складу комплексного лікування було включено напівсинтетичний антибіотик широкого спектру дії. Вкажіть препарат:

Рифампіцин

Ампіцилін

Цефотаксим

Лінкоміцин

Еритроміцин

2344 / 6307
При хворобі Вільсона-Коновалова порушується транспорт міді, що призводить до накопичення цього металу в клітинах мозку та печінки. З порушенням синтезу якого білку це пов’язано?

Гаптоглобін

Металотіонеїн

Сидерофілін

Транскобаламін

Церулоплазмін

2345 / 6307
Хворий впродовж трьох років безрезультатно лікувався з приводу значного зниження кислотності шлункового соку. Його пригнічувала поява на білизні, постелі члеників, що рухались і самостійно виповзали з анального отвору. Який найбільш імовірний діагноз?

Теніаринхоз

Цистицеркоз

Опісторхоз

Теніоз

Гіменолепідоз

2346 / 6307
При нестачі біотину спостерігається порушення синтезу вищих жирних кислот. Утворення якого із зазначених метаболітів може бути порушено при цьому?

Сукциніл КоА

Серотонін

Аланін

Малоніл КоА

Піруват

2347 / 6307
У чоловіка 60-ти років після інсульту настав тривалий сон. Ураження яких структур ЦНС найбільш імовірно призвело до цього стану?

Чорна субстанція

Висхідна частина РФ

Прецентральна звивина

V-IX пари черепних нервів

Мозочок

2348 / 6307
У чоловіка виявлене захворювання, яке зумовлене домінантним геном, локалізованим у Ххромосомі. У кого із дітей буде це захворювання, якщо дружина здорова?

Тільки у дочок

Тільки у синів

У половини синів

У половини дочок

У всіх дітей

2349 / 6307
Недостатність в організмі мікроелементу селену проявляється кардіоміопатією. Імовірною причиною такого стану є зниження активності такого селенвмісного ферменту:

Лактатдегідрогеназа

Цитохромоксидаза

Каталаза

Сукцинатдегідрогеназа

Глутатіонпероксидаза

2350 / 6307
Недостатність в організмі лінолевої та ліноленової кислот призводить до ушкоджень шкіри, випадіння волосся, сповільненого загоювання ран, тромбоцитопенії, зниження опірності до інфекційних захворювань. Порушення синтезу яких речовин найімовірніше зумовлює вказані симптоми?

Кортикостероїди

Інтерферони

Ейкозаноїди

Інтерлейкіни

Катехоламіни

2351 / 6307
Людина потрапила у крижану воду й швидко загинула в результаті різкого переохолодження. Це відбулося тому, що в даному випадку значно збільшилась віддача тепла організмом таким шляхом:

Конвекція

Теплопроведення

Радіація

Теплопроведення і радіація

2352 / 6307
Хворий 39-ти років з алкогольним цирозом печінки скаржиться на задишку, загальну слабкість. Встановлено зниження артеріального тиску, розширення поверхневих вен передньої стінки живота, спленомегалію. Яке порушення гемодинаміки спостерігається у хворого?

Синдром портальної гіпертензії

Недостатність правого шлуночка серця

Тотальна серцева недостатність

Колапс

Недостатність лівого шлуночка серця

2353 / 6307
У жінки, що тривало приймала антибіотики з приводу кишкової інфекції, розвинулось ускладнення з боку слизової порожнини рота у вигляді запального процесу і білого нальоту, у якому під час бактеріологічного дослідження були виявлені дріжджеподібні грибки Candida albicans. Який з перерахованих препаратів показаний для лікування цього ускладнення?

Поліміксин

Тетрациклін

Флуконазол

Бісептол

Фуразолідон

2354 / 6307
До лікаря акушера-гінеколога звернулась вагітна жінка, у якої діагностували мегалобластну анемію. Який з нижченаведених засобів доцільно призначити?

Метилурацил

Ціанокобаламін

Ілауцин

Пентоксил

Стрептокіназа

2355 / 6307
Хвора 45-ти років звернулась із скаргами на облисіння. При огляді: шкіра голови плямисто-коричневого кольору, малорухлива, тоненька, щільна, вкрита роговими лусочками. При гістологічному дослідженні - гіперкератоз, в дермі склероз, периваскулярні лімфомакрофагальні інфільтрати, атрофія потових та сальних залоз. У крові виявлені LE-клітини. Який найбільш імовірний діагноз?

Системна склеродермія

Вузликовий періартеріїт

Ревматоїдний артрит

Ревматизм

Системний червоний вовчак

2356 / 6307
При дослідженні гостроти слуху в коваля виявили втрату слуху на 50% у діапазоні низьких частот і майже нормальну гостроту слуху в діапазоні високих частот. Порушення яких структур слухової системи призвело до такого стану?

Кортієв орган - ближче до гелікотреми

Барабанна перетинка

Середня частина кортієвого органу

М’язи середнього вуха

Кортієв орган - ближче до овального віконця

2357 / 6307
При відборі для ревакцинації вакциною БЦЖ у школяра поставлено пробу Манту, яка виявилася негативною. Результат проби свідчить про такі особливості імунітету до туберкульозу:

Відсутність клітинного імунітету

Відсутність антитоксичного імунітету

Наявність клітинного імунітету

Відсутність гуморального імунітету

Наявність гуморального імунітету

2358 / 6307
Чоловік звернувся до лікаря з приводу безпліддя. Має високий зріст, зниження інтелекту, недорозвинення статевих залоз. У епітелії слизової оболонки порожнини рота виявлений статевий хроматин (1 тільце Барра). Про яку патологію можна думати?

Акромегалія

Синдром Клайнфельтера

Синдром Іценка-Кушинга

Синдром Ді-Джорджи

Адреногенітальний синдром

2359 / 6307
Який стан може розвинутися через 15-30 хвилин після повторного введення антигену внаслідок підвищеного рівня антитіл, переважно IgE, які адсорбуються на поверхні клітин-мішеней - тканинних базофілів (тучних клітин) та базофілів крові?

Анафілаксія

Сироваткова хвороба

Гіперчутливість уповільненого типу

Імунно-комплексна гіперчутливість

Антитіло-залежна цитотоксичність

2360 / 6307
У жінки 22-х років через 5 годин після вживання морепродуктів на шкірі тулуба та дистальних відділів кінцівок з’явились маленькі сверблячі папули, які частиною зливаються між собою. Через добу висипка самовільно зникла. Назвіть механізм гіперчутливості, що полягає в основі даних змін:

Клітинна цитотоксичність

Антитілоопосередкований клітинний цитоліз

Імунокомплексна гіперчутливість

Атопія (місцева анафілаксія)

Системна анафілаксія

2361 / 6307
Хворий на гіпертонічну хворобу разом з безсольовою дієтою та з антигіпертензивними засобами, довгий час приймав гідрохлортіазид, що зумовило порушення електролітного балансу. Яке порушення внутрішнього середовища виникло у хворого?

Гiперкалiємiя

Гiпермагнiємiя

Гіпохлоремічний алкалоз

Збільшення об’єму циркулюючої крові

Метаболічний ацидоз

2362 / 6307
У 49-річної жінки після видалення лімфатичних вузлів правої аксилярної ділянки з приводу раку грудної залози через півроку виявлено збільшення правої верхньої кінцівки в об’ємі та значне її ущільнення, гладка та напружена шкіра, через щілиноподібні дефекти якої на поверхню витікає прозора рідина. Діагностуйте вид порушення лімфообігу:

Гостра місцева лімфедема

Хронічна вроджена місцева лімфедема

Хронічна набута місцева лімфедема

Гостра загальна лімфедема

Хронічна загальна лімфедема

2363 / 6307
У сироватці хворого виявлений імуноферментним методом HBsAg. При якому захворюванні виявлення даного ангигену має діагностичне значення?

Сказ

Вірусний гепатит В

ВІЛ

Кір

Натуральна віспа

2364 / 6307
При повторному введенні алергену починається виділення гістаміну тучними клітинами крові. До якого рівня реактивності відноситься така відповідь організму?

Клітинний

Молекулярний

Органний

Системний

Субклітинний

2365 / 6307
В експерименті на кролику введення пірогеналу призвело до підвищення у тварини температури тіла. Яка з перерахованих речовин відіграє роль вторинного пірогену, що бере участь у механізмі виникнення лихоманкової реакції?

Піромен

Брадикінін

Імуноглобулін

Гістамін

Інтерлейкін-1

2366 / 6307
При авторадіографічному дослідженні епітелію тонкої кишки було виявлено, що його повне оновлення відбувається протягом 3-х діб за рахунок проліферації малодиференційованих клітин. Вкажіть їх локалізацію:

Верхівка ворсинок

Основа ворсинок

Бічна поверхня ворсинок

Дно крипт

Власна пластинка слизової оболонки

2367 / 6307
Лікар при дослідженні мазку крові у пацієнта з анемією встановив діагноз - спадкова гемолітична анемія Мінковського-Шофара. Виявлення у крові яких характерних клітин надало можливість лікарю встановити діагноз?

Мегалоцити

Поліхроматофіли

Мікросфероцити

Анізоцити

Пойкілоцити

2368 / 6307
У хворого з клінічними симптомами гіпотиреозу, щитоподібна залоза збільшена удвічі, при пальпації щільна, з горбистою поверхнею. При гістологічному дослідженні - поряд з атрофією фолікулів залози відмічається дифузна інфільтрація паренхіми лімфоцитами, плазматичними клітинами з утворенням фолікулів і посилене розростання сполучної тканини. Вкажіть найбільш імовірний діагноз:

Зоб Хашімото

Ендемічний зоб

Фіброзний зоб

Спорадичний зоб

Дифузний токсичний зоб

2369 / 6307
При загостренні ревматоїдного артриту хворому, в анамнезі якого супутній хронічний гастрит, призначений целекоксиб. Чим обумовлено зменшення побічної дії препарату на травний тракт?

Пригнічення фосфодіестерази

Переважаюче пригнічення циклооксигенази-2

Переважаюче пригнічення циклооксигенази-1

Переважаюча стимуляція аденіла-тциклази

Пригнічення фосфоліпази А2

2370 / 6307
В експерименті на кролі встановлено, що об’єм кисню, який споживається головним мозком за 1 хвилину, дорівнює об’єму CO2 , який виділяється клітинами мозку в кров. Це свідчить, що у клітинах головного мозку має місце:

Окиснення білків

Гіпокапнія

Гіпоксія

Окиснення жирів

Окиснення вуглеводів

2371 / 6307
При активації запального процесу, деяких аутоімунних та інфекційних захворюваннях у плазмі крові різко зростає рівень білків гострої фази. Який із наведених нижче білків здатний утворювати гель при охолодженні сироватки?

а2-макроглобін

Церулоплазмін

С-реактивний білок

Гаптоглобін

Кріоглобулін

2372 / 6307
Хворому поставлено діагноз газова гангрена. Після ідентифікації збудника досліджуваний матеріал необхідно знищити. Який метод слід використати?

Тиндалізація

Стерилізація текучою парою

Пастеризація

Кип’ятіння

Стерилізація парою під тиском

2373 / 6307
Фенілкетонурія - це захворювання, яке зумовлено рецесивним геном, що локалізується в аутосомі. Батьки є гетерозиготами за цим геном. Вони вже мають двох хворих синів і одну здорову доньку. Яка імовірність, що четверта дитина, яку вони очікують, народиться теж хворою?

50%

25%

100%

0%

75%

2374 / 6307
У пацієнта з підвищеним артеріальним тиском, тремором, тахікардією, була діагностовано доброякісна пухлина мозкової речовини наднирників. Гіперсекреція якого гормону викликає таку симптоматику?

Тироксин

Адреналін

Глюкагон

Соматотропін

Інсулін

2375 / 6307
В пробірку, що містить розчин NaCl 0,9%, додали краплю крові. Що відбудеться з еритроцитами?

Біологічний гемоліз

Осмотичний гемоліз

Зморшкування

Залишаться без змін

Набухання

2376 / 6307
До косметолога звернулася пацієнтка зі скаргами на появу чорних цяток на обличчі. Після обстеження було встановлено, що поява цяток пов’язана з порушенням виділення секрету сальних залоз. Який тип секреції характерний для цих залоз?

Мерокриновий

Макроапокриновий

Голокриновий

Мерокриновий та мікроапокриновий

Мікроапокриновий

2377 / 6307
У пацієнта з хронічним захворюванням нирок розвинулась ниркова недостатність. Який з показників найбільш імовірно свідчить про порушення реабсорбції в канальцях в даному випадку?

Гіперазотемія

Гіпо- та ізостенурія

Лейкоцитурія

Гематурія

Зниження кліренсу

2378 / 6307
У людей, які постійно проживають в гірській місцевості, адаптація до 'кисневого голодування' здійсню ється шляхом полегшеної віддачі кисню гемоглобіном внаслідок:

Підвищеного утворення 2,3- дифосфогліцерату в еритроцитах

Зниженого утворення 2,3- дифосфогліцерату в еритроцитах

Підвищення pH крові

Зниження температури крові

Зростання парціального тиску CO2

2379 / 6307
У хворого 40-ка років ознаки гірської хвороби: запаморочення, задишка, тахікардія, рН крові - 7,50, pCO2 -30 мм рт.ст., зсув буферних основ +4 ммоль/л. Яке порушення кислотноосновного стану має місце?

Газовий алкалоз

Видільний ацидоз

Негазовий ацидоз

Негазовий алкалоз

Газовий ацидоз

2380 / 6307
Після ремонту автомобіля в закритому приміщенні при працюючому двигуні у чоловіка з’явилися задишка, запаморочення, акроціаноз, частота дихання 24-26/хв. Газовий склад крові: pO2 - 60 мм рт.ст., pCO2 - 30 мм рт.ст.; у крові наявний карбоксигемоглобін. Про який вид гіпоксії можна думати?

Тканинна

Гіпоксична

Гемічна

Циркуляторна

Респіраторна

2381 / 6307
При обстеженні у хворого виявлене порушення чутливості шкіри в ділянці передньої поверхні шиї. Який нерв уражений?

Малий потиличний

Надключичні

Великий вушний

Поперечний нерв шиї

Шийна петля

2382 / 6307
У вагітної жінки 26-ти років після тривалого блювання було зареєстровано зниження об’єму циркулюючої крові. Про яку зміну загальної кількості крові може йти мова?

Олігоцитемічна гіперволемія

Олігоцитемічна гіповолемія

Проста гіповолемія

Поліцитемічна гіповолемія

Поліцитемічна гіперволемія

2383 / 6307
У хворої дитини гінгівіт, спричинений анаеробною інфекцією. Яку групу протимікробних засобів потрібно призначити для лікування?

Аміноглікозиди

Поліміксини

Нітрофурани

Нітроімідазоли

Сульфаніламіди

2384 / 6307
Пацієнту, який знаходився в клініці з приводу пневмонії, ускладненої плевритом, у складі комплексної терапії вводили преднізолон. Протизапальна дія цього синтетичного глюкокортикоїда пов’язана з блокуванням вивільнення арахідонової кислоти шляхом гальмування такого ферменту:

Циклооксигеназа

Пероксидаза

Фосфоліпаза А2

Ліпоксигеназа

Фосфоліпаза C

2385 / 6307
Встановлено, що в клітинах організмів відсутні мембранні органели та їх спадковий матеріал не має нуклеосомної організації. Що це за організми?

Еукаріоти

Віруси

Прокаріоти

Найпростіші

Аскоміцети

2386 / 6307
У постраждалого в автомобільній аварії припинилося грудне дихання при збереженні діафрагмального. На якому рівні найбільш імовірно пошкоджено спинний мозок?

VI-VII шийні сегменти

I-II поперекові сегменти

XI-XII грудні сегменти

I-II крижові сегменти

I-II шийні сегменти

2387 / 6307
Хлопчик на другому році життя став часто хворіти на респіраторні захворювання, стоматити, гнійничкові ураження шкіри. Навіть невеликі пошкодження ясен і слизової ускладнюються запаленням, що протікає тривало. Встановлено, що у крові дитини практично відсутні імуноглобуліни усіх класів. Зниження функціональної активності якої клітинної популяції лежить в основі описаного синдрому?

Нейтрофіли

Т-лімфоцити

NK-лімфоцити

В-лімфоцити

Макрофаги

2388 / 6307
На електронній фотографії представлена органела, що являє собою великий поліпротеазний комплекс, що складається з трубкоподібної та двох регуляторних частин, які розташовані на обох кінцях органели. Остання виконує функцію протеолізу. Назвіть цю органелу:

Рибосома

Включення

Протеасома

Комплекс Гольджі

Центріоль

2389 / 6307
У хворого після перенесеної черепно-мозкової травми порушений акт ковтання. Який відділ мозку постраждав?

Таламус

Проміжний мозок

Середній мозок

Кінцевий мозок

Довгастий мозок

2390 / 6307
Дитина 6-ти років знаходиться на стаціонарному лікуванні з діагнозом алергічного риніту. В крові: зміни в лейкоцитарній формулі. Кількість яких клітин лейкоцитарного ряду може бути збільшена?

Нейтрофіли

В-лімфоцити

Еозинофіли

Т-лімфоцити

Базофіли

2391 / 6307
У хворого 69-ти років на шкірі в ділянці нижньої повіки з’явилося невелике бляшкоподібне утворення з наступним виразкуванням, яке було оперативно видалене. При мікроскопічному дослідженні утворення: в дермі шкіри комплекси з атипових епітеліальних клітин; периферії клітини розташовані перпендикулярно до базальної мембрани. Клітини темні, призматичної полігональної форми з гіперхромними ядрами з частими мітозами. Іноді зустрічаються утворення, подібні до волосяного фолікула. Яка гістологічна форма рака у хворого?

Недиференційований рак

Плоскоклітинний рак з ороговінням

Базально-клітинний рак

Аденокарцинома

Плоскоклітинний рак без ороговіння

2392 / 6307
Анатомічний мертвий простір - це частина повітря, яка залишається в повітроносних шляхах після видиху. В якій із наведених нижче ситуацій відбудеться зменшення анатомічного мертвого простору?

Поворот лежачого пацієнта на правий бік

Дихання через рот

Нахил голови вперед

Накладання трахеостоми

Поворот лежачого пацієнта на лівий бік

2393 / 6307
До кардіологічного відділення надійшов хворий з інтенсивним болем, який зумовлений інфарктом міокарда. Для купірування болю було вирішено потенціювати дію анальгетика нейролептиком. Який з перерахованих нейролептиків найбільш придатний у даному випадку?

Сульпірид

Галоперидол

Аміназин

Трифтазин

Дроперидол

2394 / 6307
У 12-річної дитини непереносимість ряду харчових продуктів. Їх вживання викликає алергічну реакцію у вигляді висипань на шкірі, що сверблять. Який протигістамінний засіб слід призначити, щоб не заважати шкільним заняттям дитини?

Еуфілін

Лоратадин

Димедрол

Диклофенак

Ефедрин

2395 / 6307
У хворого на слизовій оболонці ясен виразка овальної форми з припіднятими краями хрящоподібної щільності. Дно виразки м’ясисто-червоного забарвлення з нашаруваннями сірого кольору. При мікроскопічному дослідженні - проліферація ендотелію дрібних судин, периваскулярна лімфоплазмоцитарна інфільтрація. Про яке захворювання йдеться?

Травматична виразка

Ерозивно-виразкова лейкоплакія

Виразка-рак

Сифіліс

Виразково-некротичний гінгівіт

2396 / 6307
Стоматолог призначив пацієнту препарати кальцію для профілактики карієсу. Лікар не знав, що пацієнт хворіє на хронічну серцеву недостатність і приймає серцеві глікозиди. Що відбудеться в результаті поєднання препаратів кальцію і серцевих глікозидів?

Уповільниться метаболізм серцевих глікозидів

Знизиться активність і токсичність серцевих глікозидів

Прискориться виведення серцевих глікозидів

Препарати не взаємодіють

Збільшиться активність і токсичність серцевих глікозидів

2397 / 6307
Тривале лікування гіпофункції щитоподібної залози спричинило загальну дистрофію, карієс зубів, тахікардію, тремор кінцівок. Який лікарський засіб викликав зазначені побічні ефекти?

Тирокальцитонін

Преднізолон

Паратиреоїдин

L-тироксин

Хумулін

2398 / 6307
У жінки встановлено діагноз - рак шийки матки. З яким вірусом може бути асоційована ця патологія?

Varicella-Zoster вірус

Цитомегаловірус

Аренавірус

Папілома вірус

Вірус простого герпеса тип 2

2399 / 6307
У хворого хлопчика 12-ти років вміст холестерину в сироватці крові до 25 ммоль/л. В анамнезі - спадкова сімейна гіперхолестеринемія, причиною якої є порушення синтезу білків-рецепторів до:

Ліпопротеїнів високої щільності

Хіломікронів

Ліпопротеїнів дуже низької щільності

Ліпопротеїнів проміжної щільності

Ліпопротеїнів низької щільності

2400 / 6307
При дослідженні зовнішнього дихання лікар попросив пацієнта здійснити максимально глибокий видих після максимально глибокого вдиху для визначення такого показника:

Життєва ємність легень

Загальна ємність легень

Киснева ємність крові

Функціональна залишкова ємність

Резервний об’єм видиху

2401 / 6307
Хворий 47-ми років з діагнозом вогнищевий туберкульоз верхньої долі правої лєгені, в скиді комбінованої терапії одержує ізоніазид. Через деякий час пацієнт почав пред’являти скарги на м’язову слабкість, зниження шкірної чутливості, порушення зору, координації рухів. Який вітамінний препарат доцільно використати для усунення даних явищ?

Вітамін В12

Вітамін В6

Вітамін A

Вітамін C

Вітамін D

2402 / 6307
Хворий 58-ми років впродовж 9-ти років хворіє на цукровий діабет, отримує для корекції гіперглікемії інсулін-семіленте. 10 днів тому почав лікування гіпертонічної хвороби анаприліном. Через годину після прийому антигіпертензивного препарату розвинулась гіпоглікемічна кома. Який механізм виникнення гіпоглікемії за умови прийому анаприліну?

Пригнічення глікогенолізу

Збільшення періоду напіввиведення інсуліну-семіленте

Зменшення періоду напіввиведення глюкагону

Збільшення біодоступності інсуліну-семіленте

Зменшення всмоктування глюкози

2403 / 6307
Похідні птерину (аміноптерин і метотрексат) - є конкурентними інгібіторами дигідрофолатредуктази, внаслідок чого вони пригнічують регенерацію тетрагідрофолієвої кислоти з дигідрофолату. Ці лікарські засоби призводять до гальмування міжмолекулярного транспорту одновуглецевих груп. Біосинтез якого полімеру при цьому пригнічується?

Білок

Глікозаміноглікани

Гангліозиди

ДНК

Гомополісахариди

2404 / 6307
При підозрі на туберкульоз хворій дитині зробили пробу Манту Через 24 години у місці введення алергену з’явились припухлість, гіперемія і болісність. Які основні компоненти визначають цю реакцію організму?

Мононуклеари, Т-лімфоцити і лімфокіни

В-лімфоцити, IgM

Гранулоцити, Т-лімфоцити і IgG

Макрофаги, В-лімфоцити і моноцити

Плазматичні клітини, Т-лімфоцити і лімфокіни

2405 / 6307
Біохімічний аналіз сироватки крові пацієнта з гепатолентикулярною дегенерацією (хвороба Вільсона-Коновалова) виявив зниження вмісту церулоплазміну. У цього пацієнта в сироватці крові буде підвищена концентрація таких іонів:

Кальцій

Мідь

Калій

Фосфор

Натрій

2406 / 6307
В процесі метаболізму в організмі людини виникають активні форми кисню, у тому числі супероксидний аніон-радикал О2 . Цей аніон інактивується за допомогою ферменту:

Глутатіонпероксидаза

Супероксиддисмутаза

Пероксидаза

Каталаза

Глутатіонредуктаза

2407 / 6307
Хлопчик 10-ти років знаходиться у лікарні з підозрою на харчову токсикоінфекцію. При посіві фекалій хворого на середовище Ендо виросла велика кількість безбарвних колоній. Який мікроорганізм можна з найбільшою імовірністю ВИКЛЮЧИТИ з числа можливих збудників захворювання?

Escherichia coli

Yersinia enterocolitica

Pseudomonas aeruginosa

Salmonella enteritidis

Proteus vulgaris

2408 / 6307
Чоловік 23-х років після ДТП надійшов до лікарні у важкому стані із черепно-мозковою травмою. Дихання характеризується судомним тривалим вдихом, який переривається коротким видихом. Для якого типу дихання це характерно?

Апнейстичне

Кусмауля

Гаспінг-дихання

Біота

Чейн-Стокса

2409 / 6307
У хлопчика 2-х років з вираженим геморагічним синдромом відсутній антигемофільний глобулін А (фактор VIII) у плазмi крові. Яка фаза гемостазу первинно порушена у цього хворого?

Ретракція кров’яного згустку

Перетворення протромбіну в тромбін

Внутрішній механізм активації протромбінази

Перетворення фібриногену в фібрин

Зовнішній механізм активації протромбінази

2410 / 6307
Щуру в плевральну порожнину введено 0,5 мл повітря. Який тип недостатності дихання виникає в даному випадку?

Дисрегуляторне порушення альвеолярної вентиляції

Обструктивне порушення альвеолярної вентиляції

Дифузійний

Перфузійний

Рестриктивне порушення альвеолярної вентиляції

2411 / 6307
У хворого виявлено протозойне захворювання, при якому вражений головний мозок і спостерігається втрата зору. У крові знайдені одноклітинні півмісяцевої форми з загостреним кінцем. Збудником цього захворювання є:

Токсоплазма

Лямблія

Амеба

Трихомонада

Лейшманія

2412 / 6307
Юнак 17-ти років звернувся до медико-генетичної консультації з приводу відхилень у фізичному і статевому розвитку. При мікроскопії клітин слизової оболонки рота виявлене одне тільце Барра. Вкажіть найбільш імовірний каріотип юнака:

47, XYY

47,18+

45, Х0

47, XXY

47,21+

2413 / 6307
У спортсмена внаслідок довільної затримки дихання на 40 секунд зросли частота серцевих скорочень та системний артеріальний тиск. Реалізація яких механізмів регуляції зумовлює зміни показників?

Умовні симпатичні рефлекси

Безумовні парасимпатичні рефлекси

Умовні парасимпатичні рефлекси

Безумовні симпатичні рефлекси

2414 / 6307
Жінці 54-х років поставили попередній діагноз: інфаркт міокарда. Характерною ознакою даного захворювання є суттєве підвищення в крові активності такого ферменту:

Каталаза

Креатинфосфокіназа

Г-6-ФДГ

Аргіназа

Альфа-амілаза

2415 / 6307
Дитина попросила батька надути гумову кульку якомога більше за один видих. Яким з перелічених об’ємів повітря скористується батько?

Ємність вдиху

Функціональна залишкова ємність

Життєва ємність легень

Резервний об’єм вдиху

Загальна ємність легень

2416 / 6307
На розтині тіла хворого 43-х років, померлого від ревматизму, поверхня епікарду ворсиста, покрита плівками сірого кольору, що легко відділяються. Після їх відділення визначається набрякла повнокровна поверхня епікарду. Який найбільш імовірний діагноз?

Гнійний перикардит

Проліферативний перикардит

Фібринозний перикардит

Катаральний перикардит

Геморагічний перикардит

2417 / 6307
При розтині тіла померлого виявлена гіперплазія кісткового мозку плоских і трубчастих кісток (піоїдний кістковий мозок), спленомегалія (6 кг), гепатомегалія (5 кг), збільшення всіх груп лімфатичних вузлів. Якому захворюванню відповідають виявлені зміни?

Хронічний лімфолейкоз

Справжня поліцитемія

Мієломна хвороба

Хронічний мієлолейкоз

Лімфогрануломатоз

2418 / 6307
Хворий 61-го року страждає на цироз печінки. Варикозні розширення яких вен з системи порто-кавальних анастомозів можуть мати місце у хворого?

V subcostalis

V. circumflexa ilium profunda

V femoralis

V epigastrica superficialis

Vv. intercostales posteriores

2419 / 6307
Електрофоретичне дослідження сироватки крові хворого на пневмонію показало збільшення однієї з білкових фракцій. Вкажіть її:

α2 -глобуліни

Альбуміни

β-глобуліни

γ-глобуліни

α1 -глобуліни

2420 / 6307
Хворий 34-х років страждає на туберкульоз легень з дитинства. Йому призначена операція. Під час операції у хворого була видалена середня доля правої легені. Які сегменти були видалені?

Верхній і передній

Верхній язичковий і нижній язичковий

Задній і передній

Медіальний базальний і латеральний базальний

Латеральний і медіальний

2421 / 6307
У потерпілого 35-ти років встановлено гнійне запалення жовчного міхура. В який відділ очеревинної порожнини потрапить гній під час розриву жовчного міхура при його типовому положенні?

Лівий бічний канал

Чепцева сумка

Печінкова сумка

Верхній дванадцятипалий закуток

Передшлункова сумка

2422 / 6307
При мікроскопії мазка фекалій школяра виявлені жовто-коричневого кольору яйця з горбкуватою оболонкою. Якому гельмінту вони належать?

Аскарида

Стьожак широкий

Гострик

Ціп’як карликовий

Волосоголовець

2423 / 6307
В експерименті на тварині досліджують серцевий цикл. Закриті усі клапани серця. Якій фазі циклу відповідає такий стан?

Асинхронного скорочення

Швидкого наповнення

Ізометричного скорочення

Протодіастолічний період

Повільного наповнення

2424 / 6307
На основі лабораторного аналізу у хворого підтверджено діагноз - подагра. Для встановлення діагнозу було проведено визначення вмісту:

Сечовини в крові та сечі

Залишкового азоту в крові

Сечової кислоти в крові та сечі

Креатиніну в сечі

Аміаку в сечі

2425 / 6307
Ціаністий калій є отрутою, від якої смерть організму наступає миттєво. На які ферменти в мітохондріях діє ціанистий калій:

НАД+ - залежні дегідрогенази

Флавінові ферменти

Цитохромоксидаза (аа3)

Цитохром Р-450

Цитохром В5

2426 / 6307
Для лікування деяких інфекційних захворювань, викликаних бактеріями, застосовуються сульфаніламідні препарати, що блокують синтез фактора росту бактерій. Назвіть механізм їх дії:

Є алостеричними ферментами

Є алостеричними інгібіторами ферментів

Беруть участь в окисно-відновних процесах

Є антивітамінами параамінобензойної кислоти

Інгібують всмоктування фолієвої кислоти

2427 / 6307
У різнороба 55-ти років, який 28 років пропрацював на хімічному заводі в цеху зі шкідливими умовами виробництва, спостерігаються часті кровотечі із слизової оболонки носової порожнини. За рахунок яких артерій це відбувається?

Передня і задня решітчасті артерії

Війчасті артерії

Надочноямкова артерія

Передня мозкова артерія

Очна артерія

2428 / 6307
Після травми передньої поверхні верхньої третини передпліччя у хворого утруднення пронації, послаблення долонного згинання кисті та порушення чутливості шкіри 1-3 пальців на долоні. Який нерв ушкоджено?

n. radialis

n. medianus

n. cutaneus antebrachii medialis

n. musculocutaneus

n. ulnaris

2429 / 6307
Юнак 15-ти років доставлений до приймального відділєння у непритомному стань Об’єктивно: на зовнішні подразники хворий не реагує, дихання періодичне за типом Чейн-Стокса, зіниці звужені, зіничний рефлекс відсутній. Було встановлено, що дані симптоми обумовлені використанням морфіну. Назвіть засіб для антидотної терапії:

Налоксон

Кальцію хлорид

Протаміну сульфат

Апоморфіну гідрохлорид

Унітіол

2430 / 6307
При гістологічному дослідженні вузла у видаленій молочній залозі серед рясної строми виявлені різних розмірів і форми комплекси атипових поліморфних епітеліальних клітин з наявністю просвітів у центрі комплексів. Клітини з великими ядрами, збільшеним числом ядерець і ядерцевих організаторів, наявністю атипових мітозів. Який попередній діагноз?

Солідний рак

Фіброаденома молочної залози

Аденокарцинома

Плоскоклітинний незроговілий рак

Недиференційований поліморфно-клітинний рак

2431 / 6307
У чоловіка 63-х років, що багато років страждав на атеросклероз і переніс раніше інфаркт міокарда, розвинувся напад загрудинного болю. Хворий госпіталізований, через 3 дні, на фоні прогресуючої серцево-судинної недостатності, помер. Під час розтину тіла у ділянці задньої стінки лівого шлуночка і міжшлуночкової перегородки виявлена ділянка білого кольору близько 3 см у діаметрі, волокниста, западаюча, з чіткою межею. Прозектор трактував ці зміни як:

Дистрофія міокарда

Вогнищевий кардіосклероз

Інфаркт міокарда

Ішемія міокарда

Міокардит

2432 / 6307
23-х років вдень раптово підвищилася температура до 39,5°C і через 6 годин повернулася до норми. На другу добу напад повторився і температура досягла 41,5°C. Період апірексії настав через 8 годин. Який тип температурної кривої?

Септичний

Виснажуючий

Постійний

Послаблюючий

Переміжний

2433 / 6307
Під час оперативного втручання на фоні використання гігронію різко знизився артеріальний тиск. Представники яких груп лікарських препаратів можуть нормалізувати артеріальний тиск?

Гангліоблокатори

M-холіноміметики

α-адреноміметики

α-адреноблокатори

Н-холіноміметики

2434 / 6307
При глікогенозі - хворобі Гірке - порушується перетворення глюкозо-6-фосфату на глюкозу, що призводить до накопичення глікогену в печінці та нирках. Дефіцит якого ферменту є причиною захворювання?

Гексокіназа

Альдолаза

Глюкозо-6-фосфатаза

Глікогенсинтетаза

Фосфорилаза

2435 / 6307
При проведені наукового експерименту дослідник зруйнував структуру однієї з частин клітини, що порушило здатність клітини до поділу. Яка структура була зруйнована найбільш імовірно?

Мітохондрії

Центросома

Пластичний комплекс

Глікокалікс

Мікрофібрили

2436 / 6307
У хворого з дихальною недостатністю рН крові 7,35. Визначення рСO2 показало наявність гіперкапнії. При дослідженні рН сечі відзначається підвищення її кислотності. Яка форма порушення кислотноосновного стану в даному випадку?

Ацидоз метаболічний, компенсований

Ацидоз метаболiчний, декомпенсований

Ацидоз газовий, компенсований

Алкалоз газовий, компенсований

Алкалоз газовий, декомпенсований

2437 / 6307
При аутопсії тіла чоловіка середніх років, що тривало страждав на бронхоектатичну хворобу, виявлено, що обидва наднирника різко з6ільшені в об’ємі за рахунок кіркового шару. Наднирники щільні, бліді, сального вигляду. Мікроскопічно по ходу ретикулярної строми, у стінках судин - відкладання безструктурних, гомогенних, еозинофільних мас і конго-рот - позитивних мас. Який процес зумовив ці зміни?

Мукоїдне набухання

Гіаліноз

Фібриноїдне набухання

Ліпідоз

Амілоїдоз

2438 / 6307
При бактеріологічному дослідженні сечі хворого на пієлонефрит виділені мікроорганізми, що утворюють на м’ясо-пептонному агарі жовто-зелений пігмент і характерний запах. Як вони називаються?

Протеї

Ешеріхії

Клебсієли

Азотобактерії

Псевдомонади

2439 / 6307
Дитина 4-х років вдихнула ґудзик, який за допомогою бронхоскопу був видалений з правого головного бронху. Який епітелій бронху найбільш імовірно ушкоджений стороннім предметом?

Багатошаровий незроговілий

Одношаровий плоский

Одношаровий низькопризматичний

Одношаровий багаторядний війчастий

Перехідний

2440 / 6307
Жінка 53-х років хвора на гіпертонічну хворобу тривалий час лікувалася гіпотензивними засобами. Останнім часом стан її погіршився: з’явились нудота, набряк повік та губ, уртикарні висипання на шкірі, блювання, брадикардія, пронос. Який препарат викликає ці явища?

Клофелін

Дибазол

Резерпін

Анаприлін

Бензогексоній

2441 / 6307
У хворого 48-ми років, на тлі хронічного гломерулонефриту, розвинулася стійка артеріальна гіпертензія. Вкажіть групу найбільш ефективних лікарських засобів для лікування цього пацієнта:

Антагоністи кальцію

а-адреноблокатори

Інгібітори ангіотензинперетворюючого ферменту

Гангліоблокатори

Міотропні спазмолітики

2442 / 6307
Чоловік 54-х років госпіталізований до нейрохірургічного відділення зі скаргами на відсутність чутливості шкіри нижньої повіки, латеральної поверхні носа, верхньої губи. Лікар при огляді встановив запалення другої гілки трійчастого нерва. Через який отвір виходить із черепа ця гілка?

Остистий отвір

Круглий отвір

Овальний отвір

Верхня очноямкова щілина

Рваний отвір

2443 / 6307
Чоловік 64-х років із хронічним закрепом приймав щодня одне драже 'Бісакоділ'. Через деякий час для отримання ефекту він змушений був приймати по 2 драже. Як називається такий тип зміни дії лікарських речовин?

Кумуляція

Сенсибілізація

Ідіосинкразія

Звикання

Лікарська залежність

2444 / 6307
У чоловіка 22-х років високого росту та астенічної будови тіла з ознаками гіпогонадизму, гінекомастією та зменшеною продукцією сперми (азооспермія) виявлено каріотип 47 XXY. Який спадковий синдром супроводжується такою хромосомною аномалією?

Віскотта-Олдрича

Луї-Барра

Тернера

Дауна

Клайнфельтера

2445 / 6307
У товщі шкіри макроскопічно знайдена і видалена щільна пухлина, рухлива. При мікроскопії вона представлена хаотично розташованими пучками колагенових волокон з невеликою кількістю веретеноподі6них клітин. Яка пухлина видалена?

Лейоміома

Ліпома

Іломус-ангіома

Щільна фіброма

Меланома

2446 / 6307
Хвора 26-ти років звернулася до лікаря зі скаргами на появу у випорожненнях білих плоских рухливих утворів, які нагадують локшину. При лабораторному дослідженні виявлені членики з такою характеристикою: довгі, вузькі, з розміщеним поздовжньо каналом матки, яка має 17-35 бічних відгалужень з кожного боку. Який вид гельмінтів паразитує у кишечнику жінки?

Hymenolepis nana

Diphyllobothrium latum

Echinococcus granulosus

Taenia solium

Taeniarhynchus saginatus

2447 / 6307
У жінки 30-ти років при гістологічному дослідженні шийного лімфовузла виявлено порушення малюнка внаслідок розростання епітеліоїдних, лімфоїдних клітин і макрофагів з ядрами у вигляді підкови, в центрі деяких скупчень клітин - безструктурні ділянки блідо-рожевого кольору з уламками ядер. Для якого захворювання характерні такі зміни?

Туберкульоз

Сифіліс

Метастаз пухлини

Лімфогрануломатоз

Актиномікоз

2448 / 6307
В експерименті у тварини в результаті проведеного перетинання депресорного нерва та руйнування каротидних клубочків розвинулась стійка гіпертензія. З порушенням якої функції нервової системи пов’язане це явище?

Трофічна

Вища нервова діяльність

Вегетативна

Сенсорна

Рухова

2449 / 6307
У хворого 20-ти років видалена пухлина лобної частки правої півкулі головного мозку діаметром 5 см, яка була нечітко відмежована від довколишньої тканини. На розрізі - однорідного вигляду, гістологічно - складається із зіркоподібних клітин, численні відростки яких утворюють густі сплетіння. Яка пухлина була у хворого?

Астроцитома

Хоріоїдпапілома

Гангліоневрома

Епендімома

Олігодендрогліома

2450 / 6307
У хворого 20-ти років з жовтяницею встановлено: підвищення у плазмі крові вмісту загального білірубіну за рахунок непрямого (вільного), в калі та сечі - високий вміст стеркобіліну, рівень прямого (зв’язаного) білірубіну в плазмі крові в межах норми. Про який вид жовтяниці можна думати?

Жовтяниця немовлят

Гемолітична

Хвороба Жильбера

Механічна

Паренхіматозна (печінкова)

2451 / 6307
У хлопчика зі спадково обумовленими вадами зразу ж після народження спостерігався характерний синдром, який називають 'крик кішки'. У ранньому дитинстві малюк мав 'нявкаючий' тембр голосу. Під час дослідження каріотипу цієї дитини було виявлено:

Додаткову Y-хромосому

Додаткову Х-хромосому

Нестачу Х-хромосоми

Делецію короткого плеча 5-ї хромосоми

Додаткову 21-у хромосому

2452 / 6307
При визначенні групи крові за системою АВ0 за допомогою стандартних сироваток були отримані наступні результати: аглютинацію еритроцитів викликали сироватки I, II та III груп. Яка група досліджуваної крові?

Неможливо визначити

AB(IV)

B(III)

O(I)

A(II)

2453 / 6307
Жінка 68-ми років, що страждає на атеросклероз, госпіталізована до хірургічного відділення з приводу розлитого гнійного перитоніту. Під час операції діагностовано тромбоз брижових артерій. Яка найбільш імовірна причина перитоніту?

Стаз

Немічний інфаркт

Геморагічний інфаркт

Ішемія ангіоспастична

Ішемія компресійна

2454 / 6307
Для утворення транспортної форми амінокислот для синтезу білка необхідно:

Ревертаза

Рибосома

м-РНК

Аміноацил-тРНК-синтетаза

ГТФ

2455 / 6307
Перебуваючи у робочому відрядженні в одній із країн тропічної Африки, лікар зіткнувся зі скаргами місцевого населення з приводу хвороби дітей 10-14 років, що супроводжується стійкими лихоманками, які не мають правильного чергування, виснаженням, анемією, збільшенням печінки і селезінки. Враховуючи місцеві умови, що пов’язані з великою кількістю москітів, можна передбачити що це:

Сонна хвороба

Балантидіаз

Вісцеральний лейшманіоз

Хвороба Чагаса

Токсоплазмоз

2456 / 6307
Під час хірургічного втручання на органах черевної порожнини сталася рефлекторна зупинка серця. Де знаходиться центр рефлексу?

Середній мозок

Кора великих півкуль

Проміжний мозок

Спинний мозок

Довгастий мозок

2457 / 6307
У чоловіка 36-ти років розвинулася гарячка, яка супроводжувалася зміщенням установчої точки терморегуляційного центру на більш високий рівень, з послідовним чергуванням наступних стадій: Incrementi, fastigii, decrementi. При якому захворюванні можуть спостерігатися подібні зміни?

Гостра пневмонія

Гіпертрофія міокарда

Акромегалія

Ренальний діабет

Цукровий діабет

2458 / 6307
Одна з форм вродженої патології супроводжується гальмуванням перетворення фенілаланіну в тирозин. Біохімічною ознакою хвороби є накопичення в організмі деяких органічних кислот, зокрема:

Фенілпіровиноградна

Глутамінова

Молочна

Лимонна

Піровиноградна

2459 / 6307
У чоловіка 41-го року відзначаються періодичні напади серцебиття (пароксизми), сильне потовиділення, напади головного болю. При обстеженні виявлена гіпертензія, гіперглікемія, підвищення основного обміну, тахікардія. При якій патології наднирників спостерігається подібна картина?

Гіперфункція кори наднирників

Гіперфункція мозкового шару

Гіпофункція мозкового шару

Гіпофункція кори наднирників

Первинний альдостеронізм

2460 / 6307
У дитини 10-ти років через 2 тижні після перенесеної ангіни розвинувся нефритичний синдром (протеїнурія, гематурія, циліндрурія), що свідчить про ураження базальної мембрани клубочків нирок. Який найбільш імовірний механізм лежить в основі ушкодження базальної мембрани?

Гранулематозний

Реагіновий

Антитільний

Цитотоксичний

Імунокомплексний

2461 / 6307
Жінка 24-х років звернулася до медико-генетичної консультації з приводу оцінки ризику захворювання на гемофілію у її дітей. Її чоловік страждає на гемофілію. Під час збору анамнезу виявилося, що у сім’ї жінки не було випадків гемофілії. Вкажіть ризик народження хворої дитини:

Відсутній

25%

100%

75%

50%

2462 / 6307
При тиреотоксикозі підвищується продукція тиреоїдних гормонів Т3 та Т4, розвиваються схуднення, тахікардія, психічне збудження та інше. Як саме впливають тиреоїдні гормони на енергетичний обмін в мітохондріях клітин?

Блокують субстратне фосфорилювання

Роз‘єднують окислення та окисне фосфорилювання

Блокують дихальний ланцюг

Активують субстратне фосфорилювання

Активують окисне фосфорилювання

2463 / 6307
У людини внаслідок тривалого перебування у горах на висоті 3000 м над рівнем моря збільшилась киснева ємкість крові. Безпосередньою причиною цього є посилене утворення в організмі:

2,3-дифосфогліцерату

Карбгемоглобіну

Катехоламінів

Еритропоетинів

Лейкопоетинів

2464 / 6307
У хворого 45-ти років при рентгенологічному обстеженні була виявлена кила міжхребцевого диску грудного відділу хребетного стовпа. Який вид з’єднання між хребцями зазнав патологічних змін?

Діартроз

Синостоз

Геміартроз

Синсаркоз

Синхондроз

2465 / 6307
Чоловіку 44-х років за вимогою діагностичних тестів зробили лімфографію органів грудної порожнини. Хірург встановив, що пухлина вразила орган, з лімфатичних судин якого лімфа безпосередньо переходить у грудну протоку. Який це орган?

Лівий головний бронх

Стравохід

Серце

Осердя

Трахея

2466 / 6307
Виникнення нижчеперерахованих захворювань пов’язане із генетичними факторами. Назвіть патологію із спадковою схильністю:

Дальтонізм

Цукровий діабет

Фенілкетонурія

Хорея Гентінгтона

Серпоподібноклітинна анемія

2467 / 6307
Серологічна діагностика інфекційних захворювань заснована на специфічній взаємодії антитіл з антигенами. Як називається серологічна реакція, при якій високодисперсні антигени адсорбовані на еритроцитах?

Реакція нейтралізації

Реакція преципітації

Реакція непрямої (пасивної) гемаглютинації

Реакція зв’язування комплементу

Реакція гемадсорбції

2468 / 6307
Чоловік 42-х років з ураженням ліктьового нерва не може звести до серединної лінії II і V пальці. Функція яких м’язів при цьому порушена?

Тильні міжкісткові м’язи

Короткий долонний м’яз

Долонні міжкісткові м’язи

Відвідний м’яз мізинця

Червоподібні м’язи

2469 / 6307
На електронній мікрофотографії епідермісу шкіри серед клітин кубічної форми виділяються паросткові клітини, в цитоплазмі яких добре розвинутий апарат Гольджі, багато рибосом і меланосом. Назвіть ці клітини:

Клітини Лангеганса

Тканинні базофіли

Меланоцити

Клітини Меркеля

Кератиноцити

2470 / 6307
Потужність, що розвиває м’яз, недостатня для піднімання вантажу. Який вид скорочення м’яза має місце у даному випадку?

Ексцентричне

Тетанічне

Концентричне

Ізометричне

Ізотонічне

2471 / 6307
Через рік після субтотальної резекції шлунка з приводу виразки малої кривизни виявлені зміни в аналізі крові - анемія, лейко- і тромбоцитопенія, КП=1.3, наявність мегалобластів та мегалоцитів. Дефіцит якого фактору обумовив розвиток цієї анемії?

Хлороводнева кислота

Муцин

Фактор Касла

Пепсин

Іастрин

2472 / 6307
У мазку з нальоту на мигдаликах хворого з підозрою на дифтерію виявлено палички синього кольору з потовщеннями на полюсах. Який метод фарбування мазків було використано?

Грама

Пнса

Леффлера

Нейссера

Буррі

2473 / 6307
Хвора 66-ти років скаржиться на 6іль у гомілці, який підсилюється при ходьбі. Об’єктивно: набряк та почервоніння по ходу вени. Лікар призначив антикоагулянт прямої дії для місцевого вживання. Який препарат можна використати з цією метою?

Мазь гепаринова

Тромбін

Мазь саліцилова

Мазь бутадіонова

Мазь троксевазинова

2474 / 6307
У хлопчика 6-ти років відзначається змішана інвазія аскаридами та гостриками. Який протиглисний препарат слід призначити для одноразового прийому?

Фенасал

Аміноакрихін

Насіння гарбуза

Піперазину адипінат

Мебендазол

2475 / 6307
Дитина народилася в стані асфіксії. Який препарат необхідно ввести новонародженому для стимуляції дихання?

Етимізол

Атропін

Прозерин

Празозин

Лобелін

2476 / 6307
У пацієнта 38-ми років при прогресуванні стафілококового гнійного періодонтиту виникло гнійне запалення кістково-мозкових просторів альвеолярного відростка, а потім тіла нижньої щелепи. Мікроскопічно кісткові балки витончені, вогнища некрозу, кісткові секвестри, оточені сполучнотканинною капсулою. Який найбільш імовірний діагноз?

Пародонтома

Хронічний остеомієліт

Хронічний фіброзний періостит

Гострий остеомієліт

Гнійний періостит

2477 / 6307
Еритроцити людини не містять мітохондрій. Який основний шлях утворення АТФ в цих клітинах?

Креатинкіназна реакція

Окислювальне фосфорилювання

Анаеробний гліколіз

Аденілаткіназна реакція

Аеробний гліколіз

2478 / 6307
У чоловіка 50-ти років запалення яєчка ускладнилось його водянкою. Необхідне оперативне втручання. Яку з оболонок яєчка останньою розтинає хірург під час операції?

Зовнішня сім’яна фасція

М’яз-підіймач яєчка

Внутрішня сім’яна фасція

М ’ясиста оболонка

Парієтальний листок піхвової оболонки яєчка

2479 / 6307
У жінки 52-х років при обстеженні було виявлено зниження кількості еритроцитів у крові та підвищення рівня вільного гемоглобіну в плазмі крові (гемоглобінемія). КП- 0,85. Який вид анемії спостерігається у хворої?

Хронічна постгеморагічна

Спадкова гемолітична

Набута гемолітична

Анемія внаслідок порушення еритропоезу

Гостра постгеморагічна

2480 / 6307
У хворого 28-ми років тривале блювання призвело до зневоднення організму. Підвищена секреція якого гормону перш за все сприятиме збереженню води в організмі?

Альдостерон

Тироксин

Вазопресин

Кальцитонін

Соматостатин

2481 / 6307
При проведенні операції на тонкій кишці лікар виявив ділянку слизової оболонки, де на фоні колових складок була присутня поздовжня складка. Який відділ тонкої кишки має таку будову?

Pars descendens duodeni

Pars horizontalis duodeni

Початковий eiddrn jejunum

Pars ascendens duodeni

Дистальний вiддiл ileum

2482 / 6307
У хворого нормально забарвлений кал, у складі якого знаходиться велика кількість вільних жирних кислот. Причиною цього є порушення наступного процесу:

Секреція ліпаз

Гідроліз жирів

Жовчоутворення

Всмоктування жирів

Жовчовиділення

2483 / 6307
При термометрії встановлено, що температура відкритих ділянок шкіри на 1-1,5° нижче за температуру поруч розташованих ділянок, закритих одягом з натуральних тканин. Причиною цього є те, що одяг, перш за все, зменшує тепловіддачу таким шляхом:

Радіація

Проведення

Конвекція

Випаровування

2484 / 6307
На препараті м’якої мозкової оболонки виявляється судина, у стінці якої відсутня середня оболонка, зовнішня оболонка зрощена з оточуючою тканиною, внутрішня оболонка побудована із базальної мембрани та ендотелію. Що це за судина?

Вена м’язового типу зі слабким розвитком м’язових елементів

Артеріола

Артерія м’язового типу

Артерія змішаного типу

Вена волокнистого типу

2485 / 6307
В основі розвитку імунних і алергічних реакцій організмом застосовуються однакові механізми відповіді імунної системи на антиген. Визначте основну відмінність алергічних реакцій від імунних?

Шляхи потрапляння в організм

Кількість антигену, що потрапляє

Розвиток пошкодження тканин

Особливість будови антигенів

Спадкова схильність

2486 / 6307
Чоловік 38-ми років поступив до неврологічного відділення зі скаргами на погіршення пам’яті та розумової працездатності після перенесеної травми голови. Запропонуйте лікарський засіб для покращення метаболізму головного мозку:

Анальгін

Меридил

Пірацетам (ноотропіл)

Сиднокарб

Кофеїн

2487 / 6307
У хлопчика 8-ми років виник біль у горлі, підвищилась температура тіла. На 2-й день від початку захворювання виявлені висипання червоного кольору у вигляді дрібних, густо розташованих плям, величиною з макове зерно. Вони покривають все тіло, за винятком носо-губного трикутника. При огляді порожнини рота - в зіві яскраве почервоніння, мигдалики збільшені, язик малиново-червоний. Який найбільш імовірний діагноз?

Аденовірусна інфекція

Дифтерія зіва

Стрептококова ангіна

Кір

Скарлатина

2488 / 6307
Юнака 15-ти років вжалила бджола. На місці укусу визначається набряк, гіперемія, підвищення температури. Назвіть ініціальний патогенетичний фактор запального набряку:

Зниження онкотичного тиску крові

Підвищення кров’яного тиску в капілярах

Підвищення проникності мікросудин

Порушення лімфовідтоку

Підвищення осмотичного тиску у вогнищі запалення

2489 / 6307
Хворий 49-ти років був доставлений до лікарні в коматозному стані. В анамнезі - цукровий діабет. Об’єктивно: дихання Кусмауля, зниження артеріального тиску, у видихуваному повітрі запах ацетону. Після проведеної невідкладної терапії стан покращився. Який препарат було введено хворому?

Букаркам

Інсулін

Глібенкламід

Адреналін

Ізадрин

2490 / 6307
Катіонні глікопротеїни є основними компонентами слини привушних залоз. Які амінокислоти обумовлюють їх позитивний заряд?

Аспартат, аргінін, глутамат

Аспартат, глутамат, гліцин

Цистеїн, гліцин, пролін

Глутамат, валін, лейцин

Лізин, аргінін, гістидин

2491 / 6307
На мікропрепараті очного яблука плода спостерігається пошкодження рогівки. Частина якого зародкового листка була уражена в процесі ембріонального розвитку?

Дерматом

Нефротом

Мезодерма

Ектодерма

Ентодерма

2492 / 6307
Жінці 55-ти років для лікування ІХС був призначений β-адреноблокатор. Через деякий час у неї з’явився кашель, бронхоспазм. Якому з перелічених засобів притаманна така побічна дія?

Метопролол

Фенігідин

Атенолол

Талінол

Анаприлін

2493 / 6307
Студентка 22-х років звернулася до лікаря із скаргами на підвищення температури тіла до 38°C, слабкість, біль у горлі. Об’єктивно: язик вкритий білим нальотом. Які гістологічні структури язика беруть участь в утворенні цього нальоту?

Епітелій грибоподібних сосочків

Сполучнотканинна основа всіх сосочків язика

Епітелій жолобкуватих сосочків

Епітелій ниткоподібних сосочків

Епітелій листоподібних сосочків

2494 / 6307
Досліджуються рецептори, інформація від яких прямує до кори без участі таламусу. Які це рецептори?

Дотикові

Слухові

Зорові

Смакові

Нюхові

2495 / 6307
У здорових батьків, спадковість яких не обтяжена, народилась дитина з чисельними вадами розвитку. Цитогенетичний аналіз виявив у соматичних клітинах дитини трисомію за 13-ю хромосомою (синдром Патау). З яким явищем пов’язане народження такої дитини?

Рецесивна мутація

Порушення гаметогенезу

Домінантна мутація

Соматична мутація

Хромосомна мутація

2496 / 6307
На препараті представлено орган, вкритий сполучнотканинною капсулою, від якої відходять трабекули. В органі можна розрізнити кіркову речовину, де містяться лімфатичні вузлики та мозкову речовину, представлену тяжами лімфоїдних клітин. Який орган представлений на препараті?

Тимус

Селезінка

Лімфатичний вузол

Мигдалики

Червоний кістковий мозок

2497 / 6307
До біорегуляторів клітинних функцій ліпідної природи належать тромбоксани. Джерелом для синтезу цих сполук є:

Арахідонова кислота

Пальмітинова кислота

Пальмітоолеїнова кислота

Фосфатидна кислота

Стеаринова кислота

2498 / 6307
Жінка 25-ти років звернулася зі скаргами на дисменорею та безпліддя. При обстеженні виявлено: зріст жінки 145 см, недорозвинені вторинні статеві ознаки, на шиї крилоподібні складки. При цитологічному дослідженні в соматичних клітинах не виявлено тілець Барра. Який діагноз встановив лікар?

Синдром Клайнфельтера

Синдром трисомії Х

Синдром Шерешевського-Тернера

Синдром Морріса

2499 / 6307
У чоловіка 36-ти років хворого на туберкульоз легень непереносимість аміноглікозидного антибіотика амікацину. Який протитуберкульозний антибіотик можна включити до складу комплексної терапії у даному випадку?

Стрептоміцин

Рифампіцин

Бензилпеніцилін

Амоксицилін

Канаміцин

2500 / 6307
У жінки 60-ти років після емоційної реакції, яка була викликана гнівом, виник напад загрудинного болю. На ЕКГ були встановлені ознаки порушення коронарного кровообігу. Який вид порушень міг спричинити це явище?

Артерiальна гіпєрємія

Венозна гіперемія

Венозний стаз

Справжній стаз

Ангіоспастична ішемія

2501 / 6307
Для запобігання нападів гострого панкреатиту лікар призначив трасілол (контрікал, гордокс), який є інгібітором:

Гастриксину

Хімотрипсину

Трипсину

Карбоксипептидази

Еластази

2502 / 6307
Хворий 58-ми років помер від прогресуючої серцевої недостатності. На розтині: серце розширене у поперечнику, мляве, м’яз на розрізі нерівномірного кровонаповнення, пістрявий. При гістологічному дослідженні: у міокарді повнокров’я, у стромі лімфогістіоцитарні інфільтрати, що розсувають кардіоміоцити. Виявлені морфологічні зміни свідчать про:

Інфаркт міокарда

Жирову дистрофії міокарда

Кардіосклероз

Венозне повнокров’я

Негнійний проміжний міокардит

2503 / 6307
Хвора 39-ти років, з цукровим діабетом в анамнезі, госпіталізована до клініки у прекоматозному стані кетоацидотичного типу. Збільшення вмісту якого метаболіту до цього призвело?

Аспартат

Альфа-кетоглутарат

Малонат

Цитрат

Ацетоацетат

2504 / 6307
У хворих із синдромом набутого імунодефіциту (СНІД) різко знижується імунологічна реактивність, що проявляється розвитком хронічних запальних процесів, інфекційних захворювань, пухлинного росту. Клітини якого типу ушкоджує ВІЛ-інфекція, внаслідок чого знижується імунний захист?

Т-супресори

Т4-хелпери

Т8-ефектори

Природні кілери (NK)

В-лімфоцити

2505 / 6307
Під час аналізу електрокардіограми встановлено збільшення тривалості і амплітуди зубця S. Деполяризація якої ділянки серця порушена у хворого?

Верхівка серця

Базальні відділи шлуночків

Середня і нижня третина міжшлуно-чкової перегородки

Бокові стінки шлуночків

Передсердя

2506 / 6307
У хворого 42-х років з хронічною серцевою недостатністю, незважаючи на терапію кардіотонічними засобами і тіазидовим діуретиком, зберігаються набряки і виникла загроза асциту. Який препарат слід призначити для підсилення діуретичного ефекту застосованих ліків?

Фуросемід

Амілорид

Манітол

Спіронолактон

Клопамід

2507 / 6307
У синтезі пуринових нуклеотидів беруть участь деякі амінокислоти, похідні вітамінів, фосфорні ефіри рибози. Коферментна форма якого вітаміну є переносником одновуглецевих фрагментів в синтезі пуринових нуклеотидів?

Фолієва кислота

Рибофлавін

Пантотенова кислота

Піридоксин

Нікотинова кислота

2508 / 6307
На дослідження в бактеріологічну лабораторію було відправлено випорожнення хворої дитини грудного віку, з яких виділена культура ентеропатогенних кишкових паличок О55К59. На основі яких критеріїв виділена культура віднесена до ЕПКП О55?

Визначення фаговару

Антигенні властивості

Культуральні ознаки

Морфологічні ознаки

Біохімічні властивості

2509 / 6307
У померлої дитини 3-х років за життя мала місце менінгіальна симптоматика, На розтині в м’якій мозковій оболонці макроскопічно виявлені просоподібні вузлики, які мікроскопічно представлені осередком казеозного некрозу з валами епітеліоїдних, лімфоїдних клітин, між якими зустрічаються великі клітини з ядрами на периферії у вигляді півмісяця. Який найбільш імовірний менінгіт у дитини?

Туберкульозний

Бруцельозний

Менінгококовий

Грипозний

Сифілітичний

2510 / 6307
З метою оцінки адаптації до фізичного навантаження лікар провів обстеження робітників після виконання важкої праці. Які зміни в загальному аналізі крові можна виявити?

Анемія

Зсув лейкоцитарної формули вліво

Лейкопенія

Гіпоальбумінемія

Перерозподільчий лейкоцитоз

2511 / 6307
У чоловіка 34-х років під час бійки виникла зупинка серця внаслідок сильного удару у верхню ділянку передньої черевної стінки. Який із зазначених механізмів регуляції спричинив зупинку серця?

Симпатичні умовні рефлекси

Парасимпатичні безумовні рефлекси

Симпатичні безумовні рефлекси

Парасимпатичні умовні рефлекси

Периферичні рефлекси

2512 / 6307
Після фармакологічної блокади іонних каналів мембрани нервового волокна потенціал спокою зменшився з -90 до -80 мВ. Які канали було заблоковано?

Натрієві

Кальцієві

Магнієві

Калієві

Хлорні

2513 / 6307
У хворого 49-ти років на гострий панкреатит виникала загроза некрозу підшлункової залози, що супроводжувалось надходженням у кров і тканини активних панкреатичних протеїназ і розщеплення тканинних білків. Які захисні фактори організму можуть інгібувати ці процеси?

Церулоплазмін, трансферин

Імуноглобуліни

Гемоплексин, гаптоглобін

α2-макроглобулін, α1 -антитрипсин

Кріоглобулін, інтерферон

2514 / 6307
У чоловіка 36-ти років після перенесеної стрептококової інфекції діагностовано гострий гломерулонефрит. Найбільш імовірно, що ураження базальної мембрани ниркових тілець виникає внаслідок алергічної реакції такого типу:

Імунокомплексна

Стимулююча

Анафілактична

Цитотоксична

Сповільнена

2515 / 6307
Хворому 46-ти років на ревматоїдний поліартрит призначили нестероїдний протизапальний засіб. Але, з причини супутнього захворювання, через деякий час препарат відмінили. Яке захворювання є протипоказанням до призначення цієї групи препаратів?

Виразкова хвороба

Пневмонія

Бронхіт

Радикуліт

Мігрень

2516 / 6307
Чоловік 53-х років звернувся зі скаргами на гострий біль у правому підребер’ї. При огляді лікар звернув увагу на пожовтілі склери хворого. Лабораторні аналізи показали підвищену активність АЛТ та негативну реакцію на стеркобілін у калі. Для якого захворювання характерні такі симптоми?

Хронічний коліт

Гепатит

Жовчнокам’яна хвороба

Гемолітична жовтяниця

Хронічний гастрит

2517 / 6307
До інфекційної лікарні доставлено літнього чоловіка, безпритульного. Скарги на високу температуру, запаморочення, висип на шкірі. Враховуючи, що хворий страждає також на педикульоз, лікар запідозрив висипний тиф. Який метод діагностики найдоцільніше використати для підтвердження діагнозу?

Серологічний

Мікроскопічний

Вірусологічний

Бактеріологічний

Алергологічний

2518 / 6307
У ході хірургічної операції на нирці із заднього доступу (з боку спини) виникла необхідність перетиснути ниркову артерію. У якій послідовності розташовані елементи ниркової ніжки у її воротах ззаду наперед?

Сечовід, артерія, вена

Артерія, сечовід, вена

Вена, сечовід, артерія

Вена, артерія, сечовід

Артерія, вена, сечовід

2519 / 6307
У лікарню наприкінці робочого дня доставлений робітник 'гарячого' цеху, який скаржиться на головний біль, запаморочення, нудоту, загальну слабкість. Об’єктивно: свідомість збережена, шкірні покриви гіперемовані, сухі, гарячі на дотик. ЧСС- 130/хв. Дихання часте, поверхневе. Яке порушення процесів терморегуляції, найбільш імовірно, виникло у людини в даній ситуації?

Посилення тепловіддачі та теплопродукції

Зниження теплопродукції без змін тепловіддачі

Посилення тепловіддачі та зниження теплопродукції

Посилення теплопродукції без змін тепловіддачі

Зниження тепловіддачі

2520 / 6307
В експерименті подразнюють гілочки симпатичного нерва, які іннервують серце. Це призвело до збільшення сили серцевих скорочень, тому що через мембрану типових кардіоміоцитів збільшився:

Вихід іонів калію

Вхід іонів калію

Вхід іонів кальцію

Вхід іонів кальцію та калію

Вихід іонів кальцію

2521 / 6307
Під час тривалого бігу у чоловіка 35-ти років виникла гостра серцева недостатність. Які зміни іонного складу спостерігаються у серцевому м’язі при цьому стані?

Зменшення в позаклітинному просторі іонів K + і Mg2+

Збільшення в позаклітинному просторі іонів Na+ і Ca2+

Накопичення в клітинах міокарда іонів K + і Mg2+

Накопичення в клітинах міокарда іонів Na+ і Ca2+

Зменшення в клітинах міокарда іонів Na+ і Ca2+

2522 / 6307
Дитині 14-ти років поставили діагноз: бруцельоз. В контакті з хворими тваринами вона не була. Як дитина могла заразитись?

Через сире молоко

Через брудні руки

Через воду

Під час ін’єкції

Через немиті овочі і фрукти

2523 / 6307
При гострому інфаркті в міокарді виникає декілька зон в осередку інфаркту: зона некрозу, зона ішемічного пошкодження і зона ішемії. Зоні пошкодження на ЕКГ відповідає:

Комплекс QRS типу QS

Глибокий зубець Q

Негативний зубець T

Зниження зубця R

Зміщення сегменту RS — T вище ізолінії

2524 / 6307
Жінка 38-ми років звернулася до ендокринологічної клініки з виразним тремором кінцівок. Гіперпродукція, якого гормону здатна викликати такі порушення?

АКТГ

Інсулін

Соматостатин

Адреналін

Тироксин

2525 / 6307
Чоловік 60-ти років був госпіталізований до стаціонару з діагнозом отруєння грибами, де і помер на 12-й день з ознаками гострої печінкової недостатності. На секції - макроскопічно: печінка дрябла, в’яла, жовто-сірого кольору, глинястого вигляду як на поверхні, так і на розтині; мікроскопічно: ділянки некрозу з аутолітичним розпадом і утворенням жиро-білкового детриту в центрі та вузькою смужкою гепатоцитів у стані жирової дистрофії на периферії печінкових часточок. Який найбільш імовірний діагноз?

Хронічна токсична дистрофія печінки

Хвороба Вільсона-Коновалова

Спадковий пігментний гепатоз

Токсична дистрофія печінки в стадії червоної дистрофії

Токсична дистрофія печінки в стадії жовтої дистрофії

2526 / 6307
У хворого 32-х років у заочеревинному просторі виявлено пухлиноподібне утворення розміром 17х15 см з проростанням його в брижу, яке хірург цілком вилучити не зміг. Макроскопічно: тканина утворення на розрізі жовтувата, м’яка, з осередками некрозу і ослизнення. При мікроскопічному дослідженні виявлено: клітини з вираженим поліморфізмом ядер, з наявністю патологічних мітозів, цитоплазма клітин світла, дрібновакуолізована, при фарбуванні суданом вакуолі дають позитивну реакцію. Визначте пухлину:

Ліпосаркома

Гібернома

Фібросаркома

Фіброма

Ліпома

2527 / 6307
У юнака 16-ти років діагностовано хворобу Марфана. При дослідженні встановлено: порушення розвитку сполучної тканини, будови кришталика ока, аномалії серцевосудинної системи, арахнодактилія. Яке генетичне явище зумовило розвиток цієї хвороби?

Кодомінування

Неповне домінування

Множинний алелізм

Комплементарність

Плейотропія

2528 / 6307
При алкаптонурії відбувається надмірне виділення гомогентизинової кислоти із сечею. З порушенням метаболізму якої амінокислоти пов’язано виникнення цього захворювання?

Фенілаланін

Тирозин

Аланін

Аспарагін

Метіонін

2529 / 6307
У кішки з децеребраційною ригідністю потрібно знизити тонус м’язів. Цього можна досягти шляхом:

Подразнення вестибулярних ядер Дейтерса

Подразнення ампулярних вестибуло-рецепторів

Подразнення вестибулослухового нерва

Подразнення отолітових вестибуло-рецепторів

Руйнування вестибулярних ядер Дейтерса

2530 / 6307
Під дією негативних чинників довкілля порушена функція міосателітоцитів. Зміну якої функції всього м’язового волокна слід очікувати в даному випадку?

Скоротливий термогенез

Скорочення

Розслаблення

Трофіка

Регенерація

2531 / 6307
У жінки 23-х років після аборту з’явилась маткова кровотеча. Мікроскопічно у вишкребі з порожнини матки знайдені ворсини хоріона, що нагадують грона винограду. Мікроскопічно: набряк строми ворсин з утворенням безліч кіст, що супроводжуються проліферацією епітелію та синцитію ворсин, залишки плоду і плодових оболонок. Який найбільш імовірний діагноз?

Плацентарний поліп

Хоріонепітеліома

Ендометрит

Вагітність

Міхуровий занесок

2532 / 6307
У лабораторії особливо небезпечних інфекцій проводиться мікроскопічне дослідження патологічного матеріалу від хворого з підозрою на чуму, забарвленого за Гінсом-Буррі. Яку властивість збудника дозволяє визначити даний метод?

Капсулоутворення

Кислотостійкість

Наявність зерен волютину

Лугостійкість

Спороутворення

2533 / 6307
Фільтрат бульйонної культури збудників газової анаеробної інфекції розлили по пробірках, додали видові антитоксичні сироватки, витримали впродовж 40 хвилин при кімнатній температурі. Для визначення виду анаероба тепер необхідно:

Додати у пробірки преципітуючу діагностичну сироватку

Додати у пробірки еритроцитарний діагностикум

Вміст про6ірок посіяти на щільні поживні середовища

Додати у пробірки аглютинуючу діагностичну сироватку

Ввести тваринам вміст пробірок

2534 / 6307
Для постійного внутрішньовенного ведення ліків хворому краще встановити підключичний катетер. В якому анатомічному утворенні на шиї потрібно шукати підключичну вену?

Spatium interscalenum

Spatium interaponeuroticum suprasternale

Fossa retromandibularis

Spatium antescalenum

2535 / 6307
У тварини збільшений тонус м’язів-розгиначів. Це є наслідком посиленої передачі інформації до мотонейронів спинного мозку такими низхідними шляхами:

Ретикулоспінальні

Латеральні кортикоспінальні

Вестибулоспінальні

Медіальні кортикоспінальні

Руброспінальні

2536 / 6307
У пацієнта 21-го року при очікуванні екстракції зуба виникло сильне відчуття страху. Який з препаратів йому слід призначити для усунення цього відчуття?

Карбамазепін

Етимізол

Аміназин

Анальгін

Діазепам

2537 / 6307
У потерпілого 35-ти років з травмою голови у скроневій ділянці діагностовано епідуральну гематому. Яка з артерій найімовірніше пошкоджена?

Середня мозкова

Задня вушна

Поверхнева скронева

Середня оболонкова

Передня оболонкова

2538 / 6307
В результаті травми порушено цілісність переднього корінця спинного мозку. Які відростки яких нейронів при цьому пошкоджені?

Аксони чутливих нейронів

Аксони рухових нейронів

Дендрити рухових нейронів

Дендрити чутливих нейронів

Дендрити вставних нейронів

2539 / 6307
У чоловіка відмічається випадіння функції медіальних половин сітківки. Який відділ провідного шляху зорового аналізатора уражений?

Лівий зоровий нерв

Лівий зоровий тракт

Правий зоровий тракт

Зорове перехрестя

Правий зоровий нерв

2540 / 6307
В експерименті необхідно оцінити рівень збудливості тканини. Для цього доцільно визначити:

Критичний рівень деполяризації

Потенціал спокою

Амплітуду ПД

Тривалість ПД

Поріг деполяризації

2541 / 6307
Хворий 33-х років звернувся до стоматолога зі скаргами на те, що після лікування солями важких металів у нього з’явились сірі плями на слизовій оболонці ротової порожнини. Який засіб слід застосувати для усунення побічних ефектів?

Унітіол

Діамантовий зелений

Калію перманганат

Налоксон

Метиленовий синій

2542 / 6307
При визначенні повітряної та кісткової провідності звуку було встановлено, що у пацієнта ліве вухо краще сприймає звук при кістковому його проведенні, що могло бути пов’язане з захворюванням:

Середнього вуха зліва

Зовнішнього вуха справа

Середнього вуха справа

Внутрішнього вуха зліва

Внутрішнього вуха справа

2543 / 6307
В хронічному експерименті на щурах стимулювали електричним струмом паравентрикулярні та супраоптичні ядра гіпоталамуса. Яку поведінкову реакцію це спричинило у тварин?

Зменшення споживання води

Збільшення споживання води

Збільшення споживання їжі

Зменшення споживання їжі

Відмова від їжі та рідини

2544 / 6307
Чоловік 55-ти років доставлений до реанімаційного відділення без свідомості. Зі слів родичів стало відомо, що хворий помилково випив спиртовий розчин невідомого виробника. За даними проведеного обстеження діагностовано отруєння метиловим спиртом. Який антидот необхідно використати в даному випадку?

Ацетилцистеїн

Протаміну сульфат

Тетурам

Етанол

Налоксон

2545 / 6307
Внаслідок ДТП у потерпілої 37-ми років виникло неутримання сечі. Які сегменти спинного мозку пошкоджені?

L1 — L2

Th1 — L1

Th2 — Th5

S2 - S4

Th1 — Th5

2546 / 6307
При обстеженні жінки 56-ти років, що хвора на цукровий діабет 1-го типу, виявлене порушення білкового обміну, що при лабораторному дослідженні крові проявляється аміноацидемією а клінічно - уповільненням загоєння ран і зменшенням синтезу антитіл. Який з перерахованих механізмів викликає розвиток аміноацидемії?

Підвищення онкотичного тиску в плазмі крові

Підвищення протеолізу

Зменшення концентрації амінокислот у крові

Гіперпротеїнемія

Збільшення ліпопротеїдів високої щільності

2547 / 6307
Дитина 6-ти років під час гри порізала ногу осколком скла і була направлена у поліклініку для введення протиправцевої сироватки. З метою попередження розвитку анафілактичного шоку лікувальну сироватку вводили за методом Безредка. Який механізм лежить в основі подібного способу гіпосенсибілізації організму?

Стимуляція імунологічної толерантності до антигену

Стимуляція синтезу антиген-специфічних IgG

Зв’язування фіксованих на тучних клітинах IgE

Блокування синтезу медіаторів у тучних клітинах

Зв’язування рецепторів до IgE на тучних клітинах

2548 / 6307
У дитини 6-ти років гостро виникло захворювання з високою температурою тіла, вираженою інтоксикацією і геморагічною висипкою на шкірі. Смерть настала від гострої надниркової недостатності. На розтині морфологічні зміни визначалися головним чином важкістю ДВЗ-синдрому і ендотоксичного шоку. У шкірі виявлені вогнища некрозу, діапедезні крововиливи, у судинах дерми фібринові тромби. У наднирниках масивні крововиливи. Який найбільш імовірний діагноз?

Менінгококцемія

Грип

Висипний тиф

Скарлатина

Кір

2549 / 6307
Хворому на контактний дерматит необхідно призначити антигістамінний лікарський засіб, що не викликає сонливості. Назвіть цей препарат:

Супрастин

Ранітидин

Димедрол

Лоратадин

Дипразин

2550 / 6307
Недостатність в організмі лінолевої та ліноленової кислот призводить до ушкоджень шкіри, випадіння волосся, сповільненого загоювання ран, тромбоцитопенії, зниження опірності до інфекційних захворювань. Порушення синтезу яких речовин найімовірніше зумовлює вказані симптоми?

Ейкозаноїди

Інтерферони

Кортикостероїди

Інтерлейкіни

Катехоламіни

2551 / 6307
Прикладом специфічних паразитів людини є малярійний плазмодій, гострик дитячий і деякі інші. Джерелом інвазії таких паразитів завжди є людина. Такі специфічні паразити людини викликають захворювання, які називаються:

Антропонозні

Мультифакторіальні

Антропозоонозні

Інфекційні

Зоонозні

2552 / 6307
Хворому 63-х років з атонією сечового міхура лікар призначив препарат, дозу якого хворий самостійно збільшив. З’явились підвищене потовиділення, салівація, діарея, м’язові спазми. Препарат якої групи був призначений?

Реактиватори холінестерази

Холіноміметики

Іангліоблокатори

Токолітики

Адреноблокатори

2553 / 6307
Для лікування піодермії лікар призначив вакцину, яка виготовлена зі штаму бактерій, виділеного від хворого. До якого типу вакцин належить даний препарат?

Асоційована вакцина

Хімічна вакцина

Автовакцина

Атенуйована вакцина

Генно-інженерна вакцина

2554 / 6307
У біоптаті нирки 45-річного чоловіка, що має хронічну хворобу нирок, виявлено: склероз, лімфоплазмоцитарна інфільтрація стінок мисок та чашок, дистрофія та атрофія канальців. Збережені канальці розширені, розтягнені колоїдоподібними масами, епітелій сплющений ('щитоподібна' нирка). Який діагноз найбільш імовірний?

Іломерулонефрит

Тубуло -інтерстиційний нефрит

Нефросклероз

Хронічний пієлонефрит

Іострий пієлонефрит

2555 / 6307
У водія 60-ти років довготривала хронічна патологія легень призвела до недостатності клапанів легеневої артерії і трикуспідального клапану, недостатності кровообігу за правошлуночко-вим типом. Який тип артеріальної гіпертензії є причиною перевантаження серця об’ємом?

Рефлексогенна гіпертензія

Центрально-ішемічна гіпертензія

Легенева гіпертензія

Сольова гіпертензія

Ессенціальна гіпертензія

2556 / 6307
Під час футбольного матчу між вболівальниками виникла сутичка. На фоні негативних емоцій в одного учасника сутички були розширені зіниці й підвищене серцебиття. Активація якої системи організму забезпечує такі вегетативні зміни при негативних емоціях?

Соматична нервова

Метасимпатична нервова

Парасимпатична нервова

Симпато-адреналова

Гіпоталамо-гіпофізарно-тиреоїдна

2557 / 6307
При дослідженні гостроти слуху в коваля виявили втрату слуху на 50% у діапазоні низьких частот і майже нормальну гостроту слуху в діапазоні високих частот. Порушення яких структур слухової системи призвело до такого стану?

Середня частина кортієвого органу

Барабанна перетинка

М’язи середнього вуха

Кортієв орган - ближче до гелікотреми

Кортієв орган - ближче до овального віконця

2558 / 6307
У хворого 26-ти лет, що приймав високоефективний протитуберкульозний засіб, наприкінці курсу лікування розвинувся периферичний неврит. Який препарат викликав ускладнення?

Флориміцину сульфат

Ізоніазид

Ципрофлоксацин

Рифампіцин

Етамбутол

2559 / 6307
У людини після довільної тривалої затримки дихання збільшилися частота й глибина дихання. Які зміни у складі крові стали причиною цього?

Зниження pC02

Підвищення p02

Підвищення pH

Зниження p02

Підвищення pC02

2560 / 6307
Стеатоз виникає внаслідок накопичення триацилгліцеролів у гепатоцитах. Одним з механізмів розвитку цього захворювання є зменшення утилізації нейтрального жиру ЛПДНЩ. Які ліпотропні речовини попереджують розвиток стеатозу?

Метіонін, B6 , B12

Аланін, B1 , PP

Валін, B3 , B2

Iзолейцин, B1 , B2

Аргінін, B2 , B3

2561 / 6307
Який стан може розвинутися через 15-30 хвилин після повторного введення антигену внаслідок підвищеного рівня антитіл, переважно IgE, які адсорбуються на поверхні клітин-мішеней - тканинних базофілів (тучних клітин) та базофілів крові?

Антитіло-залежна цитотоксичність

Анафілаксія

Імунно-комплексна гіперчутливість

Гіперчутливість уповільненого типу

Сироваткова хвороба

2562 / 6307
У дівчинки 18-ти років через 5 годин після вживання морепродуктів на шкірі тулуба та дистальних відділів кінцівок з’явились маленькі сверблячі папули, які частиною зливаються між собою. Через добу висипка самовільно зникла. Назвіть механізм гіперчутливості, що полягає в основі даних змін:

Клітинна цитотоксичність

Імунокомплексна гіперчутливість

Антитілоопосередкований клітинний цитоліз

Атопія (місцева анафілаксія)

Системна анафілаксія

2563 / 6307
Чоловік 46-ти років, хворий на гіпертонічну хворобу, разом з безсольовою дієтою та з антигіпертензивними засобами довгий час приймав гідрохлортіазид, що зумовило порушення електролітного балансу. Яке порушення внутрішнього середовища виникло у хворого?

Гіпохлоремічний алкалоз

Збільшення об’єму циркулюючої крові

Гіперкаліємія

Гіпермагніємія

Метаболічний ацидоз

2564 / 6307
Синтез і-РНК проходить на матриці ДНК з урахуванням принципу комплементарності. Якщо триплети у ДНК наступні - АТГ-ЦГТ, то відповідні кодони і-РНК будуть:

АТГ-ЦГТ

УАГ-ЦГУ

АУГ-ЦГУ

УАЦ-ГЦА

ТАГ-УГУ

2565 / 6307
При декарбоксилюванні глутамату утворюється нейромедіатор гамма-аміномасляна кислота (ГАМК). При розпаді ГАМК перетворюється у метаболіт циклу лимонної кислоти, яким є:

Фумарат

Сукцинат

Лимонна кислота

Малат

Оксалоацетат

2566 / 6307
У дитячому дошкільному закладі напередодні новорічних свят було зареєстровано спалах кишкової інфекції. При бактеріологічному дослідженні випорожнень хворих патогенних бактерій не було виділено. При електронній мікроскопії виявлено утворення округлої форми з чітким обідком і товстою втулкою, які нагадують колесо. Вкажіть найбільш імовірний збудник даної інфекції:

E.coli

Rotavirus

P.vulgaris

Coxsacki-virus

Adenovirus

2567 / 6307
Жінка 28-ми років доставлена до приймального відділення зі скаргами на різкий біль у нижній ділянці живота. Гінеколог при огляді припустив позаматкову вагітність. Яке анатомічне утворення потрібно пропунктувати для підтвердження діагнозу?

Матково-міхурова заглибина

Передміхуровий простір

Міхурово-прямокишкова заглибина

Прямокишково-маткова заглибина

Міжсигмоподібний закуток

2568 / 6307
При гістологічному дослідженні біоптатів, взятих з потовщених країв виразки шлунка, виявлені невеликі гніздові скупчення різко атипових гіперхромних невеликих епітеліальних клітин, які розташовані серед дуже розвиненої строми. Визначте пухлину:

Недиференційована саркома

Аденома

Скіррозний недиференційований рак

Медулярний рак

Аденокарцинома

2569 / 6307
Жінка 26-ти років поступила у пологове відділєння в терміні вагітності 40 тижнів. Шийка матки розкрита, але скорочення матки відсутнє. Лікар дав засіб гормональної природи для посилення пологової діяльності. Назвіть засіб:

Тестостерон

Гідрокортизон

Естрон

Окситоцин

АКТГ

2570 / 6307
У хворого 47-ми років виникла кишкова коліка на фоні гіпертонічної хвороби. Засоби якої з перерахованих груп найбільш доцільно використати для її купірування у даній ситуації?

Адреноміметики

Міотропні спазмолітики

Симпатоміметики

Антихолінестеразні засоби

М-холиноміметики

2571 / 6307
Віддачу тепла яким шляхом збільшують люди, які приймають прохолодний душ у спекотні дні?

Теплопроведення

Випромінювання

Конвекція

Випаровування поту

2572 / 6307
Пацієнт доставлений в інфекційне відділення з попереднім діагнозом ботулізм. У бактеріологічній лабораторії провели одну з імунних реакцій на вміст ботулінічного токсину в досліджуваному матеріалі. Як називається ця реакція?

Аглютинації

Нейтралізації

Зв’язування комплементу

Подвійної імунної дифузії

Іммобілізації

2573 / 6307
У недоношеного немовляти спостерігається жовтяниця. З нестачею у нього якого ферменту це пов’язано?

Кисла фосфатаза

НАД+ -дегідрогеназа

Каталаза

Лужна фосфатаза

УДФ-трансглюкуронідаза

2574 / 6307
У студента 19-ти років запальний процес у крилопіднебінній ямці. Інфекція поширилась в середню черепну ямку. Через яке анатомічне утворення розповсюдилась інфекція?

Foramen rotundum

Foramen sphenopalatinum

Canalis palatinus minor

Canalis palatinus major

Canalis ptherygoideus

2575 / 6307
У жінки 36-ти років має місце гіповітаміноз B2. Причиною виникнення специфічних симптомів (ураження епітелію, слизових, шкіри, рогівки ока) імовірно є дефіцит:

Цитохромоксидази

Флавінових коферментів

Цитохрому В

Цитохрому С

Цитохрому А1

2576 / 6307
Пацієнт 22-х років скаржиться на підвищену больову чутливість задньої третини язика і на порушення смаку в цій ділянці. Ураження якого нерва має місце?

Під’язиковий

Додатковий

Язикоглотковий

Лицевий

Трійчастий

2577 / 6307
На електронній мікрофотографії представлена клітина нейрального походження. Термінальна частина дендрита клітини має циліндричну форму і складається з 1000 замкнутих мембранних дисків. Яка клітина зображена на мікрофотографії?

Колбочкова нейросенсорна

Нейрон кори великих півкуль

Нейрон передніх рогів спинного мозку

Паличкова нейросенсорна

Нейрон спинномозкового вузла

2578 / 6307
Порушення процесів мієлінізації нервових волокон призводить до неврологічних розладів і розумової відсталості. Такі симптоми характерні для спадкових і набутих порушень обміну:

Фосфатидної кислоти

Холестерину

Сфінголіпідів

Вищих жирних кислот

Нейтральних жирів

2579 / 6307
Хворому хірург видалив порожнинне утворення печінки діаметром 2 см. Встановлено, що стінка порожнини утворена щільною волокнистою сполучною тканиною, вміст являє собою каламутну, густу, жовтувато-зеленуватого кольору рідину з неприємним запахом, яка мікроскопічно складається переважно з поліморфноядерних лейкоцитів. Якому патологічному процесу відповідають такі морфологічні зміни?

Гострий абсцес

Флегмона

Емпієма

Хронічний абсцес

2580 / 6307
У клітині в гранулярній ЕПС відбувається етап трансляції, при якому спостерігається просування і-РНК щодо рибосоми. Амінокислоти з’єднуються пептидними зв’язками в певній послідовності - відбувається біосинтез поліпептиду. Послідовність амінокислот у поліпептиді буде відповідати послідовності:

Нуклеотидів р-РНК

Кодонів і-РНК

Нуклеотидів т-РНК

Антикодонів р-РНК

Антикодонів т-РНК

2581 / 6307
При травмі периферичних нервів виникає м’язова атрофія, кістки стають порозними і ламкими, на шкірі і слизових виникають виразки. Яка функція нервових системи уражується у даному випадку?

Вегетативна

Чутлива

Рухова

Трофічна

Вища нервова діяльність

2582 / 6307
У вагітної жінки 24-х років після тривалого блювання було зареєстровано зниження об’єму циркулюючої крові. Про яку зміну загальної кількості крові може йти мова?

Проста гіповолемія

Олігоцитемічна гіперволемія

Олігоцитемічна гіповолемія

Поліцитемічна гіповолемія

Поліцитемічна гіперволемія

2583 / 6307
Робочий комунальної служби спустився в каналізаційний колодязь без засобів захисту і через деякий час знепритомнів. Лікарями швидкої допомоги діагностовано отруєння сірководнем. Який вид гіпоксії при цьому розвинувся?

Гемічний

Тканинний

Циркуляторний

Перевантажувальний

Респіраторний

2584 / 6307
У хворого 48-ми років на хронічний гломерулонефрит наявні набряки, АТ-210/100 мм рт.ст., ЧСС- 85/хв., межі серця розширені. Який механізм розвитку артеріальної гіпертензії є головним?

Гіперфункція серця

Активація ренин-ангіотензин-альдостеронової системи

Підвищення ОЦК

Підвищення активності симпатичного відділу нервової системи

Підвищення продукції вазопресину

2585 / 6307
Обстежуваний знаходиться у фазі повільнохвильового глибокого сну. Про це свідчить реєстрація на ЕЕГ таких хвиль:

Тета -хвилі

Дельта-хвилі

Бета-хвилі

Альфа-веретена

Альфа-хвилі

2586 / 6307
Потерпілий 19-ти років доставлений до травматологічного відділення з різаною раною трапецієподібного м’яза. Яка з фасцій шиї формує піхву для даного м’яза?

Передхребтова пластинка шийної фасції

Сонна піхва шийної фасції

М ’язова частина передтрахейної пластинки шийної фасції

Вісцеральна частина передтрахейної пластинки шийної фасції

Поверхнева пластинка шийної фасції

2587 / 6307
У хворого діагностовано ГРВІ. У сироватці крові знайдено імуноглобуліни класу М. Який період інфекційного процесу в даному випадку?

Мікро6оносійство

Інкубаційний

Гострий

Реконвалесценція

Продромальний

2588 / 6307
При огляді дівчинки 5-ти років лікар помітив на мигдаликах сірувату плівку. Мікроскопія мазків, пофарбованих за Нейссером показала наявність коринебактерій дифтерії. Яка морфологічна особливість була найбільш суттєвою для встановлення виду збудника?

Полярно розташовані гранули волютину

Наявність спор, діаметр яких перевищує діаметр клітини

Наявність капсули

Розташування клітин збудника у вигляді штахетника

Локалізація збудника всередині макрофагів

2589 / 6307
На розтині тіла хворого, що помер від серцевої недостатності, в інтимі аорти і коронарних судин виявлені жовтого кольору плями і смуги, що вибухають над поверхнею інтими сірувато-жовтуваті бляшки, а також вогнищеве виразкування даних бляшок з наявністю крововиливів, тромбозів і вогнищ звапнування. Для якого захворювання характерні вказані зміни судин?

Гіпертонічна хвороба

Атеросклероз

Сифілітичний мезаортит

Вузликовий періартеріїт

2590 / 6307
У результаті побутової травми у пацієнта виникла значна крововтрата, що супроводжувалося зниженням артеріального тиску. Дія яких гормонів забезпечує швидке відновлення кров’яного тиску, викликаного крововтратою?

Адреналін, вазопресин

Окситоцин

Альдостерон

Кортизол

Статеві

2591 / 6307
Жінка 31-го року хворіє на ВІЛ-інфекцію на стадії СНІД. На шкірі нижніх кінцівок, слизової оболонки піднебіння з’явились рудувато-червоні плями, яскраво-червоні вузлики різних розмірів. Один з вузликів взято на гістологічне дослідження. Виявлено багато хаотично розташованих тонкостінних судин, вистелених ендотелієм; пучки веретеноподібних клітин з наявністю гемосидерину. Яка пухлина розвинулась у хворої?

Фібросаркома

Лімфангіома

Гемангіома

Саркома Капоші

Лімфома Беркіта

2592 / 6307
Анатомічний мертвий простір - це частина повітря, яка залишається в повітроносних шляхах після видиху. В якій із наведених нижче ситуацій відбудеться зменшення анатомічного мертвого простору?

Поворот лежачого пацієнта на лівий бік

Дихання через рот

Поворот лежачого пацієнта на правий бік

Нахил голови вперед

Накладання трахеостоми

2593 / 6307
У чоловіка 29-ти років з ножовим пораненням шиї визначається кровотеча. При первинній обробці рани встановлено, що пошкоджена судина, розташована вздовж латерального краю груднинно-ключично-соскоподібного м’яза. Визначте цю судину:

V jugularis externa

V. jugularis interna

V jugularis anterior

carotis externa

2594 / 6307
У хворого переливання крові ускладнилося розвитком гемотрансфузійного шоку. Назвіть тип алергічної реакції, що лежить в основі даної патології.

Цитотоксичний

Анафілактичний

Імунокомплексний

Рецептороопосередкований

Гіперчутливість сповільненого типу

2595 / 6307
Лікарі-інфекціоністи широко застосовують антибіотики, які інгібують синтез нуклеїнових кислот. Який етап біосинтезу гальмує рифампіцин?

Транскрипція в прокаріотах і еукаріотах

Ініціація транскрипції в прокаріотах

Термінація транскрипції в прокаріотах і еукаріотах

Сплайсинг у прокаріотах і еукаріотах

Реплікація в прокаріотах

2596 / 6307
Підшлункова залоза - орган змішаної секреції. Ендокринно продукує бета-клітинами гормон інсулін, який впливає на обмін вуглеводів. Як він впливає на активність глікогенфосфорилази (ГФ) і глікогенсинтетази (ГС)?

Пригнічує ГФ, активує ГС

Пригнічує ГФ і ГС

Не впливає на активність ГФ і ГС

Активує ГФ і Гс

Активує ГФ, пригнічує ГС

2597 / 6307
Відомо, що введення в організм людини лікарського препарату дикумаролу викликає різке зниження в крові вмісту протромбіну і ряду інших білкових факторів згортання крові. Антивітаміном якого вітаміну є дикумарол?

Вітамін E

Вітамін C

Вітамін P

Вітамін K

Вітамін H

2598 / 6307
До лікарні звернулися робітники з приводу того, що на тілі ними були знайдені паразити сірого кольору, довжиною близько 3 мм. Вони викликали неприємне подразнення шкіри, в місцях укусів виникали свербіж, біль, папули синього кольору, крововиливи. У окремих робітників піднялася температура. На шкірі були крововиливи. Яке захворювання найбільш імовірно?

Короста

Міаз шкірний

Саркоптоз

Педикульоз

Фтіріоз

2599 / 6307
Пацієнту 40-ка років з ознаками емоційної лабільності та пов’язаним з ними порушенням сну призначено нітразепам. З чим пов’язаний снодійний ефект цього засобу?

Інгібування ефектів збуджуючих амінокислот

Стимуляція H1 -гістамінових рецепторів

Активація ГАМК-ергічної системи

Блокада опіатних рецепторів

Пригнічення серотонінергічної нейромедіації

2600 / 6307
У хлопчика 11-ти років вміст холестерину в сироватці крові до 25 ммоль/л. В анамнезі - спадкова сімейна гіперхолестеринемія, причиною якої є порушення синтезу білків-рецепторів до:

Ліпопротеїнів проміжної щільності

Хіломікронів

Ліпопротеїнів високої щільності

Ліпопротеїнів дуже низької щільності

Ліпопротеїнів низької щільності

2601 / 6307
У сироватці крові пацієнта встановлено підвищення активності гіалуронідази. Визначення якого біохімічного показника сироватки крові дозволить підтвердити припущення про патологію сполучної тканини?

Білірубін

Галактоза

Глюкоза

Сечова кислота

Сіалові кислоти

2602 / 6307
При мікроскопії мікропрепарату з виділень хворої хронічним кольповагінітом лікар виявив округлої форми та еліпсоподібні клітини, що брунькуються, розміром 3-6 мкм. Про збудника якої грибкової хвороби може йти мова в даному випадку?

Криптококоз

Кокцидіоз

Кандидоз

Мікроспорія

Епідермофітія

2603 / 6307
У пацієнта з ознаками коліту виділена чиста культура бактерій, яка за морфологічними, культуральними та біохімічними властивостями віднесена до роду шигел. Яку з названих реакцій доцільно застосувати для серологічної ідентифікації культури?

Зв’язування комплементу

Затримки гемаглютинації

Непрямої гемаглютинації

Преципітації

Аглютинації з діагностичними сироватками

2604 / 6307
У пацієнтки 20-ти років встановлено діагноз - СНІД. Які популяції клітин найбільш чутливі до вірусу імунодефіциту людини?

Т-хелпери

Гепатоцити

Епітеліоцити

Ендотеліоцити

В-лімфоцити

2605 / 6307
Після загоєння рани на її місці утворився рубець. Яка речовина є основним компонентом цього різновиду сполучної тканини?

Хондроітин-сульфат

Еластин

Кератансульфат

Гіалуронова кислота

Колаген

2606 / 6307
У спортсмена внаслідок довільної затримки дихання на 40 секунд зросли частота серцевих скорочень та системний артеріальний тиск. Реалізація яких механізмів регуляції зумовлює зміни показників?

Безумовні симпатичні рефлекси

Безумовні парасимпатичні рефлекси

Умовні симпатичні рефлекси

Умовні парасимпатичні рефлекси

2607 / 6307
У малярійного плазмодія - збудника триденної малярії, розрізняють два штами: південний та північний. Вони відрізняються тривалістю інкубаційного періоду: у південного він короткий, а у північного - довгий. В цьому проявляється виражена дія такого добору:

Рушійний

Штучний

Дизруптивний

Стабілізуючий

Статевий

2608 / 6307
Струми надвисокої частоти (НВЧ), що застосовуються у фізіотерапії, не викликають збудження, а зумовлюють лише тепловий ефект на тканини. Як можна пояснити це явище?

Розвивається акомодація

Тривалість стимулу менше порога

Стимул поступає у фазу відносної рефрактерності

Інтенсивність стимулу менше порога

Стимул поступає у фазу абсолютної рефрактерності

2609 / 6307
У чоловіка 35-ти років феохромоцитома. В крові виявляється підвищений рівень адреналіну та норадреналіну, концентрація вільних жирних кислот зросла в 11 разів. Активація якого ферменту під впливом адреналіну підвищує ліполіз?

Ліпопротеїдліпаза

ТАГ-ліпаза

Фосфоліпаза А2

Фосфоліпаза С

Холестеролестераза

2610 / 6307
В результаті травми пошкоджений спинний мозок (з повним розривом) на рівні першого шийного хребця. Що відбудеться з диханням?

Не змінюється

Зростає частота

Припиняється

Зменшується частота

Зростає глибина

2611 / 6307
У людини з хронічним захворюванням нирок порушена їх видільна функція. рН венозної крові становить 7,33. Для корекції кислотно-лужного стану пацієнту доцільно внутрішньовенно ввести розчин:

Бікарбонату натрію

Глюкози

Хлориду кальцію

Хлориду калію

Хлориду натрію

2612 / 6307
При обтураційній жовтяниці і жовчних норицях часто спостерігається протромбінова недостатність. З дефіцитом в організмі якого вітаміну це пов’язано?

K

E

В6

A

C

2613 / 6307
При розтині тіла померлого виявлена гіперплазія кісткового мозку плоских і трубчастих кісток (піоїдний кістковий мозок), спленомегалія (6 кг), гепатомегалія (5 кг), збільшення всіх груп лімфатичних вузлів. Якому захворюванню відповідають виявлені зміни?

Лімфогрануломатоз

Хронічний лімфолейкоз

Хронічний мієлолейкоз

Справжня поліцитемія

Мієломна хвороба

2614 / 6307
У молодої жінки в зв’язку з гострими болями в клубовій ділянці, вилучена маткова труба з локальним розширенням її середньої третини, що заповнено кров’ю. При гістологічному дослідженні в отворі труби знайдені хоріальні ворсини, великі поля еритроцитів з домішком лейкоцитів. Який найбільш імовірний діагноз?

Трубна вагітність

Гнійний сальпінгіт

Крововилив в маткову трубу

Геморагічний сальпінгіт

Гострий гнійний сальпінгіт

2615 / 6307
При дослідженні крові хворого виявлено значне збільшення активності МВ-форм КФК (креатинфосфокінази) та ЛДГ-1. Яку патологію можна припустити?

Інфаркт міокарда

Холецистит

Панкреатит

Ревматизм

Гепатит

2616 / 6307
До лікарні надійшла дитина з діагнозом 'стафілококовий сепсис'. На яке живильне середовище потрібно посіяти кров хворого з метою виділення збудника?

Цукрово-пептонний бульйон

М ’ясо-пептонний агар

Жовчно-сольовий агар

Середовище Бучіна

Середовище Плоскірьова

2617 / 6307
У чоловіка 62-х років видалено нирку, у якій при макроскопічному дослідженні виявлено пухлину у вигляді вузла діаметром до 8 см. Тканина пухлини на розрізі строката, з множинними крововиливами, некрозами. Гістологічно: пухлина складається із світлих клітин, які утворюють альвеолярні і сосочкові структури, помірно виражений інвазивний ріст. У багатьох клітинах пухлини визначаються патологічні мітози, гіперхромні ядра. Діагностуйте виявлену пухлину нирки:

Світлоклітинна аденома

Світлоклітинний рак

Нефробластома

Ацидофільна аденома з малігнізацією

Аденокарцинома

2618 / 6307
Безпосередньо після переходу з горизонтального положення у вертикальне у чоловіка частота серцевих скорочень збільшилась на 15 скорочень за хвилину. Які механізми регуляції переважно зумовлюють цю зміну?

Умовні та безумовні симпатичні рефлекси

Катехоламіни

Безумовні симпатичні рефлекси

Симпатичні рефлекси і катехоламіни

Умовні симпатичт рефлекси

2619 / 6307
У людини з масою 80 кг після тривалого фізичного навантаження об’єм циркулюючої крові зменшився, гематокрит - 50%, загальний білок крові - 80 г/л. Такі показники крові є наслідком, перш за все:

Збільшення вмісту білків у плазмі

Збільшення онкотичного тиску плазми

Втрати води з потом

Збільшення кількості еритроцитів

Збільшення діурезу

2620 / 6307
Після перенесеного запального процесу хворий став помічати слабкість при згинанні кисті в ділянці 1,2, 3 і 4 пальців, зменшення об’єму м’яза підвищення великого пальця. При обстеженні виявлено порушення больової і температурної чутливості в ділянці поверхні долоні 1, 2, 3 і променевої поверхні четвертого пальців. Який із нервів уражений?

Променевий

М’язово-шкірний

Присередній шкірний нерв передпліччя

Ліктьовий

Серединний

2621 / 6307
У чоловіка 53-х років діагностовано сечокам’яну хворобу з утворенням уратів. Цьому пацієнту призначено аллопурінол, який є конкурентним інгібітором ферменту:

Уриділтрансфераза

Ксантиноксидаза

Дигідроурацилдегідрогеназа

Уреаза

Уратоксидаза

2622 / 6307
В експерименті на тварині досліджують серцевий цикл. Закриті усі клапани серця. Якій фазі циклу відповідає такий стан?

Повільного наповнення

Асинхронного скорочення

Протодіастолічний період

Швидкого наповнення

Ізометричного скорочення

2623 / 6307
Крива дисоціації оксигемоглобіну зміщена вправо. Які зміни в організмі людини можуть бути причиною цього?

Алкалоз

Гіпокапнія

Гіпертермія

Збільшення концентрації 2,3-дифосфогліцерату в еритроцитах

Гіпоксемія

2624 / 6307
При операції правосторонньої лобектомії хірург підійшов до кореня правої легені з метою окремого виділення і обробки його складових. Вкажіть порядок розміщення елементів кореня правої легені з якими лікар зустрінеться при виділенні та обробці в напрямку зверху вниз?

Легенева вена, легеневі артерії, бронх

Діафрагмальний нерв, бронх, бронхіальні артерія і вена

Легенева артерія, бронх, легеневі вени

Бронх, легенева артерія, легеневі вени

Бронх, легенева артерія, діафрагмальний нерв

2625 / 6307
Ціаністий калій є отрутою, від якої смерть організму наступає миттєво. На які ферменти в мітохондріях діє ціанистий калій?

НАД+ - залежні дегідрогенази

Флавінові ферменти

Цитохром Р-450

Цитохром В5

Цитохромоксидаза (аа3)

2626 / 6307
При бактеріоскопічному дослідженні носоглоткового слизу дитини 2,5 років, хворої на назофарингіт, виявлені грамнегативні диплококи, схожі за формою на кавові зерна. Які структури організму дитини найбільш імовірно будуть уражені, якщо ці мікроорганізми проникнуть у кров?

Ниркові гломерули

Сечо-статеві шляхи

Лімфатичні вузли

Оболонки мозку

Сєрцєві клапани

2627 / 6307
У хворого під час трахеотомії виникла виражена кровотеча. Яка артерія була травмована під час операції?

A. laryngea superior

A. thyroidea ima

A. thyroidea superior

A. laryngea inferior

A. thyroidea inferior

2628 / 6307
При різноманітних захворюваннях рівень активних форм кисню різко зростає, що призводить до руйнування клітинних мембран. Для запобігання цьому використовують антиоксиданти. Найпотужнішим природнім антиоксидантом є:

Жирні кислоти

Вітамін D

Альфа-токоферол

Гліцерол

Глюкоза

2629 / 6307
При обстеженні чоловіка 40-ка років було встановлено діагноз: гіпохромна анемія. Який препарат треба призначити для лікування?

Ферковен

Вікасол

Пентоксил

Гепарин

Ціанокобаламін

2630 / 6307
У неврологічне відділення з приводу мозкового крововиливу поступив хворий 62-х років. Об’єктивно: стан важкий. Спостерігається наростання глибини і частоти дихання, а потім його зменшення до апное, після чого цикл дихальних рухів відновлюється. Який тип дихання у хворого?

Біота

Кусмауля

Чейна-Стокса

Апнейстичне

Гаспінг-дихання

2631 / 6307
Хворому проведено субтотальну субфасціальну резекцію щитоподібної залози. У післяопераційному періоді тривалий час зберігається охриплість голосу. Який нерв ушкоджено в ході операції?

Зворотній гортанний нерв

Язиковий нерв

Верхній гортанний нерв

Нижньощелепний нерв

Під’язиковий нерв

2632 / 6307
Чоловік 53-х років доставлений у стаціонар у непритомному стані. Об’єктивно: шкіра суха, дихання часте поверхневе, запах ацетону відсутній, Ps- 126/хв., АТ- 70/40 мм рт.ст. Вміст глюкози у крові 48 ммоль/л, реакція сечі на ацетон негативна. Для якого із перелічених станів найбільш характерні симптоми у хворого?

Токсична кома

Лактацидемічна кома

Колапс

Гіперосмолярна кома

Гіперкетонемічна кома

2633 / 6307
У гістопрепараті представлений паренхіматозний орган, що має кіркову і мозкову речовину. Кіркова утворена тяжами епітеліоцитів, між якими проходять кровоносні капіляри. Тяжі формують три зони. Мозкова речовина складається з хромафіноцитів і венозних синусоїдів. Який орган має дані морфологічні ознаки?

Лімфатичний вузол

Наднирник

Нирки

Щитоподібна залоза

Тимус

2634 / 6307
На спеціальному живильному середовищі після посіву виділень гною з уретри виросли ніжні блакитні колонії. При мікроскопії препаратів з них виявлені грамнегативні бобоподібні диплококи. Збудником якої хвороби вони є?

Хламідіоз

Меліоїдоз

Гонорея

Сифіліс

Туляремія

2635 / 6307
Жінка 68-ми років скаржиться на відсутність рухів у правих руці і нозі. Чотири місяці тому перенесла інсульт. Об’єктивно: рухи в правих кінцівках відсутні, тонус м’язів їх підвищений. Який стан спостерігається у хворої?

Моноплегія

Геміплегія

Тетраплегія

Параплегія

2636 / 6307
У пораненого перев’язали плечову артерію в нижній третині плеча. За рахунок яких артерій можливе відновлення кровопостачання передпліччя і кисті?

М’язові гілки плечової артерії

Передні і задні огинаючі артерії плеча

Підлопаткова, передня міжкісткова артерія

Глибока артерія плеча, ліктьовіколатеральні артерії

Грудонадплечова, задня огинаюча артерія плеча

2637 / 6307
У стінці бронху при гістологічному дослідженні чітко визначаються залози, хрящові острівці та багаторядний циліндричний миготливий епітелій. Які бронхи досліджують?

Малі

Термінальні бронхіоли

Головні

Великі

Середні

2638 / 6307
Під час обстеження у хворої встановлене ураження дорсальної частини мосту, порушена функція жування. Ядро якого нерва уражене?

Ядро під’язикового нерва

Подвійне ядро блукаючого нерва

Рухове ядро лицевого нерва

Рухове ядро трійчастого нерва

Мостове ядро трійчастого нерва

2639 / 6307
Глікоген, що надійшов з їжею, гідролізувався у шлунково-кишковому тракті. Який кінцевий продукт утворився в результаті цього процесу?

Галактоза

Фруктоза

Лактоза

Лактат

Глюкоза

2640 / 6307
При бактеріологічному дослідженні сечі хворого на пієлонефрит виділені мікроорганізми, що утворюють на м’ясо-пептонному агарі жовто-зелений пігмент і характерний запах. Як вони називаються?

Ешеріхії

Протеї

Азотобактерії

Псевдомонади

Клебсієли

2641 / 6307
У хворої 28-ми років виявлена пухлина яєчника і показана операція по його видаленню. При здійсненні її необхідно розтинати зв’язку, що з’єднує яєчник з маткою. Яку зв’язку повинен перерізати хірург?

Lig. Suspensorium ovarii

Lig. umbilicale laterale

Lig. Cardinali

Lig. Ovarii proprium

Lig. Latum uteri

2642 / 6307
Для підвищення результатів спортсмену рекомендували застосовувати препарат, який містить у собі карнітин. Який процес в найбільшому ступені активується карнітином?

Синтез стероїдних гормонів

Синтез кетонових тіл

Транспорт жирних кислот у мітохондрії

Тканинне дихання

Синтез ліпідів

2643 / 6307
Хвора 26-ти років звернулася до лікаря зі скаргами на появу у випорожненнях білих плоских рухливих утворів, які нагадують локшину. При лабораторному дослідженні виявлені членики з такою характеристикою: довгі, вузькі, з розміщеним поздовжньо каналом матки, яка має 17-35 бічних відгалужень з кожного боку. Який вид гельмінтів паразитує у кишечнику жінки?

Diphyllobothrium latum

Taenia solium

Taeniarhynchus saginatus

Hymenolepis nana

Echinococcus granulosus

2644 / 6307
У жінки 30-ти років при гістологічному дослідженні шийного лімфовузла виявлено порушення малюнка внаслідок розростання епітеліоїдних, лімфоїдних клітин і макрофагів з ядрами у вигляді підкови, в центрі деяких скупчень клітин - безструктурні ділянки блідо-рожевого кольору з уламками ядер. Для якого захворювання характерні такі зміни?

Лімфогрануломатоз

Туберкульоз

Метастаз пухлини

Актиномікоз

Сифіліс

2645 / 6307
Хворому з прогресуючою м’язовою дистрофією було проведено біохімічне дослідження сечі. Поява якої речовини у великій кількості в сечі може підтвердити захворювання м’язів у даного хворого?

Креатинін

Порфірини

Гіпурова кислота

Креатин

Сечовина

2646 / 6307
При пункційній біопсії в трансплантованій нирці виявлена дифузна інфільтрація строми лімфоцитами, плазмоцитами, лімфобластами, плазмобластами, а також некротичний артеріїт. Який патологічний процес розвинувся у трансплантаті?

Пієлонефрит

Ішемічне пошкодження нирки

Гломерулонефрит

Імунне відторгнення

Тубулонекроз

2647 / 6307
Який механізм тепловіддачі найбільш ефективно спрацьовує при перебуванні людини в умовах 80% вологості повітря та температурі навколишнього середовища +35oC?

Потовиділення

Теплопровідність

Конвекція

Радіація

2648 / 6307
При обстеженні чоловіка 45-ти років, що тривалий час перебував на рослинній дієті, виявлено негативний азотистий баланс. Яка особливість раціону стала причиною цього явища?

Надмірна кількість вуглеводів

Недостатня кількість білків

Недостатня кількість жирів

Недостатня кількість жирів і білків

Надмірна кількість води

2649 / 6307
До клініки поступив чоловік 40-ка років, якого укусила гадюка. Де переважно буде проходити гемоліз еритроцитів у цьому випадку?

У паренхімі нирок

У клітинах селезінки

У клітинах печінки

У кістковому мозку

У кровоносному руслі

2650 / 6307
При клінічному обстеженні пацієнта 70-ти років виявлено порушення рухових функцій, що пов’язано з віковими змінами у гіаліновому хрящі. Які вікові зміни викликали обмеження рухів у суглобах?

Відкладання солей кальцію в міжклітинній речовині

Потовщення охрястя

Збільшення гідрофільності основної речовини

Збільшення кількості хрящових клітин

Збільшення кількості ізогенних груп

2651 / 6307
Хворому 42-х років для лікування бактеріальної пневмонії призначено амоксицилін. Вкажіть, який механізм бактерицидної дії препарату?

Пригнічення внутрішньоклітинного синтезу білка

Пригнічення SH-груп ферментів мікроорганізмів

Пригнічення синтезу клітинної стінки мікроорганізмів

Порушення проникливості цитоплазматичної мембрани

Антагонізм із параамінобензойною кислотою

2652 / 6307
У жінки обмежений кровотік у нирках, підвищений артеріальний тиск. Гіперсекреція якого гормону зумовила підвищення тиску?

Ренін

Еритропоетин

Адреналін

Вазопресин

Норадреналін

2653 / 6307
У чоловіка 41-го року відзначаються періодичні напади серцебиття (пароксизми), сильне потовиділення, напади головного болю. При обстеженні виявлена гіпертензія, гіперглікемія, підвищення основного обміну, тахікардія. При якій патології наднирників спостерігається подібна картина?

Гіперфункція кори наднирників

Гіперфункція мозкового шару

Первинний альдостеронізм

Гіпофункція кори наднирників

Гіпофункція мозкового шару

2654 / 6307
Пацієнт 16-ти років, що страждає на хворобу Іценко-Кушінга, консультований з приводу надмірної ваги тіла. При опитуванні з’ясувалося, що енергетична цінність спожитої їжі складає 1700-1900 ккал/добу. Яка провідна причина ожиріння у даному випадку?

Гіподинамія

Надлишок глюкокортикоїдів

Нестача інсуліну

Нестача глюкокортикоїдів

Надлишок інсуліну

2655 / 6307
У людини з нападом бронхоспазму необхідно зменшити вплив блукаючого нерва на гладеньку мускулатуру бронхів. Які мембранні циторецептори доцільно заблокувати для цього?

α- та β-адренорецептори

Н-холінорецептори

β-адренорецептори

М-холінорецептори

α-адренорецептори

2656 / 6307
У новонародженої дитини виявлено наступну патологію: аномалія розвитку нижньої щелепи та гортані, що супроводжується характерними змінами голосу, а також мікроцефалія, вада серця, чотирьохпалість. Найбільш імовірною причиною таких аномалій є делеція:

21-ої хромосоми

Короткого плеча 7-ої хромосоми

Короткого плеча 9-ої хромосоми

Короткого плеча 5-ої хромосоми

Короткого плеча 11-ої хромосоми

2657 / 6307
У жінки з важкою інтоксикацією, зумовленою сепсисом, який і послужив безпосередньою причиною смерті, на розтині виявлене 'тигрове серце'. Мікроскопічно в цитоплазмі кардіоміоцитів виявлені ліпіди. Який морфогенетичний механізм розвитку переважно лежить в основі даної дистрофії?

Трансформація

Декомпозиція

Інфільтрація

Спотворений синтез

2658 / 6307
Чоловік 42-х років з ураженням ліктьового нерва не може звести до серединної лінії II і V пальці. Функція яких м’язів при цьому порушена?

Тильні міжкісткові м’язи

Відвідний м’яз мізинця

Червоподібні м’язи

Короткий долонний м’яз

Долонні міжкісткові м’язи

2659 / 6307
Хворому з гострою недостатністю надниркових залоз був призначений лікарський препарат, після застосування якого у нього з’явилися скарги на біль у кістках (двічі були переломи), часті простудні хвороби, набряки, повільне загоювання ран. Який препарат міг спричинити такі явища?

Ретаболіл

Тестостерон

Преднізолон

Естріол

Спіронолактон

2660 / 6307
У подружжя народився син, хворий на гемофілію. Батьки здорові, а дідусь за материнською лінією також хворий на гемофілію. Визначте тип успадкування ознаки.

Рецесивний, зчеплений зі статтю

Аутосомно-домінантний

Аутосомно-рецесивний

Неповне домінування

Домінантний, зчеплений зі статтю

2661 / 6307
Хворому з діагнозом цукровий діабет II типу ендокринолог призначив глібенкламід. Вкажіть основний механізм дії цього засобу:

Стимулює секрецію інсуліну бета-клітинами острівців Лангерганса

Активує транспорт глюкози в клітину

Пригнічує глюконеогенез

Підсилює метаболізм глюкози

Підсилює захоплення глюкози периферичними тканинами

2662 / 6307
У мазку з нальоту на мигдаликах хворого з підозрою на дифтерію виявлено палички синього кольору з потовщеннями на полюсах. Який метод фарбування мазків було використано?

Грама

Буррі

Леффлера

Гінса

Нейссера

2663 / 6307
Дитина народилася в стані асфіксії. Який препарат необхідно ввести новонародженому для стимуляції дихання?

Лобелін

Атропін

Етимізол

Празозин

Прозерин

2664 / 6307
Хворий похилого віку страждає на хронічний закреп, в основі якого лежить гіпотонія товстого кишечнику. Який препарат слід призначити хворому?

Натрію сульфат

Касторова олія

Прозерин

Бісакодил

Ацеклідин

2665 / 6307
У пацієнта 38-ми років при прогресуванні стафілококового гнійного періодонтиту виникло гнійне запалення кістково-мозкових просторів альвеолярного відростка, а потім тіла нижньої щелепи. Мікроскопічно кісткові балки витончені, вогнища некрозу, кісткові секвестри, оточені сполучнотканинною капсулою. Який найбільш імовірний діагноз?

Гнійний періостит

Хронічний остеомієліт

Гострий остеомієліт

Пародонтома

Хронічний фіброзний періостит

2666 / 6307
У немовляти виявлено мікроцефалію. Лікарі вважають, що це пов’язано з застосуванням жінкою під час вагітності актиноміцину Д. На які зародкові листки в першу чергу подіяв цей тератоген?

Ентодерма

Ентодерма та мезодерма

Усі листки

Ектодерма

Мезодерма

2667 / 6307
У чоловіка 65-ти років впродовж 15-ти років була виражена артеріальна гіпертензія. Останнім часом систолічний тиск почав знижуватися, а діастолічний залишився підвищеним. Який гемодинамічний тип артеріальної гіпертензії у хворого?

Нормокінетичний

Еукінетичний

Гіперкінетичний

Гіпокінетичний

2668 / 6307
У жінки 52-х років при обстеженні було виявлено зниження кількості еритроцитів у крові та підвищення рівня вільного гемоглобіну в плазмі крові (гемоглобінемія). КП- 0,85. Який вид анемії спостерігається у хворої?

Хронічна постгеморагічна

Спадкова гемолітична

Гостра постгеморагічна

Анемія внаслідок порушення ери-тропоезу

Набута гемолітична

2669 / 6307
У бактеріологічній лабораторії проводиться дослідження якості питної води. Її мікробне число виявилося близько 100. Які мікроорганізми враховувалися при цьому?

Ентеропатогенні бактерії та вїруси

Бактерії групи кишкової палички

Умовно-патогенні мікроорганізми

Всі бактерії, що виросли на живильному середовищі

Бактерії, патогенні для людей та тварин

2670 / 6307
В гістологічному препараті визначається орган, стінка якого утворена трьома оболонками. Внутрішня оболонка складається з ендотелію і тонкого підендотеліального шару. Зовнішня оболонка найтовстіша. Який орган представлений у препараті?

Матка

Серце

Вена

Артерія

Сечовід

2671 / 6307
Лімфоцит уражений ретровірусом ВІЛ (СНІД). В цьому випадку напрямок потоку інформації в клітині буде:

і-РНК -> поліпептид -> ДНК

РНК -> ДНК -> і-РНК -> поліпептид

Поліпептид -> РНК -> ДНК -> і-РНК

ДНК -> і-РНК -> поліпептид -> ДНК

ДНК -> поліпептид -> і-РНК

2672 / 6307
Під час судово-медичної експертизи жінки, яка загинула у автокатастрофі, знайдено ембріон на стадії ранньої гаструли. Назвіть місце його локалізації при умові нормального розвитку:

Маткова частина яйцепроводу

Яєчник

Черевна порожнина

Стінка матки

Ампульна частина яйцепроводу

2673 / 6307
В основі розвитку імунних і алергічних реакцій організмом застосовуються однакові механізми відповіді імунної системи на антиген. Визначте основну відмінність алергічних реакцій від імунних?

Шляхи потрапляння в організм

Спадкова схильність

Розвиток пошкодження тканин

Особливість будови антигенів

Кількість антигену, що потрапляє

2674 / 6307
У пацієнта внаслідок черепно-мозкової травми знижена шкірна чутливість. Яка ділянка кори великого мозку може бути ураженою?

Передня центральна звивина

Задня центральна звивина

Поясна звивина

Потилична ділянка

Лобна ділянка кори

2675 / 6307
При розтині трупа чоловіка 47-ми років, померлого раптово, в інтимі черевного відділу аорти знайдені осередки жовтого кольору у вигляді плям та смуг, що не вибухають над поверхнею інтими. При фарбуванні суданом ІІІ спостерігається жовтогаряче забарвлення. Для якої стадії атеросклерозу характерні такі зміни?

Стадія утворення атероматозної виразки

Атерокальцинозу

Атероматозу

Ліпосклерозу

Ліпоїдозу

2676 / 6307
При огляді ротової порожнини стоматолог виявив появу у дитини перших великих нижніх кутніх зубів. Який вік дитини?

8-9 років

4-5 років

6-7 років

10-11 років

12-13 років

2677 / 6307
У батьків, хворих на гемоглобінопатію (аутосомно-домінантний тип успадкування), народилася здорова дівчинка. Які генотипи батьків?

Обоє гетерозиготні за геном гемоглобінопатії

Обоє гомозиготні за геном гемоглобінопатії

У обох батьків ген гемоглобінопатії відсутній

Мати гетерозиготна за геном гемоглобінопатії, у батька цей ген відсутній

Батько гетерозиготний за геном гемоглобінопатії, у матері цей ген відсутній

2678 / 6307
Хворому з артеріальною гіпертензією було призначено один з антигіпертензивних засобів. Артеріальний тиск нормалізувався, однак хворого почав турбувати постійний сухий кашель. Який з перерахованих препаратів має таку по6ічну дію?

Клофелін

Лізиноприл

Резерпін

Ніфедипін

Анаприлін

2679 / 6307
У 30-річного померлого наркомана, який страждав на ВІЛ-інфекцію, при патоморфологічному дослідженні виявлено, що обидві легені ущільнені, темно-бордовосірі, мало повітряні, міжальвеолярні перетинки густо інфільтровані лімфоцитами, частина альвеолоцитів трансформовані у великі клітини з центрально розташованим круглим ядром зі світлим обідцем, які нагадують 'совине око'. Яка опортуністична інфекція викликала пневмонію у даного хворого?

Пневмоциста карінії

Цитомегаловірус

Герпес-вірус

Атипова мікобактерія

Токсоплазма

2680 / 6307
У пацієнта у результаті тривалого блювання відбувається значна втрата шлункового соку, що є причиною порушення кислотно-лужного стану в організмі. Яка з перерахованих форм порушення кислотно-лужного стану має місце?

Газовий алкалоз

Газовий ацидоз

Негазовий алкалоз

Негазовий ацидоз

Метаболічний ацидоз

2681 / 6307
Хвора звернулася зі скаргами на болі у правій латеральній ділянці живота. Під час пальпації визначається щільне, нерухоме, пухлиноподібне утворення. У ділянці якого відділу травної трубки можлива наявність пухлини?

Colon transversum

Colon sigmoideum

Colon descendens

Caecum

Colon ascendens

2682 / 6307
На практичному занятті з мікробіології студентам запропоновано пофарбувати суміш бактерій за методикою Грама та пояснити механізм фарбування. Які морфологічні структури бактерій зумовлюють грамнегативне та грампозитивне фарбування бактерій?

Цитоплазма

Капсула

Джгутики

Клітинна стінка

ЦПМ

2683 / 6307
Студентка 22-х років звернулася до лікаря із скаргами на підвищення температури тіла до 38oC, слабкість, біль у горлі. Об’єктивно: язик вкритий білим нальотом. Які гістологічні структури язика беруть участь в утворенні цього нальоту?

Епітелій листоподібних сосочків

Сполучнотканинна основа всіх сосочків язика

Епітелій ниткоподібних сосочків

Епітелій грибоподібних сосочків

Епітелій жолобкуватих сосочків

2684 / 6307
Досліджуються рецептори, інформація від яких прямує до кори без участі таламусу. Які це рецептори?

Слухові

Смакові

Зорові

Дотикові

Нюхові

2685 / 6307
Препарат 'Гептрал', який використовують при хворобах печінки, містить Sаденозилметіонін. Ця активна амінокислота бере участь у синтезі:

Холестерину

Жовчних кислот

Фосфоліпідів

Триацилгліцеролів

Гему

2686 / 6307
Хвора 39-ти років, з цукровим діабетом в анамнезі, госпіталізована до клініки у прекоматозному стані кетоацидотичного типу. Збільшення вмісту якого метаболіту до цього призвело?

Ацетоацетат

Малонат

Цитрат

Альфа-кетоглутарат

Аспартат

2687 / 6307
В хірургічне відділення доставлено пацієнта з рiзаною раною медіального краю передпліччя. При обстеженні виявлено, що в хворого перерізано ліктьовий м’яз-згинач зап’ястка і ліктьовий м’яз-розгинач зап’ястка. Які з рухів кисті будуть порушені у хворого?

Приведення

Відведення

Згинання

Розгинання і відведення

Розгинання

2688 / 6307
Для діагностування деяких хромосомних хвороб використовують визначення статевого хроматину. Назвіть хворобу, при якій потрібне це визначення:

Хвороба Брутона

Гемофілія

Синдром Шерешевського-Тернера

Трисомія Е

Хвороба Дауна

2689 / 6307
У хворої після видалення матки розвинулась гостра анурія (немає виділення сечі). Які анатомічні структури найімовірніше було пошкоджено при операції?

Цибулинно-губчастий м’яз

Сечівник

Сечоводи

Зовнішній сфінктер сечівника

Внутрішній сфінктер сечівника

2690 / 6307
Дихальний коефіцієнт у хворого складає 0,7. Це свідчить, що у клітинах людини переважає:

Окислення білків

Окислення жирів

Змішане окислення жирів та білків

Змішане окислення жирів та вуглеводів

Окислення вуглеводів

2691 / 6307
У чоловіка при обстеженні виявлено порушення кровообігу міокарда лівого передсердя. У басейні якої артерії відбулись порушення кровообігу?

Права вінцева

Ліва вінцева

Права та ліва вінцеві

Передня міжшлуночкова гілка лівої вінцевої артерії

2692 / 6307
Лікар-цитогенетик при виготовленні метафазної пластинки обробив культуру лейкоцитів гіпотонічним (0,56%) розчином хлориду калію. Після цього відбулося набухання клітин і розрив клітинної мембрани за рахунок надходження води до клітини. Який механізм транспорту має місце в цьому випадку?

Фагоцитоз

Дифузія

Ендоосмос

Піноцитоз

Полегшена дифузія

2693 / 6307
При зовнішньому дослідженні трупа чоловіка 69-ти років, який помер 4 години тому, патологоанатом відмітив, що м’язи померлого мають дуже щільну консистенцію, суглоби згинаються та розгинаються важко. Як називається ця патологоанатомічна ознака смерті?

Трупне висихання

Трупне розкладання

Трупне заклякання

Трупне охолодження

Трупні гіпостази

2694 / 6307
Швидкість проведення збудження нервовими волокнами становить 120 м/сек. Який з наведених чинників, перш за все, забезпечує таку швидкість?

Наявність мієлінової оболонки

Малий поріг деполяризації

Велика амплітуда потенціалу дії

Великий фактор надійності

Великий потенціал спокою

2695 / 6307
У померлої дитини 3-х років за життя мала місце менінгіальна симптоматика, На розтині в м’якій мозковій оболонці макроскопічно виявлені просоподібні вузлики, які мікроскопічно представлені осередком казеозного некрозу з валами епітеліоїдних, лімфоїдних клітин, між якими зустрічаються великі клітини з ядрами на периферії у вигляді півмісяця. Який найбільш імовірний менінгіт у дитини?

Туберкульозний

Грипозний

Бруцельозний

Сифілітичний

Менінгококовий

2696 / 6307
Лікар записав в історії хвороби, що у хворого дихання поверхневе (знижена глибина дихання). Це означає, що зменшеним є такий показник зовнішнього дихання:

Дихальний об’єм

Ємність вдиху

Життєва ємність легень

Функціональна залишкова ємність

Хвилинний об’єм дихання

2697 / 6307
При проведенні амніоцентезу в клітинах плоду виявлено по 2 тільця статевого хроматину (тільця Барра). Для якого захворювання характерна дана ознака?

Синдром Шерешевського-Тернера

Синдром Клайнфельтера

Синдром Дауна

Синдром Патау

Трисомія X

2698 / 6307
Відомо, що типові патологічні процеси розвиваються за однаковими закономірностями в різних органах і тканинах та у різних видів тварин. Яке з перерахованих явищ можна віднести до типового патологічного процесу?

Гіпертонічна хвороба

Інфаркт міокарда

Пухлина

Непрохідність кишківника

Туберкульоз

2699 / 6307
У хворого 49-ти років на гострий панкреатит виникала загроза некрозу підшлункової залози, що супроводжувалось надходженням у кров і тканини активних панкреатичних протеїназ і розщеплення тканинних білків. Які захисні фактори організму можуть інгібувати ці процеси?

Імуноглобуліни

Гемоплексин, гаптоглобін

Кріоглобулін, інтерферон

α2-макроглобулін, α1-антитрипсин

Церулоплазмін, трансферин

2700 / 6307
У 19-місячної дитини із затримкою розвитку та проявами самоагресії, вміст сечової кислоти в крові -1,96 ммоль/л. При якому метаболічному порушенні це спостерігається?

Синдром набутого імунодефіциту

Хвороба Гірке

Подагра

Синдром Леша-Ніхана

Хвороба Іценко-Кушінга

2701 / 6307
Чоловік 53-х років звернувся зі скаргами на гострий біль у правому підребер’ї. При огляді лікар звернув увагу на пожовтілі склери хворого. Лабораторні аналізи показали підвищену активність АЛТ та негативну реакцію на стеркобілін у калі. Для якого захворювання характерні такі симптоми?

Гемолітична жовтяниця

Хронічний гастрит

Хронічний коліт

Гепатит

Жовчнокам’яна хвороба

2702 / 6307
Основними тригерами, що включають ефекторні системи клітини у відповідь на дію гормонів, є протеїнкінази, які змінюють каталітичну активність певних регуляторних ферментів шляхом АТФ-залежного фосфорилювання. Який із наведених ферментів є активним у фосфорильованій формі?

Глікогенсинтаза

ГОМГ-КоА-редуктаза

Піруваткіназа

Глікогенфосфорилаза

Ацетил-КоА-карбоксилаза

2703 / 6307
У пацієнта встановлено порушення синтезу та виділення вазопресину. В якому відділі нефрона найбільше порушиться процес сечоутворення?

Тонка частина петлі Генле

Збірна трубочка

Клубочок

Товста частина петлі Генле

Проксимальний звивистий каналець

2704 / 6307
При обстеженні 2-х місячної дитини педіатр звернула увагу, що плач дитини нагадує котячий крик. Діагностовані мікроцефалія і вада серця. За допомогою цитогенетичного метода з’ясований каріотип дитини 46, XX, 5р-. Дане захворювання є наслідком такого процесу:

Транслокація

Дуплікація

Плейотропія

Делеція

Інверсія

2705 / 6307
В експерименті подразнюють гілочки симпатичного нерва, які іннервують серце. Це призвело до збільшення сили серцевих скорочень, тому що через мембрану типових кардіоміоцитів збільшився:

Вхід іонів кальцію та калію

Вхід іонів кальцію

Вихід іонів кальцію

Вхід іонів калію

Вихід іонів калію

2706 / 6307
У пацієнта 60-ти років виявлено збільшення порогу сприймання звуків високої частоти. Зміна функцій яких структур слухового аналізатора зумовлює виникнення цього порушення?

Євстахієвої труби

Барабанної перетинки

Органу Корті ближче до овального віконця

М’язів середнього вуха

Органу Корті ближче до гелікотре-ми

2707 / 6307
У пацієнта виявлено кишкову непрохідність, знижений апетит, нудоту, блювання. На основі проведеної лабораторної діагностики встановлено дифілоботріоз. Зараження відбулось через вживання:

Яловичини

Свинини

Риби

Яєць

Крабів та раків

2708 / 6307
В інфекційну клініку доставлено хворого із проявами лихоманки, що повторюється вдруге з інтервалом 2 дні. В краплі крові, зафарбованій за Романовським-Гімзою, виявлено звивисті клітини синьо-фіолетового кольору. Який мікроорганізм викликав захворювання?

Leptospira interrogans

Borrelia recurentis

Plasmodium vivax

Treponema pallidum

Ricketsia typhi

2709 / 6307
У юнака 18-ти років діагностовано хворобу Марфана. При дослідженні встановлено: порушення розвитку сполучної тканини, будови кришталика ока, аномалії серцево- судинної системи, арахнодактилія. Яке генетичне явище зумовило розвиток цієї хвороби?

Кодомінування

Неповне домінування

Множинний алелізм

Комплементарність

Плейотропія

2710 / 6307
При алкаптонурії відбувається надмірне виділення гомогентизинової кислоти із сечею. З порушенням метаболізму якої амінокислоти пов’язано виникнення цього захворювання?

Фенілаланін

Аланін

Аспарагін

Тирозин

Метіонін

2711 / 6307
Чоловік 30-ти років, водій за професією, страждає на алергічний риніт із загостренням у весняний період. Лікар призначив хворому антигістамінний засіб з незначним седативним ефектом та тривалістю дії близько 24 годин. Який із перерахованих засобів було призначено?

Гепарин

Димедрол

Окситоцин

Вікасол

Лоратадин

2712 / 6307
Хворий госпіталізований з небезпекою поширення запального процесу з потиличної ділянки в порожнину черепа. Крізь яке анатомiчне утворення можливе це поширення?

Остистий отвір

Круглий отвір

Виростковий канал

Овальний отвір

Тім ’яний отвір

2713 / 6307
Після введення пеніцилину в хворого розвився набряк Квінке. Який препарат екстреної терапії необхідно ввести хворому?

Сульфацил-натрій

Рифампіцин

Преднізолон

Но-шпа

Аскорбінова кислота

2714 / 6307
Під час електронномікроскопічного дослідження біоптату гепатоцитів на біліарному полюсі виявлено велику кількість плоских цистерн, сплющених у центральній частині й розширених на периферії, та дрібних міхурців із секреторними гранулами. Назвіть цю структуру:

Комплекс Гольджі

Піноцитозні міхурці

Мікротрубочки

Лізосома

Ендоплазматична сітка

2715 / 6307
У кішки з децеребраційною ригідністю потрібно знизити тонус м’язів. Цього можна досягти шляхом:

Подразнення отолітових вестибулорецепторів

Подразнення вестибулярних ядер Дейтерса

Подразнення ампулярних вестибулорецепторів

Подразнення вестибулослухового нерва

Руйнування вестибулярних ядер Дейтерса

2716 / 6307
При гастробіопсії у хворого встановлена метаплазія поверхневого епітелію слизової оболонки, який замість циліндричного набув вигляду кишкового. Разом з тим спостерігається склероз на місці залоз слизової оболонки та лімфогістіоцитарна інфільтрація. Про яке захворювання шлунка можна думати?

Корозивний гастрит

Хронічний гастрит з ураженням залоз без атрофії

Ерозивний гастрит

Поверхневий хронічний гастрит

Хронічний атрофічний гастрит

2717 / 6307
При дослідженні імунного статусу людини обов’язково визначають кількість імуноглобулінів різних класів. Яка з перерахованих реакцій використовується для цього?

Бласттрансформації

Ланцюгово-полімеразна

Подвійної імунодифузії

Радіальної імунодифузії

Оберненої непрямої гемаглютинації

2718 / 6307
У хворого, що страждає на спадкову хворобою Хартнупа, спостерігаються пелагроподібні ураження шкіри, порушення розумового розвитку в результаті нестачі нікотинової кислоти. Причиною цього захворювання є порушення такого процесу:

Декарбоксилювання триптофану

Всмоктування і реабсорбція в нирках триптофану

Трансамінування фенілаланіну

Всмоктування і реабсорбція в нирках метіоніну

Всмоктування і реабсорбція цистеїну

2719 / 6307
Хворий помилково прийняв надмірну дозу тироксину. До яких змін секреції тиреоліберину та тиреотропіну це призведе?

Секреція гормонів зменшиться

Секреція тиреотропіну збільшиться, тиреоліберину - зменшиться

Секреція тиреоліберину збільшиться, тиреотропіну - зменшиться

Секреція гормонів збільшиться

Змін секреції гормонів не буде

2720 / 6307
Проводять дуоденальне зондування. Що із наведеного доцільно ввести людині під шкіру, щоб суттєво збільшити надходження до дванадцятипалої кишки жовчі?

Соматостатин

Секретин

Гастрин

Нейротензин

Холецистокінін-панкреозимін

2721 / 6307
У чоловіка 32-х років, хворого на пневмонію, спостерігається закупорка харкотинням дихальних шляхів. В організмі хворого при цьому буде розвиватися така зміна кислотнолужної рівноваги:

Змін не буде

Респіраторний ацидоз

Респіраторний алкалоз

Метаболічний ацидоз

Метаболічний алкалоз

2722 / 6307
При аналізі ЕКГ виявлено випадіння деяких серцевих циклів PQRST. Наявні зубці і комплекси не змінені. Назвіть вид аритмії:

Синоатріальна блокада

Атріовентрикулярна блокада

Миготлива аритмія

Передсердна екстрасистола

Внутрішньопередсердна блокада

2723 / 6307
У хворої встановлено порушення виділення тиреотропного гормону гіпофіза. Зі зниженням функцій якої частки гіпофіза це пов’язано?

Lobus anterior

Infundibulum

Lobus posterior

Pars intermedia

2724 / 6307
У пацієнтки 23-х років після використання нової губної помади з’явилися набряк і свербіння губ, а через 2 дні - кірочки на червоній облямівці губ. Який тип алергічної реакції найбільш імовірний?

Стимулюючий

Цитотоксичний

Анафілактичний

Сповільнений

Імунокомплексний

2725 / 6307
У хворого лікар виявив накопичення рідини в плевральній порожнині справа над куполом діафрагми. У якому анатомічному утворі накопичилась рідина?

Реброво-діафрагмальний лівий синус

Діафрагмально-середостінний синус

Реброво-середостінний синус

Реброво-діафрагмальний правий синус

2726 / 6307
При регенерації епітелію слизової оболонки порожнини рота (розмноження клітин) відбулася реплікація (авторепродукція) ДНК за напівконсервативним механізмом. При цьому нуклеотиди нової нитки ДНК є комплементарними до:

Інтронних ділянок гену

Материнської нитки

Змістовних кодонів

Ферменту РНК-полімерази

Ферменту ДНК-полімерази

2727 / 6307
В результаті травми порушено цілісність переднього корінця спинного мозку. Які відростки яких нейронів при цьому пошкоджені?

Дендрити вставних нейронів

Дендрити чутливих нейронів

Аксони рухових нейронів

Дендрити рухових нейронів

Аксони чутливих нейронів

2728 / 6307
При визначенні повітряної та кісткової провідності звуку було встановлено, що у пацієнта ліве вухо краще сприймає звук при кістковому його проведенні, що могло бути пов’язане з захворюванням:

Внутрішнього вуха зліва

Зовнішнього вуха справа

Середнього вуха справа

Внутрішнього вуха справа

Середнього вуха зліва

2729 / 6307
Чоловік 55-ти років доставлений до реанімаційного відділення без свідомості. Зі слів родичів стало відомо, що хворий помилково випив метиловий спирт. Який антидот необхідно використати в даному випадку?

Протаміну сульфат

Ацетилцистеїн

Налоксон

Етанол

Тетурам

2730 / 6307
Хвора на ревматоїдний артрит після трьохтижневого лікування преднізолоном почала скаржитись на перебої в ро6оті серця. З чим пов’язаний розвиток даного небажаного ефекту препарату?

Гіпокаліємія

Гіперурікемія

Гіперглікемія

Гіперкаліємія

Гіпоглікемія

2731 / 6307
Для профілактики атеросклерозу, ішемічної хвороби серця, порушень мозкового кровообігу рекомендується споживання жирів із високим вмістом поліненасичених жирних кислот. Однією з таких жирних кислот є:

Стеаринова

Лінолева

Олеїнова

Лауринова

Пальмітоолеїнова

2732 / 6307
Обстежуваний знаходиться у фазі швидкохвильового сну. При цьому на ЕЕГ реєструється:

δ-хвиля

β-хвиля

α-хвиля

θ-хвиля

α-веретено

2733 / 6307
Внаслідок ДТП у потерпілої 37-ми років виникло неутримання сечі. Які сегменти спинного мозку пошкоджені?

Th1 — L1

L1 — L2

S2 - S4

Th1 — Th5

Th2 — Th5

2734 / 6307
Жінці 26-ти років, хворій на бронхіт, призначили засіб етіотропної терапії - антибіотик широкого спектру дії. Який це препарат?

Амброксол

Інтерферон

Доксициклін

Дексаметазон

БЦЖ-вакцина

2735 / 6307
При обстеженні жінки 56-ти років, що хвора на цукровий діабет 1-го типу, виявлене порушення білкового обміну, що при лабораторному дослідженні крові проявляється аміноацидемією а клінічно - уповільненням загоєння ран і зменшенням синтезу антитіл. Який з перерахованих механізмів викликає розвиток аміноацидемії?

Підвищення онкотичного тиску в плазмі крові

Гіперпротеїнемія

Підвищення протеолізу

Збільшення ліпопротеїдів високої щільності

Зменшення концентрації амінокислот у крові

2736 / 6307
Хворому на туберкульоз легень було призначено препарат з групи антибіотиків, що відноситься до високоефективних протитуберкульозних засобів. Який препарат був призначений?

Рифампіцин

Цефалексин

Тетрациклін

Циклосерин

ПАСК

2737 / 6307
При дослідженні тимуса дитини 5-ти років, що померла від гострої деструктивної стафілококової пневмонії, виявлено зменшення маси залози до 3,0 г. При гістологічному дослідженні в тимусі знайдено: зменшення часточок залози, значне зменшення кількості лімфоцитів, з колапсом строми часточок, інверсія шарів, кистоподібне збільшення тілець Гассаля. Який з перелічених діагнозів найбільш імовірний?

Дисплазія тимусу

Акцидентальна інволюція тимусу

Тимомегалія

Агенезія тимусу

Гіпоплазія тимусу

2738 / 6307
При обстеженні хворого окуліст виявив збільшення часу адаптації ока до темряви. Нестача якого вітаміну може бути причиною такого симптому?

B2

B6

K

C

A

2739 / 6307
Хворому 63-х років з атонією сечового мiхура лікар призначив препарат, дозу якого хворий самостійно збільшив. З’явились підвищене потовиділення, салівація, діарея, м’язові спазми. Препарат якої групи був призначений?

Адреноблокатори

Токолітики

Гангліоблокатори

Холіноміметики

Реактиватори холінестерази

2740 / 6307
На мікропрепараті плівки пухкої сполучної тканини видно клітину овальної форми, зі світлою цитоплазмою і великим ядром що містить специфічний малюнок гетерохроматину у вигляді циферблату годинника (або спиць колеса). Яка клітина у полі зору?

Адипоцит

Плазмоцит

Фібробласт

Тканинний базофіл

Макрофаг

2741 / 6307
Для лікування піодермії лікар призначив вакцину, яка виготовлена зі штаму бактерій, виділених від хворого. До якого типу вакцин належить даний препарат?

Асоційована вакцина

Атенуйована вакцина

Генноінженерна вакцина

Аутовакцина

Хімічна вакцина

2742 / 6307
У хворого скарги на загальну слабкість, підвищену втому, зниження апетиту і маси тіла. В анамнезі часті пневмонії. На підставі клінічних даних та результатів дослідження периферійної крові у нього діагностовано хронічний лімфолейкоз. Які дегенеративні зміни лейкоцитів характерні для даного захворювання?

Тільця Князькові-Деле

Токсична зернистість

Палички Ауера

Тіні Боткіна-Гумпрехта

Зерна Амато

2743 / 6307
У чоловіка 72-х років довготривала хронічна патологія легень призвела до недостатності клапанів легеневої артерії і трикуспідального клапану, недостатності кровообігу за правошлуночковим типом. Який тип артеріальної гіпертензії є причиною перевантаження серця об’ємом?

Легенева гіпертензія

Центрально-ішемічна гіпертензія

Есенціальна гіпертензія

Рефлексогенна гіпертензія

Сольова гіпертензія

2744 / 6307
Хворому для лікування серцевої недостатності було призначено серцевий глікозид. Яка супутня патологія може сприяти кумуляції серцевих глікозидів?

Гіперацидний гастрит

Гіпертонічна хвороба

Ниркова недостатність

Гіпоацидний гастрит

Анорексія

2745 / 6307
Чоловік 40-ка років хворіє на гіперацидний гастрит з нічними голодними болями. Призначте хворому лікарський засіб - блокатор гістамінових Н2 -рецепторів III покоління, який знизить виділення хлористоводневої кислоти (особливо вночі) та збільшить утворення захисного слизу:

Пірензепін

Платифіліну гідротартрат

Фамотидин

Атропіну сульфат

Метацин

2746 / 6307
П’ятирічна дитина-правша після черепно-мозкової травми на деякий час втратила здатність розмовляти, але через тривалий час ця здатність у неї відновилась. Яка півкуля була травмована й за рахунок якої властивості ЦНС дітей відновлення мови стало можливим?

Ліва півкуля, пластичність

Обидві півкулі, інертність

Права півкуля, пластичність

Права півкуля, рухливість

Ліва півкуля, інертність

2747 / 6307
Чоловік 35-ти років помер у приймальному відділенні лікарні, куди був доставлений у шоковому стані. На розтині тіла виявлено аневризму дуги аорти з її розривом та масивною кровотечею. Мікроскопічно: в медії аорти скупчення лімфоцитів та плазмоцитів, поодинокі гігантські клітини Пирогова-Лангханса, фібробласти. Спостерігаються також дрібні вогнища некрозу та руйнування еластичних структур стінки аорти. Яке захворювання зумовило такі зміни?

Туберкульоз

Висипний тиф

Сифіліс

Атеросклероз

2748 / 6307
У дитини 5-ти років розвинулось гостре респіраторне захворювання, яке супроводжувалось кашлем, виділенням значної кількості слизу із носа. Який тип запалення у хворої дитини?

Фібриноїдне

Гнилісне

Геморагічне

Гнійне

Катаральне

2749 / 6307
Чоловікові 58-ми років зроблено операцію з приводу раку простати. Через 3 місяці йому проведено курс променевої та хіміотерапії. До комплексу лікарських препаратів входив 5- фтордезоксиуридин - інгібітор тимідилатсинтази. Синтез якої речовини блокується цим препаратом?

р-РНК

і-РНК

т-РНК

ДНК

2750 / 6307
Стеатоз виникає внаслідок накопичення триацилгліцеролів у гепа-тоцитах. Одним з механізмів розвитку цього захворювання є зменшення утилізації нейтрального жиру ЛПДНЩ. Які ліпотропні речовини попереджують розвиток стеатозу?

Аргінін, B2 , B3

Валін, B3 , B2

Метіонін, B6 , B12

Аланін, B1 , PP

Ізолейцин, B1 , B2

2751 / 6307
У пацієнта, що прибув з ендемічного за малярією району, підвищилася температура тіла, відзначається головний біль, озноб, загальне нездужання - симптоми, що характерні й для звичайної застуди. Які лабораторні дослідження необхідно провести, щоб підтвердити або спростувати діагноз 'малярія'?

Дослідження пунктату лімфовузлів

Аналіз сечі

Мікроскопія пунктату червоного кісткового мозку

Мікроскопія мазків крові

Дослідження спинномозкової рідини

2752 / 6307
У чоловіка 40-ка років внаслідок посиленого гемолізу еритроцитів підвищився вміст заліза в плазмі крові. Який білок забезпечує його депонування в тканинах?

Гаптоглобін

Альбумін

Транскортин

Феритин

Трансферин

2753 / 6307
У дитячому дошкільному закладі напередодні новорічних свят було зареєстровано спалах кишкової інфекції. При бактеріологічному дослідженні випорожнень хворих патогенних бактерій не було виділено. При електронній мікроскопії виявлено утворення округлої форми з чітким обідком і товстою втулкою, які нагадують колесо. Вкажіть найбільш імовірний збудник даної інфекції:

Rotavirus

E.coli

Coxsacki-virus

Adenovirus

Pvulgaris

2754 / 6307
У біоптаті щитоподібної залози виявлено атрофію паренхіматозних елементів, дифузну інфільтрацію тканини залози лімфоцитами та плазматичними клітинами з утворенням в ній лімфоїдних фолікулів. Для якого захворювання є характерними наведені ознаки?

Аденома щитоподібної залози

Тиреоїдит Ріделя

Ендемічний зоб

Тиреоїдит Хасімото

Хвороба Базедова

2755 / 6307
При гістологічному дослідженні біоптатів, взятих з потовщених країв виразки шлунка, виявлені невеликі гніздові скупчення різко атипових гіперхромних невеликих епітеліальних клітин, які розташовані серед дуже розвиненої строми. Визначте пухлину:

Медулярний рак

Аденокарцинома

Недиференційована саркома

Скіррозний недиференційований рак

Аденома

2756 / 6307
При розтині тіла померлого у інфекційному відділенні виявлено: фібринозногнійний перитоніт; в слизовій оболонці клубової кишки багаточислені виразкові дефекти овальної форми до 3-5 см, які розташовані вздовж кишки і повторюють форму пейєрової бляшки, краї виразок рівні, заокруглені, дно чисте, представлене м’язовою або серозною оболонкою. В дні двох виразок знайдені отвори до 0,3 см в діаметрі. Для якого захворювання найбільш характерні ці зміни?

Хвороба Крона

Дизентерія

Паратиф

Неспецифічний виразковий коліт

Черевний тиф

2757 / 6307
Хворий 78-ми років з хворобою Паркінсона приймає препарати левадопи (наком). Яка антипаркінсонічна дія у цього засобу?

Адреноблокуюча

М-холіноблокуюча

М-холіноміметична

Блокуюча гістамінергічна дія

Допамінергічна

2758 / 6307
У хворого 47-ми років виникла кишкова коліка на фоні гіпертонічної хвороби. Засоби якої з перерахованих груп найбільш доцільно використати для її купірування у даній ситуації?

М-холиноміметики

Міотропні спазмолітики

Адреноміметики

Симпатоміметики

Антихолінестеразні засоби

2759 / 6307
Під час ректороманоскопії хворого зі скаргами на діарею виявлено, що слизова оболонка прямої і сигмоподібної кишок різко гіперемована, набрякла, вкрита великою кількістю слизу, а у деяких ділянках вкрита плівчастими накладаннями зеленуватого кольору. Про яке захворювання можна думати?

Амебіаз

Дизентерія

Черевний тиф

Холера

Сальмонельоз

2760 / 6307
У хворого на ЕКГ виявлено збільшення тривалості комплексу QRS. Наслідком чого це може бути?

Порушення провідності у атріовентрикулярному вузлі

Збільшення збудливості передсердь

Збільшення часу охоплення збудженням передсердь

Збільшення збудливості шлуночків та передсердь

Збільшення часу охоплення збудженням шлуночків

2761 / 6307
Хвороба Андерсена належить до групи спадкових хвороб, що розвиваються внаслідок уродженої недостатності синтезу певних ферментів глікогенолізу. Недостатність якого ферменту є молекулярною основою цього глікогенозу?

Глюкозо-6-фосфатази

Лізосомальні глікозидази

Глікогенсинтаза

Фосфофруктокіназа

Аміло(1,4-1,6)трансглікозидаза

2762 / 6307
У хворого з варикозним розширенням вен під час огляду нижніх кінцівок відзначається: ціаноз, пастозність, зниження температури шкіри, поодинокі петехії. Який розлад гемодинаміки має місце у хворого?

Обтураційна ішемія

Тромбоемболія

Компресійна ішемія

Артеріальна гіперемія

Венозна гіперемія

2763 / 6307
Після прийому сульфаніламідів у хворого виникли лихоманка, блювання і стул з кров’ю. У крові: лейк.-0,9 • 109 /л (гранул.- 0,7 • 109 /л), лейкоаглютиніни. Який з термінів найбільш точно характеризує виявлені зміни у крові?

Агранулоцитоз

Гемодилюція

Лейкоз

Лейкопенія

2764 / 6307
При гістологічному дослідженні органів і тканин померлої від ниркової недостатності молодої жінки, у якої прижиттєво виявлявся високий титр антинуклеарних антитіл, виявлені поширені фібриноїдні зміни в стінках судин. Відмічається ядерна патологія з вакуолізацією ядер, каріоре-ксисом, утворенням гематоксилінових тілець. Який найбільш імовірний діагноз?

Системний червоний вовчак

Вузликовий періартеріїт

Облітеруючий ендартеріїт

Гіпертонічна хвороба

Атеросклероз

2765 / 6307
У недоношеного немовляти спостерігається жовтяниця. З нестачею у нього якого ферменту це пов’язано?

Лужна фосфатаза

Кисла фосфатаза

УДФ-трансглюкуронідаза

Каталаза

НАД+- дегідрогеназа

2766 / 6307
У хворого пухлина грудного відділу стравоходу. Куди можуть безпосередньо метастазувати пухлинні клітини?

Nodi intercostales

Nodi mediastinales

Nodi hepatici

Nodi gastrici

Ductus thoracicus

2767 / 6307
При дослідженні вмісту дванадцятипалої кишки людини знайдені найпростіші грушоподібної форми з парними ядрами, чотирма парами джгутиків. Між ядрами - дві опірні нитки, з вентрального боку розташований присмоктувальний диск. Який представник найпростіших виявлений у хворого?

Трихомонада кишкова

Лямблія

Токсоплазма

Трипаносома

Лейшманія

2768 / 6307
36-ти років має місце гіповітаміноз B2 168. . Причиною виникнення специфічних симптомів (ураження епітелію, слизових, шкіри, рогівки ока) імовірно є дефіцит:

Цитохрому В

Цитохрому С

Цитохромоксидази

Цитохрому А1

Флавінових коферментів

2769 / 6307
При диспансерному обстеженні у хворого знайдено цукор в сечі. Який найбільш імовірний механізм виявлених змін, якщо вміст цукру в крові нормальний?

Порушення фільтрації глюкози в клубочковому відділі нефрона

Недостатня продукція інсуліну підшлунковою залозою

Порушення реабсорбції глюкози в канальцях нефрона

Гіперпродукція глюкокортикоїдів наднирниками

Інсулінорезистентність рецепторів клітин

2770 / 6307
На розтині в серці виявлено наступні зміни: великий осередок некрозу білого кольору з червоною облямівкою, який захоплює всю товщу серцевого м’яза. На зовнішній оболонці серця - ознаки фібринозного перикардиту. Який найбільш імовірний діагноз?

Субепікардіальний інфаркт міокарда

Субендокардіальний інфаркт міокарда

Інтрамуральний інфаркт міокарда

Трансмуральний інфаркт міокарда

Міокардит

2771 / 6307
На електронній мікрофотографії представлена клітина нейрального походження. Термінальна частина дендрита клітини має циліндричну форму і складається з 1000 замкнутих мембранних дисків. Яка клітина зображена на мікрофотографії?

Нейрон передніх рогів спинного мозку

Колбочкова нейросенсорна

Паличкова нейросенсорна

Нейрон кори великих півкуль

Нейрон спинномозкового вузла

2772 / 6307
Порушення процесів мієлінізації нервових волокон призводить до неврологічних розладів і розумової відсталості. Такі симптоми характерні для спадкових і набутих порушень обміну:

Сфінголіпідів

Холестерину

Фосфатидної кислоти

Вищих жирних кислот

Нейтральних жирів

2773 / 6307
У хворого 15-ти років концентрація глюкози натще 4,8 ммоль/л, через годину після цукрового навантаження - 9,0 ммоль/л, через 2 години - 7,0 ммоль/л, через 3 години - 4,8 ммоль/л. Ці показники характерні для такого захворювання:

Хвороба Іценко-Кушінга

Цукровий діабет I типу

Цукровий діабет II типу

Прихований цукровий діабет

2774 / 6307
У чоловіка 48-ми років виявлено порушення периферичного кровообігу з обмеженням припливу артеріальної крові, при цьому має місце збліднення даної ділянки, зниження місцевої температури. Це порушення називається:

Сладж

Реперфузійний синдром

Венозна гіперемія

Ішемія

Стаз

2775 / 6307
Хворому хірург видалив порожнинне утворення печінки діаметром 2 см. Встановлено, що стінка порожнини утворена щільною волокнистою сполучною тканиною, вміст являє собою каламутну, густу, жовтувато-зеленуватого кольору рідину з неприємним запахом, яка мікроскопічно складається переважно з поліморфноядерних лейкоцитів. Якому патологічному процесу відповідають такі морфологічні зміни?

Хронічний абсцес

Емпієма

Флегмона

Гострий абсцес

2776 / 6307
Лікар призначив пацієнту з хронічним бронхітом відхаркувальний засіб, який діє шляхом розщеплення дисульфідних зв’язків глікозаміногліканів харкотиння, зменшуючи цим його в’язкість, проте попередив хворого про можливий бронхоспазм при його використанні. Який засіб був призначений?

Ацетилцистеїн

Натрію гідрокарбонат

Бромгексин

Трава термопсису

Лібексин

2777 / 6307
Лікар призначив хворому з гострою серцевою недостатністю не-глікозидний кардіотонічний засіб, який безпосередньо стимулює β1 -адренорецептори міокарда, що збільшує кровообіг, діурез. Застосовується лише внутрішньовенно крапельно внаслідок швидкої інактивації в організмі. Який препарат призначив лікар?

Анаприлін

Добутамін

Адреналін

Корглікон

Дигоксин

2778 / 6307
Чоловік 40-ка років перебував у пульмонологічному відділенні з приводу рецидивуючої правосторонньої пневмонії. Помер від легенево-серцевої недостатності. На розтині в правій лєгєні визначається ділянка круглої форми 3х4 см. Вона являє собою порожнину з нерівними шорсткими краями, заповнену каламутною вершкоподібною жовто-зеленою рідиною. Мікроскопічно: стінка порожнини утворена тканиною легені з дифузною інфільтрацією лейкоцитами. Визначте патологічний процес у легені:

Гострий абсцес

Інфаркт

Хронічний абсцес

Гангрена

Емпієма

2779 / 6307
Робочий комунальної служби спустився в каналізаційний колодязь без засобів захисту і через деякий час знепритомнів. Лікарями швидкої допомоги діагностовано отруєння сірководнем. Який вид гіпоксії при цьому розвинувся?

Перевантажувальний

Тканинний

Гемічний

Циркуляторний

Респіраторний

2780 / 6307
Під час розтину тіла жінки 52-х років, яка тривалий час хворіла на жовчно-кам’яну хворобу, було знайдено: макроскопічно - печінка помірно збільшена, деформована, поверхня органу горбиста, тканина щільна, на розрізі тканина коричнева з зеленим відтінком, складається з множинних вузликів діаметром 8-10 мм. Мікроскопічно - гепатоцелюлярні вузлики оточені прошарками сполучної тканини, яка містить збільшену кількість дрібних жовчних протоків з холестазом. Діагностуйте захворювання печінки:

Токсична дистрофія печінки

Біліарний цироз печінки

Портальний цироз печінки

Холелітіаз

Постнекротичний цироз печінки

2781 / 6307
При гістологічному дослідженні біоптату, отриманого із нижньої третини стравоходу 57- річного чоловіка із симптомами тривалого рефлюксу шлункового вмісту, виявлено наступні зміни: у слизовій оболонці на місці багатошарового плоского епітелію визначається одношаровий залозистий призматичний епітелій, з ознаками продукції слизу. Вкажіть патологічний процес, який виник у слизовій оболонці:

Гіпертрофія

Організація

Регенерація

Гіперплазія

Метаплазія

2782 / 6307
Хворій жінці із захворюванням нирок, що супроводжується вираженими набряками, призначили діуретичний препарат, що пригнічує реабсорбцію в нирках іонів Na+ і води, посилює виведення нирками іонів K+ і Mg++, викликає гіперурикемію, зумовлює потужний діуретичний ефект. Назвіть цей препарат:

Спіронолактон

Тріамтерен

Аллопуринол

Діакарб

Фуросемід

2783 / 6307
Лікар-стоматолог для лікування гінгівіту призначив пацієнту препарат з протипротозойною та антибактеріальною діями, який може викликати відразу до алкоголю. Вкажіть препарат, який призначив лікар:

Метронідазол

Цефтріаксон

Тетрациклін

Лінкоміцину гідрохлорид

Левоміцетин

2784 / 6307
Аналіз ЕКГ хворого виявив відсутність зубця Р. Тривалість та амплітуда QRS комплексу та зубця Т відповідають нормі. Що є водієм ритму серця даного пацієнта?

Міокард шлуночків

Волокна Пуркін’є

Передсердно-шлуночковий вузол

Синусовий вузол

Пучок Гіса

2785 / 6307
харчування призвів до зменшення іонів Ca2+ 185. в крові. До збільшення секреції якого гормону це призведе?

Вазопресин

Тирокальцитонін

Паратгормон

Соматотропін

Тироксин

2786 / 6307
Хворій 43-х років для лікування бронхопневмонії призначена бензилпеніциліну натрієва сіль. Який з вказаних побічних ефектів найбільш характерний для даного засобу?

Анемія

Неврит слухового нерва

Агранулоцитоз

Алергічні реакції

Ураження печінки

2787 / 6307
Пацієнт із захворюванням першого верхнього різця зліва скаржиться на сильні болі шкіри в ділянці надбрівної дуги з того ж боку. Реалізація якого виду рефлексів спричиняє вказані реакції?

Вісцеро-вісцеральні

Пропріоцептивні

Вісцеро-соматичні

Сомато-вісцеральні

Вісцеро-дермальні

2788 / 6307
Потерпілий 19-ти років доставлений до травматологічного відділення з різаною раною трапецієподібного м’яза. Яка з фасцій шиї формує піхву для даного м’яза?

Поверхнева пластинка шийної фасції

Сонна піхва шийної фасції

Вісцеральна частина передтрахейної пластинки шийної фасції

Передхребтова пластинка шийної фасції

М’ язова частина передтрахейної пластинки шийної фасції

2789 / 6307
У хворого діагностовано ГРВІ. У сироватці крові знайдено імуноглобуліни класу М. Який період інфекційного процесу в даному випадку?

Мікробоносійство

Реконвалесценція

Інкубаційний

Гострий

Продромальний

2790 / 6307
У хворого 34-х років після перенесеної кишкової інфекції, викликаної сальмонелами, стали згасати симптоми захворювання. Імуноглобуліни якого класу будуть виявлені в крові хворого в період реконвалесценції?

IgM

IgG

IgA

IgE

IgD

2791 / 6307
До шпиталю було доставлено юнака 16-ти років, хворого на інсулінозалежний цукровий діабет. Рівень глюкози у крові пацієнта складав 18 ммоль/л. Хворому було введено інсулін. Дві години потому рівень глюкози зменшився до 8,2 ммоль/л, тому що інсулін:

Стимулює перетворення глюкози в печінціу глікоген та ТАГ

Стимулює розщеплення глікогену в печінці

Стимулює розщеплення глікогену у м’язах

Стимулює транспорт глюкози через плазматичні мембрани в головному мозку та печінці

Гальмує синтез кетонових тіл із глюкози

2792 / 6307
Хворий на трансмуральний інфаркт міокарда лівого шлуночка переведений до відділення реанімації у важкому стані. АТ- 70/50 мм рт.ст., ЧСС- 56/хв., ЧД- 32/хв. Зазначте головну ланку в патогенезі кардіоген-ного шоку:

Падіння периферичного судинного опору

Втрата електролітів

Падіння серцевого викиду

Крововтрата

Втрата води

2793 / 6307
Обстеження пацієнта з високим артеріальним тиском показало в нього вторинну артеріальну гіпертензію. Причиною такого стану є ренін-продукуюча пухлина нирки. Що є головною ланкою в патогенезі вторинної артеріальної гіпертензії в хворого?

Гіперпродукція кортизолу

Гіпєрпродукція інсуліну

Гіперпродукція ангіотензину 2, альдостерону

Недостатня продукція катехоламінів

Недостатня продукція вазопресину

2794 / 6307
У чоловіка 29-ти років з ножовим пораненням шиї визначається кровотеча. При первинній обробці рани встановлено, що пошкоджена судина, розташована вздовж латерального краю груднинно-ключично-соскоподібного м’яза. Визначте цю судину:

A. carotis interna

A. carotis externa

V jugularis anterior

V. jugularis interna

V jugularis externa

2795 / 6307
У хворого переливання крові ускладнилося розвитком гемотрансфузійного шоку. Назвіть тип алергічної реакції, що лежить в основі даної патології:

Рецептороопосередкований

Імунокомплексний

Гіперчутливість сповільненого типу

Анафілактичний

Цитотоксичний

2796 / 6307
Підшлункова залоза - орган змішаної секреції. Ендокринно продукує бета-клітинами гормон інсулін, який впливає на обмін вуглеводів. Як він впливає на активність глікогенфо-сфорилази (ГФ) і глікогенсинтетази (ГС)?

Пригнічує ГФ, активує ГС

Пригнічує ГФ і ГС

Активує ГФ і ГС

Не впливає на активність ГФ і ГС

Активує ГФ, пригнічує ГС

2797 / 6307
Призначення доксицикліну гідрохлориду викликало порушення симбіозу мікробної флори в кишечнику. Визначити тип порушень при антибіотикотерапії:

Бактеріоз

Ідіосинкразія

Суперінфекція

Сенсибілізація

Дисбактеріоз

2798 / 6307
Для проведення анальгезії наркотичний анальгетик застосували з препаратом бензодіазепінового ряду. Який засіб використали для потенціювання анальгезії?

Імізин

Хлорпротіксен

Трифтазин

Діазепам

Карбамазепін

2799 / 6307
У хворого стенокардія. Який антиангінальний засіб протипоказаний для лікування, якщо в пацієнта алергія на йод?

Дротаверин

Нітросорбід

Аміодарон

Нітрогліцерин

Верапаміл

2800 / 6307
Молода людина 25-ти років споживає надмірну кількість вуглеводів (600 г на добу), що перевищує її енергетичні потреби. Який процес буде активуватися в організмі людини у даному випадку?

Ліполіз

Гліколіз

Ілюконеогенез

Ліпогенез

Окиснення жирних кислот

2801 / 6307
В шкірі виявлена щільна, рухома, чітко відмежована від оточуючих тканин пухлина. На розрізі вона білого кольору, представлена волокнистою тканиною. Мікроскопічно: хаотично переплетені колагенові волокна, клітин мало. Що це за пухлина?

Гістіоцитома

Міома

Дерматофіброма

Фіброма

Десмоїд

2802 / 6307
Для корекції артеріального тиску при колаптоїдному стані хворому було введено мезатон. Який механізм дії даного препарату?

Блокує α-адренорецептори

Стимулює α-адренорецептори

Блокує β-адренорецептори

Стимулює α- і β-адренорецептори

Стимулює β-адренорецептори

2803 / 6307
Після переходу до змішаного харчування у новонародженої дитини виникла диспепсія з діареєю, метеоризмом, відставанням у розвитку. Біохімічна основа даної патології полягає у недостатності:

Целюлази

Ліпази та креатинкінази

Лактази та целобіази

Трипсину та хімотрипсину

Сахарази та ізомальтази

2804 / 6307
У чоловіка 52-х років діагностовано системний амебіаз з ураженням кишківника, печінки, легенів. Який препарат слід призначити?

Метронідазол

Ентеросептол

Хінгамін

Хініофон

Тетрациклін

2805 / 6307
У пацієнта з ознаками коліту виділена чиста культура бактерій, яка за морфологічними, культуральними та біохімічними властивостями належить до роду шигел. Яку з названих реакцій доцільно застосувати для серологічної ідентифікації культури?

Затримки гемаглютинації

Аглютинації з діагностичними сироватками

Зв’язування комплементу

Непрямої гемаглютинації

Преципітації

2806 / 6307
Для лікування урогенітальних інфекцій використовують хінолони - інгібітори ферменту ДНК-гірази. Який процес порушується під дією хінолонів у першу чергу?

Реплікація ДНК

Зворотна транскрипція

Репарація ДНК

Ампліфікація генів

Рекомбінація генів

2807 / 6307
У хворого з аневризмою правої підключичної артерії спостерігається захриплість голосу. З подразненням якого нерву це може бути пов’язано?

N.laringeus superior sinister

N.laringeus superior dexter

N.laringeus reccurens dexter

N.laringeus inferior sinister

N.laringeus reccurens sinister

2808 / 6307
Під час операції в печінці хворого виявлені дрібні пухирці малих розмірів з незначною кількістю рідини, які щільно прилягають один до одного. Який гельмінтоз виявився у хворого?

Фасціольоз

Дікроцеліоз

Альвеококоз

Опісторхоз

Клонорхоз

2809 / 6307
У хлопчика 3-х років з вираженим геморагічним синдромом відсутній антигемофільний глобулін А (фактор VIII) у плазмі крові. Яка фаза гемостазу первинно порушена у цього хворого?

Ретракція кров’яного згустку

Перетворення протромбіну в тромбін

Зовнішній механізм активації протромбінази

Перетворення фібриногену в фібрин

Внутрішній механізм активації протромбінази

2810 / 6307
До лікаря звернувся чоловік 27-ми років. При огляді було виявлено збільшення кистей, стоп та нижньої щелепи. Крім того спостерігалися деформація суглобів (kiphosis), гормональні порушення (імпотенція, атрофія яєчок). Функція якої залози порушена?

Шишкоподібне тіло

Передня частка гіпофізу

Прищитоподібні залози

Щитоподібна залоза

Надниркові залози

2811 / 6307
У чоловіка 25-ти років діагностований гострий дифузний гломерулонефрит. З анамнезу: за 18 днів до прояву хвороби переніс ангіну. Який механізм ураження ниркових клубочків буде спостерігатися у цьому випадку?

Нефротоксичний

Імунний

Медикаментозний

Ішемічний

2812 / 6307
У чоловіка 35-ти років феохромоцитома. В крові спостерігається підвищений рівень адреналіну та норадреналіну, концентрація вільних жирних кислот зросла в 11 разів. Активація якого ферменту під впливом адреналіну підвищує ліполіз?

Фосфоліпаза А2

Фосфоліпаза С

Ліпопротеїдліпаза

ТАГ -ліпаза

Холестеролестераза

2813 / 6307
Жінку 44-х років втяла оса, внаслідок чого розвинувся шок. В анамнезі - тяжка алергічна реакція на укус оси. Об’єктивно: РS- 179/хв, слабкий, АТ- 80/40 мм рт.ст., ЧД-26/хв. Яка провідна ланка патогенезу анафілактичного шоку?

Зменшення ударного об’єму серця

Зменшення об’єму циркулюючої крові

Біль

Зниження периферійного опору судин

Тахікардія

2814 / 6307
Фекалії дитини, що хворіє на ентерит, емульгують в фізіологічному розчині і краплю емульсії наносять на елективне середовище: 10% молочно-сольовий, або жовтковосольовий агар. Який мікроорганізм передбачається виділити?

Стрептокок

Клебсієла

Ентерокок

Стафілокок

Кишкова паличка

2815 / 6307
У хворої 63-х років діагностований інсулінонезалежний цукровий діабет. Ендокринолог почав лікування з призначення глібенкламіду. Вкажіть механізм дії цього засобу:

Гальмує транспорт глюкози до клітин

Активує в-клітини острівців Лангерганса

Підсилює руйнування білків

Стимулює гіпоталамічні центри

Зменшує всмоктування глюкози в кишківнику

2816 / 6307
У хворого коса пахвинна грижа. Яке анатомічне утворення стало слабким місцем передньої черевної стінки?

Медіальна пахвинна ямка

Стегнова ямка

Латеральна пахвинна ямка

Надміхурова ямка

Пахвинний трикутник

2817 / 6307
Людині внутрішньовенно ввели 0,5 л ізотонічного розчину лікарської речовини. Які з рецепторів насамперед відреагують на зміну водно-сольового балансу організму?

Волюморецептори порожнистих вен і передсердь

Осморецептори гіпоталамусу

Барорецептори дуги аорти

Натрієві рецептори гіпоталамусу

Осморецептори печінки

2818 / 6307
До лікарні надійшла дитина з діагнозом 'стафілококовий сепсис'. На яке живильне середовище потрібно посіяти кров хворого з метою виділення збудника?

Жовчно-сольовий агар

Цукрово-пептонний бульйон

Середовище Плоскірева

М ’ясо-пептонний агар

Середовище Бучіна

2819 / 6307
При бактеріоскопічному дослідженні носоглоткового слизу дитини 2,5 років, хворої на назофарингіт, виявлені грамнегативні диплококи, схожі за формою на кавові зерна. Які структури організму дитини найбільш імовірно будуть уражені, якщо ці мікроорганізми проникнуть у кров?

Ниркові гломерули

Серцеві клапани

Оболонки мозку

Лімфатичні вузли

Сечостатеві шляхи

2820 / 6307
У хлопчика 2-х років спостерігається збільшення в розмірах печінки та селезінки, катаракта. В крові підвищена концентрація цукру, але тест толерантності до глюкози в нормі. Спадкове порушення обміну якої речовини є причиною цього стану?

Фруктоза

Галактоза

Мальтоза

Глюкоза

Сахароза

2821 / 6307
В сечі новонародженого визначається цитрулін та високий рівень аміаку. Вкажіть, утворення якої речовини, найімовірніше, порушене у цього малюка:

Аміак

Креатин

Креатинін

Сечовина

Сечова кислота

2822 / 6307
У хворого, що звернувся до лікарню зі скаргами на пронос, діагностували амебну дизентерію. До комплексного лікування був включений доксациклін. Назвіть вид дії призначеного препарату:

Етіотропна дія

Основна дія

Рефлекторна дія

Незворотня дія

Пряма дія

2823 / 6307
Під час обстеження у хворої встановлене ураження дорсальної частини мосту, порушена функція жування. Ядро якого нерва уражене?

Рухове ядро трійчастого нерва

Мостове ядро трійчастого нерва

Ядро під’язикового нерва

Подвійне ядро блукаючого нерва

Рухове ядро лицевого нерва

2824 / 6307
з дихальною недостатністю рН крові 7,35. Визначення рСO2 24. показало наявність гіперкапнії. При дослідженні рН сечі відзначається підвищення її кислотності. Яка форма порушення кислотноосновного стану в даному випадку?

Ацидоз метаболічний, компенсований

Ацидоз метаболічний, декомпенсований

Алкалоз газовий, компенсований

Алкалоз газовий, декомпенсований

Ацидоз газовий, компенсований

2825 / 6307
При бактеріологічному дослідженні сечі хворого на пієлонефрит виділені мікроорганізми, що утворюють на м’ясо-пептонному агарі жовто-зелений пігмент і мають характерний запах. Як вони називаються?

Клебсієли

Псевдомонади

Протеї

Азотобактерії

Ешеріхії

2826 / 6307
У хворого впродовж 10-ти днів має місце підвищена температура, напади характерного кашлю. Лікар призначив посів слизу з носоглотки на середовище КВА. Який мікроорганізм передбачається виявити?

Клебсієла

Стафілокок

Лістерія

Паличка коклюшу

Палочка інфлуенци

2827 / 6307
У нейрохірургічне відділення надійшов 54-річний чоловік зі скаргами на відсутність чутливості шкіри нижньої повіки, латеральної поверхні носа, верхньої губи. Лікар при огляді встановив запалення другої гілки трійчастого нерва. Через який отвір виходить з черепа ця гілка?

Остистий отвір

Рваний отвір

Круглий отвір

Овальний отвір

Верхня очноямкова щілина

2828 / 6307
Пацієнт 46-ти років звернувся до лікаря зі скаргами на болі в дрібних суглобах ніг та рук. Суглоби збільшені, мають вигляд потовщених вузлів. У сироватці встановлено підвищений вміст уратів. Це може бути спричинене:

Порушенням обміну піримідинів

Порушенням обміну амінокислот

Порушенням обміну ліпідів

Порушенням обміну пуринів

Порушенням обміну вуглеводів

2829 / 6307
У хворого з жовтяницею встановлено: підвищення у плазмі крові вмісту загального білірубіну за рахунок непрямого (вільного), в калі та сечі - високий вміст стеркобіліну, рівень прямого (зв’язаного) білірубіну в плазмі крові в межах норми. Про який вид жовтяниці можна думати?

Механічна

Паренхіматозна (печінкова)

Гемолітична

Жовтяниця немовлят

Хвороба Жильбера

2830 / 6307
У хворого з тромбофлебітом нижніх кінцівок раптово після навантаження виникли задишка, різкий біль у грудях, ціаноз, набухання шийних вен. Яке найбільш імовірне порушення кровообігу виникло у хворого?

Тромбоемболія вінцевих судин

Тромбоемболія легеневої артерії

Тромбоемболія ворітної вени

Тромбоемболія мезентеріальних судин

Тромбоемболія судин головного мозку

2831 / 6307
При штовханні штанги спортсмен закидає голову назад для максимального підвищення тонусу м’язів-розгиначів верхніх кінцівок. Де розташовані центри рефлексів, які при цьому виникають?

Ядра Дейтерса

Базальні ганглії

Червоні ядра

Спинний мозок

Рухова кора

2832 / 6307
Хворий 65-ти років, що страждає на атеросклероз, госпіталізований до хірургічного відділення з приводу розлитого гнійного перитоніту. Під час операції діагностовано тромбоз брижових артерій. Яка найбільш імовірна причина перитоніту?

Стаз

Геморагічний інфаркт

Ішемія ангіоспастична

Ішемія компресійна

Iшемiчний інфаркт

2833 / 6307
У крові чоловіка 26-ти років виявлено 18% еритроцитів сферичної, сплощеної, кулястої та остистої форм. Інші еритроцити були у формі двоввігнутих дисків. Як називається таке явище?

Патологічний пойкілоцитоз

Фізіологічний анізоцитоз

Еритроцитоз

Фізіологічний пойкілоцитоз

Патологічний анізоцитоз

2834 / 6307
Хвора страждає від болю в ногах та набряків. В ході обстеження хворої на медіальній поверхні стегна спостерігається набряк, збільшення розміру вен, утворення вузлів. З боку якої вени спостерігається патологія?

V tibialis

V saphena parva

V saphena magna

V femoralis

V profunda femoris

2835 / 6307
Знешкодження ксенобіотиків (лікарських засобів, епоксидів, ареноксидів, альдегідів, нітропохідних тощо) та ендогенних метаболітів (естрадіолу, простагландинів, лейкотрієнів) відбувається в печінці шляхом їх кон’югації з:

Фосфоаденозином

Гліцином

Глутатіоном

Аспарагіновою кислотою

S-Аденозилметіоніном

2836 / 6307
У хворого на гіпертонічну хворобу виявлено високий рівень реніну в крові. Якому з гіпотензивних засобів слід надати перевагу в цьому випадку?

Лізиноприл

Анаприлін

Ніфедипін

Празозин

Дихлотіазид

2837 / 6307
На останньому місяці вагітності вміст фібриногену в плазмі крові в 2 рази вище за норму. Яку швидкість осідання еритроцитів слід при цьому очікувати?

5-10 мм/годину

3-12 мм/годину

0-5 мм/годину

10-15 мм/годину

40-50 мм/годину

2838 / 6307
На розтині у дитини знайдена некротична ангіна, флегмона шиї, гнійний отит, гнійний менінгіт. Ці зміни найбільш характерні для:

Токсичної скарлатини

Дифтерії зіву

Септичної скарлатини

Отогенного сепсису

Менінгококової інфекції

2839 / 6307
У хворого відзначаються періодичні напади серцебиття (пароксизми), сильне потовиділення, напади головного болю. При обстеженні виявлена гіпертензія, гіперглікемія, підвищення основного обміну, тахікардія. При якій патології наднирників спостерігається подібна картина?

Гіпофункція мозкового шару

Гіперфункція мозкового шару

Первинний альдостеронізм

Гіперфункція кори наднирників

Гіпофункція кори наднирників

2840 / 6307
На енцефалограмі людини зареєстровано дельта-ритм. У якому стані вона перебуває?

Активної бадьорості

Засинання

Пасивної бадьорості

Парадоксального сну

Повільного сну

2841 / 6307
У людини з нападом бронхоспазму необхідно зменшити вплив блукаючого нерва на гладеньку мускулатуру бронхів. Які мембранні циторецептори доцільно заблокувати для цього?

М-холінорецептори

α- та β-адренорецептори

β-адренорецептори

Н-холінорецептори

α-адренорецептори

2842 / 6307
Необхідно оцінити рівень збудливості нерва у хворого. Для цього доцільно визначити для нерва наступну величину:

Амплітуда потенціалу дії

Порогова сила подразника

Потенціал спокою

Критичний рівень деполяризації

Тривалість потенціалу дії

2843 / 6307
Хворому з кардіогенним шоком, гіпотензією, ядухою і набряками ввели неглі-козидний кардіотонік. Який саме препарат був введений хворому?

Етимізол

Добутамін

Кофеїн-бензоат натрію

Кордіамін

Бемегрид

2844 / 6307
Біосинтез пуринового кільця відбувається на рибозо-5-фосфаті шляхом поступового нарощення атомів азоту і вуглецю та замикання кілець. Джерелом рибозофосфату є наступний процес:

Пентозофосфатний цикл

Глюконеогенез

Гліконеогенез

Глікогеноліз

Гліколіз

2845 / 6307
У жінки з важкою інтоксикацією, спричиненою сепсисом, який і послужив безпосередньою причиною смерті, на розтині виявлене 'тигрове серце'. Який морфогенетичний механізм розвитку переважно лежить в основі даної дистрофії?

Трансформація

Декомпозиція

Спотворений синтез

Інфільтрація

2846 / 6307
Хворий 32-х років з ураженням ліктьового нерва не може звести до серединної лінії ІІ і V пальці. Функція яких м’язів при цьому порушена?

Відвідний м’яз мізинця

Червоподібні м’язи

Тильні міжкісткові м’язи

Короткий долонний м’яз

Долонні міжкісткові м’язи

2847 / 6307
Який з нижченаведених сечогінних засобів слід призначити хворому з первинним гіперальдостеронізмом?

Фуросемід

Триамтерен

Маніт

Гіпотіазид

Спіронолактон

2848 / 6307
У мазку з нальоту на мигдаликах хворого з імовірною дифтерією виявлено палички синього кольору з потовщеннями на полюсах. Який метод фарбування мазків було використано?

Леффлера

Нейссера

Буррі

Грама

Гінса

2849 / 6307
При розтині померлого, який хворів на туберкульоз, у верхній частці правої легені знайдено порожнину розмірами 3х2 см, яка сполучається з бронхом. Стінка порожнини щільна, має три шари: внутрішній -піогенний, середній - шар туберкульозної грануляційної тканини, зовнішній - сполучнотканинний. Який діагноз найбільш імовірний?

Туберкульома

Фіброзно-осередковий туберкульоз

Гострий осередковий туберкульоз

Фіброзно-кавернозний туберкульоз

Гострий кавернозний тубеокульоз

2850 / 6307
Дитина народилася в стані асфіксії. Який препарат необхідно ввести новонародженому для стимуляції дихання?

Прозерин

Лобелін

Празозин

Етимізол

Атропін

2851 / 6307
Хворий похилого віку страждає на хронічний закреп, в основі якого лежить гіпотонія товстого кишківника. Який препарат слід призначити хворому?

Касторова олія

Натрію сульфат

Ацеклідин

Прозерин

Бісакодил

2852 / 6307
При обстеженні в аналізі крові пацієнта виявлено лейкоцитоз, лімфоцитоз, клітини Боткіна-Гумпрехта на тлі анемії. Про яку хворобу слід думати лікарю?

Інфекційний мононуклеоз

Лімфогранулематоз

Мієломна хвороба

Хронічний лімфолейкоз

Гострий мієлолейкоз

2853 / 6307
У жінки 37-ми років протягом року періодично виникали інфекційні захворювання бактеріального генезу, їх перебіг був вкрай тривалим, ремісії - короткочасними. При обстеженні виявлена гіпогамаглобулінемія. Порушення функції яких клітин може бути прямою її причиною?

Макрофаги

Плазматичні клітини

Лімфоцити

Нейтрофіли

Фагоцити

2854 / 6307
При гістологічному дослідженні щитоподібної залози визначається значна інфільтрація лімфоцитами з утворенням лімфоїдних фолікулів, руйнування паренхіматозних елементів, розростання волокон сполучної тканини. Для якого захворювання характерна така картина?

Паренхіматозний зоб

Ендемічний зоб

Дифузний токсичний зоб

Зоб Хашимото

Колоїдний зоб

2855 / 6307
У чоловіка 50-ти років при обстеженні було виявлено зниження кількості еритроцитів у крові та підвищення рівня вільного гемоглобіну в плазмі крові (гемоглобінемія). КП становив 0,85. Який вид анемії спостерігається у хворого?

Анемія внаслідок порушення еритропоезу

Спадкова гемолітична

Хронічна постгеморагічна

Гостра постгеморагічна

Набута гемолітична

2856 / 6307
У хворого 68-ми років, який страждає на серцеву недостатність та впродовж тривалого часу приймає препарати наперстянки, з’явилися явища інтоксикації, які швидко нівелювалися застосуванням донатора сульфгідрильних груп унітіолу. Який механізм терапевтичної дії цього засобу?

Реактивує натрій-калієву-АТФ-азу мембран міокардіоцитів

Сповільнює надходження натрію до міокардіоцитів

Підвищує енергозабезпечення міокарду

Гальмує вивільнення калію з міокардіоцитів

Зменшує накопичення іонізованого кальцію

2857 / 6307
При визначенні мікробного числа повітря у лікарняній палаті виявилося, що воно становить 1500 клітин/м3 . Які групи мікроорганізмів враховувалися при цьому?

Всі патогенні та умовно-патогенні бактерії

Бактерії та віруси - збудники респіраторних інфекцій

Всі бактерії, що виросли на живильному середовищі

Стафілококи та гемолітичні стрептококи

Збудники госпітальних інфекцій

2858 / 6307
При проведенні операції на тонкій кишці лікар виявив ділянку слизової оболонки, де на фоні колових складок була присутня поздовжня складка. Який відділ тонкої кишки має таку будову?

Pars descendens duodeni

Дистальний відділ ileum

Pars horizontalis duodeni

Pars ascendens duodeni

Початковий відділ jejunum

2859 / 6307
В гістологічному препараті визначається орган, стінку якого утворюють три оболонки. Внутрішня оболонка складається з ендотелія та тонкого підендотеліального шару. Зовнішня оболонка найтовща. Вкажіть, який орган представлено у препараті?

Сечовід

Матка

Вена

Серце

Артерія

2860 / 6307
У хворого нормально забарвлений кал, у складі якого є велика кількість вільних жирних кислот. Причиною цього є порушення:

Гідролізу жирів

Жовчовиділення

Секреції ліпаз

Всмоктування жирів

Жовчоутворення

2861 / 6307
У новонародженого малюка педіатр виявив, що отвір крайньої плоті за величиною не перевищує діаметр сечовивідного каналу і голівка статевого члена не може виходити через такий отвір. Як називається цей стан?

Парафімоз

Гіпоспадія

Епіспадія

Гермафродитизм

Фімоз

2862 / 6307
Під час судово-медичної експертизи жінки, яка загинула у автокатастрофі, знайдено ембріон на стадії ранньої гаструли. Назвіть місце його локалізації за умови нормального розвитку:

Яєчник

Стінка матки

Черевна порожнина

Маткова частина яйцепроводу

Ампульна частина яйцепроводу

2863 / 6307
В основі розвитку імунних і алергічних реакцій організмом застосовуються однакові механізми відповіді імунної системи на антиген. Визначте основну відмінність алергічних реакцій від імунних:

Особливість будови антигенів

Кількість антигену, що потрапляє

Спадкова схильність

Шляхи потрапляння антигенів до організму

Розвиток пошкодження тканин

2864 / 6307
У хворої 49-ти років відзначається обмеження довільних рухів у лівих кінцівках. Тонус м’язів у лівих руці та нозі підвищений за спастичним типом, посилені місцеві сухожилкові рефлекси, виявляються патологічні рефлекси. Який найбільш імовірний механізм призвів до розвитку м’язової гіпертонії та гіперрефлексії?

Гальмування мотонейронів кори головного мозку

Активація збуджувальних впливів з вогнища інсульту

Зниження гальмівних низхідних впливів

Активація мотонейронів внаслідок інсульту

Активація синаптичної передачі імпульсів

2865 / 6307
Чоловік 50-ти років хворіє на хронічний бронхіт, скаржиться на задишку під час фізичного навантаження, постійний кашель з відходженням харкотиння. При обстеженні діагностовано ускладнення - емфізема легень. Чим вона зумовлена?

Зменшення альвеолярної вентиляції

Зменшення розтяжності легень

Зниження еластичних властивостей легень

Зменшення перфузії легень

Порушення вентиляційно-перфузійного співвідношення в легенях

2866 / 6307
У чоловіка 58-ми років, померлого при наростаючих явищах хронічної серцевої недостатності, діагностовано ревматичний гранульоматозний міокардит. Мікроскопічно в міокарді спостерігаються гранульоми, що складаються з макрофагів з гіперхромними ядрами та світлою цитоплазмою, в центрі - осередок некрозу. Який характер має некроз в середині гранульоми?

Казеозний

Жировий

Колікваційний

Ценкеровський

Фібриноїдний

2867 / 6307
У хворого з довготривалим запаленням слизової оболонки носової порожнини з’явилися симптоми ураження слизової оболонки верхньощелепної пазухи (гайморит). Через яке утворення носової порожнини стало можливим розповсюдження інфекції?

Клино-піднебінний отвір

Решітчаста лійка

Решітчасті комірки

Клино-решітчаста заглибина

Верхньощелепний отвір

2868 / 6307
Хворому з артеріальною гіпертензією було призначено один з антигіпертензивних засобів. Артеріальний тиск нормалізувався, однак хворого почав турбувати постійний сухий кашель. Який з перерахованих препаратів має таку побічну дію?

Анаприлін

Ніфедипін

Клофелін

Лізиноприл

Резерпін

2869 / 6307
В анотації до препарату вказано, що він містить антигени збудника черевного тифу, адсорбовані на стабілізованих еритроцитах барана. З якою метою використовують цей препарат?

Для виявлення антитіл в реакції Відаля

Для виявлення антитіл в реакції непрямої гемаглютинації

Для серологічної ідентифікації збудника черевного тифу

Для виявлення антитіл в реакції гальмування гемаглютинації

Для виявлення антитіл в реакції зв’язування комплементу

2870 / 6307
До дерматолога звернувся хворий зі скаргами на появу гнійничків на шкірі обличчя та шиї. При лабораторному аналізі вмісту гнійних фолікул було виявлено рухомі червоподібні паразити. Вкажіть збудника, який викликав це захворювання:

Блоха людська

Личинка мухи

Блощиця ліжкова

Коростяний свербун

Залозниця вугрова

2871 / 6307
На розтині тіла хлопчика 8-ми років, що хворів на дифтерію зіва та мигдаликів і помер на другий тиждень від початку захворювання, виявлено зміни в міокарді у вигляді дрібновогнищевих некрозів міокардиоцитів, набряку строми з незначною лімфоцитарною інфільтрацією. Діагностуйте вид міокардиту:

Вогнищевий проміжний ексудативний

Альтеративний

Інтерстиційний

Септичний

Гранулематозний

2872 / 6307
У хворого діагностовано септичний ендокардит. Температура тіла протягом 5-ти днів коливалася в межах 39,5°С - 40,2°С. На 6-й день на тлі різкого зниження температури до 35,2°С розвинувся колапс. Який головний механізм колапсу?

Поліурія

Посилене потовиділення

Вазодилатація

Гіпервентиляція

Тахікардія

2873 / 6307
У пацієнта у результаті тривалого блювання відбувається значна втрата шлункового соку, що є причиною порушення кислотно-лужного стану в організмі. Яка з перерахованих форм порушення кислотно-лужного стану має місце?

Негазовий ацидоз

Метаболічний ацидоз

Негазовий алкалоз

Газовий алкалоз

Газовий ацидоз

2874 / 6307
Хворий 55-ти років хворіє на хронічний гломерулонефрит протягом 15-ти років. Які зміни складу крові або сечі найбільш характерно свідчать про обмеження секреторної функції нирок?

Гіпо-, ізостенурія

Гіперглікемія

Протеїнурія

Гіпопротеїнемія

Гіперазотемія

2875 / 6307
У хворого в обох щелепах рентгенологічно виявлено численні дефекти у вигляді гладкостінних округлих отворів. При гістологічному дослідженні - явища остеолізису і остеопорозу при явищах слабкого кісткоутворення. В сечі хворого знайдено білок БенсДжонса. Назвіть захворювання:

Гострий мієлолейкоз

Мієломна хвороба

Хронічний еритромієлоз

Хронічний мієлолейкоз

Гострий недиференційований лейкоз

2876 / 6307
Хворий 30-ти років звернувся до лікаря зі скаргами на підвищення температури тіла до 38°С, слабкість, біль у горлі. Об’єктивно: язик вкритий білим нальотом. Які гістологічні структури язика беруть участь в утворенні цього нальоту?

Епітелій жолобкуватих сосочків

Епітелій грибоподібних сосочків

Сполучнотканинна основа всіх сосочків язика

Епітелій листоподібних сосочків

Епітелій ниткоподібних сосочків

2877 / 6307
Досліджуються рецептори, інформація від яких прямує до кори без участі таламусу. Які це рецептори?

Смакові

Нюхові

Зорові

Дотикові

Слухові

2878 / 6307
На розтині тіла чоловіка похилого віку, який протягом останніх 2-х тижнів страждав на гострий розлад кишківника, виявлені зміни у прямій та сигмоподібній кишках: на поверхні слизової оболонки спостерігається коричнево-зелена плівка. Стінка кишки потовщена, порожнина різко звужена. Мікроскопічно виявляється проникаючий на різну глибину некроз слизової оболонки, некротичні маси пронизані нитками фібрину, з лейкоцитарною інфільтрацією. Який з перерахованих діагнозів найбільш імовірний?

Фібринозний коліт

Фолікулярний коліт

Виразковий коліт

Катаральний коліт

2879 / 6307
У вагітної жінки взяли кров для підтвердження клінічного діагнозу 'токсоплазмоз'. Яка з перерахованих серологічних реакцій має діагностичне значення?

Реакція гемадсорбції

Реакція нейтралізації

Реакція гальмування гемаглютинації

Реакція аглютинації

Реакція зв’язування комплементу

2880 / 6307
У здорових батьків, спадковість яких не обтяжена, народилася дитина з множинними вадами розвитку. Цитогенетичний аналіз виявив у соматичних клітинах дитини трисомію за 18-ю хромосомою (синдром Едвардса). З яким явищем пов’язане народження такої дитини?

Домінантною мутацією

Хромосомною мутацією - дуплікацією

Соматичною мутацією у ембріона

Впливом тератогенних факторів

Нерозходженням пари хромосом під час гаметогенезу

2881 / 6307
Під час об’єктивного обстеження хворого з діагнозом: атеросклеротичний міокардіосклероз, лікар встановив феномен дефіциту пульсу. При якій формі порушення серцевого ритму спостерігається такий феномен?

Синусова екстрасистолія

Ідіовентрикулярний ритм

Брадикардія

Миготлива аритмія

Передсердно-шлуночковий ритм

2882 / 6307
При аналізі родоводу пробанда виявлено, що ознака проявляється з однаковою частотою у представників обох статей і хворі наявні у всіх поколіннях (по вертикалі), а по горизонталі - у сибсів (братів і сестер пробанда) з відносно великих родин. Який тип успадкування досліджуваної ознаки?

Зчеплений з У-хромосомою

Автосомно-домінантний

Зчеплений з Х-хромосомою, рецесивний

Зчеплений з Х-хромосомою, домінантний

Автосомно-рецесивний

2883 / 6307
До клініки доставлено хвору на цукровий діабет, госпіталізовано у прекоматозному стані кетоацидотичного типу. Збільшення вмісту якого метаболіту до цього призвело?

Ацетоацетат

α-кетоглутарат

Аспартат

Цитрат

Малонат

2884 / 6307
В хірургічне відділення доставлено пацієнта з різаною раною медіального краю передпліччя. При обстеженні виявлено, що в хворого перерізано ліктьовий м’яз-згинач зап’ястка і ліктьовий м’яз-розгинач зап’ястка. Які з рухів кисті будуть порушені у хворого?

Відведення

Розгинання і відведення

Розгинання

Приведення

Згинання

2885 / 6307
При огляді хворого хірург встановив поранення в області двох верхніх третин правої нирки. Цілісність якого органу слід перевірити при цьому, беручи до уваги син-топію правої нирки?

Тонка кишка

Печінка

Підшлункова залоза

Шлунок

Низхідна ободова кишка

2886 / 6307
Хворому на гострий інфаркт міокарда у комплексній терапії було призначено гепарин. Через деякий час після введення даного препарату з’явилася гематурія. Який антагоніст гепарину необхідно ввести хворому для усунення даного ускладнення?

Вікасол

Неодикумарин

Амінокапронова кислота

Фібриноген

Протаміну сульфат

2887 / 6307
У синтезі пуринових нуклеотидів беруть участь деякі амінокислоти, похідні вітамінів, фосфорні ефіри рибози. Коферментна форма якого вітаміну є переносником одновуглецевих фрагментів в цьому синтезі?

Нікотинова кислота

Пантотенова кислота

Піридоксин

Рибофлавін

Фолієва кислота

2888 / 6307
У жінки на шкірі шиї спостерігається новоутворення на ніжці, кулястої форми, зморшкувате. У біоптаті з нього виявлена надмірна проліферація покривного епітелію та строми шкіри у вигляді сосочків, збережена полярність клітин, їх комплексність та базальна мембрана, характерний тканинний атипізм. Який найбільш імовірний діагноз?

Невус

Саркома

Фіброма

Рак

Папілома

2889 / 6307
У хворого 75-ти років, який довгий час страждав на атеросклероз церебральних судин, на аутопсії виявлені: тромбоз правої середньої мозкової артерії, великий осередок неправильної форми сірого кашоподібного розм’якшення мозкової тканини. Який патологічний процес розвинувся в спинному мозку?

Геморагічний інфаркт

Гума мозку

Коагуляційний некроз

Ішемічний інфаркт

2890 / 6307
У померлої дитини 4-х років при житті була наявна менінгіальна симптоматика, На розтині в м’якій мозковій оболонці макроскопічно виявлені просоподібні вузлики, які мікроскопічно представлені осередком казеозного некрозу з валами епітеліоїдних, лімфоїдних клітин, між якими зустрічаються великі клітини з ядрами на периферії у вигляді півмісяця. Який найбільш імовірний менінгіт у дитини?

Сифілітичний

Туберкульозний

Менінгококовий

Бруцельозний

Грипозний

2891 / 6307
На препараті представлено орган ендокринної системи, зовні вкритий сполучнотканинною капсулою, від якої всередину органа відходять перегородки, що ділять його на часточки. Кожна часточка складається з двох видів клітин - нейросекреторних пінеалоцитів - полігональних клітин з відростками, локалізованих центрально, та гліоцитів (астроцитів) - на периферії. Що за орган представлено на препараті?

Гіпофіз

Щитоподібна залоза

Мозкова речовина наднирників

Епіфіз

Гіпоталамус

2892 / 6307
У здорових батьків народилася дитина з синдромом Патау. Який метод медичної генетики дасть змогу віддиференціювати дану спадкову хворобу від її фенокопії?

Близнюковий

Цитогенетичний

Визначення статевого хроматину

Біохімічний

Дерматогліфічний

2893 / 6307
Лікар записав в історії хвороби, що у хворого дихання поверхневе (знижена глибина дихання). Це означає, що зменшеним є такий показник зовнішнього дихання:

Дихальний об’єм

Життєва ємність легень

Хвилинний об’єм дихання

Функціональна залишкова ємність

Ємність вдиху

2894 / 6307
У хворого 65-ти років діагностовано хворобу Паркінсона. Який засіб, що підвищує вміст дофаміну, слід йому призначити?

Циклодол

Амізил

Атропину сульфат

Скополаміну гідробромід

Леводопа

2895 / 6307
Після фармакологічної блокади іонних каналів мембрани нервового волокна потенціал спокою зменшився з -90 до -80 м Які канали було заблоковано?

Натрієві

Кальцієві

Калієві

Магнієві

Хлорні

2896 / 6307
У студента, який раптово зустрів кохану дівчину, збільшився системний артеріальний тиск. Посилена реалізація яких рефлексів спричинила таку зміну тиску?

Умовні симпатичні та парасимпатичні

Безумовні симпатичні

Безумовні парасимпатичні

Умовні симпатичні

Умовні парасимпатичні

2897 / 6307
Вагітна жінка під час пологів втратила близько 800 мл крові. Спостерігається тахікардія, артеріальний тиск 100/70 мм рт.ст., тахіпное до 28/хв. Який тип гіпоксії розвивається первинно в такій клінічній ситуації ?

Серцево-судинна

Змішана

Кров’яна

Дихальна

Тканинна

2898 / 6307
Основними тригерами, що вмикають ефекторні системи клітини у відповідь на дію гормонів, є протеїнкінази, які змінюють каталітичну активність певних регуляторних ферментів шляхом АТФ-залежного фосфорилювання. Який з наведених ферментів є активним у фосфори-льованій формі?

Глікогенфосфорилаза

Ацетил-КоА-карбоксилаза

ГОМГ-КоА-редуктаза

Піруваткіназа

Ілікогенсинтаза

2899 / 6307
У пацієнта виявлено: поганий апетит, нудота, блювання, анемія. На основі проведеної лабораторної діагностики встановлено дифілоботріоз. Зараження відбулося через вживання:

Свинини

Крабів та раків

Риби

Яловичини

Яєць

2900 / 6307
Очищення слизової оболонки дихальних шляхів від пилу і мікроорганізмів відбувається завдяки мукоциліарному транспорту - переміщенню слизу поверхнею епітелію. Які клітини забезпечують цей механізм очищення?

Бронхіолярні екзокриноцити

Війчасті та келихоподібні

Дендритні

Щіточкові

Ендокринні та базальні

2901 / 6307
На електронномікроскопічній фотографії поперечного зрізу волокна чітко візуалізуються декілька осьових циліндрів з мезаксонами. Яке це волокно?

Нервове мієлінове

Еластичне

Нервове безмієлінове

Ретикулярне

Колагенове

2902 / 6307
Інозитолтрифосфати в тканинах організму утворюються в результаті гідролізу фосфатидилінозитолдифосфатів і відіграють роль вторинних посередників (месенджерів) в механізмі дії гормонів. Їхній вплив у клітині спрямований на:

Вивільнення іонів кальцію з клітинних депо

Гальмування фосфодіестерази

Гальмування протеїнкінази С

Активацію протеїнкінази А

Активацію аденілатциклази

2903 / 6307
В ході експерименту було продемонстровано підвищення активності β-галактозидази після внесення лактози до культурального середовища з Е.соlі. Яка ділянка лактозного оперону стає розблокованою від репресору за цих умов?

Структурний ген

Промотор

Регуляторний ген

Оператор

Праймер

2904 / 6307
Під час електронномікроскопічного дослідження біоптату гепатоцитів на біліарному полюсі виявлено велику кількість плоских цистерн, сплющених у центральній частині й розширених на периферії, та дрібних міхурців із секреторними гранулами. Назвіть цю структуру:

Ендоплазматична сітка

Мікротрубочки

Лізосома

Піноцитозні міхурці

Комплекс Гольджі

2905 / 6307
При гастробіопсії у хворого встановлена метаплазія поверхневого епітелію слизової оболонки, який замість циліндричного набув вигляду кишкового. Разом з тим спостерігається склероз на місці залоз слизової оболонки та лімфогістіоцитарна інфільтрація. Про яке захворювання шлунка можна думати?

Хронічний атрофічний гастрит

Хронічний гастрит з ураженням залоз без атрофії

Поверхневий хронічний гастрит

Ерозивний гастрит

Корозивний гастрит

2906 / 6307
У жінки 23-х років після аборту з’явилася маткова кровотеча. Мікроскопічно у зіскобі з порожнини матки знайдені ворсини хоріона, що нагадують грона винограду. Мікроскопічно: набряк строми ворсин з утворенням безліч кіст, що супроводжуються проліферацією епітелію та синцитію ворсин, залишки плоду і плодових оболонок. Який найбільш імовірний діагноз?

Ендометрит

Вагітність

Плацентарний поліп

Хоріонепітеліома

Міхуровий занесок

2907 / 6307
У дитячому колективі проведено планову вакцинацію проти кору. Яким методом можна перевірити ефективність проведеної вакцинації?

Вірусоскопічний

Вірусологічний

Серологічний

Біологічний

Алергопроба

2908 / 6307
Хворому з лихоманкою та висипкою на шкірі після обстеження за допомогою серологічних реакцій поставлено діагноз - фасціольоз. Було встановлено, що хворий заразився шляхом споживання сирої води з річки. Яка стадія життєвого циклу фасціоли інвазійна для людини?

Яйце

Адолескарій

Метацеркарій

Фіна

Мірацидій

2909 / 6307
Хворий помилково прийняв надмірну дозу тироксину. До яких змін секреції тиреоліберину та тиреотропіну це призведе?

Секреція гормонів зменшиться

Секреція тиреотропіну збільшиться, тиреоліберину - зменшиться

Змін секреції гормонів не буде

Секреція тиреоліберину збільшиться, тиреотропіну - зменшиться

Секреція гормонів збільшиться

2910 / 6307
Турист нещодавно повернувся з країн Середньої Азії, де є багато москітів. У нього на шкірі з’явилися невеликі виразки з нерівними краями. В цьому випадку можна припустити наступне захворювання:

Токсоплазмоз

Специфічний міаз

Дерматотропний лейшманіоз

Демодекоз

Скабіес

2911 / 6307
У потерпілого з травмою голови у скроневій ділянці діагностовано епідуральну гематому. Яка з артерій найімовірніше пошкоджена?

Задня вушна

Передня оболонкова

Поверхнева скронева

Середня мозкова

Середня оболонкова

2912 / 6307
У дитини спостерігається затримка фізичного та розумового розвитку, глибокі порушення з боку сполучної тканини внутрішніх органів, у сечі виявлено кератансульфати. Обмін яких речовин порушений?

Глікозаміноглікани

Гіалуронова кислота

Еластин

Колаген

Фібронектин

2913 / 6307
При аналізі ЕКГ виявлено випадіння деяких серцевих циклів PQRST. Наявні зубці та комплекси не змінені. Назвіть вид аритмії:

Передсердна екстрасистола

Атріовентрикулярна блокада

Внутрішньопередсердна блокада

Синоатріальна блокада

Миготлива аритмія

2914 / 6307
До лікарні доставлено хворого з отруєнням інсектицидом - ротеноном. Яка ділянка мітохондріального ланцюга переносу електронів блокується цією речовиною?

НАДН-коензим О-редуктаза

Коензим О-цитохром С-редуктаза

Сукцинат-коензим О-редуктаза

АТФ -синтетаза

Цитохром С-оксидаза

2915 / 6307
В ході регенерації епітелію слизової оболонки порожнини рота (розмноження клітин) відбулася реплікація (авторепродукція) ДНК за напівконсервативним механізмом. При цьому нуклеотиди нової нитки ДНК є комплементарними до:

Змістовних кодонів

Материнської нитки

Ферменту РНК-полімерази

Інтронних ділянок гену

Ферменту ДНК-полімерази

2916 / 6307
Серед антиатеросклеротичних препаратів, що застосовуються з метою профілактики та лікування атеросклерозу, є левостатин. Він діє шляхом:

Пригнічення всмоктування холестерину в кишківнику

Гальмування біосинтезу холестерину

Усіма наведеними шляхами

Активації метаболізму холестерину

Стимулювання екскреції холестерину з організму

2917 / 6307
При визначенні повітряної та кісткової провідності звуку було встановлено, що у пацієнта ліве вухо краще сприймає звук при кістковому його проведенні, що могло бути пов’язано з захворюванням:

Середнього вуха справа

Середнього вуха зліва

Внутрішнього вуха зліва

Зовнішнього вуха справа

Внутрішнього вуха справа

2918 / 6307
В хронічному експерименті на щурах стимулювали електричним струмом паравентрикулярні та супраоптичні ядра гіпоталамуса. Яка поведінкова реакція спостерігалася у тварин?

Збільшення споживання води

Зменшення споживання їжі

Збільшення споживання їжі

Відмова від їжі та рідини

Зменшення споживання води

2919 / 6307
У хворого 45-ти років на тлі трансмурального інфаркту міокарда розвинулася гостра лівошлуночкова недостатність. Який лікарський засіб доцільно застосувати у даній ситуації для покращення помпової функції серця?

Промедол

Ефедрин

Ізадрин

Добутамін

Еуфілін

2920 / 6307
При зниженні активності ферментів антиоксидантного захисту посилюються процеси перекисного окиснення ліпідів клітинних мембран. При нестачі якого мікроелементу знижується активність глутатіонпероксидази?

Селен

Молібден

Мідь

Марганець

Кобальт

2921 / 6307
У хворого з нагноєнням рани при бактеріологічному дослідженні ранового вмісту виявлено грамнегативну паличку, яка на МПА утворює напівпрозорі слизові колонії синьо-зеленого кольору з перламутровим відтінком. Культура має специфічний запах фіалок або жасмину. Який вид збудника виділений з рани хворого?

S. pyogenes

S. faecalis

P vulgaris

S. aureus

P aeruginosa

2922 / 6307
У хворого під час комп’ютерної томографії грудної клітки діагностовано пухлину заднього нижнього середостіння. Яка з перерахованих структур стиснута пухлиною?

Aorta thoracica

Trachea

Vena cava superior

N. phrenicus

Arcus aortae

2923 / 6307
Під час операції холецистектомії у хірурга виникла необхідність визначити топографію загальної жовчної протоки. Злиттям яких проток утворюється дана анатомічна структура?

Загальної печінкової і лівої печінкової проток

Загальної печінкової і міхурової проток

Лівої печінкової і міхурової проток

Правої та лівої печінкових проток

Загальної печінкової і правої печінкової проток

2924 / 6307
Внаслідок ДТП у потерпілої 37-ми років виникло неутримання сечі. Які сегменти спинного мозку пошкоджені?

S2 - S4

Th1 — L1

L1 — L2

Th1 — Th5

Th2 — Th5

2925 / 6307
При обстеженні жінки 56-ти років, що хвора на цукровий діабет 1-го типу, виявлене порушення білкового обміну, що при лабораторному дослідженні крові проявляється аміноацидемією, а клінічно - уповільненням загоєння ран і зменшенням синтезу антитіл. Який з перерахованих механізмів викликає розвиток аміноацидемії?

Гіперпротеїнемія

Збільшення ліпопротеїдів високої щільності

Підвищення онкотичного тиску в плазмі крові

Зменшення концентрації амінокислот у крові

Підвищення протеолізу

2926 / 6307
У людини в результаті патологічного процесу збільшена товщина гематоальвеолярного бар’єру. Безпосереднім наслідком цього буде зменшення:

Альвеолярної вентиляції легень

Кисневої ємності крові

Дифузійної здатності легень

Резервного об’єму видиху

Хвилинного об’єму дихання

2927 / 6307
Недостатність в організмі мікроелементу селену проявляється кардіоміопатією. Імовірною причиною такого стану є зниження активності такого селенвмісного ферменту:

Лактатдегідрогеназа

Каталаза

Цитохромоксидаза

Глутатіонпероксидаза

Сукцинатдегідрогеназа

2928 / 6307
При дослідженні тимуса дитини 5-ти років, що померла від гострої деструктивної стафілококової пневмонії, виявлено зменшення маси залози до 3,0 г. При гістологічному дослідженні в тимусі знайдено: зменшення часточок залози, значне зменшення кількості лімфоцитів з колапсом строми часточок, інверсія шарів, кистоподібне збільшення тілець Гассаля. Який з перерахованих діагнозів найбільш імовірний?

Дисплазія тимусу

Агенезія тимусу

Гіпоплазія тимусу

Тимомегалія

Акцидентальна інволюція тимусу

2929 / 6307
У хворого внаслідок хронічного обструктивного бронхіту на тлі задишки, тахікардії та ціанозу під час дослідження газового складу крові виявлено розвиток гіпоксемії та гіперкапнії. Яке порушення зовнішнього дихання спостерігається у хворого?

Гіперперфузія

Гіпоперфузія

Гіпервентиляція

Гіпердифузія

Гіповентиляція

2930 / 6307
Хлопчику 5-ти років був встановлений діагноз - міастенія. Оберіть препарат з групи антихолінестеразних засобів, який покращує нервово-м’язову передачу:

Прозерин

Галантаміну гідробромід

Алоксим

Ацеклідин

Армін

2931 / 6307
Хворому для лікування серцевої недостатності було призначено серцевий глікозид. Яка супутня патологія може сприяти кумуляції серцевих глікозидів?

Гіпоацидний гастрит

Анорексія

Гіпертонічна хвороба

Ниркова недостатність

Гіперацидний гастрит

2932 / 6307
П’ятирічна дитина-правша після черепно-мозкової травми на деякий час втратила здатність розмовляти, але через тривалий час ця здатність у неї відновилась. Яка півкуля була травмована й за рахунок якої властивості ЦНС дітей відновлення мови стало можливим?

Обидві півкулі, інертність

Ліва півкуля, інертність

Права півкуля, пластичність

Ліва півкуля, пластичність

Права півкуля, рухливість

2933 / 6307
При дослідженні гостроти слуху в коваля виявили втрату слуху на 50% у діапазоні низьких частот і майже нормальну гостроту слуху в діапазоні високих частот. Порушення яких структур слухової системи призвело до такого стану?

Барабанна перетинка

Кортієв орган - ближче до овального віконця

Кортієв орган - ближче до гелікотреми

М’язи середнього вуха

Середня частина кортієвого органу

2934 / 6307
В підводному човні під час занурення порушилася система подачі кисню. У підводників збільшилися частота дихання і серцевих скорочень. Який вид гіпоксії розвинувся у підводників?

Гіпоксична

Серцево-судинна

Тканинна

Кров’яна

Дихальна

2935 / 6307
Сучасні антиатеросклеротичні препарати застосовуються з метою профілактики та лікування атеросклерозу. Такі препарати як гемфіброзил та фенфібрат гальмують біосинтез холестерину шляхом інгібування ферменту:

Гексокиназа

Ацилтрансфераза

Ілюкозо-6-фосфатаза

Ацил-КоА-холестеринацилтрансфераза

в-ГОМК-редуктаза

2936 / 6307
Хворий на атеросклероз приймає антисклеротичний засіб - фенофібрат. Який механізм дії має цей засіб?

Інгібує абсорбцію холестерину в ШКТ

Знижує рівень хіломікронів

Поліпшує мікроциркуляцію крові

Підвищує захоплення ліпопротеїдів низької щільності та блокує біосинтез ендогенного холестерину

Поновлює негативний електричний заряд ендотелію судин

2937 / 6307
Хворий на гіпертонічну хворобу разом з безсольовою дієтою та з антигіпертензивними засобами, довгий час приймав гідрохлортіазид, що зумовило порушення електролітного балансу. Яке порушення внутрішнього середовища виникло у хворого?

Гіпермагніємія

Гіперкаліємія

Збільшення об’єму циркулюючої крові

Гіпохлоремічний алкалоз

Метаболічний ацидоз

2938 / 6307
При лабораторному дослідженні крові пацієнта виявлено, що вміст білків у плазмі становить 40 г/л. Як це впливає на транскапілярний обмін води в мікроциркуляторному руслі?

Обмін не змінюється

Зменшуються фільтрація і реабсорбція

Збільшуються фільтрація і реабсорбція

Зменшується фільтрація, збільшується реабсорбція

Збільшується фільтрація, зменшується реабсорбція

2939 / 6307
При гістологічному дослідженні біоптатів, взятих з потовщених країв виразки шлунка, виявлені невеликі гніздові скупчення різко атипових гіперхромних невеликих епітеліальних клітин, які розташовані серед дуже розвиненої строми. Визначте пухлину:

Аденома

Медулярний рак

Недиференційована саркома

Скіррозний недиференційований рак

Аденокарцинома

2940 / 6307
У хворого на ЕКГ виявлено збільшення тривалості комплексу QRS. Наслідком чого це може бути?

Збільшення часу охоплення збудженням шлуночків

Порушення провідності у атріовентрикулярному вузлі

Збільшення збудливості передсердь

Збільшення збудливості шлуночків та передсердь

Збільшення часу охоплення збудженням передсердь

2941 / 6307
В ході авторадіографічного дослідження епітелію тонкої кишки було виявлено, що його повне оновлення відбувається протягом 3-х діб за рахунок проліферації малодиференційованих клітин. Вкажіть їх локалізацію:

Власна пластинка слизової оболонки

Основа ворсинок

Дно крипт

Верхівка ворсинок

Бічна поверхня ворсинок

2942 / 6307
У хворого пухлина грудного відділу стравоходу. Куди можуть безпосередньо метастазувати пухлинні клітини?

Nodi intercostales

Ductus thoracicus

Nodi hepatici

Nodi gastrici

Nodi mediastinales

2943 / 6307
При загостренні ревматоїдного артриту хворому, в анамнезі якого супутній хронічний гастрит, призначений целекоксиб. Чим обумовлено зменшення побічної дії препарату на травний тракт?

Пригнічення фосфоліпази А2

Переважне пригнічення циклооксигенази-2

Пригнічення фосфодіестерази

Переважне пригнічення циклооксигенази-1

Переважна стимуляція аденілатциклази

2944 / 6307
Дитина 4-х років госпіталізована в ортопедичне відділення з переломом гомілки зі зсувом. Перед репозицією уламків необхідна анальгезія. Який препарат слід обрати?

Омнопон

Панадол

Промедол

Анальгін

Морфіну гідрохлорид

2945 / 6307
При напруженій фізичній роботі у м’язовій тканині накопичується молочна кислота, яка дифундує в кров і підхоплюється печінкою та серцем. Який процес забезпечує відновлення запасів глікогену в м’язах?

Орнітиновий цикл

Цикл трикарбонових кислот

Пентозофосфатний шлях

Цикл Корі

Цикл лимонної кислоти

2946 / 6307
Фенілкетонурія - це захворювання, яке зумовлено рецесивним геном, що локалізується в аутосомі. Батьки є гетерозиготами за цим геном. Вони вже мають двох хворих синів і одну здорову доньку. Яка імовірність, що четверта дитина, яку вони очікують, народиться теж хворою?

50%

0%

100%

75%

25%

2947 / 6307
Визначення Х-хроматину в соматичних клітинах використовується для експресдіагностики спадкових захворювань, пов’язаних зі зміною кількості статевих хромосом. Який каріотип чоловіка, переважна більшість клітин якого містять одну грудочку Ххроматину:

47, ХХУ

48, ХХХУ

45, Х0

49, ХХХХУ

46, ХУ

2948 / 6307
36-ти років має місце гіповітаміноз В2 148. . Причиною виникнення специфічних симптомів (ураження епітелію, слизових оболонок, шкіри, рогівки ока) імовірно є дефіцит:

Флавінових коферментів

Цитохрому С

Цитохромоксидази

Цитохрому А1

Цитохрому В

2949 / 6307
На розтині в серці виявлено наступні зміни: великий осередок некрозу білого кольору з червоною облямівкою, який захоплює всю товщу серцевого м’яза. На зовнішній оболонці серця - ознаки фібринозного перикардиту. Який найбільш імовірний діагноз?

Субендокардіальний інфаркт міокарда

Трансмуральний інфаркт міокарда

Інтрамуральний інфаркт міокарда

Субепікардіальний інфаркт міокарда

Міокардит

2950 / 6307
В пробірку, що містить розчин NaCl 0,9%, додали краплю крові. Що відбудеться з еритроцитами?

Біологічний гемоліз

Зморшкування

Осмотичний гемоліз

Набухання

Залишаться без змін

2951 / 6307
До косметолога звернулася пацієнтка зі скаргами на появу чорних цяток на обличчі. Після обстеження було встановлено, що поява цяток пов’язана з порушенням виділення секрету сальних залоз. Який тип секреції характерний для цих залоз?

Макроапокриновий

Мерокриновий

Голокриновий

Мікроапокриновий

Мерокриновий та мікроапокриновий

2952 / 6307
У хворого 40-ка років ознаки гірської хвороби: запаморочення, задишка, тахікардія, рН крові - 7,50, pCO2 - 30 мм рт.ст., зсув буферних основ +4 ммоль/л. Яке порушення кислотно-основного стану має місце?

Газовий ацидоз

Негазовий алкалоз

Негазовий ацидоз

Видільний ацидоз

Газовий алкалоз

2953 / 6307
У чоловіка 48-ми років виявлено порушення периферичного кровообігу з обмеженням припливу артеріальної крові, при цьому має місце зблідніння даної ділянки, зниження місцевої температури. Це порушення називається:

Реперфузійний синдром

Венозна гіперемія

Стаз

Ішемія

Сладж

2954 / 6307
Хворому хірург видалив порожнинне утворення печінки діаметром 2 см. Встановлено, що стінка порожнини утворена щільною волокнистою сполучною тканиною, вміст являє собою каламутну, густу, жовтувато-зеленого кольору рідину з неприємним запахом, яка мікроскопічно складається переважно з поліморфноядерних лейкоцитів. Якому патологічному процесу відповідають такі морфологічні зміни?

Флегмона

Гострий абсцес

Хронічний абсцес

Емпієма

2955 / 6307
Лікар призначив пацієнту з хронічним бронхітом відхаркувальний засіб, який діє шляхом розщеплення дисульфідних зв’язків глікозаміногліканів харкотиння, зменшуючи цим його в’язкість, проте попередив хворого про можливий бронхоспазм при його використанні. Який засіб був призначений?

Ацетилцистеїн

Натрію гідрокарбонат

Бромгексин

Трава термопсису

Лібексин

2956 / 6307
Лікар призначив хворому з гострою серцевою недостатністю неглікозидний кардіотонічний засіб, який безпосередньо стимулює β1 -адренорецептори міокарда, що збільшує кровообіг, діурез. Застосовується лише внутрішньовенно крапельно внаслідок швидкої інактивації в організмі. Який препарат призначив лікар?

Корглікон

Добутамін

Адреналін

Анаприлін

Дигоксин

2957 / 6307
У молодого подружжя народилася дитина з енцефалопатією. Лікар встановив, що хвороба пов’язана з порушенням мітохондріальної ДНК. Як успадковуються мітохондріальні патології?

Від обох батьків усіма дітьми

Від матері всіма дітьми

Від батька тільки синами

Від матері тільки синами

Від батька тільки дочками

2958 / 6307
При розтині тіла жінки, померлої від хронічної ниркової недостатності, в слизовій оболонці товстої кишки виявлені сіро-жовті плівки, що щільно з’єднані з підлеглим шаром, при відокремленні яких утворюються виразки. Мікроскопічно: глибока ділянка некрозу пронизана нитками фібрину. Визначте вид запалення:

Крупозне

Катаральне

Гнійне

Гнильне

Дифтеритичне

2959 / 6307
Чоловік 40-ка років перебував у пульмонологічному відділенні з приводу рецидивуючої правосторонньої пневмонії. Помер від легенево-серцевої недостатності. На розтині в правій легені визначається ділянка круглої форми 3х4 см. Вона являє собою порожнину з нерівними шорсткими краями, заповнену каламутною вершкоподібною жовто-зеленою рідиною. Мікроскопічно: стінка порожнини утворена тканиною легені з дифузною інфільтрацією лейкоцитами. Визначте патологічний процес у легені:

Гострий абсцес

Хронічний абсцес

Емпієма

Гангрена

Інфаркт

2960 / 6307
Під час розтину тіла жінки 52-х років, яка тривалий час хворіла на жовчнокам’яну хворобу, було знайдено: макроскопічно - печінка помірно збільшена, деформована, поверхня органу горбиста, тканина щільна, на розрізі тканина коричнева з зеленим відтінком, складається з множинних вузликів діаметром 8-10 мм. Мікроскопічно - гепатоцелюлярні вузлики оточені прошарками сполучної тканини, яка містить збільшену кількість дрібних жовчних протоків з холестазом. Діагностуйте захворювання печінки:

Холелітіаз

Постнекротичний цироз печінки

Портальний цироз печінки

Токсична дистрофія печінки

Біліарний цироз печінки

2961 / 6307
Встановлено, що в клітинах організмів відсутні мембранні органели та їх спадковий матеріал не має нуклеосомної організації. Що це за організми?

Найпростіші

Аскоміцети

Віруси

Прокаріоти

Еукаріоти

2962 / 6307
Аналіз ЕКГ хворого виявив відсутність зубця P. Тривалість та амплітуда QRS комплексу та зубця Т відповідають нормі. Що є водієм ритму серця даного пацієнта?

Волокна Пуркіньє

Синусовий вузол

Пучок Гіса

Передсердно-шлуночковий вузол

Міокард шлуночків

2963 / 6307
Під час виконання оперативного втручання на гомілці хірург виділяє задню стінку canalis cruropopliteus. Яка анатомічна структура її утворює?

M.soleus

M.gastrocnemius

M.tibialis posterior

M.tibialis anterior

M.plantaris

2964 / 6307
Хворій 43-х років для лікування бронхопневмонії призначена бензилпеніциліну натрієва сіль. Який з вказаних побічних ефектів найбільш характерний для даного засобу?

Агранулоцитоз

Алергічні реакції

Ураження печінки

Анемія

Неврит слухового нерва

2965 / 6307
Після лікування високоефективним протитуберкульозним засобом у жінки 48ми років виникли явища невриту зорового нерва, порушення пам’яті, судоми. Який із зазначених протитуберкульозних препаратів приймала хвора?

Ізоніазид

Канаміцину сульфат

Етамбутол

ПАСК

Рифампіцин

2966 / 6307
Хлопчик на другому році життя став часто хворіти на респіраторні захворювання, стоматити, гнійничкові ураження шкіри. Навіть невеликі пошкодження ясен і слизової оболонки ускладнюються запаленням, що протікає тривало. Встановлено, що у крові дитини практично відсутні імуноглобуліни всіх класів. Зниження функціональної активності якої клітинної популяції лежить в основі описаного синдрому?

Нейтрофіли

В-лімфоцити

NK-лімфоцити

Макрофаги

Т-лімфоцити

2967 / 6307
Пацієнт із захворюванням першого верхнього різця зліва скаржиться на сильний біль шкіри в ділянці надбрівної дуги з того ж боку. Реалізація якого виду рефлексів спричиняє вказані реакції?

Пропріоцептивні

Вісцеро-вісцеральні

Вісцеро-дермальні

Вісцеро-соматичні

Сомато-вісцеральні

2968 / 6307
Під час роботи лікарю - стоматологу доводиться довго стояти на ногах, що може призвести до застою крові у венах нижніх кінцівок та їх варикозного розширення. З порушенням якого механізму венозного припливу крові до серця це пов’язано?

Відсутність скорочення скелетних м’язів

Градієнт тиску

Залишкова рушійна сила серця

Присмоктувально-тисковий помповий ефект діафрагми на органи черевної

Присмоктувальний ефект грудної клітки

2969 / 6307
. У хворого 34-х років після перенесеної кишкової інфекції, викликаної сальмонелами, стали згасати симптоми захворювання. Імуноглобуліни якого класу будуть виявлені в крові хворого в період реконвалесценції?

IgG

IgE

IgM

IgA

IgD

2970 / 6307
На плановий прийом до педіатра батьки привели дитину віком 13 місяців. Під час повного огляду лікар перевірив розвиток II сигнальної системи дитини. Назвіть період, коли у людини вперше з’являються ознаки розвитку II сигнальної системи:

2,5-3 роки

1,5-2 роки

2-2,5 роки

3-5 років

6-12 місяців

2971 / 6307
У хворого спостерігається пухлина тканин орбіти позаду очного яблука. Зазначено порушення акомодації та звуження зіниці ока. Яке анатомічне утворення ушкоджено?

N. opticus

N. trochlearis

N. nasociliaris

N. lacrimalis

Ganglion ciliare

2972 / 6307
До серцево-судинного відділення надійшов хворий зі скаргами на постійний головний біль у потиличній ділянці, шум у вухах, запаморочення. При обстеженні: АТ-180/110 мм рт.ст., ЧСС- 95/хв. Рентгенологічно визначено звуження однієї з ниркових артерій. Активація якої з перерахованих систем викликала гіпертензивний стан хворого?

Імунна

Кінінова

Ренін-ангіотензинова

Симпатоадреналова

Гемостатична

2973 / 6307
Хворого на трансмуральний інфаркт міокарда лівого шлуночка переведено до відділення реанімації у важкому стані. АТ-70/50 мм рт.ст., ЧСС- 56/хв., ЧД- 32/хв. Зазначте головну ланку в патогенезі кардіогенного шоку:

Падіння серцевого викиду

Падіння периферичного судинного опору

Крововтрата

Втрата води

Втрата електролітів

2974 / 6307
У чоловіка 29-ти років з ножовим пораненням шиї спостерігається кровотеча. При первинній обробці рани встановлено, що пошкоджена судина, розташована вздовж латерального краю груднинно-ключично-соскоподібного м’яза. Визначте цю судину:

V jugularis anterior

V jugularis interna

A. carotis interna

V jugularis externa

A. carotis externa

2975 / 6307
Після перелому нижньої щелепи постраждалий відзначає втрату чутливості шкіри у ділянці підборіддя і нижньої губи. Який нерв був пошкоджений?

Infraorbitalis

Maxillaris

Facialis

Mentalis

Buccalis

2976 / 6307
У хворого переливання крові ускладнилося розвитком гемотрансфузійного шоку. Назвіть тип алергічної реакції, що лежить в основі даної патології:

Анафілактичний

Гіперчутливість сповільненого типу

Рецептороопосередкований

Імунокомплексний

Цитотоксичний

2977 / 6307
Підшлункова залоза - орган змішаної секреції. Ендокринно продукує бета-клітинами гормон інсулін, який впливає на обмін вуглеводів. Як він впливає на активність глікогенфосфорилази (ГФ) і гліко-генсинтетази (ГС)?

Не впливає на активність ГФ і ГС

Пригнічує ГФ і ГС

Активує ГФ, пригнічує ГС

Активує ГФ і ГС

Пригнічує ГФ, активує ГС

2978 / 6307
До приймального відділення інфекційної лікарні надійшов чоловік 25-ти років. Діагноз: СНІД. Ураження яких клітин зумовлює стан імунодефіциту?

Т-хелпери

Т-супресори

Т-кіллери

Гладкі клітини (тканинні базофіли)

Плазмоцити

2979 / 6307
Тривале лікування гіпофункції щитоподібної залози спричинило загальну дистрофію, карієс зубів, тахікардію, тремор кінцівок. Який лікарський засіб викликав зазначені побічні ефекти?

Паратиреоїдин

Преднізолон

Тирокальцитонін

L-тироксин

Хумулін

2980 / 6307
Молода людина 25-ти років споживає надмірну кількість вуглеводів (600 г на добу), що перевищує її енергетичні потреби. Який процес буде активуватися в організмі людини у даному випадку?

Ліполіз

Ліпогенез

Окиснення жирних кислот

Гліколіз

Ілюконеогенез

2981 / 6307
У пацієнта встановлено гіповітаміноз фолієвої кислоти, що може призвести до порушення синтезу:

Пуринових нуклеотидів та холестерину

Тимідилових нуклеотидів та жирних кислот

Пуринових та тимідилових нуклеотидів

Гема та креатину

Цитрату та кетонових тіл

2982 / 6307
У людини виявлена пухлина одного з відділів головного мозку, внаслідок чого в неї порушена здатність підтримувати нормальну температуру тіла. Яка структура головного мозку пошкоджена?

Гіпоталамус

Мозочок

Чорна субстанція

Стріатум

Таламус

2983 / 6307
До складу харчових раціонів обов’язково входять продукти, в яких є клітковина. Відомо, що вона не перетравлюється ферментами травного тракту й не засвоюється організмом. Яку роль відіграє ця речовина?

Стимулює моторну функцію травного каналу

Гальмує процеси виділення ферментів травних соків

Гальмує секреторну функцію травного каналу

Гальмує всмоктувальну функцію травного каналу

Гальмує моторну функцію травного каналу

2984 / 6307
До лікарні звернувся чоловік 50-ти років з розладами пам’яті, болісними відчуттями по ходу нервових стовбурів, зниженням інтелектуальних функцій, порушеннями з боку серцево-судинної системи і явищами диспепсії. В анамнезі хронічний алкоголізм. Дефіцит якого вітаміну може викликати ці симптоми?

Ніацин

Тіамін

Кальциферол

Рибофлавін

Ретинол

2985 / 6307
По приїзді групи експертів на місце злочину виявлено тіло без ознак життя. В ході дослідження крові загиблого виявлена велика концентрація іонів ціанової кислоти. Інгібування якого комплексу дихального ланцюга мітохондрій стало причиною смерті?

І

III

II

V

IV

2986 / 6307
Вивчення організму мешканця Паміру виявило високий рівень основного обміну, розширення грудної клітки, зростання кисневої ємності крові за рахунок збільшення еритроцитів, високий вміст гемоглобіну. До якого адаптивного екологічного типу слід віднести цього чоловіка?

Пустельний

Гірський

Арктичний

Субтропічний

Тропічний

2987 / 6307
Пацієнт звернувся зі скаргами на гострий біль у правому підребер’ї. При огляді лікар звернув увагу на пожовтіння склер хворого. Лабораторно: підвищена активність АлАТ та негативна реакція на стеркобілін в калі. Для якого захворювання характерні такі симптоми?

Гепатит

Хронічний гастрит

Хронічний гастродуоденіт

Гемолітична жовтяниця

Хронічний коліт

2988 / 6307
Жінка 30-ти років народила хлопчика з розщепленням верхньої губи ('заяча губа' 'вовча паща'). При додатковому обстеженні виявлені значні порушення нервової, серцевосудинної систем та зору. При дослідженні каріотипу діагностована трисомія за 13-ю хромосомою. Який синдром наявний у хлопчика?

Едвардса

Дауна

Шерешевського-Тернера

Патау

Клайнфельтера

2989 / 6307
До медико-генетичної консультації звернулася жінка 30-ти років у якої в ядрах більшості клітин епітелію слизової оболонки щоки було виявлено по два тільця Барра. Який попередній діагноз можна встановити?

Трисомія за 21-ю хромосомою

Трисомія за X-хромосомою

Моносомія за X-хромосомою

Трисомія за 18-ю хромосомою

Трисомія за 13-ю хромосомою

2990 / 6307
При розтині хворої 28-ми років, що померла від уремії, виявлені збільшені строкаті нирки з осередками крововиливів. Патогістологічно в судинних клубочках виявлені гематоксилінові тільця, капілярні мембрани клубочків у вигляді дротяних петель, гіалінові тромби та осередки фібриноїдного некрозу. За патогенезом гіперчутливість якого типу лежить в основі описаної хвороби?

Гіперчутливість II типу (антитілозалежна)

Гіперчутливість I типу (анафілактична)

Гіперчутливість IV типу (клітинна цитoтоксичність)

Гіперчутливість V типу (гранулематоз)

Гіперчутливість III типу (імунокомплексна)

2991 / 6307
На розтині померлого було виявлено наявність крові в тонкій та товстій кишках (1,5-2 л). В нижніх відрізках клубової кишки стінка мала 'брудні' виразки на місці групових лімфатичних фолікулів. Решта пейєрових бляшок була зеленуватого кольору з демаркаційним запаленням навколо. Було діагностовано черевний тиф. Для якого періоду черевного тифу найбільш характерні описані морфологічні зміни кишки й ускладнення?

Мозкоподібного набухання

Чистих виразок

Утворення виразок

Загоювання виразок

Некроз

2992 / 6307
Ефективна діагностика носійства збудників кишкових інфекцій ґрунтується на виявленні антитіл до певних антигенів бактерій в реакції непрямої гемаглютинації. Який стандартний препарат слід застосувати у цій реакції?

Еритроцити барана й гемолітичну сироватку

Моноклональні антитіла

Монорецепторні діагностичні сироватки

Еритроцитарні діагностикуми з адсорбованими антигенами бактерій

Антитіла проти імуноглобулінів основних класів

2993 / 6307
У туриста під час тривалого перебування на спекоті відбулася значна втрата води, що супроводжувалося різким зниженням діурезу. Посилення секреції яких гормонів відбувається при цьому?

Ілюкокортикоїди й інсулін

Вазопресин й альдостерон

Тироксин і трийодтиронін

Адреналін і норадреналін

Серотонін і дофамін

2994 / 6307
Після травми на рентгенограмі тазу виявлена тріщина кістки, яка має затульну борозну. Яка це кістка?

Крижова

Лобкова

Клубова

Сіднична

Куприкова

2995 / 6307
У пацієнта стоматологічного відділення виявлено хворобу Педжета, що супроводжується деградацією колагену. Вирішальним фактом для постановки діагнозу було виявлення у сечі хворого підвищеного рівня:

Серину

Оксипроліну

Аргініну

Триптофану

Аланіну

2996 / 6307
Школяр 8-ми років звернувся до стоматолога з герпетичним висипанням на нижній губі. Який найбільш ефективний засіб слід призначити?

Фурадонін

Кетоконазол

Ацикловір

Оксацилін

Ампіцилін

2997 / 6307
У пацієнта перед кардіологічною операцією зареєстровано тиск у всіх відділах серця. Який тиск в лівому шлуночку під час діастоли?

0 мм рт.ст.

120 мм рт.ст.

80 мм рт.ст.

100 мм рт.ст.

40 мм рт.ст.

2998 / 6307
Хворий 47-ми років впродовж останніх 3-х років хворіє на туберкульоз легень, скаржиться на задишку, важкість в області правого боку грудної стінки, температуру тіла 37,7°С. Виявлено правобічний ексудативний плеврит. Який тип клітин передбачається у плевральному пунктаті?

Еозинофіли

Еритроцити

Нейтрофіли

Лімфоцити

Атипові клітини

2999 / 6307
У п’ятирічного хлопчика спостерігалися малий зріст, розумове відставання, обмежені рухи, грубі риси обличчя. ЦІ особливості стали помітними з 18-місячного віку. У нього виявили дефіцит L-ідуронідази. Обмін яких сполук порушено?

Глікозаміноглікани

Нуклеотиди

Фосфоліпіди

Вітаміни

Білки

3000 / 6307
Дитячий стоматолог мав контакт з хворим на дифтерію підлітком. Час останньої імунізації лікаря проти дифтерії - 12 років. Який препарат необхідно ввести стоматологу?

Жива вакцина

Інтерферон

Антитоксична протидифтерійна сироватка

Хімічна вакцина

Рекомбінантна вакцина

3001 / 6307
У хворого після перелому верхньої третини плечової кістки розвинувся параліч задньої групи м’язів плеча i передпліччя. Який нерв пошкоджено?

Пахвовий

Серединний

Ліктьовий

Променевий

М’язово-шкірний

3002 / 6307
Хворому з ревматоїдним артритом тривалий час вводили гідрокортизон. У нього з’явилися гіперглікемія, поліурія, глюкозурія, спрага. Ці ускладнення лікування є наслідком активації процесу:

Гліколіз

Глікогеноліз

Глюконеогенез

Ліполіз

Глікогенез

3003 / 6307
Для корекції артеріального тиску при колаптоїдному стані хворому було введено мезатон. Який механізм дії даного препарату?

Блокує β-адренорецептори

Стимулює β-адренорецептори

Стимулює α-адренорецептори

Стимулює α- і β-адренорецептори

Блокує α-адренорецептори

3004 / 6307
На електронній мікрофотографії представлені структури у вигляді відкритих міхурців, внутрішня поверхня яких вистелена одношаровим епітелієм, який утворений респіраторними та секреторними клітинами. Які це структури?

Альвеоли

Бронхіоли

Термінальні бронхіоли

Ацинуси

Альвеолярні ходи

3005 / 6307
В гістопрепараті представлений паренхіматозний орган, поверхневий шар кіркової речовини якого формують клубочки, утворені ендокриноцитами. Якому органу належить дана морфологічна ознака?

Щитовидна залоза

Наднирник

Лімфатичний вузол

Селезінка

Яєчник

3006 / 6307
Після переходу до змішаного харчування у новонародженої дитини виникла диспепсія з діареєю, метеоризмом, відставанням у розвитку. Біохімічна основа даної патології полягає у недостатності:

Трипсину та хімотрипсину

Целюлази

Лактази та целобіази

Сахарази та ізомальтази

Ліпази та креатинкінази

3007 / 6307
Хворому перед операцією було введено дитилін (лістенон) і проведено інтубацію. Дефіцит якого ферменту в організмі хворого подовжує дію м’язового релаксанту?

K-Na-АТФ-аза

N-ацетилтрансфераза

Сукцинатдегідрогеназа

Псевдохолінестераза

Карбангідраза

3008 / 6307
У хворого із захворюванням печінки виявлено зниження вмісту протромбіну в крові. Це призведе, перш за все, до порушення:

Судинно-тромбоцитарного гемостазу

Другої фази коагуляційного гемостазу

Антикоагулянтних властивостей крові

Фібринолізу

Першої фази коагуляційного гемостазу

3009 / 6307
У хворого із сечокам’яною хворобою виникли нестерпні спастичні болі. Для попередження больового шоку йому ввели разом з атропіном наркотичний анальгетик, що не має спазмогенного ефекту. Який це був препарат?

Пірітрамід

Промедол

Етилморфіну гідрохлорид

Трамадол

Морфіну гідрохлорид

3010 / 6307
Яким буде скорочення м’язів верхньої кінцівки при намаганні підняти непосильний вантаж?

Ауксотонічне

Ізотонічне

Одиночне

Фазичне

Ізометричние

3011 / 6307
Для лікування урогенітальних інфекцій використовують хінолони - інгібітори ферменту ДнК-гірази. Який процес порушується під дією хінолонів у першу чергу?

Зворотна транскрипція

Реплікація ДНК

Рекомбінація генів

Ампліфікація генів

Репарація ДНК

3012 / 6307
Жінку госпіталізовано в клініку з симптомами гострого живота. При обстеженні виникла підозра на позаматкову вагітність. Яке з анатомічних утворень таза необхідно пропунктувати для підтвердження діагнозу?

Excavatio rectouterina

Excavatio rectovesicalis

Processus vaginalis peritonei

Fossa ischiorectalis

Excavatio vesicouterina

3013 / 6307
Щуру в плевральну порожнину введено 0,5 мл повітря. Який тип недостатності дихання виникає в даному випадку?

Обструктивне порушення альвеолярної вентиляції

Перфузійний

Рестриктивне порушення альвеолярної вентиляції

Дифузійний

Дисрегуляторне порушення альвеолярної вентиляції

3014 / 6307
Дитина 10-ти років страждає на стафілококовий дерматит. Лікування бензилпеніциліном не дало результатів. Призначення комбінованого препарату пеніциліну з клавулановою кислотою дало швидке одужання. Яка причина позитивної дії цього препарату?

Гальмування аденозиндезамінази

Блокада транслокази

Гальмування транспептидази

Інактивація β-лактамази

Активація фосфодіестерази

3015 / 6307
У хворого спостерігається типова для нападу малярії клінічна картина: озноб, жар, проливний піт. Яка стадія малярійного плазмодію найімовірніше буде виявлена в крові хворого в цей час?

Спороциста

Мерозоїт

Спорозоїт

Мікро- або макрогамети

Оокінета

3016 / 6307
У немовляти спостерігаються епілептиформні судоми, викликані дефіцитом вітаміну В6. Це спричинено зменшенням у нервовій тканині гальмівного медіатора - γ-аміномасляної кислоти. Активність якого ферменту знижена при цьому?

Глутаматсинтетаза

Глутаматдегідрогеназа

Глутаматдекарбоксилаза

Піридоксалькіназа

Аланінамінотрансфераза

3017 / 6307
Жінку 44-х років вжалила оса внаслідок чого розвинувся шок. В анамнезі вже була важка алергічна реакція на жалення оси. Об’єктивно: пульс -179/хв., слабкий, АТ- 80/40 ммрт.ст., ЧД-26/хв. Яка провідна ланка патогенезу анафілактичного шоку?

Тахікардія

Зниження периферійного опору судин

Біль

Зменшення ударного об’єму серця

Зменшення об’єму циркулюючої крові

3018 / 6307
У препараті в одній з судин мікроциркуляторного русла середня оболонка утворена 1-2 шарами гладеньких міоцитів, які розташовані поодинці і мають спіралеподібний напрямок. Зовнішня оболонка представлена тонким шаром пухкої волокнистої сполучної тканини. Вкажіть вид судини:

Венула

Капіляр

Посткапіляр

Артеріоловенулярний анастомоз

Артеріола

3019 / 6307
Фекалії дитини, що хворіє на ентерит, емульгують в фізіологічному розчині і краплю емульсії наносять на елективне середовище: 10% молочно-сольовий, або жовтково-сольовий агар. Який мікроорганізм передбачається виділити?

Стафілокок

Кишкова паличка

Стрептокок

Клебсієла

Ентерокок

3020 / 6307
При обстеженні хворого виявлена характерна клініка колагенозу Вкажіть, збільшення якого показника сечі характерне для цієї патології:

Глюкоза

Мінеральні солі

Аргінін

Солі амонію

Гідроксипролін

3021 / 6307
При патологічних процесах, які супроводжуються гіпоксією, відбувається неповне відновлення молекули кисню в дихальному ланцюзі і накопичення пероксиду водню. Вкажіть фермент, який забезпечує його руйнування:

Кетоглутаратдегідрогеназа

Каталаза

Сукцинатдегідрогеназа

Аконітаза

Цитохромоксидаза

3022 / 6307
Електрофоретичне дослідження сироватки крові хворого пневмонією показало збільшення одної з білкових фракцій. Вкажіть її:

γ-глобуліни

α2 -глобуліни

β-глобуліни

α1 -глобуліни

Альбуміни

3023 / 6307
При травмі в області тазу у хворого на рентгенологічному знімку виявлено некроз головки стегнової кістки. Під час травми кульшового суглоба було пошкоджено зв’язку:

Лобкова-стегнова

Клубово-стегнова

Сіднично-стегнова

Головки стегнової кістки

3024 / 6307
Людині внутрішньовенно ввели 0,5 л ізотонічного розчину лікарської речовини. Які з рецепторів насамперед прореагують на зміни водно-сольового балансу організму?

Осморецептори гіпоталамусу

Волюморецептори порожнистих вен і передсердь

Осморецептори печінки

Барорецептори дуги аорти

Натрієві рецептори гіпоталамуса

3025 / 6307
В судово-медичній експертизі широко використовується метод дактилоскопії, який оснований на тому, що сосочковий шар дерми визначає строго індивідуальний малюнок на поверхні шкіри. Яка тканина утворює цей шар дерми?

Жирова тканина

Пухка волокниста неоформлена сполучна частина

Щільна неоформлена сполучна тканина

Ретикулярна тканина

Щільна оформлена сполучна тканина

3026 / 6307
Важливою складовою частиною ниркового фільтраційного бар’єру є тришарова базальна мембрана, яка має спеціальну сітчасту будову її середнього електроннощільного шару. Де міститься ця базальна мембрана?

Проксимальні канальці

Капіляри перитубулярної капілярної сітки

Ниркове тільце

Тонкі канальці

Дистальні прямі канальці

3027 / 6307
Чоловік 42-х років помер при явищах вираженої інтоксикації і дихальної недостатності. На розтині: тканина легень у всіх відділах строката, з множинними дрібновогнищевими крововиливами та вогнищами емфіземи. Гістологічно у легенях: геморагічна бронхопневмонія з абсцедуванням, у цитоплазмі клітин епітелію бронхів еозинофільні і базофільні включення. Діагностуйте виявлене на секції захворювання:

Крупозна пневмонія

Часткова пневмонія

Плевропневмонія

Грип

Стафілококова бронхопневмонія

3028 / 6307
В експерименті подразнюють скелетний м’яз серією електричних імпульсів. Який вид м’язового скорочення виникне, якщо кожний наступний імпульс припадає на період вкорочення поодинокого м’язового скорочення?

Серія поодиноких скорочень

Зубчастий тетанус

Асинхронний тетанус

Суцільний тетанус

Контрактура м’яза

3029 / 6307
У чоловіка 53-х років діагностовано сечокам’яну хворобу з утворенням уратів. Цьому пацієнту призначено аллопурінол, який є конкурентним інгібітором ферменту:

Уриділтрансфераза

Ксантиноксидаза

Уреаза

Дигідроурацилдегідрогеназа

Уратоксидаза

3030 / 6307
Мати зауважила занадто темну сечу у її 5-річної дитини. Дитина скарг не висловлює. Жовчних пігментів у сєчі не виявлено. Поставлено діагноз алкаптонурія. Дефіцит якого ферменту має місце у дитини?

Декарбоксилаза фенілпірувату

Оксидаза гомогентизинової кислоти

Фенілаланінгідроксилаза

Тирозиназа

Оксидаза оксифенілпірувату

3031 / 6307
До дерматолога звернулася пацієнтка із скаргами на екзематозне ураження шкіри рук, що з’являється після контакту з миючим засобом ”Лотос” Використання гумових рукавичок запобігає цьому. Патологічна реакція шкіри зумовлена активацією:

Моноцитів

Базофілів

Нейтрофілів

В-лімфоцитів

Т-лімфоцитів

3032 / 6307
У хворої 36-ти років, яка лікувалася сульфаніламідами з приводу респіраторної вірусної інфекції, в крові гіпорегенераторна нормохромна анемія, лейкопенія, тромбоцитопенія. В кістковому мозку - зменшення кількості мієлокаріоцитів. Яка це анемія?

Гіпопластична

Постгеморагічна

Залізодефіцитна

Гемолітична

В12-фолієводефіцитна

3033 / 6307
Чоловік 38-ми років раптово помер. На розтині: у задній стінці лівого шлуночка серця виявлено інфаркт міокарда. Які найбільш імовірні зміни у будові міокардіоцитів можна побачити у вогнищі інфаркту мікроскопічно?

Вуглеводна дистрофія

Жирова дистрофія

Каріолізис

Білкова дистрофія

Звапнування

3034 / 6307
У відділення реанімації надійшов чоловік 47-ми років з діагнозом інфаркт міокарда. Яка з фракцій лактатдегідрогенази (ЛДГ) буде переважати в сироватці крові впродовж перших двох діб захворювання?

ЛДГ2

ЛДГ4

ЛДГ5

ЛДГ1

ЛДГ3

3035 / 6307
У хлопчика 2-х років спостерігається збільшення в розмірах печінки та селезінки, катаракта. В крові підвищена концентрація цукру, але тест толерантності до глюкози в нормі. Спадкове порушення обміну якої речовини є причиною цього стану?

Фруктоза

Глюкоза

Мальтоза

Сахароза

Галактоза

3036 / 6307
У неврологічне відділення з приводу мозкового крововиливу поступив хворий 62-х років. Об’єктивно: стан важкий. Спостерігається наростання глибини і частоти дихання, а потім його зменшення до апное, після чого цикл дихальних рухів відновлюється. Який тип дихання у хворого?

Апнейстичне

Чейна-Стокса

Кусмауля

Гаспінг-дихання

Біота

3037 / 6307
Глікоген, що надійшов з їжею, гідролізується у шлунково-кишковому тракті. Який кінцевий продукт утворюється в результаті цього процесу?

Глюкоза

Лактат

Лактоза

Галактоза

Фруктоза

3038 / 6307
Після перенесеного геморагічного інсульту у хворого розвинулася кіста головного мозку. Через 2 роки помер від післягрипозної пневмонії. На секції трупа виявлено в мозку кісту із стінками біло-іржавого відтінку, реакція Перлса позитивна. Який з процесів найбільш імовірний у стінці кісти?

Місцевий гемомеланоз

Первинний гемохроматоз

Місцевий гемосидероз

Загальний гемосидероз

Інфільтрація білірубіну

3039 / 6307
При розтині тіла померлого чоловіка 73-х років, який довго страждав на ішемічну хворобу серця з серцевою недостатністю, знайдено: ’’мускатна” печінка, бура індурація легень, ціанотична індурація нирок та селезінки. Який з видів порушення кровообігу найбільш імовірний?

Гостре загальне венозне повнокрів’я

Хронічне малокрів’я

Хронічне загальне венозне повнокрів’я

Артеріальна гіперемія

Гостре малокрів’я

3040 / 6307
У нейрохірургічне відділення поступив 54-річний чоловік із скаргами на відсутність чутливості шкіри нижньої повіки, латеральної поверхні зовнішнього носа, верхньої губи. Лікар при огляді встановлює запалення другої гілки трійчастого нерва. Через який отвір виходить із черепа ця гілка?

Остистий отвір

Овальний отвір

Круглий отвір

Верхня очноямкова щілина

Рваний отвір

3041 / 6307
Чоловіку 18-ти років з приводу флегмони плеча було зроблено внутрішньом’язову ін’єкцію пеніциліну. Після цього у нього з’явилися тахікардія, ниткоподібний пульс, АТ знизився до 80/60 мм рт.ст. Який вид фармакологічної реакції розвинувся?

Центральна дія

Рефлекторна дія

Периферична дія

Анафілаксія

Потенціювання

3042 / 6307
Після побутової травми у пацієнта 18-ти років з’явились постійні запаморочення, ністагм очей, скандована мова, невпевнена хода. Це свідчить про порушення функції:

Вестибулярних ядер

Мозочка

Рухової кори

Базальних гангліїв

Чорної субстанції

3043 / 6307
Хворий 65-ти років, що страждає на атеросклероз, госпіталізований до хірургічного відділення з приводу розлитого гнійного перитоніту. Під час операції діагностовано тромбоз брижових артерій. Яка найбільш імовірна причина перитоніту?

Ішемія компресійна

Стаз

Ішемічний інфаркт

Геморагічний інфаркт

Ішемія ангіоспастична

3044 / 6307
Хворому з прогресуючою м’язовою дистрофією було проведено біохімічне дослідження сечі. Поява якої речовини у великій кількості в сечі може підтвердити захворювання м’язів у даного хворого?

Сечовина

Креатин

Гіпурова кислота

Креатинін

Порфірини

3045 / 6307
У крові чоловіка 26-ти років виявлено 18% еритроцитів сферичної, сплощеної, кулястої та остистої форм. Інші еритроцити були у формі двоввігнутих дисків. Як називається таке явище?

Фізіологічний пойкілоцитоз

Еритроцитоз

Патологічний пойкілоцитоз

Патологічний анізоцитоз

Фізіологічний анізоцитоз

3046 / 6307
Хвора страждає від болю в ногах та набряків. В ході обстеження хворої на медіальній поверхні стегна спостерігається набряк, збільшення розміру вен, утворення вузлів. З боку якої вени спостерігається патологія?

V profunda femoris

V saphena parva

V tibialis

V saphena magna

V femoralis

3047 / 6307
Під час підготовки пацієнта до операції на серці проведено вимірювання тиску в камерах серця. В одній з них тиск протягом серцевого циклу змінювався від 0 до 120 мм рт.ст. Назвіть цю камеру серця:

Правий шлуночок

Ліве передсердя

Лівий шлуночок

Праве передсердя

3048 / 6307
У жінки обмежений кровотік у нирках, підвищений артеріальний тиск. Гіперсекреція якого гормону зумовила підвищений тиск?

Ренін

Еритропоетин

Вазопресин

Норадреналін

Адреналін

3049 / 6307
На останньому місяці вагітності вміст фібриногену в плазмі крові в 2 рази вище за норму. Яку швидкість осідання еритроцитів слід при цьому очікувати?

5-10 мм/годину

0-5 мм/годину

40-50 мм/годину

3-12 мм/годину

10-15 мм/годину

3050 / 6307
Пацієнт 16-ти років, що страждає на хворобу Іценко-Кушінга, консультований з приводу надмірної ваги тіла. При опитуванні з’ясувалося, що енергетична цінність спожитої їжі складає 1700-1900 ккал/добу. Яка провідна причина ожиріння у даному випадку?

Нестача глюкокортикоїдів

Нестача інсуліну

Надлишок інсуліну

Гіподинамія

Надлишок глюкокортикоїдів

3051 / 6307
Людина зробила спокійний видих. Як називається об’єм повітря, який міститься у неї в легенях при цьому?

Залишковий об’єм

Резервний об’єм видиху

Функціональна залишкова ємкість легень

Дихальний об’єм

Життєва ємність легень

3052 / 6307
Людина зробила максимально глибокий видих. Як називається об’єм повітря, що знаходиться в її легенях після цього?

Ємність вдиху

Залишковий об’єм

Альвеолярний об’єм

Резервний об’єм видиху

Функціональна залишкова ємність легень

3053 / 6307
В експерименті на ссавці зруйнували певну структуру серця, що призвело до припинення проведення збудження від передсердь до шлуночків. Що саме зруйнували?

Синоатріальний вузол

Волокна Пуркін’є

Ніжки пучка Гіса

Атріовентрикулярний вузол

Пучок Гіса

3054 / 6307
Необхідно оцінити рівень збудливості нерва у хворого. Для цього доцільно визначити для нерва наступну величину:

Критичний рівень деполяризації

Тривалість потенціалу дії

Порогова сила подразника

Потенціал спокою

Амплітуда потенціалу дії

3055 / 6307
Хворому з гострою недостатністю надниркових залоз був призначений лікарський препарат після якого у нього з’явилися скарги на біль в кістках (двічі були переломи), часті простудні хвороби, набряки, повільне загоювання ран. Який препарат міг спричинити такі явища?

Ретаболіл

Естріол

Тестостерон

Спіронолактон

Преднізолон

3056 / 6307
Під час розтину тіла мертвонародженої дитини виявлено аномалію розвитку серця: шлуночки не розмежовані, з правої частини виходить суцільний артеріальний стовбур. Для яких хребетних характерна подібна будова серця?

Риби

Амфібії

Птахи

Рептилії

Ссавці

3057 / 6307
Який з нижченаведених сечогінних засобів слід призначити хворому з первинним гіперальдостеронізмом?

Спіронолактон

Маніт

Триамтерен

Гіпотіазид

Фуросемід

3058 / 6307
В результаті травми хворий не може підняти руку до горизонтального рівня. Який м’яз постраждав?

Триголовий

Дельтоподібний

Найширший м’яз спини

Трапецієподібний

Двоголовий

3059 / 6307
В експерименті збільшили проникність мембрани збудливої клітини для іонів калію. Які зміни електричного стану мембрани при цьому виникнуть?

Деполяризація

Змін не буде

Потенціал дії

Локальна відповідь

Гіперполяризація

3060 / 6307
При обстеженні в аналізі крові пацієнта виявлено лейкоцитоз, лімфоцитоз, клітини Боткіна- Гумпрехта на тлі анемії. Про яку хворобу слід думати лікарю?

Лімфогранулематоз

Мієломна хвороба

Хронічний лімфолейкоз

Інфекційний мононуклеоз

Гострий мієлолейкоз

3061 / 6307
У жінки 37-ми років протягом року періодично виникали інфекційні захворювання бактеріального генезу, їх перебіг був вкрай тривалим, ремісії - короткочасними. При обстеженні виявлена гіпогамаглобулінемія. Порушення функції яких клітин може бути прямою її причиною?

Лімфоцити

Макрофаги

Плазматичні клітини

Фагоцити

Нейтрофіли

3062 / 6307
У хворого 37-ми років на фоні тривалого застосування антибіотиків спостерігається підвищена кровоточивість при невеликих пошкодженнях. У крові - зниження активності факторів згортання крові ІІ, VII, IX, X, подовження часу згортання крові. Недостатністю якого вітаміну обумовлені зазначені зміни?

Вітамін Е

Вітамін D

Вітамін С

Вітамін А

Вітамін К

3063 / 6307
Хворий 18-ти років звернувся до лікарні із скаргами на шум та больові відчуття у вусі. Об’єктивно - у хворого гостре респіраторне захворювання, риніт. Крізь який отвір глотки інфекція потрапила до барабанної порожнини та викликала її запалення?

Барабанний отвір слухової труби

Зів

Вхід до гортані

Глотковий отвір слухової труби

Хоани

3064 / 6307
У хворого 68-ми років, який страждає на серцеву недостатність та впродовж тривалого часу приймає препарати наперстянки, з’явилися явища інтоксикації, які швидко нівелювалися застосуванням донатора сульфгідрильних груп унітіолу. Який механізм терапевтичної дії цього засобу?

Сповільнює надходження натрію до міокардіоцитів

Іальмує вивільнення калію з міокардіоцитів

Підвищує енергозабезпечення міокарду

Реактивує натрій-калієву-АТФ-азу мембран міокардіоцитів

Зменшує накопичення іонізованого кальцію

3065 / 6307
Охолодження тіла людини у воді виникає значно швидше, ніж на повітрі, тому, що у воді значно ефективнішою є віддача тепла шляхом:

Теплопроведення

Конвекції

Випаровування поту

Тепловипромінювання

3066 / 6307
У бактеріологічній лабораторії проводиться дослідження якості питної води. Її мікробне число виявилося близько 100. Які мікроорганізми враховувалися при цьому?

Бактерії групи кишкової палички

Ентеропатогенні бактерії та віруси

Всі бактерії, що виросли на живильному середовищі

Умовно-патогенні мікроорганізми

Бактерії, патогенні для людей та тварин

3067 / 6307
У юнака 20-ти років діагностовано спадковий дефіцит УДФ-глюкуронілтрансферази. Підвищення якого показника крові підтверджує діагноз?

Тваринний індикан

Уробілін

Непрямий (некон’югований) білірубін

Прямий (кон’югований) білірубін

Стеркобіліноген

3068 / 6307
Хворому 50-ти років з хронічною серцевою недостатністю і тахиаритмією призначили кардіотонічний препарат. Який з препаратів призначили хворому?

Добутамін

Дофамін

Дигоксин

Аміодарон

Мілдронат

3069 / 6307
У людини порушено всмоктування продуктів гідролізу жирів. Причиною цього може бути дефіцит у порожнині тонкої кишки:

Ліполітичних ферментів

Жовчних пігментів

Іонів натрію

Жиророзчинних вітамінів

Жовчних кислот

3070 / 6307
У відповідь на сильне швидке скорочення м’яза спостерігається його рефлекторне розслаблення. З подразнення яких рецепторів починається ця рефлекторна реакція?

М’язові веретена

Сухожилкові рецептори Гольджі

Дотикові рецептори

Больові рецептори

Суглобові рецептори

3071 / 6307
У хворого, що переніс 5 років тому субтотальну резекцію шлунка, розвинулась В12-фолієводефіцитна анемія. Який механізм є провідним у розвитку такої анемії?

Відсутність зовнішнього фактора Касла

Дефіцит фолієвої кислоти

Дефіцит транскобаламіну

Порушення всмоктування вітаміну В в тонкій кишці

Відсутність внутрішнього фактора Касла

3072 / 6307
Чоловік 50-ти років хворіє на хронічний бронхіт, скаржиться на задишку під час фізичного навантаження, постійний кашель з відходженням харкотиння. При обстеженні діагностовано ускладнення - емфізема легень. Чим вона зумовлена?

Зниження еластичних властивостей легень

Зменшення розтяжності легень

Порушення вентиляційно-перфузійного співвідношення в легенях

Зменшення перфузії легень

Зменшення альвеолярної вентиляції

3073 / 6307
При розтині трупа чоловіка 47-ми років, померлого раптово, в інтимі черевного відділу аорти знайдені осередки жовтого кольору у вигляді плям та смуг, що не вибухають над поверхнею інтими. При фарбуванні суданом III спостерігається жовтогаряче забарвлення. Для якої стадії атеросклерозу характерні такі зміни?

Ліпоїдозу

Стадія утворення атероматозної виразки

Атерокальцинозу

Ліпосклерозу

Атероматозу

3074 / 6307
У клініку госпіталізовано хворого з діагнозом карциноїду кишечника. Аналіз виявив підвищену продукцію серотоніну, який утворюється з амінокислоти триптофан. Який біохімічний механізм лежить в основі даного процесу?

Утворення парних сполук

Декарбоксилювання

Дезамінування

Трансамінування

Мікросомальне окиснення

3075 / 6307
У батьків, хворих на гемоглобінопатію (аутосомно-домінантний тип успадкування), народилася здорова дівчинка. Які генотипи батьків?

Обоє гетерозиготні за геном гемоглобінопатії

Обоє гомозиготні за геном гемоглобінопатії

Батько гетерозиготний за геном гемоглобінопатії, у матері цей ген відсутній

У обох батьків ген гемоглобінопатії відсутній

Мати гетерозиготна за геном гемоглобінопатії, у батька цей ген відсутній

3076 / 6307
Хворому з артеріальною гіпертензією було призначено один з антигіпертензивних засобів. Артеріальний тиск нормалізувався, однак хворого почав турбувати постійний сухий кашель. Який з перерахованих препаратів має таку побічну дію?

Резерпін

Ніфедипін

Лізиноприл

Клофелін

Анаприлін

3077 / 6307
В анотації до препарату вказано, що він містить антигени збудника черевного тифу, адсорбовані на стабілізованих еритроцитах барана. З якою метою використовують цей препарат?

Для серологічної ідентифікації збудника черевного тифу

Для виявлення антитіл в реакції зв’язування комплементу

Для виявлення антитіл в реакції гальмування гемаглютинації

Для виявлення антитіл в реакції Відаля

Для виявлення антитіл в реакції непрямої гемаглютинації

3078 / 6307
При розтині тіла померлого чоловіка 48-ми років в ділянці 1-го сегменту правої легені виявлено круглий утвір діаметром 5 см з чіткими контурами, оточений тонким прошарком сполучної тканини, виповнений білими крихкими масами. Діагностуйте форму вторинного туберкульозу:

Гострий кавернозний туберкульоз

Фіброзно-кавернозний туберкульоз

Гострий вогнищевий туберкульоз

Казеозна пневмонія

Туберкулома

3079 / 6307
У хворого діагностовано септичний ендокардит. Температура тіла протягом 5-ти днів коливалася в межах 39,5oC -40,2oC. На 6-й день на тлі різкого зниження температури до35,2oC розвинувся колапс. Який головний механізм колапсу?

Гіпервентиляція

Тахікардія

Поліурія

Вазодилатація

Посилене потовиділення

3080 / 6307
Хвора звернулася зі скаргами на болі у правій латеральній ділянці живота. Під час пальпації визначається щільне, нерухоме, пухлиноподібне утворення. У ділянці якого відділу травної трубки можлива наявність пухлини?

Colon sigmoideum

Colon ascendens

Colon descendens

Caecum

Colon transversum

3081 / 6307
У хворого струс головного мозку, що супроводжується повторним блюванням і задишкою. При обстеженні відзначено: рН - 7,62; pCO - 40 мм рт.ст. Яке порушення кислотно-основного стану є у хворого?

Газовий алкалоз

Газовий ацидоз

Негазовий ацидоз

Негазовий алкалоз

3082 / 6307
У хворого в обох щелепах рентгенологічно виявлено численні дефекти у вигляді гладкостінних округлих отворів. При гістологічному дослідженні - явища остеолізису і остеопорозу при явищах слабкого кісткоутворення. В сечі хворого знайдено білок Бенс-Джонса. Назвіть захворювання:

Хронічний еритромієлоз

Хронічний мієлолейкоз

Гострий недиференційований лейкоз

Мієломна хвороба

Гострий мієлолейкоз

3083 / 6307
Катіонні глікопротеїни є основними компонентами слини привушних залоз. Які амінокислоти обумовлюють їх позитивний заряд?

Глутамат, валін, лейцин

Лізин, аргінін, гістидин

Цистеїн, гліцин, пролін

Аспартат, аргінін, глутамат

Аспартат, глутамат, гліцин

3084 / 6307
На мікропрепараті очного яблука плода спостерігається пошкодження рогівки. Частина якого зародкового листка була уражена в процесі ембріонального розвитку?

Дерматом

Нефротом

Ентодерма

Мезодерма

Ектодерма

3085 / 6307
На практичному занятті з мікробіології студентам запропоновано пофарбувати суміш бактерій за методикою Грама та пояснити механізм фарбування. Які морфологічні структури бактерій зумовлюють грамнегативне та грампозитивне фарбування бактерій?

Цитоплазма

ЦПМ

Джгутики

Клітинна стінка

Капсула

3086 / 6307
У вагітної жінки взяли кров для підтвердження клінічного діагнозу 'токсоплазмоз' Яка з перерахованих серологічних реакцій має діагностичне значення?

Реакція аглютинації

Реакція зв’язування комплементу

Реакція гальмування гемаглютинації

Реакція нейтралізації

Реакція гемадсорбції

3087 / 6307
У здорових батьків, спадковість яких не обтяжена, народилася дитина з множинними вадами розвитку. Цитогенетичний аналіз виявив у соматичних клітинах дитини трисомію за 18-ю хромосомою (синдром Едвардса). З яким явищем пов’язане народження такої дитини?

Хромосомною мутацією - дуплікацією

Соматичною мутацією у ембріона

Домінантною мутацією

Впливом тератогенних факторів

Нерозходженням пари хромосом під час гаметогенезу

3088 / 6307
На препараті представлено орган, вкритий сполучнотканинною капсулою, від якої відходять трабекули. В органі можна розрізнити кіркову речовину, де містяться лімфатичні вузлики та мозкову речовину, представлену тяжами лімфоїдних клітин. Який орган представлений на препараті?

Лімфатичний вузол

Селезінка

Червоний кістковий мозок

Тимус

Мигдалики

3089 / 6307
При аналізі родоводу пробанда виявлено, що ознака проявляється з однаковою частотою у представників обох статей і хворі наявні у всіх поколіннях (по вертикалі), а по горизонталі - у сибсів (братів і сестер пробанда) з відносно великих родин. Який тип успадкування досліджуваної ознаки?

Зчеплений з Х-хромосомою, домінантний

Автосомно-домінантний

Зчеплений з Х-хромосомою, рецесивний

Автосомно-рецесивний

Зчеплений з Y-хромосомою

3090 / 6307
У 60-річного пацієнта було виявлено гіперглікемію і глюкозурію. Для лікування хворого лікар призначив препарат для приймання всередину. Який це препарат?

Глібенкламід

Окситоцин

Фуросемід

Корглікон

Панкреатин

3091 / 6307
Хворий 58-ми років помер від прогресуючої серцевої недостатності. На розтині: серце розширене у поперечнику, мляве, м’яз на розрізі нерівномірного кровонаповнення, пістрявий. При гістологічному дослідженні: у міокарді повнокров’я, у стромі лімфогістіоцитарні інфільтрати, що розсувають кардіоміоцити. Виявлені морфологічні зміни свідчать про:

Венозне повнокров’я

Інфаркт міокарда

Негнійний проміжний міокардит

Жирову дистрофію міокарда

Кардіосклероз

3092 / 6307
У важкоатлета при підйомі штанги відбувся розрив грудної лімфатичної протоки. Вкажіть найбільш імовірне місце ушкодження:

Заднє середостіння

Ділянка попереково-крижового сполучення

Ділянка аортального отвору діафрагми

Місце впадіння у венозний кут

Ділянка шиї

3093 / 6307
При пошкодженні клітини іонізуючим випромінюванням вмикаються механізми захисту і адаптації. Який механізм відновлення порушеного внутрішньоклітинного гомеостазу реалізується при цьому?

Активація антиоксидантної системи

Накопичення Na+ в клітинах

Гіпертрофія мітохондрій

Пригнічення аденілатциклази

Активація Са-опосередкованих клітинних функцій

3094 / 6307
У хворої після видалення матки розвинулась гостра анурiя (немає виділення сечі). Які анатомiчнi структури найімовірніше було пошкоджено при операції?

Цибулинно-губчастий м’яз

Сечівник

Зовнішній сфінктер сечівника

Внутрішній сфінктер сечівника

Сечоводи

3095 / 6307
У хворої внаслідок запалення порушена ендокринна функція фолікулярних клітин фолікулів яєчника. Синтез яких гормонів буде пригнічений?

Лютропін

Фолістатин

Прогестерон

Естрогени

Фолікулостимулюючий гормон

3096 / 6307
У чоловіка при обстеженні виявлено порушення кровообігу міокарда лівого передсердя. У басейні якої артерії відбулись порушення кровообігу?

Передня міжшлуночкова гілка лівої вінцевої артерії

Ліва вінцева

Права вінцева

Права та ліва вінцеві

3097 / 6307
Чоловік 36-ти років, лісник за фахом, через тиждень після тривалого перебування у весняному лісі гостро захворів - гарячка, головний біль, порушення свідомості,епілептиформні напади, смерть розвинулася на 3-й день хвороби. На розтині тіла: набрякголовного мозку, множинні точкові геморагії; під час мікроскопічного дослідження - периваскулярний та перицелюлярний набряк, множинні периваскулярні, переважно лімфоцитарні, інфільтрати. Діагностуйте основне захворювання:

Менінгококова інфекція

Церебро-васкулярна хвороба

Поліомієліт

Гнійний енцефаліт

Кліщовий енцефаліт

3098 / 6307
При зовнішньому дослідженні трупа чоловіка 69-ти років, який помер 4 години тому, патологоанатом відмітив, що м’язи померлого мають дуже щільну консистенцію, суглоби згинаються та розгинаються важко. Як називається ця патологоанатомічна ознака смерті?

Трупне охолодження

Трупне розкладання

Трупне заклякання

Трупні гіпостази

Трупне висихання

3099 / 6307
Хворому з метою попередження жирової дистрофії печінки лікар призначив ліпотропний препарат - донор метильних груп. Це імовірно:

Глюкоза

Холестерин

S-Аденозилметіонін

Валін

Білірубін

3100 / 6307
Хворому на гострий інфаркт міокарда у комплексній терапії було призначено гепарин. Через деякий час після введення даного препарату з’явилась гематурія. Який антагоніст гепарину необхідно ввести хворому для усунення даного ускладнення?

Вікасол

Протаміну сульфат

Амінокапронова кислота

Фібриноген

Неодикумарин

3101 / 6307
У хворого виявили злоякісну анемію. Терапія внутрішньо-м’язовим введення вітаміну Б12 давала нетривалий нестійкий ефект поліпшення складу крові. Пацієнт - завзятий рибаль і часто вживає самостійно виловлену і недостатньо термічно оброблену рибу. Який діагноз можна припустити?

Парагонімоз

Анкілостомоз

Дифілоботріоз

Ентеробіоз

Трихоцефальоз

3102 / 6307
У синтезі пуринових нуклеотидів беруть участь деякі амінокислоти, похідні вітамінів, фосфорні ефіри рибози. Коферментна форма якого вітаміну є переносником одновуглецевих фрагментів в цьому синтезі?

Піридоксин

Нікотинова кислота

Пантотенова кислота

Фолієва кислота

Рибофлавін

3103 / 6307
У тварини зруйнували отолітові вестибулорецептори. Які з наведених рефлексів зникнуть внаслідок цього у тварини?

Статокінетичні при рухах з лінійним прискоренням

Первинні орієнтувальні

Статокінетичні при рухах з кутовим прискоренням

Міотатичні

Випрямлення тулуба

3104 / 6307
Лікар записав в історії хвороби, що у хворого дихання поверхневе (знижена глибина дихання). Це означає, що зменшеним є такий показник зовнішнього дихання:

Хвилинний об’єм дихання

Життєва ємність легень

Функціональна залишкова ємність

Ємність вдиху

Дихальний об’єм

3105 / 6307
До лікаря звернувся студент з проханням призначити препарат для лікування алергічного риніту, який виник у нього під час цвітіння липи. Який засіб можна застосувати?

Лоратадин

Норадреналіну гідротартрат

Лозартан

Анаприлін

Амброксол

3106 / 6307
Хворий похилого віку страждає на хронічний закреп, в основі якого лежить гіпотонія товстої кишки. Який препарат слід призначити хворому?

Новокаїнамід

Натрію сульфат

Атропіну сульфат

Бісакодил

Касторова олія

3107 / 6307
При лабораторному дослідженні дитини виявлено підвищений вміст у крові та сечі лейцину, валіну, ізолейцину та їх кетопохідних. Сеча має характерний запах кленового сиропу. Недостатність якого ферменту характерно для цього захворювання?

Амінотрансфераза

Глюкозо-6-фосфатаза

Фосфофруктомутаза

Фосфофруктокіназа

Дегідрогеназа розгалужених амінокислот

3108 / 6307
При обстеженні 2-х місячної дитини педіатр звернула увагу, що плач дитини нагадує котячий крик. Діагностовані мікроцефалія і вада серця. За допомогою цитогенетичного метода з’ясований каріотип дитини: 46, XX, 5р. Дане захворювання є наслідком такого процесу:

Транслокація

Плейотропія

Інверсія

Дуплікація

Делеція

3109 / 6307
Під час аутопсії тіла хворого, котрий помер від тяжкого ексикозу, на ґрунті профузної діареї, виявлені такі зміни: слизова оболонка прямої та сигмоподібної кишок на всьому протязі вкрита сіро-білими плівчастими нашаруваннями, що міцно з’єднані з підлеглими тканинами, між плівками розташовані множинні великі й поверхневі виразки, вкриті згортками крові. Мікроскопічно діагностовано фібринозно-виразковий коліт. Яке захворювання проявляється такими змінами?

Сальмонельози

Дизентерія

Ієрсініоз

Стафілококова інфекція

Колі-інфекція

3110 / 6307
У лікарню до кінця робочого дня доставлений працівник 'гарячого' цеху, який скаржиться на головний біль, запаморочення, нудоту, загальну слабкість. Свідомість збережена, шкірні покриви гіперемовані, сухі, гарячі на дотик. ЧСС- 130/хв. Дихання часте, поверхневе. Яке порушення процесів регуляції тепла найімовірніше виникло у людини у даній ситуації?

Посилення тепловіддачі і зниження теплопродукції

Зниження теплопродукції без зміни тепловіддачі

Посилення тепловіддачі і теплопродукції

Посилення теплопродукції без змін тепловіддачі

Зниження тепловіддачі

3111 / 6307
В експерименті подразнюють гілочки симпатичного нерва, які іннервують серце. Це призвело до збільшення сили серцевих скорочень, тому що через мембрану типових кардіоміоцитів збільшився:

Вихід іонів калію

Вхід іонів кальцію та калію

Вхід іонів калію

Вихід іонів кальцію

Вхід іонів кальцію

3112 / 6307
У пацієнта 60-ти років виявлено збільшення порогу сприймання звуків високої частоти. Зміна функцій яких структур слухового аналізатора зумовлює виникнення цього порушення?

М’язів середнього вуха

Органу Корті ближче до овального віконця

Органу Корті ближче до гелікотреми

Євстахієвої труби

Барабанної перетинки

3113 / 6307
У 3-річної дитини тривале підвищення температури, збільшені лімфовузли, у крові - значне підвищення кількості лімфоцитів. Методом ІФА виявлено антиген віруса Епштейна-Бара. Який діагноз можна поставити на основі вказаного?

Інфекційний мононуклеоз

Цитомегаловірусна інфекція

Іерпетична аденопатія

Лімфома Беркета

Іенералізована інфекція, викликана herpes-zoster

3114 / 6307
У дитини виявлена схильність до ожиріння, яка є результатом діатезу. Назвіть вид діатезу, при якому частіше може розвинутись ожиріння:

Лімфатико-гіпопластичний

Ексудативно-катаральний

Нервово-артритичний

Астенічний

3115 / 6307
У юнака 18-ти років діагностовано хворобу Марфана. При дослідженні встановлено порушення розвитку сполучної тканини, будови кришталика ока, аномалії серцево-судинної системи, арахнодактилію. Прикладом якого генетичного явища є ця хвороба?

Кодомінування

Неповне домінування

Комплементарність

Множинний алелізм

Плейотропія

3116 / 6307
Ураження хворого одноразовою дозою іонізуючого випромінювання спричинило розвиток кістково-мозкової форми променевої хвороби. Які патологічні прояви з боку крові будуть характерними в період удаваного благополуччя?

Наростаюча лімфопенія, лейкопенія

Тромбоцитопенія, лейкоцитоз

Тромбоцитопенія, анемія

Анемія, лейкопенія

Перерозподільчий лейкоцитоз, лім-фоцитоз

3117 / 6307
У хворого на черевний тиф при проведенні серологічного дослідження (реакція Відаля) виявлено О- і Н-аглютитиніни у титрі 1:800 і 1:200 відповідно. Це свідчить про:

Неможливість підтвердити діагноз

Початок захворювання

Проведене щеплення

Раніше перенесене захворювання

Період реконвалесценції

3118 / 6307
У дитячому колективі проведено планову вакцинацію проти кору. Яким методом можна перевірити ефективність проведеної вакцинації?

Серологічний

Біологічний

Вірусоскопічний

Алергопроба

Вірусологічний

3119 / 6307
У клінічній практиці застосовують для лікування туберкульозу препарат ізоніазид - антивітамін, який здатний проникати у туберкульозну палочку. Туберкулостатичний ефект обумовлений порушенням процесів реплікації, окисно-відновних реакцій завдяки утворенню несправжнього коферменту з:

ФМН

ФАД

КоО

НАД

ТДФ

3120 / 6307
При огляді хворої лікар-гінеколог відмітив симптоми запалення статевих шляхів, у мазку взятому із піхви, виявлено грушоподібні найпростіші з шипом, з передньої частини відходять джгутики, наявна ундулююча мембрана. Яке захворювання підозрює лікар у хворої?

Лямбліоз

Токсоплазмоз

Кишковий трихомоноз

Урогенітальний трихомоноз

Балантидіоз

3121 / 6307
У чоловіка 32-х років, хворого на пневмонію, спостерігається закупорка харкотинням дихальних шляхів. В організмі хворого при цьому буде розвиватися така зміна кислотно-лужної рівноваги:

Респіраторний алкалоз

Метаболічний ацидоз

Змін не буде

Респіраторний ацидоз

Метаболічний алкалоз

3122 / 6307
При аналiзi ЕКГ виявлено випадіння деяких серцевих циклів PQRST. Наявні зубці та комплекси не змінені. Назвать вид аритмії:

Передсердна екстрасистола

Синоатріальна блокада

Внутрішньопередсердна блокада

Миготлива аритмія

Атріовентрикулярна блокада

3123 / 6307
У хворої встановлено порушення виділення тиреотропного гормону гіпофіза. Зі зниженням функцій якої частки гіпофіза це пов’язано?

Infundibulum

Pars intermedia

Lobus anterior

Lobus posterior

3124 / 6307
Стресовий стан і больове відчуття у пацієнта перед візитом до стоматолога супроводжуються анурією (відсутністю сечовиділення). Це явище зумовлене збільшенням:

Секреції адреналіну та зменшенням вазопресину

Активності антиноціцептивної системи

Секреції вазопресину та зменшенням адреналіну

Активності парасимпатичної нервової системи

Секреції вазопресину та адреналіну

3125 / 6307
Введення знеболюючого пацієнту перед екстракцією зуба призвело до розвитку анафілактичного шоку, який супроводжувався розвитком олігурії. Який патогенетичний механізм зумовив зменшення діурезу в даній клінічній ситуації?

Зменшення кількості функціонуючих нефронів

Збільшення онкотичного тиску крові

Зниження гідростатичного тиску в капілярах клубочків

Підвищення гідростатичного тиску в капсулі Шумлянського-Боумена

Пошкодження клубочкового фільтру

3126 / 6307
Хвора на ревматоїдний артрит після трьохтижневого лікування преднізолоном почала скаржитись на перебої в роботі серця. З чим пов’язаний розвиток даного небажаного ефекту препарату?

Гіпокаліємія

Гіперкаліємія

Гіпоглікемія

Гіперурікемія

Гіперглікемія

3127 / 6307
У хворого 45-ти років на тлі трансмурального інфаркту міокарда розвинулася гостра лівошлуночкова недостатність. Який лікарський засіб доцільно застосувати у даній ситуації для покращення помпової функції серця?

Промедол

Добутамін

Ізадрин

Еуфілін

Ефедрин

3128 / 6307
Для профілактики атеросклерозу, ішемічної хвороби серця, порушень мозкового кровообігу рекомендується споживання жирів із високим вмістом поліненасичених жирних кислот. Однією з таких жирних кислот є:

Лауринова

Пальмітоолеїнова

Олеїнова

Стеаринова

Лінолева

3129 / 6307
У хворого з нагноєнням рани при бактеріологічному дослідженні ранового вмісту виявлено грамнегативну паличку, яка на МПА утворює напівпрозорі слизові колонії синьо-зеленого кольору з перламутровим відтінком. Культура має специфічний запах фіалок або жасмину. Який вид збудника виділений з рани хворого?

S. faecalis

P aeruginosa

S. pyogenes

P vulgaris

S. aureus

3130 / 6307
Після обстеження пацієнта в клініці нервових хвороб встановлена відсутність звуження зіниці при дії світла. З ураженням яких структур головного мозку це пов’язано?

Ретикулярні ядра середнього мозку

Червоні ядра середнього мозку

Ядра гіпоталамуса

Вегетативні ядра 3 пари черепно-мозкових нервів

Ретикулярш ядра довгастого мозку

3131 / 6307
У хворого під час комп’ютерної томографії грудної клітки діагностовано пухлину заднього нижнього середостіння. Яка з перерахованих структур стиснута пухлиною?

Aorta thoracica

Trachea

N. phrenicus

Vena cava superior

Arcus aortae

3132 / 6307
Під час операції холецистектомії у хірурга виникла необхідність визначити топографію загальної жовчної протоки. Злиттям яких проток утворюється дана анатомічна структура?

Загальної печінкової і лівої печінкової проток

Загальної печінкової і правої печінкової проток

Правої та лівої печінкових проток

Лівої печінкової і міхурової проток

Загальної печінкової і міхурової проток

3133 / 6307
Дитина 6-ти років під час гри порізала ногу осколком скла і була направлена у поліклініку для введення протиправцевої сироватки. З метою попередження розвитку анафілактичного шоку лікувальну сироватку вводили за методом Безредка. Який механізм лежить в основі подібного способу гіпосенсибілізації організму?

Блокування синтезу медіаторів у тучних клітинах

Зв’язування фіксованих на тучних клітинах IgE

Зв’язування рецепторів до IgE на тучних клітинах

Стимуляція імунологічної толерантності до антигену

Стимуляція синтезу антиген-специфічних IgG

3134 / 6307
Хворому лікар призначив протикашльовий препарат центральної дії, який є алкалоїдом мачку жовтого. Діє на кашльовий центр вибірково, не пригнічуючи дихання, не затримує виділення харкотиння. Не викликає обстипації і лікарської залежності. Можна призначати дітям. Визначте препарат:

Бромгексин

Лібексин

Окселадин

Кодеїну фосфат

Глауцину гідрохлорид

3135 / 6307
Хлопчику 5-ти років був встановлений діагноз - міастенія. Оберіть препарат з групи антихолінестеразних засобів, який покращує нервово-м’язову передачу:

Іалантаміну гідробромід

Прозерин

Ацеклідин

Алоксим

Армін

3136 / 6307
Хворому 63-х років з атонією сечового міхура лікар призначив препарат, дозу якого хворий самостійно збільшив. З’явились підвищене потовиділення, салівація, діарея, м’язові спазми. Препарат якої групи був призначений?

Токолітики

Адреноблокатори

Гангліоблокатори

Холіноміметики

Реактиватори холінестерази

3137 / 6307
Під час розтину тіла жінки 28-ми років, яка померла від геморагічного шоку, виявлено: гемоперитонеум, права маткова труба збільшена у розмірах, багряна, з наскрізним дефектом стінки, її дилятований просвіт виповнений темно-червоними згортками крові. Гістологічно у слизовій оболонці труби та серед м’язових клітин визначаються пласти великих світлих децидуальних клітин, у м’язовій оболонці та серед згортків крові у просвіті труби - ворсинки хоріона. Діагностуйте патологію вагітності:

Порушена трубна вагітність

Порушена маткова вагітність

Інтерлігаментарна вагітність

Деструюючий міхурцевий занесок

Черевна вагітність

3138 / 6307
У біоптаті нирки 45-річного чоловіка, що має хронічну хворобу нирок, виявлено: склероз, лімфо-плазмоцитарна інфільтрація стінок мисок та чашок, дистрофія та атрофія канальців. Збережені канальці розширені, розтягнені колоїдоподібними масами, епітелій сплющений (’’щитоподібна” нирка). Який діагноз найбільш імовірний?

Нефросклероз

Гломерулонефрит

Хронічний пієлонефрит

Гострий пієлонефрит

Тубуло-інтерстиційний нефрит

3139 / 6307
Чоловік 40 років хворіє на гіперацидний гастрит з нічними голодними болями. Призначте хворому лікарський засіб - блокатор гістамінових H2-рецепторів III покоління, який знизить виділення хлористоводневої кислоти (особливо вночі) та збільшить утворення захисного слизу:

Метацин

Фамотидин

Платифіліну гідротартрат

Пірензепін

Атропіну сульфат

3140 / 6307
Під час футбольного матчу між вболівальниками різних команд виникла сутичка. На фоні негативних емоцій в одного учасника сутички були розширені зіниці й підвищене серцебиття. Активація якої системи регуляції функцій організму забезпечує такі вегетативні зміни при негативних емоціях?

Соматична нервова

Парасимпатична нервова

Метасимпатична нервова

Гіпоталамо-гіпофізарно-тиреоїдна

Симпато-адреналова

3141 / 6307
У хворого 32-х років після оперативного втручання розвинувся гнійний процес. Із гною рани виділено культуру S. aureus. Який з перерахованих тестів найдоцільніше використати для диференціації S. aureus від S. epidermidis?

Гемоліз на кров’яному агарі

Оксидазний тест

Колір колонії

Ферментація арабінози

Плазмокоагулазна активність

3142 / 6307
Чоловікові 58-ми років зроблено операцію з приводу раку простати. Через 3 місяці йому проведено курс променевої та хіміотерапії. До комплексу лікарських препаратів входив 5-фтордезоксиуридин - інгібітор тимідилатсинтази. Синтез якої речовини блокується цим препаратом?

р-РНК

і-РНК

т-РНК

ДНК

3143 / 6307
В підводному човні під час занурення порушилася система подачі кисню. У підводників збільшилися частота дихання і серцевих скорочень. Який вид гіпоксії розвинувся у підводників?

Дихальна

Кров’яна

Тканинна

Гіпоксична

Серцево-судинна

3144 / 6307
Синтез і-РНК проходить на матриці ДНК з урахуванням принципу комплементарності. Якщо триплети у ДНК наступні - АТГ-ЦГТ, то відповідні кодони і-РНК будуть:

АУГ-ЦГУ

УАГ-ЦГУ

ТАГ-УГУ

УАЦ-ГЦА

АТГ-ЦГТ

3145 / 6307
У процесі фібринолізу кров’яний тромб розсмоктується. Розщеплення нерозчинного фібрину відбувається шляхом його гідролізу під дією протеолітичного ферменту плазміну, який наявний у крові в неактивній формі плазміногену. Активується плазміноген шляхом обмеженого протеолізу за участю фермента:

Трипсин

Урокіназа

Ентерокіназа

Хімотрипсин

Пепсин

3146 / 6307
У біоптаті щитоподібної залози виявлено атрофію паренхіматозних елементів, дифузну інфільтрацію тканини залози лімфоцитами та плазматичними клітинами з утворенням в ній лімфоїдних фолікулів. Для якого захворювання є характерними наведені ознаки?

Ендемічний зоб

Тиреоїдит Хасімото

Аденома щитоподібної залози

Тиреоїдит Ріделя

Хвороба Базедова

3147 / 6307
У хворого з жовтяницею встановлено: підвищення у плазмі крові вмісту загального білірубіну за рахунок непрямого (вільного), в калі і сечі - високий вміст стеркобіліну, рівень прямого (зв’язаного) білірубіну у плазмі крові в межах норми. Який вид жовтяниці має місце у хворого?

Хвороба Жильбера

Паренхіматозна

Механічна

Гемолітична

3148 / 6307
При гістологічному дослідженні біоптатів, взятих з потовщених країв виразки шлунка, виявлені невеликі гніздові скупчення різко атипових гіперхромних невеликих епітеліальних клітин, які розташовані серед дуже розвиненої строми. Визначте пухлину:

Аденома

Недиференційована саркома

Аденокарцинома

Медулярний рак

Скіррозний недиференційований рак

3149 / 6307
Жінка 69-ти років довго хворіла на атеросклероз. Поступила в хірургічне відділення з симптомами гострого живота. При лапаротомії виявлені: тромбоз мезентеріальної артерії, петлі тонкої кишки набряклі, багряно-чорного кольору, на їх серозній оболонці фібринозні нашарування. Який патологічний процес розвинувся у кишці хворої?

Секвестр

Немічний інфаркт

Коагуляційний некроз

Суха гангрена

Волога гангрена

3150 / 6307
Під час ректороманоскопії хворого зі скаргами на діарею виявлено, що слизова оболонка прямої і сигмоподібної кишок різко гіперемована, набрякла, вкрита великою кількістю слизу, а у деяких ділянках вкрита плівчастими накладаннями зеленуватого кольору. Про яке захворювання можна думати?

Холера

Дизентерія

Черевний тиф

Амебіаз

Сальмонельоз

3151 / 6307
При копрологічному дослідженні у працівників кав’ярні лікарями санітарно- епідеміологічної станції були виявлені округлі цисти, характерною ознакою яких є наявність чотирьох ядер. Імовірніше за все у цих працівників безсимптомно паразитує:

Лямблія

Кишкова трихомонада

Амеба кишкова

Балантидій

Дизентерійна амеба

3152 / 6307
У хворого з варикозним розширенням вен під час огляду нижніх кінцівок відзначається: ціаноз, пастозність, зниження температури шкіри, поодинокі петехії. Який розлад гемодинаміки має місце у хворого?

Артеріальна гіперемія

Обтураційна ішемія

Тромбоемболія

Компресійна ішемія

Венозна гіперемія

3153 / 6307
При гістологічному дослідженні органів і тканин померлої від ниркової недостатності молодої жінки, у якої прижиттєво виявлявся високий титр антинуклеарних антитіл, виявлені поширені фібриноїдні зміни в стінках судин. Відмічається ядерна патологія з вакуолізацією ядер, каріорексисом, утворенням гематоксилінових тілець. Який найбільш імовірний діагноз?

Атеросклероз

Системний червоний вовчак

Облітеруючий ендартеріїт

Гіпертонічна хвороба

Вузликовий періартеріїт

3154 / 6307
Мікоплазми є своєрідною групою мікроорганізмів, що відносяться до родини Mycoplasmataceae і що мають властивості як бактерій, так і вірусів. Назвіть одну особливість мікоплазм, яка відрізняє їх від бактерій і вірусів:

Відсутність клітинної стінки

Висока ферментативна активність

Внутрішньоклітинний паразитизм

Спосіб розмноження

Відсутність клітинної будови

3155 / 6307
Після довготривалого вживання антибіотиків у хворого на слизовій ротової порожнини появилися округлі білі плями, на язику білий наліт. Який мікроорганізм імовірно спричинив дані симптоми?

Лактобацили

Ентерокок

Кишкова паличка

Стрептокок

Гриби роду Candida

3156 / 6307
Фенілкетонурія - це захворювання, яке зумовлено рецесивним геном, що локалізується в аутосомі. Батьки є гетерозиготами за цим геном. Вони вже мають двох хворих синів і одну здорову дочку. Яка імовірність, що четверта дитина, яку вони очікують народиться теж хворою?

75%

25%

100%

0%

50%

3157 / 6307
При диспансерному обстеженні у хворого знайдено цукор в сечі. Який найбільш імовірний механізм виявлених змін, якщо вміст цукру в крові нормальний?

Недостатня продукція інсуліну підшлунковою залозою

!нсулінорезистентність рецепторів клітин

Порушення реабсорбції глюкози в канальцях нефрона

Порушення фільтрації глюкози в клубочковому відділі нефрона

Гіперпродукція глюкокортикоїдів наднирниками

3158 / 6307
В пробірку, що містить розчин NaCl 0,9%, додана крапля крові. Що відбудеться з еритроцитами?

Осмотичний гемоліз

Набухання

Зморшкування

Залишаться без змін

Біологічний гемоліз

3159 / 6307
У хворого 40-ка років ознаки гірської хвороби: запаморочення, задишка, тахікардія, рН крові - 7,50, pCO2 - 30 мм рт.ст., зсув буферних основ +4 ммоль/л. Яке порушення кислотно-основного стану має місце?

Негазовий алкалоз

Видільний ацидоз

Газовий ацидоз

Газовий алкалоз

Негазовий ацидоз

3160 / 6307
У хворого 15-ти років концентрація глюкози натще - 4,8 ммоль/л, через годину після цукрового навантаження - 9,0 ммоль/л, через 2 години - 7,0 ммоль/л, через 3 години - 4,8 ммоль/л. Ці показники характерні для такого захворювання:

Цукровий діабет I типу

Хвороба !ценко-Кутінга.

Цукровий діабет II типу

Прихований цукровий діабет

3161 / 6307
Лікар призначив хворому з гострою серцевою недостатністю не-глікозидний кардіотонічний засіб, який безпосередньо стимулює в1-адренорецептори міокарда, що збільшує кровообіг, діурез. Застосовується лише внутрішньовенно крапельно внаслідок швидкої інактивації в організмі. Який препарат призначив лікар?

Анаприлін

Дигоксин

Добутамін

Адреналін

Корглікон

3162 / 6307
Чоловік 40-ка років перебував у пульмонологічному відділенні з приводу рецидивуючої правосторонньої пневмонії. Помер від легенево-серцевої недостатності. На розтині в правій легені визначається ділянка круглої форми 3х4 см. Вона являє собою порожнину з нерівними шорсткими краями, заповнену каламутною вершкоподібною жовто-зеленою рідиною. Мікроскопічно: стінка порожнини утворена тканиною легені з дифузною інфільтрацією лейкоцитами. Визначте патологічний процес у легені:

Гострий абсцес

!нфаркт

Емпієма

Гангрена

Хронічний абсцес

3163 / 6307
У вагітної жінки 24-х років після тривалого блювання було зареєстровано зниження об’єму циркулюючої крові. Про яку зміну загальної кількості крові може йти мова?

Олігоцитемічна гіповолемія

Поліцитемічна гіперволемія

Поліцитемічна гіповолемія

Олігоцитемічна гіперволемія

Проста гіповолемія

3164 / 6307
Пацієнту, який знаходився в клініці з приводу пневмонії, ускладненої плевритом, у складі комплексної терапії вводили преднізолон. Протизапальна дія цього синтетичного глюкокортикоїда пов’язана з блокуванням вивільнення арахідонової кислоти шляхом гальмування:

Ліпоксигенази

Фосфоліпази С

Фосфоліпази A2

Циклооксигенази

Пероксидази

3165 / 6307
У чоловіка 64-х років, який тривалий час курив і вживав міцні спиртні напої, на боковій поверхні язика виявили подібний на виразку утвір із білої, помірно щільної тканини розмірами 5х3 см. При гістологічному дослідженні біоптату виявили, що утвір побудований із клітини, які формують солідні структури і тяжі, що нагадують за будовою багатошаровий плоский епітелій, в якому клітини з вираженим поліморфізмом, з великими атиповими ядрами з патологічними мітозами. Діагностуйте виявлене у чоловіка захворювання:

Плоскоклітинний незроговілий рак

Лейкоплакія

Еритроплакія

Плоскоклітинний зроговілий рак

Рак на місці

3166 / 6307
Обстежуваний знаходиться у фазі повільнохвильового глибокого сну. Про це свідчить реєстрація на ЕЕГ таких хвиль:

Альфа-хвилі

Тета -хвилі

Бета-хвилі

Дельта-хвилі

Альфа-веретена

3167 / 6307
Лікар-стоматолог для лікування гінгівіту призначив пацієнту препарат з протипротозойною та антибактеріальною діями, який може викликати відразу до алкоголю. Вкажіть препарат, який призначив лікар:

Метронідазол

Лінкоміцину гідрохлорид

Цефтріаксон

Тетрациклін

Левоміцетин

3168 / 6307
Спеціальний режим харчування призвів до зменшення іонів Ca2+ в крові. До збільшення секреції якого гормону це призведе?

Паратгормон

Тироксин

Тирокальцитонін

Вазопресин

Соматотропін

3169 / 6307
У пацієнта після переохолодження на губах з’явились герпетичні висипання. Для лікування призначений крем ацикловіру, терапевтичний ефект якого пояснюється здатністю:

Гальмувати активність ДНК-полімерази

Уповільнювати реплікацію РНК- і ДНК-геномних вірусів

Порушувати функцію зворотної транскриптази

Гальмувати активність протеази

Інгібувати нейрамінідазу

3170 / 6307
На плановий прийом до педіатра батьки привели дитину віком 13 місяців. Під час повного огляду лікар перевірив розвиток II сигнальної системи дитини. Назвіть період, коли у людини вперше з’являються ознаки розвитку II сигнальної системи:

6-12 місяців

2-2,5 роки

2,5-3 роки

3-5 років

1,5-2 роки

3171 / 6307
У сироватці крові новонародженого виявлено антитіла до вірусу кору. Про наявність якого імунітету це може свідчити?

Штучний активний

Штучний пасивний

Природний активний

Спадковий, видовий

Природний пасивний

3172 / 6307
У хворого спостерігається пухлина тканин орбіти позаду очного яблука. Зазначено порушення акомодації та звуження зіниці ока. Яке анатомічне утворення ушкоджено?

N. lacrimalis

N. nasociliaris

N. opticus

Ganglion ciliare

N. trochlearis

3173 / 6307
У хворого виявлено зміну функції привушної слинної залози. Який з вузлів вегетативної нервової системи віддає післявузлові симпатичні волокна для неї?

Ganglion cervicale superius

Ganglion oticum

Ganglion pterygopalatinum

Ganglion cervicothoracicum

Ganglion submandibulare

3174 / 6307
Жінка 31-го року хворіє на В!Л- інфекцію на стадії СНІД. На шкірі нижніх кінцівок, слизової оболонки піднебіння з’явились рудувато-червоні плями, яскраво-червоні вузлики різних розмірів. Один з вузликів взято на гістологічне дослідження. Виявлено багато хаотично розташованих тонкостінних судин, вистелених ендотелієм, пучки веретеноподібних клітин з наявністю ге-мосидерину. Яка пухлина розвинулась у хворої?

Фібросаркома

Лімфангіома

Лімфома Беркіта

Гемангіома

Саркома Капоші

3175 / 6307
У альпініста, що піднявся на висоту 5200 м, розвинувся газовий алкалоз. Що є причиною його розвитку?

Гіпервентиляція легенів

Гіповентиляція легенів

Зниження температури навколишнього середовища

Гіпероксемія

Гіпоксемія

3176 / 6307
У хворого хронічна нежить. Набряк слизової оболонки носової порожнини призводить до порушення функції рецепторів нюхового нерва, які розташовані в нюховій ділянці носової порожнини. Через яке утворення волокна цього нерва потрапляють в передню черепну ямку?

Foramen ethmoidale anterior

Foramen incisivum

Foramen ethmoidale posterior

Lamina cribrosa os ethmoidale

Foramen sphenopalatinum

3177 / 6307
На гістологічному препараті в складі видовженої структури, обмеженої плазмолемою, по периферії розташовані численні ядра, а в цитоплазмі наявна поперечна посмугованість. Яка це структура?

Синцитіотрофобласт

Міосимпласт

Кардіоміоцит

Гладкий міоцит

Колагенове волокно

3178 / 6307
У спортсмена легкоатлета (бігуна на довгі дистанції) під час змагань розвинулась гостра серцева недостатність. В результаті чого виникла ця патологія?

Порушення вінцевого кровообігу

Перевантаження серця об’ємом

Патологія перикарда

Прямого пошкодження міокарда

Перевантаження серця опором

3179 / 6307
У пацієнта встановлено гіповітаміноз фолієвої кислоти, що може призвести до порушення синтезу:

Гема та креатину

Пуринових та тимідилових нуклеотидів

Тимідилових нуклеотидів та жирних кислот

Цитрату та кетонових тіл

Пуринових нуклеотидів та холестерину

3180 / 6307
Спадкова гіперліпопротеїнемія І типу обумовлена недостатністю ліпопротеїнліпази. Підвищення рівня яких транспортних форм ліпідів в плазмі навіть натщесерце є характерним?

Хіломікрони

Модифіковані ліпопротеїни

Ліпопротеїни високої густини

Ліпопротеїни дуже низької густини

Ліпопротеїни низької густини

3181 / 6307
У людини виявлена пухлина одного з відділів головного мозку, внаслідок чого в неї порушена здатність підтримувати нормальну температуру тіла. Яка структура головного мозку пошкоджена?

Гіпоталамус

Мозочок

Стріатум

Чорна субстанція

Таламус

3182 / 6307
До лікарні звернувся чоловік 50-ти років з розладами пам’яті, болісними відчуттями по ходу нервових стовбурів, зниженням інтелектуальних функцій, порушеннями з боку серцево-судинної системи і явищами диспепсії. В анамнезі хронічний алкоголізм. Дефіцит якого вітаміну може викликати ці симптоми?

Ретинол

Тіамін

Кальциферол

Ніацин

Рибофлавін

3183 / 6307
Пацієнт звернувся зі скаргами на гострий біль у правому підребер’ї. При огляді лікар звернув увагу на пожовтіння склер хворого. Лабораторно: підвищена активність АлАТ та негативна реакція на стеркобілін в калі. Для якого захворювання характерні такі симптоми?

Гепатит

Гемолітична жовтяниця

Хронічний гастродуоденіт

Хронічний гастрит

Хронічний коліт

3184 / 6307
У хворого після тривалого психоемоційного напруження спостерігається підвищення артеріального тиску, що супроводжується серцебиттям, кардіалгіями, головним болем, запамороченням. Домінуючим у формуванні артеріальної гіпертензії у даному випадку є збільшення:

Частоти серцевих скорочень

Об’єму циркулюючої крові

Тонусу артеріол

Серцевого викиду

Тонусу венул

3185 / 6307
Незалежно від расової чи етнічної належності у людини розвивається комплекс морфофункціональних, біохімічних, імунологічних ознак, які обумовлюють кращу біологічну пристосованість людини до відповідного фізичного середовища. Який тип біологічної реакції представлений у людини?

Тропічний тип

Гірський тип

Адаптивний тип

Арктичний тип

Тип зони помірного клімату

3186 / 6307
Для знеболювання використовують новокаїн, під дією якого нервове волокно втрачає здатність проводити збудження. Який мембранно-іонний механізм дії цього препарату?

Блокування калієвих іоноселективних каналів

Блокування натрій-протонного насоса

Блокування кальцієвих іоноселектив-них каналів

Блокування калій-натрієвого насоса

Блокування натрієвих іоноселективних каналів

3187 / 6307
У хворого 41-го року відзначається гіпонатріємія, гіперкаліємія, дегідратація, зниження артеріального тиску, м’язова слабкість, брадикардія, аритмія. З порушенням функцій яких гормонів це пов’язано?

Тиреоїдні

Кортикостероїди

Статеві гормони

Гормони мозкової речовини наднирників

Гормони підшлункової залози

3188 / 6307
При розтині хворої 28-ми років, що померла від уремії, виявлені збільшені строкаті нирки з осередками крововиливів. Патогістологічно в судинних клубочках виявлені гематоксилінові тільця, капілярні мембрани клубочків у вигляді дротяних петель, гіалінові тромби та осередки фібриноїдного некрозу. За патогенезом гіперчутливість якого типу лежить в основі описаної хвороби?

Гіперчутливість V типу (гранулематоз)

Гіперчутливість IV типу (клітинна цитотоксичність)

Гіперчутливість II типу (антитілозалежна)

Гіперчутливість III типу (імунокомплексна)

Гіперчутливість I типу (анафілактична)

3189 / 6307
Хвора 37-ми років померла під час нападу експіраторної задухи, що був спричинений контактом з екзогенним алергеном (пилок амброзії). При гістологічному дослідженні в просвіті бронхів спостерігаються скупчення слизу, в стінці бронхів багато тучних клітин (лаброцитів), більшість з яких у стані де-грануляції, багато еозинофілів. До патогенезуякого типу реакцій гіперчутливості можна віднести описані зміни?

III типу (імунокомплексна)

II типу (антитілозалежна)

I типу (анафілактична)

V типу (гранулематоз)

IV типу (клітинна цитотоксичність)

3190 / 6307
Оперуючи на наднирниковій залозі, хірург зупиняє кровотечу з артерії, що відходить до наднирникової залози від черевної аорти. Назвіть цю артерію:

A. suprarenalis media

A. uprarenalis inferior

A. suprarenalis superior

A. renalis

A. phrenica inferior

3191 / 6307
Хворому на паратиф А інфекціоніст на 3-му тижні захворювання призначив повторне бактеріологічне дослідження. Який матеріал слід взяти для виділення збудника?

Харкотиння

Випорожнення

Ліквор

Блювотні маси

Кров

3192 / 6307
Пацієнтка тривалий час приймала снодійний засіб нітразепам. Після відміни препарату у неї розвинулись безсоння, зниження апетиту, агресивність. Як називається такий стан?

Кумуляція

Ейфорія

Тахіфілаксія

Абстиненція

Сенсибілізація

3193 / 6307
Хворому на шизофренію призначено аміназин для купіювання психічного стану. Вкажіть механізм дії препарату:

Блокада ГАМК-рецепторів

Блокада Д2-дофамінових рецепторів

Стимуляція опіоїдних рецепторів

Стимуляція серотонінових рецепторів

3194 / 6307
Хворому на пневмонію призначений антибіотик із групи макролідів. Вкажіть цей препарат:

Тетрациклін

Гентаміцин

Стрептоміцин

Азитроміцин

Ампіцилін

3195 / 6307
В клініку доставлено чоловіка з травмою спини. Під час обстеження виявлено перелом хребців грудного відділу. Під час об’єктивного огляду нейрохірургом виявлено: нижче рівня перелому з правого боку відсутня глибока чутливість, з лівого боку – порушена температурна та тактильна чутливість. Яке ураження з боку спинного мозку є у хворого?

Судомний синдром

Анестезія

Синдром Броун Секара

Парастезія

Хвороба Паркінсона

3196 / 6307
Через 8 днів після хірургічної операції у пацієнта розвинувся правець. Лікар запідозрив, що причиною став контамінований збудником правця шовний матеріал, який був доставлений в бактеріологічну лабораторію. Яке живильне середовище необхідно використовувати для первинного посіву шовного матеріалу?

Сабуро

Ендо

Гіса

Кітт-Тароцці

ЖСА

3197 / 6307
Після проведеної операції на шиї хворий втратив чутливість в її передній ділянці. Яка гілка шийного сплетення була пошкоджена під час операції?

Малий потиличний нерв

Поперечний нерв шиї

Надключичні нерви

Шийна петля

Великий вушний нерв

3198 / 6307
У пацієнта перед кардіологічною операцією зареєстровано тиск у всіх відділах серця. Який тиск в лівому шлуночку під час діастоли?

100 мм рт.ст.

80 мм рт.ст.

120 мм рт.ст.

40 мм рт.ст.

0 мм рт.ст.

3199 / 6307
При обстеженні жінки виявили перелом однієї з кісток черепа, ускладнений кровотечею з поперечної пазухи твердої оболонки мозку. Назвіть кістку, на якій розташовується борозна цієї пазухи:

Клиноподібна

Тім’яна

Лобова

Потилична

Скронева

3200 / 6307
Батьки - глухонімі, але глухота у дружини залежить від аутосомно-рецесивного гена, а у чоловіка виникла внаслідок тривалого прийому антибіотиків у дитинстві. Яка імовірність народження глухої дитини в родині, якщо батько гомозиготний за аллелю нормального слуху?

25%

100%

10%

0%

75%

3201 / 6307
Тривале перебування в умовах спеки викликало у людини спрагу. Сигналізація від яких рецепторів, перш за все, зумовила її розвиток?

Осморецептори гіпоталамусу

Натрiєвi рецептори гіпоталамусу

Барорецептори дуги аорти

Глюкорецептори гіпоталамусу

Осморецептори печінки

3202 / 6307
Хворому з ревматоїдним артритом тривалий час вводили гідрокортизон. У нього з’явилися гіперглікемія, поліурія, глюкозурія, спрага. Ці ускладнення лікування є наслідком активації такого процесу:

Ліполіз

Гліколіз

Глікогеноліз

Глікогенез

Глюконеогенез

3203 / 6307
Для зняття марення і галюцинацій у хворої на шизофренію лікар використав аміназин. Який механізм антипсихотичної дії препарату?

Стимуляція холінергічних процесів в ЦНС

Блокада адренергічних і дофамінергічних процесів в ЦНС

Стимуляція адренергічних і дофамінергічних процесів в ЦНС

Інгібування зворотнього нейронального захоплення МАО

Блокада холінергічних процесів в ЦНС

3204 / 6307
При підозрі на туберкульоз хворій дитині зробили пробу Манту. Через 24 години у місці введення алергену з’явились припухлість, гіперемія і болісність. Які основні компоненти визначають цю реакцію організму?

Гранулоцити, Т-лімфоцити і IgG

В-лімфоцити, IgM

Мононуклеари, Т-лімфоцити і лімфокіни

Плазматичні клітини, Т-лімфоцити і лімфокіни

Макрофаги, В-лімфоцити і моноцити

3205 / 6307
Хворому перед операцією було введено дитилін (лістенон) і проведено інтубацію. Дефіцит якого ферменту в організмі хворого подовжує дію м’язового релаксанту?

Псевдохолінестераза

N-ацетилтрансфераза

Карбангідраза

K-Аа-АтФ-аза

Сукцинатдегідрогеназа

3206 / 6307
Хлопчик 12-ти років знаходиться у лікарні з підозрою на харчову токсикоінфекцію. При посіві фекалій хворого на середовище Ендо виросла велика кількість безбарвних колоній. Який мікроорганізм можна з найбільшою імовірністю ВИКЛЮЧИТИ з числа можливих збудників захворювання?

Salmonella enteritidis

Pseudomonas aeruginosa

Proteus vulgaris

Escherichia coli

Yersinia enterocolitica

3207 / 6307
У хворого внаслідок хронічного захворювання органів дихання, що супроводжується задишкою, тахікардією і ціанозом, при дослідженні газового складу крові виявлений розвиток гіпоксемії і гіперкапнії. Внаслідок якого з порушень зовнішнього дихання виникли ці зміни?

Гіпердифузія

Гіповентиляція

Гіпервентиляція

Гіперперфузія

Гіпоперфузія

3208 / 6307
У тварини через 2 тижні після експериментального звуження ниркової артерії підвищився артеріальний тиск. З посиленням дії на судини якого чинника гуморальної регуляції це пов’язане?

Вазопресин

Кортизол

Альдостерон

Ангіотензин II

Дофамін

3209 / 6307
У чоловіка 43-х років, що помер у хірургічному відділенні при явищах розлитого гнійного перитоніту, на розтині у дистальному відділі тонкого кишечника виявлені пейєрові бляшки, що виступають у просвіт, поверхня деяких з них вкрита струпом, коричнево-зеленуватого кольору, у центрі деяких бляшок наявні глибокі дефекти, що сягають серозного шару. Для якого захворювання характерні зміни, виявлені у тонкому кишечнику?

Стафілококовий ентерит

Туберкульоз кишечника

Дизентерія

Черевний тиф

Хвороба Крона

3210 / 6307
У хворого виявлено ожиріння, гірсутизм, «місяцеподібне» обличчя, рубці багряного кольору на шкірі стегон. Артеріальний тиск -180/110 мм рт. ст., глюкоза крові - 17,2ммоль/л. При якій зміні продукції гормонів наднирників можлива така картина?

Гіпопродукція адреналіну

Гіпопродукція глюкокортикоїдів

Гіпопродукція мінералокортикоїдів

Гіперпродукція мінералокортикоїдів

Гіперпродукція глюкокортикоїдів

3211 / 6307
У хворого, що тривало приймав глюкокортикоїди, в результаті відміни препарату виникло загострення наявного захворювання, зниження артеріального тиску, слабкість. Ці явища можна пов’язати з розвитком:

Звикання до препарату

Кумуляцією

Гіперпродукцією АКТГ

Недостатності наднирників

Сенсибілізацією

3212 / 6307
Тварині, сенсибілізованій туберкуліном, внутрішньоочеревинно введений туберкулін. Через 24 години при лапаротомії виявлено венозну гіперемію та набряк очеревини. Умазках-відбитках з очеревини велика кількість лімфоцитів та моноцитів. Яке запалення має місце у тварини?

Фібринозне

Серозне

Гнійне

Алергічне

Асептичне

3213 / 6307
У хворого спостерігаються біль голови , висока температура, озноб, кашель. З харкотиння виділили палички овоїдної форми з біполярним забарвленням, грам-негативні, у мазку з бульйонної культури розташовані ланцюжками, на агарі утворюються колонії R-форми. Це характерне для збудника такого захворювання:

Туберкульоз

Стрептококова ангіна

Менінгококовий назофарингіт

Дифтерія

Чума

3214 / 6307
До лікарні потрапили пацієнти зі скаргами: слабкість, болі в кишечнику, розлади травлення. Після дослідження фекалій були виявлені кулясті цисти з чотирма ядрами. Для якого найпростішого характерні такі цисти?

Амеба ротова

Балантидій

Амеба кишкова

Кишкова трихомонада

Амеба дизентерійна

3215 / 6307
У доношеного новонародженого спостерігається жовте забарвлення шкіри та слизових оболонок. Імовірною причиною цього стану може бути тимчасова нестача такого ферменту:

Гемсинтетаза

Уридинтрансфераза

УДФ-глюкуронілтрансфераза

Білівердинредуктаза

Гемоксигеназа

3216 / 6307
У немовляти спостерігаються епілептиформні судоми, викликані дефіцитом вітаміну В6. Це спричинено зменшенням у нервовій тканині гальмівного медіатора - гамма-аміномасляної кислоти. Активність якого ферменту знижена:

Аланінамінотрансфераза

Глутаматдекарбоксилаза

Піридоксалькіназа

Глутаматсинтетаза

Глутаматдегідрогеназа

3217 / 6307
У молодої особи впродовж року прогресувала ниркова недостатність зі смертельним наслідком. При розтині тіла виявлені великі строкаті нирки з червоними дрібними краплинами в жовто-сірому корковому шарі. Гістологічно в клубочках виявлені «півмісяці» з проліферуючого нефротелія. Який найбільш імовірний діагноз?

Гнійний нефрит

Швидкопрогресуючий гломерулонефрит

Амілоїдоз

Крововилив в нирки

Рак нирки

3218 / 6307
У хворого відсутній зір, але зіничний рефлекс реалізується нормально. Де може знаходитись зона пошкодження?

Зорова кора

Соматосенсорна кора

Нижні горбики чотиригорбкового тіла

Зоровий перехрест

Верхні горбики чотиригорбкового тіла

3219 / 6307
Хворий з 30-річним стажем роботи у шахті помер від легенево-серцевої недостатності, що наростала. На розтині легені збільшені у розмірах, щільної консистенції, на розрізі у них велика кількість вузликів розмірами з просяні зерна і більше, щільних, сіруватого і сірувато-чорного кольору, місцями вузлики зливаються у більш великі ділянки. Назвіть найбільш імовірне захворювання:

Антрако-силікоз

Бериліоз

Алюміноз

Азбестоз

Сидероз

3220 / 6307
Для серодіагностики черевного тифу ставлять реакцію, при якій до різних розведень сироватки хворого добавляють дiагностикуми трьох видів мікроорганізмів і результат якої оцінюють за наявності осаду із склеєних бактерій. Ця реакція відома під назвою:

Відаля

Райта

Асколі

Вассермана

Борде-Жангу

3221 / 6307
У хворого напад тахікардії. Які мембранні циторецептори кардіоміоцитів доцільно заблокувати, щоб припинити напад?

Бета-адренорецептори

М- та Н-холінорецептори

Альфа-адренорецептори

Н-холінорецептори

М-холінорецептори

3222 / 6307
У хворого на ЕКГ виявлено, що тривалість інтервалу RR дорівнює 1,5 с, частота серцевих скорочень - 40/хв. Що є водієм ритму серця?

Пучок Гіса

Права ножка Гіса

Синусовий вузол

Атріовентрикулярний вузол

Ліва ножка Гіса

3223 / 6307
У хворого за лабораторними даними визначили аскаридоз. Який засіб треба призначити?

Гентаміцин

Піперазин

Мебендазол

Фенасал

Фуразолідон

3224 / 6307
Хворий не може відвести від тулуба верхню кінцівку. Який м’яз НЕ ВИКОНУЄ свою функцію?

Великий коловий м’яз

Малий коловий м’яз

Підосний м’яз

Дельтоподібний м’яз

Найширший м’яз спини

3225 / 6307
Пункцію сечового міхура через передню черевну стінку, не займаючи очеревину, можливо виконати:

Тільки у жінок

Тільки у чоловіків

Тільки у дітей

При пустому сечовому міхурі

При наповненому сечовому міхурі

3226 / 6307
В судово-медичній експертизі широко використовується метод дактилоскопії, який оснований на тому, що сосочковий шар дерми визначає строго індивідуальний малюнок на поверхні шкіри. Яка тканина утворює цей шар дерми?

Пухка волокниста неоформлена сполучна тканина

Жирова тканина

Ретикулярна тканина

Щільна неоформлена сполучна тканина

Щільна оформлена сполучна тканина

3227 / 6307
Важливою складовою частиною ниркового фільтраційного бар’єру є тришарова базальна мембрана, яка має спеціальну сітчасту будову її середнього електроннощільного шару. Де міститься ця базальна мембрана?

Ниркове тільце

Проксимальні канальці

Тонкі канальці

Капіляри перитубулярної капілярної сітки

Дистальні прямі канальці

3228 / 6307
Чоловік 42-х років помер при явищах вираженої інтоксикації і дихальної недостатності. На розтині: тканина легень у всіх відділах строката, з множинними дрібновогнищевими крововиливами та вогнищами емфіземи. Гістологічно у легенях: геморагічна бронхопневмонія з абсцедуванням, у цитоплазмі клітин епітелію бронхів еозинофільні і базофільні включення. Діагностуйте виявлене на секції захворювання:

Стафілококова бронхопневмонія

Крупозна пневмонія

Плевропневмонія

Часткова пневмонія

Грип

3229 / 6307
В експерименті подразнюють скелетний м’яз серією електричних імпульсів. Який вид м’язового скорочення виникне, якщо кожний наступний імпульс припадає на період вкорочення поодинокого м’язового скорочення?

Контрактура м’яза

Серія поодиноких скорочень

Зубчастий тетанус

Суцільний тетанус

Асинхронний тетанус

3230 / 6307
У хворого, який тривалий час страждав на переміжну кульгавість, тканини пальців стопи сухі, чорного кольору, нагадують мумію. На невеликій відстані від почорнілої ділянки розташована двоколірна лінія (червоний колір прилягає до практично незмінених тканин, а біло-жовтий колір - до змінених тканин). Який вид некрозу у даного хворого?

Пролежень

!нфаркт

Мацерація

Гангрена

Секвестр

3231 / 6307
У пацієнта, що звернувся до лікаря, спостерігається жовте забарвлення шкіри, сеча - темна, кал темно-жовтого кольору. Підвищення концентрації якої речовини буде спостерігатися в сироватці крові?

Вердоглобін

Кон’югований білірубін

Вільний білірубін

Мезобілірубін

Білівердин

3232 / 6307
До дерматолога звернулася пацієнтка із скаргами на екзематозне ураження шкіри рук, що з’являється після контакту з миючим засобом «Лотос» Використання гумових рукавичок запобігає цьому. Патологічна реакція шкіри зумовлена активацією:

Базофілів

Нейтрофілів

Моноцитів

Т-лімфоцитів

В -лімфоцитів

3233 / 6307
У хворого на тимому (пухлину вилочкової залози) спостерігається ціаноз, розширення підшкірної венозної сітки і набряк м’яких тканин обличчя, шиї, верхньої половини тулуба і верхніх кінцівок. Який венозний стовбур перетиснено пухлиною?

Внутрішня яремна вена

Зовнішня яремна вена

Підключична вена

Верхня порожниста вена

Передня яремна вена

3234 / 6307
В результаті травми у чоловіка 47-ми років пошкоджені передні корінці спинного мозку. Відростки яких нейронів пошкоджені?

Дендрити чутливих псевдоуніполярних

Дендрити рухових і аксони ядер бокових стовпів

Дендрити і аксони чутливих псевдоуні-полярних

Аксони чутливих псевдоуніполярних

Аксони нейронів рухових соматичних та вегетативних ядер

3235 / 6307
Під час розтину померлого 43-х років, що страждав на ІХС з розвитком інфаркту міокарда, патологоанатом виявив набряк легень. Які патологічні зміни могли зумовити набряк легень?

Ішемія малого кола

Гостре загальне малокрів’я

Стаз крові

Гостра правошлуночкова недостатність

Гостра лівошлуночкова недостатність

3236 / 6307
У чоловіка 40-ка років ушкоджені міжпальцеві проміжки на ногах: шкіра мокне, відшаровується, з’явилися тріщини. При посіві зі шкрібу шкіри на середовище Сабуро виросли пухнасті колонії, білі зверху та зеленувато-жовті внизу. У мазках з верхньої частини колоній видно конідії у вигляді «дубинок» з 1-5 клітинами. Які ще органи найбільш імовірно може уразити цей збудник?

Підшкірна клітковина

Волосся

Слизова статевих шляхів

Лімфатичні судини

Нігті

3237 / 6307
При ендоскопічному дослідженні у хворого з хронічним ентероколітом (запалення кишки) спостерігається відсутність специфічних структур рельєфа тонкої кишки. Які компоненти визначають особливості рельєфа слизової оболонки цього органу?

Поля, складки, ямки

Поля, ворсинки

Циркулярні складки, ворсинки та крипти

Косо-спіральні складки

Гаустри, ворсинки, крипти

3238 / 6307
Під час операції на головному мозку відмічено, що подразнення певних зон кори великих півкуль викликало у хворого і тактильні і температурні відчуття. На яку звивину діяли подразники?

Верхня латеральна

Поясна

Постцентральна

Парагіпокампова

Прецентральна

3239 / 6307
Хворому проведено субтотальну субфасціальну резекцію щитоподібної залози. У післяопераційному періоді тривалий час зберігається охриплість голосу. Який нерв ушкоджено в ході операції?

Нижньощелепний нерв

Зворотній гортанний нерв

Під’язиковий нерв

Язиковий нерв

Верхній гортанний нерв

3240 / 6307
В результат виснажуючої м’язової роботи у робочого значно зменшилась буферна ємність крові. Надходженням якої речовини у кров можна пояснити це явище?

1,3-бісфосфогліцерат

Лактат

3-фосфоглицерат

Піруват

Альфа-кетоглутарат

3241 / 6307
В біоптаті бронха хворого, який зловживає палінням, в потовщеній слизовій оболонці виявлено хронічний обструктивний бронхіт з ознаками трансформації одношарового війчастого епітелія в багатошаровий плоский епітелій. Який з процесів найбільш імовірний?

Гіпертрофія епітелію

Гіперплазія епітелію

Метаплазія

Плоскоклітинний рак

Лейкоплакія

3242 / 6307
У хворого 45-ти років при аналізі ЕКГ встановлено: ритм синусовий, число передсердних комплексів більше числа шлуночкових комплексів; прогресуюче подовження інтервалу P-Q від комплексу до комплексу; випадіння окремих шлуночкових комплексів; зубці P та комплекси QRST без змін. Назвіть тип порушення серцевого ритму:

Атріовентрикулярна блокада I ступеня

Атріовентрикулярна блокада II ступеня

Синоаурікулярна блокада

Повна атріовентрикулярна блокада

Внутрішньопередсердна блокада

3243 / 6307
Хворий з діагнозом цукровий діабет, вранці натще отримав призначену дозу інсуліну пролонгованої дії. Пропустив черговий прийом їжі і невдовзі відчув слабкість, біль голови ,запаморочення, пітливість, тремтіння тіла, судоми, відчуття голоду, явища гіпоглікемії. Застосування глюкози стан не полегшило. Який препарат необхідно ввести для купірування даного стану?

Пренізолон

Норадреналін

Гідрокортизон

Адреналін

Тріамцінолон

3244 / 6307
У чоловіка 25-ти років з переломом основи черепа виділяється великий об’єм сечі з низькою відносною щільністю. Причиною змін сечоутворення є порушення синтезу такого гормону:

Вазопресин

Адренокортикотропний гормон

Окситоцин

Тиреотропний гормон

Соматотропний гормон

3245 / 6307
Розпочинається імплантація бластоцисти людини. Як називається період ембріогенезу, що розпочинається одночасно з імплантацією?

Дроблення

Гаструляція

Диференціювання

Інвагінація

Гістогенез

3246 / 6307
Дитина 8-ми років готувалася до тонзилектомії. Аналіз крові показав, що час згортання збільшено (до 7 хвилин). Який препарат за 5 днів до операції слід включити до комплексу лікарських засобів підготовчого періоду в першу чергу?

Вікасол

Дицинон

Амінокапронова кислота

Кальцію хлорид

Фібриноген

3247 / 6307
У дитини, що страждає на пілоростеноз, що супроводжується частим блюванням, розвинулись ознаки зневоднення організму. Яка форма порушення кислотно-основного стану може розвиватися у даному випадку?

Метаболічний ацидоз

Газовий алкалоз

Негазовий алкалоз

Газовий ацидоз

Негазовий ацидоз

3248 / 6307
У хворого після автомобільної травми артеріальний тиск - 70/40 мм рт. ст. Хворий у непритомному стані. За добу виділяє близько 550 мл сечі. Яке порушення функції нирок спостерігається у хворого?

Гострий дифузний гломерулонефрит

Хронічна ниркова недостатність

Гостра ниркова недостатність

Тубулопатія

Пієлонефрит

3249 / 6307
У гістопрепараті яєчника жінки визначаються структури, що мають велику порожнину. Овоцит І порядку в них оточений прозорою оболонкою, променистим вінцем і розташований у яйценосному горбику, стінка утворена шаром фолікулярних клітин i текою. Вкажіть, якій структурі яєчника належать дані морфологічні ознаки:

Зрілий (третинний) фолікул

Первинний фолікул

Примордіальний фолікул

Жовте тіло

Атретичне тіло

3250 / 6307
Хворий переніс повторний інтрамуральний інфаркт міокарда. Після лікування та реабілітації виписаний у задовільному стані під нагляд дільничного терапевта. Через 2роки загинув у автомобільній катастрофі. Який характер патологічного процесу в міокарді було встановлено на розтині?

Атрофія

Некроз

Дрібновогнищевий кардіосклероз

Гіперплазія

Крупновогнищевий кардіосклероз

3251 / 6307
При бактеріологічному дослідженні гною з післяопераційної рани виділені мікроорганізми, які дали ріст на цукрово-кров’яному агарі через 7-10 днів в анаеробних умовах: колонії S-форми, блискучі, чорного кольору з неприємним запахом. При мікроскопії виявлені поліморфні грамнегативні палички. Які мікроорганізми могли викликати цей нагнійний процес?

Кишкова паличка

Клостридії

Фузобактерії

Бактероїди

Велонели

3252 / 6307
У 12-річного хлопчика в сечі виявлено високий вміст усіх амінокислот аліфатичного ряду. При цьому відмічена найбільш висока екскреція цистину та цистеїну. Крім того, УЗД нирок показало наявність каменів у них. Виберіть можливу патологію:

Фенілкетонурія

Цистит

Алкаптонурія

Хвороба Хартнупа

Цистинурія

3253 / 6307
Цикл Кребса відіграє важливу роль у реалізації глюкопластичного ефекту амінокислот. Це зумовлено обов’язковим перетворенням безазотистого залишку амінокислот у:

Сукцинат

Малат

Фумарат

Оксалоацетат

Цитрат

3254 / 6307
Після прийому препарату у хворого з серцевою недостатністю зменшилася частота скорочень серця, пульс став кращого наповнення, зменшилися набряки, збільшився діурез. Вкажіть, який препарат приймав хворий:

Резерпін

Анаприлін

Дигоксин

Верапаміл

Дилтіазем

3255 / 6307
У пацієнта 65-ти років з тривалими скаргами, характерними для хронічного гастриту, у периферичній крові виявлені мегалоцити, у кістковому мозку мегалобластичний еритропоез. Який найбільш імовірний діагноз?

Залізодефіцитна анемія

B12-фолієводефіцитна анемія

Апластична анемія

Гемолітична анемія

Гіпопластична анемія

3256 / 6307
З калу та блювотних мас від хворого з підозрою на холеру були виділені культури вібріонів. Проведення якої реакції дозволить визначити вид мікроба, що викликав це захворювання?

Аглютинації Відаля

Аглютинації з сироватками, що містять О-антитіла

Преципітації

Аглютинації з сироватками, що містять Н-антитіла

Пасивної гемаглютинації з еритроци-тарним антигенним діагностикумом

3257 / 6307
У дитини на слизовій оболонці щік та язика виявлені білуваті плями, які нагадують молоко, що скипілося. У виготовлених препаратах-мазках знайдені грампозитивні овальні дріжджоподібні клітини. Які це збудники?

Актиноміцети

Фузобактерії

Дифтерійна паличка

Гриби роду Кандіда

Стафілококи

3258 / 6307
При обстеженні юнака з розумовою відсталістю виявлено євнухоїдну будову тіла, недорозвиненість статевих органів. В клітинах порожнини рота - статевий хроматин. Який метод генетичного дослідження слід застосувати для уточнення діагнозу?

Популяційно-статистичний

Дерматогліфіка

Клініко-генеалогічний

Біохімічний

Цитологічний

3259 / 6307
У хворого з жовтяницею встановлено: підвищення у плазмі крові вмісту загального білірубіну за рахунок непрямого (вільного), в калі та сечі - високий вміст стеркобіліну, рівень прямого (зв’язаного) білірубіну в плазмі крові в межах норми. Про який вид жовтяниці можна думати?

Механічна

Гемолітична

Жовтяниця немовлят

Хвороба Жильбера

Паренхіматозна (печінкова)

3260 / 6307
Кухар в результаті необачності обпік руку парою. Підвищення концентрації якої речовини викликало почервоніння, набряклість та болючість ураженої ділянки шкіри?

Галактозамін

Лізин

Тіамін

Глутамін

Гістамін

3261 / 6307
Дитина доставлена в санпропускник в стані асфіксії. При огляді в гортані виявлені білуваті плівки, що обтурують просвіт та легко видаляються. Лікар запідозрив дифтерію. Про яке запалення гортані йдеться?

Крупозне

Дифтеритичне

Катаральне

Гнійне

Серозне

3262 / 6307
У хворого із запаленням легень спостерігається підвищення температури тіла. Яка біологічно активна речовина відіграє провідну роль у виникненні цього прояву хвороби?

Інтерлейкін-I

Гістамін

Лейкотрієни

Серотонін

Брадикінін

3263 / 6307
Хворий 12-ти років поступив в клініку з гемартрозом колінного суглоба, з раннього дитинства страждає кровоточивістю. Яка хвороба у хлопчика?

Геморагічний васкуліт

Тромбоцитопенічна пурпура

B12 фолієво-дефіцитна анемія

Гемолітична анемія

Гемофілія

3264 / 6307
У хворого видалено 12-палу кишку. Це призведе до зменшення секреції, перш за все, такого гормону:

Соматостатин

Гастрин

Холецистокінін-секретин

Нейротензин

Гістамін

3265 / 6307
В клітині, яка мітотично ділиться, спостерігається розходження дочірніх хроматид до полюсів клітини. На якій стадії мітотичного циклу знаходиться клітина?

Профаза

Анафаза

Телофаза

Метафаза

Інтерфаза

3266 / 6307
Знешкодження ксенобіотиків (лікарських засобів, епоксидів, ареноксидів, альдегідів, нітропохідних тощо) та ендогенних метаболітів (естрадіолу, простагландинів, лейкотрієнів) відбувається в печінці шляхом їх кон’югації з:

Гліцином

S-Аденозилметіоніном

Аспарагіновою кислотою

Фосфоаденозином

Глутатіоном

3267 / 6307
Вітамін A у комплексі зі специфічними циторецепторами проникає через ядерні мембрани, індукує процеси транскрипції, що стимулює ріст та диференціювання клітин. Ця біологічна функція реалізується наступною формою вітаміну A:

Транс -ретиналь

Цис-ретиналь

Каротин

Ретинол

Транс-ретиноєва кислота

3268 / 6307
Одна з форм вродженої патології супроводжується гальмуванням перетворення фенілаланіну в тирозин. Біохімічною ознакою хвороби є накопичення в організмі деяких органічних кислот, у тому числі такої кислоти:

Піровиноградна

Ілутамінова

Молочна

Фенілпіровиноградна

Лимонна

3269 / 6307
Хворому встановлено діагноз - активний вогнищевий туберкульоз легень. Вкажіть, який із препаратів йому найбільш доцільно призначити в першу чергу

Сульфален

Ізоніазид

Етоксид

Циклосерин

Етіонамід

3270 / 6307
В експерименті на спинному мозку при збудженні альфа-мотонейронів згиначів встановлено гальмування альфа-мотонейронів м’язів-розгиначів. Який вид гальмування лежить в основі цього явища?

Зворотнє

Реципрокне

Латеральне

Пресинаптичне

Деполяризаційне

3271 / 6307
У хворого, що страждає на важку форму порушення водно-сольового обміну, настала зупинка серця у діастолі. Який найбільш імовірний механізм зупинки серця у діастолі?

Гіпонатріємія

Гіпокаліємія

Дегідратація організма

Гіпернатріємія

Гіперкаліємія

3272 / 6307
Встановлено ураження ВІЛ Т-лімфоцитів. При цьому фермент вірусу зворотня транскриптаза (РНК-залежна ДНК-полімераза) каталізує синтез:

і-РНК на матриці вірусного білку

ДНК на вірусній р-РНК

Вірусної і-РНК на матриці ДНК

Вірусної РНК на матриці ДНК

ДНК на матриці вірусної і-РНК

3273 / 6307
Людина зробила спокійний видих. Як називається об’єм повітря, який міститься у неї в легенях при цьому?

Дихальний об’єм

Резервний об’єм видиху

Залишковий об’єм

Життєва ємність легень

Функціональна залишкова ємкість легень

3274 / 6307
Людина зробила максимально глибокий видих. Як називається об’єм повітря, що знаходиться в її легенях після цього?

Ємність вдиху

Резервний об’єм видиху

Функціональна залишкова ємність легень

Залишковий об’єм

Альвеолярний об’єм

3275 / 6307
У людини необхідно оцінити стан клапанів серця. Яким з інструментальних методів дослідження доцільно скористатися для цього?

Сфігмографія

Зондування судин

Фонокардіографія

Електрокардіографія

Флебографія

3276 / 6307
Чоловіку 40-ка років за вимогою діагностичних тестів зробили лімфографію органів грудної порожнини. Хірург встановив, що пухлина вразила орган, з лімфатичних судин якого лімфа безпосередньо переходить в грудну протоку. Який це орган?

Лівий головний бронх

Серце

Трахея

Стравохід

Осердя

3277 / 6307
При гістологічному дослідженні біоптату шкіри виявлені гранульоми, які складаються з макрофагальних вузликів з наявністю лімфоцитів та плазматичних клітин. Крім того, зустрічаються великі макрофаги з жировими вакуолями, які містять запакованих у вигляді куль збудників захворювання (клітини Вірхова). Грануляційна тканина добре васкуляризована. Для якого захворювання характерна описана гранульома?

Сап

Риносклерома

Туберкульоз

Лепра

Сифіліс

3278 / 6307
При мікроскопічному дослідженні біоптату шкіри виявляються гранульоми, які складаються з епітеліоїдних клітин, оточених в основному Т-лімфоцитами. Серед епітеліоїдних клітин розташовуються поодинокі гігантські багатоядерні клітини типу Пирогова-Лангханса. В центрі гранульом виявляються ділянки казеозного некрозу. Кровоносні судини відсутні. Для якого захворювання характерні описані гранульоми?

Лепра

Сап

Туберкульоз

Риносклерома

Сифіліс

3279 / 6307
Ліквідатору наслідків аварії на Чорнобильські АЕС, що отримав велику дозу опромінення, проведено трансплантацію кісткового мозку. Через деякий час після проведеної операції у пацієнта діагностовано розвиток реакції трансплантат проти хазяїна. Які антигени послужили пусковим механізмом виникнення цієї реакції?

Антигени системи Rh еритроцитів ліквідатора

Антигени системи HLA клітин організму донора

Антигени HBs, HBc, Hbe

Антигенами системи ABO еритроцитів ліквідатора

Антигени системи HLA клітин організму ліквідатора

3280 / 6307
Під час гістологічного дослідження стулок мітрального клапана серця жінки 30-ти років було встановлено, що ендотеліальні клітини вогнищево десквамовані, в цих ділянках на поверхні стулки розташовані дрібні тромботичні нашарування, сполучна тканина стулки з явищами мукоїдного набухання з ділянками склерозу та васкуляризації. Діагностуйте вид клапанного ендокардиту:

Поворотньо-бородавчастий

Дифузний

Поліпозно-виразковий

Гострий бородавчастий

Фібропластичний

3281 / 6307
При органічних пошкодженнях головного мозку пам’ять може покращити:

Діазепам

Кофеїн

Мезапам

Пірацетам

Нітразепам

3282 / 6307
У хворої під час травми стався розрив лобкового симфізу. Який тип з’єднання постраждав?

Синхондроз

Діартроз

Іеміартроз

Синостоз

Синдесмоз

3283 / 6307
У хворого діагностовано інфаркт задньої частини міжшлуночкової перегородки. В ділянці якої кровоносної судини виникло порушення кровообігу?

R. interventricularis posterior

R. atrialis intermedius

R. marginalis dexter

R. circumflexus

R. marginalis sinister

3284 / 6307
У дитячому відділенні інфекційної клініки хлопчику поставлено діагноз «дифтерія» Який препарат потрібно ввести хворому в першу чергу?

Протидифтерійна антитоксична сироватка

Дифтерійний анатоксин

АКДП

АДП

TABte

3285 / 6307
У студента перед екзаменом виникла тахікардія. Які зміни на ЕКГ будуть свідчити про її наявність?

Подовження інтервалу R-R

Подовження сегменту Q-T

Розширення комплексу QRS

Укорочення інтервалу R-R

Подовження інтервалу P-Q

3286 / 6307
При розтині тіла чоловіка, померлого від опікової хвороби, знайдено набряк головного мозку, збільшення печінки, а також нирок, кірковий шар яких широкий, блідо-сірий, мозковий - повнокровний. Мікроскопічно: некроз епітелію канальців головних відділів з деструкцією базальних мембран, набряк інтерстицію з лейкоцитарною інфільтрацією та крововиливами. Який з перелічених діагнозів найбільш імовірний?

Мієломна нирка

Некротичний нефроз

Пієлонефрит

Тубулоінтерстиціальний нефрит

Подагрична нирка

3287 / 6307
У хворої 59-ти років з нейроциркуляторною (первинною) артеріальною гіпотензією купіровано гіпотонічний криз підшкірним введенням 1 мл 20% розчину кофеїну бензоатунатрію. Механізм дії цього препарату полягає у тому, що він блокує:

Моноаміноксідазу

Альфа-адренорецептори

Пуринові рецептори

Бета-адренорецептори

Аденозинові рецептори

3288 / 6307
Хворий похилого віку страждає на хронічний закреп, в основі якого лежить гіпотонія товстого кишечнику. Який препарат слід призначити хворому?

Натрію сульфат

Ацеклідин

Бісакодил

Касторова олія

Прозерин

3289 / 6307
У жінки через 6 місяців після пологів розвинулася маткова кровотеча. При гінекологічному огляді у порожнині матки виявлена тканина темно-червоного кольору змножинними порожнинами, що нагадує «губку» При мікроскопічному дослідженні пухлини у лакунах крові виявлені атипові світлі епітеліальні клітини Лангханса і гігантські клітини синцитіотрофобласту. Назвіть пухлину:

Плоскоклітинний незроговілий рак

Аденокарцінома

Фіброміома

Хоріонепітеліома

Міхурний занесок

3290 / 6307
У хворого спостерігається гемералопія (куряча сліпота). Яка з перерахованих речовин володітиме лікувальною дією?

Креатин

Кератин

Каротин

Карнозин

Карнітин

3291 / 6307
Хворий 18-ти років звернувся до лікарні із скаргами на шум та больові відчуття у вусі. Об’єктивно: у хворого гостре респіраторне захворювання, риніт. Крізь який отвір глотки інфекція потрапила до барабанної порожнини та викликала її запалення?

Барабанний отвір слухової труби

Хоани

Вхід до гортані

Зів

Глотковий отвір слухової труби

3292 / 6307
Хворий 50-ти років страждає на гіпертонічну хворобу. Під час фізичного навантаження у нього з’явилося відчуття слабкості, нестачі повітря, синюшність слизової оболонки губ, шкіри обличчя. Дихання супроводжувалося відчутними на відстані вологими хрипами. Який механізм лежить в основі виникнення такого синдрому?

Хронічна лівошлуночкова недостатність

Гостра лівошлуночкова недостатність

Тампонада серця

Хронічна правошлуночкова недостатність

Колапс

3293 / 6307
Внаслідок тривалого голодування в організмі людини швидко зникають резерви вуглеводів. Який з процесів метаболізму за цих умов поновлює вміст глюкози в крові?

Аеробний гліколіз

Глікогеноліз

Анаеробний гліколіз

Глюконеогенез

Пентозофосфатний шлях

3294 / 6307
У чоловіка 40-ка років було встановлено діагноз: серпоподібноклітинна анемія. Який механізм приводить до зменшення кількості еритроцитів в крові у цього хворого?

Нестача заліза в організмі

Позасудинний гемоліз

Нестача вітаміну B12 і фолієвої кислоти

Внутрішньосудинний гемоліз

Нестача білка

3295 / 6307
У жінки 52-х років при обстеженні було виявлено зниження кількості еритроцитів у крові та підвищення рівня вільного гемоглобіну в плазмі крові (гемоглобінемія). КП- 0,85. Який вид анемії спостерігається у хворої?

Спадкова гемолітична

Анемія внаслідок порушення еритропоезу

Хронічна постгеморагічна

Іостра постгеморагічна

Набута гемолітична

3296 / 6307
Лікування туберкульозу здійснюється за допомогою комбінованої хіміотерапії, що поєднує речовини різного механізму дії. Який з протитуберкульозних засобів пригнічує транскрипцію ДНК в РНК мікобактерій?

Рифампіцин

Ізоніазид

Етіонамід

ПАСК

Стрептоміцин

3297 / 6307
На тканину діють електричним імпульсом катодного напрямку, амплітуда якого дорівнює 70% порогу. Які зміни мембранного потенціалу клітин це викличе?

Потенціал дії

Часткова деполяризація

Гіперполяризація

Змін не буде

3298 / 6307
У людини визначили величину енерговитрат. У якому стані знаходилась людина, якщо її енерговитрати виявилися меншими за основний обмін?

Нервове напруження

Сон

Спокій

Легка робота

Відпочинок

3299 / 6307
При визначенні основного обміну з’ясовано, що його величина у досліджуваного менша за належну величину на 7%. Це означає, що інтенсивність процесів енергетичного метаболізму у досліджуваного:

Помірно знижена

Помірно підвищена

Нормальна

Суттєво підвищена

Суттєво знижена

3300 / 6307
Внаслідок руйнування певних структур стовбуру мозку тварина втратила орієнтувальні рефлекси. Які структури було зруйновано?

Чотиригорбкові тіла

Червоні ядра

Вестибулярні ядра

Чорна речовина

Медіальні ядра ретикулярної формації

3301 / 6307
У новонародженого малюка педіатр виявив, що отвір крайньої плоті за величиною не перевищує діаметр сечовивідного каналу і голівка статевого члена не може виходити через такий отвір. Як називається цей стан?

Епіспадія

Гіпоспадія

Парафімоз

Гермафродитизм

Фімоз

3302 / 6307
Психологічне дослідження встановило: у людини добра здатність швидко пристосовуватися до нового оточення, добра пам’ять, емоційна стійкість, висока працездатність. Найімовірніше, ця людина:

Холерик

Флегматик

Сангвінік

Меланхолік

Флегматик з елементами меланхоліка

3303 / 6307
Хворий на ішемічну хворобу серця не повідомив лікаря, що в нього бувають напади бронхоспазму. Лікар призначив препарат, після прийому якого напади стенокардії стали рідше, але напади бронхоспазму стали частіше. Який препарат був призначений?

Дилтіазем

Нітрогліцерин

Анаприлін

Верапаміл

Нітросорбід

3304 / 6307
До ендокринолога звернулась хвора 45-ти років із скаргами на підвищення апетиту, сухість слизових оболонок ротової порожнини, збільшення діурезу. При обстеженні вперше виявлено інсулін незалежний діабет. Який з названих препаратів доцільно призначити хворій?

Окситоцин

Інсулін

Адіурекрин

Вазопресин

Глібенкламід

3305 / 6307
Дитина 7-ми років поступила в інфекційне відділення зі скаргами на різкий біль у горлі, утруднення під час ковтання, підвищення температури тіла до 39oC, набряк шиї. Об’єктивно: мигдалики збільшені, їх слизова оболонка повнокровна, вкрита великою кількістю плівок білувато-жовтого кольору, які щільно прилягають до слизової оболонки. При спробі зняти плівку залишається глибокий дефект, який кровоточить. Який вид запалення має місце?

Гнійне

Крупозне

Серозне

Геморагічне

Дифтеритичне

3306 / 6307
У хворого на хронічний дифузний гломерулонефрит розвинулася хронічна недостатність нирок. В термінальній стадії ХНН розвивається оліго- та анурія, що спричиняється:

Зниженням маси діючих нефронів

Зменшенням фільтраційного тиску та фільтрації

Збільшенням реабсорбції води в дистальних канальцях

Дисемінованим внутрішньосудинним зсіданням крові

Ішемією коркової речовини нирок внаслідок спазму судин

3307 / 6307
При розтині тіла померлого чоловіка 48-ми років в ділянці 1-го сегменту правої легені виявлено круглий утвір діаметром 5 см з чіткими контурами, оточений тонким прошарком сполучної тканини, виповнений білими крихкими масами. Діагностуйте форму вторинного туберкульозу:

Туберкулома

Гострий вогнищевий туберкульоз

Фіброзно-кавернозний туберкульоз

Казеозна пневмонія

Гострий кавернозний туберкульоз

3308 / 6307
На препарат представлено орган, вкритий сполучнотканинною капсулою, від якої відходять трабекули. В органі можна розрізнити кіркову речовину, де містяться лiмфатичнi вузлики та мозкову речовину, представлену тяжами лімфоїдних клітин. Який орган представлений на препараті?

Тимус

Селезінка

Мигдалики

Червоний кістковий мозок

Лімфатичний вузол

3309 / 6307
В регуляції фізіологічних функцій беруть участь іони металів. Один із них отримав назву «король месенджерів» Таким біоелементом посередником є:

Zn++

Fe+++

K+

Na+

Са++

3310 / 6307
Хворий звернувся до лікаря зі скаргами на болі в кульшовому суглобі та у всіх привідних м’язах і шкірі над ними при рухах. Який нерв ушкоджено?

Латеральний шкірний нерв стегна

Стегновий нерв

Клубово-пахвинний нерв

Затульний нерв

Задній шкірний нерв стегна

3311 / 6307
Хворому 35-ти років для обстеження очного дна був призначений атропіну сульфат у вигляді очних крапель. Для відновлення акомодації йому закрапали пілокарпіну гідрохлорид, але це не дало бажаного ефекту. Що є причиною відсутності ефекту?

Звикання

Двосторонній антагонізм

Односторонній антагонізм

Синергізм

Тахіфілаксія

3312 / 6307
В поліклініку до лікаря звернулася жінка 32-х років зі скаргами на відсутність в неї лактації після народження дитини. Дефіцитом якого гормону, найбільш імовірно, можна пояснити дане порушення?

Тиреокальцитонін

Пролактин

Глюкагон

Соматотропін

Вазопресин

3313 / 6307
У хворого пухлина черевної порожнини, що стискає нижню порожнисту вену. Який кава- кавальний анастомоз на передній стінці живота забезпечить відтік венозної крові?

Між верхніми і нижніми прямокишковими венами

Між верхньою і нижньою надчеревними венами

Між пупковою і верхньою надчеревною венами

Між непарною і напівнепарною венами

Між хребетними і пупковими венами

3314 / 6307
У людини під дією мутагенного фактору з’явилась велика кількість мутантних клітин. Але більшість з них були розпізнані і знищені клітинами:

Стовбуровими

В -лімфоцитами

Т-лімфоцитами супресорами

Т-лімфоцитами кілерами

Плазмобластами

3315 / 6307
При пошкодженні клітини іонізуючим випромінюванням вмикаються механізми захисту і адаптації. Який механізм відновлення порушеного внутрішньоклітинного гомеостазу реалізується при цьому?

Пригнічення аденілатциклази

Гіпертрофія мітохондрій

Накопичення Na+ в клітинах

Активація Са-опосередкованих клітинних функцій

Активація антиоксидантної системи

3316 / 6307
На гістологічному препараті легень видно структуру діаметром близько 0,5 мм, слизова оболонка якої вкрита одношаровим кубічним війковим епітелієм, у якому зустрічаються секреторні клітини Клара, війчасті клітини, мікроворсинчасті. Вкажіть структуру:

Малий бронх

Термінальна бронхіола

Альвеола

Альвеолярний хід

Середній бронх

3317 / 6307
Чоловік 36-ти років, лісник за фахом, через тиждень після тривалого перебування у весняному лісі гостро захворів -гарячка, головний біль, порушення свідомості, епілептиформні напади, смерть розвинулася на 3-й день хвороби. На роз- тині тіла: набряк головного мозку, множинні точкові геморагії; під час мікроскопічного дослідження - периваскулярний та перицелюлярний набряк, множинні периваскулярні,переважно лімфоцитарні, інфільтрати. Діагностуйте основне захворювання:

Церебро-васкулярна хвороба

Менінгококова інфекція

Кліщовий енцефаліт

Поліомієліт

Гнійний енцефаліт

3318 / 6307
У хворого з флегмоною передпліччя при мікробіологічному аналізі ексудату в зоні запалення визначена присутність стрептококів. Які клітини будуть переважати в ексудаті?

Еозинофільні гранулоцити

Лімфоцити

Базофільні гранулоцити

Моноцити

Нейтрофільні гранулоцити

3319 / 6307
У п’ятимісячної дівчинки виявлено застійні явища у легенях. При обстеженні виявлено зв’язок між висхідною аортою та легеневою артерією, що в нормі спостерігається у деяких земноводних і плазунів. Назвіть цю природжену ваду розвитку:

Розвиток правої дуги аорти

Незрощення боталової протоки

Дефект міжпередсердної перегородки

Транспозиція магістральних судин

Дефект міжшлуночкової перегородки

3320 / 6307
Хворому на гострий інфаркт міокарда у комплексній терапії було призначено гепарин. Через деякий час після введення даного препарату з’явилася гематурія. Який антагоніст гепарину необхідно ввести хворому для усунення даного ускладнення?

Амінокапронова кислота

Вікасол

Неодикумарин

Протаміну сульфат

Фібриноген

3321 / 6307
До лікарні звернулася жінка з дитиною, у якої на голові була гангренозна рана. Лікар при огляді виявив у рані білих червоподібних личинок комах. Яка комаха могла їх відкласти?

Вольфартова муха

Муха-жигалка

Комарі

Москіти

Блохи

3322 / 6307
На дослідження в бактеріологічну лабораторію було відправлено випорожнення хворої дитини грудного віку, з яких виділена культура ентеропатогенних кишкових паличок О55К59. На основі яких критеріїв виділена культура віднесена до ЕПКП О55?

Культуральні ознаки

Морфологічні ознаки

Біохімічні властивості

Антигенні властивості

Визначення фаговару

3323 / 6307
У чоловіка 30-ти років перед операцією визначили групову належність крові. Кров резус- позитивна. Реакцію аглютинації еритроцитів не викликали стандартні сироватки груп 0а/3 (I), А/3 (II), Ва (III). Досліджувана кров належить до групи:

Ар (II)

АВ (IV)

Ва (III)

0ав (I)

3324 / 6307
Швидкість проведення збудження нервовими волокнами становить 120 м/с. Який з наведених чинників, перш за все, забезпечує таку швидкість?

Малий поріг деполяризації

Наявність мієлінової оболонки

Великий фактор надійності

Велика амплітуда потенціалу дії

Великий потенціал спокою

3325 / 6307
У тварини зруйнували отолітові вестибулорецептори. Які з наведених рефлексів зникнуть внаслідок цього у тварини?

Статокінетичні при рухах з лінійним прискоренням

Первинні орієнтувальні

Міотатичні

Випрямлення тулуба

Статокінетичні при рухах з кутовим прискоренням

3326 / 6307
У хворого з ВШ-інфекцією визначена ураження шкіри нижніх кінцівок у вигляді множинних пухлинних вузликів синюшно-червоного кольору, що зливаються і створюють поверхневі виразки. При дослідженні біоптату шкіри виявлено новоутворення кровоносних судин, що створюють порожнини різної форми і величини та побудоване з ендотелію, багато пучків веретеноподібних клітин. Діагностуйте найімовірніший характер шкірної патології:

Лімфома шкіри

Запальний дерматит

Базаліома

Саркома Капоші

Дерматомікоз

3327 / 6307
У хворого з патологією серцево-судинної системи розвинулись набряки на нижніх кінцівках. Який механізм розвитку серцевого набряку?

Порушення лімфовідтоку

Підвищення гідростатичного тиску в венулах

Підвищення гідростатичного тиску в артеріолах

Підвищення онкотичного тиску плазми крові

Зниження осмотичного тиску плазми крові

3328 / 6307
У хворої при лікуванні гіпертонічної хвороби на фоні прийому препарату виник сухий кашель. Для якого препарату характерна така побічна дія?

Резерпін

Дротаверин

Но-шпа

Лізиноприл

Гідрохлортіазид

3329 / 6307
У фібробластах шкіри дитини із хворобою Дауна виявлено 47 хромосом. Визначте тип аномалії:

Полісомія Y

Трисомія 18

Трисомія Х

Трисомія 13

Трисомія 21

3330 / 6307
При операції у хлопчика 12-ти років видалений апендикс, який надіслано патологу на дослідження. Макроскопічно: апендикс в дистальному відділі з булавоподібним стовщенням діаметром 3 см, при розрізі якого вилилася прозора жовтувата рідина, стінка апендикса стоншена. Мікроскопічно: атрофія всіх шарів апендикса, ознак запалення немає. Який найбільш імовірний діагноз?

Водянка апендикса

Емпієма апендикса

Флегмонозний апендицит

Хронічний апендицит

Міксоглобульоз апендикса

3331 / 6307
У лікарню наприкінці робочого дня доставлений робітник «гарячого» цеху, який скаржиться на головний біль, запаморочення, нудоту, загальну слабкість. Об’єктивно: свідомість збережена, шкірні покриви гіперемовані, сухі, гарячі на дотик. Частота серцевих скорочень - 130/хв. Дихання часте, поверхневе. Яке порушення процесів терморегуляції, найбільш імовірно, виникло у людини в даній ситуації"?

Зниження теплопродукції без змін тепловіддачі

Посилення тепловіддачі і теплопродукції

Посилення теплопродукції без змін тепловіддачі

Посилення тепловіддачі і зниження теплопродукції

Зниження тепловіддачі

3332 / 6307
При розтині трупа чоловіка 48-ми років виявлено, що кістковий мозок пласких кісток, діафізів та епіфізів трубчастих кісток соковитий, сіро-червоний або сіро-жовтий гноєподібний (піоїдний кістковий мозок). Селезінка масою - 7 кг. На розрізі вона темно-червоного кольору, з ішемічними інфарктами. Всі лімфатичні вузли збільшені, м’які, сіро-червоного кольору. В печінці жирова дистрофія і лейкемічні інфільтрати. Який найбільш імовірний діагноз?

Мієломна хвороба

Хронічний мієлоїдний лейкоз

Гострий мієлоїдний лейкоз

Лімфогранулематоз

Гострий лімфоїдний лейкоз

3333 / 6307
У 3-річної дитини тривале підвищення температури, збільшені лімфовузли, у крові - значне підвищення кількості лімфоцитів. Методом ІФА виявлено антиген віруса Епштейна-Бара. Який діагноз можна поставити на основі вказаного?

Генералізована інфекція, викликана herpes-zoster

Інфекційний мононуклеоз

Герпетична аденопатія

Лімфома Беркета

Цитомегаловірусна інфекція

3334 / 6307
В бактеріологічну лабораторію поступив досліджуваний матеріал (промивні води, в’ялена риба домашнього приготування), взятий у хворого з підозрою на ботулізм. На яке поживне середовище слід зробити первинний посів матеріалу?

Сироватковий агар

Середовище Кітта-Тароцці

Цукровий м’ясо-пептонний бульйон

Цукрово-кров’яний агар

Цукровий м’ясо-пептонний агар

3335 / 6307
У юнака 18-ти років діагностовано хворобу Марфана. При дослідженні встановлено: порушення розвитку сполучної тканини, будови кришталика ока, аномалії серцево-судинної системи, арахнодактилія. Яке генетичне явище зумовило розвиток цієї хвороби?

Кодомінування

Неповне домiнування

Множинний алелiзм

Комплементарність

Плейотропія

3336 / 6307
У бактерій встановлений процес кон’югації, при якому між бактерiями утворюється цитоплазматичний місток, по якому з клітини-донора до клітини-реціпієнта надходять плазміди і фрагменти молекули ДНК. Яке значення цього процесу?

Підвищує гетерозиготність

Забезпечує обмін речовинами між клітинами

Ліквідує небажані мутації

Сприяє активізації мутаційного процесу

Забезпечує обмін генетичного матеріалу

3337 / 6307
Дослідженнями останніх десятиліть встановлено, що безпосередніми «виконавцями» апоптозу в клітині є особливі ферменти - каспази. В утворенні одного з них бере участь цитохром С. Вкажіть його функцію в нормальній клітині:

Компонент H + - АТФ-азной системи

Компонент піруватдегідрогеназної системи

Фермент дихального ланцюга переносу електронів

Фермент ЦТК

Фермент бета-окислювання жирних кислот

3338 / 6307
У відділення реанімації був доставлений хворий з інфарктом міокарда. Який препарат необхідно ввести хворому для лікування больового шоку?

Целекоксиб

Налоксон

Парацетамол

Промедол

Анальгін

3339 / 6307
Жінка 52-х років, хвора на рак молочної залози, пройшла курс променевої терапії. Розмір пухлини зменшився. Який з наведених механізмів ушкодження клітини найбільше обумовлює ефективність променевої терапії?

Утворення вільних радикалів

Лізис NK-клітинами

Мутагенез

Тромбоз судин

Гіпертермія

3340 / 6307
Після операції на кишечнику у хворого з’явились симптоми отруєння аміаком за типом печінкової коми. Який механізм дії аміаку на енергозабезпечення ЦНС?

Гальмування ЦТК в результаті зв’язування альфа-кетоглутарату

Гальмування бета-окиснення жирних кислот

Роз’єднування окисного фосфорилювання

Гальмування гліколізу

Інактивація ферментів дихального ланцюга

3341 / 6307
У пацієнта перфоративна виразка передньої стінки шлунка. В яке похідне очеревини попаде вміст шлунка?

Передшлункова сумка

Правий брижовий синус

Лівий брижовий синус

Чепцева сумка

Печінкова сумка

3342 / 6307
При огляді хворої лікар-гінеколог відмітив симптоми запалення статевих шляхів, у мазку взятому із піхви, виявлено грушоподібні найпростіші з шипом, з передньої частини відходять джгутики, наявна ундулююча мембрана. Яке захворювання підозрює лікар у хворої?

Токсоплазмоз

Лямбліоз

Балантидіоз

Урогенітальний трихомоноз

Кишковий трихомоноз

3343 / 6307
Жінка звернулася до лікаря зі скаргами на утруднення рухів язика. Обстеження головного мозку за допомогою ЯМР показало, що у хворої крововилив в нижньому відділі довгастого мозку. Про пошкодження якого ядра довгастого мозку у хворої можна думати?

Одиноке ядро

Подвійне ядро

Нижнє слиновидільне ядро

Ядро додаткового нерва

Ядро під’язикового нерва

3344 / 6307
При мікроскопічному дослідженні виявляється паренхіматозний орган, в якому епітеліальні тяжі формують клубочкову, пучкову та сітчасту зони. Центральна частина органу представлена скупченнями хроматофінних клітин. Визначте орган:

Гіпофіз

Епіфіз

Щитоподібна залоза

Надниркова залоза

Печінка

3345 / 6307
У відділення реанімації поступив хворий після ДТП з однобічним пневмотораксом. Який вид дихання спостерігається у даному випадку?

Поверхневе рідке

Глибоке часте

Поверхневе часте

Поверхневе

Асфіктичне

3346 / 6307
У хворої виявлена ракова пухлина шийки матки. У які регіонарні лімфатичні вузли можливе розповсюдження метастазів?

Верхні і нижні брижові

Поперекові і ниркові

Навколоміхурні і поперекові

Пахвинні і здухвинні

3347 / 6307
При огляді порожнини рота дитини педіатр виявила наявність нижніх медіальних різців. Дитина розвивається нормально. Визначте її вік:

10-12 місяців

8-9 місяців

13-14 місяців

6-7 місяців

3348 / 6307
Серед антиатеросклеротичних препаратів, що застосовуються з метою профілактики та лікування атеросклерозу, є левостатин. Він діє шляхом:

Гальмування біосинтезу холестерину

Усіма наведеними шляхами

Стимулювання екскреції холестерину з організму

Активації метаболізму холестерину

Пригнічення всмоктування холестерину в кишківнику

3349 / 6307
При зниженні активності ферментів антиоксидантного захисту посилюються процеси перекисного окиснення ліпідів клітинних мембран. При нестачі якого мікроелементу знижується активність глутатіонпероксидази?

Марганець

Молібден

Селен

Мідь

Кобальт

3350 / 6307
Хворий 75-ти років був оперований з приводу рака передміхурової залози, помер раптово на 4-ту добу після оперативного втручання. При розтині тіла померлого у просвітах головного стовбура і біфуркації легеневої артерії були виявлені і легко видалені крихкі маси темно-червоного кольору з тьмяною поверхнею. Такі ж маси знаходились у порожнині правого шлуночка серця. Який різновид порушення кровообігу призвів до раптової смерті хворого?

!нфаркт міокарда

Тромбоемболія легеневої артерії

Тромбоз легеневої артерії

Парадоксальна емболія

Тканинна емболія

3351 / 6307
При обстеженні жінки 56-ти років, що хвора на цукровий діабет 1-го типу, виявлене порушення білкового обміну, що при лабораторному дослідженні крові проявляється аміноацидемією а клінічно - уповільненням загоєння ран і зменшенням синтезу антитіл. Який з перерахованих механізмів викликає розвиток аміноацидемії?

Підвищення протеолізу

Зменшення концентрації амінокислот у крові

Збільшення ліпопротеїдів високої щільності

Підвищення онкотичного тиску в плазмі крові

Гіперпротеїнемія

3352 / 6307
При недостатності вітаміну C спостерігається порушення структури колагенових волокон. Яка стадія їх синтезу порушується при цьому?

Утворення гідроксипроліну і гідроксилізину

Агрегація тропоколагену і утворення фібрил

Приєднання глюкозних і галактозних залишків

Відщеплення сигнальних олігопептидів

Утворення гама-карбоксиглутамату

3353 / 6307
У людини в результаті патологічного процесу збільшена товщина гематоальвеолярного бар’єру. Безпосереднім наслідком цього буде зменшення:

Хвилинного об’єму дихання

Дифузійної здатності легень

Альвеолярної вентиляції легень

Кисневої ємності крові

Резервного об’єму видиху

3354 / 6307
Недостатність в організмі мікроелементу селену проявляється кардіоміопатією. Імовірною причиною такого стану є зниження активності такого селенвмісного ферменту:

Лактатдегідрогеназа

Сукцинатдегідрогеназа

Глутатіонпероксидаза

Цитохромоксидаза

Каталаза

3355 / 6307
У хворого після перенесеного ішемічного інсульту стали неможливими довільні рухи у правих кінцівках, спостерігається гіперрефлексія. Під час пальпації визначається підвищений тонус м’язів кінцівок. Яка форма порушення рухової функції має місце у хворого?

Периферичний парез

Мозочкова атаксія

Периферичний параліч

Центральний параліч

Тетанія

3356 / 6307
У жінки, що тривало приймала антибіотики з приводу кишкової інфекції, розвинулось ускладнення з боку слизової порожнини рота у вигляді запального процесу і білого нальоту, у якому під час бактеріологічного дослідження були виявлені дріжджеподібні грибки Candida albicans. Який з перерахованих препаратів показаний для лікування цього ускладнення?

Фуразолідон

Тетрациклін

Поліміксин

Бісептол

Флуконазол

3357 / 6307
Досліджуються клітини червоного кісткового мозку людини, які належать до клітинного комплексу, що постійно діляться. Який процес забезпечує генетичну ідентичність цих клітин:

Мутація

Мітоз

Трансплантація

Репарація

Мейоз

3358 / 6307
Хвора 57-ми років для лікування гіпертонічної хвороби тривалий час приймала анаприлін. Побічні ефекти спонукали пацієнтку відмовитись від прийому препарату, що призвело до розвитку гіпертонічного кризу і нападу стенокардії. Як називається ускладнення, яке виникло?

Звикання

Тахіфілаксія

Лікарська залежність

Синдром відміни

Сенсибілізація

3359 / 6307
При лабораторному дослідженні крові пацієнта виявлено, що вміст білків у плазмі становить 40 г/л. Як це впливає на транскапілярний обмін води в мікроциркуляторному руслі?

Збільшується фільтрація, зменшується реабсорбція

Зменшується фільтрація, збільшується реабсорбція

Зменшуються фільтрація і реабсорбція

Збільшуються фільтрація і реабсорбція

Обмін не змінюється

3360 / 6307
У хворого спостерігається тремтіння рук, що пов’язане з хворобою Паркінсона. Дефіцит якого медіатора в стріопалідарних структурах призводить до таких симптомів?

Субстанція Р

Норадреналін

Дофамін

ГАМК

Серотонін

3361 / 6307
У хворого на жовчно-кам’яну хворобу спостерігаються ознаки холемічного синдрому. Який симптом з перерахованих зумовлений відсутністю надходження жовчі у кишечник?

Астенія

Гіпотонія

Шкірний свербіж

Стеаторея

Брадикардія

3362 / 6307
У людини після гострої крововтрати виникло відчуття спраги. Зміна якого гомеостатичного параметру викликала це відчуття?

Підвищення осмотичного тиску рідин організму

Підвищення онкотичного тиску рідин організму

Зменшення об’єму позаклітинної рідини

Зниження онкотичного тиску рідин організму

Зниження осмотичного тиску рідин організму

3363 / 6307
Студент використав консервовану донорську кров для визначення часу її зсідання. Однак, будь-якого позитивного результату він отримати не зміг. Причиною цього є відсутність в крові:

Фі6риногєну

Вітаміну K

Іонізованого кальцію

Тромбопластину

Фактора Хагемана

3364 / 6307
Введення тварині екстракту тканини передсердя посилює виділення натрію з сечею. Дія якої біологічно активної речовини стала причиною такого стану?

Серотонін

Адреналін

Калійкреїн

Натрійуретичний гормон

Ілюкокортикоїд

3365 / 6307
Хворій жінці із захворюванням нирок, що супроводжується вираженими набряками, призначили діуретичний препарат, що пригнічує реабсорбцію в нирках іонів Na+ і води, посилює виведення нирками іонів K+ і Mg++, викликає гіперурикемію, зумовлює потужний діуретичний ефект. Назвіть цей препарат:

Діакарб

Аллопуринол

Спіронолактон

Тріамтерен

Фуросемід

3366 / 6307
Хворому призначили антибіотик хлорамфенікол (левомецитин), який порушує у мікроорганізмів синтез білку шляхом гальмування процесу:

Ампліфікація генів

Транскрипція

Утворення полірибосом

Процесінг

Елонгація трансляції

3367 / 6307
До офтальмолога звернулася жінка зі скаргами на шкірний свербіж і набряклість повік. При обстеженні виявлено членистоноге червоподібної форми величиною 0,4 мм. Суцільний щиток вкриває передню частину тіла, тіло має поперечну смугастість. Ноги короткі, лапки з двома кігтиками. Який діагноз може встановити лікар?

Фтиріоз

Демодекоз

Факультативний міаз

Педікульоз

Короста

3368 / 6307
У хворого 69-ти років на шкірі в ділянці нижньої повіки з’явилося невелике бляшкоподібне утворення з наступним виразкуванням, яке було оперативно видалене. При мікроскопічному дослідженні утворення: в дермі шкіри комплекси з атипових епітеліальних клітин; периферії клітини розташовані перпендикулярно до базальної мембрани. Клітини темні, призматичної полігональної форми з гіперхромними ядрами з частими мітозами. Іноді зустрічаються утворення, подібні до волосяного фолікула. Яка гістологічна форма раку у хворого?

Недиференційований рак

Базально-клітинний рак

Плоскоклітинний рак з ороговінням

Аденокарцинома

Плоскоклітинний рак без ороговіння

3369 / 6307
Після перелому нижньої щелепи постраждалий відзначає втрату чутливості шкіри у ділянці підборіддя і нижньої губи. Який нерв був пошкоджений?

Mentalis

Buccalis

Infraorbitalis

Facialis

Maxillaris

3370 / 6307
У молодого чоловіка внаслідок подразнення сонячного сплетення запальним процесом (солярит) підвищена функціональна активність залоз шлунка, що виражається, зокрема, у збільшенні продукції хлоридної кислоти. Яка з вказаних нижче речовин викликає гіперхлоргідрію у даному випадку?

Калікреїн

Гастрин

Глюкагон

Гастроінгібуючий пептид

Урогастрон

3371 / 6307
У альпініста, що піднявся на висоту 5200 м, розвинувся газовий алкалоз. Що є причиною його розвитку?

Гіпоксемія

Зниження температури навколишнього середовища

Гіповентиляція легенів

Гіпероксемія

Гіпервентиляція легенів

3372 / 6307
Реакції міжмолекулярного транспорту одновуглецевих радикалів є необхідними для синтезу білків та нуклеїнових кислот. З якого з наведених нижче вітамінів утворюється кофермент, потрібний для вказаних вище реакцій?

Рибофлавін

Фолієва кислота

Аскорбінова кислота

Тіамін

Пантотенова кислота

3373 / 6307
Прокаріотичні та еукаріотичні клітини характеризуються здатністю до по- ділу. Поділ прокаріотичних клітин відрізняється від поділу еукаріотичних, але існує молекулярний процес, який лежить в основі цих поділів. Який це процес?

Ампліфікація генів

Транскрипція

Трансляція

Реплікація ДНК

Репарація

3374 / 6307
Для проведення анальгезії наркотичний анальгетик застосували з препаратом бензодіазепінового ряду. Який засіб використали для потенціювання анальгезії?

Імізин

Діазепам

Хлорпротіксен

Карбамазепін

Трифтазин

3375 / 6307
Молода людина 25-ти років споживає надмірну кількість вуглеводів (600 г на добу), що перевищує її енергетичні потреби. Який процес буде активуватися в організмі людини уданому випадку?

Ліпогенез

Глюконеогенез

Ліполіз

Гліколіз

Окиснення жирних кислот

3376 / 6307
Юнак 15-ти років скаржиться на загальну слабкість, запаморочення, швидку стомлюваність. При обстеженні виявлено еритроцити зміненої форми, кількість їх знижена. Попередній діагноз: серпоподібно-клітинна анемія. Який тип мутації зумовлює розвиток цього патологічного стану?

Хромосомна аберація

Інверсія

Точкова мутація

Делеція

Мутація зсуву рамки зчитування

3377 / 6307
Хворому із больовим синдромом в суглобах постійно призначають аспірин. Який з перерахованих ферментів він пригнічує?

Циклооксигеназа

Фосфоліпаза С

Ліпооксигеназа

Фосфоліпаза А2

Фосфоліпаза Д

3378 / 6307
Препарати групи вітаміну В2 призначають при захворюваннях шкіри. Завдяки наявності якої структури в його складі визначається здатність до окиснення-відновлення?

Ізоалоксазин

Аденін

Залишок фосфорної кислоти

Рибозофосфат

Рибітол

3379 / 6307
Під час автомобільної аварії людина отримала сильний удар в епігастральну ділянку, внаслідок чого виникла зупинка серця. Що могло стати причиною таких змін серцевої діяльності?

Збільшене виділення альдостерону

Збільшене виділення кортизолу

Підвищення тонусу блукаючого нерва

Підвищення тонусу симпатичної нервової системи

Збільшене виділення адреналіну

3380 / 6307
У хворого з синдромом Іценко-Кушинга спостерігаються стійка гіперглікемія та глюкозурія. Синтез та секреція якого гормону збільшені у цього хворого?

Альдостерон

Кортизол

Тироксин

Адреналін

Глюкагон

3381 / 6307
У хворого має місце хронічний запальний процес мигдаликів. За рахунок якого біохімічного процесу у вогнищі запалення підтримується концентрація НАДФН, необхідного для реалізації механізму фагоцитозу?

Орнітиновий цикл

Гліколіз

Цикл Корі

Пентозо-фосфатний шлях

Цикл Кребса

3382 / 6307
Щоденно в організмі людини 0,5% всього гемоглобіну перетворюється на метгемоглобін. Який фермент, що міститься в еритроцитах, каталізує відновлення метгемоглобіну до гемоглобіну?

Глюкуронілтрансфераза

Білівердинредуктаза

Метгемоглобінтрансфераза

Метгемоглобінредуктаза

Гемоксигеназа

3383 / 6307
У хворого з гемолітичною анемією виявлено дефіцит піруваткінази в еритроцитах. За цих умов причиною розвитку гемолізу еритроцитів є:

Надлишок К + в еритроцитах

Нестача Na+ в еритроцитах

Генетичні дефекти глікофорину А

Зменшення активності Na+, К+ -АТФ-ази

Дефіцит спектрину

3384 / 6307
Хвора 37-ми років померла під час нападу експіраторної задухи, що був спричинений контактом з екзогенним алергеном (пилок амброзії). При гістологічному дослідженні в просвіті бронхів спостерігаються скупчення слизу, в стінці бронхів багато тучних клітин (лаброцитів), більшість з яких у стані дегрануляції, багато еозинофілів. До патогенезу якого типу реакцій гіперчутливості можна віднести описані зміни?

IV типу (клітинна цитотоксичність)

I типу (анафілактична)

V типу (гранулематоз)

II типу (антитілозалежна)

III типу (імунокомплексна)

3385 / 6307
У хворого запалення присереднього надвиростка плечової кістки (епіконділіт). Який нерв залучений у процес?

N. medianus

N. axillaris

N. radialis

N. musculocutaneus

N. ulnaris

3386 / 6307
Який вітамінний препарат доцільно призначити пацієнту зі скаргами на зниження гостроти зору в сутінках, сухість шкіри, часті застудні захворювання, ламкість волосся?

Ретинолу ацетат

Пантотенова кислота

Тіаміну хлорид

Нікотинова кислота

Ергокальциферол

3387 / 6307
43-х річний капітан корабля скаржиться на прояви сезонної алергії. Який лікарський засіб, що не має снодійного ефекту, можна йому призначити?

Дипразин

Ванкоміцин

Лоратадин

Мікосептин

Супрастин

3388 / 6307
У жінки 52-х років артеріальна гіпертензія ускладнилась правобічною геміплегією і втратою мови. Яка зона головного мозку є імовірно найбільш ураженою?

Права передня звивина

Ліва скронева частка

Потилична частка

Ліва передня звивина і ліва скронева частка

Ліва передня звивина

3389 / 6307
Хворому на хронічний бронхіт призначили муколітичний препарат, що підвищує синтез сурфактанту. Який препарат був використаний?

Лібексин

Кодеїн

Амброксол

Мукалтин

Фенотерол

3390 / 6307
Ефективна діагностика носійства збудників кишкових інфекцій ґрунтується на виявленні антитіл до певних антигенів бактерій в реакції непрямої гемаглютинації. Який стандартний препарат слід застосувати у цій реакції'?

Еритроцитарні діагностикуми з адсорбованими антигенами бактерій

Моноклональні антитіла

Еритроцити барана й гемолітичну сироватку

Антитіла проти імуноглобулінів основних класів

Монорецепторні діагностичні сироватки

3391 / 6307
На розтині тіла померлого, який страждав на гіпертонічну хворобу, у лівій гемісфері мозку виявлена порожнина округлої форми 4х5 см із іржавою стінкою, заповнена жовтуватою прозорою рідиною. Назвіть патологію, яка розвинулась у головному мозку хворого:

Геморагічне просякнення

Абсцес

Гематома

Немічний інфаркт

Кіста

3392 / 6307
Клінічні ознаки хвороби у пацієнта почалися через 24 години після вживання рибних консервів і включали нудоту, слабкість, сухість у роті. Потім приєдналися двоїння зображення, афагія, проблеми з диханням. Чим обумовлена така симптоматика при ботулізмі?

Вплив нейротоксину

Активація цАМФ

Розмноження збудника у ШКТ

Вплив ентеротоксину

Вплив ендотоксину

3393 / 6307
У хворого з підозрою на «озену» з носоглотки були виділені грамнегативні палички, які утворювали капсулу на поживному середовищі. Які мікроорганізми спричинили хворобу?

Клебсієли

Шигели

Хламідії

Сальмонели

Мікоплазми

3394 / 6307
До травматологічного пункту доставлено хворого з пошкодженням м’язів нижніх кінцівок. За рахунок яких клітин можлива репаративна регенерація м’язових волокон і відновлення функції м’язів?

Плазмоцити

Міосателітоцити

Фібробласти

Ендотеліоцити

Адипоцити

3395 / 6307
У чоловіка 25-ти років на 4-й день після крововтрати в крові зросла кількість поліхроматофільних еритроцитів, ретикулоцитів і навіть з’явились поодинокі нормобласти. Про що свідчить такі гематологічні зміни?

Процеси регенерації випереджають дозрівання клітин

Знижена продукція еритропоезу в нирках

Ослаблені процеси еритропоезу

Зменшено об’єм плазми крові

Підвищена спорідність гемоглобіну до кисню

3396 / 6307
Хворому на туберкульоз легень призначений препарат першого ряду, який спричинив розвиток невриту лицьового нерва та порушення рівноваги. Вкажіть цей лікарський засіб:

Цефазолін

Ізоніазид

Левоміцетин

Бісептол

Фуразолідон

3397 / 6307
Хірург під час операції на щитоподібній залозі перев’язав верхню щитоподібну артерію. Гілку якої судини перев’язав лікар?

A. carotis externa

A. carotis interna

A. lingualis

A. pharyngea ascendens

A. facialis

3398 / 6307
Батьки - глухонімі, але глухота у дружини залежить від аутосомно-рецесивного гена, а у чоловіка виникла внаслідок тривалого прийому антибіотиків у дитинстві. Яка імовірність народження глухої дитини в родині, якщо батько гомозиготний за аллелю нормального слуху?

10%

75%

25%

0%

100%

3399 / 6307
До лікарні потрапив футболіст з ушкодженням поверхневого пахвинного кільця та розривом двох ніжок, що його обмежують. Похідним якої анатомічної структури вони є?

Апоневроз поперечного м’яза живота

Апоневроз внутрішнього косого м’яза живота

Апоневроз зовнішнього косого м’яза живота

Міжножкові волокна

Власна фасція живота

3400 / 6307
У потерпілого перелом у ділянці бічної поверхні променево-зап’ясткового суглоба. Де найімовірніше відбувся перелом?

Шилоподібний відросток променевої кістки

Нижня третина плечової кістки

Головчаста кістка

Шилоподібний відросток ліктьової кістки

Гачкувата кістка

3401 / 6307
Під час гіпертонічного кризу хворому ввели магнію сульфат, в результат чого відбулося різке зниження артеріального тиску. Введенням якого препарату можна запобiгти по6ічному ефекту сульфату магнію?

Натрію сульфат

Кальцію хлорид

Трилон Б

Натрію бромід

Калію хлорид

3402 / 6307
Паренхіма аденогіпофізу представлена трабекулами, утвореними залозистими клітинами. Серед аденоцитів є клітини з гранулами, які забарвлюються основними барвниками і містять глікопротеїди. Які це клітини?

Гонадотропоцити, тиротропоцити

Хромофобні клітини

Меланотропоцити

Соматотропоцити

Мамотропоцити

3403 / 6307
У хворого із сечокам’яною хворобою виник нестерпний спастичний біль. Для попередження больового шоку йому ввели разом з атропіном наркотичний анальгетик, що не має спазмогенного ефекту. Який це препарат?

Промедол

Піритрамід

Морфіну гідрохлорид

Трамадол

Етилморфіну гідрохлорид

3404 / 6307
До травматологічного пункту доставлено постраждалого після ДТП з діагнозом: закритий перелом середньої третини стегна зі зміщенням. З метою репозиції кісткових уламків хворому введено 10 мл 2% розчину дитиліну в/в, внаслідок чого розвинулося тривале апное та міорелаксація. Дефіцитом якого ферменту зумовлена вказана фармакогенетична ферментопатія?

Глюкозо-6-фосфатдегідрогеназа

Псевдохолінестераза

Метгемоглобінредуктаза

Уридіндифосфоглюкуронова трансфераза

N-ацетилтрансфераза

3405 / 6307
Підлітку, що перебував у стані тяжкого алкогольного сп’яніння, лікар швидкої допомоги серед інших заходів здійснив внутрішньом’язове введення розчину кофеїну. Поясніть, який принцип взаємодії між алкоголем та кофеїном пояснює доцільність даної маніпуляції:

Синергізм

Потенціація

Сумація ефектів

Конкурентний антагонізм

Фізіологічний антагонізм

3406 / 6307
Захворювання бері-бері - це класична форма недостатності вітаміну тіаміну. Активна форма його синтезується за допомогою ферменту з класу:

Трансфераз

Ізомераз

Оксидоредуктаз

Ліаз

Гідролаз

3407 / 6307
Після введення пірогену у хворого підвищилася температура тіла, шкірні покриви стали блідими, холодними на дотик, з’явилася остуда, збільшилося споживання кисню. Яка зміна у процесах терморегуляції буде спостерігатися, насамперед, під час описаного періоду лихоманки?

Тепловіддача та теплопродукція перебувають у рівновазі

Зниження тепловіддачі

Збільшення теплопродукції

Зниження теплопродукції

3408 / 6307
Епідеміологічне дослідження поширення пухлин виявило високу кореляцію розвитку пухлин легень з тютюнопалінням. З дією якого хімічного канцерогену найімовірніше пов’язаний розвиток даного виду патології?

3,4-бензопірен

Афлатоксин

Діетилнітрозамін

Метилхолантрен

Ортоаміноазотолуол

3409 / 6307
В ході експерименту у білого щура моделювався набряк легені шляхом введення адреналіну. Який патогенетичний механізм розвитку набряку є провідним в даному випадку?

Лімфогенний

Мембраногенний

Токсичний

Колоїдно-осмотичний

Гідродинамічний

3410 / 6307
У хворого інфаркт міокарда. Активність якого ферменту буде значно підвищена в сироватці крові хворого в перші години?

Креатинфосфокіназа МВ

ЛДГ4

АЛТ

ЛДГ5

ACT

3411 / 6307
У хворого спостерігається атонія м’язів. Назвіть фермент м’язової тканини, активність якого може бути знижена при такому стані:

Транскетолаза

Креатинфосфокіназа

Глутамінтрансфераза

Амілаза

Каталаза

3412 / 6307
Пацієнтові, що звернувся до травмпункту у зв’язку з травмою, отриманою під час роботи на присадибній ділянці, лікар призначив введення правцевого анатоксину. Який імунітет сформується у даного пацієнта після введення препарату?

Антимікробний активний

Антитоксичний активний

Нестерильний

Антимікробний пасивний

Антитоксичний пасивний

3413 / 6307
При яких групах крові батьків за резус-фактором можливий розвиток резус-конфлікту під час вагітності?

Жінка Rh(-), чоловік Rh(+) (гомозигота)

Жінка Rh(+) (гетерозигота), чоловік Rh(+) (гомозигота)

Жінка Rh(+), чоловік Rh(+) (гетерозигота)

Жінка Rh(+), чоловік Rh(+) (гомозигота)

Жінка Rh(-), чоловік Rh(-)

3414 / 6307
Здорова людина перебуває в небезпечному за захворюванням на малярію районі. Який із зазначених препаратів необхідно призначити з метою особистої хіміопрофілактики малярії?

Хінгамін

Тетрациклін

Бісептол

Метронідазол

Сульфален

3415 / 6307
У хворого спостерігається типова для нападу малярії клінічна картина: остуда, жар, проливний піт. Яка стадія малярійного плазмодію найімовірніше буде виявлена в крові хворого в цей час?

Оокінета

Спороциста

Спорозоїт

Мікро- або макрогамети

Мерозоїт

3416 / 6307
Юнак 15-ти років скаржиться на загальну слабкість, запаморочення, швидку стомлюваність. В ході обстеження виявлено еритроцити зміненої форми, кількість їх знижена. Попередній діагноз: серпоподібноклітинна анемія. Який тип мутації зумовлює розвиток цього патологічного стану?

Інверсія

Мутація зсуву рамки зчитування

Точкова мутація

Делеція

Хромосомна аберація

3417 / 6307
Хворій виконують операцію на щитоподібній залозі. Гілки яких артерій повинен перев’язати хірург під час операції?

Верхня щитоподібна та щитошийний стовбур

Верхня та нижня гортанна

Верхня та нижня щитоподібна

Верхня щитоподібна та висхідна шийна

Висхідна шийна та глибока шийна

3418 / 6307
У хворого 75-ти років з позачеревної клітковини видалена пухлина розмірами 16х8х6 см. Мікроскопічно: анаплазовані жирові клітини з ознаками клітинного атипізму, поліморфізму. Зустрічаються величезні спотворені клітини, що мають у цитоплазмі жирові крапельки. Найімовірнішим є такий діагноз:

Ліпосаркома

Міосаркома

Ліпома

Фібросаркома

Мезотеліома

3419 / 6307
Внаслідок травми у людини ушкоджений отолітовий апарат внутрішнього вуха. На які подразники НЕ ЗМОЖЕ реагувати ця людина?

Рух з лінійним прискоренням

Світлові

Рух з кутовим прискоренням

Дотикові

Звукові

3420 / 6307
У хворого напад тахікардії. Які мембранні циторецептори кардіоміоцитів доцільно заблокувати, щоб припинити напад?

М-холінорецептори

α-адренорецептори

β-адренорецептори

М- та Н-холінорецептори

Н-холінорецептори

3421 / 6307
Під час патологічних процесів, які супроводжуються гіпоксією, відбувається неповне відновлення молекули кисню в дихальному ланцюзі і накопичення пероксиду водню. Вкажіть фермент, який забезпечує його руйнування:

Каталаза

Кетоглутаратдегідрогеназа

Цитохромоксидаза

Аконітаза

Сукцинатдегідрогеназа

3422 / 6307
У хворого після травми виникла необхідність введення протиправцевої сироватки, але проба на чутливість до сироватки виявилася позитивною. Специфічну гіпосенсибілізацію у хворого слід виконати за допомогою введення:

Наркотичних речовин, що знижують чутливість

Роздільної дози специфічного алергену

Лікувальних доз антигістамінних препаратів

Мінімальних доз специфічного алергену

Фізіологічних доз глюкокортикоїдів

3423 / 6307
Водій автомобіля дістав травму грудної клітки внаслідок удару об рульове колесо. Яка з перерахованих артерій найімовірніше може бути ушкодженою?

A. thoracica interna

А. subscapularis

А. vertebralis

А. thyroidea superior

А. suprascapularis

3424 / 6307
У вагітної жінки розвинувся токсикоз з тяжким повторним блюванням шлунковим вмістом протягом доби. Наприкінці доби почали проявлятися тетанічні судоми та зневоднення організму. Який розлад кислотно-лужної рівноваги викликав дані зміни?

Газовий ацидоз

Негазовий видільний алкалоз

Негазовий видільний ацидоз

Негазовий метаболічний ацидоз

Газовий алкалоз

3425 / 6307
На ізольованому серці шляхом охолодження припиняють функціонування окремих структур. Яку структуру охолодили, якщо серце внаслідок цього спочатку припинило скорочення, а далі відновило їх із частотою у 2 рази меншою за вихідну?

Ніжки пучка Гіса

Атріовентрикулярний вузол

Волокна Пуркіньє

Пучок Гіса

Синоатріальний вузол

3426 / 6307
У жінки 30-ти років хвилинний об’єм крові у стані спокою становить 5 л/хв. Який об’єм крові проходить у неї за 1 хвилину крізь судини легень?

3,75 л

2,5 л

1,5 л

2,0 л

5 л

3427 / 6307
Після перенесеного запального захворювання у хворого виникло неповне відведення очного яблука в латеральну сторону. Який нерв у хворого пошкоджений?

Відвідний

Окоруховий

Лицевий

Блоковий

Зоровий

3428 / 6307
У дитини, що померла від дифтерії з явищами серцевої недостатності, на розтині виявлено, що порожнини серця розширені в поперечнику; м’яз серця тьмяний, в’ялий, на розрізі строкатий, з жовтуватими ділянками. У цитоплазмі деяких кардіоміоцитів зі збереженою цитоплазмою спостерігаються дрібні вакуолі. На заморожених зрізах вакуолі забарвлюються у оранжевий колір суданом 3. Який вид дистрофії виявлений у кардіоміоцитах?

Балонна

Гідропічна

Вуглеводна

Жирова

Гіаліново-крапельна

3429 / 6307
У хворого на тимому (пухлину загрудинної залози) спостерігаються ціаноз, розширення підшкірної венозної сітки і набряк м’яких тканин обличчя, шиї, верхньої половини тулуба і верхніх кінцівок. Який венозний стовбур перетиснено пухлиною?

Зовнішня яремна вена

Передня яремна вена

Підключична вена

Верхня порожниста вена

Внутрішня яремна вена

3430 / 6307
Хворій 20-ти років в зв’язку з ревматизмом призначено тривалий прийом аспірину. Який структурний компонент слизової оболонки шлунку найбільшою мірою забезпечить її захист від ушкодження?

Одношаровий призматичний залозистий епітелій

М’язова тканина

Багатошаровий плаский незроговілий епітелій

Багатошаровий війчастий епітелій

Сполучна тканина

3431 / 6307
У хворої 36-ти років, яка лікувалася сульфаніламідами з приводу респіраторної вірусної інфекції, в крові спостерігаються гіпорегенераторна нормохромна анемія, лейкопенія, тромбоцитопенія. В кістковому мозку - зменшення кількості мiєлокарiоцитiв. Яка це анемія?

Постгеморагічна

Гемолітична

В 12-фолієводефіцитна

Гіпопластична

Залізодефіцитна

3432 / 6307
Чоловік 38-ми років раптово помер. На розтині: у задній стінці лівого шлуночка серця виявлено інфаркт міокарда. Які найбільш імовірні зміни у будові міокардіоцитів можна побачити у вогнищі інфаркту мікроскопічно?

Вуглеводна дистрофія

Білкова дистрофія

Каріолізис

Жирова дистрофія

Звапнування

3433 / 6307
У лікарню надійшла робітниця хімічного підприємства з ознаками отруєння. У волоссі цієї жінки знайдено підвищену концентрацію арсенату, який блокує ліпоєву кислоту. Вкажіть, порушення якого процесу є найімовірнішою причиною отруєння:

Мікросомальне окиснення

Окислювальне декарбоксилювання ПВК

Знешкодження супероксидних іонів

Відновлення органічних перекисів

Відновлення метгемоглобіну

3434 / 6307
Під час операції на головному мозку помічено, що подразнення певних зон кори великих півкуль викликало у хворого і тактильні і температурні відчуття. На яку звивину діяли подразники?

Поясна

Постцентральна

Прецентральна

Верхня латеральна

Парагіпокампова

3435 / 6307
З метою аналгезії можуть бути використані речовини, що імітують ефекти морфіну, але виробляються в ЦНС. Вкажіть таку речовину:

Соматоліберин

Вазопресин

β-ендорфін

Кальцитонін

Окситоцин

3436 / 6307
У хворого через 12 годин після гострого нападу загрудинного болю спостерігається різке підвищення активності АсАТ в сироватці крові. Вкажіть патологію, для якої характерне це зміщення:

Інфаркт міокарду

Вірусний гепатит

Цукровий діабет

Нецукровий діабет

Колагеноз

3437 / 6307
У дитини, що народилася від пізнього шлюбу, спостерігаються малий зріст, відставання у розумовому розвитку, товстий 'географічний' язик, вузькі очні щілини, пласке обличчя з широкими вилицями. Що найімовірніше спричинило розвиток вказаного синдрому?

Внутрішньоутробний імунний конфлікт

Внутрішньоутробна інфекція

Пологова травма

Внутрішньоутробна інтоксикація

Спадкова хромосомна патологія

3438 / 6307
Людина стоїть у кімнаті в легкому одязі; температура повітря +14°C. Вікна і двері зачинені. Яким шляхом організм людини віддає найбільше тепла?

Випаровування

Теплорадіація

Конвекція

Перспірація

Теплопроведення

3439 / 6307
До медико-генетичної консультації звернулася жінка за рекомендацією гінеколога з приводу відхилень фізичного і статевого розвитку. В ході мікроскопії клітин слизової оболонки ротової порожнини не знайдено статевого хроматину. Який найбільш імовірний діагноз?

Синдром Шерешевського-Тернера

Синдром Клайнфельтера

Хвороба Дауна

Трисомія за Х-хромосомою

Хвороба Реклінгаузена

3440 / 6307
35-річному хворому на атопічний дерматит був призначений лоратадин. Визначте механізм дії цього препарату:

Блокує Н1-гістамінові рецептори

Стимулює М-холінорецептори

Блокує β-адренорецептори

Блокує ГАМК-рецептори

Стимулює дофамінові рецептори

3441 / 6307
Розпочинається імплантація бластоцисти людини. Як називається період ембріогенезу, що розпочинається одночасно з імплантацією?

Дроблення

Гістогенез

Інвагінація

Диференціювання

Гаструляція

3442 / 6307
Лікар швидкої допомоги був викликаний до жінки 40-ка років з приводу нападу бронхіальної астми з явищами стенокардії, лікар ввів хворій належний препарат. Який із наведених препаратів найбільш ефективний для невідкладної допомоги?

Ефедрин

Адреналін

Сальбутамол

Платифілін

Атропін

3443 / 6307
Який препарат слід призначити хворому, у якого з’явилося безсоння внаслідок появи висипань алергічного характеру з почервонінням, набряком та сильним свербінням?

Хлоралгідрат

Фенобарбітал

Димедрол

Нітразепам

Натрію оксибутират

3444 / 6307
В дитячому садку проведені планові щеплення вакциною проти кору. Яким методом можна перевірити формування післявакцинального імунітету?

Алергічний

Серологічний

Бактеріоскопічний

Вірусологічний

Бактеріологічний

3445 / 6307
Після курсу терапії хворому на виразку дванадцятипалої кишки лікар пропонує вживати соки з капусти та картоплі. Вміст яких речовин в цих продуктах сприяє профілактиці та загоєнню виразок?

Вітамін Б

Вітамін K

Вітамін C

Пантотенова кислота

Вітамін U

3446 / 6307
У дитини з білявим волоссям, блідою шкірою відмічається збільшений тонус м’язів, судоми та розумова відсталість. В крові підвищений рівень фенілаланіну. Який з перелічених методів необхідно застосувати для встановлення діагнозу цієї ензимопатії?

Біохімічний

Генеалогічний

Популяційно-статистичний

Електрофізіологічний

Цитогенетичниий

3447 / 6307
Хворому з больовим синдромом в суглобах постійно призначають аспірин. Який з перерахованих ферментів він пригнічує?

Циклооксигеназа

Фосфоліпаза С

Фосфоліпаза D

Фосфоліпаза А2

Ліпооксигеназа

3448 / 6307
Робота шахтарів у забої часто спричинює антракоз. Який вид дихальної недостатності може розвинутися при цьому?

Обструктивний

Рестриктивний

Торакальний

Діафрагмальний

Дисрегуляторний

3449 / 6307
До гінекологічного відділення госпіталізована жінка з клінікою гострого живота, припускається позаматкова вагітність. Крізь яке анатомічне утворення або його частину гінеколог буде виконувати пункцію черевної порожнини?

Fornix vaginae posterior

Rectum

Cervix uteri

Fornix vaginae anterior

3450 / 6307
У дитини 6-ти місяців спостерігається різке відставання в психомоторному розвитку, бліда шкіра з екзематозними змінами, біляве волосся, блакитні очі, напади судом. Найточніше встановити діагноз у цієї дитини дозволить визначення в крові і сечі концентрації такої речовини:

Валін

Триптофан

Гістидин

Лейцин

Фенілпіруват

3451 / 6307
У хворого діагностували легеневу форму туберкульозу. Який з наведених лікарський засіб використовують для лікування туберкульозу?

Норсульфазол

Ізоніазид

Фуразолідон

Тетрациклін

Пеніцилін

3452 / 6307
Дитина доставлена в санпропускник в стані асфіксії. Під час огляду в гортані виявлені білуваті плівки, що обтурують просвіт та легко видаляються. Лікар припускає дифтерію. Про яке запалення гортані йдеться?

Гнійне

Катаральне

Крупозне

Серозне

Дифтеритичне

3453 / 6307
В бактеріологічній лабораторії досліджувалися консерви, які спричинили тяжку токсикоінфекцію. В результаті мікроскопії культури з середовища Кітта-Тароцці виділені грампозитивні палички, схожі на тенісну ракетку. Який діагноз встановить лікар?

Хламідіоз

Туляремія

Ботулізм

Дизентерія

Черевний тиф

3454 / 6307
Хворому на глаукому призначили пілокарпіну гідрохлорид в очних краплях. До якої фармакологічної групи належить цей препарат?

Міорелаксанти

М-холіноміметики

Гангліоблокатори

М-холіноблокатори

α-адреноблокатори

3455 / 6307
Під час автомобільної аварії людина зазнала сильного удару в епігастральну ділянку, внаслідок чого виникла зупинка серця. Що могло стати причиною таких змін серцевої діяльності?

Підвищене виділення альдостерону

Підвищене виділення адреналіну

Підвищення тонусу блукаючого нерва

Підвищене виділення кортизолу

Підвищення тонусу симпатичної нервової системи

3456 / 6307
До лікаря-інфекціоніста на прийом прийшов хворий зі скаргами на лихоманку, яка триває три дні, загальну слабкість, безсоння, погіршення апетиту. Лікар припускає черевний тиф. Для підтвердження діагнозу найдоцільніше призначити виділення:

Копрокультури

Мієлокультури

Білікультури

Гемокультури

Уринокультури

3457 / 6307
У здорової дорослої людини швидкість проведення збудження через атріовентрикулярний вузол дорівнює 0,02-0,05 м/с. Атріовентрикулярна затримка забезпечує:

Одночасність скорочення обох передсердь

Одночасність скорочення обох шлуночків

Достатню силу скорочення шлуночків

Достатню силу скорочення передсердь

Послідовність скорочення передсердь та шлуночків

3458 / 6307
В клітині, яка мітотично ділиться, спостерігається розходження дочірніх хроматид до полюсів клітини. На якій стадії мітотичного циклу перебуває клітина?

Телофаза

Метафаза

Профаза

Анафаза

Інтерфаза

3459 / 6307
На розтині тіла померлого виявлено, що вся права легеня збільшена, щільна, на плеврі нашарування фібрину, на розрізі тканина сірого кольору, з якої стікає каламутна рідина. Для якого захворювання легенів характерна така картина?

Інтерстиціальна пневмонія

Гангрена легені

Вогнищева пневмонія

Фіброзувальний альвеоліт

Крупозна пневмонія

3460 / 6307
Вкажіть антибіотик, що утворює хелатні сполуки з іонами кальцію, заліза, алюмінію, які не абсорбуються з тонкого кишківника:

Ампіциліну натрієва сіль

Хіноксидин

Гентаміцину сульфат

Доксицикліну гідрохлорид

Амоксиклав

3461 / 6307
Аналіз крові хворого на цукровий діабет показав наявність молочної кислоти у концентрації 2,5 ммоль/л. Яка кома розвинулася у хворого?

Гіпоглікемічна

Гіперглікемічна

Гіперкетонемічна

Лактацидемічна

Гіперосмолярна

3462 / 6307
Підвищення внутрішньочерепного тиску у хворого з церебральною гематомою обумовило надмірну активність блукаючого нерва (ваготонію) та зміну частоти серцевих скорочень. Який вид аритмії серця виникає при цьому?

Синусова тахікардія

Синусова брадикардія

Передсердно-шлуночкова блокада

Пароксизмальна тахікардія

Шлуночкова екстрасистолія

3463 / 6307
В ході обстеження людини необхідно визначити, яка частка альвеолярного повітря оновлюється під час кожного вдиху. Який з наведених показників необхідно розрахувати для цього?

Життєва ємність легень

Коефіцієнт легеневої вентиляції

Хвилинний об’єм дихання

Функціональна залишкова ємність легень

Хвилинна альвеолярна вентиляція

3464 / 6307
У людини необхідно оцінити стан клапанів серця. Яким з інструментальних методів дослідження доцільно скористатися для цього?

Електрокардіографія

Сфігмографія

Зондування судин

Флебографія

Фонокардіографія

3465 / 6307
Швидкою допомогою в приймальне відділення доставлений хворий з кривавим блюванням. В анамнезі цироз печінки. Пошкодження яких вен найбільш ймовірне вданому випадку?

Стравохідні

Верхні брижові

Нижні брижові

Селезінкові

Печінкові

3466 / 6307
Ліквідатору наслідків аварії на Чорнобильській АЕС, що отримав велику дозу опромінення, проведено трансплантацію кісткового мозку. Через деякий час після проведеної операції у пацієнта діагностовано розвиток реакції 'трансплантат проти хазяїна'. Які антигени стали пусковим механізмом виникнення цієї реакції?

Антигени HBs, HBc, Hbe

Антигени системи HLA-клітин організму донора

Антигени системи Rh еритроцитів ліквідатора

Антигени системи HLA-клітин організму ліквідатора

Антигенами системи AB0 еритроцитів ліквідатора

3467 / 6307
В експерименті ізольований м’яз жаби ритмічно подразнюють електричними імпульсами. Кожний наступний імпульс припадає на період розслаблення попереднього скорочення. Яке скорочення виникає?

Зубчастий тетанус

Тонічне

Одиночне

Суцільний тетанус

Асинхронне

3468 / 6307
Під час розтину тіла мертвонародженої дитини виявлено аномалію розвитку серця: шлуночки не розмежовані, з правої частини виходить суцільний артеріальний стовбур. Для яких хребетних характерна подібна будова серця?

Птахи

Ссавці

Амфібії

Рептилії

Риби

3469 / 6307
Хворому на сепсис призначили антимікробний препарат з групи фторхінолонів. Оберіть його серед наведених препаратів:

Ципрофлоксацин

Ампіцилін

Цефалексин

Цефпіром

Метронідазол

3470 / 6307
Під дією медіатора на постсинаптичну мембрану нервової клітини розвинулася гіперполяризація. Збільшення проникності мембрани для яких іонів може викликати такі зміни?

Кальцію

Магнію

Натрію і калію

Калію

Натрію

3471 / 6307
Щоденно в організмі людини 0,5% всього гемоглобіну перетворюється на метгемоглобін. Який фермент, що міститься в еритроцитах, каталізує відновлення метгемоглобіну до гемоглобіну?

Метгемоглобінтрансфераза

Білівердинредуктаза

Ілюкуронілтрансфераза

Гемоксигеназа

Метгемоглобінредуктаза

3472 / 6307
У хворого спостерігається гемералопія (куряча сліпота). Яка з перерахованих речовин матиме лікувальну дію?

Карнітин

Карнозин

Кератин

Креатин

Каротин

3473 / 6307
У хворого з серцевою недостатністю виникла аритмія у вигляді генерації позачергових імпульсів в пучку Гіса. Порушення якої функції серцевого м’язу спостерігається в даному випадку?

Автоматизм

Скоротливість

Збудливість та провідність

Збудливість

Провідність

3474 / 6307
Хворий 50-ти років страждає на гіпертонічну хворобу. Під час фізичного навантаження у нього з’явилося відчуття слабкості, нестачі повітря, синюшність слизової оболонки губ, шкіри обличчя. Дихання супроводжувалося чутними на відстані вологими хрипами. Який механізм лежить в основі виникнення такого синдрому?

Колапс

Хронічна правошлуночкова недостатність

Хронічна лівошлуночкова недостатність

Гостра лівошлуночкова недостатність

Тампонада серця

3475 / 6307
На слизовій оболонці правого піднебінного мигдалика спостерігається безболісна виразка з гладеньким лакованим дном та рівними краями хрящеподібної консистенції. Мікроскопічно: запальний інфільтрат, що складається з лімфоцитів, плазмоцитів, невеликої кількості нейтрофілів та епітеліоїдних клітин, наявність ендо- та периваскуліту. Про яке захворювання йдеться?

Сифіліс

Актиномікоз

Туберкульоз

Скарлатина

Дифтерія зіву

3476 / 6307
У юнака 20-ти років діагностовано спадковий дефіцит УДФ-глюкуронілтрансферази. Підвищення якого показника крові підтверджує цей діагноз?

Тваринний індикан

Стеркобіліноген

Уробілін

Непрямий (некон’югований) білірубін

Прямий (кон’югований) білірубін

3477 / 6307
Хворому 50-ти років з хронічною серцевою недостатністю і тахіаритмією призначили кардіотонічний препарат. Який з препаратів призначили хворому?

Добутамін

Аміодарон

Дигоксин

Дофамін

Мілдронат

3478 / 6307
Офтальмолог з діагностичною метою (розширення зіниць для огляду очного дна) використав 1% розчин мезатону. Мідріаз, викликаний препаратом, обумовлений:

Активацією α1-адренорецепторів

Блокадою α1-адренорецепторів

Активацією М-холінорецепторів

Активацією β1-адренорецепторів

Активацією α2-адренорецепторів

3479 / 6307
Чоловік 55-ти років, що скаржиться на біль в ділянці нирок, надійшов в лікарню. В ході ультразвукового обстеження пацієнта виявлено наявність ниркових каменів. Наявність в сечі якої з наведених речовин є найбільш імовірною причиною утворення каменів уданого пацієнта?

Сечова кислота

Білірубін

Білівердин

Креатинін

Уробілін

3480 / 6307
В ході визначення енерговитрат організму людини встановлено, що дихальний коефіцієнт дорівнює 1,0. Це означає, що у клітинах досліджуваного переважно окиснюються:

Вуглеводи

Жири

Білки

Білки і вуглеводи

Вуглеводи та жири

3481 / 6307
Чоловік 43-х років доставлений у лікарню з ознаками черевного тифу. Хворий нещодавно був у місцевості, епідемічній за даним захворюванням, де пив некип’ячену воду з колодязя. З моменту появи перших симптомів пройшло 3 дні. У цей період збудник може бути виявлений у:

Крові

Жовчному міхурі

Тонкій кишці

Лімфоїдній тканині кишківника

Шлунку

3482 / 6307
У хворого запалення присереднього надвиростка плечової кістки (епікондиліт). Який нерв залучений у процес?

N. axillaris

N. ulnaris

N. musculocutaneus

N. medianus

N. radialis

3483 / 6307
У холодну погоду з вітром люди мерзнуть швидше, ніж за відсутності вітру. Причиною цього є те, що вітер збільшує, насамперед, віддачу тепла таким шляхом:

Теплопроведення

Конвекція

Випаровування

Радіація

3484 / 6307
Хворий помер від інтоксикації на 4-у добу після вживання сирих яєць. На розтині: слизова оболонка шлунка і тонкої кишки запалена, вкрита слизовим ексудатом; в легенях, головному мозку і печінці знайдені абсцеси. Який діагноз найбільш імовірний?

Черевний тиф

Сальмонельоз (септична форма)

Сальмонельоз (інтестинальна форма)

Сальмонельоз (черевнотифозна форма)

Дизентерія

3485 / 6307
Під час визначення групової належності крові за системою АВ0 аглютинацію еритроцитів досліджуваної крові викликали стандартні сироватки I та II груп та не викликала сироватка III групи. Якою є група крові?

А (II) бета

В (III) альфа

Неможливо визначити

0 (I) альфа, бета

АВ (IV)

3486 / 6307
На мікропрепараті серця розрізняються клітини, які розташовані у вигляді світлих тяжів, мають невелику кількість міофібрил, включення глікогену та ексцентрично локалізоване ядро. Які це клітини?

Волокна Пуркіньє

Скоротливі

Провідні пейсмекерні

Ендокринні

Провідні перехідні

3487 / 6307
Чоловік 40-ка років скаржиться на загальну слабкість, головний біль, кашель з виділенням мокротиння, задишку. Після клінічного огляду й обстеження поставлено діагноз: пневмонія. Який тип гіпоксії має місце у хворого?

Тканинна

Респіраторна

Ішемічна

Циркуляторна

Гіпоксична

3488 / 6307
До хірурга звернувся чоловік 60-ти років, що тривалий час хворів на цукровий діабет. Тканини правої стопи були чорного кольору, щільні, з чіткими краями. Який діагноз поставив хірург?

Волога гангрена

Іазова гангрена

Суха гангрена

Пролежень

Трофічна виразка

3489 / 6307
У жінки 52-х років артеріальна гіпертензія ускладнилася правобічною геміплегією і втратою мови. Яка зона головного мозку є імовірно найбільш ураженою?

Ліва передня звивина і ліва скронева частка

Права передня звивина

Ліва скронева частка

Ліва передня звивина

Потилична частка

3490 / 6307
Хворий 62-х років блідий, всі групи лімфовузлів збільшені. В крові: Hb- 60 г/л, еритроцити - 1,9 Т/л, лейкоцити - 29 Г/л, тромбоцити - 110 Г/л. Лейкоцитарна формула: сегментоядерні лейкоцити - 10%, лімфоцити - 8%, моноцити - 2%, бластних клітин - 80%. Цитохімічні дослідження бластних клітин: позитивна реакція на глікоген, негативна – на ліпіди і пероксидазу. Дайте заключення про патологію:

Гострий промієлоцитарний лейкоз

Гострий мегакаріоцитарний лейкоз

Гострий мієлобластний лейкоз

Гострий монобластний лейкоз

Гострий лімфобластний лейкоз

3491 / 6307
До лікарні надійшов пацієнт з перфоративною виразкою задньої стінки шлунка. Який елемент очеревини хірург під час операції повинен ретельно обстежити?

Лівий бічний канал

Передшлункова сумка

Печінкова сумка

Права брижова пазуха

Чепцева сумка

3492 / 6307
У померлого від інтоксикації чоловіка під час розтину тіла виявлено збільшення стегна в об’ємі; на поверхні шкіри нориці з виділенням в’язкої жовто-зеленої рідини; стегнова кістка потовщена та деформована, має місце утворення секвестрів. При мікроскопічному дослідженні: секвестральні порожнини оточені грануляційною та сполучною тканиною з наявністю нейтрофілів, кістково-мозкові канали облітеровані, компактна пластинка потовщена. Який з перерахованих діагнозів найбільш імовірний?

Хронічний гнійний остеомієліт

Гострий гематогенний остеомієліт

Паратиреоїдна остеодистрофія

Остеопороз

Туберкульозний остеомієліт

3493 / 6307
Хворому на хронічний бронхіт призначили муколітичний препарат, що підвищує синтез сурфактанту. Який препарат був використаний?

Фенотерол

Амброксол

Лібексин

Кодеїн

Мукалтин

3494 / 6307
Внаслідок травми низу передньої стінки живота у хворої ушкоджена зв’язка, що розташована у пахвинному каналі. Яка це зв’язка?

Ligamentum inguinale

Ligamentum latum uteri

Ligamentum lacunare

Ligamentum teres uteri

Ligamentum ovarium proprium

3495 / 6307
Після перенесеного ГРЗ у хворої спостерігається утруднене носове дихання, підвищена температура, головний біль, сльозовиділення, болючість під час пальпації в ділянці fossa canina справа. Запаленням якої пазухи ускладнилося захворювання?

Cellulae ethmoidales posteriores

Sinus sphenoidalis dexter

Sinus maxillaris dexter

Cellulae ethmoidales anteriores

Sinus frontalis dexter

3496 / 6307
Під час профілактичного огляду у деяких робітників, що працюють на виробництві кам’яновугільних смол, у ротовій порожнині виявлені ділянки стовщення і зроговіння слизової оболонки, переважно щік, білястого кольору, з шорсткою поверхнею, безболісні. Про яку патологію йдеться?

Стоматит

Глосит

Лейкоплакія

Папіломатоз

3497 / 6307
У хворого на рентгенограмі легень виявлено затемнення. В ході діагностичної експрес-біопсії лімфатичного вузла бронха виявлено казеозний некроз, навколо якого розташовані епітеліоїдні та лімфоїдні пласти з домішками багатоядерних гігантських клітин. Вкажіть причину лімфаденіту:

Туберкульоз

Сифіліс

Метастази раку

Аденовірусна інфекція

Пневмонія

3498 / 6307
На розтині тіла померлого в підкоркових ядрах правої півкулі головного мозку спостерігається порожнина неправильної форми 5х3,5 см, заповнена червоними згустками крові та розм’якшеною тканиною мозку. Назвіть патологію, яка розвинулася у головному мозку:

Астроцитома

Геморагічне просякнення

Абсцес

Ішемічний інфаркт

Гематома

3499 / 6307
У хворого діагностовано пухлину мозку, яка розміщена в ділянці 'пташиної шпори. Порушення якої функції розвинеться у хворого, якщо пухлина буде активно розвиватися?

Дотикова чутливість

Нюх

Смак

Зір

Слух

3500 / 6307
Хворому 35-ти років для обстеження очного дна був призначений атропіну сульфат у вигляді очних крапель. Для відновлення акомодації йому закрапали пілокарпіну гідрохлорид, але це не дало бажаного ефекту. Що є причиною відсутності ефекту?

Звикання

Синергізм

Односторонній антагонізм

Двосторонній антагонізм

Тахіфілаксія

3501 / 6307
Хвора 38-ми років померла під час нападу бронхіальної астми, який не вдалося зняти. В ході гістологічного дослідження у просвіті бронхів виявлені накопичення слизу, в стінці бронхів численні лаброцити, багато з них у стані дегрануляції, а також велика кількість еозинофілів. Який патогенез (механізм розвитку) цих змін у бронхах?

Гранулематоз

Атопія

Цитотоксична, цитолітична дія антитіл

Імунокомплексний механізм

Клітинно обумовлений цитоліз

3502 / 6307
У хворого 23-х років після перенесеної ангіни розвинувся сечовий синдром (гематурія, протеїнурія, лейкоцитурія). В пункційній біопсії нирок виявлена картина інтракапілярного проліферативного гломерулонефриту, а електронномікроскопічно виявлені великі субепітеліальні депозити. Який патогенез цього захворювання?

Цитотоксична, цитолітична дія антитіл

Гранулематоз

Атопія

Клітинно обумовлений цитоліз

Імунокомплексний механізм

3503 / 6307
У хворого, який на тлі атеросклерозу переніс ішемічний інсульт, спостерігається порушення рухової функції у вигляді геміплегії. Яка з перерахованих ознак є характерною для уражених кінцівок при даній патології?

Гіпотонус м’язів

Трофічні розлади

Гіпорефлексія

Гіпертонус м’язів

3504 / 6307
Через три тижні після пересадки серця у пацієнта різко погіршився стан. Смерть настала від гострої серцевої недостатності. Гістологічно в міокарді виявлено васкуліт, некрози та лімфоцитарну інфільтрацію. Назвіть причину таких змін:

Гостре відторгнення трансплантату

Хронічне відторгнення трансплантату

Інфаркт міокарда

Найгостріше відторгнення трансплантату

Некротизуючий васкуліт

3505 / 6307
У хворого з флегмоною передпліччя в ході мікробіологічного аналізу ексудату в зоні запалення виявлена присутність стрептококів. Які клітини будуть переважати в ексудаті?

Лімфоцити

Еозинофільні гранулоцити

Нейтрофільні гранулоцити

Моноцити

Базофільні гранулоцити

3506 / 6307
Важливим джерелом утворення аміаку в головному мозку є дезамінування АМФ. Яка амінокислота відіграє основну роль у зв’язуванні аміаку в нервовій тканині?

Глутамат

Аланін

Аргінін

Ізолейцин

Лізин

3507 / 6307
У хворої 19-ти років з дитинства спостерігалося зниження гемоглобіну до 9095 г/л. Аналіз крові під час госпіталізації: еритроцити - 3,2 • 1012/л, гемоглобін - 85 г/л, КП- 0,78;лейкоцити - 5,6 • 109/л, тромбоцити - 210 • 109/л. В мазку: анізоцитоз, пойкілоцитоз, мішенеподібні еритроцити. Ретикулоцити - 6%. Лікування препаратами заліза не ефективне. Яку патологію системи крові можна припустити в даному випадку?

Ферментопатія

Серпоподібноклітинна анемія

Мембранопатія

Таласемія

Фавізм

3508 / 6307
Жінці 50-ти років, яка хворіє на тромбофлебіт, ввели гепарин, що спровокував кишкову кровотечу. Який препарат потрібно призначити?

Стрептокіназу

Натрію цитрат

Вікасол

Протаміну сульфат

Кислоту амінокапронову

3509 / 6307
До хірургічного відділення ЦРЛ надійшов хворий з колотою раною стопи, яку він отримав під час косовиці. Який специфічний препарат необхідно застосувати з метою екстреної пасивної імунопрофілактики правця?

Антибіотики

Антитоксична сироватка

Протиправцева вакцина

Вакцина АКДП

Анатоксин

3510 / 6307
Під час серцевого нападу чоловік в автобусі втратив свідомість, з’явилися судоми. Лікар швидкої допомоги виявив на ЕКГ, що частота скорочення передсердь перевищує частоту скорочення шлуночків. Що може бути причиною даного стану?

Порушення автоматії СА-вузла

Порушення проведення збудження між передсердями

Виникнення гетеротропних вогнищ збудження

Повна поперечна блокада проведення збудження

Порушення автоматії АВ-вузла

3511 / 6307
На розтині тіла померлого, який страждав на гіпертонічну хворобу, у лівій півкулі мозку виявлена порожнина округлої форми 4х5 см з іржавою стінкою, заповнена жовтуватою прозорою рідиною. Назвіть патологію, яка розвинулася у головному мозку хворого:

Ішемічний інфаркт

Абсцес

Геморагічне просякнення

Кіста

Гематома

3512 / 6307
Під час постсинтетичного періоду мітотичного циклу було порушено синтез білків тубулінів. До яких наслідків це може призвести?

Скорочення тривалості мітозу

Порушення репарації ДНК

Порушення формування веретена поділу

Порушення спіралізації хромосом

Порушення цитокінезу

3513 / 6307
У хворого 40-ка років, доставленого в травматологічне відділення, виявлено поранення правої кисті на долонній поверхні. Під час обстеження травмований не може самостійно звести розведені II, IV та V пальці. Які м’язи пошкоджені?

Короткий долонний

Червоподібні

Долонні міжкісткові

Довгий долонний

Тильні міжкісткові

3514 / 6307
На аутопсії тіла жінки, що хворіла на хронічну дизентерію, в ході гістологічного дослідження внутрішніх органів в стромі та паренхімі міокарда, нирок, в слизовій оболонці шлунка та в сполучній тканині легень виявлені аморфні відкладення фіолетового кольору, що дають позитивну реакцію за Коссом. Яке ускладнення розвинулося у хворої?

Дистрофічне звапніння

Гіаліноз

Амілоїдоз

Метаболічне звапніння

Метастатичне звапніння

3515 / 6307
У хворого з патологією серцево-судинної системи розвинулися набряки на нижніх кінцівках. Який механізм розвитку серцевого набряку?

Підвищення гідростатичного тиску в венулах

Підвищення онкотичного тиску плазми крові

Порушення лімфовідтоку

Підвищення гідростатичного тиску в артеріолах

Зниження осмотичного тиску плазми крові

3516 / 6307
Під час проведення хірургічних маніпуляцій було використано новокаїн з метою знеболення. Через 10 хвилин у хворого з’явилися блідість шкірних покривів, задишка, гіпотензія. Яку алергічну реакцію можна припустити?

Анафілактична

Імунокомплексна

Клітинно-опосередкована

Цитотоксична

Стимулююча

3517 / 6307
У 19-місячної дитини із затримкою розвитку та проявами самоагресії, вміст сечової кислоти в крові - 1,96 ммоль/л. При якому метаболічному порушенні це спостерігається?

Подагра

Синдром набутого імунодефіциту

Хвороба Гірке

Хвороба Іценко-Кушинга

Синдром Леша-Ніхана

3518 / 6307
Дитина 9-ти місяців харчується сумішами, незбалансованими за вмістом вітаміну В6. У дитини спостерігається пелагроподібний дерматит, судоми, анемія. Розвиток судом може бути пов’язаний з дефіцитом утворення:

Гістаміну

Дофаміну

Серотоніну

ДОФА

ГАМК

3519 / 6307
У хлопчика 12-ти років видалено апендикс, який надіслано патологу на дослідження. Макроскопічно: апендикс в дистальному відділі з булавоподібним потовщенням діаметром 3 см, при розрізі якого вилилася прозора жовтувата рідина, стінка апендикса стоншена. Мікроскопічно: атрофія всіх шарів апендикса, ознак запалення немає. Який найбільш імовірний діагноз?

Емпієма апендикса

Флегмонозний апендицит

Хронічний апендицит

Міксоглобульоз апендикса

Водянка апендикса

3520 / 6307
У дитини 7-ми років, яка неодноразово хворіла на стрептококову ангіну, лікар припускає розвиток ревматизму. Призначено серологічне дослідження. Наявність антитіл до якого з стрептококових антигенів найімовірніше підтвердить передбачуваний діагноз?

С-вуглевод

М-білок

Еритрогенний токсин

Капсульний полісахарид

О-стрептолізин

3521 / 6307
У хворого з попереднім діагнозом 'озена' з носоглотки були виділені грам-негативні палички, які утворювали капсулу на поживному середовищі. Які мікроорганізми спричинили хворобу?

Сальмонели

Шигели

Хламідії

Клебсієли

Мікоплазми

3522 / 6307
У зв’язку з крововтратою пацієнту введено 1 л розчину хлориду натрію з концентрацією 150 ммоль/л. Внаслідок цього, насамперед, зменшиться:

Осмотичний тиск внутрішньоклітинний

Онкотичний тиск крові

Онкотичний тиск міжклітинної рідини

Осмотичний тиск міжклітинної рідини

Осмотичний тиск крові

3523 / 6307
У хворого кровотеча з поперечної ободової кишки. Чим кровопостачається цей відділ товстої кишки?

Arteria sigmoidea

Arteria ileocolica

Arteria mesenterica inferior

Arteria colica media

Arteria lienalis

3524 / 6307
Чоловік протягом 3-х років працював в одній з африканських країн. Після переїзду до України звернувся до офтальмолога зі скаргами на біль в очах, набряки повік, сльозоточивість і тимчасове послаблення зору. Під кон’юнктивою ока були виявлені гельмінти розмірами 30-50 мм, які мали видовжене ниткоподібне тіло. Який діагноз може поставити лікар?

Філяріоз

Трихоцефальоз

Дифілоботріоз

Ентеробіоз

Аскаридоз

3525 / 6307
Хворому призначена ендоскопія 12-палої кишки. В результаті виявлено запалення великого дуоденального сосочка і порушення виділення жовчі в просвіт кишки. У якому відділі 12-палої кишки виявлені порушення?

Висхідна частина

Верхня частина

Низхідна частина

Горизонтальна частина

Цибулина

3526 / 6307
Чоловік з гострим міокардитом помер від серцево-судинної недостатності. В ході мікроскопічного дослідження внутрішніх органів виявлені: плазморагія, набряк, стази в капілярах, численні крововиливи, а також дистрофічні зміни в паренхімі. Наслідком чого є дані зміни?

Хронічний загальний венозний застій

Гострий загальний венозний застій

Загальне артеріальне повнокрів’я

Місцеве артеріальне повнокрів’я

ДВС-синдром

3527 / 6307
Хворому на гепатит для попередження уражень печінки призначили вітаміноподібну речовину холін. Його лікувальний ефект пов’язаний з:

Активацією глікогенсинтази

Пригніченням синтезу холестерину

Пригніченням синтезу ацетонових тіл

Активацією глікогенфосфорилази

Ліпотропною дією

3528 / 6307
Дослідженнями останніх десятиліть встановлено, що безпосередніми 'виконавцями' апоптозу в клітині є особливі ферменти - каспази. В утворенні одного з них бере участь цитохром С. Вкажіть його функцію в нормальній клітині:

Компонент піруватдегідрогеназної системи

Компонент H + - АТФ-азної системи

Фермент в-окиснення жирних кислот

Фермент дихального ланцюга переносу електронів

Фермент ЦТК

3529 / 6307
Процес біосинтезу білка є енергозалежним. Вкажіть, який макроергічний субстрат безпосередньо використовується в цьому процесі на стадії елонгації:

ГТФ

УТФ

ЦТФ

АДФ

АТФ

3530 / 6307
З метою профілактики гепатиту В групі стоматологів ввели вакцину, яка являє собою генно-інженерний HBs-антиген. Від якого ще інфекційного агента захищає таке щеплення?

Вірус гепатиту С

Вірус імунодефіциту людини

Вірус Коксакі, група В

Вірус грипу, тип В

Вірус дельта

3531 / 6307
У хворого з ознаками інтоксикації та ниркової недостатності виявлені у сечі рухливі мікроорганізми з численними дрібними завитками, які забарвилися за Романовським-Гімзою у рожевий колір. З анамнезу відомо, що хворий кілька днів тому купався у відкритій водоймі. Яке захворювання можна припустити?

Псевдотуберкульоз

Сифіліс

Туберкульоз

Лептоспіроз

Бруцельоз

3532 / 6307
У жінки під час мейозу відбулося порушення розходження аутосом. Утворилася яйцеклітина з зайвою 18-ю хромосомою. Яйцеклітина запліднюється нормальним сперматозооном. У майбутньої дитини буде синдром:

Патау

Клайнфельтера

Едвардса

Шерешевського-Тернера

Дауна

3533 / 6307
Для виявлення джерела інфікування хворих у хірургічному відділенні патогенним стафілококом було проведено дослідження з застосуванням стандартних стафілококових бактеріофагів. Яке дослідження чистих культур, виділених із різних джерел, було проведено?

Фагоіндикація

Фагоідентифікація

Фагодіагностика

Фаготипування

Фаготерапія

3534 / 6307
У новонародженої дитини спостерігається зниження інтенсивності смоктання, часте блювання, гіпотонія. У сечі та крові значно підвищена концентрація цитруліну. Який метаболічний процес порушений?

Глюконеогенез

Гліколіз

ЦТК

Цикл Корі

Орнітиновий цикл

3535 / 6307
Після споживання солоної їжі у людини значно зменшилася кількість сечі. Підвищена секреція якого гормону призвела до зменшення діурезу?

Ренін

Натрійуретичний

Вазопресин

Альдостерон

Ангіотензин-II

3536 / 6307
У результаті радіаційного випромінювання були ушкоджені стовбурові гемопоетичні клітини. Утворення яких клітин сполучної тканини буде порушено?

Меланоцити

Макрофаги

Адипоцити

Фібробласти

Перицити

3537 / 6307
Через 8 днів після хірургічної операції у пацієнта розвинувся правець. Лікар припускає, що причиною став контамінований збудником правця шовний матеріал, який був доставлений в бактеріологічну лабораторію. Яке поживне середовище необхідно використовувати для первинного посіву шовного матеріалу?

ЖСА

Ендо

Сабуро

Кітта-Тароцці

Гіса

3538 / 6307
У пацієнта має місце пошкодження волокон дев’ятої пари черепних нервів (язикоглотковий нерв). Формування якого відчуття буде порушено?

Усі смакові відчуття

Кисле

Солоне

Гірке

Солодке

3539 / 6307
Чоловік 70-ти років хворіє на атеросклероз судин нижніх кінцівок та ішемічну хворобу серця. Під час обстеження виявлено порушення ліпідного складу крові. Надлишок яких ліпопротеїнів є головною ланкою в патогенезі атеросклерозу?

Холестерину

Високої щільності

Хіломікронів

Низької щільності

Проміжної щільності

3540 / 6307
У хворого на есенціальну артеріальну гіпертензію розвинувся гіпертонічний криз, що призвело до нападу серцевої астми. Який механізм серцевої недостатності є провідним вданому випадку?

Абсолютна коронарна недостатність

Перевантаження серця збільшеним об’ємом крові

Пошкодження міокарда

Перевантаження серця підвищеним опором

Порушення надходження крові до серця

3541 / 6307
Стресовий стан і больове відчуття у пацієнта перед візитом до стоматолога супроводжуються анурією (відсутністю сечовиділення). Це явище зумовлене збільшенням:

Секреції вазопресину та зменшенням адреналіну

Активності антиноціцептивної системи

Секреції вазопресину та адреналіну

Активності парасимпатичної нервової системи

Секреції адреналіну та зменшенням вазопресину

3542 / 6307
У хворого під час прийому їжі виникла асфіксія внаслідок закупорки трахеї стороннім тілом. Яка форма порушення зовнішнього дихання спостерігається у хворого?

Первинно дискінетична

Обструктивна

Дифузно-пневмонозна

Вентиляційно-рестриктивна

Дифузно-рестриктивна

3543 / 6307
Під час дослідження епітелію шкіри з’ясувалося, що він складається з кількох шарів клітин. Епітеліоцити зовнішнього шару не мають ядер. Який це епітелій?

Багатошаровий кубічний

Перехідний

Багаторядний війчастий

Багатошаровий плаский зроговілий

Багатошаровий плаский незроговілий

3544 / 6307
Введення пацієнту знеболювального перед екстракцією зуба призвело до розвитку анафілактичного шоку, який супроводжувався розвитком олігурії. Який патогенетичний механізм зумовив зменшення діурезу в даній клінічній ситуації?

Підвищення гідростатичного тиску в капсулі Шумлянського-Боумена

Збільшення онкотичного тиску крові

Пошкодження клубочкового фільтру

Зменшення кількості функціонуючих нефронів

Зниження гідростатичного тиску в капілярах клубочків

3545 / 6307
Після видалення зуба у пацієнта виникла кровотеча. Аналіз крові виявив зниження протромбінового індексу. Дефіцит якого вітаміну може бути причиною такого стану?

C

K

D

A

B

3546 / 6307
У пацієнтів для оцінки ефективності дихання використовують показник функціональної залишкової ємності. З яких наступних об’ємів вона складається?

Резервний об’єм вдиху та залишковий

Резервний об’єм вдиху та дихальний

Резервний об’єм видиху та дихальний

Резервний об’єм видиху та залишковий

Резервний об’єм вдиху, дихальний, залишковий

3547 / 6307
В хронічному експерименті на щурах стимулювали електричним струмом паравентрикулярні та супраоптичні ядра гіпоталамуса. Яка поведінкова реакція спостерігалася у тварин?

Відмова від їжі та рідини

Збільшення споживання їжі

Зменшення споживання води

Зменшення споживання їжі

Збільшення споживання води

3548 / 6307
Після обстеження пацієнта в клініці нервових хвороб встановлена відсутність звуження зіниці під дією світла. З ураженням яких структур головного мозку це пов’язано?

Вегетативні ядра 3 пари черепно-мозкових нервів

Червоні ядра середнього мозку

Ретикулярні ядра довгастого мозку

Ретикулярні ядра середнього мозку

Ядра гіпоталамуса

3549 / 6307
Аспірин інгібує синтез простагландинів, завдяки блокуванню активності циклооксигенази. Яка жирна кислота необхідна для цього синтезу?

Арахідонова

Стеаринова

Лінолева

Ліноленова

Пальмітинова

3550 / 6307
Лікар встановив у хворого наявність ураження парасимпатичного ядра IX пари черепних нервів. Яке ядро уражене в даному випадку?

Nucleus tractus solitarii

Nucleus thoracicus

Nucleus ambiguus

Nucleus salivatorius inferior

Nucleus salivatorius superior

3551 / 6307
Хворому лікар призначив протикашльовий препарат центральної дії, який є алкалоїдом мачку жовтого. Діє на кашльовий центр вибірково, не пригнічує дихання, не затримує виділення харкотиння. Не викликає обстипації і лікарської залежності. Можна призначати дітям. Визначте цей препарат:

Бромгексин

Глауцину гідрохлорид

Кодеїну фосфат

Лібексин

Окселадин

3552 / 6307
У хворого після резекції шлунка з приводу виразкової хвороби перистальтика кишківника не відновилася. Який лікарський засіб доцільно призначити хворому для відновлення моторики?

Атенолол

Метилурацил

Резерпін

Прозерин

Гігроній

3553 / 6307
Коли людина проходить повз їдальню та чує дзвін посуду, у неї виділяється слина. Реалізація якого рефлексу зумовлює цю реакцію?

Умовний природний

Умовний штучний

Умовний інструментальний

Умовний орієнтовний

Безумовний орієнтовний

3554 / 6307
До медико-генетичної консультації звернувся юнак з приводу відхилень у фізичному і статевому розвитку. В ході мікроскопії клітин слизової оболонки рота виявлені тільця Барра. Вкажіть найбільш імовірний каріотип юнака:

47, 21+

47,18+

47, ХХY

45, ХО

47, ХYY

3555 / 6307
У хворого з яскраво вираженою жовтушністю шкіри, склер та слизових оболонок, сеча має колір темного пива, кал світлий. У крові підвищений вміст прямого білірубіну, в сечі визначається білірубін. Який тип жовтяниці у хворого?

Обтураційна

Паренхіматозна

Екскреційна

Гемолітична

Кон’югаційна

3556 / 6307
У реанімаційне відділення надійшов хворий з гострим інфарктом міокарда, якому для зменшення болю ввели морфіну гідрохлорид. Механізм дії морфіну гідрохлориду обумовлений:

Стимуляцією аденілатциклази

Блокадою гістамінових рецепторів

Стимуляцією опіатних рецепторів

Блокадою фосфодіестерази

Блокадою холінестерази

3557 / 6307
Хворому у ЛОР-відділенні проведено біопсію слизової оболонки носа. Гістологічно виявлено продуктивне запалення з формуванням гранульом, у складі яких переважають лімфоцити і плазмоцити, спостерігаються еозинофільні гіаліноподібні кулі та великі макрофаги із світлою цитоплазмою (клітини Мікуліча). Який збудник міг викликати описані зміни?

Респіраторно-синцитіальний вірус

Золотистий стафілокок

Мікобактерія лепри

Бліда трепонема

Паличка Волковича-Фріша

3558 / 6307
У працівників хімічних комбінатів, де виробляють органічні розчинники, які здатні розчиняти фосфоліпіди, часто розвиваються захворювання легень. Який компонент аерогематичного бар’єру при цьому пошкоджується в першу чергу?

Респіраторні альвеолоцити

Секреторні альвеолоцити

Альвеолярні макрофаги

Септальні клітини

Сурфактант

3559 / 6307
У людини після довільної тривалої затримки дихання збільшилися частота й глибина дихання. Які зміни в крові, насамперед, стали причиною цього?

Підвищення рО2

Підвищення рСО2

Зниження рО2

Зниження рСО2

Підвищення рН

3560 / 6307
У відділення надійшла дитина з носовою кровотечею та меленою в калі. Зі слів матері, відбулося отруєння дитини кумаринами, які застосовувалися для боротьби з щурами. Введення якого засобу припинить кровотечу у дитини?

Адреналін

Вікасол

Фраксипарин

Фепранон

Тромбін

3561 / 6307
Внаслідок дефіциту УФО-ендонуклеази порушується репарація ДНК і виникає таке захворювання:

Альбінізм

Пігментна ксеродермія

Фенілкетонурія

Серпоподібноклітинна анемія

Подагра

3562 / 6307
В експериментальної тварини зроблено двобічну перерізку блукаючих нервів. Що відбудеться з диханням?

Стане частим і поверхневим

Стане рідким і глибоким

Зупиниться в фазі видиху

Не зміниться

Зупиниться в фазі вдиху

3563 / 6307
Жінка 69-ти років довго хворіла на атеросклероз. Надійшла до хірургічного відділення з симптомами гострого живота. В ході лапаротомії виявлені: тромбоз мезентеріальної артерії, петлі тонкої кишки набряклі, багряно-чорного кольору, на їх серозній оболонці фібринозні нашарування. Який патологічний процес розвинувся у кишці хворої?

Секвестр

Ішемічний інфаркт

Суха гангрена

Волога гангрена

Коагуляційний некроз

3564 / 6307
До лікарні потрапив футболіст з ушкодженням поверхневого пахвинного кільця та розривом двох ніжок, що його обмежують. Похідним якої анатомічної структури вони є?

Власна фасція живота

Апоневроз зовнішнього косого м’яза живота

Апоневроз поперечного м’яза живота

Апоневроз внутрішнього косого м’яза живота

Міжніжкові волокна

3565 / 6307
В ході копрологічного дослідження у працівників кав’ярні лікарями санітарно- епідеміологічної станції були виявлені округлі цисти, характерною ознакою яких є наявність чотирьох ядер. Імовірніше за все у цих працівників безсимптомно паразитує:

Кишкова трихомонада

Балантидій

Дизентерійна амеба

Лямблія

Амеба кишкова

3566 / 6307
При деяких гельмінтозах людина може сама виявити гельмінта, оскільки зрілі членики збудника можуть активно виповзати з ануса людини. Це характерно для:

Ехінококозу

Теніозу

Гіменолепідозу

Дифілоботріозу

Теніаринхозу

3567 / 6307
Хворий на хронічну серцеву недостатність протягом декількох місяців приймав дигітоксин; в процесі дигіталізації з’явилися такі симптоми: головний біль, нудота, діарея, втрата апетиту, порушення кольорового сприйняття, брадикардія. Який антидот доцільно призначити для зменшення симптомів інтоксикації?

Атропіну сульфат

Унітіол

Преднізолон

Адреналіну гідрохлорид

Налоксон

3568 / 6307
У сироватці крові новонародженого виявлено антитіла до вірусу кору. Про наявність якого імунітету це свідчить?

Спадковий, видовий

Штучний пасивний

Штучний активний

Природний пасивний

Природний активний

3569 / 6307
Під час реплікації ДНК один із її ланцюгів синтезується із запізненням. Що визначає дану особливість синтезу?

Великі розмірами ДНК-полімерази

Відсутність трифосфонуклеотидів

Антипаралельність ланцюгів

Необхідність репарації

Компліментарність ланцюгів

3570 / 6307
Фермент оксидаза D-амінокислот каталізує дезамінування тільки D-амінокислот. Яка властивість ферментів виявляється при цьому?

Відносна специфічність

Стереохімічна специфічність

Абсолютна специфічність

Залежність від рН

Термолабільність

3571 / 6307
У хворого хронічний нежить. Набряк слизової оболонки носової порожнини призводить до порушення функції рецепторів нюхового нерва, які розташовані в нюховій ділянці носової порожнини. Через яке утворення волокна цього нерва потрапляють до передньої черепної ямки?

Lamina cribrosa os ethmoidale

Foramen ethmoidale posterior

Foramen ethmoidale anterior

Foramen incisivum

Foramen sphenopalatinum

3572 / 6307
На гістологічному препараті в складі видовженої структури, обмеженої плазмолемою, по периферії розташовані численні ядра, а в цитоплазмі наявна поперечна посмугованість. Яка це структура?

Міосимпласт

Колагенове волокно

Гладенький міоцит

Кардіоміоцит

Синцитіотрофобласт

3573 / 6307
A woman complains of раіп іп the right іlіас region. Palpation detects there a soft, movie, and painful intestine. What intestine is being palpated?

Rectum

Sigmoid colon

Jejunum

Ascendmg colon

Cecum

3574 / 6307
An experiment has demonstrated that after exposure to ultrav radiation the dermal cells of the patients with xeroderma pigmentosum are slower to restore the native DNA structure than they are in the healthy individuals due to deficiency of the DNA repair enzyme. What enzyme takes part in the repair process?

DNA polymerase III holoenzyme

DNA gyrase

RNA ligase

Endonuclease

Primase

3575 / 6307
Blood serum analysis of the patient with acute hepatitis shows increased levels of alanine aminotransferase (ALT) and aspartate aminotransferase (AST). What changes on the cellular level can result in such developments?

Disturbed energy supply to the cells

Disturbed intercellular interactions

Cell destruction

Damage to the genetic apparatus of the cells

Disturbed cellular enzyme systems

3576 / 6307
The disease onset occurred 3 days ago. The patient complains of body temperature up to 38°C, stomachache, and frequent loose bloody stools. Bacillary dysentery was clinically diagnosed in the patient. What method of microbiological diagnostics would be advisable in this case and what samples should be obtained from the patient to confirm this diagnosis?

Serology, blood

Bacterioscopy, blood

Bacterioscopy, feces

Bacteriology, urine

Bacteriology, feces

3577 / 6307
In the morning a man diagnosed with diabetes mellitus received a prescribed dose of long- acting insulin on an empty stomach. He missed his regular meal and soon after that he developed weakness, headache, and vertigo, body tremors, convulsions, feeling of hunger, and signs of hypoglycemia. Glucose administration did not improve the patient’s condition. What medicine should be administered to provide quick relief to the patient?

Hydrocortisone

Triamcinolone

Noradrenaline

Prednisolone

Adrenaline

3578 / 6307
A man complaining of memory deterioration, reduced mental capabilities, sleep disorders, and vertigo was brought to the neurologic department. The patient explains these symptoms as consequences of the brain concussion received in a traffic accident 2 years ago. Choose the drug that can improve the patient’s brain metabolism and would be the most advisable in this case:

Piracetam

Caffeine

Sodium oxybutirate

Cordiamin (Nikethamide)

Sydnocarb (Mesocarb)

3579 / 6307
Autopsy of the body shows that the soft meninges of the deceased individual are plethoric, thickened, opaque, and yellow-green colored. What type of exudative inflammation can be characterized by such changes in the soft meninges?

Serous

Hemorrhagic

Catarrhal

Fibrinous

Suppurative

3580 / 6307
Mass mortality of rodents was observed in one of the mountain villages. Simultaneously there occurred a disease outbreak in the local population. The disease manifested by rapidly progressive fever up to 40°C, marked intoxication, and enlargement of inguinal lymph nodes. Smear preparations made from autopsy specimens contained gram-negative ovoid bacilli with bipolar staining. What microorganism is the causative agent of this disease?

Francisella tularensis

Bacillus anthracis

Clostridia

Yersinia pestis

Staphylococcus

3581 / 6307
A 36-year-old woman suffers from a connective tissue disease (collagenosis). What metabolite is the most likely to be increased in her urine?

Urobilinogen

Creatinine

Urea

Indican

Oxyproline

3582 / 6307
A 34-year-old woman was diagnosed with hereditary spherocytosis (hereditary microspherocytic hemolytic anemia, Minkowski-Chauffard syndrome). What mechanism leads to erythrocyte hemolysis in the patient?

Membranopathy

Bone marrow hypoplasia

Enzymopathy

Autoimmune disorder

Hemoglobinopathy

3583 / 6307
A patient has an incised wound on the posterior surface of his thigh and is unable to flex his lower leg. What muscles were damaged?

Biceps, gracilis, adductor

Biceps, adductor, gracilis

Semitendinosus, adductor, gracilis

Semitendinosus, semimembranosus, biceps

Semimembranosus, semitendinosus, gracilis

3584 / 6307
After intensive training session an athlete presents with significant loss off vascular tone in the working muscles. These changes are caused by accumulation of the following in the vessels:

Serotonin

Metabolites

Renin-angiotensin

Natriuretic hormone

Histamine

3585 / 6307
The cell was exposed to mutagenic factor which resulted in DNA molecule losing 2 nucleotide pairs. What type of mutation occurred in the DNA?

Translocation

Duplication

Replication

Deletion

Inversion

3586 / 6307
Permeability of the excitable cell membrane to potassium ions was increased in an experiment. What changes in the membrane potential will occur?

No changes

Depolarization

Hyperpolarization

Action potential

Local response

3587 / 6307
An unconscious patient was brought into the hospital. The smell of acetone can be detected from the patient’s mouth. Blood glucose - 25 mmol/L, ketone bodies -0.57 mmol/L. What hormone deficiency can result in the development of this condition?

Aldosterone

Somatotropin

Thyroxin

Insulin

Glucocorticoids

3588 / 6307
A patient has developed systemic (megaloblastic) anemia despite eating a balanced diet. The day before he underwent a gastric surgical resection. The anemia in this patient is caused by the deficiency of:

Folic acid

Vitamin PP

Vitamin C

Protein

Castle factor

3589 / 6307
A patient was hospitalized with diagnosis of an intestinal carcinoid. Laboratory analysis detects increased synthesis of serotonin from tryptophan. This process is based on the following biological mechanism:

Deaminization

Transamination

Microsomal oxidation

Decarboxylation

Formation of paired compounds

3590 / 6307
A patient with ciliary arrhythmia was prescribed digoxin. What mechanism of action of this medicine results in its antiarrhythmic effect?

Decreased permeability of the cell membrane to calcium

Decreased sympathetic influences

Inhibition of Na, K-ATPase

Increased potassium concentration in the cardiomyocytes

Decreased permeability of the cell membrane to sodium

3591 / 6307
Thoracic duct rupture has occurred in a weightlifter, when he was lifting the bar. The injury to the thoracic duct is the most likely to be localized in the area of:

Aortic hiatus

Posterior mediastinum

Venous angle junction

Neck

Lumbosacral junction

3592 / 6307
A man was brought to the hospital in a severe condition: facial edemas, myalgia, high temperature, and respiratory distress. History-taking revealed that the patient’s family regularly consumes untested pork. What helminth can be the cause of such symptoms?

Trichinella spiralis

Diphyllobothrium latum

Strongyloides stercoralis

Ancylostoma duodenale

Taeniarynchus saginatus

3593 / 6307
Blood sample poured into a test tube has clotted within 6 minutes. The test tube was then put into a thermostat and in 24 hours the blood clot was destroyed due to activation of:

Plasmins

Kallikreins

Antithrombins

Heparin

Kinins

3594 / 6307
Autopsy of the body of a 48-year-old man shows that the bone marrow in the flat bones, as well as in the cylindrical bone diaphyses and epiphyses, is moist, colored gray-red or gray-yellow, and puriform (pyoid bone marrow). The spleen weight is 7 kg; it is dark red on section, with signs of ischemic infarctions. All the lymph nodes are enlarged, soft, and gray-red in color. In the liver there are signs of fatty degeneration and leukemic infiltrates. What is the most likely diagnosis?

Acute lymphoid leukemia

Lymphogranulomatosis

Multiple myeloma

Acute myeloid leukemia

Chronic myeloid leukemia

3595 / 6307
A 52-year-old woman suffering from breast cancer had undergone a course of radiation therapy. As a result the tumor diminished in size. What mechanism of cell damage ensures effectiveness of radiation therapy?

Formation of free radicals

NK cell-induced lysis

Vascular thrombosis

Mutagenesis

Hyperthermia

3596 / 6307
A person has memorized a phone number for a short period of time (a few seconds). After making a call the person was unable to reproduce this sequence of numbers. In this case the process of memorizing was based on the following type of memory:

Medium-term memory (episodic buffer)

Long-term memory

Short-term memory

Secondary and tertiary memory

Iconic memory

3597 / 6307
A 27-year-old woman suffering from rheumatic heart disease since her childhood has developed cardiac decompensation and died. Autopsy shows mitral stenosis; mitral valve cusps are sharply thickened, sclerotic, and fused together along the closure line. Specify the type of endocarditis in this case:

Acute verrucous endocarditis

Ulcero-polypoid endocarditis

Recurrent verrucous endocarditis

Diffuse endocarditis

Fibroplastic endocarditis

3598 / 6307
A patient with ischemic heart disease was prescribed a calcium channel blocker. Name this drug:

Altiopril

Nitroglycerine

Carvedilol

Thiotriazolin

Amlodipine

3599 / 6307
Analysis detects glucose and amino acids in the primary urine. In the residual urine they are absent due to tubular reabsorption of these substances. Where in the nephron does this process occur?

Proximal convoluted tubule

Henle’s loop

Macula densa

Distal convoluted tubule

Collecting duct

3600 / 6307
Histologic specimen of an ovary shows a follicle in its cortical substance. The follicle consists of a primary oocyte, transparent membrane, and one layer of prismatic cells. Specify the type of the follicle:

Primordial

Ovulating

Tertiary

Secondary

Primary

3601 / 6307
При деяких гельмінтозах людина може сама виявити гельмінта, оскільки зрілі членики збудника можуть активно виповзати з ануса людини. Це характерно для такого захворювання:

Ехінококоз

Дифілоботріоз

Теніоз

Гіменолепідоз

Теніарінхоз

3602 / 6307
У хворого з невритом стегнового нерва порушено згинання стегна та розгинання гомілки у колінному суглобі. Функція якого м'яза при цьому порушена?

Двоголовий м’яз стегна

Півсухожилковий м'яз

Триголовий м'яз стегна

Напівперетинчастий м'яз

Чотирьохголовий м'яз стегна

3603 / 6307
У жінки 62-х років розвинулася катаракта (помутніння кришталика) на фоні цукрового діабету. Який тип модифікації білків має місце при діабетичній катаракті?

Обмежений протеоліз

Фосфорилювзння

Глікозилювання

Метилювання

АДФ-рибозилюзання

3604 / 6307
У людини внаслідок травми мозку сталося вимкнення грудного дихання зі збереженням діафрагмального. У разі якої локалізації травми це може відбутися?

На рівні 1-го шийного сегмента спинного мозку

Між шийними та грудними сегментами спинного мозку

На рівні 8-го ірудного сегмента спинного мозку

На рівні варолієвого моста

На рівні ретикулярної формації стовбура

3605 / 6307
У полі зору цистоскопа гладка поверхня слизової оболонки без складок. Яка частина сечового міхура у полі зору?

Шийка.

Дно

Міхуровий трикутник

Верхівка.

Тіло.

3606 / 6307
Хворому на туберкульоз призначено антибіотик олігоміцин. Назвіть процес, який інгібірує цей препарат при розмноженні туберкульозної палички.

Трансамінування.

Транскрипція.

Окислювальне фосфорилювання.

Трансляція.

Реплікація.

3607 / 6307
У хворого, який на тлі атеросклерозу переніс ішемічний інсульт, спостерігається порушення рухової функції у вигляді геміплегії. Яка з перелічених ознак є характерною при даній патології для уражених кінцівок?

Трофічні розлади

Відсутність трофічних розладів

Гіпорефлексія

Гіпертонус м'язів

Гіпотонус м'язів

3608 / 6307
У хворого з ІХС виникли порушення серцевого ритму, збільшився рівень глюкози у крові. Поруч з антиангінальними засобами, лікар призначив вітамінний препарат. Який з вітамінних засобів має кардіотрофічний та гіпоглікемічний ефект?

Ціанокобаламін

Ретинол

Рибофлавін

Ергокальциферол

Тіамін

3609 / 6307
Під час експерименту, внаслідок уведення тварині синтетичного аналога тиреоїдних гормонів, збільшилася частота серцевих скорочень, що опосередковано:

Парасимпатичною нервовою системою

Соматичною нервовою системою

Симпатичною нервовою системою

Метасимпатичною нервовою системою

3610 / 6307
При мікроскопічному дослідженні біоптату шкіри виявляються гранульоми, які складаються з епітеліоїдних клітин, оточених в основному Т-лімфоцитами. Серед епітеліоїдних клітин розташовуються поодинокі гігантські багатоядерні клітини типу Пирогова-Лангханса. В центрі деяких гранульом виявляються ділянки казеозного некрозу. Кровоносні судини відсутні. Для якого захворювання характерні зазначені зміни?

Сифіліс

Риносклерома

Лепра

Туберкульоз

Сап

3611 / 6307
Хворий на хронічний пієлонефрит помер від хронічної ниркової недостатності. При житті аускультативно відмічено "шум тертя перикарду? На розтині виявлено. що епікард тьмяний, шорсткий, ніби покритий волосяним покривом. Який перикардит за характером запалення має місце?

Гнильний

Фібринозний

Гнійний

Серозний

Катаральний

3612 / 6307
Реалізація загального адаптаційного синдрому здійснюється переважно через нейроендокринну систему. Якій з ланок цієї системи належить провідна роль у патогенезі реакції, що розвішається?

Гіпофізарно-адреногенітальна

Гіпофізарно-юкстагломерулярна

Гіпофізарно-інсулярна

Гіпофізарно-тиреоїдна

Гіпофізарно-адреналова

3613 / 6307
При операції у хлопчика 12-ти років видалений апендикс, який надіслано патологу на дослідження. Макроскопічно: апендикс в дистальному відділі з булавоподібним стовщенням діаметром 3 см, при розрізі якого вилилася прозора жовтувата рідина, стінка апендикса стоншена. Мікроскопічно: атрофія всіх шарів апендикса, ознак запалення немає. Який найбільш імовірний діагноз?

Емпієма апендикса

Флегмонозний апендицит

Водянка апендикса

Хронічний апендицит

Міксоглобульоз апендикса

3614 / 6307
При гістологічному дослідженні шийного лімфатичного вузла було виявлено повнокрів'я та набухання коркового шару, в мозковій речовині наявна велика кількість плазматичних клітин, зменшення кількості лімфоцитів, значна макрофагальна реакція. Назвіть характер змін в лімфатичному вузлі:

Гострий лімфаденіт

Лімфогранулематоз

Антигенна стимуляція лімфоїдної тканини

Лімфома

3615 / 6307
Під час розтину тіла дівчики 9 років у верхівці правої легені субплеврально було знайдене вогнище казеозного некрозу діаметром 15 мм, біфуркаційні лімфатичні вузли були збільшені містили дрібні вогнища некрозу коагуляційного типу. Мікроскопічно - у легеневому вогнищі та в лімфатичних вузлах навколо некротичних мас були розташовані епітеліоїдні клітини, лімфоцити та поодинокі багатоядерні гігантські клітини. Діагностуйте захворювання.

Вторинний вогнищевий туберкульоз

Первинний туберкульоз

Гематогенний генералізований туберкульоз

Гематогенний туберкульоз з переважним ураженням легень

Вторинний фіброзно-вогнищевий туберкульоз

3616 / 6307
У хворого з яскраво вираженою жовтяничністю шкіри, склер, слизових оболонок сеча має колір темного пива, кал світлий. У крові підвищений вміст прямого білірубіну, в сечі визначається білірубін. Який тип жовтяниці у хворого?

Кон'югаційна

Обтураційна

Екскреційна

Паренхіматозна

Гемолітична

3617 / 6307
При макро-мікроскопічному дослідженні ділянки великогомілкової кістки та м’яких тканин довкола виявлено дифузне гнійне запалення; що захоплює кістковий мозок, гаверсові канали та періост осередки некрозу Про яке захворювання слід думати?

Хвороба Педжета

Остеонекроз

Хронічний гематогений остеомієліт

Гострий гематогений остеомієліт

Паратиреоїдна остеодистрофія

3618 / 6307
У хворого 65-ти років під час неврологічного обстеження виявлено крововилив у межах верхньої скроневої звивини. У зоні кровопостачання якої артерії воно знаходиться?

Задня мозкова артерія

Передня сполучна артерія

Передня мозкова артерія

Основна артерія

Середня мозкова артерія

3619 / 6307
При анемії в периферичній крові визначаються дегенеративні і регенеративні форми еритроцитів. Назвіть регенеративні форми еритроцитів.

Гіперхромні еритроцити

Сфероцити

Мікроцити

Ретикулоцити

Пойкіяоцити

3620 / 6307
Внаслідок дефіциту УФО-ендонуклеази порушується репарація ДНК і виникає таке захворювання:

Пігментна ксеродермїя

Альбінізм

Подагра

Серпоподібноклітинна анемія

Фенілкетонурія

3621 / 6307
В лікуванні системного захворювання сполучної тканини (склеродермія) необхідно використовувати десенсибілізуючу, протизапальну та імунодепресивну дії ліків. Якій групі засобів притаманні всі ці ефекти?

Адреноблокатори

Анаболічні стероїди

Адреноміметичні засоби

Стероїдні протизапальні

Антигістамінні засоби

3622 / 6307
У відділення реанімації поступив хворий після ДТП з однобічним пневмотораксом. Який вид дихання спостерігається у даному випадку?

Поверхневе часте

Глибоке часте

Поверхневе

Асфіктичне

Поверхневе рідке

3623 / 6307
При патології нирок в сечі з'являються патологічні складові частини. Поява яких патологічних складових частин сечі свідчить про підвищення проникності клубочкової мембрани?

Піурія

Алкаптонурія

Протеїнурія

Аміноацидурія

Глюкозурія

3624 / 6307
На розтині померлого в підкоркових ядрах правої півкулі головного мозку визначається порожнина неправильної форми 5 х 3,5 см, заповнена червоними згортками крові і розм’якшеною тканиною мозку. Назвіть патологію, яка розвинулась у головного мозку?

Ішемічний інфаркт.

Геморагічне просякнення.

Гематома.

Абсцес.

Кіста.

3625 / 6307
У хворої при гастродуоденоскопії виявили множинні виразки шлунка. Діагностовано синдром Золлінгера-Еллісона. Надлишок якого гормону має місце в даному випадку?

Інсулін

Окситоцин

Кортизол

Тироксин

Гастрин

3626 / 6307
Дівчина хвора на цукровий діабет, чекає на донорську нирку. Яке ускладнення діабету в неї є причиною хронічної ниркової недостатності?

Нейропатія

Ретинопатія

Атеросклероз

Макроангіопатія

Мікроангіопатія

3627 / 6307
На розтині тіла чоловіка, яким номер від черевного тифу, в клубовій кишці виявлені дефекти, розташовані по всій довжині кишки, краї їх рівні, дно утворене м’язовим шаром. Яка зі стадій черевного тифу діагностована?

Мозкоподібного набухання

Некрозу

Утворення виразок

Чистих виразок

Загоєння

3628 / 6307
При гастритах, як правило, ушкоджуються залози слизової оболонки шлунку. За рахунок яких клітин можлива їх регенерація?

Головні

Шийкові мукоцити

Ендокринні

Парієтальні

Додаткові

3629 / 6307
На секції тіла жінки 76-ти років були знайдені ознаки хронічного бронхіту та легеневого серця, печінка збільшена в розмірах, щільна, тканина на розрізі строката. Мікроскопічно: просвіти центральних вен та синусоїдних капілярів розширені. повнокровні, а гепатоцити на периферії часточок з жировою дистрофією. Який вид розладу кровообігу викликав зміни печінки?

Загальна венозна гіперемія

Місцева венозна гіперемія

Загальна артеріальна гіперемія

Загальне малокрів’я

Місцева артеріальна гіперемія

3630 / 6307
Через 6 годин після інфаркту міокарда у хворого в крові піднялася активність лактатдегідрогенази. Наявність якого ізоферменту слід чекати у цьому випадку?

ЛДГ2

ЛДГ4

ЛДГ1

ЛДГЗ

ЛДГ3

3631 / 6307
У жінки народилася мертва дитина з множинними вадами розвитку. Яке протозойне захворювання могло спричинити внутрішньоутробну загибель плоду? 1

Токсоплазмоз

Трихомоніаз

Амебіаз

Балантидіаз

Лямбліоз

3632 / 6307
Відповідно до сучасної концепції атерогенезу «Response to injury», атеросклероз є проявом хронічного запалення в інтимі артерій. З якою стадією запалення пов’язано формування фіброзних бляшок при атеросклерозі:

Проліферація

Вторинна альтерація

Первинна альтерація

Ексудація

Трансформація

3633 / 6307
В експерименті на ізольованій збудливій клітині необхідно отримати збільшення мембранного потенціалу спокою (гіперполяризацію). Для цього доцільно викликати активацію таких іонних каналів:

Натрієві

Натрієві та кальцієві

Кальцієві

Калієві та натрієві

Калієві

3634 / 6307
У пацієнтки на поверхні шкіри лівого кута лопатки знайдено утвір з наступними характеристиками: 5-7 мм діаметром, кулястої форми, твердої консистенції, який має широку основу та сосочкову поверхню. Гістологічно: клітини плоского епітелію. що нерівномірно розростаються, кількість шарів збільшена, надлишкова кількість кровоносних судин. Вкажіть патологічний процес:

Аденокарцинома

«Рак на місці»

Папілома

Аденома

Гідроаденома

3635 / 6307
І.М. Сєченов встановив, що втомлена кінцівка відновлює працездатність швидше, якщо в період відпочинку друга кінцівка працює. Це дало можливість розробити вчення про:

Втому

Активний відпочинок

Песимум

Оптимум

Парабіоз

3636 / 6307
Жінка 40-ка років звернулася із скаргами на неможливість розгинати стопу і пальці, що створює труднощі при ходьбі. Об’єктивно: ступня звисає, дещо повернена всередину, пальці зігнуті ('кінська стопа'), чутливість втрачена на зовнішній поверхні гомілки і тильній поверхні стопи. Який нерв уражений?

Стегновий

Сідничний

Великогомілковий

Спільний малогомілковий

Підшкірний

3637 / 6307
Хворий звернувся до лікаря з пораненням проксимальної фаланги пальця, яка ускладнилася флегмоною долоні. Гній заповнив спільну синовіальну піхву для згиначів, в якій лежать сухожилки поверхневого та глибокого згиначів пальців. На якому пальці було ушкодження проксимальної фаланги?

I

V

IV

ІІІ

II

3638 / 6307
При обстеженні лікарем хворої людини на основі клінічних даних був поставлений діагноз парагрип. Яким із перелічених способів може передаватись це захворювання?

Фекально-оральний

Повітряно-крапельний

Трансмісивний

Парентеральний

Через укус тварин

3639 / 6307
Жінка 35-ти років розпочала голодувати. Депо яких поживних речовин використовується у початковий період голодування і як при цьому змінюється дихальний коефіцієнт (ДК)?

Жири, ДК наближається до 0,72

Білки, ДК наближається до 0,7

Жири, ДК наближається до 0,85

Білки, ДК наближається до 1

Вуглеводи, ДК наближається до 1

3640 / 6307
У пацієнтки 26-ти років висипання на шкірі, свербіж після вживання цитрусових. Призначте лікарський засіб з блокаторів Н1-гістамінорецепторів:

Менадіону натрію біосульфат

Парацетамол

Кислота ацетилсаліцилова

Метамізол

Дифенгідрамін

3641 / 6307
Хворому при безсонні, викликаному емоційними розладами, лікар призначив засіб, що викликає сон за рахунок транквілізуючої дії. Який снодійний препарат був призначений пацієнту?

Бромізовал

Етамінал-натрій

Нітразепам

Хлоралгідрат

Фенобарбітал

3642 / 6307
Біологічне окислення та знешкодження ксенобіотиків відбувається за рахунок гемвмісних ферментів. Який метал є обов'язковою складовою цих ферментів?

Со

Мn

Zn

Мg

Fe

3643 / 6307
На гістологічному препараті представлений зріз судини, що характеризується правильною круглою формою; зіяє, стінка складається з 3-х оболонок, де у середній спостерігається наявність 40-50 вікончастих еластичних мембран. Яка судина представлена на препараті?

Кровоносний капіляр

Вена м’язового типу

Артерія еластичного типу

Артерія змішаного типу

Артерія м'язового типу

3644 / 6307
У жінки при профілактичному огляді виявили пухлину молочної залози. Результати біопсії підтвердили наявність злоякісної пухлини. Який основний механізм інфільтративного росту злоякісної пухлини?

Порушення контактного гальмування

Підвищення активності кейлонів в клітині

Збільшення адгезивності пухлинних клітин

Здатність пухлинних клітин до амебоїдного руху

Підвищення ферментативної активності лізосом

3645 / 6307
При обстеженні хворого 70-ти років виявлено цукровий діабет II типу. Яким препарат доцільно призначити хворому?

Глібенкламід

Паратиреоїдин

Інсулін

Мерказоліл

Кортизон

3646 / 6307
Людина стоїть у кімнаті в легкому одязі, температура повітря +14°С, вікна і двері зачинені. Яким шляхом вона віддає найбільше тепла?

Перспірація

Конвекція

Теплорадіація

Теплопроведення

Випаровування

3647 / 6307
Чоловіка 49 років доставили з місця автомобільної аварії в лікарню в непритомному стані. Шкірні покриви бліді, пульс частий і поверхневий. Переломів кісток і пошкодження головного мозку не виявлено. При пункції черевної порожнини отримано значну кількість крові. Первинною причиною тяжкого стану потерпілого є

Еритропенія

Гіповолемія

Гіпоінсулінемія

Гіпопротеїнемія

Гіпокатріємія

3648 / 6307
Під час бігу на довгі дистанції скелетна мускулатура тренованої людини використовує глюкозу з метою отримання енергії АТФ для м'язового скорочення. Вкажіть основний процес утилізації глюкозиза цих умов;

Глікогенез

Глікогеноліз

Аеробне окиснення

Глюконеогенез

Анаеробний гліколіз

3649 / 6307
Центральну роль в обміні амінокислот у нервовій тканині відіграє глутамінова кислота. Це пов'язано з тим, що дана амінокислота:

Використовується для синтезу глюкози

Використовується для синтезу ацетонових тіл

Використовується для синтезу ліків

Використовується для синтезу нейроспецифічних білків

Зв’язує аміак з утворенням глутаміну

3650 / 6307
Після споживання солоної їжі в людини значно зменшилася кількість сечі. Який з указаних гормонів уплинув на функцію?

Окситоцин

Соматостатин

Адреналін

Антидіуретичний

АКТГ

3651 / 6307
Внаслідок руйнування певних структур стовбуру мозку тварина втратила орієнтувальні рефлекси. Які структури було зруйновано?

Чотиригорбкова структура

Чорна речовина

Вестибулярні ядра

Червоні ядра

Медіальні ядра ретикулярної формації

3652 / 6307
При дії окислювачів (перекис водню, оксиди азоту та інші), гемоглобін, до складу якого входить Fе2+, перетворюється на сполуку, що містить FеЗ+. Ця сполука не здатна переносити кисень і має назву?

Карбоксигемоглобін

Глікозильований гемоглобін

Оксигемоглобін

Метгемоглобін

Карбгемоглобін

3653 / 6307
Під час постсинтетичного періоду мітотичного циклу було порушено синтез білків тубулінів. До яких наслідків це може призвести?

Порушення репарації ДНК

Порушення формування веретена поділу

Скорочення тривалості мітозу

Порушення цитокінезу

Порушення спіралізації хромосом

3654 / 6307
Хворому із загальним набряковим синдромом на тлі серцевої недостатності призначено фуросемід. Вплив на який процес забезпечує його терапевтичний ефект в цьому випадку?

Синтез натрієвих каналів в дистальних канальцях

Активний транспорт іонів крізь базальну мембрану

Карбоангідразу

Нирковий кровотік

Транспорт іонів крізь апікальну мембрану

3655 / 6307
Під дією УФ-опромінення та інших факторів можуть відбуватися зміни в структурі ДНК. Репарація молекули ДНК досягається узгодженою дією всіх наступних ферментів, ЗА ВИНЯТКОМ:

ДНК-глікозидаза

Аміноацил-тРНК-синтетаза

ДНК-лігаза

Ендонуклеаза

ДНК-полімераза

3656 / 6307
При мікроскопічному дослідженні нирок померлої від ниркової недостатності жінки 36-ти років, в клубочках виявлено проліферацію нефротелію капсули, подоцитів та макрофагів з утворенням "півмісяців", некроз капілярних петель, фібринові тромби в їх просвітах, а також склероз та гіаліноз клубочків, атрофію канальців та фіброз строми нирок. Який з перелічених діагнозів найбільш вірогідний?

Підгострий гломерулонефрит

Мембранозна нефропатія

Хронічний гломерулонефрит

Гострий гломерулонефрит

Фокальний сегментарний склероз

3657 / 6307
У клітинах мозку лисиці, яка була спіймана в межі міста, виявлені включення у вигляді тілець Бабеша-Негрі. Джерелом якого захворювання є ця тварина?

Інфекційний мононуклеоз

Кліщовий енцефаліт

Грип

Вітряна віспа

Сказ

3658 / 6307
Для підвищення результатів спортсмену рекомендували застосовувати препарат, який містить у собі карнітин. Який процес в найбільшому ступені активується карнітином?

Синтез ліпідів

Синтез кетонових тіл

Тканинне дихання

Транспорт жирних кислот у мітохондрії

Синтез стероїдних гормонів

3659 / 6307
У хворого з хронічним гіперацидним гастритом з'явився біль у суглобах. Для полегшення болю, враховуючи супутню патологію, був призначений целекоксиб. Вибіркова дія цього препарату на певний фермент забезпечує відсутність впливу на слизову шлунка. Назвіть цей фермент:

Калікреїн

Циклооксигеназа 1

Фосфоліпаза А2

Фосфоліназа С

Циклооксигеназа 2

3660 / 6307
На щурах моделювали опіки. Збільшилось утворення гістаміну, серотоніну, кінінів. оксиду азоту. Який тип причинно-наслідкових відносин у патогенезі єднає ці зміни з розвитком артеріальної гіперемії та підвищення проникності судин?

Circulus vitiosus

Дивергенція

'Пряма лінія'

Конвергенція

Саногенез

3661 / 6307
До лікарні поступив пацієнт з перфоративною виразкою задньої стінки шлунка. Який елемент очеревини під час операції хірург повинен ретельно обстежити?

Праву брижозу пазуху

Печінкову сумку

Чепцеву сумку

Передшлункову сумку

Лівий бічний канал

3662 / 6307
Основний спосіб зняття гіпертонічного кризу – швидке і значне зниження судинного опору. Ін’єкційні препарати якої групи адреноблокаторів можуть бути використані для цього?

Симпатолітики

a-Адреноблокатори

Кардіоселективні В-адреноблокатори

Неселективні В-адреноблокатори

3663 / 6307
У хворого збільшений основний обмін, підвищена температура тіла, тахікардія у стані спокою. Причиною цього може бути підвищена функція :

Щитовидної залози.

Статевих залоз.

Нейрогіпофізу.

Кіркової речовини наднирників.

Підшлункової залози.

3664 / 6307
ГІід час гістологічного дослідження видаленого збільшеного шийного лімфатичного вузла встановлено, що структура його стерта, лімфоїдні фолікули відсутні. Картина його одноманітна та представлена великою кількістю лімфобластів, серед яких наявні клітини з патологічним поділом. Інші групи лімфатичних вузлів і кістковий мозок не змінені. Виявлені зміни найбільш характерні для:

Лімфогранулематозу

Хронічного лімфолейкозу

Саркоїдозу

Неспецифічного гіперпластичного лімаденіту

Лімфосаркоми

3665 / 6307
Хворий після прийому жирної їжі відчуває нудоту, млявість. З часом з явилися ознаки стеатореї. В крові, холестерин - 9,2 ммоль/л. Причиною такого стану є нестача:

Хіломікронів

Фосфоліпідів

Жовчних кислот

Тригліцеридів

Жирних кислот

3666 / 6307
Хворому поставлений діагноз: гострий пієлонефрит із запаленням стінок ниркових чашечок і мисок. Який епітелій зазнав пошкодження?

Багаторядний війчастим

Багатошаровий плоский зроговілий

Одношаровий призматичний

Багатошаровий кубічний

Перехідний

3667 / 6307
До отоларинголога звернувся хворий у якого під час огляду виявлено неповне змикання голосових зв’зок при фонації. Голосова щілина при цьому приймає форму овалу.
Функція якого м’яза гортані порушена у хворого?

М. aryttenoideus transversus

М. vocalis

М. cricoaryttenoideus posterior

М. thyroaryttenoideus

М. cricoaryttenoideus lateralis

3668 / 6307
У людини, яка обертається на каруселі виникли збільшення частоти серцевії скорочень, потовиділення, нудота. З подразненням яких рецепторів, перш за все це пов’язано?

Вестибулярні

Зорові

Слухові

Дотикові

Пропріоцептори

3669 / 6307
Для лікування захворювань, збудники яких виділяють екзотоксин, застосовують антитоксичні сироватки. Для лікування якого з перерахованих захворювань антитоксичну сироватку застосувати НЕМОЖЛИВО?

Дифтерія

Правець

Ботулізм

Туберкульоз

Газова гангрена

3670 / 6307
Під час підйому пішки на 5 поверх у людини підвищився артеріальний тиск. Причиною є збільшення:

В'язкості крові

Хвилинного об'єму крові

Об’єму циркулюючої крові

Вмісту іонів в плазмі крові

Кількості функціонуючих капілярів

3671 / 6307
У людини з масою 80 кг після тривалого фізичного навантаження об’єм циркулюючої крові зменшився, гематокрит - 50%, загальний білок крові - 80 г/л. Такі показники крові є наслідком, перш за все:

Збільшення змісту білків в плазмі

Збільшення онкотичного тиску плазми

Втрати води з потом

Збільшення діурезу

Збільшення кількості еритроцитів

3672 / 6307
Лікар-гематолог призначив пацієнту з кровотечею коагулянт, який діє шляхом підвищення синтезу протромбіну та інших факторів зсідання крові переважно в печінці і є синтетичним водорозчинним вітаміном. Який препарат призначив лікар?

Тромбін

Кальцію хлорид

Етамзилат

Менадіону натрію біосульфат

Гепарин

3673 / 6307
Людину 35-ти років вкусила бджола. На місці укусу комахи визначається набряк, гіперемія, підвищення температури. Назвіть ініціальний патогенетичний фактор запального набряку:

Підвищення кров'яного тиску в капілярах

Підвищення осмотичного тиску у вогнищі запалення

Зниження онкотичного тиску крові

Підвищення проникності мікросудин

Порушення лімфовідтоку

3674 / 6307
У підлітка 12-ти років, який хворіє на бронхіальну астму, розвинувся тяжкий напад астми: виражена експіраторна задишка. блідість шкірних покровів. Який вид порушення альвеолярної вентиляції має місце?

Торако-діафрагмальний

Рестриктивннй

Обструктивний

Центральний

Нервово-м’язовий

3675 / 6307
Хірург під час операції на щитоподібній залозі перев’язав верхню щитоподібну артерію. Гілку якої судини перев’язав лікар?

A. lingualis

А. carotis externa

А. carotis interna

A. pharyngеа ascendens

A. facialis

3676 / 6307
У хворого відсутній зір, але зіничний рефлекс реалізується нормально. Де може знаходитись зона пошкодження?

Зорова кора

Зоровий перехрест

Нижні горбики чотиригорбкового тіла

Соматосенсорна кора

Верхні горбки чотиригорбкового тіла

3677 / 6307
У чоловіка і його сина інтенсивно росте волосся по краю вушних раковин. Це явище спостерігалося також у батька дідуся за батьківською лінією. Який тип успадкування зумовлює це?

Аутосомно-рецесивний

Рецесивний, зчеплений з X-хромосомою

Домінантний, зчеплений з Х-хромосомою

Зчеплений з У-хромосомою

Аутосомно-домінантний

3678 / 6307
У жінки 42 років, яка перенесла операцію на нирці, після наркозу розвинулися явища рекураризації і припинилося дихання. Як міорелаксант був застосований дитилін. Яки засіб найбільш доцільно застосовувати для відновлення тонусу м'язів?

Галантоміну гідробромід

Кофеїн

Плазму крові

Прозерин

Стрихніну нітрат

3679 / 6307
До хірургічного відділення ЦРЛ надійшов хворий з колотою раною стопи, яку він отримав під час косовиці. Який специфічний препарат необхідно застосувати з метою екстреної пасивної імунопрофілактики правця?

Анатоксин

Протиправцева вакцина

Вакцина АКДП

Антибіотики

Антитоксична сироватка

3680 / 6307
У хворого на туберкульоз легень непереносимість аміноглікозидного антибіотика амікацину. Який протитуберкульозний антибіотик можна включити до складу комплексної терапії у даному випадку?

Амоксицилін

Бензилпеніцилін

Канзміцин

Рифампіцин

Стрептоміцин

3681 / 6307
У хворого пухлина черевної порожнини, що стискає нижню порожнисту вену. Який кава-кавальний анастомоз на передній стінці живота забезпечить відтік венозної крові?

Між верхніми і нижніми прямокишковими венами

Між непарною і напівнепарною венами

Між верхньою і нижньою надчеревними венами

Між пупковою і верхньою надчеревною венами

Між хребетними і пупковими венами

3682 / 6307
Хворий на сімейну гіперліпідемію, викликану дефіцитом рецепторів ЛПНЩ, вживав інгібітори бета-гідроксиметил-глутарил-КоА-редуктази. Цей препарат сприяє:

Підвищенню рівня триацилгліцеролів крові

Підвищенню рівня сквалену в клітинах

Підвищенню рівня клітинної ацилхолестеролацилтрансферази

Зниженню рівня холестеролу крові

Зниженню клітинного вмісту бета-ГМГ-КоА

3683 / 6307
Людина 28 років споживає надмірну кількість вуглеводів (600 г на добу), що перевищує ії енергетичні потреби. Який процес буде активуватися у даному випадку?

Глюконеогенез

Ліполіз

Гліколіз

Окислення жирних кислот

Ліпогенез

3684 / 6307
Під час розтину померлого 43-х років, що страждав на ІХС з розвитком інфаркту міокарда, патологоанатом виявив набряк легень. Які патологічні зміни могли зумовити набряк легень?

Ішемія малого кола

Гостре загальне малокрїв'я

Стаз крові

Гостра правошлуночкова недостатність

Гостра лівошлуночкова недостатність

3685 / 6307
У 50-х роках у Західній Європі від матерів, які приймали в якості снодійного талідоміл, народилося кілька тисяч дітей з відсутністю або недорозвиненням кінцівок, порушенням будови скелета, іншими вадами. Яка природа даної патології?

Триплоїдів

Генна мутація

Трисомія

Тератогенна дія

Моносомія

3686 / 6307
Для підтвердження діагнозу: гострий атрофічний кандидоз проведено мікроскопію мазків, виготовлених із шкрібку нальоту на слизовій оболонці щоки, який взятий від хворої жінки. Виявлено овальної форми мікроорганізми, що брунькуються. Який метод забарвлення використав бактеріолог для фарбування мазків із досліджуваного матеріалу?

Грама

Морозова

Романовського-Гімзе

Ожешка

Нейсера

3687 / 6307
В ході обстеження людини необхідно визначити, яка частка альвеолярного повітря оновлюється під час кожного вдиху. Який з наведених показників необхідно розрахувати для цього?

Хвилинний об'єм дихання

Хвилинна альвеолярна вентиляція

Життєва ємність легень

Функціональна залишкова ємність легень

Коефіцієнт легеневої вентиляції

3688 / 6307
У медико-генетичну консультацію звернувся чоловік з приводу безпліддя. В ядрах більшості клітин епітелію слизової оболонки щоки у нього виявлено одне тільце Барра. Про який синдром може йти мова?

Клайнфельтера

Шерешевського-Тернера

Дауна

Едвардса

Патау

3689 / 6307
На 8-й день після введення протиправцевої сироватки у пацієнта піднялась температура. він став скаржитися на біль у суглобах та свербіж шкіри. Який механізм цього ускладнення?

Антитілозалежна цитотоксичність

Клітинна цитотоксичність

Гіперчутливість уповільненого типу

Анафілаксія

Імунокомилексна гіперчутливість

3690 / 6307
Хворий 58-ми років надійшов у торакальне відділення з кровохарканням: при бронхоскопії виявлено звуження правого стовбурового бронха внаслідок розростання сірувато-білої тканини. В ході біопсії виявлена пухлина, що складається з дрібних клітин овальної й округлої форми з гіперхромними ядрами та незначною цитоплазмою, клітини справляють враження «голих ядер» Клітини пухлини ростуть пластами, тяжами. Гістологічна будова пухлини свідчить про те, що у хворого:

Перехідноклітинний рак

Пласкоклітинний незроговілий рак

Пласкоклітинний зроговілий рак

Базальноклітинний рак

Недиференційований рак

3691 / 6307
У хворого 40-ка років в результаті щелепно-лицьової травми порушилася функція під'язикової і підщелепної залоз зліва - залози почали декретувати невелику кількість густої слини. Функція якого нерва порушена?

Трійчастий

Язикоглотковий

Блукаючий

Лицьовий

3692 / 6307
Під час проведення морфологічного дослідження периферичної крові хворого було помічено, що у еритроцитів забарвлена лише периферична частина, а в центрі є незабарвлене прояснення. Кольоровий показник – 0,56. Яка анемія найбільш ймовірна у цього пацієнта?

В12-фолієводефіцитна

Апластична

Сидеробластна

Залізодефіцитна

Е. Гемолітична

3693 / 6307
Підвищення внутрішньочерепного тиску у хворого з церебральною гематомою обумовило надмірну активність блукаючого нерва (ваготонію) та зміну частоти серцевих скорочені. Який вид аритмії серця виникає при ньому?

Пароксизмальна тахікардія

Передсердно-шлуночкова блокада

Синусова тахікардія

Синусова брадикардія

Шлуночкова екстрасистолія

3694 / 6307
Хворий на хронічний алкоголізм на вулиці втратив свідомість. Було діагностовано гіпоглікемію внаслідок порушення процесу глюконеогенезу. Які з наступних пар ферментів є необхідними для цього процесу?

Фосфоенолпіруваткарбоксикіназа і глюкокіназа

Фруктозо-1.6-діфосфатаза і піруваткарбоксилаза

Глюкозо-6-фосфатаза і піруватдегідрогеназа

Глюкозо-6-фосфатаза та фосфофруктокіназа

Піруваткіназа і піруваткарбоксилаза

3695 / 6307
У студента через добу після іспиту в аналізі крові виявили лейкоцитоз без істотної зміни в лейкоцитарній формулі Який механізм найімовірніше зумовив розвиток виявленої пиши в периферичній крові?

Уповільнення еміграції лейкоцитів до тканин

Зменшення руйнування лейкоцитів

Перерозподіл лейкоцитів в організмі

Посилення лейкопоезу

3696 / 6307
Юнак 15-ти років після переохолодження був доставлений в лікарню зі скаргами на біль, озноб. При огляді «гусяча шкіра», блідість, температура поступово підвищується. Яка стадія гарячки спостерігається у хворого?

St. incrementi

St. fastigii

St. inflamenti

St. absorbi

St. decrementі

3697 / 6307
На розтині тіла чоловіка, який хворів на фіброзно-кавернозний туберкульоз і помер від ниркової недостатності, виявлені збільшені в розмірах нирки, щільні на дотик, сіруватого кольору, на розрізі мають сальний вигляд. Який діагноз можна припустити?

Нефросклероз

Хронічний пієлоневрит

Гломерулонефрит

Мієломна нирка

Амілоїдоз нирок

3698 / 6307
Знешкодження ксенобіотиків (лікарських засобів, епоксидів, ареноксидів, альдегідів, нітропохідних тощо) та ендогенних метаболітів (естрадіолу, простагландинів, лейкотрієнів) відбувається в печінці шляхом їх кон'югації з:

Гліцином

Фосфоаденозином

Б-Аденозилметіоніном

Аспарагіновою кислотою

Глутатіоном

3699 / 6307
У хворого після травми виникла необхідність введення протиправцевої сироватки, але проба на чутливість до сироватки виявилася позитивною. Специфічну гіпосенсибілізацію у хворого слід виконати за допомогою введення:

Фізіологічних доз глюкокортикоїдів

Мінімальних доз специфічного алергену

Наркотичних речовин, що знижують чутливість

Роздільної дози специфічного алергену

Лікувальних доз антигістамінних препаратів

3700 / 6307
У людини частота серцевих скорочень постійно утримується на рівні 40 разів за хвилину. Що є водієм ритму серця у неї?

Волокна Пуркін'є

Атріовентрикулярний вузол

Ніжки пучка Гіса

Пучок Гіса

Синоатріальний вузол

3701 / 6307
Внаслідок захворювання нирок у пацієнта відмічаються набряки. В аналізах сечі масивна протеїнурія. Який механізм є основним у виникненні набряків у такого пацієнта?

Зниження онкотичного тиску лімфи

Зниження онкотичного тиску плазми крові

Зниження фільтраційного тиску в нирках

Зниження онкотичного тиску тканин

Підвищення осмотичного тиску плазми крові

3702 / 6307
Бактеріологічний метод діагнотики був використаний для підтвердження діагнозу: газова гангрена у хворого. Які живільні середовища необхідно використовувати для культивуваня збудника в цьому випадку?

Вільсона-Блера, Кітта-Тароцці

ЖСА, кров’яний агар

Лужний агар

Ендо, Левіна, Плоскірєва

МПА, МПБ

3703 / 6307
Внаслідок дефіциту вітаміну D у дитини визначається симптом рахіту. Зниження активності якого ферменту крові спостерігається при цьому?

Холінестераза

Креатинкіназа

а-амілаза

Кисла фосфатаза

Лужна фосфатаза

3704 / 6307
Хворому на глаукому призначили пілокарпіну гідрохлорид в очних краплях. До якої фармакологічної групи належить цей препарат?

Міорелаксанти

а-адреноблокатори

Гангліоблокатори

М-холіноміметики

М -холіноблокатори

3705 / 6307
Робота шахтарів у забої часто спричинює антракоз. Який вид дихальної недостатності може розвинутися при цьому?

Рестриктивний

Торакальний

Обструктивний

Дисрегуляторний

Діафрагмальний

3706 / 6307
У хворого з синдромом Іценко-Кушинга спостерігаються стійка гіперглікемія та глюкозурія. Синтез та секреція якого гормону збільшені у цього хворого?

Тироксин

Глюкагон

Адреналін

Альдостерон

Кортизол

3707 / 6307
Хвора 38-ми років померла під час нападу бронхіальної астми, що не вдалося купірувати. Під час гістологічного дослідження в просвіті бронхів виявлені скупчення слизу, в стінці бронхів багато тучних клітин (лаброцитів), багато з них у стані дегрануляції, а також багато еозинофілів. Який патогенез цих змін у бронхах?

Імунокомплексний механізм

Гранулематоз

Атопія

Клітинно обумовлений цитоліз

Цитотоксична, цитолітична дія антитіл

3708 / 6307
У жінки під час пологів в зв'язку з крововтратою визначили групу крові, Реакція аглютинації еритроцитів відбулася зі стандартними сироватками груп О (I), А (II) і не відбулася зі стандартною сироваткою групи В (III). Досліджувана кров належить до групи

АВ (IV)

В (III)

О (І)

А (II)

3709 / 6307
В ділянці хромосоми гени розташовані в такій послідовності: АВСDЕFG. В результаті дії радіоактивного випромінювання відбулася перебудова, після чого ділянка хромосоми має наступний вигляд: АВDЕFG. Яка мутація відбулася?

Мутація

Інсерція

Делеція

Інверсія

Дуплікація

3710 / 6307
Для оцінки придатності води для пиття проведено бактеріологічне дослідження. Який показник характеризує кількість бактерій групи кишкових паличок, що знаходяться в 1 л води?

Мікробне число

Колі-індекс

Титр колі-фага

Перфрінгенс-титр

Колі-титр

3711 / 6307
У хворого з клінічними ознаками імунодефіциту проведено імунологічні дослідження. Виявлено значне зниження кількості клітин, що утворюють розетки з еритроцитами барана. Який висновок слід зробити на основі даних аналізу?

Недостатність клітин-ефекторів гуморального імунітету

Зниження рівня системи комплементу

Зниження рівня Т-лімфоцитів

Зниження рівня Б-лімфоцитів

Зниження рівня натуральних кїлерів (УХ-клітин)

3712 / 6307
На певному етапі онтогенезу людини між кровоносними системами матері і плоду встановлюється фізіологічний зв'язок. Цю функцію виконує провізорний орган:

Амніон

Алантоїс

Серозна оболонка

Жовтковий мішок

Плацента

3713 / 6307
Встановлено ураження вірусом ВІЛ Т-лімфоцитів. При цьому фермент вірусу зворотня траскриптаза (РНК-залежна ДНК-полімераза) каталізує синтез:

Вірусної і-РНК на матриці ДНК

ДНК на матриці вірусної і-РНК

і-РНК на матриці вірусного білка

Вірусної ДНК на матриці ДНК

ДНК на вірусній р-РНК

3714 / 6307
При дослідженні людини у вертикальній позі встановлено, що в альвеолах верхівок легень парціальний тиск кисню складає 140 мм рт. ст. Причиною цього є те, що у даних відділах легень:

Перфузія переважає над вентиляцією

Перфузія і вентиляція врівноважені

Вентиляція відсутня

Вентиляція переважає над перфузією

3715 / 6307
У гістопрепараті яєчника жінки визначаються структури, що мають велику порожнину. Овоцит І порядку в них оточений прозорою оболонкою, променистим вінцем і розташований у яйценосному горбику, стінка утворена шаром фолікулярних клітин і текою. Вкажіть, якій структурі яєчника належать дані морфологічні ознаки:

Первинний фолікул

Зрілий (третинний) фолікул

Примордіальний фолікул

Атретичне тіло

Жовте тіло

3716 / 6307
На заняттях з лікувальної фізкультури лікар-фізіотерапевт запропонував юнакам відхилитись назад і дістати долонями підлогу. Яка зв’язка запобігає надмірному розгинанню хребтового стовпа?

Міжпоперечна

Надостьова

Задня повздовжня

Жовта

Передня повздовжня

3717 / 6307
На розтині жінки 23 років, помершої при нирковій недостатності, виявлено на шкірі лиця “червоний метелик”, на мітральному клапані дрібні до 0.2 см червонувато-рожеві бородавчаті нашарування, в нирках осередки фібриноїдного некрозу в клубочках, потовщення базальних мембран капілярів клубочків у вигляді “дротяних петель”, гематоксилінові тільця, каріорексис. Яке захворювання стало причиною смерті хворої?

Системний червоний вовчак

Системна склеродермія

Ревматизм

Ревматоїдний артрит

Вузликовий периартериит

3718 / 6307
В експерименті у кролика було видалено верхній шийний вузол симпатичного стовбура. На боці видалення спостерігається почервоніння і підвищення температури шкіри голови. Яка форма порушень периферичного кровообігу розвинулась у кроля?

Стаз

Венозна гіперемія

Метаболічна артеріальна гіперемія

Нейропаралітична артеріальна гіперемія

Нейротонічна артеріальна гіперемія

3719 / 6307
Експериментатору необхідно якнайшвидше виробити умовний рефлекс у собаки. На базі якого безумовного рефлекса доцільно виробляти умовний?

Статевого

Травного

Орієнтувального

Міотатичного

Захисного

3720 / 6307
У дитини, що часто хворіє на ангіни і фарингіти, відзначається збільшення лімфовузлів та селезінки. Зовнішній вигляд характеризується пастозністю і блідістю, м’язова тканина розвинена слабко. У крові спостерігається лімфоцитоз. Як називається такий вид діатезу?

Астенічний

Геморагічний

Ексудативно-катаральний

Нервово-артритичний

Лімфатико-гіпопластичний

3721 / 6307
У хворого з підозрою на ботулізм необхідно визначити тип екзотоксину, що циркулює в крові. Яка реакція може бути використана з цією метою?

Реакція пасивної гемаглютинації

Реакція зв’язування комплементу

Реакція нейтралізації

Реакція гальмування гемаглютинації

Реакція преципітації

3722 / 6307
У хворого після травми коліна гомілку у зігнутому під прямим кутом положенні можна зміщувати вперед і назад подібно до висувної шухляди. Які зв'язки розірвані?

Схрещені зв'язки коліна

Поперечна зв'язка коліна

Малогомілкова колатеральна

Великогомілкова колатеральна

Коса підколінна

3723 / 6307
Підлітку, що перебував у стані важкого алкогольного сп'яніння, лікар швидкої допомоги серед інших заходів здійснив внутрішньом'язове введення розчину кофеїну. Поясніть на основі якого принципу дії дана маніпуляція є доцільною:

Синергізм

Фізіологічний антагонізм

Потенціація

Сумація ефектів

Конкурентний антагонізм

3724 / 6307
Чоловік 43-х років доставлений у лікарню з ознаками черевного тифу. Хворий нещодавно був у місцевості, епідемічній за даним захворюванням, де пив некип'ячену воду з колодязя. З моменту появи перших симптомів пройшло З дні. У цей період збудник може бути виявлений у:

Лімфоїдній тканині кишківника

Тонкій кишці

Шлунку

Жовчному міхурі

Крові

3725 / 6307
Офтальмолог з діагностичною метою (розширення зіниць для огляду очного дна) використав 1% розчин мезатону. Мідріаз, викликаний препаратом, обумовлений:

Блокада альфа-1 адренореципторів.

Активація альфа-1 адренорецепторів.

Активація М-холінорецепторів

Активація альфа-2 адренорецепторів.

Активація 6ета-1 адренорецепторів

3726 / 6307
Зменшення тиску в каротидному синусі спричиняє наступні ефекти:

Рефлекторне гіперпное

Рефлекторну брадикардію

Падіння венозного тиску

Зростання частоти серцевих скорочень

Рефлекторне зростання венозного тиску

3727 / 6307
З метою профілактики гепатиту В групі стоматологів ввели вакцину, яка являє собою генно-інженерний НВs-антиген. Від якого ще інфекційного агента захищає таке щеплення?

Вірус дельта

Вірус грипу, тип В

Вірус Коксакі, група В

Вірус гепатиту С

Вірус імунодефіциту людини

3728 / 6307
Після прийому препарату у хворого з серцевою недостатністю зменшилася частота скорочень серця, пул став кращого наповнення, зменшилися набряки, збільшився діурез. Вкажить, який препарат приймав хворий:

Резерпін

Дилтіазем

Дигоксин

Анаприлін

Верапаміл

3729 / 6307
У чоловіка, померлого від внутрішньої кровотечі (гемоперитонеум). в печінці субкапсулярно виявлено губчастий вузол темно-червоного кольору розмірами 15x10 см, добре відмежований від навколишньої тканини. Мікроскопічно: тканина вузла складається з великих судинних тонкостінних порожнин, вистелених ендотеліальними клітинами та заповнених рідкою або згорнутою кров'ю. Встановіть вид пухлини:

Лімфангіома

Венозна гемангіома

Капілярна гемангіома

Гемангіоперицитома

Кавернозна гемангіома

3730 / 6307
Людина вийшла з кондиційованого приміщення на вулицю, де температура повітря дорівнює +40°С, вологість повітря - 60%. Віддача тепла з організму на вулиці буде здійснюватися за рахунок:

Проведення

Випаровування поту

Конвекції

Радіації

3731 / 6307
До приймального відділення доставлений хворий зі скаргами на сухість в роті, світлобоязнь та порушення зору. Об'єктивно: шкіра гіперемована, суха, зіниці розширені, тахікардія. При подальшому обстеженні був встановлений діагноз: отруєння алкалоїдами беладонни. Який лікарський засіб доцільно застосувати?

Прозерин

Армій

Пілокарпін

Дипіроксим

Ацеклідин

3732 / 6307
Хворий 63 років звернувся до невропатолога зі скаргою на те, що протягом трьох місяців не може виконувати столярні роботи, які потребують точності виконання тому що права рука робить багато неціленаправленних рухів. При дослідженні виявлено, що у хворого пошкоджена:

Gyrus angularis.

Gyrus supramarginalis.

Gyrus temporalis superior.

Gyrus precentralis.

Gyrus postcentralis.

3733 / 6307
У новонародженої дитини спостерігаються зниження інтенсивності смоктання, часте блювання, гіпотонія. У сечі та крові значно підвищена концентрація цитруліну. Який метаболічний процес порушений?

Цикл Корі

Орнітиновий цикл

Глюконеогенез

Гліколіз

ЦТК

3734 / 6307
На ізольованому серці шляхом охолодження припиняють функціонування окремих структур. Яку структуру охолодили, якщо серце внаслідок цього спочатку припинило скорочення, а далі відновили її з частотою, у 2 рази меншою за вихідну?

Синоатріальний вузол

Пучок Гіса

Ніжки пучка Гіса

Атріозетрикулярний вузол

Волокна Пуркін'є

3735 / 6307
Жінка літнього віку перенесла сильний стрес. У крові різко збільшилась концентрація адреналіну і норадреналіну. Які ферменти каталізують процес інактивації катехоламінів?

Моноамінооксидази

Пептидази

Тирозиназа

Карбоксилаза

Глікозидази

3736 / 6307
У новонародженої дитини після годування молоком спостерігалися диспептичні розлади (диспепсія, блювота). При годуванні розчином глюкози ці явища зникали. Вкажіть фермент, що бере участь в перетравленні вуглеводів, недостатня активність якого приводить до вказаних розладів.

Мальтаза.

Амілаза.

Ізомальтаза.

Сахараза.

Лактаза.

3737 / 6307
У хворого, що страждає на важку форму порушення водно- сольового обміну, настала зупинка серця в діастолі. Який найбільш вірогідний механізм зупинки серця в діастолі?

Гіперкаліємія

Гіпернатріемія

Дегідратація організму

Гіпокаліємія

Гіпонатріємія

3738 / 6307
Після перенесеного запального захворювання у хворого виникло неповне відведення очного яблука в латеральну сторону. Який нерв у хворого пошкоджений?

Лицевий

Блоковий

Зоровий

Відвідний

Окоруховий

3739 / 6307
В аналізі крові лаборант виявив без’ядерні форменні елементи у вигляді двовгнутих дисків. Назвіть їх:

Нейтрофіли

Еозинофізи

Лімфоцити

Еритроцити

Моноцити

3740 / 6307
Пацієнт, що прийшов на прийом, скаржиться на свербіж між пальцями. Лікар поставив діагноз - скабієз. Які членистоногі можуть спричиняти це захворювання?

Коростяний свербун.

Собачий кліщ.

Тайговий кліщ.

Дермацентор.

Селищний кліщ.

3741 / 6307
Студент дістав завдання розрахувати альвеолярну вентиляцію. Для цього йому необхідні знати наступні показники зовнішнього дихання:

Дихальний об’єм, резервний об’єм вдиху, резервний об’єм видиху

Хвилинний об’єм дихання, частота дихання, дихальний об’єм

Дихальний об’єм, об’єм мертвого простору, частота дихання

Об’єм мертвого простору, життєва ємність легень, дихальний об’єм

Частота дихання, життєва ємність легень, резервний об’єм вдиху

3742 / 6307
Аспірин інгібує синтез простагландинів, завдяки блокуванню активності циклооксигенази. Яка жирна кислота необхідна для цього синтезу?

Лінолева

Арахідонова

Стеаринова

Ліноленова

Пальмітинова

3743 / 6307
Характерними ознаками холери є втрата організмом великої кількості води та іонів натрію. Який механізм лежить в основі виникнення діареї при цьому?

Посилення секреції реніну клітинами ниркових артеріол

Гальмування синтезу вазопресину з гіпоталамусі

Окиснення альдостерону в корі наднирникїв

Активація аденілатциклази ентероцитів

Посилення синтезу кортикотропіну

3744 / 6307
Під час пологової діяльності при важкому прорізуванні голівки плоду, щоб уникнути розриву промежини, виконують розсічення отвору піхви біля основи великої статевої губи. Який м’яз промежини при цьому розсікають?

Поверхневий поперечний м’яз

Зовнішній сфінктер прямої кишки

Цибулинно-губчастий м’яз

Глибокий поперечний м’яз

Сіднично-печеристий м'яз

3745 / 6307
У працівників хімічних комбінатів, де виробляють органічні розчинники, які здатні розчиняти фосфоліпіди, часто розвиваються захворювання легень Який компонент аерогематичного бар'єру при цьому пошкоджується в першу чергу?

Респіраторні альвеолоцити

Секреторні альвеолоцити

Септальні клітини

Альвеолярні макрофаги

Сурфактант

3746 / 6307
У пацієнта після переохолодження у ділянці крил носа та верхньої губи з'явились герпетичні висипання. Для лікування була застосована мазь. Який противірусний засіб містить застосована мазь?

Дексаметазон

Ацикловір

Азидотимідин

Інтерферон

Індометацин

3747 / 6307
Під час розтину порожнин серця на внутрішній стінці були виявлені гребінцеві м'язи. Які відділи порожнин серця розкриті ?

Правий і лівий шлуночок

Ліве передсердя і лівий шлуночок

Ліве вушко і лівий шлуночок

Праве і ліве вушко

Праве передсердя і правий шлуночок

3748 / 6307
Під час ендоскопічного дослідження лікар виявив порушення цілісності стінки шлунка в межах слизової оболонки. Вкажіть. яким типом епітелію в нормі вистелена з середини стінку шлунка:

Псевдобагатошаровий

Багатошаровий плоский зроговілий

Одношаровий призматичний залозистий

Перехідний

Багатошаровий плоский незроговілий

3749 / 6307
Хворому 50-ти років після операції з метою прискорення загоєння рани місцево був призначений препарат, який має репаративну та імуностимулюючу активність. Визначте препарат:

Дексаметазон

Діазолін

Метилурацил

Циклоспорин

Меркаптопурин

3750 / 6307
Хворому на сепсис призначте антимікробний препарат з групи фторхінолонів. Оберіть його серед наведених препаратів

Метронідазол

Цефпіром

Цефалексин

Ципрофлоксацин

Ампіцилін

3751 / 6307
A 24-year-old man undergoes surgery and during the operation, an organ is excised and sent for histological evaluation. A light microscopic examination roved s Ihe organ encased by thin connective tissue capsule that enters the substance oi the lobes to further subdivide the organ into irregular lobular units. Bach lobule contains a cluster of follicles filled with colloid. Follicular epithelium consists of low columnar, cuboidal or squamous cells depending on the level of activity of the follicle. Which of the following organs does this tissue most likely belong to?

Parathyroid gland

Parotid gland

Thyroid gland

Thymus

Pancreas

3752 / 6307
A 23-year-old woman presents to the emergency department complaining of bloody diarrhea, fatigue and confusion. A few days earlier, she went to a fast food restaurant for a birthday party. Her friends are experiencing similar symptoms. Laboratory studies show anemia. Which of the following would you most likely obtain for microbiologic testing?

Bile

Blood

Cerebrospinal fluid

Urine

Stool

3753 / 6307
An 18-year-old girl comes to her physician with concern about her health because she has not achieved menarche. She denies any significant weight loss, changes in mood, or changes in her appetite. She mentions that her mother told her about mild birth defects, but she cannot recall the specifics. Past medical history and family history are benign. On physical examination, the patient is short in stature, has a short and webbed neck and wide chest. Staining of buccal smear reveals absence of Barr bodies in the nucleus of epithelial cells. A urine pregnancy test is negative. Which of the following genetic disorders is the most likely cause of this patient’s condition?

Cri du chat ("cat-cry”) syndrome

Edwards syndrome

Klinefelter syndrome

Palau syndrome

Turner syndrome

3754 / 6307
A 38-year-old woman, who was diagnosed with systemic lupus erythematosus (SLE) 3 years ago, comes to her physician with a complaint of facial swelling and decreased urination that she first noticed 2 weeks ago. She currently takes azathioprine and corticosteroid. Her vital signs show blood pressure 150/90 mm Hg, pulse – 91/min., temperature – 36.8 C and respiratory rate – 15/min. On physical examination, the doctor notices erythematous rash on her face exhibiting a butterfly pattern. The laboratory studies reveal hypertriglyceridemia and proteinuria. Which of the following is the most likely mechanism of SLE`s complication in this patient?

Increased plasma oncotic pressure

Acute infection of the kidney

Immune complex-mediated glomerular disease

Decrease in renal blood flow (ischemic nephropathy)

3755 / 6307
A 37-year-old man is admitted to hospital with mental confusion and disorientation. His wile reports he became more irritable and forgetlul in the past year. In addition, she notes that be became a vegan a year ago and currently, his diet consists of starchy foods like potatoes, corn and leafy vegetables. GI symptoms include anorexia, diarrhea and vomiting. He has glossitis and skin lesions that appear as vesicles over the extremities. Eczemalike lesions around the mouth, as well as desquamation and roughened skin over the hands are also present. Neurologic examination reveals symmetrical hypesthesia for all types of sensation in both upper and lower extremities in a ’’gloves and socks” distribution. Deficiency in diet of which of the following amino acids is the most likely cause of this condition?

Arginine

Histidine

Threonine

Tryptophan

Lysine

3756 / 6307
A 60-year old man with a history of hypertension, diabetes and hyperlipidemia had a sudden onset a right-sided weakness. By the time the ambulance arrived, he had difficulty speaking. Unfortunately, the patient died within the next two hours and an autopsy was performed immediately. The gross examination of the cerebral left hemisphere showed brain swelling, widened, gyri and poorly demarcated gray-white junction. Which of the following is the most likely cause of this patient`s death?

Tumor

Ischemic stroke

Abscess

Intracerebral hemorrhage

Cyst

3757 / 6307
A 34-year-old-man visits dentist complaining of toothache. After a dental procedure that onvolved extraction of several teeth, he develops severe bleeding lasting more than 15 minutes. He has a history of chronic hepatitis. Which of the following is the most likely cause of prolonged bleeding in this patient?

Hypocalcemia

Trombocytopenia

Hypoalbuminemia

Hypofibrinogenemia

3758 / 6307
A 43-year-old man seeks evaluation at emergency department with complaints of fever with chills, melaise, diffuse abdominal pain for over a week, diarrhea and loss of appetite. He says that his symptoms are progressively getting worse. He recalls that the fever began slowly and climbed its way- up stepwise to the current 39.8°C. His blood pressure is 110/70 mm Hg. A physical exam reveals a coated tongue, enlarged spleen and rose spots on the abdomen. Serologic study shows the agglutinin O titre of 1:200 by the Widal test. Which of the following is the most likely causative organism for this patient's condition?

Enterohemorrhagic E. coli

Leptospira interrogans

Salmonella typhi

Mycobacterium tuberculosis

Vibrio cholerae

3759 / 6307
A 64-year-old man presents with a tremor in his legs and arms. He says he has had the tremor for "many years", but it has worsened in the last year. The tremor is more prominent at rest and nearly disappears on movement. His daughter mentions that his movements have become slower. The patient is afebrile and vital signs are within normal limits. On physical examination, the patient is hunched over and his face is expressionless throughout examination. There is a "pillrolling" resting tremor that is accentuated when the patient is asked to clench the contralateral hand and alleviated by finger nose testing. When asked to walk across the room, the patient has difficulty taking the first step, has a stooped posture and takes short rapid shuffling steps. A doctor initiates pharmacotherapy and the drug of first line, levodopa, is prescribed. Which of the following is the most likely mechanism of action of this drug?

Activation of M2-cholinergic receptors

Cholinesterase inhibition

Inhibition of M2-cholinergic receptors

Stimulation of dopamine production

3760 / 6307
A 65-year-old woman presents to the emergency department because of shortness of breath and chest pain that started a few hours ago. She did not have a lever, expectoration, or any accompanying symptoms. She has a history of right leg deep vein thrombosis that occurred 5 years ago. Some time later, she dies of severe respiratory distress. A pulmonary autopsy specimen reveals red loose mass that is lodged in the bifurcation of the pulmonary trunk with extensions into both the left and right main pulmonary arteries. Which of the following is the most likely diagnosis?

Pneumonia

Myocardial infarction

Pneumothorax

Tromboembolism

3761 / 6307
An 11-year-old girl is brought to the doctor’s office by her mother who stales her daughter has been weak with swollen face for 5 days. The mother slates her daughter had always been healthy and active until the initiation of symptoms. Upon inquiry, the girl describes a foamy appearance of her urine but denies blood in urine, urinary frequency at night, or pain during urination. Physical examination reveals generalized swelling of the face and pitting edema on the lower limbs. Laboratory study shows proteinuria and microscopic hematuria. Which of the following is the most likely cause of findings in the laboratory study of urine?

Increased glomerular hydrostatic pressure

Increased hydrostatic pressure in Bowman's capsule

Increased plasma oncotic pressure

Increased permeability across the glomerular capillary wall

3762 / 6307
A group of researchers aimed to study cardiac physiology found that overstretching of atria in the heart leads to decreased sodium reabsorption in the distal convoluted tubule and increase in glomerular filtration rate. Which of the following is the most likely cause ot physiologic effects discovered by researchers?

Aldosterone

Antidiuretic hormone

Renin

Natriuretic peptide

Angiotensin

3763 / 6307
After dehelmintization, a 35-year-old man passed a 3.5 m tapeworm during a bowel movement. A stool examination reveals scolex with four suckers and hooks. Mature proglottids are static with up to 12 primary uterine branches. Which of the following is the most likely diagnosis?

Diphyllobothriasis

Taeniasis

Echinococcosis

Opisthorchiasis

3764 / 6307
A 14-year old girl presents to the emergency department for evaluation of an "infected leg". She states there is no history of trauma but mentions she had a history of sickle cell disease. On physical examination, her upper part of right shin is very painful, red, swollen and hot. Her temperature is 39.2°C. An X-ray shows focal bony lysis and loss of trabecular architecture in the metaphysis of right tibia, increased activity of which of the following cells is the most likely cause of bone reabsorption in this patient?

Osteoclasis

Osteoblasts

Chondrocytes

Chondroblasts

Osteocytes

3765 / 6307
A male neonate born to a 24-year-old primigravida had jaundice at 8 hours f life The neonates red blood cell time was A+, while the mothers RBC type was 0+. Laboratory studies revealed elevated titer of mother's anti-A antibody, normal erythrocyte glucose-6-phosphate and negative sickle cell test. The infant’s hemoglobin was 106 g/L. Which of the following is the most likely cause of infant’s jaundice?

Decrease in hemoglobin level

Rh incompatibility

Sickle cell disease

Hyperbilirubinemia

Glucose-6-phosphate dehydrogenase (G6PD) deficiency

3766 / 6307
A 16-year-old girl concerned about her sexual development comes to the physician. She mentions that she has still not had a menstrual period. However, she is otherwise a healthy girl with no significant medical problems since birth. On physical examination, her vital signs are stake. She does not have pubic hair and her breast fe sliehtlv elevated with areola remaining in contour with surrounding breast. Which of the following is the most likely cause of this abnormal physical development?

Hyperthyroidism

Hypothyroidism

Ovarian insufficiency

Adrenal medulla hyperfunction

Pancreatic islet insufficiency

3767 / 6307
A 46-year-old man presents with fatigue and joint pain in his lingers and wrists lor the last 2 months. The pain is present in both hands and the wrists are swollen. Furthermore, he describes morning stiffness in his joints lasting about 2 hours, which improves with use. His past medical history reveals he has been successfully treated for H. pylori related ulcers last year. He denies smoking and stopped drinking when his gastric symptoms started. Which of the following drugs is the best choice for his joint`s pain management?

Celecoxib

Aspirin

Paracetamol

Morphine

Prednisone

3768 / 6307
A 68-year-old man comes to his physician with complaints of severe fatigue and altered sensations in his extremities. Past medical history is remarkable for chronic gastritis. He drinks alcohol almost every day. His blood pressure is 130/80 mm Hg, heart rate is 95/min., respiratory rate -14/min. and temperature of 37.1 C. His heart has a regular rate and rhythm, his lungs are clear to auscultation bilaterally. Neurologic examination reveals loss of touch and vibration sense in both upper and lower limbs. Laboratory investigation results include a hemoglobin of 80 g/L, Mean Corpuscular Volume (MCV) of 115 fL (the reference range is 80-100 fL) and White Blood Cells (WBC) of 3.0=109/L. Which of the following is the most likely diagnosis?

Vitamin A deficiency

Iron deficiency

Vitamin C deficiency

Viitamin B12 deficiency

3769 / 6307
A 6-year-old boy is brought to the pediatrician by his mother, who complains of low-grade fever, chronic cough and night sweats in her child. She describes the cough as productive, producing white sputum that is sometimes streaked with blood. She also says thet her son has lost some weight in the last month. His vital sings include blood pressure of 115/75 mm Hg, heart rate of 110/min., respiratory rate of 18/min and temperature ot 36.6 C. On physical examination, the patient is ill looking. Pulmonary auscultation reveals some fine crackles 'in the right upper lobe. The pediatrician suspects an active infection and performs Mantoux test. Intradermal injection of which of the following substances has been most likely used by pediatrician for screening test in this clinical case?

Tuberculin

Bacillus Calmcttc-Guerin (BCG) vaccine

Diphtheria-tetanus toxoids-acellular pertussis vaccine (DTaP)

Tetanus and diphtheria toxoids vaccine (Td)

3770 / 6307
A 37-year old female presents to the clinic complaining of severe pain in her left wrist and tingling sensation in her left thumb, index finger, and middle finger, and some part of her ring finger. Hie pain started as an occasional throb and she could ignore it or take ibuprofen but now the pain is much worse and wakes her up at night. She works as a typist and her pain mostly increases alter typing all day. Her right wrist and fingers are line Nerve conduction studies reveal ncive compression. Which of the following nerves is most likely compressed in this patient?

Radial nerve

Ulnar nerve

Median nerve

Musculocutaneous nerve

Axillary nerve

3771 / 6307
A 45-year-old woman comes to her physician with complaints of excessive fatigue and weakness. She says that these symptoms have been present for the past month. On further questioning, she admits having lost 3 kilograms in the last 2 weeks. On physical examination, she is a tired-appearing thin woman. Hyperpigmentation is present over many areas of her body, most prominently over the face, neck and back of hands (areas exposed to light). Increased production of which of the following hormones is the most likely cause of hyperpigmentation in this patient?

Gonadotropins

Melanocytc-stimulating hormone (MSH)

Growth hormone (GII)

Thyroid-stimulaiing hormone (TSII)

B-Lipolropin

3772 / 6307
A 40-year-old woman dies of intracerebral hemorrhage alter hypertensive emergency. During an autopsy, the pathologist reveals severe obesity, excess of body hair and wide purplish stria on the abdomen. Microscopic examination of pituitary gland reveals hyperplastic acini populated by a homogenous cluster of deeply basophilic cells. Which of the following was the most likely underlying disease

Hyperthyroidism

Sheehan's syndrome

Cushing disease

Arterial hypertension

3773 / 6307
A 56-year-old man presents for a checkup. The patient says he has to urinate quite frequently, but denies any dysuria or pain on urination. Past medical history is significant for diabetes mellitus type 2 and hypertension, both managed medically. Current medications are metformin, aspirin, rosuvastaiin. captopril and furosemide Laboratory findings arc significant for the following: Glycated Hemoglobin (Hb A1c) - 8.0%. Fasting Blood Glucose - 12 mmol L. His doctor decides to add glibenclamide to the therapy. Which of the following is the most likely mechanism of this drug's action?

Stimulation of insulin release

Inhibition of insulin release

Stimulation of glucose reuptake by the cell

3774 / 6307
After dehelmintization, a 35-year-old man passed a 3.5 m tapeworm during a bowel movement. A stool examination reveals scolex with four suckers and hooks. Mature proglottids are static with up to 12 primary uterine branches. Which of the following is the most likely diagnosis?

Diphyllobothriasis

Echinococcosis

Opisthorchiasis

Taeniasis

3775 / 6307
A mother of a 4-month-old male infant brought him to pediatrician with complaints of food rejection and weight loss. He started having trouble latching onto his bottle. He has also become extremely lethargic. Examination reveals diminished muscle tone in all four limbs, and hepatosplenomegaly. An ophthalmoscopic exam reveals macular cherry red spots. During the next few weeks, hepatosplenomegaly progresses, the boy fails to thrive, and he continues to reject food. Chest X-ray shows a reticulonodular pattern and calcified nodules. Biopsy of the liver shows foamy histiocytes. A Niemann-Pick disease is suspected. Which of the following is the most likely deficient enzyme in this patient?

Glucose-6-phosphatase

Galactocerebrosidase

Phenylalanine-hydroxylase

Sphingomyelinase

Glucocerebrosidase

3776 / 6307
An unidentified surgical specimen is received for histopathologic analysis. A portion of the specimen is cut and stained with hematoxylin and eosin. Under the microscope, you see an organ encapsulated by dense connective tissue that extends to the deeper areas by way of the trabecular extensions. The organ can be subdivided into two regions: a cortex with lymphoid nodules and medulla with medullary cords populated by plasma cells, B-cells and T-cells. Which of the following structures is most likely the origin of this surgical specimen?

Lymph node

Thymus

Tonsils

Bone marrow

Spleen

3777 / 6307
A team of medical students is performing research on phases of cell cycle. During one of the mitotic phases the cell is nearly done dividing, the chromosomes decondense and two nuclei begin to form around them. Which of the following phases most likely takes place in ihe cell?

Metaphase

Prophase

Telophase

Anaphase

3778 / 6307
A 20-year-old female comes to the clinic after missing her last 2 periods. Her cycles are usually regular, occurring at 28-30 day interval with moderate bleeding and some abdominal discomfort. She also complains of progressively diminishing peripheral vision. Her doctor reveals loss of vision in the lateral halves of both eyes. Involvement of which of the following structures would you most likely expect to be the reason of bitemporal hemianopsia?

Right optic tract

Left optic nerve

Left optic tract

Optic chiasm

Right optic nerve

3779 / 6307
A 54-year-old woman has a total thyroidectomy for papillary thyroid carcinoma. 11 hours after operation she complains of tingling around her mouth. On physical examination, the Trousseau's sign and Chvostek’s sign are present. Her condition rapidly deteriorates with laryngospasm and focal seizures. The suigeon suggests surgical destruction of parathyroid glands. Which of the following is the most likely cause of this patient`s neurologic abnormality?

Hypophosphatemia

Hyperchloremia

Hyperkalemia

Hyponatremia

Hypocalcemia

3780 / 6307
Хворому призначена ендоскопія 12-палої кишки. В результаті виявлено запалення великого дуоденального сосочка і порушення виділення жовчі в просвіт кишки. У якому відділі 12-палої кишки виявлені порушення?

Горизонтальна частина

Цибулина

Низхідна частина

Верхня частина

Висхідна частина

3781 / 6307
При аналізі ЕКГ встановлено: ритм синусовий, число передсердних комплексів перевищує число шлуночкових комплексів; прогресуюче подовження інтервалу Р-Q від комплексу до комплексу; випадання окремих шлуночкових комплексів, через що після зубця Р йде довга пауза; зубці Р та комплекси QRST без змін. Назвіть тип порушення серцевого ритму:

Неповна атріовентрикулярна блокада II ступеня

Неповна атріовентрикулярна блокада І ступеня

Повна атріовентрикулярна блокада

Неповна атріовентрикулярна блокада III ступеня

Синоатріальна блокада

3782 / 6307
Чоловік 55-ти років, що скаржиться на біль в ділянці нирок, надійшов в лікарню. В ході ультразвукового обстеження пацієнта виявлено наявність ниркових каменів. Наявність в сечі якої з наведених речовин є найімовірнішою причиною утворення каменів у цього пацієнта?

Уробілін

Креатинін

Білівердин

Білірубін

Сечова кислота

3783 / 6307
У 70-річної людини швидкість поширення пульсової хвилі виявилася суттєво вище, ніж у 25-річної. Причиною цього є зниження:

Еластичності судинної стінки

Серцевого викиду

Частоти серцевих скорочень

Швидкості кровотоку

Артеріального тиску

3784 / 6307
Чоловік внаслідок транспортної аварії втратив багато крові, свідомість затьмарена, низький кров’яний тиск. При цьому у нього компенсаторно активується ренін-ангіотензинова система, що веде до:

Підвищення згортання крові

Гіперпродукції альдостерону

Посилення еритропоезу

Гіперпродукції вазопресинуАпо В48

Посилення серцевих скорочень

3785 / 6307
При дослідженні людини у вертикальній позі встановлено, що в альвеолах верхівок легень парціальний тиск кисню становить 140 мм рт.ст. Причиною цього є те, що у цих відділах легень:

Перфузія переважає над вентиляцією

Перфузія та вентиляція врівноважені

Вентиляція переважає над перфузією

Вентиляція відсутня

3786 / 6307
Хворий протягом останнього року став відзначати підвищену втомлюваність, загальну слабкість. Аналіз крові: еритроцити -4,1•1012/л, Нb-119 г/л, к.п.- 0,87, лейкоцити - 57•109/л, лейкоформула: Ю- 0, П- 0, С- 9%, Е- 0, Б- 0, лімфобласти - 2%, пролімфоцити - 5%, лімфоцити - 81%, М- 3%, тромбоцити - 160•109/л. В мазку: нормохромія, велика кількість тіней Боткіна-Гумпрехта. Про яку патологію системи крові свідчить дана гемограма?

Хронічний монолейкоз

Гострий лімфобластний лейкоз

Хронічний лімфолейкоз

Гострий мієлобластний лейкоз

Хронічний мієлолейкоз

3787 / 6307
У хворого з синдромом Іценка- Кушинга спостерігаються стійка гіперглікемія та глюкозурія. Синтез та секреція якого гормону підвищені у цього хворого?

Тироксин

Альдостерон

Адреналін

Кортизол

Глюкагон

3788 / 6307
Для лікування стрептодермії лікар призначив хворому мазь, що містить антибіотик групи тетрацикліну, і рекомендував обмежити перебування на сонці. Чим небезпечна інсоляція?

Підвищенням токсичності тетрацикліну

Розвитком стійкості збудника до антибіотика

Зниженням активності тетрацикліну

Генералізацією процесу

Розвитком фотосенсибілізації

3789 / 6307
На аутопсії тіла жінки, що хворіла на хронічну дизентерію, в ході гістологічного дослідження внутрішніх органів в стромі та паренхімі міокарда, нирок, в слизовій оболонці шлунка та в сполучній тканині легень виявлені аморфні відкладення фіолетового кольору, що дають позитивну реакцію за Коссом. Яке ускладнення розвинулося у хворої'?

Гіаліноз

Амілоїдоз

Метастатичне звапніння

Дистрофічне звапніння

Метаболічне звапніння

3790 / 6307
У 12-річного хлопчика в сечі виявлено високий вміст усіх амінокислот аліфатичного ряду. При цьому відзначена найвища екскреція цистину та цистеїну. Крім того, УЗД нирок показало наявність у них каменів. Виберіть можливу патологію:

Цистинурія

Хвороба Хартнупа

Фенілкетонурія

Цистит

Алкаптонурія

3791 / 6307
При мікроскопічному дослідженні біоптату нирки виявлено вогнища, в центрі яких розташовані зернисті еозинофільні маси, оточені інфільтратом з лімфоцитів, епітеліоїдних клітин та поодиноких клітин Пирогова-Лангханса. Виберіть патологічний процес, що найповніше відповідає зазначеним змінам:

Казеозний некроз

Альтеративне запалення

Проліферація та диференціювання макрофагів

Коагуляційний некроз

Гранулематозне запалення

3792 / 6307
У дитини 8-ми років через 5 днів після контакту з хворим на вітряну віспу з’явилося нездужання, підвищилася температура, незабаром з’явилися характерні висипи. Який термін найточніше характеризує період, що передував появі перших ознак захворювання?

Інфекційний

Контагіозний

Неспецифічний

Інкубаційний

Латентний

3793 / 6307
Хворому, після радіоактивного опромінення, лікар рекомендував збільшити в раціоні вміст рослинних олій - джерела полієнових жирних кислот. Назвіть кислоту, що містить три подвійних зв’язки:

Міристинова

Арахідонова

Ліноленова

Олеїнова

Лауринова

3794 / 6307
Хвора 55-ти років з гострим нападом печінкової кольки надійшла в гастроентерологічне відділення. Об’єктивно: температура тіла - 38°С, склери, слизові та шкіра іктеричні, сеча темна, кал безбарвний. Скарги на свербіння шкіри. Яка причина жовтяниці у цієї хворої?

Обтурація жовчних проток

Деструкція гепатоцитів

Посилений розпад еритроцитів

Порушення ліпідного обміну

Тривале вживання продуктів, багатих на каротин

3795 / 6307
Хвора 57-ми років для лікування гіпертонічної хвороби тривалий час приймала анаприлін. Побічні ефекти спонукали пацієнтку відмовитись від прийому препарату, що призвело до розвитку гіпертонічного кризу і нападу стенокардії. Як називається ускладнення, яке виникло?

Сенсибілізація

Звикання

Синдром відміни

Лікарська залежність

Тахіфілаксія

3796 / 6307
У хворої після операції видалення ракової пухлини молочної залози та регіонарних лімфовузлів розвинувся набряк руки, який був пов’язаний з лімфатичною недостатністю. Який це вид лімфатичної недостатності за механізмом виникнення?

Резорбційна

Механічна

Динамічна

Спастична

Акінетична

3797 / 6307
У жінки, що тривало приймала антибіотики з приводу кишкової інфекції, розвинулось ускладнення з боку слизової порожнини рота у вигляді запального процесу і білого нальоту, у якому під час бактеріологічного дослідження були виявлені дріжджеподібні грибки Сапdіda аlbісаns. Який з перерахованих препаратів показаний для лікування цього ускладнення?

Бісептол

Фуразолідон

Тетрациклін

Поліміксин

Флуконазол

3798 / 6307
В результаті експресії окремих компонентів геному клітини зародка набувають характерних для них морфологічних, біохімічних та функціональних особливостей. Яку назву має цей процес?

Диференціювання

Індукція

Рецепція

Капацитація

Детермінація

3799 / 6307
Жінка 54-х років звернулася до лікаря зі скаргами на непереносимість курячих яєць, що з’явилася нещодавно. Антигістамінні препарати, призначені лікарем, дещо покращували стан хворої. Які антитіла могли сприяти розвитку цієї реакції?

IgG

ІgМ

IgA

ІgD

IgE

3800 / 6307
В результаті інфікування рани після видалення зуба у хворого виникло її нагноєння. Для очищення рани, як один із засобів, рекомендовано полоскання рота гіпертонічним розчином NaCl. Який процес лежить в основі цього способу очищення?

Механічне очищення

Бактерицидна дія

Осмотична активність солі

Гемостатична дія

Денатурація білка

3801 / 6307
Під час підйому пішки на 5-й поверх у людини підвищився артеріальний тиск. Причиною є збільшення:

Хвилинного об’єму крові

Кількості функціонуючих капілярів

Об’єму циркулюючої крові

В’язкості крові

Вмісту іонів в плазмі крові

3802 / 6307
На 9-й день після введення протиправцевої сироватки з приводу брудної рани стопи в пацієнта підвищилася температура тіла до 38°С, з’явився біль у суглобах, висип, свербіння шкіри. З боку крові спостерігаються лейкопенія і тромбоцитопенія. Який тип алергічної реакції розвинувся?

Клітинно-опосередкований

Цитотоксичний

Імунокомплексний

Анафілактичний

Стимулюючий

3803 / 6307
Тварині в експерименті перерізали передні корінці п’яти сегментів спинного мозку. Які зміни відбудуться в зоні іннервації?

Втрата дотикової чутливості

Гіперчутливість

Втрата пропріоцептивної чутливості

Втрата температурної чутливості

Втрата рухів

3804 / 6307
У потерпілого травма черепа з порушенням дірчастої пластинки решітчастої кістки. Який нерв може бути ушкоджений?

N. оlfactorius

N.opticus

N.abducens

N. oрthalmicus

N. trосhlіаris

3805 / 6307
У хворого 50-ти років через місяць після перенесеного інфаркту міокарда розвинулася серцева недостатність. Невідповідність між навантаженням на серце та його здатністю виконувати роботу пов’язана в цьому випадку з:

Підвищеним опором вигнанню крові в легеневому стовбурі

Перевантаженням серця

Підвищеним опором вигнанню крові в аорті

Ушкодженням міокарда

Змішаною формою недостатності серця

3806 / 6307
У хворого після резекції шлунка розвинулась мегалобластична анемія. Який препарат необхідно призначити хворому?

Вітамін В6

Заліза лактат

Ферковен

Вітамін В12

Аскорбінову кислоту

3807 / 6307
У жінки 62-х років розвинулася катаракта (помутніння кришталику) на тлі цукрового діабету. Вкажіть, який тип модифікації білків має місце при діабетичній катаракті:

Глікозилювання

Фосфорилювання

Метилювання

АДФ-рибозилювання

Обмежений протеоліз

3808 / 6307
Розділ фармакології, що вивчає всмоктування, розподіл та елімінацію ліків в організмі людини, називається:

Фармакодинаміка

Токсикологія

Фармакокінетика

Імунофармакологія

Фармакогенетика

3809 / 6307
При пульмонологічному обстеженні виникла необхідність визначити частину повітря, яка обмінюється в легенях за один дихальний цикл. Цей показник називається:

Об’єм мертвого простору

Хвилинна легенева вентиляція

Дихальний коефіцієнт

Коефіцієнт легеневої вентиляції

Функціональна остаточна ємність

3810 / 6307
Для кращого огляду дна очного яблука лікар закрапав в кон’юнктиву ока пацієнта розчин атропіну. Це призвело до розширення зіниці через блокаду таких мембранних циторецепторів:

Альфа-адренорецепторів

М-холінорецепторів

Н-холінорецепторів

Н2-рецепторів

Бета-адренорецепторів

3811 / 6307
Жінка 38-ми років надійшла в хірургічне відділення з приводу розлитого гнійного перитоніту. Під час операції виявлена перфорація виразки клубової кишки, виразку було ушито. Черевну порожнину дреновано. Через 4 дні хвора померла. На розтині в клубовій кишці наявні виразки в пеєрових бляшках, розташовані уздовж кишки, дно деяких з них має поперечну смугастість. Для якого захворювання характерні такі виразки?

Черевного тифу

Туберкульозу кишечника

Дизентерії

Пухлини тонкої кишки

Амебіазу кишечника

3812 / 6307
Хворий помер від наростаючої легенево-серцевої недостатності. При гістологічному дослідженні виявлено дифузне ураження легенів з інтерстиціальним набряком та інфільтрацією інтер-стиціальної тканини лімфоцитами, макрофагами, плазмоцитами; наявні пневмофіброз та панацинарна емфізема. Поставте діагноз:

Ателектаз легенів

Хронічний бронхіт

Бронхопневмонія

Фіброзивний альвеоліт

Бронхіальна астма

3813 / 6307
В процесі розвитку у дитини хребет поступово набув два лордози та два кіфози. Це пояснюється розвитком здатності до:

Повзання

Прямоходіння

Плавання

Лежання

Сидіння

3814 / 6307
Судово-медичний експерт при розтині тіла 20-річної дівчини встановив смерть внаслідок отруєння ціанідами. Порушення якого процесу, найімовірніше, стало причиною смерті?

Транспорту амінокислот

Синтезу сечовини

Тканинного дихання

Синтезу гемоглобіну

Транспорту кисню гемоглобіном

3815 / 6307
В лікарню звернувся 65-річний хворий зі скаргами на часте сечовиділення зі слідами крові. При посіві сечі на МПА виросли великі колонії з характерним квітковим запахом, які мали зелено- синій пігмент. В мазку з колоній наявні грамнегативні палички. Який збудник спричинив запальний процес сечостатевої системи?

Proteus vulgaris

Escherichia coli

Pseudomonas aeruginosa

Кlebsiella ozaenae

Hafnia alvei

3816 / 6307
Чоловік з гострим міокардитом помер від серцево-судинної недостатності. В ході мікроскопічного дослідження внутрішніх органів виявлені: плазморагія, набряк, стази в капілярах, численні крововиливи, а також дистрофічні зміни в паренхімі. Наслідком чого є дані зміни?

Гострий загальний венозний застій

Хронічний загальний венозний застій

ДВ3-синдром

Загальне артеріальне повнокрів’я

Місцеве артеріальне повнокрів’я

3817 / 6307
В епідермісі є клітини, що виконують захисну функцію та мають моноцитарний генез. Які це клітини?

Кератиноцити базального шару

Кератиноцити остистого шару

Меланоцити

Клітини Лангерганса

Кератиноцити зернистого шару

3818 / 6307
Чоловік 40-ка років скаржиться на загальну слабкість, головний біль, кашель із виділенням мокротиння, задишку. Після клінічного огляду й обстеження поставлено діагноз пневмонія. Який тип гіпоксії має місце у хворого?

Гіпоксична

Тканинна

Гемічна

Циркуляторна

Респіраторна

3819 / 6307
У підлітка після перенесеного інфекційного захворювання з’явилася різко виражена аритмія з вкороченням інтервалу R-R під час вдиху і подовженням його під час видиху. Що лежить в основі цього виду аритмії?

Коливання тонусу блукаючого нерва під час акту дихання

Рефлекс Бейнбріджа

Порушення функції провідності серця

Порушення скоротливої функції серця

Порушення функції збудливості серця

3820 / 6307
Жінці 58-ми років проведене повне видалення матки з придатками, після чого виділення сечі припинилося. При цистоскопії: міхур сечі не містить, з устів сечоводів сеча не надходить. Який відділ сечовидільної системи було ушкоджено в ході операції?

Uretra

Rеп

Ureter

Vesica urinaria

Pelvis renalis

3821 / 6307
У чоловіка 30-ти років перед операцією визначили групову належність крові. Кров резус-позитивна. Реакцію аглютинації еритроцитів не викликали стандартні сироватки груп 0αβ (І), Аβ (II), Вα (III). Досліджувана кров належить до групи:

Aβ (II)

АВ (IV)

0αβ (І)

Вα (III)

3822 / 6307
У лікарню машиною швидкої допомоги доставлено хворого в стані коми. В анамнезі цукровий діабет. При обстеженні спостерігається шумне прискорене дихання, при якому глибокі вдихи чергуються з посиленими видохами за участю експіраторних м’язів. Яка форма порушення зовнішнього дихання спостерігається?

Стенотичне дихання

Дихання Куссмауля

Дихання Біота

Апнейстичне дихання

Дихання Чейна-Стокса

3823 / 6307
У хворого після черепно-мозкової травми, під час якої була ушкоджена мозочкова ділянка, розвинулися порушення часової та просторової координації рухів. Яка патологія розвинулася у хворого?

Абазія

Астазія

Парез

Дисметрія

Атаксія

3824 / 6307
Пацієнт 55-ти років звернувся до лікаря зі скаргами на часті судоми. Встановлено, що тривалий час він працює у гарячому цеху в умовах високих температур. Порушення якого виду обміну призвело до цього стану?

Вуглеводного

Ліпідного

Вітамінного

Білкового

Водно-сольового

3825 / 6307
У результаті радіаційного випромінювання були ушкоджені стовбурові гемопоетичні клітини. Утворення яких клітин сполучної тканини буде порушено?

Адипоцити

Фібробласти

Перицити

Меланоцити

Макрофаги

3826 / 6307
У потерпілого пошкоджений м’яз, апоневроз якого утворює пахвинну зв’язку. Назвіть цей м’яз:

Внутрішній косий м’яз живота

Зовнішній косий м’яз живота

Прямий м’яз живота

Пірамідальний м’яз

Поперечний м’яз живота

3827 / 6307
При обстеженні хворого похилого віку виявлено моторну афазію. Де локалізований осередок пошкодження головного мозку?

Кутова звивина

Постцентральна звивина

Центр Брока

Прецентральна звивина

Звивина Гешля

3828 / 6307
У сільського жителя на кисті правої руки з’явилася неболюча припухлість, яка набула в центрі чорного кольору. При мікроскопічному дослідженні виявлено великі грампозитивні палички, розташовані ланцюжками. Який мікроорганізм міг спричинити це захворювання?

Clostridium botulinum

Bacillus cereus

Clostridium tetani

Bacillus anthracis

Mycobacterium tuberculosis

3829 / 6307
Симбіотична теорія пояснює походження еукаріотичних клітин переходом до аеробного дихання. Це відбулося внаслідок проникнення в клітину аеробних бактерій, які в процесі еволюції перетворилися на:

Лізосоми

Комплекс Гольджі

Мітохондрїї

Рибосоми

Пероксисоми

3830 / 6307
Оглядаючи дитину 6-ти років, лікар помітив на глоткових мигдаликах сірувату плівку, при спробі видалення якої виникла помірна кровотеча. Бактеріоскопія мазків з мигдаликів показала наявність грампозитивних бактерій булавоподібної форми. Які симптоми можуть виникнути у дитини у найближчі дні, якщо не буде проведене специфічне лікування?

Папульозні висипи на шкірі

Хвилеподібна лихоманка

Набряк легенів

Токсичні ураження серцевого м’язу, печінки, нирок

Дуже сильний нападоподібний кашель

3831 / 6307
Деякі тяжкі захворювання нирок супроводжуються еритропенією. Який механізм цього явища найімовірніший?

Аліментарний дефіцит Fе2+

Порушення синтезу еритропоетинів

Збільшення діурезу

Порушення функціонування печінки

Підвищене руйнування еритроцитів в селезінці

3832 / 6307
У померлого, що понад 20 років працював на шахті з видобутку кам’яного вугілля, при розтині виявлено ущільнені легені сіро-чорного кольору зі значними ділянками новоутвореної сполучної тканини та наявністю великої кількості макрофагів із пігментом чорного кольору в цитоплазмі. Який з перерахованих діагнозів найімовірніший?

Талькоз

Антракоз

Антракосилікоз

Сидероз

Силікоантракоз

3833 / 6307
У крові резус-негативної жінки під час вагітності виявлені специфічні білки, здатні руйнувати резус-позитивні еритроцити плода. Як називається цей захисний компонент організму матері?

Сироватка

Фактор некрозу пухлини

Резус-фактор

Гормон

Антитіло

3834 / 6307
Хворий 50-ти років скаржиться на поліурію, спрагу, протягом доби випиває до 15 літрів рідини. При обстеженні виявлено: вміст глюкози крові - 4,8 ммоль/л, сеча безбарвна, відносна щільність - 1,002-1,004, цукор і білок відсутні. Яка імовірна причина поліурії?

Дефіцит альдостерону

Надлишок тиреоїдних гормонів

Дефіцит вазопресину

Дефіцит тиреоїдних гормонів

Надлишок альдостерону

3835 / 6307
У пацієнта перфоративна виразка передньої стінки шлунка. В яке похідне очеревини потрапить вміст шлунка?

Печінкова сумка

Передшлункова сумка

Правий брижовий синус

Чепцева сумка

Лівий брижовий синус

3836 / 6307
За умов тривалої інтоксикації тварин тетрахлорметаном було визначене суттєве зниження активності аміноацил- тРНК-синтетаз в гепатоцитах. Який метаболічний процес порушується в цьому випадку?

Посттрансляційна модифікація пептидів

Біосинтез білків

Посттранскрипційна модифікація РНК

Транскрипція РНК

Реплікація ДНК

3837 / 6307
У хворого зі швидко наростаючою внутрішньомозковою гіпертензією діагностована пухлина мозку. Під час операції видалена пухлина тім’яно-скроневої частки, м’якої консистенції, строката на розрізі. Гістологічно пухлина побудована з поліморфних гіперхромних клітин з утворенням псевдорозеток та великої кількості судин, ділянками некрозів і крововиливами. Поставте діагноз:

Менінгіома

Олігодендрогліома

Гліобластома

Астроцитома

Арахноїдендотеліома

3838 / 6307
Експериментатору необхідно якнайшвидше виробити умовний рефлекс у собаки. На базі якого безумовного рефлексу доцільно виробляти умовний?

Захисний

Міотатичний

Орієнтувальний

Статевий

Травний

3839 / 6307
У хворого 60-ти років під час об’єктивного обстеження виявлено набряки на ногах, асцит, збільшення печінки, що свідчить про недостатність кровообігу за правошлуночковим типом. В анамнезі перенесений ревматизм. Одним з основних факторів розвитку набряків є активація системи ренін-ангіотензин- альдостерон, яка є наслідком:

Ацидозу

Зменшення хвилинного об’єму серця

Поліцитемічної гіперволемїї

Розширення посткапілярних вен

Утруднення дифузії речовин

3840 / 6307
У п’ятимісячної дівчинки виявлено застійні явища у легенях. При обстеженні виявлено зв’язок між висхідною аортою та легеневою артерією, що в нормі спостерігається у деяких земноводних і плазунів. Назвіть цю природжену ваду розвитку:

Незрощення боталової протоки

Дефект міжпередсердної перегородки

Дефект міжшлуночкової перегородки

Розвиток правої дуги аорти

Транспозиція магістральних судин

3841 / 6307
Під час проведення морфологічного дослідження периферичної крові хворого було помічено, що у еритроцитів забарвлена лише периферична частина, а в центрі є незабарвлене прояснення. Кольоровий показник - 0,56. Яка анемія найімовірніша у цього пацієнта?

Гемолітична

Залізодефіцитна

Сидеробластна

В₁₂ фолієводефіцитна

Апластична

3842 / 6307
Під час хірургічної операції виникла необхідність масивного переливання крові. Група крові потерпілого - III (В) Rh(+). Якого донора треба вибрати?

І(0)Rh(-)

IV (АВ) Rh(-)

III (В) Rh(-)

III (В) Rh(+)

II (А) Rh(+)

3843 / 6307
У людини внаслідок лікування антибіотиками виник дисбактеріоз товстого кишечника. Яких вітамінів, синтезованих бактеріями в товстому кишечнику, буде менше надходити до організму?

Вітаміни Р і С

Вітамін К та вітаміни групи В

Аскорбінова кислота

Вітаміни А та Е

Вітамін В

3844 / 6307
Лікар швидкої допомоги встановив у хворого діагноз гіпертонічний криз. Для усунення цього стану хворому парентерально введено засіб, який крім гіпотензивної має також протисудомну дію, а при ентеральному введенні - проносну і жовчогінну. Назвіть цей препарат:

Натрію нітропрусид

Клофелін

Пентамін

Дибазол

Магнію сульфат

3845 / 6307
Жінка 62-х років скаржиться на частий біль грудного відділу хребта, переломи ребер. Лікар припустив мієломну хворобу (плазмоцитому). Який з перерахованих нижче лабораторних показників буде мати найбільше діагностичне значення?

Гіпоглобулінемія

Протеїнурія

Гіперальбумінемія

Гіпопротеїнемія

Парапротеїнемія

3846 / 6307
У чоловіка, померлого від внутрішньої кровотечі (гемоперитонеум), в печінці субкапсулярно виявлено губчастий вузол темно-червоного кольору розмірами 15x10 см, добре відмежований від навколишньої тканини. Мікроскопічно: тканина вузла складається з великих судинних тонкостінних порожнин, вистелених ендотеліальними клітинами та заповнених рідкою або згорнутою кров’ю. Встановіть вид пухлини:

Венозна гемангіома

Кавернозна гемангіома

Гемангіоперицитома

Лімфангіома

Капілярна гемангіома

3847 / 6307
Після пошкодження мозку у людини порушене сприйняття зорової інформації. В якому відділі кори сталося пошкодження?

Потилична ділянка кори

Тім’яна ділянка кори

Передня центральна звивина

Скронева ділянка кори

Задня центральна звивина

3848 / 6307
Новонароджений, який з’явився на світ у домашніх умовах без кваліфікованої медичної допомоги, і мати якого не проходила у період вагітності необхідних обстежень, на другу добу доставлений в лікарню з гострим гнійним кон’юнктивітом. При мікроскопії виділень з очей дитини знайдені грамнегативні диплококи, розташовані всередині лейкоцитів та поза клітинами. Який мікроорганізм є найімовірнішим збудником захворювання?

Neisseria gonorrhoae

Соrynebacterіит dірhtheriае

Staphylococcus aureus

Сhlamydia trachomatis

Pseudomonas aeruginosa

3849 / 6307
Фізіологи встановили, що кількість еритроцитів у крові залежить від функціонального стану червоного кісткового мозку й тривалості життя еритроцита. Який термін життя еритроцита в периферичній крові в середньому?

70 діб

220 діб

120 діб

150 діб

50 діб

3850 / 6307
Жінка 35-ти років звернулася до лікаря зі скаргами на дратівливість, тривожність, швидку втомлюваність, безсоння. Для усунення неврозу лікар призначив пацієнтці транквілізатор діазепам. Вкажіть фармакодинамічний ефект діазепаму, що дав можливість застосувати його за цих обставин:

Міорелаксантний

Протисудомний

Анксіолітичний

Антипсихотичний

Психостимулювальний

3851 / 6307
В колективі дошкільної установи одночасно виявили декілька дітей з катаральним запаленням слизової оболонки трахеї та бронхів. За клінічними проявами припущено коклюшну інфекцію. Яким чином, як правило, передається коклюш?

Трансмісивним

Аліментарним

Повітряно-пиловим

Повітряно-краплинним

Контактним

3852 / 6307
У хворого на тромбофлебіт нижніх кінцівок з’явилися біль в грудній клітці, кровохаркання, наростаюча дихальна недостатність, при явищах якої він помер. На розтині діагностовані множинні інфаркти легенів. Яка найімовірніша причина їх розвитку в цьому випадку?

Тромбоз бронхіальних артерій

Тромбоз гілок легеневої артерії

Тромбоемболія бронхіальних артерій

Тромбоз легеневих вен

Тромбоемболія гілок легеневої артерії

3853 / 6307
У хворого видалили щитоподібну залозу, яка була значно збільшена в розмірах, щільно-еластичної консистенції, з горбистою поверхнею. При гістологічному дослідженні в паренхімі залози визначається дифузна лімфо-плазмоцитарна інфільтрація з формуванням лімфоїдних фолікулів з гермінативними центрами, атипія та метаплазія фолікулярного епітелію, вогнища склеротичних змін паренхіми. Який діагноз найімовірніший?

Вузловий зоб

Дифузний еутиреоїдний зоб

Дифузний тиреотоксичний зоб

Аутоімунний тиреоїдит

Тиреоїдит де Кервена

3854 / 6307
У патогенезі розвитку II типу гіперліпопротеїнемії (сімейна гіперхолестеролемія) провідну роль відіграє дефіцит рецепторів до апобілка ЛПНЩ. Назвіть його:

Апо ВСІ

Апо А1

Апо СІІ

Апо В48

Апо В100

3855 / 6307
Хвороба Куру характеризується тремором і атаксією; при хворобі Крейтцфельдта-Якоба спостерігаються розвиток атаксії і деменції. Доведено, що ці захворювання викликаються:

Токсичними продуктами навколишнього середовища

Пріонами

Бактеріями, позбавленими клітинної стінки

Повільними вірусами

Грибами

3856 / 6307
У хворої діагностовано пухлину головки підшлункової залози, порушення венозного відтоку з деяких органів черевної порожнини. Яка венозна судина була здавлена пухлиною?

Ниркова вена

Права шлункова вена

Ліва шлункова вена

Ворітна вена печінки

Нижня порожниста вена

3857 / 6307
У дитини спостерігаються невротичні симптоми: запаморочення, слабкість, головний біль, який супроводжується нудотою, болем в правому підребер’ї, частими позивами на дефекацію. При лабораторному дослідженні дуоденального вмісту виявлено грушоподібні най-простіші з двома ядрами, 4-ма парами джгутиків, а у фекаліях овальної форми цисти. Яке захворювання у дитини?

Лямбліоз

Кишковий трихомоноз

Балантидіаз

Токсоплазмоз

Амебіаз

3858 / 6307
Психологічне дослідження встановило: у людини добра здатність швидко пристосовуватися до нового оточення, добра пам’ять, емоційна стійкість, висока працездатність. Найімовірніше, ця людина:

Флегматик з елементами меланхоліка

Меланхолік

Сангвінік

Флегматик

Холерик

3859 / 6307
В організмі людини визначено порушення обміну мелатоніну. Це може бути пов’язано з нестачею амінокислоти, з якої мелатонін синтезується. Яка це амінокислота?

ДОФА

Гістидин

Триптофан

Глутамат

Аланін

3860 / 6307
У хворих із синдромом набутого імунодефіциту (СНІД) різко знижується імунологічна реактивність, що проявляється розвитком хронічних запальних процесів, інфекційних захворювань, пухлинного росту. Клітини якого типу ушкоджує ВІЛ-інфекція, внаслідок чого знижується імунний захист?

Т-супресори

В-лімфоцити

Т8-ефектори

Природні кілери (NК)

Т4-хелпери

3861 / 6307
При глікогенозі (хворобі Гірке) пригнічується перетворення глюкозо-6- фосфату в глюкозу, що супроводжується порушенням розпаду глікогену в печінці. Дефіцит якого ферменту є причиною цього захворювання?

Фосфоглюкомутази

Глікогенфосфорилази

Глюкозо-6-фосфатази

Глюкозо-6-фосфатдегідрогенази

Фосфофруктокінази

3862 / 6307
У хворого шкіра чутлива до сонячного світла. Назвіть це спадкове захворювання, зумовлене дефектами ферментів системи репарації ДНК:

Пігментна ксеродермія

Порфірія

Хвороба Леша-Ніхана

Альбінізм

Вітиліго

3863 / 6307
Ряд антибіотиків є специфічними інгібіторами процесу трансляції в мікроорганізмах. Робота яких органел порушується при цьому?

Мікротрубочок

Рибосом

Пероксисом

Лізосом

Мітохондрій

3864 / 6307
При недостатньому харчуванні часто має місце білкове голодування, яке проявляється зниженням вмісту білка в плазмі крові й розвитком набряків. За рахунок яких білків плазми більшою мірою розвивається зниження онкотичного тиску при цьому?

Бета-глобулінів

Гамма-глобулінів

Альбумінів

Альфа-глобулінів

Фібриногену

3865 / 6307
Для профілактики атеросклерозу, ішемічної хвороби серця та порушень мозкового кровообігу людина повинна одержувати 2-6 г незамінних поліненасичених жирних кислот на добу. Ці кислоти необхідні для синтезу:

Жовчних кислот

Стероїдів

Вітамінів групи D

Простагладинів

Адреналіну

3866 / 6307
Жінка 35-ти років розпочала голодування. Депо яких поживних речовин використовується у початковий період голодування і як при цьому змінюється дихальний коефіцієнт (ДК)?

Жири, ДК наближається до 0,72

Вуглеводи, ДК наближається до 1

Жири, ДК наближається до 0,85

Білки, ДК наближається до 1

Білки, ДК наближається до 0,7

3867 / 6307
На мікропрепараті серця розрізняємо кардіоміоцити зірчастої форми з центрально розташованим ядром, розвиненими гранулярною ендоплазматичною сіткою, апаратом Гольджі та специфічними гранулами. З цими клітинами пов’язана така функція:

Захисна

Ендокринна

Скорочення

Проведення імпульсу

Регенераторна

3868 / 6307
43-річний хворий надійшов в нефрологічне відділення з масивними набряками. Два роки лікувався амбулаторно, при цьому постійно відзначався підвищений артеріальний тиск. Двічі лікувався преднізолоном, з позитивним ефектом. У сечі: відносна щільність - 1017, білок - 4,0 г/л, еритроцити - 15-18 в полі зору (вилужені), лейкоцити - 5-7 в полі зору. Яка переважно функція нирок порушена у хворого?

Фільтраційна

Секреторна

Інкреторна

Концентраційна

Реабсорбційна

3869 / 6307
Під час трьох вагітностей у жінки спостерігалися викидні. З анамнезу відомо, що жінка протягом тривалого часу проживала в сім’ї, де була кішка. Яким одноклітинним паразитом, що міг бути причиною викиднів, могла заразитися жінка?

Лямблія

Амеба

Трихомонада

Балантидій

Токсоплазма

3870 / 6307
Хворий звернувся до лікаря зі скаргами на пронос і біль в животі протягом 5-ти днів, підвищену температуру тіла до 37,5°С. Бактеріологічно встановлений діагноз амебна дизентерія. Вкажіть препарат вибору для лікування цього захворювання:

Ітраконазол

Ацикловір

Хітамін

Метронідазол

Фурацилін

3871 / 6307
Хворій 39-ти років, яка протягом 8-ми років не може завагітніти, порадили звернутися до ендокринолога. При обстеженні у хворої виявлено екзофтальм, тремор повік, тахікардію. Захворювання якої ендокринної залози супроводжується такими симптомами?

Епіфіза

Підшлункової

Надниркових

Статевих

Щитоподібної

3872 / 6307
На заняттях з лікувальної фізкультури лікар-фізіотерапевт запропонував юнакам відхилитися назад і дістати долонями до підлоги. Яка зв’язка запобігає надмірному розгинанню хребтового стовпа?

Жовта

Надостьова

Міжпоперечна

Передня поздовжня

Задня поздовжня

3873 / 6307
Дитина 3-х років померла від гострої пневмонії на тлі хронічної серцевої недостатності. На аутопсії: дефект міжшлуночкової перегородки, стеноз устя легеневої артерії, гіпертрофія правого шлуночка серця, декстрапозиція аорти. Яка вада серця у дитини була встановлена в ході аутопсії?

Пентада Фалло

Синдром Марфана

Синдром Патау

Тетрада Фалло

3874 / 6307
У хворого, який помер від уремії, на розтині виявлена деформація хребетного стовпа з різким обмеженням рухливості. Суглобові хрящі дрібних суглобів хребта зруйновані, є виражені ознаки тривалого поточного хронічного запалення в тканинах суглобів, порожнини суглобів заповнені сполучною тканиною, місцями кістковою з формуванням анкілозів. В аорті, серці та легенях наявні хронічне запалення та вогнищевий склероз. У нирках спостерігається амілоїдоз. Який діагноз у цьому випадку найімовірніший?

Хвороба Педжета (деформівний остоз)

Анкілозивний спондилоартрит (хвороба Бехтєрєва)

Паратиреоїдна остеодистрофія

Остеопетроз (мармурова хвороба)

Ревматоїдний артрит

3875 / 6307
При дефіциті біотину синтез вищих жирних кислот знижується. Внаслідок недостатньої активності якого ферменту це відбувається?

Еноїлредуктази

Цитратсинтетази

Бета-кетоацилредуктази

Ацетил-КоА-карбоксилази

Піруватдегідрогенази

3876 / 6307
Хворому поставили попередній діагноз інфаркт міокарда. Характерною ознакою цього захворювання є суттєве підвищення в крові активності:

Г-6-ФДГ

Кретинфосфокінази

Каталази

Аргінази

Альфа-амілази

3877 / 6307
Хворому на крупозну пневмонію внутрішньом’язово ввели бензилпеніцилін-натрій. Через кілька хвилин у пацієнта розвинувся анафілактичний шок. Який лікарський засіб необхідно ввести хворому?

Адреналіну гідрохлорид

Норадреналіну гідротартрат

Ефедрин

Кофеїн-бензоат натрію

Мезатон

3878 / 6307
Хворий переніс повторний інтрамуральний інфаркт міокарда. Після лікування та реабілітації був виписаний у задовільному стані під нагляд дільничного терапевта. Через 2 роки загинув у автомобільній катастрофі. Який характер патологічного процесу в міокарді було встановлено на розтині?

Дрібновогнищевий кардіосклероз

Атрофія

Великовогнищевий кардіосклероз

Гіперплазія

Некроз

3879 / 6307
У хворого діагностовано пухлину мозку, яка розміщена в ділянці 'пташиної шпори'. Порушення якої функції виникне у хворого, якщо пухлина буде активно розвиватися?

Дотикова чутливість

Зір

Слух

Смак

Нюх

3880 / 6307
У хворого з хронічним гіперацидним гастритом з’явився біль у суглобах. Для полегшення болю, враховуючи супутню патологію, був призначений целекоксиб. Вибіркова дія цього препарату на певний фермент забезпечує відсутність впливу на слизову шлунка. Назвіть цей фермент:

Циклооксигеназа 2

Калікреїн

Циклооксигеназа 1

Фосфоліпаза А2

Фосфоліпаза С

3881 / 6307
Чим пояснити той факт, що для лікування туберкульозу доза ізоніазиду підбирається індивідуально, з обов’язковим контролем після перших прийомів препарату його вмісту в сечі?

Подразнювальною дією препарату

Гіперглікемією, що виникає на тлі приймання препарату

Розвитком гемолітичної анемії

Генетично обумовленою швидкістю ацетилювання препарату у різних людей

Розвитком ниркової недостатності

3882 / 6307
У хворого щорічно навесні та на початку літа в період цвітіння трав і дерев розвивається гостре катаральне запалення кон’юнктиви очей та слизової носової порожнини. Активація та екзоцитоз яких клітинних елементів лежить в основі цього синдрому?

Макрофагів

Тканинних базофілів

Тромбоцитів

Ендотеліальних клітин

Нейтрофілів

3883 / 6307
Хворий 13-ти років скаржиться на загальну слабкість, запаморочення, втомлюваність. Спостерігається відставання у розумовому розвитку. При обстеженні виявлено високу концентрацію валіну, ізолейцину, лейцину в крові та сечі. Сеча має специфічний запах. Що може бути причиною такого стану?

Хвороба Аддісона

Гістидинемія

Тирозиноз

Хвороба кленового сиропу

Базедова хвороба

3884 / 6307
У дитини 6-ти років захворювання почалося гостро з різкого катару в зіві та на мигдаликах, який поширився на слизову оболонку рота, язик ('малиновий язик') та глотку. На поверхні мигдаликів некрози. Місцями внаслідок відторгнення некротичних мас утворюються виразки. Шийні лімфовузли збільшені. На тілі, за винятком носогубного трикутника, спостерігається дрібнокрапковий висип яскраво-червоного кольору. Яке захворювання можна припустити?

Ангіна

Менінгококовий назофарингіт

Кір

Скарлатина

Дифтерія

3885 / 6307
Клінічне обстеження хворого дозволило встановити попередній діагноз рак шлунка. В шлунковому соці виявлено молочну кислоту. Який тип катаболізму глюкози має місце у ракових клітинах?

Анаеробний гліколіз

Пентозофосфатний цикл

Глюкозо-аланіновий цикл

Аеробний гліколіз

Глюконеогенез

3886 / 6307
При вимірюванні артеріального тиску у чоловіка 56-ти років встановлено зростання діастолічного артеріального тиску до 100 мм рт.ст. Від якого з наведених факторів переважно залежить величина діастолічного артеріального тиску?

Периферичного опору судин

Об’єму циркулюючої крові

Швидкості кровотоку

Величини ударного об’єму лівого шлуночка

Величини кінцево-діастолічного об’єму лівого шлуночка

3887 / 6307
Артеріальна гіпертензія у хворої 44-х років обумовлена наявністю феохромоцитоми - пухлини мозкового шару наднирників. Антигіпертензивні засоби якої групи найдоцільніше буде призначити?

Альфа-адреноблокатори

Антагоністи кальцію

Бета-адреноблокатори

Гангліоблокатори

Симпатолітики

3888 / 6307
Бактеріологічний метод діагностики був використаний для підтвердження діагнозу газова гангрена у хворого. Які живильні середовища необхідно використовувати для культивування збудника в цьому випадку?

Ендо, Левіна, Плоскірєва

Вільсона-Блера, Кітта-Тароцці

ЖСА, кров’яний агар

Лужний агар

МПА, МПБ

3889 / 6307
Хворому з діагнозом цукровий діабет II типу ендокринолог призначив глібенкламід. Вкажіть основний механізм дії цього засобу:

Пригнічує глюконеогенез

Підсилює метаболізм глюкози

Стимулює секрецію інсуліну бета- клітинами острівців Лангерганса

Активує транспорт глюкози в клітину

Підсилює захоплення глюкози периферичними тканинами

3890 / 6307
Хворому 35-ти років для обстеження очного дна був призначений атропіну сульфат у вигляді очних крапель. Для відновлення акомодації йому закрапали пілокарпіну гідрохлорид, але це не дало бажаного ефекту. Що є причиною відсутності ефекту?

Синергізм

Двосторонній антагонізм

Тахіфілаксія

Односторонній антагонізм

Звикання

3891 / 6307
До хірургічного відділення ЦРЛ надійшов хворий з колотою раною стопи, яку він отримав під час косовиці. Який специфічний препарат необхідно застосувати з метою екстреної пасивної імунопрофілактики правця?

Протиправцева вакцина

Антитоксична сироватка

Вакцина АКДП

Антибіотики

Анатоксин

3892 / 6307
В медико-генетичну консультацію за рекомендацією андролога звернувся чоловік 35-ти років з приводу відхилень фізичного і психічного розвитку. Об’єктивно встановлено: високий зріст, астенічна будова тіла, гінекомастія, розумова відсталість. При мікроскопії клітин слизової оболонки ротової порожнини знайдено в 30% статевий хроматин (одне тільце Барра). Який діагноз найімовірніший?

Хвороба Реклінгаузена

Синдром Клайнфельтера

Хвороба Іценка-Кушинга

Хвороба Дауна

Синдром Ді Джорджі

3893 / 6307
В психіатрії для лікування ряду захворювань ЦНС використовують біогенні аміни. Вкажіть препарат цієї групи, який є медіатором гальмування:

Таурин

Гамма-аміномасляна кислота

Серотонін

Дофамін

Гістамін

3894 / 6307
В експерименті на жабі вивчали міотатичний рефлекс. Однак при роз-тягненні скелетного м’яза, його рефлекторне скорочення не відбулося. На по-рушення функції яких рецепторів слід звернути увагу?

Сухожильних органів Гольджі

Тактильних

Больових

Суглобових

М’язових веретен

3895 / 6307
У жінки під час мейозу відбулося порушення розходження аутосом. Утворилася яйцеклітина із зайвою 18-ю хромосомою. Яйцеклітина запліднюється нормальним сперматозооном. У майбутньої дитини буде синдром:

Едвардса

Патау

Клайнфельтера

Шерешевського-Тернера

Дауна

3896 / 6307
На розтині звертає на себе увагу наступне: тіло молодого чоловіка високого зросту, виражена блідість та еластичність шкіри, доліхоцефалічна будова голови, подовження і контрактура пальців верхніх і нижніх кінцівок (нагадують кінцівки павука), незначне вдавлення грудини всередину, помірно виражений сколіоз. При дослідженні органів грудної порожнини виявили розрив аневризми висхідної частини аорти. Яке захворювання, найімовірніше, мало місце в цьому випадку?

Атеросклероз аорти

Сифілітичний мезоаортит

Синдром Марфана

Синдром Гудпасчера

3897 / 6307
Пацієнт був доставлений до лікарні з такими симптомами: запаморочення, сухість в роті, зіниці сильно розширені, порушення акомодації, тахікардія, утруднення сечовипускання, атонія кишечника. Передозування якого препарату могло викликати ці симптоми?

Каптоприл

Празозин

Фуросемід

Атропіну сульфат

Клофелін

3898 / 6307
На розтині тіла померлого виявлено, що вся права легеня збільшена, щільна, на плеврі нашарування фібрину, на розрізі тканина сірого кольору, з якої стікає каламутна рідина. Для якого захворювання легенів характерна така картина?

Інтерстиціальна пневмонія

Вогнищева пневмонія

Крупозна пневмонія

Гангрена легені

Фіброзивний альвеоліт

3899 / 6307
При деяких гельмінтозах людина може сама виявити гельмінта, оскільки зрілі членики збудника можуть активно виповзати з ануса людини. Це характерно для:

Теніаринхозу

Гіменолепідозу

Теніозу

Ехінококозу

Дифілоботріозу

3900 / 6307
Чоловік 35-ти років захворів гостро, відзначалося підвищення температури до 39°С, з’явилися нежить, кашель, сльозотеча. При огляді слизова носоглотки набрякла, гіперемована з рясним слизовиділенням. Який вид запалення розвинувся в носоглотці?

Катаральне

Фібринозне

Серозне

Гнійне

Геморагічне

3901 / 6307
У чоловіка 33-х років як наслідок спинномозкової травми порушена больова та температурна чутливість, що обумовлено пошкодженням такого шляху:

Спіноталамічного

Латерального спінокортикального

Медіального спінокортикального

Заднього спіномозочкового

Переднього спіномозочкового

3902 / 6307
Під час бігу на короткі дистанції у нетренованої людини виникає м’язова гіпоксія. До накопичення якого метаболіту в м’язах це призводить?

Оксалоацетату

Ацетил-КоА

Кетонових тіл

Лактату

Глюкозо-6-фосфату

3903 / 6307
Студент перкуторно визначає межу серця, яка проектується на передню грудну клітку на рівні хрящів третього ребра. Яку межу серця визначив студент?

Верхню

Праву

Ліву

Верхівку

Нижню

3904 / 6307
У хворого з підозрою на «озену» з носоглотки були виділені грамнегативні палички, які утворювали капсулу на поживному середовищі. Які мікроорганізми спричинили хворобу?

Клебсієли

Сальмонели

Шигели

Хламідії

Мікоплазми

3905 / 6307
Жінка літнього віку перенесла сильний стрес. У крові різко збільшилась концентрація адреналіну і норадреналіну. Які ферменти каталізують процес інактивації катехоламінів?

Глікозидази

Пептидази

Тирозиназа

Карбоксилази

Моноамінооксидази

3906 / 6307
У пацієнта, який тривалий час вживає препарати, що блокують вироблення ангіотензину II, виникли брадикардія, порушення серцевого ритму. Можливою причиною цих розладів є:

Гіпернатріємія

Гіпокаліємія

Гіпокальціємія

Гіперкальціємія

Гіперкаліємія

3907 / 6307
Хворому 68-ми років в комплекс лікування атеросклерозу, ускладненого ішемічною хворобою серця, лікар включив гіполіпідемічний засіб, який знижує вміст в крові переважно тригліцеридів. Який із вказаних препаратів було призначено хворому?

Інсулін

Фенофібрат

Глібенкламід

Анаприлін

Преднізолон

3908 / 6307
У дитини, яка часто хворіє на ангіни та фарингіти, відзначається збіль-шення лімфовузлів і селезінки. Зовнішній вигляд характеризується пастозністю та блідістю, м’язова тканина розвинена слабко. У крові спостерігається лімфоцитоз. Як називається такий вид діатезу?

Лімфатико-гіпопластичний

Геморагічний

Ексудативно-катаральний

Астенічний

Нервово-артритичний

3909 / 6307
У здорової дорослої людини проводять зондування порожнин серця і великих судин. Де розташований зонд, якщо протягом серцевого циклу зареєстровані зміни тиску від 0 до 120 мм рт.ст.?

Передсердя

Лівий шлуночок

Легенева артерія

Аорта

Правий шлуночок

3910 / 6307
Під час гінекологічного обстеження пацієнтці був поставлений діагноз ендометрит (запалення ендометрію). Яка частина маткової стінки уражена запальним процесом?

Серозна оболонка

Адвентиційна оболонка

Навколоматкова клітковина

Слизова оболонка

М ’язова оболонка

3911 / 6307
Хворому на туберкульоз призначено бактерицидний антибіотик, побічним ефектом якого є забарвлення біологічних рідин у червоний колір. Визначте препарат:

Стрептоміцину сульфат

Етамбутол

Рифампіцин

Протіонамід

Ізоніазид

3912 / 6307
Аміак є дуже отруйною речовиною, особливо для нервової системи. Яка речовина бере особливо активну участь у знешкодженні аміаку в тканинах мозку?

Лізин

Аланін

Глутамінова кислота

Гістидин

Пролін

3913 / 6307
Офтальмолог з діагностичною метою (розширення зіниць для огляду очного дна) використав 1% розчин мезатону. Мідріаз, викликаний препаратом, обумовлений:

Блокадою α₁-адренорецепторів

Активацією β₁ -адренорецепторів

Активацією М-холінорецепторів

Активацією α₂-адренорецепторів

Активацією α₁-адренорецепторів

3914 / 6307
В результаті травми у чоловіка 47- ми років пошкоджені передні корінці спинного мозку. Відростки яких нейронів пошкоджені?

Аксони чутливих псевдоуніполярних нейронів

Дендрити чутливих псевдоуніполярних нейронів

Дендрити рухових нейронів й аксони ядер бокових стовпів

Аксони нейронів рухових соматичних та вегетативних ядер

Дендрити й аксони чутливих псевдоуніполярних нейронів

3915 / 6307
У дитини 7-ми років підвищилася температура тіла до 39°С, з’явилися нежить, кон’юнктивіт і кашель. На шкірі відзначалися великоплямисті висипи, при огляді порожнини рота спостерігаються білуваті висипи на слизовій оболонці щік. Раптово з’явилося важке дихання і настала смерть при явищах асфіксії. Який діагноз найімовірніший?

Дифтерія

Кір

Грип

Менінгококовий назофарингіт

Скарлатина

3916 / 6307
У відділення реанімації надійшов хворий після ДТП з однобічним пневмотораксом. Який вид дихання спостерігається у цьому випадку?

Поверхневе рідке

Глибоке часте

Асфіктичне

Поверхневе часте

Поверхневе

3917 / 6307
Під дією УФ-опромінення та інших факторів можуть відбуватися зміни в структурі ДНК. Репарація молекули ДНК досягається узгодженою дією всіх наступних ферментів, ЗА ВИНЯТКОМ:

Аміноацил-тРНК-синтетази

Ендонуклеази

ДНК-полімерази

ДНК-глікозидази

ДНК-лігази

3918 / 6307
Реалізація загального адаптаційного синдрому здійснюється переважно через нейроендокринну систему. Якій з ланок цієї системи належить провідна роль у патогенезі реакції, що розвивається?

Гіпофізарно-адреналова

Гіпофізарно-юкстагломерулярна

Гіпофізарно-адреногенітальна

Гіпофізарно-інсулярна

Гіпофізарно-тиреоїдна

3919 / 6307
Кросинговер - це обмін ділянками гомологічних хромосом у процесі клітинного поділу, переважно в профазі першого мейотичного поділу, іноді в мітозі. Від чого залежить частота кросинговеру?

Від типу хромосоми

Від довжини хромосоми

Від зовнішніх факторів

Від кількості хромосом

Від відстані між генами

3920 / 6307
У людини вимірюють енерговитрати натщесерце, лежачи, в умовах фізичного і психічного спокою, при температурі комфорту. В який час енерговитрати будуть найбільшими?

17-18 годин вечора

7-8 годин ранку

3-4 години ранку

14-16 годин дня

10-12 годин дня

3921 / 6307
Відомо, що в ході альтерації у вогнищі запалення утворюються біологічно активні речовини-медіатори запалення, які відіграють важливу роль у патогенезі цього патологічного процесу. До утворення яких медіаторів призводить активація ліпооксигенази?

Гістаміну

Лейкотрієнів

Простагландинів

Простацикліну

Тромбоксану

3922 / 6307
Внаслідок захворювання нирок у пацієнта відзначаються набряки. В аналізах сечі визначається масивна протеїнурія. Який механізм є основним у виникненні набряків у такого пацієнта?

Підвищення осмотичного тиску плазми крові

Зниження онкотичного тиску тканин

Зниження онкотичного тиску плазми крові

Зниження онкотичного тиску лімфи

Зниження фільтраційного тиску в нирках

3923 / 6307
У дитини при обстеженні виявлено набряклість обличчя та розширення вен верхньої частини тулуба. На рентгенограмі спостерігається збільшення тіні органів середостіння. Ураження якого органу імуногенезу призвело до виникнення клінічних проявів захворювання?

Передні середостінні лімфатичні вузли

Кістковий мозок грудини

Задні середостінні лімфатичні вузли

Діафрагмальні лімфатичні вузли

Тимус

3924 / 6307
Тривалий вплив на організм людини токсичних речовин призвів до руйнування органел, які відповідають за синтез білків у гепатоцитах печінки. Які органели здійснюють синтез білків у гепатоцитах?

Лізосоми

Мітохондрії

Рибосоми

Пероксисоми

Агранулярна ендоплазматична сітка

3925 / 6307
У хворого після травми в скроневій ділянці виявлена епідуральна гематома. Яка артерія була пошкоджена?

Глибока вушна артерія

Верхня барабанна артерія

Середня оболонна артерія

Нижня барабанна артерія

Глибока скронева артерія

3926 / 6307
У хворого після травми втрачена чутливість шкіри на долонній поверхні 5-го пальця лівої кисті. Який нерв пошкоджений?

N. ulnaris

N. musculo-cutaneus

N. radialis

N. теdianus

N. ахillaris

3927 / 6307
Для прискорення загоєння рани слизової оболонки в ротовій порожнині хворому призначено препарат, який являє собою термостабільний білок, що міститься у людини в сльозах, слині, грудному молоці матері, а також його можна виявити в свіжознесеному курячому яйці. Відомо, що він є фактором природної резистентності організму. Назвіть цей білок:

Iнтерлейкін

Iнтерферон

Комплемент

Лізоцим

Iманін

3928 / 6307
В тубдиспансері у хворого на інфільтративну форму туберкульозу легень, який лікувався ізоніазидом, проявилися симптоми В6-гіповітамінозу. Чому ізоніазид призводить до цього явища?

Прискорюється елімінація

Сповільнюється всмоктування вітаміну

Iзоніазид є антагоністом вітаміну В6

Прискорюється біотрансформація

Утворюється міцний зв’язок з білками плазми крові

3929 / 6307
В аналізі крові 35-річного хворого: Нb = 58 г/л, еритроцити = 1,3•1012/л, колірний показник = 1,3, лейкоцити = 2,8•109/л, тромбоцити = 1,1•109/л, ретикулоцити = 2%, ШОЕ = 35 мм/год. Визначаються полісегментовані нейтрофіли, а також тільця Жоллі, кільця Кебота, мегалоцити. Яка це анемія?

Залізодефіцитна

Гемолітична

Постгеморагічна

Гіпопластична

В12-фолієводефіцитна

3930 / 6307
72-year-old female patient suffers from a long-standing heart failure. She presents to the hospital with a cough and dyspnea on exertion for 1 week. Her symptoms worsen at night and she has noticed that her sputum is pink and frothy. Chest auscultation reveals bilateral line crepitations over the lung bases. The physician prescribes a drug that reduces preload. Which of the following is the most likely drug prescribed by the physician?

cetazolamide

Furosemide

Triamterene

Clopamide

Spironolactone

3931 / 6307
A 12-year-old boy presents with progressive clumsiness and difficulty walking. He walks 'like a drunken-man' and has experienced frequent falls. His muscle tone and strength in all 4 limbs are slightly decreased. When he is asked to stand with his eyes closed and with both feet close together, he sways from side to side, unable to stand still. Which of the following brain regions is most likely affected and caused the symptoms described above?

Red nucleus

Substantia nigra

Reticular formation

Right hemisphere

Cerebellum

3932 / 6307
A doctor refers the patient to a gastroenterologist for a stomach acid test and an upper gastrointestinal endoscopy, which revealed that this patient is a heavy acid producer and has a gastric peptic ulcer. Which of the following is the most likely causative organism for this patient’s condition?

Shigella

Leptospira

Listeria

Helicobacter

Salmonella

3933 / 6307
A 21-year-old woman who is a medical student is undergoing evaluation after sticking herself with a needle while drawing blood from a patient. In this case a medical professional is at high risk of getting a blood-transmitted infection. Which of the following diseases is least likely to be transmitted via blood?

HBV

AIDS

SLE

HIV

HCV

3934 / 6307
A 38-year-old woman, who was diagnosed with systemic lupus erythematosus (SLE) 3 years ago, comes to her physician with a complaint of facial swelling and decreased urination that she first noticed 2 weeks ago. She currently takes azathioprine and a corticosteroid. Her vital signs show blood pressure - 150/90 mm Hg, pulse - 91/min., temperature - 36.8°С and respiratory rate - 15/min. On physical examination, the doctor notices erythematous rash on her face exhibiting a butterfly pattern. The laboratory studies reveal hypercholesterolemia, hypertriglyceridemia and proteinuria. Which of the following is the most likely mechanism of SLE’s complication in this patient?

Immune complex-mediated glomerular disease

Acute infection of the kidney

Decrease in renal blood flow (ischemic nephropathy)

Increased plasma oncotic pressure

3935 / 6307
A 24-year-old man undergoes a surgery and during the operation, an organ is excised and sent for histological evaluation. A light microscopic examination reveals the organ encased by thin connective tissue capsule that enters the substance of the lobes to further subdivide the organ into irregular lobular units. Each lobule contains a cluster of follicles filled with colloid. Follicular epithelium consists of low columnar, cuboidal or squamous cells, depending on the level of activity of the follicle. Which of the following organs does this tissue most likely belong to?

Thymus

Pancreas

Thyroid gland

Parotid gland

Parathyroid gland

3936 / 6307
A researcher is investigating the relationship between inflammatory mediators. He performs an experiment, investigating the effect of nonsteroidal anti-inflammatory drugs (NSAIDs) on patients with high-grade fever. His research indicates that certain NSAIDs act as competitively reversible inhibitors of the cyclooxygenase (COX) enzymes. It is known that COX catalyzes the formation of prostaglandins from a certain molecule that itself is derived from the cellular phospholipid bilayer by phospholipase A2. Which of the following molecules is a precursor of an inflammatory mediators mentioned above?

Palmitic acid

Tyrosine

Cholesterol

Arachidonic acid Proopiomelanocortin

3937 / 6307
A medical student studies a waste disposal system in human epithelial cells. During electronic microscopy he reveals the spherical vesicles, surrounded by a membrane and containing many different hydrolytic enzymes. The main function of these organelles is to provide intracellular digestion and protective reactions of the cell. Which of the following organelles is mentioned above?

Endoplasmatic reticulum

Lysosomes

Centrosomes

Mitochondria

Ribosomes

3938 / 6307
A biology graduate student is performing an experiment in the immunology laboratory. He studies a blood cell count from a patient with acute appendicitis, which shows an increase in the number of cells having a multilobed nucleus and multiple cytoplasmic granules. These cells engulf pathogens or necrotic tissue and help in the degradation of foreign products. Which of the following procescsses is seen in the cell desbribed above?

Passive diffusion

Phagocytosis

Osmosis

Parietal digestion

Pinocytosis

3939 / 6307
A patient suffers from a condition which, is characterized by a restriction in blood supply to tissues which leads to inadequate oxygen delivery to cells and contravention of cell metabolism. It is often caused by partial or total blockage of arteries. Which of the following is developed in this patient?

Spasm

Inflammation

Embolism

Hypoxia

Ischemia

3940 / 6307
A bone marrow biopsy of an 8-year-old girl shows a group of cells which has undergone the process of pyknosis and loss of the nucleus during its differentiation. Which of the following types of hemopoiesis is characterised by the morphological changes described above?

Granulocytopoesis

Lymphocytopoesis Thrombocytopoesis

Erytropoesis

Monocytopoesis

3941 / 6307
The healthcare provider performs a complete blood count to find out if the bone marrow is making the right number of blood cells. He studies a blood cell that has no nucleus and has a function to react to bleeding of an injured blood vessel by clumping, thereby initiating a blood clot. Which of the following is the main object of testing?

Platelets Stem cells

Prothrombine

Macrophages

Leukocytes

3942 / 6307
With total starvation the only source of water for the body is the oxidation process of organic compounds. Which of the following substances under these conditions is the main source of endogenic water?

Carbohydrates

Glycoproteins

Proteins

Lipids

Lipoproteins

3943 / 6307
An 18-year-old girl comes to her physician with concern about her health because she has not achieved menarche. She denies any significant weight loss, changes in mood, or changes in her appetite. She mentions that her mother told her about mild birth defects, but she cannot recall the specifics. Past medical history and family history are benign. On physical examination, the patient is short in stature, has a short and webbed neck and wide chest. Staining of buccal smear reveals absence of Barr bodies in the nucleis of epithelial cells. A urine pregnancy test is negative. Which of the following genetic disorders is the most likely cause of this patient’s condition?

Edwards syndrome

Klinefelter syndrome

Patau syndrome

Turner syndrome

Cri du chat ('cat-cry') syndrome

3944 / 6307
A 6-year-old boy is brought to the pediatrician by his mother, who complains of low-grade fever, chronic cough and night sweats in her child. She describes the cough as productive, producing white sputum that is sometimes streaked with blood. She also says that her son has lost some weight in the last month. His vital signs include blood pressure of 115/75 mm Hg, heart rate of 110/min., respiratory rate of 18/min. and temperature of 36.6°On physical examination, the patient is ill looking. Pulmonary auscultation reveals some fine crackles in the right upper lobe. The pediatrician suspects an active infection and performs Mantoux test. Intradermal injection of which of the following substances has been most likely used by pediatrician for the screening test in this clinical case?

Diphtheria-tetanus toxoids-acellular pertussis vaccine (DTaP)

Bacillus Calmette-Guerin (BCG) vaccine

Tuberculin

Tetanus and diphtheria toxoids vaccine (Td)

3945 / 6307
A research group is investigating a complex of three enzymes. They have created cultures of myocytes derived from high-performance college athletes and simulated starvation conditions. After the experiment they concluded that during starvation the amount of this complex in the muscle tissue was higher. The complex converts pyruvate into acetyl-coenzyme-which enters the citric acid cycle (Krebs cycle) under aerobic conditions. This reaction also involves the further reduction of NAD+ molecules into NADH. An activating effect of which of the following enzymes is described above?

Hexokinase

Lactate dehydrogenase

Phosphofructokinase

Phosphorylase

Pyruvate dehydrogenase

3946 / 6307
This action is an involuntary and nearly instantaneous movement in response to a stimulus. It is made possible by neural pathways which can act on an impulse before that impulse reaches the brain and does not require conscious thought. Which of the following actions is described above?

Reflex

Initiation

Neuralgia

Defense

Tetanus

3947 / 6307
A 16-year-old girl concerned about her sexual development comes to the physician. She mentions that she has still not had a menstrual period. However, she is otherwise a healthy girl with no significant, medical problems since birth. On physical examination, her vital signs are stable. She does not have pubic hair and her breast is slightly elevated with areola remaining in contour with surrounding breast. Which of the following is the most likely cause of this abnormal physical development?

Pancreatic islet insufficiency

Adrenal medulla hyperfunction

Ovarian insufficiency

Hypothyroidism

Hyperthyroidism

3948 / 6307
A 28-year-old male patient presents to the physician with pain and discomfort in his right lower extremity. On physical exam, palpation of the patient’s pulses reveals the absence of pulsation in the dorsalis pedis and tibialis posterior arteries, while the pulsation on the femoral artery is present. The skin of his extremity is also pale and cold/ Which of the following arteries is most likely damaged in this patient?

External iliac artery

Popliteal artery

Deep femoral artery

Descending genicular artery

Tibial artery

3949 / 6307
A pathologist studies a specimen of the small bean-shaped structure which is the part of human immune system. In a cross section it consists of an outer layer (cortex) and inner layer (medulla), and is surrounded by a fibrous capsule and subscapular sinus and is about 1.8 cm long. Which of the following is being studied by the pathologist?

Parathyroid gland

Lymph node

Spleen

Salivary gland

Thymus

3950 / 6307
A 27-year-old female presents with a severe sore throat, hoarseness, painful swallowing and low-grade fever. On intraoral examination, a large grey membrane is noticed on the oropharynx. Removal of the membrane reveals a bleeding oedematous mucosa. Which of the following is the most likely diagnosis?

Streptococcal pharyngitis

Diphtheria

Meningococcal disease

Scarlet fever

Measles

3951 / 6307
The doctor evaluates his patient’s spyrography. One of the evaluation parameters represents the normal amount of air displaced between normal inhalation and exhalation without any extra efforts or appointments. Which of the following is being evaluated in this case?

Tidal volume

Vital volume

Total lung capacity

Inspiratory capacity

Residual volume

3952 / 6307
A 37-year-old man is admitted to a hospital with mental confusion and disorientation. His wife reports he became more irritable and forgetful in the past year. In addition, she notes that he became a vegan a year ago, and currently, his diet consists of starchy foods like potatoes, corn, and leafy vegetables. GI symptoms include anorexia, diarrhea and vomiting. He has glossitis and skin lesions that appear as vesicles over the extremities. Eczema-like lesions around the mouth, as well as desquamation and roughened skin over the hands, are also present. Neurologic examination reveals symmetrical hypesthesia for all types of sensation in both upper and lower extremities in a 'gloves and socks' distribution. Deficiency in the diet the of which of the following amino acids is the most likely cause of this condition?

Arginine

Threonine

Histidine

Tryptophan

Lysine

3953 / 6307
The main function of the human glands are to produce and release substances that perform a specificfunction in the body. According to the classification there are endocrine and exocrine glands. But also there are glands that may be classified as both. Which of the following glands can be endo-and exocrine simultaneously?

Gastrointestinal

Sebaceous

Lacrimal

Parathyroid

Salivary

3954 / 6307
Physiological solution is an isotonic diluent used to maintain cell integrity and viability in procedures that -require preparation of test suspension of organisms. This sterile solution in water provides osmotic protection for microbial cells. Which of the following concentrations of NaCl is considered to be physiological?

10%

0.9%

5%

9%

9%

3955 / 6307
A 60-year-old man with a history of hypertension, diabetes and hyperlipidemia had a sudden onset of right-sided weakness. By the time the ambulance arrived, he had difficulty speaking. Unfortunately, the patient died within the next 2 hours and an autopsy was performed immediately. The gross examination of the cerebral left hemisphere showed brain swelling, widened gyri and poorly demarcated gray-white junction. Which of the following is the most likely cause of this patient’s death?

Intracerebral hemorrhage

Ischemic stroke

Tumor

Cyst

Abscess

3956 / 6307
A 14-year-old girl presents to the emergency department for evaluation of an 'infected leg'. She states there is no history of trauma but mentions she had a history of sickle cell disease/ On physical examination, her upper part of the right shin is very painful, red, swollen, and hot. Her temperature is 39.2°An X-ray shows focal bony lysis and loss of trabecular architecture in the metaphysis of the right tibiIncreased activity of which of the following cells is the most likely cause of bone reabsorption in this patient?

Osteocytes

Osteoblasts

Osteoclasts

Chondrocytes

Chondroblasts

3957 / 6307
A 46-year-old man presents with fatigue and joint pain in his fingers and wrists for the last 2 months. The pain is present in both hands and the wrists are swollen. Furthermore, he describes morning stiffness in his joints lasting about 2 hours, which improves with use. His past medical history reveals he has been successfully treated for H. pylori related ulcers last year. He denies smoking and stopped drinking when his gastric symptoms started. Which of the following drugs is the best choice for his joints’ pain management?

Morphine

Aspirin

Prednisone

Paracetamol

Celecoxib

3958 / 6307
A 20-year-old female comes to the clinic after missing her last 2 periods. Her cycles are usually regular, occurring at 28-30 day interval with moderate bleeding and some abdominal discomfort. She also complains of progressively diminishing peripheral vision. Her doctor reveals loss of vision in the lateral halves of both eyes. Involvement of which of the following structures would you most likely expect to be the reason of bitemporal hemianopsia?

Left optic tract

Left optic nerve

Optic chiasm

Right optic nerve

Right optic tract

3959 / 6307
A 25-year-old male presents to an emergency department with suddenonset right-sided pleuritic chest pain and breathlessness. On examination, he has arterial blood saturation of 90% in air and is afebrile. An X- ray shows a lung edge visible in the right hemithorax, beyond which no lung markings are seen. Which of the following conditions has most likely occured in this patient?

Pneumothorax

Pleurisy

Aerophagy

Pneumonia

Rib fracture

3960 / 6307
На прийом до терапевта прийшов чоловік 37-ми років зі скаргами на періодичні інтенсивні больові напади у суглобах великого пальця стопи та їх припухлість. При аналізі сечі встановлено її різко кислий характер і рожеве забарвлення. З наявністю яких речовин можуть бути пов’язані такі зміни сечі?

Хлоридів

Амонієвих солей

Солей сечової кислоти

Фосфату кальцію

Сульфату магнію

3961 / 6307
У хворого на цукровий діабет з’явився різкий біль у правій стопі. При огляді великий палець стопи чорного кольору, тканини стопи набряклі, осередки відшарування епідермісу, виділення з неприємним запахом. Яка клініко- морфологічна форма некрозу розвинулася в хворого?

Інфаркт

Секвестр

Іангрена волога

Пролежень

Гангрена суха

3962 / 6307
На розтині трупа жінки 63-х років, що страждала на ревматизм і комбіновану мітральну ваду, виявлено, що стулки мітрального клапана різко потовщені, зрощені між собою, кам’янистої щільності, під час розрізання визначається хрускіт. Який патологічний процес зумовив кам’янисту щільність клапана серця?

Метаболічне звапнення

Амілоїдоз

Фібриноїд

Метастатичне звапнення

Дистрофічне звапнення

3963 / 6307
У клітину потрапив вірус грипу. Трансляція при біосинтезі вірусного білка в клітині буде здійснюватися:

У лізосомах

На каналах гладенької ендоплазматичної сітки

У клітинному центрі

На полірибосомах

У ядрі

3964 / 6307
У обстежуваного визначили дихальний об’єм (500 мл), частоту дихання (15 за хвилину), об’єм мертвого простору (100 мл). Скільки повітря пройде у нього за хвилину через альвеоли?

1500 мл

7500 мл

9000 мл

7400 мл

6000 мл

3965 / 6307
Експериментатору необхідно якнайшвидше виробити умовний рефлекс у собаки. На базі якого безумовного рефлексу доцільно виробляти умовний?

Орієнтувальний

Травний

Міотатичний

Захисний

Статевий

3966 / 6307
Подразнення правого блукаючого нерва спричинило різке сповільнення атріовентрикулярного проведення. На ЕКГ про це буде свідчити подовження:

Комплексу QRST

Зубця Т

Інтервалу RR

Зубця Р

Інтервалу PQ

3967 / 6307
Хворому було призначено препарат. Через декілька діб дія препарату значно знизилась і для отримання початкового ефекту потрібно збільшити дозу речовини. Вкажіть назву цього типу зміни дії лікарської речовини:

Звикання

Ідіосинкразія

Кумуляція

Лікарська залежність

Тахіфілаксія

3968 / 6307
У жінки 62-х років розвинулася катаракта (помутніння кришталику) на тлі цукрового діабету. Вкажіть, який тип модифікації білків має місце при діабетичній катаракті:

Фосфорилювання

Обмежений протеоліз

Метилювання

АДФ-рибозилювання

Ілікозилювання

3969 / 6307
При кесаревому розтині у хворої 32-х років, внаслідок сильної кровотечі і не-можливості її зупинити, оператор вимушений був видалити матку. За рахунок яких судин здійснюється кровопостачання матки?

Arteria obturatoria

Arteria pudenda interna

Arteria clitoridis

Arteria sacralis lateralis

Arteria uterina

3970 / 6307
У 6-місячної дитини спостерігались часті та сильні підшкірні кровотечі. Призначення синтетичного аналога вітаміну К (вікасола) дало позитивний ефект. В гамма-карбоксилюванні глута- мінової кислоти якого з перерахованих нижче білків згортальної системи крові бере участь цей вітамін?

Фібриногену

Фактора Розенталя

Фактора Хагемана

Протромбіну

Антигемофільного глобуліну А

3971 / 6307
У хворого виявлена серповидноклі- тинна анемія. Заміна якої амінокислоти в поліпептидному ланцюгу НЬ на валін призводить до цього захворювання?

Лейцину

Аспарагінової кислоти

Аргініну

Ілутамінової кислоти

Треоніну

3972 / 6307
Під час підйому пішки на 5-й поверх у людини підвищився артеріальний тиск. Причиною є збільшення:

В’язкості крові

Кількості функціонуючих капілярів

Об’єму циркулюючої крові

Хвилинного об’єму крові

Вмісту іонів в плазмі крові

3973 / 6307
В приймальне відділення надійшла дитина 1,5 років з ознаками отруєння нітратами: стійкий ціаноз, задишка, судоми. Утворення якої форми гемоглобіну лежить в основі цих симптомів?

Метгемоглобіну

Редукованого гемоглобіну

Карбоксигемоглобіну

Оксигемоглобіну

Карбгемоглобіну

3974 / 6307
Внаслідок росту пухлини в порожнину III шлуночка головного мозку у пацієнта розвиваються вегетативні розлади у вигляді порушення сну, терморегуляції, всіх видів обміну, нецукровий діабет. Подразнення ядер якої ділянки головного мозку викликало ці симптоми?

Довгастого мозку

Моста

Покришки середнього мозку

Ніжок мозку

Гіпотапамуса

3975 / 6307
Хворому призначено препарат дигоксин. Через декілька днів у хворого виявлено ознаки передозування цим препаратом, його вміст у крові значно перевищував верхню межу терапевтичної концентрації. Як називається такий варіант дії лікарських речовин?

Антагонізм

Тахікардія

Кумуляція

Звикання

Потенціювання

3976 / 6307
В результаті травми у чоловіка 47-ми років пошкоджені передні корінці спинного мозку. Відростки яких нейронів пошкоджені?

Дендрити й аксони чутливих псевдоуніполярних нейронів

Дендрити рухових нейронів й аксони ядер бокових стовпів

Дендрити чутливих псевдоуніполярних нейронів

Аксони нейронів рухових соматичних та вегетативних ядер

Аксони чутливих псевдоуніполярних нейронів

3977 / 6307
У зоопарк доставлені антилопи з Африки. В їх крові виявлено Trypanosoma brucei gambiense. Чи становлять ці тварини епідеміологічну небезпеку?

Небезпека лише для хижаків

Епідеміологічної небезпеки немає

Небезпека для домашніх тварин і людини

Небезпека для інших антилоп

Небезпека лише для людини

3978 / 6307
Після травматичного ураження промежини у потерпілого спостерігається нетримання сечі. Який м’яз був травмований?

Глибокий поперечний м’яз промежини

Сіднично-печеристий м’яз

Поверхневий поперечний м’яз промежини

Внутрішній стискач відхідника

Зовнішній стискач сечівника

3979 / 6307
Хворого з явищами енцефалопатії госпіталізували в неврологічний стаціонар і виявили кореляцію між наростанням енцефалопатії і речовинами, які надходять з кишечника в загальний крово- тік. Яка сполука, що утворюється в кишечнику, може бути причиною такого стану?

Індол

Бутират

Орнітин

Ацетоацетат

Біотин

3980 / 6307
До клініки доставили пацієнта 32-х років з масивною крововтратою внаслідок ДТП. Ps- 110/хв., ЧДР- 22 за 2 хв, АТ- 100/60 мм рт.ст. Яка зміна крові з перерахованих буде найбільш характерною через 1 годину після крововтрати?

Гіпохромія еритроцитів

Еритропенія

Гіпопротеїнемія

Лейкопенія

Гіповолемія

3981 / 6307
Людина отримала електротравму При цьому струм пройшов крізь серцевий м’яз. Які небезпечні порушення в роботі серця можуть виникнути у цій ситуації, що вимагають термінового втручання?

Фібриляція шлуночків

Екстрасистолія

Тахікардія

Атріовентрикулярна блокада

Брадикардія

3982 / 6307
Після тривалого вживання одного з лікарських препаратів у хворого розвинулась підвищена чутливість до простудних захворювань. Який з вказаних нижче лікарських засобів міг сприяти зниженню імунітету?

Сустак-форте

Клофелін

Настойка женьшеню

Преднізолон

Резерпін

3983 / 6307
Дитина 5-ти років після невдалого падіння з дивану скаржиться на різкий біль у шиї, що виникає при найменших рухах. Огляд виявив припухлість, порушення цілісності в ділянці правої ключиці, крепітацію. Що слід ввести в першу чергу для швидкого усунення болю і заспокоєння дитини?

Анальгін

Парацетамол

Кодеїн

Діазепам

Промедол

3984 / 6307
Чоловіку 70-ти років, який страждає на хворобу Паркінсона, був призначений препарат леводопа. Через тиждень стан хворого значно покращився. Який механізм дії лікарського засобу?

Іальмування гістамінергічної системи

Активація дофамінової системи

Активація енкефалінової системи

Іальмування серотонінергічної системи

Іальмування холінергічної системи

3985 / 6307
Жінка 37-ми років звернулася до гінеколога з приводу запального процесу піхви, який супроводжується свербінням та пінистими виділеннями. Бакана- ліз встановив наявність трихомонадної інфекції. Який засіб найбільш ефективний у даному випадку?

Метронідазол

Ністатин

Ампіцилін

Нітроксолін

Еритроміцин

3986 / 6307
При обстеженні хворого на гемофілію виявлено зміну деяких показників крові. Яка з перерахованих ознак відповідає цьому захворюванню?

Еозинофілія

Тромбоцитопенія

Час згортання крові вповільнений

Еритроцитоз

Афібриногенемія

3987 / 6307
У хворого, який страждав на хронічний алкоголізм і цироз печінки, розвинулася профузна кровотеча з варикозно розширених вен стравоходу, в результаті чого настала смерть. На аутопсії печінка дрібногорбиста, зменшена в розмірах, щільна, жовтуватого кольору. Під час гістологічного дослідження кріостатних зрізів печінки (забарвлення гематоксиліном і еозином) в гепатоцитах виявляються великі оптично порожні вакуолі, в яких міститься речовина, що забарвлюється в чорний колір при застосуванні осмієвої кислоти. Оптично порожні вакуолі гепатоцитів - це:

Гідропічна дистрофія

Алкогольний гіалін (тільця Меллорі)

Великокрапельна жирова дистрофія

Псевдовакуолі гіалоплазми

Включення гіаліну

3988 / 6307
Дослідник в ході мікроскопічного і електронно-мікроскопічного вивчення печінки звернув увагу, що деякі окремо розташовані клітини розпалися на дрібні фрагменти, оточені мембраною. У деяких з них є органели, інші включають в себе фрагменти ядра, що розпало-ся. Запальна реакція навколо була відсутня. Він розцінив ці зміни як прояв:

Дистрофії

Атрофії

Апоптозу

Некрозу

Гіпоплазії

3989 / 6307
У гістопрепараті визначається орган, що складається з сірої та білої речовини. Сіра речовина розташовується на периферії й має 6 шарів: молекулярний, зовнішній зернистий, пірамідний, внутрішній зернистий, гангліонарний і шар поліморфних клітин. Визначте утворення, якому належать дані морфологічні ознаки:

Кора великих півкуль

Довгастий мозок

Спинномозковий вузол

Спинний мозок

Мозочок

3990 / 6307
Машиною швидкої допомоги доставлено до приймального відділення лікарні людину з гострим отруєнням морфіном. Який із найбільш специфічних антагоністів необхідно застосувати в цьому випадку?

Налоксон

Омнопон

Метацин

Пентазоцин

Дипіроксим

3991 / 6307
Хворий переніс повторний інтраму- ральний інфаркт міокарда. Після лікування та реабілітації був виписаний у задовільному стані під нагляд дільничного терапевта. Через 2 роки загинув у автомобільній катастрофі. Який характер патологічного процесу в міокарді було встановлено на розтині?

Некроз

Гіперплазія

Дрібновогнищевий кардіосклероз

Атрофія

Великовогнищевий кардіосклероз

3992 / 6307
Хвора на бронхіальну астму приймала протягом 3-х місяців преднізолон у таблетках. Внаслідок значного покращення стану раптово припинила його прийом. Розвиток якого ускладнення ви- сокоймовірний у цьому випадку?

Шлункової кровотечі

Гіпотонії

Синдрому відміни

Синдрому Іценка-Кушинга

Ожиріння верхньої половини тулуба

3993 / 6307
У клініку госпіталізований хворий з отруєнням. Встановлено, що в печінці порушені механізми детоксикації. Які з органел гепатоцитів в першу чергу обумовили цей стан?

Мітохондрїї

Гранулярна ендоплазматична сітка

Агранулярна ендоплазматична сітка

Комплекс Іольджі

Рибосоми

3994 / 6307
На земній кулі існують території (біохімічні провінції) частина населення яких страждає на ендемічний зоб. Дефіцит якого біоелемента в грунті, воді та харчових продуктах викликає це захворювання?

Мідь

Цинк

Кобальт

Залізо

Йод

3995 / 6307
У 12-річного хлопчика в сечі виявлено високий вміст усіх амінокислот аліфатичного ряду. При цьому відзначена найвища екскреція цистину та цистеїну. Крім того, УЗД нирок показало наявність у них каменів. Виберіть можливу патологію:

Цистинурія

Цистит

Фенілкетонурія

Хвороба Хартнупа

Алкаптонурія

3996 / 6307
Внаслідок захворювання нирок у пацієнта відзначаються набряки. В аналізах сечі визначається масивна протеїну- рія. Який механізм є основним у виникненні набряків у такого пацієнта?

Зниження фільтраційного тиску в нирках

Зниження онкотичного тиску тканин

Зниження онкотичного тиску плазми крові

Зниження онкотичного тиску лімфи

Підвищення осмотичного тиску плазми крові

3997 / 6307
У пацієнта з гнійничковими ураженнями шкіри виділено збудник, який на кров’яному агарі утворює жовті колонії округлої форми, середніх розмірів, оточені зоною гемолізу. В мазках з колоній містяться коки, розташовані скупченнями неправильної форми, Гр+. Ви-ділена культура є оксидазо- та каталазопозитивною, ферментує маніт, синтезує плазмокоагулазу Якому виду з нижче- перерахованих відповідає виділений збудник?

Staphylococcus aureus

Streptococcus agalactiae

Staphylococcus saprophyticus

Staphylococcus epidermidis

Streptococcus pyogenes

3998 / 6307
У хворого на цукровий діабет виникла значна спрага, дисфагія та порушення психічної діяльності. Який тип розладів водно-електролітного балансу характеризує поява вказаних ознак?

Дегідратація гіперосмотична

Дегідратація ізоосмотична

Дегідратація гіпоосмотична

Гідратація ізоосмотична

Гідратація гіпоосмотична

3999 / 6307
Потерпілий обробляв рослини розчином речовини з інсектицидною дією без індивідуальних засобів захисту. Через деякий час у нього почалося сильне виділення слини, поту, сліз, біль у животі, понос. При огляді виявлено міоз. Речовина, що викликала отруєння, належить до групи:

Солі міді

Н-холіноміметики

Нітрати

Антихолінестеразні засоби

Органічні сполуки хлору

4000 / 6307
Пацієнт 64-х років має гостру серцеву недостатність, артеріальний тиск - 80/60 мм рт.ст., добовий діурез - 530 мл, істотно збільшену концентрацію сечовини та креатиніну в крові. Назвіть патогенетичний механізм розвитку азотемії та олигурії:

Гіпернатріємія

Збільшення продукції вазопресину

Зменшення об’єму циркулюючої крові

Спазм приносних артеріол клубочка

Зменшення фільтраційного тиску в нирках

4001 / 6307
Людина довгий час проживала в умовах високогір’я. Які зміни в кровоносній системі будуть у неї?

Зниження кількості лейкоцитів

Збільшення діаметра кровоносних судин

Вповільнення пульсу

Збільшення кількості гемоглобіну

Збільшення кількості лейкоцитів

4002 / 6307
При підйомі в гори у альпініста розвинулась ейфорія, яка замінилася головним болем, запамороченням, серцебиттям, задишкою, що перейшла в апное. Яке порушення кислотно-лужної рівноваги розвинулось в даному випадку?

Іазовий ацидоз

Газовий алкалоз

Негазовий ацидоз

Негазовий алкалоз

Видільний алкалоз

4003 / 6307
У дівчинки 6-ти років пастозність повік, губ, шиї, слизової оболонки язика виникла після того, як вона з’їла апельсин. Раніше на апельсини виникали висипання на шкірі, свербіння. Який патогенетичний механізм є провідним у розвитку набряку у дівчинки?

Підвищення гідростатичного тиску крові в капілярах

Підвищення онкотичного тиску тканинної рідини

Зниження онкотичного тиску крові

Порушення лімфовідтоку

Підвищення проникності капілярів

4004 / 6307
На прийом до лікаря прийшов пацієнт дуже високого зросту, з довгими товстими пальцями рук, великою нижньою щелепою і відвислою нижньою губою. Підвищену секрецію якого гормону якої залози можна припустити?

Гормонів щитоподібної залози

Гонадотропного гормону передньої частки гіпофіза

Антидіуретичного гормону задньої частки гіпофіза

Гормонів наднирників із групи глюко- кортикоїдів

Соматотропного гормону передньої частки гіпофіза

4005 / 6307
Під час гістологічного дослідження щитоподібної залози, видаленої в ході операції, виявлена деструкція й атрофія фолікулів, дифузна лімфоцитарна інфільтрація з формуванням лімфоїдних фолікулів в стромі. До якої групи захворювань належить такий тиреоїдит?

Аутоімунних

Вірусних

Викликаних фізичними факторами

Бактеріальних

Інфекційно-алергічних

4006 / 6307
Дівчинка 4-х років на 3-ю добу від початку захворювання на дифтерію померла від справжнього крупу На ауто- псії слизова оболонка гортані, трахеї та бронхів потовщена, набрякла, тьмяна, вкрита сіруватою плівкою, що легко відокремлюється. Визначити вид ексудативного запалення гортані:

Змішане

Катаральне

Серозне

Гнійне

Фібринозне

4007 / 6307
У хворого внаслідок отруєння сулемою розвинулася гостра ниркова недостатність, перебіг якої включав 4 стадії: перша - початкова, друга - оліго-, анурії, четверта - одужання. Як називається третя стадія гострої ниркової недостатності?

Патохімічна

Поліурична

Ішемічна

Метаболічна

Іемодинамічна

4008 / 6307
Злоякісна гіперхромна анемія хвороба Бірмера виникає внаслідок нестачі вітаміну В12. Який біоелемент входить до складу цього вітаміну?

Залізо

Кобальт

Магній

Цинк

Молібден

4009 / 6307
У хворого час від часу суттєво знижується вміст гемоглобіну і еритроцитів в крові, з’являється жовтяниця. З анамнезу з’ясовано, що ці напади завжди виникають після вживання кінських бобів. Яка з форм анемії має місце у даного хворого?

Залізодефіцитна анемія

Спадкова мембранопатія

Набута гемолітична анемія

Спадкова ферментопатія

Спадкова гемоглобінопатія

4010 / 6307
На розтині тіла померлого виявлено, що вся права легеня збільшена, щільна, на плеврі нашарування фібрину, на розрізі тканина сірого кольору, з якої стікає каламутна рідина. Для якого захворювання легенів характерна така картина?

Іангрена легені

Вогнищева пневмонія

Фіброзивний альвеоліт

Крупозна пневмонія

Інтерстиціальна пневмонія

4011 / 6307
На розтині хворого, який багато років працював на шахті і помер від хронічної легенево-серцевої недостатності, виявлено, що легені малоповітряні, значно ущільнені, склерозовані, верхівки емфізематозно змінені, поверхня сіро- чорного кольору, на розрізі тканина легенів аспідно-чорного кольору. Від якої хвороби настала смерть?

Талькоз

Антракоз

Алюміноз

Силікоз

Асбестоз

4012 / 6307
У потерпілого травма внаслідок прямого удару по внутрішній поверхні середньої третини гомілки. Перелом якого анатомічного утворення найбільш імовірний?

Проксимальний епіфіз великогомілкової кістки

Дистальний епіфіз великогомілкової кістки

Діафіз великогомілкової кістки

Проксимальний епіфіз малогомілкової кістки

Дистальний епіфіз малогомілкової кістки

4013 / 6307
При мікроскопічному дослідженні легені недоношеної дитини виявлено спадання стінок альвеол через відсутність сурфактанту. Вкажіть, з порушенням розвитку яких клітин стінки альвеоли це пов’язано:

Фібробластів

Альвеолярних макрофагів

Альвеолоцитів II типу

Секреторних клітин

Альвеолоцитів І типу

4014 / 6307
Хворий 70-ти років, який страждав на цукровий діабет та переніс інфаркт міокарда, помер при явищах прогресуючої серцево-судинної недостатності. На розтині ціанотична індурація селезінки та нирок, бура індурація легень та мускатна печінка. Який вид порушення кровообігу обумовив зміни внутрішніх органів? 

Загальна артеріальна гіперемія після анемії

Загальна хронічна венозна гіперемія

Місцева хронічна венозна гіперемія

Загальна гостра венозна гіперемія

Артеріальна ішемія в результаті перерозподілу крові

4015 / 6307
На слизовій оболонці мигдаликів та м’якого піднебіння виявляються білувато-сірого кольору плівки, які щільно з’єднані з підлеглою тканиною. При спробі зняти плівку на її місці виникає глибокий дефект тканини. Визначити патологічний процес, який виник на слизовій оболонці мигдаликів та м’якого піднебіння:

Гнійне запалення

Крупозне запалення

Дифтеритичне запалення

Змішане запалення

Серозне запалення

4016 / 6307
.3 метою ранньої діагностики вагітності досліджується сеча жінки. З’явлення яких гормонів в сечі свідчить про ймовірну вагітність?

Тестостерон

Прогестерон

Хоріонічний гонадотропін

Естріол

17-бета-естрадіол

4017 / 6307
Людина знепритомніла у салоні автомобіля, де тривалий час очікувала приятеля при ввімкненому двигуні. В крові у неї знайдено сполуку гемоглобіну. Яку саме?

Оксигемоглобін

Дезоксигемоглобін

Карбоксигемоглобін

Метгемоглобін

Карбгемоглобін

4018 / 6307
При обстеженні людини встановлено, що хвилинний об’єм серця дорівнює 3500 мл, систолічний об’єм - 50 мл. Якою є у людини частота серцевих скорочень?

80 скорочень за хвилину

60 скорочень за хвилину

50 скорочень за хвилину

90 скорочень за хвилину

70 скорочень за хвилину

4019 / 6307
У людини внаслідок хронічного захворювання печінки суттєво порушена її білковосинтезуюча функція. До зменшення якого параметру гомеостазу це призведе?

Іематокритний показник

Онкотичний тиск плазми крові

Осмотичний тиск

pH

Щільність крові

4020 / 6307
Унітіол є антидотом і застосовується, зокрема, при отруєннях солями важких металів. Як називається такий тип взаємодії лікарських речовин?

Неконкурентний антагонізм

Синергоантагонізм

Фізичний антагонізм

Фізіологічний антагонізм

Хімічний антагонізм

4021 / 6307
Хворому на туберкульоз в комплексній терапії призначено препарат - похідне гідразиду ізонікотинової кислоти. Визначити цей препарат:

Канаміцин

Стрептоміцину сульфат

Рифампіцин

Ізоніазид

Цефалоридин

4022 / 6307
Робітник тваринницької ферми гостро захворів і при наростаючих явищах інтоксикації помер. На розтині встановлено: селезінка збільшена, в’яла, на розрізі темно-вишневого кольору, зішкріб пульпи рясний. М’які мозкові оболонки на склепінні та основі мозку набряклі, просякнуті кров’ю, мають темно-червоний колір ('шапочка кардинала'). Мікроскопічно: серозно- геморагічне запалення оболонок і тканин головного мозку з руйнуванням стінок дрібних судин. Поставте діагноз:

Туляремія

Холера

Сибірка

Бруцельоз

Чума

4023 / 6307
Під час розтину тіла жінки, яка померла внаслідок пухлинної дисемінації муцинозної цистаденокарциноми і тривалий час мала вимушене положення в ліжку, були знайдені великі ділянки некрозу шкіри та підлеглих м’яких тканин крижової ділянки. Назвіть форму некрозу:

Сирнистий некроз

Інфаркт

Воскоподібний (ценкеровський) некроз

Пролежень

Секвестр

4024 / 6307
На розтині чоловіка, який помер на 5-у добу захворювання черевним тифом, виявлені наступні зміни: групові фолікули клубової кишки збільшені, повнокровні і виступають над слизовою оболонкою, на їх поверхні видно борозди та звивини. Гістологічно: повнокрів’я і набряк тканини, наявність гранульом, які складаються з великих клітин зі світлою цитоплазмою і містять черевнотифозні палички. Про який період місцевих змін при черевному тифі можна думати?

Стадія некрозу

Стадія утворення виразок

Стадія мозкоподібного набухання

Стадія загоювання виразок

Стадія чистих виразок

4025 / 6307
У клініці хворому було пересаджено нирку. Які з перерахованих клітин імунної системи можуть безпосередньо впливати на клітини трансплантату?

Т-хелпери

Тимоцити

В-лімфоцити

Плазмобласти

Т-кілери

4026 / 6307
На препараті селезінки виявляється судина, стінка якої складається з базальної мембрани з ендотелієм, середня оболонка відсутня, зовнішня оболонка зрощена зі сполучнотканинними прошарками селезінки. Що це за судина?

Артерія еластичного типу

Артерія м’язового типу

Артеріола

Вена безм’язового типу

Вена м’язового типу зі слабким розвитком м’язових елементів

4027 / 6307
У хворого спостерігається збільшений тонус артеріол за нормальних показників роботи серця. Як це вплине на величину артеріального тиску?

Зросте переважно діастолічний

Зменшиться переважно систолічний

Тиск не зміниться

Зменшиться переважно діастолічний

Зросте переважно систолічний

4028 / 6307
При дослідженні людини у вертикальній позі встановлено, що в альвеолах верхівок легень парціальний тиск кисню становить 140 мм рт.ст. Причиною цього є те, що у цих відділах легень:

Вентиляція переважає над перфузією

Вентиляція відсутня

Перфузія та вентиляція врівноважені

Перфузія переважає над вентиляцією

Е. -

4029 / 6307
Офтальмолог з діагностичною метою (розширення зіниць для огляду очного дна) використав 1 % розчин мезатону. Мідріаз, викликаний препаратом, обумовлений:

Активацією α2-адренорецепторів

Активацією М-холінорецепторів

Активацією β1-адренорецепторів

Блокадою α1-адренорецепторів

Активацією α1-адренорецепторів

4030 / 6307
Чоловік 55-ти років, що скаржиться на біль в ділянці нирок, надійшов в лікарню. В ході ультразвукового обстеження пацієнта виявлено наявність ниркових каменів. Наявність в сечі якої з наведених речовин є найімовірнішою причиною утворення каменів у цього паці-єнта?

Сечова кислота

Білівердин

Креатинін

Білірубін

Уробілін

4031 / 6307
В осередку запалення утворюється біогенний амін, що має судинорозширювальну дію. Назвіть його:

Серотонін

Гістамін

ДОФА

ГАМК

Триптамін

4032 / 6307
При розтині померлого від чуми хворого на фоні геморагічного синдрому знайдено геморагічний некроз шкіри стегна, лімфангіт, пахвинний геморагічний лімфаденіт. Назвіть форму чуми:

Первинно-легенева

Шкірно-бубонна

Первинно-септична

Бубонна

Геморагічна

4033 / 6307
Психологічне дослідження встановило: у людини добра здатність швидко пристосовуватися до нового оточення, добра пам’ять, емоційна стійкість, висока працездатність. Найімовірніше, ця людина:

Флегматик з елементами меланхоліка

Флегматик

Сангвінік

Холерик

Меланхолік

4034 / 6307
Чоловік 40-ка років скаржиться на загальну слабкість, головний біль, кашель із виділенням мокротиння, задишку. Після клінічного огляду й обстеження поставлено діагноз пневмонія. Який тип гіпоксії має місце у хворого?

Тканинна

Гемічна

Циркуляторна

Респіраторна

Гіпоксична

4035 / 6307
Хвора 45-ти років госпіталізована з підозрою на пухлину матки. Після обстеження встановлено діагноз - фіброміома матки. В якому шарі матки розміщується ця пухлина?

Mesometrium

Parametrium

Myometrium

Perimetrium

Endometrium

4036 / 6307
При дослідженні амніотичної рідини, одержаної при амніоцентезі (прокол амніотичної оболонки), виявлені клітини, ядра яких містять статевий хроматин (тільце Барра). Про що з зазначеного це може свідчити?

Розвиток плода чоловічої статі

Трисомія

Розвиток плода жіночої статі

Генетичні порушення в розвитку плода

Поліплоїдія

4037 / 6307
У хворої 43-х років після чергового загострення ревмокардиту з’явилися ознаки декомпенсації серцевої діяльності з виникненням набряків на ногах і асциту. Затримці води в організмі хворої сприяло підвищення продукції:

Тироксину

Кортизолу

Інсуліну

Альдостерону

Кортикотропіну

4038 / 6307
У хворого спостерігається порушення зору - гемералопія ('куряча сліпота'). Який вітамінний препарат треба вживати хворому, щоб відновити зір?

Токоферолу ацетат

Тіаміну хлорид

Вікасол

Піридоксин

Ретинолу ацетат

4039 / 6307
Хвора 38-ми років надійшла в реанімаційне відділення в непритомному стані. Рефлекси відсутні. Цукор крові - 2,1 ммоль/л. В анамнезі: цукровий діабет з 18-ти років. Яка кома має місце у хворої?

Гіпоглікемічна

Лактацидемічна

Гіперглікемічна

Гіперосмолярна

Кетоацидотична

4040 / 6307
У збудливій клітині заблокували іонні канали, внаслідок чого клітина з часом повністю втратила потенціал спокою. Які канали заблокували?

Кальцієві

Натрієві

Калієві

Хлорні

Калієві та натрієві

4041 / 6307
У збудливій клітині заблокували іонні канали. Це не змінило суттєво рівень потенціалу спокою, але клітина втратила здатність до генерації ПД. Які канали заблокували?

Хлорні

Натрієві

Калієві

Кальцієві

Натрієві та калієві

4042 / 6307
У хворого діагностовано пухлину мозку, яка розміщена в ділянці 'пташиної шпори'. Порушення якої функції виникне у хворого, якщо пухлина буде активно розвиватися?

Слух

Нюх

Смак

Дотикова чутливість

Зір

4043 / 6307
У хворого виявлено порушення периферичного кровообігу, основою якого є обмеження припливу артеріальної крові. При цьому має місце збліднення даної ділянки, зниження місцевої температури. Це обумовлене:

Артеріальною гіперемією

Ішемією

Венозною гіперемією

Лімфостазом

Сладж-феноменом

4044 / 6307
Хворому 35-ти років для обстеження очного дна був призначений атропіну сульфат у вигляді очних крапель. Для відновлення акомодації йому закрапали пілокарпіну гідрохлорид, але це не дало бажаного ефекту Що є причиною відсутності ефекту?

Двосторонній антагонізм

Тахіфілаксія

Односторонній антагонізм

Звикання

Синергізм

4045 / 6307
В процесі експерименту на собаці виникла необхідність підвищити збудливість серцевого м’язу За допомогою введення якого іону можливо досягти бажаного ефекту?

Mg2+

К+ С Fe2+

Са2+

С1~

4046 / 6307
Гістони - ядерні білки, які пов’язані з ДНК іонним зв’язком. Які амінокислоти обумовлюють основний характер гісто- нів?

Іліцин та аланін

Лейцин та ізолейцин

Серин і цистеїн

Пролін і метіонін

Аргінін і лізин

4047 / 6307
Для діагностування деяких хромосомних хвороб використовують визначення статевого хроматину. Назвіть хворобу, при якій потрібне це визначення:

Синдром Шерешевського-Тернера

Синдром Патау

Хвороба Брутона

Хвороба Дауна

Гемофілія

4048 / 6307
До лікаря звернулась жінка зі скаргами на те, що після операції видалення щитоподібної залози у неї спостерігається порушення мови, захриплість. Пошкодження якого нерва під час операції могло стати причиною цього явища?

Зворотного гортанного нерва

Під’язикового нерва

Язикового нерва

Язиково-глоткового нерва

Верхнього гортанного нерва

4049 / 6307
У жінки, хворої на злоякісну пухлину молочної залози, лікар виявив збільшення регіонарних лімфатичних вузлів. Яка група вузлів підлягає ураженню при даній патології в першу чергу?

Діафрагмальні вузли

Пахвинні вузли

Шийні вузли

Пахвові вузли

Ліктьові вузли

4050 / 6307
До хірургічного відділення ЦРЛ надійшов хворий з колотою раною стопи, яку він отримав під час косовиці. Які специфічні препарати необхідно застосувати з метою екстреної активно-пасивної імунопрофілактики правця?

Вакцина АКДП

Антибіотики

Антитоксична сироватка та анатоксин

Протиправцева вакцина

Інтерферон

4051 / 6307
У п’ятимісячної дівчинки виявлено застійні явища у легенях. При обстеженні виявлено зв’язок між висхідною аортою та легеневою артерією, що в нормі спостерігається у деяких земноводних і плазунів. Назвіть цю природжену ваду розвитку:

Дефект міжшлуночкової перегородки

Дефект міжпередсердної перегородки

Транспозиція магістральних судин

Незрощення боталової протоки

Розвиток правої дуги аорти

4052 / 6307
Лікар-дослідник у складі альпіністської експедиції піднявся у базовий табір, розташований на висоті 5000 м. На 3-й день перебування у нього з’явилися ознаки гірської хвороби: задишка, головний біль, втрата апетиту, загальна слабкість, ціаноз. Який тип гіпоксії має місце в цьому випадку?

Циркуляторна

Гіпоксична

Змішана

Тканинна

Іемічна

4053 / 6307
У чоловіка 30-ти років перед операцією визначили групову належність крові. Кров резус-позитивна. Реакцію аглютинації еритроцитів не викликали стандартні сироватки груп 0оД (І), АЗ (II), Beu (ПІ). Досліджувана кров належить до групи:

AB (IV)

ОоД (І)

Во (III)

АД (II)

4054 / 6307
Локальне порушення кровопостачання міокарда призвело до порушення у клітинах синтезу АТФ та до припинення роботи натрій-калієвих насосів. Найбільш імовірним наслідком цього буде така зміна потенціалу спокою клітин у зоні пошкодження:

Несуттєве зменшення

Відсутність змін

Суттєве збільшення

Зникнення

Збільшення

4055 / 6307
На аутопсії тіла жінки, що хворіла на хронічну дизентерію, в ході гістологічного дослідження внутрішніх органів в стромі та паренхімі міокарда, нирок, в слизовій оболонці шлунка та в сполучній тканині легень виявлені аморфні відкладення фіолетового кольору, що дають позитивну реакцію за Коссом. Яке ускладнення розвинулося у хворої?

Метаболічне звапніння

Метастатичне звапніння

Гіаліноз

Амілоїдоз

Дистрофічне звапніння

4056 / 6307
Розпад глікогену в печінці стимулюється адреналіном. Який вторинний ме- сенджер (посередник) при цьому утворюється в клітині?

ц-ГМФ

Діацилгліцерол

ц-АМФ

CO D.NO

4057 / 6307
Дитина 3-х років із симптомами стоматиту, гінгівіту, дерматиту відкритих ділянок шкіри була госпіталізована. При обстеженні встановлено спадкове порушення транспорту нейтральних амінокислот у кишечнику. Нестачею якого вітаміну будуть зумовлені дані симптоми?

Вітаміну А

Пантотенової кислоти

Біотину

Кобаламіну

Ніацину

4058 / 6307
Хворий ходить хитаючись, широко розставляючи ноги. У нього знижений тонус м’язів рук і ніг, скандована мова. У якому відділі головного мозку локалізується ураження?

Червоне ядро

Шкаралупа

Мозочок

Моторна кора

Хвостате ядро

4059 / 6307
Встановлено, що токсична дія ціанідів виявляється у гальмуванні клітинного дихання. Який органоїд клітини є найбільш чутливим до цих отрут?

Рибосоми

Комплекс Іольджі

Мітохондрїї

Лізосоми

Клітинний центр

4060 / 6307
Юнак 17-ти років страждає на фурункульоз, викликаний умовно- патогенним Staphylococcus epidermidis. Яке дослідження найдоцільніше провести, щоб правильно вибрати препарат для лікування цього хворого?

Визначити антигенні властивості

Дослідити біохімічні властивості

Виявити фактори патогенності

Визначити фаговар

Скласти антибіотикограму

4061 / 6307
У хворого впродовж 10-ти днів має місце підвищена температура, напади характерного кашлю. Лікар призначив посів слизу з носоглотки на середовище казеїнова-вугільний агар. Який мікроорганізм передбачається виявити?

Стафілокок

Лістерію

Паличку коклюшу

Паличку інфлюенци

Клебсіелу

4062 / 6307
Хворому призначена ендоскопія 12- палої кишки. В результаті виявлено запалення великого дуоденального сосочка і порушення виділення жовчі в просвіт кишки. У якому відділі 12-палої кишки виявлені порушення?

Іоризонтальна частина

Низхідна частина

Цибулина

Верхня частина

Висхідна частина

4063 / 6307
У дитини, яка часто хворіє на ангіни та фарингіти, відзначається збільшення лімфовузлів і селезінки. Зовнішній вигляд характеризується пастозністю та блідістю, м’язова тканина розвинена слабко. У крові спостерігається лімфо- цитоз. Як називається такий вид діатезу?

Нервово-артритичний

Геморагічний

Лімфатико-гіпопластичний

Ексудативно-катаральний

Астенічний

4064 / 6307
У жінки 35-ти років, яка протягом З місяців обмежувала кількість продуктів у харчовому раціоні, спостерігається зменшення маси тіла, погіршення фізичного стану та розумової діяльності, з’явилися набряки. Дефіцит яких харчових речовин міг призвести до таких змін?

Білків

Жирів

Мікроелементів

Вуглеводів

Вітамінів

4065 / 6307
Хворому діагностовано недостатність мітрального клапана. Де даний клапан розміщується?

В місці виходу аорти

Між лівим і правим шлуночками

Між лівим і правим передсердями

Між лівим передсердям і лівим шлуночком

Між правим передсердям і правим шлуночком

4066 / 6307
При проведенні оперативного втручання на шиї хірургу потрібно виділити зовнішню сонну артерію. Що є анатомічним орієнтиром для встановлення місця початкового відділу вказаної судини в ділянці шиї?

Місце початку грудинно-ключично- соскоподібного м’яза

Верхній край щитоподібного хряща

Кут нижньої щелепи

Яремна вирізка

Нижній край щитоподібного хряща

4067 / 6307
У 18-річної дівчини в сільському районі Індії розвився профузний пронос з втратою рідини до 8 літрів на добу. Який з наведених нижче мікроорганізмів може бути збудником захворювання?

Shigella dysenteriae

Vibrio cholerae

Campylobacter jejuni

Salmonella typhi

Ентеропатогенна Escherichia coli

4068 / 6307
Хворий госпіталізований з попереднім діагнозом 'черевний тиф'. Яке живильне середовище з перерахованих можна використати для виділення гемо- культури?

Середовище Левенштейна-Иенсена

МПБ

Жовчний бульйон

Жовтково-сольовий агар

Кров ’яний агар

4069 / 6307
При обстеженні лікар встановив, що дисфункція кори головного мозку пацієнта викликана ураженням мережі нейронів мозкового стовбура, яка підтримувала активність кори великих півкуль. Які структури мозку уражені?

Базальні ядра

Ядра гіпоталамуса

Ретикулярна формація

Ядра таламуса

Ядра мозочка

4070 / 6307
У пацієнта перфоративна виразка передньої стінки шлунка. В яке похідне очеревини потрапить вміст шлунка?

Печінкова сумка

Чепцева сумка

Передшлункова сумка

Лівий брижовий синус

Правий брижовий синус

4071 / 6307
У результаті радіаційного випромінювання були ушкоджені стовбурові гемопоетичні клітини. Утворення яких клітин сполучної тканини буде порушено?

Макрофаги

Меланоцити

Фібробласти

Адипоцити

Перицити

4072 / 6307
Реалізація загального адаптаційного синдрому здійснюється переважно через нейроендокринну систему. Якій з ланок цієї системи належить провідна роль у патогенезі реакції, що розвивається?

Гіпофізарно-юкстагломерулярна

Гіпофізарно-адреналова

Гіпофізарно-адреногенітальна

Гіпофізарно-тиреоїдна

Гіпофізарно-інсулярна

4073 / 6307
У відділення реанімації надійшов хворий після ДТП з однобічним пневмотораксом. Який вид дихання спостерігається у цьому випадку?

Поверхневе рідке

Асфіктичне

Поверхневе часте

Поверхневе

Глибоке часте

4074 / 6307
У хворого зі скаргами на полідипсію, поліфагію, поліурію визначили високий рівень глюкози крові і наявність її у сечі. Яке захворювання можна припустити?

Акромегалія

Цукровий діабет

Хвороба Аддісона

Інсулінома

Нецукровий діабет

4075 / 6307
Хворому на активну форму туберкульозу призначили ізоніазид. Який вітамінний препарат необхідно застосовувати для профілактики побічної дії ізоніазиду?

Ретинолу ацетат

Токоферолу ацетат

Піридоксину гідрохлорид

Рутин

Ціанокобаламін

4076 / 6307
Відомо, що однією з причин виникнення мембранного потенціалу спокою є різниця концентрації іонів по обидві сторони клітинної мембрани. Який механізм забезпечує іонну асиметрію всередині і зовні клітини?

Піноцитоз

Фільтрування

Полегшена дифузія

Активний транспорт

Дифузія

4077 / 6307
Людина застосовувала дихальну методику, яка заснована на проведенні частого та поверхнього дихання, що супроводжується гіповентиляцією. Внаслідок чого при цьому розвивається респіраторний ацидоз?

Надлишку О2 в крові

Надлишку С*О2 в крові

Нестачі СО2 в крові

Нестачі О2 в крові

4078 / 6307
Під дією УФ-опромінення та інших факторів можуть відбуватися зміни в структурі ДНК. Репарація молекули ДНК досягається узгодженою дією всіх наступних ферментів, ЗА ВИНЯТКОМ:

ДНК-глікозид ази

ДНК-полімерази

Ендонуклеази

ДНК-лігази

Аміноацил-тРНК-синтетази

4079 / 6307
Батько і син мають різні мітохондрі- альні геноми. Це пояснюється тим, що ці геноми:

Мають різну експресивність

Успадковуються від матері

Не успадковуються

Мають високий рівень мутабільності

Підлягають комбінативній мінливості

4080 / 6307
У жінки, що тривало приймала антибіотики з приводу кишкової інфекції, розвинулось ускладнення з боку слизової порожнини рота у вигляді запального процесу і білого нальоту, у якому під час бактеріологічного дослідження були виявлені дріжджеподібні грибки Candi-da albicans. Який з перерахованих препаратів показаний для лікування цього ускладнення?

Бісептол

Флуконазол

Тетрациклін

Поліміксин

Фуразолідон

4081 / 6307
Хвора 57-ми років для лікування гіпертонічної хвороби тривалий час приймала анаприлін. Побічні ефекти спонукали пацієнтку відмовитись від прийому препарату, що призвело до розвитку гіпертонічного кризу і нападу стенокардії. Як називається ускладнення, яке виникло?

Сенсибілізація

Тахіфілаксія

Лікарська залежність

Звикання

Синдром відміни

4082 / 6307
При деяких гельмінтозах людина може сама виявити гельмінта, оскільки зрілі членики збудника можуть активно виповзати з ануса людини. Це характерно для:

Теніозу

Дифілоботріозу

Гіменолепідозу

Теніаринхозу

Ехінококозу

4083 / 6307
Жінка літнього віку перенесла сильний стрес. У крові різко збільшилась концентрація адреналіну і норадреналі- ну. Які ферменти каталізують процес інактивації катехоламінів?

Моноамінооксидази

Тирозиназа

Ілікозидази

Пептидази

Карбоксилаза

4084 / 6307
У чоловіка, померлого від внутрішньої кровотечі (гемоперитонеум), в печінці субкапсулярно виявлено губчастий вузол темно-червоного кольору розмірами 15x10 см, добре відмежований від навколишньої тканини. Мікроскопічно: тканина вузла складається з великих судинних тонкостінних порожнин, вистелених ендотеліальними клітинами та заповнених рідкою або згорнутою кров’ю. Встановіть вид пухлини:

Венозна гемангіома

Лімфангіома

Кавернозна гемангіома

Іемангіоперицитома

Капілярна гемангіома

4085 / 6307
На заняттях з лікувальної фізкультури лікар-фізіотерапевт запропонував юнакам відхилитися назад і дістати долонями до підлоги. Яка зв’язка запобігає надмірному розгинанню хребтового стовпа?

Передня поздовжня

Міжпоперечна

Жовта

Задня поздовжня

Надостьова

4086 / 6307
Вивчається мітотичний поділ клітин епітелію ротової порожнини. Встановлено, що в клітині диплоїдний набір хромосом. Кожна хромосома складається з двох максимально спіралізованих хроматид. Хромосоми розташовані у площині екватору клітини. Така картина характерна для стадії мітозу:

Метафаза

Профаза

Прометафаза

Анафаза

Телофаза

4087 / 6307
У хворого з хронічним гіперацидним гастритом з’явився біль у суглобах. Для полегшення болю, враховуючи супутню патологію, був призначений целекоксиб. Вибіркова дія цього препарату на певний фермент забезпечує відсутність впливу на слизову шлунка. Назвіть цей фермент:

Циклооксигеназа 1

Фосфоліпаза А2

Калікреїн

Фосфоліпаза С

Циклооксигеназа 2

4088 / 6307
Після травми печінки у хворого з’явились симптоми отруєння аміаком за типом печінкової коми. Як аміак діє на енергозабезпечення ЦНС?

Інгібування окисного фосфорилюван- ня

Інактивація ферментів дихального ланцюга

Блокування ЦТК внаслідок зв’язування альфа-кетоглутарату

Іальмування гліколізу

Іальмування бета-окислення жирних кислот

4089 / 6307
Бактеріологічний метод діагностики був використаний для підтвердження діагнозу газова гангрена у хворого. Які живильні середовища необхідно використовувати для культивування збудника в цьому випадку?

Вільсона-Блера, Кітта-Тароцці

Ендо, Левіна, Плоскірєва

ЖСА, кров’яний агар

Лужний агар

МПА, МПБ

4090 / 6307
У дівчинки 6-ти років виражені ознаки гемолітичної анемії. При біохімічному аналізі еритроцитів встановлено дефіцит ферменту глюкозо-6- фосфатдегідрогенази. Порушення якого метаболічного процесу відіграє головну роль у розвитку цієї патології?

Пентозофосфатного шляху

Глюконеогенезу

Анаеробного гліколізу

Тканинного дихання

Окисного фосфорилювання

4091 / 6307
Скорочення поперечно-посмугованих м’язів неможливе без кальцію. Яку роль відіграє цей іон в утворенні актино- міозинових містків?

З’єднується з адренорецепторами

З’єднується з серотоніновими рецепторами

З’єднується з холінорецептором

З’єднується з гістаміновими рецепторами

З’єднується з тропоніном

4092 / 6307
У хворого з синдромом Іценка- Кушинга спостерігаються стійка гіперглікемія та глюкозурія. Синтез та секреція якого гормону підвищені у цього хворого?

Адреналін

Тироксин

Глюкагон

Альдостерон

Кортизол

4093 / 6307
Обмеження споживання води призвело до зневоднення організму. Який механізм активується для збереження води в організмі?

Збільшення секреції соматостатину

Зменшення секреції альдостерону

Зменшення секреції кальцитоніну

Збільшення секреції тироксину

Збільшення секреції вазопресину

4094 / 6307
У хворого на подагру виявлено значне підвищення рівня сечової кислоти в крові. Кінцевим продуктом обміну яких речовин є сечова кислота?

Глобулінів

Альбумінів

Тригліцеридів

Жирних кислот

Пуринових основ

4095 / 6307
При тривалому використанні одного з гормональних препаратів у хворого виявлено ознаки остеопорозу, ерозії слизової шлунка, гіперглікемію, зменшення рівня АКТГ в крові. Препарат якої групи гормонів міг викликати ці явища?

Ілюкокортикоїдів

Иодвмісних гормонів

Статевих гормонів

Кальцитоніну

Мінералокортикоїдів

4096 / 6307
Людина потрапила в ситуацію, що пов’язана з емоційним напруженням. У результаті цього в неї в крові підвищився рівень адреналіну і, як наслідок, збільшилась сила серцевих скорочень. Яким чином адреналін збільшує силу серцевих скорочень?

Знижує тонус блукаючих нервів

Викликає збудження барорецепторів

Збуджує бета-адренорецептори

Впливає на барорецептори

Знижує збудливість клітин- пейсмекерів

4097 / 6307
У хворого на малярію після вживання протималярійного препарату при- махіну розвинулася гемолітична анемія. Спадкова недостатність якого ферменту в еритроцитах спостерігається при цьому?

Фруктозо-1-фосфатальдолази

Тріозофосфатізомерази

Ліпази

Ілюкозо-6-фосфатдегідрогенази

Фосфофруктокінази

4098 / 6307
Під час проведення морфологічного дослідження периферичної крові хворого було помічено, що у еритроцитів забарвлена лише периферична частина, а в центрі є незабарвлене прояснення. Кольоровий показник - 0,56. Яка анемія найімовірніша у цього пацієнта?

Залізодефіцитна

Іемолітична

Сидеробластна

Ві2 фолієводефіцитна

Апластична

4099 / 6307
На фельдшерський пункт звернулась жінка з приводу рваної рани правої кисті, що була спричинена власним мисливським собакою, який загинув через 5 днів від сказу. Які препарати потрібно використати для профілактики сказу у вкушеної жінки?

Інтерферон

Антибіотики

Антирабічна сироватка-і-антибіотики

Антирабічна вакцина+антирабічний імуноглобулін

Вакцина БЦЖ

4100 / 6307
У хворого, який звернувся в шкірно- венерологічний диспансер, виявлений твердий шанкр. Яке мікробіологічне дослідження дозволить поставити діагноз сифілісу на даному етапі хвороби?

Бактеріологічний

Темнопольна мікроскопія

Алергологічний

Біологічний

Серологічний

4101 / 6307
У фермера, що має стадо кіз, спостерігається лихоманка неясного походження. Найімовірніше, збудником захворювання є:

Histoplasma capsulatum

Clostridium novyi

S. aureus

Т pallidum

Brucella melitensis

4102 / 6307
Жінка 35-ти років розпочала голодування. Депо яких поживних речовин використовується у початковий період голодування і як при цьому змінюється дихальний коефіцієнт (ДК)?

Білки, ДК наближається до 0,7

Вуглеводи, ДК наближається до 1

Жири, ДК наближається до 0,85

Білки, ДК наближається до 1

Жири, ДК наближається до 0,72

4103 / 6307
У хворого діагностовано паразитарне захворювання, яке супроводжується клінічною картиною В12- фолієводефіцитної анемії. Який гельмінт спричинив дане захворювання?

Taenia solium

Taeniarhynchus saginatus

Diphyllobothrium latum

Hymenolepis nana

Fasciola hepatica

4104 / 6307
У хворого з підозрою на озену з носоглотки були виділені грамнегатив- ні палички, які утворювали капсулу на поживному середовищі. Які мікроорганізми спричинили хворобу?

Хламідії

Клебсієли

Мікоплазми

Сальмонели

Шигели

4105 / 6307
Чоловік з гострим міокардитом помер від серцево-судинної недостатності. В ході мікроскопічного дослідження внутрішніх органів виявлені: плазмора- гія, набряк, стази в капілярах, численні крововиливи, а також дистрофічні зміни в паренхімі. Наслідком чого є дані зміни?

Загальне артеріальне повнокрів’я

ДВЗ -синдром

Хронічний загальний венозний застій

Місцеве артеріальне повнокрів’я

Гострий загальний венозний застій

4106 / 6307
При складних методах фарбування виявляють особливості хімічного складу бактеріальної клітини або наявність певних структур. Який з перерахованих методів фарбування є основним та найчастіше вживаним?

Романовського-Гімзи

Грама

Ціля-Нільсена

Ожешко

Леффлера

4107 / 6307
Пацієнтці 68-ми років з туберкульозом легень призначено антибіотик, прийом якого може викликати забарвлення сечі та сльозної рідини в червоний колір. Назвіть його:

Амоксиклав

Етамбутол

Рифампіцин

Тетрациклін

Етіонамід

4108 / 6307
У жінки під час мейозу відбулося порушення розходження аутосом. Утворилася яйцеклітина із зайвою 18-ю хромосомою. Яйцеклітина запліднюється нормальним сперматозооном. У майбутньої дитини буде синдром:

Патау

Шерешевського-Тернера

Клайнфельтера

Дауна

Едвардса

4109 / 6307
У клініку госпіталізовано хворого з діагнозом карцинома кишечника. Для більшості карцином характерна підвищена продукція і секреція серотоніну. Відомо, що ця речовина утворюється з триптофану в ході реакції:

Дезамінування

Утворення парних сполук

Мікросомального окислення

Трансамінування

Декарбоксилювання

4110 / 6307
After severe emotional strain a 45-year-old man suddenly developed constricting pain in the area of his heart. The pain was irradiating into his left arm, neck, and left shoulder blade. His fase was pale and covered in cold sweat. Nitroglycerine was able to relieve the pain. What condition developed in the patient in this case?

Angina pectoris

Gastric ulcer perforation

Myocardial infarction

Stroke

Psychogenic shock

4111 / 6307
A 23-year-old woman was brought into the emergency department complaining of bloody diarrhea, tiredness, and dizziness. A few days ago she went to a fast food reataurant for a birthday party. Her friends are experiencing similar symptoms. Laboratory studies show anemia. What samples should be obtained for microbiologic testing in this case?

Blood

Stool

Cerebrospinal fluid

Bile

Urine

4112 / 6307
Histological microslide shows a gastrointestinal organ. The wall of this organ consists of 4 layers: mucosal, submucosal, muscular, and serous. The muscular layer has folds and pits. What organ has such appearance?

Duodenum

Small intestine

Esophagus

Stomach

Appendix

4113 / 6307
A 2-year-old child with delayed physical and mental development was brought to the hospital. The child’s parents are the most concerned by frequent profuse vomiting that occurs in their child after eating. Laboratory testing detected phenylpyruvic acid in the child’s urine. What type of metabolism is disturbed, causing this patology?

Carbohydrate metabolism

Water and electrolyte metabolism

Phosphorus and calcium metabolism

Amino acid metabolism

Lipid metabolism

4114 / 6307
The heart is a muscular organ that pumps blood through the body. This function is carried out by valves, muscles, and vessels. What valve is located between the left ventricle and left atrium?

Aortic

Non-return

Bicuspid

Pulmonary

Tricuspid

4115 / 6307
During your physiology class, the professor asks you to explain the effect of various hormones and neurotransmitters on the metabolism of glucose in the human body. You open your report with the statement that the use of glucose by the cells is preceded by its transport from the intercellular substance into the cell. What hormone is most likely responsible for the glucose uptake by the cell?

Glucagon

Adrenaline

Aldosterone

Thyroxine

Insulin

4116 / 6307
A 25-year-old man came to the family doctor complaining of the loss of taste sensation in the front two- thirds of his tongue. The doctor determined that this condition was caused by the damage to a certain nerve. What nerve is likely to be functionally impaired in this case?

Chorda tympani

Vagus

Accessory nerve

Hypoglossal nerve

Glossopharyngeal nerve

4117 / 6307
A 28-year-old man complains of nausea, vomiting, and right-sided subcostal pain. Objectively, his skin and sclerae are icteric, he has elevated body temperature, enlarged liver, dark urine, fecal hypocholia, hyperbilirubinemia (direct and indirect bilirubin), bilirubinuria, urobilinuria, hypoproteinemia, and decreased blood coagulability. What condition can be characterized by these changes?

Acute cholecystisis

Hepatocellular parenchymal jaundice

Post-hepatic jaundice

Acute pancreatitis

Pre-hepatic hemolytic jaundice

4118 / 6307
A soft tissue node obturated the patient’s middle lobe bronchus, leading to the development of right middle lobe atelectasis. In the area of obturation, bronchial biopsy detected proliferations of atypical glandular epithelium with pathological mitoses. Epithelium penetrates into the underlaying tissues and cartilage. What disease is the most likely in this case?

Bronchial sarcoma

Inflammatory polyp

Bronchitis deformans

Bronchogetic lung cancer

Bronchial epithelial dysplasia

4119 / 6307
The main functions of neural tissue can be described as communication and integration. What is the basic unit of neural tissue that ensures its functioning?

Dendrite

Axon

Neuron

Nucleus

Myelin sheath

4120 / 6307
The patient has developed a refractive error, where the focal point of the image is located not precisely on the retina, but anterior to it. This causes distant objects to be blurry, while close objects often appear normal. Among the other signs there are headaches and eye strain. This condition is associated with retinal detachment, cataract, and glaucoma. Name this condition:

Farsightedness

Astigmatism

Blindness

Emmetropia

Nearsightedness

4121 / 6307
A 20-year-old woman came to the clinic after missing her last 2 periods. Her cycles are usually regular, occurring at 28-30 day interval with moderate bleeding and occasional abdominal pain. She also complains of progressively diminishing peripheral vision. Her doctor determined the loss of vision in the lateral halves of both retinas. What structure is likely to be alfected, causing bitemporal hemianopsia in the patient?

Right optic nerve

Optic chiasm

Left optic nerve

Right optic tract

Left optic tract

4122 / 6307
A woman was bitten by a venomous snake (Macrovipera lebetinis). Autopsy of her body shows marked intravascular hemolysis. Her pancreas, bone marrow, and lymph nodes are brown. Microscopy detects abundant brown pigment in the macrophage cytoplasm. What pigment accumulates in the tissues in such cases?

Lipofuscin

Hematin

Hemosiderin

Bilirubin

Hematoidin

4123 / 6307
Histological microslide shows an organ consisting of gray and white matter. The gray matter is located in the center and is made up of bundle neurons (neurocytus funicularis), radicular neurons (neurocytus radiculatus), and interneurons. Such morphology is characteristic of the following organ?

Medulla oblongata

Cerebellum

Cerebral hemispheres

Spinal cord

Spinal ganglion

4124 / 6307
A molecular biologist studies various molecules. One of them is a polymeric molecule essential in various biological roles in coding, decoding, regulation, and expression of genes. It is a helical single­stranded molecule folded onto itself. Which of the following molecules is being studied?

ATP

ADP

RNA

HLA

DNA

4125 / 6307
There is a system that maintains the balanced state of multicellular organisms. Its main functions are to ensure an adequate physiological response and to defend the body against infectious agents and other unintended invasions. What system has such functions?

Lymphatic system

Immune system

Endocrine system

Nervous system

Homeostatic system

4126 / 6307
A team of medical students researches the phases of cell cycle. During one of the mitotic phases the cell has nearly completed its division, the chromosomes decondense, and two nuclei begin to form around them. What phase is likely observed in the cell at this moment?

Telophase

Metaphase

Anaphase

Prophare

4127 / 6307
A 34-yeat-old man visits his dentist complaining of a toothache. After a dental procedure that involved extraction of several teeth, he developed a severe bleeding lasting more than 15 minutes. He has a history of chromic hepatitis C. What is the most likely cause of the prolonged bleeding in this patient?

Hypofibrinogenemia

Hypoalbuminemia

Thrombocytopenia

Hypocalcemia

4128 / 6307
Medical examination in an army recruitment center detected a 15-year-old boy with tall stature, eunuchoid body proportions, gynecomastia, and female pattern of pubic hain growth. The boy has fat deposits on the thighs, no facial fair growth, high-pitched voice, and below average IQ. He was diagnosed with Klinefelter syndrome. What karyotype corresponds with this disease?

46, XY

47, XYY

47, XXY

45, X0

47, XY, +18

4129 / 6307
A man with signs of peritonitis was brought to the admission room. He has a 12-year-long history of peptic ulcer disease with ulcer localization on the posterior gastric wall. He was diagnosed with gasiric ulcer perforation. What anatomical structure is likely to be contaminated with gastric content in this case?

Canalis lateralis sinister

Bursa omentalis

Bursa pregastrica

Bursa hepatica

Canalis lateralis dexter

4130 / 6307
A 46-year-old man complains of tiredness and pain in the joints of his fingers and wrists. These signs are observed for the last 2 months. The pain is present in both hands and the wrists are swollen. Furthermore, he describes morning stiffness in his joints, lasting about 2 hours, which improves with use. His past medical history reveals that he was successfully treated for H. pylori - related ulcer last year. He denies smoking and stopped drinking when his GI symptoms started. What drug is the best choice for his joint pain management?

Prednisolone

Morphine

Aspirin

Patacelanol

Celecoxib

4131 / 6307
A 27-year-old woman complains of insomnia, irritability, hand tremor, acute weight loss despite high appetite, and constant fever with body temperature of 37,2-37,5 Co. What endocrine gland is likely to be functionally impaired in this case?

Parathyroid gland

Thyroid gland

Adrenal glands

Neurohypophysis

Pancreas

4132 / 6307
General practitioner performs physical examination of the patient. In the course of the examination it is necessary to measure the palpable regular expansion of a superficial artery caused by the ejection of blood into the vessels through heart contractions. What sign does the doctor measure?

Pulse

Blood pressure

Saturaion

Heart electricity

Heartbeat

4133 / 6307
A man has signs of acute respiratory viral disease. His physician referred him for microbiological testing. A mucus smear obtained trom the patient’s tonsils contains spherical microorganisins arranged in short chains. The patient was diagnosed with tonsillitis. What microorganisms were detected in the smear from the patient's tonsils?

Tetracocci

Staphylococci

Diplococci

Micrococci

Streptocci

4134 / 6307
Muscle relaxant tubocurarine was administered during a surgery. After the surgery the patient's breathing is not restored. What antidote should he be given to resore his breathing?

Clophelin (Clonidine)

Atropine sulfate

Bemegride

Proserin (Neostigmine)

Anaprilin (Propranolol)

4135 / 6307
A 45-yeat-old woman came to her physician with complaints of extreme tiredness and weakness. She says that these symptoms lasts for a month already. Within the last 2 weeks she losts 3 kilograms. Objectively, she is tired-looking thin woman. Skin hyperpigmentation is observed in many areas of her body, most prominently on the face, neck, and the backs of her hands (the areas exposed to light). What hormone is produccd in an excess in this patient, most likely causing the hyperpigmentation?

Growth hormone (GH)

P-Lipotopin

Thyroid-stimulating hormone (TSH)

Gonadotropins

Melanocyte-stimulating hormone (MSH)

4136 / 6307
A 50-year-old woman complains of constant thist. She drinks large amounts of liquid and has increased diuresis. Her blood glucose is 12 mmol/L. Her urine contains glucose. What endocrine organ is likely to be functionally impaired in this patient?

Thyroid gland

Adrenal glands

Neurohypophysis

Parathyroid gland

Pancreas

4137 / 6307
An unidentified surgical specimen is received fot histopathologic analysis. A portion of the specimen is cut and stained with hematoxylin and eosin. Under the microscope, you see an organ encapaulated by dense connective tissue that extends to the deeper areas by way of the trabecular extensions. The organ can be subdivided into two regions: a cortex with lymphoid nodules and medulla with medullary cords populated by plasma cells, B cells, and Tcells. What anatomical structure is the most likely origin of this surgical specimen?

Spleen

Bone marrow

Thymus

Lymph node

Tonsils

4138 / 6307
A 65-year-old woman was brought into the emergency departmnent because of shortness of breath and chest pain that started a few hours ago. She denied having fever, expectoration, or any accompanying symptoms. She had a 5-year-long history of deep vein thrombosis in her thin. After a time, she died of respiratory distress. Autopsy reveals red loose masses lodged in the bifurcation of the pulmonary trunk with extensions into both the left and the right pulmonary arteries. Whnt is the most likely diagnosis?

Pneumonia

Pneumothorax

Thromboembolism

Myocardial infarction

4139 / 6307
A 54-year-old woman had a total thyroidectomy lor papillary thyroid carcinoma. 11 hours after operation she complained of tingling around her mouth. On physical examination, the Trousseau's sign and Chvostek's sign are present. Her condition rapidly deteriorates with laryngospasm and focal seizures. The surgeon suspects surgical destruction of the parathyroid glands. What is the most likely cause of this patient's neurological condition?

Hyperkalemia

Hypophosphatemia

Hyperchloremia

Hypocalcemia

Hyponatremia

4140 / 6307
Унаслідок обтурації жовчовивідної протоки у хворого зменшилося надходження жовчі в 12-палу кишку, що призвело до порушення всмоктування:

Білків

Білків та вуглеводів

Вуглеводів

Мінеральних солей

Жирів

4141 / 6307
У хворого на шкірі обличчя поступово розвилась бляшка з некрозом і виразкою в центрі. Під час патогістологічного дослідження біоптату виявлено розростання атипових епітеліальних клітин із великою кількістю патологічних мітозів. Який діагноз найімовірніший?

Рак шкіри

Трофічна виразка

Папілома

Фіброма

Саркома

4142 / 6307
Речовини виводяться з клітини внаслідок з’єднання мембранної структури апарату Гольджі з цитолемою. Уміст такої структури викидається за межі клітини. Цей процес має назву:

Ендоцитоз

Осмос

Екзоцитоз

Полегшена дифузія

Піноцитоз

4143 / 6307
У гематологічному відділенні хворому на лейкоз лікар призначив 5-фторурацил, який:

Інгібує синтез ДНК

Прискорює реплікацію

Стимулює ДНКазу

Інгібує транскрипцію

Інгібує трансляцію

4144 / 6307
Під час операції у хворого виявили дивертикул Меккеля. У якому відділі шлунково-кишкового тракту він може розміщуватися?

Сигмоподібна кишка

Дванадцятипала кишка

Порожня кишка

Клубова кишка

Сліпа кишка

4145 / 6307
У хворого з хронічним гіперацидним гастритом з’явився біль у суглобах. Для полегшення болю, ураховуючи супутню патологію, був призначений целекоксиб. Вибіркова дія цього препарату на певний фермент забезпечує відсутність впливу на слизову шлунка. Назвіть цей фермент:

Циклооксигеназа 2

Фосфоліпаза А2

Циклооксигеназа 1

Калікрсїн

Фосфоліпаза С

4146 / 6307
Під час дослідження людини у вертикальній позі встановлено, що в альвео-лах верхівок легень парціальний тиск кисню становить 140 мм рт.ст. Причиною цього є те, що в цих відділах легень:

Вентиляція відсутня

Перфузія та вентиляція врівноважені

Вентиляція переважає над перфузією

Перфузія переважає над вентиляцією

4147 / 6307
У жінки 28 років із гнійно-серозним кон’юнктивітом і скаргами на біль під час сечовипускання взято для дослідження зішкріб із кон’юнктиви. Під час мікроскопії в цитоплазмі епітеліальних клітин виявлено включення. Під час посіву патологічного матеріалу на живильні середовища росту не виявлено, але під час зараження курячих ембріонів у жовтковий мішок вдалося виділити культуру збудника. Який із наведених мікроорганізмів є найімовірнішим збудником?

Мікоплазми

Гемолітичний стрептокок

Хламідії

Дифтероїди

Аденовіруси

4148 / 6307
У 70-ті роки науковці встановили, що причиною важкої жовтяниці новонароджених є порушення зв’язування білірубіну в гепатоцитах. Яка речовина використовується для утворення кон’югату?

Молочна кислота

Сечова кислота

Сірчана кислота

Піровиноградна кислота

Глюкуронова кислота

4149 / 6307
Під час операції на щитоподібній залозі з приводу захворювання на Базедову хворобу помилково були видалені паращитовидні залози. Виникли судоми, тетанія. Обмін якого біоелемента було порушено?

Магнія

Заліза

Натрія

Кальція

Калія

4150 / 6307
Жінка 35 років розпочала голодування. Депо яких поживних речовин використовується у початковий період голодування і як у цьому разі змінюється дихальний коефіцієнт (ДК)?

Білки, ДК наближається до 0,7

Жири, ДК наближається до 0,72

Білки, ДК наближається до 1

Жири, ДК наближається до 0,85

Вуглеводи, ДК наближається до 1

4151 / 6307
На розтині чоловіка 52 років, який тривалий час страждав на туберкульо-зний простатит і помер від менінгоен- цефаліту, у м’яких оболонках основи і бічних поверхонь головного мозку, селезінці, нирках, печінці виявлено вели-ку кількість щільних, сірого кольору вузликів діаметром 0,5 - 1мм. Гістологі-чне дослідження показало, що вузлики складаються з епітеліоїдних, лімфоїдних і нечисленних гігантських клітин з ядрами, розташованими на периферії клітин, що мають вигляд підкови. Виявлені зміни свідчать про:

Вторинний туберкульоз

Міліарний туберкульоз

Великовогнищевий дисемінований туберкульоз

Найгостріший туберкульозний сепсис

Септикопіємію

4152 / 6307
Пацієнту 33 роки. Хворіє 10 років. Періодично звертається до лікаря зі скаргами на гострий біль в животі, судоми, порушення зору. У його родичів спостерігаються подібні симптоми. Сеча червоного кольору. Госпіталізований із діагнозом 'гостра переміжна порфірія'. Причиною захворювання може бути порушення біосинтезу:

Простагландинів

Інсуліну

Гему

Жовчних кислот

Колагену

4153 / 6307
Цукровий діабет у хворої 35 років ускладнився виникненням і розвитком катаракти, ймовірною причиною якої є:

Клітинна дегідратація

Дефіцит НАДФ • Н2 та гальмування синтезу жирних кислот

Глікозилювання білків кришталика і накопичення сорбітолу

Зниження спорідненості білків з киснем

Порушення зв’язування рецепторів з інсуліном

4154 / 6307
В експерименті на щурах електричне подразнення головного мозку викликало у голодних тварин відмову від споживання їжі. Яку структуру стимулювали?

Бліду кулю

Мигдалеподібні ядра

Латеральну ділянку гіпоталамуса

Гіпокамп

Вентромедіальне ядро гіпоталамуса

4155 / 6307
Під час обстеженя пацієнта встановили сильний, урівноважений, інертний тип вищої нервової діяльності за Павловим. До якого виду темпераменту за Гіппократом відноситься пацієнт?

Меланхолічний

Холеричний

Сангвінічний

Флегматичний

4156 / 6307
У бактеріологічній лабораторії досліджувалися консерви, які стали причиною важкої токсикоінфскцїї. Під час мікроскопії культури із середовища Кітта- Тароцці виявлені грампозитивні споро- утворювачі палички, схожі на тенісну ракетку. Який діагноз поставить лікар?

Туляремія

Дизентерія

Ботулізм

Черевний тиф

Хламідіоз

4157 / 6307
У нормальних умовах кровообіг матері і плода не має прямих зв’язків. Кров ембріона протікає по судинах ворсинок хоріона, а материнська циркулює в міжворсинковому просторі ендометрію матки. Укажіть, що розділяє кров плода і матері:

Аморфний фібриноїд Рора

Сполучнотканинні перегородки

Фібриноподібна оксифільна маса (фібриноїд Лангханса)

Гемохоріальний бар’єр

Замикальна пластинка основної відпадаючої оболонки матки

4158 / 6307
У хворого на бронхіальну астму виникла гостра недостатність дихання. Який тип недостатності дихання виникає в цьому разі?

Обструктивне порушення альвеолярної вентиляції

Перфузійний

Рестриктивне порушення альвеолярної вентиляції

Дисрегуляторне порушення альвеолярної вентиляції

Дифузний

4159 / 6307
У 25-річного пацієнта на тлі гострого запалення легень із температурою тіла 40,1°С визначено лейкоцитоз 14,9 г/л із чітким зсувом лейкоформули вліво. Який із наведених чинників безпосередньо підвищує в кістковому мозку і проліферацію, і диференціацію лейкоцитів?

Інтерлейкін-10

Пухлинний некротичний чинник

Колонієстимулюючий чинник

Простациклін

Інтерлейкін-1

4160 / 6307
У хворого на мікросфероцитарну гемолітичну анемію (хворобу Мінковського-Шоффара) внаслідок підвищення проникності мембрани еритроцитів у клітину надходять іони натрію й вода. Еритроцити набувають форми сфероцитів і легко руйнуються. Який провідний механізм пошкодження еритроцитів у цьому разі?

Електролітно-осмотичний

Протеїновий

Кальцієвий

Ацидотичний

Нуклеїновий

4161 / 6307
У дорослої людини системний артеріальний тиск знизився з 120/70 до 90/50 мм рт.ст., що викликало рефлекторне звуження судин. У якому із за-значених органів звуження судин буде найбільшим?

Нирки

Головний мозок

Кишечник

Наднирники

Серце

4162 / 6307
У людини трапляється спадкова хвороба, симптомокомплекс якої поєднує в собі цироз печінки та дистрофічні процеси головного мозку. Вона супроводжується зменшенням вмісту церулоплазміну в плазмі крові та порушенням обміну міді в організмі. Це хвороба:

Німанна-Піка

Жильбера

Тея-Сакса

Вільсона-Коновалова

Марфана

4163 / 6307
У людини внаслідок тривалого голодування швидкість клубочкової фільтрації зросла на 20%. Найімовірніша причина змін фільтрації в таких умовах є:

Збільшення ниркового плазмотоку

Збільшення коефіцієнта фільтрації

Збільшення системного артеріального тиску

Збільшення проникності ниркового фільтру

Зменшення онкотичного тиску плазми крові

4164 / 6307
У відповідь на розтягнення м’яза спостерігається його рефлекторне скоро-чення. З подразнення яких рецепторів починається ця рефлекторна реакція?

Дотикові рецептори

Сухожилкові рецептори Гольджі

Суглобові рецептори

Больові рецептори

М’язові веретена

4165 / 6307
Для розслаблення скелетних м’язів, щоб зіставити відламки стегнової кіс-тки, що утворилися внаслідок перелому, хворому був введений міореалаксант, що призвело до зупинки дихання. Після введення свіжої цитратної крові дихання відновилося. Який міореалаксант ввели хворому?

Атракурію бесилат

Тубокурарину хлорид

Дитилін

Панкуронію бромід

Піпекуронію бромід

4166 / 6307
У чоловіка 55 років, який протягом багатьох років страждав на недоста-тність мітрального клапану, виникла гостра серцева недостатність. Який патофізіологічний варіант недостатності серця спостерігається у цьому разі?

Перевантаження серця об’ємом

Перевантаження серця тиском

Коронарогенне ушкодження серця

Гостра тампонада серця

Гіпоксичне ушкодження серця

4167 / 6307
У крові хворого з лейкопенією виявлено антилейкоцитарні антитіла. Який тип алергічної реакції за Кумбсом і Джеллом виник у цьому разі?

Цитотоксичний

Стимулюючий

Гіперчутливість сповільненого типу

Імунокомплексний

Анафілактичний

4168 / 6307
Для профілактики та лікування тромбозу застосовуються засоби, що знижують згортання крові (антикоагулянти). Укажіть антикоагулянт, у разі передозування якого застосовують як антагоніст протаміну сульфат:

Неодикумарин

Натрію гідроцитрат

Синкумар

Фснілін

Гепарин

4169 / 6307
Людина в стані спокою штучно примушує себе дихати часто і глибоко впродовж 3-4 хвилин. Як це відбивається на кислотно-лужній рівновазі організму?

Виникає змішаний ацидоз

Виникає метаболічний алкалоз

Виникає дихальний алкалоз

Виникає метаболічний ацидоз

Виникає дихальний ацидоз

4170 / 6307
У жінки 38 років розвинувся напад бронхіальної астми. Який із наведених бронхолітиків є ефективним для надання невідкладної допомоги та належить до групи бета-2-адреноміметиків?

Адреналін

Платифілін

Сальбутамол

Сальметерол

Іпратропію бромід

4171 / 6307
Під час УЗД вагітної в серцево- судинній системі плоду порушень не виявлено, артеріальна протока функціонує. Визначте, які судини вона з’єднує:

Пупкову вену з аортою

Легеневий стовбур із нижньою порожнистою веною

Легеневий стовбур з аортою

Пупкову вену із пупковою артерією

Легеневий стовбур із верхньою порожнистою веною

4172 / 6307
Під час огляду у хворого виявилося запалення анатомічного утворення, що врівноважує тиск між барабанною порожниною та глоткою. Назвіть це утворення:

Слухова труба

Зовнішній слуховий прохід

Соскоподібна печера

Внутрішнє вухо

Внутрішній слуховий прохід

4173 / 6307
Парубок звернувся до лікарні зі скаргами на порушення сечовипускання. Під час обстеження зовнішніх статевих органів виявлено, що сечівник розщеплений зверху і сеча витікає крізь цей отвір. Який вид аномалії розвитку зовнішніх статевих органів спостерігається у цьому разі?

Гіпоспадія

Епіспадія

Гермафродитизм

Фімоз

Парафімоз

4174 / 6307
Аналіз родоводу дитини з міотонічною дистрофією дав можливість уста-новити, що захворювання виявляється у кожному поколінні однаково у осіб обох статей, батьки однаковою мірою передають захворювання дітям. Якщо один із батьків хворий (гетерозигота), а другий - здоровий, ризик народження хворої дитини становитиме 50%. Визначте тип успадкування захворювання:

Х-зчеплене домінантне успадкування

Аутосомно-рецесивний

Х-зчеплене рецесивне успадкування

Аутосомно-домінантний

У-зчеплене успадкування

4175 / 6307
Хворому призначена ендоскопія 12- палої кишки. Унаслідок цього виявлено запалення великого дуоденального сосочка і порушення виділення жовчі в просвіт кишки. У якому відділі 12-палої кишки виявлені порушення?

Висхідна частина

Горизонтальна частина

Верхня частина

Цибулина

Низхідна частина

4176 / 6307
Молодому чоловікові було видано позитивну відповідь на наявність у нього антитіл до ВІЛ (використаний ІФА), проте обстежуваний наполягає на дослідженні, яке б достовірно довело наявність у нього антитіл до цього вірусу. Яке дослідження слід провести для підтвердження діагнозу?

РПГА

РГГА

РІФ

Імуноблотинг

ПЛР

4177 / 6307
Хворому проведено трепанацію і вишкрібання осередків відростка скроневої кістки через його гнійне запалення, яке перейшло з середнього вуха. На якому відростку здійснено хірургічне втручання?

Processus mastoideus

Processus styloideus

Processus zygomaticus

Processus prerygoideus

4178 / 6307
Дитина 2 років випила очні краплі з домашньої аптечки. Стан важкий, значне пото- і слиновиділення, астматичне дихання, кашель, зіниці різко звужені, тони серця глухі, брадикардія. Перистальтика кишечника посилена, пронос, артеріальний тиск знижений. Яким препаратом викликане отруєння?

Пілокарпіну гідрохлорид

Анаприлін

Атропін

Платифіліну гідротартрат

Сульфацил-натрій

4179 / 6307
Хворому перед екстракцією зуба була проведена провідникова анестезія лідокаїном, після введення якого з’явилися набряк і гіперемія навколо місця уколу, свербіння шкіри, загальна слабкість, гіпотензія, рухове збудження. Визначте, як називається ускладнення, що виникло:

Лікарська залежність

Тахіфілаксія

Алергічна реакція

Токсична дія

Толерантність

4180 / 6307
У студента медінституту, госпіталізованого в інфекційне відділення на 2-гу добу захворювання, припускають інфекційний мононуклеоз. Який результат лабораторного дослідження може підтвердити діагноз у цього студента в день госпіталізації?

Виявлення 4-разового наростання антитіл до вірусу Епштейна-Барр

Виявлення антитіл до цитомегаловірусу

Виявлення ІgМ-антитіл до вірусу Епштейна-Барр

Виявлення ІgМ-антитіл до вірусу простого герпесу

Ізоляція (виділення) вірусу герпесу

4181 / 6307
Для лікування кропив’янки з метою усунення сверблячого висипу на шкірі хворому призначений димедрол. Який механізм забезпечує його ефективність у цьому разі?

Прискорення руйнування гістаміну

Конкурентна блокада Н1-рецепторів

Незалежний антагонізм з гістаміном

Пригнічення вивільнення гістаміну

Інгібіція синтезу гістаміну

4182 / 6307
Юнак 25 років звернувся до лікаря зі скаргами на загальну слабкість, швидку втомлюваність, дратівливість, зниження працездатності, кровоточивість ясен. Недостатність якого вітаміну може мати місце у цьому разі?

Рибофлавін

Аскорбінова кислота

Тіамін

Ретинол

Фолієва кислота

4183 / 6307
Після накладання джгута у досліджуваного виявили точкові крововиливи. З порушенням функції яких клітин крові це пов’язано?

Нейтрофіли

Еритроцити

Моноцити

Тромбоцити

Лімфоцити

4184 / 6307
У пацієнта спостерігаються птоз (опущення повіки), розбіжна косоокість, порушення акомодації, розширення зіниць. Ядра якої пари черепних нервів уражені?

VI

VII

III

IV

V

4185 / 6307
Сечокам’яна хвороба ускладнилася виходом камінця з нирки. На якому рівні сечовода, найімовірніше, він може зупинитися?

У мисці

У середній черевній частині

На 2 см вище впадіння в сечовий міхур

На межі черевної та тазової частин

На 5 см вище тазової частини

4186 / 6307
У чоловіка 64 років спостерігаються симптоми різкого порушення процесів обміну речовин та енергії. Під час проведення комп’ютерної томографії в одній із ділянок головного мозку виявлена пухлина. Яка структура головного мозку, що відіграє значну роль у регуляції процесів обміну речовин, може бути уражена в цьому разі?

Таламус

Ретикулярна формація

Червоне ядро

Чорна субстанція

Гіпоталамус

4187 / 6307
Чоловік 40 років, м’ясник, номер від сепсису. На правій щоці його визначається конусоподібний, щільний, темно- червоний інфільтрат 6 см із чорною кірочкою в центрі. Права половина обличчя, шиї різко набряклі, щільні. Під час мікроскопічного дослідження в інфільтраті визначається вкрай гостре серозно- геморагічне запалення, у центрі інфільтрату спостерігається некроз епідермісу і підлеглих шарів. Який діагноз поставив патологоанатом?

Фурункул

Чума

Туляремія

Сибірка

Флегмона шиї

4188 / 6307
Хлопчик на другому році життя став часто хворіти на респіраторні захворювання, гноячкові ураження шкіри. Установлено, що в крові дитини практично відсутні імуноглобуліни всіх класів. Зниження функціональної активності якої клітинної популяції лежить в основі описаного синдрому?

Е. Нейтрофілів

NK-лімфоцитів

В-лимфоцитів

Т-лімфоцитів

Макрофагів

4189 / 6307
Моделюючи запалення на брижі жаби, спостерігали крайове стояння лейкоцитів та їх еміграцію крізь судинну стінку. Який із наведених факторів обумовлює цей процес?

Збільшення онкотичного тиску в осередку запалення

Вплив хемотаксичних речовин

Збільшення гідростатичного тиску в судинах

Зниження онкотичного тиску в судинах

Зменшення гідростатичного тиску в судинах

4190 / 6307
Дівчинка 11 років прийшла на прийом до лікаря із мамою, яка скаржиться на слабкість та набряклість обличчя її дитини протягом 3 днів. Мати стверджує, що до початку симптомів її дитина завжди була здоровою та активною. Під час фізикального обстеження виявлено генералізований набряк обличчя, якщо натискати на набряк нижніх кінцівок, залишається ямка, яка поступово згладжується. Під час збору анамнезу дівчинка зазначає пінистий вигляд сечі, але заперечує домішки крові у сечі, ніктурію або біль під час сечовиділення. Лабораторні дослідження виявили протеїнурію та мікрогематурію. Що з наведеного є найімовірнішою причиною змін у лабо-раторному аналізі сечі?

Підвищення онкотичного тиску плазми крові

Підвищення гідростатичного тиску у капсулі Шумлянського-Боумена

Підвищення гідростатичного тиску у клубочку

Підвищення проникності стінки клубочкового капіляру

4191 / 6307
Застосування еубіотика коліцину з лікувальною і профілактичною метою пов'язане з особливістю нспатогенних кишкових паличок синтезувати коліцин - речовину білкової природи, що пригнічує ріст патогенних видів мікроорганізмів. Яка структура кодує здатність бактеріальної клітини синтезувати коліцини?

Мeзосома

Ядро

Нуклеоїд

Рибосома

Плазміда

4192 / 6307
Хворому з неоперабельним раком легені, що супроводжується болем, який важко переносити, лікар призначив знеболювальний засіб. На цьому тлі у хворого виникла непрохідність кишечника. Який зі знеболювальних препаратів міг зумовити описане ускладнення?

Омнопон

Фентаніл

Промедол

Анальгін

Морфін

4193 / 6307
На прийомі у лікаря хворий повідомив, що самостійно застосовує протиалергійний препарат, який ефективно зменшує прояви алергії, проте викликає значну сонливість. Який препарат із групи блокаторів НІ-рецепторів застосовує хворий?

Кромолін-натрій

Лоратадин

Димедрол

Ранітидин

Тавегіл

4194 / 6307
Досить часто причиною набутих імунодефіцитів є інфекційне ураження організму, під час якого збудники розмножуються безпосередньо в клітинах імунної системи і руйнують їх. Виберіть серед наведених ті захворювання, за яких має місце вищезгадане:

Інфекційний мононуклеоз, СНІД

Поліомієліт, гепатит А

Ку-гарячка, висипний тиф

Туберкульоз, мікобактеріоз

Дизентерія, холера

4195 / 6307
Регуляція експресії генів здійснюється за допомогою різних механізмів. Назвіть ділянки ДНК, у разі індукції яких активується експресія гену:

Сайленсер

Спейсср

Термінатор

Енхансер

Атенюатор

4196 / 6307
Гемоглобін дорослої людини (НЬАІ) - білок-тетрамер, який складається з двох альфа- та двох бета-пептидних ланцюгів. Яку назву має така структура цього білка?

Третинна

Вторинна

Четвертинна

Первинна

4197 / 6307
У чоловіка ЗО років перед операцією визначили групову належність кро-ві. Кров резус-позитивна. Реакцію аглютинації еритроцитів не викликали стандартні сироватки груп Оа/3 (І), Ар (II), Ва (III). Досліджувана кров належить до групи:

АВ (IV)

0аb (І)

Ва (III)

Аb (II)

4198 / 6307
Недостатня продукція мінералокортикоїдів (Аддісонова хвороба) супроводжується м’язовою слабкістю, що зумовлена підвищеним виділенням із сечею іонів:

Гідрогену

Кальцію

Магнію

Натрію

Калію

4199 / 6307
У хворого після загострення хронічного калькульозного холециститу гостро розвинулася жовтяниця. Під час ЕКГ-обстеження звернено увагу, що на тлі правильного синусового ритму (ЧСС - 51/хв.) періодично з’являються екстрасистоли. Який механізм, найімовірніше, викликав порушення електричної активності серця?

Пошкоджуюча дія жовчних кислот на синусовий вузол

Подразнення рецепторів блукаючого нерва жовчними кислотами

Подразнення провідної системи токсинами, які не були знешкоджені в печінці

Пошкоджуюча дія жовчних кислот на міокард

Подразнювальна дія жовчних кислот на синусовий вузол

4200 / 6307
У пацієнта з аускультацією серця прослуховується патологічний піум у другому міжребер’ї праворуч від грудини. Ураження якого клапана можна припустити?

Легеневого

Аортального

Тристулкового

Мітрального

4201 / 6307
У новонародженої дитини спостерігається зниження інтенсивності смоктання, часте блювання, гіпотонія. У сечі та крові значно підвищена концентрація цитруліну. Який метаболічний процес порушений?

ЦТК

Орнітиновий цикл

Гліколіз

Цикл Корі

Глюконеогенез

4202 / 6307
У хворої дитини з підозрою на дифтерію було взято на дослідження ви-ділення ураженої слизової оболонки зіву. Приготовано і забарвлено мазок. Під час мікроскопії виявлено жовті палички з темно-синіми потовщеннями на кінцях. Який структурний елемент мікробної клітини визначається у виявлених мікроорганізмів?

Зерна волютину

Спори

Капсула

Плазміди

Джгутики

4203 / 6307
На розтині тіла померлої виявлено такі морфологічні прояви: стеноз ліво-го атріовентрикулярного отвору, недостатність мітрального клапана. Гістологічно в міокарді спостерігається вогнищевий кардіосклероз, наявність квітучих гранульом Ашоффа-Талалаєва. Який із наведених нижче діагнозів найімовірніший?

Системний червоний вовчак

Вузликовий периартеріїт

Дерматоміозит

Склеродермія

Ревматизм

4204 / 6307
Експериментальній тварині після попередньої сенсибілізації підшкірно введено дозу антигену. У місці ін’єкції розвинулось фібринозне запалення з альтерацією стінок судин, основної речовини та волокнистих структур сполучної тканини у вигляді мукоїдного та фібриної- дного набухання, фібриноїдного некрозу. Яка імунологічна реакція має місце?

Гіперчутливість сповільненого типу

Гранульоматоз

Гіперчутливість негайного типу

Нормергічна реакція

Реакція трансплантаційного імунітету

4205 / 6307
На електронній мікрофотографії ділянки нирки у стінці приносної та вино-сної артеріол визначаються клітини з великими секреторними гранулами в цитоплазмі. Визначте структурне утворення нирки, до складу якого входять ці клітини:

Дистальний відділ нефрона

Ниркове тільце

Юкстагломерулярний апарат

Петля нефрона

Проксимальний відділ нефрона

4206 / 6307
Для розвитку гарячкових станів характерним є зростання рівня білків 'го-строї фази” - церулоплазміну, фібриногену, С-реактивного протеїну. Укажіть можливий механізм цього явища:

Стимулювальний вплив ІЛ-1 на гепатоцити

Проліферативна дія ІЛ-2 на Т-лімфо- цити

Руйнівна дія підвищеної температури на клітини організму

Дегрануляція тканинних базофілів

4207 / 6307
Вивчаючи під електронним мікроскопом клітини підшлункової залози, було знайдено структури, які поділяють клітину на велику кількість комірок, каналів, цистерн та поєднані із плазмолемою. Укажіть ці органели:

Рибосоми

Ендоплазматична сітка

Комплекс Гольджі

Центросоми

Мітохондрїї

4208 / 6307
Хворий з хронічною серцево- судинною недостатністю приймав ди- гоксин. Після призначення додаткової терапії розвинулися явища інтоксикації серцевими глікозидами. Який препарат може викликати підсилення інтоксикації серцевими глікозидами?

Розчин глюкози

Магнію хлорид

Калію хлорид

Аспаркам

Кальцію хлорид

4209 / 6307
Надмірне споживання вуглеводів (600 г на добу), що перевищує енергетичні потреби у людини 28 років, буде супроводжуватися активацією:

Гліколізу

Бета-окисленню жирних кислот

Ліполізу

Ліпогенезу

Глюконеогенезу

4210 / 6307
Під час лабораторного дослідження дихальної функції крові встановлено, що має місце погіршення транспорту нею СО^. З дефіцитом якого ферменту це може бути пов’язано?

Карбоангідрази

2,3-дифосфогліцсрату

Аденілатциклази

Фосфорилази

Протеїнкінази

4211 / 6307
Під час ревізії черевної порожнини виявлено венозну кровотечу з печінково- дванадцятипалої зв’язки. Яку з вен пошкоджено?

Селезінкову вену

Нижню брижову вену

Верхню брижову вену

Нижню порожнисту вену

Ворітну вену печінки

4212 / 6307
Жінка 49 років тривалий час хворіла на хронічний гломерулонефрит, унаслідок чого настала смерть. На розтині встановлено, що нирки мають розміри 7x3x2,5 см, масу 65,0 г, щільні, дрібнозернисті. Також виявлено фібринозне запалення серозних і слизових оболонок, дистрофічні зміни паренхіматозних органів, набряк головного мозку. Яке ускладнення призвело до вказаних змін серозних оболонок і внутрішніх органів?

Уремія

Сепсис

Анемія

ДВЗ-синдром

Тромбоцитопенія

4213 / 6307
Хворому на стрептококову пневмонію призначено протимікробний засіб, що порушує побудову мікробної оболонки. Який це препарат?

Гентаміцину сульфат

Доксицикліну гідрохлорид

Азитроміцин

Еритроміцин

Бензилпеніциліну натрієва сіль

4214 / 6307
У препараті 10-дснного зародка людини видно 2 міхурці, що контактують між собою (амніотичний та жовтковий). Як називається структура, що лежить у місці їх контакту?

Амніотична ніжка

Дах жовткового міхурця

Дно амніотичного міхурця

Зародковий щиток

Позазародкова мезодерма

4215 / 6307
У 35-річного наркомана, який тривало хворів на фіброзно-кавернозний туберкульоз легень, під час патологоана- томічного дослідження встановлено, що нирки та селезінка збільшені в розмірах, щільної консистенції, на розрізі тканина із сірим відтінком та 'сальним” блиском. Мікроскопічно вичвлено в білій та червоній пульпі селезінки, у інтерстиції та мезангіумі клубочків нирок депозити Конго-рот-позитивних мас. Діагностуйте вид ураження внутрішніх органів:

Вторинний амілоїдоз

Дифузний гіаліноз

Старечий амілоїдоз

Ідіопатичний амілоїдоз

Локальний пухлиноподібний амілоїдоз

4216 / 6307
Під час лікування хронічного набрякового синдрому фуроссмідом у хворого виникло порушення катіонного складу плазми крові. Який засіб слід використовувати для його корегування?

Натрію хлорид

Натрію гідрокарбонат

Калію хлорид

Кальцію хлорид

Літію карбонат

4217 / 6307
У хворого 56 років із серцевою недостатністю спостерігається набряк стоп та гомілок, шкіра в місці набряку бліда і холодна. Яка провідна ланка патогенезу набряку у хворого?

Зменшення онкотичного тиску в капілярах

Підвищення гідростатичного тиску ввенулах

Позитивний водний баланс

Підвищення проникності капілярів

Порушення лімфовідтоку

4218 / 6307
Під час гістологічного дослідження біоптату печінки 67-річного чоловіка, який тривалий час хворіє на хронічну дифузну обструктивну емфізему, було знайдено такі морфологічні зміни: центральні вени дилатовані; у центрі часточок синусоїди гіперемовані, з ознаками капілярізації; частина гепатоцитів дистрофічно змінена; помірний периваску- лярний склероз; перипортально спостерігаються гепатоцити з ознаками жирової дистрофії, Діагностуйте вид ураження печінки:

'Гусяча” печінка

Жировий гепатоз

'Кремнієва” печінка

Портальний цироз

'Мускатна” печінка

4219 / 6307
Судово-медичний аналіз останків тіл царської сім’ї, ідентифікація останків тіла українського журналіста Георгія Гон- гадзе були здійснені за допомогою методу ДНК-діагностики - ланцюгової полі- меразної реакції, в основу якої покладено:

Аналіз нуклеотидного складу мРНК

Ампліфікацію генів

Аналіз нуклеотидного складу тРНК

Аналіз амінокислотного складу білків

Аналіз нуклеотидного складу рРНК

4220 / 6307
Під час реєстрації ЕКГ хворого з гіперфункцією щитоподібної залози зареєстровано збільшення частоти серцевих скорочень. Скорочення якого елемента ЕКГ про це свідчить?

Сегмента Р-Q

Інтервалу Р-Q

Комплексу QRS

Інтервалу Р-Т

Інтервалу R-R

4221 / 6307
Мікротравмування слизової оболонки порожнини рота виникають щодня під час вживання їжі, проте кровотеча швидко припиняється за допомогою:

Антигепаринового фактору

Тромбопластину

Лізоциму

Муцину

Гепарину

4222 / 6307
У хворого зіниця звужена і у разі зменшення освітлення не розширюється. У якому місці відбулося ураження центральної нервової системи?

Бічний ріг сірої речовини спинного мозку на рівні С VIII - Тh І

Покрив ніжки середнього мозку на рівні верхніх горбиків покришки

Бічне колінчасте тіло проміжного мозку

Верхні горбики покришки середнього мозку

Основа ніжки середнього мозку на рівні нижніх горбиків покришки

4223 / 6307
Хворий 38 років, який страждає на шизофренію, тривалий час перебував на лікуванні психотрошшми засобами. Він звернувся до лікаря зі скаргами на порушення координації рухів, тремор рук, сонливість. Яка група препаратів може викликати такий симптомокомплекс?

Нейролептики

Транквілізатори

Психомоторні стимулятори

Адаптогени

Антидепресанти

4224 / 6307
У хворого виявлено вивих кришталика та павукоподібні пальці. Який син-дром діагностує лікар, беручи до уваги ще й порушення розвитку сполучної тканини, форми кисті та стопи хворого?

Марфана

Дауна

Шерешсвського-Тернера

Клайнфельтера

Трисомїї X

4225 / 6307
У хворого з опіковою хворобою як ускладнення розвинувся ДВЗ-синдром. Яку стадію ДВЗ-синдрому можна припустити, якщо відомо, що час згортання крові хворого за Лі-Уайтом становить менше, ніж 3 хвилини?

Гіпeркоагуляції

Перехідну

Гіпокоагуляцїї

Фібринолізу

Термінальну

4226 / 6307
У хворого з двобічним ураженням надниркових залоз з’явилося темне ко-ричневе забарвлення шкірних покривів. Під час гістохімічного дослідження шкіри реакція Перлса негативна. Який пігмент зумовив зміну кольору шкіри?

Порфірин

Гемосидерин

Ліпофусцин

Білівердин

Меланін

4227 / 6307
Аміак особливо токсичний для ЦНС людини. Укажіть головний шлях його знешкодження в нервовій тканині:

Синтез солей амонію

Синтез сечовини

Синтез глутаміну

Трансамінування

Утворення парних сполук

4228 / 6307
На відміну від нервових клітин, які зазвичай не розмножуються, стовбурові клітини можуть відновлюватися багато разів. Процес багаторазового відновлення клітин називають:

Проліферація

Атрофія

Гіпертрофія

Апоптоз

Диференціація

4229 / 6307
Після хімічного опіку у хворого розвинувся стеноз стравоходу. Відбулося різке схуднення через утруднене вживання їжі. У крові виявлено еритроцити - 3,0 • 1012/л, Нb - 106 г/л, загальний білок -57 г/л. Який вид голодування у хворого?

Неповне

Водне

Абсолютне

Білкове

Повне

4230 / 6307
Реалізація загального адаптаційного синдрому здійснюється переважно через нейроендокринну систему. Якій із ланок цієї системи належить провідна роль у патогенезі реакції, що розвивається?

Гіпофізарно-тиреоїдна

Гіпофізарно-адреналова

Гіпофізарно-юкстагломерулярна

Гіпофізарно-інсулярна

Гіпофізарно-адреногенітальна

4231 / 6307
В обстежуваного відсутній колінний рефлекс. Укажіть рівень ушкодження спинного мозку:

III - IV поперекові сегменти

IX - X грудні сегменти

І - II поперекові сегменти

V - VII шийні сегменти

VII - VIII грудні сегменти

4232 / 6307
Чоловіка 45 років протягом останніх 3 років турбував сухий кашель, наростали задишка, легенева недостатність, швидка втрата ваги. На розтині встановлено легеневе серце. У легенях різко виражений фіброз із наявністю порожнин, що створюють картину 'медових стільників'. Гістологічно спостерігається інтерстиціальний фіброз з вираженою інфільтрацією строми лімфогістіоцита- ми з домішкою нейтрофілів. Установіть діагноз:

Пиловий пневмосклероз

Бронхоектатична хвороба

Післязапальний пневмосклероз

Хронічна бульозна емфізема

Бронхіальна астма

4233 / 6307
У гістопрепараті представлений орган, у власній пластинці слизової обо-лонки якого розташовані прості трубчасті залози, що складаються переважно з головних і парієтальних, а також слизових, шийкових ендокринних клітин. Укажіть вид залоз:

Кардіальні затози стравоходу

Пілоричні залози шлунка

Власні залози шлунка

Кардіальні залози шлунка

Власні залози стравоходу

4234 / 6307
У людини збільшена вентиляція легень унаслідок фізичного навантаження. Який з наведених показників зовнішнього дихання у неї значно більший, ніж у стані спокою?

Резервний об’єм вдиху

Загальна ємність легенів

Життєва ємність легенів

Резервний об’єм видиху

Дихальний об’єм

4235 / 6307
До стаціонару надійшов хворий з отруєнням грибами, серед яких випадково виявився мухомор. Крім промивання шлунка, активованого вугілля і сольових проносних усередину, а також інфузійної дезінтоксикаційної терапії, хворому призначили ін’єкції атропіну сульфату, унаслідок чого симптоми отруєння значно послабилися. Укажіть тип взаємодії мускарину (алкалоїду мухомора) і атропіну сульфату:

Прямий функціональний однобічний антагонізм

Хімічний антагонізм

Непрямий функціональний антагонізм

Фізико-хімічний антагонізм (антидоти-зм)

Опосередкований функціональний антагонізм

4236 / 6307
У пацієнта перфоративна виразка передньої стінки шлунка. У яке похідне очеревини потрапить вміст шлунка?

Лівий брижовий синус

Передшлункова сумка

Чепцева сумка

Печінкова сумка

Правий брижовий синус

4237 / 6307
Унаслідок токсичного ушкодження клітин печінки з порушенням білково- синтезуючої функції у пацієнта спостерігаються різко знижені кількість альбумінів у плазмі крові та онкотичний тиск. Яке явище буде наслідком цих змін?

Збільшення темпу діурезу

Зменшення густини крові

Поява набряків

Зменшення діурезу

Зменшення ШОЕ

4238 / 6307
Офтальмолог із діагностичною метою (розширення зіниць для огляду очного дна) використав 1% - й розчин мезатону. Мідріаз, викликаний препаратом, обумовлений:

Активацією а2-адренорецепторів

Блокадою а1-адренорецеиторів

Активацією М-холінорецепторів

Активацією bі-адренорецепторів

Активацією а1-адренорецепторів

4239 / 6307
Хворий 55 років обстежувався у ендокринолога через порушення ендокринної функції підшлункової залози, що виявляється у зменшенні гормона глюкагону в крові. Функція яких клітин цієї залози порушена в цьому разі?

РР-клітини

А-клітини

В-клітини

D-клітини

D1-клітини

4240 / 6307
У чоловіка, померлого від внутрішньої кровотечі (гемоперитонеум), у печінці субкапсулярно виявлено губчастий вузол темно-червоного кольору розмірами 15 х 10 см, добре відмежований від навколишньої тканини. Мікроскопічно виявлено, що тканина вузла складається з великих судинних тонкостінних порожнин, вистелених ендотеліальними клітинами та заповнених рідкою або згорнутою кров’ю. Установіть вид пухлини:

Капілярна гемангіома

Лімфангіома

Кавернозна гемангіома

Гсмангіопсрицитома

Венозна гемангіома

4241 / 6307
Аналіз крові виявив знижений вміст гемоглобіну. Яка функція крові порушиться?

Забезпечення імунітету

Транспорт поживних речовин

Транспорт гормонів

Зсідання

Транспорт газів

4242 / 6307
Пацієнт упродовж 15 років хворіє на бронхіальну астму. Які зміни лейкоцитарної формули може знайти лікар у цього пацієнта?

Лейкоцитоз

Базофілія

Зсув лейкоцитарної формули вліво

Лейкопенія

Еозинофілія

4243 / 6307
У хворого з підозрою на черевний тиф упродовж двох тижнів захворювання лабораторний діагноз не був встановлений. Який матеріал треба направити до лабораторії для бактеріологічного дослідження на третьому тижні?

Промивні води шлунка

Фекалії та сечу

Слиз із зіву

Харкотиння

Слиз із носу

4244 / 6307
Чоловік 40 років скаржиться на загальну слабкість, головний біль, кашель із виділенням мокротиння, задишку. Після клінічного огляду й обстеження поставлено діагноз ”пневмонія'. Який тип гіпоксії має місце у хворого?

Гемічна

Циркуляторна

Тканинна

Гіпоксична

Респіраторна

4245 / 6307
Після проведення туберкулінової проби (проба Манту) у дитини за 48 годин на місці введення туберкуліну утворилася папула діаментром до 10 мм. Який механізм гіперчутливості лежить в основі описаних змін?

Клітинна цитотоксичність

Анафілаксія

Гранулематоз

Імунокомплсксна цитотоксичність

Антитілозалежна цитотоксичність

4246 / 6307
Хворого доставили до клініки у коматозному стані. В анамнезі вказано цукровий діабет І типу впродовж 5 років. Об’єктивно: дихання шумне, глибоке, у видихуваному повітрі відчутно запах ацетону. Уміст глюкози у крові -18,2 ммоль/л, кетонових тіл - 100 мкмоль/л. Для якого ускладнення цього захворювання характерні такі розлади?

Гіперглікемічна кома

Печінкова кома

Кетоацидотична кома

Гіперосмолярна кома

Гіпоглікемічна кома

4247 / 6307
Хворому зі стрептококовою інфекцією ясен було призначено препарат, що містить у своїй структурі b-лактамне кільце. Який препарат із наведених належить до цієї групи?

Стрептоміцину сульфат

Бензилпеніцилін

Рифампіцин

Еритроміцин

Левоміцетин

4248 / 6307
Внутрішньоклітинне дослідження біопотенціалів ізольованої культури тканини показало, що потенціали дії, які виникають у клітинах, характеризуються наявністю плато тривалістю до 300 мс у фазі рeполяризації. Яку тканину досліджували?

Скоротливий міокард

Скелетний м’яз

Атипові кардіоміоцити

Гладенький м’яз

Нервове волокно

4249 / 6307
У хворого діагностовано бронхіальну астму. Які зміни показників вентиляції легень будуть спостерігатися?

Збільшення об’єму форсованого видиху

Збільшення життєвої ємності легень

Зменшення залишкового об’єму легень

Зменшення об’єму форсованого видиху

Збільшення резервуарного об’єму видиху

4250 / 6307
У хворого з гострим циститом під час дослідження сечі виявили лейкоцити й багато грамнегативних паличок. Під час посіву виросли колонії слизового характеру, які утворювали зелений, розчинний пігмент. Який мікроорганізм, імовірно, є причиною захворювання?

КІеbsіеllа рпеитопіае

Sаlтопеllа епtеrіtіdіs

Рrоtеиs тіrаbіlіs

Рsеиdотопаз аеrиgіпоsа

Еsсhеrіhіа соli

4251 / 6307
У дитини, яка часто хворіє на ангіни та фарингіти, спостерігається збільшення лімфовузлів і селезінки. Зовнішній вигляд характеризується пастозністю та блідістю, м’язова тканина роз-винена слабко. У крові спостерігається лімфоцитоз. Як називається такий вид діатезу?

Геморагічний

Нервово-артритичний

Лімфатико-гіпопластичний

Ексудативно-катаральний

Астенічний

4252 / 6307
Під час патоморфологічного дослідження жовчного міхура після холецистектомії виявлено, що розміри його збільшені, стінки потовщені, серозна оболонка тьмяна та повнокровна, у порожнині міхура містяться в’язкі жовто- зелені маси. Мікроскопічно в стінці міхура спостерігається дифузна інфільтрація сегментоядерними нейтрофілами. Яка форма холециститу найімовірніша?

Хронічний

Гострий флегмонозний

Гострий гангренозний

Гранульоматозний

Гострий катаральний

4253 / 6307
Унаслідок радіаційного випромінювання були ушкоджені стовбурові гемо- поетичні клітини. Утворення яких клітин сполучної тканини буде порушено?

Макрофаги

Адипоцити

Остеобласти

Фібробласти

Меланоцити

4254 / 6307
Гіповітаміноз С призводить до зменшення утворення органічного матриксу, порушення синтезу коллагену, тому що цей вітамін бере участь у процесах:

Карбоксилювання лізину

Гідроксилювання проліну

Карбоксилювання проліну

Гідроксилювання аргініну

4255 / 6307
Хвора 24 років потрапила до лікарні зі скаргами на головний біль, біль у поперековій ділянці, набряки на обличчі, загальну слабкість. Місяць тому перенесла ангіну. На час надходження: АТ - 180/110 мм рт.ст., у сечі вичвлено виражену протеінурію, мікрогематурію, лейко- цитурію. На яку форму гіпертензії страждає хвора?

Ендокринну

Ниркову

Гіпертонічну хворобу

Есенціальну

Первинну

4256 / 6307
У хворої похилого віку вже 4 рази траплявся перелом кісток верхньої кінцівки. З підвищенням місткості якої речовини пов’язане підвищення крихкості кісток у такому віці?

Неорганічних речовин

Органічних речовин

Сполучної речовини

Міжклітинної рідини

Води

4257 / 6307
У хворого 70 років перебіг атеросклерозу ускладнився тромбозом судин нижніх кінцівок, виникла гангрена пальців лівої стопи. Початок тромбоутворення, найімовірніше, пов’язаний з:

Пeрeтворeнням протромбіну в тромбін

Перетворенням фібриногену в фібрин

Зниженням синтезу гепарину

Адгезією тромбоцитів

Активацією протромбінази

4258 / 6307
Повільне наповнення шлунка чи сечового міхура в межах фізіологічної норми не викликає підвищення тиску в цих органах. Яка властивість гладеньких м’язів лежить в основі цього явища?

Скоротливість

Автоматія

Рефрактерність

Збудливість

Пластичність

4259 / 6307
Відомо, що в осіб із генетично обумовленою недостатністю глюкозо-6- фосфатдегідрогенази еритроцитів у відповідь на призначення деяких протималярійних препаратів може розвиватися гемоліз еритроцитів. Як називається цей прояв атипових реакцій на лікарські засоби?

Сенсибілізація

Тахіфілаксія

Толерантність

Алергія

Ідіосинкразія

4260 / 6307
Жінка 38 років скаржиться на постійну спрагу, часте сечовиділення, зниження апетиту, головний біль. Сеча безбарвна, прозора, слабокислої реакції, не містить глюкози. Добовий діурез до 12 л. Нестача якого гормона може бути причиною цього стану?

Вазопресину

Передсердного натрійуретичного фактору

Глюкагону

Норадреналіну

Інсуліну

4261 / 6307
Під час дослідження каріотипу п’ятирічного хлопчика виявлено 46 хромосом. Одна з хромосом 15-ї пари довша від звичайної, тому що до неї приєднана ділянка хромосоми з 21-ї пари. Укажіть вид мутації, що має місце в цього хлопчика:

Поліплоїдія

Дуплікація

Транслокація

Інверсія

Делеція

4262 / 6307
Хворий переніс повторний інтраму- ральний інфаркт міокарда. Після лікування та реабілітації був виписаний у задовільному стані під нагляд дільничого терапевта. За 2 роки загинув у автомобільній катастрофі. Який характер патологічного процесу в міокарді було вста-новлено на розтині?

Дрібновогнищевий кардіосклероз

Атрофія

Некроз

Гіперплазія

Великовогнищевий кардіосклероз

4263 / 6307
У експериментальних тварин із раціону виключили ліпоєву кислоту. Водночас у них спостерігалося пригнічення піруватдегідрогеназного комплексу. Чим є ліпоєва кислота для цього ферменту?

Продуктом

Коферментом

Інгібітором

Алостеричним регулятором

Субстратом

4264 / 6307
Унаслідок закупорки загальної жовчної протоки (установлено рентгенологічно) надходження жовчі до дванадцятипалої кишки припинилося. Слід очікувати на порушення:

Абсорбції білків

Секреції соляної кислоти в шлунку

Гальмування слиновиділення

Гідролізу вуглеводів

Емульгування жирів

4265 / 6307
Антибіотик олігоміцин інгібірує АТФ-синтазу. У якому процесі бере участь цей фермент?

У синтезі білків

У циклі трикарбонових кислот

У субстратному фосфорилюванні

У синтезі нуклеїнових кислот

В окисному фосфорилюванні

4266 / 6307
У жінки 62 років розвинулася катаракта (помутніння кришталику) на тлі цукрового діабету. Укажіть, який тип модифікації білків має місце у разі при діабетичної катаракти:

Глікозилювання

Обмежений протеоліз

Фосфорилювання

АДФ-рибозилювання

Метилювання

4267 / 6307
У жінки 50 років у яєчнику виявлено новоутворення округлої форми, діаметром 2,5 см. На розрізі в ньому є порожнина з прозорою світло-жовтою рідиною, внутрішня поверхня порожнини гладенька. Установіть макроскопічну форму пухлини:

Інфільтрат

Вузол

Вузол із некрозом у центрі

Виразка

Кіста

4268 / 6307
Після перенесеного менінгоенцефа- літу у хворої виявлені залишкові явища такі як ураження лицевого нерва з правого боку. Порушення якої групи м’язів буде спостерігатися за цієї патології?

Глибоких м’язів шиї

Надпід'язикових м’язів

Мімічних м’язів

Підпід’язикових м’язів

Жувальних м’язів

4269 / 6307
У жінки під час мейозу відбулося порушення розходження аутосом. Утворилася яйцеклітина із зайвою 18-ю хромосомою. Яйцеклітина запліднюється нормальним сперматозооном. У майбутньої дитини буде синдром:

Едвардса

Дауна

Клайнфсльтсра

Патау

Шерешевського-Тернера

4270 / 6307
Унаслідок травми мозку у жінки порушена функція епіфізу. Які з наведених функції будуть порушені у цієї жінки?

Цикл сон-неспання

Менструальний цикл

Частота серцевих скорочень

Серцевий цикл

Частота дихання

4271 / 6307
У жінки з ішемічною хворобою серця ЧСС на ЕКГ становить 230/хв., зубець Р деформований, шлуночкові комплекси без змін. Які порушення серцевого ритму в хворої?

Тріпотіння передсердь

Миготлива аритмія

Шлуночкова екстрасистола

Передсердна пароксизмальна тахікардія

Фібриляція шлуночків

4272 / 6307
Хворий 15 років надійшов до алергологічного відділення з діагнозом 'бронхіальна астма'. Надмірне утворення яких антитіл обумовлює розвиток основних клінічних симптомів?

ІgD

ІgА

ІgЕ

IgM

4273 / 6307
Хворий надійшов у реанімаційне відділення в стані гіпоксії, що виникла внаслідок аспірації блювотних мас. Об’єктивно: стан важкий, шкірні покриви вологі, бліді, з акроціанозом. Спостерігаються тахіпное, тахікардія, зниження АТ. Який із наведених симптомів гіпоксії, що розвивається гостро, належить до термінових захисно-пристосувальних реакцій організму?

Розвиток акроціанозу

Підвищення потовиділення

Блідість шкірних покривів

Зниження АТ

Тахікардія

4274 / 6307
Для окислення жирних кислот необхідна транспортна система, що включає аміноспирт, який транспортує жирні кислоти крізь мембрану мітохондрій. Назвіть цю сполуку:

Карнозин

Карнітин

Кардіоліиін

Карбомоїлфосфат

Крсатинін

4275 / 6307
Для усунення больового синдрому пацієнту з інфарктом міокарда лікар призначив анальгетичний препарат. Стан хворого поліпшився, але з часом з’явилися ейфорія, міоз. Лікар виявив пригнічення дихання. Який лікарський засіб був призначений?

Ібупрофен

Баралгін

Парацстамол

Морфіну гідрохлорид

Мелоксикам

4276 / 6307
На аутопсії тіла жінки, яка хворіла на хронічну дизентерію, під час гістологічного дослідження внутрішніх органів у стромі та паренхімі міокарда, нирок, у слизовій оболонці шлунка та в сполучній тканині легень виявлені аморфні відкладення фіолетового кольору, що дають позитивну реакцію за Коссом. Яке ускладнення розвинулося у хворої?

Метаболічне звапніння

Амілоїдоз

Метастатичне звапнення

Дистрофічне звапнення

Гіаліноз

4277 / 6307
Зі слизових оболонок і харкотиння хворого, який тривалий час приймав імунодепресанти, були виділені великі грампозитивні овальні клітини з брунькуванням, розташовані хаотично, та довгасті клітини, розташовані ланцюжками. Який збудник виділений?

Ієрсинії

Актиноміцети

Стрсптобактерії

Стрептококи

Кандиди

4278 / 6307
У 12-річного хлопчика в сечі виявлено високий вміст усіх амінокислот аліфатичного ряду. Водночас зафіксована найвища екскреція цистину та цистеїну. Крім того, УЗД нирок показало наявність у них каменів. Виберіть можливу патологію:

Алкаптонурія

Хвороба Хартнупа

Сечокам’яна хвороба

Цистинурія

Фенілкетонурія

4279 / 6307
У жінки з групою крові III (В), Rh (-) народилася дитина з групою крові II (А). У дитини діагностовано гемолітичну хворобу новонароджених, спричинену резус-конфліктом. Яка група крові та резус-фактор можливі у батька?

II (А), Rh (-)

III (В), Rh (+)

II (А), Rh (+)

І (0), Rh (+)

І (0), Rh (-)

4280 / 6307
Унаслідок травми відбувся розрив нюхових волокон, які виходять із порожнини носа. Крізь яку кістку проходять ці волокна?

Верхню щелепу

Носову

Решітчасту

Клиноподібну

Нижню носову раковину

4281 / 6307
Хворому 35 років для обстеження очного дна був призначений атропіну сульфат у формі очних крапель. Для відновлення акомодації йому закрапали пілокарпіну гідрохлорид, але це не дало бажаного ефекту. Що є причиною відсутності ефекту?

Однобічний антагонізм

Тахіфілаксія

Двобічний антагонізм

Звикання

Синергізм

4282 / 6307
Під дією декарбоксилаз утворюються біогенні аміни. Який біогенний амін запускає багатостадійний механізм регуляції секреції НСl у шлунку?

Серотонін

Глутамін

ГАМК

Гістамін

Дофамін

4283 / 6307
На спеціальному живильному середовищі після посіву гною, що виділяється з уретри, виросли ніжні блакитнуваті колонії. При мікроскопії препаратів з них виділені грамнегативні бобовидні диплококи. Збудником якої хвороби вони є?

Туляремії

Меліоїдозу

Хламідіозу

Гонореї

Сифілісу

4284 / 6307
Молекули зрілої іРНК у клітині є носієм генетичної інформації про послідовність з’єднання між собою певних амінокислот. Це означає, що в молекулах іРНК закодована:

Первинна структура ліпідів

Первинна структура білка

Вторинна структура вуглеводів

Первинна структура полінуклеотидів

Первинна структура вуглеводів

4285 / 6307
Під час споживання їжі, що містить білок глютен, розвивається целіакія (глютенова хвороба), яка характеризується дегенерацією кишкових ворсинок із втратою їх абсорбтивної функції, діареєю і стеатореєю, здуттям живота, втратою ваги та іншими позакишковими проявами. Глютен є білком:

Яєць

Кукурудзи

Суниць

Рису

Пшениці

4286 / 6307
Чоловік 48 років помер при явищах серцевої недостатності. Під час макроскопічного дослідження серця встановлено, що стулки мітрального клапану ущільнені, потовщені та помірно деформовані. Мікроскопічно виявлено пучки колагенових фібрил гомогенізовані, еозинофільні, оточені незначним ма- крофагальним інфільтратом, метахро- мазія не виявляється. У разі забарвлення пікрофуксином виявляються вогнища жовтого кольору. Діагностуйте вид ураження сполучної тканини:

Мукоїдне набухання

Фібриноїдне набухання

Склероз

Гіаліноз

Амілоїдоз

4287 / 6307
У дитини 15 років за 14 днів після перенесеної ангіни з’явилися набряки на обличчі зранку, підвищення артеріального тиску, сеча має вигляд 'м’ясних помиїв'. Імуногістохімічне дослідження біоптату нирки виявило відкладання імунних комплексів на базальних мембранах капілярів та у мезангії клубочків. Яке захворювання розвинулося у пацієнта?

Гострий пієлонефрит

Ліпоїдний нефроз

Гострий інтерстиціальний нефрит

Некротичний нефроз

Гострий гломерулонефрит

4288 / 6307
У хворого з підозрою на озену з носоглотки були виділені грамнегативні палички, які утворювали капсулу на поживному середовищі. Які мікроорганізми спричинили хворобу?

Хламідїї

Мікоплазми

Сальмонели

Шигeли

Клебсієли

4289 / 6307
Жінка літнього віку зазнала сильного стресу. У крові різко збільшилася концентрація адреналіну і норадреналіну. Які ферменти каталізують процес інактивації катехоламінів?

Тирозинази

Глікозидази

Карбоксилази

Пептидази

Моноаміноксидази

4290 / 6307
Під час макро- та мікроскопічних досліджень ділянки великогомілкової кістки та м'яких тканин довкола виявлено дифузне гнійне запалення, що захоплює кістковий мозок, гаверсові канали та периост, осередки некрозу. Яке захворювання можна припустити?

Хвороба Педжета

Остеонекроз

Гострий гематогений остеомієліт

Паратиреоїдна остеодистрофія

Хронічний гематогений остеомієліт

4291 / 6307
Під час аналізу крові виявлено фізіологічний лейкоцитоз. Причиною цього може бути те, що перед дослідженням людина:

Поснідала

Випила води

Не снідала

Запальний процес

Палила тютюн

4292 / 6307
У дитини 14-ти років був виявлений позитивний азотистий баланс. Що з наведеного може може бути причиною цього?

Голодування

Ріст організму

Значні фізичні навантаження

Зниження вмісту білка у їжі

Емоційне напруження

4293 / 6307
До клініки надійшла дитина 4-х років з ознаками тривалого білкового голодування: затримка росту, анемія, набряки, розумова відсталість. Причиною розвитку набряків у цієї дитини є зниження синтезу:

Ліпопротеїнів

Альбумінів

Гемоглобіну

Глобулінів

Глікопротеїнів

4294 / 6307
У чоловіка 58 років є ознаки атеросклеротичного ураження серцево-судинної системи. Збільшення якого з наведених показників біохімічного аналізу крові найхарактерніше для цього стану:

Активності аланінамінотрансферази

Хіломікронів

Рівня ЛПНЩ

Активності сукципатдегідрогенази

Рівня ЛПВЩ

4295 / 6307
Чоловік 38 років поступив у терапевтичне відділення з діагнозом: правосторонній ексудативний плеврит. Відкачана з плевральної порожнини грудної клітки рідина прозора, має відносну щільність 1.020; містить 55 г\л, білка альбуміно-глобуліновий коефіцієнт-1.6; загальна кількість клітин-2.8. в 1 мкл; pH-6.5.Який тип ексудату має місце у хворого?

Гнилісний

Фібринозний

Гнійний

Серозний

Геморагічний

4296 / 6307
Новонароджений, який з’явився на світ у домашніх умовах без кваліфікованої медичної допомоги і маги якого не ііроходила у період вагітності відповідних обстежень, на другу добу доставлений до лікарні з гострим гнійним кон’юнктивітом. Під час мікроскопії виділень з очей дитини виявлені грамнегативні диплококи, розташовані всередині лейкоцитів та поза клітинами. Який мікроорганізм є найімовірнішим збудником захворювання?

Corynebacterium diphtheriae

Chlamydia trachomatis

Pseudomonas aeruginosa

Staphylococcus aureus

Neisseria gonorrhoae

4297 / 6307
Під час мікроскопічного дослідження легені недоношеної дитини виявлено спадання стінок альвеол через відсутність сурфактанту. Укажіть, з порушенням розвитку яких клітин стінки альвеоли це пов’язано:

Секреторних клітин

Фібробластів

Альвеолярних макрофагів

Альвеолоцитів II типу

Альвеолоцитів І типу

4298 / 6307
У потерпілого колото-різана рана нижнього відділу задньої стінки пахвової ямки. Які м’язи пошкоджено?

Триголовий м’яз плеча

Дельтоподібний м’яз

Великий грудний м’яз

Підостний м’яз

Найширший м’яз спини

4299 / 6307
42-річний чоловік захворів гостро після переохолодження. Захворювання супроводжувалось серцево-дихальною недостатністю, від якої він помер. Під час розтину виявлено, що уся права легеня збільшена, гепатизована, на плеврі значні фібринозні накладення. На розрізі легеня сірого кольору, зерниста, з поверхні стікає каламутна рідина. Гістологічне дослідження виявило гостре запалення з наявністю у просвітах альвеол гнійно-фібринозного ексудату. Який із наведених діагнозів найімовірніший?

Крупозна пневмонія

Ідіопатичний фіброзуючий альвеоліт

Інтерстиційна міжальвеолярна пневмонія

Осередкова пневмонія

Стафілококова пневмонія

4300 / 6307
На ізольованому серці шляхом охолодження припиняють функціонування окремих структур. Яку структуру охолодили, якщо серце внаслідок цього спочатку припинило скорочення, а потім відновило їх із частотою у 2 рази меншою ніж вихідна?

Ніжки пучка Гіса

Волокна Пуркіньє

Атріовентрикулярний вузол

Синоатріальний вузол

Пучок Гіса

4301 / 6307
Жінці проведена операція з приводу позаматкової (трубної) вагітності. Гілки яких артерій повинен перев’язати хірург під час операції?

Маткової і яєчникової

Маткової і нижньої міхурової

Нижньої міхурової і яєчникової

Маткової і верхньої міхурової

Верхньої міхурової і яєчникової

4302 / 6307
У крові резус-негативної жінки під час вагітності виявлені специфічні білки, здатні руйнувати резус-позитивні еритроцити плода. Як називається цей захисний компонент організму матері?

Антитіло

Сироватка

Фактор некрозу пухлини

Резус-фактор

Гормон

4303 / 6307
Жінка 26 років через рік після важких пологів з кровотечею скаржиться на загальну слабкість, втрату маси тіла на 18 кг, відсутність менструацій. Об'єктивно: гіпоплазія молочних залоз. Діагностовано хвороба Сіммондса. Що являється основним механізмом втрати ваги у жінки?

Зниження продукції гормонів аденогіпофізу

Гіпопаратиреоз

Зниження функції статевих залоз

Гіпотиреоз

Зниження функції кіркового шару наднирників

4304 / 6307
Останніми роками застосовується метод геноіндикації збудників, що дає можливість виявити в досліджуваних зразках фрагменти нуклеїнових кислот патогенів. Виберіть із наведених реакцій ту, яка підходить для цього:

Реакція преципітації

Полімеразна ланцюгова реакція

Реакція наростання титру фага

Радіоімунний аналіз

Імуноферментний аналіз

4305 / 6307
Під час експерименту на жабі вивчали міотатичний рефлекс. Однак піл час розтягнення скелетного м’яза, його рефлекторне скорочення не відбулося. На порушення функції яких рецепторів слід звернути увагу?

Сухожильних органів Гольджі

Больових

Суглобових

Тактильних

М’язових веретен

4306 / 6307
У чоловіка, хворого на цукровий діабет, виникли значна спрага, дисфагія та порушення психічної діяльності. Який тип розладів водно-електроліпюго балансу характеризує поява цих ознак?

Гідратація ізоосмотична

Гідратація гіпоосмотична

Дегідратація гіперосмотична

Дегідратація ізоосмотична

Дегідратація гіпоосмотична

4307 / 6307
Пацієнту після видалення щитоподібної залози призначили препарат замісної терапії. Виберіть із наведених цей медикаментозний засіб:

Калію йодид

Кортикотропін

Протирелш

L-тироксин

Мерказоліл

4308 / 6307
На розтині чоловіка, який тривалий час хворів на пресенільне недоумство, виявлено атрофію кори головного мозку, стоншення переважно лобових, скроневих та потиличних часток. Мікроскопічно спостерігається: в корі атрофованих часток мозку виявляють старечі бляшки, пошкоджені нейрони, тільця Хірано. Який діагноз найімовірніший?

Хвороба Альцгеймера

Розсіяний склероз

Хвороба Паркінсона

Церебральний атеросклероз

Хвороба Кройцфельда-Якоба

4309 / 6307
Під час копрологічного дослідження встановлено, що кал знебарвлений, у ньому знайдено краплі нейтрального жиру. Найбільш імовірною причиною цього є порушення:

Процесів всмоктування в кишечнику

Секреції підшлункового соку

Надходження жовчі в кишечник

Секреції кишкового соку

Кислотності шлункового соку

4310 / 6307
Унаслідок гострої ниркової недостатності у чоловіка виникла олігурія. Яка добова кількість сечі відповідає симптому?

100-500 мл

1000-1500 мл

1500-2000 мл

50-100 мл

500-1000 мл

4311 / 6307
На аутопсії хворого, померлого від отруєння етиленгліколем, нирки дещо збільшені у розмірах, набряклі, їх капсула знімається дуже легко, кіркова речовина широка, блідо-сіра, мозкова речовина - темно-червона. Яка патологія нирок розвинулась у хворого?

Гострий пієлонефрит

Гострий гломерулонефрит

Гострий тубуло-інтерстиціальний нефрит

Ліпоїдний нефроз

Некротичний нефроз

4312 / 6307
Людина стоїть у кімнаті в легкому одязі. Іемпература повітря 14°С. Вікна і двері зачинені. Яким шляхом організм людини віддає найбільше тепла у таких умовах?

Теплопроведення

Випаровування

Теплорадіація

Перспірація

Конвекція

4313 / 6307
В епідермісі є клітини, що виконують захисну функцію та мають моноцитарний генез. Які цс клітини?

Кератиноцити базального шару

Кератиноцити зернистого шару

Меланоцита

Кератиноцити остистого шару

Клітини Лангерганса

4314 / 6307
Для лікування злоякісних пухлин призначають метотрексат - структурний аналог фолієвої кислота, який є конкурентним інгібітором дигідрофолатредуктази і тому гальмує синтез:

Гііцсрофосфатидів

Жирних кислот

Глікогену

Моносахаридів

Нуклеотидів ДНК

4315 / 6307
На заняттях із лікувальної фізкультури лікар-фізіотерапевт запропонував юнакам відхилитися назад і дістати долонями підлогу. Яка зв’язка запобігає надмірному розгинанню хребтового стовпа?

Передня поздовжня

Жовта

Задня поздовжня

Надостьова

Міжпоперечпа

4316 / 6307
Під час дослідження біоптату виявлена гранульома, що складається з лімфоцитів, плазматичних клітин, макрофагів із пінистою цитоплазмою (клітинами Мікуліча), багато гіалінових куль. Яке захворювання можна припустити?

Сифіліс

Риносклерома

Туберкульоз

Актиномікоз

Лепра

4317 / 6307
Жінці під час пологів перелили кров донора, який прибув із Анголи. За два тижні у рецишєнтки виникла пропасниця. Було запідозрено малярію. Яке лабораторне дослідження потрібно провести для підтвердження діагнозу?

Вивчення лейкоцитарної формули

Визначення збудника методом посіву крові на живильне середовище

Вивчення мазка і товстої краплі крові для виявлення еритроцитарних стадій збудника

Проведення серологічних досліджень

Вивчення пунктату лімфатичних вузлів

4318 / 6307
В умовах гострого експерименту кролику зробили перев’язку ниркової артерії. Унаслідок цього у тварини значно зріс рівень артеріального тиску, ще є результатом збільшення секреції.

Вазопресину

Адреналіну

Натріиуретичного гормону

Норадреналіну

Реніну

4319 / 6307
Жінка 40 років звернулася зі скаргами на неможливість розігнути стопу і пальці, що створює труднощі під час ходіння. Об'єктивно спостерігається: ступня звисає, дещо повернена всередину, пальці її зігнуті («кінська стопа»), чутливість втрачена на зовнішній поверхні гомілки та тильній поверхні стопи. Який нерв уражений?

Стегновий

Сідничний

Підшкірний

Великогомілковий

Спільний малогомілковий

4320 / 6307
Під час повторної постановки реакції аглютинації Відаля виявлено наростання титрів антитіл до О-антигенів S. typhi в сироватці пацієнта з 1:100 до 1:400. Як можна тлумачити отримані результати?

Хворіє па черевний тиф

Раніше був щеплений проти черевного тифу

Є гострим носієм черевнотифозних мікробів

Є хронічним посієм черевнотифозних мікробів

Раніше перехворів на черевний тиф

4321 / 6307
Аналіз сироватки крові пацієнта з гострим гепатитом показує підвищений рівень аланінамінотрансферази (АЛТ) та аспартатамінотрансферази (АСТ). Які зміни на клітинному рівні призвести до таких показників?

Порушення клітинних ферментних систем

Порушення міжклітинних взаємодій

Порушення енергопостачання клітин

Пошкодження генетичного апарату клітин

Руйнування клітин

4322 / 6307
У пацієнта з бронхіальною астмою за допомогою шкірних алергічних проб установлено сенсибілізацію алергеном тополиного пуху. Який фактор імунної системи відіграє вирішальну роль у розвитку цього імунопатологічного стану?

IgE

Сенсибілізовані Т-лімфоцити

IgD

IgG

IgM

4323 / 6307
Під час планового обстеження у вагітної взята кров з вени для постановки реакції Вассермана. Реакція виявилася позитивною. Позашлюбні статеві зв’язки вагітна та її чоловік заперечують. Що потрібно зробити, щоб підтвердити або спростувати діагностування сифілісу?

Зробити мазок з уретри

Поставити реакцію іммобілізації блідих трепонем

Поставити реакцію зв’язування комплементу

Повторити реакщю Вассермана

Поставити осадові реакції

4324 / 6307
У разі травматичного пошкодження верхніх кінцівок можливий розвиток дегенерації нервових волокон, яка супроводжується розпадом осьових циліндрів і мієліну. За допомогою яких нервових структур відбувається відновлення мієліну в процесі регенерації?

Ендоневриту

Нейролемоцитів (Шваннівських клітин)

Псриневриту

Астроцитів

Лізаксону

4325 / 6307
У чоловіка колаптоїдний стан через зниження тонусу периферичних судин. Який препарат найефективніший у цій ситуації?

Празозин

Ізадрин

Прозерин

Мезатон

Клофелін

4326 / 6307
Людина постійно живе високо в горах. Яку зміну показників крові можна виявити у неї?

Зменшення колірного показника

Зниження Кількості рстикулоцитів

Еритроцитоз

Появу в крові еритробластів

Зниження показників вмісту гемоглобіну

4327 / 6307
Людині, у якої напад бронхоспазму, треба зменшити вплив блукаючого нерва на гладеньку мускулатуру бронхів. Які мембранні циторецептори доцільно заблокувати для цього?

α-адренорецептори

Н-холінорецептори

β-адренорецептори

М-холінорецептори

α- та β-адренорецептори

4328 / 6307
Чоловік 35 років звернувся до лікарні зі скаргою на носові кровотечі. Лікар призначив пацієнту препарат, який є коагулянтом непрямого типу дії. Визначте цей препарат:

Вікасол

Фраксипарин

Заліза сульфат

Кислота амінокапронова

Тромбін

4329 / 6307
У чоловіка під час профілактичного обстеження на медіальній стінці лівої пахвової западини виявлено збільшений лімфовузол метастатичного походження. Укажіть найімовірнішу локалізацію первинної пухлини:

Шлунок

Молочна залоза

Легеня

Щитоподібна залоза

Піднижньощелепна слинна залоза

4330 / 6307
Чоловік 55 років прооперований із приводу гострого апендициту. На 5-ту добу, піднявшись із ліжка, відчув брак повітря. У нього розвинувся різкий ціаноз обличчя. Пацієнт втратив свідомість. Після безрезультатної реанімації констатована смерть. На розтині виявлена тромбоемболія легеневого стовбура. Що могло бути найімовірнішим джерелом тромбоемболії?

Тромбоз вен нижніх кінцівок

Тромбоз брижових артерій

Тромбоз ворітної вени

Тромбоз у лівому шлуночку серця

Кулястий тромб передсердя

4331 / 6307
У людей похилого віку часто спостерігається демінералізація кісток (зниженим вміст іонів кальцію). Причиною цього може бути знижена секреція:

Тироксину

Паратгормону

Тиреокальцитоніну

Інсуліну

Альдостерону

4332 / 6307
У жінки після операції видалення ракової пухлини молочної залози та регіонарних лімфовузлів розвинувся набряк руки, який був пов'язаний із лімфатичною недостатністю. Який це вид лімфатичної недостатності за механізмом виникнення?

Спастична

Механічна

Резорбційна

Динамічна

Акінетична

4333 / 6307
Дитина 3 років померла від гострої пневмонії на глі хронічної серцевої недостатності. Під час аутопсії виявлено: дефект міжпілуночкової перегородки, стеноз гирла легеневої артерії, гіпертрофія правого шлуночка серця, декстрапозиція аорти. Яка вада серця у дитини була встановлена під час аутопсії?

Пентада Фалло

Синдром Марфана

Тетрада Фалло

Синдром Патау

4334 / 6307
Чоловікові для введення лікувальної дози протиправцевої сироватки було зроблено пробу на чутливість, яка виявилась позитивною. Специфічну гіпосенсибілізацію хворому найправильніше провести введенням:

Антигістамінних препаратів

Дозволеної дози протиправцевої сироватки

Глюкокортикоїдів

Малих доз протиправцевої сироватки

Імунодепресантів

4335 / 6307
Артеріальна гіпертензія у жінки 44 років обумовлена наявністю феохромоцитоми - пухлини мозкового шару наднирників. Антигіпсртензивні засоби якої групи найдоцільніше буде призначити?

Бета-адреноблокатори

Альфа-адреноблокатори

Гангліоблокатори

Симпатолітики

Антагоністи кальцію

4336 / 6307
На ЕЕГ у потиличних відведеннях зареєстровано альфа-ритм. Яким є стан досліджуваного?

Спокій із розплющеними очима

Спокій із заплющеними очима

Глибокий сон

Стрес

Стан наркозу

4337 / 6307
44.Фермент оксидаза D-амінокислоі каталізує дезамінування лише D-амінокислот. Яка властивість ферментів тоді виявляється?

Термолабільність

Відносна специфічність

Стереохімічна специфічність

Абсолютна специфічність

Залежність від pH

4338 / 6307
Чоловікові, у якого є підозра на прогресуючу м’язову дистрофію, було зроблено аналіз сечі. Яка сполука в сечі підтверджує діагноз цього пацієнта?

Порфирін

Міоглобін

Креатин

Креатинін

Колаген

4339 / 6307
У чоловіка 50 років за місяць після перенесеного інфаркту міокарда розвинулася серцева недостатність. Невідповідність між навантаженням на серце та його здатністю виконувати роботу пов’язана в цьому разі з:

Підвищеним опором вигнанню крові в легеневому стовбурі

Змішаною формою серцевої недостатності

Ушкодженням міокарда

Перевантаженням серця

Підвищеним опором вигнанню крові в аорті

4340 / 6307
Пацієнтці, у якої наявні ротеїнурія гематурія, набряки і артеріальна гіпертензія, поставлено діагноз: гострий гломерулонефрит. Назвіть алергічні реакції за Кумбсом і Джеллом, які у 80% випадків є причиною гломерулонефриту

Гіперчутливості сповільненого типу

Анафілактивні

Імунокомплексні

Стимулювального типу

Цитотоксичні

4341 / 6307
Хворому на активну форму туберкульозу призначили ізоніазид. Який вітамінний препарат необхідно застосовувати для профілактики побічної дії ізоніазиду?

Піридоксину гідрохлорид

Рутин

Ретинолу ацетат

Ціанокобаламін

Токоферолу ацетат

4342 / 6307
Під час розтину тіла дівчинки 9 років у верхівці правої легені субплеврально було виявлено вогнище казеозного некрозу діаметром 15 мм, біфуркаційні лімфатичні вузли збільшені, містять дрібні вогнища некрозу коагуляційного типу Мікроскопічно спостерігається: у легеневому вогнищі та в лімфатичних вузлах навколо некротичних мас розташовані спітсліоїдиі клітини, лімфоцити та поодинокі багатоядерні гігантські клітини. Діагностуйте захворювання:

Вторинний фіброзно-вогнищевий туберкульоз

Первинний туберкульоз

Гематогенний генералізований туберкульоз

Гематогенний туберкульоз із переважним ураженням легень

Вторинний вогнищевий туберкульоз

4343 / 6307
Під час запальних процесів в органіці починається синтез білків «гостро. фази». Які речовини є стимуляторами їх синтезу?

Імуноглобуліни

Біогенні аміни

Інтерферони

Інтерлейкін-1

Ангіотензини

4344 / 6307
У хворого внаслідок зловживання алкоголем розвинулась жирова дистрофія печінки. Порушення якої ланки обміну ліпідів має місце в даному випадку?

Обмін жирів у жировій клітковині

Всмоктування жирів

Проміжного обміну ліпідів

Бета - окислення ліпідів

Транспортування жирів

4345 / 6307
У людини виявлено протозойне захворювання, унаслідок якого вражений головний мозок і спостерігається втрата зору. У крові знайдено одноклітинні півмісяцевоі форми із загостреним кінцем. Збудником цього захворювання є:

Амеба

Лямблія

Трихомонада

Лейшманія

Токсоплазма

4346 / 6307
У новонародженого хлопчика виявлена полідактилія, мікроцефалія, незарощення верхньої губи та верхнього піднебіння, а також гіпертрофія паренхіматозних органів. Вказані вади відповідають синдрому Патау. Яка найбільш ймовірна причина даної патології?

Часткова моносомія

Трисомія 21-ої хромосоми

Трисомія 13-ої хромосоми

Нерозходження статевих хромосом

Трисомія 18-ої хромосоми

4347 / 6307
Під час глікогенозу (хвороби Гірке) порушується перетворення глюкозо-6-фосфату на глюкозу, що призводить до накопичення глікогену в печінці та нирках. Дефіцит якого ферменту є причиною захворювання?

Альдолаза

Глюкозо-6-фосфатаза

Гексокіназа

Глікогенсинтетаза

Фосфорилаза

4348 / 6307
Жінка 40 років звернулась до лікаря зі скаргами на болі в дрібних суглобах ніг і рук. Суглоби збільшені, мають вигляд потовщених вузлів. У сироватці крові виявлено підвищений вміст уратів. Причиною є порушення обміну:

Амінокислот

Піримідинів

Ліпідів

Пуринів

Вуглеводів

4349 / 6307
Розділ фармакології, що вивчає всмоктування, розподіл та елімінацію ліків в організмі людини, називається:

Фармакогенетика

Фармакодинаміка

Імунофармакологія

Токсикологія

Фармакокінетика

4350 / 6307
3 хімічного виробництва до токсико логічного відділешія доставлено чоловіка з отруєнням ртуттю. Який препарат слід використати у цій ситуації?

Ізонітрозин

Ентсросорбснт СКН

Унітіол

Налоксон

Активоване вугілля

4351 / 6307
У чоловіка, хворого на гіпертонічну хворобу, спіронолактон викликав виражений терапевтичний ефект. Його терапевтична активність обумовлена ослабленням дії:

Альдостерону

Ангіотензину II

Ангіотензинперетворювального ферменту

Реніну

Брадикініну

4352 / 6307
Фенілкетонурія успадковується як аутосомно-рецесивна ознака. У здорових батьків народилася дитина, хвора на фенілкетонурію. Які генотипи батьків?

аа х аа

Аа х аа

АА х АА

АА х Аа

Аа х Аа

4353 / 6307
Хворого з явищами енцефалопатії госпіталізували до неврологічного стаціонару і виявили кореляцію між наростанням енцефалопатії і речовинами, що надходять із кишечнику до загального кровотоку. Які з'єднання, що утворюються в кишечнику, можуть викликати ендотоксемію?

Орнітин

Індол

Бутират

Ацетоацетат

Біотин

4354 / 6307
В організмі людини визначене порушення обміну мелатоніну. Це може бути пов’язано з нестачею амінокислоти, з якої мелатонін синтезується. Яка це амінокислота?

ДОФА

Триптофан

Гістидин

Глутамат

Аланін

4355 / 6307
Чоловіку 57 років, після обстеження був поставлений діагноз - В12 дефіцитна анемія, назначене лікування. Через 3 доби був зроблений контрольний аналіз крові. Що буде найбільш адекватним критерієм підвищення еритропоезу?

Підвищення кількості ретикулоцитів

Підвищення кількості тромбоцитів

Зниження кольорового показника

Підвищення рівня гемоглобіну

Підвищення кількості лейкоцитів

4356 / 6307
У разі зниження концентрації Na+ у плазмі крові в нирках посилюється його реабсорбція. Який основний механізм регуляції стимулює цей процес?

Симпатичні рефлекси

Альдостерон

Натрійуретичний гормон

Ренін

Парасимпатичні рефлекси

4357 / 6307
Хірург виявив у чоловіка болючість у правій пахвинній ділянці. Захворювання яких органів має місце?

Підшлункова залоза і дуоденум

Апендикс, сліпа кишка

Дуоденум, тонка кишка

Висхідна ободова кишка, права нирка

Пряма кишка і сліпа кишка

4358 / 6307
У хворої дитини з явищами гнійного керато-кон'юнктивіту лікар-офтальмолог запідозрив бленорею (гонорейний кон'юнктивіт). Якими методами лабораторної діагностики слід скористатися для підтвердження діагнозу?

Біологічним та алергічним

Серологічним та алергічним

Мікроскопічним та бактеріологічним

Мікроскопічним та серологічним

Біологічним та методом фагодіагностики

4359 / 6307
При вимірюванні артеріального тиску у чоловіка віком 56 років встановлено зростання діастолічного артеріального тиску до 100 мм рт. ст. Від якого із наведених факторів в основному залежить величина діастолічного артеріального тиску?

Величини кінцево-діастолічного об'єму лівого шлуночка

Об'єму циркулюючої крові

Швидкості кровотоку

Величини ударного об'єму лівого шлуночка

Периферичного опору судин

4360 / 6307
У чоловіка з клінічними ознаками імунодефіциту проведено імунологічні дослідження. Виявлено значне зниження кількості клітин, що утворюють розетки з еритроцитами барана. На основі даних аналізу можна зробити висновок, що у пацієнта наявне зниження кількості:

Т-лімфоцитів

Компонентів системи комплементу

Натуральних кілерів (NK-клітини)

В-лімфоцитів

Клітин-ефекторів гуморального імунітету

4361 / 6307
При мікроскопічному дослідженні біопсії нирки виявлено вогнища, в центрі яких знаходяться зернисті еозинофільні маси, оточені інфільтратом з лімфоцитів, епітеліоїдних клітин та поодиноких клітин Пирогова-Лангханса. Виберіть патологічний процес, що найбільш повно відповідає зазначеним змінам:

Проліферація та диференціювання макрофагів

Коагуляційний некроз

Альтеративне запалення

Гранулематозне запалення

Казеозний некроз

4362 / 6307
Чоловік 63 років звернувся до невропатолога зі скаргою на те, що протягом трьох місяців не може здійснювати столярні роботи, які потребують точності виконання, тому що права рука робить багато нецілеспрямованих рухів. Під час дослідження виявлено, що у хворого пошкоджена:

Gyrus temporalis superior

Gyrus angularis .

Gyrus precentralis

Gyrus postcentralis

Gyrus supramarginalis

4363 / 6307
У пацієнта під час обстеження зліва в V міжребірї на 1-2 см латеральншіе від середньоключичної Ліни під час аускультації краще прослуховується І іон, ніж П. Закриттям якого клапана цс обумовлено?

Правим трьохстулковим клапаном

Двостулковим та тристулковим клапанами

Півмісяцсвим клапаном аорти

Лівим двостулковим клапаном

Півмісяцсвим клапаном легеневого стовбура

4364 / 6307
На рентгенограмі газу видно всі три частини тазової кістки, відокремлені широкими проміжками, що відповідають невидимому на рентгенограмах хрящу. Про який вік суб'єкта можна говорити?

До 50 років

До 36 років

До 16 років

До 40 років

До 46 років

4365 / 6307
Подразнення слизової оболонки бронхів викликає кашльовий рефлекс. Де замикається рефлекторна дуга цього рефлексу?

Передні роги спинного мозку

Уставні нейрони на рівні спинного мозку

Середній мозок

Бічні роги спинного мозку

Довгастий мозок

4366 / 6307
Чоловіку, який страждає на гіпертонічну хворобу (AT - 200/110 мм pт. ст.), з-поміж препаратів комплексної терапії лікар призначив апаприлін (пропранолол). За 2 тижні після початку приймання цього препарату пацієнт став скаржитися на відчуття задухи, утруднення дихання. Поясніть можливу причину ускладнень і оберіть вашу тактику в цій ситуації:

Блокада бета-2-адренорецепторів. Призначити селективний бста-1-адрсно- блокатор

Міотропна бпонхоспастична дія Призначити еуфілін

Збудження М-холінорецспторів. Ппи- зїіачити атропіп

Блокада бета-1-адренорецепторів. Призначити селективний бета-2-адрено- блокатор

Алергічна реакція. Препарат скасувати, призначити антигістамінні засоби

4367 / 6307
Відповідно до сучасної концепції атерогенезу «Response to injury», атеросклероз є проявом хроні’пюго запалення в інтимі артерій. З якою стадією запалення пов’язане формування фіброзних бляшок у разі атеросклерозу?

Проліферація

Вторинна альтерація

Трансформація

Ексудація

Первинна альтерація

4368 / 6307
У людини трапляються хвороби, пов’язані із порушенням розщеплення і накопичення у клітинах глікогену, ліпідів та ін. Причиною виникнення цих спадкових хвороб є відсутність відповідних ферментів у:

Ядрі

Мікротрубочках

Лізосомах

Ендоплазматичній сітці

Мітохондріях

4369 / 6307
Для комплексної терапії туберкульозу було призначено препарат, що є антагоністом вітаміну Вб. Назвіть цей препарат:

Ізоніазид

Етамбутол

Стрептоміцин

Левофлоксацип

ПАСК

4370 / 6307
79.На одній зі стадій клітинного циклу хромосоми досягають полюсів клітини, Дссиіралізуються, навколо них формуються ядерні оболонки, відновлюється ядерце. У якій фазі мітозу перебуває клітина?

Метафаза

Прометафаза

Профаза

Телофаза

4371 / 6307
Під час мікроскопії мазка, приготованого з досліджуваного матеріалу від хворої дитини з підозрою на дифтерію і забарвленого за Неиссером, виявлено палички снітло-коричневого кольору з темно-синіми включеннями на кінцях. Який структурний елемент мікробної клітини виявлено?

Ядерна субстанція

Спори

Зерна волютину

Джгутики

Капсула

4372 / 6307
81. Судово-медичний експерт під час розтину тіла 20-річноі дівчини встановив, що смерть настала внаслідок отруєння ціанідами. Порушення якого процесу, найімовірніше, було причиною смерті дівчини?

Транспорту кисню гемоглобіном

Синтезу гемоглобіну

Тканинного дихання

Синтезу сечовини

Транспор ту водню за допомогою малат-аспартатного механізму

4373 / 6307
Під час перетворення глюкози в пентозному циклі утворюються фосфати різних моносахаридів. Яка із цих речовин може бути використана для синтезу нуклеїнових кислот?

Еритрозо-4-фосфат

Пентозо-5-фосфат

Рибулозо-5-фосфат

Седогептулозо-7-фосфат

Рибозо-5-фосфат

4374 / 6307
На розтині виявлено зменшені в розмірах нирки, вага 50,0 г, їх поверхня дрібнозерниста, кора рівномірно стоншена. Під час мікроскопічного дослі дження спостерігається: стінки артеріол значно потовщені внаслідок відкладен ня в них гомогенних безструктурних рожевого кольору мас, просвіт їх різко звужений, клубочки зменшені, склсрозовані, канальці атрофічні. Для якого захворювання характерні описані зміни?

Гіпертонічна хвороба

Пієлонефрит зі зморщуванням нирок

Амілоїдоз нирок

Хронічний гломерулонефрит

Гострий некротичний нефроз

4375 / 6307
Чоловіку з гострим гастроентероколітом після проведених бактеріологічних досліджень установлено діагноз холера. Який антибіотик є засобом вибору у разі цього захворювання?

Цефалексин

Пеніцилін

Оксацилін

Ампіцилін

Доксициклін

4376 / 6307
До дермато-венерологічного диспансера надійшла жінка ЗО років із діагнозом: первишшй сифіліс. Який засіб найдоцільніше їй призначити?

Цефамізин

Тетрациклін

Феноксимстилпеніцилін

Левоміцетин

Бензилпеніцилін

4377 / 6307
У дитячому садку планується проведення вакцинації проти кашлюку. Який із наведених нижче препаратів слід використати для цього?

Типоспецифічна сироватка

Вакцина АКДП

Вакцина БЦЖ

АДП анатоксин

Нормальний гамма-глобуліп

4378 / 6307
У дитини 3 років, яка страждає на квашіоркор, спостерігається порушення зроговіння епідермісу та збільшення його злущення, є жирова інфільтрація печінки. Який тип голодування спостерігається у цьому разі?

Жирове

Білкове

Вуглеводне

Мінеральне

Енергетичне

4379 / 6307
Під час огляду пацієнта лікар припустив синдром Іценка-Кушинга. Визначення якої речовини в крові пацієнта підтвердить припущення лікаря

Альдостерону

17-кетостсроідш

Адреналіну

Токоферолу

Кортизону

4380 / 6307
У крові пацієнта виявлено підвищення активності ЛДГ4, ЛДГ5, АлАТ, карбамоїлорнітинтрансферази. В якому органі можна передбачити розвиток патологічного процесу?

У нирках

У сполучній тканині

У печінці (можливий гепатит)

У скелетних м’язах

У серцевому м’язі (можливий інфаркт міокарда)

4381 / 6307
У чоловіка, який хворіє на верхній тип ожиріння, клінічно тривало спостерігалися артеріальна гіпертонія, гіперглікемія, глюкозурія. Смерть настала через крововилив у головний мозок. Під час патоморфологічного дослідження виявлені базофільна аденома гіпофіза і гіперплазія кори наднирникових залоз. Який діагноз є найправильнішим?

Адипозогенітальна дистрофія

Акромегалія

Гіпофізарний нанізм

Хвороба Іценка-Кушинга

Цукровий діабет

4382 / 6307
Учням 1-го класу поставлена проба Манту. З 35 учнів у 15 проба Мапту була негативною. Що треба зробити дітям із негативною пробою Манту?

Дослідити сироватку крові

Зробити повторну пробу

Увести антирабічну вакцину

Увести вакцину БЦЖ

Увести антитоксичну сироватку

4383 / 6307
В ядрі клітини з молекули незрілої і-РНК утворилася молекула зрілої і-РНК. яка має менший розмір, ніж незріла і-РНК. Сукупність етанів цього перетворення має назву:

Рекогніція

Процесинг

Реплікація

Термінація

Трансляція

4384 / 6307
У медичній практиці широкого застосування набули курареподібні речовини (міорелаксанти). Про яку побічну дію повинен пам’ятати лікар у разі їх застосування?

Розслаблення дихальних м’язів

Зупинка серця

Напади судом

Тромбоутворення

Розлади мозкового кровообігу

4385 / 6307
Цитохімічне дослідження виявило високий вміст гідролітичних ферментів у цитоплазмі. Про активність яких органел із наведених свідчить цей факт?

Клітинного центру

Ендоплазматичної сітки

Лізосом

Полісоми

Мітохондрій

4386 / 6307
Під час фіброгастродуодсноскопії лікарю необхідно оглянути великий сосочок 12-палої кишки. Який анатомічний утвір може бути орієнтиром для його виявлення?

Циркулярні складки 12-палої кишки

Дуоденальні залози

Печінково-дуоденальна зв’язка

Цибулина 12-палої кишки

Поздовжня складка 12-палої кишки

4387 / 6307
У чоловіка під час обстеження в клініці виявлено різке зниження показників гемоглобіну. Яка функція крові порушується?

Імморальна

Трофічна

Дихальна

Гомеостатична

Захисна

4388 / 6307
У жінки 32 років після перенесеного міокардиту виявлено під час електрокардіографічного дослідження порушення серцевого ритму (ритм не синусовий). Функції яких кардіоміоцитів порушені?

Пейсмекерних клітин

Провідних кардіоміоцитів ніжок пучка Гіса

Провідних кардіоміоцитів пучка Гіса

Перехідних провідних кардіоміоцитів

Скоротливих кардіоміоцитів

4389 / 6307
У дитини 7-ми років підвищилась температура тіла до 39oC, з’явилися нежить, кон’юнктивіт, кашель. На шкірі відзначались великоплямисті висипання, під час огляду порожнини рота - білуваті висипання на слизовій оболонці щік. Раптово з’явилось важке дихання і настала смерть при явищах асфіксії. Який найбільш імовірний діагноз?

Грип

Менінгококовий назофарингіт

Скарлатина

Дифтерія

Кір

4390 / 6307
Під час аналізу крові виявлено фізіологічний лейкоцитоз. Причиною цього може бути те, що перед дослідженням людина:

Палила тютюн

Поснідала

Випила води

Запальний процес

Не снідала

4391 / 6307
У хворого 30 років на електрокардіограмі відмічено зниження амплітуди зубця R. Що означає цей зубець на ЕКГ?

Деполяризацію шлуночків

Реполяризацію шлуночків

Поширення збудження від передсердь до шлуночків

Деполяризацію передсердь

Електричну діастолу серця

4392 / 6307
У пацієнта після травми виникли паралічі, розлади больової чутливості справа. Зліва паралічі відсутні, але порушена больова і температурна чутливість. Яка причина такого явища?

Пошкодження стовбура мозку

Одностороннє ураження спинного мозку з лівої сторони

Пошкодження мозочка

Пошкодження рухової зони кори головного мозку

Одностороннє ураження спинного мозку з правої сторони

4393 / 6307
У чоловіка, у якого яскраво виражена жовтушність шкіри, склер та слизових оболонок, сеча має колір темного пива, кал ахолічний. У крові виявлено підвищений вміст прямого білірубіну. У сечі - білірубін. Який тип жовтяниці у пацієнта?

Гемолітична

Паренхіматозна

Екскреційна

Обтураційна

Кон'югаційна

4394 / 6307
Показники аналізу крові 35-річного чоловіка: НЬ — 58 г/л, еритроцити — 1,3 • 10,2/л, колірний показник - 1,3, лейкоцити - 2,8 • 109/л, тромбоцити - 1,1* 109/л, ретикулоцити - 2%, ШОЕ - 35 мм/год. Визначаються полісегментовані нейтрофіли, а також тільця Жоллі, кільця Кебота. мегалоцити. Яка це анемія?

Постгеморагічна

Гіпопластична

В12-фолієводефіцитна

Гемолітична

Залізодефіцитна

4395 / 6307
Для лікування стрептодермії лікар призначив хворому мазь, що містить антибіотик групи тетрацикліну, і рекомендував обмежити перебування на сонці. Чим небезпечна інсоляція?

Зниженням активності тетрацикліну

Розвитком стійкості збудника до антибіотика

Генералізацією процесу

Підвищенням токсичності тетрацикліну

Розвитком фотосенсибілізації

4396 / 6307
У чоловіка гострий гломерулонефрит. Поява якої речовини в сечі свідчить про ушкодження базальної мембрани капілярів клубочків нирок у разі цієї патології?

Білку

Лейкоцитів

17-кетостероїдів

Ппокозн

Креатину

4397 / 6307
У дитини, яка страждає на геморагічний синдром, діагностована гемофілія. Вона зумовлена дефіцитом фактора:

IX (Крістмаса)

II (протромбіну)

VІІІ (антигемофільного глобуліну)

XI (протромбопластину)

XII (Хагемана)

4398 / 6307
Унітіол є антидотом і застосовується, зокрема, у разі отруєнь солями важких металів. Як називається такий тин взаємодії лікарських речовин?

Неконкурентний антагонізм

Хімічний антагонізм

Синергоантагонізм

Фізичний антагонізм

Фізіологічний антагонізм

4399 / 6307
Після введення місцевого анестетика у пацієнта розвинувся анафілактичний шок. Який механізм порушення кровообігу є провідним у цьому разі?

Активація симнато-адреналової системи

Гіперволемія

Зниження скоротливої функції серця

Зниження тонусу судин

Підвищення тонусу сулин

4400 / 6307
Для підвищення результатів спортсмену рекомендували застосовувати препарат, який містить карштин. Який процес найбільше активується карнітином?

Транспорт жирних кислот у мітохондрії

Синтез кетонових тіл

Синтез ліпідів

Синтез стероїдних гормонів

Тканинне дихання

4401 / 6307
Розтин тіла показує, що м’які мозкові оболонки померлої людини є повноцінними, потовщеннями, непрозорими та мають жовто-зелений колір. Який тип ексудативного запалення може характеризуватися такими змінами в м’яких мозкових оболонках?

Гнійне

Катаральне

Серозне

Геморагічне

Фібринозне

4402 / 6307
До клініки надійшла дитина 4 роківз ознаками тривалого білкового голодування: затримка росту, анемія, набряки, розумова відсталість. Виберіть причину розвитку набряків у дитини:

Зниження синтезу глобулінів

Зниження синтезу альбумінів

Зниження синтезу глікопротеїнів

Зниження синтезу гемоглобіну

Зниження синтезу ліпопротеїиів

4403 / 6307
товщі шкіри макроскопічно виявлено рухливу пухлину. Мікроскопічно спостерігаються хаотично розташовані пучки колагенових волокон із невеликою кількістю веретеноподібних клітин. Яка пухлина виявлена?

Меланома

Ліпома

Фіброма

Лейоміома

Гломус-ангіома

4404 / 6307
113. Під час обстеження у чоловіка похилого віку виявлено моторну афазію. Де локалізований осередок пошкодження головного мозку?

Постцентральна звивина

Звивина Гешля

Центр Брока

Кутова звивина

Прецентральна звивина

4405 / 6307
Чоловіку, після інфаркту міокарда, призначено ацетилсаліцилову кислоту, 75 мт щоденно. З якою метою призначено препарат?

Зменшення запалення

Зменшення болю

Розширсшія коронарних судин

Зменшення агрегації тромбоцитів

Зниження температури

4406 / 6307
Чоловік 25 років звернувся до приймального покою зі скаргами на нежить та головний біль протягом 4 днів. Після обстеження йому встановили діагноз: фронтит. Через який носовий хід інфекція потрапила до лобової пазухи?

Верхній

Носоглотковий

Нижній

Середній

Загальний

4407 / 6307
Злоякісна гіперхромна анемія – хвороба Бірмера – виникає внаслідок нестачі вітаміну В12. Який біоелемент входить до складу цього вітаміну?

Кобальт

Молібден

Цинк

Залізо

Магній

4408 / 6307
Хвора 13 років знаходиться на стаціонарному лікуванні в гематологічному відділенні обласної дитячої лікарні з діагнозом залізодефіцитна анемія. Який тип гіпоксії має місце у цієї хворої?

Циркуляторна

Дихальна

Змішана

Гемічна

Тканинна

4409 / 6307
При макро-мікроскопічному дослідженні ділянки великогомілкової кістки та м’яких тканин довкола виявлено дифузне гнійне запалення; що захоплює кістковий мозок, гаверсові канали та періост осередки некрозу Про яке захворювання слід думати?

Гострий гематогений остеомієліт

Хронічний гематогений остеомієліт

Паратиреоїдна остеодистрофія

Хвороба Педжета

Остеонекроз

4410 / 6307
Жінка 62 років скаржиться на частий біль у ділянці грудної клітки та хребта, переломи ребер. Лікар припустив мієломну хворобу (плазмоцитому). Який з перерахованих нижче лабораторних показників буде мати найбільше діагностичне значення?

Гiперальбумiнемiя

Гіпогло6улінємія

Парапротеїнемiя

Протєїнурія

Гіпопротеїнемія

4411 / 6307
Чоловік, хворий на бронхіальну астму, тривалий час приймає преднізолон. Який механізм дії препарату?

Пригнічення активності дигідрофолатредуктази

Блокада натрієвих каналів

Блокада лсйкотрієнових рецепторів

Гальмування активності фосфоліпази А2

Блокада гістамінових рецепторів

4412 / 6307
Тривале вживання великих доз аспірину викликає пригнічення синтезу простагландинів в результаті зниження активності фермента:

Пероксидази

5-ліпоксигенази

Фосфодіестерази

Циклооксигенази

Фосфоліпази А2

4413 / 6307
У дитини, що одужує після кору, развинулася пневмонія, викликана умовно-патогенними бактеріями. Яка найбільш імовірна форма цієї інфекції?

Суперінфекція

Реінфекція

Персистуюча інфекція

Госпітальна інфекція

Вторинна інфекція

4414 / 6307
Клінічне обстеження чоловіка дало можливість установити попередній діагноз: рак шлунка. У шлунковому соці виявлено молочну кислоту. Який тип катаболізму глюкози має місце в ракових клітинах?

Аеробний гліколіз

Анаеробний гліколіз

Глюконеогенез

Пентозофосфатний цикл

Глюкозо-аланіновий цикл

4415 / 6307
Чоловік 62 роки помер від хронічної ниркової недостатності. На секції виявлено: нирки зменшені, щільної консистенції, фіброзна капсула знімається важко, поверхня зерниста, коркова речовина витончена. Гістологічно - в частині клубочків проліферація ендотеліальних та мезангіальних клітин, потовщення мембран капілярів клубочків, деякі клубочки слерозовані та гіалізовані. В стромі нечисленні лімфоїдні інфільтрати. Який імовірний діагноз?

Пієлонефрит

Артеріолосклеротичний нефросклероз (первинно зморщена нирка)

Атеросклеротично зморщена нирка

Хронічний гломерулонефрит

Гострий екстракапілярний ексудативний гломерулонефрит

4416 / 6307
126. Чоловік не може розігнути коліно, у нього відсутня шкірна чутливість на передній поверхні стегна. Неврологічне обстеження виявило ураження нерва. Який із наведених нервів є ураженим?

Верхнього сідничного

Статево-стегнового

Стегнового

Заратульного

Сідничного

4417 / 6307
Із сироватки крові людини виділили п'ять ізоферментних форм лактатдегідрогенази і вивчили їх властивості. Яка властивість доводить, що виділені ізоферментні форми того самого ферменту?

Однакова молекулярна маса

Однакова електрофоретична рухливість

Каталізують ту саму реакцію

Однакова тканинна локалізація

Однакові фізико-хімі’нгі властивості

4418 / 6307
Пацієнту, у якого закритий перелом плечової кістки, накладено гіпсову пов’язку. Наступного дня з’явилися припухлість, синюшність і похолодніння кисті травмованої руки. Про який розлад периферичного кровообігу свідчать ці ознаки?

Венозна гіперемія

Тромбоз

Артеріальна гіперемія

Емболія

Ішемія

4419 / 6307
У пацієнта спостерігається збільшений тонус артеріол за нормальних показників роботи серця. Як це вплине на величину артеріального тиску?

Зменшиться переважно систолічний

Зменшиться переважно діастолічний

Зросте переважно діастолічний

Зросте переважно систолічний

Тиск не зміниться

4420 / 6307
130. У дитини, яка часто хворіє на ангіни та фарингіти, спостерігається збільшення лімфовузлів і селезінки. Зовнішній вигляд характеризується пастозністю та блідістю, м’язова тканина розвинена слабко. У крові виявлено лімфоцитоз. Як називаєгься такий вид діатезу?

Ексудативно-катаральний

Геморагічний

Астенічний

Нервово-аріритичний

Лімфатико-гіпопластичний

4421 / 6307
Під час експерименту треба оцінити рівень збудливості тканини. Для цього доцільно визначити величину:

Тривалості потенціалу дії

Потенціалу спокою

Порогу деполяризації

Амплітуди потенціалу дії

4422 / 6307
132. Під час мікроскопії нирки патоло- тЗанатом виявив, що приблизно у 80% клубочків спостерігається розмноження епітелію зовнішнього листка капсули Шумлянського, що формує «півмісяці». Він дійшов висновку що така картина відповідає:

Фібропластичному гломерулонефриту

Екстракапілярному проліферативному (швидкопрогресуючому) гломерулонефриту

Інтракапілярному проліферативному гломерулонефриту

Інтракапілярному ексудативному гломерулонефриту

Екстракапілярному ексудативному гломерулонефриту

4423 / 6307
Лікар-діетолог радить пацієнту під час лікування перніциозної анемії включити до раціону напівсиру печінку. Наявність якого вітаміну у цьому продукті стимулює процес кровотворення?

В2

В12

В1

Н

C

4424 / 6307
Під час обстеження у пацієнта спостерігається гіперглікемія, кетонурія, поліурія, гіперстенурія та глюкозурія. Яка форма порушення кислотно-лужної рівноваги спостерігається в цій ситуації?

Метаболічний ацидоз

Змішаний алкалоз

Негазовий алкалоз

Метаболічний алкалоз

Газовий ацидоз

4425 / 6307
У чоловіка, хворого на тимому (пухлину тимусу), спостерігаються ціаноз, розширення підшкірної венозної сітки і набряк м яких тканин обличчя ши верхньої половини тулуба і верхніх кініцнок. Який венозний стовбур непеснено пухлиною?

Передня яремна вена

Зовнішня яремна вена

Внутрішня яремна вена_

Підключична вена

Верхня порожниста вена

4426 / 6307
У гістологічному препараті паренхіма органа представлена часточками, які мають форму шестигранних призм і складаються з анастомозуючих пластинок, між якими лежать синусоїдні капіляри. що радіально сходяться до центральної вени. Який анатомічний орган має таку морфологічну будову?

Тймус

Підшлункова залоза

Лімфатичний вузол

Селезінка

Печінка

4427 / 6307
У клітину проник вірус грипу. Трансляція під час біосинтезу вірусного білка в клітині буде здійснюватися:

На каналах гладенької ендоплазматичної сітки

У ядрі

У клітинному центрі

У лізосомах

На полірибосомах

4428 / 6307
У чоловіка на 2-3-й де нь після операції ушивання кровоточивої виразки шлунка виявились ознаки атонії кишечника та затримка сечовиділення. Виберіть препарат для усунення ускладнення:

Прозерин

Атропін

Гістамін

Папаверин

Пілокарпін

4429 / 6307
Юнак 17 років страждає на фурункульоз, викликаний умовно-патогенним Staphylococcus epidermidis. Яке дослідження найдоцільніше провести, щоб правильно вибрати препарат для лікування цього хворого?

Визначити фаговар

Визначити антигенні властивості

Скласти антибіотикограму

Виявити фактори патогенності

Дослідити біохімічні властивості

4430 / 6307
У 30-ти річного чоловіка, який хворів на гостре респіраторне захворювання та загинув при явищах гострої легенево-серцевої недостатності, під час розтину знайдено фібринозно-геморагічне запалення в слизовій оболонці гортані та трахеї, деструктивний панбронхит, збільшені легені, яки мають пістрявий вигляд за рахунок абсцесів, крововиливів, некрозу. Який з перелічених діагнозів найбільш вірогідний?

Аденовірусна інфекція

Респіраторно-синцитіальна інфекція

Грип

Кір

Парагрип

4431 / 6307
У пацієнта 18 років при лабораторному обстеженні виявлено наявність глюкози в сечі при нормальній концентрації її в плазмі крові. Найвірогіднішою причиною цього є порушення:

Секреції інсуліну

Секреції глюкокортикоїдів

Клубочкової фільтрації

Канальцсвої секреції

Канальцсвої реабсорбції

4432 / 6307
Чоловік, хворий на ішемічну хворобу серця, не повідомив лікаря, що у нього трапляються напади бронхоспазму. Лікар призначив препарат, після приймання якого напади стенокардії порідшали, але почастішали напади бронхоспазму. У кажіть, який препарат був призначений:

Нітротросорбід

Дилтіазем

Верапаміл

Анаприлін

Атенолол

4433 / 6307
Під час мікроскопічного дослідження прямої кишки виявлено великі вогнища некрозу слизової оболонки, некротичні маси просякнуті фібрином, що утворює плівку. Слизова і підслизова оболонки на периферії ділянок некрозу повнокровні, набряклі, з крововиливами і лейкоцитарною інфільтрацією. Яке захворювання можна припустити?

Черевний тиф

Сальмонельоз

Амебіаз

Дизентерія

Холера

4434 / 6307
Молекула зрілої інформаційної РНК має меншу довжину, ніж відповідний ген молекули ДНК. Неінформативні послідовності нуклеотидів про-іРНК видаляються під час нроцесингу. Яку назву мають ці ділянки?

Інтрони

Екзони

Трапскриптони

Мутони

Кластери

4435 / 6307
У людини внаслідок травми мозку сталося вимкнення грудного дихання зі збереженням діафрагмального. У разі якої локалізації травми це може відбутися?

На рівні ретикулярної формації стовбура

Між шийними та грудними сегментами спинного мозку

На рівні 1-го шийного сегмента спинного мозку

На рівні варолієвого моста

На рівні 8-го ірудного сегмента спинного мозку

4436 / 6307
Жінка 25-ти років скаржиться на постійний біль у ділянці серця, задишку під час рухів, загальну слабкість. Об’єктивно: шкіра бліда та холодна, акроціаноз. Ps - 96/хв., АТ - 105/70 мм рт.ст. Межа серця зміщена на 2 см вліво. Перший тон над верхівкою серця послаблений, систолічний шум над верхівкою. Діагностовано недостатність мітрального клапана серця. Чим обумовлене порушення кровообігу?

Перевантаженням міокарда збільшеним об’ємом крові

Збільшенням об'єму судинного русла

Пошкодженням міокарда

Перевантаженням міокарда підвищеним опором відтоку крові

Зниженням об’єму крові, що циркулює

4437 / 6307
У жінки 28 років виявлена позаматкова вагітність, яка ускладнилася розривом маткової труби. У який простір очеревини можливе попадання крові?

Правий брижовий синус

Прямокишково-матковий

Лівий брижовий синус

Міхурово-матковий

Міжсигмоподібний синус

4438 / 6307
Потерпілий обробляв рослини розчином речовини з інсектицидною дією без індивідуальних засобів захисту. За деякий час у нього почалося сильне виділення слини, поту, сліз, біль у животі, пронос. Під час огляду виявлено міоз. Речовина, що викликала отруєння, належить до групи:

Антихолінестеразні засоби

Солі міді

Н-холіноміметики

Органішіі сполуки хлору

Нітрати

4439 / 6307
У жінки 45 років, яка тривалий час хворіє на бронхіальну астму, виник напад задухи. Який патогенетичний механізм має це явище?

Втрата еластичності легеневої тканини

Порушення перфузїї легеневої тканини

Спазм дрібних бронхів

Зниження чутливості дихального центру

Порушення рухомості грудної клітки

4440 / 6307
A patient has arterial hypertension. What long-acting drug from the group of calcium channel blockers should be prescribed?

Pyrroxanum

Atenolol

Amlodipine

Reserpine

Octadine

4441 / 6307
A drycleaner's worker has been found to have hepatic steatosis. This pathology can be caused by the disruption of synthesis of the following substance:

Tristearin

Urea

Phosphatidic acid

Cholic acid

Phosphatidylcholine

4442 / 6307
What condition may develop 15 30 minutes after re-administration of the antigen as a result of the increased level of antibodies, mainly IgE, that are adsorbed on the surface of target cells, namely tissue basophils (mast cells) and blood basophils?

Delayed-type hypersensitivity

Immune complex hyperresponsiveness

Anaphylaxis

Antibody-dependent cytotoxicity

Serum sickness

4443 / 6307
A patient underwent a course of treatment for atherosclerosis. Laboratory tests revealed an increase in the anti atherogenic lipoprotein fraction in the blood plasma. The treatment efficacy increase in: confirmed by the

HDL

VLDL

LDL

IDL

Chylomicrons

4444 / 6307
A patient has been found to have a marked dilatation of saphenous veins in the region of anterior abdominal wall around the navel. This is a symptom of pressure increase in the following vessel:

V. mesenterica superior

V. cava superior

V. mesenterica inferior

V. cava inferior

V. portae hepatis

4445 / 6307
An electron micrograph shows a cell-to-cell adhesion consisting, in each cell, of an attachment plaque The intercellular space is filled with electron-dense substance including transmembrane fibrillar Specify this adhesion: structures.

Nexus

Adherens junction

Synapse

Desmosome

Tight junction

4446 / 6307
A person with the fourth blood group (genotype IAIB) has in erythrocytes both antigen A controlled by allele IA and antigen В controlled by allele IB. This phenomenon is an example of the following gene interation:

Codominance

Semidominance

Polymery

Epistasls

Complementarity

4447 / 6307
A specimen shows an organ covered with the connective tissue capsule with trabeculae radiating inward the organ. There is also cortex containing some lymph nodules, and medullary cords made of lymphoid cells. What organ is under study?

Red bone marrow

Spleen

Lymph node

Tonsils

Thymus

4448 / 6307
One of the factors that cause obesity is the inhibition of fatty acids oxidation due to:

Choline deficiency

Lack of carbohydrates in the diet

Impaired phospholipid synthesis

Low level of carnitine

Excessive consumption of fatty foods

4449 / 6307
The resuscitation unit has admitted a patient in grave condition. It is known that he had mistakenly taken sodium fluoride which blocks cytochrome oxidase. What type of hypoxia developed in the patient?

Hypoxic

Tissue

Cardiovascular

Respiratory

Hemic

4450 / 6307
A 30-yearold patient has dyspnea fits, mostly at night. He has been diagnosed with bronchial asthma. What type of aller reaction according to the Gell-Coombs classification is most likely in this case?

Immune complex

Cytotoxic

Delayed-type hypersensitivity

Stimulating

Anaphylactic

4451 / 6307
Hepatitis В is diagnosed through laboratory tests that determine the presence of HBA-DNA in blood serum of the patient. What reference method is applied for this purpose?

Hybridization signal amplification method

Polymerase chain reaction

Ligase chain reaction method

ELISA diagnostic method

Hybridization method

4452 / 6307
A 37-year-old female patient complains of headache, vertigo, troubled sleep, numbness of limbs. For the last 6 years she has been working at the gas-discharge lamp producing factory in the lead processing shop. Blood test findings: low hemoglobin and RBC level, serum iron concentration exceeds the norm by several times. Specify the type of anemia: Minkowsky-Shauffard disease

Iron refractory anemia

Metaplastic anemia

Hypoplastic anemia

Correct answer

Iron-deficiency anemia

4453 / 6307
A patient complains that at the bare mention of the tragic events that once occurred in his life he experiences tachycardia, dyspnea and an abrupt rise of blood pressure. What structures of the CNS are responsible for these cardiorespiratory reactions in this patient?

Quadrigemina of mesencephalon

Specific thalamic nuclei

Lateral hypothalamic nuclei

Cerebellum

Cerebral cortex

4454 / 6307
A 49-year-old man complains of pain in his metatarsophalangeal joints and joint deformation. In blood hyperuricemy can be observed. X ray has revealed metatarsophalangeal joint space narrowing, erosion, periarticular calcification of the both joints, osteoporosis. Microscopy has revealed inflammatory granulomatous reaction surrounding necrotizing masses in the area of the first metatarsophalangeal joint. Choose the most likely diagnosis:

Hyperparathyroidism

Rheumatoid arthritis

Urolithiasis

Gout (podagra)

Pyrophosphate arthropathy

4455 / 6307
A young woman suddenly developed fever up to 39°C accompanied by a strong headache. Examination revealed marked nuchal rigidity. Spinal puncture was performed. Gram-stained of cerebrospinal fluid smear contained many neutrophils and Gram-positive diplococci. What bacteria could be the cause of this disease?

Pseudomonas aeruginosa

Haemophilus influenza

Staphylococcus aureus

Streptococcus pneumonia

Neisseria meningitidis

4456 / 6307
During ventricular systole, the cardiac muscle does not respond to additional stimulation because it is in the phase of:

Subnormal excitability

There is no correct answer

Hyperexcitability

Absolute refractoriness

Relational refractoriness

4457 / 6307
Histologic specimen of a kidney demonstrates cells closely adjoined to the renal corpuscle in the distal convoluted tubule. Their basement membrane is extremely thin and has no folds. These cells sense the changes in sodium content of urine and influence renin secretion occurring in juxtaglomerular cells. Name these cells:

Macula densa cells

Mesangial cells

Glomerular capillary endothelial cells

Juxtaglomerular cells

Podocytes

4458 / 6307
As a result of a continuous chronic encephalopathy, a patient has developed spontaneous motions and a disorder of torso muscle tone. These are the symptoms of the disorder of the following conduction tract:

Tractus corticonuclearrs

Tractus spinothalamicus

Tractus tectospinalis

Tractus rubrospinalis

Tractus corticospinalis

4459 / 6307
Work in a mine is known to cause inhalation of large amounts of coal dust. Inhaled coal dust can be detected in the following pulmonary cells:

Secretory epithelial cells

Respiratory epithelial cells

Pericapillary cells

Alveolar macrophages

Capillary endothelial cells

4460 / 6307
Inherited diseases, such as mucopolysaccharidoses, are manifested in metabolic disorders of connective tissue, bone and joint pathologies. The sign of this disease is the excessive urinary excretion of the following substance:

Glucose

Lipids

Glycosaminoglycans

Amino acids

Urea

4461 / 6307
A patient consulted a physician about chest pain, cough, fever. Roentgenography of lungs revealed eosinophilic infiltrates that were found to contain larvae What kind of helminthiasis are these presentations typical of?

Echinococcosis

Fascioliasis

Ascariasts

Trichinosis

Cysticercosis

4462 / 6307
A patient underwent surgical removal of a cavitary liver lesion 2 cm in diameter. It was revealed that the cavity wall was formed by dense fibrous connective tissue; the cavity contained muddy, thick, yellowish greenish fluid with an unpleasant odor Microscopically, the fluid consisted mainly of polymorphonuclear leukocytes. What pathological process are these morphological changes typical for?

Acute abscess

Empyema

Phlegmon

Chronic abscess

4463 / 6307
Due to the use of poor-quality measles vaccine for preventive vaccination, ej 1-year-old child developed an autoimmune renal injury. The urine was found to contain macromolecular proteins. What process of urine formation was disturbed?

Filtration

Secretion and filtration

Secretion

Reabsorption

Reabsorption and secretion

4464 / 6307
A 41-year-old male patient has a history of recurrent attacks of heartbeats (paroxysms), profuse sweating, headaches. Examination revealed hypertension, hyperglycemia, increased basal metabolic rate, and tachycardia. These clinical presentations are typical for the following adrenal pathology:

Primary aldosteronism

Hypofunction of the adrenal cortex

Hyperfunction of the adrenal cortex

Hyperfunction of the medulla

Hypofunction of the medulla

4465 / 6307
A 12-yearold child has a viral infection complicated by obstructive bronchitis. Bronchospasm can be eliminated by inhalations of a drug from the following pharmacological group:

M-anticholinergics

B2-agonists

Analeptics

N - c cholinomimetics

b2-adrenergic blockers

4466 / 6307
Prolonged treatment of hypothyroidism has caused general dystrophy, dental caries, tachycardia, tremor of extremities. What drug is the cause of these side effects?

Parathyreoidinum

Thyrocalcitonin

L-thyroxin

Prednisolone

Humulin (Human insulin)

4467 / 6307
During pathomorphological kidney investigation of a patient, who for a long time had been suffering from osteomyelitis and died from progressing renal failure, the following was revealed: deposits of homogeneous eosinophilic masses in glomerular mesangium, arterial and arteriolar walls, and stroma, which became red when stained with Congo red. What pathological process is this?

Carbohydrate degeneration

Amyloidosis

Hyalinosis

Calcinosis

Mucoid swelling

4468 / 6307
During cell division, DNA replication occurs by a signal from the cytoplasm, and a certain portion of the DNA helix unwinds and splits into two individual strains. What enzyme facilitates this process?

Ligase

Restrictase

Helicase

DNA polymerase

RNA polymerase

4469 / 6307
During appendectomy a patient had the a. appendicularis figated. This vessel branches from the following artery:

A. mesenterica inferior

A. ileocolica

A. colica media

A. colica dextra

A. sigmoidea

4470 / 6307
Хвора 36-ти років страждає на колагеноз. Збільшення вмісту якого метаболіту найбільш вірогідно буде встановлено у сечі?

Індикану

Уробіліногену

Сечовини

Оксипроліну

Креатиніну

4471 / 6307
Дівчина 19-ти років, хвора на цукровий діабет, чекає на донорську нирку. Яке ускладнення діабету є причиною хронічної ниркової недостатності?

Мікроангіопатія

Атеросклероз

Ретинопатія

Макроангіопатія

Невропатія

4472 / 6307
Під час розтину трупа чоловіка 48-ми років виявлено, що кістковий мозок пласких кісток, діафізів та епіфізів трубчастих кісток соковитий, сіро-червоний або сіро-жовтий гноєподібний (піоїдний кістковий мозок). Селезінка масою - 7 кг. На розрізі вона темно-червоного кольору, з ішемічними інфарктами. Всі лімфатичні вузли збільшені, м’які, сіро-червоного кольору. В печінці жирова дистрофія і лейкемічні інфільтрати. Який найбільш імовірний діагноз?

Лімфогранулематоз

Хронічний мієлоїдний лейкоз

Гострий лімфоїдний лейкоз

Гострий мієлоїдний лейкоз

Мієломна хвороба

4473 / 6307
Дослідників, які вивчають фізіологію серця, виявили, що надмірне розтягнення передсердь спричинює зниження реабсорбції натрію у дситальному звивистому канальці та підвищення швидкості клубочкової фільтрації. Що з наведенного є наймовірнішою причиною фізіологічних змін. виявлених дослідниками?

Вазопресин

Альдостерон

Натрійуретичний гормон

Ренін

Ангіотензин

4474 / 6307
У дитини 5 років відбулася інвазія гельмінтів, що призвело до сенсибілізації організму. Які показники лейкоцитарної формули підтвердять цей процес?

Збільшення нейтрофілів

Збільшення еозинофілів

Зменшення базофілів

Зменшення еозинофілів

Збільшення базофілів

4475 / 6307
Під час бактеріологічного дослідження гнійного матеріалу на МПЛ виросли великі безбарвні слизові колонії, які за 24 години з доступом сонячного світла утворили зелено-блакитний водорозчинний пігмент. Бактеріоскопія виявила грамнегативні палички. Чиста культура цього мікроорганізму має запах жасмину та стійкість до більшості антибіотиків. Культура якого мікроорганізму, ймовірно, виділена?

Proteus vulgaris

Klebsiella osaenae

Brucella abortus

Pseudomonas aureginosa

Yersinia pestis

4476 / 6307
Чоловік 50 років, у якого наявна хвороба Паркінсона, застосовує лікарський засіб, який утворюється в організмі з тирозину і є попередником дофаміну. Пацієнт зазначає зменшення тремтіння кінцівок, збільшення об’єму рухів, поліпшення уваги. Який препарат використовує пацієнт?

Скополамін

Леводопа

Натрію вальпроат

Циклодол

Діазепам

4477 / 6307
Жінка 28 років потрапила до інфекційної лікарні з приводу пожовтіння шкіри, склер, слизових оболонок. Лабораторно встановлено підвищення рівня прямого білірубіну в крові. У сечі виявлені уробі-ліноген і білірубін. Для якого з наведених захворювань характерні такі зміни?

Інфаркт нирки

Туберкульоз нирки

Механічна жовтяниця

Паренхіматозна жовтяниця

Гемолітична жовтяниця

4478 / 6307
У жінки, яка хворіє на ішемічну хворобу серця, розвинувся кардіосклероз, що супроводжується аритмією серця. Назвіть вид аритмії, за якого одночасно порушуються автоматизм і провідність:

Миготлива аритмія

Передсердно-шлуночкова блокада

Внутрішньопередсердна блокада

Синусова аритмія

Синоатріальна блокада

4479 / 6307
Під час розтину тіла чоловіка, померлого від гострої серцево-судинної недостатності, виявлені заповнена кров’ю порожнина серцевої сорочки та розплавлений серцевого м’яза бічної поверхні лівого шлуночка. Яка безпосередня причина смерті?

Розрив хронічної аневризми серця

Гостра лівошлуночкова недостатність

Гемотомпонада перикарда

Фібриляція шлуночків

Кардіогснний шок

4480 / 6307
Чоловікові, у якого підозра на системне захворювання сполучної тканини, зробили біопсію нирок і скелетного м'язу. Гістологічно в тканині нирки і м'язі виявлено поширений фібриноїдний некроз внутрішньої і середньої стінок дрібних артерій з їх нейтрофільною інфільтрацією. Виражена інфільтрація визначається в периваскулярній тканині та адвентиціальній оболонці судин, деякі артеріоли тромбовані. Укажіть найімовірніший діагноз:

Гіпертонічна хвороба

Облітеруючий ендартеріїт

Системний червоний вовчак

Атеросклероз

Вузликовий периартеріїт

4481 / 6307
В експериментальної тварини зроблено двобічну перерізку блукаючих нервів. Що станеться з її диханням?

Зупиниться у фазі вдиху

Стане частим і поверхневим

Стане рідким і глибоким

Не зміниться

Зупиниться у фазі видиху

4482 / 6307
У жінки, яка має клінічні ознаки імунодефіциту і незмінену кількість та функціональну активність Т- і В-лімфоцитів, під час обстеження виявлено дефект на молекулярному рівні, через який порушена функція антигенпрезентації імунокомпетентним клітинам. Дефект структур яких клітин є можливим?

О-лімфоцити

Т-лімфоцити, В-лімфоцити

NK-клітини

Макрофаги, моноцити

Фібробласти, Т-лімфоцити, В-лімфоцити

4483 / 6307
Вагітна жінка захворіла на пневмонію; термін вагітності 20 тижнів. Який хіміотерапевтичний препарат можна порадити лікарю для призначення хворій без загрози для розвитку плоду?

Сульфален

Левоміцетин

Офлоксацин

Гентаміцин

Бензилпеніцилін

4484 / 6307
Для серологічної діагностики черевного тифу використовують реакцію Відаля. Який механізм взаємодії антигенів та антитіл лежить в її основі?

Гемоліз

Іммобілізація бактерій

Преципітація

Бактеріоліз

Аглютинація

4485 / 6307
У молодої жінки діагностовано позаматкову вагітність. В якому з перерахованих органів відбувається запліднення?

Порожнина малого тазу

Порожнина матки

Ампула маткової труби

Перешийок маткової труби

Порожнина піхви

4486 / 6307
У важкоатлета під час піднімання штанги стався розрив грудного лімфатичного протоку. Укажіть найімовірніше місце пошкодження:

Ділянка попереково-крижового сполучення

Ділянка аортального отвору діафрагми

Ділянка шиї

Заднє середостіння

Місце впадіння у венозний кут

4487 / 6307
У жінки спостерігаються симптоми запального процесу сечостатевих шляхів. У мазку із слизової оболонки піхви виявлено великі одноклітинні організми грушоподібної форми із загостреним шипом на задньому кінці тіла, великим ядром та ундулюючою мембраною. Які найпростіші знайдені в мазку?

Lamblia intestinalis

Trichomonas vaginalis

Trichomonas buccalis

Trichomonas hominis

Trypanosoma gambiense

4488 / 6307
Мати 4-місячного хлопчика звернулася до педіатра зі скаргами на відмову дитини від їжі та втрату ваги. Дитина з труднощами захоплює пляшечку. Хлопчик також став украй млявим. Під час обстеження виявлено знижений топус м'язів у всіх кінцівках та гепатоспленомегалію. Офтальмоскопією виявлено макулярні вишнево-червоні плямки. Наступні 2 тижні гепатоспленомегалія прогресує, хлопчик погано набирає вагу та продовжує відмовлятися від їжі. На рентгенограмі органів грудної клітки ретикулонодулярний візерунок із кальцифіконаними вузликами. Біопсія печінки виявила клітини Німанна-Піка. Дефіцит якого з наведених ферментів, найімовірніше, успадковано цією дитиною?

Фенілаланін гідроксилаза

Галактоцереброзидаза

Сфінгомієліназа

Глюкоцереброзидаза

Глюкозо-6-фосфатаза

4489 / 6307
Вислуховуючи тони серця, лікар виявив у пацієнта функціональні порушення двостулкового (мітрального) клапана. У якому місці його вислуховував лікар?

У другому міжребер’ї зліва біля груди-ші

Біля мечоподібного відростка

У другому міжреберії справа біля грудини

Біля місця прикріплення реберного хряща праворуч

На верхівці серця

4490 / 6307
В гістологічному препараті визначається орган, стінка якого містить три оболонки. Найтовстіша середня оболонка утворена анастомозуючими волокнами, що утворені ланцюжком клітин, які в зоні контакту формують вставні диски. Яку функцію виконують ці клітини?

Захисну

Провідну

Скоротливу

Секреторну

Трофічну

4491 / 6307
Фолдинг - посттрансляційна модифікація білку. Який механізм фолдингу пепсину головних клітин слизової оболонки шлунка?

Дистилювання

Фосфорилювання

Частковий протеоліз

Ковалентна модифікація

Метилювання

4492 / 6307
При розтині тіла жінки 40-ка років, що померла від крововиливу у головний мозок під час гіпертонічного кризу, знайдено: ожиріння за верхнім типом, гіпертрихоз та гірсутизм, стрії шкіри, стегон та живота. В передній долі гіпофізу - базофільна аденома. Який з перелічених діагнозів найбільш імовірний?

Хвороба Іценко-Кушинга

Аліментарне ожиріння

Гіпертонічна хвороба

Церебральне ожиріння

Хвороба Сімондса

4493 / 6307
Рівень теплопродукції у людини, яка перебуває в стані глибокого наркозу із застосуванням міорелаксантів і гангліоблокаторів:

Підвищується

Спочатку не змінюється, а потім підвищується

Не змінюється

Знижується

4494 / 6307
Під час експерименту збільшили проникність мембрани збудливої клітини для іонів калію. Які зміни електричного стану мембрани у цьому разі виникнуть?

Деполяризація

Локальна відповідь

Гіперполяризація

Потенціал дії

Змін не буде

4495 / 6307
У жіночій консультації обстежується жінка, у якої було декілька мимовільних викиднів. На підставі клініко-спідеміологічного анамнезу було припущено хронічний токсоплазмоз. Яке лабораторне дослідження буде найефективнішим для підтвердження діагнозу?

Шкірно-алергічна проба

Серологічні реакції

Мікроскопія мазка фекалій

Мікроскопія піхвового мазка

Мікроскопія мазка крові

4496 / 6307
У клініці встановлено, що під час вагітності тяжкість симптомів ревматоїдного артриту різко знижується. Прискорення секреції яких гормонів, які мають протизапальну дію, спостерігається у цьому разі?

Глюкокортикоїдів

Гонадотропних

Естрогенів

Катехоламінів

Йодованих щитоподібної залози

4497 / 6307
Пацієнт протягом останнього року став помічати підвищену втомлюваність, загальну слабкість. Результати аналізу крові: еритроцити -4,1- 1012/л, НЬ - 119 г/л, кольоровий показник - 0,87, лейкоцити - 57 • 109/л, лейкоформула: юні -0%. паличкоядерні -0%, сегментоядерні - 9%, еозинофіли - 0%. базофіли - 0%, лімфобласти - 2%, пролімфоцити -5%, лімфоцити - 81%, моноцити - 3%, тромбоцити - 160 • 109/л. У мазку виявлено: нормохромія, велика кількість тіней Боткіна-Гумпрехта. Про яку патологію системи крові свідчить така гемограма?

Хронічний лімфолейкоз

Хронічний мієлолейкоз

Гострий лімфобластний лейкоз

Хронічний монолейкоз

Гострий мієлобластний лейкоз

4498 / 6307
Клітину лабораторної тварини піддали надмірному рентгенівському опроміненню. У результаті утворились білкові фрагменти в цитоплазмі. Які органели клітини візьмуть участь у їх утилізації?

Лізосоми

Ендоплазматичний ретикулум

Комплекс Гольджі

Клітинний центр

Рибосоми

4499 / 6307
На аутопсії померлої, яка тривалий час хворіла цистітом і дискінезією сечоводів, виявлено морфологічні ознаки уремії. Нирка була нерівномірно рубцево-зморщена. У просвіті мисок містилися дрібні уратні камені і пісок. Гістологічно виявлено “щитовидну нирку', вогнища інтерстиційного запалення. Який із нижчеперерахованих діагнозів є найбільш вірогідний?

Атеросклеротично зморщена нирка

Хронічний пієлонефрит

Гострий пієлонефрит

Первинно зморщена нирка

Амілоїдно зморщена нирка

4500 / 6307
У дівчинки 9 років, яка скаржиться на біль у животі, виявлено гострий апендицит. Як змінюється клітинний склад периферичної крові у разі гострого запалення?

Лімфоцитоз

Нейтропенія

Нейтрофільоз

Еритроцитоз

Лімфопенія

4501 / 6307
До ЛОР-відділення потрапив пацієнт із запаленням слизової оболонки верхнього носового ходу та задньо-верхнього відділу перегородки носу. Він скаржиться на порушення нюху. Тіло якого нейрона нюхового аналізатора уражене?

Другого

Третього

П’ятого

Першого

Четвертого

4502 / 6307
Юнак 16 років, потерпілий в автомобільній аварії, перебуває на лікуванні в стаціонарі після великої крововтрати. На 10-й день у крові збільшилася кількість регенераторних форм еритроцитів. Які це клітини?

Мегалобласта

Шизоцити

Ретикулоцита

Мегалоцита

Ехіноцити

4503 / 6307
На розтині тіла жінки 52 років, яка померла внаслідок легеневої недостатності, у легенях виявлено такі зміни: обидві частки лівої легені збільшені, щільні, безповітряні. На розрізі в різних відділах визначаються жовтуватого кольору великі вогнища з тьмяною поверхнею, що злегка кришиться, на плеврі є сіруватого кольору плівчасті відкладення. Під час гістологічного дослідження спостерігається: просвіти альвеол заповнені серозно-фібринозним і фібринозним ексудатом із наявністю безструктурних ділянок в ексудаті та тканині легенів. Серед безструктурних ділянок визначаються уламки ядер. Для якого захворювання характерні такі зміни в легенях?

Крупозної пневмонії

Пневмоніеподібного раку легені

Казеозної пневмонії

Інфільтративно-пневмонічного туберкульозу

Лімфогранулематозу легенів

4504 / 6307
Чоловікові встановлено діагноз цукрового діабету 2-го типу. Концентрація глюкози в крові - 16 ммоль/л. Яка з наведених ознак достовірно свідчить про цей тип захворювання?

Інсулінорсзистентність тканин

Антитіла до інсуліну

Абсолютна інсулінова недостатність

Гіперглікемія

Порушення тесту толерантності до глюкози

4505 / 6307
1910 року Раус під час експерименту отримав саркому курей шляхом введення їм безклітанного фільтрату, отриманого із саркоми курки. Який метод експериментального моделювання використовував автор?

Гетеротрансплантація

Гомотрансплантація

Експлантація

Індукування

Ізотрансплантація

4506 / 6307
У підлітка після перенесеного інфекційного захворювання з’явилася різко виражена аритмія з вкороченням інтервалу R-R під час вдиху і подовження його під час видиху. Що лежить в основі даного виду аритмії?

Порушення фунції збудливості серця

Коливання тонусу блукаючого нерва під час акту дихання

Порушення функції провідності серця

Вплив мікробних токсинів на серце

Рефлекс Бейнбріджа

4507 / 6307
Офтальмолог з діагностичною метою (розширення зіниць для огляду очного дна) використав 1% розчин мезатону. Мі-дріаз, викликаний препаратом, обумовлений:

Активацією М-холінорецепторів

Активацією α1-адренорецепторів

Активацією β-адренорецепторів

Блокадою α1-адренорецепторів

Активацією α2-адренорецепторів

4508 / 6307
У дитини 9 років, яка хворіла на вірусний грип, на 5-ту добу захворювання з’явилися сильний головний біль, нудота, запаморочення, менінгеальні ознаки. Смерть настала за добу від наростаючого набряку мозку. Під час розтину порожнини черепа виявлено: м’які мозкові оболонки набряклі, повнокровні, дифузно просякнуті яскраво-червоного кольору рідиною. Яке ускладнення грипу можна припустити?

Крововилив у мозок

Серозний менінгіт

Венозна гіперемія оболонок мозку

Гнійний лептоменінгіт

Геморагічний менінгіт

4509 / 6307
При аналізі крові у хворого залишковий азот склав 48 ммоль / л, сечовина 15,3 ммоль / л. Про захворювання якого органу свідчать результати цього аналізу?

Нирки

Печінка

Селезінка

Кишечник

Шлунок

4510 / 6307
У пацієнта після проходження третього курсу променевої терапії розвинулася двостороння абсцедивна пневмонія, яка призвела до його смерті. Під час мікроскопічного дослідження селезінки, лімфатичних вузлів виявлено різке зменшення розмірів лімфатичних фолікулів із виснаженням переважно В-залежних зон лімфоцитів. Який із наведених імуно-патологічних процесів найімовірніший?

Органоспецифічна аутоімунна хвороба

Органонсспсцифічна аутоімунна хвороба

Аутоіммунна хвороба проміжного типу

Вторинний імунодефіцитний синдром

Первинний імунодефіцитний синдром

4511 / 6307
Дівчинка 11 років прийшла на прийом до лікаря із мамою, яка скаржиться на слабкість та набряклість обличчя її дитини протягом 3 днів. Мати стверджує, що до початку симптомів її дитина завжди була здоровою та активною. Під час фізикального обстеження виявлено генералізований набряк обличчя, натискання набряку нижніх кінцівок залишає ямку, яка поступово згладжується. Під час збору анамнезу дівчинка зазначає пінистий вигляд сечі, але заперечує домішки крові у сечі, ніктурію або біль під час сечовиділення. Лабораторні дослідження виявили протеінурію та мікрогематурію. Що з наведеного є найімовірнішою причиною змін у лабораторному аналізі сечі?

Підвищення гідростатичного тиску в клубочку

Підвищення онкотичного тиску плазми крові

Підвищення проникності крізь стінку клубочкового капіляру

Підвищення гідростатичного тиску в капсулі Шумлянського-Боумена

4512 / 6307
У чоловіка, який має алкогольне ураження печінки, порушені процеси біотрансформації ксенобіотиків та ендогенних токсичних сполук. Зниження активності якого хромопротеїну може бути причиною цього?

Цитохрому C1

Гемоглобіну

Цитохрому Р-450

Цитохрому b

Цитохромоксидази

4513 / 6307
У лабораторії під час мікроскопії харкотиння пацієнта, хворого на пневмонію, випадково виявлено личинки гельмінти. Під час аналізу крові виявлена еозинофілія. Який гельмінтоз можна припустити?

Парагонімоз

Ентеробіоз

Ошсторхоз

Трихоцефальоз

Аскаридоз

4514 / 6307
Пацієнту, який страждає на хронічні закрепи, було призначено синтетичний препарат, послаблювальннй ефект якого реалізується шляхом посилення перистальтики кишечника. Назвіть цей лікарський засіб:

Алохол

Рицинова олія

Кальцію хлорид

Контрикал

Бісакодил

4515 / 6307
У чоловіка, який помер від уремії, на розтині виявлена деформація хребетного стовпа з різким обмеженням рухливості. Суглобові хрящі дрібних суглобів хребта зруйновані, є виражені ознаки тривалого поточного хронічного запалення в тканинах суглобів, порожнини суглобів заповнені сполучною тканиною, місцями кістковою з формуванням анкілозів. В аорті, серці та легенях наявні хронічне запалення та вогнищевий склероз. У нирках спостерігається амілоїдоз. Який діагноз у цьому разі найімовірніший?

Хвороба Педжета (деформівний остоз)

Паратиреоїдна остеодистрофія

Анкілозивний спондилоартрит (хвороба Бехтерева)

Остеопетроз (мармурова хвороба)

Ревматоїдний артрит

4516 / 6307
Чоловікові, хворому на сифіліс, призначили лікарський засіб, в основі механізму дії якого є порушення утворення муреіну, що призводить до загибелі збудникВизначте препарат:

Бійохінол

Доксацикліну гідрохлорид

Азітроміцин

Ципрофлоксацин

Бензилпеніциліну натрієва сіль

4517 / 6307
Взаємозв'язок між ендокринною і нервовою системами значною мірою здійснюється за допомогою рилізинг-гормонів. Укажіть місце їх продукування:

Нейрогіпофіз

Епіфіз

Таламус

Гіпоталамус

Аденогіпофіз

4518 / 6307
Абсолютний дефіцит вітаміну К в організмі призводить до:

Порушення адгезії тромбоцитів

Дисбактеріозу кишечника

Гіпокоагуляції

Гіперкоагуляції

4519 / 6307
Найпоширенішим ускладненням застосування сечогінних засобів є гіпокаліємія. Якому діуретику властива калій-зберігальна дія?

Фуросемід

Спіронолактон

Пентамін

Циклометіазид

Гідрохлортіазид

4520 / 6307
Жінка протягом 8 років хворіла на а туберкульоз, померла в стаціонарі при ознаках хронічної ниркової недостатності. На розтині – нирки збільшені, поверхня розрізу має сальний вигляд; гістологічно виявлено масивні відкладання безструктурних гомогенних еозинофільних мас, при забарвленні конго рот спостерігається їх виражена метахромазія. Який патологічний процес розвинувся у нирках?

Токсичний нефрит на фоні антибіотикотерапії

Нефросклероз

Гострий гломерулонефрит

Вторинний амілоїдоз

Гематогенний туберкульоз нирок

4521 / 6307
Чоловік скаржиться, що згадування про минулі трагічні події в його житті викликає у нього тахікардію, задишку і різке зростання артеріального тиску. Які структури ЦНС забезпечують такі кардіореспіраторні реакції у цього пацієнта?

Чогиригорбкове тіло середнього мозку

Кора великих півкуль

Мозочок

Специфічні ядра таламуса

Латеральні ядра гіпоталамуса

4522 / 6307
На електронній мікрофотографії клітини видно дві різних органели, які руйнують білки. Що це за органели?

ибосома

ероксисоми та рибосоми

Лізосоми та протеасоми.

ндоплазматична сітка та мікрофіламенти

омплекс Гольджі та мікротрубочки

4523 / 6307
Жінка страждає на тяжку форму бронхіальної астми. За призначенням лікаря тривалий час отримує гормональну терапію. До яких ускладнень може призвести тривала терапія глюкортикоїдами?

Нирковокам’яної хвороби

Феохромоцитоми

Хвороби Іценка-Купіинга

ГЬстрої надниркової недостатності

Артеріальної гіпертензії

4524 / 6307
Пацієнту під час нападу стенокардії призначили лікарський засіб у таблетках під язик. За 1 хвилину стискальний біль у серці послабшав, але з’явився головний біль, що пульсує. Визначте цей препарат:

Анаприлін

Папаверин

Верапаміл

Нітрогліцерин

Фенігідин

4525 / 6307
В ділянці хромосоми гени розташовані в такій послідовності: ABCDEFG. В результаті дії радіоактивного випромінювання відбулася перебудова, після чого ділянка хромосоми має наступний вигляд: ABDEFG. Яка мутація відбулася?

Дуплікація

Інверсія

Інсерція

Мутація

Делеція

4526 / 6307
У пацієнтки 26-ти років висипання на шкірі, свербіж після вживання цитрусових. Призначте лікарський засіб з блокаторів Н1-гістамінорецепторів:

Менадіону натрію біосульфат

Метамізол

Парацетамол

Дифенгідрамін

Кислота ацетилсаліцилова

4527 / 6307
До терапевтичного відділення з приводу ревмокарди та надійшов чоловік, у якого в анамнезі в минулому була виразкова хвороба шлунка. Який препарат слід призначити хворому з групи ненаркотичних анальгетиків із протизапальною дією, щоб не спровокувати виразковий процес у шлунку?

Піроксикам

Ібупрофен

Целекоксиб

Диклофенак

Індомстацин

4528 / 6307
У жінки під час мейозу відбулося порушення розходження аутосом. Утворилася яйцеклітина з зайвою 18-ю хромосомою. Яйцеклітина запліднюється нормальним сперматозооном. У майбутньої дитини буде синдром:

Дауна

Клайнфельтера

Шерешевського-Тернера

Едвардса

Патау

4529 / 6307
У хворої на дифтерію дитини через 10 днів після введення антитоксичної протидифтерійної сироватки з’явилася висипка на шкірі, яка супроводжувалася сильним свербежем, підвищенням температура тіла до 38oC, появою болю у суглобах. Яку причину цих явищ можна припустити?

Гіперчутливість уповільненого типу

Сироваткова хвороба

Анафілактична реакція

Атопія

Контактна алергія

4530 / 6307
Чоловік, який хворіє на алкогольний цироз печінки, скаржиться на загальну слабкість, задишку. Установлено зниження артеріального тиску, асцит, розширення поверхневих вен передньої стінки живота, спленомсгалію. Яке порушення гемодинаміки спостерігається у пацієнта?

Синдром портальної гіпертензії

Недостатність лівого шлуночка серця

Колапс

Тотальна серцева недостатність

Недостатність правого шлуночка серця

4531 / 6307
Підлітку, що перебував у стані тяжкого алкогольного сп’яніння, лікар швидкої допомоги серед інших заходів здійснив внутрішньом’язове введення розчину кофеїну. Поясніть, який принцип взаємодії між алкоголем та кофеїном пояснює доцільність даної маніпуляції:

Конкурентний антагонізм

Синергізм

Потенціація

Фізіологічний антагонізм

Сумація ефектів

4532 / 6307
До приймального відділення звернувся хворий зі скаргами на задишку. Температура тіла 39°С. На рентгенограмі легенів спостерігається двостороння пневмонія. Який вид гіпоксії розвинувся у пацієнта?

Кров'яна

Дихальна

Тканинна

Серцево-судинна

Гіпоксична

4533 / 6307
Для лікування хвороби Паркінсона застосовують попередник дофаміну - ДОФА. З якої амінокислоти утворюється ця активна речовина?

Триптофану

Цистеїну

Аланіну

Гістидину

Тирозину

4534 / 6307
Чоловік 43 років звернувся до відділення невідкладної допомоги зі скаргами на гарячку з ознобом, нездужання, розлитий абдомінальний біль протягом тижня, діарею та втрату апетиту. Він зазначає, що його симптоми прогресивно стають гіршими. Він згадує, що гарячка розпочиналася повільно та підвищувалася до поточних 39,8°С поступово. Його артеріальний тиск - 110/70 мм рт. ст. Фізикальне обстеження виявило вкритий нальотом язик, збільшену селезінку та розеольозиий висип па животі. Під час проведення реакції аглютинації Відаля з О-діагностикумом установлено, що вона позитивна в розведенні 1:200. Який із мікроорганізмів, найімовірніше, є причиною описаного стану пацієнта?

Ентерогеморагічна Е. coli

Leptospira interrogans

Salmonella typhi

Mycobacterium tuberculosis

Vibrio cholerae

4535 / 6307
Під час розтину тіла жінки, яка померла внаслідок пухлинної дисемінації (множинні метастази та карциноз очеревини) муцинозної цистаденокарциноми і тривалий час мала вимушене положення в ліжку, були знайдені велики ділянки некрозу шкіри та підлеглих м’яких тканин крижової ділянці. Діагностуйте вид некрозу.

Пролежень

Сирнистий некроз

Інфаркт

Секвестр

Воскоподібний (ценкероський) некроз

4536 / 6307
Під час гістологічного дослідження слизової оболонки шлунка вивчали будову тканини, що вкриває поверхню слизової оболонки. Ця тканина не містить кровоносних судин, а її клітини розташовані на базальній мембрані. Яка тканина вкриває поверхню слизової оболонки шлунка?

Ретикулярна

Епітеліальна

М'язова

Лімфоїдна

Сполучна

4537 / 6307
Унаслідок перенесеного енцефаліту у чоловіка розвинувся параліч м’язів очного яблука. Лікар встановив, що у пацієнта ушкоджене ядро окорухового нерва. В якому відділі головного мозку відбувається патологічний процес?

Середньому мозку

Довгастому мозку

Проміжному мозку

Мосту

Мозочку

4538 / 6307
У чоловіка 47 років, який має хронічне захворювання печінки, лікар запідозрив гепатит С. Який патологічний матеріал потрібно дослідити для підтвердження діагнозу?

Фекалії

Жовч

Пунктат лімфовузлів

Кров

Біоптат печінки

4539 / 6307
Чоловік 68 років переніс інфаркт міокарду. Під час ЕКГ-обстеження спостерігається прогресивне збільшення тривалості інтервалу PQ аж до випадіння комплексу QRS, після чого інтервал PQ відновлюється. З порушенням якої функції серця пов'язане таке порушення серцевого ритму?

Автоматизму

Збудливості

Провідності

Скоротливості

4540 / 6307
Під дією УФ-опромінення та інших факторів можуть відбуватися зміни в структурі ДНК. Репарація молекули ДНК досягається узгодженою дією всіх наступних ферментів, ЗА ВИНЯТКОМ:

Аміноацил-тРНК-синтетаза

Ендонуклеаза

ДНК-глікозидаза

ДНК-полімераза

ДНК-лігаза

4541 / 6307
З метою серологічної діагностики черевного тифу проводять постановку реакції, під час якої до різних розведень сироватки хворого додають діагностикуми трьох видів мікроорганізмів і результат цієї реакції оцінюють за утворенням аглютинату. Назвіть цю реакцію за автором:

Закса-Вітебського

Райта

Відаля

Оухтерлоні

Вассермана

4542 / 6307
У гістопрепараті визначається орган, що складається із сірої та білої речовини. Сіра речовина розташовується в центрі і складається з пучкових, корінцевих та асоціативних нейронів. Назвіть орган, для якого характерні дані морфологічні ознаки.

Спинний мозок

Спинномозковий ганглій

Мозочок

Довгастий мозок

Великі півкулі головного мозку

4543 / 6307
До хірургічного відділення поступив хворий з ножевим пораненням грудної клітини справа та пневмотораксом (проникнення повітря у плевральну порожнину). Перкуторно нижня межа правої легені по середньоключичній лінії піднялася на рівень III ребра. Де у нормі вона повинна знаходитися?

VI ребро

IX ребро

VII ребро

VIII ребро

4544 / 6307
Взаємодія адреналіну з α‎-адренорецепторами підвищує внутрішньоклітинний вміст інозитолтрифосфату (ІФЗ), який впливає на вміст цитоплазматичного Са2. Утворення ІФЗ відбувається під впливом:

Фосфоліпази А2

Фосфоліпази С

Аденілатциклази

ТАГ-ліпази

ЛП-ліпази

4545 / 6307
У хворого, який на тлі атеросклерозу переніс ішемічний інсульт, спостерігається порушення рухової функції у вигляді геміплегії. Яка з перерахованих ознак є характерною для уражених кінцівок при даній патології?

Трофічні розлади

Гіпорефлексія

Гіпотонус м’язів

Гіпертонус м’язів

4546 / 6307
Під час експерименту треба виявити наявність збудження в м’язі. Для цього потрібно зареєструвати:

Концентрацію іонів

Силу скорочення

Механоміограму

Тривалість скорочення

Електроміограму

4547 / 6307
Під час глікогенозу (хвороби Гірке) пригнічується перетворення глюкозо-6-фосфату на глюкозу, що супроводжується порушенням розпаду глікогену в печінці. Дефіцит якого ферменту є причиною цього захворювання?

Фосфофруктокінази

Глюкозо-6-фосфатази

Глюкозо-6-фосфатдегідрогенази

Фосфоглюкомутази

Глікогенфосфорилази

4548 / 6307
У працівників хімічних комбінатів, де виробляють органічні розчинники, які здатні розчиняти фосфоліпіди, часто розвиваються захворювання легень. Який компонент аерогематичного бар’єру при цьому пошкоджується в першу чергу?

Респіраторні альвеолоцити

Сурфактант

Секреторні альвеолоцити

Септальні клітини

Альвеолярні макрофаги

4549 / 6307
Під час видалення правого яєчника хірург замість яєчникової артерії прев’язав орган, розміщений поряд. Який орган перев’язав хірург?

Нижню порожнисту вену

Загальну клубову артерію

Аорту

Сечовід

4550 / 6307
У жінки діагностовано пухлину головки підшлункової залози, порушення венозного відтоку із деяких органів черевної порожнини. Який венозний сосуд було здавлено пухлиною?

Ліва шлункова вена

Права шлункова вена

Ниркова вена

Нижняя полая

Ворітна вена

4551 / 6307
Який препарат слід призначити хворому, у якого з’явилося безсоння внаслідок появи висипань алергічного характеру з почервонінням, набряком та сильним свербінням?

Димедрол

Хлоралгідрат

Нітразепам

Фенобарбітал

Натрію оксибутират

4552 / 6307
Чоловік 60 років помер від правошлу-ночнокової недостатності. Під час розтину спостерігається: легені збільшені в об’ємі, у ділянці верхівок визначаються субплевральні міхури діаметром до 1 см. Гістологічно в паренхімі виявлено стоншення, а подекуди повне руйнування стінок альвеол. Спостерігаються також ознаки супутнього бронхіоліту. Який це вид легеневої патології?

Хронічний бронхіт

Пневмонія

Пневмосклероз

Емфізема легень

Бронхіальна астма

4553 / 6307
Під час розтину тіла чоловіка 56 років, який тривалий час хворів на хронічний гломерулонефрит, на поверхні епікарда і перикарда виявили сірувато-білуваті ворсинчасті нашарування («волосате» серце). Про який патологічний процес йдеться?

Серозне запалення

Геморагічне запалення

Фібринозне запалення

Гнійне запалення

Катаральне запалення

4554 / 6307
До фізіологічних властивостей серцевого м’язу людини належать усі наведені, крім:

Скоротливість

Еластичність

Збудливість

Автоматія

Провідність

4555 / 6307
Злоякісна пухлина привушної слинної залози спричинила пошкодження артерії, яка проходе через її паренхіму. Яка це артерія?

Висхідна глоткова

Лицева

Верхньощелепна

Задня вушна

Поверхнева скронева

4556 / 6307
У чоловіка після травми хребта спостерігається відсутність довільних рухів, сухожилкових рефлексів, чутливості тільки нижніх кінцівок. Який механізм порушень та у якому відділі хребта була травма?

Центральний параліч, куприковий відділ

Спінальний шок, шийний відділ

Спінальний шок, грудний відділ

Периферичний параліч, шийний відділ

4557 / 6307
У пацієнта синусова тахікардія. Для відновлення ритму лікар призначив пацієнтові препарати калію та рекомендував споживання продуктів, багатих на калій. Який механізм дії калію на серце?

Підвищує пейсмекерну активність синусового вузла

Активує симпатичний відділ автономної нервової системи

Активує парасимпатичний відділ автономної нервової системи

Знижує пейсмскерну активність синусового вузла

Пригнічує симпатичний відділ автономної нервової системи

4558 / 6307
У крові хворого виявили С- реактивний білок, який за хімічною природою є глікопротеїном. Про яку патологію це свідчить?

Порфірія

Лейкопенія

Анемія

Тромбоцитопенія

Ревматизм

4559 / 6307
До лікарні машиною швидкої допомоги доставлено хворого в стані коми. В анамнезі указано цукровий діабет. Під час обстеження спостерігається шумне прискорене дихання, у якому глибокі вдихи чергуються із посиленими видохами за участю експіраторних м’язів. Яка форма порушення зовнішнього дихання спостерігається?

Дихання Чейна-Стокса

Дихання Куссмауля

Дихання Біота

Стенотичне дихання

Апнейстичне дихання

4560 / 6307
До травматологічного відділення потрапив потерпілий із травмою зап’ястка та клінікою ушкодження нерва, що проходить у каналі зап’ястка. Який це нерв?

N. axillaris

N. musculocutaneus

N. ulnaris

N. medianus

N. radialis

4561 / 6307
Чоловіку для лікування хронічного обструктивного бронхіту до складу комплексної терапії треба включити муколітик. Виберіть цей препарат із запропонованих:

Вінкристин

Ацетилцистеїн

Калію оротат

Пантогам

Циклофосфамід

4562 / 6307
Жінка 38 років померла під час нападу бронхіальної астми, який не вдалося зняти. В ході гістологічного дослідження у просвіті бронхів виявлені накопичення слизу, в стінці бронхів численні лаброци-ти, багато з них у стані дегрануляції, а також велика кількість еозинофілів. Який патогенез (механізм розвитку) цих змін у бронхах?

Імунокомплексний механізм

Клітинно обумовлений цитоліз

Гранулематоз

Атопія

Цитотоксична, цитолітична дія антитіл

4563 / 6307
Скорочення поперечно-посмугованих м'язів неможливе без кальцію. Яку роль відіграє цей іон в утворенні актино-міозинових містків?

З’єднується з гістаміновими рецепторами

З'єднується із тропоніном

З’єднується із серотоніновими рецепторами

З’єднується з холінорецептором

З’єднується з адренорецепторами

4564 / 6307
Жінка з I (O) Rh- групою крові вийшла заміж за чоловіка з IV (AB) Rh+ групою крові. Який варіант групи крові і резус-фактора можна очікувати у дітей?

IV (AB) Rh+

I (O) Rh+

IV (AB) Rh-

III (B) Rh+

I (O) Rh-

4565 / 6307
У дитини спостерігається spina bifida у ділянці 12-го грудного хребця. Незрощення (щілина) якої анатомічної структури призвела до розвитку цієї аномалії?

Міжхрсбцевого диска

Дуги хребця

Остистого відростка

Суглобових відростків

Тіла хребця

4566 / 6307
У померлого, що понад 20 років працював на шахті зі здобуття кам(яного вугілля, при розтині знайдені ущільнені легені сіро-чорного кольору зі значними ділянками новоутвореної сполучної тканини. Який з перелічених діагнозів найбільш вірогідний?

Силікоантракоз

Талькоз

Антракосилікоз

Сидероз

Антракоз

4567 / 6307
Для дегідратаційної терапії у разі набряку мозку та легень призначають препарат із потужною натрійуретичною дією. Укажіть цей препарат:

Фуросемід

Теофілін

Спіронолактон

Манії

Кислота етакринова

4568 / 6307
В одному з гірських селищ мала місце масова загибель гризунiв. Одночасно хворіло населення цієї мiсцевостi. Хвороба супроводжувалася швидким підвищенням to до 40oС, вираженою інтоксикацією, збільшенням пахових лiмфовузлiв. У препаратах-мазках з трупного матеріалу виявлені грам-негативні палички овоїдної форми з біполярним забарвленням. Якi мікроорганізми є збудниками цього інфекційного захворювання?

Клостридії

Збудник сибірки

Паличка чуми

Стафілокок

Збудник туляремії

4569 / 6307
У чоловіка 60 років під час об'єктивного обстеження виявлено набряки на ногах, асцит, збільшення печінки, що свідчить про недостатність кровообігу за правошлуночковим типом. В анамнезі вказаний перенесений ревматизм. Одним з основних факторів розвитку набряків є активація системи ренінангіотензин-альдостерон, яка є наслідком:

Ацидозу

Розширення посткапілярних вен

Зменшення хвилинного об’єму серця

Утруднення дифузії речовин

Поліцитемічної гіпсрволсмії

4570 / 6307
Унаслідок дефіциту УФО-ендонуклеази порушується репарація ДНК і виникає такс захворювання:

Пігментна ксеродермія

Фенілкетонурія

Альбінізм

Серпоподібноклітинна анемія

Подагра

4571 / 6307
Клітини водія ритму серця мають специфічну фазу ПД:

Швидку систолічну деполяризацію

Систолічну рсполяризацію

Повільну діастолічну деполяризацію

Швидку діастолічну деполяризацію

4572 / 6307
Швидка допомога доставила до лікарні непритомного водія, який проспав у кабіні автомобіля з увімкненим двигуном протягом тривалого часу. Під час огляду шкірні покриви яскраво червоного кольору. Яка причина непритомності?

Накопичення вуглекислоти

Накопичення метгемоглобіну

Накопичення відновленого гемоглобіну

Накопичення карбгемоглобіну

Накопичення карбоксигемоглобіну

4573 / 6307
Підвищення вмісту молібдену в організмі призводить до інтенсивного утворення та накопичення сечової кислоти та її солей у тканинах і синовіальних оболонках суглобів, що є причиною розвитку «молібденової» подагри. Надлишок молібдену може так впливати на обмін пуринів, бо останній є складовою частиною і активатором ферменту:

Альдегідоксидази

Сульфітоксидази

Аденінфосфорибозилтрансферази

Гіпоксантингуанінфосфорибозилтрансферази

Ксантиноксидази

4574 / 6307
Жінка 56 років госпіталізована до хірургічного відділення з клінікою гострого живота. Під час операції виявлено, що частина тонкої кишки протяжністю 80 см має темно-червоний колір, серозна оболонка її тьмяна, шорстка, у черевній порожнині геморагічна рідина. Який розлад кровообігу виявлено у пацієнтки?

Крововилив у стінку тонкої кишки

Місцеве артеріальне повнокрів’я

Місцеве венозне повнокрів’я

Ішемічний інфаркт тонкої кишки

Геморагічний інфаркт тонкої кишки

4575 / 6307
Чоловіку призначена ендоскопія 12-палої кишки. В результаті виявлено запалення великого дуоденального сосочка і порушення виділення жовчі в просвіт кишки. У якому відділі 12-палої кишки виявлені порушення?

Верхня частина

Висхідна частина

Горизонтальна частина

Низхідна частина

Цибулина

4576 / 6307
У пацієнта попри повноцінне харчування розвинулася гіперхромна (мегалобластична) анемія. Напередодні він переніс операцію з резекції шлунка. Яка причина анемії?

Нестача фолієвої кислоти в їжі

Нестача вітаміну РР в їжі

Нестача вітаміну С в їжі

Нестача фактора Касла

Нестача білка в їжі

4577 / 6307
У чоловіка після повернення з роботи спостерігається дертя в горлі, біль у м’язах, підвищення температури тіла до 38,5°. Який механізм підвищення температури є найбільш раннім на першій стадії формування лихоманки?

Шкірна вазоконстрикпія

Активація окислювальних процесів

Зменшення потовиділення

М’язовий термогенез

Збільшення ЧСС

4578 / 6307
До клініки госпіталізовано чоловіка з діагнозом: карциноїд кишечника. Аналіз виявив підвищену продукцію серотоніну. Відомо, що ця речовина утворюється з амінокислоти триптофану. Який біохімічний механізм лежить в основі цього процесу?

Утворення парних сполук

Трансамінування

Декарбоксилювання

Мікросомальне окиснення

Дезамінування

4579 / 6307
Чоловікові, який хворіє на перитоніт, до комплексної антибактеріальної терапії включено препарат, що належить до похідних нітроімідазолу. Який із наведених препаратів потрібно призначити?

Бактрим

Норфлоксацин

Уросульфан

Метронідазол

Лінезолід

4580 / 6307
Гормон гіпофіза проопіомеланокортин (ПОМК) є попередником декількох тропних гормонів. Під час якого процесу він перетворюється на кортикотропін?

Апотомії

Дихотомії

Окислення

Відновлення

Обмеженого протеолізу

4581 / 6307
У чоловіка, який потрапив до неврологічного відділення, було виявлено посилення процесів гальмування в центральній нервовій системі. Надлишок якого медіатора може призвести до цього?

Адреналін

Норадреналін

ГАМК

Ацетилхолін

Дофамін

4582 / 6307
Новонародженому 1 місяць. У нього спостерігається заднє тім’ячко. Коли воно повинне зарости?

На 5 місяці життя

На 2-3 місяці життя

На 4 місяці життя

На другому році життя

На 6 місяці життя

4583 / 6307
У молодої жінки спостерігається потовщення шиї, екзофтальм, пульс -110/хв. Які додаткові дослідження треба провести для встановлення діагноза?

Томографічнс дослідження наднирко них залоз

Дослідження вмісту катехоламінів крові

Дослідження вмісту ТЗ і Т4

Тест навантаження глюкозою

Ультразвукове дослідження яєчників

4584 / 6307
У жінки під час профілактичного огляду виявили пухлину молочної залози. Результати біопсії підтвердили наявність злоякісної пухлини. Який основний механізм інфільтративного росту злоякісної пухлини?

Порушення контактного гальмування

Підвищення активності кейлонів у клітині

Підвищення ферментативної активності лізосом

Збільшення адгсзивності пухлинних клітин

Здатність пухлинних клітин до амебоїдного руху

4585 / 6307
Одним з біохімічних механізмів ускладнень цукрового діабету є посилення зв’язування глюкози з білками, що називається:

Глюкозметилюванням

Сульфурилюванням

Глюкозамінуванням

Гідроксилюванням

Глікозилюванням

4586 / 6307
Під час реплікації ДНК один із її ланцюгів синтезується із запізненням. Що визначає дану особливість синтезу?

Відсутність трифосфонуклеотидів

Компліментарність ланцюгів

Антипаралельність ланцюгів

Необхідність репарації

Великі розмірами ДНК-полімерази

4587 / 6307
У процесі старіння лабільність м’яза зменшується, змінюється величина мембранного потенціалу. Це зумовлено:

Активацією хеморецепторів

Зміною інтенсивності процесів обміну та транспортом К+ у м'язових клітинах

Закриттям потенціалзалежних Са2+ каналів

Зміною транспорту К+ у м’язових клітинах

Зміною інтенсивності процесів обміну

4588 / 6307
Міокард являє собою функціональний синтицій, і збудження, що виникає у будь-якій ділянці, поширюється на всі інші відділи. У зв’язку з цією особливістю, збудження в серці підлягає закону:

Анрепа

Франка-Старлінга

Усе або нічого

Лапласа

4589 / 6307
Чоловікові, хворому на алергічний дерматит, лікар призначив антигістамінний засіб, який не впливає на ЦНС. Який це засіб?

Лоратадин

Трифтазин

Зафірлукаст

Дипразин

Супрастин

4590 / 6307
У чоловіка, який має певні симптоми. які виникли після травми ноги лопатою, був запідозрений правець. На яке середовище треба виконати посів досліджуваного матеріалу?

Бордс-Жангу

Плоскірєва

Казеїново-вугільний агар

Леніна

Кітта-Тароцці

4591 / 6307
У пацієнта відзначено підвищену чутливість до світла, ураження відкритих ділянок шкіри, неврологічний біль. У сечі спостерігається неспецифічне виведення порфіринів. Це захворювання належить до групи порфірій і є наслідком порушення:

Утворення уробіліну

Розпаду гемоглобіну

Засвоєння вітаміну РР

Синтезу гемоглобіну

Засвоєння рибофлавіну

4592 / 6307
Під час дослідження матеріалу випадку кишкової непрохідності, викликаного стенозом термінального відділу тонкої кишки, виявлено потовщення стінки та щілиноподібні поздовжні виразки слизової. Під час мікроскопії виявлено формування гранульом, які складаються з епітеліо'їдних клітин, лімфоцитів та поодиноких клітин типу Пирогова-Лангханса у всій товщі кишки. Укажіть можливе захворювання:

Хронічний ентерит

Хвороба Крона

Амебна дизентерія

Черевний тиф

Хвороба Уїппла

4593 / 6307
Для диференційної діагностики у пацієнтнатще було взято кров на аналіз. Рівень глюкози виявився в межах норми. Назвіть клітини підшлункової залози які виробляють гормони, що регулююіь рівень глюкози в крові:

Центроацинозні клітини

Екзокринні панкреатоцити

РР-клітини

В-клітини

Фіброцити

4594 / 6307
Для поліпшення трофіки серцевого м’яза пацієнту призначено кардонат, до складу якого входить кокарбоксилаза (тіаміндифосфат) - коферментна форма вітаміну:

B5

B6

B1

B2

B12

4595 / 6307
Після опромінення високою дозою радіації у підлітка значно постраждала лімфоїдна система, стався розпад великої кількості лімфоцитів. Відновлення нормальної формули крові можливе завдяки діяльності залози:

Печінки

Щитоподібної залози

Підшлункової залози

Тимусу

Наднирників

4596 / 6307
Жінка має виражену прогресивну м'язову дистрофію. Назвіть показники азотного обміну в сечі, які характерні для такого стану:

Амонійні солі

Креатинін

Сечовина

Креатин

Сечова кислота

4597 / 6307
Лікар призначив пацієнту вітамін B6. Це забезпечить належні умови для синтезу:

Амінокислот та ТАГ

Кетонових тіл та біогенних амінів

ТАГ та фосфоліпідів

Холестерину та амінокислот

Амінокислот та біогенних амінів

4598 / 6307
Фермент, що з’єднується із субстратом, взаємодіє з ним тільки частиною молекули. Назвіть її:

Кофактор

Ділянка поліпептидного ланцюга

Алостсричний центр

Активний центр

Кофермент

4599 / 6307
У жінки, у якої наявний тиреотоксикоз, спостерігається роз’єднання окисного фосфорилювання. До яких змін це призводить?

Зменшення теплоутворення

Збільшення теплоутворення

Збільшення утворення АТФ

Теплоутворення не змінюється

3меншення частки вільного окислення

4600 / 6307
У збудливій клітині заблокували іонні канали. Це суттєво не змінило рівень потенціалу спокою, але клітина втратила здатність до генерації ПД. Які канали заблоковано?

Кальцієві

Натрієві та калієві

Хлорні

Натрієві

Калієві

4601 / 6307
Електролітичне руйнування вентромедіальних ядер гіпоталамуса в експериментальної тварини через деякий час після операції призвело до:

Гіпертонусу метасимпатичної системи

Гіпертонусу парасимпатичної системи

Схуднення

Гіпертонусу симпатичної системи

Ожиріння

4602 / 6307
Під час взаємодії норадреналіну з бета-адренорецепторами скоротливих кардіоміоцитів активується фермент, який зумовлює відкриття кальцієвих каналів і вхід іонів кальцію в кардіоміоцит, що призводить до позитивного інотро-пного ефекту. Назвіть цей фермент:

Гуанілатциклаза

Аденілатциклаза

Фосфатаза

Карбоангідраза

Трансредуктаза

4603 / 6307
На розтині тіла жінки 63 років, яка страждала на ревматизм і комбіновану мітральну ваду, виявлено, іцо стулки міірального клапана різко потовщені, зрощені між собою, кам’янистої щільності. Під час розрізання визначається хрускіт. Який патологічний процес зумовив кам’янисту щільність клапана серця?

Метаболічне звапнення

Метастатичне звапнення

Дистрофічне звапнення

Амілоїдоз

Фібриноїд

4604 / 6307
Під час гіпертонічного кризу хворому ввели магнію сульфат, в результат чого відбулося різкє зниження артеріального тиску. Введенням якого препарату можна запобiгти по6ічному ефекту сульфату магнію?

Трилон Б

Кальцію хлорид

Натрію бромід

Калію хлорид

Натрію сульфат

4605 / 6307
Лікарем швидкої допомоги у пацієнта було діагностовано анафілактичний шок, що супроводжувався бронхоспазмом. Виділення якої біологічно активної речовини тканинними базофілами зумовлює таку клінічну симптоматику?

Лейкотрієни

Брадикінін

Гепарин

Гістамін

Простагландини

4606 / 6307
Гіпоксичне пошкодження півкуль мозочка призвело до порушення рухових функцій. У чому виявляються зміни, що виникли?

Ригідність

Атетоз

Порушення координації рухів

Хорея

Фібриляція

4607 / 6307
Під час експерименту тварині зменшили кровопостачання нирок. За деякий час у тварини різко підвищився кровяний тиск. Що було причиною цього явища?

Зменшення ефективного фільтраційного тиску

Порушення процесів реабсорбції в проксимальних звивистих канальцях

Порушення процесів реабсорбції в дистальних звивистих канальцях

Підвищення продукції реніну

Зменшення клубочкової фільтрації

4608 / 6307
Батьки для профілактики кишкових інфекцій у дитини 3-х років тривало застосовували антибіотики. Через місяць стан дитини погіршився. У крові - виражена лейкопенія і гранулоцитопенія. Який найбільш вірогідний механізм виявлених змін у крові?

Аутоімунний

Перерозподільний

Віковий

Мієлотоксичний

Гемолітичний

4609 / 6307
До інфекційного відділення надійшов чоловік із діагнозом: холера. Яка основна група антибіотиків для лікування цього захворювання?

Тетрацикліни

Пеніциліни

Цефалоспорини

Аміноглікозиди

Макроліди

4610 / 6307
У пацієнта після курсу лікування атеросклерозу в плазмі крові лабораторно доведено збільшення рівня ан-тиатерогенної фракції ліпопротеїнів. Збільшення рівня яких ліпопротеїнів підтверджує ефективність терапії захворювання?

ЛПНЩ

ЛПДНЩ

Хіломікрони

ЛПВЩ

ЛППЩ

4611 / 6307
Після вживання м’ясної консерви у школяра з’явилися неврологічні симптоми. Був поставлений діагноз: ботулізм. Які екстрені методи лікування необхідно використати?

Введення сульфаніламідних препаратів

Призначення проносних засобів

Введення антибіотиків

Введення антиботулінічної сироватки

Введення антиботулінічної вакцини

4612 / 6307
Чоловікові, який хворіє на бронхопневмонію, лікар призначив ацетилцистеїн. Визначте показання до його застосування:

Судоми

Бронхіальна астма

Асфіксія новонароджених

Бронхіт із харкотинням

Серцева недостатність

4613 / 6307
Після перенесеної травми голови у хворого бувають напади інтенсивного болю в ділянці обличчя і судоми жувальних м’язів. Який нерв найімовірніше травмований?

N. abducens

N. trigeminus

N. oculomotorius

N. olphactorius

N. facialis

4614 / 6307
У людини діагностовано галактоземію - хворбу накопичення. Цю хворобу можна діагностувати за допомогою якого методу?

Генеалогічного

Цитогенетичного

Біохімічного

Популяційно-статистичного

Близнюкового

4615 / 6307
У жінки діагностовано гострий період захворювання на ГРВІ. Імуноглобуліни якого класу, ймовірно, будуть присутні в крові пацієнтки?

IgE

IgG

IgM

IgD

IgA

4616 / 6307
У людини внаслідок лікування антибіотиками виник дисбактеріоз товстого кишечника. Яких вітамінів, синтезованих бактеріями в товстому кишечнику, буде менше надходити до організму?

Вітамін D

Вітамін К та вітаміни групи 8

Вітаміни А та Е

Вітаміни Р і С

Аскорбінова кислота

4617 / 6307
У потерпілого виявлено ножове поранення м’язів передньої стінки пахвової ямки. Які м’язи грудної клітки формують цю стінку?

M. serratus anterior, m. subclavius

M. pectoralis minor, m. pectoralis major

M. serratus anterior, m. pectoralis minor

М. pectoralis minor, т. subclavius

M. serratus anterior, m. pectoralis major

4618 / 6307
У жінки 42 років, яка перенесла операцію на нирці, після наркозу розвинулися явища рекураризації і припинилося дихання.

Кофеїн

Стрихніну нітрат

Галантоміну гідробромід

Прозерин

Плазму крові

4619 / 6307
На 8-й день після введення протиправцевої сироватки з приводу брудної рани стопи у пацієнта підвищилася температура тіла до 380С, з’явилися біль у суглобах, висипка, свербіж. У крові - лейкопенія і тромбоцитопенія. Який тип алергічної реакції розвинувся?

Анафілактична

Імунокомплексна гіперчутливість

Стимулююча

Цитотоксична

Гіперчутливість уповільненого типу

4620 / 6307
After severe emotional strain a 45-year-old man suddenly developed constricting pain in the area of his heart. The pain was irradiating into his left arm, neck, and left shoulder blade. His fase was pale and covered in cold sweat. Nitroglycerine was able to relieve the pain. What condition developed in the patient in this case?

Myocardial infarction

Gastric ulcer perforation

Angina pectoris

Psychogenic shock

Stroke

4621 / 6307
A 23-year-old woman was brought into the emergency department complaining of bloody diarrhea, tiredness, and dizziness. A few days ago she went to a fast food reataurant for a birthday party. Her friends are experiencing similar symptoms. Laboratory studies show anemia. What samples should be obtained for microbiologic testing in this case?

Urine

Blood

Bile

Stool

Cerebrospinal fluid

4622 / 6307
Histological microslide shows a gastrointestinal organ. The wall of this organ consists of 4 layers: mucosal, submucosal, muscular, and serous. The muscular layer has folds and pits. What organ has such appearance?

Esophagus

Duodenum

Appendix

Small intestine

Stomach

4623 / 6307
A 2-year-old child with delayed physical and mental development was brought to the hospital. The child’s parents are the most concerned by frequent profuse vomiting that occurs in their child after eating. Laboratory testing detected phenylpyruvic acid in the child’s urine. What type of metabolism is disturbed, causing this patology?

Lipid metabolism

Amino acid metabolism

Water and electrolyte metabolism

Carbohydrate metabolism

Phosphorus and calcium metabolism

4624 / 6307
The heart is a muscular organ that pumps blood through the body. This function is carried out by valves, muscles, and vessels. What valve is located between the left ventricle and left atrium?

Aortic

Non-return

Tricuspid

Pulmonary

Bicuspid

4625 / 6307
During your physiology class, the professor asks you to explain the effect of various hormones and neurotransmitters on the metabolism of glucose in the human body. You open your report with the statement that the use of glucose by the cells is preceded by its transport from the intercellular substance into the cell. What hormone is most likely responsible for the glucose uptake by the cell?

Adrenaline

Aldosterone

Glucagon

Thyroxine

Insulin

4626 / 6307
A 25-year-old man came to the family doctor complaining of the loss of taste sensation in the front two- thirds of his tongue. The doctor determined that this condition was caused by the damage to a certain nerve. What nerve is likely to be functionally impaired in this case?

Hypoglossal nerve

Chorda tympani

Accessory nerve

Vagus

Glossopharyngeal nerve

4627 / 6307
A 28-year-old man complains of nausea, vomiting, and right-sided subcostal pain. Objectively, his skin and sclerae are icteric, he has elevated body temperature, enlarged liver, dark urine, fecal hypocholia, hyperbilirubinemia (direct and indirect bilirubin), bilirubinuria, urobilinuria, hypoproteinemia, and decreased blood coagulability. What condition can be characterized by these changes?

Pre-hepatic hemolytic jaundice

Acute cholecystisis

Hepatocellular parenchymal jaundice

Post-hepatic jaundice

Acute pancreatitis

4628 / 6307
A soft tissue node obturated the patient’s middle lobe bronchus, leading to the development of right middle lobe atelectasis. In the area of obturation, bronchial biopsy detected proliferations of atypical glandular epithelium with pathological mitoses. Epithelium penetrates into the underlaying tissues and cartilage. What disease is the most likely in this case?

Bronchial epithelial dysplasia

Bronchial sarcoma

Inflammatory polyp

Bronchogetic lung cancer

Bronchitis deformans

4629 / 6307
The main functions of neural tissue can be described as communication and integration. What is the basic unit of neural tissue that ensures its functioning?

Axon

Dendrite

Neuron

Myelin sheath

Nucleus

4630 / 6307
The patient has developed a refractive error, where the focal point of the image is located not precisely on the retina, but anterior to it. This causes distant objects to be blurry, while close objects often appear normal. Among the other signs there are headaches and eye strain. This condition is associated with retinal detachment, cataract, and glaucoma. Name this condition:

Nearsightedness

Astigmatism

Emmetropia

Blindness

Farsightedness

4631 / 6307
A 20-year-old woman came to the clinic after missing her last 2 periods. Her cycles are usually regular, occurring at 28-30 day interval with moderate bleeding and occasional abdominal pain. She also complains of progressively diminishing peripheral vision. Her doctor determined the loss of vision in the lateral halves of both retinas. What structure is likely to be alfected, causing bitemporal hemianopsia in the patient?

Right optic tract

Left optic tract

Right optic nerve

Left optic nerve

Optic chiasm

4632 / 6307
A woman was bitten by a venomous snake (Macrovipera lebetinis). Autopsy of her body shows marked intravascular hemolysis. Her pancreas, bone marrow, and lymph nodes are brown. Microscopy detects abundant brown pigment in the macrophage cytoplasm. What pigment accumulates in the tissues in such cases?

Hemosiderin

Hematoidin

Lipofuscin

Hematin

Bilirubin

4633 / 6307
Histological microslide shows an organ consisting of gray and white matter. The gray matter is located in the center and is made up of bundle neurons (neurocytus funicularis), radicular neurons (neurocytus radiculatus), and interneurons. Such morphology is characteristic of the following organ?

Cerebral hemispheres

Spinal ganglion

Medulla oblongata

Cerebellum

Spinal cord

4634 / 6307
A molecular biologist studies various molecules. One of them is a polymeric molecule essential in various biological roles in coding, decoding, regulation, and expression of genes. It is a helical single­stranded molecule folded onto itself. Which of the following molecules is being studied?

RNA

DNA

HLA

ATP

ADP

4635 / 6307
There is a system that maintains the balanced state of multicellular organisms. Its main functions are to ensure an adequate physiological response and to defend the body against infectious agents and other unintended invasions. What system has such functions?

Homeostatic system

Lymphatic system

Nervous system

Immune system

Endocrine system

4636 / 6307
A team of medical students researches the phases of cell cycle. During one of the mitotic phases the cell has nearly completed its division, the chromosomes decondense, and two nuclei begin to form around them. What phase is likely observed in the cell at this moment?

Anaphase

Prophare

Metaphase

Telophase

4637 / 6307
A 34-yeat-old man visits his dentist complaining of a toothache. After a dental procedure that involved extraction of several teeth, he developed a severe bleeding lasting more than 15 minutes. He has a history of chromic hepatitis C. What is the most likely cause of the prolonged bleeding in this patient?

Hypoalbuminemia

Hypofibrinogenemia

Thrombocytopenia

Hypocalcemia

4638 / 6307
Medical examination in an army recruitment center detected a 15-year-old boy with tall stature, eunuchoid body proportions, gynecomastia, and female pattern of pubic hain growth. The boy has fat deposits on the thighs, no facial fair growth, high-pitched voice, and below average IQ. He was diagnosed with Klinefelter syndrome. What karyotype corresponds with this disease?

45, X0

47, XXY

46, XY

47, XY, 18

47, XYY

4639 / 6307
A man with signs of peritonitis was brought to the admission room. He has a 12-year-long history of peptic ulcer disease with ulcer localization on the posterior gastric wall. He was diagnosed with gasiric ulcer perforation. What anatomical structure is likely to be contaminated with gastric content in this case?

Bursa hepatica

Canalis lateralis dexter

Canalis lateralis sinister

Bursa pregastrica

Bursa omentalis

4640 / 6307
A 46-year-old man complains of tiredness and pain in the joints of his fingers and wrists. These signs are observed for the last 2 months. The pain is present in both hands and the wrists are swollen. Furthermore, he describes morning stiffness in his joints, lasting about 2 hours, which improves with use. His past medical history reveals that he was successfully treated for H. pylori - related ulcer last year. He denies smoking and stopped drinking when his GI symptoms started. What drug is the best choice for his joint pain management?

Prednisolone

Patacelanol

Morphine

Aspirin

Celecoxib

4641 / 6307
A 27-year-old woman complains of insomnia, irritability, hand tremor, acute weight loss despite high appetite, and constant fever with body temperature of 37,2-37,5°C. What endocrine gland is likely to be functionally impaired in this case?

Thyroid gland

Pancreas

Neurohypophysis

Parathyroid gland

Adrenal glands

4642 / 6307
General practitioner performs physical examination of the patient. In the course of the examination it is necessary to measure the palpable regular expansion of a superficial artery caused by the ejection of blood into the vessels through heart contractions. What sign does the doctor measure?

Heart electricity

Blood pressure

Pulse

Saturaion

Heartbeat

4643 / 6307
A man has signs of acute respiratory viral disease. His physician referred him for microbiological testing. A mucus smear obtained trom the patient’s tonsils contains spherical microorganisins arranged in short chains. The patient was diagnosed with tonsillitis. What microorganisms were detected in the smear from the patient's tonsils?

Streptocci

Micrococci

Tetracocci

Staphylococci

Diplococci

4644 / 6307
Muscle relaxant tubocurarine was administered during a surgery. After the surgery the patient's breathing is not restored. What antidote should he be given to resore his breathing? Anaprilin (Propranolol)

Bemegride

Correct answer

Clophelin (Clonidine)

Atropine sulfate

Proserin (Neostigmine)

4645 / 6307
A 45-yeat-old woman came to her physician with complaints of extreme tiredness and weakness. She says that these symptoms lasts for a month already. Within the last 2 weeks she losts 3 kilograms. Objectively, she is tired-looking thin woman. Skin hyperpigmentation is observed in many areas of her body, most prominently on the face, neck, and the backs of her hands (the areas exposed to light). What hormone is produccd in an excess in this patient, most likely causing the hyperpigmentation? Thyroid-stimulating hormone (TSH)

Correct answer.

Growth hormone (GH)

Melanocyte-stimulating hormone (MSH)

P-Lipotopin

Gonadotropins

4646 / 6307
A 50-year-old woman complains of constant thist. She drinks large amounts of liquid and has increased diuresis. Her blood glucose is 12 mmol/L. Her urine contains glucose. What endocrine organ is likely to be functionally impaired in this patient?

Parathyroid gland

Thyroid gland

Neurohypophysis

Adrenal glands

Pancreas

4647 / 6307
An unidentified surgical specimen is received fot histopathologic analysis. A portion of the specimen is cut and stained with hematoxylin and eosin. Under the microscope, you see an organ encapaulated by dense connective tissue that extends to the deeper areas by way of the trabecular extensions. The organ can be subdivided into two regions: a cortex with lymphoid nodules and medulla with medullary cords populated by plasma cells, B cells, and Tcells. What anatomical structure is the most likely origin of this surgical specimen?

Thymus

Tonsils

Lymph node

Bone marrow

Spleen

4648 / 6307
A 65-year-old woman was brought into the emergency departmnent because of shortness of breath and chest pain that started a few hours ago. She denied having fever, expectoration, or any accompanying symptoms. She had a 5-year-long history of deep vein thrombosis in her thin. After a time, she died of respiratory distress. Autopsy reveals red loose masses lodged in the bifurcation of the pulmonary trunk with extensions into both the left and the right pulmonary arteries. What is the most likely diagnosis?

Thromboembolism

Myocardial infarction

Pneumonia

Pneumothorax

4649 / 6307
A 54-year-old woman had a total thyroidectomy lor papillary thyroid carcinoma. 11 hours after operation she complained of tingling around her mouth. On physical examination, the Trousseau's sign and Chvostek's sign are present. Her condition rapidly deteriorates with laryngospasm and focal seizures. The surgeon suspects surgical destruction of the parathyroid glands. What is the most likely cause of this patient's neurological condition?

Hyperchloremia

Hypophosphatemia

Hypocalcemia

Hyperkalemia

Hyponatremia

4650 / 6307
В ході гістологічного дослідження біоптату, отриманого з нижньої третини стравоходу 57-річного чоловіка з симптомами тривалого рефлюксу шлункового вмісту, виявлено наступні зміни: у слизовій оболонці на місці багатошарового плаского епітелію спостерігається одношаровий залозистий призматичний епітелій, з ознаками продукції слизу. Вкажіть патологічний процес, який виник у слизовій оболонці:

Регенерація

Організація

Гіпертрофія

Метаплазія

Гіперплазія

4651 / 6307
У хворого після резекції шлунка з приводу виразкової хвороби перистальтика кишківника не відновилася. Який лікарський засіб доцільно призначити хворому для відновлення моторики?

Гігроній

Атенолол

Метилурацил

Прозерин

Резерпін

4652 / 6307
В сироватці крові при постановці імуноферментної реакції був визначений НВ8-антиген. При якому захворюванні зустрічається даний антиген?

Вірусний гепатит В

Туберкульоз

Сифіліс

СНІД

Вірусний гепатит А

4653 / 6307
У хворого на крупозну пневмонію має місце гарячка з температурою тіла 39°С, при цьому добові коливання температури не перевищували 1°С впродовж 9-ти діб. До якого типу температурних кривих відноситься ця гарячка?

Постійна

Гектична

Поворотна

Гіперпіретична

Ремітуюча

4654 / 6307
У хворого 67-ми років з клінічним діагнозом хронічного бронхіту, пневмосклерозу, серцево-легеневої недостатності взято біоптат з підозрілої ділянки слизової оболонки правого бронха. Гістологічно встановлено клітинний і тканинний атипізм, появу структур у вигляді 'ракових перлин’.’ Якому патологічному процесу відповідають зазначені гістологічні зміни?

Плоскоклітинна метаплазія слизової бронху

Хронічний поліпозний бронхіт

Плоскоклітинний рак бронху з ороговінням

Гострий бронхіт

Бронхоектаз

4655 / 6307
Після нападу бронхіальної астми хворому проведено дослідження периферичної крові. Які зміни очікуються?

Лімфоцитоз

Еозинофілія

Тромбоцитопенія

Лейкопенія

Еритроцитоз

4656 / 6307
Людина знаходиться в середовищі з темепературою 38 градусів С, відносною вологістю повітря 50%. Які шляхи тепловіддачі зумовлюють підтримку постійної температури ядра тіла за цих умов?

Конвекція і теплопроведення

Конвекція

Теплопроведення

Випаровування

Радіація

4657 / 6307
В клітинах здорової печінки активно синтезуються глікоген ТА білки. Які типи органел добре розвинуті?

Мітохондрії

Клітинний центр

Гранулярна та агранулярна ЕПС

Пероксисоми

Лізосоми

4658 / 6307
У людини з четвертою групою крові (генотип ІАІВ) в еритроцитах одночасно присутні антиген А, який контролюється алелем ІА, і антиген В - продукт експресії алеля ІВ. Прикладом якої взаємодії генів є дане явище?

Полімерії

Неповного домінування

Кодомінування

Епістазу

Комплементарності

4659 / 6307
До лікаря звернувся хворий 30-ти років з імовірною на ВІЛ-інфекцію. Для уточнення діагнозу лікар запропонував провести полімеразну ланцюгову реакцію. Основним процесом в даному дослідженні є:

Транскрипція

Хромосомна мутація

Генна мутація

Генетична рекомбінація

Ампліфікація генів

4660 / 6307
Після травми у хворого спостерігається порушення моторних центрів, що регулюють діяльність м’язів голови. В яких ділянках кори півкуль в нормі локалізується відповідний центр?

Надкрайова звивина

Нижня частина передцентральної звивини

Верхня частина передцентральної звивини

Верхня тім'яна часточка

Кутова звивина

4661 / 6307
Під час спортивних змагань боксер отримав сильний удар у живіт, що привело до нокауту через короткочасне падіння артеріального тиску. Які фізіологічні механізми викликали цей стан?

Підсилення парасимпатичних впливів

Раптова зміна кількості рідини у організмі

Ішемія ЦНС

Зміна транскапілярного обміну

Подразнення симпатичних нервів

4662 / 6307
Хворий висловлює скарги на біль у верхньому відділі пупкової ділянки. Пальпаторно визначається рухлива болісна кишка. Яку кишку пальпує лікар?

Клубова

Сигмоподібна

Поперечно-ободова

Дванадцятипала

Порожня

4663 / 6307
У людини при переході зі світлого приміщення в темне відбувається розширення зіниць. Який з наведених рефлексів зумовлює цю реакцію?

Симпатичний умовний

Метасимпатичний

Парасимпатичний умовний

Парасимпатичний безумовний

Симпатичний безумовний

4664 / 6307
Чоловіку 35-ти років з виразковою хворобою зроблено резекцію антрального відділу шлунку. Секреція якого гастроінтестинального гормону внаслідок операції буде порушена найбільше?

Холецистокінін

Гастрин

Секретин

Нейротензин

Гістамін

4665 / 6307
У хворого на виразкову хворобу шлунка під час мікроскопії мазка-відбитка з біоптату слизової виявлені грамнегативні напівзігнуті дугоподібні мікроби, тест на уреазну активність позитивний. Які мікроби були виявлені у хворого?

Спірили

Спірохети

Хелікобактерії

Трепонеми

Лептоспіри

4666 / 6307
Для лікування ревматоїдного артриту жінці 65-ти років у складі комплексної терапії був призначений гормональний препарат, який має імунодепресивну дію. Визначте цей препарат:

Супрастин

Рибофлавін

Преднізолон

Тималін

Ферковен

4667 / 6307
У чоловіка 41-го року відзначаються періодичні напади серцебиття (пароксизми), сильне потовиділення, напади головного болю. При обстеженні виявлена гіпертензія, гіперглікемія, підвищення основного обміну, тахікардія. При якій патології наднирників спостерігається подібна картина?

Гіпофункція кори наднирників

Гіперфункція кори наднирників

Гіперфункція мозкового шару

Первинний альдостеронізм

Гіпофункція мозкового шару

4668 / 6307
На гістологічному препараті нирки в дистальному звивистому канальці виявляються клітини, які щільно прилягають до ниркового тільця. Базальна мембрана їх дуже тонка і не утворює складок. Ці клітини відчувають зміни вмісту натрію у сечі та впливають на секрецію реніну юкстагломерулярними клітинами. Які це клітини?

Подоцити

Юкстагломерулярні клітини

Клітини щільної плями

Мезангіальні клітини

Ендотелій капілярів клубочка

4669 / 6307
Хворий 67-ми років вживає з їжею переважно яйця, сало, вершкове масло, молоко, м’ясо. У крові виявлено холестерин 12,3 ммоль/л, загальні ліпіди - 8,2 г/л, підвищені фракції ліпопротеїнів низької щільності (ЛПНЩ). Яка гіпер-ліпопротеїнемія спостерігається у хворого?

Холестерин, гіперліпопротеїнемія

Гіперліпопротеїнемія І типу

Гіперліпопротеїнемія ІІа типу

Гіперліпопротеїнемія IIb типу

Гіперліпопротеїнемія IV типу

4670 / 6307
У підлітка 12-ти років, який хворіє на бронхіальну астму, виник тяжкий напад астми: виражена експіраторна задишка, блідість шкірних покривів. Який вид порушення альвеолярної вентиляції має місце?

Центральний

Нервово-м’язовий

Обструктивний

Торако-діафрагмальний

Рестриктивний

4671 / 6307
У хворого травма у ділянці передньої поверхні переднього драбинчастого м’яза. Функція якого нерва може бути порушена?

Плечове сплетення

Додатковий

Зворотній гортанний

Діафрагмальний

Блукаючий

4672 / 6307
Пацієнту, що хворіє на цукровий діабет та алергічний дерматит, лікар призначив фторований гормональний препарат в мазі. На питання хворого, про переваги призначеного препарату над маззю гідрокортизону лікар пояснив, що:

Діє короткочасно

Діє слабше

Коштує дешевше

Призначений препарат практично немає резорбтивної дії

Посилює синтез інсуліну

4673 / 6307
На розтині у потиличній частці головного мозку виявлена порожнина 2,5x1,5 см, заповнена прозорою рідиною, стінка її гладенька, буруватого кольору. Який процес розвинувся у головному мозку?

Абсцес мозку

Вада розвитку мозку

Сіре розм’якшення мозку

Кіста на місці сірого розм’якшення

Кіста на місці крововиливу

4674 / 6307
Під час дослідження коронарних артерій виявлені атеросклеротичні бляшки з кальцинозом, що закривають просвіт на 1/3. У м’язі дрібні множинні білуваті прошарки сполучної тканини. Який процес виявлено в міокарді?

Післяінфарктний кардіосклероз

Інфаркт міокарда

Тигрове серце

Міокардит

Дифузний кардіосклероз

4675 / 6307
У хворого запалення легенів ускладнилось ексудативним плевритом. В якому з перелічених анатомічних утворень переважним чином може накопичуватися рідина?

sinus obliquus pericardii

sinus costomediastinalis pleurae

sinus phrenicomediastinalis pleurae

sinus costodiaphragmaticus pleurae

sinus transversus pericardii

4676 / 6307
До інфекційного відділення поступив хворий з ознаками жовтяниці внаслідок зараження вірусом гепатиту. Який з перерахованих показників є суворо специфічним, що відрізняє паренхіматозну жовтяницю від решти?

Холалемія

Підвищення рівня АлАТ, АсАТ

Білірубінурія

Гіпербілірубінемія

Уробілінурія

4677 / 6307
У хворого з дихальною недостатністю рН крові 7,35. Визначення рСО. показало наявність гіперкапнії. При дослідженні рН сечі відзначається підвищення її кислотності. Яка форма порушення кислотно-основного стану в даному випадку?

Ацидоз метаболічний, компенсований

Алкалоз газовий, компенсований

Ацидоз метаболічний, декомпенсований

Алкалоз газовий, декомпенсований

Ацидоз газовий, компенсований

4678 / 6307
В хірургічне відділення доставлено чоловіка 35 років з гнійною раною на шиї попереду трахеї ( в ділянці передвісцерального простору). Куди може розповсюджуватись інфекція, якщо хворому терміново не зроблять операцію?

В надгрудинний міжапоневротичний простір

В ретровісцеральний простір

В грудну порожнину - в заднє середостіння

В грудну порожнину - в середнє середостіння

В грудну порожнину - переднє середостіння

4679 / 6307
Який стан може розвинутися через 15-30 хвилин після повторного введення антигену внаслідок підвищеного рівня антитіл, переважно ІдЕ, які адсорбуються на поверхні клітин-мішеней - тканинних базофілів (тучних клітин) та базофілів крові?

Імунно-комплексна гіперчутливість

Сироваткова хвороба

Гіперчутливість уповільненого типу

Антитіло-залежна цитотоксичність

Анафілаксія

4680 / 6307
До медико-генетичної консультації звернулися батьки хворої дівчинки 5 років. Після дослідження каріотипу виявили 46 хромосом. Одна з хромосом 15-ї пари була довша від звичайної, тому що до неї приєдналася хромосома з 21-ї пари. Який вид мутації має місце в цієї дівчинки?

Нестача

Інверсія

Дуплікація

Делеція

Транслокація

4681 / 6307
У хлопчика 2-х років діагностована хвороба Дауна. Які зміни в хромосомах можуть бути причиною цієї хвороби?

Трисомія по 18-й хромосомі

Трисомія по Х-хромосомі

Трисомія по 21-й хромосомі

Трисомія по 13-й хромосомі

Моносомія по Х-хромосомі

4682 / 6307
93 Досліджують процеси тепловіддачі у роздягненої людини при кімнатній температурі. З'ясовано, що за таких умов найбільша кількість тепла віддається шляхом:

Випаровування

Конвекції

Теплорадіацїї

Теплопроведення

4683 / 6307
У зв’язку з вираженим больовим синдромом хворому призначено наркотичний анальгетик. Вкажіть обраний препарат:

Морфін

Німесулід

Індометацин

Анальгін

Димексид

4684 / 6307
В матеріалі, взятому від хворої людини, знайдено декілька видів мікроорганізмів (стафілококи та стрептококи різних видів), які стали причиною захворювання. Як називається такий вид інфекції?

Реінфекція

Вторинна інфекція

Змішана інфекція

Коінфекція

Суперінфекція

4685 / 6307
Хворий 30-ти років після перенесеного вірусного гепатиту В став скаржитися на тривалі носові кровотечі. Призначення якого засобу буде найбільш раціональним для корекції цього стану?

Фраксіпарин

Дипіридамол

Аспаркам

Вікасол

Фолієва кислота

4686 / 6307
Активація ряду факторів системи гемостазу здійснюється шляхом приєднання до них іонів кальцію. Наявність якого структурного компонента у їх складі забезпечує приєднання іонів кальцію?

Моноамінодикарбонові кислоти

у-карбоксиглутамінова кислота

Гідроксипролін

у-аміномасляна кислота

у-оксимасляна кислота

4687 / 6307
Екзофтальм, що спостерігається при тиреотоксикозі, обумовлений накопиченням в ретробульбарній тканині речовин, які мають високу здатністю зв’язувати воду. Які це речовини?

Креатин

Фосфоліпіди

Холестерин

Глікозаміноглікани

АТФ

4688 / 6307
У дівчинки 3-х років з затримкою психічного розвитку діагностовано сфінголіпідоз (хвороба Німана-Піка). Порушення синтезу якої речовини спостерігається при цьому?

Цераміди

Гангліозиди

Сфінгозин

Сфінгомієліназа

Глікозилтрансфераза

4689 / 6307
У людини через 10 хвилин після початку інтенсивної фізичної роботи кількість еритроцитів у крові збільшилася з 4,0*1012/л до 4,5*1012/л. Що є основною причою цього?

Вихід еритроцитів з депо

Збільшення хвилинного об’єму крові

Активація еритропоезу

Втрата води організмом

Пригнічення руйнування еритроцитів

4690 / 6307
У жінки, яка відпочивала на дачі, відразу після укусу оси виник біль, через кілька хвилин на шкірі в місці укусу з’явився пухир, еритема і сильне свербіння, а ще через деякий час -кропив’янка, експіраторна задишка. Внаслідок дії яких факторів у хворої розвинулась експіраторна задишка?

Адреналін

Фактор Хагемана

Гістамін

Норадреналін

Лізосомальні ферменти

4691 / 6307
У хворого, який скаржиться на поліурію і полідипсію, знайдено цукор в сечі. Вміст цукру в плазмі крові у нормі. З чим пов'язаний механізм глюкозурії у хворого?

Пперпродукція глюкокортикоїдів наднирниками

Порушення фільтрації глюкози в клубом новому відділі нефрону

Недостатня продукція інсуліну підшлунковою залозою

Порушення реабсорбції глюкози в канальцях нефрону

Інсулінорезистентність рецепторів клітин

4692 / 6307
На електронній фотографії представлена органела, що являє собою великий поліпротеазний комплекс, що складається з трубкоподібної та двох регуляторних частин, які розташовані на обох кінцях органели. Остання виконує функцію протеолізу. Назвіть цю органелу.

Комплекс Гольджі

Протеасома.

Включення

Рибосома

Центріоль

4693 / 6307
У чоловіка виявлене захворювання, яке зумовлене домінантним геном, локалізованим у Х-хромосомі. У кого із дітей буде це захворювання, якщо дружина здорова?

Тільки у синів

Тільки у дочок

У всіх дітей

У половини дочок

У половини синів

4694 / 6307
У пацієнта, що прибув з ендемічного за малярією району, підвищилася температура тіла, відзначається головний біль, озноб, загальне нездужання -симптоми, що характерні й для звичайної застуди. Які лабораторні дослідження необхідно провести, щоб підтвердити або спростувати діагноз 'малярія'?

Дослідження спинномозкової рідини

Мікроскопія пунктату червоного кісткового мозку

Дослідження пунктату лімфовузлів

Мікроскопія мазків крові

Аналіз сечі

4695 / 6307
При повторному введенні алергену починається виділення гістаміну тучними клітинами крові. До якого рівня реактивності відноситься така відповідь організму?

Органний

Клітинний

Субклітинний

Молекулярний

Системний

4696 / 6307
У хлопчика 7 р., із дрібнокрапчастим яскраво-рожевим висипом на гіперемованому фоні шкіри лоба, шиї, внизу живота, підколінних ямках, носо-губний трикутник блідий. В ротоглотці — відмежована яскраво-червона гіперемія, мигдалики набряклі, рихлі, в лакунах є гній, малиновий язик. Шийні лімфовузли збільшені, щільні, болючі. Поставити діагноз:

Дифтерія

Інфекційний мононуклеоз

Коклюш

Скарлатина

Краснуха

4697 / 6307
У жінки 45-ти років артеріальна гіпертензія с високою концентрацією ангіотензину-2 у крові. Який з перерахованих антигіпертензивних засобів найбільш показаний у цьому випадку?

Метопролол

Резерпін

Верапаміл

Празозин

Лізиноприл

4698 / 6307
Відомо, що робота в шахті пов’язана з вдиханням значної кількості вугільного пилу. В яких клітинах легень можна виявити вугільний пил?

Альвеолярних макрофагах

Перицитах капілярів

Респіраторних епітеліоцитах

Ендотеліоцитах капілярів

Секреторних епітеліоцитах

4699 / 6307
Хворий 40-ка років страждає на бронхіальну астму та скаржиться на тахікардію протягом тривалого часу. Вкажіть найбільш оптимальний в даній ситуації препарат для купірування бронхоспазму:

Адреналіну гідрохлорид

Ізадрин

Ефедрину гідрохлорид

Орципреналіну сульфат

Сальбутамол

4700 / 6307
Хворий доставлений в хірургічне відділення з різаною раною передньої поверхні плеча у нижній третині. В нього порушена функція згинання в плечовому і ліктьовому суглобах. Це є результатом ушкодження такого м’язу:

Ліктьовий м’яз

Дзьобоподібно-плечевий м’яз

Дельтоподібний м’яз

Трьохголовий м’яз плеча

Двоголовий м’яз плеча

4701 / 6307
У хворого на цукровий діабет розвинулась діабетична нефропатія з розвитком уремії. Швидкість клубочкової фільтрації 9 мл/хв. Який найбільш імовірний механізм зниження швидкості клубочкової фільтрації і розвитку хронічної ниркової недостатності у пацієнта?

Розвиток ацидозу в тканинах

Зменшення маси діючих нефронів

Закупорка просвіту канальців нефрону гіаліновими циліндрами

Зниження системного артеріального тиску

Спазм артеріол

4702 / 6307
У хворої на бронхіальну астму вірусне інфікування спровокувало астматичний статус зі смертельним наслідком. При гістологічному дослідженні легень виявлено: спазм і набрякання бронхіол, в їх стінках виражена інфільтрація лімфоцитами, еозинофілами та іншими лейкоцитами, а також дегрануляція лаброцитів. Який механізм гіперчутливості лежить в основі зазначених змін?

Реагінова реакція

Запальний

Імунокомплексний

Аутоімунний

Імунозумовлений клітинний цитоліз

4703 / 6307
На розтиш тїла жінки з середньої Азії, що тривалий час страждає на виснажуючу лихоманку, були виявлені збільшені' печінка і селезінка аспідно-чорного кольору. Кістковий мозок гіперплазований, такого ж кольору. Кора головного мозку сіро-димчатого кольору. Для якого захворювання характерні такі зміни?

Гепатит

Сепсис

Малярія

Висипний тиф

СНЩ

4704 / 6307
14. У лабораторному експерименті на собаці вивчали будову центральних від ділів слухової сенсорної системи. Була зруйнована одна з структур середнього мозку. Собака втратив орієнтувальний рефлекс на звукові сигнали. Яка структура була зруйнована?

Верхні горбики чотиригорбикового тіла

Червоне ядро

Ядра ретикулярної формації

Чорна речовина

Нижні горбики чотиригорбикового тіла

4705 / 6307
Чоловіку 40-ка років за результатами діагностичних тестів зробили лімфографію органів грудної порожнини. Хірург встановив, що пухлина вразила орган, з лімфатичних судин якого лімфа безпосередньо переходить в грудну протоку. Який це орган?

Трахея

Осердя

Лівий головний бронх

Серце

Стравохід

4706 / 6307
В лікарню надійшов потерпілий з рваною раною гомілки, яка виникла внаслідок того, що його покусала хвора на сказ тварина. Яку вакцину необхідно ввести для попередження сказу?

Антирабічна вакцина

АКДП

АДП

БЦЖ

TABte

4707 / 6307
У людини внаслідок патологічного процесу збільшена товщина альвеолокапіляр-ної мембрани. Безпосереднім наслідком цього буде зменшення у людини:

Альвеолярної вентиляції легень

Дифузійної здатності легень

Резервного об’єму видиху

Кисневої ємності крові

Хвилинного об’єму дихання

4708 / 6307
В експерименті подразнюють гілочки симпатичного нерва, які інервують серце. Це призвело до збільшення сили серцевих скорочень, тому що через мембрану типових кардіоміоцитів збільшився:

Вхід іонів калію

Вихід іонів кальцію

Вхід іонів кальцію та калію

Вхід іонів кальцію

Вихід іонів калію

4709 / 6307
Застосування антикоагулянтного засобу викликало крововиливи на слизових оболонках, макрогематурію, зменшення часу згортання крові. Введення протаміну сульфату усунуло ці порушення. Який засіб було призначено?

Фібринолізин

Вікасол

Неодикумарин

Гепарин

Натрію цитрат

4710 / 6307
У хворого 47-ми років виникла кишкова коліка на фоні гіпертонічної хвороби. Засоби якої з перерахованих груп найбільш доцільно використати для її купірування у даній ситуації?

Антихолінестеразні засоби

Адреноміметики

М-холиноміметики

Міотропні спазмолітики

Симпатоміметики

4711 / 6307
Чоловік 65-ти років раптово втратив зір на одне око в зв'язку з відшаруванням сітківки. Хворому була проведена енуклеація. Під час гістологічного дослідження видаленого очі в сітківці і судинній оболонці виявлені комплекси атипових клітин з вираженим поліморфізмом клітин і ядер, з помірною кількістю мітозів, враховуючи патологічні. У цитоплазмі клітин і міжклітинної середовищі виявляється пігмент коричневого кольору, що дає позитивну реакцію ДОФА. Реакція Перлса негативна. Який найбільш ймовірний діагноз?

меланома

пігментний невус

Цістоцеркоз

Хвороба Вільсона-Коновалова

крововилив

4712 / 6307
У вагітної жінки виявлені IgM до вірусу краснухи, на підставі чого акушер-гінеколог рекомендував перервати вагітність через високу ймовірність тератогенного впливу на плід. Важливим було виявлення саме Ig M, так як імуноглобуліни цього класу:

Мають найбільшу молекулярну вагу

Можуть долати плацентарний бар'єр

Є основним фактором противірусного захисту

Є показником свіжого зараження

Пов'язані з анафілактичними реакціями

4713 / 6307
У хворого, внаслідок тривалого хронічного захворювання головного мозку, виникли мимовільні рухи, порушився тонус м’язів тулуба. На порушення якого провідного шляху вказують ці симптоми?

Tractus spinothalamicus

Tractus corticonuclearis

Tractus tectospinalis

Tractus corticospinalis

Tractus rubrospinalis

4714 / 6307
У хворого, який знаходиться у клініці з ознаками отруєння ртуттю, у нирках відмічаються наступні процеси: вогнищеві некротичні зміни канальців головних відділів, набряк, лейкоцитарна інфільтрація та геморагії інтерстицію, венозний застій. Який стан розвився у хворого?

Гострий пієлонефрит

Хронічний пієлонефрит

Хронічна ниркова недостатність

Гострий некротичний нефроз

Гострий гломерулонефрит

4715 / 6307
У хворого висока температура, задуха, біль у правій частині грудної клітини. Плевральна пункція дала 700 мл в'язкої рідини жовто-зеленого кольору. Який патологічний процес розвився у плевральній порожнині?

Серозний плеврит

Емпієма плеври

Карциноматоз плеври

Бронхопневмонія

Геморрагичний плеврит

4716 / 6307
Онкологічному хворому призначили препарат метотрексат, до якого з часом клітини-мішені пухлини втратили чутливість. Експресія гену якого ферменту при цьому змінюється?

Тиміназа

Дезаміназа

Фолатоксидаза

Фолатдекарбоксилаза

Дегідрофолатредуктаза

4717 / 6307
Невпізнання хворим предметів при їх обмацуванні виникло після черепно-мозкової травми. Який відділ мозку ушкоджено?

Прецентральна звивина

Скронева частка

Постцентральна звивина

Мозочок

Потилична частка

4718 / 6307
При патологічних процесах, які супроводжуються гіпоксією, відбувається неповне відновлення молекули кисню в дихальному ланцюзі і накопичення пероксиду водню. Вкажіть фермент, який забезпечує його руйнування:

Сукцинатдегідрогеназа

Цитохромоксидаза

Аконітаза

Каталаза

Кетоглутаратдегідрогеназа

4719 / 6307
В анамнезі жінки зазначено три викидні, внаслідок четвертої вагітності народилася дитина з ураженням центральної нервової системи та очей, збільшенням лімфовузлів та селезінки. Відомо, що дома у жінки живуть дві кішки. Мікроскопічне дослідження мазків крові та пунктатів лімфовузлів виявило в клітинах тільця у формі півмісяця, у яких один кінець загострений і має утворення у вигляді присоски, а інший - заокруглений. Який паразит виявлений у жінки?

Plasmodium vivax

Trichomonas hominis

Toxoplasma gondii

Lamblia intestinalis

Balantidium coli

4720 / 6307
Жінка 45-ти років, перукар, скаржиться на болі у ногах, що з’являються після роботи, ввечері та вночі. При огляді хворої виявлено варикозне розширені вени на присередній поверхні гомілки та стегна. Яка вена та її безпосередні протоки розширені?

Стегнова

Глибока вена стегна

Мала підшкірна

Велика підшкірна

Передня великогомілкова вена

4721 / 6307
При обстежені пацієнта встановили сильний, врівноважений, інертний тип вищої нервової діяльності за Павловим. Якому темпераменту за Гіппократом відповідає пацієнт?

Сангвінік

Холерик

Меланхолік

Флегматик

4722 / 6307
У чоловіка 30-ти років перед операцією визначили групу крові. Кров резус-позитивна. Реакція аглютинації еритроцитів не відбулася зі стандартними сироватками груп 0 (І), А (II), В (III). Досліджувана кров належить до групи:

0 (І)

В (III)

А (II)

АВ (IV)

4723 / 6307
У підлітка 13-ти років під час проведення рентгенографічного дослідження кульшового суглоба виявлено зону просвітлення шириною 3 мм між голівкою та діафізом стегнової кістки. Як слід оцінити таку ситуацію?

Як вивих головки стегнової кістки

Як тріщину шийки стегнової кістки

Як перелом шийки стегнової кістки

Як артефакт на рентгенологічній плівці

Як норму (незавершений процес окостеніння)

4724 / 6307
У жінки з III (Б), Rh- групою крові народилась дитина з II (A) групою крові. У дитини діагностовано гемолітичну хворобу новонародженого внаслідок резус-конфлікту. Яка група крові за системою АБо та резус-належність можливі у батька?

I (O), Rh+

III (Б), Rh+

II (A), Rh+

I (O), Rh-

II (A), Rh-

4725 / 6307
Одним із факторів, що призводить до ожиріння, є пригнічення швидкості окислення жирних кислот внаслідок:

Надмірного вживання жирної їжі

Через недостатність синтезу фосфо-ліпідів

Через недостатність холіну

Зниження вмісту карнітину

Через недостатність вуглеводів у харчуванні

4726 / 6307
Хворий після перенесеного епідемічного паротиту схуднув, постійно відчуває спрагу, п'є багато води, відмічає часте сечовиділення, підвищений апетит, шкірний свербіж, слабкість, фурункульоз. У крові: глюкоза - 16 ммоль/л, кетонових тіл - 100 мкмоль/л; глюкозурія. Яке захворювання розвинулось у пацієнта?

Нецукровий діабет

Інсулінонезалежний цукровий діабет

Стероїдний діабет

Інсулінозалежний цукровий діабет

Цукровий діабет недостатнього харчування

4727 / 6307
Під час вивчення фаз мітотичного циклу корінця цибулі знайдено клітину, в якій хромосоми лежать в екваторіальній площині, утворюючи зірку. На якій стадії мітозу перебуває клітина?

Телофаза

Інтерфаза

Профаза

Метафаза

Анафаза

4728 / 6307
Після попередньої сенсибілізації експериментальній тварині підшкірно ввели дозу антигену. У місці ін'єкції розвинулось фібринозне запалення з альтерацією стінок судин основної речовини та волокнистих структур сполучної тканини у вигляді мукоїдного та фібриноїдного набухання і некрозу. Яка імунологічна реакція має місце?

Гранулематоз

Нормергічна реакція

Гіперчутливість сповільненого типу

Гіперчутливість негайного типу

Реакція трансплантаційного імунітету

4729 / 6307
У гістологічному препараті паренхіма органа представлена лімфоїдною тканиною, яка утворює лімфатичні вузлики; останні розташовуються дифузно і містять центральну артерію. Яке анатомічне утворення має таку морфологічну будову?

Лімфатичний вузол

Тимус

Мигдалик

Червоний кістковий мозок

Селезінка

4730 / 6307
У пацієнта виявлено різке розширення підшкірних вен в ділянці передньої черевної стінки навколо пупка. Підвищений тиск у якій судині сприяє цій симптоматиці?

V mesenterica inferior

V mesenterica superior

V portae hepatis

V cava superior

V cava inferior

4731 / 6307
У хворого виявлено зниження вмісту іонів магнію, які потрібні для прикріплення рибосом до гранулярної ендоплазматичної сітки. Відомо, що це призводить до порушення біосинтезу білка. Який саме етап біосинтезу білка буде порушено?

Термінація

Реплікація

Активація амінокислот

Трансляція

Транскрипція

4732 / 6307
До токсикологічного відділення госпіталізовано жінку з отруєнням невідомою речовиною. Яка група лікарських засобів може бути застосована для зменшення всмоктування і надходження отрути в організм?

Антихолінестеразні

Нейролептики

Антиоксиданти

Органічні нітрати

Адсорбенти

4733 / 6307
У хворого пухлиною пошкоджено піраміди довгастого мозку. У якому з провідних шляхів порушиться проведення нервових імпульсів?

Tr. corticopontinus

Tr. corticonuclearis

Tr. spinocerebellaris

Tr. dentatorubralis

Tr. corticospinalis

4734 / 6307
Під час оформлення дитини до школи для вирішення питання про необхідність ревакцинації поставлена проба Манту, яка виявилася негативною. Про що свідчить даний результат проби?

Про відсутність антитоксичного імунітету до туберкульозу

Про відсутність антитіл до туберкульозних бактерій

Про наявність антитіл до туберкульозних бактерій

Про наявність клітинного імунітету до туберкульозу

Про відсутність клітинного імунітету до туберкульозу

4735 / 6307
У хірургічне відділення лікарні був прийнятий хворий з підозрою на абсцес печінки. Хворий тривалий час знаходився у відрядженні в одній з африканських країн і неоднарозово хворів на гостре шлунково-кишкове захворювання. Яке протозойне захворювання може бути в хворого?

Малярія

Лейшманіоз

Амебіаз

Трипаносомоз

Токсоплазмоз

4736 / 6307
У пацієнта порушена функція нирок. Для перевірки стану фільтраційної здатності нирок йому призначено визначення кліренсу:

Глутаміну

!ндолу

Гідрокарбонату

Сечової кислоти

Креатиніну

4737 / 6307
Обстеження пацієнта з високим артеріальним тиском виявило у нього вторинну артеріальну гіпертензію. З’ясовано, що причиною такого стану пацієнта є гормонально активна пухлина кори наднирників. Гіперпродукція якого гормону є причиною вторинної артеріальної гіпертензії у хворого?

Глюкагон

Адреналін

Кортизол

Тироксин

Інсулін

4738 / 6307
Тривале лiкування гiпофункцiї щитоподiбної залози спричинило загальну дистрофiю, карiєс зубiв, тахiкардiю, тремор кiнцiвок. Який засiб викликав побiчнi ефекти?

Хумулiн

Паратиреоїдин

Преднiзолон

Тирокальцитонiн

L-тироксин

4739 / 6307
При обстеженні у алерголога хворому встановлений діагноз - поліноз. У який спосіб можна провести специфічну десенсибілізацію?

Ілюкокортикоїди

Атигістамінні препарати

Дробне введенням алергену

Введення фізіологічного розчину

4740 / 6307
У хірургічному відділенні перев'язувальні матеріали стерилізували в автоклаві. За недоглядом медсестри режим стерилізації був порушений і температура в автоклаві досягла 100оС замість належних 120оС. Які мікроорганізми можуть зберігати життєздатність у таких умовах?

Сальмонели та клебсієли

Бацили та клостридії

Плісняві та дріжджеподібні грибки

Стафілококи та стрептококи

Коринебактерії та мікобактерії

4741 / 6307
Жінці 26-ти років, хворій на бронхіт, призначили засіб етіотропної терапії антибіотик широкого спектру дії. Який це препарат?

Доксициклін

Дексаметазон

Амброксол

Інтерферон

БЦЖ-вакцина

4742 / 6307
У хворого на алкоголізм почався алкогольний психоз із вираженим психомоторним збудженням. Який препарат з групи нейролептиків слід призначити для швидкої допомоги?

Натрію бромід

Аміназин

Резерпін

Діазепам

Фторотан

4743 / 6307
У препараті яєчника поряд з фолікулами різного порядку виявляються атретичні тіла і розвинуте жовте тіло. Якій стадії оваріально-ментруального циклу відповідає такий стан у яєчнику?

Передменструальна

Постменструальна

Менструальна

Регенераторна

Росту фолікула

4744 / 6307
Пацієнта госпіталізовано з попреднім діагнозом «гепатит В». Для діагностики захворювання зачточовано серологічну реакцію, яка грунтується на взаємодії антигена з антитілом, хімічно зв’язаним з пероксидазою або лужною фосфатазою. Яку назву має використана серологічна реакція?

Реакція зв’язування комплементу

Реакція імунофлюоресценції

Радіоімунологічний метод

Реакція імобілізації

Імуноферментний аналіз

4745 / 6307
Чоловік 63 років, страждає раком стравоходу, метастази в лімфатичні вузли середостіння, ракова кахексія. Яка патогенетична стадія пухлинного процесу чоловіка?

ініціації

трансформації

промоції

прогресії

4746 / 6307
У чоловіка швидкість клубочкової фільтрації 180 мл/хв (норма -125±25 мл/хв).Причиною цього може бути зменшення:

Гідростатичного тиску крові в капілярах клубочків

Онкотичного тиску плазми крові

Проникності ниркового фільтру

Ниркового кровотоку

Ефективного фільтраційного тиску

4747 / 6307
У немовляти присутній мікроцефалія. Лікарі вважають, що це пов'язано із застосуванням жінкою під час вагітності актиноміцину D. На які зародкові листки подіяв цей тератоген?

Ектодерма

Мезодерма

Ентодерма

Ентодерми і мезодерма

Усі листки

4748 / 6307
Основна маса азоту з організму виводиться у вигляді сечовини. Зниження активності якого ферменту в печінці призводить до гальмування синтезу сечовини і нагромадження амоніаку в крові і тканинах?

Уреаза

Пепсин

Карбамоїлфосфатсинтаза

Амілаза

Аспартатамінотрансфераза

4749 / 6307
У хворого на артеріальну гіпертензію наслідком гіпертонічного кризу стала гостра серцева недостатність. Який механізм серцевої недостатності є головним в даному випадку?

Перевантаження серця опором

Перевантаження серця припливом крові

Абсолютна коронарна недостатність

Ушкодження міокарда

Відносна коронарна недостатність

4750 / 6307
14. При розтині померлого, який хворів на туберкульоз, у верхній частці правої легені знайдено порожнину розмірами 3x2 см, яка сполучається з бронхом. Стінка порожнини щільна, має три шари: внутрішній -піогенний, середній - шар туберкульозної грануляційної тканини, зовнішній - сполучнотканинний. Який діагноз найбільш імовірний?

Фіброзно-осередковий туберкульоз

Туберкульома

Гострий осередковий туберкульоз

Фіброзно-кавернозний туберкульоз

Гострий кавернозний тубеокульоз

4751 / 6307
У жінки 30-ти років з’явилися ознаки вірилізму (ріст волосся на тілі, облисіння скронь, порушення менструального циклу). Гіперпродукція якого гормону може спричинити такий стан?

Пролактин

Тестостерон

Релаксин

Естріол

Окситоцин

4752 / 6307
У хворого хлопчика 12-ти років вміст холестерину в сироватці крові до 25 ммоль/л. В анамнезі - спадкова сімейна гіперхолестеринемія, причиною якої є порушення синтезу білків-рецепторів до:

Ліпопротеїнів проміжної щільності

Хіломікронів

Ліпопротеїнів дуже низької щільності

Ліпопротеїнів високої щільності

Ліпопротеїнів низької щільності

4753 / 6307
Під дією негативних чинників довкілля порушена функція міосателітоцитів. Зміну якої функції всього м’язового волокна слід очікувати в даному випадку?

Регенерація

Скоротливий термогенез

Трофіка

Скорочення

4754 / 6307
Чоловік внаслідок ДТП отримав травму і втратив велику кількість крові. Які зміни периферичної крові будуть найбільш імовірні на 2-й день після травми?

Анізоцитоз

Еритропенія

Значний ретикулоцитоз

Гіпохромія

Пойкілоцитоз

4755 / 6307
Типовими проявами харчового отруєння, спричиненого С.ЬоІиІіпит є двоїння в очах, порушення ковтання та дихання. Ці симптоми розвиваються внаслідок:

Адгезії збудника до рецепторів на ентероцитах

Дії ентеротоксину

Активації аденілатциклази

Дії екзотоксину

Розвитку ентеротоксичного шоку

4756 / 6307
Підліток 15 років, скаржиться на недостачу повітря, загальну слабкість, серцебиття. ЧСС 130/хв, АТ-100/60 мм рт. ст. на ЕКГ, комплекс QRS нормальної форми та тривалості. Число зубців Р та шлункових комплексів однакове, зубець Т злитий з зубцем Р. Яка аритмія серця спостерігається у підлітка?

Тремтіння передсердь

Синусова тахікардія

Мерехтіння передсердь

Синусова екстрасистолія

Передсердна пароксизмальна тахікардія

4757 / 6307
У хворого після інфаркту міокарда передсердя і шлуночки стали скорочуватись незалежно одне від одного: з частотою 60-70 та 35-40 за хвилину. Назвіть вид блокади в даному випадку:

Внутрішньошлуночкова

Повна передсердно-шлуночкова

Неповна передсердно-шлуночкова

Внутрішньопередсердна

Сино-атріальна

4758 / 6307
Коензим А бере участь в багатьох важливих реакціях метаболізму. Похідним якого вітаміну він є?

Пантотенова кислота

Кальциферол

Тіамін

Убіхінон

Ніацин

4759 / 6307
Жінці 46-ти років планується провести об’ємне оперативне втручання в щелепно-лицьовій області. Відомо, що хвора схильна до підвищеної гемокоа-гуляції. Який природний антикоагулянт може бути застосований з метою запобігання тромбоутворенню?

Фібринолізин

Жодна з наведених речовин

Гепарин

Гірудин

Цитрат натрію

4760 / 6307
Хворий у непритомному стані доставлений бригадою швидкої допомоги у лікарню. Об'єктивно: рефлекси відсутні, періодично з’являються судоми, дихання нерівномірне. Після лабораторного обстеження було діагностовано печінкову кому. Нагромадження якого метаболіту є суттєвим для появи розладів центральної нервової системи?

Глутамін

Сечовина

Білірубін

Гістамін

Амоніак

4761 / 6307
Під час внутрішньовенної трансфузії фізіологічним розчином стан хворого різко погіршився і на фоні гострої ядухи настала смерть. При розтині померлого виявлено гостре венозне повнокров'я внутрішніх органів з різко розширеними правими відділами серця. При проколі правого шлуночка під водою виявляються пухирці. Який патологічний процес та його вид виник у хворого?

Тканинна емболія

Жирова емболія

Повітряна емболія

Тромбоемболія

ГІазова емболія

4762 / 6307
При первинному посіві води на 1% пе-птонну воду, через 6 годин на поверхні середовища виявлений ріст - ніжна плівка. Для збудника якого захворювання характерні такі культуральні властивості?

Чума

Дизентерія

Псевдотуберкульоз

Холера

Туберкульоз

4763 / 6307
Робітник одного із сільськогосподарських підприємств гостро занедужав і при наростаючих явищах інтоксикації помер. На розтині тіла: селезінка збільшена, в'яла, на розрізі - темно-вишневого кольору, дає надмірний зішкріб пульпи. М’які мозкові оболонки на склепінні та основі мозку набряклі, просякнуті кров'ю ('червоний чепець” або 'шапочка кардинала'). Мікроскопічно: серозно-геморагічне запалення оболонок і тканин мозку. Який найбільш імовірний діагноз?

Бруцельоз

Чума

Холера

Туляремія

Сибірка

4764 / 6307
В клініку поступила молода жінка зі скаргами на різкий біль внизу живота. При обстеженні у лікаря виникла підозра на розрив маткової труби в результаті позаматкової вагітності. Під час пункції через заднє склепіння піхви шприці виявили кров. В якому з утво-рень очеревини жінки накопичується кров в даному випадку?

Позадусліпокишковому поглибленні.

Прямокишково-матковому поглибленні.

Прямокишково-міхурному поглибленні.

Міхурно-матковому поглибленні.

Міжсигмовидному поглибленні.

4765 / 6307
Чоловік 42 років страждає ревматоїдним артритом До комплексу призначених йому лікувальних препаратів включений аспірин - інгібітор. З якої кислоти утворюються простагландини?

Арахідоново

Нейрамінової

Ліноленової

Пропіонової

Лінолевої

4766 / 6307
У вагітної жінки акушер-гінеколог вимірює розміри таза. За допомогою циркуля була виміряна відстань між двома клубовими гребенями. Який розмір таза був визначений?

Conjugata vera

Conjugata anatomica

Distantia cristarum

Distantia spinarum

Distantia throchanterica

4767 / 6307
Під час гістологічного дослідження стулок мітрального клапана серця жінки 30-ти років було встановлено, що ендотеліальні клітини вогнищево десквамовані, в цих ділянках на поверхні стулки розташовані дрібні тромботичні нашарування, сполучна тканина стулки з явищами мукоїдного набухання з ділянками склерозу та васкуляризації. Діагностуйте вид клапанного ендокардиту:

Дифузний

Поліпозно-виразковий

Гострий бородавчастий

Поворотньо-бородавчастий

Фібропластичний

4768 / 6307
До лікарні доставлено жінку з симптомами гострого апендициту, що супроводжується напруженням м'язів у правій здухвинній ділянці. Який тип вегетативних рефлексів забезпечує виникнення даного симптому?

Вісцеро-вісцеральні

Вісцеро-соматичні

Дермато-вісцеральні

Вісцеро-дермальні

Сомато-вісцеральні

4769 / 6307
Мембранний потенціал спокою клітини змінився з -85 до -90 мВ. Причиною цього може бути активація таких каналів мембрани клітини:

Калієві

Калієві та кальцієві

Калієві та натрієві

Натрієві

Кальцієві

4770 / 6307
До щелепно-лицьового відділення надійшов хворий з переломом нижньої щелепи. Було вирішено з'єднання кісток провести хірургічним методом під наркозом. Після внутрішньовенного введення міорелаксанту спостерігались короткочасні фібрилярні скорочення м'язів обличчя хворого. Який міорелаксант було застосовано?

Діазепам

Пипекуроній бромід

Дитилін

Тубокурарина хлорид

Меліктин

4771 / 6307
У жінки 35-ти років проведено біопсію вузлових утворень молочної залози. В ході гістологічного дослідження відзначається посилена проліферація епітелію дрібних протоків та ацинусів з формуванням залозистоподібних структур, різної форми та розмірів, які розташовані у фіброзній стромі. У ділянках залозистої проліферації ознак клітинної атипії не виявлено. Який найбільш імовірний діагноз?

Мастит

Інвазивна протокова карцинома

Кістозна мастопатія

Фіброаденома

Аденокарцинома

4772 / 6307
У неврологічному відділенні з приводу оперізуючого лишаю проходить курс лікування жінка 50-ти років. Внаслідок реактивації якого вірусу виникло це захворювання?

Вірусу кору

Вірусу простого ГЕРПЕСУ 1-го типу

Вірусу цитомегалії

Вірусу простого ГЕРПЕСУ 2-го типу

Вірусу Зостер (вітряної віспи)

4773 / 6307
У пацієнта 28-ми років, який знаходиться в пульмонологічному відділенні, діагностовано емфізему легенів, яка виникла внаслідок розщеплення міжальвеолярних перетинок тканинним трипсином. Уроджена недостатність якого білка спричинила розвиток захворювання?

Гаптоглобін

аі-протеїназний інгібітор

Трансферин

а2-макроглобулін

Кріоглобулін

4774 / 6307
Хворому на гострий інфаркт міокарда у комплексній терапії було призначено гепарин. Через деякий час після введення даного препарату з’явилася гематурія. Який антагоніст гепарину необхідно ввести хворому для усунення даного ускладнення?

Амінокапронова кислота

Неодикумарин

Протаміну сульфат

Фібриноген

Вікасол

4775 / 6307
У хворого, який проходить курс лікувального голодування, нормальний рівень глюкози у крові підтримується головним чином за рахунок глюконеогенезу. З якої амінокислоти при цьому у печінці людини найбільш активно синтезується глюкоза?

Глутамінова кислота

Аланін

Лізин

Лейцин

Валін

4776 / 6307
Хворому для зупинки кровотечі призначили препарат - прямий коагулянт. При введенні розчину пацієнт скаржився на біль по ходу вени, відчуття жару, серцебиття. Назвіть препарат, який викликає такі симптоми:

Стрептокіназа

Кальцію хлорид

Ергокальциферол

Пентоксил

Гірудин

4777 / 6307
Які рецептори слід заблокувати у людини перед проведенням бронхоскопії, щоб зменшити вплив блукаючого нерва на гладкі м’язи бронхів?

Н-холінорецептори

М-холінорецептори

а-адренорецептори

в-адренорецептори

а- і в-адренорецептори

4778 / 6307
У працівника хімчистки виявлена жирова дистрофія печінки. Порушення синтезу якої речовини може призвести до даної патології?

Тристеарин

Фосфатидилхолін

Холева кислота

Фосфатидна кислота

Сечовина

4779 / 6307
У хворого виявлені спленомегалія, геморагічний діатез. Периферична кров не змінена. При пункції груднини: кістковий мозок представлений тільки мієлобластами. Про яке захворювання йдеться?

Лімфоцитарний лейкоз

Мієлобластний лейкоз

Мієлоцитарний лейкоз

Лімфосаркома

Лімфобластний лейкоз

4780 / 6307
У людей, адаптованих до дії високої зовнішньої температури, посилене потовиділення не супроводжується втратою з потом великої кількості хлориду натрію. Дія якого гормону на потові залози спричинює цей результат?

Альдостерон

Тироксин

Вазопресин

Натрійуретичний

0% Кортизол

4781 / 6307
В відділення черепно-мозкової травми потрапив хворий з пошкодженням великого крила клиноподібної кістки. Лінія перелому пройшла через ОСТИСТИЙ отвір. Яка судина постраждала?

Латеральна крилоподібна артерія

Середня оболонна артерія

Поверхнева скронева артерія

Задня глибока скронева артерія

Передня глибока скронева артерія

4782 / 6307
Хворому для зупинки кровотечі призначили препарат - прямий коагулянт. При введенні розчину пацієнт скаржився на біль по ходу вени, відчуття жару, серцебиття. Назвіть препарат, який викликає такі симптоми:

Ергокальциферол

Гірудин

Кальцію хлорид

Пентоксил

Стрептокіназа

4783 / 6307
Жінці 23-х років в комплексному лікування ксерофтальмії лікар призначив ретинолу ацетат, але, дізнавшись, що пацієнтка знаходиться на 8-му тижні вагітності, відмінив зазначений лікарський засіб. Яка можлива дія віта-мінопрепарату спонукала лікаря переглянути призначення?

Утеротонічна

Тератогенна

Токсична

Канцерогенна

Мутагенна

4784 / 6307
Надмірна секреція певного гормону спостерігалася у хворого при феохромоцитомі - пухлині, що походить з мозкової речовини надниркових залоз. Як він називається?

Інсулін

Тироксин

Глюкагон

Соматотропін

Адреналін

4785 / 6307
У пацієнта 36-ти років після дорожньої травми виникли параліч м’язів кінцівок справа, втрата больової і температурної чутливості зліва, часткове зниження тактильної чутливості з обох сторін. Для ураження якого відділу мозку вказані зміни є найбільш характерними?

Ліва половина спинного мозку

Рухова кора зліва

Передні стовпи спинного мозку

Задні стовпи спинного мозку

Права половина спинного мозку

4786 / 6307
Після ін’єкції інсуліну рівень глюкози крові знижується протягом декількох секунд. Це відбувається завдяки активації такого процесу:

Синтез ліпідів

Транспорт глюкози до клітин

Пентозофосфатний цикл

Гліколіз

Синтез глікогену

4787 / 6307
В ході біохімічного аналізу еритроцитів немовляти встановлено виражену недостатність глутатіонпероксидази і низький рівень відновленого глутатіону. Який патологічний стан може розвинутися у цієї дитини?

Серпоподібно-клітинна анемія

Гемолітична анемія

Мегалобластна анемія

Перніціозна анемія

Залізодефіцитна анемія

4788 / 6307
Жінка 30-ти років хворіє близько року, коли вперше з'явився біль у ділянці суглобів, їх припухлість, почервоніння шкіри над ними. Попередній діагноз - ревматоїдний артрит. Зміна якого компоненту в структурі білка сполучної тканини є однією з причин цього захворювання?

Міозин

Колаген

Муцин

Тропонін

Овоальбумін

4789 / 6307
При розтині померлого, який хворів на туберкульоз, у верхній частці правої легені знайдено порожнину розмірами 3x2 см, яка сполучається з бронхом. Стінка порожнини щільна, має три шари: внутрішній -піогенний, середній - шар туберкульозної грануляційної тканини, зовнішній - сполучнотканинний. Який діагноз найбільш імовірний?

Фіброзно-кавернозний туберкульоз

Гострий кавернозний тубеокульоз

Туберкульома

Фіброзно-осередковий туберкульоз

Гострий осередковий туберкульоз

4790 / 6307
Під час розтину тіла померлого в черевній порожнині виявлено близько 2,0 л гнійної рідини. Очеревина тьмяна, з сіруватим відтінком, на серозній оболонці кишок є сіруватого кольору нашарування, що легко знімаються. Найімовірніше у хворого був такий перитоніт:

Туберкульозний

Геморагічний

Фібринозно-гнійний

Серозний

4791 / 6307
У пацієнта на ЕКГ спостерігається значне розширення зубця R (до 0,18 с). Це зумовлено зменшенням швидкості проведення збудження такими структурами серця:

Лівий шлуночок

Шлуночки

Передсердя

Атріо-вентрикулярний вузол

Правий шлуночок

4792 / 6307
A patient has peptic ulcer of the stomach. What medicine can decrease the secretion of hydrochloric acid and pepsin by blocking the H2 receptors?

Physostigmine

Phthalazol (Phthalylsulfathiazole)

Famotidine

Fluvoxamine

Phenobarbital

4793 / 6307
Roentgenological examination of skull base bones revealed enlargement of sellar cavity, thinning of anterior clinoid processes, destruction of different parts, destruction of different parts of sella turcica. Such bone destruction might be caused by a tumour of the following wndocrinous gland:

Epiphysis

Thyroid gland

Adrenal glands

Hypophysis

Thymus gland

4794 / 6307
A veterinarian, working at cattle farm, visited doctor and complained of pain in joints, fever, weakness, night sweats. He has been ill for a month. According to complaints and patient profession doctor suspected brucellosis. Which material collected from this patient should be exameneed in ordinary bacteriological laboratory?

Cerebrospinal fluid

Urine

Vomiting mass

Blood serum

Feces

4795 / 6307
A bone marrow biopsy of an 8-year old girl shows a group of cells which has undergone the process ofpyknosis and loss of the nucleus dunng iis differentiation. Which of the following types of hemopoiesis is characterised by the morphological changes described above?

Erytropoesis

Monocytopoesis

Granulocytopoesis

Lymphocytopoesis

Thrombocylopoesis

4796 / 6307
A 46-year-old man presents with fatigue and joint pain in his fingers and wrists for the last 2 months. The pain is present in both hands and the wrists are swollen. Furthermore, he describes morning stiffness in his joints lasting about 2 hours, which improves with use. His past medical history reveals he has been successfully treated for II. pylori related ulcers last year. He denies smoking and stopped drinking when his gastric symptoms started. Which of the following drugs is the best choice for his joint pain management?

Aspirin

Paracetamol

Morphine

Prednisone

Celecoxib

4797 / 6307
A 16-year-old girl concerned about her sexual development comes to the physician. She mentions that she has still not had a menstrual period. However, she is otherwise a healthy girl with no significant medical problems since birth. On physical examination, her vital signs are stable. She does not have pubic hair and her breast is slightly elevated with areola remaining in contour with surrounding breast. Which of the following is the most likely cause of this abnormal physical development?

Adrenal medulla hyperfunction

Hypothyroidism

Pancreatic islet insufficiency

Hyperthyroidism

Ovarian insufficiency

4798 / 6307
An 18-year-old girl comes to her physician with concern about her health because she has not achieved menarche. She denies any significant weight loss, changes in mood, or changes in her appetite. She mentions that her mother told her about mild birth defects, but she cannot recall the specifics. Past medical history and familyhistory are benign. On physical examination, the patient is short in stature, has a short and webbed neck and wide chest. Staining of buccal smear reveals absence of Barr bodies in the nucleus of epithelial cells. A urine pregnancy test is negative. Which of the following genetic disorders is the most likely cause of this patient's condition?

Patau syndrome

Cri du chat ('cat-cry') syndrome

Turner syndrome

Edwards syndrome

Klinefelter syndrome

4799 / 6307
A 14-year old girl presents to the emergency department for evaluation of an 'infected lell'. She states there is no history of trauma but mentions she had a history of sickle cell disease. On physical examination, her upper part of right shin is very painful, red. swollen and hot. Her temperature is 39.2°C. An X-ray shows focal bony lysis and loss of trabecular architecture in the metaphysis of right tibia. Increased activity of which of the following cells is the most likely cause of bone reabsorption in this patient?

Osteoblasts

Chondrocytes

Osteoclasts

Osteocytes

Chondroblasts

4800 / 6307
A 24-year-old man undergoes surgery and during the operation, an organ is excised and sent for histological evaluation. A light microscopic examination reveals the organ encased by thin connective tissue capsule that enters the substance of the lobes to further subdivide the organ into irregular lobular units. Each lobule contains a cluster of follicles filled with colloid. Follicular epithelium consists of low columnar, cuboidal or squamous cells depending on the level of activity of the follicle. Which of the following organs does this tissue most likely belong to?

Parotid gland

Parathyroid gland

Thyroid gland

Pancreas

Thymus

4801 / 6307
A 6-year-old boy is brought to the pediatrician by his mother, who complains of low-grade fever, chronic cough and night sweats in her child. She describes the cough as productive, producing white sputum that is sometimes streaked with blood. She also says that her son has lost some weight in the last month. His vital signs include blood pressure of 115/75 mm Hg, heart rate of 110/min., respiratory rate of 18/min. and temperature of 36.6oC. On physical examination, the patient is ill looking. Pulmonary auscultation reveals some fine crackles in the right upper lobe. The pediatrician suspects an active infection and performs Mantoux test. Intradermal injection of which of the following substances has been most likely used by pediatrician for screening test in this clinical case?

Tuberculin

Bacillus Calmette-Guerin (BCG) vaccine

Diphtheria-tetanus toxoids-acellular pertussis vaccine (DTaP)

Tetanus and diphtheria toxoids vaccine(Td)

4802 / 6307
A team of medical students is performing research on phases of cell cycle. During one of the mitotic phases the cell is nearly done dividing, the chromosomes décondensé and two nuclei begin to form around them. Which of the following phases most likely takes place in the cell?

Anaphase

Metaphase

Prophase

Telophase

4803 / 6307
A 60-year old man with a history of hypertension, diabetes and hyperlipidemia had a sudden onset of right-sided weakness. By the time the ambulance arrived, he had difficulty speaking. Unfortunately, the patient died within the next 2 hours and an autopsy was performed immediately. The gross examination of the cerebral left hemisphere showed brain swelling, widened gyri and poorly demarcated gray-white junction. Which of the following is the most likely cause of this patient's death?

Abscess

Intracerebral hemorrhage

Thmor

Ischemic stroke

Cyst

4804 / 6307
A 38-year-old woman, who was diagnosed with systemic lupus erythematosus (SLE) 3 years ago, comes to her physician with a complaint of facial swelling and decreased urination that she first noticed 2 weeks ago. She currently takes azathioprine and corticosteroid. Her vital signs show blood pressure 150/90 mm Hg, pulse – 91/min., temperature – 36.8 C and respiratory rate – 15/min. On physical examination, the doctor notices erythematous rash on her face exhibiting a butterfly pattern. The laboratory studies reveal hypertriglyceridemia and proteinuria. Which of the following is the most likely mechanism of SLE`s complication in this patient?

Acute infection ol-the kidney

Immune complex-mediated glomerular disease

Decrease in renal blood flow (ischemic nephropathy)

Increased plasma oncotic pressure

4805 / 6307
A 20-year-old female comes to the clinic after missing her last 2 periods. Her cycles are usually regular, occurring at 28-30 day interval with moderate bleeding and some abdominal discomfort. She also complains of progressively diminishing peripheral vision. Her doctor reveals loss of vision in the lateral halves of both eyes. Involvement of which of the following structures would you most likely expect to be the reason of bitemporal hemianopsia?

Right optic tract

Left optic nerve

Left optic tract

Optic chiasm

Right optic nerve

4806 / 6307
A 24-year-old man undergoes surgery and during the operation, an organ is excised and sent for histological evaluation. A light microscopic examination roved s Ihe organ encased by thin connective tissue capsule that enters the substance oi the lobes to further subdivide the organ into irregular lobular units. Bach lobule contains a cluster of follicles filled with colloid. Follicular epithelium consists of low columnar, cuboidal or squamous cells depending on the level of activity ol the follicle. Which of the following organs does this tissue most likely belong to?

Parathyroid gland

Thymus

Pancreas

Parotid gland

Thyroid gland

4807 / 6307
A 37-year-old man is admitted to a hospital with mental confusion and disorientation. His wife reports he became more irritable and forgetful in the past year. In addition, she notes that he became a vegan a year ago, and currently, his diet consists of starchy foods like potatoes, corn, and leafy vegetables. Gl symptoms include anorexia, diarrhea and vomiting. He has glossitis and skin lesions that appear as vesicles over the extremities. Eczema-like lesions around the mouth, as well as desquamation and roughened skin over the hands, are also present. Neurologic examination reveals symmetrical hypesthesia for all types of sensation in both upper and lower extremities in a 'gloves and socks' distribution. Deficiency in the diet the of which of the following amino acids is the most likely cause of this condition?

Arginine

Lysine

Histidine

Threonine

Tryptophan

4808 / 6307
Під час визначення основного обміну у пацієнта виявлено, що його величина менша за належну на 7%. Яка інтенсивність процесів енергетичного метаболізму у пацієнта?

Суттєво підвищена

Помірно підвищена

Нормальна

Помірно знижена

Суттєво знижена

4809 / 6307
Під час розтину тіла померлого чоловіка з діагнозом: гіпертонічна хвороба, у лівій гемісфері мозку виявлено порожнину округлої форми 4х5 см із іржавою стінкою, яка заповнена жовтуватою прозорою рідиною. Який патологічний процес розвинувся у головному мозку чоловіка?

Абсцес

Гематома

Кіста

Ішемічний інфаркт

Геморагічний інфаркт

4810 / 6307
Унаслідок тривалого голодування в організмі людини швидко зникають резерви вуглеводів. Який метаболічний процес поновлює вміст глюкози в кровi?

Пентозофосфатний шлях

Глюконеогенез

Анаеробний гліколіз

Аеробний гліколіз

Глікогеноліз

4811 / 6307
У людини визначили величину енерговитрат. У якому стані перебуває людина, якщо її енерговитрати виявилися менші за основний обмін?

Відчуває нервове напруження

Спить

Перебуває в стані спокою

Виконує легку роботу

Відпочиває

4812 / 6307
Пацієнту призначили антибіотик левоміцитин (хлорамфенікол), який пригнічує синтез білка мікроорганізмів шляхом гальмування процесу:

Процесингу

Елонгації трансляції

Ампліфікації генів

Утворення полірибосом

Транскрипції

4813 / 6307
Пацієнтка віком 37 років померла під час нападу експіраторної задухи, що був спричинений контактом із екзогенним алергеном (пилок амброзії). Під час гістологічного дослідження в просвіті бронхів спостерігаються скупчення слизу, у стінці бронхів багато тучних клітин (лаброцитів), більшість із яких у стані дегрануляції, багато еозинофілів. До патогенезу якого типу реакцій гіперчутливості можна віднести описані зміни?

І типу (анафілактична)

ІІІ типу (імунокомплексна)

IV типу (клітинна цитотоксичність)

II типу (антитілозалежна)

V типу (гранулематоз)

4814 / 6307
До інфекційного відділення госпіталізовано семирічну дитину госпіталізовано, яка скаржиться на різкий біль у горлі під час ковтання. Об'єктивно спостерігається: підвищення температури тіла до 39°С, набряк шиї, мигдалики збільшені, їх слизова оболонка гіперемічна, вкрита великою кількістю плівок білувато-жовтого кольору, які щільно прилягають до слизової оболонки. Під час спроби зняти плівку залишається глибокий дефект тканини, який кровоточить. Який вид запалення у дитини?

=Дифтеритичне

Крупозне

Серозне

Гнійне

Геморагічне

4815 / 6307
Важливою складовою частиною ниркового фільтраційного бар'єру є тришарова базальна мембрана, середній електронно-щільний шар якої має сітчасту будову. Де розташована ця базальна мембрана?

У проксимальних канальцях

У дистальних звивистих канальцях

У нирковому тільці

У тонких канальцях

У дистальних прямих канальцях

4816 / 6307
Одна з форм вродженої патології супроводжується гальмуванням перетворення фенілаланіну в тирозин. Біохімічною ознакою цього захворювання є накопичення в організмі деяких органічних кислот. Укажіть кислоту, яка накопичуватиметься в організмі у цьому разі.

Глутамінова

Лимонна

Молочна

Фенілпіровиноградна

Піровиноградна

4817 / 6307
Реакції міжмолекулярного транспорту одновуглецевих радикалів є обов'якзовими для синтезу білків та нуклеїнових кислот. Коферментна форма якого вітаміну є переносником одновуглецевих фрагментів у цьому синтезі?

Аскорбінової кислоти

Рибофлавіну

Пантотенової кислоти

Фолієвої кислоти

Tiaмiнy

4818 / 6307
У пацієнта з діагнозом: гемолітична анемія, виявлено дефіцит піруваткінази в еритроцитах. Яка причина розвитку гемолізу еритроцитів за цих умов?

Генетичні дефекти глікофорину А

Зменшення активності Na+, К+-АТФ-ази

Надлишок к в еритроцитах

Дефіцит спектрину

Нестача Na в еритроцитах

4819 / 6307
Симптоми захворювання виявилися у пацієнта за 24 години після вживання рибних консервів. Він відчуває нудоту, слабкість, сухість у роті, двоїння в очах, афагію, утруднення дихання. Що зумовлює таку симптоматику при ботулізмі?

Вплив нейротоксину

Вплив ендотоксину

Активація цАМФ

Розмноження збудника у ШКТ

Вплив ентеротоксину

4820 / 6307
На гістологічному препараті легень видно структуру діаметром близько 0,5 мм, слизова оболонка якої вкрита одношаровим кубічним війковим епітелієм, у якому зустрічаються секреторні клітини Клара, війчасті клітини, мікроворсинчасті. Вкажіть структуру:

Альвеола

Малий бронх

Середній бронх

Термінальна бронхіола

Альвеолярний хід

4821 / 6307
Від виснажливої силової роботи у робітника значно зменшилася буферна ємність крові. Накопичення у крові якого метаболіту може спричинити зміну цього показника?

Альфа-кетоглутарату

1,3-бісфосфогліцерату

3-фосфогліцерату

Пірувату

Лактату

4822 / 6307
До ендокринолога звернулася пацієнтка віком 45 років, у якої спостерігаються: підвищення апетиту, сухість слизових оболонок ротової порожнини, збільшення діурезу. Під час обстеження вперше виявлено інсулінозалежний діабет. Який лікарський засіб треба призначити жінці?

Інсулін

Окситоцин

Глібенкламід

Вазопресин

Адіурекрин

4823 / 6307
Під час обстеження жінки віком 56 років з діагнозом: цукровий діабет 1-го типу, виявлено порушення білкового обміну, що під час лабораторного дослідження крові проявляється аміноацидемією, зменшенням синтезу антитіл, а клінічно уповільненням загоєння ран. Який механізм розвитку аміноацидемії?

Підвищення протеолізу

Збільшення ліпопротеїдів висо

Зменшення концентрації амінокислот у крові

ої щільності Підвищення онкотичного тиску в плазмі крові

Гіперпротеїнемія

4824 / 6307
Під час оперативного втручання на головному мозку виявлено, що подразнення певних зон кори великих півкуль спричинило у пацієнта тактильні та температурні відчуття. На яку звивину діяли подразники?

Поясну

Верхню латеральну

Постцентральну

Прецентральну

Парагіпокампову

4825 / 6307
У пацієнта, який тривалий час приймав глюкокортикоїди, унаслідок відміни препарату виникло загострення існую чого захворювання, спостерігається зниження артеріального тиску та слабкість. З розвитком якого патологічного стану можна пов'язати ці явища?

Недостатністю наднирників

Сенсибілізацією

Гіперпродукцію АКТГ

Звиканням до лікарського засобу

Кумуляцією

4826 / 6307
У чоловіка віком 25 років унаслідок перелому основи черепа виділяється велика кількість сечі з низькою відносною щільністю. Порушення синтезу якого гормону є причиною змін у регулюванні процесу сечоутворення? Соматотропного гормону

Адренокортикотропого гормону

Окситоцину

Вазопресину

Тиреотропного гормону

4827 / 6307
Як називається об'єм повітря, що залишається в легенях після спокійного видиху?

Життєва ємність легень

Залишковий об'єм

Функціональна залишкова ємність легень

Дихальний об'єм

Резервний об'єм видиху

4828 / 6307
Під час мікроскопічного дослідження виявлено паренхіматозний орган, в якому епітеліальні тяжі формують клубочкову, пучкову та сітчасту зони. Центральна частина органа представлена скупченнями хромафінних клітин. Визначте цей орган.

Надниркова залоза

Eпіфіз

Щитоподібна залоза

Печінка

Гіпофіз

4829 / 6307
До лікаря звернулася жінка віком 32 роки зі скаргами на відсутність у неї лактації після народження дитини. Дефіцит якого гормону найвірогідніше спричинив це порушення?

Вазопресину

Соматотропіну

Тиреокальцитоніну

Глюкагону

Пролактину

4830 / 6307
У клітині, у якій відбувається мітотичний поділ, спостерігається розходження дочірніх хроматид до полюсів клітини. Визначте стадію мітозу.

Анафаза

Метафаза

Інтерфаза

Профаза

Телофаза

4831 / 6307
Прокаріотичні та еукаріотичні клітини характеризуються здатністю до поділу. Поділ прокаріотичних клітин відрізняється від поділу еукаріотичних, але існує молекулярний процес, що лежить в основі обох поділів. Назвіть цей процес.

Транскрипція

Трансляція

Ампліфікація генів

Реплікація ДНК

Репарація

4832 / 6307
Для забезпечення анальгезії наркоємний анальгетик застосували разом з препаратом бензодіазепінового ряду. Який лікарський засіб треба використати для потенціювання анальгезії?

Карбамазепін

Імізин

Хлорпротиксен

Трифтазин

Діазепам

4833 / 6307
У жінки, яка тривалий час приймала антибіотики, розвинулось ускладнення на слизовій оболонці порожнини рота: запальний процес з білим нальотом. Під час бактеріологічного дослідження нальоту виявлено дріжджеподібні грибки Candida albicans. Який лікарський засіб треба використати для лікування цього ускладнення?

Флуконазол

Бісептол

Поліміксин

Фуразолідон

Тетрациклін

4834 / 6307
У чоловіка спостерігається пухлина черевної порожнини, що здавлює нижню порожнисту вену. Який кава-кавальний анастомоз на передній стінці живота забезпечить відтік венозної крові?

Між верхніми і нижніми прямокишковими венами

Між верхньою і нижньою надчеревними венами

Між пупковою і верхньою надчеевною венами

Між непарною і напівнепарною венами

Між хребетними і пупковими венами

4835 / 6307
Ліквідатору наслідків аварії на Чорнобильській АЕС, що отримав велику дозу опромінення, проведено трансплантацію кісткового мозку. Через деякий час після проведеної операції у пацієнта діагностовано розвиток реакції `трансплантат проти хазяїна`. Які антигени стали пусковим механізмом виникнення цієї реакції?

Антигенами системи AB0 еритроцитів ліквідатора

Антигени системи Rh еритроцитів ліквідатора

Антигени системи HLA-клітин організму ліквідатора

Антигени системи HLA-клітин організму донора

Антигени HBs, HBc, Hbe

4836 / 6307
У пацієнта з діагнозом: хронічний дифузний гломерулонефрит, розвинулася хронічна ниркова недостатність. На термінальній стадії ХНН розвивається оліго- та анурія, що спричиняється:

Зниженням кількості функціонуючих нефронів

Ішемією коркової речовини нирок унаслідок спазму судин

Дисемінованим внутрішньосудинним зсіданням крові

Збільшенням реабсорбції води в дистальних канальцях

Зменшенням фільтраційного тиску та фільтрації

4837 / 6307
Тривале перебування в умовах спеки викликало у людини спрагу. Збудження яких рецепторів, перш за все, сприяло розвитку такої реакції?

Механорецепторів вестибулярного апарата

Барорецепторів дуги аорти

Осморецепторів гіпоталамусу

Глюкорецепторів гіпоталамусу

Хеморецепторів дуги аорти

4838 / 6307
Цикл Кребса виконує важливу роль у реалізації глюкопластичного ефекту амінокислот. Це зумовлено обов'язковим перетворенням безазотистого залишку амінокислот у:

Сукцинат

Цитрат

Оксалоацетат

Малат

Фумарат

4839 / 6307
Пацієнту з діагнозом: гострий інфаркт міокарда, у комплексній терапії призначено гепарин. За деякий час після введення лікарського засобу у пацієнта з'явилася гематурія. Який антагоніст гепарину треба ввести пацієнту для усунення цього ускладнення?

Фібриноген

Вікасол

Протаміну сульфат

Неодикумарин

Амінокапронову кислоту

4840 / 6307
Щоденно в організмі людини 0,5% Усього гемоглобіну перетворюється на метгемоглобін. Який фермент, що місти ться в еритроцитах, каталізує реакцію відновлення метгемоглобіну до гемоглобіну?

Білівердинредуктаза

Метгемоглобінредуктаза

Гемоксигеназа

Глюкуронілтрансфераза

Метгемоглобінтрансфераза

4841 / 6307
У хворого 20-ти років з жовтяницею встановлено: підвищення у плазмі крові вмісту загального білірубіну за рахунок непрямого (вільного), в калі та сечі - високий вміст стеркобіліну, рівень прямого (зв’язаного) білірубіну в плазмі крові в межах норми. Про який вид жовтяниці можна думати?

Гемолітична

Паренхіматозна (печінкова)

Хвороба Жильбера

Механічна

Жовтяниця немовлят

4842 / 6307
У п'ятимісячної дівчинки спостерігаються застійні явища у легенях. Під час обстеження виявлено зв'язок між висхідною аортою та легеневою артерією, що в нормі спостерігається у деяких земноводних і плазунів. Назвіть цю вроджену ваду розвитку:

Транспозиція магістральних судин

Дефект міжпередсердної перегородки

Незарощення боталової протоки

Дефект міжшлуночкової перегородки

Коарктація аорти

4843 / 6307
Уведення тварині екстракту тканини передсердя посилює виділення натрію із сечею. Дія якої біологічно активної речовини є причиною такого стану?

Адреналіну

Калікреїну

Натрійуретичного гормону

Глюкокортикоїдів

Серотоніну

4844 / 6307
Під час бактеріологічного дослідження калу та блювотних у мас пацієнта виявлено культуру холерного вібріону. Проведення якої реакції дасть змогу визначити вид мікроба, що спричинив це захворювання?

Пасивної гемаглютинації з еритроцитарним антигенним діагностикумом

Преципітації

Аглютинації з сироватками, що містять О-антитіла

Аглютинації з сироватками, що містять Н-антитіла

Аглютинації Відаля

4845 / 6307
Чоловік віком 25 років вживає надмірну кількість вуглеводів (600 г на добу), що перевищує його енергетичні потреби. Який метаболічний процес буде активуватися в організмі чоловіка у цьому разі?

Ліпогенез

Ліполіз

Окиснення жирних кислот

Гліколіз

Глюконеогенез

4846 / 6307
У фібробластах шкіри дитини з діагнозом хвороба Дауна, виявлено 47 хромосом. Визначте тип аномалії.

Трисомія 21

Трисомія Х

Трисомія 13

Полісомія Ү

Трисомія 18

4847 / 6307
Під час гістологічного дослідження стулок мітрального клапана серця жінки віком 30 років встановлено, що ендотеліальні клітини вогнищево десквамовані, у цих ділянках на поверхні стулки розташовані дрібні тромботичні нашарування, сполучна тканина стулки з явищами мукоїдного набухання з ділянками склерозу та васкуляризації. Який ймовірно вид клапанного ендокардиту в пацієнтки?

Фібропластичний

Поліпозно-виразковий

Поворотньо-бородавчастий

Дифузний

Гострий бородавчастий

4848 / 6307
Безпосередніми «виконавцями» апоптозу в клітині є особливі ферменти каспази. В утворенні одного з них бере участь цитохром С. Яка його функція в нормальній клітині?

Фермент дихального ланцюга переносу електронів

Фермент ЦТК

Компонент Н+ -АТФ-азної системи

Компонент піруватдегідрогеназної системи

Фермент бета-окислювання жирних кислот

4849 / 6307
Пацієнт віком 65 років тривалий час скаржиться на симптоми, характерні для хронічного гастриту. Під час лабораторного аналізу в крові виявлено мегалоцити, у кістковому мозку виявлений мегалобластичний еритропоез. Який діагноз є найімовірнішим у цьому разі?

Гіпопластична анемія

Залізодефіцитна анемія

Гемолітична анемія

Апластична анемія

В12-фолієводефіцитна анемія

4850 / 6307
До лікарні госпіталізовано потерпілого під час ДТП з артеріальним тиском 70/40 мм рт. ст. Пацієнт без свідомості. Добовий діурез близько 550 мл. Яке порушення функції нирок розвинулося у чоловіка?

Гостра ниркова недостатність

Пієлонефрит

Тубулопатія

Гострий дифузний гломерулонефрит

Хронічна ниркова недостатність

4851 / 6307
На зовнішню поверхню мембрани збудливої тканини діють електричним імпульсом катодного напрямку, амплітуда якого дорівнює 70% порогу. Які зміни мембранного потенціалу клітин це зумовить?

Часткову деполяризацію

Змін не буде

Гіперполяризацію

Потенціал дії

4852 / 6307
Хірург під час операції на щитоподібній залозі перев'язав верхню щитоподібну артерію. Гілку якої судини перев'язав лікар?

A. pharyngea ascendens

A. lingualis

A. carotis interna

A carotis externa

A. facialis

4853 / 6307
Хворій жінці із захворюванням нирок, що супроводжується вираженими набряками, призначили діуретичний препарат, що пригнічує реабсорбцію в нирках іонів Na+ і води, посилює виведення нирками іонів K+ і Mg++, викликає гіперурикемію, зумовлює потужний діуретичний ефект. Назвіть цей препарат:

Тріамтерен

Аллопуринол

Спіронолактон

Фуросемід

Діакарб

4854 / 6307
У пацієнта із синдромом Іценко-Кушинга спостерігаються стійка гіперглікемія та глюкозурія. Синтез та секреція якого гормону збільшені в організмі пацієнта?

Адреналіну

Кортизолу

Альдостерону

Тироксину

Глюкагону

4855 / 6307
У пацієнта спостерігається ожиріння, гірсутизм, 'місяцеподібне обличчя', рубці багряного кольору на шкірі стегон. Артеріальний тиск 180/110 мм рт.ст., глюкоза крові 17,2 ммоль/л. Під час якої зміни продукції гормонів наднирників можливі такі симптоми?

Гіпопродукції глюкокортикоїдів

Гіпопродукції адреналіну

Гіперпродукції глюкокортикоїдів

Гіпопродукції мінералокортикоїдів

Гіперпродукції мінералокортикоїдів

4856 / 6307
У пацієнта похилого віку спостерігається хронічний закреп через гіпотонію товстого кишечника. Який лікарський засіб треба йому призначити?

Ацеклідин

Прозерин

Касторову олію

Натрію сульфат

Бісакодил

4857 / 6307
При розтині чоловіка, померлого від опікової хвороби, знайдено набряк головного мозку, збільшення печінки, а також нирок, кірковий шар яких широкий, блідо-сірий, мозковий – повнокровний. Мікроскопічно: некроз канальців головних відділів з деструкцією базальних мембран, набряк інтерстицію з лейкоцитарною інфільтрацією та крововиливами. Який з перелічених діагнозів найбільш вірогідний?

Мієломна нирка

Тубулоінтерстиціальний нефрит

Пієлонефрит

Подагрична нирка

Некротичний нефроз

4858 / 6307
В експерименті на спинному мозку під час збудження альфа-мотонейронів м'язів-згиначів спостерігається гальмування альфа-мотонейронів м'язів-розгиначів. Який вид гальмування є причиною цього явища?

Зворотне

Реципрокне

Пресинаптичне

Деполяризаційне

Латеральне

4859 / 6307
Пацієнту встановлено діагноз: аскаридоз. Який лікарський засіб треба йому призначити?

Фуразолідон

Піперазин

Мебендазол

Фенасал

Гентаміцин

4860 / 6307
У доношеної новонародженої дитини спостерігається жовте забарвлення шкіри та слизових оболонок. Тимчасова недостатність якого ферменту може бути імовірною причиною такого стану дитини?

УДФ-глюкуронілтрансферази

Гемоксигенази

Гемсинтетази

Білівердинредуктази

Уридилтрансферази

4861 / 6307
У тварини за 2 тижні після експериментального моделювання стенозу ниркової артерії спостерігається підвищення артеріального тиску. З посиленням дії якого чинника гуморальної регуляції на судини це пов'язано?

Альдостерону

Дофаміну

Вазопресину

Кортизолу

Ангіотензину II

4862 / 6307
Пацієнт з діагнозом: ішемічна хвороба серця, не повідомив сімейному лікарю, що в нього бувають напади бронхоспазму. Лікар призначив засіб, після прийому якого напади стенокардії трапляються рідше, але напади бронхоспазму почастішали. Який ймовірно лікарський засіб було призначено?

Верапаміл

Нітросорбід

Анаприлін

Дилтіазем

Нітрогліцерин

4863 / 6307
У новонародженої дитини отвір крайньої плоті статевого члена за величою не перевищує діаметр сечовивідного каналу і перешкоджає вільному виходу голівки статевого члена. Як називається цей патологічний стан у дитини?

Епіспадія

Гіпоспадія

Парафімоз

Гермафродитизм

Фімоз

4864 / 6307
У чоловіка віком 43 роки, який помер у хірургічному відділенні від дифузного гнійного перитоніту, під час розтину тіла виявили пейєрові бляшки у дистальному відділі стінки тонкого кишечника. Поверхня деяких із них укрита струпом коричнево-зеленуватого кольору, у центрі деяких бляшок наявні глибокі дефекти, що досягають серозного шару. Для якого захворювання характерні наведені зміни у тонкому кишечнику чоловіка?

Черевний тиф

Дизентерія

Туберкульоз кишечника

Стафілококовий ентерит

Хвороба Крона

4865 / 6307
Швидкість проведення збудження нервовими волокнами становить 120 м/с. Який з наведених чинників, перш за все, забезпечує таку швидкість?

Велика амплітуда потенціалу дії

Малий поріг деполяризації

Великий потенціал спокою

Великий фактор надійності

Наявність мієлінової оболонки

4866 / 6307
Для діагностики черевного тифу проводять серологічну реакцію, під час якої до різних розведень сироватки крові пацієнта додають діагностикуми трьох видів мікроорганізмів і результат реакції оцінюють за наявністю осаду у вигляді білих пластівців. Як називається така реакція? Реакція Борде-Жангу

Реакція Райта

Реакція Відаля

Реакція Вассермана

Реакція Асколі

4867 / 6307
Пацієнт через падіння не може відвести верхню кінцівку вище, ніж горизонтальний рівень. Який м'яз було травмовано?

Великий круглий

Підостьовий

Дельтоподібний

Малий круглий

Найширший м'яз спини

4868 / 6307
До лікарні госпіталізовано футболіста з ушкодженням поверхневого пахвинного кільця та розривом двох ніжок, що його обмежують. Чим утворена передня стінка пахвинного каналу?

Власна фасція живота

Міжніжкові волокна

Апоневроз зовнішнього косого м'яза живота

Апоневроз внутрішнього косого м'яза живота

Апоневроз поперечного м'яза живота

4869 / 6307
Хлопчика віком 12 років госпіталізовано до лікарні з попереднім діагнозом: харчова токсикоінфекція. Під час посіву фекалій дитини на середовище Ендо виросла велика кількість безбарвних колоній. Який мікроорганізм можна виключити з-поміж можливих збудників захворювання?

Proteus vulgaris

Yersinia enterocolitica

Pseudomonas aeruginosa

Escherichia coli

Salmonella enteritidis

4870 / 6307
У родині, де батьки глухонімі, глухота у дружини пов'язана з аутосомно-рецесивним геном, а у чоловіка виникла внаслідок тривалого прийому антибіотиків у дитинстві. Яка ймовірність народження глухої дитини в цій родині, якщо батько гомозиготний за алелем нормального слуху?

10%

100%

25%

0%

75%

4871 / 6307
Серед гіполіпідемічних препаратів, що застосовуються з метою профілактики та лікування атеросклерозу, є ловастатин. Який механізм дії цього лікарського засобу?

Активація метаболізму холестерину

Пригнічення всмоктування холестерину в кишківнику

Гальмування біосинтезу холестерину

Усіма наведеними шляхами

Стимулювання екскреції холестерину з організму

4872 / 6307
До травматологічного відділення госпіталізовано пацієнта, у якого спостерігається пошкодження м'язів нижніх кінцівок. Які клітини мають здатність до репаративної регенерації м'язових волокон і відновлення функції м'язів?

Плазмоцити

Фібробласти

Міосателітоцити

Ендотеліоцити

Адипоцити

4873 / 6307
До лікарні госпіталізовано пацієнта зі скаргами на слабкість, болі в кишечнику. Під час лабораторного дослідження фекалій виявлено кулясті цисти з чотирма ядрами. Для якого виду найпростіших характерні такі цисти?

Амеби дизентерійної

Кишкової трихомонади

Амеби кишкової

Балантидію

Амеби ротової

4874 / 6307
Пацієнту діагностовано хронічний бронхіт та призначено муколітичний препарат, що підвищує синтез сурфактанту. Який лікарський засіб було використано для лікування пацієнта?

Амброксол

Лібексин

Фенотерол

Кодеїн

Мукалтин

4875 / 6307
У регуляції фізіологічних функцій беруть участь іони металів. Один із них отримав назву «король месенджерів». Таким біоелементом-посередником є:

Na+

K+

Zn++

Fe+++

Ca++

4876 / 6307
У пацієнта з діагнозом: пневмонія, спостерігається підвищення температури тіла. Яка біологічно активна речовина відіграє провідну роль у виникненні цього симптому захворювання?

Серотонін

Лейкотрієни

Інтерлейкін-1

Гістамін

Брадикінін

4877 / 6307
У пацієнтки під час лікування гіпертонічної хвороби на тлі прийому антигіпертензивного препарату виник сухий кашель. Для якого лікарського засобу характерна така побічна дія?

Лізиноприл

Резерпін

Но-шпа

Дротаверин

Гідрохлортіазил

4878 / 6307
Пацієнт віком 75 років був оперований із приводу рака передміхурової залози. Помер раптово на 4-ту добу після оперативного втручання. Під час розтину тіла чоловіка у просвітах головного стовбура і біфуркаці легеневої артерії були виявлені та легко видалені крихкі маси темно-червоного кольору з тьмяною поверхнею. Такі ж маси були у порожнині правого шлуночка серця. Яке порушення кровообігу спричинило раптову смерть пацієнта? Тромбоз легеневої артерії

Тромбоемболія легеневої артерії

Інфаркт міокарда

Парадоксальна емболія

Тканинна емболія

4879 / 6307
Лікування туберкульозу здійснюється за допомогою комбінованої хіміотерапії, що поєднує речовини різного механізму дії. Який протитуберкульозний препарат пригнічує транскрипцію ДНК в РНК мікобактерій?

Етіонамід

Рифампіцин

ПАСК

Стрептоміцин

Ізоніазид

4880 / 6307
У потерпілого спостерігається перелом у ділянці бічної поверхні променево-зап'ясткового суглоба. У якому місці найімовірніше відбувся перелом?

Нижня третина плечової кістки

Шилоподібний відросток ліктьової кістки

Головчаста кістка

Шилоподібний відросток променевої кістки

Гачкувата кістка

4881 / 6307
Пацієнту діагностовано флегмону передпліччя. Під час мікробіологічного аналізу ексудату із зони запалення виявлено стрептококи. Які клітини будуть переважати в ексудаті?

Еозинофільні гранулоцити

Лімфоцити

Моноцити

Нейтрофільні гранулоцити

Базофільні гранулоцити

4882 / 6307
У дитячому відділенні інфекційної лікарні хлопчику встановлено діагноз: дифтерія. Який препарат треба ввести пацієнту насамперед:

Дифтерійний анатоксин

АКДП

TABte

Протидифтерійну антитоксичну сироватку

АДП

4883 / 6307
Восьмирічній дитині рекомендовано проведення тонзилектомії. Лабораторний аналіз крові виявив, що час згортання крові збільшено до 7 хвилин. Який лікарський засіб треба призначити дитині для зменшення крововтрати під час операції?

Кальцію хлорид

Дицинон

Вікасол

Фібриноген

Амінокапронову кислоту

4884 / 6307
У пацієнта, який тривалий час хворів на переміжну кульгавість, об'єктивно спостерігається: шкіра пальців стопи суха, чорного кольору, нагадує тканину мумії. На невеликій відстані від почорнілої ділянки розташована двоколірна лінія (червоний колір прилягає до майже незмінених тканин, а біло-жовтий колір до змінених тканин). Який вид некрозу розвинувся у пацієнта?

Пролежень

Гангрена

Інфаркт

Секвестр

4885 / 6307
У пацієнта діагностована перфоративна виразка передньої стінки шлунку. До якого простору черевної порожнини потраплятиме вміст шлунку?

Печінкова сумка

Правий брижовий синус

Чепцева сумка

Передшлункова сумка

Лівий брижовий синус

4886 / 6307
Вітамін А у комплексі зі специфічними циторецепторами проникає через ядерні мембрани, індукує процеси транскрипції, що стимулює ріст та диференціацію клітин. Якою формою вітаміну А реалізується ця біологічна функція?

Цис-ретиналь

Ретинол

Транс-ретиналь

Трансретиноєва кислота

Каротин

4887 / 6307
У разі нестачі вітаміну С виникає порушення структури колагенових волокон. Яка стадія їх синтезу порушується?

Агрегація тропоколагену і утворення фібрил

Відщеплення сигнальних олігопептидів

Утворення гама-карбоксиглутамату

Приєднання глюкозних і галактозних залишків

Утворення гідроксипроліну і гідроксилізину

4888 / 6307
У чоловіка після гострої крововтрати виникло відчуття спраги. Зміна якого гомеостатичного параметра викликала це відчуття організму?

Зниження осмотичного тиску рідин організму

Зменшення об'єму позаклітинної рідини

Зниження онкотичного тиску рідин організму

Підвищення осмотичного тиску рідин організму

Підвищення онкотичного тиску рідин

4889 / 6307
У студента перед іспитом збільшилася ЧСС. Які зміни на ЕКГ характерні для тахікардії?

Подовження сегменту Q-T

Розширення комплексу QRS

Подовження інтервалу R-R

Подовження інтервалу P-Q

Укорочення інтервалу R-R

4890 / 6307
Під час експерименту дослідники подразнюють скелетний м'яз серією електричних імпульсів. Який вид м'язового скорочення виникне, якщо кожний наступний імпульс потрапляє на період поодинокого м'язового скорочення?

Асинхронний тетанус

Серія поодиноких скорочень

Зубчастий тетанус

Контрактура м'яза

Суцільний тетанус

4891 / 6307
У пацієнта з діагнозом: серцева недостатність, розвинулися набряки на нижніх кінцівках. Який механізм розвитку серцевого набряку?

Підвищення гідростатичного тиску в артеріолах

Порушення лімфовідтоку

Зниження осмотичного тиску плазми крові

Підвищення гідростатичного тиску в венулах

Підвищення онкотичного тиску плазми крові

4892 / 6307
Під час мікроскопічного дослідження біоптату шкіри спостерігаються гранульоми, що складаються з епітеліоїдних клітин, які оточені в основному Т- лімфоцитами. Серед епітеліоїдних клітин розташовуються поодинокі гігантські багатоядерні клітини Пирогова-Лангханса. У центрі гранульом виявляються ділянки казеозного некрозу. Кровоносні судини відсутні. Для якого захворювання характерні такі гранульоми?

Туберкульозу

Лепри

Риносклероми

Сифілісу

Бруцельозу

4893 / 6307
У бактеріологічній лабораторії досліджують матеріал (промивні води, в'ялена риба домашнього приготування), який взяли у пацієнта із симптомами ботулізму. На яке поживне середовище треба посіяти матеріал для виявлення чистої культури?

Цукровий м'ясо-пептонний агар

Середовище Кітта-Тароцці

Цукровий м'ясо-пептонний бульйон

Цукрово-кров'яний агар

Сироватковий агар

4894 / 6307
У пацієнта з діагнозом: жовчнокам'яна хвороба, спостерігаються симптоми холемічного синдрому. Який симптом зумовлений відсутністю надходження жовчі у кишечник?

Стеаторея

Шкірний свербіж

Брадикардія

Астенія

Гіпотонія

4895 / 6307
Під час пошкодження клітини іонізуючим випромінюванням вмикаються механізми захисту і адаптації. Який механізм відновлення порушеного внутрішньоклітинного гомеостазу реалізується у цьому разі?

Накопичення Na+ у клітинах

Пригнічення аденілатциклази

Активація антиоксидантної системи

Гіпертрофія мітохондрій

Активація Са-опосередкованих клітинних функцій

4896 / 6307
Пацієнту з діагнозом: туберкульоз легень, для лікування призначено протитуберкульозний препарат першого ряду, що спричинив розвиток невриту лицьового нерва та порушення рівноваги. Визначте цей лікарський засіб.

Левоміцетин

Ізоніазид

Цефазолін

Бісептол

Фуразолідон

4897 / 6307
При підозрі на туберкульоз хворій дитині зробили пробу Манту Через 24 години у місці введення алергену з’явились припухлість, гіперемія і болісність. Які основні компоненти визначають цю реакцію організму?

Мононуклеари, Т-лімфоцити і лімфокіни

В-лімфоцити, IgM

Плазматичні клітини, Т-лімфоцити і лімфокіни

Гранулоцити, Т-лімфоцити і IgG

Макрофаги, В-лімфоцити і моноцити

4898 / 6307
Пацієнту перед оперативним втручанням уведено дитилін (суксаметоній) і проведено інтубацію трахеї. Дефіцит якого ферменту в організмі пацієнта продовжує дію міорелаксанту?

Псевдохолінестерази

K-Na-ATФ-ази

N-ацетилтрансферази

Сукцинатдегідрогенази

Карбоангідрази

4899 / 6307
У гістологічному препараті тканини бронха пацієнта, який зловживає тютюнопалінням, у потовщеній слизовій оболонці виявлено хронічний обструктивний бронхіт із ознаками трансформації одношарового війчастого епітелію в багатошаровий плоский епітелій. Для якого патологічного процесу характерна така перебудова епітелію?

Лейкоплакії

Гетероплазії

Гіпертрофії

Метаплазії

Гіперплазії

4900 / 6307
Під час розтину тіла померлого чоловіка віком 48 років у ділянці 1-го сегмента правої легені виявлено кругле утворення діаметром 5 см із чіткими контурами, оточене тонким прошарком сполучної тканини, заповнене білими крихкими масами. Для якої форми вторинного туберкульозу характерні такі ознаки?

Гострий кавернозний туберкульоз

Гострий вогнищевий туберкульоз

Фіброзно-кавернозний туберкульоз

Туберкульома

Казеозна пневмонія

4901 / 6307
Під час розтину тіла чоловіка віком 48 років виявлено, що кістковий мозок плоских кісток, діафізів та епіфізів трубчастих кісток соковитий, сіро-червоний або сіро-жовтий гноєподібний (піоїдний кістковий мозок). Селезінка масою 7 кг, на розрізі темно-червоного кольору, з ішемічними інфарктами. Усі лімфатичні вузли збільшені, м'які, сіро-червоного кольору. У гепатоцитах печінки жирова дистрофія і лейкемічні інфільтрати. Який найбільш імовірний діагноз?

Мієломна хвороба

Лімфогранулематоз

Гострий мієлоїдний лейкоз

Гострий лімфоїдний лейкоз

Хронічний мієлоїдний лейкоз

4902 / 6307
Під час зниження активності ферментів антиоксидантного захисту посилюються процеси перекисного окиснення ліпідів клітинних мембран. Нестача якого мікроелемента сприяє зниженню активності глутатіонпероксидази?

Селену

Молібдену

Кобальту

Марганцю

Міді

4903 / 6307
Унаслідок руйнування деяких структур стовбуру мозку тварина втратила орієнтувальні рефлекси, що виникають у відповідь на зорові та слухові подразнення. Які структури було зруйновано?

Вестибулярні ядра

Червоні ядра

Ядра покрівлі середнього мозку

Чорну речовину

Медіальні ядра ретикулярної формації

4904 / 6307
Під час лабораторного дослідження крові пацієнта виявлено, що вміст білків у плазмі становить 40 г/л. Як це вплине на транскапілярний обмін води в мікроциркуляторному судинному руслі?

Збільшиться і фільтрація, і реабсорбція

Зменшиться фільтрація, збільшиться реабсорбція

Зменшиться і фільтрація, і реабсорбція

Обмін не зміниться

Збільшиться фільтрація, зменшиться реабсорбція

4905 / 6307
Унаслідок перелому нижньої щелепи у постраждалого спостерігається втрата чутливості шкіри у ділянці підборіддя і нижньої губи. Який нерв найімовірніше було травмовано?

Infraorbitalis

Buccalis

Facialis

Mentalis

Maxillaris

4906 / 6307
Чоловік віком 42 роки помер при явищах вираженої інтоксикації та дихальної недостатності. Під час розтину тіла виявлено: тканина легень у всіх відділах строката, з множинними дрібновогнищевими крововиливами та вогнищами емфіземи. Під час гістологічного дослідження легень виявлено: геморагічна бронхопневмонія з абсцедуванням, у цитоплазмі клітин епітелію бронхів еозинофільні та базофільні включення. Яке захворювання розвинулося у пацієнта?

Стафілококова бронхопневмонія

Грип

Часткова пневмонія

Плевропневмонія

Крупозна пневмонія

4907 / 6307
У дитини на слизовій оболонці щік та язика спостерігаються білуваті плями. У виготовлених препаратах-мазках виявлено грампозитивні овальні дріжджоподібні клітини. Визначте ці збудники.

Фузобактерії

Гриби роду Кандіда

Актиноміцети

Стафілококи

Дифтерійна паличка

4908 / 6307
Пацієнту виконано субтотальну субфасціальну резекцію щитоподібної залози. У післяопераційний період у нього тривалий час зберігається осиплість голосу. Який нерв було пошкоджено під час операції?

Верхній гортанний

Під'язиковий

Язиковий

Нижньощелепний

Зворотній гортанний

4909 / 6307
У пацієнта спостерігається жовте забарвлення шкіри, сеча темна, кал темно-жовтого кольору. Підвищення концентрації якої речовини буде виявлено в сироватці крові у цьому разі?

Вільного білірубіну

Кон'югованого білірубіну

Стеркобіліногену

Мезобілірубіну

Білівердину

4910 / 6307
У чоловіка під дією мутагенного фактору з'явилася велика кількість мутантних клітин. Проте більшість із них були розпізнані і знищені. Які клітини відповідають за розпізнання та знищення мутантних клітин?

Плазмобласти

Т-лімфоцити-хелпери

Т-лімфоцити-супресори

В-лімфоцити

Т-лімфоцити-кілери

4911 / 6307
У жінки віком 52 роки артеріальна гіпертензія ускладнилася правобічною геміплегією і втратою мови. Яка зона кори головного мозку є ймовірно найбільш ураженою?

Ліва передня звивина

Ліва передня звивина і ліва скронева частка

Ліва скронева частка

Права передня звивина

Потилична частка

4912 / 6307
До відділення реанімації госпіталізовано пацієнта із діагнозом: інфаркт міокарда. Який лікарський засіб треба ввести пацієнту для купірування больового синдрому?

Целекоксиб

Промедол

Парацетамол

Анальгін

Налоксон

4913 / 6307
Кухар на робочому місці під час приготування їжі отримав опік руки парою. Підвищення концентрації якої речовини викликало почервоніння, набряк та біль ураженої ділянки шкіри?

Галактозаміну

Тіаміну

Гістаміну

Глутаміну

Лізину

4914 / 6307
Пацієнтка віком 57 років для лікування гіпертонічної хвороби тривалий час приймала анаприлін. Через виникнення побічних ефектів пацієнтка відмовилася від прийому лікарського засобу, що спричинило розвиток гіпертонічної кризи та нападу стенокардії. Як називається ускладнення, що виникло в пацієнтки?

Тахіфілаксія

Звикання

Сенсибілізація

Синдром відміни

Лікарська залежність

4915 / 6307
На дослідження в бактеріологічну лабораторію було відправлено випорожнення хворої дитини грудного віку, з яких виділена культура ентеропатогенних кишкових паличок О55К59. на основі яких критеріїв виділена культура віднесена до ЕПКП О552 ?

Біохімічних

Морфологічних

Культуральних

Антигенних

4916 / 6307
Юнаку віком 18 років встановлено діагноз: хвороба Марфана. Під час клініко-лабораторного дослідження виявлено порушення розвитку сполучної тканини, будови кришталика ока, аномалії серцево-судинної системи, арахнодактилія. Який варіант взаємодії генів різних алельних пар сприяє розвитку цього захворювання?

Кодомінування

Неповне домінування

Комплементарність

Плейотропія

Множинний алелізм

4917 / 6307
Пацієнту, якому діагностовано ревматоїдний артрит, упродовж тривалого часу вводили гідрокортизон. У пацієнта з'явилися гіперглікемія, поліурія, глюкозурія, відчуття спраги. Наслідком активації якого процесу є поява цих симптомів?

Глікогенолізу

Глюконеогенезу

Гліколізу

Ліполізу

Глікогенезу

4918 / 6307
Унаслідок ДТП жінка отримала сильний удар в епігастральну ділянку, через що у неї сталася зупинка серця. Яка ймовірна причина зупинки серцевої діяльності?

Збільшене виділення адреналіну

Збільшене виділення кортизолу

Підвищення тонусу блукаючого нерва

Підвищення тонусу симпатичної нервової системи

Збільшене виділення альдостерону

4919 / 6307
Дитину госпіталізовано до лікарні в стані асфіксії. Під час огляду гортані виявлено білуваті плівки, що обтурують просвіт дихальних шляхів та легко видаляються. Для якого виду запалення гортані характерні такі ознаки?

Дифтеритичне

Крупозне

Серозне

Катаральне

Гнійне

4920 / 6307
Пацієнта віком 12 років госпіталізовано до лікарні з гемартрозом колінного суглоба. Він із раннього дитинства хворіє на підвищену кровоточивість. Який діагноз у хлопчика?

Геморагічний васкуліт

Гемофілія

Гемолітична анемія

Тромбоцитопенічна пурпура

В12 фолієво-дефіцитна анемія

4921 / 6307
Під час розтину тіла померлого чоловіка віком 43 роки було виявлено набряк легень. З анамнезу відомо, що пацієнт хворів на ІХС із розвитком інфаркту міокарда. Яка ймовірна причина набряку легень?

Стаз крові

Гостра лівошлуночкова недостатність

Гостре загальне малокрів'я

Ішемія малого кола

Гостра правошлуночкова недостатність

4922 / 6307
У трирічної дитини спостерігається підвищення температури, збільшені лімфовузли. Під час лабораторного дослідження загального аналізу крові виявлено лімфоцитоз. Методом ІФА виявлено антиген віруса Епштейна-Бара. Яке захворювання найімовірніше розвинулося у дитини?

Лімфома Беркета

Герпетична аденопатія

Інфекційний мононуклеоз

Цитомегаловірусна інфекція

Генералізована інфекція, викликана herpes-zoster

4923 / 6307
У дитини, якій діагностовано пілоростеноз, об'єктивно спостерігається часте блювання та ознаки зневоднення організму. Яка форма порушення кислотно-основного стану може розвинутися у цьому разі?

Негазовий ацидоз

Негазовий алкалоз

Газовий алкалоз

Газовий ацидоз

Метаболічний ацидоз

4924 / 6307
У пацієнтки виявлено рак шийки матки. У які регіонарні лімфатичні вузли можливе розповсюдження метастазів?

Поперекові та ниркові

Навколоміхурові та поперекові

Верхні та нижні брижові

Пахвинні та здухвинні

4925 / 6307
У чоловіка внаслідок патологічного процесу збільшена товщина гематоальвеолярного бар'єра. Найімовірнішим наслідком цього буде зменшення:

Альвеолярної вентиляції легень

Дифузійної здатності легень

Кисневої ємності крові

Резервного об'єму видиху

Хвилинного об'єму дихання

4926 / 6307
У пацієнта спостерігається тремор рук, пов'язаний із хворобою Паркінсона. Дефіцит якого медіатора в стріопалідарній системі спричиняє такий симптом?

ГАМК

Дофаміну

Субстанції Р

Норадреналіну

Серотоніну

4927 / 6307
За якої умови можливо виконати пункцію сечового міхура через передню черевну стінку, не займаючи очеревину?

Тільки у чоловіків

У разі спустошеного сечового міхура

У разі наповненого сечового міхура

Тільки у дітей

Тільки у жінок

4928 / 6307
У судово-медичній експертизі використовується метод дактилоскопії, який базується на тому, що сосочковий шар дерми визначає індивідуальний малюнок на поверхні шкіри. Яка тканина утворює цей шар дерми?

Щільна неоформлена сполучна тканина

Жирова тканина

Пухка волокниста неоформлена сполучна тканина

Щільна оформлена сполучна тканина

Ретикулярна тканина

4929 / 6307
У жінки впродовж року прогресувала ниркова недостатність, що призвело до смерті. Під час розтину тіла померлої виявлено великі строкаті нирки з червоними дрібними краплями в жовто-сірому кірковому шарі. Під час гістологічного дослідження в клубочках спостерігаються «півмісяці» з проліферуючого нефротелія. Для якого захворювання характерні такі результати дослідження?

Швидкопрогресуючого гломерулонефриту

Амілоїдозу

Раку нирки

Гнійного нефриту

Крововиливу в нирки

4930 / 6307
У пацієнта спостерігається напад тахікардії. Які мембранні циторецептори кардіоміоцитів треба заблокувати, щоб припинити напад?

M- та Н-холінорецептори

M-холінорецептори

Бета-адренорецептори

Альфа-адренорецептори

Н-холінорецептори

4931 / 6307
В альпініста, який піднявся на висоту 5200 м, розвинувся газовий алкалоз. Що є причиною розвитку цього патологічного стану?

Гіперканнія

Зниження температури навколишнього середовища

Гіпервентиляція легенів

Гіпоксемія

Гіповентиляція легенів

4932 / 6307
У пацієнта з тимомою (пухлиною вилочкової залози) об'єктивно спостерігається: ціаноз, розширення підшкірної венозної сітки, набряк м'яких тканин обличчя, шиї, верхньої половини тулуба і верхніх кінцівок. Який венозний стовбур стиснений пухлиною?

Зовнішня яремна вена

Підключична вена

Передня яремна вена

Верхня порожниста вена

Внутрішня яремна вена

4933 / 6307
У пацієнта, який має важку форму порушення водно-сольового обміну, сталася зупинка серця у діастолі. Який найімовірніший механізм цього патологічного стану?

Гіпонатріємія

Гіперкаліємія

Гіпернатріємія

Гіпокаліємія

Гіпомагніємія

4934 / 6307
У жінки віком 52 роки під час проведення загального аналізу крові виявлено зниження кількості еритроцитів та підвищення рівня вільного гемоглобіну в плазмі крові (гемоглобінемія), КП - 0,85. Який вид анемії спостерігається у пацієнтки?

Гостра постгеморагічна

Хронічна постгеморагічна

Анемія внаслідок порушення еритропоезу

Набута гемолітична

Спадкова гемолітична

4935 / 6307
Пацієнту віком 50 років встановлено діагноз: гіпертонічна хвороба. Під час фізичного навантаження у нього з'явилося відчуття слабкості, нестачі повітря, ціаноз слизової оболонки губ та шкіри обличчя. Аускультативно виявлено вологі хрипи в нижніх відділах легень. Який механізм є причиною виникнення такого синдрому?

Гостра лівошлуночкова недостатність

Тампонада серця

Хронічна лівошлуночкова недостатність

Колапс

Хронічна правошлуночкова недостатність

4936 / 6307
Знешкодження ксенобіотиків (лікарських засобів, епоксидів, ареноксидів, альдегідів, нітропохідних тощо) та ендогенних метаболітів (естрадіолу, простагландинів, лейкотрієнів) відбувається в печінці шляхом їх кон’югації з:

Глутатіоном

Аспарагіновою кислотою

Фосфоаденозином

S-Аденозилметіоніном

Гліцином

4937 / 6307
Під час ендоскопічного дослідження у пацієнта, якому діагностовано хронічний ентероколіт, спостерігається відсутність специфічних структур слизової оболонки тонкого кишечника. Які компоненти характеризують особливості рельєфу слизової оболонки цього органа?

Гаустри, ворсинки, крипти

Косо-спіральні складки

Поля, ворсинки

Циркулярні складки, ворсинки та крипти

Поля, складки, ямки

4938 / 6307
У чоловіка 30-ти років перед операцією визначили групу крові. Кров резус-позитивна. Реакція аглютинації еритроцитів не відбулася зі стандартними сироватками груп 0 (I), А (II), В (III). Досліджувана кров належить до групи:

Неможливо визначити

0 (I)

В (III)

АВ (IV)

А (II)

4939 / 6307
Пацієнту, який відчуває біль у суглобах, постійно призначають аспірин. Який фермент інгібує дія цього лікарського заcобу:

Фосфоліпазу А2

Фосфоліпазу Д

Ліпооксигеназу

Циклооксигеназу

Фосфоліпазу С

4940 / 6307
Під час проведення ЕКГ-дослідження виявлено, що тривалість інтервалу RR дорівнює 1,5 с, частота серцевих скорочень 40/хв. Що є водієм ритму серця у цьому разі?

Ліва ніжка Гіса

Атріовентрикулярний вузол

Синусовий вузол

Права ніжка Гіса

Пучок Гіса

4941 / 6307
На гістологічному препараті представлено орган, що вкритий сполучнотканинною капсулою, від якої всередину відходять трабекули. В органі спостерігається кіркова речовина, де містяться лімфатичні вузлики та мозкова речовина, представлена тяжами лімфоїдних клітин. Який орган представлений на гістопрепараті?

Червоний кістковий мозок

Лімфатичний вузол

Мигдалики

Селезінка

Тимус

4942 / 6307
У пацієнта віком 45 років під час проведення ЕКГ виявлено: ритм синусовий, кількість передсердних комплексів більша ніж шлуночкових; прогресуюче подовження інтервалу Р-Q від комплексу до комплексу; випадіння окремих шлуночкових комплексів; зубці Р та комплекси QRST без змін. Визначте тип порушення серцевого ритму.

Атріовентрикулярна блокада II ступеня

Повна атріовентрикулярна блокада

Атріовентрикулярна блокада I ступеня

Синоатріальна блокада

Внутрішньопередсердна блокада

4943 / 6307
Чоловіку віком 40 років проведено лімфографію органів грудної порожнини. Виявлено пухлину, що вразила орган, з лімфатичних судин якого лімфа безпосередньо переходить у грудну протоку. Визначте цей орган:

Осердя

Серце

Лівий головний бронх

Трахея

Стравохід

4944 / 6307
Для усунення марення і галюцинацій у пацієнтів з діагнозом: шизофренія, використовують аміназин. Який механізм антипсихотичної дії цього лікарського засобу?

Блокада холінергічних процесів у ЦНС

Стимуляція холінергічних процесів у ЦНС

Блокада адренергічних і дофамінергічних процесів у ЦНС

Інгібування зворотнього нейронального захоплення МАО

Стимуляція адренергічних і дофамінергічних процесів у ЦНС

4945 / 6307
У пацієнта спостерігаються головний біль, підвищення температури тіла, озноб, кашель. Під час бактеріологічного дослідження мокротиння виявлено палички овоїдної форми з біполярним забарвленням, грамнегативні, у мазку з бульйонної культури розташовані ланцюжками, на агарі утворюються колонії R-форми. Для збудника якого захворювання характерні такі показники?

Стрептококової ангіни

Туберкульозу

Менінгококового назофарингіту

Чуми

Дифтерії

4946 / 6307
У пацієнта спостерігається запалення медіального надвиростка плечової кістки (епікондиліт). Який нерв залучено у запальний процес?

N. medianus

N. axillaris

N. ulnaris

N. radialis

N. musculocutaneus

4947 / 6307
Пацієнт після перенесеного ішемічного інсульту не може здійснювати довільні рухи правими кінцівками, спостерігається гіперрефлексія. Під час пальпації визначається підвищений тонус м'язів кінцівок. Яка форма порушення рухової функції спостерігається у пацієнта?

Мозочкова атаксія

Тетанія

Периферичний параліч

Периферичний парез

Центральний параліч

4948 / 6307
У чоловіка віком 43 роки з'явилися симптоми сезонної алергії. Який лікарський засіб, що не має снодійного ефекту, треба йому призначити?

Дипразин

Ванкоміцин

Супрастин

Мікосептин

Лоратадин

4949 / 6307
У пацієнта спостерігається хронічний запальний процес мигдаликів. Для реалізації механізму фагоцитозу у вогнищі запалення потрібен НАДФН. Унаслідок якого біохімічного процесу у вогнищі запалення підтримується потрібна концентрація НАДФН?

Гліколізу

Циклу Корі

Орнітинового циклу

Пентозо-фосфатного шляху

Циклу Кребса

4950 / 6307
До лікарні звернулася мати з дитиною. На голові у дитини спостерігається гангренозна рана. Під час огляду у рані виявлено білі червоподібні личинки відкласти? Яка комаха найімовірніше могла їх відкласти?

Муха-жигалка

Москіти

Комарі

Блохи

Вольфартова муха

4951 / 6307
Під час гістологічного дослідження біоптату шкіри виявлено гранульоми, які складаються з макрофагальних вузликів із наявністю лімфоцитів та плазматичних клітин. Крім того, зустрічаються великі макрофаги з жировими вакуолями, які містять збудників захворювання, запакованих у вигляді куль (клітини Вірхова). Грануляційна тканина добре васкуляризована. Для якого захворювання характерна описана гранульома?

Сифілісу

Лепри

Бруцельозу

Туберкульозу

Риносклероми

4952 / 6307
Жінка віком 52 роки з діагнозом: рак грудної залози, пройшла курс променевої терапії. Розмір пухлини зменшився. Який механізм ушкодження клітини найімовірніше обумовлює ефективність променевої терапії?

Лізис NK-клітинами

Утворення вільних радикалів

Гіпертермія

Тромбоз судин

Мутагенез

4953 / 6307
У лікарню наприкінці робочого дня доставлений робітник 'гарячого' цеху, який скаржиться на головний біль, запаморочення, нудоту, загальну слабкість. Об’єктивно: свідомість збережена, шкірні покриви гіперемовані, сухі, гарячі на дотик. Частота серцевих скорочень - 130/хв. Дихання часте, поверхневе. Яке порушення процесів терморегуляції, найбільш імовірно, виникло у людини в даній ситуації?

Посилення тепловіддачі і зниження теплопродукції

Зниження теплопродукції без змін тепловіддачі

Посилення тепловіддачі і теплопродукції

Зниження тепловіддачі

Посилення теплопродукції без змін тепловіддачі

4954 / 6307
Між бактеріями під час процесу кон'югації утворюється цитоплазматичний місток. По цьому містку із клітини-донора до клітини-реципієнта надходять плазміди і фрагменти молекули ДНК. Яка біологічна роль цього процесу?

Забезпечення обміну речовинами між клітинами

Забезпечення обміну генетичного матеріалу

Підвищення гетерозиготності

Сприяння активізації мутаційного процесу

Ліквідація небажаних мутацій

4955 / 6307
У пацієнта внаслідок хронічного захворювання органів дихання, що супроводжується задишкою, тахікардією і ціанозом, під час дослідження газового складу крові виявлено розвиток гіпоксемії і гіперкапнії. Яке порушення зовнішнього дихання спричинило зміни в газовому складі крові?

Гіпердифузія

Гіпервентиляція

Гіперперфузія

Гіпоперфузія

Гіповентиляція

4956 / 6307
Пацієнту встановлено діагноз: інфаркт задньої частини міжшлуночкової перетинки. У ділянці якої кровоносної судини виникло порушення кровообігу?

R. marginalis sinister

R. circumflexus

R. interventricularis posterior

R. marginalis dexter

R. atrialis intermedius

4957 / 6307
Чоловік віком 36 років за тиждень після тривалого перебування у весняному лісі раптово захворів. Спостерігається гарячка, головний біль, порушення свідомості, епілептиформні напади. Смерть настала на 3-й день захворювання. Під час розтину тіла виявлено набряк головного мозку та множинні точкові геморагії. Під час мікроскопічного дослідження виявлено периваскулярний та перицелюлярний набряк, множинні периваскулярні, переважно лімфоцитарні, інфільтрати. Яке захворювання розвинулося у чоловіка?

Поліомієліт

Кліщовий енцефаліт

Церебро-васкулярна хвороба

Менінгококова інфекція

Цитомегаловірусний енцефаліт

4958 / 6307
У крові пацієнта, який хворіє на лейкопенію, виявили антилейкоцитарні антитіла. Який тип алергічної реакції за Кумбсом і Джеллом виник у чоловіка?

Цитотоксичний

Стимулюючий

Анафілактичний

Гіперчутливість сповільненого типу

Імунокомплексний

4959 / 6307
У пацієнта віком 43 роки розвиток гострого панкреатиту супроводжується порушенням прохідності загальної жовчної протоки. До якого ускладнення це може призвести?

Гемолітичної жовтяниці

Портальної гіпертензії

Паренхіматозної жовтяниці

Печінкової коми

Механічної жовтяниці

4960 / 6307
Юнак віком 15 років скаржиться на загальну слабкість, запаморочення, швидку стомлюваність. Під час обстеження виявлено еритроцити зміненої форми, кількість їх знижена. Попередній діагноз: серповидноклітинна анемія. Яка амінокислотна заміна в гемоглобіні зумовлює розвиток цього патологічного стану?

Глутамату на аспартат

Глутамату на валін

Глутамату на аланін

Валіну на аспартат

Валіну на глутамат

4961 / 6307
До кардіологічного відділення госпіталізовано чоловіка з інфарктом міокарда в гострій фазі. Який фермент застосовують для лізису тромбів у коронарних судинах у перші години?

Лізоцим

Стрептокіназу

Хімотрипсин

Лідазу

Трипсин

4962 / 6307
Організми, які в процесі еволюції не створили захисту від H2O2, живуть лише в анаеробних умовах. Які з нижченаведених ферментів можуть руйнувати пероксид водню?

Флавінзалежні оксидази

Оксигеназа та гідроксилаза

Пероксидаза та каталаза

Оксигеназа та каталаза

Цитохромоксидаза, цитохром b5

4963 / 6307
Під час наркозу у хворого виникла загроза набряку мозку. Який препарат треба ввести у цьому разі?

Спіронолактон

Тріамтерен

Дофамін

Фуросемід

Натрію бромід

4964 / 6307
У дитини, яку годували синтетичними сумішами, з'явилися ознаки недостатності вітаміну B1. У яких реакціях бере участь цей вітамін?

Окислювальне декарбоксилування кетокислот

Декарбоксилювання амінокислот

Трансамінування амінокислот

Гідроксилювання проліну

Окислювально-відновні реакції

4965 / 6307
У хлопчика спостерігається велика щілина між різцями. Відомо, що ген, відповідальний за розвиток такої аномалії, домінантний. У рідної сестри цього хлопчика зуби звичайного положення. За генотипом дівчинка буде:

Гетерозигота

Гомозигота домінантна

Тригетерозигота

Гомозигота рецесивна

Дигетерозигота

4966 / 6307
Під час проведення місцевого знеболення у пацієнта виник анафілактичний шок. Який із нижченаведених препаратів треба ввести хворому?

Преднізолон

Адреналіну гідрохлорид

Супрастин

Атропіну сульфат

Діазепам

4967 / 6307
Пацієнту, який хворіє на подагру, лікар призначив алопуринол, що призвело до зниження концентрації сечової кислoти. Яка властивість алопуринолу забезпечує цей терапевтичний ефект?

Уповільнення реутилізації піримідинових нуклеотидів

Прискорення синтезу нуклеїнових кислот

Прискорення катаболізму піримідинових нуклеотидів

Збільшення швидкості виведення азотовмісних речовин

Конкурентне інгібування ксантиноксидази

4968 / 6307
У дванадцятирічної дитини за 2 тижні після перенесеної ангіни розвинувся нефритичний синдром, що свідчить про ураження базальної мембрани клубочків нирок. Який механізм найімовірніше лежить в основі пошкодження базальної мембрани?

Антитільний

Реагіновий

Імунокомплексний

Гранулематозний

Цитотоксичний

4969 / 6307
На тлі тривалого введення гепарину у пацієнта розвинулася шлункова кровотеча. Укажіть специфічний антидот гепарину, який треба терміново застосувати.

Бемегрид

Дипіроксим

Протаміну сульфат

Натрію цитрат

Вікасол

4970 / 6307
У пацієнта, який хворіє на крупозну пневмонію, спостерігається гарячка з температурою тіла 39°С. Водночас добові коливання температури не перевищують 1°С упродовж 9 діб. До якого типу температурних кривих відноситься гарячка у чоловіка?

Ремітуюча

Поворотна

Гектична

Постійна

Гіперпіретична

4971 / 6307
Під час дослідження біоптату шкіри виявлено гранульоми, до складу яких входять лімфоцити та макрофаги. Серед макрофагів виділяються великі клітини з жировими включеннями, які вміщують у собі спаковані у вигляді куль мікроорганізми (клітини Вірхова). Для якого захворювання характерний такий тип гіперчутливості?

Риносклерома

Лепра

Сифіліс

Висипний тиф

Туберкульоз

4972 / 6307
Під час гістологічного дослідження біоптату, який отримали із нижньої третини стравоходу чоловіка віком 57 років із симптомами тривалого рефлюксу шлункового вмісту, виявлено такі зміни: у слизовій оболонці на місці багатошарового плоского епітелію визначається одношаровий залозистий призматичний епітелій, з ознаками продукції слизу. Укажіть патологічний процес, який виник у слизовій оболонці пацієнта.

Гіпертрофія

Організація

Регенерація

Метаплазія

Гіперплазія

4973 / 6307
Жінка скаржиться на погіршення зору. Під час обстеження у пацієнтки виявлено ожиріння та гіперглікемію натще серце. Яке ускладнення основної хвороби може бути причиною втрати зору?

Гломерулопатія

Діабетична макроангіопатія

Діабетична нейропатія

Атеросклероз

Діабетична мікроангіопатія

4974 / 6307
Пацієнт скаржиться на біль у верхньому відділі пупкової ділянки. Пальпаторно визначається рухлива болісна кишка. Яку кишку пальпує лікар?

Дванадцятипалу

Сигмоподібну

Поперечно-ободову

Порожню

Клубову

4975 / 6307
В умовному експерименті дія токсичної речовини порушує механізм передавання нервового імпульсу між нейронами. Яка структура забезпечує виконання цієї функції?

Нейролема

Синапс

Мітохондрія

Нейрофібрила

Субстанція Ніссля

4976 / 6307
Під час дослідження стегнової кістки виявлено хронічне гнійне запалення компактної речовини та кісткового мозку, утворення секвестрів. Для якого захворювання характерні такі симптоматичні явища?

Мієломної хвороби

Остеомієліту

Ретикулосаркоми

Остеобластокластоми

Періоститу

4977 / 6307
Отруєння ботуліністичним токсином, який блокує вхід іонів кальцію до нервоних закінчень аксонів мотонейронів, є небезпечним для життя. Яка загроза виника?

Виникнення блювоти

Виникнення проносу

Розлад тонусу судин

Зупинка серця

Зупинка дихання

4978 / 6307
Під час систоли шлуночків серцевий м'яз не відповідає на додаткове подразнення тому. У якій фазі він перебуває?

Підвищеної збудливості

Відносної рефрактерності

Абсолютної рефрактерності

Субнормальної збудливості

4979 / 6307
Під час фізичного й емоційного навантаження людина менш чутлива до болю. Визначте, які механізми пригнічення болю активуються в організмі людини у цьому разі.

Парасимпатична система

Ноцицептивна система

Антиноцицептивна система

Функції щитоподібної залози

Функції надниркових залоз

4980 / 6307
Під час підйому в гори у людини збільшується частота дихання і прискорюється серцебиття. Яка причина таких процесів в організмі людини?

Підвищення парціального тиску СО2

Зниження парціального тиску О2

Підвищення рН крові

Збільшення концентрації азоту в повітрі

Підвищення вологості повітря

4981 / 6307
У хлопця віком 18 років виявлено збільшення щитоподібної залози, що супроводжується підвищеним обміном речовин та збільшенням частоти пульсу. Ці ознаки спостерігаються при гіперсекреції гормону тироксину. Які органели клітин щитоподібної залози найбільш відповідальні за секрецію та виділення гормонів?

Лізосоми

Рибосоми

Комплекс Гольджі

Мітохондрії

Центросоми

4982 / 6307
Сибірка - гостре небезпечне інфекційне захворювання, що легко поширюється та, за сприятливих умов, залишається багато років життєдіяльним і збеpiгає патогенність. Яка особливість збудника сибірки робить його небезпечним упродовж тривалого часу?

Утворює полісахаридну капсулу

Утворює спори

Утворює джгутики

Утворює протеїнову капсулу

Належить до актиноміцетів

4983 / 6307
У підлітка віком 13 років під час проведення рентгенографічного дослідження кульшового суглоба виявлено зону просвітлення між голівкою та діафізом стегнової кістки шириною 3 мм. Як треба розцінювати цю ситуацію?

Як норму (незавершений процес окостеніння)

Як вивих головки стегнової кістки

Як тріщину шийки стегнової кістки

Як артефакт на рентгенологічній плівці

Як перелом шийки стегнової кістки

4984 / 6307
Пацієнт скаржиться, що під час згадування про минулі трагічні події в його житті, у нього виникають тахікардія, задишка, підвищення артеріального тиску. Які структури центральної нервової системи (ЦНС) забезпечують такі кардіореспіраторні реакції у пацієнта?

Кора великих півкуль

Чотиригорбкове тіло середнього мозку

Специфічні ядра таламуса

Мозочок

Латеральні ядра гіпоталамуса

4985 / 6307
У вагітної жінки лікар-акушер-гінеколог вимірює розміри таза. За допомогою циркуля була виміряна відстань між двома клубовими гребенями. Який розмір великого таза визначали?

Distantia spinarum

Distantia cristarum

Conjugata vera

Conjugata anatomica

Distantia throchanterica

4986 / 6307
Під час посмертного розтину тіла дев'ятирічної дитини у слизовій оболонці прямої кишки виявлено множинні неправильної форми дефекти різної глибини з нерівними краями, а також сіро-білі плівки, щільно спаяні з підлеглою тканиною. Для якого захворювання характерні такі результати аутопсії?

Дизентерія

Амебіаз

Черевний тиф

Сальмонельоз

Холера

4987 / 6307
У дитини з гемолітичною хворобою новонародженого розвинулася енцефалопатія. Збільшення вмісту якої речовини в крові спричинило ураження центральної нервової системи (ЦНС)?

Комплексу білірубін-альбуміну

Білірубін-глюкуроніду

Вільного білірубіну

Жовчної кислоти

Вердоглобіну

4988 / 6307
Пацієнтам з ішемічною хворобою серця призначають невеликі дози аспірину, який інгібує синтез активатора агрегації тромбоцитів тромбоксану А2. 3 якої речовини утворюється тромбоксан А2?

Глутамінова кислота

Малонова кислота

Оцтова кислота

Гомогентизинова кислота

Арахідонова кислота

4989 / 6307
Під час дослідження ізольованого кардіоміоциту (КМЦ) встановлено, що він не генерує імпульси збудження автоматично. Визначте, з якої структури серця отримано КМЦ.

Волокна Пуркін'є

Атріо-вентрикулярний вузол

Сино-атріальний вузол

Шлуночки

Пучок Гіса

4990 / 6307
У пацієнта діагностовано порушення периферичного кровообігу, основою якого є обмеження припливу артеріальної крові. Спостерігається збліднення цієї ділянки та зниження місцевої температури. Яким захворюванням це обумовлено?

Артеріальною гіперемією

Ішемією

Венозною гіперемією

Сладж-феноменом

Лімфостазом

4991 / 6307
Вкажіть організми, в клітинах яких відсутні мембранні органели та їх спадковий матеріал не має нуклеосомної організації:

Еукаріоти

Прокаріоти

Аскоміцети

Віруси

Найпростіші

4992 / 6307
У біоптаті дужок м'якого піднебіння, який взяли на аналіз через підозру на пухлину (макроскопічно визначалася виразка зі щільним дном), виявлено некроз слизової оболонки з інфільтрацією підслизового шару лімфоцитами, епітеліоїдними клітинами, плазматичними клітинами, поодинокими нейтрофілами, наявність вираженого ендо- та периваскуліту. Для якого захворювання характерні ці симптоми?

Дифтерії зіву

Виразкового стоматиту

Афтозного стоматиту

Первинного сифілісу

Виразково-некротичного стоматиту

4993 / 6307
У зв'язку з випадком дифтерії виникла потреба провести запобіжні щеплення. Який препарат треба використати для створення штучного активного імунітету?

Антидифтерійну сироватку

Специфічний імуноглобулін

Вакцину АКДП

Вакцину з убитих бактерій

Дифтерійний анатоксин

4994 / 6307
У жінки виявлено ущільнення в молочній залозі. У якому напрямку під час операції має провести розріз лікар-хірург для зменшення травмування дольки?

Поперечно

Радіально

Дугоподібно

Вертикально

4995 / 6307
Під час посмертного розтину тіла чоловіка, який проходив військову службу на підводному атомному човні, виявлено такі патології: спустошення в кістковому мозку (панмієлофтиз), анемія, лейкопенія, тромбоцитопенія, розпад лімфоцитів у лімфатичних вузлах, селезінці, лімфатичному апараті шлунково-кишкового тракту, крововиливи наднирники. Яке захворювання розвинулося у чоловіка?

Вібраційна хвороба

Гостра променева хвороба

Гостра анемія

Гострий лейкоз

Кесонна хвороба

4996 / 6307
Чоловіка віком 62 роки госпіталізовано до лікарні через масивний крововилив у мозок. AT- 70/30 мм рт. ст., ЧСС - 120/хв, ЧД- 4/хв, свідомість і реакція на зовнішні подразники відсутні. Визначте стан пацієнта.

Оглушення

Колапс

Шок

Кома

Агонія

4997 / 6307
Чоловік звернувся до лікаря з приводу безпліддя. Має високий зріст, зниження інтелекту, недорозвинення статевих залоз. У епітелії слизової оболонки порожнини рота виявлено статевий хроматин (1 тільце Барра). Для якого захворювання характерні ці симптоматичні явища?

Синдром Клайнфельтера

Синдром Ді Джорджи

Синдром Іценка-Кушинга

Акромегалія

Адреногенітальний синдром

4998 / 6307
Під час експерименту оцінюють рівень збудливості тканини. Який показник треба визначити для цього?

Критичний рівень деполяризації

Амплітуду ПД

Поріг деполяризації

Тривалість ПД

Потенціал спокою

4999 / 6307
Під час гістологічного дослідження лімфовузла в ділянці заднього трикутника шиї пацієнта віком 18 років морфолог виявив скупчення клітин: поодиноких багатоядерних клітин Березовського-Штернберга, великих та малих клітин Ходжкіна, численних лімфоцитів, поодиноких плазматичних клітин, еозинофілів. Яке захворювання розвинулося у пацієнтa?

Лімфоцитарна лімфома

Пухлина Беркітта

Лімфогранулематоз

Нодулярна лімфома

Хронічний лімфоїдний лейкоз

5000 / 6307
Під час обстеження пацієнта лікар використовує анатомічний розподіл передньої стінки живота на ділянки для більш ретельного діагностування хвороб. Скільки таких ділянок є в межах передньобокової стінки живота?

4

5

3

9

8

5001 / 6307
У пацієнтки, яка упродовж 15 років хворіє вираженою гіпертензією, з'явилися задишка, серцебиття, знизився систолічний тиск, діастолічний залишився без змін. Який основний механізм виникнення серцевої недостатності у жінки?

Ушкодження міокарда

Порушення регуляції серцевої діяльності

Перевантаження серця збільшеним опором викиду крові

Перевантаження серця збільшеним об'ємом крові

Порушення проведення імпульсу по міокарду

5002 / 6307
Територію колишнього худобомогильника, що не використовувався понад 50 років, планується передати під житлове будівництво. Однак дослідження грунту виявило наявність життєздатних спор збудника особливо небезпечного захворювання. Який із мікроорганізмів міг зберігатися у грунті упродовж такого тривалого часу?

Francisella tularensis

Brucella abortus

Mycobacterium bovis

Bacillus anthracis

5003 / 6307
За умов гострої кровотечі організм втрачає не тільки рідину, але й електроліти. Який із наведених нижче засобів у вигляді розчину можна використати як простий кровозамінник?

Натрію бромід

Натрію хлорид

Альбумін

Кальцію хлорид

Натрію нуклеотид

5004 / 6307
У пацієнта діагностована стеаторея. Вкажіть, з порушенням надходження у кишечник яких речовин це може бути пов'язано?

Жирів

Трипсину

Жовчних кислот

Амілази

Хімотрипсину

5005 / 6307
Чоловіка віком 30 років, який хворіє на цукровий діабет 1-го типу, було госпіталізовано у стані коми. Лабораторні дослідження виявили гіперглікемію, кетонемію. Яке з наведених нижче метаболічних порушень може бути у цього пацієнта?

Респіраторний ацидоз

Нормальний кислотно-лужний стан

Метаболічний алкалоз

Респіраторний алкалоз

Метаболічний ацидоз

5006 / 6307
Чоловіка госпіталізували до інфекційного відділення на 2-у добу захворювання з попереднім діагнозом: інфекційний мононуклеоз. Який результат лабораторного дослідження може підтвердити діагноз у пацієнта в день госпіталізації?

Ізоляція (виділення) вірусу герпесу

Виявлення IgM антитіл до вірусу Епштейна-Барр

Виявлення антитіл до цитомегаловіруcy

Виявлення IgM-антитіл до вірусу простого герпесу

Виявлення 4-кратного наростання антитіл до вірусу простого герпесу

5007 / 6307
У дорослої людини системний артеріальний тиск знизився з 120/70 до 90/50 мм рт. ст., що викликало рефлекторне звуження судин. У якому із нижченаведених органів звуження судин буде найменшим?

Печінка

Скелетні м'язи

Шкіра

Кишечник

Серце

5008 / 6307
До травматологічного пункту госпіталізовано постраждалого після ДТП з діагнозом: закритий перелом середньої третини стегна зі зміщенням. Для репозиції кісткових уламків хворому введено 10 мл 2% розчину дитиліну в/в, унаслідок чого розвинулося тривале апное та міорелаксація. Дефіцитом якого ферменту зумовлена ця фармакогенетична ферментопатія?

Уридіндифосфоглюкуронова трансферази

Псевдохолінестерази

N-ацетилтрансферази

Метгемоглобінредуктази

Глюкозо-6-фосфатдегідрогенази

5009 / 6307
Жінка віком 37 років скаржиться на головний біль, запаморочення, поганий сон, затерпання кінцівок. Упродовж 6 років працює на заводі газорозрядних ламп у свинцевому цеху. Під час лабораторного аналізу крові виявлено знижену кількість еритроцитів та гемоглобіну, підвищений в декілька разів уміст сироваткового заліза. Який вид анемії у жінки?

Метапластична

Залізодефіцитна

Анемія Мінковського-Шоффара

Гіпопластична

Залізорефрактерна

5010 / 6307
Пацієнту з діагнозом: неоперабельний рак легені, лікар онколог призначив знеболювальний лікарський засіб, який викликав побічну реакцію. У хворого спостерігаються явища непрохідності кишечника. Який знеболювальний препарат міг викликати таке ускладнення?

Фентаніл

Анальгін

Морфін

Промедол

Омнопон

5011 / 6307
У дванадцятирічного хлопчика, який хворіє на грип, діагностовано респіраторний мікоплазмоз. Який вид інфекції розвинувся у дитини?

Рецидив

Ятрогенна інфекція

Суперінфекція

Змішана інфекція

Аутоінфекція

5012 / 6307
До приймального відділення лікарні госпіталізували юнака у непритомному стані. Діагностовано отруєння морфіном. У нього спостерігається поверхневе та рідке дихання, що зумовлено пригніченням дихального центру. Який тип недостатності дихання виник у пацієнта?

Дифузійний

Перфузійний

Вентиляційний рестриктивний

Вентиляційний обструктивний

Вентиляційний дисрегуляторний

5013 / 6307
У пацієнта спостерігається випадіння функції медіальних половин сітківки. Який відділ провідного шляху зорового аналізатора уражений?

Лівий зоровий тракт

Зорове перехрестя

Правий зоровий нерв

Правий зоровий тракт

Лівий зоровий нерв

5014 / 6307
Під час дослідження нирки виявлено. що вона набрякла, повнокровна, капсула легко знімається, порожнини мисок і чашечок розширені, заповнені каламутною сечею, їх слизова оболонка тьмяна, з вогнищами крововиливів. На розрізі тканина нирки строката, жовто-сірі ділянки оточені зоною повнокрів'я і геморагій. Якому захворюванню відповідає такий макроскопічний вид нирки?

Амілоїдоз нирок

Гострий пієлонефрит

Полікістоз нирок

Нефролітіаз

Гострий гломерулонефрит

5015 / 6307
У вівчаря, який пас отару овець під охороною собак, за деякий час з'явився біль у грудях і кровохаркання. Рентгенологічно у легенях виявлено фіни гельмінта кулястої форми. Імунологічна реакція Касоні позитивна. Укажіть гельмінта, який міг спричинити це захворювання.

Печінковий сисун

Стьожак широкий

Ціп'як карликовий

Ехінокок

Ціп'як озброский

5016 / 6307
У дівчини віком 18 років виявлено диспропорції тіла, крилоподібні складки шкіри на шиї, недорозвиненість яєчників, у ядрах клітин букального епітелію відсутні тільця Барра. Для якого захворювання характерні такі симптоматичні явища?

Синдром Клайнфельтера

Синдром Патау

Синдром котячого крику

Синдром Едвардса

Синдром Шерешевського-Тернера

5017 / 6307
Пацієнту з діагнозом: хронічний бронхіт, призначили муколітичний та відхаркувальний лікарський засіб. Вкажіть цей препарат:

Глауцину гідрохлорид

Анаприлін

Парацетамол

Кодеїн

Амброксол

5018 / 6307
У жінки віком 46 років, яка хворіє на жовчнокам'яну хворобу, розвинулася жовтяниця. Сеча набула темно-жовтого кольору, а кал став ахолічний. Концентрація якої речовини в сироватці крові зросте найбільше?

Кон'югованого білірубіну

Вільного білірубіну

Мезобілірубіну

Білівердину

Уробіліногену

5019 / 6307
До лікаря-стоматолога звернувся пацієнт зі скаргами на тризм (не може відкривати рот повністю). З анамнезу чоловіка відомо про колоту рану нижньої кінцівки. Для якого інфекційного захворювання характерні такі симптоми?

Правець

Ранева анаеробна інфекція

Кашлюк

Туляремія

Сказ

5020 / 6307
У пацієнта після перенесеного порушення мозкового кровообігу розвинувся параліч. Оберіть антихолінестеразний засіб для призначення хворому.

Ацеклітин

Бензогексоній

Кордіамін

Прозерин

Метацин

5021 / 6307
У чоловіка віком 52 роки спостерігаються лихоманка та біль у суглобах. Обидва перші плюсно-фалангові суглоби деформовані, набряклі, почервонілі. Сечовина крові висока. Пацієнту діагностовано подагру. Який основний патогенетичний фактор розвитку цього захворювання?

Гіперазотемія

Гіперурикемія

Аргінінсукцинатурія

Гіпераміноацидемія

Цитрулінурія

5022 / 6307
У чоловіка внаслідок патологічного процесу збільшена товщина альвеоло-капілярної мембрани. Безпосереднім наслідком цього буде зменшення у хворого:

Дифузійної здатності легень

Хвилинного об'єму дихання

Альвеолярної вентиляції легень

Кисневої ємності крові

Резервного об'єму видиху

5023 / 6307
До лікаря звернувся пацієнт із періодонтитом нижнього кутнього зуба. Виявлено, що запальний процес уже поширився на лімфатичні вузли. Які лімфовузли найперші були уражені запальним процесом?

Бічні шийні

Піднижньощелепні

Лицеві

Підборідні

Передні шийні

5024 / 6307
Мама одинадцятирічної дівчинки скаржиться на слабкість та набряклість обличчя її дитини уродовж 3-х днів, хоча до цього вона завжди була здоровою та активною. Під час фізикального обстеження виявлено генералізований набряк обличчя, якщо натискати на набряк нижніх кінцівок, залишається ямка, яка поступово згладжується. Під час збору анамнезу дівчинка зазначає пінистий вигляд сечі, але заперечує домішки крові у сечі, ніктурію або біль під час сечовиділення. Лабораторні дослідження виявили протеїнурію та мікрогематурію. Що з нижченаведеного є ймовірною причиною змін у лабораторному аналізі сечі?

Підвищення гідростатичного тиску в капсулі Шумлянського-Боумена

Підвищення онкотичного тиску плазми крові

Підвищення проникності стінки клубочкового капіляра

Підвищення гідростатичного тиску в клубочку

5025 / 6307
Гормони кортикостероїди регулюють процеси адаптації цілісного організму до змін умов навколишнього середовища та підтримання внутрішнього гомеостазу. Який гормон активує гіпоталамо-гіпофізарно-надниркову систему?

Соматоліберин

Соматостатин

Тироліберин

Кортиколіберин

Кортикостатин

5026 / 6307
Чотирирічна дитина померла від легенево-серцевої недостатності. Під час аутопсії виявлено ущільнення підшлункової залози, у бронхах обтураційні ателектази та бронхоектази, у печінці - жирова інфільтрація. Мікроскопічно в підшлунковій залозі спостерігається: кістозно розширені протоки, атрофія залозистої паренхіми, дифузний фіброз та лімфо-гістіоцитарна інфільтрація. Для якого захворювання характерні такі результати аутопсії?

Хвороба Гіршпрунга

Інфекційна фетопатія

Рамстопатія

Ембріонатія

Фетальний муковісцидоз

5027 / 6307
Ключовою реакцією синтезу жирних кислот є утворення малоніл-КоА. Який метаболіт є джерелом синтезу малоніл-KoA?

Сукциніл-КоА

Малонат

Цитрат

Ацетил-КоА

Ацил-КоА

5028 / 6307
У чоловіка діагностовано алкаптонурію. Укажіть фермент, дефект якого є причиною цієї патології.

Піруватдегідрогеназа

Глутаматдегідрогеназа

ДОФА-декарбоксилаза

Оксидаза гомогентизинової кислоти

Фенілаланінгідроксилаза

5029 / 6307
У пацієнта на шкірі обличчя поступово розвинулася бляшка з некрозом і виразкою в центрі. Під час патогістологічного дослідження біоптату виявлено розростання атипових епітеліальних клітин із великою кількістю патологічних мітозів. Для якого захворювання характерні такі клініко-лабораторні показники?

Саркома

Рак шкіри

Фіброма

Папілома

Трофічна виразка

5030 / 6307
У пацієнта віком 67 років клінічний діагноз: хронічний бронхіт, пневмосклероз, серцево-легенева недостатність. З підозрілої ділянки слизової оболонки правого бронха взято біоптат. Гістологічно встановлено: клітинний і тканинний атипізм, поява структур, що мають вигляд `ракових перлин`. Для якого захворювання характерні ці гістологічні зміни?

Бронхоектаз

Плоскоклітинна метаплазія слизової бронху

Плоскоклітинний рак бронху з ороговінням

Гострий бронхіт

Хронічний поліпозний бронхіт

5031 / 6307
Під час розтину тіла померлої півторарічної дитини виявлено: геморагічні висипання на шкірі, помірна гіперемія та набряк слизової носоглотки, дрібні крововиливи в слизових оболонках і внутрішніх органах, різкі дистрофічні зміни в печінці, міокарді, гострий некротичний нефроз, масивні крововиливи в наднирниках. Для якого захворювання характерні ці симптоматичні явища?

Скарлатина

Дифтерія

Висипний тиф

Менінгококова інфекція

Kip

5032 / 6307
Типовими проявами харчового отруєння спричиненого C. botulinum є двоїння в очах, порушення ковтання та дихання. Унаслідок якого процесу розвиваються ці симптоми?

Активації аденілатциклази

Дії екзотоксину

Дії ентеротоксину

Адгезії збудника до рецепторів на ентероцитах

Розвитку ентеротоксичного шоку

5033 / 6307
Для дослідження білків сироватки крові застосовують різні фізичні та фізико-хімічні методи. За допомогою якого методу можна розділити альбуміни і глобуліни сироватки крові?

Полярографії

Діалізу

Спектрофотометрії

Електрофорезу

Рефрактометрії

5034 / 6307
Призначення доксицикліну гідрохлориду зумовило порушення симбіозу мікробної флори в кишечнику. Вкажіть тип порушень при антибіотикотерапії.

Бактеріоз

Дизбактеріоз

Сенсибілізація

Ідіосинкразія

Суперінфекція

5035 / 6307
У препараті паренхіматозного органа розрізняють нечітко відмежовані часточки шестигранної форми, у центрі яких знаходиться вена, а в міжчасточковій сполучній тканині проходять тріади (артерія, вена і вивідна протока). Вкажіть який цей орган:

Підшлункова залоза

Селезінка

Щитоподібна залоза

Печінка

Тимус

5036 / 6307
У новонародженої дитини спостерігається зниження інтенсивності смоктання, часте блювання, гіпотонія. У сечі та крові значно підвищена концентрація цитруліну. Який метаболічний процес порушений?

Гліколіз

Цикл Корі

ЦТК

Глюконеогенез

Орнітиновий цикл

5037 / 6307
Після дегельмінтизації у пацієнта віком 35 років під час випорожнення кишечника виділився стрічковий хробак довжиною 3,5 м. Дослідження калу виявило сколекс із чотирма присосками та гачками. Зрілі членики гельмінта нерухомі та мають до 12 бічних гілок матки. Яке захворювання ймовірно у чоловіка?

Teніоз

Дифілоботріоз

Опісторхоз

Ехінококоз

5038 / 6307
Пацієнт віком 48 років перебуває у стані непритомності. Перед цим пацієнт кілька разів втрачав свідомість із появою судом. На ЕКГ зубці Р та деформовані комплекси QRS не пов'язані між собою. Частота скорочень передсердь близько 70/хв, шлуночків - 25-30/хв. Яка аритмія виникла у пацієнта?

Внутрішньопередсердна блокада

Внутрішньошлуночкова блокада

Повна передсердно-шлуночкова блокада

Передсердно-шлуночкова блокада 1 ступеня

Передсердно-шлуночкова блокада 2 ступеня

5039 / 6307
Під час виконання операції на черевній порожнині хірург виділив зв'язку, що проходить від передньої стінки черевної порожнини (пупка) до нижньої поверхні печінки. Визначте, яка це зв'язка печінки.

Венозна

Кругла

Трикутна

Серпоподібна

Вінцева

5040 / 6307
Пацієнту, який хворіє на легеневий туберкульоз, призначено найбільш ефективне специфічне лікування - протитуберкульозний антибіотик. Укажіть цей препарат.

Бактрім

Доксициклін

Рифампіцин

Стрептоцид

Фуразолідон

5041 / 6307
У жінки віком 40 років під час обстеження виявлено підвищений рівень основного обміну. Надлишок якого з нижченаведених гормонів зумовлює цей стан?

Соматостатину

Тиреокальцитоніну

Альдостерону

Трийодтироніну

Глюкагону

5042 / 6307
Пацієнту віком 35 років під час хірургічного втручання ввели препарат із групи деполяризуючих міорелаксантів. Укажіть, який засіб було введено.

Атракуріум

Дитилін

Рокуроній

Аміназин

Прозерин

5043 / 6307
Під час обстеження жінки віком 56 років, яка хворіє на цукровий діабет 1 го типу, виявлено порушення білкового обміну. Під час лабораторного дослідження крові це виявляється аміноацидемією, а клінічно - уповільненням загоєння ран і зменшенням синтезу антитіл. Який із наведених нижче механізмів викликає розвиток аміноацидемії?

Гіперпротеїнемія

Підвищення онкотичного тиску в плазмі крові

Зменшення концентрації амінокислот у крові

Збільшення ліпопротеїдів високої щільності

Підвищення протеолізу

5044 / 6307
Унаслідок закупорки загальної жовчної протоки (встановлено рентгенологічно) надходження жовчі в дванадцятипалу кишку припинилося. Яке порушення ліпідного обміну може відбутися?

Гідролізу вуглеводів

Секреції соляної кислоти

Гальмування слиновиділення

Емульгування жирів

Всмоктування білків

5045 / 6307
У жінки віком 45 років артеріальна гіпертензія з високою концентрацією ангіотензину-2 у крові. Який із нижченаведених антигіпертензивних засобів треба рекомендувати у цьому разі?

Лізиноприл

Верапаміл

Празозин

Метопролол

Анаприлин

5046 / 6307
До медико-генетичної консультації звернулися батьки п'ятирічної хворої дівчинки. Під час дослідження каріотипу виявили 46 хромосом. Одна із хромосом 15-ї пари була довша за звичайну, тому що до неї приєдналася ділянка хромосоми з 21-ї пари. Який вид мутації у цієї дівчинки?

Транслокація

Дуплікація

Нестача

Делеція

Інверсія

5047 / 6307
Чоловіку, який хворіє на паркінсонізм, призначили препарат Леводопа. Приймання ліків швидко поліпшило стан пацієнта. Який механізм дії цього лікарського засобу?

Антихолінестеразна дія

Блокада М-холінорецепторів

Стимуляція синтезу дофаміну

Стимуляція дофамінових рецепторів

Стимуляція М-холінорецепторів

5048 / 6307
У чоловіка віком 60 років, який хвоpий на хронічний гепатит, часто спостерігаються кровотечі з носа і ясен, спонтанно з'являються геморагічні висипання на шкірі і слизових оболонках. Що є причиною виникнення цих явищ?

Поява в крові макроглобулінів і кріоглобулінів

Зменшення синтезу протромбіну і фібриногену

Зменшення в крові рівня холінестерази

Зменшення утворення сироваткових альбумінів

Підвищення вмісту амінотрансфераз

5049 / 6307
Пацієнт скаржиться на різке збільшення діурезу (до 5-7 л сечі за добу). Обстеження виявило, що у пацієнта зменшена секреція вазопресину. Які клітини мають недостатню секреторну активність?

Ендокриноцити проміжної частки гіпофіза

Клітини туберальної частини гіпофіза

Нейросекреторні клітини гіпоталамуса

Ендокриноцити передньої частки гіпофіза

Пітуїцити

5050 / 6307
Дев'ятирічний хлопчик захворів гостро: з'явився біль у горлі, підвищилася температура до 39,5°С. Наступного дня виявлено дифузні висипи на усій шкірі, за винятком носогубного трикутника. Під час огляду порожнини рота спостерігаються малиновий язик, «палаючий зів», некротична ангіна. Який діагноз найбільш імовірний?

Дифтерія

Грип

Kip

Скарлатина

Менінгококцемія

5051 / 6307
Унаслідок руйнування певних структур стовбура мозку тварина втратила орієнтувальні рефлекси у відповідь на сильні світлові подразники. Які структури мозку було зруйновано?

Передні горбки чотиригорбкового тіла

Задні горбки чотиригорбкового тіла

Червоні ядра

Чорна речовина

Вестибулярні ядра

5052 / 6307
Від хворого виділені нерухомі бактерії овоїдної форми з біполярним забарвленням, які в організмі утворюють ніжну капсулу, на агарі утворюють колонії з мутно-білим центром, оточеним фестончатою облямівкою, що нагадує мереживо. Продукують «мишачий токсин». Для збудника якого інфекційного захворювання характерні ці властивості?

Кашлюку

Чуми

Сибірки

Туляремії

Бруцельозу

5053 / 6307
Під час обстеження зовнішніх статевих органів у хлопчика виявлено повне закриття уретри зверху. Сечівник залишається відкритим знизу і має вигляд невеликої щілини. Який вид аномалії розвитку зовнішніх статевих органів спостерігається у дитини?

Парафімоз

Епіспадія

Фімоз

Гіпоспадія

Гермафродитизм

5054 / 6307
У разі спадкового дефекту ферментів синтезу гема хворий має підвищену чутливість шкіри до сонячного світла, сечу червоного кольору. Накопичення яких метаболітів обміну гемоглобіну викликає ці симптоми?

Мезобіліногенів

Стеркобіліногенів

Білірубіну

Порфіриногенів

Уробіліногенів

5055 / 6307
Тромбоз коронарної артерії спричинив розвиток інфаркту міокарда. Які механізми ушкодження клітин є домінуючими під час цього захворювання?

Протеїнові

Ацидотичні

Кальцієві

Електролітно-осмотичні

Ліпідні

5056 / 6307
У шестирічної дитини спостерігається затримка росту, порушення процесів окостеніння, декальцифікація зубів. Яка ймовірна причина цього?

Інсулінова недостатність

Дефіцит вітаміну D

Авітаміноз С

Гіпертиреоз

Зменшення продукції глюкагону

5057 / 6307
У родині здорових батьків, які нещодавно повернулися з Африки, народилася дитина з ознаками анемії та невдовзі померла. Обстеження виявило, що еритроцити дитини мають аномальну півмісяцеву форму. Хвороба успадковується за аутосомно-рецесивним типом. Визначте можливі генотипи батьків.

aa x aa

Аа x Aa

AA х AA

Aa x аa

Aa х AA

5058 / 6307
У пацієнта порушена моторна функція язика. З патологією якого нерва це пов'язано?

Трійчастого

Під'язикового

Лицьового

Блукаючого

Язикоглоткового

5059 / 6307
У чоловіка на тлі інфаркту міокарда виникла пароксизмальна шлуночкова тахікардія. Який із нижченаведених протиаритмічних засобів треба обрати, щоб не зменшити серцевий викид?

Новокаїнамід

Лідокаїну гідрохлорид

Верапаміл

Анаприлін

Калію хлорид

5060 / 6307
У пацієнтки під час мікроскопії мазків, які приготовили з виділень із піхви, виявлено грамнегативні бобовидні диплококи. Який попередній діагноз можна встановити?

Гонорея

Мікоплазмоз

Сифіліс

Токсоплазмоз

Хламідіоз

5061 / 6307
Пацієнт за чотири місяці після перенесеного інсульту розпочав реабілітацію. У нього об'єктивно спостерігаються ознаки центрального паралічу, рухи у правій руці і нозі відсутні, тонус м'язів на цих кінцівках підвищений, місцеві рефлекси посилені. Яким терміном можна описати стан пацієнта?

Моноплегія

Тетраплегія

Геміплегія

Параплегія

Функціональний параліч

5062 / 6307
Чоловік віком 35 років скаржиться на нежить та втрату відчуття запаху упродовж тижня. Під час обстеження виявлено, що у носовій порожнині велика кількість слизу, що вкриває слизову оболонку та блокує нюхові рецептори. Де в носовій порожнині розташовані ці рецептори?

Нижня носова раковина

Верхня носова раковина

Присінок носу

Загальний носовий хід

Середня носова раковина

5063 / 6307
У чоловіка, який хворів на мітральний стеноз, під час посмертного розтину тіла виявлено ущільнені легені бурого кольору. Для якого патологічного процесу в легенях характерні ці симптоми?

Гемосидерозу

Гемомеланозу

Гемохроматозу

Ліпофусцинозу

Жовтяниці

5064 / 6307
У людини під час переходу зі світлого приміщення в темне відбувається розширення зіниць. Який із наведених нижче рефлексів зумовлює цю реакцію?

Симпатичний умовний

Симпатичний безумовний

Парасимпатичний безумовний

Метасимпатичний

Парасимпатичний умовний

5065 / 6307
Трьохрічна дитина поїла полуниць. За деякий час у неї з'явилися висипання та свербіж. Що буде спостерігатися у лейкоцитарній формулі крові дитини?

Лімфоцитоз

Нейтрофільний лейкоцитоз

Езинофілія

Моноцитоз

Лімфоцитопенія

5066 / 6307
У пацієнта з ознаками кишкової інфекції (блювання, діарея, біль у черевній порожнині) упродовж трьох діб наростали ознаки інтоксикації, з'явилася папульозна висипка на відкритих ділянках тіла, яка поширилася на тулуб. Попередній діагноз: псевдотуберкульоз. Який метод лабораторної діагностики дає можливість підтвердити діагноз на першому тижні захворювання?

Мікроскопічний

Серологічний

Бактеріологічний

Алергічний

Біологічний

5067 / 6307
Серед лімфоцитів розрізняють популяцію клітин, що мають мембранні рецептори до IgM, під впливом специфічних антигенів активуються, мітотично розмножуються, диференціюються у плазматичні клітини, що виробляють антитіла (імуноглобуліни). Як називаються ці клітини?

Т-лімфоцити супресори

В-лімфоцити

Т-лімфоцити пам'яті

Т-лімфоцити кілери

5068 / 6307
Надмолекулярний мультиферментний комплекс, інтегрований у ліпідний шар внутрішньої мембрани мітохондрій, що створює умови для перебігу окисно-відновних реакцій, це:

Карбоксипептидаза

G-білок-трансдуктор

Гексокіназа

Дихальний ланцюг

Піруваткіназа

5069 / 6307
Під час розтину тіла чоловіка, який помер від набряку легень, у міокарді було виявлено велике вогнище жовто-сірого кольору, а в коронарній артерії свіжий тромб. Для якого захворювання характерні такі симптоматичні явища?

Атеросклероз

Кардіосклероз

Інфаркт міокарда

Кардіоміопатія

Міокардит

5070 / 6307
Трьохрічну дитину госпіталізовано до реанімаційного відділення інфекційної лікарні. Під час огляду спостерігається: стан важкий, шкіра та слизові оболонки сухі, тургор тканин знижений. З анамнезу відомо, що протягом доби після вживання недоброякісної їжі в дитини була профузна діарея та багаторазове блювання. Який вид порушення водно-сольового обміну розвинувся у дитини?

Ізоосмолярна гіпергідратація

Гіпоосмолярна дегідратація

Гіпоосмолярна гіпергідратація

Гіперосмолярна гіпергідратація

Ізоосмолярна дегідратація

5071 / 6307
Під час хірургічної операції на піднижньощелепній слинній залозі у пацієнта почалася кровотеча. Пошкодження якої артерії, топографічно пов'язаної з цією залозою, може викликати кровотечу?

Сонної

Поверхневої скроневої

Лицевої

Верхньощелепної

Висхідної глоткової

5072 / 6307
Жінка віком 65 років, яка хворіє на тромбофлебіт глибоких вен гомілки, раптово померла. Під час розтину тіла в загальному стовбурі та біфуркації легеневої артерії спостерігаються вільно лежачі червоні пухкі маси із тьмяною гофрованою поверхнею. Який патологічний процес у легеневій артерії виявлено?

Тканинну емболію

Жирову емболію

Тромбоз

Емболію сторонніми тілами

Тромбоемболію

5073 / 6307
Провідними симптомами первинного гіперпаратиреозу є остеопороз та ураження нирок із розвитком сечокам'яної хвороби. Які речовини складають основу каменів під час цього захворювання?

Сечова кислота

Фосфат кальцію

Холестерин

Цистин

Білірубін

5074 / 6307
У пацієнта віком 18 років після перенесеної пневмонії, яку лікували В-лактамним антибіотиком, виник кандидоз. Який протигрибковий засіб треба призначити?

Стрептоміцин

Ампіцилін

Бісептол

Флуконазол

Фталазол

5075 / 6307
До кардіологічного відділення госпіталізували пацієнта зі скаргами на постійний головний біль у потиличній ділянці, шум у вухах, запаморочення. Об'єктивно спостерігається: АТ 180/110 мм рт. ст., ЧСС - 95/хв. Рентгенологічно виявлено звуження однієї з ниркових артерій. Активація якої з нижченаведених систем викликала гіпертензивний стан у пацієнта?

Ренін-ангіотензивна

Кінінова

Симпатоадреналова

Імунна

Гемостатична

5076 / 6307
Білому щуру ввели під шкіру сулему в дозі 5 мг/кг маси тіла. За 24 години в плазмі крові концентрація креатиніну збільшилася в декілька разів. Який механізм ретенційної азотемії у цьому разі?

Збільшення утворення креатиніну в м'язах

Зниження клубочкової фільтрації

Зростання реабсорбції креатиніну

Збільшення секреції креатиніну в канальцях нирок

Зростання клубочкової фільтрації

5077 / 6307
На гістологічному препараті видно судину, стінка якої складається з ендотелію, базальної мембрани та пухкої сполучної тканини. Визначте тип судини.

Гемокапіляр

Артерія

Вена м'язового типу

Вена безм'язового типу

Лімфокапіляр

5078 / 6307
У пацієнта, який упродовж тривалого часу мав незбалансоване харчування з малою кількістю білка, розвинулася жирова інфільтрація печінки. Вкажіть речовину, відсутність якої у їжі могла бути причиною цього стану.

Аланін

Метіонін

Оцтова кислота

Біотин

Холестерин

5079 / 6307
За 6 годин після первинного посіву води на 1%-у пептонну воду, на поверхні середовища з'явилася ніжна голубувата плівка. Для збудника якого захворювання характерні такі культуральні властивості?

Псевдотуберкульозу

Чуми

Холери

Туберкульозу

Дизентерії

5080 / 6307
Під час дослідження вмісту дванадцятипалої кишки чоловіка виявлено найпростіші грушоподібної форми з парними ядрами та чотирма парами джгутиків. Між ядрами дві опірні нитки, з вентрального боку розташований присмоктувальний диск. Якого представника найпростіших виявили у пацієнта?

Трихомонаду кишкову

Трипаносому

Лейшманію

Токсоплазму

Лямблію

5081 / 6307
До приймального відділення госпіталізовано чоловіка у непритомному стані. Шкіра холодна, зіниці звужені, дихання утруднене, відзначається періодичність за типом Чейна-Стокса, артеріальний тиск знижений, сечовий міхур переповнений. Встановлено діагноз: отруєння морфіном. Який препарат треба застосувати як антагоніст?

Бемегрид

Цититон

Унітіол

Налоксон

Тіосульфат натрію

5082 / 6307
У жінки віком 64 роки порушені тонкі рухи пальців рук, розвинута м'язова ригідність, тремор. Лікар-невропатолог діагностував хворобу Паркінсона. Ураження яких структур головного мозку спричинило це захворювання?

Таламуса

Червоних ядер

Чорної субстанції

Мозочка

Ретикулярної формації

5083 / 6307
У пацієнта внаслідок черепно-мозкової травми знижена шкірна чутливість. Яка ділянка кори великого мозку ймовірно уражена?

Передня центральна звивина

Лобна ділянка кори

Задня центральна звивина

Потилична ділянка

Поясна звивина

5084 / 6307
Під час експерименту досліджували рефлекс згинання у спинальної жаби, який викликали шляхом подразнення поодинокими електричними імпульсами силою нижче порогової, але частота генерації цих імпульсів була такою, що рефлекс виявлявся. Який процес у нервових центрах спостерігається під час експерименту?

Порогова сумація

Пресинаптична сумація

Постсинаптична сумація

Послідовна (часова) сумація

Просторова сумація

5085 / 6307
Унаслідок тривалого перебування людини в горах на висоті 3000 м над рівнем моря у неї збільшилася киснева ємність крові. Посилене утворення в організмі яких речовин є безпосередньою причиною цього?

Лейкопоетинів

Катехоламінів

Еритропоетинів

2,3-дифосфогліцерату

Карбгемоглобіну

5086 / 6307
Виявлено ураження вірусом ВІЛ Т- лімфоцитів. Водночас фермент вірусу — зворотна транскриптаза (РНК-залежна ДНК-полімераза) - каталізує синтез:

і-РНК на матриці вірусного білку

Вірусної РНК на матриці ДНК

ДНК на матриці вірусної РНК

Вірусної ДНК на матриці ДHK

Вірусного білку на матриці вірусної РНК

5087 / 6307
У жінки віком 40 років хвороба Іценка-Кушинга стероїдний діабет. Під час біохімічного обстеження виявлено гіперглікемію та гіпохлоремію. Який із наведених нижче процесів активується у таких хворих найперше?

Глікогеноліз

Глюконеогенез

Реабсорбція глюкози

Транспорт глюкози в клітину

Гліколіз

5088 / 6307
У пацієнта віком 30 років під час лабораторного аналізу крові виявлено: кількість еритроцитів 6 на 10в12/л, гемоглобіну - 10,55 ммоль/л. Діагностовано хворобу Вакеза. Укажіть провідну ланку патогенезу.

Дефіцит заліза

Гіпоксія

Дефіцит вітаміну В12

Ацидоз

Гіперплазія еритроїдного відростка пухлинної природи

5089 / 6307
Який препарат треба призначити для індивідуальної хіміопрофілактики маляpiї?

Примахін

Лаферон

Хінгамін

Хлоридин

Пірантел

5090 / 6307
Симптоми пелагри (гіповітаміноз PP) особливо різко виражені у хворих із недостатнім білковим харчуванням, тому що попередником нікотинаміду в органíзмі людини є одна з незамінних амінокислот. Укажіть цю амінокислоту.

Аргінін

Гістидин

Треонін

Лізин

Триптофан

5091 / 6307
У пацієнта під час гематологічного дослідження виявлено: еритроцити - 2,8 на 10в12/л, Hb 80 г/л, КП 0,85, ретикулоцити - 0,1%, тромбоцити - 160 тис. в мкл, лейкоцити - 60 на 10^9/л, базофіли 2%, еозинофіли 8%, промієлоцити 5%, мієлоцити - 5%, юні - 16%, паличкоядерні - 20%, сегментоядерні - 34%, лімфоцити - 5%, моноцити - 5%. Для якого захворювання крові характерні такі результати дослідження?

Хронічний мієлолейкоз

Гострий мієлолейкоз

Гемолітична анемія

Гіпопластична анемія

Недиференційований лейкоз

5092 / 6307
Чоловіку віком 40 років, який хворіє на туберкульоз легень, призначено ізоніазид. Дефіцит якого вітаміну може розвинутися унаслідок тривалого вживання цього лікарського засобу?

Tiaміну

Фолієвої кислоти

Біотину

Кобаламіну

Піридоксину

5093 / 6307
У чоловіка спостерігаються періодичні напади серцебиття (пароксизми), сильне потовиділення, напади головного болю. Під час обстеження виявлено гіпертензію, гіперглікемію, підвищення основного обміну, тахікардію. Для якого захворювання наднирників характерні ці симптоми?

Гіпофункція кори наднирників

Первинний альдостеронізм

Гіперфункція мозкового шару

Гіперфункція кори наднирників

Гіпофункція мозкового шару

5094 / 6307
Чоловіку, який має онкологічне захворювання, буде проведено оперативне втручання на низхідному відділі ободової кишки. Укажіть найважливіше джерело його кровопостачання.

Нижня брижова артерія

Середня ободова артерія

Верхня брижова артерія

Селезінкова артерія

Черевний стовбур

5095 / 6307
Під час тривалого використання лікарського засобу у хворого можуть спостерігатися остеопороз, ерозія слизової шлунку, гіпокаліємія, затримка натрію і води в організмі, зменшення вмісту кортикотропіну у крові. Який препарат може викликати таку побічну дію?

Преднізолон

Дигоксин

Індометацин

Рифампіцин

Гіпотіазид

5096 / 6307
Для того щоб схуднути, жінка обмежувала кількість продуктів у харчовому раціоні. За 3 місяці в неї з'явилися набряки та збільшився діурез. Дефіцит яких компонентів їжі спричинив ці ускладнення?

Білків

Вуглеводів

Жирів

Вітамінів

Мінеральних речовин

5097 / 6307
Під час видалення жовчного міхура в лігатуру замість однієї а. cystyca була вигнута артерія, перев'язка якої зумовила некроз правої частки печінки та смерть пацієнта. Яка артерія була помилково перев'язана разом з a. cystyca?

A. gasro-duodenalis

Ramus sinister a. hepatica propria

Ramus dexter a. hepatica propria

A. hepatica communis

A. pancreato-duodenalis sup

5098 / 6307
У гістологічному препараті паренхіма органа представлена лімфоїдною тканиною, яка утворює лімфатичні вузлики. Вони розташовуються дифузно і містять центральну артерію. Яке анатомічне утворення має таку морфологічну будову?

Тимус

Печінка

Червоний кістковий мозок

Селезінка

Лімфатичний вузол

5099 / 6307
Під час експерименту подразнюють гілочки симпатичного нерва, які іннервують серце. Це спричинило збільшення сили серцевих скорочень, тому що через мембрану типових кардіоміоцитів збільшився:

Вхід іонів кальцію та калію

Вихід іонів кальцію

Вихід іонів калію

Вхід іонів кальцію

Bxiд ioнів кaлiю

5100 / 6307
У пацієнта із порушенням серцевого ритму під час обстеження на ЕКГ спостерігаються: ЧСС 60/хв, інтервал РQ подовжений, періодичне випадіння комплексу QRS. Яке порушення серцевого ритму виявлено?

Синдром слабкості синусового вузла

Неповна АѴ блокада II ступеня

Неповна AV блокада I ступеня

Блокада правої ніжки пучка Гіса

Повна AV блокада

5101 / 6307
Відомо, що шкіра людини дуже міцна на розрив і складається з епітеліальної та двох видів сполучної тканин. Яка з нижченаведених тканин забезпечує міцність шкіри?

Перехідний епітелій

Пухка сполучна тканина

Багатошаровий плоский епітелій

Одношаровий епітелій

Щільна неоформлена сполучна

5102 / 6307
Жінка віком 50 років, яка довго хворіла на гломерулонефрит, при наростаючих явищах інтоксикації (уремії) померла. Під час розтину тіла виявлено: стінка товстої кишки потовщена, на слизовій оболонці сірувато-білувата плівка, яка тісно зрощена з підлеглою тканиною. На плеврі наявні сіруваті плівки, які легко відділяються. Які види запалення виявлено?

У товстій кишці - серозний катар, на плеврі - крупозне

У товстій кишці дифтеритичне, на плеврі - крупозне

У товстій кишці - гнійне, на плеврі - крупозне

У товстій кишці - крупозне, на плеврі - дифтеритичне

У товстій кишці - гнійне, на плеврі - дифтеритичне

5103 / 6307
Депресії та емоційні розлади є наслідком нестачі у головному мозку норадреналіну, серотоніну та інших біогенних амінів. Збільшення їх умісту в синапсах можна досягти за допомогою антидепресантів, які гальмують фермент:

Фенілаланін-4-монооксигеназа

Оксидаза L-амінокислот

Оксидаза Д-амінокислот

Моноамінооксидаза

Діамінооксидаза

5104 / 6307
Пацієнту призначили ферментний препарат, що містить цитохром С. Який процес посилюється під дією цього ферменту?

Синтез фосфоліпідів

Гліколіз

Тканинне дихання

Синтез колагену

Цикл трикарбонових кислот

5105 / 6307
У пацієнта з клінічними ознаками первинного імунодефіциту виявлено порушення функції антигенпрезентації імунокомпетентним клітинам. Дефект структур яких клітин можливий у цьому разі?

В-лімфоцити

Макрофаги

Т-лімфоцити

Фібробласти

О-лімфоцити

5106 / 6307
До терапевтичного відділення лікарні госпіталізовано пацієнта з діагнозом: виразкова хвороба шлунку з гіперацидним синдромом. Препарат із якої групи лікарських засобів треба використати у цьому разі для комплексної терапії?

Блокатори кальцієвих каналів

Блокатори Н2-гістамінових рецепторів

Стероїдні протизапальні засоби

Блокатори H1-гістамінових рецепторів

Нестероїдні протизапальні засоби

5107 / 6307
У жінки віком 30 років з'явилися ознаки вірилізму (ріст волосся на тілі, облисіння скронь, порушення менструального циклу). Гіперпродукція якого гормону може спричинити такий стан?

Релаксину

Окситоцину

Естріолу

Прогестерону

Тестостерону

5108 / 6307
Незалежно від расової чи етнічної належності у людини розвивається комплекс морфофункціональних, біохімічних, імунологічних ознак, які обумовлюють кращу біологічну пристосованість людини до відповідного фізичного середовища. Який тип біологічної реакції представлений у людини?

Тропічний тип

Адаптивний тип

Гірський тип

Арктичний тип

Тип зони помірного клімату

5109 / 6307
У гематологічному відділенні хворому на лейкоз лікар-онколог призначив 5-фторурацил. Який механізм дії цього препарату?

Інгібує трансляцію

Стимулює ДНКазу

Інгібує синтез ДНК

Інгібує транскрипцію

Прискорює реплікацію

5110 / 6307
На електронномікроскопічній фотографії поперечного зрізу волокна чітко візуалізуються декілька осьових циліндрів з мезаксонами. Яке це волокно?

Нервове мієлінове

Колагенове

Ретикулярне

Еластичне

Нервове безмієлінове

5111 / 6307
У пацієнта, який тривалий час хворів на переміжну кульгавість, об'єктивно спостерігається: шкіра пальців стопи суха, чорного кольору, нагадує тканину мумії. На невеликій відстані від почорнілої ділянки розташована двоколірна лінія (червоний колір прилягає до майже незмінених тканин, а біло-жовтий колір --- до змінених тканин). Який вид некрозурозвинувся у пацієнта?

Інфаркт

Пролежень

Гангрена

Секвестр

---

5112 / 6307
Пацієнту призначили ферментний препарат, що містить цитохром С. Який процес посилюється під дією цього ферменту?

Тканинне дихання

Цикл трикарбонових кислот

Гліколіз

Синтез колагену

Синтез фосфоліпідів

5113 / 6307
У альпініста, що піднявся на висоту 5200 м, розвинувся газовий алкалоз. Що є причиною його розвитку?

Гіповентиляція легенів

Гіпоксемія

Гіпероксемія

Зниження температури навколишньогосередовища

Гіпервентиляція легенів

5114 / 6307
Піддослідній тварині ввели блокатор цитохромоксидази, що призвело до її миттєвої смерті. Яка сполука калію може викликати таку реакцію?

Сульфат

Цианід

Оксалат

Фосфат

Нітрит

5115 / 6307
У хворого внаслідок хронічного обструктивного бронхіту на тлі задишки, тахікардії та ціанозу під час дослідження газового складу крові виявлено розвиток гіпоксемії та гіперкапнії. Яке порушення зовнішнього дихання спостерігається у хворого?

Гіперперфузія

Гіпоперфузія

Гіповентиляція

Гіпервентиляція

Гіпердифузія

5116 / 6307
Під час розтину тіла дівчинки 9-ти років у верхівці правої легені субплеврально було знайдене вогнище казеозного некрозу діаметром 15 мм, біфуркаційні лімфатичні вузли збільшені, містять дрібні вогнища некрозу коагуляційного типу. Мікроскопічно: у легеневому вогнищі та в лімфатичних вузлах навколо некротичних мас розташовані епітеліоїдні клітини, лімфоцити та поодинокі багатоядерні гігантські клітини. Діагностуйте захворювання:

Гематогенний генералізований туберкульоз

Вторинний фіброзно-вогнищевий туберкульоз

Гематогенний туберкульоз з переважним ураженням легень

Вторинний вогнищевий туберкульоз

Первинний туберкульоз

5117 / 6307
Патогенні бактерії здатні активно проникати у внутрішнє середовище організму та інтенсивно поширюватися в тканинах. Який фермент забезпечує інвазивні властивості бактерій?

Каталаза

Лактаза

Гіалуронідаза

Оксидоредуктаза

Плазмокоагулаза

5118 / 6307
У хворого після тривалого психоемоційного напруження спостерігається підвищення артеріального тиску, що супроводжується серцебиттям, кардіалгіями, головним болем, запамороченням. Домінуючим у формуванні артеріальної гіпертензії у даному випадку є збільшення:

Серцевого викиду

Частоти серцевих скорочень

Тонусу артеріол

Об’єму циркулюючої крові

Тонусу венул

5119 / 6307
У неврологічне відділення з приводу мозкового крововиливу поступив хворий 62-х років. Об’єктивно: стан важкий. Спостерігається наростання глибини і частоти дихання, а потім його зменшення до апное, після чого цикл дихальних рухів відновлюється. Який тип дихання у хворого?

Апнейстичне

Біота

Гаспінг-дихання

Чейна-Стокса

Кусмауля

5120 / 6307
У дитини, що померла від дифтерії з явищами серцевої недостатності, на розтині виявлено, що порожнини серця розширені в поперечнику; м’яз серця тьмяний, в’ялий, на розрізі строкатий, з жовтуватими ділянками. У цитоплазмі деяких кардіоміоцитів зі збереженою цитоплазмою спостерігаються дрібні вакуолі. На заморожених зрізах вакуолі забарвлюються у оранжевий колір суданом 3. Який вид дистрофії виявлений у кардіоміоцитах?

Вуглеводна

Гідропічна

Балонна

Жирова

Гіаліново-крапельна

5121 / 6307
На аутопсії тіла жінки, що хворіла на хронічну дизентерію, в ході гістологічного дослідження внутрішніх органів в стромі та паренхімі міокарда, нирок, в слизовій оболонці шлунка та в сполучній тканині легень виявлені аморфні відкладення фіолетового кольору, що дають позитивну реакцію за Коссом. Яке ускладнення розвинулося у хворої?

Амілоїдоз

Метаболічне звапніння

Гіаліноз

Дистрофічне звапніння

Метастатичне звапніння

5122 / 6307
У жінки при профілактичному огляді виявили пухлину молочної залози. Результати біопсії підтвердили наявність злоякісної пухлини. Який основний механізм інфільтративного росту злоякісної пухлини?

Здатність пухлинних клітин до амебоїдного руху

Підвищення ферментативної активності лізосом

Порушення контактного гальмування

Збільшення адгезивності пухлинних клітин

Підвищення активності кейлонів в клітині

5123 / 6307
У дитини 6-ти місяців спостерігається різке відставання в психомоторному розвитку, бліда шкіра з екзематозними змінами, біляве волосся, блакитні очі, напади судом. Найточніше встановити діагноз у цієї дитини дозволить визначення в крові і сечі концентрації такої речовини:

Лейцин

Триптофан

Валін

Гістидин

Фенілпіруват

5124 / 6307
Глікоген, що надійшов з їжею, гідролізується у шлунково-кишковому тракті. Який кінцевий продукт утворюється в результаті цього процесу?

Глюкоза

Фруктоза

Лактоза

Галактоза

Лактат

5125 / 6307
У чоловіка 48-ми років виявлено порушення периферичного кровообігу з обмеженням припливу артеріальної крові, при цьому має місце зблідніння даної ділянки, зниження місцевої температури. Це порушення називається:

Ішемія

Сладж

Реперфузійний синдром

Венозна гіперемія

Стаз

5126 / 6307
Під час дослідження каріотипу п'ятирічного хлопчика виявлено 46 хромосом. Одна з хромосом 15-ї пари довша від звичайної, тому що до неї приєднана ділянка хромосоми з 21-ї пари. Укажіть вид мутації, що має місце в цього хлопчика:

Дуплікація

Делеція

Поліплоїдія

Інверсія

Транслокація

5127 / 6307
До травматологічного пункту доставлено хворого з пошкодженням м’язів нижніх кінцівок. За рахунок яких клітин можлива репаративна регенерація м’язових волокон і відновлення функції м’язів?

Плазмоцити

Міосателітоцити

Фібробласти

Адипоцити

Ендотеліоцити

5128 / 6307
До офтальмолога звернулася жінка зі скаргами на шкірний свербіж і набряклість повік. При обстеженні виявлено членистоноге червоподібної форми величиною 0,4 мм. Суцільний щиток вкриває передню частину тіла, тіло має поперечну смугастість. Ноги короткі, лапки з двома кігтиками. Який діагноз може встановити лікар?

Педікульоз

Фтиріоз

Демодекоз

Факультативний міаз

Короста

5129 / 6307
Тромбоз коронарної артерії спричинив розвиток інфаркту міокарда. Які механізми ушкодження клітин є домінуючими під час цього захворювання?

Кальцієві

Ацидотичні

Електролітно-осмотичні

Ліпідні

Протеїнові

5130 / 6307
У пацієнта спостерігається запальний гнійний процес шкіри першого міжпальцевого проміжку ноги. Які лімфатичні вузли є регіонарними для цієї ділянки та реагують болем і припухлістю?

Задні великогомілкові та підколінні

Поверхневі та глибокі пахвинні

Передні великогомілкові

Зовнішні клубові

Поверхневі пахвинні

5131 / 6307
У новонародженої дитини спостерігається зниження інтенсивності смоктання, часте блювання, гіпотонія. У сечі та крові значно підвищена концентрація цитруліну. Який метаболічний процес порушений?

Глюконеогенез

Цикл Корі

Гліколіз

Орнітиновий цикл

ЦТК

5132 / 6307
Після проведеної операції на шиї хворий втратив чутливість в її передній ділянці. Яка гілка шийного сплетення була пошкоджена під час операції?

Надключичні нерви

Великий вушний нерв

Малий потиличний нерв

Поперечний нерв шиї

Шийна петля

5133 / 6307
У гістопрепараті яєчника жінки визначаються структури, що мають велику порожнину. Овоцит I порядку в них оточений прозорою оболонкою, променистим вінцем і розташований у яйценосному горбику, стінка утворена шаром фолікулярних клітин і текою. Вкажіть, якій структурі яєчника належать дані морфологічні ознаки:

Примордіальний фолікул

Жовте тіло

Атретичне тіло

Зрілий (третинний) фолікул

Первинний фолікул

5134 / 6307
Основними тригерами, що вмикають ефекторні системи клітини у відповідь на дію гормонів, є протеїнкінази, які змінюють каталітичну активність певних регуляторних ферментів шляхом АТФ-залежного фосфорилювання. Який з наведених ферментів є активним у фосфорильованій формі?

Глікогенсинтаза

ГОМГ-КоА-редуктаза

Ацетил-КоА-карбоксилаза

Глікогенфосфорилаза

Піруваткіназа

5135 / 6307
По приїзді групи експертів на місце злочину виявлено тіло без ознак життя. В ході дослідження крові загиблого виявлена велика концентрація іонів ціанової кислоти. Інгібування якого комплексу дихального ланцюга мітохондрій стало причиною смерті?

V

II

I

III

IV

5136 / 6307
Унаслідок радіаційного випромінювання були ушкоджені стовбурові гемопоетичні клітини. Утворення яких клітин сполучної тканини буде порушено?

Адипоцити

Меланоцити

Макрофаги

Остеобласти

Фібробласти

5137 / 6307
У пацієнта хромосомна хвороба --- синдром Клайнфельтера. Загальна кількість хромосом у нього становить 47 (каріотип XXY). У соматичних клітинах виявляється статевий хроматин, кількість якого дорівнює числу X-хромосом мінус 1. Як називається статевий хроматин у соматичних клітинах?

Тільця Князькова-Деле

Тільця Жоллі

Тільця Барра

Тільця Меллорі

Кільця Кебота

5138 / 6307
Жінка 35-ти років розпочала голодувати. Депо яких поживних речовин використовується у початковий період голодування і як при цьому змінюється дихальний коефіцієнт (ДК)?

Білки, ДК наближається до 1

Вуглеводи, ДК наближається до 1

Білки, ДК наближається до 0,7

Жири, ДК наближається до 0,72

Жири, ДК наближається до 0,85

5139 / 6307
Під час обстеження у пацієнта діагностовано поліноз. Яким способом можна виконати специфічну десенсибілізацію?

Уведенням алергену в мікродозах

Уведенням фізіологічного розчину

Уведенням глюкокортикоїдів

---

Уведенням антигістамінних препаратів

5140 / 6307
Фермент лецитин-холестерин-ацилтрансфераза (ЛХАТ) каталізує реакцію утворення ефірів холестерину за рахунок перенесення залишку жирної кислоти з позиції С-2 холінфосфатиду (лецитину) на холестерин. З якою жирною кислотою холестерин утворює ефіри?

Міристиновою

Лауриновою

Пальмітиновою

Стеариновою

Лінолевою

5141 / 6307
У сироватці крові новонародженого виявлено антитіла до вірусу кору. Про наявність якого імунітету це свідчить?

Штучний пасивний

Природний активний

Штучний активний

Спадковий, видовий

Природний пасивний

5142 / 6307
У клітині, у якій відбувається мітотичний поділ, спостерігається розходження дочірніх хроматид до полюсів клітини. Визначте стадію мітозу.

Анафаза

Метафаза

Інтерфаза

Профаза

Телофаза

5143 / 6307
Паренхіма аденогіпофізу представлена трабекулами, утвореними залозистими клітинами. Серед аденоцитів є клітини з гранулами, які забарвлюються основними барвниками і містять глікопротеїди. Які це клітини?

Мамотропоцити

Соматотропоцити

Гонадотропоцити, тиротропоцити

Меланотропоцити

Хромофобні клітини

5144 / 6307
У хворого на ЕКГ виявлено, що тривалість інтервалу RR дорівнює 1,5 с, частота серцевих скорочень - 40/хв. Що є водієм ритму серця?

Синусовий вузол

Пучок Гіса

Права ножка Гіса

Ліва ножка Гіса

Атріовентрикулярний вузол

5145 / 6307
Пацієнту зі стрептококовою інфекцією ясен було призначено препарат, що містить у своїй структурі $\beta$-лактамне кільце. Який препарат із наведених нижче відноситься до цієї групи?

Еритроміцин

Стрептоміцину сульфат

Левоміцетин

Бензилпеніцилін

Рифампіцин

5146 / 6307
За якої умови можливо виконати пункцію сечового міхура через передню черевну стінку, не займаючи очеревину?

Тільки у чоловіків

У разі спустошеного сечового міхура

У разі наповненого сечового міхура

Тільки у жінок

Тільки у дітей

5147 / 6307
До приймального відділення лікарні госпіталізували юнака у непритомному стані. Діагностовано отруєння морфіном. У нього спостерігається поверхневе та рідке дихання, що зумовлено пригніченням дихального центру. Який тип недостатності дихання виник у пацієнта?

Перфузійний

Вентиляційний обструктивний

Дифузійний

Вентиляційний дисрегуляторний

Вентиляційний рестриктивний

5148 / 6307
У пацієнта стоматологічного відділення виявлено хворобу Педжета, що супроводжується деградацією колагену. Вирішальним фактом для постановки діагнозу було виявлення у сечі хворого підвищеного рівня:

Аланіну

Триптофану

Оксипроліну

Серину

Аргініну

5149 / 6307
Внаслідок травми низу передньої стінки живота у хворої ушкоджена зв’язка, що розташована у пахвинному каналі. Яка це зв’язка?

Ligamentum ovarium proprium

Ligamentum lacunare

Ligamentum latum uteri

Ligamentum teres uteri

Ligamentum inguinale

5150 / 6307
До травматологічного пункту доставлено постраждалого після ДТП з дiагнозом: закритий перелом середньої третини стегна зі зміщенням. З метою репозиції кісткових уламків хворому введено 10 мл 2% розчину дитиліну в/в, внаслідок чого розвинулося тривале апное та міорелаксація. Дефіцитом якого ферменту зумовлена вказана фар- макогенетична ферментопатія?

Глюкозо-6-фосфатдегідрогеназа

Уридіндифосфоглюкуронова трансфераза

N-ацетилтрансфераза

Псевдохолінестераза

Метгемоглобінредуктаза

5151 / 6307
Чоловіку віком 35 років для обстеження очного дна був призначений атропіну сульфат у вигляді очних крапель. Для відновлення акомодації пацієнту закапали пілокарпіну гідрохлорид, але це не дало бажаного ефекту. Що є причиною відсутності ефекту?

Однобічний антагонізм

Звикання

Двобічний антагонізм

Тахіфілаксія

Синергізм

5152 / 6307
У пацієнта з діагнозом: пневмонія, спостерігається підвищення температури тіла. Яка біологічно активна речовина відіграє провідну роль у виникненні цього симптому захворювання?

Iнтерлейкін-I

Лейкотрієни

Гістамін

Брадикінін

Серотонін

5153 / 6307
У дитини, що страждає на пілоростеноз, що супроводжується частим блюванням, розвинулись ознаки зневоднення організму. Яка форма порушення кислотно-основного стану може розвиватися у даному випадку?

Газовий ацидоз

Негазовий алкалоз

Газовий алкалоз

Негазовий ацидоз

Метаболічний ацидоз

5154 / 6307
У біоптаті щитоподібної залози виявлено атрофію паренхіматозних елементів, дифузну інфільтрацію тканини залози лімфоцитами та плазматичними клітинами з утворенням в ній лімфоїдних фолікулів. Для якого захворювання є характерними наведені ознаки?

Тиреоїдит Хасімото

Хвороба Базедова

Тиреоїдит Ріделя

Ендемічний зоб

Аденома щитоподібної залози

5155 / 6307
Пацієнту для усунення кишкової коліки призначено препарат атропіну сульфат. Яке захворювання може бути протипоказанням для застосування цього препарату?

Гіпотонія

Глаукома

Синусова брадикардія

Запаморочення

Бронхіальна астма

5156 / 6307
Ключовою реакцією синтезу жирних кислот є утворення малоніл-КоА. Який метаболіт є джерелом синтезу малоніл-КоА?

Цитрат

Сукциніл-КоА

Ацил-КоА

Малонат

Ацетил-КоА

5157 / 6307
Хворому з артеріальною гіпертензією було призначено один з антигіпертензивних засобів. Артеріальний тиск нормалізувався, однак хворого почав турбувати постійний сухий кашель. Який з перерахованих препаратів має таку побічну дію?

Ніфедипін

Клофелін

Лізиноприл

Анаприлін

Резерпін

5158 / 6307
В ході експерименту було продемонстровано підвищення активності β-галактозидази після внесення лактози до культурального середовища з E.coli. Яка ділянка лактозного оперону стає розблокованою від репресору за цих умов?

Промотор

Структурний ген

Регуляторний ген

Оператор

Праймер

5159 / 6307
При анемії в периферичній крові визначаються дегенеративні і регенеративні форми еритроцитів. Які з наведених форм еритроцитів є регенеративними?

Гіперхромні еритроцити

Ретикулоцити

Сфероцити

Пойкілоцити

Мікроцити

5160 / 6307
Юнаку віком 18 років встановлено діагноз: хвороба Марфана. Під час клініко-лабораторного дослідження виявлено порушення розвитку сполучної тканини, будови кришталика ока, аномалії серцево-судинної системи, арахнодактилія. Який варіант взаємодії генів різних алельних пар сприяє розвитку цього захворювання?

Кодомінування

Неповне домінування

Множинний алелізм

Плейотропія

Комплементарність

5161 / 6307
У пацієнта з аускультацією серця прослуховується патологічний шум у другому міжребер'ї праворуч від грудини. Ураження якого клапана можна припустити?

Легеневого

Аортального

--

Мітрального

Тристулкового

5162 / 6307
У хворого в обох щелепах рентгенологічно виявлено численні дефекти у вигляді гладкостінних округлих отворів. При гістологічному дослідженні - явища остеолізису і остеопорозу при явищах слабкого кісткоутворення. В сечі хворого знайдено білок Бенс-Джонса. Назвіть захворювання:

Хронічний еритромієлоз

Хронічний мієлолейкоз

Гострий недиференційований лейкоз

Мієломна хвороба

Гострий мієлолейкоз

5163 / 6307
У 70-ті роки науковці встановили, що причиною важкої жовтяниці новонароджених є порушення зв'язування білірубіну в гепатоцитах. Яка речовина використовується для утворення кон'югату?

Сечова кислота

Сірчана кислота

Глюкуронова кислота

Піровиноградна кислота

Молочна кислота

5164 / 6307
Пацієнту перед оперативним втручанням уведено дитилін (суксаметоній) і проведено інтубацію трахеї. Дефіцит якого ферменту в організмі пацієнта продовжує дію міорелаксанту?

Карбангідрази

Сукцинатдегідрогенази

N-ацетилтрансферази

K-Na-АТФ-ази

Псевдохолінестерази

5165 / 6307
Хворому перед екстракцією зуба була проведена провідникова анестезія лідокаїном, після введення якого з'явилися набряк і гіперемія навколо місця уколу, свербіння шкіри, загальна слабкість, гіпотензія, рухове збудження. Визначте, як називається ускладнення, що виникло:

Алергічна реакція

Лікарська залежність

Толерантність

Тахіфілаксія

Токсична дія

5166 / 6307
На розтині в серці виявлено наступні зміни: великий осередок некрозу білого кольору з червоною облямівкою, який захоплює всю товщу серцевого м’яза. На зовнішній оболонці серця - ознаки фіброзного перикардиту. Який найбільш імовірний діагноз?

Трансмуральний інфаркт міокарда

Інтрамуральний інфаркт міокарда

Субепікардіальний інфаркт міокарда

Субендокардіальний інфаркт міокарда

Міокардит

5167 / 6307
Лікар-хірург здійснює первинну хірургічну обробку глибокої різаної рани латеральної поверхні колінного суглоба. Яку зв'язку треба зшити лікарю?

mph{Lig. popliteum arcuatum

mph{Lig. patellae

mph{Lig. collaterale tibiale

mph{Lig. popliteum obliquum

mph{Lig. collaterale fibulare

5168 / 6307
Після операції на кишечнику у хворого з’явились симптоми отруєння аміаком за типом печінкової коми. Який механізм дії аміаку на енергозабезпечення ЦНС?

Гальмування ЦТК в результаті зв’язування альфа-кетоглутарату

Роз’єднування окисного фосфорилювання

Гальмування гліколізу

Гальмування бета-окиснення жирних кислот

Інактивація ферментів дихального ланцюга

5169 / 6307
Серед антиатеросклеротичних препаратів, що застосовуються з метою профілактики та лікування атеросклерозу, є левостатин. Він діє шляхом:

Пригнічення всмоктування холестерину вкишківнику

Активації метаболізму холестерину

Гальмування біосинтезу холестерину

Усіма наведеними шляхами

Стимулювання екскреції холестерину з організму

5170 / 6307
Хворий з хронічною серцево-судинною недостатністю приймав дигоксин. Після призначення додаткової терапії розвинулися явища інтоксикації серцевими глікозидами. Який препарат може викликати підсилення інтоксикації серцевими глікозидами?

Кальцію хлорид

Аспаркам

Магнію хлорид

Розчин глюкози

Калію хлорид

5171 / 6307
У пацієнта віком 43 роки розвиток гострого панкреатиту супроводжується порушенням прохідності загальної жовчної протоки. До якого ускладнення це може призвести?

Паренхіматозної жовтяниці

Гемолітичної жовтяниці

Портальної гіпертензії

Механічної жовтяниці

Печінкової коми

5172 / 6307
У хворого 69-ти років на шкірі в ділянці нижньої повіки з’явилося невелике бляшкоподібне утворення з наступним виразкуванням, яке було оперативно видалене. При мікроскопічному дослідженні утворення: в дермі шкіри комплекси з атипових епітеліальних клітин; периферії клітини розташовані перпендикулярно до базальної мембрани. Клітини темні, призматичної полігональної форми з гіперхромними ядрами з частими мітозами. Іноді зустрічаються утворення, подібні до волосяного фолікула. Яка гістологічна форма раку у хворого?

Недиференційований рак

Плоскоклітинний рак без ороговіння

Аденокарцинома

Базально-клітинний рак

Плоскоклітинний рак з ороговінням

5173 / 6307
У чоловіка, померлого від внутрішньої кровотечі (гемоперитонеум), у печінці субкапсулярно виявлено губчастий вузол темно-червоного кольору розмірами 15 х 10 см, добре відмежований від навколишньої тканини. Мікроскопічно виявлено, що тканина вузла складається з великих судинних тонкостінних порожнин, вистелених ендотеліальними клітинами та заповнених рідкою або згорнутою кров'ю. Установіть вид пухлини:

Лімфангіома

Капілярна гемангіома

Кавернозна гемангіома

Венозна гемангіома

Гемангіоперицитома

5174 / 6307
На розтині тіла хлопчика 8-ми років, що хворів на дифтерію зіва та мигдаликів і помер на другий тиждень від початку захворювання, виявлено зміни в міокарді у вигляді дрібновогнищевих некрозів міокардиоцитів, набряку строми з незначною лімфоцитарною інфільтрацією. Діагностуйте вид міокардиту:

Вогнищевий проміжний ексудативний

Гранулематозний

Альтеративний

Септичний

Інтерстиційний

5175 / 6307
У пацієнтів для оцінки ефективності дихання використовують показник функціональної залишкової ємності. З яких наступних об’ємів вона складається?

Резервний об’єм вдиху, дихальний, залишковий

Резервний об’єм вдиху та дихальний

Резервний об’єм вдиху та залишковий

Резервний об’єм видиху та дихальний

Резервний об’єм видиху та залишковий

5176 / 6307
Аміак особливо токсичний для ЦНС людини. Укажіть головний шлях його знешкодження в нервовій тканині:

Синтез глутаміну

Трансамінування

Утворення парних сполук

Синтез сечовини

Синтез солей амонію

5177 / 6307
Дослідженнями останніх десятиліть встановлено, що безпосередніми 'виконавцями' апоптозу в клітині є особливі ферменти - каспази. В утворенні одного з них бере участь цитохром С. Вкажіть його функцію в нормальній клітині:

Компонент піруватдегідрогеназної системи

Фермент β-окиснення жирних кислот

Фермент ЦТК

Компонент H+ - АТФ-азної системи

Фермент дихального ланцюга переносу електронів

5178 / 6307
Під час підготовки пацієнта до операції на серці проведено вимірювання тиску в камерах серця. В одній з них тиск протягом серцевого циклу змінювався від 0 до 120 мм рт.ст. Назвіть цю камеру серця:

Ліве передсердя

Праве передсердя

Правий шлуночок

Лівий шлуночок

5179 / 6307
У людини з нападом бронхоспазму необхідно зменшити вплив блукаючого нерва на гладеньку мускулатуру бронхів. Які мембранні циторецептори доцільно заблокувати для цього?

α- та β-адренорецептори

Н-холінорецептори

β-адренорецептори

М-холінорецептори

α-адренорецептори

5180 / 6307
У дитини, якій діагностовано пілоростеноз, об'єктивно спостерігається часте блювання та ознаки зневоднення організму. Яка форма порушення кислотно-основного стану може розвинутися у цьому разі?

Негазовий алкалоз

Негазовий ацидоз

Газовий алкалоз

Газовий ацидоз

Метаболічний ацидоз

5181 / 6307
У дитини, що народилася від пізнього шлюбу, спостерігаються малий зріст, відставання у розумовому розвитку, товстий 'географічний' язик, вузькі очні щілини, пласке обличчя з широкими вилицями. Що найімовірніше спричинило розвиток вказаного синдрому?

Внутрішньоутробна інтоксикація

Пологова травма

Спадкова хромосомна патологія

Внутрішньоутробний імунний конфлікт

Внутрішньоутробна інфекція

5182 / 6307
Під час лікування хронічного набрякового синдрому фуросемідом у хворого виникло порушення катіонного складу плазми крові. Який засіб слід використовувати для його корегування?

Натрію хлорид

Кальцію хлорид

Калію хлорид

Натрію гідрокарбонат

Літію карбонат

5183 / 6307
У хворого з гострим циститом під час дослідження сечі виявили лейкоцити й багато грамнегативних паличок. Під час посіву виросли колонії слизового характеру, які утворювали зелений, розчинний пігмент. Який мікроорганізм, імовірно, є причиною захворювання?

Escherihia coli

Proteus mirabilis

Salmonella enteritidis

Pseudomonas aeruginosa

Klebsiella pneumoniae

5184 / 6307
На розтині тіла чоловіка, який помер від черевного тифу, в клубовій кишці виявлені дефекти, розташовані по всій довжині кишки, краї їх рівні, дно утворене м’язовим шаром. Яка зі стадій черевного тифу діагностована?

Чистих виразок

Загоєння

Некрозу

Утворення виразок

Мозкоподібного набухання

5185 / 6307
У хворого на туберкульоз легень непереносимість аміноглікозидного антибіотика амікацину. Який протитуберкульозний антибіотик можна включити до складу комплексної терапії у даному випадку?

Стрептоміцин

Амоксицилін

Рифампіцин

Бензилпеніцилін

Канаміцин

5186 / 6307
Інозитолтрифосфати в тканинах організму утворюються в результаті гідролізу фосфатидилінозитолдифосфатів і відіграють роль вторинних посередників (месенджерів) в механізмі дії гормонів. Їхній вплив у клітині спрямований на:

Активацію протеїнкінази А

Гальмування фосфодіестерази

Гальмування протеїнкінази С

Вивільнення іонів кальцію з клітинних депо

Активацію аденілатциклази

5187 / 6307
У жінки виявлено ущільнення в молочній залозі. У якому напрямку під час операції має провести розріз лікар-хірург для зменшення травмування дольки?

Вертикально

---

Поперечно

Дугоподібно

Радіально

5188 / 6307
Під час розтину тіла померлого 43-х років, що страждав на ІХС з розвитком інфаркту міокарда, патологоанатом виявив набряк легень. Які патологічні зміни могли зумовити набряк легень?

Ішемія малого кола

Гостра правошлуночкова недостатність

Стаз крові

Гостре загальне малокрів’я

Гостра лівошлуночкова недостатність

5189 / 6307
У доношеної новонародженої дитини спостерігається жовтяне забарвлення шкіри та слизових оболонок. Тимчасова недостатність якого ферменту може бути імовірною причиноютакого стану дитини?

УДФ-глюкуронілтрансферази

Уридилтрансферази

Гемоксигенази

Білівердинредуктази

Гемсинтетази

5190 / 6307
До отоларинголога звернувся хворий, у якого під час огляду гортані виявлено неповне змикання голосових зв’язок при фонації. Голосова щілина при цьому приймає форму овалу. Функція якого м’яза гортані порушена у хворого?

M. thyroaryttenoideus

M. cricoaryttenoideus posterior

M. aryttenoideus transversus

M. vocalis

M. cricoaryttenoideus lateralis

5191 / 6307
Унаслідок перелому нижньої щелепи у постраждалого спостерігається втрата чутливості шкіри у ділянці підборіддя і нижньої губи. Який нерв найімовірніше було травмовано?

Facialis

Buccalis

Infraorbitalis

Mentalis

Maxillaris

5192 / 6307
Який з нижченаведених сечогінних засобів слід призначити хворому з первинним гіперальдостеронізмом?

Гіпотіазид

Спіронолактон

Маніт

Триамтерен

Фуросемід

5193 / 6307
У людини збільшується вентиляція легень під час фізичного навантаження. Який показник зовнішнього дихання стає значно більшим ніж у стані спокою?

Резервний об'єм видиху

Життєва ємність легень

Резервний об'єм вдиху

Загальна ємність легень

Дихальний об'єм

5194 / 6307
У чоловіка 30 років перед операцією визначили групову належність крові. Кров резус-позитивна. Реакцію аглютинації еритроцитів не викликали стандартні сироватки груп 0$\alpha\beta$ (I), Аβ (II), Вα (III). Досліджувана кров належить до групи:

αβ (I)

АВ (IV)

--

Вα (III)

Аβ (II)

5195 / 6307
До лікарні звернулася мати з дитиною. На голові у дитини спостерігається гангренозна рана. Під час огляду у рані виявлено білі червоподібні личинки комах. Яка комаха найімовірніше могла їх відкласти?

Комарі

Муха-жигалка

Вольфартова муха

Москіти

Блохи

5196 / 6307
Хворому 35 років для обстеження очного дна був призначений атропіну сульфат у формі очних крапель. Для відновлення акомодації йому закрапали пілокарпіну гідрохлорид, але це не дало бажаного ефекту. Що є причиною відсутності ефекту?

Однобічний антагонізм

Синергізм

Звикання

Тахіфілаксія

Двобічний антагонізм

5197 / 6307
Парубок звернувся до лікарні зі скаргами на порушення сечовипускання. Під час обстеження зовнішніх статевих органів виявлено, що сечівник розщеплений зверху і сеча витікає крізь цей отвір. Який вид аномалії розвитку зовнішніх статевих органів спостерігається у цьому разі?

Епіспадія

Парафімоз

Фімоз

Гіпоспадія

Гермафродитизм

5198 / 6307
У десятирічної дитини за два тижні після перенесеної ангіни з'явився біль на ділянці нирок, незначні набряки, розвинувся сечовий синдром. Проведено нефробіопсію та мікроскопічно виявлено інтракапілярне проліферативне запалення. Під час електронної мікроскопії виявлено великі електронно-щільні субепітеліальні депозити (<<горби>>). Яке захворювання розвинулося у дитини?

Мембранозний гломерулонефрит

Гломерулонефрит, що швидко прогресує

Постінфекційний гломерулонефрит

Гострий гнійний проміжний нефрит

Ліпоїдний нефроз

5199 / 6307
Під час роботи лікарю стоматологу доводиться довго стояти на ногах, що може призвести до застою крові у венах нижніх кінцівок та їх варикозного розширення. З порушенням якого механізму венозного припливу крові до серця це пов’язано?

Відсутність скорочення скелетних м’язів

Градієнт тиску

Залишкова рушійна сила серця

Присмоктувальний ефект грудної клітки

Присмоктувально-тисковий помповий ефект діафрагми на органи черевної порожнини

5200 / 6307
Важливим джерелом утворення аміаку в головному мозку є дезамінування АМФ. Яка амінокислота відіграє основну роль у зв’язуваннї аміаку в нервовій тканині?

Аланін

Лізин

Глутамат

Ізолейцин

Аргінін

5201 / 6307
Під час проведення ЕКГ-дослідження виявлено, що тривалість інтервалу RR дорівнює 1,5 с, частота серцевих скорочень --- 40/хв. Що є водієм ритму серця у цьому разі?

Ліва ножка Гіса

Синусовий вузол

Пучок Гіса

Атріовентрикулярний вузол

Права ножка Гіса

5202 / 6307
У хворого видалено 12-палу кишку. Це призведе до зменшення секреції, перш за все, такого гормону:

Соматостатин

Гістамін

Гастрин

Нейротензин

Холецистокінін-секретин

5203 / 6307
Чотирирічна дитина померла від легенево-серцевої недостатності. Під час аутопсії виявлено ущільнення підшлункової залози, у бронхах --- обтураційні ателектази та бронхоектази, у печінці --- жирова інфільтрація. Мікроскопічно в підшлунковій залозі спостерігається: кістозно розширені протоки, атрофія залозистої паренхіми, дифузний фіброз та лімфо-гістіоцитарна інфільтрація. Для якого захворювання характерні такі результати аутопсії?

Хвороба Гіршпрунга

Фетальний муковісцидоз

Ембріопатія

Гаметопатія

Інфекційна фетопатія

5204 / 6307
У дитини з білявим волоссям, блідою шкірою відмічається збільшений тонус м’язів, судоми та розумова відсталість. В крові підвищений рівень фенілаланіну. Який з перелічених методів необхідно застосувати для встановлення діагнозу цієї ензимопатії?

Електрофізіологічний

Цитогенетичниий

Популяційно-статистичний

Біохімічний

Генеалогічний

5205 / 6307
Реалізація загального адаптаційного синдрому здійснюється переважно через нейроендокринну систему. Яка з ланок цієї системи відіграє провідну роль у патогенезі реакції, що розвивається?

Гіпофізарно-юкстагломерулярна

Гіпофізарно-адреногенітальна

Гіпофізарно-інсулярна

Гіпофізарно-адреналова

Гіпофізарно-тиреоїдна

5206 / 6307
У вагітної жінки взяли кров для підтвердження клінічного діагнозу 'токсоплазмоз'. Яка з перерахованих серологічних реакцій має діагностичне значення?

Реакція нейтралізації

Реакція гемадсорбції

Реакція гальмування гемаглютинації

Реакція аглютинації

Реакція зв’язування комплементу

5207 / 6307
Хворому для лікування серцевої недостатності було призначено серцевий глікозид. Яка супутня патологія може сприяти кумуляції серцевих глікозидів?

Гіпертонічна хвороба

Гіперацидний гастрит

Гіпоацидний гастрит

Ниркова недостатність

Анорексія

5208 / 6307
При розтині тіла жінки, померлої від хронічної ниркової недостатності, в слизовій оболонці товстої кишки виявлені сіро-жовті плівки, що щільно з’єднані з підлеглим шаром, при відокремленні яких утворюються виразки. Мікроскопічно: глибока ділянка некрозу пронизана нитками фібрину. Визначте вид запалення:

Гнійне

Дифтеритичне

Гнильне

Крупозне

Катаральне

5209 / 6307
Хворий на гіпертонічну хворобу разом з безсольовою дієтою та з антигіпертензивними засобами, довгий час приймав гідрохлортіазид, що зумовило порушення електролітного балансу. Яке порушення внутрішнього середовища виникло у хворого?

Гіперкаліємія

Гіпохлоремічний алкалоз

Метаболічний ацидоз

Гіпермагніємія

Збільшення об’єму циркулюючої крові

5210 / 6307
Хлопчика віком 12 років госпіталізовано до лікарні з попереднім діагнозом: харчова токсикоінфекція. Під час посіву фекалій дитини на середовище Ендо виросла велика кількість безбарвних колоній. Який мікроорганізмможна виключити з-поміж можливих збудників захворювання?

Escherichia coli

Salmonella enteritidis

Proteus vulgaris

Pseudomonas aeruginosa

Yersinia enterocolitica

5211 / 6307
У хворого відзначаються періодичні напади серцебиття (пароксизми), сильне потовиділення, напади головного болю. При обстеженні виявлена гіпертензія, гіперглікемія, підвищення основного обміну, тахікардія. При якій патології наднирників спостерігається подібна картина?

Первинний альдостеронізм

Гіпофункція кори наднирників

Гіперфункція кори наднирників

Гіпофункція мозкового шару

Гіперфункція мозкового шару

5212 / 6307
Хворому на туберкульоз легень призначений препарат першого ряду, який спричинив розвиток невриту лицьового нерва та порушення рівноваги. Вкажіть цей лікарський засіб:

Левоміцетин

Ізоніазид

Цефазолін

Бісептол

Фуразолідон

5213 / 6307
У чоловіка при обстеженні виявлено порушення кровообігу міокарда лівого передсердя. У басейні якої артерії відбулись порушення кровообігу?

Права та ліва вінцеві

Передня міжшлуночкова гілка лівої вінцевоїартерії

Права вінцева

Ліва вінцева

5214 / 6307
При аналізі ЕКГ виявлено випадіння деяких серцевих циклів PQRST. Наявні зубці та комплекси не змінені. Назвіть вид аритмії:

Передсердна екстрасистола

Внутрішньопередсердна блокада

Синоатріальна блокада

Миготлива аритмія

Атріовентрикулярна блокада

5215 / 6307
У хворого спостерігається типова для нападу малярії клінічна картина: остуда, жар, проливний піт. Яка стадiя малярійного плазмодію найімовірніше буде виявлена в крові хворого в цей час?

Оокінета

Мікро- або макрогамети

Мерозоїт

Спорозоїт

Спороциста

5216 / 6307
До медико-генетичної консультації звернулася жінка за рекомендацією гінеколога з приводу відхилень фізичного і статевого розвитку. В ході мікроскопії клітин слизової оболонки ротової порожнини не знайдено статевого хроматину. Який найбільш імовірний діагноз?

Хвороба Реклінгаузена

Синдром Клайнфельтера

Синдром Шерешевського-Тернера

Хвороба Дауна

Трисомія за Х-хромосомою

5217 / 6307
Вкажіть організми в клітинах яких відсутні мембранні органели та їх спадковий матеріал не має нуклеосомної організації.

Віруси

Найпростіші

Аскоміцети

Еукаріоти

Прокаріоти

5218 / 6307
У хворого на ЕКГ виявлено збільшення тривалості комплексу QRS. Наслідком чого це може бути?

Порушення провідності у атріовентрикулярномувузлі

Збільшення часу охоплення збудженням передсердь

Збільшення збудливості передсердь

Збільшення часу охоплення збудженням шлуночків

Збільшення збудливості шлуночків та передсердь

5219 / 6307
У хворого з підозрою на ботулізм необхідно визначити тип екзотоксину, що циркулює в крові. Яка реакція може бути використана з цією метою?

Реакція нейтралізації

Реакція пасивної гемаглютинації

Реакція преципітації

Реакція гальмування гемаглютинації

Реакція зв’язування комплементу

5220 / 6307
Під час вживання внутрішньо 100 мл 25% (насиченого) розчину сірчанокислої магнезії, з'являється багато рідкого калу. Чому виникає такий проносний ефект?

Стимулюється виділення гормонів 12-палої кишки

Збільшується осмотичний тиск у кишечнику

Стимулюється секреція шлункового соку

Гальмується робота кишечника

Зменшується осмотичний тиск

5221 / 6307
Багато слизових оболонок людини продукують фермент, що викликає лізис бактерій. Його виявляють у слізній рідині, слині та в слизі шлунково-кишкового тракту. Вкажіть цей фермент.

Лізоцим

Гіалуронідаза

Фібринолізин

Комплемент

Опсонін

5222 / 6307
У мазку з нальоту на мигдаликах хворого з імовірною дифтерією виявлено палички синього кольору з потовщеннями на полюсах. Який метод фарбування мазків було використано?

Леффлера

Нейссера

Гінса

Буррі

Грама

5223 / 6307
У зв’язку з крововтратою пацієнту введено 1 л розчину хлориду натрію з концентрацією 150 ммоль/л. Внаслідок цього, насамперед, зменшиться:

Онкотичний тиск крові

Осмотичний тиск міжклітинної рідини

Онкотичний тиск міжклітинної рідини

Осмотичний тиск внутрішньоклітинний

Осмотичний тиск крові

5224 / 6307
У новонародженої дитини спостерігається зниження інтенсивності смоктання, часте блювання, гіпотонія. У сечі та крові значно підвищена концентрація цитруліну. Який метаболічний процес порушений?

ЦТК

Цикл Корі

Глюконеогенез

Гліколіз

Орнітиновий цикл

5225 / 6307
У хворого струс головного мозку, що супроводжується повторним блюванням і задишкою. При обстеженні відзначено: рН - 7,62; pCO2 - 40 мм рт.ст. Яке порушення кислотно-основного стану є у хворого?

Газовий ацидоз

Негазовий алкалоз

Негазовий ацидоз

Газовий алкалоз

5226 / 6307
У дитини, яка часто хворіє на ангіни та фарингіти, спостерігається збільшення лімфовузлів і селезінки. Зовнішній вигляд характеризується пастозністю та блідістю, м'язова тканина розвинена слабко. У крові спостерігається лімфоцитоз. Як називається такий вид діатезу?

Астенічний

Нервово-артритичний

Геморагічний

Лімфатико-гіпопластичний

Ексудативно-катаральний

5227 / 6307
У п'ятимісячної дівчинки спостерігаються застійні явища у легенях. Під час обстеження виявлено зв'язок між висхідною аортою та легеневою артерією, що в нормі спостерігається у деяких земноводних і плазунів. Назвіть цю вроджену ваду розвитку.

Транспозиція магістральних судин

Дефект міжшлуночкової перегородки

Дефект міжпередсердної перегородки

Коарктація аорти

Незарощення боталової протоки

5228 / 6307
Для усунення марення і галюцинацій у пацієнтів з діагнозом: шизофренія, використовують аміназин. Який механізм антипсихотичної дії цього лікарського засобу?

Блокада холінергічних процесів у ЦНС

Стимуляція адренергічних і дофамінергічних процесів у ЦНС

Блокада адренергічних і дофамінергічних процесів у ЦНС

Інгібування зворотнього нейронального захоплення МАО

Стимуляція холінергічних процесів у ЦНС

5229 / 6307
Під час посмертного розтину тіла жінки виявлено стеноз лівого атріовентрикулярного отвору та недостатність мітрального клапана. Гістологічно в міокарді виявлено вогнищевий кардіосклероз та наявність квітучих гранульом Ашоффа-Талалаєва. Який із наведених нижче діагнозів найімовірніший?

Дерматоміозит

Системний червоний вовчак

Вузликовий периартеріїт

Ревматизм

Склеродермія

5230 / 6307
На розтині померлого було виявлено наявність крові в тонкій та товстій кишках (1,5-2 л). В нижніх відрізках клубової кишки стінка мала 'брудні' виразки на місці групових лімфатичних фолікулів. Решта пейєрових бляшок була зеленуватого кольору з демаркаційним запаленням навколо. Було діагностовано черевний тиф. Для якого періоду черевного тифу найбільш характерні описані морфологічні зміни кишки й ускладнення?

Чистих виразок

Мозкоподібного набухання

Утворення виразок

Загоювання виразок

Некроз

5231 / 6307
У чоловіка 64-х років, який тривалий час курив і вживав міцні спиртні напої, на боковій поверхні язика виявили подібний на виразку утвір із білої, помірно щільної тканини розмірами 5х3 см. При гістологічному дослідженні біоптату виявили, що утвір побудований із клітини, які формують солідні структури і тяжі, що нагадують за будовою багатошаровий плоский епітелій, в якому клітини з вираженим поліморфізмом, з великими атиповими ядрами з патологічними мітозами. Діагностуйте виявлене у чоловіка захворювання:

Лейкоплакія

Рак на місці

Плоскоклітинний зроговілий рак

Плоскоклітинний незроговілий рак

Еритроплакія

5232 / 6307
При обстеженні 2-х місячної дитини педіатр звернула увагу, що плач дитини нагадує котячий крик. Діагностовані мікроцефалія і вада серця. За допомогою цитогенетичного метода з’ясований каріотип дитини: 46, XX, 5р. Дане захворювання є наслідком такого процесу:

Інверсія

Дуплікація

Плейотропія

Делеція

Транслокація

5233 / 6307
Під час посмертного розтину тіла чоловіка віком 62 роки виявлено надклапанний розрив аорти з тампонадою серця. Гістологічне дослідження висхідного відділу аорти виявило в зовнішній і середній оболонках інфільтрати з лімфоїдних, плазматичних, епітеліоїдних клітин, вогнища некрозу в середній оболонці, проліферацію адвентиціальних і ендотеліальних клітин та судин зовнішньої оболонки. Для якої патології характерні такі зміни в аорті?

Септичного аортиту

Ревматичного аортиту

Атеросклерозу

Гіпертонічної хвороби

Сифілітичного аортиту

5234 / 6307
Хворому перед операцією було введено дитилін (лістенон) і проведено інтубацію. Дефіцит якого ферменту в організмі хворого подовжує дію м’язового релаксанту?

Карбангідраза

Сукцинатдегідрогеназа

N-ацетилтрансфераза

K-Na-АТФ-аза

Псевдохолінестераза

5235 / 6307
У вагітної жінки розвинувся токсикоз з тяжким повторним блюванням шлунковим вмістом протягом доби. Наприкінці доби почали проявлятися тетанічні судоми та зневоднення організму. Який розлад кислотно-лужної рівноваги викликав дані зміни?

Негазовий видільний ацидоз

Газовий алкалоз

Негазовий метаболічний ацидоз

Негазовий видільний алкалоз

Газовий ацидоз

5236 / 6307
У результаті радіаційного випромінювання ушкоджено стовбурові гемопоетичні клітини. Утворення яких клітин сполучної тканини буде порушено?

Меланоцитів

Адипоцитів

Макрофагів

Перицитів

Фібробластів

5237 / 6307
Пацієнт віком 65 років тривалий час скаржиться на симптоми, характерні для хронічного гастриту. Під час лабораторного аналізу в крові виявлено мегалоцити, у кістковому мозку --- мегалобластичний еритропоез. Якийдіагноз є найімовірнішим у цьому разі?

Гемолітична анемія

Гіпопластична анемія

Апластична анемія

B₁₂-фолієводефіцитна анемія

Залізодефіцитна анемія

5238 / 6307
Під час пологової діяльності при важкому прорізуванні голівки плоду, щоб уникнути розриву промежини, виконують розсічення отвору піхви біля основи великої статевої губи. Який м’яз промежини при цьому розсікають?

Цибулинно-губчастий м’яз

Зовнішній сфінктер прямої кишки

Глибокий поперечний м’яз

Поверхневий поперечний м’яз

Сіднично-печеристий м’яз

5239 / 6307
У хворої при лікуванні гіпертонічної хвороби на фоні прийому препарату виник сухий кашель. Для якого препарату характерна така побічна дія?

Дротаверин

Но-шпа

Лізиноприл

Гідрохлортіазид

Резерпін

5240 / 6307
В експерименті на спинному мозку при збудженні альфа-мотонейронів згиначів встановлено гальмування альфа- мотонейронів м’язів-розгиначів. Який вид гальмування лежить в основі цього явища?

Пресинаптичне

Латеральне

Реципрокне

Зворотнє

Деполяризаційне

5241 / 6307
Хворий 58-ми років надійшов у торакальне відділення з кровохарканням; при бронхоскопії виявлено звуження правого стовбурового бронха внаслідок розростання сірувато-білої тканини. В ході біопсії виявлена пухлина, що складається з дрібних клітин овальної й округлої форми з гіперхромними ядрами та незначною цитоплазмою, клітини справляють враження 'голих ядер'. Клітини пухлини ростуть пластами, тяжами. Гістологічна будова пухлини свідчить про те, що у хворого:

Недиференційований рак

Перехідноклітинний рак

Пласкоклітинний зроговілий рак

Пласкоклітинний незроговілий рак

Базальноклітинний рак

5242 / 6307
Молекули зрілої іРНК у клітині є носієм генетичної інформації про послідовність з'єднання між собою певних амінокислот. Це означає, що в молекулах іРНК закодована:

Первинна структура ліпідів

Вторинна структура вуглеводів

Первинна структура вуглеводів

Первинна структура полінуклеотидів

Первинна структура білка

5243 / 6307
Характерними ознаками холери є втрата організмом великої кількості води та іонів натрію. Що є основою біохімічної дії холерного токсину?

Активація аденілатциклази в епітелії тонкої кишки

Посилення секреції реніну клітинами нирковихартеріол

Посилення синтезу кортикотропіну

Окислення альдостерону в корі наднирників

Гальмування синтезу вазопресину у гіпоталамусі

5244 / 6307
В результаті травми у чоловіка 47-ми років пошкоджені передні корінці спинного мозку. Відростки яких нейронів пошкоджені?

Аксони нейронів рухових соматичних та вегетативних ядер

Дендрити рухових і аксони ядер бокових стовпів

Аксони чутливих псевдоуніполярних

Дендрити і аксони чутливих псевдоуніполярних

Дендрити чутливих псевдоуніполярних

5245 / 6307
Чоловіку віком 40 років, який хворіє на туберкульоз легень, призначено ізоніазид. Дефіцит якого вітаміну може розвинутися унаслідок тривалого вживання цього лікарського засобу?

Кобаламіну

Піридоксину

Тіаміну

Фолієвої кислоти

Біотину

5246 / 6307
При обстеженні жінки виявили перелом однієї з кісток черепа, ускладнений кровотечею з поперечної пазухи твердої оболонки мозку. Назвіть кістку, на якій розташовується борозна цієї пазухи:

Тім’яна

Лобова

Потилична

Клиноподібна

Скронева

5247 / 6307
У хворого з серцевою недостатністю виникла аритмія у вигляді генерації позачергових імпульсів в пучку Гіса. Порушення якої функції серцевого м’язу спостерігається в даному випадку?

Автоматизм

Збудливість та провідність

Провідність

Скоротливість

Збудливість

5248 / 6307
У підлітка 12-ти років, який хворіє на бронхіальну астму, розвинувся тяжкий напад астми: виражена експіраторна задишка, блідість шкірних покровів. Який вид порушення альвеолярної вентиляції має місце?

Центральний

Обструктивний

Торако-діафрагмальний

Рестриктивний

Нервово-м’язовий

5249 / 6307
У хлопчика спостерігається велика щілина між різцями. Відомо, що ген, відповідальний за розвиток такої аномалії, домінантний. У рідної сестри цього хлопчика зуби звичайного положення. За генотипом дівчинка буде:

Гомозигота домінантна

Дигетерозигота

Гомозигота рецесивна

Тригетерозигота

Гетерозигота

5250 / 6307
У спортсмена легкоатлета (бігуна на довгі дистанції) під час змагань розвинулась гостра серцева недостатність. В результаті чого виникла ця патологія?

Перевантаження серця опором

Перевантаження серця об’ємом

Порушення вінцевого кровообігу

Патологія перикарда

Прямого пошкодження міокарда

5251 / 6307
У здорових батьків, спадковість яких не обтяжена, народилася дитина з множинними вадами розвитку. Цитогенетичний аналіз виявив у соматичних клітинах дитини трисомію за 18-ю хромосомою (синдром ЕдвардсA). З яким явищем пов’язане народження такої дитини?

Хромосомною мутацією - дуплікацією

Впливом тератогенних факторів

Соматичною мутацією у ембріона

Домінантною мутацією

Нерозходженням пари хромосом під час гаметогенезу

5252 / 6307
До лікарні госпіталізовано пацієнта з попереднім діагнозом: гострий панкреатит. Активність якого ферменту в крові та сечі буде різко підвищена у цьому разі?

Аланінамінотрансферази

Креатинфосфокінази

Аспартатамінотрансферази

Лактатдегідрогенази

Альфа-амілази

5253 / 6307
Хвора 24 років потрапила до лікарні зі скаргами на головний біль, біль у поперековій ділянці, набряки на обличчі, загальну слабкість. Місяць тому перенесла ангіну. На час надходження: АТ -- 180/110 мм рт.ст., у сечі вичвлено виражену протеінурію, мікрогематурію, лейкоцитурію. На яку форму гіпертензії страждає хвора?

Первинну

Ниркову

Ендокринну

Гіпертонічну хворобу

Есенціальну

5254 / 6307
У клініку госпіталізовано хворого з діагнозом карциноїду кишечника. Аналіз виявив підвищену продукцію серотоніну, який утворюється з амінокислоти триптофан. Який біохімічний механізм лежить в основі даного процесу?

Мікросомальне окиснення

Дезамінування

Трансамінування

Декарбоксилювання

Утворення парних сполук

5255 / 6307
У чоловіка 25-ти років діагностований гострий дифузний гломерулонефрит. З анамнезу: за 18 днів до прояву хвороби переніс ангіну. Який механізм ураження ниркових клубочків буде спостерігатися у цьому випадку?

Нефротоксичний

Медикаментозний

Імунний

Ішемічний

5256 / 6307
Спеціальний режим харчування призвів до зменшення іонів Ca2+ в крові. До збільшення секреції якого гормону це призведе?

Соматотропін

Тирокальцитонін

Тироксин

Паратгормон

Вазопресин

5257 / 6307
Хворому з прогресуючою м’язовою дистрофією було проведено біохімічне дослідження сечі. Поява якої речовини у великій кількості в сечі може підтвердити захворювання м’язів у даного хворого?

Порфірини

Креатинін

Гіпурова кислота

Сечовина

Креатин

5258 / 6307
У пацієнта травма кульшового суглоба. Рентгенологічно чітко виявлено внутрішньосуглобовий крововилив. Яка звʼязка розірвана?

Lig. transversum acetabuli

Lig. ischiofemorale

Lig. capitis femoris

Zona orbicularis

Lig. iliofemorale

5259 / 6307
Виникнення нижченаведених захворювань повʼязують із генетичними факторами. Укажіть патологію зі спадковою схильністю.

Цукровий діабет

Дальтонізм

Фенілкетонурія

Хорея Гантінгтона

5260 / 6307
За 5-8 днів після застосування значної кількості лікувальної сироватки у пацієнта виявлено шкірний висип, свербіж, набряклість, біль у суглобах, підвищення температури тіла. У сечі зʼявився білок. Пацієнту встановлено діагноз: сироваткова хвороба. Який важливий фактор патогенезу цього захворювання?

Активація Т-кілерів

Накопичення в крові циркулюючих імунних комплексів

Дегрануляція тканинних базофілів

Активація макрофагів

Цитоліз клітин крові

5261 / 6307
Під час обстеження пацієнта віком 70 років виявлено порушення рухових функцій. Лікар повʼязує це із віковими змінами в гіаліновому хрящі. Які зміни викликали обмеження рухів у суглобах?

3більшення гідрофільності основної речовини

Збільшення кількості ізогенних груп

Відкладання солей кальцію в міжклітинній речовині

Збільшення кількості хрящових клітин

5262 / 6307
У дитячому садку в більшості дітей раптово з'явилися симптоми гастроентериту. Виявлено, що симптоми виникли за кілька годин після вживання молочної каші. У ході бактеріологічного дослідження блювотних мас та залишків каші, виділено золотистий стафілокок. Яке дослідження потрібно провести для уточнення джерела інфекції?

Визначити здатність штамів до токсиноутворення

Виконати дослідження обладнання харчоблоку

Визначити наявність антитіл у хворих дітей

Виконати фаготипування виділених штамів

5263 / 6307
У пацієнта з розладом мозкового кровотоку спостерігаються порушення акту ковтання під час вживання рідкої їжі. Який відділ мозку пошкоджено?

Мозочок

Середній мозок

Довгастий мозок

Проміжний мозок

Шийний відділ спинного мозку

5264 / 6307
Нітрогліцерин використовується для розширення коронарних судин та зменшення болю в серці. В організмі людини він розпадається з утворенням нітроген оксиду (NO). Активність якого ферменту підвищується в цьому разі?

Гуанілатциклази

Фосфоліпази С

Ліпооксигенази

Циклооксигенази

5265 / 6307
У пацієнта діагностовано мегалобластичну анемію на фоні атрофічного гастриту. Який препарат, введений лише парентерально, буде профілактикою цієї патології?

Преднізолон

Ціанокобаламін

Прозерин

Целекоксиб

5266 / 6307
У звʼязку зі спалахом дифтерії виникла необхідність провести запобіжні щеплення. Який препарат потрібно використати для створення штучного активного імунітету?

Антидифтерійну сироватку

Дифтерійний анатоксин

Вакцину АКДП

Вакцину з вбитих бактерій

Специфічний імуноглобулін

5267 / 6307
У результаті травмування хребта у пацієнта відсутня пропріоцептивна чутливість нижньої половини тіла та нижніх кінцівок. Пошкодження якого провідного шляху може бути причиною цього?

Тr. spino-cerebellaris ventralis (Говерса)

Tr. spino-thalamicus anterior

Fasciculus cuneatus (Бурдаха)

Fasciculus gracilis (Голля)

5268 / 6307
Під час дослідження залишкового азоту виявлено, що азот сечовини значно знижений. Для захворювання якого органа це характерно?

Кишечника

Печінки

Шлунка

Серця

5269 / 6307
Під час лабораторної діагностики гепатиту В, у крові пацієнта визначають наявність вірусної ДНК. За допомогою якої з нижчезазначених реакцій це встановлюють?

Імуноферментного аналізу

Реакції гальмування гемаглютинації

Полімеразної ланцюгової реакції

Реакції непрямої гемаглютинації

5270 / 6307
Пацієнту з діагнозом: глаукома, лікар призначив препарат, що знижує внутрішньоочний тиск. Укажіть цей лікарський засіб.

Прозерин

Феназепам

Норадреналіну гідротартрат

Анаприлін

Анальгін

5271 / 6307
Чому під час першого вдиху новонародженого обсяг повітря, яке видихається, є у 2-3 рази меншим, ніж обсяг, який вдихається?

Формується резерв вдиху

Формується загальна ємність легень

Формується функціональна залишкова ємність легень

Формується дихальний обʼєм

5272 / 6307
Після порушення кровопостачання головного мозку пацієнт утратив здатність до написання літер і цифр. В якій частині мозку виникла патологія?

Lobus frontalls

Lobus occipitalis

Insula

Lobus parietalls

5273 / 6307
Під час профілактичного огляду пацієнта виявлено потовщення шиї, екзофтальм, підвищення температури тіла, пульс - 110/хв. Уміст яких гормонів доцільно визначити у крові?

Катехоламінів

Тироксину

Статевих гормонів

Інсуліну

5274 / 6307
У новонародженої дитини підвищена чутливість до інфекцій. Який імуноглобулін проникає крізь плацентарний бар'єр і забезпечує гуморальний імунітет немовлят?

IgD

IgE

IgM

IgG

IgA

5275 / 6307
Пацієнту віком 40 років, який хворіє на туберкульоз легень, призначено ізоніазид. Нестача якого вітаміну може розвинутися внаслідок тривалого прийому цього препарату?

Піридоксину

Біотину

Кобаламіну

Тіаміну

Фолієвої кислоти

5276 / 6307
У ході біохімічного аналізу крові пацієнта виявлено: натрій 115 ммоль/л, хлориди 85 ммоль/л, глюкоза 6 ммоль/л, загальний білок 65 г/л. Зменшення якого показника стане наслідком цих змін?

pH крові

Осмотичного тиску крові

Онкотичного тиску крові

Обʼєму циркулюючої крові

Швидкості осідання еритроцитів

5277 / 6307
Після введення великої дози антитіл до базальної мембрани клубочків нирок, у піддослідної тварини розвинувся гострий гломерулонефрит. Який вид алергічної реакції за класифікацією Кумбса і Джелла лежить в основі цієї патології?

Гіперчутливість сповільненого типу

Стимулювальний

Імунокомплексний

Цитотоксичний

5278 / 6307
У пацієнта на шкірі обличчя поступово виникла бляшка з некрозом і виразкою в центрі. Під час проведення патогістологічного дослідження біоптату виявлено розростання атипових епітеліальних клітин із великою кількістю патологічних мітозів. Для якого захворювання характерні такі прояви?

Рак шкіри

Папілома

Саркома

Трофічна виразка

Фіброма

5279 / 6307
Пацієнта віком 45 років шпиталізовано до лікарні зі скаргами на температуру, біль під час дихання, задуху та кашель. Після лабораторної та рентгенодіагностики вставлено діагноз: плеврит. Для евакуації ексудату призначено плевральну пункцію. В якому місці плевральної порожнини знаходиться найбільша кількість ексудату?

Реберно-діафрагмальний синус

Під куполом плеври

Реберно-медіастинальний синус

Під коренем легенів

5280 / 6307
Пацієнт із діагнозом: гіпертонічна хвороба приймає еналаприл. Який механізм дії цього гіпотензивного препарату?

Антагоніст ангіотензинових рецепторів

Інгібітор фосфодієстерази

Інгібітор ангіотензинперетворювального ферменту

Інгібітор циклооксигенази

Блокатор Ca2+-каналів

5281 / 6307
Під час фіброгастродуоденоскопії лікарю необхідно оглянути великий сосочок дванадцятипалої кишки. Яке анатомічне утворення може бути орієнтиром для його виявлення?

Поздовжня складка дванадцятипалої кишки

Дуоденальні залози

Цибулина дванадцятипалої кишки

Циркулярні складки дванадцятипалої кишки

5282 / 6307
У чоловіка, який надійшов до неврологічного відділення, діагностовано крововиливи у стовбур мозку. Цей стан супроводжувався порушенням дрібної моторики кистей рук, амімією обличчя та збільшенням тонусу соматичних мʼязів. Ураження якої з перерахованих структур стовбуру мозку могло викликати цей стан?

Чорної субстанції

Вестибулярного латерального ядра Дейтерса

Вестибулярного медіального ядра Швальбе

Ядра третьої пари черепно-мозкових нервів

5283 / 6307
У пацієнта, який хворіє на цукровий діабет другого типу, але гіперглікемічна кома (рівень глюкози крові - 56 ммоль/л). Яке патологічне явище в цьому разі лежить в основі порушення функцій клітин мозку пацієнта?

Гіпогідратація клітин мозку

Токсичне ураження

Іонний дисбаланс

Енергодефіцит

Гіпергідратація клітин мозку (набухання)

5284 / 6307
У пацієнта діагностовано пелагру. В якому типі реакцій важливу роль відіграє вітамін РР?

Дегідрування

Дезамінування

Гідроксилювання

Трансамінування

Декарбоксилювання

5285 / 6307
У пацієнта віком 60 років, який хворіє на артеріальну гіпертензію, цукровий діабет і гіперліпідемією, раптово розвинулася слабкість у правій половині тіла. На час прибуття екстреної (швидкої) медичної допомоги, йому вже було важко говорити. За 2 години після виникнення цього ускладнення пацієнт помер. Макроскопічне дослідження лівої півкулі мозку виявило набряк головного мозку, розширення звивин та нечіткі межі між білою та сірою речовиною. Яка патологія стала ймовірною причиною смерті пацієнта?

Геморагічний інсульт

Ішемічний інсульт

Kicтa

Пухлина

Абсцес

5286 / 6307
У немовляти після народження в судинній системі відбуваються зміни, повʼязані із переходом від плацентарного кровообігу до легеневого. Яка з кровоносних судин у ході цього перетворюється на круглу звʼязку печінки?

Ductus arteriosus

Ductus venosus

A. umbilicalis dextra

V. umbilicalis

5287 / 6307
У пацієнта віком 40 років після щелепно-лицевої травми порушилася функція підʼязичної та підщелепної залоз зліва. У цих залозах відзначається гіпосалівація. Функція якої пари нерв порушена?

XII пари

Х пари

VII пари

VI пари

5288 / 6307
У пацієнта віком 27 років виявлено патологічні зміни печінки і головного мозку. У плазмі крові спостерігається різке зниження вмісту міді, а в сечі її підвищення. Встановлено діагноз: хвороба Вільсона. Активність якого ферменту необхідно дослідити в сироватці крові для підтвердження діагнозу?

Карбоангідрази

Ксантиноксидази

Алкогольдегідрогенази

Лейцинамінопептидази

Церулоплазміну

5289 / 6307
Із розвитком медичної генетики зʼявилася можливість одужання за допомогою дієтотерапії при деяких спадкових хворобах, які раніше вважалися невиліковними. Яке з нижченаведених захворювань можна вилікувати за допомогою дієтотерапії?

Ахондроплазію

Фенілкетонурію

Дальтонізм

Хворобу Тея-Сакса

5290 / 6307
Арахідонова кислота, як незамінний компонент їжі, є попередником біологічно активних речовин. Які сполуки синтезуються з цієї кислоти?

Адреналін

Трийодтиронін

Холін

Етаноламін

Тромбоксани

5291 / 6307
У пацієнта перебіг геморагічного шоку ускладнився розвитком гострої ниркової недостатності. Укажіть провідну ланку в механізмі розвитку цього ускладненя?

Розвиток ДВ3-синдрому

Викид у кров вазопресину

Централізація кровообігу із виникненням ішемії нирок

Підвищення проникності стінки капілярів

Активація симпатоадреналової системи

5292 / 6307
У пацієнта сироватка крові має молочний вигляд. Під час біохімічного дослідження виявлено високий рівень триацилгліцеролів і хіломікронів. Спадковий дефект якого ферменту викликає цей стан?

Ліпопротеїнлінази

Панкреатичної ліпази

Фосфоліпази

Тканинної гормончутливої ліпази

Фосфодіестерази

5293 / 6307
У пацієнта віком 50 років після резекції шлунка посилилися процеси гниття білків у кишечнику. Підвищення якого показника в сечі вказує на цей стан?

Сечової кислоти

Оксипроліну

Стеркобіліногену

Креатиніну

Тваринного індикану

5294 / 6307
Пацієнту з діагнозом: гіпертензивна хвороба, у комплексній терапії призначено гідрохлортіазид. Який механізм дії цього препарату сприяє зниженню артеріального тиску?

Зменшення виділення іонів натрію та води

Блокада ферменту карбоангідрази

Збільшення утворення ангіотензину II

Збільшення виділення іонів натрію та води

5295 / 6307
Відомо, що селезінка є «кладовищем еритроцитів». Що відбувається з еритроцитами червоної пульпи, які гинуть?

Поглинаються макрофагами

Відбувається лізис ферментами гігантських клітин сторонніх тіл

Накопичуються в червоній пульпі

Потрапляють у кровотік

5296 / 6307
У пацієнта розгладжена права носогубна складка, розширена права очноямкова щілина (її не вдається замружити, тому що повіки не змикаються), виникають труднощі під час розмови та вживання їжі (їжа застрягає між щокою і зубами). Який нерв уражено?

N. glossopharyngeus sinister

N. facialis dexter

N. abduceus dexter

N. trigeminus dexter

5297 / 6307
У синтезі та виділенні медіаторів запалення беруть участь ряд клітин крові та сполучної тканини. Укажіть клітини, в яких синтезується інтерлейкіи-1.

Тканинні базофіли

Макрофаги

Еозинофільні гранулоцити

Лімфоцити

5298 / 6307
Пацієнта віком 57 років шпиталізували у важкому стані. За показниками біохімічного аналізу крові виявлено рН - 7,33, рСО2 артеріальної крові – 36 мм.рт.ст., SB-17 ммоль/л, ВЕ становить +6 ммоль/л. Який тип порушення кислотно-лужного балансу спостерігається у цьому разі?

Декомпенсований негазовий ацидоз

Компенсований газовий алкалоз

Компенсований негазовий алкалоз

Декомпенсований негазовий алкалоз

Компенсований негазовий ацидоз

5299 / 6307
У пацієнта дефіцит вітаміну D. Всмоктування якого мікроелементу порушиться?

Води

Натрію

Заліза

Кальцію

5300 / 6307
Першим етапом діагностування хвороб, які зумовлені порушенням обміну речовин, є скринінг-метод, після якого використовують точніші методи дослідження ферментів і амінокислот. Яку назву має цей метод?

Цитогенетичний

Імунологічний

Біохімічний

Гібридизація соматичних клітин

5301 / 6307
У палаті інтенсивної терапії інфекційного відділення лікар відзначає у пацієнта з менінгококовим менінгітом періоди тахіпное, які змінюються тривалими періодами апное. Який це тип патологічного дихання?

Дихання Біота

Дихання Грокко

Апнейстичне лихання

Гаспінг-дихання

5302 / 6307
Якщо концентрація Na+ у плазмі крові знижується, у нирках посилюється його реабсорбція. Який основний механізм регуляції стимулює цей процес?

Ренін

Симпатичні рефлекси

Натрійуретичний гормон

Альдостерон

5303 / 6307
Пацієнту встановлено попередній діагноз: токсоплазмоз. Який біологічний матеріал використано для діагностики цього захворювання?

Харкотиння

Сечу

Кров

Дуоденальний вміст

5304 / 6307
За два тижні після переливання крові у реципієнта виникла лихоманка. Яке протозойне захворювання може припустити лікар в цій ситуації?

Токсоплазмоз

Лейшманіоз

Трипаносомоз

Малярію

Амебіаз

5305 / 6307
Надмолекулярний мультиферментний комплекс, інтегрований у ліпідний шар внутрішньої мембрани мітохондрій, що створює умови для перебігу окисно-відновних реакцій. Укажіть із нижченаведеного.

Карбоксипептидаза

Піруваткіназа

Гексокіназа

Дихальний ланцюг

5306 / 6307
Гальмування ноцицептивної інформації відбувається за участі багатьох медіаторів. Що з нижченаведеного до цього не відноситься?

ГАМК

Серотонін

Глутамат

Ендорфін

5307 / 6307
Пацієнтці встановлено діагноз: первинний сифіліс. Який із нижченаведених препаратів треба їй призначити?

Левоміцетин

Тетрациклін

Бензилпеніцилін

Цефамезин

5308 / 6307
Під час розтину тіла чоловіка віком 50 років виявлено такі зміни: права легеня помірно щільна в усіх відділах, на розрізі тканина безповітряна, дрібнозерниста, сухувата. Вісцеральна плевра має нашарування фібрину сіро-коричневого кольору. Для якого захворювання характерні такі патологічні зміни?

Туберкульозу

Крупозної пневмонії

Інтерстиціальної пневмонії

Бронхопневмонії

5309 / 6307
Потерпілому після ДТП ампутовано нижню кінцівку. Упродовж тривалого часу він відчував нестерпний біль в ампутованій кінцівці. Який вид болю виник у пацієнта?

Каузалгічний

Відбитий

Рефлекторний

Фантомний

5310 / 6307
Дитина віком 6 років померла від дихальної недостатності внаслідок паралічу дихальної мускулатури. Під час гістологічного дослідження грудного відділу спинного мозку виявлено повнокрівʼя, стертий малюнок сірої речовини, крапельні крововиливи, дрібні ділянки розм'якшення тканини мозку, що западають, а також запалення з проліферацією нейроглії навколо загиблих нейронів. Для якого захворювання характерні такі патологічні зміни?

Поліомієліту

Цитомегалії

Менінгококової інфекції

Токсоплазмозу

5311 / 6307
Пацієнт не розуміє зверненої до нього мови. Після обстеження лікар встановив діагноз: сенсорна афазія. Яка ділянка нервової системи уражена?

Середня скронева звивина

Верхня скронева звивина

Верхня лобова звивина

Середня лобова звивина

5312 / 6307
У пацієнтки віком 36 років спостерігається місяцеподібне обличчя, ожиріння верхньої частини тулуба, стриї на передній черевній стінці, гірсутизм, гіперглікемія та глюкозурія. Для якої патології характерні такі ознаки?

Вторинного гіперальдостеронізму

Синдрому Іценка-Кушинга

Первинного гіперальдостеронізму

Феохромоцитоми

5313 / 6307
У медичній практиці для профілактики алкоголізму широко використовують тетурам, який є інгібітором альдегіддегідрогенази. Підвищення у крові якого метаболіту викликає відразу до алкоголю?

Малонового альдегіду

Ацетальдегіду

Метанолу

Пропіонового альдегіду

5314 / 6307
Пацієнт скаржиться на різке збільшення діурезу (до 5-7 л сечі за добу). Обстеження виявило, що у нього зменшена секреція вазопресину. Які клітини мають недостатню секреторну активність?

Ендокриноцити передньої частки гіпофіза

Клітини туберальної частини гіпофіза

Ендокриноцити проміжної частки гіпофіза

Нейросекреторні клітини гіпоталамуса

5315 / 6307
Пацієнтка звернулася до лікаря зі скаргами на ущільнення у верхньо-бічній ділянці правої молочної залози. Які лімфатичні вузли повинен перевірити лікар, щоб переконатися в нерозповсюдженні патологічного процесу?

Міжреберні

Пахвові

Передні середостінні

Білягрудинні

5316 / 6307
Надмірне утворення вільних радикалів спричиняє пошкодження клітин. Укажіть неферментативний фактор антиоксидантної системи захисту клітини.

Ціанокобаламін

Вітамін E

Супероксиддисмутаза

Глутатіонредуктаза

5317 / 6307
Під час розтину тіла плода від другої вагітності Rh-негативної матері, який помер внутрішньоутробно на сьомому місяці вагітності, виявлено поширені набряки підшкірної клітковини та головного мозку, асцит, збільшену печінку та селезінку, атрофію тимуса та гіпертрофію міокарда. Мікроскопічно спостерігається: осередки екстрамедулярного кровотворення, дистрофічні та некробіотичні зміни у внутрішніх органах. Для якого

Післяпологової жовтяничної форми гемолітичної хвороби

Анемічної форми гемолітичної хвороби

Набрякової форми гемолітичної хвороби

Вродженої жовтяничної форма гемолітичної хвороби

5318 / 6307
Під час емоційного перевантаження у пацієнтки віком 30 років частота серцевих скорочень сягнула 112/хв. Яка структура провідної системи серця спричинила цей стан?

Внутрішньошлуночковий вузол

Гілки пучка Гіса

Пучок Гса

Волокна Пуркіньє

Синоатріальний вузол

5319 / 6307
Під час субмікроскопічного дослідження клітини виявлено, що її цитоплазма містить багато лізосом, фагосом, піноцитозних міхурців. Інші органели розвинені помірно. Яку функцію може виконувати така клітина?

Реабсорбція іонів натрію

Депонування іонів кальцію

Синтез ліпідів

Фагоцитоз

5320 / 6307
Пацієнту віком 57 років після обстеження встановлено діагноз: В12-дефіцитна анемія. За 3 доби після призначеного лікування зроблено контрольний аналіз крові. Який критерій укаже на підвищення еритропоезу?

Підвищення рівня гемоглобіну

Підвищення кількості ретикулоцитів

Зниження кольорового показника

Підвищення кількості лейкоцитів

5321 / 6307
Пацієнт із бронхіальною астмою тривалий час приймає преднізолон. Який механізм дії цього препарату?

Пригнічення активності дигідрофолатредуктази

Блокада лейкотрієнових рецепторів

Гальмування активності фосфоліпази А

Блокада гістамінових рецепторів

5322 / 6307
Швидкість окиснення субстратів клітинного палива і транспорту електронів до кисню дихальним ланцюгом регулюється за допомогою дихального контролю. Від чого він залежить?

Від співвідношення АТФ/АМ

Від концентрації АДФ

Від співвідношення АТФ/(АДФ+АМФ)

Від концентрації АМФ

5323 / 6307
На які зміни з боку ізольованого серця жаби можна очікувати після введення у перфузійний розчин надлишкової кількості хлориду кальцію?

Зменшення сили скорочень

Зупинка серця в діастолі

Збільшення частоти і сили скорочень

Збільшення сили скорочень

5324 / 6307
У пацієнта під час огляду спостерігаються кальциноз шкіри, синдром Рейно, порушення моторики стравоходу, склеродактилія та телеангіоектазії. Ці зміни називаються «синдром CREST». Для якого захворювання характерні такі симптоми?

Дерматоміозит

Системна склеродермія

Ревматоїдний артрит

Подагричний артрит

5325 / 6307
У пацієнта добовий діурез становить 7 літрів. Рівень глюкози крові в нормі. Порушення секреції якого гормону є причиною цього стану?

Глюкагону

Інсуліну

Тироксину

Вазопресину

5326 / 6307
Пацієнту з неоперабельним раком легені, що супроводжується болем, який важко терпіти, лікар призначив знеболювальний засіб. За кілька днів після приймання цього препарату у пацієнта виникли явища непрохідності кишечника. Який знеболювальний препарат міг спричинити таке ускладнення?

Морфін

Фентаніл

Анальгін

Омнопон

5327 / 6307
Коли чужорідний агент потрапляє в організм, синтез двох класів імуноглобулінів починається майже паралельно, але в одного з них зростання і паління кількості концентрації відбувається швидше. Які це два класи імуноглобулінів?

IgA i IgG

IgG i lgD

IgM i lgD

IgM i IgG

IgA i lgD

5328 / 6307
Лімфоцит уражено ретровірусом ВІЛ (СНІД). Який напрямок течії інформації у клітині в цьому разі?

РНК > ДНК > іРНК > поліпептид

ДНК > іРНК > поліпептид > ДНК

іРНК > поліпептид > ДНК

ДНК > поліпептид > іРНК

5329 / 6307
У пацієнта із порушенням серцевого ритму під час проведення ЕКГ спостерігається наступне: ЧСС - 60/хв, інтервал РО подовжений, періодичне випадіння комплексу QRS. Яке порушення серцевого ритму виявлено у пацієнта?

Повна AV-блокада

Блокада правої ніжки пучка Гіса

Неповна AV-блокада II ступеня

Синдром слабкості синусового вузла

5330 / 6307
Під час оперативного втручання із застосуванням інгаляційного наркозу та міорелаксантів, лікар-анестезіолог помітив у пацієнта швидке підвищення температури тіла (до 43°С). Яка патологія розвинулася у пацієнта?

Гіпертермічний синдром

Фізична гіпертермія

Інфекційна гарячка

Перегрівання

5331 / 6307
Пацієнтка віком 32 роки скаржиться на безпліддя, порушення менструального циклу, хронічну ановуляцію та полікістоз яєчників. Під час бімануального обстеження з обох сторін виявлено збільшені яєчники щільної консистенції, розмірами 5х6 см. Ці ж дані підтверджено при УЗД. Під час гістологічного дослідження виявлено потовщену фіброзну капсулу яєчника, що вкриває незліченні кістозні фолікули, вистелені гранульозними клітинами з гіперпластичною лютеїновою внутрішньою оболонкою (текою). Жовтих тіл немає. Що спричинило цей стан у пацієнтки?

Хронічний двосторонній аднексит

Передменструальний синдром

Синдром полікістозних яєчників (синдром Штейна-Левенталя)

Андробластома яєчників

5332 / 6307
До лікаря звернувся пацієнт зі скаргами на свербіння між пальцями рук і на животі, що посилюється вночі. Під час огляду на шкірі виявлено тоненькі смужки сірого кольору та дрібний висип. Який імовірний збудник цього захворювання?

Sarcoptes scabiei

Ixodes persulcatus

Ixodes ricinus

Ornitodorus papillipes

5333 / 6307
На третьому тижні ембріогенезу центральна частина клітин епібласту (ектодерми) прогинається і починається процес нейруляції. В якому напрямку диференціюється решта клітин ектодерми?

Соміти

Шкіра

Кишки

Хорда

5334 / 6307
У пацієнтки під час обстеження клітин слизової оболонки щоки не знайдено статевого хроматину. Визначте ймовірне захворювання у пацієнтки.

Трисомія X

Хвороба Вільсона-Коновалова

Хвороба Шерешевського-Тернера

Синдром Дауна

5335 / 6307
Пацієнтці віком 30 років, яка перебуває на 32 тижні вагітності, у складі комплексної терапії виразкової хвороби шлунка лікар призначив антибіотик. Який із наведених нижче препаратів можна рекомендувати у разі виникнення такого ускладнення?

Тетрациклін

Азитроміцин

Левоміцетин

Бензилпеніцилін

5336 / 6307
Під час експерименту на клітину подіяли тетраетиламонієм, який блокує калієві іоноселективні канали. Як це позначиться на мембранному потенціалі?

Потенціал спокою зникне

Потенціал дії не виникне

Потенціал спокою збільшиться

Потенціал спокою не зміниться

5337 / 6307
Пацієнт віком 60 років звернувся до лікаря зі скаргами на біль у суглобах. У сироватці крові пацієнта виявлено підвищення концентрації С-реактивного білка та оксипроліну. Для якого захворювання характерні ці симптоми?

Цукровий діабет

Подагра

Гепатит

Ревматизм

5338 / 6307
У пацієнта з ознаками недостатності мітрального клапана в анамнезі відзначалися напади ревматизму, які супроводжувалися запальними явищами в суглобах. Яке з патологічних явищ у цього пацієнта належить до категорії патологічний стан?

Ревматизм

Ревмокардит

Запалення суглобів

Недостатність мітрального клапана

Артрит

5339 / 6307
Пацієнт упродовж 15 років хворіє на бронхіальну астму. Які зміни лейкоцитарної формули у пацієнта може спричинити це захворювання?

Лейкоцитоз

Еозинофілію

Зсув лейкоцитарної формули вліво

Лейкопенію

5340 / 6307
Відомо, що ацетилсаліцилова кислота та глюкокортикоїди мають виражену протизапальну дію. Укажіть на синтез яких біологічно активних речовин не впливає ацетилсаліцилова кислота на відміну від глюкокортикоїдів.

Лейкотрієнів

Тромбоксанів

Простагландинів F

Простациклінів

5341 / 6307
Потенціал спокою був підвищений відповідно до активації іонних каналів зовнішньої мембрани збудливої клітини. Які канали активовані?

Натрієві

Калієві

Кальцієві

Хлорні

5342 / 6307
У пацієнтки віком 49 років після тривалого стояння набрякли ноги. Яка ймовірна причина виникнення набряків?

Зменшення гідростатичного тиску крові у венах

3меншення гідростатичного тиску крові в артеріях

Підвищення гідростатичного тиску крові у венах

Збільшення онкотичного тиску плазми крові

5343 / 6307
Під час гематологічного дослідження крові пацієнта, отримано такі результати: еритроцити - 2,8*10^12/л, НЬ - 80 г/л, КП - 0,85, ретикулоцити - 0,1%, тромбоцити - 160 тис. в мкл, лейкоцити - 60 • 10^9, базофіли - 2%, еозинофіли - 8%, промієлоцити - 5%, мієлоцити-5%, юні - 16%, паличкоядерні – 20% , сегментоядерні - 34%, лімфоцити - 5%, моноцити - 5%. Про яку форму патології крові свідчать ці результати дослідження?

Гіпопластична анемія

Гемолітична анемія

Хронічний мієлолейкоз

Гострий мієлолейкоз

Недиференційований лейкоз

5344 / 6307
На гістологічному препараті представлені три нейрони: псевдоуніполярний, біполярний та мультиполярний. Скільки аксонів можливо визначити в кожного з вищенаведених типів клітин?

Два

Чотири

Жодного

Один

5345 / 6307
Під час хірургічного лікування стегнової грижі лікар виділяє поверхневий отвір стегнового каналу. Яка анатомічна структура його утворює?

Hiatus saphenus

Fascia pectinea

Fossa femoralis

Arcus iliopectineus

5346 / 6307
За клінічними показниками пацієнту призначено піридоксальфосфат. Для корекції яких процесів рекомендовано цей препарат?

Синтезу білка

Окисного декарбоксилювання кетокислот

Синтезу пуринових і піримідинових основ

Трансамінування і декарбоксилювання амінокислот

Дезамінування пуринових нуклеотидів

5347 / 6307
У пацієнта відбувся напад бронхоспазму. Які мембранні циторецептори гладеньких мʼязів бронхів треба стимулювати для покращення стану пацієнта?

альфа-адренорецентори

бета-адренорецентори

Н2-гістамінові репентори

Н-холінорецентори

5348 / 6307
Під час падіння із висоти пацієнт отримав перелом стегнової кістки. Який препарат треба призначити для зняття больового синдрому?

Морфіну гідрохлорид

Ацетилсаліцилову кислоту

Пентамін

Димедрол

5349 / 6307
Молекула зрілої інформаційної РНК має меншу довжину, ніж відповідний ген молекули ДНК. Неінформативна послідовність нуклеотидів про-іРНК видаляється під час процесингу. Укажіть назву цих ділянок.

Екзони

Транскриптони

Мутони

Кластери

Інтрони

5350 / 6307
Під час адаптації до перебування в горах, в еритроцитах зростає синтез 2,3-дифосфогліцерату. Що стимулює ця речовина в організмі людини?

Утворення оксигемоглобіну

Дисоціацію оксигемоглобіну

Тканинне дихання

Окисне фосфорилювання

5351 / 6307
У пацієнтки віком 45 років, якій діагностовано пієлонефрит, лікар виявив стійку артеріальну гіпертензію. Який механізм виникнення цього стану?

Активація процесів ацидогенезу і амоніогенезу в нирках

Активація центральних холінергічних механізмів

Активація ренін-ангіотензинової системи

Активізація синтезу ангіотензинази в тканинах нирки

5352 / 6307
До лікаря-дерматолога звернулася пацієнтка віком 22 роки зі скаргами на гнійне висипання на обличчі та спині. Із анамнезу відомо, що у неї виявлено Н.pylori. Враховуючи таку супутню патологію, лікар призначив антибактеріальний препарат, який буде діяти як на збудників інфекцій мʼяких тканин, так і на H. pylori. Який антибактеріальний препарат призначив лікар?

Кларитроміцин

Ізоніазид

Озельтамівір

Флуконазол

5353 / 6307
Пацієнту із цукровим діабетом другого типу лікар призначив пероральний препарат, який разом із гіпоглікемічним ефектом також має гіпохолестеринемічну дію. Препарат відноситься до похідних сульфонілсечовини. Укажіть цей препарат.

Глібенкламід

Акарбоза

Новокаїнамід

Інсулін

Преднізолон

5354 / 6307
У пацієнта, який довгий час незбалансовано харчувався, уживав малу кількість білка, розвинулася жирова інфільтрація печінки. Відсутність якої речовини в їжі призвела до цього стану?

Холестерин

Оцтова кислота

Біотин

Аланін

Метіонін

5355 / 6307
Під час аутопсії тіла пацієнта з діагнозом: мієломна хвороба, який помер з явищами серцевої недостатності, виявлено збільшене в розмірах серце з розширеними порожнинами. Міокард блідий, має щільну консистенцію і воскоподібний блиск. Мікроскопічно під час забарвлення конго-рот визначається позитивна реакція. Для якої патології серця характерні такі результати дослідження?

Гіпертензивне серце

Ожиріння серця

Сальне серце

5356 / 6307
Пацієнту встановлено діагноз: виразкова хвороба шлунка із підвищеною кислотністю. Під час ендоскопічного та бактеріологічного досліджень виділено бактерії роду Helicobacter. Завдяки якій своїй властивості ці мікроорганізми не гинуть у кислому середовищі шлунка?

Уреазній активності

Здатності утворювати капсулу

Стійкості до ванкоміцину

Оксидазній активності

5357 / 6307
Група дослідників, що вивчають фізіологію серця, виявили, що надмірне розтягнення передсердь серця веде до зниження реабсорбції натрію в дистальному звивистому канальці та підвищення швидкості клубочкової фільтрації. Що із перерахованого є ймовірною причиною фізіологічних змін, виявлених дослідниками?

Ренін

Ангіотензин

Натрійуретичний гормон

Вазопресин

5358 / 6307
Під час дослідження біоптату виявлено гранульому, що складається з лімфоцитів, плазматичних клітин, макрофагів із пінистою цитоплазмою (клітини Мікуліча) та великої кількості галінових куль. Для якого захворювання характерні такі показники?

Сифілісу

Туберкульозу

Риносклероми

Актиномікозу

5359 / 6307
До приймального відділення шпиталізовано пацієнта в непритомному стані. Обʼєктивно спостерігається: шкіра холодна, зіниці звужені, дихання ускладнене, відзначається періодичність дихання за типом Чейна-Стокса, артеріальний тиск знижений, сечовий міхур переповнений. Встановлено діагноз: отруєння морфіном. Який препарат треба застосувати в якості антагоніста?

Тіосульфат натрію

Унітіол

Налоксон

Бемегрид

5360 / 6307
Лікар вимірює зовнішні розміри таза та визначає відстань між великими вертлюгами стегнових кісток. Який розмір визначив лікар?

Conjugata externa

Diameter oblique

Distantia intertrochanterica

Distantia intercristalis

5361 / 6307
Пацієнт віком 34 роки звернувся до лікаря-стоматолога із приводу зубного болю. Після видалення декількох зубів почалася кровотеча, що тривала понад 15 хвилин. Із анамнезу відомо: у пацієнта хронічний гепатит С. Яка ймовірна причина продовження кровотечі?

Зниження вмісту фібриногену в крові

Гіпокальціємія

Тромбоцитопенія

5362 / 6307
До інфекційного відділення шпиталізовано пацієнта із попереднім діагнозом: черевний тиф. Хворіє впродовж трьох днів. Температура - 39°С. Який метод лабораторної діагностики треба застосувати для підтвердження діагнозу?

Виділення гемокультури

Виділення уринокультури

Виділення копрокультури

Серологічний метод

5363 / 6307
У пацієнта, який отримав множинні травми під час ДТП, розвинувся шок із гострою нирковою недостатністю, у результаті чого він помер. Під час розтину тіла виявлено: обидві нирки збільшені в розмірах та набряклі; корковий шар блідо-сірий, мозковий - темно-червоний. Який патологічний процес спричинив ниркову недостатність

Гідронефроз

Амілоїдоз

Некротичний нефроз

Гострий гломерулонефрит

5364 / 6307
Для зменшення суглобового болю пацієнтка прийняла одночасно по таблетці парацетамолу та диклофенаку натрію. Який вид взаємодії ліків використала пацієнтка для самолікування?

Антагонізм неконкурентний

Антагонізм конкурентний

Адитивний синергізм

Синергоантагонізм

5365 / 6307
Пацієнт екстрено звернувся до лікаря із приводу численних укусів, отриманих від собаки. Який матеріал треба направити до мікробіологічної лабораторії для дослідження та встановлення діагнозу: сказ?

Мазок-відбиток з рогівки ока пацієнта

Спинномозкову рідину пацієнта

Мозок тварини, яка покусала пацієнта

Парні сироватки пацієнта

5366 / 6307
Цукровий діабет матері призвів до народження дитини з вродженими вадами опорнорухової, серцево-судинної та нервової систем. Вплив яких факторів спричинив розвиток діабетичної ембріопатії?

Тератогенних

Канцерогенних

Мутагенних

Екзогенних

Антропогенних

5367 / 6307
Пацієнт із діагнозом: ревматоїдний артрит, декілька тижнів приймав препарати глюкокортикостероїдів, а потім раптово припинив їх приймати. Яке ускладнення може виникнути в цьому разі?

Гіперглікемія

Загострення хронічних інфекційних процесів

Синдром відміни

Виразкування слизової оболонки шлунка і дванадцятипалої кишки

5368 / 6307
Під час дослідження фаз клітинного циклу, на одній із стадій мітозу клітина майже завершила поділ, а хромосоми, навколо яких починають формуватися ядра, деспіралізуються. Укажіть фазу клітинного циклу.

Анафаза

Телофаза

Профаза

5369 / 6307
Яка сполука є кінцевим продуктом катаболізму пуринових нуклеотидів у людини?

Сечова кислота

Ксантин

Гіпоксантин

Пурин

Алантоїн

5370 / 6307
Під час розтину тіла померлого чоловіка виявлено, що речовина головного мозку та лімфатичних вузлів має темно-сірий колір, а печінка та селезінка значно збільшені. Гістологічно в них спостерігаються гемомеланоз і гемосидероз. Із анамнезу відомо про періодичні напади гарячки. Для якого захворования характерні такі патогістологічні зміни?

Септицемія

Аддісонова хвороба

Гемолітична анемія

Малярія

5371 / 6307
Під час розтину тіла пацієнта, який помер після абдомінальної операції, виявлено численні тромби у венах малого таза. Клінічно зафіксовано тромбоемболічний синдром. Де потрібно шукати тромбоемболи?

У легеневих артеріях

У венах нижніх кінцівок

У лівому шлуночку серця

У портальній вені

5372 / 6307
Під час аварії на виробництві пацієнт зазнав токсичного впливу калію ціаніду, що спричинило блокаду цитохромоксидази. До якого патологічного процесу це призвело?

Гіпоксичної гіпоксіяї

Тканинної гіпоксії

Дихальної гіпоксії

Гемічної гіпоксії

Циркуляторної гіпоксії

5373 / 6307
Пацієнта госпіталізували до лікарні в коматозному стані. Із анамнезу відомо, що він хворіє на цукровий діабет другого типу впродовж 5 років. Обʼєктивно спостерігається: дихання шумне, глибоке, із рота відчувається запах ацетону. Уміст глюкози у крові становить 15,2 ммоль/л, кетонових тіл = 100 мкмоль/л. Для якого ускладнення цукрового діабету характерні такі симптоми?

Кетоацидотичної коми

Гіперглікемічної коми

Гіперосмолярної коми

Гіпоглікемічної коми

5374 / 6307
Продуктами гідролізу та модифікації деяких білків є біологічно активні речовини - гормони. Із якого з нижченаведених білків у гіпофізі утворюються ліпотропін, кортикотропін, меланотропін та ендорфін?

Нейростромін

Проопіомеланокортин (ПОМК)

Тиреоглобулін

Нейроглобулін

5375 / 6307
Гострий герпетичний гінгівостоматит є найпоширенішою первинною інфекцією, яку спричиняє вірус простого герпесу першого типу. Який матеріал треба взяти лікарю-стоматологу для лабораторного підтвердження діагнозу?

Мокротиння

Сечу

Рідину з везикул

Слину

5376 / 6307
Унаслідок хронічних захворювань при розростанні сполучної тканини у паренхімі печінки (фіброз) спостерігається порушення циркуляції крові у класичних часточках. Який напрямок руху крові в таких часточках?

Від центру до периферії

Від периферії до центру

Від основи до вершини

Від вершини до основи

5377 / 6307
У пацієнта під час огляду встановлено наявність атетозу та хореї. Яка структура ЦНС імовірно уражена?

Смугасте тіло

Довгастий мозок

Мозочок

Лімбічна система

5378 / 6307
В експерименті на собаці вивчали будову центральних відділів слухової сенсорної системи. У наслідок руйнування однієї зі структур головного мозку собака втратив орієнтувальний рефлекс на звукові сигнали. Яку структуру зруйновано?

Нижні горбки чотиригорбкового тіла

Латеральні колінчасті тіла

Медіальні колінчасті тіла

Верхні горбки чотиригорбкового тіла

5379 / 6307
У дитини віком 12 років за 5 днів після захворювання на грип з'явився сильний головний біль, нудота, запаморочення та менінгеальні прояви. Дитина померла за добу після появи набряку мозку, що наростав. Під час розтину порожнини черепа виявлено: мʼякі мозкові оболонки набряклі, повнокровні, дифузно просякнуті рідиною яскраво-червоного кольору, звивини і борозни згладжені. Яке ускладнення грипу можна припустити в цьому разі?

Серозний менінгіт

Крововилив у мозок

Геморагічний менінгіт

Гнійний лептоменінгіт

5380 / 6307
На розтині легенева тканина має комірчастий вигляд через мішковидні та циліндричні розширення бронхів. Мікроскопічно у стінці цих бронхів спостерігається лейкоцитарна інфільтрація із переважанням нейтрофілів. Еластичні мʼязові волокна та хрящові пластини частково зруйновані і заміщені сполучною тканиною. Прилегла легенева тканина має осередки запалення, поля фіброзу та склерозу судин і ознаки емфіземи. У серці

Хронічний бронхіт

Пневмофіброз

Емфізема легень

Бронхоектатична хвороба

5381 / 6307
У дитини із хворобою Гірке спостерігаються сповільнений ріст і збільшення печінки та нирок. У крові знижений вміст глюкози та підвищений вміст жирів і сечової кислоти. Відсутність якого ферменту викликає цей вид глікогенозу?

Глікогенсинтетази

Фосфофруктокінази

Печінкової фосфорилази

Глюкозо-6-фосфат-дегідрогенази

5382 / 6307
У пацієнта віком 30 років, який шпиталізований із діагнозом: гострий гломерулонефрит, спостерігалася протеїнурія. Яке порушення спричинило це явище?

Зниження онкотичного тиску крові

Підвищення проникності клубочкової мембрани

Зниження кількості нефронів, що функціонують

Затримка виведення продуктів азотистого обміну

5383 / 6307
Пацієнту, який займається мисливством, на підставі анамнезу та наявних клінічних симптомів, лікар встановив попередній діагноз: бубонна чума. Який механізм зараження при цій клінічній формі захворювання?

Аліментарний

Трансмісивний

Повітряно-крапельний

Фекально-оральний

5384 / 6307
У пацієнта виявлено сліпоту кіркового походження. Тромбоз якої артерії в нього розвинувся?

Середньої мозкової

Передньої ворсинчастої

Задньої сполучної

Задньої мозкової

5385 / 6307
У пацієнта віком 30 років, на електрокардіограмі виявлено зниження амплітуди зубця R. Що означає цей зубець на ЕКГ?

Деполяризацію шлуночків

Електричну діастолу серця

Поширення збудження від передсердь до шлуночків

Реполяризацію шлуночків

5386 / 6307
Під час виконання оперативного втручання на колінному суглобі лікар повинен памʼятати про артерію, що віддає найбільшу кількість гілок для утворення артеріальної суглобової сітки. Укажіть цю артерію.

A. tibialis anterior

A. tibialis posterior

A. poplitea

A. femoralis

5387 / 6307
У чотирирічної дитини спостерігаються гепатомегалія та гіпоглікемія. Біохімічним методом виявлено дефіцит глюкозо-6-фосфатази. Яке захворювання ймовірно у пацієнта?

Хвороба Гірке

Хвороба Помпе

Алкаптонурія

Хвороба Тея-Сакса

5388 / 6307
У пацієнта зупинилося дихання внаслідок травми в ділянці потилиці. Що могло стати причиною апное?

Розрив спинного мозку нижче п'ятого шийного сегмента

Ушкодження мозочка

Розрив між середнім і довгастим мозком

Ушкодження довгастого мозку

5389 / 6307
Пацієнту з діагнозом: гострий інфаркт міокарда, призначено антикоагулянтну терапію. Вимірювання якого показника системи згортання крові потрібне в разі приймання гепарину, щоб попередити можливі ускладнення через його передозування?

Міжнародного нормалізованого відношення

Активованого часткового тромбопластинового часу

ШОЕ

Протромбінового часу

Концентрації фібриногену

5390 / 6307
У пацієнта виявлено змішану глистну інвазію: аскаридоз кишечника та трематодоз печінки. Який протигельмінтний препарат доцільно призначити у цьому разі?

Пірантел

Мебендазол

Піперазину адипінат

Хлоксил

5391 / 6307
У пацієнта, який пройшов тривалий курс лікування глюкокортикоїдами, виявлено виразки у шлунку. Укажіть провідний механізм розвитку цієї патології.

Збільшення продукції простагландинів El, E2

Збільшення секреції та кислотності шлункового соку

Зниження тонусу парасимпатичної нервової системи

Зниження гістаміну в слизовій оболонці шлунка

5392 / 6307
Збудник гепатиту D (дельта-агент) є дефектним вірусом, який може реплікуватися лише в клітинах, що вже інфіковані одним із нижченаведених вірусів. Укажіть цей вірус.

Гепатиту В

Імунодефіциту людини

Гепатиту E

Гепатиту А

Етштейна-Барр

5393 / 6307
Пацієнту встановлено діагноз: водянка яєчка (гідроцеле). Яка оболонка яєчка утворює порожнину, усередині якої накопичується рідина?

Tunica dartos

Fascia spermatica interna

Tunica vaginalis testis

Tunica albuginea

5394 / 6307
Пацієнт віком 55 років прооперований із приводу гострого апендициту. 3а 5 днів після підняття з ліжка відчув нестачу повітря. У нього розвинувся різкий ціаноз обличчя. Пацієнт втратив свідомість. Після безрезультатної реанімації була констатовано смерть. На розтині виявлено тромбоемболію легеневого стовбура. Яке ймовірне джерело тромбоемболії?

Тромбоз у лівому шлуночку серця

Тромбоз брижових артерій

Тромбоз вен нижніх кінцівок

Тромбоз ворітної вени

5395 / 6307
Генний апарат людини містить близько 30 тисяч генів, а кількість варіантів антитіл сягає мільйонів. Який механізм використовується для утворення нових генів, що відповідають за синтез такої кількості антитіл?

Реплікація ДНК

Ампліфікація генів

Репарація ДНК

Рекомбінація генів

5396 / 6307
Біотерорист надіслав у поштовому конверті порошок, що імовірно містить збудника сибірки. Яка властивість збудника сибірки робить його небезпечним упродовж тривалого часу?

Утворює спори

Утворює протеїнову капсулу

Утворює джгутики

Утворює полісахаридну капсулу

5397 / 6307
У пацієнта з ослабленим імунітетом, віком 36 років, діагностовано простий герпес слизової оболонки губ. Як компонент комплексної терапії лікар призначив йому препарат місцевого застосування, що має противірусну активність. Укажіть цей лікарський засіб.

Ацикловір

Ремантадин

Тималін

Інтерферон

5398 / 6307
Під час розтину тіла чоловіка віком 67 років, виявлено ознаки фібринозного запалення в товстому кишечнику. Для якого захворювання характерні такі патологічні зміни?

Амебіазу

Черевного тифу

Холери

Дизентерїї

Балантидіазу

5399 / 6307
До інфекційної лабораторії надійшли випорожнення пацієнта з діагнозом холера. Який метод мікробіологічної діагностики потрібно застосувати, щоб підтвердити чи спростувати цей діагноз?

Біологічний

Бактеріологічний

Алергічний

Вірусологічний

5400 / 6307
Аміак особливо токсичний для ЦНС людини. Укажіть головний шлях його знешкодження в нервовій тканині.

Синтез глутаміну

Синтез сечовини

Утворення парних сполук

Трансамінування

Синтез солей амонію

5401 / 6307
Синдром Верніке-Корсакова зазвичай спостерігається у хронічних алкоголіків, харчовий раціон яких містить мало вітамінів, що спричиняє зниження активності транскетолази. Дефіцит якого вітаміну до цього призводить?

Кобаламіну

Ретинолу

Тіаміну

Рибофлавіну

5402 / 6307
Під час дослідження стегнової кістки виявлено хронічне гнійне запалення компактної речовини та кісткового мозку, а також утворення секвестрів. При якому захворюванні розвиваються такі зміни?

Остеомієліт

Остеобластокластома

Періостит

Ретикулосаркома

5403 / 6307
Стан вагітної жінки ускладнився токсикозом. Під час лабораторного обстеження виявлено кетонурію. Яка речовина зʼявилася в сечі пацієнтки?

Креатинін

Лактат

Урати

Ацетоацетат

5404 / 6307
Пацієнту з діагнозом: гіпертонічна хвороба (АТ - 200/110 мм рт. ст.) серед препаратів комплексної терапії лікар призначив анаприлін (пропранолол). За 2 тижні після початку приймання цього лікарського засобу, пацієнт почав скаржитися на відчуття задухи та ускладнене дихання. Що є можливою причиною ускладнень і яку тактику потрібно вибрати в цій ситуації?

Блокада бета2-адренореценторів. Призначити селективний бета1-адреноблокатор

Міотропна бронхоспастична дія. Призначити еуфілін

Блокада бета1-адренорецепторів. Призначити селективний бета2-адреноблокатор

Алергічна реакція. Приймання препарату відмінити, призначити антигістамінні засоби

Збудження М-холінорецепторів. Призначити атропін

5405 / 6307
За результатами аналізу ЕКГ треба визначити водія ритму серця. На підставі якого показника це можна зробити?

Напрямок зубця R

Напрямок зубця Р

Напрямок зубця О

Амплітуду зубця Р

Амплітуду зубця R

5406 / 6307
Пацієнту, якому діагностовано гастрит із підвищеною секрецією, лікар призначив блокатор протонної помпи. Яке фізіологічне обґрунтування цього призначення лікаря?

Блокується робота К+/Na+ помпи

Знижується надходженя іонів Н+ у порожнину шлунка

Знижується надходження іонів Н+ у парієтальні клітини

Знижується метаболізм парієтальних клітин

5407 / 6307
Пацієнтці з діагнозом: ішемічна хвороба серця, призначено ацетилсаліцилову кислоту. Синтез якої ендогенної речовини зумовлює антиагрегантну дію цього препарату?

Тромбоксану А2

Простагландину I2 (простацикліну)

Лейкотрієнів

Простагландину E1

Простагландину E2

5408 / 6307
У людини синтез у лімбічній системі мозку диоксифенілаланіну (ДОФА) зумовлює виникнення почуття страху. З якої амінокислоти синтезується ДОФА?

Триптофану

Тирозину

Глутамінової кислоти

5-окситриптофану

Лізину

5409 / 6307
У хворого на цукровий діабет після ін'єкції інсуліну настала втрата свідомості, судоми. Який результат дав біохімічний аналіз крові на вміст глюкози?

3,3 ммоль/л

10 ммоль/л

8,0 ммоль/л

2,5 ммоль/л

5,5 ммоль/л

5410 / 6307
Під час розтину тіла чоловіка віком 50 років виявлено такі зміни: права легеня помірно щільна в усіх відділах, на розрізі тканина безповітряна, дрібнозерниста, сухувата. Вісцеральна плевра має нашарування фібрину сіро-коричневого кольору. Для якого захворювання характерні такі патологічні зміни?

Туберкульозу

Інтерстиціальної пневмонії

Крупозної пневмонії

Бронхопневмонії

Пневмофіброзк

5411 / 6307
Відомо, що в осіб із генетично обумовленою недостатністю глюкозо-6-фосфатдегідрогенази еритроцитів у відповідь на призначення деяких протималярійних препаратів може розвиватися гемоліз еритроцитів. Як називається цей прояв атипових реакцій на лікарські засоби?

Алергія

Ідіосинкразія

Толерантність

Тахіфілаксія

Сенсибілізація

5412 / 6307
У 47-річного чоловіка діагностовано: артрит великого пальця правої ноги і камені в нирках, що складаються з сечової кислоти. Пацієнт приймає алопуринол. Який біохімічний дефект, імовірно, був виявлений у цнього?

Порушення метаболізму піримідинів

Порушення метаболізму арахідонової кислоти

Підвищення рівня лейкотрієнів

Дефект синтезу сечовини

Порушення метаболізму пуринів

5413 / 6307
На розтині тіла жінки 72-х років, що страждала на ревматоїдний артрит і померла від уремії, були виявлені збільшені нирки, щільної консистенції, блідо-сірого кольору, з блискучою сальною поверхнею розрізу. На основі макроскопічних змін патологоанатом припустив:

Хронічний пієлонефрит

Хронічний гломерулонефрит

Первинно зморщені нирки

Атеросклеротичний нефросклероз

Амілоїдоз нирки

5414 / 6307
У пацієнта діагностовано мегалобластичну анемію на фоні атрофічного гастриту. Який препарат, введений лише парентерально, буде профілактикою цієї патології?

Ціанокобаламін

Пірацетам

Прозерин

Целекоксиб

Преднізолон

5415 / 6307
Пацієнту віком 55 років для лікування легеневої форми туберкульозу, в рамках комплексної терапії, призначено протитуберкульозний засіб. Який лікарський засіб виявляє антимікробну активність виключно до мікобактерій туберкульозу?

Стрептоміцину сульфат

Циклосерин

Гатіфлоксацин

Ізоніазид

Канаміцину сульфат

5416 / 6307
Під час розтину тіла чоловіка, який помер із явищами легенево-серцевої недостатності, виявлено, що бронхи деформовані, із мішковидними вип'ячуваннями стінки та гнійним запаленням. У серці виявлено гіпертрофію правого шлуночка, у нирках - амілоїдоз. Із анамнезу відомо, що протягом 8 років пацієнт скаржився на кашель із гнійним мокротинням та задуху, а його пальці рук нагадували барабанні палички. Якому захворюванню характерні такі патологічні зміни?

Хронічному бронхіту

Абсцесу

Бронхоектатичній хворобі

Гострому броніту

Туберкульозу

5417 / 6307
Повільне наповнення шлунка чи сечового міхура в межах фізіологічної норми не викликає підвищення тиску в цих органах. Яка властивість гладеньких м'язів лежить в основі цього явища?

Збудливість

Автоматія

Рефрактерність

Скоротливість

Пластичність

5418 / 6307
Під час гістологічного дослідження вушка серця у пацієнта зі стенозом мітрального клапана, були виявлені гранульоми Ашоффа-Талалаєва. Про який генез вади серця свідчить гістологічне дослідження?

Сифілітичний

Ревматичний

Септичний

Атеросклеротичний

Природжений

5419 / 6307
Дівчинка 6-ти років захворіла гостро. З'явилися біль у горлі, висока температура, приєднався дрібнокрапковий висип на шкірі. Під час огляду порожнини рота виявлена різка гіперемія зіву, малиновий язик, збільшені яскраво-червоного кольору мигдалики з сіруватими і жовтуватими тьмяними вогнищами, які розповсюджуються на перитонзилярні тканини, збільшені підщелепні лімфатичні вузли. Для якого захворювання характерні дані зміни?

Дифтерія зіву

Дифтерія гортані

Менінгококовий назофарингіт

Кір

Скарлатина

5420 / 6307
Під час обстеження пацієнтки на медіальній поверхні стегна спостерігає-ться припухлість, збільшення розмірів вен, утворення вузлів. Укажіть, яка вена вражена патологічним процесом?

V. iliaca externa

V. saphena magna

V. saphena parva

V. poplitea

V. femoralis

5421 / 6307
Порушення процесів розщеплення білків у тонкій кишці зумовлено порушенням активності трипсину та хімотрипсину. Від наявності якого фактора залежить активність цих ферментів?

Ентерокіназа

Солі Na^+

Жовчні кислоти

Пепсин

Соляна кислота

5422 / 6307
Під час захворювань внутрішніх орга-нів пацієнт часто набуває вимушених поз (наприклад, згинання і приведення нижніх кінцівок до живота). До якого виду рефлексів належить цей рефлекс?

Вісцеродермальні рефлекси

Дерматовісцеральні рефлекси

Моторновісцеральні рефлекси

Вісцеровісцеральні рефлекси

Вісцеромоторні рефлекси

5423 / 6307
Вірус імунодефіциту людини належить до родини ретровірусів. Що найхарактерніше для цієї родини вірусів?

Реакція імуноферментного аналізу для виявлення антигенів

Нуклеїнова кислота не інтегрується в геном хазяїна

Це прості віруси, що вражають тільки людину

Радіоімунний аналіз для виявлення антигенів

Наявність ферменту зворотної транскриптази

5424 / 6307
У слизовій кишки хірургом виявлені скупчені лімфоїдні вузлики (Пеєрові бляшки). Яка це кишка?

Сліпа

Клубова

Порожня

Пряма

Дванадцятипала

5425 / 6307
До лікарні госпіталізовано пацієнта з попереднім діагнозом: гострий панкреатит. Активність якого ферменту в крові та сечі буде різко підвищена у цьому разі?

Аспартатамінотрансферази

Аланінамінотрансферази

Креатинфосфокінази

Альфа-амілази

Лактатдегідрогенази

5426 / 6307
Після введення великої дози антитіл до базальної мембрани клубочків нирок, у піддослідної тварини розвинувся гострий гломерулонефрит. Який вид алергічної реакції за класифікацією Кумбса і Джелла лежить в основі цієї патології?

Стимулювальний

Гіперчутливість сповільненого типу

Імунокомплексний

Цитотоксичний

Анафілактичний

5427 / 6307
Реалізація загального адаптаційного синдрому здійснюється переважно через нейроендокринну систему. Якій з ланок цієї системи належить провідна роль у патогенезі реакції, що розвивається?

Гіпофізарно-адреналова

Гіпофізарно-тиреоїдна

Гіпофізарно-юкстагломерулярна

Гіпофізарно-інсулярна

Гіпофізарно-адреногенітальна

5428 / 6307
У пацієнта, який хворіє грипом спостерігаються: висока температура, задишка, тахікардія. Як зміниться спорідненість Hb до кисню за таких умов?

Зменшується

Зростає

Змін не відбувається

Спочатку зростає, а потім зменшується

5429 / 6307
У пацієнта під час обстеження виявлено порушення роботи вузлів провідної системи серця. У басейні якої артерії відбулися порушення кровообігу?

Лівої вінцевої артерії

Огинаючій гілці лівої вінцевої артерії

Правої вінцевої артерії

Передньої міжшлуночкової гілки лівої вінцевої артерії

Правої та лівої вінцевих артеріях

5430 / 6307
У значної кількості пацієнтів перед відвідуванням стоматолога виникає тривожність, страх, пригнічений настрій. Посилення виділення якого медіатора центральної нервової системи може зменшити ці порушення психоемоційного стану у людини?

Серотоніну

Ацетилхоліну

Дофаміну

ГАМК

Норадреналіну

5431 / 6307
У гістологічному препараті біоптату епідермісу шкіри здорової дорослої людини в базальному шарі видно клітини, що діляться. Який процес забезпечують ці клітини?

Фізіологічну регенерацію

Диференціювання

Репаративну регенерацію

Адаптацію

Апоптоз

5432 / 6307
Дефіцит якого вітаміну проявляється симптомокоплексом, що називається пелагрою і характеризується тріадою: дерматит, діарея, деменція?

Вітамін В1

Вітамін А

Вітамін С

Вітамін В2

Вітамін РР

5433 / 6307
Група дослідників, що вивчають фізіологію серця, виявили, що надмірне розтягнення передсердь серця веде до зниження реабсорбції натрію в дистальному звивистому канальці та підвищення швидкості клубочкової фільтрації. Що із перерахованого є ймовірною причиною фізіологічних змін, виявлених дослідниками?

Ренін

Вазопресин

Натрійуретичний гормон

Ангіотензин

Альдостерон

5434 / 6307
У чоловіка, який помер у віці 38 років, виявлено множинні тромбоемболії різних органів, у тканині серця макрофагальні гранульоми у клапанному і пристіночному ендокарді, із його ураженням і накладенням тромботичних мас, а також макрофагальні гранульоми в міокарді. Для якого захворювання характерні такі паталогічні зміни?

Гіпертонічній хворобі

Ревматизму

Атеросклерозу

Інфекційному міокардиту

Системному червоному вовчаку

5435 / 6307
Тривале вживання великих доз аспірину викликає пригнічення синтезу простагландинів через зниження активності фермента:

Пероксидази

Циклооксигенази

5-ліпоксигенази

Фосфодіестерази

Фосфоліпази А_2

5436 / 6307
Хворому для обробки опіків призначили лікарський засіб, антисептичні властивості якого пов’язані з утворенням атомарного кисню в присутності органічних речовин. Препарату властива також в’яжуча (протизапальна) дія, обумовлена утворенням альбумінатів. Визначте препарат:

Натрію гідрокарбонат

Калію перманґанат

Водню пероксид

Спирт етиловий

Хлоргексидину біглюконат

5437 / 6307
У пацієнта, який отримав множинні травми під час ДТП, розвинувся шок із гострою нирковою недостатністю, у результаті чого він помер. Під час розтину тіла виявлено: обидві нирки збільшені в розмірах та набряклі; корковий шар блідо-сірий, мозковий - темно-червоний. Який патологічний процес спричинив ниркову недостатність?

Амілоїдоз

Гострий пієлонефрит

Некротичний нефроз

Гідронефроз

Гострий гломерулонефрит

5438 / 6307
Пацієнту встановлено діагноз: водянка яєчка (гідроцеле). Яка оболонка яєчка утворює порожнину, усередині якої накопичується рідина?

Tunica dartos

Tunica vaginalis testis

Fascia spermatica interna

Tunica albuginea

Fascia spermatica externa

5439 / 6307
Поверхня суглобів вкрита тканиною, в якій відсутні кровоносні судини. У міжклітинній речовині цієї тканини багато води, глікозаміногліканів та протеогліканів. Клітини цієї тканини утворюють ізогенні групи. Назвіть цю тканину.

Жирова

Ретикулярна

Хрящова

Власне сполучна

Кісткова

5440 / 6307
У чоловіка і його сина інтенсивно росте волосся по краю вушних раковин. Це явище спостерігалося також у батька і дідуся за батьковою лінією. Який тип успадкування зумовлює це?

Рецесивний, зчеплений з Х-хромосомою

Зчеплений з Y-хромосомою

Домінантний, зчеплений з Х-хромосо-мою

Аутосомно-домінантний

Аутосомно-рецесивний

5441 / 6307
У пацієнта виявлено порушення серцевого ритму. Для відновлення ритму лікар призначив хворому антагоністи кальцію. Як чином іони кальцію впливають на міокард?

Зменшують силу і частоту серцевих скорочень

Активують симпатичний відділ автономної нервової системи

Збільшують силу і частоту серцевих скорочень

Пригнічують симпатичний відділ автономної нервової системи

Активують парасимпатичний відділ автономної нервової системи

5442 / 6307
При розвитку колапсу спостерігаються розлади функцій нервової системи, порушення газообмінної функції легень, розлади системи крові і гемостазу. Первинне виникнення якої гіпоксії зумовлює ці прояви колапсу?

Циркуляторна

Тканинна

Гемічна

Гіпоксична

Дихальна

5443 / 6307
Хворому на гіпертонічну хворобу лікар призначив препарат, що пригнічує ангіотензин-перетворюючий фермент (АПФ). Який препарат було призначено?

Ніфедипін

Лізиноприл

Холестирамін

Лозартан

Карведилол

5444 / 6307
Процес тканинного дихання супроводжується окисненням органічних сполук і синтезом макроергічних молекул. У яких органелах відбувається цей процес?

Пероксисомах

Рибосомах

Комплексі Гольджі

Лізосомах

Мітохондріях

5445 / 6307
В лабораторію на дослідження доставлений травний сік, рН якого становить 2,2. Який це травний сік найімовірніше?

Жовч

Шлунковий

Кишковий

Слина

Підшлунковий

5446 / 6307
У каріотипі матері 45 хромосом. Встановлено, що це пов'язано з транслокацією 21-ї хромосоми на 14-ту. Яке захворювання найімовірніше буде у дитини, якщо каріотип батька нормальний?

Синдром Моріса

Синдром Патау

Синдром Едвардса

Синдром Дауна

Синдром Клайнфельтера

5447 / 6307
У пацієнтки віком 45 років, якій діагностовано пієлонефрит, лікар виявив стійку артеріальну гіпертензію. Який механізм виникнення цього стану?

Активізація синтезу ангіотензинази в тканинах нирки

Активація процесів ацидогенезу і амоніогенезу в нирках

Активація центральних холінергічних механізмів

Активація ренін-ангіотензинової системи

Зниження парціального тиску кисню в тканинах нирки

5448 / 6307
Під час реєстрації ЕКГ хворого з гіперфункцією щитоподібної залози зареєстровано збільшення частоти серцевих скорочень. Скорочення якого елемента ЕКГ про це свідчить?

Комплексу QRS

Сегмента P-Q

Інтервалу P-Q

Інтервалу R-R

Інтервалу P-T

5449 / 6307
Яка головна функція пентозофосфатного шляху в жировій тканині?

Знешкодження ксенобіотиків

Продукція рибозофосфатів

Генерація енергії

Генерація НАДФН2

Окислення глюкози до кінцевих продуктів

5450 / 6307
Серед дітей школи-інтернату спостерігаються випадки захворювання ангіною. Під час мікроскопічного дослідження мазків з мигдаликів, зафарбованих за методом Нейсера виявлені тонкі палички жовтого кольору з темно-коричневими зернами на кінцях, що розміщуються у вигляді римської цифри п'ять. Яку інфекцію необхідно запідозрити в цьому випадку?

Скарлатину

Інфекційний мононуклеоз

Тонзиліт

Лістеріоз

Дифтерію

5451 / 6307
Під дією декарбоксилаз утворюються біогенні аміни. Який біогенний амін запускає багатостадійний механізм регуляції секреції НСl у шлунку?

ГАМК

Дофамін

Гістамін

Серотонін

Глутамін

5452 / 6307
У досліді вивчали просторовий поріг шкірної чутливості. В яких ділянках шкіри він найбільший?

Плече

Обличчя

Спина

Тильна поверхня кисті

Гомілка

5453 / 6307
У мазку з матеріалу, взятого від пацієн-та, у якого підозрюється дифтерія, виявлені жовті палички з синіми зернами на кінцях. Який спосіб забарвлення було використано?

За Леффлером

За Цілем-Нільсеном

За Козловським

За Нейссером

За Романовським

5454 / 6307
Жінка 57-ми років померла від гострої легенево-серцевої недостатності. Під час розтину патологоанатом запідозрив жирову емболію легеневої артерії. Яким методом забарвлення мікропрепаратів потрібно скористатися для підтвердження такого діагнозу?

Пікрофуксином

Метиленовим синім

Гематоксилін-еозином

Конго-рот

Суданом III

5455 / 6307
Під час обстеження у пацієнта спостерігається гіперглікемія, кетонурія, поліурія, гіперстенурія та глюкозурія. Яка форма порушення кислотно-лужної рівноваги спостерігається в цій ситуації?

Метаболічний алкалоз

Змішаний алкалоз

Метаболічний ацидоз

Негазовий алкалоз

Газовий ацидоз

5456 / 6307
У чоловіка 45-ти років, що лікувався від застудного захворювання великими дозами парацетамолу, з'явилися симптоми олігурії, азотемії. Через 5 днів настала смерть при явищах ГНН. При гістологічному дослідженні нирок виявлений дифузний набряк проміжної тканини мозкового шару нирок, його інфільтрація лімфоцитами, еозинофілами, окремими нейтрофілами, руйнування епітелію канальців, клубочки мало змінені. Найбільш імовірний тип ураження нирок:

Пієлонефрит

Гострий гломерулонефрит

Нефротичний синдром

Тубулоінтерстиційний нефрит

Некронефроз

5457 / 6307
У дитини віком 12 років за 5 днів після захворювання на грип з'явився сильний головний біль, нудота, запаморочення та менінгеальні прояви. Дитина померла за добу після появи набряку мозку, що наростав. Під час розтину порожнини черепа виявлено: м'які мозкові оболонки набряклі, повнокровні, дифузно просякнуті рідиною яскраво-червоного кольору, звивини і борозни згладжені. Яке ускладнення грипу можна припустити в цьому разі?

Венозна гіперемія оболонок мозку

Геморагічний менінгіт

Гнійний лептоменінгіт

Серозний менінгіт

Крововилив у мозок

5458 / 6307
За рік після субтотальної резекції шлунку через виразку малої кривизни, виявлені зміни в аналізі крові: анемія, лейко- і тромбоцитопенія, КП - 1,3, наявність мегалобластів та мегалоцитів. Дефіцит якого фактору обумовив розвиток цієї патології?

Фактора Касла

Пепсину

Муцину

Хлороводневої кислоти

Гастрину

5459 / 6307
Чоловік не може розігнути коліно, у нього відсутня шкірна чутливість на передній поверхні стегна. Неврологічне обстеження виявило ураження нерва. Який із наведених нервів є ураженим?

Сідничного

Статево-стегнового

Затульного

Стегнового

Верхнього сідничного

5460 / 6307
У хворого на гіпертонічну хворобу при систематичному лікуванні гіпотензивним препаратом з'явився кашель. Який із нижчеперерахованих засобів може бути причиною побічної дії?

Лізиноприл

Апресин

Дибазол

Верапаміл

Празозин

5461 / 6307
Під час розтину тіла чоловіка віком 70 років, який помер від загального виснаження, виявлено зменшені у розмірах серце і печінка жовто-бурого кольору. За результатами мікроскопічного дослідження встановлено: в цитоплазмі кардіоміоцитів та гепатоцитів навколо ядер дрібні зерна пігменту бурого кольору, які дають негативну реакцію на залізо. Якому пігменту це властиво?

Гемосидерину

Гемомеланіну

Гематоїдину

Білірубіну

Ліпофусцину

5462 / 6307
У хворого 56 років із серцевою недостатністю спостерігається набряк стоп та гомілок, шкіра в місці набряку бліда і холодна. Яка провідна ланка патогенезу набряку у хворого?

Порушення лімфовідтоку

Підвищення проникності капілярів

Підвищення гідростатичного тиску в венулах

Зменшення онкотичного тиску в капілярах

Позитивний водний баланс

5463 / 6307
У результаті точкового крововиливу в сітківку ока пацієнт втратив здатність бачити предмети в центрі поля зору. В якому місці сітківки відбувся крововилив?

Ціліарна частина сітківки

Сліпа пляма

Райдужна частина сітківки

Судинна оболонка

Жовта пляма

5464 / 6307
У хворого на діабет 60-ти років в крові виявлено кетоацидоз. Біохімічним наслідком кетоацидозу за умов цукрового діабету є зменшення рівня використання ацетил-Ко-А внаслідок дефіциту:

Аланіну

Аспартату

Ацетату

Сукцинату

Оксалоацетату

5465 / 6307
До інфекційного відділення шпиталізовано дитину 5 років з високою температурою тіла та плямисто-папульозним висипом на шкірі. Лікар встановив діагноз: кір. За допомогою серологічних методів дослідження в сироватці крові були виявлені специфічні антитіла. Імуноглобуліни якого класу свідчать про гостру (початкову) стадію вірусної інфекції?

IgЕ

lgM

ІgA

IgD

ІgG

5466 / 6307
Стероїдні гормони полегшують зв'язування РНК-полімерази з промотором гена. Який етап синтезу білка у цьому разі активується?

Сплайсинг

Посттрансляційна модифікація

Процесинг

Транскрипція

Трансляція

5467 / 6307
До пульмонологічного відділення госпіталізовано хворого з гнійним бронхітом. У складі комплексної терапії йому призначили препарат для розрідження харкотиння і полегшення його відходження. Назвіть цей препарат:

Настоянка валеріани

Преднізолон

Ацетилцистеїн

Холосас

Морфіну гідрохлорид

5468 / 6307
Людину 35-ти років вкусила бджола. На місці укусу комахи визначається набряк, гіперемія, підвищення температури. Назвіть ініціальний патогенетичний фактор запального набряку:

Зниження онкотичного тиску крові

Підвищення осмотичного тиску у вогнищі запалення

Порушення лімфовідтоку

Підвищення кров'яного тиску в капілярах

Підвищення проникності мікросудин

5469 / 6307
Послідовність триплетів у ДНК виз-начає порядок розташування амінокислот у молекулі білка. Як називається така властивість генетичного коду?

Неперекривність

Колінеарність

Триплетність

Виродженість

Універсальність

5470 / 6307
Пацієнт, який хворіє на хронічну серцеву недостатність, упродовж тривалого часу приймав у таблетках дигоксин, який є серцевим глікозидом середньої тривалості дії. Це спричинило зниження гостроти зору та нудоту, що виникає час від часу. Яка властивість препарату зумовлює таку побічну дію?

Потенціювання

Толерантність

Кумуляція

Сенсибілізація

Лікарська залежність

5471 / 6307
У пацієнта після переохолодження у ділянці крил носа та верхньої губи з'явились герпетичні висипання. Для лікування була застосована мазь. Який противірусний засіб містить застосована мазь?

Азидотимідин

Дексаметазон

Ацикловір

Індометацин

Інтерферон

5472 / 6307
Жінка 35-ти років розпочала голодувати. Депо яких поживних речовин використовується у початковий період голодування і як при цьому змінюється дихальний коефіцієнт (ДК)?

Жири, ДК наближається до 0,72

Білки, ДК наближається до 0,7

Вуглеводи, ДК наближається до 1

Жири, ДК наближається до 0,85

Білки, ДК наближається до 1

5473 / 6307
До лікаря-стоматолога звернулася мати 10-річного хлопчика, хворого на гнійний гінгівіт, з запитанням: чи можна застосувати для лікування хвороби препарати з групи фторхінолонів. Лікар дав негативну відповідь, мотивуючи це тим, що фторхінолони:

Викликають пошкодження дентину

Сприяють виведенню кальцію з кісток і зубів

Проявляють припікаючу дію на слизові оболонки

Провокують кровотечу з ясен

Викликають пошкодження хрящової тканини у дітей

5474 / 6307
Гальмування ноцицептивної інформації відбувається за участі багатьох медіаторів. Що з нижченаведеного до цього не відноситься?

Глутамат

ГАМК

Серотонін

Норадреналін

Ендорфін

5475 / 6307
В експерименті після обробки нервово-м'язового препарата жаби курареподібною речовиною скорочення м'яза у відповідь на електричну стимуляцію нерва зникли. Яка функція клітинної мембрани м'яза порушується курареподібними препаратами?

Рецепція медіаторів у нервово-м'язовому синапсі

Створення бар'єру між середовищем клітини та навколишньою міжклітинною рідиною

Створення електричних потенціалів по обидва боки мембрани

Підтримання внутрішньої структури клітини, її цитоскелета

Зміна проникності для різних речовин

5476 / 6307
Аналіз прозорої рідини лимонно-жовтого кольору, отриманої у пацієнта під час пункції черевної порожнини, дав такий результат: відносна щільність - 1012, альбумін - 1%, глобуліни - 0,2%, фібриногену немає, лейкоцити - 1-3 в п/з. Рідина стерильна, стояла впродовж години і не згорнулася. Яке з наведених явищ супроводжується такими характеристиками?

Асцит внаслідок застою крові в портальній системі

Емпієма очеревини

Запалення очеревини

Перитоніт

Набрякова форма гемолітичної хвороби новонароджених

5477 / 6307
Однією з патогенетичних ланок у розвитку променевої хвороби є інтенсифікація процесів вільнорадикального окиснення речовин. Які речовини є основним джерелом утворення вільних радикалів?

Ліпіди

Вуглеводи

Білки

Вода

Гормони

5478 / 6307
Пацієнт із діагнозом: ревматоїдний артрит, декілька тижнів приймав препарати глюкокортикостероїдів, а потім раптово припинив їх приймати. Яке ускладнення може виникнути в цьому разі?

Синдром відміни

Гіперглікемія

Загострення хронічних інфекційних процесів

Виразкування слизової оболонки шлунка і дванадцятипалої кишки

Підвищення артеріального тиску

5479 / 6307
Учням 1-го класу поставлена проба Манту. З 35 учнів у 15 проба Манту була негативною. Що треба зробити дітям із негативною пробою Манту?

Зробити повторну пробу

Увести вакцину БЦЖ

Дослідити сироватку крові

Увести антирабічну вакцину

Увести антитоксичну сироватку

5480 / 6307
Чоловіка 45 років протягом останніх 3 років турбував сухий кашель, наростали задишка, легенева недостатність, швидка втрата ваги. На розтині встановлено легеневе серце. У легенях різко виражений фіброз із наявністю порожнин, що створюють картину ''медових стільників''. Гістологічно спостерігається інтерстиціальний фіброз з вираженою інфільтрацією строми лімфогістіоцитами з домішкою нейтрофілів. Установіть діагноз:

Пиловий пневмосклероз

Бронхіальна астма

Післязапальний пневмосклероз

Хронічна бульозна емфізема

Бронхоектатична хвороба

5481 / 6307
Під час повторної постановки реакції аглютинації Відаля виявлено наростання титрів антитіл до О-антигенів S. typhi в сироватці пацієнта з 1:100 до 1:400. Як можна тлумачити отримані результати?

Є хронічним носієм черевнотифозних мікробів

Хворіє на черевний тиф

Раніше перехворів на черевний тиф

Раніше був щеплений проти черевного тифу

Є гострим носієм черевнотифозних мікробів

5482 / 6307
Пацієнту для лікування лямбліозу призначено синтетичний протипротозойний засіб з групи похідних імідазолу. Перед цим лікар суворо попередив пацієнта про те, що під час лікування забороняється вживати алкогольні напої. Який препарат був призначений?

Метронідазол

Фуразолідон

Хінгамін

Тетрациклін

Метациклін

5483 / 6307
Гіповітаміноз С призводить до зменшення утворення органічного матриксу, порушення синтезу коллагену, тому що цей вітамін бере участь у процесах:

Карбоксилювання лізину

Гідроксилювання проліну

Гідроксилювання аргініну

Карбоксилювання проліну

5484 / 6307
У пацієнта гемералопія (порушення зорової адаптації у темряві). Який вітамінний засіб впливає на синтез зорового пурпуру та покращує зір?

Ергокальциферол

Ретинолу ацетат

Нікотинова кислота

Токоферолу ацетат

Ціанокобаламін

5485 / 6307
Людину 30-ти років вкусила бджола. На місці укусу визначається набряк, гіперемія, підвищення температури. Назвіть ініціальний патогенетичний фактор запального набряку:

Підвищення проникності мікросудин

Підвищення кров'яного тиску в капілярах

Підвищення осмотичного тиску у вогнищі запалення

Порушення лімфовідтоку

Зниження онкотичного тиску крові

5486 / 6307
Під час розтину тіла плода від другої вагітності Rh-негативної матері, який помер внутрішньоутробно на сьомому місяці вагітності, виявлено поширені набряки підшкірної клітковини та головного мозку, асцит, збільшену печінку та селезінку, атрофію тимуса та гіпертрофію міокарда. Мікроскопічно спостерігається: осередки екстрамедулярного кровотворення, дистрофічні та некробіотичні зміни у внутрішніх органах. Для якого захворювання характерні такі патологічні зміни?

Набрякової форми гемолітичної хвороби

Вродженої жовтяничної форма гемолітичної хвороби

Післяпологової жовтяничної форми гемолітичної хвороби

Анемічної форми гемолітичної хвороби

Геморагічної хвороби новонароджених

5487 / 6307
Чоловік 45-ти років звернувся до лікаря зі скаргою на бляшкоподібне утворення на шиї. Гістологічно в біоптаті шкіри виявлено пухлинні клітини, які розташовані гніздами, мають круглу і овальну форму з вузьким обідком базофільної цитоплазми та схожі на клітини базального шару епідермісу. Вкажіть назву пухлини:

Трихоепітеліома

Сірінгоаденома

Епідермальний рак

Гідраденома

Базаліома

5488 / 6307
У жінки віком 50 років гнійне запалення шийки матки. Під час бактеріоскопічного дослідження гнійних виділень з шийки матки виявлено: грамнегативні бобовидні диплококи, що знаходяться як в середині, так і поза лейкоцитами. Назвіть збудника цього гнійного запалення.

Candida albicans

Calymmatobacterium granulomatis

Chlamidia trachomatis

Haemophilus vaginalis

Neisseria gonorroeae

5489 / 6307
Порушення процесів розщеплення ліпідів у тонкому кишечнику зумовлено порушенням активності ліпази. Який із факторів активує ліпазу?

Пепсин

Ентерокіназа

Соляна кислота

Жовчні кислоти

Солі Na^+

5490 / 6307
Хворий 59-ти років госпіталізований у кардіологічне відділення в тяжкому стані з діагнозом гострий інфаркт міокарда в ділянці задньої стінки лівого шлуночка та перегородки, початковий набряк легень. Який механізм є первинним у виникненні набряку легень у пацієнта?

Лівошлуночкова недостатність

Гіпоксемія

Легенева артеріальна гіпертензія

Легенева венозна гіпертензія

Зниження альвеоло-капілярної дифузії кисню

5491 / 6307
Чоловік тривалий час працює в нафтопереробній промисловості. Який з наведених класів канцерогенів зустрічається в його оточенні?

Нітрозаміни

Поліциклічні ароматичні вуглеводні

Канцерогени біологічного походження

Аміни

Аміноазосполуки

5492 / 6307
У чоловіка, хворого на гіпертонічну хворобу, спіронолактон викликав виражений терапевтичний ефект. Його терапевтична активність обумовлена ослабленням дії:

Ангіотензинперетворювального ферменту

Реніну

Брадикініну

Ангіотензину ІІ

Альдостерону

5493 / 6307
Пацієнту для усунення кишкової коліки призначено препарат атропіну сульфат. Яке захворювання може бути протипоказанням для застосування цього препарату?

Бронхіальна астма

Запаморочення

Синусова брадикардія

Глаукома

Гіпотонія

5494 / 6307
У пацієнтки віком 36 років спостерігається місяцеподібне обличчя, ожиріння верхньої частини тулуба, стриї на передній черевній стінці, гірсутизм, гіперглікемія та глюкозурія. Для якої патології характерні такі ознаки?

Феохромоцитоми

Первинного гіперальдостеронізму

Синдрому Конна

Синдрому Іценка-Кушинга

Вторинного гіперальдостеронізму

5495 / 6307
У 35-річного наркомана, який тривало хворів на фіброзно-кавернозний туберкульоз легень, під час патологоанатомічного дослідження встановлено, що нирки та селезінка збільшені в розмірах, щільної консистенції, на розрізі тканина із сірим відтінком та ''сальним'' блиском. Мікроскопічно вичвлено в білій та червоній пульпі селезінки, у інтерстиції та мезангіумі клубочків нирок депозити Конго-рот-позитивних мас. Діагностуйте вид ураження внутрішніх органів:

Дифузний гіаліноз

Локальний пухлиноподібний амілоїдоз

Ідіопатичний амілоїдоз

Старечий амілоїдоз

Вторинний амілоїдоз

5496 / 6307
Хворий на гіпертонічну хворобу, який лікувався гіпотіазидом, скаржиться на загальну слабкість, втрату апетиту, серцебиття. Спостерігаються гіпотонія м'язів, в'ялі паралічі, послаблення перистальтики кишечника. Що може бути причиною такого стану?

Гіпонатріємія

Гіпокаліємія

Гіперкаліємія

Гіперкальціємія

Гіперурикемія

5497 / 6307
У підлітка 12-ти років, який хворіє на бронхіальну астму, розвинувся тяжкий напад астми: виражена експіраторна задишка, блідість шкірних покровів. Який вид порушення альвеолярної вентиляції має місце?

Центральний

Обструктивний

Рестриктивний

Нервово-м'язовий

Торако-діафрагмальний

5498 / 6307
У жінки періодично виникають напади артеріальної гіпертензії, що супроводжуються головним болем, частим серцебиттям, виразною пітливістю, різким болем в надчеревній ділянці, збільшенням рівня глюкози в плазмі крові. В плазмі крові та сечі виявлений високий рівень метанефринів. Для якого пухлинного захворювання найхарактерніші такі симптоми?

Пухлина яєчників

Аденома щитоподібної залози

Феохромоцитома

Рак шлунку

Аденома прищитоподібних залоз

5499 / 6307
У немовляти після народження в судинній системі відбуваються зміни, пов'язані із переходом від плацентарного кровообігу до легеневого. Яка з кровоносних судин у ході цього перетворюється на круглу зв'язку печінки?

Ductus venosus

A) umbilicalis dextra

A) umbilicalis sinistra

V. umbilicalis

Ductus arteriosus

5500 / 6307
Під час розтину тіла пацієнта, який помер після абдомінальної операції, виявлено численні тромби у венах малого таза. Клінічно зафіксовано тромбоемболічний синдром. Де потрібно шукати тромбоемболи?

У лівому шлуночку серця

У легеневих артеріях

У головному мозку

У портальній вені

У венах нижніх кінцівок

5501 / 6307
Чоловік 50-ти років пережив сильний стрес. У крові різко збільшилась концентрація адреналіну і норадреналіну. Які ферменти каталізують процес інактивації останнього?

Пептидази

Карбоксилаза

Моноамінооксидази

Тирозиназа

Глікозидази

5502 / 6307
Для комплексної терапії туберкульозу було призначено препарат, що є антагоністом вітаміну В_6. Назвіть цей препарат:

Ізоніазид

ПАСК

Стрептоміцин

Етамбутол

Левофлоксацин

5503 / 6307
Під час експерименту треба оцінити рівень збудливості тканини. Для цього доцільно визначити величину:

Потенціалу спокою

Тривалості потенціалу дії

Амплітуди потенціалу дії

Порогу деполяризації

5504 / 6307
У хворого збільшений основний обмін, підвищена температура тіла, тахікардія у стані спокою. Причиною цього може бути підвищена функція:

Статевих залоз

Нейрогіпофізу

Щитоподібної залози

Кіркової речовини наднирників

Підшлункової залози

5505 / 6307
Хворому на туберкульоз призначено комплексне лікування, до складу якого входить ізоніазид. До якої групи препаратів за хімічною будовою належить цей засіб?

Похідне парааміносаліцилової кислоти

Похідне параамінобензойної кислоти

Напівсинтетичний антибіотик

Похідне гідразиду ізонікотинової кислоти

Антибіотик групи стрептоміцину

5506 / 6307
У 50-х роках у Західній Європі від матерів, які приймали в якості снодійного талідомід, народилося кілька тисяч дітей з відсутністю або недорозвиненням кінцівок, порушенням будови скелета, іншими вадами. Яка природа даної патології?

Генна мутація

Трисомія

Моносомія

Триплоїдія

Тератогенна дія

5507 / 6307
У гістологічному препараті стінки очного яблука визначається структура, що складається з ланцюга трьох нейронів. Тіла цих нейронів формують зовнішній, внутрішній ядерний і гангліонарний шари. Яка структура ока має таку морфологічну будову?

Склера

Війчасте тіло

Сітківка

Судинна оболонка

5508 / 6307
У пацієнта відбувся напад бронхоспазму. Які мембранні циторецептори гладеньких м'язів бронхів треба стимулювати для покращення стану пацієнта?

beta-адренорецептори

alpha-адренорецептори

Н-холінорецептори

М-холінорецептори

5509 / 6307
В експерименті на ізольованій збудливій клітині необхідно отримати збільшення мембранного потенціалу спокою (гіперполяризацію). Для цього доцільно викликати активацію таких іонних каналів:

Калієві та натрієві

Калієві

Натрієві

Натрієві та кальцієві

Кальцієві

5510 / 6307
У дитини встановлено діагноз: хвороба Тея-Сакса, що пов'язана з порушенням обміну речовин. Порушення якого обміну речовин зумовлює цю хворобу?

Мінерального

Амінокислотного

Ліпідного

Білкового

5511 / 6307
Під час мікроскопічного дослідження випорожнень пацієнта з явищами профузного проносу, багаторазового блювання та наростаючої інтоксикації, було виявлено Грам-негативні палички, що нагадували кому і розташовувались групами, у вигляді табунців риб. Культура збудника була виділена через середовище накопичення - 1% пептонну воду, де вона утворювала ніжну плівку. Збудник якого захворювання був виділений?

Кишкового єрсиніозу

Шигельозу

Холери

Сальмонельозу

5512 / 6307
Під час обстеження пацієнта була вияв-лена зменшена кількість імуноглобулінів. Порушення функції яких клітин імунної системи пацієнта може бути причиною цього?

Т-хелперів

Т-кілерів

Т-супресорів

Плазматичних клітин

5513 / 6307
Чоловік віком 37 років, який палить протягом 19 років, звернувся із скаргами на постійний кашель. Під час біопсії бронха виявлено: ознаки хронічного запалення, потовщення слизової оболонки, трансформація одношарового війчастого епітелію в багатошаровий плоский епітелій. Який патологічний процес спостерігається у пацієнта?

Метаплазія

Дисплазія

Гіпертрофія епітелію

Лейкоплакія

5514 / 6307
Пацієнту з туберкульозом призначено антибіотик. Під час застосування препарату можливе виникнення порушень функції печінки, лейкопенія, забарвлення біологічних рідин у червоний колір. Який лікарський засіб був призначений?

Піразинамід

Рифампіцин

Циклосерин

Натрію парааміносаліцилат

Ізоніазид

5515 / 6307
У пацієнта, який хворіє на цукровий діабет другого типу, розвинулася гіперглікемічна кома (рівень глюкози крові - 56 ммоль/л). Яке патологічне явище в цьому разі лежить в основі порушення функцій клітин мозку пацієнта?

Гіпергідратація клітин мозку (набухання)

Токсичне ураження

Гіпогідратація клітин мозку

Енергодефіцит

Іонний дисбаланс

5516 / 6307
Пацієнту проведено оперативне втручання з протезування мітрального клапана. Оберіть препарат для попередження тромбоутворення для цього пацієнта.

Парацетамол

Дипіридамол

Варфарин

Урокіназа

5517 / 6307
Пацієнту з діагнозом: гіпертонічна хвороба (АТ - 200/110 мм рт. cт.) серед препаратів комплексної терапії лікар призначив анаприлін (пропранолол). За 2 тижні після початку приймання цього лікарського засобу, пацієнт почав скаржитися на відчуття задухи та ускладнене дихання. Що є можливою причиною ускладнень і яку тактику потрібно вибрати в цій ситуації?

Блокада beta_2-адренорецепторів. Призначити селективний beta_1-адреноблокатор

Міотропна бронхоспастична дія. Призначити еуфілін

Збудження М-холінорецепторів. Призначити атропін

Алергічна реакція. Приймання препарату відмінити, призначити антигістамінні засоби

Блокада beta_1-адренорецепторів. Призначити селективний beta_2-адреноблокатор

5518 / 6307
Пацієнту, який лікувався доксицикліном, лікар порадив не вживати молочних продуктів. Чому лікар дав таку рекомендацію пацієнту?

Порушується процес перетравлення їжі

Збільшується ризик дисбактеріозу

Сповільнюється абсорбція антибіотика

Не засвоюються молочні продукти

5519 / 6307
Під час обстеження пацієнта виявлено поліп у дистальному відділі товстої кишки. Визначте локалізацію поліпа.

Colon transversum

Rectum

Colon sigmoideum

Colon descendens

5520 / 6307
До приймального відділення шпиталізовано пацієнта в непритомному стані. Об'єктивно спостерігається: шкіра холодна, зіниці звужені, дихання ускладнене, відзначається періодичність дихання за типом Чейна-Стокса, артеріальний тиск знижений, сечовий міхур переповнений. Встановлено діагноз: отруєння морфіном. Який препарат треба застосувати в якості антагоніста?

Бемегрид

Унітіол

Тіосульфат натрію

Цититон

Налоксон

5521 / 6307
На розтині тіла жінки 49-ти років, що померла від хронічної ниркової недостатності, виявлено: нирки ущільнені, зменшені, строкаті, з ділянками крововиливів. Мікроскопічно: у ядрах епітелію канальців гематоксилінові тільця, потовщення базальних мембран капілярів клубочків, які мають вигляд дротяних петель, подекуди в капілярах - гіалінові тромби та вогнища фібриноїдного некрозу. Який найбільш імовірний діагноз?

Амілоїдоз

Ревматизм

Атеросклеротичний нефросклероз

Системний червоний вовчак

5522 / 6307
Дівчина 15-ти років скаржиться на швидку втомлюваність, слабкість, серцебиття. В аналізі крові виявлено: кількість еритроцитів - 2,5 г/л, Hb - 50 г/л, КП - 0,6, анізоцитоз зі схильністю до мікроцитозу. Вміст заліза в сироватці крові становив 5,4 ммоль. Яка гематологічна патологія спостерігається у цьому випадку?

Залізодефіцитна анемія

Гостра постгеморагічна анемія

В12 і фолієводефіцитна анемія

Набута гемолітична анемія

5523 / 6307
Під час дослідження процесів травлення in vitro спостерігався набряк білкового субстрату. Який компонент шлункового соку забезпечує таке перетворення білка?

Гастриксин

Хлористоводнева кислота

Слиз

Трипсин

5524 / 6307
У щура з хронічною нирковою недостатністю виявлено остеопороз, патологічна кальцифікація внутрішніх органів, артеріа-льна гіпертензія. З посиленням дії якого гормону пов'язані ці порушення?

Трийодтиронін

Тироксин

Кальцитонін

Паратгормон

5525 / 6307
Внутрішньоклітинне дослідження біопотенціалів ізольованої культури тканини показало, що потенціали дії, які виникають у клітинах, характеризуються наявністю плато тривалістю до 300 мс у фазі реполяризації. Яку тканину досліджували?

Скелетний м'яз

Атипові кардіоміоцити

Нервове волокно

Скоротливий міокард

5526 / 6307
У хворого на туберкульоз легень непереносимість аміноглікозидного антибіо-тика амікацину. Який протитуберкульозний антибіотик можна включити до складу комплексної терапії у даному випадку?

Рифампіцин

Канаміцин

Амоксицилін

Бензилпеніцилін

5527 / 6307
В експерименті на щурах електричне подразнення головного мозку викликало у голодних тварин відмову від споживання іжі. Яку структуру стимулювали?

Латеральну ділянку гіпоталамуса

Мигдалеподібні ядра

Вентромедіальне ядро гіпоталамуса

Бліду кулю

5528 / 6307
В комплексному лікуванні гіпертонічної хвороби пацієнту призначено сечогінний препарат. За кілька днів АТ знизився але виникли ознаки гіпокаліємії. Який препарат міг викликати такі ускладнення?

Еналаприл

Фуросемід

Метопролол

Спіронолактон

5529 / 6307
У хворого в інфекційному відділенні спостерігається підвищення температури до 39^oC) При ЕКГ дослідженні інтервал R-R вкорочений, зубець Р збережений перед кожним комплексом QRS. ЧСС- 120/хв. Порушення якої властивості серцевого м'яза призвело до вказаної форми патології ритму?

Провідність і засвоєння ритму

Засвоєння ритму

Провідність

Автоматизм і провідність

Автоматизм

5530 / 6307
У пацієнта з COVID-19 відмічається посилення продукції білків, що пригнічують трансляцію в інфікованих клітинах шляхом індукції синтезу протеїнкіназ, які фосфорилюють фактори ініціації eIF-2. Вкажіть ці білки.

Альбуміни

Протеази

Інтерферони

Інтегрини

5531 / 6307
Формування великої кількості імуноглобулінів з різною антигенною специфічністю відбувається з невеликої кількості генів. Завдяки якому процесу це є можливим?

Транслокація

Рекомбінація генів

Делеція

Реплікація

5532 / 6307
В травматологічний пункт доставлений хворий з пораненням великого поперекового м'яза. Хворий втратив можливість розгинати гомілку в колінному суглобі. Який нерв ушкоджений в цьому випадку?

Затульний

Клубово-пахвинний

Стегновий

Клубово-підчеревний

5533 / 6307
У пацієнта тривале блювання призвело до зневоднення організму. Підвищення секреції якого гормону за цих умов, перш за все, забезпечує збереження води в організмі?

Натрійуретичний

Вазопресин

Адреналін

Кальцитонін

5534 / 6307
У дитини 14-ти років, хворої на дифтерію, у період кризи при різкому падінні температури на фоні тахікардії АТ складає 70/50 мм рт.ст. До якої форми порушення судинного тонусу відноситься дане явище?

Гостра гіпотензія

Вегето-судинна дистонія

Хронічна гіпотензія

5535 / 6307
Жінка віком 44 роки скаржиться на загальну слабість, біль в ділянці серця, значне збільшення маси тіла. Об'єктивно спостерігається: обличчя місяцеподібне, гірсутизм, АТ - 165/100 мм рт. ст., зріст - 164 см, вага - 103 кг, переважне накопичення жиру на шиї, верхньому плечовому поясі, животі. Що є основним патогенетичним механізмом ожиріння у жінки?

Зниження продукції тиреоїдних гормонів

Підвищення продукції мінералокортикоїдів

Підвищення продукції глюкокортикоїдів

Підвищення продукції інсуліну

5536 / 6307
У пацієнта розгладжена права носогубна складка, розширена права очноямкова щілина (її не вдається замружити, тому що повіки не змикаються), виникають труднощі під час розмови та вживання їжі (їжа застрягає між щокою і зубами). Який нерв уражено?

N. abduceus dexter

N. trigeminus dexter

N. vagus dexter

N. glossopharyngeus sinister

N. facialis dexter

5537 / 6307
Відомо, що ацетилсаліцилова кислота та глюкокортикоїди мають виражену протизапальну дію. Укажіть на синтез яких біологічно активних речовин не впливає ацетилсаліцилова кислота на відміну від глюкокортикоїдів.

Тромбоксанів

Простагландинів Е

Лейкотрієнів

Простациклінів

5538 / 6307
Під час обстеження у пацієнта виявлено, що верхівковий поштовх зміщений на 3,5 см ліворуч від лівої середньоключичної лінії. Які камери серця при цьому гіпертрофовані?

Праве передсердя

Правий шлуночок

Лівий шлуночок

Ліве передсердя

5539 / 6307
Пацієнт із діагнозом: гіпертонічна хвороба приймає еналаприл. Який механізм дії цього гіпотензивного препарату?

Блокатор Са^++-каналів

Інгібітор циклооксигенази

Інгібітор ангіотензинперетворювального ферменту

Антагоніст ангіотензинових рецепторів

5540 / 6307
В аналізі крові лаборант виявив без'я-дерні формені елементи у вигляді двовгнутих дисків. Назвіть їх:

Еритроцити

Лімфоцити

Моноцити

Еозинофіли

5541 / 6307
Хворий 55 років обстежувався у ендокринолога через порушення ендокринної функції підшлункової залози, що виявляється у зменшенні гормона глюкагону в крові. Функція яких клітин цієї залози порушена в цьому разі?

А-клітини

РР-клітини

В-клітини

D1-клітини

5542 / 6307
Інститут геронтології людям похилого віку радить вживати комплекс вітамінів, який містить вітамін E) Яку головну функцію він виконує?

Антиневрична

Антиоксидантна

Антискорбутна

Антидерматитна

5543 / 6307
У пацієнта віком 60 років крововилив у головний мозок спричинив тривалий сон. Пошкодження якої структури найімовірніше призвело до цього стану?

Гіпокампу

Кори великих півкуль

Ретикулярної формації

Чорної субстанції

5544 / 6307
Лімфоцит уражено ретровірусом ВІЛ (СНІД). Який напрямок течії інформації у клітині в цьому разі?

Поліпептид > РНК > ДНК > іРНК

ДНК > iРНК > поліпептид > ДНК

РНК > ДНК > іРНК > поліпептид

ДНК > поліпептид > iРНК

іРНК > поліпептид > ДНК

5545 / 6307
Для підвищення результатів спортсмену рекомендували застосовувати препарат, що містить карнітин. Який процес найбільшою мірою активізується карнітином?

Тканинне дихання

Транспорт жирних кислот в мітохондрії

Синтез кетонових тіл

Синтез ліпідів

Синтез стероїдних гормонів

5546 / 6307
У чоловіка 30 років, який хворів на дизентерію, що була підтверджена бактеріологічно, з'явилися ознаки парапроктиту. Про які найімовірніші місцеві зміни йдеться у цього пацієнта?

Фолікулярний коліт

Фібринозний коліт

Утворення виразок

Неспецифічний виразковий коліт

5547 / 6307
Хворому для профілактики ранової інфекції перед операцією було призначено синтетичний протипротозойний засіб, який має високу активність на Helicobacter pylori. Визначте препарат:

Ацикловір

Ізоніазид

Хінгамін

Метронідазол

5548 / 6307
В організмі людини резерви вуглеводнів локалізуються переважно в печінці та скелетних м'язах. Який із цих резервів мобілізується для підтримання рівня глюкози крові під час голодування?

Крохмаль

Глікоген печінки

Глікоген м'язів

Амілопектин

5549 / 6307
На електронній мікрофотографії біопсійного матеріалу представлені структури, до складу яких входять сурфактант, альвеолоцити I типу, базальна мембрана і фенестрований ендотелій капілярів. До якого гістогематичного бар'єра в організмі людини відносяться ці структури?

Гематолікворного

Гематоенцефалічного

Гематотимусного

Аерогематичного

5550 / 6307
При розтині тіла 6-місячної дитини, померлої від сепсису, виявлена відсутність тимусу, зменшення розмірів та маси селезінки. При мікроскопічному дослідженні селезінки спостерігається відсутність периартеріальних Т-залежних зон фолікулів зі спустошенням червоної пульпи; у лімфатичних вузлах - відсутність перикортикальної зони, яка переважно представлена Т-лімфоцитами. В-зони у периферійних імунних органах розвинуті нормально. Який це патологічний процес?

Синдром Ді Джорджі (недостатність клітинного імунітету)

Синдром Брутона (недостатність гуморального імунітету)

ВІЛ-інфекція

Акцидентальна інволюція тимусу

Синдром Гланцмана-Рінікера (недостатність клітинного та гуморального імунітету)

5551 / 6307
Під час розтину тіла чоловіка віком 38 років, який помер раптово, в інтимі черевного відділу аорти спостерігаються ділянки жовтуватого кольору, які не підвищуються над її поверхнею. Гістологічно в інтимі виявлено велику кількість ксантомних клітин, які суданом III забарвлюються в яскравий помаранчевий колір. Для якої стадії атеросклерозу характерні такі патологічні показники?

Ліпосклероз

Звиразкування

Ліпоїдоз

Атероматоз

Атерокальциноз

5552 / 6307
До лікаря-невропатолога звернулася хвора 52-х років із скаргами на втрату чутливості шкіри правої половини обличчя в ділянці нижньої повіки, спинки носа та верхньої губи. Яка гілка якого нерва при цьому ушкоджена?

Очний нерв трійчастого нерва

Верхньощелепний нерв трійчастого нерва

Барабанна струна лицевого нерва

Нижньощелепний нерв трійчастого нерва

5553 / 6307
Дитина має ідіосинкразію на лікарську речовину. Чим зумовлена ця патологічна реакція на ліки?

Супутнім захворюванням органу-мішені

Виснаженням субстрату, з яким взаємодіє лікарська речовина

Спадковою ензимопатією

Пригніченням мікросомальних ферментів печінки

5554 / 6307
У дитини з явищами гнійного кератокон'юн-ктивіту лікар-офтальмолог запідозрив бленорею (гонорейний кон'юнктивіт). Якими методами лабораторної діагностики слід скористатися для підтвердження діагнозу?

Серологічним та алергічним

Мікроскопічним та серологічним

Мікроскопічним та бактеріологічним

Біологічним та методом фагодіагнос-тики

5555 / 6307
У чоловіка 39-ти років спостерігається підвищений ризик розвитку інфекційних процесів, гіперкератоз, порушення присмеркового зору. Який вітамінний препарат необхідно призначити?

Ретинолу ацетат

Ергокальциферол

Токоферолу ацетат

Піридоксину гідрохлорид

5556 / 6307
Карбоксибіотин - коферментна форма вітаміну H. У якому процесі, що відбувається в організмі людини, бере участь цей вітамін?

Біосинтез вищих жирних кислот

ЦТК

Декарбоксилювання амінокислот

Гідроксилювання проліну

5557 / 6307
Чоловік 39-ти років, прооперований у зв'язку з виразковою хворобою шлунка, помер через 7 днів після операції. На розтині листки очеревини повнокровні, тьмяні, вкриті масивними жовтувато-зеленуватими плівчастими накладеннями, в очеревинній порожнині біля 300 мл густої зеленувато-жовтуватої рідини. Який патологічний процес було знайдено в очеревинній порожнині?

Спайкова хвороба

Фібринозно-гнійний перитоніт

Фібринозно-геморагічний перитоніт

Фібринозно-серозний перитоніт

5558 / 6307
У хворого на мікросфероцитарну гемолітичну анемію (хворобу Мінковського-Шоффара) внаслідок підвищення проникності мембрани еритроцитів у клітину надходять іони натрію й вода. Еритроцити набувають форми сфероцитів і легко руйнуються. Який провідний механізм пошкодження еритроцитів у цьому разі?

Кальцієвий

Нуклеїновий

Електролітно-осмотичний

Ацидотичний

Протеїновий

5559 / 6307
У жінки 45-ти років має місце недостатня секреція ферменту ентерокінази. Порушення якої травної функції може викликати дефіцит ентерокінази?

Гідроліз вуглеводів

Всмоктування вітамінів

Гідроліз жирів

Всмоктування жирів

Гідроліз білків

5560 / 6307
Чоловікові 32 років діагностовано гостру променеву хворобу. Лабораторно встановлено різке зниження рівня серотоніну в тромбоцитах. Найімовірнішою причиною зниження тромбоцитарного серотоніну є порушення процесу декарбоксилювання:

Тирозину

Піровиноградної кислоти

Серину

Гістидину

5-окситриптофану

5561 / 6307
У гістопрепараті представлена залоза. У часточках визначаються ацинуси, секреторні клітини яких мають дві зони: базальну - гомогенну базофільну і апікальну - зимогенну оксифільну. Який орган має наведені морфологічні ознаки?

Під'язикова слинна залоза

Печінка

Підщелепна слинна залоза

Підшлункова залоза

Привушна слинна залоза

5562 / 6307
Під час оперативного втручання з приводу пахвинної кили хірург виділяє поверхневе пахвинне кільце. Похідні апоневрозу якого з м'язів утворюють більшість його стінок?

M. obliquus externus abdominis

M. obliquus internus abdominis

M. rectus abdominis

M. transversus abdominis

M. psoas major

5563 / 6307
У чоловіка віком 30 років під час проведення ЕКГ виявлено наступні зміни: ділянка аномального скорочення, в якій відсутній зубець Р, комплекс QRS деформований, зубець Т негативний і протилежно направлений комплексу QRS. Який патологічний стан спостерігається у пацієнта?

Пароксизмальна тахікардія

Шлуночкова екстрасистолія

Синусова аритмія

Атріовентрикулярна екстрасистолія

Передсердна екстрасистолія

5564 / 6307
Унаслідок укусу отруйної змії у людини може розвинутися гемолітична жовтяниця. При цьому в крові потерпілого зростає насамперед такий показник.

Білірубін прямий (кон'югований)

Вільні амінокислоти

Білірубін непрямий (некон'югований)

Сечова кислота

Сечовина

5565 / 6307
Пацієнт віком 56 років скаржиться на гострий напад болю в ділянці правого гомілковостопного суглоба. Об'єктивно виявлено: суглоб набряклий та гарячий на дотик, а у крові визначається підвищений рівень сечової кислоти. Для лікування цієї патології лікар призначив препарат, який інгібує ксантиноксидазу. Укажіть цей лікарський засіб.

Теофілін

Уролесан

Алопуринол

Преднізолон

Діакарб

5566 / 6307
Пацієнтка віком 65 років госпіталізована до кардіологічного відділення клініки з діагнозом: синдром Дресслера. З анамнезу відомо, що у пацієнтки був інфаркт міокарда. Які додатокві клініко-лабораторні показники будуть підтвердження дігнозу: синдром Дреслера?

Збільшення рівня автоантитіл крові

Гарячка

Збільшення ШОЕ

Лейкоцитоз

Збільшення активності аспартатамінотрансферази в крові

5567 / 6307
На електронній мікрофотограмі ниркового тільця між капілярами судинного клубочка визначаються відростчасті клітини, в цитоплазмі яких наявна велика кількість філаментів. Назвіть дані клітини:

Фібробласти

Юкстагломерулярні

Юкставаскулярні

Мезангіальні

Адвентиційні

5568 / 6307
У пацієнта спостерігається перфораційна виразка задньої стінки шлунку. Куди виділятиметься вміст шлунку у цьому разі?

У печінкову сумку

У лівий брижовий синус

У передшлункову сумку

У чепцеву сумку

У правий брижовий синус

5569 / 6307
У 25-річного пацієнта на тлі гострого запалення легень із температурою тіла 40,1^oC визначено лейкоцитоз 14,9 г/л із чітким зсувом лейкоформули вліво. Який із наведених чинників безпосередньо підвищує в кістковому мозку і проліферацію, і диференціацію лейкоцитів?

Колонієстимулюючий чинник

Пухлинний некротичний чинник

Простациклін

Інтерлейкін-10

Інтерлейкін-1

5570 / 6307
У пацієнта внаслідок комбінованої терапії хронічної серцевої недостатності дигітоксином та фуросемідом розвинулась різка м'язова слабкість. Які електролітні порушення будуть спостерігатися в крові у цьому разі?

Гіпокаліємія

Гіперкальціємія

Гіпокальціємія

Гіперкаліємія

Гіпохлоремія

5571 / 6307
Хворий на хронічний алкоголізм на вулиці втратив свідомість. Було діагностовано гіпоглікемію внаслідок порушення процесу глюконеогенезу. Які з наступних пар ферментів є необхідними для цього процесу?

Глюкозо-6-фосфатаза та фосфофруктокіназа

Глюкозо-6-фосфатаза і піруватдегідрогеназа

Фруктозо-1,6-діфосфатаза і піруваткарбоксилаза

Фосфоенолпіруваткарбоксикіназа і глюкокіназа

Піруваткіназа і піруваткарбоксилаза

5572 / 6307
У пацієнта віком 30 років, на електрокардіограмі виявлено зниження амплітуди зубця R. Що означає цей зубець на ЕКГ?

Поширення збудження від передсердь до шлуночків

Електричну діастолу серця

Деполяризацію шлуночків

Реполяризацію шлуночків

Деполяризацію передсердь

5573 / 6307
У пацієнта віком 40 років після щелепно-лицевої травми порушилася функція під'язичної та підщелепної залоз зліва. У цих залозах відзначається гіпосалівація. Функція якої пари нерв порушена?

VII пари

VI пари

XI пари

Х пари

XIІ пари

5574 / 6307
Після тривалої безбілкової дієти у дівчини з'явилися набряки. Про зменшення яких білкових фракцій крові свідчить цей стан?

Фібріногену

Плазміногену

Трансферину

Глобулінів

Альбумінів

5575 / 6307
У 45-річного чоловіка, який гостро захворів на пневмонію, на 6-й день хвороби розвинувся набряк легень, який став причиною смерті. Під час розтину виявлено, що уражена вся верхня частка правої легені. Ця частка збільшена, щільна, із фібринозними нашаруваннями на плеврі, на розрізі сірого кольору, з поверхні розрізу стікає каламутна рідина. Мікроскопічно спостерігаються: у просвіті альвеол - фібрин, нейтрофіли, макрофаги, гемолізовані еритроцити. Який тип пневмонії спостерігається у пацієнта?

Респіраторний дистрес-синдром дорослих

Гіпостатична пневмонія

Вірусна пневмонія

Стафілококова бронхопневмонія

Крупозна пневмонія

5576 / 6307
У жінки 32 років після перенесеного міокардиту виявлено під час електрокардіографічного дослідження порушення серцевого ритму (ритм не синусовий). Функції яких кардіоміоцитів порушені?

Провідних кардіоміоцитів ніжок пучка Гіса

Провідних кардіоміоцитів пучка Гіса

Перехідних провідних кардіоміоцитів

Скоротливих кардіоміоцитів

Пейсмекерних клітин

5577 / 6307
У пацієнта виявлено гнійний запальний процес у ділянці стегна (післяін'єкційний абсцес). Які лімфатичні вузли збільшуються у цьому разі?

Задні шийні

Підколінні

Паратрахеальні

Пахвинні

Піднижньощелепні

5578 / 6307
У коледжі навчається юнак 16 років із сільської місцевості. Під час планового проведення реакції Манту виявилося, що у нього вона негативна. Яка тактика лікаря буде найраціональнішою?

Терміново ізолювати юнака від навчального колективу

Провести прискорену діагностику туберкульозу методом Прайса

Повторити реакцію за 1 місяць

Зробити щеплення БЦЖ

Провести серодіагностику туберкульозу

5579 / 6307
За клінічними показниками пацієнту призначено піридоксальфосфат. Для корекції яких процесів рекомендовано цей препарат?

Синтезу білка

Окисного декарбоксилювання кетокислот

Дезамінування пуринових нуклеотидів

Трансамінування і декарбоксилювання амінокислот

Синтезу пуринових і піримідинових основ

5580 / 6307
Пацієнт віком 45 років після травми правого плеча внаслідок падіння не може відвести праву руку до горизонтального рівня. Пошкодження якого м'язу викликало вказане обмеження руху?

Двоголового м'яза плеча

Великого круглого

Підосного

Дельтоподібного

Плечового

5581 / 6307
Чоловік 62 років помер від хронічної ниркової недостатності. Під час секції виявлено: нирки зменшені, щільної консистенції, фіброзна капсула знімається важко, поверхня зерниста, коркова речовина стоншена. Гістологічно спостерігається: у частині клубочків проліферація ендотеліальних та мезангіальних клітин, потовщення мембран капілярів клубочків, деякі клубочки склерозовані та гіалізовані. У стромі - нечисленні лімфоїдні інфільтрати. Який імовірний діагноз?

Хронічний гломерулонефрит

Атеросклеротично зморщена нирка

Гострий екстракапілярний ексудакти-вний гломерулонефрит

Артеріолосклеротичний нефросклероз (первинно зморщена нирка)

Пієлонефрит

5582 / 6307
До ізольованої нервової клітини ссавця повністю припинили надходження кисню. Як у цьому разі зміниться потенціал спокою?

Змін не буде

Суттєво зменшиться

Зникне

Збільшиться несуттєво

Суттєво збільшиться

5583 / 6307
У пацієнта з діагнозом: хронічний гломерулонефрит, розвинулася стійка артеріальна гіпертензія. Укажіть групу лікарських засобів для лікування цього пацієнта.

Міотропні спазмалітики

Антагоністи кальцію

Англіоблокатори

alpha-аденоблокатори

Інгібітори ангіотензинперетворюючого ферменту

5584 / 6307
При дослідженні однієї з видалених під час операції надниркових залоз виявили великі клітини, які імпрегнуються розчином двухромовокислого калію. Який гормон синтезують ці клітини?

Холецистокінін

Секретин

Альдостерон

Адреналін

Тироксин

5585 / 6307
Пацієнтка з діагнозом: бронхіальна астма, тривалий час лікувалася глюкокортикоїдами. Після різкого припинення лікування відчула погіршення стану, яке проявлялось падінням артеріального тиску і відновленням нападів бронхіальної астми. Для якого патологічного стану є характерним наявність таких симптомів?

Сенсибілізація

Акумуляція

Синдром відміни

Тахіфілаксія

5586 / 6307
У пацієнтки діагностували кістковомозкову форму гострої променевої хвороби. Який з перерахованих гематологічних симптомів спостерігатиметься в періоді розпалу?

Зсув лейкоцитарної формули вліво

Еритроцитоз

Відносна лімфопенія

Панцитопенія

Відносний лімфоцитоз

5587 / 6307
Чоловіку 45-ти років, хворому на гостру пневмонію, був призначений антибіо-тик з групи пеніцилінів. При проведенні проби на індивідуальну чутливість до антибіотику виникла алергічна реакція. Яким із наведених препаратів краще лікувати хворого?

Ципрофлоксацин

Біцилін-5

Бензилпеніцилін

Феноксиметилпеніцилін

Еритроміцин

5588 / 6307
Під час лабораторного обстеження чоловіка 54 років було встановлено, що його кліренс інуліну становить 120 мл/хв. Це означає, що в нього в нормі такий показник:

Нирковий плазмотік

Швидкість клубочкової фільтрації

Нирковий кровотік

Канальцева секреція

Канальцева реабсорбція

5589 / 6307
Під час бактеріологічного дослідження сечі пацієнта з гострим циститом виділено грамнегативні рухомі палички, які на м'ясо-пептонному агарі утворюють великі слизові колонії зелено-блакитного кольору із запахом карамелі або жасмину. Який мікроорганізм найімовірніше спричинив захворювання?

Кlebsiella osaenае

Staphylococcus aureus

Рsеudоmоnаs aeruginosa

Escherichia coli

Рrоtеus vulgaris

5590 / 6307
Під час аутопсії тіла пацієнта з діагнозом: мієломна хвороба, який помер з явищами серцевої недостатності, виявлено збільшене в розмірах серце з розширеними порожнинами. Міокард блідий, має щільну консистенцію і воскоподібний блиск. Мікроскопічно під час забарвлення конго-рот визначається позитивна реакція. Для якої патології серця характерні такі результати дослідження?

Тигрове серце

Сальне серце

Гіпертензивне серце

Ожиріння серця

5591 / 6307
Швидкість окиснення субстратів клітинного палива і транспорту електронів до кисню дихальним ланцюгом регулюється за допомогою дихального контролю. Від чого він залежить?

Від співвідношення СО_2/О_2

Від співвідношення АТФ/АМФ

Від концентрації АДФ

Від співвідношення АТФ/(АДФ+АМФ)

Від концентрації АМФ

5592 / 6307
Пацієнту з діагнозом: глаукома, лікар призначив препарат, що знижує внутрішньоочний тиск. Укажіть цей лікарський засіб.

Анаприлін

Норадреналіну гідротартрат

Прозерин

Анальгін

Феназепам

5593 / 6307
На розтині виявлено зменшені в розмірах нирки, вага 50,0 г, їх поверхня дрібнозерниста, кора рівномірно стоншена. Під час мікроскопічного дослідження спостерігається: стінки артеріол значно потовщені внаслідок відкладення в них гомогенних безструктурних рожевого кольору мас, просвіт їх різко звужений, клубочки зменшені, склерозовані, канальці атрофічні. Для якого захворювання характерні описані зміни?

Амілоїдоз нирок

Хронічний гломерулонефрит

Гіпертонічна хвороба

Гострий некротичний нефроз

Пієлонефрит зі зморщуванням нирок

5594 / 6307
До сімейного лікаря звернувся пацієнт із скаргою на сильний головний біль. Об'єктивно спостерігається: АТ - 220/110 мм рт. ст., ЧСС - 88/хв, ЧД - 18/хв. Діагностовано неускладнений гіпертензивний криз. Для надання невідкладної допомоги лікар обирає препарат групи блокаторів кальцієвих каналів короткої тривалості дії. Який препарат призначив лікар?

Лерканідипін

Амлодипін

Лацидипін

Ніфедипін

Фелодипін

5595 / 6307
На щурах моделювали опіки. Збільшилось утворення гістаміну, серотоніну, кінінів, оксиду азоту. Який тип причинно-наслідкових відносин у патогенезі єднає ці зміни з розвитком артеріальної гіперемії та підвищення проникності судин?

Circulus vitiosus

''Пряма лінія''

Саногенез

Дивергенція

Конвергенція

5596 / 6307
На основі клінічних даних пацієнту вставлено попередній діагноз: гострий панкреатит. Вкажіть біохімічний тест, який підтверджує цей діагноз.

Активність амінотрансфераз крові

Активність амілази крові

Активність лужної фосфатази крові

Рівень креатиніну в крові

Активність кислої фосфатази крові

5597 / 6307
У чоловіка 32-х років високий зріст, гінекомастія, жіночий тип оволосіння, високий голос, розумова відсталість, безпліддя. Попередній діагноз - синдром Клайнфельтера. Для його уточнення необхідно дослідити:

Лейкоцитарну формулу

Родовід

Групу крові

Сперматогенез

Каріотип

5598 / 6307
Хворому з травмою ока лікар призначив засіб, що забезпечує тривале (до 10 діб) розслаблення акомодаційних м'язів. Вкажіть цей препарат:

Пірензепін

Атропіну сульфат

Скополаміну гідробромід

Метацин

Пілокарпіну гідрохлорид

5599 / 6307
Хворого доставила в стаціонар швидка допомога з попереднім діагнозом гострий панкреатит. Визначити активність якого ферменту в крові та сечі необхідно для підтвердження цього діагнозу?

Альфа-амілази

АсАТ

Лактатдегідрогенази

АлАТ

Холінестерази

5600 / 6307
У хворого високий артеріальний тиск внаслідок збільшеного тонусу судин. Для зниження тиску доцільно призначити блокатори:

Н1-рецепторів

alpha-адренорецепторів

beta-адренорецепторів

М-холінорецепторів

alpha- та beta-адренорецепторів

5601 / 6307
Для оцінки придатності води для пиття проведено бактеріологічне дослідження. Який показник характеризує кількість бактерій групи кишкових паличок, що знаходяться в 1 л води?

Титр колі-фага

Колі-індекс

Мікробне число

Колі-титр

Перфрінгенс-титр

5602 / 6307
У дитини 2,5 років з підозрою на дифтерію зіву взяли мазок із слизової та зробили посів на зсілу кінську сироватку. Яке призначення цього етапу мікробіологічної діаг-ностики?

Визначення серотипу токсину

Виявлення токсигенності

Дослідження антигенних властивостей

Виділення чистої культури

Визначення біохімічних властивостей

5603 / 6307
При обстеженні хворого виявлено зменшення кількості лейкоцитів, еритроцитів і гемоглобіну в периферичній крові, появу великих клітин (мегалобластів). Дефіцит якого вітаміну може до цього призводити?

Біотин

Фолієва кислота

Ніацин

Рибофлавін

Аскорбінова кислота

5604 / 6307
У 20-річної дівчини, яка страждає на поліпоз кишечника, у анамнезі вказано часті грибкові та вірусні захворювання. Недостатність якої ланки імунної системи є найімовірнішою в цьому разі?

Натуральних кілерів

В-лімфоцитів

Фагоцитів

Комплементу

Т-лімфоцитів

5605 / 6307
Хворий з пухлиною товстої кишки скаржиться на біль в животі, випорожнення з домішками крові, загальну слабкість. В крові: гемоглобін - 97 ммоль/л, еритроцити - 2,8·10^12/л, КП- 0,66, мікроцитоз, пойкілоцитоз, помірна кількість поліхроматофільних еритроцитів. Який вид анемії у хворого?

Арегенераторна

Гіпохромна

Гіперхромна

Мегалобластична

Гіперрегенераторна

5606 / 6307
В приймальне відділення доставили хворого з тепловим ударом. Які з наведених захисно-компенсаторних реакцій розвиваються при цьому?

Стійка гіперглікемія

Спазм вінцевих судин

Розширення периферійних судин

Звуження периферійних судин

Підвищення ЧСС

5607 / 6307
Чоловік 25 років скаржиться на загальну слабкість, озноб, біль у горлі. Об'єктивно спостерігається: почервоніння в ділянці мигдаликів. Температура тіла - 38,6^oС. Які з наведених клітин є головним джерелом ендогенних пірогенів, що викликають гарячку у пацієнта?

В-лімфоцити

Еозинофіли

Базофіли

Тучні клітини

Нейтрофіли

5608 / 6307
У пацієнтки екзофтальм, схуднення, тахікардія, негативний азотистий баланс, підвищена кількість вільних жирних кислот та глюкози в крові. Для якої дисфункції характерні такі симптоми?

Гіперфункції щитоподібної залози

Цукрового діабету

Гіперпродукції СТГ

Гіпофункції щитоподібної залози

Гіпофункції надниркових залоз

5609 / 6307
В експерименті вимірювали величину кровотоку (мл/хв) у різних органах. Який із нижченаведених органів має найбільшу величину кровотоку на 100 г маси?

Мозок

Печінка

Шлунок

Нирки

Щитоподібна залоза

5610 / 6307
Молодий чоловік 21-го року скаржиться на слабкість, підвищення температури до 38-40^oC) Об'єктивно: печінка і селезінка збільшені. В крові: Hb- 100 г/л, еритроцити - 2,9·10^12/л, лейкоцити - 4,4·10^9/л, тромбоцити - 48 ·10^9/л, нейтрофіли с/я - 17%, лімфоцити - 15%, бластні клітини - 68%. Всі цитохімічні реакції бластних клітин негативні. Дайте гематологічний висновок:

Хронічний мієлолейкоз

Гострий еритромієлоз

Гострий лімфобластний лейкоз

Гострий мієлобластний лейкоз

Недиференційований лейкоз

5611 / 6307
Під час виконання оперативного втручання на колінному суглобі лікар повинен пам'ятати про артерію, що віддає найбільшу кількість гілок для утворення артеріальної суглобової сітки. Укажіть цю артерію.

A) fibularis

A) tibialis posterior

A) femoralis

A) tibialis anterior

A) poplitea

5612 / 6307
Аналіз крові виявив знижений вміст гемоглобіну. Яка функція крові порушиться?

Транспорт поживних речовин

Транспорт гормонів

Забезпечення імунітету

Транспорт газів

Зсідання

5613 / 6307
Чоловік та його дружина гомозиготні за одним геном. Але у чоловіка домінантні алелі цього гена, а у дружини - рецесивні. Яка з наведених закономірностей спадковості спостерігатиметься у їхніх дітей?

Явище успадкування, зчепленого зі статтю

Явище зчепленого успадкування генів

Закон розщеплення

Закон незалежного успадкування ознак

Закон одноманітності гибридів

5614 / 6307
Студент дістав завдання розрахувати альвеолярну вентиляцію. Для цього йому необхідні знати наступні показники зовнішнього дихання:

Частота дихання, життєва ємність легень, резервний об'єм вдиху

Хвилинний об'єм дихання, частота дихання, дихальний об'єм

Дихальний об'єм, резервний об'єм вдиху, резервний об'єм видиху

Об'єм мертвого простору, життєва ємність легень, дихальний об'єм

Дихальний об'єм, об'єм мертвого простору, частота дихання

5615 / 6307
Для лікування злоякісних пухлин призначають метотрексат - структурний аналог фолієвої кислоти, який є конкурентним інгібітором дигідрофолатредуктази і тому гальмує синтез:

Глікогену

Моносахаридів

Гліцерофосфатидів

Нуклеотидів ДНК

Жирних кислот

5616 / 6307
Серед лімфоцитів розрізняють популяцію клітин, що мають мембранні рецептори до IgM, під впливом специфічних антигенів активуються, мітотично розмножуються, диференціюються у плазматичні клітини, що виробляють антитіла (імуноглобуліни). Як називаються ці клітини?

В-лімфоцити

Т-лімфоцити кілери

Т-лімфоцити супресори

Т-лімфоцити пам'яті

5617 / 6307
Під час обстеження у пацієнта виявлено порушення кровообігу підшлункової залози. Яка із артерій може бути пошкоджена у цьому разі?

А. dastrica sinistra

А. gastrica dextra

А. gastroepiploica dextra

А. hepatica propria

А. lienalis

5618 / 6307
Після 10-денного лікування препаратом із групи антибіотиків у пацієнта спостерігаються явища дисбактеріозу: диспептичні явища, кандидомікоз, жовтяниця, фотосенсибілізація. Антибіотик якої групи приймав хворий?

Групи аміноглікозидів

Групи тетрациклінів

Групи рифампіцину

Групи пеніцилінів

Групи цефалоспоринів

5619 / 6307
Під час обстеження пацієнта з явищами гiпертензiї з'ясувалося, що йому найбiльш доцiльно застосувати препарат, який діє на артерiальний тиск через систему ренiн-ангiотензину. Укажіть цей препарат.

Апресин

Дiбазол

Лізиноприл

Анапрiлiн

Октадiн

5620 / 6307
Унаслідок дії електричного струму на клітину скелетного м'яза відбулася деполяризація її мембрани. Рух яких іонів крізь мембрану відіграє основну роль у розвитку деполяризації?

Ca2^+

Na^+

HCO3^-

Cl^-

K^+

5621 / 6307
У хворого 71-го року з облітеруючим атеросклерозом з'явився біль в лівій стопі. До лікаря не звертався. На момент огляду стопа збільшена в об'ємі, тканини в'ялі, чорного кольору, мацеровані. Демаркаційна зона не виражена. Поставте діагноз:

Коагуляційний некроз

Волога гангрена

Муміфікація

Секвестр

Суха гангрена

5622 / 6307
На прийомі у лікаря хворий повідомив, що самостійно застосовує протиалергійний препарат, який ефективно зменшує прояви алергії, проте викликає значну сонливість. Який препарат із групи блокаторів Н1-рецепторів застосовує хворий?

Ранітидин

Димедрол

Тавегіл

Лоратадин

Кромолін-натрій

5623 / 6307
У п'ятирічної дитини з'явилися висока температура, дрібноточковий висип та кон'юнктивіт. Вона померла від пневмонії, що виникла як ускладнення основного захворювання. Під час гістологічного дослідження легень виявлено ендо-, мезо- та панбронхіт і гігантоклітинну пневмонію. Для якого захворювання характерні ці симптоматичні явища?

Скарлатина

Кір

Дифтерія

Крупозна пневмонія

Вітряна віспа

5624 / 6307
У зв'язку зі спалахом дифтерії виникла необхідність провести запобіжні щеплення. Який препарат потрібно використати для створення штучного активного імунітету?

Дифтерійний анатоксин

Антидифтерійну сироватку

Специфічний імуноглобулін

Вакцину з вбитих бактерій

Вакцину АКДП

5625 / 6307
У хворої на дифтерію дитини через 10 днів після введення антитоксичної протидифтерійної сироватки з'явилися висипання на шкірі, які супроводжувалися сильним свербінням, підвищилася температура тіла до 38^oC, з'явився біль в суглобах. Яку причину цих явищ можна припустити?

Анафілактична реакція

Сироваткова хвороба

Атопія

Гіперчутливість уповільненого типу

Контактна алергія

5626 / 6307
Під час експерименту у дослідної тварини стимулювалися барорецептори каротидного синусу. Які зміни з боку серцево-судинної системи будуть спостерігатися?

Підвищується артеріальний тиск

Підвищується секреція передсердних натрійуретричних пептидів

Знижується симпатичний тонус

Збільшується частота серцевих скорочень

Позитивний хронотропний ефект

5627 / 6307
У пацієнта хромосомна хвороба - синдром Клайнфельтера. Загальна кількість хромосом у нього становить 47 (каріотип XXY). У соматичних клітинах виявляється статевий хроматин, кількість якого дорівнює числу X-хромосом мінус 1. Як називається статевий хроматин у соматичних клітинах?

Тільця Жоллі

Тільця Меллорі

Кільця Кебота

Тільця Князькова-Деле

Тільця Барра

5628 / 6307
Одними з медіаторів запалення клітинного походження є лейкотрієни. Під впливом якого ферменту утворюються ці біологічно активні речовини?

Циклооксигенази 1

Ліпооксигенази

Тромбоксансинтетази

Циклооксигенази 2

Фосфоліпази А2

5629 / 6307
Після проведення туберкулінової проби (проба Манту) у дитини за 48 годин на місці введення туберкуліну утворилася папула діаментром до 10 мм. Який механізм гіперчутливості лежить в основі описаних змін?

Імунокомплексна цитотоксичність

Анафілаксія

Гранулематоз

Клітинна цитотоксичність

Антитілозалежна цитотоксичність

5630 / 6307
У людини трапляється спадкова хвороба, симптомокомплекс якої поєднує в собі цироз печінки та дистрофічні процеси головного мозку. Вона супроводжується зменшенням вмісту церулоплазміну в плазмі крові та порушенням обміну міді в організмі. Це хвороба:

Німанна-Піка

Вільсона-Коновалова

Марфана

Тея-Сакса

Жильбера

5631 / 6307
У пацієнта віком 50 років після резекції шлунка посилилися процеси гниття білків у кишечнику. Підвищення якого показника в сечі вказує на цей стан?

Сечової кислоти

Стеркобіліногену

Креатиніну

Оксипроліну

Тваринного індикану

5632 / 6307
У пацієнтів з діагнозом: цукровий діабет, часто спостерігаються запальні процеси, знижується регенерація та уповільнюється загоєння ран. Що є причиною цього явища?

Зниження ліполізу

Зниження протеосинтезу

Посилення катаболізму

Підвищення ліполізу

Прискорення гліконеогенезу

5633 / 6307
У пацієнта віком 30 років, який хворіє на гостре запалення підшлункової залози (панкреатит), виявлено порушення порожнинного травлення білків. Із недостатнім синтезом та виділенням підшлунковою залозою якого ферменту це зумовлено?

Дипептидаз

Трипсину

Амілази

Пепсину

Ліпази

5634 / 6307
У хворого з неврологічними порушеннями діагностована пухлина головного мозку. Під час операції видалена пухлина, що має вигляд щільного вузла, пов'язаного з твердою мозковою оболонкою. Гістологічно пухлина побудована з ендотеліоподібних клітин, тісно прилеглих одна до одної. Ваш діагноз:

Менінгіома

Гліобластома

Нейробластома

Менінгеальна саркома

Астроцитома

5635 / 6307
У пацієнта під час обстеження зліва в V міжребір'ї на 1-2 см латеральніше від середньоключичної лінії під час аускультації краще прослуховується І тон, ніж ІІ. Закриттям якого клапана це обумовлено?

Правим трьохстулковим клапаном

Лівим двостулковим клапаном

Півмісяцевим клапаном аорти

Півмісяцевим клапаном легеневого стовбура

Двостулковим та тристулковим клапанами

5636 / 6307
Як називається механізм активації неактивної форми пепсиногену в активну форму пепсин?

Фосфорилювання

Дефосфорилювання

Обмеженого протеолізу

Ацетилювання

Метилювання

5637 / 6307
Хворому на туберкульоз призначено антибіотик олігоміцин. Назвіть процес, який інгібується цим препаратом у дихальному ланцюзі мітохондрій при розмноженні туберкульозної палички:

Окисне фосфорилювання

Трансамінування

Трансляція

Реплікація

Ініціація

5638 / 6307
У п`ятирічної дитини виявлена спадкова мембранопатія (хвороба Мінковського-Шоффара). Яка зміна осмотичної резистентності еритроцитів спостерігатиметься в цьому разі?

Зниження резистентності

Збільшення зони резистентності

Розширення амплітуди резистентності

Підвищення резистентності

Зниження амплітуди резистентності

5639 / 6307
За здатністю до синтезу в організмі людини всі протеїногенні амінокислоти поділяються на замінні, незамінні та умовно замінні. Яка з наведених амінокислот є незамінною?

Серин

Глутамін

Тирозин

Фенілаланін

Пролін

5640 / 6307
Під час аналізу електрокардіограми встановлено: величина кута альфа дорівнює 80^o. Укажіть положення електричної осі серця.

Відхилена вправо

Відхилена вліво

Горизонтальне

Вертикальне

5641 / 6307
У хворого після травми коліна гомілку у зігнутому під прямим кутом положенні можна зміщувати вперед і назад подібно до висувної шухляди. Які зв'язки розірвані?

Хрестоподібні зв'язки коліна

Коса підколінна

Малогомілкова колатеральна

Великогомілкова колатеральна

Поперечна зв'язка коліна

5642 / 6307
Під час розтину тіла померлого чоловіка виявлено, що речовина головного мозку та лімфатичних вузлів має темно-сірий колір, а печінка та селезінка значно збільшені. Гістологічно в них спостерігаються гемомеланоз і гемосидероз. Із анамнезу відомо про періодичні напади гарячки. Для якого захворювання характерні такі патогістологічні зміни?

Аддісонова хвороба

Гемолітична анемія

Септицемія

Малярія

Чорна віспа

5643 / 6307
У дитини 15 років за 14 днів після перенесеної ангіни з'явилися набряки на обличчі зранку, підвищення артеріального тиску, сеча має вигляд ''м'ясних помиїв''. Імуногістохімічне дослідження біоптату нирки виявило відкладання імунних комплексів на базальних мембранах капілярів та у мезангії клубочків. Яке захворювання розвинулося у пацієнта?

Гострий інтерстиціальний нефрит

Ліпоїдний нефроз

Некротичний нефроз

Гострий пієлонефрит

Гострий гломерулонефрит

5644 / 6307
У хворої при гастродуоденоскопії виявили множинні виразки шлунка. Діагностовано синдром Золлінгера-Еллісона. Надлишок якого гормону має місце в даному випадку?

Тироксин

Кортизол

Гастрин

Інсулін

Окситоцин

5645 / 6307
При вивченні мазка крові людини з наявністю запального процесу можна бачити велику кількість округлих клітин із сегментованим ядром (три і більше сегментів) та дрібною рожево-фіолетовою зернистістю в цитоплазмі. Які це клітини крові?

Базофільні гранулоцити

Лімфоцити

Нейтрофільні гранулоцити

Еритроцити

Еозинофільні гранулоцити

5646 / 6307
У полі зору цистоскопа гладка поверхня слизової оболонки, без складок. Яка частина сечового міхура у полі зору?

Верхівка

Тіло

Міхуровий трикутник

Шийка

Дно

5647 / 6307
Для підтвердження діагнозу: гострий атрофічний кандидоз проведено мікроскопію мазків, виготовлених із шкрібку нальоту на слизовій оболонці щоки, який взятий від хворої жінки. Виявлено овальної форми мікроорганізми, що брунькуються. Який метод забарвлення використав бактеріолог для фарбування мазків із досліджуваного матеріалу?

Романовського-Гімзе

Ожешка

Морозова

Нейсера

Грама

5648 / 6307
Пацієнта шпиталізовано із попереднім діагнозом: черевний тиф. Хворіє упродовж трьох днів. Температура тіла - 39^oС. Який метод лабораторної діагностики необхідно застосувати для підтвердження діагнозу?

Серологічний метод

Виділення уринокультури

Виділення копрокультури

Виділення гемокультури

Виділення білікультури

5649 / 6307
В експерименті на собаці вивчали будову центральних відділів слухової сенсорної системи. Унаслідок руйнування однієї зі структур головного мозку собака втратив орієнтувальний рефлекс на звукові сигнали. Яку структуру зруйновано?

Латеральні колінчасті тіла

Верхні горбки чотиригорбкового тіла

Нижні горбки чотиригорбкового тіла

Червоні ядра

Медіальні колінчасті тіла

5650 / 6307
Що є етіологічним фактором цукрового діабету 1-го типу?

Міцний зв'язок інсуліну з білком

Висока активність інсулінази

Пошкодження гіпофіза

Пошкодження beta-клітин

Відсутність рецепторів до інсуліну

5651 / 6307
Дитина народилась здоровою, але через тиждень у неї з'явилося блювання, а пізніше гіпертонус м'язів, судоми, специфічний солодкий запах сечі і поту. Яке захворювання спостерігається у дитини?

Фруктозурія

Фенілкетонурія

Хвороба кленового сиропу

Гістидинемія

Хвороба Коновалова-Вільсона

5652 / 6307
У дитини 7-ми років з'явився біль у горлі, підвищилася температура тіла. На 2-й день від початку захворювання виявлені висипання червоного кольору у вигляді дрібних, густо розташованих плям завбільшки з макове зерно. Вони вкривають все тіло, за винятком носогубного трикутника. При огляді порожнини рота: в зіві яскраве почервоніння, мигдалики збільшені, язик малиново-червоний. Поставте діагноз:

Аденовірусна інфекція

Стрептококова ангіна

Кір

Дифтерія зіва

Скарлатина

5653 / 6307
Людина в стані спокою штучно примушує себе дихати часто і глибоко впродовж 3 - 4 хвилин. Як це відбивається на кислотно-лужній рівновазі організму?

Виникає змішаний ацидоз

Виникає метаболічний ацидоз

Виникає дихальний ацидоз

Виникає дихальний алкалоз

Виникає метаболічний алкалоз

5654 / 6307
Під час посмертного розтину тіла чоловіка віком 62 роки виявлено надклапанний розрив аорти з тампонадою серця. Гістологічне дослідження висхідного відділу аорти виявило в зовнішній і середній оболонках інфільтрати з лімфоїдних, плазматичних, епітеліоїдних клітин, вогнища некрозу в середній оболонці, проліферацію адвентиціальних і ендотеліальних клітин та судин зовнішньої оболонки. Для якої патології характерні такі зміни в аорті?

Сифілітичного аортиту

Септичного аортиту

Атеросклерозу

Ревматичного аортиту

Гіпертонічної хвороби

5655 / 6307
У пацієнта спостерігається: телеангіоектазія, атаксія, в аналізі крові - знижена кількість Т-лімфоцитів, відсутність IgА, зниження рівня IgG, IgМ. Для якого синдрому характерні такі показники?

Синдром Віскотта-Олдріча

Синдром Луї-Бар

Синдром Дауна

Синдром Шерешевського-Тернера

Синдром Клайнфельтера

5656 / 6307
У пацієнта після часткової резекції підшлункової залози з'явилась стеаторея, що свідчить про порушення травлення жирів в кишечнику. З недостатністю якого ферменту це пов'язано?

Гастриксину

Трипсину

Ліпази

Пепсину

Амілази

5657 / 6307
Багато слизових оболонок людини продукують фермент, що викликає лізис бактерій. Його виявляють у слізній рідині, слині та в слизі шлунково-кишкового тракту. Вкажіть цей фермент.

Опсонін

Гіалуронідаза

Лізоцим

Комплемент

5658 / 6307
При пункційній біопсії в трансплантованій нирці виявлена дифузна інфільтрація строми лімфоцитами, плазмоцитами, лімфобластами, плазмобластами, а також некротичний артеріїт. Який патологічний процес розвинувся у трансплантаті?

Iшемічне пошкодження нирки

Імунне відторгнення

Гломерулонефрит

Тубулонекроз

5659 / 6307
Під час рентгенологічного обстеження виявлено: перелом основи черепа. Лінія перелому проходить через остистий і круглий отвори. Яка кістка пошкоджена внаслідок травми?

Решітчаста

Лобова

Потилична

Клиноподібна

5660 / 6307
У пацієнта через рік після резекції 2/3 шлунка виникли скарги на блідість шкірних покривів, головні болі, запаморочення, загальну слабкість. Загальний аналіз крові: гемоглобін - 60 г/л, еритроцити - 2,4·10^12/л. Яка причина виникнення цього патологічного стану?

Підвищення секреції внутрішнього фактора Кастла

Підвищення вмісту фолієвої кислоти

Зниження секреції внутрішнього фактора Кастла

Зниження всмоктування міді

5661 / 6307
У людини в артеріальній крові напруга кисню збільшена до 104 мм рт. ст., а вуглекислого газу - зменшена до 36 мм рт. ст. Такі зміни можуть бути наслідком:

Довільної гіпервентиляції

Перебування в горах

Помірного фізичного навантаження

Затримки дихання

5662 / 6307
Під час дослідження залишкового азоту виявлено, що азот сечовини значно знижений. Для захворювання якого органа це характерно?

Шлунка

Кишечника

Печінки

Серця

5663 / 6307
Під час дослідження нирки виявлено: нирка набрякла, повнокровна, капсула легко знімається. Порожнини мисок і чашечок розширені, заповнені каламутною сечею, їх слизова оболонка тьмяна, з вогнищами крововиливів. На розрізі тканина нирки строката, жовто-сірі ділянки оточені зоною повнокров'я і геморагій. Якому захворюванню відповідає такий макроскопічний вид нирок?

Гострий гломерулонефрит

Амілоїдоз нирок

Гострий пієлонефрит

Нефролітіаз

5664 / 6307
У пацієнтки віком 64 роки виник патологічний перелом плечової кістки. Під час біопсії виявлено атипові плазматичні клітини. Рентгенологічно у місці перелому спостерігаються пухлиноподібні утворення. Якому захворюванню відповідають такі патологічні зміни?

Фіброзна дисплазія кістки

Мієломна хвороба

Метастаз аденокарциноми

Хронічний остіомієліт

5665 / 6307
На електронній мікрофотографії міокарда видно клітини відростчатої форми, що містять мало органел, але мають добре розвинену гранулярну ендоплазматичну сітку та секреторні гранули. Вкажіть ці клітини.

Пейсмекерні клітини

Секреторні кардіоміоцити

Перехідні атипові клітини

Клітини пучка Гіса

5666 / 6307
Дитина 2 років випила очні краплі з домашньої аптечки. Стан важкий, значне пото- і слиновиділення, астматичне дихання, кашель, зіниці різко звужені, тони серця глухі, брадикардія. Перистальтика кишечника посилена, пронос, артеріальний тиск знижений. Яким препаратом викликане отруєння?

Платифіліну гідротартрат

Пілокарпіну гідрохлорид

Атропін

Сульфацил-натрій

5667 / 6307
Чоловікові 54 років діагностовано великовогнищевий інфаркт міокарда передньої стінки лівого шлуночка. В якій артерії порушений кровообіг?

У задній міжшлуночковій гілці правої вінцевої артерії

У правій вінцевій артерії

У передній міжшлуночковій гілці правої вінцевої артерії

В огинальній гілці лівої вінцевої артерії

5668 / 6307
У пацієнта спостерігається травма променево-зап'ясткового суглоба. Якими кістками проксимального ряду зап'ястку (крім горохоподібної) сформовано цей суглоб?

Трапецієподібною, гачкуватою, півмісяцевою

Човноподібною, півмісяцевою, тригранною

Човноподібною, трапецієподібною, півмісяцевою

5669 / 6307
Хворий на хронічний пієлонефрит помер від хронічної ниркової недостатності. При житті аускультативно відмічено ''шум тертя перикарду''. На розтині виявлено, що епікард тьмяний, шорсткий, ніби покритий волосяним покривом. Який перикардит за характером запалення має місце?

Катаральний

Гнильний

Гнійний

Фібринозний

5670 / 6307
На ізольований нерв жаби один за одним подіяли двома подразниками порогової сили. Другий подразник потрапив у фазу деполяризації потенціалу дії. Чому в цьому разі виникне лише один потенціал дії?

Знизилася калієва проникність мембрани нерва

Другий подразник потрапив у фазу абсолютної рефрактерності

Підвищилася лабільність нерва

Знизився критичний рівень деполяризації нерва

5671 / 6307
У чоловіка внаслідок зловживання алкоголем розвинулась жирова дистрофія печінки. Порушення якої ланки обміну ліпідів має місце в цьому разі?

Обмін жирів у жировій клітковині

Транспортування жирів

Всмоктування жирів

Проміжний обмін ліпідів

Бета-окиснення ліпідів

5672 / 6307
В умовах запалення знижується сила місцевоанестезуючої дії новокаїну. В умовах якого стану у вогнищі запалення виникає порушення гідролізу солі новокаїну і вивільнення активного анестетика-основи?

Пригнічення окислювального фосфорилювання

Локального тканинного ацидозу

Локального тканинного алкалозу

Пригнічення активності карбоангідрази

5673 / 6307
Пацієнтка віком 35 років за 2 тижні після перенесеного COVID-19 проходила диспансерне спостереження. Під час проведення ЕКГ виявлено зниження вольтажу зубців, зубець Р не змінений і зв'язаний з QRS комплексом, тривалість інтервалу РQ - 0,32 с. Який тип аритмії розвинувся у пацієнтки?

Атріо-вентрикулярна блокада 1 ступеня

Атріо-вентрикулярна блокада 3 ступеня

Атріо-вентрикулярна блокада 2 ступеня

Синдром Вольфа-Паркінсона-Уайта

5674 / 6307
30-річний чоловік звернувся на консультацію до стоматолога зі скаргами на розлади жування, біль при відтягуванні нижньої щелепи назад. Лікар встановив запалення одного з жувальних м'язів. Якого саме?

Крилоподібного медіального

Скроневого (передні волокна)

Скроневого (задні волокна)

Жувального

Крилоподібного латерального

5675 / 6307
На електронній мікрофотографії ділянки нирки у стінці приносної та виносної артеріол визначаються клітини з великими секреторними гранулами в цитоплазмі. Визначте структурне утворення нирки, до складу якого входять ці клітини:

Юкстагломерулярний апарат

Петля нефрона

Проксимальний відділ нефрона

Дистальний відділ нефрона

5676 / 6307
На розтині тіла жінки 23-х років, яка померла при явищах ниркової недостатності, на шкірі обличчя виявлений ''червоний метелик'', на мітральному клапані дрібні до 0,2 см червонувато-рожеві бородавчасті нашарування, в нирках осередки фібриноїдного некрозу в клубочках, потовщення базальних мембран капілярів клубочків у вигляді ''дротяних петель'', гематоксилінові тільця, каріорексис. Яке захворювання стало причиною смерті хворої?

Системний червоний вовчак

Вузликовий періартеріїт

Системна склеродермія

Ревматоїдний артрит

Ревматизм

5677 / 6307
Першим етапом діагностування хвороб, які зумовлені порушенням обміну речовин, є скринінг-метод, після якого використовують точніші методи дослідження ферментів і амінокислот. Яку назву має цей метод?

Цитогенетичний

Біохімічний

Гібридизація соматичних клітин

Імунологічний

5678 / 6307
Хворий звернувся до стоматолога зі скаргами на гнійне запалення ясен. Який препарат буде найбільш ефективним, якщо передбачається анаеробна природа збудника?

Метронідазол

Оксацилін-натрій

Ко-тримоксазол

Гентаміцин

5679 / 6307
У хворого зіниця звужена і у разі зменшення освітлення не розширюється. У якому місці відбулося ураження центральної нервової системи?

Бічний ріг сірої речовини спинного мозку на рівні С VIII - Th I

Бічне колінчасте тіло проміжного мозку

Основа ніжки середнього мозку на рівні нижніх горбиків покришки

Верхні горбики покришки середнього мозку

5680 / 6307
У відповідь на розтягнення м'яза спостерігається його рефлекторне скорочення. З подразнення яких рецепторів починається ця рефлекторна реакція?

М'язові веретена

Суглобові рецептори

Больові рецептори

Дотикові рецептори

Сухожилкові рецептори Гольджі

5681 / 6307
Жінка 25-ти років госпіталізована в гінекологічне відділення з метою оперативного втручання з приводу пухлини яєчника. При здійсненні оперативного втручання необхідно розсікати зв'язку, що з'єднує яєчник з маткою. Яку саме?

Lig. cardinale

Lig. umbilicale laterale

Lig. suspensorium ovarii

Lig. ovarii proprium

Lig. latum uteri

5682 / 6307
Пацієнта шпиталізовано зі скаргами на періодичні напади серцебиття, які виникають раптово і так само раптово припиняються. Під час проведення ЕКГ виявлено епізод скорочень частотою 200 уд/хв, який має такі ознаки: ритм правильний, зубець Р відсутній, комплекс QRS без змін, зубець Т деформований. Укажіть вид аритмії.

Повна АВ блокада

Шлуночкова екстрасистолія

Передсердна екстрасистолія

Суправентрикулярна пароксизмальна тахікардія

АВ блокада І ст.

5683 / 6307
До інфекційної лабораторії надійшли випорожнення пацієнта з діагнозом холера. Який метод мікробіологічної діагностики потрібно застосувати, щоб підтвердити чи спростувати цей діагноз?

Алергічний

Бактеріологічний

Бактеріоскопічний

Біологічний

5684 / 6307
У пацієнтки 26-ти років висипання на шкірі, свербіж після вживання цитрусових. Призначте лікарський засіб з групи блокаторів Н1-гістамінорецепторів:

Парацетамол

Дифенгідрамін

Менадіону натрію біосульфат

Метамізол

5685 / 6307
Під час патоморфологічного дослідження жовчного міхура після холецистектомії виявлено, що розміри його збільшені, стінки потовщені, серозна оболонка тьмяна та повнокровна, у порожнині міхура містяться в'язкі жовто-зелені маси. Мікроскопічно в стінці міхура спостерігається дифузна інфільтрація сегментоядерними нейтрофілами. Яка форма холециститу найімовірніша?

Гранульоматозний

Гострий катаральний

Хронічний

Гострий флегмонозний

5686 / 6307
Внаслідок впливу гамма-випромінювання ділянка ланцюга ДНК повернулась на 180 градусів. Яка мутація відбулася в ДНК?

Дуплікація

Транслокація

Реплікація

Інверсія

5687 / 6307
Унаслідок гострої ниркової недостатності у чоловіка виникла олігурія. Яка добова кількість сечі відповідає цьому симптому?

100-500 мл

1500-2000 мл

500-1000 мл

50-100 мл

5688 / 6307
Із розвитком медичної генетики з'явилася можливість одужання за допомогою дієтотерапії при деяких спадкових хворобах, які раніше вважалися невиліковними. Яке з нижченаведених захворювань можна вилікувати за допомогою дієтотерапії?

Ахондроплазію

Фенілкетонурію

Гемофілію

Дальтонізм

5689 / 6307
У людини трапляються хвороби, пов'язані із порушенням розщеплення і накопичення у клітинах глікогену, ліпідів та ін. Причиною виникнення цих спадкових хвороб є відсутність відповідних ферментів у:

Лізосомах

Ендоплазматичній сітці

Мікротрубочках

Мітохондріях

5690 / 6307
Пацієнтка віком 25 років за місяць після пологів звернулася до лікаря зі скаргою на зменшення кількості молока. Дефіцит якого гормону вплинув на вироблення грудного молока у жінки?

Пролактину

Соматостатину

Глюкагону

Адренокортикотропного гормона

5691 / 6307
Серологічна діагностика інфекційних захворювань заснована на специфічній взаємодії антитіл з антигенами. Як називається серологічна реакція, у разі якої високодисперсні антигени адсорбовані на еритроцитах?

Реакція преципітації

Реакція непрямої (пасивної) гемаглютинації

Реакція нейтралізації

Реакція гемадсорбції

Реакція зв'язування комплементу

5692 / 6307
Для короткотривалого хірургічного втручання пацієнту було введено дроперидол та фентаніл. Вкажіть, що зумовлює можливість виключення больової чутливості при їх спільному застосуванні.

Хімічна взаємодія

Кумуляція

Антагонізм

Потенціювання

5693 / 6307
Дитина трьох років доставлена до реанімаційного відділення інфекційної лікарні. При огляді: стан важкий, шкіра та слизові оболонки сухі, тургор тканин знижений. З анамнезу відомо, що протягом доби після вживання недоброякісної їжі в дитини була профузна діарея та багаторазове блювання. Який вид порушення водно-сольового обміну розвинувся у хворої дитини?

Ізоосмолярна дегідратація

Ізоосмолярна гіпергідратація

Гіпоосмолярна дегідратація

Гіпоосмолярна гіпергідратація

5694 / 6307
При гістологічному дослідженні шийного лімфатичного вузла було виявлено повнокрів'я та набухання коркового шару, в мозковій речовині наявна велика кількість плазматичних клітин, зменшення кількості лімфоцитів, значна макрофагальна реакція. Назвіть характер змін в лімфатичному вузлі:

Антигенна стимуляція лімфоїдної тканини

Лімфома

Гострий лімфаденіт

5695 / 6307
У недоношених новонароджених порушений синтез сурфактанту. Які функції він виконує в легенях?

Підвищує опір дихальних шляхів

Зменшує поверхневий натяг стінок альвеол

Збільшує поверхневий натяг стінок альвеол

Полегшує екскурсію діафрагми

5696 / 6307
У жінки 28 років виявлена позаматкова вагітність, яка ускладнилася розривом маткової труби. У який простір очеревини можливе попадання крові?

Лівий брижовий синус

Міхурово-матковий

Міжсигмоподібний синус

Прямокишково-матковий

5697 / 6307
У людини частота серцевих скорочень постійно утримується на рівні 40 разів за хвилину. Що є водієм ритму серця?

Волокна Пуркіньє

Ніжки пучка Гіса

Синоатріальний вузол

Атріовентрикулярний вузол

Пучок Гіса

5698 / 6307
Для розслаблення скелетних м'язів, щоб зіставити відламки стегнової кістки, що утворилися внаслідок перелому, хворому був введений міореалаксант, що призвело до зупинки дихання. Після введення свіжої цитратної крові дихання відновилося. Який міореалаксант ввели хворому?

Піпекуронію бромід

Дитилін

Атракурію бесилат

Панкуронію бромід

5699 / 6307
Для лікування виразкової хвороби шлунка пацієнта призначено фамотидин. Який механізм дії цього препарату?

Вплив на йонні канали клітинних мембран

Блокада Н_2 гістамінових рецепторів

Антиферментна дія

Вплив на транспортні системи клітинних мембран

5700 / 6307
Хворий на виразкову хворобу з локалізацією виразки у дванадцятипалій кишці відзначає печію, нудоту, періодичне блювання. Який препарат з блокаторів Н2-гістамінових рецепторів потрібно йому призначити?

Дифенгідрамін

Перфеназин

Атропіну сульфат

Фамотидин

5701 / 6307
Офтальмологом у новонародженої дитини виявлено гнійні виділення з кон’юнктиви ока. При мікроскопії мазка з кон'юнктиви ока знайдено велику кількість лейкоцитів та грамнегативні бобовидні диплококи, розташовані всередині лейкоцитів. Який збудник є причиною цього захворювання?

Neisseria catarrhalis

Streptococcus pyogenes

Staphylococcus epidermitis

Neisseria gonnorrhoeae

5702 / 6307
Пацієнту встановлено діагноз: виразкова хвороба шлунка із підвищеною кислотністю. Під час ендоскопічного та бактеріологічного досліджень виділено бактерії роду Helicobacter. Завдяки якій своїй властивості ці мікроорганізми не гинуть у кислому середовищі шлунка?

Здатності утворювати капсулу

Уреазній активності

Стійкості до ванкоміцину

Каталазній активності

5703 / 6307
До реанімаційного відділення госпіталізовано пацієнта з отруєнням солями важких металів. Який антидот треба застосувати в цьому разі?

Унітіол

Атропіну сульфат

Алоксим

Налоксон

5704 / 6307
Серед мешканців тайгового селища, які займаються збором і заготівлею ягід, почастішали випадки зараження на альвеококоз. Що є джерелом інвазії при даному захворюванні?

Птахи

Хворі люди

Гризуни

Лисиці

5705 / 6307
Під час емоційного збудження частота серцевих скорочень (ЧСС) у людини 30-ти років досягла 112/хв. Зміна стану якої структури провідної системи серця є причиною збільшення ЧСС?

Пучок Гіса

Атріовентрикулярний вузол

Волокна Пуркіньє

Синоатріальний вузол

5706 / 6307
Зріст десятирічної дитини сягає 178 см, а її маса - 64 кг. Із порушенням діяльності якої ендокринної залози це пов'язано?

Надниркових залоз

Щитоподібної залози

Гіпофізу

Статевих залоз

5707 / 6307
У хворого на бронхіальну астму виникла гостра недостатність дихання. Який тип недостатності дихання виникає в цьому разі?

Обструктивне порушення альвеолярної вентиляції

Перфузійний

Рестриктивне порушення альвеолярної вентиляції

Дисрегуляторне порушення альвеолярної вентиляції

5708 / 6307
У чоловіка колаптоїдний стан через зниження тонусу периферичних судин. Який препарат найефективніший у цій ситуації?

Ізадрин

Клофелін

Празозин

Мезатон

Прозерин

5709 / 6307
Фенілкетонурія успадковується як аутосомна рецесивна ознака. У родині, де обоє батьків здорові, народилася дитина, хвора на фенілкетонурію. Які генотипи батьків?

АА х Аа

аа х аа

Аа х Аа

АА х АА

Аа х аа

5710 / 6307
У 12-річного хлопчика до перебігу грипу приєднався респіраторний мікоплазмоз. Який вид інфекції розвинувся за цих умов?

Змішана інфекція

Аутоінфекція

Рецидив

Ятрогенна інфекція

Суперінфекція

5711 / 6307
Жінка 40-ка років звернулася із скаргами на неможливість розгинати стопу і пальці, що створює труднощі при ходьбі. Об'єктивно: ступня звисає, дещо повернена всередину, пальці її зігнуті (''кінська стопа''), чутливість втрачена на зовнішній поверхні гомілки і тильній поверхні стопи. Який нерв уражений?

Сідничний

Стегновий

Підшкірний

Спільний малогомілковий

Великогомілковий

5712 / 6307
При аналізі крові людини виявлено: натрій - 115 ммоль/л, хлориди - 85 ммоль/л, глюкоза - 6 ммоль/л, загальний білок - 65 г/л. Наслідком зазначених змін, перш за все, стане зменшення:

Осмотичного тиску крові

Швидкості осідання еритроцитів

Онкотичного тиску крові

Об'єму циркулюючої крові

рН крові

5713 / 6307
Під час копрологічного дослідження встановлено, що кал знебарвлений, у ньому знайдено краплі нейтрального жиру. Найімовірнішою причиною цього є порушення:

Процесів усмоктування в кишечнику

Секреції кишкового соку

Надходження жовчі в кишечник

Кислотності шлункового соку

Секреції підшлункового соку

5714 / 6307
У хворого на аденому клубочкової зони кори наднирників (синдром Конна) спостерігаються артеріальна гіпертензія, напади судом, поліурія. Що є головною ланкою в патогенезі цих порушень?

Гіпосекреція глюкокортикоїдів

Гіперальдостеронізм

Гіперсекреція катехоламінів

Гіпоальдостеронізм

Гіперсекреція глюкокортикоїдів

5715 / 6307
Пацієнту після видалення щитоподібної залози призначили препарат замісної терапії. Виберіть із наведених цей медикаментозний засіб:

Кортикотропін

Мерказоліл

Протирелін

L-тироксин

Калію йодид

5716 / 6307
У хворого 40-ка років в результаті щелепно-лицьової травми порушилася функція під'язикової і підщелепної залоз зліва - залози почали секретувати невелику кількість густої слини. Функція якого нерва порушена?

Блукаючий

Під'язиковий

Лицьовий

Трійчастий

Язикоглотковий

5717 / 6307
Який період життєвого циклу малярійного плазмодія співпадає із появою у хворого клінічних симптомів малярії?

При виході мерозоїтів із зруйнованих еритроцитів

При проникненні мерозоїтів у еритроцити

При проникнені спорозоїтів у кров людини

У період тканинної шизогонії

Під час утворення гаметоцитів

5718 / 6307
І.М. Сєченов встановив, що втомлена кінцівка відновлює працездатність швидше, якщо в період відпочинку друга кінцівка працює. Це дало можливість розробити вчення про:

Парабіоз

Песимум

Оптимум

Втому

Активний відпочинок

5719 / 6307
У дитини 2-х років при обстеженні виявлено гепатоспленомегалію, катаракту, затримку розумового розвитку. В крові - підвищення вмісту галактози. Яка спадкова патологія у дитини?

Непереносимість дисахаридів

Гіповітаміноз D

Порфірія

Галактоземія

Фенілкетонурія

5720 / 6307
У пацієнта виявлено сліпоту кіркового походження. Тромбоз якої артерії в нього розвинувся?

Передньої мозкової

Задньої мозкової

Передньої ворсинчастої

Задньої сполучної

Середньої мозкової

5721 / 6307
Хворому із загальним набряковим синдромом на тлі серцевої недостатності призначено фуросемід. Вплив на який процес забезпечує його терапевтичний ефект в цьому випадку?

Транспорт іонів крізь апікальну мембрану

Синтез натрієвих каналів в дистальних канальцях

Нирковий кровотік

Карбоангідразу

Активний транспорт іонів крізь базальну мембрану

5722 / 6307
Людина вийшла з кондиційованого приміщення на вулицю, де температура повітря дорівнює +40^oC, вологість повітря - 60%. Віддача тепла з організму на вулиці буде здійснюватися за рахунок:

Випаровування поту

Радіації

Конвекції

Проведення

5723 / 6307
У пацієнта травма кульшового суглоба. Рентгенологічно чітко виявлено внутрішньосуглобовий крововилив. Яка зв'язка розірвана?

Zona orbicularis

Lig. iliofemorale

Lig. transversum acetabuli

Lig. ischiofemorale

Lig. capitis femoris

5724 / 6307
У чоловіка на ЕКГ виявлено збільшення тривалості інтервалу QT. Це може бути наслідком зменшення у шлуночках швидкості:

Реполяризації

Скорочення

Розслаблення

Деполяризації

Деполяризації та реполяризації

5725 / 6307
Пацієнтка віком 32 роки скаржиться на безпліддя, порушення менструального циклу, хронічну ановуляцію та полікістоз яєчників. Під час бімануального обстеження з обох сторін виявлено збільшені яєчники щільної консистенції, розмірами 5x6 см. Ці ж дані підтверджено при УЗД. Під час гістологічного дослідження виявлено потовщену фіброзну капсулу яєчника, що вкриває незліченні кістозні фолікули, вистелені гранульозними клітинами з гіперпластичною лютеїновою внутрішньою оболонкою (текою). Жовтих тіл немає. Що спричинило цей стан у пацієнтки?

Синдром полікістозних яєчників (синдром Штейна-Левенталя)

Хронічний двосторонній аднексит

Передменструальний синдром

Андробластома яєчників

Адреногенітальний синдром

5726 / 6307
Хворому з травмою передпліччя під час репозиції кісток для міорелаксації введено дитилін. Повне відновлення тонусу і функції м'язів спостерігалось більш, ніж через годину. Що може бути причиною значного подовження курареподібної дії препарату?

Генетичний дефіцит гідроксилаз

Генетичний дефіцит моноаміноксидази

Генетичний дефіцит бутирилхолінестерази

Пригнічення мікросомного окиснення

Утворення активного метаболіту

5727 / 6307
Під час перебування в погано провітрюваному приміщенні з великим скупченням людей у літнього чоловіка відбулося підвищення артеріального тиску. Який механізм розвитку такої реакції?

Депресорний рефлекс з осморецепторів

Депресорний рефлекс з хеморецепторів

Пресорний рефлекс з хеморецепторів

Пресорний рефлекс з волюморецепторів

Пресорний рефлекс з осморецепторів

5728 / 6307
Під час мікроскопічного дослідження тканини легенів виявлено ділянку запалення, яка складається з вогнища некрозу, оточеного правильними рядами епітеліоїдних та лімфоїдних клітин; наявні плазматичні клітини, макрофаги та гігантські багатоядерні клітини Пирогова-Лангханса. Укажіть вид такого запалення.

Типове продуктивне запалення

Альтеративне запалення

Запалення внаслідок лепри

Ексудативне запалення

Туберкульозне запалення

5729 / 6307
Під час патологоанатомічного дослідження спинного мозку чоловіка 70 років виявлені деструкція та зменшення кількості клітин ядер передніх рогів у шийному і грудному відділах. Які функції були порушені у цього чоловіка за життя?

Моторні функції верхніх кінцівок

Чутливість верхніх кінцівок

Моторні функції нижніх кінцівок

Чутливість і моторні функції верхніх кінцівок

Чутливість нижніх кінцівок

5730 / 6307
Хворому з великими опіками зробили пересадку донорської шкіри. На 8-му добу трансплантат став набряклим, змінився його колір; на 11 добу почав відторгатися. Які клітини беруть у цьому участь?

Еозинофіли

Базофіли

Еритроцити

В-лімфоцити

Т-лімфоцити

5731 / 6307
У пацієнта після перелому великогомілкової кістки відбувається надмірна продукція кісткової тканини - екзостоз. Визначте, який вид регенерації спостерігається у нього?

Фізіологічна

Патологічна недостатня

Репаративна

Патологічна надмірна

5732 / 6307
Мікрофлора шлунку нечисленна. Це пов'язано з кислотністю шлункового вмісту. Проте H. pylori здатна виживати в шлунку, завдяки продукції певного фермену. Назвіть цей фермент.

Уреаза

Протеаза

Гіалуронідаза

Аденілатциклаза

Ліпаза

5733 / 6307
В аналізі крові пацієнта спостерігає-ться значне підвищення рівня лімфоцитів. Який з наведених гормонів сприяє цьому?

Тирозин

Тимозин

Нейротензин

Мотилін

Соматостатин

5734 / 6307
Під час мікроскопії нирки патологоанатом виявив, що приблизно у 80% клубочків спостерігається розмноження епітелію зовнішнього листка капсули Шумлянського, що формує <<півмісяці>>. Він дійшов висновку, що така картина відповідає:

Інтракапілярному ексудативному гломерулонефриту

Екстракапілярному проліферативному (швидкопрогресуючому) гломерулонефриту

Екстракапілярному ексудативному гломерулонефриту

Фібропластичному гломерулонефриту

Інтракапілярному проліферативному гломерулонефриту

5735 / 6307
У пацієнта виявлено змішану глистну інвазію: аскаридоз кишечника та трематодоз печінки. Який протигельмінтний препарат доцільно призначити у цьому разі?

Пірантел

Левамізол

Піперазину адипінат

Мебендазол

Хлоксил

5736 / 6307
У хворого при обстеженні виявлено відсутність Т-лімфоцитів, дефекти обличчя, щитоподібної і прищитоподібних залоз, вада серця. Не розвиваються імунні реакції клітинного типу. Діагностовано синдром Ді Джорджі, який зумовлений:

Комбінованим імунодефіцитом

Гіпоплазією вилочкової залози

Гіперплазією вилочкової залози

Первинною недостатністю В-лімфоцитів

Первинною недостатністю Т-лімфоцитів

5737 / 6307
У дитини, яка часто хворіє на ангіни та фарингіти, відзначається збільшення лімфовузлів і селезінки. Зовнішній вигляд характеризується пастозністю та блідістю, м'язова тканина розвинена слабко. У крові спостерігається лімфоцитоз. Як називає-ться такий вид діатезу?

Лімфатико-гіпопластичний

Нервово-артритичний

Астенічний

Геморагічний

Ексудативно-катаральний

5738 / 6307
В комплексній терапії виразкової хвороби шлунка призначено препарат, який є конкурентним антагоністом гістамінових рецепторів. Впливаючи на Н_2-рецептори парієтальних клітин знижує індукцію хлористоводневої кислоти. Вкажіть цей препарат.

Омепразол

Пірензепін

Мізопростол

Фамотидин

Сукральфат

5739 / 6307
У пацієнта спостерігається обмеження згинання у ліктьовому суглобі, зниження тонусу двоголового м'яза плеча та втрата чутливості шкіри на передньо-латеральній поверхні передпліччя. Функція якого нерва порушена?

N. ulnaris

N. medianus

N. axillaris

N. radialis

N. musculocutaneus

5740 / 6307
У пацієнта обстежували сприйняття звуків за допомогою камертона. Під час розташування його біля зовнішнього вуха пацієнт не чув правим вухом звук камертона. Коли лікар розташував ніжку камертона на соскоподібному відростку пацієнт відчув звук. З ураженням якої частини слухової сенсорної системи це пов'язано?

Слухового нерва

Медіального колінчастого тіла

Середнього вуха

Внутрішнього вуха

Нижніх горбиків

5741 / 6307
Під час ревізії черевної порожнини виявлено венозну кровотечу з печінково-дванадцятипалої зв'язки. Яку з вен пошкоджено?

Ворітну вену печінки

Нижню брижову вену

Нижню порожнисту вену

Селезінкову вену

Верхню брижову вену

5742 / 6307
У пацієнта, який хворіє на виражений пневмосклероз, після перенесеного інфільтративного туберкульозу легень розвинулася дихальна недостатність. До якого патогенетичного типу вона відноситься?

Дисрегуляційного

Рестриктивного

Апнейстичного

Рефлекторного

Обструктивного

5743 / 6307
У деяких дорослих людей після вживання молока спостерігаються диспептичні явища. Дефіцит якого ферменту асоціюється з проблемою несприйняття молока?

Ліпази

Пептидази

Лактази

Амілази

Мальтази

5744 / 6307
У дев'ятирічного хлопчика виявлено ураження нирок та підвищенний артеріальний тиск. Підвищення рівня якого біологічно активного пептиду спричинив цей стан?

Ангіотензину ІІ

Калідину

Антидіуретичного гормона

Інсуліну

Глюкагону

5745 / 6307
Дослідження показало, що загальна кількість лейкоцитів в крові пацієнта становить 11·10^9/л; при цьому нейтрофіли становлять 80%, з них 9% - паличкоядерні. Охарактеризуйте зміни клітинного складу ''білої'' крові в зазначеному випадку:

Ядерний зсув нейтрофілів вправо

Лімфоцитоз

Ядерний зсув нейтрофілів вліво

Нейтропенія

Лейкопенія

5746 / 6307
До лікаря звернулися родичі пацієнта віком 52 років зі скаргами на те, що він не може прочитати написане, не розуміє що йому кажуть, але може розмовляти. Де локалізується зона ураження мозку?

У корі заднього відділу нижньої лобової звивини

У корі заднього відділу верхньої скроневої звивини

У гіпокампі

У корі переднього відділу верхньої скроневої звивини

5747 / 6307
У головному мозку людини утворюються ендогенні пептиди, подібні до морфію, які здатні зменшувати больові відчуття. До них відносяться:

Ліберини

Ендорфіни

Вазопресин

Окситоцин

Статини

5748 / 6307
На розтині чоловіка 52 років, який тривалий час страждав на туберкульозний простатит і помер від менінгоенцефаліту, у м'яких оболонках основи і бічних поверхонь головного мозку, селезінці, нирках, печінці виявлено велику кількість щільних, сірого кольору вузликів діаметром 0,5 - 1мм. Гістологічне дослідження показало, що вузлики складаються з епітеліоїдних, лімфоїдних і нечисленних гігантських клітин з ядрами, розташованими на периферії клітин, що мають вигляд підкови. Виявлені зміни свідчать про:

Вторинний туберкульоз

Септикопіємію

Найгостріший туберкульозний сепсис

Міліарний туберкульоз

Великовогнищевий дисемінований туберкульоз

5749 / 6307
У патогенезі холери значну роль відіграють екзо- і ендотоксини, ферменти агресії. Основним синдромом цієї хвороби є дегідратація. Які з наведених патогенетичних впливів є основною причиною зневоднення?

Відщеплення нейрамінової кислоти

Деструкція муцину

Активація аденілатциклази

Дефект фосфоліпідів мембран

Деструкція гіалуронової кислоти

5750 / 6307
У пацієнта хворого на цукровий діабет, розвинулася діабетична кома внаслідок порушення кислотно-основного стану. Який вид порушення виник у цьому разі?

Змішаний алкалоз

Респіраторний ацидоз

Метаболічний ацидоз

Негазовий алкалоз

Метаболічний алкалоз

5751 / 6307
Цукровий діабет матері призхвів до народження дитини з вродженими вадами опорно-рухової, серцево-судинної та нервової систем. Вплив яких факторів спричинив розвиток діабетичної ембріопатії?

Мутагенних

Антропогенних

Тератогенних

Канцерогенних

Екзогенних

5752 / 6307
У хворого з гострим циститом при дослідженні сечі виявили лейкоцити і багато грамнегативних паличок. При посіві виросли колонії слизового характеру, які утворювали зелений, розчинний пігмент. Який мікроорганізм, ймовірно, є причиною захворювання?

Klebsiella pneumoniae

Escherihia coli

Salmonella enteritidis

Pseudomonas aeruginosa

Proteus mirabilis

5753 / 6307
У пацієнта під час огляду встановлено наявність атетозу та хореї. Яка структура ЦНС імовірно уражена?

Мозочок

Гіпоталамус

Смугасте тіло

Довгастий мозок

Лімбічна система

5754 / 6307
У барокамері знизили тиск до 400 мм рт.ст. Як зміниться зовнішнє дихання людини, яка знаходиться у барокамері?

Зменшиться глибина і зросте частота дихання

Збільшиться глибина і частота дихання

Зменшиться глибина і частота дихання

Збільшиться глибина і зменшиться частота дихання

Залишиться без змін

5755 / 6307
У пацієнта з гіпохромною анемією в еритроцитах знаходиться 45% Hb S та 55% Hb А1. Яка форма анемії у пацієнта?

alpha-таласемія

Глюкозо-6-фосфатдегідрогеназодефіцитна

Мікросфероцитарна

Хвороба Аддісон-Бірмера

Серпоподібноклітинна анемія

5756 / 6307
У пацієнта з бронхіальною астмою за допомогою шкірних алергічних проб установлено сенсибілізацію алергеном тополиного пуху. Який фактор імунної системи відіграє вирішальну роль у розвитку цього імунопатологічного стану?

Сенсибілізовані Т-лімфоцити

IgD

IgE

IgG

IgM

5757 / 6307
У здорових батьків народився син з фенілкетонурією, але завдяки спеціалізованій дієтотерапії розвивався нормально. З якою формою мінливості пов'язане його одужання?

Комбінативна

Мутаційна

Модифікаційна

Генотипова

Соматична

5758 / 6307
При перевірці крові донорів на станції переливання крові в сироватці одного з них виявлені антитіла до вірусу імунодефіциту людини. Який метод рекомендується для підтвердження діагнозу ВІЛ-інфекції?

Імуноферментний аналіз

Радіоімунний аналіз

Вестерн-блот (імуноблотинг)

Електронна мікроскопія

Імунофлюоресценція

5759 / 6307
Під час ректороманоскопії у дитини 10 років слизові оболонки прямої та сигмовидної кишок набряклі, червоного кольору, вкриті товстим шаром слизу. Про яку патологію свідчать зазначені зміни?

Гнійне запалення

Венозне повнокрів'я

Геморагічне запалення

Катаральне запалення

Синець

5760 / 6307
Людина вживає надмірну кількість вуглеводів. Який метаболічний процес активується в організмі при цьому?

Ліпогенез

Глікогеноліз

Сечовиноутворення

Синтез нуклеїнових кислот

Ліполіз

5761 / 6307
У хворої 45-ти років напад миготливої аритмії. Хворіє на гіпертонічну хворобу II ст. Препарат вибору для припинення нападу:

Анаприлін

Строфантин

Калію хлорид

Лідокаїн

Сустак-форте

5762 / 6307
Генний апарат людини містить близько 30 тисяч генів, а кількість варіантів антитіл сягає мільйонів. Який механізм використовується для утворення нових генів, що відповідають за синтез такої кількості антитіл?

Репарація ДНК

Реплікація ДНК

Рекомбінація генів

Утворення фрагментів Оказакі

Ампліфікація генів

5763 / 6307
У пацієнта з ознаками недостатності мітрального клапана в анамнезі відзначалися напади ревматизму, які супроводжувалися запальними явищами в суглобах. Яке з патологічних явищ у цього пацієнта належить до категорії патологічний стан?

Запалення суглобів

Ревматизм

Недостатність мітрального клапана

Артрит

Ревмокардит

5764 / 6307
Пацієнт віком 15 років лікується з приводу важкої гіпербілірубінемії. У комплексі лікарських препаратів застосовують барбітурати. Синтез якої речовини вони індукують у печінці?

Вердоглобіну

Непрямого гемоглобіну

Білівердину

УДФ-глюкуронілтрансферази

Гемоксигенази

5765 / 6307
Після операції з видалення щитоподібної залози у пацієнта з'явилось заніміння кінцівок, лабораторно діагностовано гіпокальціємію. Який гормональний препарат слід призначити?

Кальцитрин

Тиреоїдин

Трийодтиронін

Тироксин

Паратиреоїдин

5766 / 6307
У пацієнта після катетеризації сечового міхура в загальному аналізі сечі з'явилася велика кількість свіжих еритроцитів, що ймовірно, пов'язано з пошкодженням найвужчої частини сечовипускного каналу. Яка частина сечівника пошкоджена?

Ближча

Перетинчаста

Дальша

Передміхурова

Губчаста

5767 / 6307
Відомо, що цей інфекційний збудник може пошкоджувати слизові, викликати запалення внутрішніх органів, сепсис, утворення синьо-зеленого гною, а також він доволі стійкий до більшості антибіотиків. Укажіть цей збудник.

Escherichia coli

Staphylococcus aureus

Streptococcus mutants

Pseudomonas aeruginosa

Proteus vulgaris

5768 / 6307
У хворого спостерігається алергічна реакція, яка супроводжується свербінням, набряками та почервонінням шкіри. Концентрація якого біогенного аміну підвищилася у тканинах?

Гамма-аміномасляної кислоти

Серотоніну

Дофаміну

Триптаміну

Гістаміну

5769 / 6307
У людини внаслідок тривалого голодування швидкість клубочкової фільтрації зросла на 20%. Найімовірнішю причиною змін фільтрації в таких умовах є:

Збільшення проникності ниркового фільтру

Зменшення онкотичного тиску плазми крові

Збільшення коефіцієнта фільтрації

Збільшення ниркового плазмотоку

Збільшення системного артеріального тиску

5770 / 6307
Пацієнту з діагнозом: туберкульоз, призначили етіотропне лікування. Який препарат доцільно вибрати з групи антибіотиків у цьому разі?

Тетрациклін

Біцилін

Рифампіцин

Цефалексин

Левоміцетин

5771 / 6307
Хірург виявив у чоловіка болючість у правій пахвинній ділянці. Захворювання яких органів має місце?

Пряма кишка і сліпа кишка

Дуоденум, тонка кишка

Підшлункова залоза і дуоденум

Апендикс, сліпа кишка

Висхідна ободова кишка, права нирка

5772 / 6307
Відомо, що не всі сенсорні сигнали сприймаються свідомо. У такий спосіб мозок відокремлює важливу інформацію від менш важливої. Який із відділів головного мозку відіграє найбільш значущу роль у цьому процесі?

Базальні ганглії

Гіпоталамус

Мозочок

Таламус

Кора великих півкуль

5773 / 6307
Чоловік, у якого підозрюють черевний тиф, надійшов до інфекційної лікарні на 3-й день захворювання. Який метод мікробіологічної діагностики варто використати для постановки діагнозу?

Метод виділення гемокультури

Метод виділення копрокультури

Метод виділення білікультури

Метод виділення урінокультури

Метод виділення збудника з ліквора

5774 / 6307
Під час експерименту, внаслідок уведення тварині синтетичного аналога тиреоїдних гормонів, збільшилася частота серцевих скорочень, що опосередковано:

Симпатичною нервовою системою

Соматичною нервовою системою

Парасимпатичною нервовою системою

Метасимпатичною нервовою системою

5775 / 6307
У травматологічне відділення поступив чоловік 35-ти років з травмою лівої кисті. При огляді встановлено: різана рана долонної поверхні лівої кисті; середні фаланги II - V пальців не згинаються. Які м'язи пошкоджені?

Червоподібні м'язи

Тильні міжкісткові м'язи

Долонні міжкісткові м'язи

Поверхневий м'яз-згинач пальців

Глибокий м'яз-згинач пальців

5776 / 6307
Головним у специфічному лікуванні анаеробних інфекцій є своєчасне введення сироватки, що містить специфічні антитіла. На нейтралізацію якого токсину спрямована дія сироватки?

Анаеробних бактерій

Антитоксину

Екзотоксину

Ентеротоксину

Анатоксину

5777 / 6307
У людини масою тіла 80 кг після тривалого фізичного навантаження об'єм циркулюючої крові знижений до 5,4 л, гематокрит - 50%, загальний білок крові - 80 г/л. Такі показники крові є наслідком, перш за все:

Втрати води з потом

Збільшення вмісту білків в плазмі

Збільшення кількості еритроцитів

Збільшення об’єму циркулюючої крові

Збільшення діурезу

5778 / 6307
Під час дослідження крові пацієнта виявлено значне збільшення активності МВ-фракції КФК (креатинфосфокінази) та ЛДГ-1. Про виникнення якої патології це може свідчити?

Панкреатиту

Ревматизму

Холециститу

Гепатиту

Інфаркту міокарда

5779 / 6307
Після порушення кровопостачання головного мозку пацієнт утратив здатність до написання літер і цифр. В якій частині мозку виникла патологія?

Lobus frontalis

Lobus occipitalis

Lobus parietalis

Insula

Lobus temporalis

5780 / 6307
Які біологічно активні речовини утворюються під час утилізації арахідонової кислоти циклооксигеназним шляхом.

Соматомедини

Інсуліноподібні фактори росту

Простагландини

Біогенні аміни

Тироксин

5781 / 6307
Стан вагітної жінки ускладнився токсикозом. Під час лабораторного обстеження виявлено кетонурію. Яка речовина з'явилася в сечі пацієнтки?

Ацетоацетат

Лактат

Урати

Креатинін

Піруват

5782 / 6307
У молодої жінки раптово виникли кашель та спазм бронхів, коли вона увійшла в приміщення з високою концентрацією тютюнового диму. Які рецептори викликали цей захисний рефлекс?

Рецептори плеври

Центральні хеморецептори

Ірритантні рецептори

Механорецептори легень

Юкстамедулярні рецептори

5783 / 6307
Унаслідок безконтрольного прийому вітамінного препарату в дитини виникли анорексія, нудота, блювання, пронос, гіпертермія, з'явилися крововиливи на шкірі та слизових, явища менінгізму. Який препарат приймала дитина?

Нікотинамід

Ретинолу ацетат

Токоферолу ацетат

Ціанокобаламін

Тіамін

5784 / 6307
Хворій 27-ми років проведена секторна резекція тканини молочної залози. При макроскопічному дослідженні виявлений чітко обмежений вузол білого кольору діа-метром 4 см, щільної консистенції. При терміновому гістологічному дослідженні пухлина складається з великої кількості фіброзної строми з її розростанням навкруги дрібних канальців, епітелій канальців лежить на базальній мембрані, зберігає полярність. Поставте діагноз:

Саркома

Рак

Періканалікулярна фіброаденома

Дисгормональні розлади

Аденокарцинома

5785 / 6307
Унаслідок травми мозку у жінки порушена функція епіфізу. Які з наведених функції будуть порушені у цієї жінки?

Частота серцевих скорочень

Частота дихання

Менструальний цикл

Серцевий цикл

Цикл сон-неспання

5786 / 6307
При взаємодії ацетилхоліну з М-холінорецепторами скоротливих кардіоміоцитів утворюється біологічно активна речовина, яка зумовлює інактивацію кальцієвих каналів, зменшення входу іонів кальцію в кардіоміоцит і розвиток негативного інотропного ефекту. Назвіть цю речовину:

Циклічний аденозиндифосфат (цАДФ)

Циклічний аденозинмонофосфат (цАМФ)

Циклічний аденозинтрифосфат (цАТФ)

Циклічний гуанозинмонофосфат (цГМФ)

Циклічний гуанозинтрифосфат (цГТФ)

5787 / 6307
Сечокам'яна хвороба ускладнилися виходом камінця з нирки. На якому рівні сечовода, найімовірніше, він може зупинитися?

На 2 см вище впадіння в сечовий міхур

На 5 см вище тазової частини

В мисці

В середній черевній частині

На межі черевної та тазової частин

5788 / 6307
У пацієнта після хронічної пневмонії розвинувся фіброз легень. Який показник легеневої вентиляції зміниться найбільше?

Збільшиться функціональна залишкова ємність легень

Збільшиться резервний об'єм вдиху

Збільшиться залишковий об'єм

Збільшиться резервний об'єм видиху

Зменшиться життєва ємність легень

5789 / 6307
Під час функціонального навантаження на велотренажері у досліджуваного збільшилася частота дихання. Що є основною причиною зміни діяльності дихального центру в цьому випадку?

Зростання кількості адреналіну в крові

Підвищення напруги O_2 у крові

Зниження напруги CO_2 у крові

Підвищення напруги CO_2 у крові

Зниження напруги O_2 у крові

5790 / 6307
Під час емоційного перевантаження у пацієнтки віком 30 років частота серцевих скорочень сягнула 112/хв. Яка структура провідної системи серця спричинила цей стан?

Пучок Гіса

Волокна Пуркіньє

Гілки пучка Гіса

Внутрішньошлуночковий вузол

Синоатріальний вузол

5791 / 6307
Пацієнт скаржиться на головний біль, утруднене дихання. За результатами рентгенологічного дослідження встановлено діагноз: фронтит (запалення лобової пазухи). В якому носовому ході під час огляду порожнини носа спостерігатимуться гнійні виділення?

Над верхньою носовою раковиною

Верхньому

Загальному

Середньому

Нижньому

5792 / 6307
У пацієнта перебіг геморагічного шоку ускладнився розвитком гострої ниркової недостатності. Укажіть провідну ланку в механізмі розвитку цього ускладненя?

Розвиток ДВЗ-синдрому

Централізація кровообігу із виникненням ішемії нирок

Активація симпатоадреналової системи

Викид у кров вазопресину

Підвищення проникності стінки капілярів

5793 / 6307
В сім'ї з двома дітьми у дитини до року на фоні підвищеної температури, виник напад спастичного кашлю. Подібна картина спостерігалася в старшої дитини дошкільного віку місяць тому. Лікар запідозрив кашлюкову інфекцію. Яким методом можна провести ретроспективну діагностику цього захворювання?

Мікроскопічний

Молекулярно-біологічний

Серологічний

Бактеріологічний

Біологічний

5794 / 6307
В ділянці хромосоми гени розташовані в такій послідовності: ABCDEFG. В результаті дії радіоактивного випромінювання відбулася перебудова, після чого ділянка хромосоми має наступний вигляд: ABDEFG. Яка мутація відбулася?

Інсерція

Дуплікація

Мутація

Делеція

Інверсія

5795 / 6307
У хворого порушений зір у бічних половинах полів зору обох очей (бітемпоральна геміанопсія). Яка нервова структура уражена?

Зорові нерви

Сітківка ока

Зорове перехрестя

Правий зоровий тракт

Лівий зоровий тракт

5796 / 6307
При визначенні групи крові за системою АВ0 аглютинацію еритроцитів досліджуваної крові викликали стандартні сироватки I та II груп і не викликала - III групи. Які аглютиногени містяться в цих еритроцитах?

А

С

А та В

D та C

В

5797 / 6307
Під час експерименту на нервово-м'язовому препараті жаби вивчають одиночні скорочення м'яза у відповідь на електричну стимуляцію нерва. Як зміниться скорочення м'яза після обробки препарату курареподібною речовиною?

Не зміняться

Збільшиться сила

Збільшиться тривалість

Зменшиться тривалість

Зникнуть

5798 / 6307
У чоловіка спостерігається прогресивна м'язова дистрофія. Назвіть показник обміну азоту сечі, характерний для такого стану.

Сечовина

Сечова кислота

Амонійні солі

Креатинін

Креатин

5799 / 6307
Пацієнт віком 45 років з діагнозом: гострий психоз, протягом місяця проходив терапію. Стан пацієнта поліпшився, але з'явилися ригідність м'язів, тремтіння рук, гіпокінезія. Який лікарський засіб викликає такі побічні реакції?

Дифенін

Діазепам

Сиднокарб

Аміназин

Хлордіазепоксид

5800 / 6307
До лікаря-офтальмолога звернулась жінка зі скаргами на погіршення зору. При обстеженні було встановлено порушення процесу акомодації. Функція якої анатомічної структури порушена у хворої?

M. ciliaris

M. sphincter pupillae

Lig. pectinatum iridis

Corpus vitreum

M. dilatator pupillae

5801 / 6307
У біоптаті потовщеної слизової оболонки носа жінки віком 29 років, яка скаржиться на утруднення носового дихання, виявлено скупчення плазматичних і епітеліоїдних клітин та лімфоцитів, серед яких багато гіалінових куль (Русселівські тільця) та великих макрофагів зі світлою цитоплазмою (клітини Мікуліча). Яке запалення розвинулося в слизовій оболонці носа пацієнтки?

Змішане

Гранулематозне

Проміжне

З утворенням поліпів і гострокінцевих кондилом

Ексудативне

5802 / 6307
Під час операції для міорелаксації пацієнта використовують курареподібні фармакологічні препарати. Який механізм їх дії?

Блокування проведення збудження нервовими волокнами

Блокування виділення ацетилхоліну з пресинаптичного відділу

Блокування виділення норадреналіну з пресинаптичного відділу

Блокування Н-холінорецепторів скелетних м'язів

Блокування М-холінорецепторів гладких м'язів

5803 / 6307
Під час лікування зуба жінці віком 30 років зробили ін'єкцію препарату, після якого вона втратила больову чутливість на декілька годин. Який механізм дії знеболювального препарату у цьому випадку?

Блокування натрієвих каналів нервових волокон

Блокування кальцієвих каналів нервових волокон

Підвищення калієвої проникності мембрани нервових волокон

Підвищення натрієвої проникності мембрани нервових волокон

5804 / 6307
У пацієнта виявлено гіперкаліємію та гіпонатріємію. Знижена секреція якого гормону може спричинити такі зміни?

Натрійуретичний

Кортизол

Вазопресин

Альдостерон

Паратгормон

5805 / 6307
Півторарічний хлопчик постійно хворіє на піодермію та тричі хворів на пневмонію. У крові виявлено: знижена кількість імуноглобулінів G та A, відсутні плазмоцити. Який вид імунодефіциту виник у дитини?

Гіпоплазія вилочкової залози

Синдром Луї-Бар

Швейцарський тип

Синдром Віскотта-Олдрича

Гіпогаммаглобулінемія Брутона

5806 / 6307
Для окислення жирних кислот необхідна транспортна система, що включає аміноспирт, який транспортує жирні кислоти крізь мембрану мітохондрій. Назвіть цю сполуку:

Карнітин

Кардіоліпін

Карбомоїлфосфат

Креатинін

Карнозин

5807 / 6307
Чоловіка 49-ти років доставили з місця автомобільної аварії в лікарню в непритомному стані. Шкірні покриви бліді, пульс частий і поверхневий. Переломів кісток і пошкодження головного мозку не виявлено. При пункції черевної порожнини отримано значну кількість крові. Первинною причиною тяжкого стану потерпілого є:

Гіпонатріємія

Еритропенія

Гіповолемія

Гіпопротеїнемія

5808 / 6307
Після зіткнення двох автомобілів у одного з водіїв спостерігається деформація в середній третині лівої гомілки, сильний біль, особливо у разі спроби рухати лівою гомілкою. З рани виступають кінці кістки із тригранним перерізом, посилюється крововтрата. Пошкодження якої кістки є найімовірнішим?

Надп'яткової кістки

Малогомілкової кістки

Стегнової кістки

Великогомілкової кістки

5809 / 6307
Лікар припускає, що пацієнт хворий на дифтерію. Під час бактеріоскопічного дослідження мазка із зіву, виявлено паличкоподібні бактерії із зернами волютину. Який етіотропний засіб є препаратом вибору у цьому разі?

Протидифтерійна антитоксична сироватка

Бактеріофаг

Еубіотик

Інтерферон

5810 / 6307
У пологове відділення надійшла жінка 26-ти років, у якої за терміном вагітності (40 тижнів) вже мають розпочатися пологи. При огляді встановлено, що шийка матки розкрита, але скорочення матки відсутнє. Лікар дав засіб гормональної природи для посилення пологової діяльності. Назвіть цей засіб:

Окситоцин

Естрон

Тестостерон

АКТГ

5811 / 6307
Під час гістологічного дослідження видаленого збільшеного шийного лімфатичного вузла встановлено, що структура його стерта, лімфоїдні фолікули відсутні. Картина його одноманітна та представлена великою кількістю лімфобластів, серед яких наявні клітини з патологічним поділом. Інші групи лімфатичних вузлів і кістковий мозок не змінені. Виявлені зміни найбільш характерні для:

Неспецифічного гіперпластичного лімфаденіту

Хронічного лімфолейкозу

Лімфосаркоми

Лімфогранулематозу

Саркоїдозу

5812 / 6307
Лікар-хірург здійснює первинну хірургічну обробку глибокої різаної рани латеральної поверхні колінного суглоба. Яку зв'язку треба зшити лікарю?

Lig. popliteum obliquum

Lig. collaterale fibulare

Lig. popliteum arcuatum

Lig. collaterale tibiale

5813 / 6307
Педіатр при огляді дитини зазначив відставання у фізичному і розумовому розвитку. В аналізі сечі був різко підвищений вміст кетокислоти, яка дає якісну кольорову реакцію з хлорним залізом. Яке порушення обміну речовин було виявлено?

Альбінізм

Алкаптонурія

Фенілкетонурія

Цистинурія

5814 / 6307
У біоптаті слизової оболонки пацієнта, хворого на бронхіальну астму, виявлено значну кількість клітин з численними метахроматичними гранулами. Вкажіть цю клітину.

Фібробласт

Макрофаг

Ретикулоцит

Тканинний базофіл

5815 / 6307
Під час перетворення глюкози в пентозному циклі утворюються фосфати різних моносахаридів. Яка із цих речовин може бути використана для синтезу нуклеїнових кислот?

Пентозо-5-фосфат

Рибулозо-5-фосфат

Еритрозо-4-фосфат

Рибозо-5-фосфат

5816 / 6307
рН артеріальної крові - 7,4; первинної сечі - 7,4; кінцевої сечі - 5,8. Зниження рН кінцевої сечі є наслідком секреції у канальцях нефрона:

Сечовини

Іонів калію

Креатиніну

Іонів водню

5817 / 6307
У клітині штучно блоковано синтез гістонових білків. Яка структура клітини буде пошкоджена?

Ядерний хроматин

Клітинна оболонка

Комплекс Гольджі

Ядерна оболонка

5818 / 6307
Досить часто причиною набутих імунодефіцитів є інфекційне ураження організму, під час якого збудники розмножуються безпосередньо в клітинах імунної системи і руйнують їх. Виберіть серед наведених ті захворювання, за яких має місце вищезгадане:

Ку-гарячка, висипний тиф

Поліомієліт, гепатит А

Дизентерія, холера

Інфекційний мононуклеоз, СНІД

5819 / 6307
У пацієнта діагностовано герпетичний стоматит. Що слід призначити в даному випадку для лікування?

Сульфацил натрію

Ацикловір

Клотримазол

Тетрациклін

5820 / 6307
До лікаря-гастроентеролога звернувся хворий зі скаргами на появу нудоти та печії після прийому їжі, стеаторею. Що може бути причиною такого стану?

Підвищення виділення ліпази

Недостатність амілази

Порушення синтезу фосфоліпази

Недостатність жовчних кислот

5821 / 6307
Хворий надійшов у нефрологічне відділення з діагнозом опущення правої нирки (нефроптоз). Яке положення відносно 12 ребра в нормі займає права нирка?

12 ребро проектується на нижній полюс

12 ребро перетинає нирку у верхній третині

12 ребро перетинає нирку у нижній третині

5822 / 6307
До стоматолога звернулася мати дитини 2-х років зі скаргами на руйнування у неї зубів. При огляді молочні зуби деформовані, уражені карієсом, біля шийки коричнева облямівка. З анамнезу встановлено, що мати під час вагітності приймала антибіотики без контролю лікаря. Вкажіть, яку групу антибіотиків, що володіє найбільш вираженою тератогенною дією, могла приймати мати?

Тетрацикліни

Цефалоспорини

Пеніциліни

Макроліди

5823 / 6307
Пацієнтку віком 48 років шпиталізовано до лікарні зі скаргами на слабкість, дратівливість, порушення сну. Об'єктивно спостерігається: шкіра та склери жовтого кольору. Під час лабараторного дослідження встановлено: в крові - холемія; випорожнення - ахолічні; сеча - темного кольору (білірубін). Яка жовтяниця спостерігається у пацієнтки?

Синдром Кріглера-Найяра

Гемолітична

Механічна

Синдром Жільбера

5824 / 6307
Пацієнтам з ішемічною хворобою серця призначають невеликі дози аспірину, який інгібує синтез активатора агрегації тромбоцитів тромбоксану А2. З якої речовини утворюється тромбоксан А2?

Арахідонова кислота

Малонова кислота

Оцтова кислота

Глутамінова кислота

5825 / 6307
У неврологічне відділення поступив хворий зі скаргами на погіршення пам'яті та розумової працездатності після перенесеної травми голови. Запропонуйте лікарський засіб для покращення метаболізму головного мозку:

Пірацетам (ноотропіл)

Кофеїн

Анальгін

Сиднокарб

Меридил

5826 / 6307
У клітинах мозку лисиці, спійманої у межах міста, виявлені включення у вигляді тілець Бабеша-Негрі. Джерелом якого захворювання була тварина?

Сказ

Кліщовий енцефаліт

Вітряна віспа

Інфекційний мононуклеоз

5827 / 6307
До біорегуляторів клітинних функцій ліпідної природи належать тромбоксани. Що є джерелом для синтезу цих сполук?

Фосфатидна кислота

Пальмітинова кислота

Пальмітоолеїнова кислота

Арахідонова кислота

5828 / 6307
Пацієнта шпиталізовано із попереднім діагнозом: дифілоботріоз. Уживання яких продуктів могло спричинити це захворювання?

Молока і яєць

Овочів і фруктів

Яловичини

Свинини

Риби

5829 / 6307
Під час гострого тромбозу показана антикоагулянтна терапія. Назвіть антикоагулянт прямої дії, який застосовується в разі загрози тромбозу.

Варфарин

Фраксипарин

Гепарин

Дипіридамол

5830 / 6307
До клініки надійшла дитина 4 років з ознаками тривалого білкового голодування: затримка росту, анемія, набряки, розумова відсталість. Виберіть причину розвитку набряків у дитини:

Зниження синтезу глобулінів

Зниження синтезу альбумінів

Зниження синтезу гемоглобіну

Зниження синтезу ліпопротеїнів

5831 / 6307
Піддослідному собаці ввели гормон, що призвело до збільшення швидкості клубочкової фільтрації за рахунок розширення приносної артеріоли і зменшення реабсорбції іонів натрію і води в канальцях нефрона. Який гормон було введено?

Тироксин

Вазопресин

Альдостерон

Передсердний натрійуретичний гормон

5832 / 6307
У разі нестачі вітаміну А у людини порушується сутінковий зір. Укажіть клітини, яким належить ця рецепторна функція.

Біполярні нейрони

Паличкові нейросенсорні клітини

Горизонтальні нейроцити

Гангліонарні нервові клітини

Колбочкові нейросенсорні клітини

5833 / 6307
Під час огляду пацієнта хірург установив наявність пораанення верхньої третини нирки. Цілісність якого органу треба перевірити в цьому разі, зважаючи на синтопію лівої нирки?

Низхідної ободової кишки

Поперечної ободової кишки

Шлунка

Печінки

5834 / 6307
Під час хірургічного лікування стегнової грижі лікар виділяє поверхневий отвір стегнового каналу. Яка анатомічна структура його утворює?

Arcus iliopectineus

Septum femorale

Hiatus saphenus

Fossa femoralis

5835 / 6307
Обстежено хворого чоловiка, госпіталізованого на 5-й день хвороби з явищами жовтяниці, болем в м'язах, ознобом, носовими кровотечами. При проведені лабораторної діагностики бактеріолог виконав темнопольну мікроскопію краплини крові хворого. Назвіть збудників хвороби:

Leptospira interrogans

Bartonella bacilloformis

Rickettsia mooseri

Calymmatobacterium granulomatis

5836 / 6307
Зі слизових оболонок і харкотиння хворого, який тривалий час приймав імунодепресанти, були виділені великі грампозитивні овальні клітини з брунькуванням, розташовані хаотично, та довгасті клітини, розташовані ланцюжками. Який збудник виділений?

Кандиди

Стрептококи

Стрептобактерії

Актиноміцети

5837 / 6307
Унаслідок закупорки загальної жовчної протоки (установлено рентгенологічно) надходження жовчі до дванадцятипалої кишки припинилося. Слід очікувати на порушення:

Секреції соляної кислоти в шлунку

Гальмування слиновиділення

Емульгування жирів

Гідролізу вуглеводів

5838 / 6307
В гістологічному препараті стінки очного яблука визначається структура, в якій відсутні кровоносні судини. Яке утворення характеризується даною морфологічною ознакою?

Циліарне тіло

Сітківка

Рогівка

Судинна оболонка

5839 / 6307
У дитини діагностовано атипову пневмонію, яка не піддається лікуванню beta-лактамними антибіотиками. Після посіву мокротиння дитини на спеціальне середовище, отримано ріст мікроорганізмів, які утворюють мікроскопічні колонії зі щільним центром. Який мікроорганізм є збудником пневмонії у дитини?

Streptococcus pneumoniae

Mycoplasma pneumoniae

Klebsiella pneumoniae

Legionella pneumophila

5840 / 6307
Дівчина, хвора на цукровий діабет, чекає на донорську нирку. Яке ускладнення діабету є в неї причиною хронічної ниркової недостатності?

Ретинопатія

Нейропатія

Макроангіопатія

Мікроангіопатія

5841 / 6307
У пацієнтки виявлено пухлину вентральної поверхні мосту. У якій артерії спостерігатиметься уповільнення кровотоку?

A) cerebri anterior

A) carotis interna

A) cerebri media

A) basilaris

5842 / 6307
Оглядаючи пацієнта лікар-невропатолог виявив відсутність колінного рефлексу під час постукування по зв’язці наколінка. Який нерв ймовірно пошкоджений?

Стегновий

Загальний малогомілковий

Затульний

Сідничний

5843 / 6307
Численні ефекти гормону росту здійснюються за допомогою білкових факторів, що утворюються в печінці під впливом соматотропіну. Як називаються ці фактори?

Ендорфіни

Соматомедини

Ліпотропіни

Атріопептини

5844 / 6307
У хворого з пiдозрою на черевний тиф упродовж двох тижнiв захворювання лабораторний дiагноз не був встановлений. Який матерiал треба направити до лабораторiі для бактерiологiчного дослiдження на третьому тижнi?

Фекалії та сечу

Слиз із носу

Промивні води шлунка

Слиз із зiву

Харкотиння

5845 / 6307
Чоловік 38 років надійшов у терапевтичне відділення з діагнозом: правосторонній ексудативний плеврит. Рідина, відкачана з плевральної порожнини грудної клітки прозора, має відносну густину 1,020, містить 55 г/л білка, альбуміно-глобуліновий коефіцієнт - 1,6; загальна кількість клітин - 2,8/мкл; pH - 6,5. Який тип ексудату наявний у хворого?

Фібринозний

Серозний

Гнійний

Геморагічний

5846 / 6307
В кардіологічне відділення поступив хворий з аритмією. Який протиаритмічний препарат доцільно призначити?

Дротаверину гідрохлорид

Фурацилін

Диклофенак-натрію

Аміодарон

5847 / 6307
Збудник гепатиту D (дельта-агент) є дефектним вірусом, який може реплікуватися лише в клітинах, що вже інфіковані одним із нижченаведених вірусів. Укажіть цей вірус.

Епштейна-Барр

Гепатиту Е

Гепатиту А

Гепатиту В

5848 / 6307
Під час огляду пацієнта віком 32 роки спостерігається диспропорційна будова скелету, збільшення надбрівних дуг, носа, губ, язика, щелепних кісток і стоп. Яка ймовірна причина розвитку цих порушень?

Збільшення рівня соматотропного гормону

Збільшення рівня тироксину

Збільшення концентрації глюкагону

Зниження концентрації інсуліну

Збільшення рівня катехоламінів

5849 / 6307
У пацієнта виявлено високий рівень альдостерону в крові. Яка з фізіологічно активних речовин найімовірніше призвела до цього?

Натрійуретичний фактор

Ангіотензин ІІ

цАМФ

цГМФ

5850 / 6307
У хворого перед хірургічним втручанням проводять визначення групи крові за системою АВ0 з використанням моноклональних антитіл до групових антигенів. Аглютинація не спостерігалася ні з анти-А, ні з анти-В реагентами. До якої групи належить досліджувана кров?

B(III)

A1(II)

A2(II)

0(I)

5851 / 6307
Під час огляду дитини у віці 11 місяців педіатр виявив викривлення кісток нижніх кінцівок та затримку мінералізації кісток черепа. Нестача якої речовини призводить до цієї патології?

Пантотенової кислоти

Біофлавоноїдів

Холекальциферолу

Рибофлавіну

Тіаміну

5852 / 6307
Юнак 15-ти років після переохолодження був доставлений в лікарню зі скаргами на біль, озноб. При огляді ''гусяча шкіра'', блідість, температура тіла поступово підвищується. Яка стадія гарячки спостерігається у хворого?

St. incrementi

St. inflamenti

St. absorbi

St. decrementi

5853 / 6307
У чоловіка, хворого на тимому (пухлину тимусу), спостерігаються ціаноз, розширення підшкірної венозної сітки і набряк м'яких тканин обличчя, шиї, верхньої половини тулуба і верхніх кінцівок. Який венозний стовбур перетиснено пухлиною?

Передня яремна вена

Підключична вена

Верхня порожниста вена

Внутрішня яремна вена

5854 / 6307
Жінка 38 років скаржиться на постійну спрагу, часте сечовиділення, зниження апетиту, головний біль. Сеча безбарвна, прозора, слабокислої реакції, не містить глюкози. Добовий діурез до 12 л. Нестача якого гормона може бути причиною цього стану?

Вазопресину

Норадреналіну

Інсуліну

Передсердного натрійуретичного фактору

5855 / 6307
За результатами аналізу ЕКГ треба визначити водія ритму серця. На підставі якого показника це можна зробити?

Напрямок зубця Q

Ампітуду зубця R

Напрямок зубця P

Ампітуду зубця P

5856 / 6307
У чоловіка 58 років є ознаки атеросклеротичного ураження серцево-судинної системи. Збільшення якого з наведених показників біохімічного аналізу крові найхарактерніше для цього стану?

Активності аланінмінотрансферази

Рівня ЛПНЩ

Рівня ЛПВЩ

Хіломікронів

Активності сукцинатдегідрогенази

5857 / 6307
Пацієнт екстрено звернувся до лікаря із приводу численних укусів, отриманих від собаки. Який матеріал треба направити до мікробіологічної лабораторії для дослідження та встановлення діагнозу: сказ?

Парні сироватки пацієнта

Мазок-відбиток з рогівки ока пацієнта

Мозок тварини, яка покусала пацієнта

Спинномозкову рідину пацієнта

5858 / 6307
На гістологічному препараті видно судину, стінка якої складається з ендотелію, базальної мембрани та пухкої сполучної тканини. Назвати тип судини:

Гемокапіляр

Вена безм'язового типу

Вена м'язового типу

Артерія

Лімфокапіляр

5859 / 6307
Пацієнту з діагнозом: артеріальна гіпертензія з явищами стенокардії, призначили антиангінальний препарат із групи антагоністів кальцію. Вкажіть цей препарат.

Амлодипін

Метопролол

Молсидомін

Пентоксифілін

Анаприлін

5860 / 6307
У хворого з ІХС виникли порушення серцевого ритму, збільшився рівень глюкози у крові. Поруч з антиангінальними засобами, лікар призначив вітамінний препарат. Який з вітамінних засобів має кардіотрофічний та гіпоглікемічний ефект?

Ергокальциферол

Рибофлавін

Тіамін

Ціанокобаламін

Ретинол

5861 / 6307
Пацієнт тривалий час застосовував великі дози сульфаніламідних препаратів. Нещодавно в нього з'явилися: значна задишка в спокої, слабкість, втрата апетиту, порушення сну. Лабораторне дослідження крові виявило наявність в крові метгемоглобіну. Який механізм інактивації гемоглобіну в еритроцитах відбувається в пацієнта?

Блокада окисних ферментних систем

Пошкодження білка в гемоглобіні

Сполучення гемоглобіну з сульфаніламідами

Блокада відновних ферментних систем

Окислення заліза в гемоглобіні

5862 / 6307
На штучному вигодовуванні перебуває дитина 9 місяців. Для вигодовування використовували суміші, незбалансовані за вмістом вітаміну B_6. У дитини спостерігаються судоми, причиною яких може бути порушення утворення:

ГАМК

Бета-аланіну

Дофаміну

Гістаміну

Серотоніну

5863 / 6307
Терапія анаприліном позитивно вплинула на динаміку хвороби у жінки 44 років, яка страждає на стенокардію. Який механізм дії цього препарату?

Зменшення енерговитрат міокарда внаслідок зниження навантаження

Блокада бета-адренорецепторів і зниження потреби міокарда в кисні

Зменшення окислювального обміну в міокарді внаслідок блокади ферментів циклу Кребса

Збільшення надходження кисню в міокард

Зниження потреби і збільшення надходження кисню в міокард

5864 / 6307
Під час розтину тіла пацієнта, померлого від розповсюдженого перитоніту, в дистальних відділах тонкої кишки виявлено численні виразки овальної форми, які розташовані вздовж кишки. Дно виразок чисте, гладеньке, утворене м'язовою або серозною оболонкою, краї виразок рівні, заокруглені. У двох виразках є перфоративні отвори діаметром до 0,5 см. Яке захворювання можна запідозрити?

Туберкульоз

Холера

Дизентерія

Висипний тиф

Черевний тиф

5865 / 6307
Піддослідній тварині ввели блокатор цитохромоксидази, що призвело до її миттєвої смерті. Яка сполука калію може викликати таку реакцію?

Цианід

Нітрит

Сульфат

Оксалат

Фосфат

5866 / 6307
Пацієнту встановлено попередній діагноз: токсоплазмоз. Який біологічний матеріал використано для діагностики цього захворювання?

Сечу

Дуодентальний вміст

Харкотиння

Фекалії

Кров

5867 / 6307
При анемії в периферичній крові визначаються дегенеративні і регенеративні форми еритроцитів. Які з наведених форм еритроцитів є регенеративними?

Пойкілоцити

Сфероцити

Мікроцити

Гіперхромні еритроцити

Ретикулоцити

5868 / 6307
Жінка 33-х років, яка тривалий час лікується з приводу хронічного поліартриту, скаржиться на підвищення артеріального тиску, зміни розподілу жирової тканини, порушення менструального циклу. Який препарат приймає хвора?

Бутадіон

Індометацин

Преднізолон

Синафлан

Беклометазон

5869 / 6307
Під час ендоскопічного дослідження лікар виявив порушення цілісності стінки шлунка в межах слизової оболонки. Вкажіть, яким типом епітелію в нормі вистелена з середини стінку шлунка:

Багатошаровий плоский зроговілий

Одношаровий призматичний залозистий

Псевдобагатошаровий

Багатошаровий плоский незроговілий

Перехідний

5870 / 6307
На заняттях з лікувальної фізкультури лікар-фізіотерапевт запропонував юнакам відхилитися назад і дістати долонями підлогу. Яка зв'язка запобігає надмірному розгинанню хребтового стовпа?

Передня поздовжня

Жовта

Задня поздовжня

Міжпоперечна

Надостьова

5871 / 6307
Пацієнт віком 49 років має підвищену концентрацію сечової кислоти в крові. Для зниження рівня сечової кислоти лікар призначив алопуринол. Конкурентним інгібітором якого ферменту є алопуринол?

Ксантиноксидази

Аденінфосфорибозилтрансферази

Гуаніндезамінази

Аденозиндезамінази

Гіпоксантинфорибозилтрансферази

5872 / 6307
Відомо, що вірус гепатиту D належить до дефектних вірусів і може репродукуватися у клітинах хазяїна лише в присутності іншого вірусу гепатиту. Яку назву має цей вірус?

Вірус гепатиту В

Вірус гепатиту А

Вірус гепатиту С

Вірус гепатиту G

Вірус гепатиту Е

5873 / 6307
Пацієнту, що хворіє на цукровий діабет та алергічний дерматит, лікар призначив фторований гормональний препарат в мазі. На питання хворого про переваги призначеного препарату над маззю гідрокортизону лікар пояснив, що:

Діє короткочасно

Посилює синтез інсуліну

Діє слабше

Призначений препарат практично не має резорбтивної дії

Коштує дешевше

5874 / 6307
Недостатня продукція мінералокортикоїдів (Аддісонова хвороба) супроводжується м'язовою слабкістю, що зумовлена підвищеним виділенням із сечею іонів:

Калію

Магнію

Натрію

Гідрогену

Кальцію

5875 / 6307
Який компонент раціону людини не перетравлюється в шлунково-кишковому тракті, але є необхідною складовою харчування?

Целюлоза

Сахароза

Білок

Ліпіди

Крохмаль

5876 / 6307
У пацієнта вміст глюкози в плазмі крові становить 15 ммоль/л, спостерігаються спрага та поліурія. Дефіцит якого гормону спричиняє такі симптоми?

Інсуліну

Соматотропіну

Кортизолу

Глюкагону

Соматоліберину

5877 / 6307
Як називаються клітини ендокринного комплексу нирки, що знаходяться під ендотелієм у стінці приносної та виносної артеріоли, у цитоплазмі містять гранули реніну, що сприяє підвищенню кров’яного тиску?

Мезангіоцити

Клітини Гурмагтіга

Інтерстиційні клітини

Юкстагломерулярні

Клітини щільної плями

5878 / 6307
У пацієнта після резекції шлунку виникла гіперхромна анемія. Який препарат необхідно призначити?

Фербітол

Коамід

Ціанокобаламін

Фолієву кислоту

Ферум-лек

5879 / 6307
Чоловік з уретритом протягом тижня самостійно лікувався антибіотиком пеніцилінового ряду, але це не покращило його стан. Проведене бактеріологічне дослідження показало, що збудником захворювання є мікоплазми. Чому препарат, який приймав пацієнт, виявився неефективним?

Збудник розмножується всередині клітин

Мікоплазми не мають клітинної стінки

Мембрана мікоплазм містить холестерин

Мікоплазми утворюють фермент, який руйнує пеніцилін

Мікоплазми не утворюють відповідних транспортних білків

5880 / 6307
Молодому чоловікові було видано позитивну відповідь на наявність у нього антитіл до ВІЛ (використаний ІФА), проте обстежуваний наполягає на дослідженні, яке б достовірно довело наявність у нього антитіл до цього вірусу. Яке дослідження слід провести для підтвердження діагнозу?

РІФ

РПГА

ПЛР

Імуноблотинг

РГГА

5881 / 6307
У людини 40-ка років після емоційного збудження виявили підвищення артеріа-льного тиску. Вкажіть можливу причину цього ефекту:

Зменшення частоти серцевих скорочень

Підвищення тонусу симпатичної нервової системи

Підвищення тонусу парасимпатичної нервової системи

Розширення артеріол

Гіперполяризація кардіоміоцитів

5882 / 6307
У 29-річної породіллі на 3-й день після пологів виник дифузний набряк правої грудної залози, болючість при пальпації, гіперемія шкіри у вказаній ділянці, підвищення температури тіла до 38^oC) При гістологічному дослідженні тканини залози виявлено: в стромі - дифузний клітинний інфільтрат, який складається з великої кількості нейтрофільних лейкоцитів, інтерстиційний набряк, гіперемія судин. Діагностуйте захворювання:

Хронічний продуктивний мастит

Хронічний гнійний мастит

Гострий апостематозний мастит

Гострий серозний мастит

Гострий флегмонозний мастит

5883 / 6307
У пацієнта з гіперфункцією щитоподібної залози інтенсивність енергетич-ного обміну підвищена, але він скаржиться на зниження фізичної сили і працездатності. З чим це пов'язано?

Накопичення кінцевих продуктів обміну в м'язах

Роз'єднання біологічного окиснення та окиснювального фосфорилювання

Збільшення вмісту АМФ в м'язах

Збільшення рівня АДФ і H_3PO_4

Серцева недостатність

5884 / 6307
У пацієнтки віком 60 років спостерігається застійна серцева недостатність зі збільшенням переднавантаження на серце. Яку біологічно активну речовину буде секретувати серце у цьому разі?

Шлунковий інгібуючий пептид

Передсердний натрійуретичний пептид

Ангіотензин ІІ

Альдостерон

Вазопресин

5885 / 6307
До лікаря-дерматолога звернулася пацієнтка віком 22 роки зі скаргами на гнійне висипання на обличчі та спині. Із анамнезу відомо, що у неї виявлено H. pylori. Враховуючи таку супутню патологію, лікар призначив антибактеріальний препарат, який буде діяти як на збудників інфекцій м'яких тканин, так і на H. pylori. Який антибактеріальний препарат призначив лікар?

Флуконазол

Кларитроміцин

Озельтамівір

Рифампіцин

Ізоніазид

5886 / 6307
У людини внаслідок травми мозку сталося вимкнення грудного дихання зі збереженням діафрагмального. У разі якої локалізації травми це може відбутися?

На рівні варолієвого моста

На рівні 1-го шийного сегмента спинного мозку

На рівні ретикулярної формації стовбура

Між шийними та грудними сегментами спинного мозку

На рівні 8-го грудного сегмента спинного мозку

5887 / 6307
У хворого на ішемічну хворобу серця виявлено підвищений вміст тригліцеридів і ліпопротеїнів низької щільності в плазмі крові. Який препарат слід призначити?

Аміодарон

Лізиноприл

Фенофібрат

Добутамін

Фамотидин

5888 / 6307
У дитини грудного віку спостерігається пігментація склер, слизових оболонок. Виділяється сеча, яка темніє на повітрі. У крові та сечі виявлено гомогентизинову кислоту. Укажіть, яке захворювання може бути причиною цього стану?

Альбінізм

Гістидинемія

Галактоземія

Цистинурія

Алкаптонурія

5889 / 6307
Генетичний дефект Na^+-залежного транспортера моносахаридів мембрани ентероцитів зумовлює розвиток у новонароджених дітей внаслідок порушення всмоктування - синдрому мальабсорбції. Порушення всмоктування якої речовини спостерігається у цьому випадку?

Сахарози

Лактози

Рибози

Галактози

Мальтози

5890 / 6307
У пацієнта діагностовано абсцес лівої пахвинної ділянки. Скаржиться на загальну слабкість, підвищення температури тіла ввечері до 38,5^oC, що нормалізується вранці, а також на болючість та почервоніння шкіри в ділянці локалізації абсцесу. Укажіть тип температурної кривої у цього пацієнта?

Febris recurrens

Febris hectica

Febris remittens

Febris continua

Febris acontinua

5891 / 6307
Унаслідок обтурації жовчовивідної протоки у хворого зменшилося надходження жовчі в 12-палу кишку, що призвело до порушення всмоктування:

Жирів

Мінеральних солей

Вуглеводів

Білків

Білків та вуглеводів

5892 / 6307
Після тижневого голодування рівень глюкози крові підтримується на постійному рівні. За рахунок якого процесу це відбувається?

Гліколіз

Фосфороліз глікогену

Глюконеогенез

ЦТК

Глікогеноліз

5893 / 6307
Під час хвороби Іценка-Кушинга (гіперфункція кори наднирників із підвищеною продукцією кортикостероїдів) виникає гіперглікемія. Який процес стимулюється у цьому разі?

Фосфороліз глікогену

Гліколіз

Глюконеогенез

Цикл Кребса

Пентозофосфатний шлях окислення глюкози

5894 / 6307
Чоловік тривалий час лікується з приводу хронічної пневмонії. Під час мікроскопічного дослідження мокротиння в мазках, зафарбованих за методом Ціля-Нільсена, виявлено червоні палички розмірами 0,25х4 мкм, розташовані поодиноко, іноді невеликими скупченнями. Наявність якого захворювання можна припустити?

Пневмококової пневмонії

Грипозної пневмонії

Туберкульозу легень

Актиномікозу легень

Кандидозу легень

5895 / 6307
Під час глікогенозу (хвороби Гірке) порушується перетворення глюкозо-6-фосфату на глюкозу, що призводить до накопичення глікогену в печінці та нирках. Дефіцит якого ферменту є причиною захворювання?

Глікогенсинтетази

Фосфорилази

Альдолази

Гексокінази

Глюкозо-6-фосфатази

5896 / 6307
Пацієнтка звернулася до лікаря зі скаргами на ущільнення у верхньо-бічній ділянці правої молочної залози. Які лімфатичні вузли повинен перевірити лікар, щоб переконатися в нерозповсюдженні патологічного процесу?

Міжреберні

Білягрудинні

Пахвові

Передні середостінні

Верхні діафрагмальні

5897 / 6307
У пацієнта діагностовано стеноз мітрального клапана. До якого патогенетичного типу недостатності серця можна віднести цей стан?

Змішана форма недостатності серця

Недостатність серця від перевантаження опором

Недостатність серця через гіпертрофію міокарда

Недостатність серця через перевантаження об'ємом

Недостатність серця через ушкодження міокарда

5898 / 6307
Пацієнту, що проживає на специфічній геохімічній території, поставлено діагноз ендемічний зоб. Який вид посттрансляційної модифікації тиреоглобуліну порушений в організмі хворого?

Метилування

Глікозилювання

Фосфорилювання

Ацетилування

Йодування

5899 / 6307
Хворому на цукровий діабет медсестра помилково ввела майже подвійну дозу інсуліну, що призвело до гіпоглікемічної коми. Який лікарський засіб необхідно ввести хворому для виведення з коми?

Норадреналін

Соматотропін

Глюкозу

Лідазу

Інсулін

5900 / 6307
Хворий з хронічною серцево-судинною недостатністю приймав дигоксин. Після призначення додаткової терапії розвинулися явища інтоксикації серцевими глікозидами. Який препарат може викликати підсилення інтоксикації серцевими глікозидами?

Калію хлорид

Розчин глюкози

Кальцію хлорид

Магнію хлорид

Аспаркам

5901 / 6307
У разі зниження концентрації Na^+ у плазмі крові в нирках посилюється його реабсорбція. Який основний механізм регуляції стимулює цей процес?

Парасимпатичні рефлекси

Альдостерон

Симпатичні рефлекси

Натрійуретичний гормон

Ренін

5902 / 6307
Рентгенологічно встановлено затемнення на ділянці верхньощелепної пазухи справа, що свідчить про наявність гною. В який носовий хід буде виділятися патологічна рідина?

У правий нижній носовий хід

У правий середній носовий хід

У правий найвищий носовий хід

У правий верхній носовий хід

У правий загальний носовий хід

5903 / 6307
Під час дослідження біоптату виявлена гранульома, що складається з лімфоцитів, плазматичних клітин, макрофагів із пінистою цитоплазмою (клітинами Мікуліча), багато гіалінових куль. Яке захворювання можна припустити?

Актиномікоз

Лепра

Туберкульоз

Риносклерома

Сифіліс

5904 / 6307
До лікаря звернулася жінка віком 25 років зі скаргами на дисменорею та безпліддя. Під час обстеження виявлено: зріст жінки 145 см, недорозвинені вторинні статеві ознаки, на шиї крилоподібні складки. В буккальному епітелії не виявлено тілець Барра. Який патологічний стан спостерігається у пацієнтки?

Синдром Морріса

Синдром Шерешевського-Тернера

Синдром трисомії Х

Синдром Дауна

Синдром Клайнфельтера

5905 / 6307
Основу неорганічної структури зубів складають кристали фосфатів кальцію. Який гормон регулює гомеостаз кальцію?

Тестостерон

Вазопресин

Паратгормон

Альдостерон

Адреналін

5906 / 6307
У новонародженого хлопчика виявлена полідактилія, мікроцефалія, незрощення верхньої губи та верхнього піднебіння, а також вади серця і судин. Під час каріотипування діагностовано синдром Патау. Яка причина цієї патології?

Трисомія 18-ї хромосоми

Моносомія Х

Трисомія 13-ї хромосоми

Трисомія 21-ї хромосоми

Нерозходження статевих хромосом

5907 / 6307
При розтині тіла померлої дитини 1,5 років виявлено: геморагічні висипання на шкірі, помірна гіперемія та набряк слизової носоглотки, дрібні крововиливи в слизових оболонках і внутрішніх органах, різкі дистрофічні зміни в печінці, міокарді, гострий некротичний нефроз, масивні крововиливи в наднирниках. Для якого захворювання найбільш характерні виявлені зміни?

Менінгококова інфекція

Скарлатина

Висипний тиф

Дифтерія

Кір

5908 / 6307
Який біохімічний маркер свідчить про інфікованість крові донора вірусним гепатитом В?

HDV

НВсAg

НВsAg

HAV

HCV

5909 / 6307
Однією з функцій центрального гальмування є відбір і обмеження надходження сенсорної інформації до кори головного мозку. Який вид гальмування забезпечує цю функцію?

Латеральне

Реципрокне

Песимальне

Пресинаптичне

Зворотне

5910 / 6307
Хворому 50-ти років після операції з метою прискорення загоєння рани місцево був призначений препарат, який має репаративну та імуностимулюючу активність. Визначте препарат:

Метилурацил

Меркаптопурин

Діазолін

Циклоспорин

Дексаметазон

5911 / 6307
Під час гістологічного дослідження міокарда було виявлено велику ділянку міокардіоцитів, у яких відсутні ядра. Цитоплазма клітин гомогенна, рожевого кольору. На периферії вогнища визначаються розширені, різко повнокровні судини і виражена інфільтрація сегментоядерними лейкоцитами. Про який процес свідчить ця гістологічна картина?

Демаркаційне запалення навколо інфаркту

Септичний розпад інфаркту

Інкапсуляцію інфаркту

Асептичний автоліз інфаркту

Організацію інфаркту

5912 / 6307
Вірус імунодефіциту людини, маючи на своїй поверхні антигени gp 41 i gp 120, взаємодіє з клітинами-мішенями організму. Виберіть серед перерахованих антигени лімфоцитів людини, з якими комплементарно зв'язується gp 120 вірусу:

CD 4

CD 28

CD 8

CD 3

CD 19

5913 / 6307
У студента через добу після іспиту в аналізі крові виявили лейкоцитоз без істотної зміни в лейкоцитарній формулі. Який механізм найімовірніше зумовив розвиток виявленої зміни в периферичній крові?

Зменшення руйнування лейкоцитів

Уповільнення еміграції лейкоцитів до тканин

Перерозподіл лейкоцитів в організмі

Посилення лейкопоезу

5914 / 6307
Чоловік віком 60 років скаржиться на біль у грудній клітці. У сироватці крові виявлено значне зростання активності ферментів: креатинфосфокінази, її МВ-ізоформи й аспартатамінотрансферази. Визначте, в якій м'язовій тканині розвивається патологічний процес.

У скелетних м'язах

У тканині легень

У тканині печінки

У гладеньких м'язах

У серцевому м'язі

5915 / 6307
Під час гемотрансфузій рекомендується переливати кров лише відповідної групи. Вкажіть, чим зумовлена приналежність до певної групи крові за системою АВ0.

Вуглеводними детермінантами мембран еритроцитів

Білковими детермінантами мембран еритроцитів

Вуглеводними детермінантами мембран лейкоцитів

Білками сироватки крові

Білково-полісахаридними компонентами лейкоцитів

5916 / 6307
З віком у людини розвивається пресбіопія (далекозорість). Причиною розвитку цього стану є:

Помутніння кришталика

Атрофія сітківки

Зменшення еластичності кришталика

Подовження очного яблука

Вкорочення очного яблука

5917 / 6307
Під час розтину тіла дівчинки 9-ти років у верхівці правої легені субплеврально було знайдене вогнище казеозного некрозу діаметром 15 мм, біфуркаційні лімфатичні вузли збільшені, містять дрібні вогнища некрозу коагуляційного типу. Мікроскопічно: у легеневому вогнищі та в лімфатичних вузлах навколо некротичних мас розташовані епітеліоїдні клітини, лімфоцити та поодинокі багатоядерні гігантські клітини. Діагностуйте захворювання:

Вторинний фіброзно-вогнищевий туберкульоз

Гематогенний туберкульоз з переважним ураженням легень

Первинний туберкульоз

Гематогенний генералізований туберкульоз

Вторинний вогнищевий туберкульоз

5918 / 6307
Пацієнтка госпіталізована до пульмонологічного відділення з діагнозом: ексудативний плеврит. В якому синусі плеври скупчуватиметься найбільша кількість запального випоту?

Діафрагмально-середостінний синус

Реберно-діафрагмальний синус

Реберно-середостінний синус

Поперечний синус перикарда

5919 / 6307
У пацієнта з асфіксією після короткочасної зупинки дихання виникли поодинокі рідкі подихи з пасивним видихом, після чого дихання зупинилось зовсім. Який тип дихання спостерігався у пацієнта?

Куссмауля

Апнейстичне

Гаспінг-дихання

Чейна-Стокса

Біота

5920 / 6307
У пацієнта порушений акт жування внаслідок ураження патологічним процесом структур, що утворюють аферентний провідник дуги відповідного рефлексу. Який нерв уражений у пацієнта?

N. vagus

N. hypoglossus

N. glossopharyngeus та n. vagus

N. trigeminus

N. glossopharyngeus

5921 / 6307
У пацієнта перебіг геморагічного шоку ускладнився розвитком гострої ниркової недостатності. Укажіть ініціюючу ланку в механізмі розвитку цього ускладнення.

Викид у кров вазопресину

Активація симпатоадреналової системи

Централізація кровообігу з виникненням ішемії нирок

Підвищення проникності стінки капілярів

Розвиток ДВЗ-синдрому

5922 / 6307
Після руйнування структур ЦНС тварина втратила орієнтувальні рефлекси. Що саме зруйнували?

Чорну речовину

Червоні ядра

Медіальні ретикулярні ядра

Латеральні вестибулярні ядра

Чотиригорбкове тіло

5923 / 6307
Людині, у якої напад бронхоспазму, треба зменшити вплив блукаючого нерва на гладеньку мускулатуру бронхів. Які мембранні циторецептори доцільно заблокувати для цього?

alpha- та beta-адренорецептори

alpha-адренорецептори

H-холінорецептори

beta-адренорецептори

М-холінорецептори

5924 / 6307
Під час обстеження пацієнта виявлено, що АТ - 180/110 мм рт. ст., ЧСС - 95 уд/хв, рентгенологічно визначено звуження однієї з ниркових артерій. Активація якої системи викликала гіпертензивний стан у цього пацієнта?

Імунної

Гемостатичної

Ренін-ангіотензивної

Кінінової

Симпатоадреналової

5925 / 6307
В травматологічне відділення звернувся чоловік 30-ти років з різаною раною підошовної ділянки лівої стопи. У хворого обмежене піднімання латерального краю стопи. Функція якого м'яза імовірніше всього порушена?

Камбалоподібний м'яз

Триголовий м'яз литки

Передній великогомілковий м'яз

Довгий малогомілковий м'яз

Довгий м'яз-згинач великого пальця стопи

5926 / 6307
Через 6 годин після інфаркту міокарда у хворого в крові піднялася активність лактатдегідрогенази. Наявність якого ізоферменту в крові слід чекати у цьому випадку?

ЛДГ4

ЛДГ1

ЛДГ5

ЛДГ2

ЛДГ3

5927 / 6307
На розтині тіла чоловіка, який хворів на фіброзно-кавернозний туберкульоз і помер від ниркової недостатності, виявлені збільшені в розмірах нирки, щільні на дотик, сіруватого кольору, на розрізі мають сальний вигляд. Який діагноз можна припустити?

Амілоїдоз нирок

Хронічний пієлонефрит

Гломерулонефрит

Мієломна нирка

Нефросклероз

5928 / 6307
Під час падіння із висоти пацієнт отримав перелом стегнової кістки. Який препарат треба призначити для зняття больового синдрому?

Парацетамол

Ацетилсаліцилову кислоту

Морфіну гідрохлорид

Пентамін

Димедрол

5929 / 6307
У гепатоцитах чоловіка, який помер від хронічного алкоголізму, виявлено збільшення кількості одномембранних органел, які містять фермент каталазу та беруть участь у процесі детоксикації. Назвіть ці органели.

Рибосоми

Мітохондрії

Лізосоми

Апарат Гольджі

Пероксисоми

5930 / 6307
Для лікування кропив'янки з метою усунення сверблячого висипу на шкірі хворому призначений димедрол. Який механізм забезпечує його ефективність у цьому разі?

Пригнічення вивільнення гістаміну

Незалежний антагонізм з гістаміном

Конкурентна блокада Н1-рецепторів

Інгібіція синтезу гістаміну

Прискорення руйнування гістаміну

5931 / 6307
Під час аутопсії чоловіка віком 54 роки, який помер з клінічними явищами розлитого фібринозно-гнійного перитоніту, в термінальному відділі клубової кишки та початковому відділі товстого кишечника виявлено: слизова оболонка з численними поздовжніми, щілиноподібними виразками та поперечними тріщинами, горбиста, має вигляд 'бруківки'. У деяких ділянках виявлено перфорацію виразок з утворенням внутрішньоочеревинних абсцесів і свищів. Для якого захворювання характерні такі зміни?

Черевний тиф

Псевдомембранозний коліт

Хвороба Менетріє

Неспецифічний виразковий коліт

Хвороба Крона

5932 / 6307
У жінки при профілактичному огляді виявили пухлину молочної залози. Результати біопсії підтвердили наявність злоякісної пухлини. Який основний механізм інфільтративного росту злоякісної пухлини?

Порушення контактного гальмування

Здатність пухлинних клітин до амебоїдного руху

Збільшення адгезивності пухлинних клітин

Підвищення ферментативної активності лізосом

Підвищення активності кейлонів в клітині

5933 / 6307
Чоловік хворіє на ішемічну хворобу серця. Унаслідок цього порушується енергетичне забезпечення серцевого м'яза. Який процес є основним джерелом енергії для міокарда?

Окиснення жирних кислот

Глікогеноліз

Глюконеогенез

Протеоліз

Гліколіз

5934 / 6307
Хворий на сімейну гіперліпідемію, викликану дефіцитом рецепторів до ЛПНЩ, вживав інгібітор бета-гідроксиметил-глутарил-КоА-редуктази. Цей препарат сприяє:

Підвищенню рівня триацилгліцеролів крові

Зниженню клітинного вмісту бета-ГМГ-КоА

Підвищенню активності клітинної ацилхолестеролацилтрансферази

Зниженню рівня холестеролу крові

Підвищенню рівня сквалену в клітинах

5935 / 6307
В альвеолах легень є спеціальні клітини через які здійснюється газообмін. Вони входять до складу аерогематичного бар'єру. Укажіть ці клітини.

Альвеолоцити ІІ-го типу

Клітини Клара

Альвеолоцити І-го типу

Мікроворсинчасті епітеліоцити

Альвеолярні макрофаги

5936 / 6307
Унаслідок порушення правил безпеки при роботі з фосфорорганічним інсек-тицидом у працівника виник бронхоспазм. Який із запропонованих бронхолітиків показаний у цій ситуації?

Ефедрин

Беротек

Адреналін

Атропін

Еуфілін

5937 / 6307
Для профілактики та лікування тромбозу застосовуються засоби, що знижують згортання крові (антикоагулянти). Укажіть антикоагулянт, у разі передозування якого застосовують як антагоніст протаміну сульфат:

Фенілін

Неодикумарин

Синкумар

Гепарин

Натрію гідроцитрат

5938 / 6307
Відповідно до сучасної концепції атерогенезу ''Response to injury'', атеросклероз є проявом хронічного запалення в інтимі артерій. З якою стадією запалення пов'язано формування фіброзних бляшок при атеросклерозі:

Первинна альтерація

Ексудація

Трансформація

Вторинна альтерація

Проліферація

5939 / 6307
Під час мікроскопічного дослідження біоптату шкіри виявляються гранульоми, що складаються з епітеліоїдних клітин, оточених в основному Т-лімфоцитами. Серед епітеліоїдних клітин розташовуються поодинокі гігантські багатоядерні клітини типу Пирогова-Лангханса. В центрі деяких гранульом виявляються ділянки казеозного некрозу. Кровоносні судини відсутні. Для якого захворювання характерні описані гранульоми?

Сапу

Риносклероми

Сифілісу

Лепри

Туберкульозу

5940 / 6307
У хворого 36-ти років одразу після перенесеного стафілококового сепсису з’явилися напади стенокардії. За допомогою коронарографії в лівій коронарній артерії було виявлено пристінковий тромбоз без ознак атеросклерозу. Утворення тромбу розпочалося внаслідок пошкодження ендотелію судини і вивільнення:

Аденозинтрифосфату

Серотоніну

Фосфоліпази А2

Аденозиндифосфату

Фактора активації тромбоцитів

5941 / 6307
У гістопрепараті представлений орган, у власній пластинці слизової оболонки якого розташовані прості трубчасті залози, що складаються переважно з головних і парієтальних, а також слизових, шийкових ендокринних клітин. Укажіть вид залоз:

Власні залози стравоходу

Власні залози шлунка

Кардіальні залози шлунка

Пілоричні залози шлунка

Кардіальні залози стравоходу

5942 / 6307
При обстеженні лікарем хворої людини на основі клінічних даних був поставлений діагноз парагрип. Яким із перелічених способів може передаватись це захворювання?

Повітряно-крапельний

Через укус тварин

Парентеральний

Фекально-оральний

Трансмісивний

5943 / 6307
Під час розтину тіла чоловіка 61 року, який страждав на ревматоїдний артрит, виявлені збільшені нирки, щільної консистенції, жовтувато-білі, з воскоподібним блиском, з ділянками рубцевих западінь на поверхні. Під час забарвлення за Конго-рот виявлено відкладення гомогенних рожевих мас у капілярних петлях клубочків, у стінках артеріол, арте-рій, у базальній мембрані канальців і в стромі. Розвитком якого процесу ускладнився ревматоїдний артрит?

Швидкопрогресуючого гломерулонефриту

Гострого некротичного нефрозу

Вторинниого амілоїдозу нирок

Фібропластичного гломерулонефриту

Постінфекційного гломерулонефриту

5944 / 6307
До медико-генетичної консультації звернулася жінка, щоб дізнатись вірогідність народження сина з гемофілією. Її чоловік страждає на це захворювання з народження. Жінка здорова і серед її предків не було хворих на гемофілію. Визначте ймовірність народження хлопчика з гемофілією в цій сім'ї.

Дорівнює 100%

Дорівнює 75%

Дорівнює 25%

Дорівнює 0%

Дорівнює 50%

5945 / 6307
У хворого зареєстрували ЕКГ. За яким її елементом лікар може оцінити процеси розповсюдження деполяризації передсердь?

Зубець P

Зубець Q

Зубець R

Зубець S

Зубець T

5946 / 6307
У хворого 65-ти років при неврологічному обстеженні виявлено крововилив у межах верхньої скроневої звивини. У зоні кровопостачання якої артерії воно знаходиться?

Середня мозкова

Передня сполучна

Задня мозкова

Основна

Передня мозкова

5947 / 6307
Чому під час першого вдиху новонародженого обсяг повітря, яке видихається, є у 2-3 рази меншим, ніж обсяг, який вдихається?

Формується життєва ємність легень

Формується резерв вдиху

Формується функціональна залишкова ємність легень

Формується загальна ємність легень

Формується дихальний об'єм

5948 / 6307
У жінки 50 років у яєчнику виявлено новоутворення округлої форми, діаметром 2,5 см. На розрізі в ньому є порожнина з прозорою світло-жовтою рідиною, внутрішня поверхня порожнини гладенька. Установіть макроскопічну форму пухлини:

Кіста

Вузол

Виразка

Вузол із некрозом у центрі

Інфільтрат

5949 / 6307
При деяких захворюваннях товстої кишки змінюються кількісні співвідношення між епітеліоцитами слизової оболонки. Які типи клітин переважають в епітелії крипт товстої кишки в нормі?

Малодиференційовані клітини

Стовпчасті ворсинчасті епітеліоцити

Келихоподібні клітини

Клітини з ацидофільними гранулами

Ендокриноцити

5950 / 6307
Після вживання жирної їжі у хворого з'являються нудота, печія, стеаторея. Причиною такого стану може бути:

Підвищене виділення ліпази

Порушення синтезу фосфоліпази

Порушення синтезу трипсину

Нестача жовчних кислот

Нестача амілази

5951 / 6307
Під час вживання внутрішньо 100 мл 25% (насиченого) розчину сірчанокислої магнезії, з'являється багато рідкого калу. Чому виникає такий проносний ефект?

Стимулюється виділення гормонів 12-палої кишки

Гальмується робота кишечника

Стимулюється секреція шлункового соку

Збільшується осмотичний тиск у кишечнику

Зменшується осмотичний тиск

5952 / 6307
Хворий 58-ми років надійшов у торакальне відділення з кровохарканням; при бронхоскопії виявлено звуження правого стовбурового бронха внаслідок розрос-тання сірувато-білої тканини. В ході біопсії виявлена пухлина, що складається з дрібних клітин овальної й округлої форми з гіперхромними ядрами та незначною цитоплазмою, клітини справляють враження ''голих ядер''. Клітини пухлини ростуть пластами, тяжами. Гістологічна будова пухлини свідчить про те, що у хворого:

Недиференційований рак

Базальноклітинний рак

Пласкоклітинний зроговілий рак

Пласкоклітинний незроговілий рак

Перехідноклітинний рак

5953 / 6307
Пацієнту встановлено діагноз: психоз. За два тижні після отримання лікарської терапії його стан поліпшився, однак невдовзі з'явилася ригідність, тремор та гіпокінезія. Який із нижченаведених препаратів може викликати такі ускладення?

Сиднокарб

Аміназин

Імізин

Хлордіазепоксид

Дифенін

5954 / 6307
Під час експерименту на собаці виникла потреба знизити збудливість міокарду. Який розчин для цього доцільно ввести тварині внутрішньовенно?

Хлориду натрію

Глюкози

Хлориду кальцію

Бікарбонату натрію

Хлориду калію

5955 / 6307
У пацієнта під час обстеження виявлено порушення сприйняття зеленого кольору. Відсутність яких клітин сітківки зумовлює таке порушення зору?

Колбочкових нейросенсорних

Біполярних нейронів

Епітеліальних пігментних

Гангліонарних нейронів

Паличкових нейросенсорних

5956 / 6307
П'ятирічна дитина гостро захворіла: висока температура, виражена інтоксикація, геморагічний висип на шкірі. Померла внаслідок гострої надниркової недостатності. Під час розтину тіла були виявлені морфологічні зміни, викликані тяжкістю ДВЗ-синдрому і ендотоксичного шоку. На шкірі виявлено вогнища некрозу, діапедезні крововиливи, фібринові тромби в судинах дерми. У надниркових залозах масивні крововиливи. Для якого захворювання характерні такі зміни?

Менінгококцемія

Скарлатина

Кір

Висипний тиф

Грип

5957 / 6307
Унаслідок токсичного ушкодження клітин печінки з порушенням білковосинтезуючої функції у пацієнта спостерігаються різко знижені кількість альбумінів у плазмі крові та онкотичний тиск. Яке явище буде наслідком цих змін?

Зменшення густини крові

Зменшення діурезу

Зменшення ШОЕ

Збільшення темпу діурезу

Поява набряків

5958 / 6307
До чого призводить екстрасистола, яка була спричинена виникненням збудження в одному зі шлуночків?

До підвищення швидкості проведення збудження по шлуночках

До компенсаторної паузи передсердь

До тривалої компенсаторної паузи шлуночка

До зниження швидкості проведення збудження по передсердях

5959 / 6307
Після гіпервентиляції у спортсмена спостерігається короткочасна зупинка дихання. Якими змінами в крові це зумовлено?

Зменшення рН

Зменшення напруги CO_2

Зменшення напруги O_2

Збільшення напруги CO_2

5960 / 6307
У дівчинки 7-ми років явні ознаки анемії. Лабораторно встановлений дефіцит піруваткінази в еритроцитах. Порушення якого процесу грає головну роль в розвитку анемії у дівчинки?

Дезамінування амінокислот

Розкладу пероксидів

Тканинного дихання

Анаеробного гліколізу

5961 / 6307
У пацієнтки на поверхні шкіри лівого кута лопатки знайдено утвір з наступними характеристиками: 5-7 мм діаметром, кулястої форми, твердої консистенції, який має широку основу та сосочкову поверхню. Гістологічно: клітини плоского епітелію, що нерівномірно розростаються, кількість шарів збільшена, надлишкова кількість кровоносних судин. Вкажіть патологічний процес:

Аденокарцинома

Аденома

''Рак на місці''

Папілома

5962 / 6307
Після накладання джгута у досліджуваного виявили точкові крововиливи. З порушенням функції яких клітин крові це пов'язано?

Тромбоцити

Нейтрофіли

Лімфоцити

Еозинофіли

5963 / 6307
У медико-генетичну консультацію звернувся чоловік з приводу безпліддя. В ядрах більшості клітин епітелію слизової оболонки щоки у нього виявлено одне тільце Барра. Про який синдром може йти мова?

Шерешевського-Тернера

Патау

Клайнфельтера

Едвардса

5964 / 6307
Під час дослідження фаз клітинного циклу, на одній із стадій мітозу клітина майже завершила поділ, а хромосоми, навколо яких починають формуватися ядра, деспіралізуються. Укажіть фазу клітинного циклу.

Телофаза

Метафаза

Профаза

Анафаза

5965 / 6307
У разі різкого підвищення тиску в аорті знижується сила та частота серцевих скорочень. У складі якого нерва проходять чутливі волокна від барорецепторів дуги аорти?

Блукаючого нерва

Нерва Павлова

Язико-глоткового нерва

Зворотного нерва

5966 / 6307
Кетонові тіла у разі діабету синтезуються в печінці з ацетил-КоА. Під час розпаду якої сполуки утворюється ацетил-КоА?

Жирних кислот

Гліцерину

Лактату

Глюкози

5967 / 6307
Для розвитку гарячкових станів характерним є зростання рівня білків ''гострої фази'' - церулоплазміну, фібриногену, С-реактивного протеїну. Укажіть можливий механізм цього явища:

Проліферативна дія ІЛ-2 на Т-лімфоцити

Стимулювальний вплив ІЛ-1 на гепатоцити

Руйнівна дія підвищеної температури на клітини організму

Дегрануляція тканинних базофілів

5968 / 6307
Який із внутрішніх органів бере найбільшу участь у гуморальній регуляції еритропоезу?

Легені

Печінка

Підшлункова залоза

Нирки

5969 / 6307
Під час гістологічного дослідження легень недоношеної дитини встановлено злипання альвеол через відсутність сурфактанту. З недостатністю розвитку яких клітин стінки альвеоли це пов'язано?

Респіраторні альвеолоцити

Секреторні альвеолоцити

Фібробластоподібні клітини

Клітини Клара

5970 / 6307
До лікаря звернулася жінка віком 25 років зі скаргами на висипання в ділянці тулуба. Лікар припускає вторинний сифіліс. Який метод діагностики треба застосувати для підтвердження попереднього діагнозу?

Вірусологічний

Біологічний

Алергічний

Серологічний

5971 / 6307
У жінки народилася мертва дитина з множинними вадами розвитку. Яке протозойне захворювання могло спричинити внутрішньоутробну загибель плоду?

Амебіаз

Токсоплазмоз

Трихомоніаз

Лямбліоз

5972 / 6307
Пацієнта шпиталізовано до реанімаційного відділення з підозрою на отруєння чадним газом. Яке похідне гемоглобіну буде виявлено під час спектрального аналізу?

Метгемоглобін

Карбоксигемоглобін

Дезоксигемоглобін

Карбгемоглобін

5973 / 6307
У хворого виявлено вивих кришталика та павукоподібні пальці. Який синдром діагностує лікар, беручи до уваги ще й порушення розвитку сполучної тканини, форми кисті та стопи хворого?

Шерешевського-Тернера

Трисомії Х

Марфана

Клайнфельтера

5974 / 6307
Тривале застосування лікарських препаратів має вплив на клітини печінки, зокрема, викликає виражену гіпертрофію агранулярного ендоплазматичного ретикулума. З якою функцією даної органели це пов'язано?

Синтез білків

Детоксикація шкідливих речовин

Утворення веретена поділу

Внутрішньоклітинне травлення

5975 / 6307
Під час планового обстеження у вагітної взята кров з вени для постановки реакції Вассермана. Реакція виявилася позитивною. Позашлюбні статеві зв'язки вагітна та її чоловік заперечують. Що потрібно зробити, щоб підтвердити або спростувати діагностування сифілісу?

Поставити реакцію іммобілізації блідих трепонем

Повторити реакцію Вассермана

Поставити реакцію зв'язування комплементу

Поставити осадові реакції

5976 / 6307
В клініку нервових хвороб було доставлено чоловіка 46-ти років з попереднім діагнозом крововилив в мозок. У хворого спостерігаються часті спонтанні рухи кінцівок, які періодично змінюються станом гіпертонуса їх м'язів. Ураженням яких структур головного мозку можна пояснити ці симптоми?

Базальних гангліїв

Лобних ділянок кори головного мозку

Гіпоталамусу

Гіпофізу

5977 / 6307
Хворий 15 років надійшов до алергологічного відділення з діагнозом ''бронхіальна астма''. Надмірне утворення яких антитіл обумовлює розвиток основних клінічних симптомів?

IgE

IgD

IgA

IgM

5978 / 6307
У хворого діагностовано бронхіальну астму. Які зміни показників вентиляції легень будуть спостерігатися?

Зменшення залишкового об'єму легень

Збільшення об'єму форсованого видиху

Збільшення життєвої ємності легень

Зменшення об'єму форсованого видиху

5979 / 6307
В кардіологічне відділення госпіталізований хворий з інфарктом міокарда в гострій фазі. Для лізису тромбів в коронарних судинах у перші години застосовують фермент:

Хімотрипсин

Стрептокіназа

Лізоцим

Лідаза

Трипсин

5980 / 6307
Під час УЗД вагітної в серцево-судинній системі плоду порушень не виявлено, артеріальна протока функціонує. Визначте, які судини вона з'єднує:

Легеневий стовбур з аортою

Легеневий стовбур із верхньою порожнистою веною

Пупкову вену з аортою

Легеневий стовбур із нижньою порожнистою веною

5981 / 6307
У пацієнта в ділянці нижньої щелепи виник болючий вузлик червоного кольору. Гістологічно спостерігається: скупчення гнійного ексудату в декількох волосяних фолікулах. Яка клініко-морфологічна форма запалення спостерігається у цьому випадку?

Абсцес

Фурункул

Карбункул

Панарицій

Флегмона

5982 / 6307
У жінки, яка має розлади менструального циклу, що супроводжуються тривалими кровотечами, виявлено гіпохромію, зниження кількості ретикулоцитів, мікроцитоз та гіпосидеремію. До якої групи за патогенезом відноситься цей вид анемії?

Залізодефіцитна

Гемолітична

Метапластична

Гіпопластична

5983 / 6307
Хвора 24 років потрапила до лікарні зі скаргами на головний біль, біль у поперековій ділянці, набряки на обличчі, загальну слабкість. Місяць тому перенесла ангіну. На час надходження: АТ - 180/110 мм рт.ст., у сечі вичвлено виражену протеінурію, мікрогематурію, лейкоцитурію. На яку форму гіпертензії страждає хвора?

Гіпертонічну хворобу

Ниркову

Ендокринну

Первинну

5984 / 6307
Мікротравмування слизової оболонки порожнини рота виникають щодня під час вживання їжі, проте кровотеча швидко припиняється за допомогою:

Лізоциму

Тромбопластину

Гепарину

Муцину

5985 / 6307
Характерними ознаками холери є втрата організмом великої кількості води та іонів натрію. Що є основою біохімічної дії холерного токсину?

Гальмування синтезу вазопресину у гіпоталамусі

Посилення секреції реніну клітинами ниркових артеріол

Окислення альдостерону в корі наднирників

Активація аденілатциклази в епітелії тонкої кишки

5986 / 6307
Хворому для купірування порушення серцевого ритму було введено препарат, який також застосовується у якості місцево анестезуючого засобу. Визначте препарат:

Анестезин

Кокаїну гідрохлорид

Лідокаїну гідрохлорид

Дикаїн

5987 / 6307
Для лікування захворювань, збудники яких виділяють екзотоксин, застосовують антитоксичні сироватки. Для лікування якого з перерахованих захворювань антитоксичну сироватку застосувати textbfНЕМОЖЛИВО?

Ботулізм

Газова гангрена

Туберкульоз

Дифтерія

5988 / 6307
У жінки 28 років із гнійно-серозним кон'юнктивітом і скаргами на біль під час сечовипускання взято для дослідження зішкріб із кон'юнктиви. Під час мікроскопії в цитоплазмі епітеліальних клітин виявлено включення. Під час посіву патологічного матеріалу на живильні середовища росту не виявлено, але під час зараження курячих ембріонів у жовтковий мішок вдалося виділити культуру збудника. Який із наведених мікроорганізмів є найімовірнішим збудником?

Аденовіруси

Дифтероїди

Мікоплазми

Хламідії

5989 / 6307
Пацієнт віком 45 років звернувся до лікаря зі скаргами на головний біль, запаморочення, часту нудоту, блювання, м'язову слабість, болі в ділянці серця. АТ - 170/110 мм рт. ст. Рівень натрію в периферичній крові - 165 ммоль/л, калію - 2,5 ммоль/л. Під час комп’ютерної томографії виявлено пухлину лівого наднирника розміром 1 см. Які зміни кислотно-основної рівноваги спостерігаються у разі цього захворювання?

Дихальний ацидоз

Дихальний алкалоз

Метаболічний алкалоз

Кислотно-основна рівновага не порушується

5990 / 6307
У новонародженої дитини підвищена чутливість до інфекцій. Який імуноглобулін проникає крізь плацентарний бар'єр і забезпечує гуморальний імунітет немовлят?

IgA

IgG

IgD

IgE

5991 / 6307
Жінка 49 років тривалий час хворіла на хронічний гломерулонефрит, унаслідок чого настала смерть. На розтині встановлено, що нирки мають розміри 7х3х2,5 см, масу 65,0 г, щільні, дрібнозернисті. Також виявлено фібринозне запалення серозних і слизових оболонок, дистрофічні зміни паренхіматозних органів, набряк головного мозку. Яке ускладнення призвело до вказаних змін серозних оболонок і внутрішніх органів?

Тромбоцитопенія

Сепсис

ДВЗ-синдром

Уремія

5992 / 6307
Працівника фермерського тваринницького господарства віком 55 років шпиталізовано до лікарні зі скаргами на високу температуру, озноб, головний біль та болі в м’язах. Пацієнт самотужки лікувався вдома протягом 9 днів після появи симптомів. На підставі клінічних ознак та анамнестичних даних лікар встановив попередній діагноз: лептоспіроз. Який досліджуваний матеріал необхідно взяти для виділення збудника для лабораторного підтвердження діагнозу?

Сечу

Слину

Рановий вміст

Спинномозкову рідину

5993 / 6307
На розтині легенева тканина має комірчастий вигляд через мішковидні та циліндричні розширення бронхів. Мікроскопічно у стінці цих бронхів спостерігається лейкоцитарна інфільтрація із переважанням нейтрофілів. Еластичні м'язові волокна та хрящові пластини частково зруйновані й заміщені сполучною тканиною. Прилегла легенева тканина має осередки запалення, поля фіброзу та склерозу судин і ознаки емфіземи. У серці спостерігається гіпертрофія правого шлуночка. Для якого захворювання характерні такі патологічні зміни?

Бронхоектатична хвороба

Хронічний бронхіт

Емфізема легень

Інтерстиціальна пневмонія

5994 / 6307
У збільшеному шийному лімфатичному вузлі дівчинки 14-ти років мікроскопічно було виявлено, що тканинна будова вузла порушена, лімфоїдні фолікули відсутні, є ділянки склерозу та вогнища некрозу, клітинний склад вузла поліморфний, присутні лімфоцити, еозинофіли, атипові клітини великих розмірів з багаточасточковими ядрами (клітини Березовського-Штернберга) та одноядерні клітини також великих розмірів. Діагностуйте захворювання:

Лімфома Беркітта

Лімфогранулематоз

Хронічний лімфолейкоз

Грибовидний мікоз

5995 / 6307
Продуктами гідролізу та модифікації деяких білків є біологічно активні речовини - гормони. Із якого з нижченаведених білків у гіпофізі утворюються ліпотропін, кортикотропін, меланотропін та ендорфін?

Проопіомеланокортин (ПОМК)

Нейростромін

Нейроглобулін

Тиреоглобулін

5996 / 6307
У батьків з нормальним слухом дві дочки та син страждають на вроджену глухоту, а 5 - здорові діти. Яким є тип успадкування глухоти:

Аутосомно-домінантний

Х-зчеплений домінантний

Х-зчеплений рецесивний

Аутосомно-рецесивний

5997 / 6307
Пацієнт отримав поранення живота зправа. Яка ділянка товстої кишки, найімо-вірніше, може бути пошкоджена?

Висхідна ободова кишка

Пряма кишка

Поперечна ободова кишка

Сигмовидна ободова кишка

5998 / 6307
У криптах слизової оболонки тонкої кишки, в ділянці дна, розташовані клітини з ацидофільними секреторними гранулами та базофільною цитоплазмою, функцією яких є участь в антибактеріальному захисті. Назвіть ці клітини:

Ендокриноцити

Шийкові мукоцити

Клітини Панета

Келихоподібні екзокриноцити

5999 / 6307
У десятирічної дитини проведено пробу Манту (із туберкуліном). Через 48 годин на місці введення препарату з'явилаcя папула розміром до 8 мм у діаметрі. Який тип реакції гіперчутливості розвинувся в дитини після введення туберкуліну?

Реакція гіперчутливості IV типу

Реакція типу феномен Артюса

Атопічна реакція

Реакція типу сироваткової хвороби

6000 / 6307
У пацієнта із порушенням серцевого ритму під час проведення ЕКГ спостерігається наступне: ЧСС - 60/хв, інтервал PQ подовжений, періодичне випадіння комплексу QRS. Яке порушення серцевого ритму виявлено у пацієнта?

Блокада правої ніжки пучка Гіса

Повна AV-блокада

Неповна AV-блокада І ступеня

Синдром слабкості синусового вузла

Неповна AV-блокада ІІ ступеня

6001 / 6307
Чоловіка віком 48 років із ознаками двостороннього спонтанного пневмотораксу, помер. Під час розтину в обох легенях виявлено: субплевральні пухирі діаметром 1-3 см, заповнені повітрям. Легені за межами пухирів мають підвищену повітряність, тканина ріжеться з хрустом. Яка патологія легень спостерігається в пацієнта?

Інтерстиціальна емфізема легень

Бульозна емфізема легень

Стареча емфізема легень

Ідіопатична емфізема легень

6002 / 6307
При деяких спадкових хворобах (наприклад синдром Кернса-Сейра) спостерігається деструкція мітохондрій. Які процеси у клітині можуть бути порушені внаслідок цього?

Поділ ядра

Синтез амінокислот

Синтез АТФ

Кросинговер

6003 / 6307
Який із нижченаведених гормонів стимулює виділення ліполітичних і протеолітичних ферментів клітинами підшлункової залози?

Альдостерон

Соматостатин

Бомбезин

Холецистокінін-панкреозимін (ХЦК-ПЗ)

6004 / 6307
У хімічному синапсі збудження передається за допомогою нейромедіатора. Які іони сприяють вивільненню медіатора в синаптичну щілину?

Натрію

Магнію

Кальцію

Хлору

6005 / 6307
У чоловіка, який хворіє на верхній тип ожиріння, клінічно тривало спостерігалися артеріальна гіпертонія, гіперглікемія, глюкозурія. Смерть настала через крововилив у головний мозок. Під час патоморфологічного дослідження виявлені базофільна аденома гіпофіза і гіперплазія кори надниркових залоз. Який діагноз є найправильнішим?

Гіпофізарний нанізм

Цукровий діабет

Хвороба Іценка-Кушинга

Акромегалія

6006 / 6307
На електронній мікрофотографії червоного кісткового мозку спостерігається мегакаріоцит. Периферична частина його цитоплазми пронизана демаркаційними каналами. Яку роль відіграють ці структури?

Відокремлення тромбоцитів

Збільшення площі поверхні клітин

Руйнування клітин

Поділ клітин

Збільшення кількості іонних каналів

6007 / 6307
У хворого на шкірі обличчя поступово розвилась бляшка з некрозом і виразкою в центрі. При патогістологічному дослідженні біоптату виявлено розростання атипових епітеліальних клітин з великою кількістю патологічних мітозів. Який найбільш імовірний діагноз?

Саркома

Папілома

Рак шкіри

Фіброма

6008 / 6307
При дослідженні стегнової кістки виявлено хронічне гнійне запалення компактної речовини та кісткового мозку, утворення секвестрів. При якому захворюванні розвиваються такі зміни?

Остеобластокластома

Ретикулосаркома

Периостит

Остеомієліт

Мієломна хвороба

6009 / 6307
Відомо, що селезінка є <<кладовищем еритроцитів>>. Що відбувається з еритроцитами червоної пульпи, які гинуть?

Потрапляють у кровотік

Відбувається лізис ферментами гігантських клітин сторонніх тіл

Поглинаються макрофагами

Накопичуються в червоній пульпі

Поглинаються нейтрофільними лейкоцитами

6010 / 6307
У пацієнта, який хворіє на рак сечового міхура, під час проведення цитостатичної хіміотерапії в аналізі крові виявлено: лейкоцити - 0,8·10^9/л, гранулоцити - 0,6·10^9/л. Яке ураження білої крові спостерігається у пацієнта?

Лейкопенія

Агранулоцитоз

Лейкоцитоз

Лейкемія

Лейкемоїдна реакція

6011 / 6307
Один із відділів центральної нервової системи має пошарове розташування нейронів, серед яких є клітини таких форм: зірчасті, веретеноподібні, горизонтальні, пірамідні. Якому відділу нервової системи відповідає така структура?

Мозочку

Кори великих півкуль головного мозку

Спинному мозку

Довгастому мозку

Гіпоталамусу

6012 / 6307
У біоптаті дужок м'якого піднебіння, взятого в зв'язку з підозрою на пухлину (макроскопічно визначалася виразка з щільним дном), виявлений некроз слизової оболонки з інфільтрацією підслизового шару лімфоцитами, епітеліоїдними клітинами, плазматичними клітинами, поодинокими нейтрофілами. Звертає увагу наявність вираженого ендо- та периваскуліту. Описані зміни характерні для:

Первинного сифілісу

Виразково-некротичного стоматиту

Дифтерії зіву

Виразкового стоматиту

Афтозного стоматиту

6013 / 6307
При дії окислювачів (перекис водню, оксиди азоту та інші), гемоглобін, до складу якого входить Fe^2+, перетворюється на сполуку, що містить Fe^3+. Ця сполука textbfНЕ ЗДАТНА переносити кисень і має назву:

Карбоксигемоглобін

Оксигемоглобін

Глікозильований гемоглобін

Метгемоглобін

Карбгемоглобін

6014 / 6307
Чоловік надійшов до хірургічного відділення із діагнозом: гострий панкреа-тит. Розпочато консервативне лікування. Призначення якого препарату є патогенетично обгрунтованим?

Панкреатину

Хімотрипсину

Контрикалу

Фібринолізину

Трипсину

6015 / 6307
В гематологічному відділенні хворому на лейкоз лікар призначив 5-фторурацил, який:

Інгібує трансляцію

Прискорює реплікацію

Стимулює ДНК-азу

Інгібує транскрипцію

Інгібує синтез ДНК

6016 / 6307
В експерименті після тривалої електричної стимуляції периферійного відрізку n. vagus було отримано короткочасну зупинку серця. Які іонні механізми у клітинах серця забезпечили цей ефект?

Зменшення вхідного току Na^+

Зменшення вихідного току K^+

Збільшення вихідного току K^+

Збільшення вхідного току Ca^2+

Збільшення вхідного току Na^+

6017 / 6307
До травматологічного відділення був доставлений пацієнт із пошкодженням передньої поверхні лівого стегна у його нижній третині. Під час обстеження виявлена рана розміром 1,5х3 см, яка розташована поперечно до стегна, на 2 см вище надколінника. У рані добре видно пошкоджене сухожилля м'яза. У пацієнта обмежене розгинання гомілки. Функція якого м'яза, найімовірніше, порушена?

M. quadriceps femoris

M. tibialis anterior

M. peroneus longus

M. triceps surae

M. extensor digitorum longus

6018 / 6307
Жінку 32 років вжалила оса. На місці укусу - набряк та гіперемія. Який механізм набряку є первинним у цьому разі?

Підвищення онкотичного тиску тканинної рідини

Підвищення проникності капілярів

Підвищення гідростатичного тиску крові в капілярах

Утруднення лімфовідтоку

Зниження онкотичного тиску крові

6019 / 6307
В експерименті при збільшенні концентрації калію в позаклітинному середовищі до внутрішньоклітинного рівня, клітина втратила збудливість. Який електрофізіологічний процес при цьому відбувся?

Збільшення виходу натрію з клітини та розвиток деполяризації

Підвищення входу калію в клітину та розвиток локальної відповіді

Підвищення виходу калію з клітини та розвиток гіперполяризації

Припинення виходу калію з клітини та зникнення потенціалу спокою

Зниження виходу калію з клітини та розвиток гіперполяризації

6020 / 6307
Уведення в організм людини лікарського препарату дикумаролу викликає різке зниження вмісту в крові протромбіну й інших білкових факторів згортання крові. Антивітаміном якого вітаміну є дикумарол?

Вітаміну Е

Вітаміну С

Вітаміну Р

Вітаміну Н

Вітаміну К

6021 / 6307
Під час лікування парадонтозу застосовують антиоксидант природного походження. Яка з наведених природних сполук використовується як антиоксидантний засіб?

Піридоксин

Тіамін

Холін

Глюконат

Токоферол

6022 / 6307
Відомо, що одним з механізмів мобілізації організму у відповідь на стресові ситуації є вироблення адренокортикотропного гормону (АКТГ), який регулює синтез і виділення гормонів кори надниркових залоз. Який гормон викликає секрецію АКТГ у передній долі гіпофіза?

Соматотропний гормон

Гормон росту

Кортікотропін-рилізинг гормон

Тиреотропний гормон

Епідермальний фактор росту

6023 / 6307
До стоматолога звернувся хворий зі скаргами на біль, почервоніння слизової оболонки порожнини рота, припухлість ясен. Було встановлено діагноз: герпетичний гінгівостоматит. Це захворювання міг викликати вірус:

Простого герпесу тип 2

Оперізуючого лишаю

Простого герпесу тип 1

Цитомегалії

Епштейна-Барр

6024 / 6307
При тривалому лікуванні голодуванням у пацієнта зменшилось співвідношення альбумінів і глобулінів в плазмі. Що з наведеного буде наслідком цих змін?

Збільшення ШОЕ

Зниження ШОЕ

Збільшення гематокриту

Зниження гематокриту

Гіперкоагуляція

6025 / 6307
Після вживання лікарського препарату-блокатора в людини підвищилася частота серцевих скорочень (ЧСС). Після натискання на очні яблука очікуваного рефлекторного зниження ЧСС не відбулося. Що заблокував препарат у клітинах водія ритму серця?

Ca2^+-канали L-типу

Бета1-адренорецептори

Швидкі Na^+-канали

Альфа1-адренорецептори

М-холінорецептори

6026 / 6307
Недостатня секреція якого ферменту зумовлює неповне перетравлювання жирів в кишково-шлунковому тракті та появу великої кількості нейтральних жирів в калових масах?

Пепсину

Панкреатичної ліпази

Амілази

Фосфоліпази

Ентерокінази

6027 / 6307
Арахідонова кислота, як незамінний компонент їжі, є попередником біологічно активних речовин. Які сполуки синтезуються з цієї кислоти?

Трийодтиронін

Холін

Адреналін

Тромбоксани

Етаноламін

6028 / 6307
Надмірне споживання вуглеводів (600 г на добу), що перевищує енергетичні потреби у людини 28 років, буде супроводжуватися активацією:

Глюконеогенезу

Бета-окисленню жирних кислот

Гліколізу

Ліпогенезу

Ліполізу

6029 / 6307
Хворому для зняття кишкової коліки призначений атропіну сульфат. Яке з названих захворювань може бути протипоказанням для застосування цього препарату?

Бронхіальна астма

Головний біль

Гіпотонія

Глаукома

Запаморочення

6030 / 6307
У мікропрепараті виготовленому з пунктату регіонарного лімфовузла пацієнта, зафарбованому за Романовським-Гімза, лікар виявив тонкі мікроорганізми з 12-14 рівномірними завитками з гострими кінцями довжиною 10-13 мкм блідо-рожевого кольору. Збудника якої інфекційної хвороби виявлено?

Сифілісу

Поворотного тифу

Лептоспірозу

Трипаносомозу

Лейшманіозу

6031 / 6307
У жінки виявлено пухлину яєчника. Показана операція. Яку зв'язку повинен перерізати хірург, щоб відділити яєчник від матки?

Власну зв'язку яєчника

Круглу зв'язку матки

Зв'язку, що підвішує яєчник

Широку зв'язку матки

Бічну пупкову зв'язку

6032 / 6307
Під час експерименту на клітину подіяли тетраетиламонієм, який блокує калієві іоноселективні канали. Як це позначиться на мембранному потенціалі?

Потенціал дії не виникне

Потенціал спокою зникне

Розвинеться гіперполяризація

Потенціал спокою збільшиться

Потенціал спокою не зміниться

6033 / 6307
Під час субмікроскопічного дослідження клітини виявлено, що її цитоплазма містить багато лізосом, фагосом, піноцитозних міхурців. Інші органели розвинені помірно. Яку функцію може виконувати така клітина?

Реабсорбція іонів натрію

Депонування іонів кальцію

Фагоцитоз

Синтез полісахаридів

Синтез ліпідів

6034 / 6307
Для підвищення результатів спортсмену рекомендували застосовувати препарат, який містить карнітин. Який процес найбільше активується карнітином?

Синтез стероїдних гормонів

Тканинне дихання

Транспорт жирних кислот у мітохондрії

Синтез ліпідів

Синтез кетонових тіл

6035 / 6307
У пацієнта виявлено порушення реабсорбції води в нирках. Із порушенням секреції якого гормону це безпосередньо пов'язано?

Тиреокальціотоніну

Натрійуретичного

Вазопресину

Паратгормону

Альдостерону

6036 / 6307
У пацієнтки за два тижні після видалення зуба відбулася регенерація багатошарового плоского епітелію. Які органели брали участь у відновленні слизової оболонки?

Центросоми

Постлізосоми

Мітохондрії

Гладкий ЕПР

Рибосоми

6037 / 6307
До лікаря звернувся пацієнт з приводу хвороби Паркінсона. Лікар обрав засіб-попередник дофаміну в ЦНС. Який з наведених препаратів має такий механізм дії?

Платифіліну гідротартрат

Метацин

Лоразепам

Леводопа

Іпратропію бромід

6038 / 6307
Під час огляду пацієнта з ранами на шкірі голови, що кровоточать, лікар виявив пошкодження тканин личинками, а також локальні місця нагноєння. Встановлено діагноз: облігатний міаз. Який збудник викликає цей стан?

Жигалка осіння

Муха вольфартова

Муха цеце

Муха хатня

Триатомовий клоп

6039 / 6307
Нітрогліцерин використовується для розширення коронарних судин та зменшення болю в серці. В організмі людини він розпадається з утворенням нітроген оксиду (NO). Активність якого ферменту підвищується в цьому разі?

Гуанілатциклази

Ліпооксигенази

Фосфоліпази С

Циклооксигенази

Аденілатциклази

6040 / 6307
Недостатність міді у людини позначається на енергетичному обміні. Дефіцит якого фермента зумовлює цей процес?

Аргінази

Лактатдегідрогенази

Сукцинатдегідрогенази

Піруваткарбоксилази

Цитохромоксидази

6041 / 6307
Чоловікові, у якого є підозра на прогресуючу м'язову дистрофію, було зроблено аналіз сечі. Яка сполука в сечі підтверджує діагноз цього пацієнта?

Креатинін

Міоглобін

Порфирін

Креатин

Колаген

6042 / 6307
Пацієнтку віком 50 років шпиталізовано із закритою черепно-мозковою травмою у ділянці потиличної кістки. Під час огляду спостерігається: порушення ходи і рівноваги, тремор рук. Яка частина головного мозку пошкоджена?

Проміжний мозок

Міст

Мозочок

Спинний мозок

Довгастий мозок

6043 / 6307
До реанімації лікарні потрапив хворий з тяжким отруєнням невідомою речовиною. Який з перерахованих препаратів необхідно використати для проведення форсованого діурезу?

Гідрохлортіазид

Спіронолактон

Тріамтерен

Ацетазоламід

Фуросемід

6044 / 6307
Пацієнту 33 роки. Хворіє 10 років. Періодично звертається до лікаря зі скаргами на гострий біль в животі, судоми, порушення зору. У його родичів спостерігаються подібні симптоми. Сеча червоного кольору. Госпіталізований із діагнозом ''гостра переміжна порфірія''. Причиною захворювання може бути порушення біосинтезу:

Колагену

Жовчних кислот

Інсуліну

Простагландинів

Гему

6045 / 6307
Профілактичні щеплення проти поліомієліту провели інактивованою вакциною, яку вводили парентерально. Які імуноглобуліни відповідають у цьому випадку за створення поствакцинального імунітету?

Ig G, секреторні Ig A

Ig M, секреторні Ig A

Ig M, Ig G

Сироваткові Ig А, Ig M

Ig Е, Ig M

6046 / 6307
На гістологічному препараті представлений зріз судини, що характеризує-ться правильною круглою формою; зіяє, стінка складається з 3-х оболонок, де у середній спостерігається наявність 40-50 вікончастих еластичних мембран. Яка судина представлена на препараті?

Кровоносний капіляр

Артерія змішаного типу

Артерія м'язового типу

Артерія еластичного типу

Вена м'язового типу

6047 / 6307
Хворій на гіпертензивну хворобу в комплексній терапії призначено гідрохлортіазид. Який механізм дії препарату сприяє зниженню артеріального тиску?

Блокада ферменту карбоангідрази

Зменшення виділення іонів натрію та води

Збільшення виділення іонів натрію та води

Збільшення утворення ангіотензину ІІ

Блокада кальцієвих каналів

6048 / 6307
Під час клінічного та біохімічного обстеження пацієнта виявлено серповидноклітинну анемію. Визначення якого компонента крові було вирішальним для встановлення діагнозу?

Метгемоглобіну

Гемоглобіну A1

Гемоглобіну S

Гемоглобіну F

Гемоглобіну С

6049 / 6307
Під час бігу на довгі дистанції скелетна мускулатура тренованої людини використовує глюкозу з метою отримання енергії АТФ для м'язового скорочення. Вкажіть основний процес утилізації глюкози за цих умов:

Глюконеогенез

Глікогенез

Аеробне окиснення

Анаеробний гліколіз

Глікогеноліз

6050 / 6307
У дитини виявлено спадкове захворювання шкіри ангідроз (відсутність потових залоз). Унаслідок цього порушені важливі функцiї шкiри - потовидiлення та терморегуляцiя. Ця вада є наслiдком порушення в ембріогенезі закладки:

Ектодерми

Спланхнотому

Склеротому

Ентодерми

Дерматому

6051 / 6307
Молекула зрілої інформаційної РНК має меншу довжину, ніж відповідний ген молекули ДНК. Неінформативні послідовності нуклеотидів про-іРНК видаляються під час процесингу. Яку назву мають ці ділянки?

Мутони

Інтрони

Транскриптони

Екзони

Кластери

6052 / 6307
У медико-генетичній консультації дослідили каріотип дитини із вадами фізичного розвитку. Виявлено трисомію хромосоми 13. Як називається цей синдром?

Патау

Едвардса

Шерешевського-Тернера

Дауна

Клайнфельтера

6053 / 6307
Під час проведення оперативного втручання із застосуванням інгаляційного наркозу та міорелаксантів лікар-анестезіолог помітив у хворого швидке підвищення температури тіла до 43^oC) Назвіть патологію, яка розвинулася у хворого:

Гіпертермічний синдром

Перегрівання

Фізична гіпертермія

Травматичний шок

Інфекційна гарячка

6054 / 6307
В лікуванні системного захворювання сполучної тканини (склеродермія) необхідно використовувати десенсибілізуючу, протизапальну та імунодепресивну дії ліків. Якій групі засобів притаманні всі ці ефекти?

Антигістамінні засоби

Адреноміметичні засоби

Анаболічні стероїди

Стероїдні протизапальні

Адреноблокатори

6055 / 6307
В експерименті досліджували впливи симпатичної та парасимпатичної системи на серцево-судинну діяльність. Внаслідок подразнення блукаючого нерву спостерігалося зниження артеріального тиску. На чому переважно ґрунтується такий ефект парасимпатичної системи?

Зниженні сили серцевих скорочень

Зниженні частоти серцевих скорочень

Розширені вен

Зниженні периферичного опору судин

Розширенні артеріол

6056 / 6307
У чоловіка збіжна косоокість. Який із м'язів очного яблука пошкоджено?

Musculus rectus oculi superior

Musculus rectus oculi medialis

Musculus rectus oculi lateralis

Musculus rectus oculi inferior

Musculus obliqus oculi superior

6057 / 6307
До відділення реанімації доставлено пацієнта, у крові якого виявлено високий вміст сульфгемоглобіну. Який тип гіпоксії наявний у цьому разі?

Гемічний тип

Циркуляторний тип

Респіраторний тип

Екзогенний тип

Тканинний тип

6058 / 6307
Під час операції на органах заднього середостіння існує ризик пошкодження нервів, розташованих поруч зі стравоходом. Назвіть ці нерви.

Язикоглоткові нерви

Діафрагмові нерви

Блукаючі нерви

Додаткові нерви

Міжреберні нерви

6059 / 6307
Після травми хворий не може розігнути руку у ліктьовому суглобі. Порушення функції якого м'яза може бути причиною цього?

M. brachialis

M. coraco-brachialis

M. biceps brachii

M. subscapularis

M. triceps brachii

6060 / 6307
У пацієнта зі спадковою гіперамоніє-мією внаслідок порушення орнітинового циклу виникла вторинна оротацидурія. Зростання якого метаболіту орнітинового циклу є причиною посиленого синтезу оротової кислоти?

Орнітину

Карбамоїлфосфату

Цитруліну

Аргініносукцинату

Сечовини

6061 / 6307
У чоловіка віком 33 роки діагностували прободіння шлунку і запалення очеревини, що призвело до напруження м’язів передньої черевної стінки - дошкоподібний живіт. Який рефлекс забезпечує цей симптом?

Вісцеровісцеральний

Кутановісцеральний

Вісцеросоматичний

Соматовісцеральний

Вісцерокутаний

6062 / 6307
У товщі шкіри макроскопічно виявлено рухливу пухлину. Мікроскопічно спостерігаються хаотично розташовані пучки колагенових волокон із невеликою кількістю веретеноподібних клітин. Яка пухлина виявлена?

Лейоміома

Фіброма

Меланома

Ліпома

Гломус-ангіома

6063 / 6307
У чоловіка віком 55 років раптово виникло сильне серцебиття та болі в серці, різка слабкість, підвищення артеріального тиску, нерегулярний пульс з дефіцитом. На ЕКГ відсутні хвилі P і різна тривалість інтервалів R-R. Яке порушення серцевого ритму у пацієнта?

Миготлива аритмія

Поперечна блокада серця

Дихальна аритмія

Екстрасистолія

Пароксизмальна тахікардія

6064 / 6307
У пацієнта порушений акт ковтання, осиплість голосу, поперхування рідкою їжею, м'яке піднебіння звисає. Які нерви іннервують м'язи м'якого піднебіння?

Язикоглотковий і під'язиковий

Лицевий і щічний

Шийне сплетення

Блукаючий і нижньощелепний

Язикоглотковий і лицевий

6065 / 6307
До лікаря звернувся юнак 16-ти років зі скаргами на свербіння між пальцями рук і на животі, яке посилювалося вночі. При огляді на шкірі були виявлені тоненькі смужки сірого кольору та дрібний висип. Який найбільш імовірний збудник цієї хвороби?

Dermacentor pictus

Ixodes persulcatus

Sarcoptes scabiei

Ixodes ricinus

Ornitodorus papillipes

6066 / 6307
У хворого після загострення хронічного калькульозного холециститу гостро розвинулася жовтяниця. Під час ЕКГ-обстеження звернено увагу, що на тлі правильного синусового ритму (ЧСС - 51/хв.) періодично з'являються екстрасистоли. Який механізм, найімовірніше, викликав порушення електричної активності серця?

Пошкоджуюча дія жовчних кислот на міокард

Подразнення провідної системи токсинами, які не були знешкоджені в печінці

Подразнювальна дія жовчних кислот на синусовий вузол

Пошкоджуюча дія жовчних кислот на синусовий вузол

Подразнення рецепторів блукаючого нерва жовчними кислотами

6067 / 6307
На рентгенограмі тазу видно всі три частини тазової кістки, відокремлені широкими проміжками, що відповідають невидимому на рентгенограмах хрящу. Про який вік суб'єкта можна говорити?

До 40 років

До 36 років

До 50 років

До 16 років

До 46 років

6068 / 6307
Чоловік 60 років систематично приймав дигоксин. Його стан спершу поліпшився, а потім став погіршуватися, розвинулися брадикардія і аритмія. Яке явище є причиною цього стану?

Тахіфілаксія

Кумуляція матеріальна

Зниження чутливості адренорецепторів

Алергія

Індукція ферментів

6069 / 6307
У життєвому циклі деяких паразитів може бути хазяїн, в якому паразит зберігає життєздатність, нагромоджується але не розвивається. Як називають цього хазяїна?

Факультативним

Додатковим

Резервуарним

Дефінітивним

Облігатним

6070 / 6307
Аміак особливо токсичний для ЦНС людини. Вкажіть головний шлях його знешкодження в нервовій тканині:

Трансамінування

Синтез солей амонію

Синтез глутаміну

Синтез сечовини

Утворення парних сполук

6071 / 6307
У чоловіка гострий гломерулонефрит. Поява якої речовини в сечі свідчить про ушкодження базальної мембрани капілярів клубочків нирок у разі цієї патології?

Білку

Лейкоцитів

17-кетостероїдів

Креатину

Глюкози

6072 / 6307
Виникнення нижченаведених захворювань пов'язують із генетичними факторами. Укажіть патологію зі спадковою схильністю.

Фенілкетонурія

Цукровий діабет

Хорея Гантінгтона

Дальтонізм

Серповидноклітинна анемія

6073 / 6307
Пацієнта госпіталізували до лікарні в коматозному стані. Із анамнезу відомо, що він хворіє на цукровий діабет другого типу впродовж 5 років. Об'єктивно спостерігається: дихання шумне, глибоке, із рота відчувається запах ацетону. Уміст глюкози у крові становить 15,2 ммоль/л, кетонових тіл - 100 мкмоль/л. Для якого ускладнення цукрового діабету характерні такі симтоми?

Гіперосмолярної коми

Гіпоглікемічної коми

Кетоацидотичної коми

Печінкової коми

Гіперглікемічної коми

6074 / 6307
Після черепно-мозкової травми у пацієнта спостерігається втрата здатності сприймати написаний текст та читати (алексія). В якій ділянці кори півкуль у нормі локалізується відповідний центр?

Gyrus parahipocampalis

Gyrus paracentralis

Gyrus angularis

Gyrus supramarginalis

Gyrus lingualis

6075 / 6307
Цукровий діабет у хворої 35 років ускладнився виникненням і розвитком катаракти, ймовірною причиною якої є:

Дефіцит НАДФ · Н2 та гальмування синтезу жирних кислот

Зниження спорідненості білків з киснем

Глікозилювання білків кришталика і накопичення сорбітолу

Клітинна дегідратація

Порушення зв’язування рецепторів з інсуліном

6076 / 6307
У дитини 10 років, через виявлену пухлину, видалено задню частку гіпофіза. Унаслідок цього виникне:

Затримка росту

Затримка розумового розвитку

Збільшення діурезу

Зменшення діурезу

Гіперглікемія

6077 / 6307
Пацієнт віком 34 роки звернувся до лікаря-стоматолога із приводу зубного болю. Після видалення декількох зубів почалася кровотеча, що тривала понад 15 хвилин. Із анамнезу відомо: у пацієнта хронічний гепатит С. Яка ймовірна причина продовження кровотечі?

Гіпокальціємія

Тромбоцитопенія

Зниження вмісту альбумінів у крові

Зниження вмісту фібриногену в крові

6078 / 6307
При дослідженні людини у вертикальній позі встановлено, що в альвеолах верхівок легень парціальний тиск кисню складає 140 мм рт.ст. Причиною цього є те, що у даних відділах легень:

Вентиляція переважає над перфузією

Вентиляція відсутня

Перфузія переважає над вентиляцією

Перфузія і вентиляція врівноважені

6079 / 6307
У пацієнтки віком 47 років міжфалангові і п'ястно-фалангові суглоби легко піддаються вивиху і підвивиху з типовим відхиленням пальців у вигляді <<плавники моржа>>. Під час мікроскопічного дослідження виявлено розростання ворсин синовіальної оболонки, руйнування хряща та формування паннуса. Яке захворювання викликає такі паталогічні зміни?

Хвороба Бехтерєва

Системний червоний вовчак

Ревматичний артрит

Остеоартроз

Ревматоїдний артрит

6080 / 6307
Надмолекулярний мультиферментний комплекс, інтегрований у ліпідний шар внутрішньої мембрани мітохондрій, що створює умови для перебігу окисно-відновних реакцій. Укажіть із нижченаведеного.

G-білок-трансдуктор

Піруваткіназа

Карбоксипептидаза

Гексокіназа

Дихальний ланцюг

6081 / 6307
Пентозофосфатний шлях окислення глюкози має важливе фізіологічне значення для синтезу нуклеотидів ДНК та РНК і деяких коферментів. З постачанням якого ферменту це пов'язано?

Рибозо-5-фосфату

Фруктозо-1-фосфату

Галактозо-1-фосфату

Гліцерол-3-фосфату

Фосфоенолпірувату

6082 / 6307
Пацієнту з діагнозом: кропив'янка призначили антигістамінний засіб - димедрол. Укажіть механізм дії цього препарату.

Вивільнення гістаміну

Утворення комплексу антиген-антиті-ло

Взаємодія гістаміну з рецепторами в органах

Синтез імуноглобулінів

Активація В-лімфоцитів

6083 / 6307
У мешканців територій із холодним кліматом у крові виявляється підвищений уміст гормону, що має пристосувальне терморегуляторне значення. Укажіть цей гормон.

Інсулін

Соматотропін

Тироксин

Кортизол

Глюкагон

6084 / 6307
У пацієнта з хворобою нирок спостерігається підвищення артеріального тиску. Які структури нирки спричиняють цей симптом?

Клітини щільної плями

Юкстагломерулярні клітини

Клітини дистальних канальців

Клітини петлі нефрона

Клітини проксимальних канальців

6085 / 6307
Юнак 15-ти років скаржиться на загальну слабкість, запаморочення, швидку стомлюваність. При обстеженні виявлено еритроцити зміненої форми, кількість їх знижена. Попередній діагноз: серповидноклітинна анемія. Яка амінокислотна заміна в гемоглобіні зумовлює розвиток цього патологічного стану?

Валіну на глутамат

Глутамату на валін

Глутамату на аспартат

Глутамату на аланін

Валіну на аспартат

6086 / 6307
У жінки віком 64 роки за тиждень після перенесеного інсульту відсутні рухи у верхній і нижній лівих кінцівках. Тонус м'язів цих кінцівок і рефлекси в них підвищені, наявні патологічні рефлекси. Яка форма паралічу у пацієнтки?

Геміплегія

Моноплегія

Параплегія

Тетраплегія

Диплегія

6087 / 6307
У систему для штучного культивування клітин введено радіоактивний ізотоп фосфору. Які органічні речовини можуть його накопичувати?

Амінокислоти

Дисахариди

Тригліцериди

Полісахариди

Нуклеїнові кислоти

6088 / 6307
Під час оперативного втручання із приводу каменів у жовчних протоках, лікар-хірург повинен знайти загальну печінкову протоку. Між листками якої зв'язки вона знаходиться?

Печінково-ниркової

Венозної

Печінково-дванадцятипалої

Круглої

Печінково-шлункової

6089 / 6307
Особлива роль цистеїну пов’язана з тим, що він входить до складу потужного антиоксиданту, що складається з трьох амінокислот та має здатність зворотно перетворюватись з відновленої форми на окислену, підтримуючи тим самим у відновленому стані SH-групи різних ферментів та біорегуляторів. Вкажіть цей антиоксидант.

Карнозин

Ретинол

Токоферол

Глутатіон

Ансерин

6090 / 6307
У медичній практиці для профілактики алкоголізму широко використовують тетурам, який є інгібітором альдегіддегідрогенази. Підвищення у крові якого метаболіту викликає відразу до алкоголю?

Ацетальдегіду

Пропіонового альдегіду

Малонового альдегіду

Метанолу

Етанолу

6091 / 6307
При хронічному передозуванні глюкокортикоїдів у хворого розвивається гіперглікемія. Назвіть процес вуглеводного обміну, за рахунок якого при цьому збільшується концентрація глюкози у крові:

Аеробний гліколіз

Глюконеогенез

Пентозофосфатний цикл

Глікогеноліз

Глікогенез

6092 / 6307
У дорослої людини системний артеріальний тиск знизився з 120/70 до 90/50 мм рт.ст., що викликало рефлекторне звуження судин. У якому із зазначених органів звуження судин буде найбільшим?

Нирки

Наднирники

Серце

Кишечник

Головний мозок

6093 / 6307
Хворому перед екстракцією зуба була проведена провідникова анестезія лідокаїном, після введення якого з'явилися набряк і гіперемія навколо місця уколу, свербіння шкіри, загальна слабкість, гіпотензія, рухове збудження. Визначте, як називається ускладнення, що виникло:

Токсична дія

Алергічна реакція

Толерантність

Лікарська залежність

Тахіфілаксія

6094 / 6307
Збільшення екскреції якої речовини з сечею свідчить про підвищення процесів гниття у кишечнику пацієнта з кишковою непрохідністю?

Креатиніну

Індикану

Сечової кислоти

Уробіліну

Сечовини

6095 / 6307
У чоловіка віком 44 років перед смертю спостерігалося різке падіння артеріального тиску. На розтині виявлено аневризму дуги аорти до 10 см у діаметрі. Інтима аорти у висхідній частині та дузі має зморшкуватий вигляд, інтима аорти відшарована; між інтимою і середнім шаром аорти є згустки крові. Мікроскопічно спостерігається: в середньому шарі великовогнищева інфільтрація лімфоїдними, плазматичними і епітеліоїдними клітинами, руйнування еластичних волокон і розростання сполучної тканини, васкуліти emph vasa vasorum. Для якого захворювання характерні такі зміни?

Ревматичний аортит

Атеросклеротична аневризма аорти

Вузликовий поліартеріїт

Неспецифічний аортоартеріїт

Сифілітичний мезаортит

6096 / 6307
На 8-й день після введення протиправцевої сироватки у пацієнта піднялась температура, він став скаржитися на біль у суглобах та свербіж шкіри. Який меха-нізм цього ускладнення?

Гіперчутливість уповільненого типу

Антитілозалежна цитотоксичність

Імунокомплексна гіперчутливість

Клітинна цитотоксичність

Анафілаксія

6097 / 6307
При гастритах, як правило, ушкоджую-ться залози слизової оболонки шлунку. За рахунок яких клітин можлива їх регенерація?

Головні

Додаткові

Парієтальні

Шийкові мукоцити

Ендокринні

6098 / 6307
У жінки 45 років в період цвітіння трав виникло гостре запальне захворювання верхніх дихальних шляхів та очей: гіперемія, набряк, слизові виділення. Збільшення кількості якого типу лейкоцитів буде найхарактернішим у цьому разі?

Базофілів

Нейтрофілів

Еозинофілів

Лімфоцитів

Моноцитів

6099 / 6307
Чоловік 32-х років, мешканець сільської місцевості, звернувся до лікаря зі скаргами на наявність болісної припухлості у підщелепній ділянці з утворенням нориці. Під час огляду хворого лікар виявив інфільтрат, у центрі якого наявна нориця з густим гнійним виділенням з крихтами білого кольору. Під час розтину інфільтрату тканини щільні, жовтувато-зеленого кольору, стільникової будови через множинні дрібні гнійники. Яке захворювання можна припустити?

Сифіліс

Туберкульоз

Актиномікоз

Лепра (проказа)

Абсцес підщелепної ділянки

6100 / 6307
У палаті інтенсивної терапії інфекційного відділення лікар відзначає у пацієнта з менінгококовим менінгітом періоди тахіпное, які змінюються тривалими періодами апное. Який це тип патологічного дихання?

Гаспінг-дихання

Дихання Чейна-Стокса

Апнейстичне дихання

Дихання Біота

Дихання Грокко

6101 / 6307
Під час мікроскопії мазка, приготованого з досліджуваного матеріалу від хворої дитини з підозрою на дифтерію і забарвленого за Нейссером, виявлено палички світло-коричневого кольору з темно-синіми включеннями на кінцях. Який структурний елемент мікробної клітини виявлено?

Спори

Капсула

Ядерна субстанція

Зерна волютину

Джгутики

6102 / 6307
У дворічної дитини виникли судоми внаслідок зниження концентрації іонів кальцію в плазмі крові. Зниженням функції якої залози це обумовлено?

Гіпофізу

Кори наднирників

Прищитоподібних залоз

Шишкоподібної залози

Тимусу

6103 / 6307
Під час гістологічного дослідження біоптату печінки 67-річного чоловіка, який тривалий час хворіє на хронічну дифузну обструктивну емфізему, було знайдено такі морфологічні зміни: центральні вени дилатовані; у центрі часточок синусоїди гіперемовані, з ознаками капілярізації; частина гепатоцитів дистрофічно змінена; помірний периваскулярний склероз; перипортально спостерігаються гепатоцити з ознаками жирової дистрофії, Діагностуйте вид ураження печінки:

Портальний цироз

''Гусяча'' печінка

''Мускатна'' печінка

''Кремнієва'' печінка

Жировий гепатоз

6104 / 6307
У тимусі, виробляється не тільки специфічний гуморальний фактор - тимозин, що викликає диференціацію клітин людського червоного кісткового мозку, але й дозрівають деякі клітини. Які це клітини?

Макрофаги

Еритроцити

В-лiмфоцити

Т-лiмфоцити

Базофіли

6105 / 6307
Під час аутопсії плода виявлено епідуральну гематому, яка утворилася внаслідок розриву серповидного синуса і намета мозочка.Проявом патології якого періоду розвитку можливе таке ушкодження?

Інтранатального періоду

Постнатального періоду

Періоду прогенеза

Антенатального періоду

Перинатального періоду

6106 / 6307
Пацієнту після видалення пухлини для проведення хіміотерапії було призначено 5-фторурацил. Яка особливість дії фторурацилу забезпечує його використання як протипухлинного препарата?

Він є активатором РНК-полімерази

Він є інгібітором РНК-полімерази

Він є інгібітором тимідилатсинтази

Він є інгібітором дигідрофолатредук-тази

6107 / 6307
При розтині хворого 40-ка років в тонкій кишці виявлені збільшені групові фолікули, поверхня їх з борознами та звивинами, малюнок нагадує головний мозок, фолікули виступають під слизовою оболонкою кишки. На розрізі вони соковиті, сіро-червоні. При мікроскопічному дослідженні помітна проліферація моноцитів, гістіоцитів, ретикулярних клітин, скупчення макрофагів, які утворюють гранульоми, лімфоцити витіснені. Для якого захворювання характерні описані зміни?

Амебіаз

Черевний тиф

Сальмонельоз

Холера

6108 / 6307
В реанімаційне відділення поступив хворий з тяжким отруєнням. Для проведення комплексу лікування необхідно виконати катетеризацію та введення лікарських речовин до підключичної вени. У якому топографічному утворенні вона знаходиться?

Spatium antesсalenum

Spatium interaрoneuroticum suprasternale

Spatium retrosternocleidomastoideus

Spatium interscalenum

6109 / 6307
До якої групи молекулярних хвороб обміну речовин належить фенілкетонурія?

Порушення метаболізму амінокислот

Спадкові хвороби обміну сполучної тканини

Порушення обміну мінеральних речовин

Порушення метаболізму вуглеводів

6110 / 6307
Як відобразиться на нервово-м'язовій передачі відкачування іонів кальцію із синаптичної щілини в експерименті?

Збільшиться викид медіатора в синаптичну щілину

Виникне деполяризація кінцевої пластини

Виникне потенціал дії кінцевої пластини

Зменшиться викид медіатора в синаптичну щілину

6111 / 6307
Під час операції на грудному відділі хребетного стовпа лікар-хірург розітнув зв'язки, що з'єднують дуги хребців. Визначте, які зв'язки розітнув хірург.

Ligg. interspinalia

Ligg. flavа

Lig. supraspinale

Ligg. intertransversaria

6112 / 6307
Як зміниться дихання в людини, яка ввійшла у приміщення із підвищеним умістом вуглекислого газу?

Зменшиться глибина дихання

Збільшиться глибина і частота дихання

Зменшиться частота дихання

Збільшиться частота дихання

6113 / 6307
У бактеріологічній лабораторії досліджувалися консерви, які стали причиною важкої токсикоінфекції. Під час мікроскопії культури із середовища Кітта-Тароцці виявлені грампозитивні спороутворювачі палички, схожі на тенісну ракетку. Який діагноз поставить лікар?

Дизентерія

Туляремія

Черевний тиф

Ботулізм

6114 / 6307
Основний спосіб зняття гіпертонічного кризу - швидке і значне зниження судинного опору. Ін'єкційні препарати якої групи адреноблокаторів можуть бути використані для цього?

Симпатолітики

alpha-адреноблокатори

Кардіоселективні beta-адреноблокатори

6115 / 6307
До лікаря-інфекціоніста звернулись батьки з хворою дитиною, які тривалий час працювали в Індії. У дитини такі симптоми: шкіра землистого кольору, втрата апетиту, в'ялість, збільшені печінка, селезінка та лімфатичні вузли. Яке протозойне захворювання можна припустити у дитини?

Амебіаз

Токсоплазмоз

Вісцеральний лейшманіоз

Балантидіаз

6116 / 6307
Під час запальних процесів в організмі починається синтез білків <<гострої фази>>. Які речовини є стимуляторами їх синтезу?

Ангіотензини

Інтерлейкін-1

Біогенні аміни

Інтерферони

6117 / 6307
Унаслідок травми відбулося пошкодження спинного мозку з повним його розривом на рівні першого шийного хребця. Як зміниться дихання потерпілого?

Зросте глибина дихання

Дихання припиниться

Зменшиться частота дихання

Зросте частота дихання

6118 / 6307
У альпіністів, які без кисневого спорядження піднімалися на гору, на висоті 5000 метрів над рівнем моря виникла тахікардія, знизився тиск, дихання стало частим. Який вид гіпоксії розвинувся?

Гіпоксемічна

Гемічна

Циркуляторна

Тканинна

6119 / 6307
Чоловік 63 років, за словами родичів, тричі втрачав свідомість. Об'єктивно спостерігається: ЧД - 18/хв., ЧСС - 45/хв., АТ - 100/70 мм рт. ст. На ЕКГ встановлено: частота зубців Р - 80/хв., частота шлуночкових комплексів - 42/хв. Який вид аритмії найімовірніший?

Синоаурикулярна блокада

AV блокада I ступеня

Синусова брадикардія

Повна AV блокада

6120 / 6307
Більша частина учасників експедиції Магеллана в Америку загинула від авітамінозу. Захворювання проявлялось загальною слабкістю, підшкірними крововиливами, випадінням зубів, кровотечею з ясен. Укажіть назву цього авітамінозу.

Пелагра

Рахіт

Скорбут (Цинга)

Поліневрит (Бері-бері)

Анемія Бірмера

6121 / 6307
Пацієнт віком 49 років скаржиться на стійке підвищення артеріального тиску (155/120 мм рт.ст.). Рекомендована гіпотензивна терапія упродовж місяця не була ефективною. Під час додаткового обстеження виявлено гіпернатріємію, гіпохлоремію та гіперплазію наднирників. Встановлено діагноз: первинний гіперальдостеронізм. Через неможливість проведення хірургічного лікування пацієнту рекомендовано фармакологічну терапію із використанням антагоніста мінералокортикоїдних рецепторів. Укажіть лікарський засіб, який рекомендовано пацієнту.

Лозартан

Спіронолактон

Метопролол

Амлодипін

6122 / 6307
Під час обстеження двохрічної дитини виявлено: відставання у фізичному та розумовому розвитку (кретинізм), зниження вмісту гормонів щитоподібної залози. Який гормональний препарат слід застосовувати у якості замісної терапії?

Метформін

Тіамазол

Дексаметазон

L-тироксин

6123 / 6307
При макро- та мікроскопічному дослідженнях ділянки великогомілкової кістки та м'яких тканин довкола виявлено дифузне гнійне запалення, що захоплює кістковий мозок, гаверсові канали та періост, осередки некрозу. Про яке захворювання слід думати?

Хвороба Педжета

Хронічний гематогений остеомієліт

Гострий гематогений остеомієліт

Паратиреоїдна остеодистрофія

Остеонекроз

6124 / 6307
Пацієнт помер від вторинної бактеріальної пневмонії. Під час аутопсії виявлено, що м'язи блідо-жовті, з численними вогнищами кальцинозу. Мікроскопічно у м'язах спостерігаються дистрофічні зміни, відсутність посмугованості та зменшений уміст глікогену. У стромі виявляються набряк та запалення, клітинний інфільтрат представлений лімфоцитами, макрофагами та плазмоцитами. Виявлено склеротичні зміни серця, легень та печінки. Якому захворюванню відповідають такі патологічні зміни?

Ценкеровському некрозу м'язів при черевному тифі

Міопатії

Дерматоміозиту (хвороба Вагнера-Унферрихта-Хеппа)

Міозиту

6125 / 6307
Безпосередньо після переходу з горизонтального положення у вертикальне у чоловіка 23-х років частота серцевих скорочень збільшилася на 15 скорочень за хвилину, систолічний тиск не змінився, діастолічний зріс на 10 мм рт.ст. Яка рефлекторна реакція виконавчих структур є причиною збільшення діастолічного тиску?

Звуження резистивних та ємнісних судин

Збільшення систолічного об'єму

Збільшення хвилинного об'єму крові

Звуження ємнісних судин

Звуження судин опору

6126 / 6307
У дитячому садку планується проведення вакцинації проти кашлюку. Який із наведених нижче препаратів слід використати для цього?

АДП анатоксин

Типоспецифічна сироватка

Вакцина АКДП

Нормальний гамма-глобулін

6127 / 6307
Під час розтину тіла чоловіка віком 67 років, виявлено ознаки фібринозного запалення в товстому кишечнику. Для якого захворювання характерні такі патологічні зміни?

Дизентерії

Амебіазу

Балантидіазу

Холери

6128 / 6307
Енергію електрохімічного потенціалу для синтезу АТФ в мітохондріях використовує ферментативна система:

АДФ-рибозилтрансфераза

Піруватдегідрогеназа

РНК-полімераза

АТФ-синтетаза

6129 / 6307
У пацієнта 18 років під час лабораторного обстеження виявлено наявність глюкози в сечі в умовах нормальної концентрації її в плазмі крові. Найімовірнішою причиною цього є порушення:

Канальцевої секреції

Секреції інсуліну

Клубочкової фільтрації

Канальцевої реабсорбції

6130 / 6307
Хворий звернувся до лікаря з приводу туберкульозу легень. Чоловік пред'являє скарги на забарвлення сечі та слізної рідини в червоний колір. Який протитуберкульозний препарат викликає дані симптоми?

Ізоніазид

Рифампіцин

Стрептоміцину сульфат

Етамбутол

Етіонамід

6131 / 6307
У чоловіка 36-ти років з підозрою на туберкульоз нирок було проведене дослідження осаду сечі. При мікроскопії знайдено кислотостійкі бактерії, але за методом Прайса корд-фактор не виявлений. Яке дослідження дозволить найбільш надійно підтвердити або спростувати попередній діагноз?

Серологічна ідентифікація збудника

Шкірна алергічна проба

Фаготипування виділеної культури

Зараження лабораторних тварин

6132 / 6307
Під час мікроскопічного дослідження прямої кишки виявлено великі вогнища некрозу слизової оболонки, некротичні маси просякнуті фібрином, що утворює плівку. Слизова і підслизова оболонки на периферії ділянок некрозу повнокровні, набряклі, з крововиливами і лейкоцитарною інфільтрацією. Яке захворювання можна припустити?

Черевний тиф

Дизентерія

Амебіаз

Холера

6133 / 6307
Пацієнта віком 45 років шпиталізовано до лікарні зі скаргами на температуру, біль під час дихання, задуху та кашель. Після лабораторної та рентгенодіагностики вставлено діагноз: плеврит. Для евакуації ексудату призначено плевральну пункцію. В якому місці плевральної порожнини знаходиться найбільша кількість ексудату?

Діафрагмально-медіастинальний синус

Реберно-медіастинальний синус

Реберно-діафрагмальний синус

Під коренем легенів

Під куполом плеври

6134 / 6307
Укажіть правильне твердження про позачергове збудження, що виникло в міокарді шлуночків.

Знижує автоматизм синусно-передсер-дного вузла

Не впливає на автоматизм синусно-передсердного вузла

Підвищує швидкість проведення збудження по робочих кардіоміоцитах

Знижує швидкість проведення збудження по робочих кардіоміоцитах

6135 / 6307
Для лікування пацієнта застосовано протизапальний лікарський засіб, який блокує дію циклооксигеназ. Укажіть цей препарат.

Тіамін

Алопурінол

Аспірин

Анальгін

6136 / 6307
Пацієнт з діагнозом: гіпертонічна хвороба, помер від гострого інфаркту міокарда. Під час аутопсії у правій гемісфері головного мозку виявлено порожнину 2 см у діаметрі, яка заповнена прозорою жовтуватою рідиною. Стінка порожнини гладка, іржаво-жовта. Яку патологію виявлено в головному мозку померлого?

Ехінокок

Туберкулому

Кісту

6137 / 6307
У пацієнта, який точно виконував рекомендації щодо дотримання певної дієти впродовж 10 днів, було проведено дослідження величини дихального коефіцієнта. Результат: ДК = 1,0. Якої дієти дотримувався пацієнт?

З підвищеним умістом білків і вуглеводів

З підвищеним умістом вуглеводів

Збалансованої

З підвищеним умістом білків і жирів

6138 / 6307
Пацієнтка скаржиться на свербіння, відчуття печіння у ділянці зовнішніх статевих органів та гнійні пінисті виділення з них. Під час лабораторного дослідження у виділеннях виявлено одноклітинні організми грушоподібної форми з чотирма джгутиками, ундулюючою мембраною і шипом на кінці тіла. Який це вид організмів?

Entamoeba gingivalis

Lamblia intestinalis

Trichomonas vaginalis

Trichomonas hominis

6139 / 6307
Під час операції хірургу треба знай-ти місце утворення ворітної печінкової вени. Укажіть це місце.

У печінково-шлунковій зв'язці

За шлунком

На задній стінці печінкової сумки

За головкою підшлункової залози

6140 / 6307
У лікарню госпіталізована хвора у важкому стані з діагнозом геморагічний інсульт в ділянці медіальної поверхні лобової частки правої півкулі головного мозку. Ушкодження якої артерії, найімовірніше, призвело до цього стану?

A) cerebri posterior

A) cerebri anterior

A) communicans posterior

A) cerebri media

6141 / 6307
У хворого на туберкульоз у біоптаті нирки при гістологічному дослідженні у вогнищі казеозного некрозу виявлені хаотично розташовані дрібні зерна хроматину. Наслідком чого є виявлені зміни?

Апоптоз

Мітотична активність ядер

Каріорексис

Каріолізис

6142 / 6307
Пацієнт віком 35 років скаржиться на нежить та головний біль упродовж 5 днів. Після обстеження йому встановлено діагноз: гайморит (запалення верхньощелепної пазухи). Крізь який носовий хід інфекція потрапила до цієї пазухи?

Носоглотковий

Загальний

Середній

Верхній

6143 / 6307
Під час гри у футбол хлопець травмував м'язи. Звернувшись до лікаря, він скаржився на нездатність розігнути гомілку. Який м'яз пошкоджено?

Напівсухожильний м'яз

Чотириголовий м'яз стегна

Грушоподібний м'яз

Квадратний м'яз попереку

Двоголовий м'яз стегна

6144 / 6307
У пацієнта під час Ro-дослідження виявлено затемнення в ділянці пазухи твердої мозкової оболони, яка проходить від півнячого гребня решітчастої кістки до внутрішнього потиличного виступу потиличної кістки. У ділянці якої пазухи виявлено патологічні зміни?

Sinus sagittalis superior

Sinus transversus

Sinus rectus

Sinus sagittalis inferior

6145 / 6307
У пацієнта під час операції виявили пухлину шлунка в первинному вогнищі малігнізації (у межах слизової оболонки). Метастази в лімфатичних вузлах і віддалені метастази відсутні. Який етап розвитку патогенезу пухлини спостерігається у пацієнта?

Промоція

Ініціація

Імунне пригнічення пухлини

6146 / 6307
Який тип вентиляційної недостатності дихання характерний для пневмотораксу?

Обструктивний

Дисрегуляторний

Рестриктивний

Змішаний

6147 / 6307
У студента через добу після іспиту під час аналізу крові виявили лейкоцитоз без істотної зміни в лейкоцитарній формулі. Визначте ймовірний механізм розвитку лейкоцитозу в периферичній крові.

Зменшення руйнування лейкоцитів

Уповільнення еміграції лейкоцитів у тканини

Посилення лейкопоезу

Перерозподіл лейкоцитів в організмі

6148 / 6307
Пацієнт, що прийшов на прийом, скаржиться на свербіж між пальцями. Лікар поставив діагноз: скабієс. Які членистоногі можуть спричиняти це захворювання?

Коростяний свербун

Собачий кліщ

Дермацентор

Тайговий кліщ

6149 / 6307
Хворий звернувся до лікаря з пораненням проксимальної фаланги пальця, яке ускладнилося флегмоною долоні. Гній заповнив спільну синовіальну піхву для згиначів, в якій лежать сухожилки поверхневого та глибокого згиначів пальців. На якому пальці було ушкодження проксимальної фаланги?

IV

V

I

II

6150 / 6307
Під час аутопсії тіла чоловіка віком 29 років, який тривалий час хворів на виразкову хворобу дванадцятипалої кишки були виявлені ознаки перитоніту, множинні стеатонекрози позаочеревинної жирової тканини та підшлункової залози. В діляці тіла підшлункової залози виявлено виразкоподібний дефект діаметром 5 мм і глибиною до 10 мм, краї якого містили некротичні маси. Яке ускладнення виразкової хвороби дванадцятипалої кишки спостерігається у цьому випадку?

Стеноз

Пенетрація

Дуоденіт

Кровотеча

6151 / 6307
Чоловік, який нещодавно повернувся з африканської країни, звернувся до лікаря-уролога зі скаргами на біль під час сечовипускання. У сечі, взятій на аналіз у денний час, були виявлені яйця з характерним шипом. Поставте діагноз.

Шистосомоз урогенітальний

Шистосомоз кишковий

Шистосомоз японський

Опісторхоз

Дикроцеліоз

6152 / 6307
У пацієнта віком 20 років періодично з'являються слабкість, жовтушність шкіри та склер. Діагностовано хворобу Мінковського-Шоффара. Що є характерним для показників крові під час цього захворювання?

Мікросфероцитоз

Агранулоцитоз

Анулоцитоз

Макроцитоз

6153 / 6307
У хворого діагностована тяжка B_12-дефіцитна анемія з порушенням кровотворення і появою в крові змінених еритроцитів. В анамнезі: тотальна резекція шлунка. Наявність яких клітин у периферичній крові дозволяє підтвердити діагноз?

Анулоцити

Нормоцити

Овалоцити

Мегалоцити

6154 / 6307
На відміну від нервових клітин, які зазвичай не розмножуються, стовбурові клітини можуть відновлюватися багато разів. Процес багаторазового відновлення клітин називають:

Гіпертрофія

Диференціація

Проліферація

Апоптоз

Атрофія

6155 / 6307
У померлого чоловіка віком 67 років виявлено: слизова оболонка ректосигмоїдального відділу товстої кишки різко набрякла, тьмяна, з множинними ерозіями, виразковими дефектами та поодинокими поліпами. За результатами гістологічного дослідження визначається: слизова оболонка різко інфільтрована лімфоцитами, плазматичними клітинами, нейтрофільними гранулоцитами та еозинофілами, які зосередженні переважно в просвіті крипт (крипт-абсцеси). Спостерігаються розростання грануляційної тканини з утворенням поліпів і склероз кишкової стінки. Для якого захворювання характерні такі результати дослідження?

Хвороба Крона

Неспецифічний виразковий коліт в стадії загострення

Дизентерія

Черевний тиф

Гострий виразковий коліт

6156 / 6307
У чоловіка, який три доби тому переніс гостру крововтрату, досліджували кров і отримали таку лейкоцитарну формулу: лейкоцити - 12·10^9/л, базофіли - 0%, еозинофіли - 3%, моноцити - 0%, юні - 3%, паличкоядерні - 12%, сегментоядерні - 62%, лімфоцити - 16%, мієлоцити - 4%. Яка форма зміни лейкоцитарного складу крові наявна в цьому разі?

Нейтрофільоз із зсувом вправо

Абсолютна моноцитопенія

Нейтрофільоз із регенеративним зсувом вліво

Абсолютна лімфоцитопенія

6157 / 6307
Гострий герпетичний гінгівостоматит є найпоширенішою первинною інфекцією, яку спричиняє вірус простого герпесу першого типу. Який матеріал треба взяти лікарю-стоматологу для лабораторного підтвердження діагнозу?

Рідину з везикул

Сечу

Кров

Мокротиння

6158 / 6307
При проведені спірограми у пацієнта встановлено зменшення частоти і глибини дихання. Це призведе до зменшення:

Резервного об'єму вдиху

Хвилинного об'єму дихання

Залишкового об'єму

Життєвої ємності легень

Резервного об'єму видиху

6159 / 6307
За 5-8 днів після застосування значної кількості лікувальної сироватки у пацієнта виявлено шкірний висип, свербіж, набряклість, біль у суглобах, підвищення температури тіла. У сечі з'явився білок. Пацієнту встановлено діагноз: сироваткова хвороба. Який важливий фактор патогенезу цього захворювання?

Активація макрофагів

Дегрануляція тканинних базофілів

Активація Т-кілерів

Цитоліз клітин крові

Накопичення в крові циркулюючих імунних комплексів

6160 / 6307
У хворого пухлина підшлункової залози. При хірургічному втручанні доступ проведений шляхом розтину lig. gastrocolicum. При цьому випадково травмована судина, що проходить у даній зв'язці. Кровотечу з якої судини був змушений зупиняти хірург?

A) gastroomentalis dexter

А. gastro-duodenalis

А. gastrica dexter

А. gastrica sinister

А. pancreatico-duodenalis sup.

6161 / 6307
Лікар-дієтолог радить пацієнту під час лікування перніциозної анемії включити до раціону напівсиру печінку. Наявність якого вітаміну у цьому продукті стимулює процес кровотворення?

Н

В_2

В_1

В_12

С

6162 / 6307
У пацієнта з'явилися ознаки запалення слизової оболонки передніх та середніх комірок решітчастої кістки. Крізь яке утворення носової порожнини стало можливим розповсюдження інфекції?

Загальний носовий хід

Нижній носовий хід

Верхній носовий хід

Середній носовий хід

Хоани

6163 / 6307
У пацієнта сироватка крові має молочний вигляд. Під час біохімічного дослідження виявлено високий рівень триацилгліцеролів і хіломікронів. Спадковий дефект якого ферменту викликає цей стан?

Фосфодіестерази

Фосфоліпази

Ліпопротеїнліпази

Панкреатичної ліпази

Тканинної гормон-чутливої ліпази

6164 / 6307
Вам запропоновані для роботи такі препарати: 1) бруцельозна нашкірна вакцина; 2) лептоспірозна вакцина; 3) вакцина БЦЖ; 4) адсорбована коклюшно-дифтерійно-правцева вакцина АКДП; 5) адсорбований правцевий анатоксин. Який вид імунітету створюють ці препарати?

Штучний пасивний імунітет

Антитоксичний імунітет

Антибактеріальний імунітет

Штучний активний імунітет

Нестерильний (інфекційний) імунітет

6165 / 6307
У пацієнта з хронічним тонзилітом виникло ускладнення - заглотковий абсцес. Куди може поширитися інфекція із заглоткового (ретровісцерального) простору?

В грудну порожнину, в переднє середостіння

В надгрудинний міжапоневротичний простір

В грудну порожнину, в середнє середостіння

В превісцеральний простір

В грудну порожнину, в заднє середостіння

6166 / 6307
Пацієнтці, яка звернулася до лікаря зі скаргами на почервоніння та свербіж шкіри обличчя після використання косметологічного крему, призначено дифенгідрамін. Який механізм протиалергічної дії препарату?

Збудження beta-адренорецепторів

Блокада Н_2-гістамінових рецепторів

Пригнічення лейкотрієнових рецепторів

Блокада Н_1-гістамінових рецепторів

Збудження Н_1-гістамінових рецепторів

6167 / 6307
Жінці під час пологів перелили кров донора, який прибув із Анголи. За два тижні у реципієнтки виникла пропасниця. Було запідозрено малярію. Яке лабораторне дослідження потрібно провести для підтвердження діагнозу?

Вивчення лейкоцитарної формули

Вивчення пунктату лімфатичних вузлів

Визначення збудника методом посіву крові на живильне середовище

Проведення серологічних досліджень

Вивчення мазка і товстої краплі крові для виявлення еритроцитарних стадій збудника

6168 / 6307
З метою перевірки крові донорів на наявність антигенів гепатиту В необхідно застосувати високочутливі методи. Яку з названих реакцій слід застосувати з вказаною метою?

Твердофазний імуноферментний аналіз

Імуноелектрофорез

Непрямої гемаглютинації

Непрямої імунофлуоресценції

Зв'язування комплементу

6169 / 6307
У десятирічної дитини за два тижні після перенесеної ангіни з'явився біль на ділянці нирок, незначні набряки, розвинувся сечовий синдром. Проведено нефробіопсію та мікроскопічно виявлено інтракапілярне проліферативне запалення. Під час електронної мікроскопії виявлено великі електронно-щільні субепітеліальні депозити (<<горби>>). Яке захворювання розвинулося у дитини?

Ліпоїдний нефроз

Мембранозний гломерулонефрит

Гломерулонефрит, що швидко прогресує

Гострий гнійний проміжний нефрит

Постінфекційний гломерулонефрит

6170 / 6307
У пацієнта сенсорна афазія, не розуміє зверненої до нього мови. Яка локалізація ураження нервової системи?

Нижня лобова звивина

Середня лобова звивина

Верхня скронева звивина

Верхня лобова звивина

Середня скронева звивина

6171 / 6307
Регуляція експресії генів здійснюється за допомогою різних механізмів. Назвіть ділянки ДНК, у разі індукції яких активується експресія гену:

Сайленсер

Термінатор

Спейсер

Атенюатор

Енхансер

6172 / 6307
В результаті травми у чоловіка 40-ка років зруйновані задні корінці спинного мозку. Які розлади будуть спостерігатися в ділянці іннервації цих корінців?

Порушення функції гладеньких м'язів

Втрата больової чутливості

Втрата температурної і вібраційної чутливості

Втрата всіх видів чутливості

Порушення функції посмугованих скелетних м'язів

6173 / 6307
Під час дослідження каріотипу п'ятирічного хлопчика виявлено 46 хромосом. Одна з хромосом 15-ї пари довша від звичайної, тому що до неї приєднана ділянка хромосоми з 21-ї пари. Укажіть вид мутації, що має місце в цього хлопчика:

Поліплоїдія

Дуплікація

Делеція

Інверсія

Транслокація

6174 / 6307
Дев'ятирічна дитина захворіла гостро: температура тіла - 39,5^oС, інтоксикація, петехіальний висип з некрозами в центрі на нижніх кінцівках, гнійне запалення оболонок ока, серцево-судинна недостатність із різким падінням артеріального тиску. Дитина померла. Під час розтину тіла було виявлено: на шкірі та у внутрішніх органах васкуліти з тромбозом, некрозами, крововиливами і гнійним запаленням; наднирники збільшені в розмірах, темно-червоні, з масивними крововиливами. Для якого захворювання характерні такі симптоми?

Гострий міліарний туберкульоз

Менінгококцемія з синдромом Вотергауза-Фредеріксена

Кір

Геморагічний васкуліт

Скарлатина

6175 / 6307
Під час огляду у хворого виявилося запалення анатомічного утворення, що врівноважує тиск між барабанною порожниною та глоткою. Назвіть це утворення:

Внутрішнє вухо

Слухова труба

Зовнішній слуховий прохід

Соскоподібна печера

Внутрішній слуховий прохід

6176 / 6307
Виконуючи оперативне втручання, хірург повинен потрапити до чепцевої сумки очеревинної порожнини через чепцевий отвір (отвір Вінслова). Які анатомічні структури обмежують його попереду?

Великий чепець

Нутрощева поверхня печінки

Верхня частина 12-палої кишки

Печінко-ниркова зв'язка

Печінково-дванадцятипалокишкова зв'язка

6177 / 6307
До загального шляху катаболізму біологічних макромолекул належить, крім циклу трикарбонових кислот і мітохондріального дихального ланцюга, процес окиснювального декарбоксилювання пірувату. Вкажіть його продукт:

Цитрат

Лактат

Ацетил-КоА

Альфа-кетоглутарат

Малоніл-КоА

6178 / 6307
У студента після вживання м'яса, консервованого в домашніх умовах, з'явились симптоми харчового отруєння Clostridium botulinum: диплопія, порушення мови та параліч дихання. Чим обумовлені такі симптоми ботулізму?

Дією нейротоксину

Ендотоксичним шоком

Секрецією ентеротоксину

Активацією аденілатциклази

Інвазією Cl. botulinum в епітелій кишечника

6179 / 6307
У пацієнта зі встановленим діагнозом: СНІД, на нижній кінцівці виявлено пухлину, що злегка підіймається над шкірою, є безболісними плямами. Гістологічно для видаленої пухлини характерні незавершений ангіогенез, проліферація веретеноподібних клітин, що утворюють пучки, які хаотично переплітаються і розростання грануляційної тканини різного ступеня зрілості з інфільтрацією імунокомпетентними клітинами. Васкулярні проліферати оточені набряклою рихлою сполучною тканиною, наявні відкладення гемосидерину. Який найімовірніший діагноз?

Гемангіоендотеліома

Ангіофібросаркома

Злоякісна перицитома

Саркома Капоши

Фібросаркома

6180 / 6307
У підшлунковій залозі синтезується трипсиноген, який під дією ентерокінази кишечника перетворюється на трипсин. У який спосіб відбувається цей процес?

Шляхом метилювання

Шляхом гідроксилювання

Шляхом фосфорилювання

Шляхом ацетилювання

Шляхом обмеженого протеолізу

6181 / 6307
Центральну роль в обміні амінокислот у нервовій тканині відіграє глутамінова кислота. Це пов'язано з тим, що дана амінокислота:

Використовується для синтезу ліків

Використовується для синтезу ацетонових тіл

Використовується для синтезу глюкози

Зв'язує аміак з утворенням глутаміну

Використовується для синтезу нейроспецифічних білків

6182 / 6307
Під дією УФ-опромінення та інших факторів можуть відбуватися зміни в структурі ДНК. Репарація молекули ДНК досягається узгодженою дією всіх наступних ферментів, textbfЗА ВИНЯТКОМ:

ДНК-полімераза

Аміноацил-тРНК-синтетаза

ДНК-лігаза

Ендонуклеаза

ДНК-глікозидаза

6183 / 6307
У хворого запалення внутрішнього вуха. Після обстеження лікар констатує ураження тіл 1-х нейронів слухового аналізатора. Де вони знаходяться?

G. spirale

G. ciliare

G. trigeminale

G. geniculi

G. vestibulare

6184 / 6307
До лікарні госпіталізовано пацієнта з діагнозом: пневмоконіоз, дихальна недостатність. Порушення якого компонента зовнішнього дихання є характерним для цієї патології?

Порушення нервової регуляції зовніш-нього дихання

Зниження вентиляції легень

Порушення перфузії легень

Зниження процесу дифузії газів

Порушення гуморальної регуляції зовнішнього дихання

6185 / 6307
У дитини виявлено три хромосоми 18-ї пари, що характеризувалося деформацією черепа (здавлений з боків, з низьким чолом і широкою виступаючою потилицею), порушеннями в розвитку опорно-рухового апарату, зрощенням пальців та порушенням у розвитку скелетної мускулатури. Яка спадкова патологія виникла в цієї дитини?

Синдром Дауна

Синдром Клайнфельтера

Синдром Едвардса

Синдром Шерешевського-Тернера

Синдром Патау

6186 / 6307
У кариотипі хворого на синдром Дауна виявлено дві популяції соматичних клітин - нормальні клітини і клітини з трисомією 21. Як називається це генетичне явище?

Модифікація

Поліплоїдія

Мозаїцизм

Генокопія

Фенокопія

6187 / 6307
Для профілактики виникнення інфекційної хвороби при відвідуванні африканської країни людині призначили хінгамін. До якої групи фармакологічних засобів належить цей препарат?

Протиглистні

Протимікозні

Протималярійні

Протитуберкульозні

Протисифілітичні

6188 / 6307
Чоловіку 57 років після обстеження було поставлено діагноз: В_12-дефіцитна анемія - та призначено лікування. За 3 доби був зроблений контрольний аналіз крові. Що буде найадекватнішим критерієм підвищення еритропоезу?

Підвищення кількості тромбоцитів

Зниження кольорового показника

Підвищення рівня гемоглобіну

Підвищення кількості ретикулоцитів

Підвищення кількості лейкоцитів

6189 / 6307
У людей похилого віку часто спостерігається демінералізація кісток (знижений вміст іонів кальцію). Причиною цього може бути знижена секреція:

Паратгормону

Альдостерону

Інсуліну

Тиреокальцитоніну

Тироксину

6190 / 6307
У пацієнта травма голови викликала втрату тактильної і температурної чутливості. Яку звивину головного мозку було пошкоджено?

Прецентральну

Постцентральну

Кутову

Надкрайову

Поясну

6191 / 6307
Лікар-гематолог призначив пацієнту з кровотечею коагулянт, який діє шляхом підвищення синтезу протромбіну та інших факторів зсідання крові переважно в печінці, є синтетичним водорозчинним вітаміном. Який препарат призначив лікар?

Гепарин

Менадіону натрію біосульфат

Тромбін

Кальцію хлорид

Етамзилат

6192 / 6307
На розтині чоловіка, який тривалий час хворів на пресенільне недоумство, виявлено атрофію кори головного мозку, стоншення переважно лобових, скроневих та потиличних часток. Мікроскопічно спостерігається: в корі атрофованих часток мозку виявляють старечі бляшки, пошкоджені нейрони, тільця Хірано. Який діагноз найімовірніший?

Церебральний атеросклероз

Розсіяний склероз

Хвороба Кройцфельда-Якоба

Хвороба Альцгеймера

Хвороба Паркінсона

6193 / 6307
Під час лікування хронічного набрякового синдрому фуросемідом у хворого виникло порушення катіонного складу плазми крові. Який засіб слід використовувати для його корегування?

Натрію хлорид

Кальцію хлорид

Калію хлорид

Натрію гідрокарбонат

Літію карбонат

6194 / 6307
У жінки спостерігається рясний пронос протягом трьох дні. Під час аналізу газового складу крові відзначається знижений вміси НСО_3. Який механізм є провідним у компенсації порушення кислотно-лужної рівноваги у пацієнтки?

Зниження вентиляції легенів

Посилення вентиляції легенів

Зниження реабсорбції бікарбонату в нирках

Зменшення виділення аміаку з сечею

6195 / 6307
У клініко-діагностичній лабораторії під час обстеження пацієнта виявлено: у сироватці крові висока активність ізофермента ЛДГ_1. Для патології якого внутрішнього органу характерні такі клініко-лабораторні показники?

Серця

Скелетних м'язів

Нирок

Підшлункової залози

Печінки

6196 / 6307
Внаслідок кульового поранення в надключичній ділянці зліва у хворого виникло порушення рухів верхньої кінцівки. Які нервові структури пошкоджені?

Шийне сплетення

Спинний мозок

Нервові корінці

Міжреброві нерви

Плечове сплетення

6197 / 6307
На розтині тіла чоловіка, який помер від черевного тифу, в клубовій кишці вия-влені дефекти, розташовані по всій довжині кишки, краї їх рівні, дно утворене м'язовим шаром. Яка зі стадій черевного тифу діагностована?

Загоєння

Мозкоподібного набухання

Утворення виразок

Чистих виразок

Некрозу

6198 / 6307
У молекулярній біології використовують метод визначення послідовності розташування нуклеотидів у молекулі ДНК за амінокислотним складом поліпептиду. На якій властивості генетичного коду базується цей метод?

Універсальність

Виродженість

Специфічність

Колінеарність

Неперекривність

6199 / 6307
Усі нестероїдні протизапальні засоби можуть пошкоджувати слизову оболонку шлунку. Для пошуку речовин, які не викликають цього ускладнення, необхідно знати що його спричиняє. Уплив на який молекулярний субстрат треба зменшити, щоб послабити вираженість цього ускладнення?

Аденілатциклазу

Лізосомальні ферменти

Циклооксигеназу-1

Калікреїн

Циклооксигеназу-2

6200 / 6307
Транскетолаза забезпечує перебіг неокиснювальної фази пентозофосфатного циклу, який веде до накопичення НАДФН і рибозо-5-фосфату, що використовуються безпосередньо для синтезу таких речовин, як:

Нуклеотиди

Вітаміни

Ліпопротеїни

Амінокислоти

Жирні кислоти

6201 / 6307
На розтині тіла чоловіка 58-ми років, який тривалий час вживав значну кількість алкоголю і помер вдома, було знайдено: макроскопічно - права легеня щільна і збільшена в розмірах, тканина на розрізі сіруватого кольору, однорідна, плевра вкрита сіруватими плівчастими нашаруваннями; мікроскопічно - порожнини альвеол містять нитки фібрину, нейтрофіли, гемолізовані еритроцити. Діагностуйте захворювання:

Казеозна пневмонія

Інтерстиціальна пневмонія

Вогнищева пневмонія

Первинний туберкульоз легень

Крупозна пневмонія

6202 / 6307
Чоловікові для введення лікувальної дози протиправцевої сироватки було зроблено пробу на чутливість, яка виявилась позитивною. Специфічну гіпосенсибілізацію хворому найправильніше провести введенням:

Глюкокортикоїдів

Антигістамінних препаратів

Малих доз протиправцевої сироватки

Дозволеної дози протиправцевої сироватки

Імунодепресантів

6203 / 6307
Експериментатору необхідно якнайшвидше виробити умовний рефлекс у собаки. На базі якого безумовного рефлексу доцільно виробляти умовний?

Орієнтувальний

Травний

Захисний

Статевий

Міотатичний

6204 / 6307
При хворобі Верніке-Корсакова, що часто спостерігається у хронічних алкоголіків, харчовий раціон яких містить мало вітамінів, визначається зниження активності транскетолази. Нестача якого вітаміну призводить до цього?

Ніацин

Тіамін

Рибофлавін

Ретинол

Кобаламін

6205 / 6307
Під час лютеїнової фази менструального циклу утворюється жовте тіло. Синтез якого гормону стимулює ця тимчасова залоза внутрішньої секреції?

Тестостерону

Альдостерону

Прогестерону

Паратгормону

Кортикостерону

6206 / 6307
У жінки віком 30 років після тривалого використання губної помади з флюоресцентною речовиною на облямівці губ розвинулася обмежена еритема та незначне лущення, а пізніше з'явилися поперечні дрібні борозни й тріщини. Після застосування спеціальних методик при мікроскопічному дослідженні цієї зони ураження в сполучній тканині спостерігаються сенсибілізовані лімфоцити і макрофаги, відбуваються явища цитолізу. Який тип імунологічної гіперчутливості розвинувся на губах пацієнтки?

Гранулематоз

III тип (імунокомплексна цитотоксичність)

I тип (реагіновий тип)

IV тип (клітинна цитотоксичність)

II тип (антитільна цитотоксичність)

6207 / 6307
У хворої 43-х років на фоні септичного шоку відзначається тромбоцитопенія, зменшення вмісту в крові фібриногену, поява продуктів деградації фібрину, поява петехіальних крововиливів. Вкажіть причину виникнення даних змін:

Екзогенна інтоксикація

Порушення вироблення тромбоцитів

Геморагічний діатез

ДВЗ-синдром

Аутоімунна тромбоцитопенія

6208 / 6307
При обстеженні хворого, який тривалий час приймає глюкокортикоїди, виявлена лімфопенія. Як можна охарактеризувати функціональний стан імунної системи пацієнта?

Імунодефіцит вторинний

Імунодефіцит вроджений

Толерантність до аутоантигенів

Анафілаксія

Імунодефіцит первинний

6209 / 6307
У пацієнта, що перебував у зоні радіаційного ураження, в крові збільшилась концентрація малонового діальдегіду, гідропероксидів. Причиною даних змін може бути:

Збільшення в організмі кисневих радикалів і активація ПОЛ

Збільшення молочної кислоти

Збільшення холестерину

Зменшення білків крові

Збільшення кетонових тіл

6210 / 6307
Під час обстеження трупів тварин поставлено попередній діагноз: сибірка. Який експрес-метод діагностики треба використати для підтвердження цього діаг-нозу?

Реакція аглютинації

Реакція зв'язування комплементу

Реакція гальмування гемаглютинації

Реакція термопреципітації

Реакція Манту

6211 / 6307
У пацієнта встановлено діагноз: водянка яєчка (збільшення кількості рідини в серозній порожнині). Між якими оболонками яєчка локалізується патологічний вміст?

Між парієтальним і вісцеральним листками піхвової оболонки яєчка

Між внутрішньою сім'яною фасцією і піхвовою оболонкою

Між м'ясистою оболонкою і внутрішньою сім'яною фасцією

Між шкірою і м'язом, що піднімає яєчко

Між шкірою і м'язистою оболонкою

6212 / 6307
У пацієнта, який помер від хронічної ниркової недостатності, листки перикарду тьмяні, з тонкими ниткоподібними нашаруваннями сірого кольору. Який патологічний процес відбувся в перикарді?

Серозне запалення

Катаральне запалення

Фібринозне запалення

Гнійне запалення

Проліферативне запалення

6213 / 6307
Вивчаючи під електронним мікроскопом клітини підшлункової залози, було знайдено структури, які поділяють клітину на велику кількість комірок, каналів, цистерн та поєднані із плазмолемою. Укажіть ці органели:

Мітохондрії

Ендоплазматична сітка

Комплекс Гольджі

Центросоми

Рибосоми

6214 / 6307
У жінки 49-ти років після тривалого стояння виявлено набряк ніг. Назвіть можливу причину появи набряків:

Зменшення гідростатичного тиску крові у венах

Збільшення онкотичного тиску плазми крові

Підвищення артеріального тиску

Підвищення гідростатичного тиску крові у венах

Зменшення гідростатичного тиску крові в артеріях

6215 / 6307
Пацієнту з діагнозом: виразкова хвороба шлунку, призначено препарат, блокатор H_2 гістамінових рецепторів ІІІ покоління. Назвіть цей препарат.

Циметидин

Ранітидин

Омепразол

Фамотидин

Роксатидин

6216 / 6307
При розслідуванні спалаху внутрішньолікарняної інфекції виділені чисті культури золотистого стафілококу від хворих, медперсоналу і деяких об'єктів навколишнього середовища. Яке дослідження необхідно провести, щоб встановити ідентичність виділених стафілококів і встановити джерело госпітальної інфекції?

Серотипування

Зараження тварин

Визначення патогенності

Визначення біовару

Фаготипування

6217 / 6307
До невропатолога звернуся чоловік 25 років зі скаргами на слабкість у ногах та порушення ходи. Лікар діагностував міастенію і призначив пацієнту ін'єкції прозерину. Який механізм дії цього препарату?

Інгібітор тормозних процесів

Холіноміметик прямої дії

Активатор синтезу ацетилхоліну

Стимулятор метаболічних процесів

Антихолінестеразна дія

6218 / 6307
Немовля відмовляється від годування груддю, збуджене, дихання неритмічне, сеча має специфічний запах ''пивної закваски'' або ''кленового сиропу''. Вроджений дефект якого ферменту викликав дану патологію?

Дегідрогеназа розгалужених alpha-кетокислот

Аспартатамінотрансфераза

УДФ-глюкуронілтрансфераза

Гліцеролкіназа

Глюкозо-6-фосфатдегідрогеназа

6219 / 6307
У 3-річної дитини з підвищеною температурою тіла після прийому аспірину спостерігається посилений гемоліз еритроцитів. Вроджена недостатність якого ферменту могла викликати у дитини гемолітичну анемію?

Глюкозо-6-фосфатдегідрогеназа

Гліцеролфосфатдегідрогеназа

Глюкозо-6-фосфатаза

Гамма-глутамілтрансфераза

Глікогенфосфорилаза

6220 / 6307
Під час аварійного підйому з глибини у водолаза розвинулися судоми із втратою свідомості. Який патогенетичний механізм є основним у розвитку цих порушень?

Токсична дія азоту

Гіперкапнія

Газова емболія

Гіпоксія

Токсична дія кисню

6221 / 6307
Для роботи серцевого м'язу необхідна енергія. Вкажіть основний субстрат, який є джерелом енергії в м'язі, що працює:

Піровиноградна кислота

Жирні кислоти

Кетоглутарова кислота

Молочна кислота

Амінокислоти

6222 / 6307
Коли чужорідний агент потрапляє в організм, синтез двох класів імуноглобулінів починається майже паралельно, але в одного з них зростання і падіння кількості концентрації відбувається швидше. Які це два класи імуноглобулінів?

IgG і IgD

IgM i IgG

IgA i IgG

IgA i IgD

IgM i IgD

6223 / 6307
Хворий надійшов у інфекційне відділення: шкіра суха, тургор шкіри знижений, випорожнення у вигляді рисового відвару. Поставлено діагноз: холера. Яке порушення водно-електролітного балансу найімовірніше виникає при даному захворюванні?

Гіпоосмотична гіпергідратація

Ізоосмотична гіпогідратація

Гіперосмотична гіпогідратація

Гіпоосмотична гіпогідратація

Гіперосмотична гіпергідратація

6224 / 6307
Чоловік віком 30 років скаржиться на пронос і біль у животі упродовж 5 днів, підвищення температури тіла до 37,5^oC, озноб. Напередодні пацієнт був у лісі та випив води з відкритого водоймища. Встановлено та бактеріологічно підтверджено діагноз: амебна дизентерія. Вкажіть лікарський засіб, який треба застосувати найперше для лікування цього захворювання.

Фуразолідон

Фталазол

Еметину гідрохлорид

Метронідазол

Левоміцетин

6225 / 6307
Надмірне утворення вільних радикалів спричиняє пошкодження клітин. Укажіть неферментативний фактор антиоксидантної системи захисту клітини.

Глюкуронідаза

Вітамін Е

Глутатіонредуктаза

Ціанокобаламін

Супероксиддисмутаза

6226 / 6307
У хворої, яка страждає на поліноз, після поїздки за місто з'явилися набряки губ і повік, сльозотеча, виділення з носа, печіння в очах. Який основний механізм розвитку цього набряку?

Порушення лімфовідтоку

Підвищення гідростатичного тиску в капілярах

Підвищення онкотичного тиску крові

Підвищення проникності капілярів

Підвищення онкотичного тиску тканин

6227 / 6307
Вживання рослин і грибів, зібраних уздовж автомобільних трас, є небезпечним через можливість отруєння свинцем. Що є головним джерелом забруднення довкілля цим хімічним елементом?

Гербіциди

Каналізаційні стоки

Хімічні добрива

Кислотні дощі

Вихлопні гази

6228 / 6307
Хворому поставлений діагноз: гострий пієлонефрит із запаленням стінок ниркових чашечок і мисок. Який епітелій зазнав пошкодження?

Багатошаровий плоский зроговілий

Перехідний

Одношаровий призматичний

Багаторядний війчастий

Багатошаровий кубічний

6229 / 6307
В крові хворого з лейкопенією знайдені антилейкоцитарні антитіла. Який тип алергічної реакції за Кумбсом і Джеллом виник у даному випадку?

Цитотоксичний

Імунокомплексний

Анафілактичний

Гіперчутливість сповільненого типу

Стимулюючий

6230 / 6307
На тлі алергічної реакції у дитини розвинувся набряк гортані. Який тип дихальної недостатності розвинувся в цьому разі?

Обструктивний тип

Дифузійний тип

Рестриктивний тип

Перфузійний тип

Дисрегуляторний тип

6231 / 6307
У жінки 62-х років розвинулася катаракта (помутніння кришталику) на фоні цукрового діабету. Вкажіть, який тип модифікації білків має місце при діабетичній катаракті:

Глікозилювання

АДФ-рибозилювання

Фосфорилювання

Обмежений протеоліз

Метилювання

6232 / 6307
У клініку звернувся пацієнт після травми голови зі скаргами на втрату раніше набутих професійних навичок (праксії). Яка ділянка кори головного мозку ушкоджена?

Gyrus temporalis superior

Gyrus angularis

Gyrus parietalis superior

Gyrus supramarginalis

Gyrus precentralis

6233 / 6307
Чоловік 48 років помер при явищах серцевої недостатності. Під час макроскопічного дослідження серця встановлено, що стулки мітрального клапану ущільнені, потовщені та помірно деформовані. Мікроскопічно виявлено пучки колагенових фібрил гомогенізовані, еозинофільні, оточені незначним макрофагальним інфільтратом, метахромазія не виявляється. У разі забарвлення пікрофуксином виявляються вогнища жовтого кольору. Діагностуйте вид ураження сполучної тканини:

Фібриноїдне набухання

Амілоїдоз

Склероз

Гіаліноз

Мукоїдне набухання

6234 / 6307
У чоловіка внаслідок поранення ділянки передньої поверхні плеча рука не згинається у ліктьовому суглобі. Який з наведених м'язів імовірно постраждав?

M. biceps brachii

M. anconeus

M. pectoralis major

M. triceps brachii

M. deltoideus

6235 / 6307
Чоловік 25 років звернувся до приймального покою зі скаргами на нежить та головний біль протягом 4 днів. Після обстеження йому встановили діагноз: фронтит. Через який носовий хід інфекція потрапила до лобової пазухи?

Верхній

Нижній

Носоглотковий

Загальний

Середній

6236 / 6307
Пацієнт скаржиться на часті кровотечі з ясен. Під час аналізу крові виявлено дефіцит ІІ фактора зсідання крові (протромбіну). Яка фаза зсідання крові насамперед порушена у людини?

Фібриноліз

Утворення протромбінази

Утворення тромбіну

Ретракція згустку

Утворення фібрину

6237 / 6307
У хворого після тривалого нападу сильних головних болів стали неможливі активні рухи лівої руки і кисті. На цих кінцівках тонус м'язів підвищений, м'язи спазмовані, спінальні сухожилкові рефлекси різко посилені, розширені зони рефлексів. Який розлад нервової системи наявний у хворого?

Млявий параліч

Екстрапірамідний параліч

Рефлекторний параліч

Центральний параліч

Периферичний параліч

6238 / 6307
У лікарні у хворого діагностована анемія. Під час збору анамнезу виявлено, що пацієнт вже декілька років хворий на виразкову хворобу шлунка. Який вид анемії, скоріше за все, у цього хворого?

Набута гемолітична анемія

Гостра постгеморагічна анемія

Серпоподібноклітинна анемія

Спадкова гемолітична анемія

Хронічна постгеморагічна анемія

6239 / 6307
У пацієнта з розладом мозкового кровотоку спостерігаються порушення акту ковтання під час вживання рідкої їжі. Який відділ мозку пошкоджено?

Мозочок

Шийний відділ спинного мозку

Проміжний мозок

Довгастий мозок

Середній мозок

6240 / 6307
У чотирирічної дитини спостерігаються гепатомегалія та гіпоглікемія. Біохімічним методом виявлено дефіцит глюкозо-6-фосфатази. Яке захворювання ймовірно у пацієнта?

Хвороба Тея-Сакса

Алкаптонурія

Хвороба Помпе

Фенілкетонурія

Хвороба Гірке

6241 / 6307
Під час гістологічного дослідження в ділянці шийки власної залози шлунку виявляються дрібні клітини, що мають високе ядерно-цитоплазматичне співвідношення і фігури мітозу. Яку функцію виконують ці клітини?

Регенерації епітелію

Секреції пепсиногену

Захисну

Секреції іонів Cl^-

Ендокринну

6242 / 6307
У померлого чоловіка 30-ти років на розтині виявлена збільшена селезінка (вага 900,0 г), збільшена печінка (вага 4000,0 г), збільшені лімфатичні вузли, кістковий мозок діафізу стегна соковитий малиново-червоного кольору. Мікроскопічно у печінці визначаються густі інфільтрати переважно по ходу портальних трактів, що складаються з незрілих кровотворних клітин з округлої форми ядром і вузьким обідком цитоплазми. Про яке захворювання можна думати?

Хронічний мієлоїдний лейкоз

Гострий мієлобластний лейкоз

Гострий лімфобластний лейкоз

Хронічний лімфолейкоз

Генералізована форма лімфогранулематозу

6243 / 6307
Фермент лецитин-холестерин-ацилтрансфераза (ЛХАТ) каталізує реакцію утворення ефірів холестерину за рахунок перенесення залишку жирної кислоти з позиції С-2 холінфосфатиду (лецитину) на холестерин. З якою жирною кислотою холестерин утворює ефіри?

Міристиновою

Лауриновою

Пальмітиновою

Стеариновою

Лінолевою

6244 / 6307
У експериментальних тварин із раціону виключили ліпоєву кислоту. Водночас у них спостерігалося пригнічення піруватдегідрогеназного комплексу. Чим є ліпоєва кислота для цього ферменту?

Кофактором

Продуктом

Інгібітором

Субстратом

Алостеричним регулятором

6245 / 6307
У дитини 14 років був виявлений позитивний азотистий баланс. Що з наведеного може бути причиною цього?

Голодування

Емоційне напруження

Ріст організму

Зниження вмісту білка в їжі

Значні фізичні навантаження

6246 / 6307
Гемоглобін дорослої людини (HbA1) - білок-тетрамер, який складається з двох альфа- та двох бета-пептидних ланцюгів. Яку назву має така структура цього білка?

Четвертинна

Первинна

Вторинна

Третинна

6247 / 6307
У хворого 60-ти років зі спонтанним переломом ребра рентгенологічно відзначаються вогнища множинного двобічного остеопорозу ребер. В біоптаті з зони перелому виявлено однорідну проліферацію клітин лімфоцитарно-плазмоцитарного типу з ознаками клітинного атипізму. Назвіть захворювання:

Гострий остеомієліт

Остеосаркома

Метастаз раку легень

Мієломна хвороба

Туберкульозний остеомієліт

6248 / 6307
Жінка 28-ми років звернулася в поліклініку зі скаргами на головний біль. Лікар запропонував їй парацетамол, взявши до уваги наявність соматичного захворювання. Яка супутня хвороба зробила необхідним призначення саме парацетамолу?

Ревматоїдний артрит

Атеросклероз

Виразкова хвороба шлунка

Холецистит

Нефрит

6249 / 6307
У жінки народилася дитина, хвора на токсоплазмоз. Вона вважає, що заразилася токсоплазмозом від подруги, яка нещодавно також народила хвору дитину. Який спосіб зараження людини токсоплазмою є неможливим?

Пиття води, зараженої ооцитами

Споживання немитих овочів

Контакт із хворою людиною

Контакт із кішкою

Уживання напівсирого м'яса зараженої свійської тварини

6250 / 6307
Внаслідок травми у людини були пошкоджені півколові канали внутрішнього вуха. На які подразники не зможе адекватно реагувати ця людина?

Світлові

Звукові

Шкірні

Рух з кутовим прискоренням

Рух з лінійним прискоренням

6251 / 6307
42-річний чоловік захворів гостро після переохолодження. Захворювання супроводжувалось серцево-дихальною недостатністю, від якої він помер. Під час розтину виявлено, що уся права легеня збільшена, гепатизована, на плеврі значні фібринозні накладення. На розрізі легеня сірого кольору, зерниста, з поверхні стікає каламутна рідина. Гістологічне дослідження виявило гостре запалення з наявністю у просвітах альвеол гнійно-фібринозного ексудату. Який із наведених діагнозів найімовірніший?

Крупозна пневмонія

Осередкова пневмонія

Інтерстиційна міжальвеолярна пневмонія

Ідіопатичний фіброзуючий альвеоліт

Стафілококова пневмонія

6252 / 6307
Під час дослідження калу виявлені яйця аскарид. Який препарат варто призначити для лікування цього пацієнта?

Тетрациклін

Мебендазол

Ністатин

Левоміцетин

Фуразолідон

6253 / 6307
В умовах гострого експерименту кролику зробили перев'язку ниркової артерії. Унаслідок цього у тварини значно зріс рівень артеріального тиску, що є результатом збільшення секреції:

Натрійуретичного гормону

Адреналіну

Вазопресину

Реніну

Норадреналіну

6254 / 6307
Хворий 67-ми років із хронічним закрепом приймав щодня одну таблетку бісакодилу. Через деякий час для отримання ефекту він змушений був приймати по 2 таблетки. Вкажіть, як називається такий тип зміни дії лікарських речовин:

Звикання

Кумуляція

Лікарська залежність

Сенсибілізація

Ідіосинкразія

6255 / 6307
У хворого в результаті крововиливу в головний мозок виявлене порушення сприйняття мови (сенсорна афазія). Ураження якої структури головного мозку можна припустити в цьому випадку?

Верхня скронева звивина

Нижня скронева звивина

Верхня лобова звивина

Задня центральна звивина

Нижня лобова звивина

6256 / 6307
У молодої людини виявлено надлишок соматотропного гормону, збільшені розміри носа, губ, вух, нижньої щелепи, кистей і стоп. Який найімовірніший діагноз?

Гіпофізарний нанізм

Адісонова хвороба

Адіпозогенітальна дистрофія

Акромегалія

Хвороба Іценко-Кушинга

6257 / 6307
Під час адаптації до перебування в горах, в еритроцитах зростає синтез 2,3-дифосфогліцерату. Що стимулює ця речовина в організмі людини?

Окисне фосфорилювання

Утворення оксигемоглобіну

Утворення карбгемоглобіну

Дисоціацію оксигемоглобіну

6258 / 6307
У хворого з опіковою хворобою як ускладнення розвинувся ДВЗ-синдром. Яку стадію ДВЗ-синдрому можна припустити, якщо відомо, що час згортання крові хворого за Лі-Уайтом становить менше, ніж 3 хвилини?

Фібринолізу

Гіперкоагуляції

Термінальну

Перехідну

Гіпокоагуляції

6259 / 6307
Активність якої ланки нервової системи підвищується під дією тиреоїдних гормонів?

Метасимпатичної ланки автономної нервової системи

Соматичної ланки нервової системи

Симпатичної ланки автономної нервової системи

Парасимпатичної ланки автономної нервової системи

6260 / 6307
У нормальних умовах кровообіг матері і плода не має прямих зв’язків. Кров ембріона протікає по судинах ворсинок хоріона, а материнська циркулює в міжворсинковому просторі ендометрію матки. Укажіть, що розділяє кров плода і матері:

Сполучнотканинні перегородки

Фібриноподібна оксифільна маса (фібриноїд Лангханса)

Замикальна пластинка основної відпадаючої оболонки матки

Гемохоріальний бар'єр

6261 / 6307
Чоловік скаржиться, що у разі згадування про минулі трагічні події в його житті у нього виникають тахікардія, задишка й різкий підйом артеріального тиску. Які структури ЦНС забезпечують ці кардіореспіраторні реакції пацієнта?

Чотиригорбкова пластинка середнього мозку

Мозочок

Кора великих півкуль

Латеральні ядра гіпоталамуса

6262 / 6307
Під час гострого експерименту собаці під наркозом увели вазопресин, унаслідок чого зменшилася кількість сечі. Як діє цей антидіуретичний гормон при його застосуванні?

Посилює реабсорбцію натрію

Зменшує реабсорбцію води

Посилює реабсорбцію води

Збільшує реабсорбцію кальцію

6263 / 6307
До приймального відділення лікарні госпіталізовано п'ятирічну дитину, у якої встановлено такі симптоматичні явища: сильне рухове збудження, марення, хрипкий голос, розширені зіниці, які не реагують на світло, суха, гаряча, гіперемована шкіра, тахікардія і тахіпное. Ці симптоми виникли після споживання дитиною ягід беладони. Препарат якої фармакологічної групи треба призначити?

Н-холінолітики

М-холіноміметики

Антихолінестеразні засоби

Реактиватори холінестерази

6264 / 6307
Пацієнтку шпиталізовано до гінекологічного відділення з підозрою на внутрішньоочеревинну кровотечу (позаматкова вагітність). Крізь який утвір проводять пункцію для термінового діагностування кровотечі?

Матковий зів

Заднє склепіння піхви

Шийку матки

Переднє склепіння піхви

6265 / 6307
Під час бактеріологічного дослідження випорожнень на середовищі Ендо виросли колонії червоного кольору з металевим блиском, які на склі аглютинувалися полівалентною ОК-сироваткою. За якими показниками можна встановити патогенний варіант кишкової палички?

За культуральними властивостями

За токсигенними властивостями

За антигенними властивостями

За морфологічними властивостями

6266 / 6307
Пацієнта шпиталізовано до інфекційного відділення з проявами лихоманки, що повторюється вдруге з інтервалом 2 дні. В краплині крові, зафарбованій за Романовським-Гімзою, виявлено звивисті клітини синьо-фіолетового кольору. Який мікроорганізмів викликав захворювання?

Borrelia recurrentis

Treponema pallidum

Plasmodium vivax

Leptospira interrogans

Rickettsia typhi

6267 / 6307
Відомо, що у разі гострого отруєння морфіном використовується специфічний антагоніст - налоксон. Який фактор являється основним у розвитку його антагоністичної дії?

Конкуренція за зв'язування з <<опіатними>> рецепторами

Пряме збудження дихального центру

Рефлекторне збудження дихального центру

Зменшення чутливості організму до морфіну

6268 / 6307
У пацієнта з підозрою на системне захворювання з ділянки підвищеної щільності і малорухливості шкіри зроблено біопсію. Під час дослідження в дермі виявлено всі види дезорганізації сполучної тканини зі слабо вираженою клітинною реакцією, грубим склерозом і гіалінозом. Для якого захворювання характерні такі патологічні зміни?

Склеродермії

Вузликового периартеріїту

Фіброми шкіри

Псоріазу

6269 / 6307
Хлопця госпіталізували на 3-тю добу від початку захворювання з попереднім діагнозом: черевний тиф. Який метод лабораторної діагностики дозволяє поставити мікробіологічний діагноз?

Мікроскопічний

Біологічний

Алергічний

Бактеріологічний

6270 / 6307
У чоловіка, який надійшов до неврологічного відділення, діагностовано крововиливи у стовбур мозку. Цей стан супроводжувався порушенням дрібної моторики кистей рук, амімією обличчя та збільшенням тонусу соматичних м'язів. Ураження якої з перерахованих структур стовбуру мозку могло викликати цей стан?

Вестибулярного латерального ядра Дейтерса

Ретикулярної формації

Чорної субстанції

Ядра третьої пари черепно-мозкових нервів

6271 / 6307
У породіллі 35 років присутній больовий синдром, пов'язаний із затримкою першого періоду пологів. Який препарат найліпше застосувати для зменшення болю?

Промедол

Кетамін

Кодеїн

Анальгін

6272 / 6307
Сестра медична вводила пацієнтці ліки в м'язи задньої поверхні плеча. Раптом пацієнтка відчула сильний біль, який поширювався на задню поверхню передпліччя. Який нерв було ушкоджено під час уведення ліків?

Серединний

Променевий

Пахвовий

Ліктьовий

6273 / 6307
Подразнення слизової оболонки бронхів викликає кашльовий рефлекс. Де замикається рефлекторна дуга цього рефлексу?

Уставні нейрони на рівні спинного мозку

Довгастий мозок

Бічні роги спинного мозку

Передні роги спинного мозку

6274 / 6307
Який препарат необхідно застосувати як антидот у разі отруєння наркотичними анальгетиками?

Протаміну сульфат

Адреналіну гідрохлорид

Тіосульфат натрію

Налоксон

6275 / 6307
Пацієнт віком 27 pоків скаржиться на в'ялість, швидку розумову та фізичну втомлюваність, диспептичні порушення. Під час обстеження виявлено: позитивні туберкулінові проби, гіпоглікемія, AT - 90/60 мм рт. ст., гіпонатріємія, гіперпігментація шкіри. У разі якої патології наднирників спостерігаються подібні явища?

Гіпофункція мозкового шару наднирників

Хвороба Аддісона

Синдром Конна

Синдром Іценка-Кушинга

6276 / 6307
Пацієнтці віком 45 років було встановлено діагноз: недостатність прищитоподібних залоз. Як зміниться робота нирок при цій патології?

Зменшиться фільтрація кальцію в ниркових клубочках

Зменшиться реабсорбція кальцію в дистальних канальцях

Збільшиться синтез простагландинів

Збільшиться синтез урокінази

6277 / 6307
Під час обстеження хлопчика виявлено, що він не здатний скласти губи трубочкою, не може задути свічку, під час сміху кутки рота не піднімаються догори, а ротова щілина розтягується в боки (поперечна посмішка). На атрофію якого м’язу вказують такі симптоми?

Щічного м'яза

Колового м'яза рота

Великого виличного м'яза

М'яза сміху

6278 / 6307
Хворому з виразковою хворобою призначено фамотидин. Кислотність шлункового соку значно знизилась. Який механізм лежить в основі дії даного препарату?

Блокада Н-холінорецепторів симпатичних гангліїв

Блокада Н2-гістамінових рецепторів

Блокада Н1-гістамінових рецепторів

Блокада М1-холінорецепторів

6279 / 6307
З метою пригнічення аутоімунних реакцій після трансплантації органів обов'язковим є проведення курсу гормонотерапії. Які гормони застосовують у цьому випадку?

Адреналін

Глюкокортикоїди

Соматотропний гормон

Мінералокортикоїди

Статеві гормони

6280 / 6307
Хворий надійшов у реанімаційне відділення в стані гіпоксії, що виникла внаслідок аспірації блювотних мас. Об'єктивно: стан важкий, шкірні покриви вологі, бліді, з акроціанозом. Спостерігаються тахіпное, тахікардія, зниження АТ. Який із наведених симптомів гіпоксії, що розвивається гостро, належить до термінових захисно-пристосувальних реакцій організму?

Блідість шкірних покривів

Зниження АТ

Розвиток акроціанозу

Тахікардія

Підвищення потовиділення

6281 / 6307
Судово-медичний експерт під час розтину тіла 20-річної дівчини встановив, що смерть настала внаслідок отруєння ціанідами. Порушення якого процесу, найімовірніше, було причиною смерті дівчини?

Тканинного дихання

Синтезу сечовини

Синтезу гемоглобіну

Транспорту водню за допомогою малат-аспартатного механізму

6282 / 6307
У чоловіка під час обстеження в клініці виявлено різке зниження показників гемоглобіну. Яка функція крові порушується?

Гуморальна

Гомеостатична

Дихальна

Трофічна

6283 / 6307
Птахи щороку перелітають з холодних регіонів до теплих, кожного разу вони прилітають до тієї ж місцевості. Який інстинкт ними керує?

Захисний

Екологічний

Інстинкт гри

Орієнтовний

6284 / 6307
Якщо у собаки тривалий час виробляти дуже тонке диференціювання, то у підсумку може настати виснаження процесу гальмування і розвиток стійкого тривалого збудження. Яка форма патології нервової системи може при цьому виникнути?

Невроз

Парез

Атетоз

Фібриляція

6285 / 6307
У дитячому садку в більшості дітей раптово з'явилися симптоми гастроентериту. Виявлено, що симптоми виникли за кілька годин після вживання молочної каші. У ході бактеріологічного дослідження блювотних мас та залишків каші, виділено золотистий стафілокок. Яке дослідження потрібно провести для уточнення джерела інфекції?

Виконати дослідження обладнання харчоблоку

Визначити наявність антитіл у хворих дітей

Виконати фаготипування виділених штамів

Визначити здатність штамів до токсиноутворення

Поставити алергічну пробу

6286 / 6307
За декілька місяців після пологів у жінки спостерігається випадіння волосся та зубів, втрата ваги, млявість. АТ, температура тіла та рівень глюкози крові знижені. Під час лабораторного дослідження виявлено, що рівень соматотропного та кортикотропного гормонів у крові знижений. Яке порушення функції гіпофізу у пацієнтки?

Акромегалія

Гіпофізарний нанізм

Пангіпопітуїтаризм

Нецукровий діабет

6287 / 6307
Під час мікроскопічного дослідження біоптату печінки жінки, у якої протягом 10 років спостерігається жировий гепатоз, виявлено: розширення та склерозування портальних і перипортальних трактів, псевдочасточки невеликих розмірів, розділені вузькими прошарками сполучної тканини, виражене середньо- і великокрапельне ожиріння цитоплазми гепатоцитів. Для якого захворювання характерні такі симтоми?

Вторинний біліарний цироз печінки

Постнекротичний цироз печінки

Неповний септальний цироз печінки

Портальний цироз печінки

6288 / 6307
Пацієнт відзначає часті проноси, особливо після вживання жирної їжі, схуднення. Лабораторні дослідження показали наявність стеатореї, кал гіпохолічний. Можлива причина такого стану:

Нестача ліпази

Обтурація жовчовивідних шляхів

Незбалансована дієта

Запалення слизової тонкого кишечника

6289 / 6307
У жінки віком 37 років протягом року періодично виникали інфекційні захворювання бактеріального генезу, їх перебіг був вкрай тривалим, ремісії - короткочасними. Під час обстеження виявлено низький рівень основних класів імуноглобулінів. Порушення функції яких клітин найімовірніше спричинило такий стан у пацієнтки?

Лімфоцитів

Плазматичних

Макрофагів

Фагоцитів

Нейтрофілів

6290 / 6307
У пацієнта з печінковою патоло-гією виникли брадикардія, зниження артеріа-льного тиску, ознаки пригнічення нервової системи. Для якої патології печінки характерна така сукупність ознак?

Гепаторенальний синдром

Синдром портальної гіпертензії

Дисхолія

Холемічний синдром

6291 / 6307
Для лікування урогенітальних інфекцій використовують хінолони - інгібітори ферменту ДНК-гірази. Який процес порушується під дією хінолонів найперше?

Транскрипція

Реплікація

Репарація

Трансляція

6292 / 6307
Пацієнту змастили кінчик язика місцевим анестетиком. Відсутність сприйняття якого смаку спостерігатиметься у цьому випадку?

Кислого

Солодкого

Солоного

Кислого та солоного

6293 / 6307
З метою встановлення токсигенності виділених від пацієнтів збудників дифтерії культури висіяли на чашку Петрі з поживним агаром по обидва боки від розташованої в центрі смужки фільтрувального паперу, змоченого протидифтерійною антитоксичною сироваткою. Після інкубації посівів в агарі між окремими культурами і смужкою фільтрувального паперу виявлено смужкоподібні ділянки помутніння середовища. Яку імунологічну реакцію було виконано?

Реакцію Кумбса

Реакцію преципітації в гелі

Реакцію кільцепреципітації

Реакцію опсонізації

6294 / 6307
В біоптаті бронха хворого 67-ми років, який 17 років страждав на хронічний бронхіт, виявлено циліндричне розширення бронхів, слизові залози кістоподібно змінені, визначаються ділянки заміщення призматичного епітелію на багатошаровий плоский. Який патологічний процес виявлено у слизовій оболонці бронхів?

Гіпертрофія

Дисплазія

Метаплазія

Гіперплазія

Гетеротопія

6295 / 6307
Один з синдромів проявляється ураженням зубів, волосся і кісток. У кожному поколінні є хворі, з однаковою частотою хворіють представники чоловічої і жіночої статі. Визначте тип успадкування синдрому:

Аутосомно-рецесивний

Зчеплений з Х-хромосомою домінантний

Зчеплений з Х-хромосомою рецесивний

Зчеплений з Y-хромосомою

Аутосомно-домінантний

6296 / 6307
Пацієнт із діагнозом: ревматичний міокардит, періодично відчуває перебої в роботі серця. На ЕКГ виявлено: нерегулярна поява ідіовентрикулярних екстрасистол. Який патогенетичний механізм зумовлює виникнення компенсаторної паузи у цьому разі?

Пригнічення функції синусового вузла

Порушення скоротливої здатності міокарда

Затримка збудження в атріовентрикулярному вузлі

Рефрактерність міокарда до сприйняття чергового імпульсу

6297 / 6307
У півторарічної дитини діагностовано імунодефіцит. Встановлено, що кількість В-лімфоцитів у нормі, але вони функціонально неактивні. Який фактор захисту ротової порожнини буде відсутній у цієї дитини?

Секреторний імуноглобулін А

Фібронектин

Інтерферон

Лактозо-пероксидазна система

6298 / 6307
У чоловіка віком 75 років із заочеревинної клітковини видалено пухлину розміром 16,0х8,0х6,5 см. Мікроскопічно виявлено анаплазовані клітини з вираженими ознаками атипізму, поліморфізму та мітотичної активності, іноді спостерігаються великі спотворені клітини з суданофільними вакуолями в цитоплазмі. Для якого захворювання характерні такі результати дослідження?

Ліпосаркоми

Лейоміосаркоми

Ліпоми

6299 / 6307
На ізольованому серці вивчалася швидкість проведення збудження у різних його ділянках. Де була виявлена найменша швидкість?

Волокна Пуркіньє

Міокард передсердь

Атріовентрикулярний вузол

Пучок Гіса

6300 / 6307
У дитини виявлено гельмінти. Які зміни в периферичній крові спостерігатимуться при такій патології?

Еозинофілія

Моноцитоз

Базофілія

Лейкоцитоз

6301 / 6307
На спеціальному живильному середовищі після посіву гною, що виділяється з уретри, виросли ніжні блакитнуваті колонії. При мікроскопії препаратів з них виділені грамнегативні бобовидні диплококи. Збудником якої хвороби вони є?

Меліоїдозу

Гонореї

Сифілісу

Хламідіозу

6302 / 6307
У ВІЛ-інфікованого пацієнта спостерігається пригнічення активності імунної системи. Ураження яких клітин найбільшою мірою обумовлює стан імунодефіциту у цього пацієнта?

Макрофагів

Т-кілерів

В-лімфоцитів

Т-хелперів

6303 / 6307
Що потрібно додати до донорської крові, законсервованої цитратом натрію, щоб спричинити її згортання?

Іони натрію

Фібриноген

Іони кальцію

Протромбін

6304 / 6307
Хворі на алкоголізм отримують основну масу калорій зі спиртними напоями. У них може виникнути характерна недостатність тіаміну, при якій спостерігаються порушення функцій нервової системи, психози, втрата пам'яті. Зі зниженням активності якого ферменту пов'язаний цей процес?

Альдолаза

Трансаміназа

Піруватдегідрогеназа

Алкогольдегідрогеназа

6305 / 6307
У пацієнта з діагнозом: цукровий діабет, виявлено підвищений вміст кетонових тіл у крові. Із якої сполуки синтезуються кетонові тіла?

Ацетил-КоА

Ацил-КоА

Оксіацил-КоА

Сукциніл-КоА

6306 / 6307
У дитини 3 років, яка страждає на квашіоркор, спостерігається порушення зроговіння епідермісу та збільшення його злущення, є жирова інфільтрація печінки. Який тип голодування спостерігається у цьому разі?

Вуглеводне

Мінеральне

Жирове

Білкове

Енергетичне

6307 / 6307
У жінки під час пологів в зв'язку з крововтратою визначили групу крові за системою АВО. Реакція аглютинації еритроцитів відбулася зі стандартними сироватками груп О (I), А (II) і не відбулася зі стандартною сироваткою групи В (III). Досліджувана кров належить до групи:

АВ (IV)

В (III)

А (II)